You are on page 1of 931

CALCULUS WITH

ANALYTIC GEOMETRY'
Answers to OddDNumbered
Exercises / SelhTests
Daniel J. Fleming
Department of Mathematics
St. Lawrence University

James J. Kaput
Department of Mathematics
Southeastern Massachusetts University

Harper & Row, Publishers


New York Hagerstown Philadelphia San Francisco London
Answers to Odd-Numbered Exercises/Self-Tests to accompany Calculus with
Analytic Geometry 0 1979 by Daniel J. Fleming and James J. Kaput.
ANSWERS TO ODD-NUMBEItED
EXERCISES / SELFlTESTS
A-2 ANSWERS TO ODD-NUMBERED EXERCISES/SELF-TESTS

Chapter 1

See. 1.1
1. [-11/2, -1/2] 3. x = 2 5. [-9,3] 7. (a) x = -3, -2; (b) x = 5 9. (-co, -2), [-1,2/5], and [4, +co)
11. (-00, -1) and (0,5/3) 13. (-3, -fi) and (fi, +co)

See. 1.2

1.

19-
;51
(u)

p5) \;. ( 6 ) 9 3.

(0, -15)
~
-17
(T,lo)

21-
-23

11.9 - 1 = 4(x - 4) 13. x = 3 1,5. ( a ) y - 1 = 4(x


5. (3, 3) 7. ( u ) -1; ( b ) y = - x

23.
-

)j
4); ( b ) y = 4~ -

(I/'& 0 )

(0, -1)
~
+6
15;
9. the line in 8 ( c ) with slope = 4
4~ - J - 15 = 0 17. y = - X
(6)

25. ( a ) y = x +
1; (b) yes; ( d ) y = x
+
1

27. (6) 3y +x - 9 = 0 29. (4, 13/2), (6,5), and (8, 7/2) 31. (a) (13, 3), (6) (-9, 13)

See. 1.3
1 -81
1. 3. 5. ( a ) parabola; ( b ) axis x = - 4,conc. up, ext. pt. (- 7>7)

x = -1/6
7. (a) (x - 2)z + (y + 3)2 = 25; (6) a circle, center at (2, -3), radius 5 9. ( a ) parabola; ( b ) axis x = 3/2, ext. pt. (3/2, 15/4) conc. down
11. no graph 13. ( u ) parabola; (b) opening right, axisy = 1, ext. pt. (0, 1) 15.

y=o

2 1. single pt. (0, 0)

(-11/6, -169/12)
x = -11/6

Review Exercises
I.x<3/4 3.21/4>x 9. - j < x < 3
5.x>-9 ll.x=-5 13. - 2 > x o r x > 1 4 / 5 15.nosolution 1 7 . ( ~ + 2 ) ~ + 4
19- 2(x - $z - 9 21- 2 ( x - +S
23. 3(x + i)z-
Jfi)' 25. ( - 3 , -2) U (0, 1/2) U ( 3 , + w )
27. ( - W , -11 U (0,$) U [l, + m ) 29. ( - 0 0 , + w ) 31. (a)
33. (a) 5; (b) (-9, -4); (6) 2; ( d ) 3y - 4x - 10 = 0; ( e ) y + 4 =
a;
(6) (4,s); ( c ) -4; ( d ) 5x + 2y - 27 = 0; ( e ) y - = {(x - 4)
-a(.
+ 9)35. (a) 5x + 3y - 15 = 0; (b) 2x - y - 6 = 0;
S
(c) 2x --y + 3 = 0 37. (x - 5)2 + (y - 6)2 = 1/9 39. ( x + 2)2 + ( y + 4)2 = 16 41. cent. (-6,8), 7 = 10 43. cent. (3, -4),
ANSWERS TO ODD-NUMBERED EXERCISES/SELF-TESTS
A-3

r = 5 45. none of these 47. st. line, slope 3 , y int. 2 49. par. opening right, axisy = - 1, ext. pt. (-5, - 1) 5 1. st. line, slope 1,
y int. 8 53. par. opening up, axis x = ext. pt. (-$, +y) .55. par. opening up, axis x = 2, ext. pt. at ( 2 , 3) 5 7 . st. line, slope 2,
-3,
y int. -5 59. vert. st. line through (46.1,O) 61. none of these 63. none of these 65. a circle, cent. at (0, 2), 7 = 2 67. a par.,
opening right, axisy = -4, ext. pt. is ( 0 , -4) 69. none of these

3. ( a ) (2, -2); ( b ) y - x + 4 = 0; (c)y- x = O

(-l/Z, -25/4)' I

4. (x - 6)' + ( y - 8)' = 100

Chapter 2

See. 2.1

x = -1; ( b ) all R except m(x) = -2; (c) 7- [-3,O]U[l,+o0) 9. ( - 0 0 , - 2 ) U ( 7 , + 0 0 ) 11.afunctionisgiven

I 1

2x 1 x+h-1 3-1
(b)
-
ZTT + 2(24? (6) + +
+ 2(x + h)'; ( d ) h--l
h + l
+ 2h2; ( e ) ___
3+1
+ 2.9 19. (u) 3x + 1; ( b ) - x - 1; (6) 2x2 + x; ( d ) -2.-
2x+ 1 '
x+4
+ __ x + 4, (3x - 4)(x + 4) (3x - 4) * 3
x # -1/2; ( e ) 2x + 1; (f)2x + 1; (9) x ; ( h ) 4x + 3 21. ( a ) 3x - 4
3
; ( b ) 3~ - 4 - -
3
> (6)
3
;(d) x + 4 ,
+
x # -4; (e) x ; (f)x ; (g) 9x - 16; (h) ___ 23. ( a ) 5 x ; ( b ) - x ; (c) 6x2; ( d ) 2/3, x # 0; ( e ) 6x; (f)6x; (g) 4x; ( h ) 9 x
9
+ 3x + 3; ( b ) x 3 + 3x - 1; (c) 2x3 + 6x + 2; ( d ) x 3 ; ( e ) 15; (f)2; (9) (x3 + 3x + 1)3 + 3(x3 + 3x + 1) + 1; (h) 2
+ +
25. ( a ) x 3
2
27. f ( x ) = (g o h)(x), where g ( x ) = $ and h(x) = 2x + 1 29. f(x) = (g o h)(x), where g ( x ) = x + 1 and h(x) = 2x
A-4 ANSWERS TO ODD-NUMBERED EXERCISES/SELF-TESTS

See. 2.2
1
1. -1/2 3. does not exist 5 . 1/(4 6)
7. -6 9. -& 11. 96 13. the answers are the same 15. lim
r--z+
~

x +2 - +m,

lim --1
+ 11 - + ffi
- -ffi 17. lirn 1 19. ( a ) lirn S(X) = 0; ( b ) lim ~ ( x =
) 1; (c) no
+2
~

m--z- x --I12 12x I-1- z-1+

See. 2.3
1. (a) 1944 ft/s; ( b ) 1836 ft/s; (c) ~ 1 7 3 2 . 3 ft/s;
2 ( d ) 432(4 + Ax); ( e ) 1728 ft/s;
(f)864x ft/s; (g) 4665.6 ft/s; ( h ) 1/3 ft 3. -2 5. 10x
-2 1 4
7. 6 ~ 2- 9. 9 9 11. 2 ~ 3+ 13. 2 ~ 3x2 - 15. - 17. - 19. ( u ) A V = -$3? A7 + 37(A7) + AT)^); ( b ) r = 2, A7 = 1
3x3 2& 3
4?r
gives A V = -
4?r * l9 ; r= 20, Ar = 1 gives A V = - * 1261; (c) 4 ~ 7 (~d ); they are equal-a sphere grows by spherical shells 21. ( a ) 144 - 32x;
3 3
(6) 4.5 s; (c) 324 ft; ( d ) x = 9, - 144 ft/s; ( e ) 324 ft; ( f )
f

See. 2.4

1- ( a ) f ( b ) sincef(x) = - 2 x , , f ( x ) > 0 for x < 0,f(x) = 0 for x = 0,f(x) < 0 for x > 0; ( c ) y - 2 = - 2 ( x - 1)

3. ( a ) f (6) sincef(x) = 3, the slope is always positive; ( c ) y + 2 = 3 ( x - 1) (same as graph)

(6) sincef(x) = - 3 2 , the slope is everywhere negative except at 0; ( c ) y = - 3 ( x - 1) 9. since lirn


S-O+
6= 0,

then lirn __
1 = + co; thus there is a vertical tangent at x =0 11. ( a ) ___
-1 ;(b)lima=-m
17.x<3 19.(u)l;(b)l;(c)l;
x-o+ 2 6 dx (x - 1)2 c-1

(d) - \/loo=: 0.05 # 1; (e) horizontal distance between points with samey coordinate is constant; (f) they are horizontally parallel,
but not vertically parallel

See. 2.5

l.5/12 3. fi 5.4 7. -1/25 9.4 11.lim


x-0
15. yes 17. f is disc. at x = 1
1 9 . f i s disc. at x = %2 21. f i s disc. at x = 2 25. S(X) = [XI is disc. at every integer value 27. definef(x) = 0 and g ( x ) = 0; then
lirn [f(x)
x-0
+
g ( x ) ] = lim [O] = 0 =f(0)
x-0
g(0) +
ANSWERS TO ODD-NUMBERED EXERClSES/SELF-TESTS A-5

Review Exercises

1. (u) x2 + 3x - 1; 3; (b) 3x3 - x2; 2; (c) -3x


;
-1 1
2; ( d ) 3x2 - 1; 2; ( e ) (3x - 1); 4; (f)3(3x - 1) - I ; 5; (g) (x2)2; 1 3. ( a ) ; x + + 5; 7;
X2

8; (d) 3x4 + 5; 8 ; ( e ) 3x4 + 5; 8; ( f ) 3(3x4 + 5)4 + 5; 3 * 8 4 + 5; (9) x ; 1 7. (a)x2 - 3x + 2; 0; (6) 4(x2 - 3x - 2); -16; (c)
x2
4
- 3x -
.
2
- I ; (d) 4; 4; ( e ) 2; 2; cf) 4; 4; (g) (x2 - 3x - 2) - 3(x2 - 3x - 2) - 2; 26 9. (u) -+ 2x2; not defined at 1; (b) 2x2-; not

defined at 1; (c)
m,not,
7 defined at 1; ( d ) d m ;not defined at 1; ( e ) 2(x - 3); not defined at 1, since g(f(l)) is impossible;
2x

(f) d z ; not defined at 1; (g) 8x4; 8 11. (a)___


3
2x
x2 - 4
2
(b) -1/3; -5;
(c) *.
x - 2
=>
- 3 ; (d) -,
-2x - 3
1;
2x - 3
x-2 -1 x + 2 3
(f) G , T ;(9) -.
2xf5 7
- 13. f(x) = (g o h)(x), where h(x) = x +
1 and g ( x ) = x 3 15. h(x) = ( f o g)(x), where g ( x ) = x2 and

f ( x ) = 3x3 + 4.9 +
2~ 17. ( a ) y = 3(2r) - 2(2r) - 2 ; ( b ) =
~ 3(3t 1) - 2(3t +
1) - 2; ( C ) Y = 3(t +
At) - 2(t At) - 2; + +
) ~2(3u) - 2 19. not a function of x 2 1. is a function of x 23. is a function of x 25. is a
( d ) y = 3(m2) - 2(w2) - 2; ( e ) y = 3 ( 3 ~ -
function of x
x2 - 5x
ifx#O
27. (a) h(x) = x - 5, x # 0; (b) h(x) =
(-5 ifx=O

x 2 - x - 12
ifx # 4
29. ( u ) h(x) = x + 3 , x # 4; (b) h ( x ) =
(7 ifx = 4

31. ( u ) limg(x) = 8 35. lim (2 3x) = -1 37. only number 31 39. (a)lim- x2 5x = -5; (6) lim x 2 - x - 12
-
- =7
x+4 x-1 2-0 x
x-4 x -4
2(x - 5) 2(x - 5) 1 2
41. lim ~ = 2; lim = -2 ~ 45. ( u ) g(x) = 1 - -; ( b ) y - 3/2 = 0 47. ( u ) f ( t ) = --; ( b ) y - 4 = -2(t - 1)
2-5+ Ix - 5) z-5- Ix - 5) X2 t3
4
49. ( a ) - =
-1 -3
. (6) x = 1 is not in the domain 51. (a)g(x) = -; ( b ) y - 1 = -3(x - 1) 53. ( u ) f ( u ) = 3u2 - IOU;
~

dx 2 m x4
(b)y +
4 = -7(u - 1) 5 5 . ( u ) h(r) = 12r; ( b ) y - 16 = 12(r - 1) 57. cont. everywhere 59. disc. at x = 2

x
X

Ix= I

Self -Test
7x - 1
1. (u) (f- g)(x) = - 2 + 5x - 1; ( b ) cf.g)(x) = (7x (f/g)(x) = -
- 1)(X* + 2x);
x2 2x
( d ) ( f o g ) ( x ) = 7(x2
(6)
+ 2x) - 1; +
( e ) (g ~ f ) ( x =
) (7x - 1) +
2(7x - 1); ( f ) (fof)(x) = 7(7x - 1) - 1 2. (a)f ; ( b ) 1/6 3. ( a ) x = - 1; (6) x = 9 4. ( u ) 20; (6) 20; (c) 20;
(d) 20 5. no 6. ( a ) cont. on [a, c) U (c, d) U (d, e ) U ( 3 , f ) U ( J ; k]; (6) diff. everywhere except at b, c, d, e,J; h ; (c) discont. at c, d, e , f

Chapter 3
See. 3.1
1. f ( x ) = -2 3. g(x) = 3 . 9 + lox 5. f ( x ) = 0.01 7. t(x) = (x2 + 1)(21x2 + 6x + 1) + 2x(7x3 + 3x2 + x - 3)
9. f ( t ) =
+ 3t2)(8t + 1) - (4t2 + l ) ( 3 t 2 + 6t)
(t3
+ 1 11. h(s) = ___
-1 13. -4 = (4x3 + x ) ( - 1 4 ~ )- ( 7 ~ + 2)(12~ + 1)
+ 3t2)2 (t3 (s + dx (4x3 + x ) 2
112
15. f y x ) = + 3)x + 1)(2x) + (x2 + 2) I] - (x + 1 ) ( 9 + 2)2x 17. g(x) = 0 - l[(x3 + 3x2)(7) + (7x - 2)(3x2 + 6x)l
(x2 *

(x2 + 3) (x3 + 3X2)2(7X - 2)2

(@ - 9)[(Y2- 5!)(4Y3 + 2) + (y4 + 2y - 1)(23~- 5)] - ( y 2 - 5y)(y4 + 2y - 1 ) * 6


19. n(y) =
(6Y - 912
21. = - 4x-5 -
(x3 + 2x + 1)2x - + 2)
23. - = (x + l)[(x4 + x3 + 1)(6x - 7) +
dU
XZ(3X2
- 7x)(4x3 + 3 2 ) ] + (3x2
( x 3 + 2x + 1)2
7(X)
dx
A-6 ANSWERS TO ODD-NUMBERED EXERCISES/SELF-TESTS

(x dy = ( x 2 - 3)(2) - ( 2 x + 4)(2x)
+ x 3 + 1)(3x2 - 7 ~ ) ( 1 ) 25. - -
(x2 -x + 1)(6x2) - (2x3 + 1)(2x - 1)
dx ( x 2 - 3)2 (x2 - x + 1)2
0 - l(2v - 324 0 - 6(2u) 1
27. f ( o ) = +- 3 1. (a) 44; (b) y - 33 = 44(x - 2); (c) y - 33 = - -(x - 2 ) 33. for all x # 0, - 1
(v2 - u3)2 (W 44

39. f ( ~= )~[u(x)][u(x)] 41.


- 2u(x) -2(4x3 + 3x2 + 2 x )
35. (u) h(t) = 120 - 32t; ( b ) 321 ft; t = 15/2 = h(x) 43. f(X) =
(6) ~

14413 (x4 + x 3 + x 2 + 3)3

19. f ( z ) = 6(z4 - 32 - 2)5(4. - 3) 2 I. f ( y ) = ( y 3 + 1) * 7(5y2 - y + 2)6( lOy - 1) + (5y2 - y + 2)? - 4 ( y 3 + 1)3(3y2)


23. g ( x ) = ( x 2 + Z X ) / ~ ( - ~ / ~ ) (+X 5~~ ) - ~ / ( 5 x + 5) + ( x 5 + 5 ~ ) - / ~ ( 1 / 5 ) ( x+
~ 2x)-I5(2x + 2)
25. m(t) = ( t + 2)1/2(1/3)(t2 + t)-2/3(2t + 1) + (t2 + t)1/3(l/2)(t + 2)-1/2(1)
I2 (h + 1) 1 - h(1) * (3h2 + h)(3h2 + 1) - (h3 + h)(6h + 1)
27. f ( h ) =
-(-2 h + l . (h + 1)2 (3h2 + h)2
1
29. f ( t ) = -(t4 + 1)-(4t3) - ( t 3 + t2 + t + 1)-5/3(3t2 + 2t + 1)
2
(x4 + l)l3(l/2)(x5 + 4)-/2(5x4) - ( x 5 + 4)1/2(1/3)(x4 + 1)-2/3(4x3)
31. h(x) = 33. (a) dy/dx = -x/y; (6) &/dy = -y/x
~ ( + ~ 1)1/312
4

35. ( a ) dy/dx = ( 2 - ~ 3Y)/(3x + 3~); (b) dx/dy = ( 3 + ~ 3y2)/(2x - 3y) 37. ( a ) dy/dx = -xy2/xTy = -y/x; (6) dx/dy = -x/y
(1/2)y1/2x-1/2 + y-1 - 1 (x + 2)2 - 2 x y z
39. ( u ) dy/dx = - ; (b) dx/dy is l/(dy/dx) 41. ( a ) dy/dx =; (6) dx/dy is l/dy/dx
(1/2)x1/2y-1/2
, , -~xy-2 2yx2
43. dy/dx = dx/dy = -1 f o r y # -x 45. 1 = dy/dx 47. ( u ) y - 5 = -2(x - 5); ( b ) yes 49. dy/dx = 2/5 51. Au = 3, Ag = 33

See. 3.3

-3 3 - 1 - 10~4- ( ~4)
7. (u) dy = -dx;
x4
(b)
10
9. ( u ) df= 2(x - 4)1/2(x5 - 1)2 .(2t 3) dt; ( b ) d f 0.000000023
-- + ~ 1 1. ( a ) dy/dx = 5x/2y;
( b ) dX/dy = 2 y / 5 ~ 13. ( u ) dy/dx = - ( 3 9 + y 2 ) / ( 2 ~ -~ 3~); ( b ) dX/dy = - ( ~ x Y - 3y2)/(3xZ +y2)
15. ( u ) dy/dx = (5x4 + 3 ~ 3 ~ ) / ( 5 y- 3 x 5 7 ; (b) dx/dy = (5y - 3 ~ 5 ~ ) / ( 5 x+ 3 x 3 3 )
17. ( u ) dy/dx = -(3x2 +
+y1/2x-1/2 y2)/(2xy + +
i j ~ l / 2 y - ~ / ~( b) ); dx/dy = - ( 2 9 + + iy1/2x-1/2 +y2)

See. 3.4
1. f ( x ) = 3x2 + 2 x , f ( x ) = 6x + 2 , f ( x ) = 6 3. g(x) = -!k3, g( x) = 6x-, g(x) = - 2 4 ~ 5. ~ f ( t ) = ft-/,f(t) =
f(t) = 7. f(x) = 0 = f ( x ) =f(x) 9. h(r) = 5(r + 4)4, h(r) = 20(r + 4)3, h(r) = 60(r + 4)*
(3y + 1 ) . 1 - y . 3
11. m(y) = = (3y + 1)-2, m(y) = -2(3Y + 1)-3(3) = - 6 ( 3 +
1)-3, m(y) = 54(3y + 1)-4,
(3Y +
13. n(n - 1 ) . . . (n - k + 15. -n(-n - 1)(-n - 2 ) ~ - % - ~17. acc. = s ( t ) = 24t2 - 8 19. acc. = f ( t ) = 24t - 6
2 1. y = 2a; conc. up if a 0, con. down if a > 0 23. - 32 <

Review Exercises.
1. y(x)= -x(l - x2)-/ 3. g(t) = ( t - 5)11*4(ti 1)-l3(7t6) (ti + + +
1)1/3*11 * ( t - 5)O S . f ( x ) = -i(x + 4)-3/2
1 x-1-415 x + l - x + l
7. f(x) =
5 x + l
-(--) (x.( 1)2 + )
9. y = ( x 2 l)50. 10(x2 - 1)9 * 2x+ ( x 2 - 1)1 * 50(x2 + 1)49. 2x +
1 1. f(x) = 4(ux2 bx + +
C ) ~ ( ~ U . Y b) +
13. f(x) = & l + (3x - 5)6)-12. 6(3x - 5)5 * 3
+
15. h(x) = 4[(x3 4x - l)(x5 - x 2 x + +
2)I3[(x3 4x - 1)(5x4 - 2x + + +
1) (x5 - x 2 x + 2)(3x2 + 4)] +
+ +
17. p(s) = f ( s 4 s3 s2)-1/4(4s3 + +
3s2 2s) 19. t(x) = 0 2 1. g(z) = # ( l l z - 4 ~ ~ ) - ~ / ~-( 82) 11
23. t(w) =
(1 +
w2)(-1) - (1 - w)2w
+
25. gyy) = (y - 1 ) 1 / 2 . + ( ~ 1)-2/3 + (y + 1)1/3-4(y - i)-1/2
(1 w2) +
27. g(x) = ( x 2 + 1 ) 5 ( ~ 7+ x ) ~ U -7 ( ~ 4+ 1 ) 6 - 4 ~ 3+ ( x 2 + 1)5(x4 + l)i*10(x7 X ) ~ ( ~ X1)~ ( x 7 + x)1(x4 + +
l)?.5(x2 + +
l)4(2x) by Sec. 3.1, +
ANSWERS TO ODD-NUMBERED EXERCISES/SELF-TESTS A-7

Ex. 45 29. t(s) = + (s - 4)g*-(-)21 ss -+ 1 1-12 [s - (s1 --( s1)2+ 1)] 3 1. f ( x ) = 6x


*9(s - 4)8

33. m(t) = 71/72 35. g ( x ) = ( x + l)l0(x2 + 1)4(x3 + 1)3x2) + (x + l)(x3 + 1 ) 4 * 5 ( x 2 + l)4(2x) + (2+ 1)5(x3 + l)4-10(x+ 1)9
37. (a)y = (3x + 2y)/(y - x ) ; ( b ) y - 2 - 2 fi= (7 4fi) *(x - 1) 39. ( a ) dy/dx = -(sXy2 + 14x)/(5y4 + 6x5);
+

Z f i
-50\/3/10
(b)y - 1 =
17
(X - m)41. dy/dx = -(2xy2 + 3 ~ $ ~ ) / ( 2 x+3 3x9) 43. ( a ) dy/dx = (y2 -y)/-(x + 8y2);
(b)Y - (T) 6 - 1
=
44
~
2fi-3
( x - 4) 45. ( a ) dy/dx = -y3/x3; (b)y - 2 = 3*(x - 2/*) 47-55. In every case, dx/+ = l/dy/dx

67. f(31) z f ( 3 2 ) +
df= 2 - b = 1.9875 69. 0.94 7 1. 0.09833 73. 0.33 75. 195.8 gal 77. ( a ) vel. inc. if t 1; vel. dec. if <
t> 1; ( b ) moving away for 0 t < <
2 and t >
3 79. f(x) = 0 = f ( x ) 8 1. if x >
0 the slopes inc. and if x 0 the slopes dec. <
) 144 - 3 2 ; ( b ) t = 9/2 S , h(9/2) = 324 ft 85. t = 96
83. ( a ) ~ ( t =

Self-Test
+ + +
dy ( x 7 x6)(l/2)(x3 4)-/(3x2) - ( x 3 4)l/(7x6 6x7 ; ( b ) -
1. (a) - =
+
dy = ( x 2 1)4/5(8)(~
- +
dx (x7 x92 + dx
3y - 2xy2 +
d2y- - ( 2 x 5 - 3~ 2~)(3(dy/dx)- 4xy(dy/dx) - 2y2) - (3y - 2xy2)(2x2(dy/dx)
2. - + 4xy - 3 + 2(dy/dx)) 3. -0.012
2 x 3 - 3x + 2y dx2 ( 2 x 3 - 3x 2y2) +
4. ( a ) x = 2; (b) -4 5 . 360/t7

Chapter 4

See. 4.1
1. vert.: x = 2, obl.:y = x + 2 3. none 5. hor.:y = 3/4 7. note: no vert. a t x = - 1/3 as u only defined for 1x121 hor.:y = 1/3
9. vert.: x = -1, hor.:y = 1 11. vert.: x = 2, obl.:y = x +
2 13. vert.: x = -9/2, obl.:y = x / 2 - 7/4 15. vert.: x = 0, hor.:y = 0
17. hor.:y = 0 19. vert.: t = 1/2 21. vert.: x = -1, obl.:y = x - 2 23. vert.: x = (3 i- a ) / 2 25. hor.:y = 1

See. 4.2

(6) C.U. on ( x 2 , xJ, (x4, a), where x5 a < <


x6, ( x 7 , x9); c.d. on (x,,, x z ) , ( u , x 7 ) ; (c) rel. max. a t x = x I , x4, x6, rel. min. at x = x3, x5, x s ; (d) all
but xq, ( e ) at x = x2, x 7 ; ( A a t x = a bet. x 5 and x6 5. ( a ) ( - 0 0 , -2), (1, + 0 0 ) ; ( b ) (-2, 1); (c) x = -2, l;f(-2) = 16,f(l) = -11
7. ( a ) inc. for all x ; ( 6 ) none 9. ( a ) (-4,0), (4. + w ) ; ( b ) ( - 0 0 , -4), (0,4); ( 6 ) rel. min. x = *4,f(?4) = 0; A. max. x = 0 , f ( 0 ) = 256

3
13. c.d. everywhere 15. c.d. for x < 3;
C.U. for x > 3; p.i. x = 3
See. 4.3
I.

13. 15. 17.


t

i-:Ij
A-8 ANSWERS TO ODD-NUMBERED EXERCISES/SELF-TESTS

31. t 33. +y

27-+ 29-

x = -1 I x = l

See. 4.4
1. no deriv. at x = 1 3. no deriv. at x = 0 5. no deriv. at x = 0 9. c = kfi 11. c = 1 13. c = &1/(3fi) 15. c =0
17. c can be any value in the interval since tangent line = graph

Review Exercises
1. l ; h o r . a s y . y = l 3. + w ; v e r t . a s y . x = O 5. + w ; v e r t . a s y . x = 2 7. - w ; o b l . a s y . y = j x - j 9.0;noasy. 11.incr.
(-W, 1/2); decr. (1/2, + w ) ; rel. max. x = 1/2 13. decr. for all x 15. inc. for all t 17. C.U. (-a, 1); c.d. (1, + w ) ; p.i. x = 1
19. c.d. (-1 fi,l / f i ) ; C.U. elsewhere; p i . x = % l / f i 21. (-6, 106)

( 2 / 3 , - 16/27)
ANSWERS TO ODD-NUMBERED EXERCISES/SELF-TESTS
A-9

33. 35.

+ x = -1:
I

I
tY

f i x

43. 45. i 47.

A
41. fY
x = -1 I $

-1 -0.4
+
X

49. 51.
f 53-
4

L,
/I
X
I c
Self-Test
2. Y
I ti
*X

-(-1 - fi,- 2 f i - 4)

4. 4.J 5. c = -I +- fi

Chapter 5

See. 5.1
120 + 7 2 6
1. ( a ) max. 0; min. -3; ( b ) max. 0; no min. 3. max. 64/3; min. -7/6 5. no max.; min.f(2 + 2 6)
= 7. max. 1400;
2 6
A-I0 ANSWERS TO ODD-NUMBERED EXERClSES/SELF-TESTS

= -9/2 11. 10 x 10 x 5 13. 1 x 1 x 1 15. the ratio of side cost to bottom cost is

lo 4w
17. t = 32/25 h after midnight 19. - 10 f i m from furnace 2 1. use -for square,
2/k (a) ~

1 +w m from furnace; ( b ) 20
T+4
2R
?Tw
___ for circle 23. 2 x 2 25. ( a ) 10 x 30 where 10 m is length of separators; (b) 8 x 28; (c) if cost > $27.50 m 27. width = -,
v + 4 fifi fi
75 1/3
depth = ~ fiR 29. cquilatcral of side f i R 3 1. (u) 7 = (%) =: 1.5m, h = -
'O0 - '7r 'I3=. 12 m; ( b ) h/D = 2(k - 1) 33. T =
fi 7T (75)
1/3
35. 7 = h = 4 37. 1 = 8 f i 3B.r=20fi,h=40 41. 1 m i d o w n r i v e r f r o m A
h = 2(+)

Sec. 5.2
1. dh/dt = 1/v 3. 16/(49v) m/min; (b) it is dec. at - 8 / ( 7 ~ ) m/min 5. 63/29 m/s 7. ( u ) 5 mph; ( b ) yes 9. 0.3 units/s
(a)

1 1. 4/3 psi/s 13. ( a ) dy/dt = 10; ( b ) dy/dt = 200; (c) ds/dt =


1 2x2 +
15. ( a ) dy/dt = 30; ( b ) dy/dt = 6 x ;
~

m
(6) ds/dt = 3(1 + + + +
2x2)(1 x')-~/' 17. ( a ) dy/dt = lOu(t); (6) dy/dt = 2xv(t); (c) ds/dt = v ( t ) ( l 2x2)(1 x 2 ) - l I 2 ; ( d ) the Chain Rule
followed by substitution of v ( t ) for dx/dt 19. dx/dt = 1/(2x - 10) 21. ( a ) dx/dt = 1/5; ( b ) dx/dt = l / m 23. dD/dt = - 1/(64v) cm/min

See. 5.3
<
1. for t -2, the particle P moves right, slowing down toward 0 speed at t = -2; for -2 t < <
1, P moves left, speeding up until t = 1,
then for 1 t < < 4, P slows down to 0 speed at t = 4, for t >
4, P moves right increasing speed 3. for t <
-2, P moves left from s = 0, inc.
speed until t = - 2 fi,then dec. speed, stopping at t = -2; then P moves right, inc. speed until t = 0, then dec. speed, stopping at t = 2;
then P moves left, inc. speed until t = 2 fi,then dec. speed, still moving left but not passing s = 0 5. moving left and slowing for t -2; <
stop at t = -2; moves right and speeds up for -2 <t < -
- fi, continues right and slows for
-1 - 6< t < -112; stop at
2 2
t = - 1/2; moves left and speeds up for - 112 <t < - fi,
i-
+ fi < t < 1; stop at t = 1; moves right
continues left and slows for
-1
2 2
and continues speeding up for t > 1 7. moving left and slowing t < fi;stop at t = - fi;moves right and speeds up
- fi< t < - 1, -
continues right and slows for 1 < t < 0; stop at t = 0; moves left and speeds for 0 < t < 1, continues left and slows for 1 < t < fi;stop at
-
t = fi;moves right and continues speeding up for t > <
fi 8. P dec. speed for t 1, stops at t = 1, then inc. speed as it nioves to the left
for t >
1 1 1. P move3 from infinite ypeed at t = - 3 to the right, slowing to a stop at t = 0, then inc. to infinite qpeed at t = 3 as it moves
left 13. AY

r
X

See. 5.4
1. ( a ) R ( x ) = 1 2 ~(6) ; P ( x ) = 50 + 1 2 . 3 ~- 0.001x3; (c) x =. 64,P(64)= $575.056; ( d ) no max. 3. ( a ) x = 300, R(300) = $22,500
5. ( a ) 170; ( b ) P ( x ) = 1800 + 20x - x2; (c) 170; ( d ) same 7. ( a ) 170; ( b ) 7225 9. ( 6 ) x = 2000
11. r = d(b + A)(b + A + a - B ) - ( b + A) 13. ( a ) 4B/day; ( b ) 1.2Q/day

Review Exercises
1. 29/6 min., 109/3 max. 3. 55/3 max., -203.5 min. 5. -27/32 min., 13.5 max. 7. no max. or min. 9. - 1/4 min., no max.
11. min. at x = 1, m a . at x = 2 13. min. at x = 0, max. at x = -5 15. min. at x = -1, max. at x = 2 17. min. at x = 0, max. at
x = 4 19. max. at x = 1/3, no min. 21. x = 4 N , y = 2 +
23. 1 = 2(9lj3 1)3'2m 25. 3 p.m. 27. ( a ) right; ( b ) slowing
31. =: -0.013 ft/min 33. -3.6ft/s 37. dh/dt = 1/(108?r) m/s 39. dh/dt = 3/(411) m/min 41. the particle moves left, slowing down
until t =.2, stopping at s(2) = 0, then moves right, inc. speed

Self-Test
1. (a) max. = 23, min. = -17; ( b ) max. = 516, min. = 1/2
ANSWERS TO ODD-NUMBERED EXERCISES/SELF-TESTS Ail 1

2.
t = 6+ Sl(zwinr
-- SDeedine
Speeding - 3. 2/9 ft/min 4. (a) 24,250; (b) 8000 at $83 each (6) 2236 5. 4 X 4 x2
- t =1/2 Slowing t = l

-
-109 -46.5 16

Chapter 6

Sec. 6.1

See. 6.2
1. ( a ) $; ( b ) f ; (6) 4.0125; ( d ) 1/2 3. ( a ) 7/4; (b) 9/4; (c) 2.01875; ( d ) 1/4 5. ( a ) 24.375; (b) 40.125; ( 6 ) 36.25; ( d ) 2 7. ( a ) 31.4375;
( h ) 114.8125; (c) 56.390625; ( d ) 1.5 9. ( a) Q;(6) (6) #; ( d ) 2 9;
11. all equal to h(d - 6 ) 13. 1300 15. 423 17. 2
19. 4 n 2 + 1 2 n + 1 1 21. 4 23. f 25. Q 27. 29. Q 31. 3 35. 6 37. 256/3
3n2

See. 6.3
- 33
1. 4 3. 675 5. ~

8
7. 1/3 9. 9(IO3/' - 1) 11. 2 13. 38 19. 288 ft (J:o(t) dt gives vert. height at t = 6, not total dist.)

2 1. S , " f ( x )dx = total profit for first h items 23. 4x3 - 6x +1

See. 6.4
1. (6) x = 5 3. ( a ) 121.75; (b) x
( a ) 15; 4.9 5. ( a ) (6) x = 1/2 7. ( a ) 16; (6) x = 2 2 / * 4;
9. ( a ) - f ; ( b ) x = & 2 / f i 11.
3 . 2113
1 3 . 0 15. --
2
17. 6.85 thousand gal 19. [ ( 3 x 2 1)2 2(3x2 1)]6x 21. F'(x) = 2x + + +
23. - ( 4 2 + 2)l/2(2) + (x4 + 2)'I2(2x) 25. 0 27. 0 29. u = x 2 - 5x 10 implies 4 2 ) = 4 = 4 3 ) +
See. 6.5

1. 13& 3. 1/12 5. 72 7. J3,[0 -(x' - 2~ - 3)]d=


~ 32/3 9. 7/15 11. 68 +

3
fi 13. R unbounded
1 5 . Jll[(y + 1) - (2y2 + y - 1)]4= 8/3 17. 1/2 19. 2 21. f' 23. 4/3

2(9 fi- 1)
25. JA[(-x3") - (2x - 3 ) ] d x = 8/5 27. 1/2 29. 8/3 31. 33. (a) 1000 35. no 37. 18 39. $200.99
5
4 1 . CS = i;
P.S. = 12

See. 6.6
271 223277 5. 7. B. 377 2 777 45977 377 2177
1. (a) - 3. (a) 11. - 13. - 15. 17. - 19. 3 1 ~ / 3 0 21. 218771 23. -
~

7 5
~ ~

15 5 3 3 3 8 2 5 10
1677 6477
25. - 27. 471 13557 31. 1677 33. ~J:z[(l
29. __ - ~ (4
x ) - - 3x - ~ ]+ rJL4(xZ + 3x - 4 ) " ~
x ~ ) dx 35. -
15 6 3
477
37. 4 0 ~ f i ( 9- ~ ~ ) ' ' ~ d y 39. -(a2
3
- r2)3/2 41. 38071.
3
43. 13671
A112 ANSWERS TO ODD-NUMBERED EXERCISES/SELF-TESTS

Review Exercises
1. 4x1l2 + x4/4 - 3x4j3/4 + C 3. - x - l - ~ - ~ /+22 ~ - ~ / ~+/ C7 5. t3 + 5t2/2 - 2t + C 7. 2 ~ ~ /+
~ /4t3l2/3
7 - 6z1I2 + C
9. &(x2 + 2)53 + C 11. -(3t + 5)-6 + C 13. (T + l)3t2/2 + C 15. ( x - 2)37/37 + C 17. --
7 +Y+
12
3 - C

19. 2(-+ 2)9/9 +C 21. (3x - 1)5/15 + C 23. 3x14/3/14 + 6x13/6/13 + C 25. 3(ax + b)43 + c 27. i ( t 2 + 21413 + c
4a
29. h ( 4 ~ 3- 516 + c 31. g t 3 - t 2 + 1 y 5 + c 33. + 1/x)3/2 + c 35. ~3 + c 37. 3.4 + 1zx3 - 5 y 3 + c
+
39. x3/3 x 2 / 2 + x + 3x-/2 +C 41. ( b ) 65.5; (c) Sp = 44; Sp = 75; ( d ) 59 + 1/6 43. f 45. 1/2 47. 68 49. 8~ 51. 264
53. 2 55. 4(233/2 - 73/2) 57. 10/3 59. 13/2 61. 0 63. x3 + -
65. y = - x2
+ 2x + 1 67. J = (2 + 2~ + 4) + 4
3 2
4x512
x4
6 9 . y = - + -- 208x ~ + 206.8 71. J ( X ) x4
= 3x2 andf(x) = - + x - -1 73. (u) 20/3 cm; ( b ) 15.75 cm; (c) 21 + 1/3 cm;
4 15 3 4 2
+ 5/12cm + l)s 79. 0 81. 2s 3s2
( d ) 10 75. h = 16t2 77. -(x3 83. 0 85. ~

m+m 87. (a) 5;


( b ) x = *$f 89. (a) 6; ( b ) t = 91. & + 1024 93. 26 + 5/6 95. 4/3 97. 32/3 99. (a) Jt(1 - x 2 ) dx; ( b ) J t x 3 d x ;
( c ) J i x 2 d x ;(d)JA(l - x 3 ) d x ; ( e ) J t ( x 2 - x 3 ) d x 101. ~ J t x d x 103. ~J;y/dy 105. ~ J t x ~ d 107.
x ~JAydy 109. ~ J i ( 1- x 6 ) d x
111. 2?rJ&.x2dx 113. 2nJ;(1 - x ) ( l - x 2 ) d x 115. 2 ~ J & ~ y-( ~l / ~ ) d y 117. 2~J;x(l - x 3 ) d x 119. 2~J;(1 - x ) ( x 2 - x 3 ) d x
121.8671 123. * ( 2 -
2 7 ~
fi) 125. -
15 3 2

Self -Test

1. (a) (x
3
- ): 3/2
+ C; (b) z( fi + 1)85 + C 2. (a) fl-fi;( b ) 0 3. 37/12 4. (u)
1
+ 2s)
5. (a) c f t ; (b) -4 a f t / s 6. (a) 21; ( b ) x =
-1 + - 2.27 7. ( u ) 33317;( b ) 3 6 ~ (G)
; 7 2 ~
4 3 5

Chapter 7

See. 7.1

17. 36,000deg 19. 0 21. - f i / 2 23. - $ f / 2 25. -2 27. ~ fi-


+ 1
29. 1

See. 7.2
1. -sin(x - 2) 3. g ( x ) = 2 sec2 ( 2 x - 1) 5. -6 cos(3x - 2)[csc2 (3x - 2)] 7. cost - sin t
9. sinx[? cot(3x)(-csc2 (3x))(3)] + cot2 (3x)(cosx) 11.
X2
] 13. ab( -sin2 0 + cos2 8)
15. 2 sec x(sec x tan x) 17. 0 19. cos(cos(3x)). (-sin 3x)(3) 2 1. 5 tan4 ( x 2 + I)(sec (x2 + 1))(2x)
23. d s i n 4x + 1 *i ~ X ) (3x cot 3x) * 3 + -*&sin
C S C - ~ / ~ (-csc 4x + l)-1/2(cos 4x)(4)

25.
+
tan(2x - 1) sin4(3x 2)cos(3x + 2) .(3) - sin5(3x + 2)sec2(2x - 1)(2)
tan2(2x 1) ~

csc2(4x - 1)$ sec2 x - G ( 2 csc(4x - l))(-csc(4x - l)cot(4x - 1))(4)


27.
csc4(4x - 1)
29. x cos(l/x)(- 1/x2) + sin(l/x)
sin x cos x cos2y cos(2x)
3 1. -sin x(dx/dt) + 2t cos t + 2 sin t 33. x cos 0(d8/dt) + sin 0(dx/dt)
35-
37.
(4 +
sin 2x)1/2
39. 1 43. n seen x tan x

45. (a) 1 49. A cosx 51. Aa8sinax 55. 0 57. 0 59. 2 61. L s e c z d e g ( x )
180
ANSWERS TO ODD-NUMBERED EXERCISES/SELF-TESTS A-13

see. 7.3
1. f-l(x) = x +1 3. f y x ) = x 5 - 1 5. f ' ( x ) = ~ "-'
3
7.8 9. - 3 11.2 13. -5 15.40 17. 1/4 19.x=3

2 1 . ~ ~ 2 5 3 . ~ ~2 51. x = 4 27.6=8 29.x=2or3 31.0 33.

(0,

See. 7.4

[ h ( x - 1)](3x2 + 1) [ln(x3 + x)] 1


-
11. ex'I'-[
(5x +
1)
+ ln(5x + l)eS2(2x) 13. (4x3 + 2x - 3)3e3" - e3"(12x2 + 2)
(4x3 + 2x - 3 ) 2
15.
ln2(x - 1)
x3
(x - 1) +x +3~2~eln2
1
17. * 2 sin(5x2 - 7x)cos(5x2 - 7x)(lOx - 7) t - sin t ) 2 1. sec x 23. e(ee)r
19. e(sint+cost)(cos
sin2(5x2 - 7 x )
25. ezeosr( -x sin x +cos x) 27. p / z 29. 2 sin(eD)cos(e2)e5 3 1. sec(e2")tan(e2") * 2eZx 33. ecasZt(2 cos t sin t ) 35. - I(ln X ) - ~ ( ~ / X )
,x - $$r+Y) l n x y
37. 2"(ln 2)n 39. -(99!)~-~"O 4 1. 43. - Y 45. --*- 47. at x = 1 49.
e,~+~) X x lny

See. 7.5

+ 8x
x-5
3. x(x + l)r-l + (x + 1)" ln(x + 1) [
5. k(x) 3x: - 1 I
qX -
5x4
2(x5 + 4x2) +
4x3+4
5(x4 + 4x + 4) 1
4
2 ( 4 ~+ 1) +
21x6
3(3x7 - 2~
-

+ 1) 1 9. f(x)[x' + 8x7 111x1 11. g(x)[(lnx)(l/x) + ln(2x)/(x - 5 ) ]


13. g(x)[2 + 21nxI 15. 2x2"(1

35. no ext. values 37. 2 k r < x < ( 2 k + l ) ~k, integer 39. (u) 15 sec3(5es + l)tan(5es + l)es d0; (b) +
24e3(t3+t4f2+2[(t5 t4)(5t4 + 4t4) dt]

Review Exercises
1. i/x+? 3 . f ( O ) = -1 =f(-3/4) 5. 32.5 7. 4th 9. 11/2 11. 6/7 13. 2 15. 1 + logb(logbx) 17. x = 2 19. x = 2
A-I4 ANSWERS TO ODD-NUMBERED EXERCISES/SELF-TESTS

(2x
- 3)
21. x = 2 23. ( 2 x + 2)erZ+2r+1 -
x2 - 3x - 5
25. e Z " - 6 . ( c ~ ~ 2 \ / X - - I ) ( ~ ) ( x- 1)-1/2 + (col\/x--1)(3x2 - 6)eZ3-@

27. etm& 3 cos2 x( -sin x ) + C O S ~xetao&(sec2 4x)4 29.


cot3(x2 + gx)[xe.' + eZ] - xez: 3 cot2(xZ + 9 x ) * [-cscz(x2 + 9 x ) ] ( 2 x + 9)
cot6(x* 9x) +
31. f c s ~ ~ / ~ ( e ~ ) [ - c s c ( e ~ ) c o33. ) ] e ~ - x ) [ - c s c 2 ( x 3 - x2)](3x2 - 2x)
t ( e ~ tan(.* + cot(x3 - x2)[sec2(x2- x)](2x - 1)
35. xez+7 + ex+7 - 12x3 37. 5r2+'+1(2~+ 1) In 5 + F 3In 2

(x5 + 4x4 - 6)(3x3 + 2 ) - (5x4 + 16x3) ln(x3 + 2x - 4)


+ zx - 4) (x3 + 2x - 4)
43. -[8 11 q x 3

x5 + 4x4 - 6 1 (x5 + 4x4 - 6)2


45. ( u ) 5 sin4(cosx)[cos(cosx ) ] ( -sin x ) ;

( b ) [cos(coss x ) ] 5 C O S x(-sinx)
~ 47. e'og3(cost) ,(1 l)(&) (-sin t ) 49. 4eCSC2t(2
csc t)(-csc t cot t ) + 5 cot4(t4+ 1)(-csc2(t4 + 1))(4t3)
5 1. 6(x) [-4x - 3 + 4 In x ] 53. $[ln(sin4u ) + u2]-1/2 sin3 u cos u + Zu] 55. -csc x

I.-.- 9 1
2cos(x + 1) - (9x 1) In2
- + 7.-] 2X
[--sin x
59. r ( x ) cos2 x -sinxIn(sinx)] 6 1 x + [(lnx)(-csc2x)
.cot~ ~
3 sin(x + 1) 3 log2(9x - 1) xz - 6

63. $ ( e Z - x2)-1/2(es - 2 x ) 65. 5eSin("2)cos(x2)(2x) 67. -


+
(sec x)(tan x ) y cos xy -1 .ex dy
69. - 71. - =
ysinx - siny
x cos xy ; (6) ey dx xcosy + cosx
ZX-y In 2 - 1OE+YIn 10 1 + csc(x +y)cot(x + y ) + sec(x -y)tan(x - y )
73. 75. - 77.
2"-Y In 2 + In 10 csc(x +y)cot(x + y ) sec(x -y)tan(x - y ) +

79.
ti 81. 83. 4y 85.

87. -._._
1
89. 0 91. -(x9
1
+ 2x)?-1(9xs + 2 ) 97. ( u ) - z r n i / s ; (6) -7r/60 mi/s 99. ( u ) et + e-' 101. Ay =. 4 = 0.01
ln(ln(x)) Inx x 45

Self -Test
(sec3 x) 4 sec* 4x etan4' - etan4' .3 sec3 x tan x 1 1
1- (a) (6) -*-. (6) 5 sin4(cos(2x - 1))cos(cos(2x - I))(-sin(Zx - 1))2;
sec6 x Inx x '

(4 74x1 -[3(x22x+ 1) +-x 12 4 + 9(3x2


5(x3 +
-
+ 2)
2x)
4(x
1
+ I) -1;cos x
(el O ; (f)?'-'
(x + I )
2x 1
1n2'
(9)- 2 see x csc x ; ( h ) 2 sec2 x tan x

-3 7r
3.- 4.x=-
xey 4
ANSWERS TO ODD-NUMBERED EXERCISES/SELF-TESTS
A-I5

Chapter 8

Sec. 8.1

1. = In 2
3. ( u ) t = 10 5. (u) t
20 In( 1/3)
= _ _ _ ; (6) 31.7 yr;
ln(1/2)
(6) yes. Z(31.7) yr 7. t = ~

ln(6/5)
9. r~32,oOOyr 1 1. 10% compounded

-
annually 13. i 13%; (6) i 1 5 . 1 % ; (c) the bank 15. 1 3 2 2 million; ( b ) -59.44 yr dv
(u) (u) 17. i 2 4.3% 2 1. (u) - = 32 - kv;
dt

(b) u = + -,32 where A = --


; (6) vo = A + -;32 32 ( e ) as k increases, terminal speed dec.
( d ) u + -; 25. t 161 min 27. no-
k k k k
the income is below the asking price 29. yes

See. 8.2

1. 2 tan(x+ 1)ll2 + C 3. $In 3 5. jln(x2 + 1) + C 7. In262 + c 9. 2(1n + c 1 1. +ex2+2x + C


~ * x)1/2

13. -+lnlcscx3 + cotx31 + C 15. h(ln(4x4 + 5))2 + C 17. e - 1 19. -4(e-l - 1) 21. - ,-1 23. - 2 , 6 + c 62

25. In[+] 27. L 3 z 2 + 1 . L + C 29. - 2 csc(& + 1) + C 3 1- -e-' + C 33. 2 + 4x2 + 41x + 204 lnlx - 51 + C
1 + e- 2 In 3 3

35. -
2
X2
+ x + 2ln12x + 11 + C 37. In - ln(1/2) 39. n(e - 1) 41. e - 1 47. 2
2
+C

See. 8.3

1. -cosx
cos3 x
+- 3
+C 3. &x + Asin 2x - & sin 4x + +C
+ 2 csc4 8 - 4cscz 8 - lnlsin 81 + C
cos3 2x 5. - Q cscF8
7. x + $tan(2x - 5) + C 9. -4 + l n f i 11. $sin4x - &sin6x+ C
13. --Qcot6x + C 15. -&cot6x - i c o t S x + C
17. - 3 C081/3 u + 4 ~ 0 8 u~ +' C
~ 19. & sec5 8 - 4 sec3 6' + C 2 1. $ex - 4 sin 2e' + C 23. 3 sec7/3 x - 3 sec1/3 x + C
3 1
5
25. -(a/8) - - + -.3 1 + 1(
*- L r - 1 27. 0 29. - + In
1
* 3 1. an2 33. 4 35.
1 1
cos 3x - -cos 7x + C
16 16 fi 64. 48 fi 2 6 14
37. i x - sin lox - 4 cos 2x - gcos 8 x + ax + &sin 6x + C 39. 4 sin 2x + Asin 12x + C
&j

See. 8.4
1
1. - ~ / 3 3. n/3 5. 3/2 7. ~ / 49. ~ / 611- -2 13. - 1 15. 7/6 17. 2 ~ / 3 19. 1 21.
2 did=
-3 -1
23. X
2 5 - ___ 27. .2t + arccos(tZ+ 1)
(x2 -1 ) d n x2 + 9 1- (t2 + 1)2
(1 + 9x2) 1 +39x2 - 18x arctan 3 x
-.-
~

-1 1 ex
31.
1x2 - 11 d(x2 - 1)2 - 1
* 2 x 33.
1 + In2 u u
35. e z . -
+
1 e2'
+ e2 arctan es 37.
(1 + 9x2)2

See. 8.5

9.
7
* 4:arctan ($)11. $ 4:-

arcsec (?)
f i x
+C 13. eTI4 - 1 15. -
245
17. L a r c t a n (2+ sinx)] + C
[L 19. arctan(e") + C 21. -arcsin
1
+C 23. a/6 25. 2 arctan(x1l2) + C
m a 2 fi
+ C 29. -arctan(csc x) + C 35. arcsin(3/4) 37. s arcsin(&
27. arcsec[(2~)~'~]
A-I6 ANSWERS TO ODD-NUMBERED EXERCISES/SELF-TESTS

Sec. 8.6

1 sinh x = 1/2, cosh x = $6, 1 2


tanh x = -, coth x = 6,
sech x = 3. 3 sinh2(x4)cosh(x4) 4x3
6 &
~

5. $(l + 2 sinh x cosh x ) - l / , [ 2 sinh2 x + 2 coshZx ] 7. (sech2(1/x))(- 1/x2) 9. sech2 +

tanh x - csch x
'Oth

arccos x cosh x - sinh x


1 1. XS'"hX [(sinh x)(l/x) + (In x ) cosh x ] 13. ~
excosh ex
15. t2(sinh 2t)2 + 2t cosh 2t 17.
sinh ex (arccos x ) ~
1
19.
\/1 - sinh2x
Vcoshx 39. 0 41. lim,,,+ coth x = + 00; limS+,- coth x = - 00 43. - 2 c s c h f i +C 45. 2(sinh 1 + 8sinh3 1)

ds 1 \/32ksinh( \/32kt)
(c) with a parachute; ( d ) 32 sech2(a t )

Review Exercises
1. -4Gyr 3. (u) 80; (6) 40; (c) 640 5. undefined 7. n/4 9. n / 4 11. -1.46 13. n/4 15. 0 17. -arcsin(n/4) 19. n/4
1 1 1 1 1
25. -
2 1. -77/4 23. -(arctan
2
x)-'/~
(-
l + x
2)
l + x
(;x-'/') 27. -csc2(ln(arcsecx))*-*
arcsec x 1x1-
/
I
1
-In 2
29. x 2 * 2 + 2xarctan2x 31.
1 4x2 + 1 + (log, x)2
33.
d l - cos6x
-3coszx(-sinx) 35.
1
1
+ (lnx)2
.-x1
+ 1)cosx
37. -*-
a2
1
+ x2 1 + x2
+ (arctanx) 39. 1
*[
-sin2x - (cosx
sin2x
] 41.
I ( -csc x cot x )
lcsc X I d&CT

43. _-.a-
1 1/2 1 (1 + sinh x ) sinh x - cosh x cosh x 47. sinh(1n see x ) *-.
1
sec x tan x
a (1 + sinh x ) ~ sec x

49- xtanhr [(tanh x ) (+ + (In x))sech2 x] 5 1. (1 - cosh 2x)2 sinh 2x - (cosh 2 x ) ( -2 sinh 2 x )
53. _L 1 - tanhx
(1 - cosh 2 ~ ) ~ [ +
2 1 tanh x (1 2-sech
tanh2 xx ) ~ 1
55. 3 Inlsec(1 + x1/3) + tan(1 + x1l3)1 + C 57. Aln/3x + 91 + C 59. -4(3x + 9)-l + C 61. 5 - -7x + -1nj3x
28 + 91 + C 63. 4
2 3 3
6 5 . 2 fi 67. 4 sin 3x - 4 sin3 3 x +C 69. j ( 1 + tan x ) 2 / 3 + C 7 1. ~
77r2+3
+C 73. $lnlcscx2 - cot x 2 / +C 75. 7 / 2
In n
2.4G1
77. $eX3+l +C 79. $(ln(3x3 +9 ~ ~+ )C )8 1. ~ ~ + In x)-2
~ -J(l +C 83. ~ +C 85.0
In 4
87. + 1) - sin ( x 2 + I) + C 89. - --
1
sin x
sin x +C 9 1. -+(& cos6 3x - Q cos8 3x1 +c 93. + sin3/2 x - 5 sin7/2 x + c

105. -- 5
fi
fiarcsec( fix) + C, if x > 0 and negative of this if x < 0 107. - 1 [arctan
2 f i
(5) - arctan

1.9. -arcsec
2
(x-3) + C 1 1 1. 4 a r ~ t a n ( e ~+~C) 113. - f arccosh(c2") +C 1 15. arcsin(l/ fi) 1 17. lnlarctan XI +C
1 1 9 . -In/
1
1
+ (1 - x)1/2
- (1 - x)1/2
I+ C 121. 2 arctan(x*/') +C 123. arcsin(4sin 7) - arcsin(Jsin5) 125. -arcsin (y)
+ C

+ C 129. In(cosh x ) + C 13 1. $ sinh 4x + & sinh3 4x + C 133. 4tanh t + C 135. -


127. arctan(sec x ) 1 5/3 +
In ~

18 1 1 / 3 + : l + c
137. iarcsin(5x - 1) + C 139. arcsinhx + C 141. arcsinh(x - 3) + C 143. J(sinh(-2) - sinh(-4))
145. In(cosh)(l) - $ tanh(1) 147. y = sinh kx, y = cosh kx 377
149. -
4
ANSWERS TO ODD-NUMBERED EXERCISES/SELF-TESTS A-I7

Self-Test

1. (a) ~ $ 6 0 . 6 5 ;( b ) ~ $ 7 7 . 8 8 2.
dt dt
3. +C 4. &x2 + In/x + 11 + C 5. j(1n s e c ~ ) +
~ 'C~

1
6. 2 Inlsec(3 + $)I + C 7. -Jcos5 2 x ( $ - -$cos22 x ) +C 8. 4 tan4x + C 9. $(arctan x)2 +C 10. -*L
arcsec
3 v 3
1 .--1
11. -1cothx +C 12. $Inl(x + 4) + [(x - $)2 - 1]1'21 + C 13. 2[arcsin(l/x)]-
(1 - (l/X)2)1'2 x2
(1 - tanh x) sinh x - cosh x ( -sech2 x)
14.
(1 - tanh x ) ~

Chapter 10
See. 10.1
1. V = ( O , O ) , F = ( 5 , O ) , d : x = - 5 3. V = ( O , O ) , F = ( - a , O ) , d : n = ) i 5. V = ( O , O ) , F = ( O , 2 O ) , d y = -20 7. V = ( O , O ) , F = ( i , O ) ,
d x = -~9. V = ( - 5 , -8), F = ( - 5 , -?), d? = - 9 1 1 - V = ( - $ j , y ) , F = ( g , y ) , d : y = - # 1 3 . y 2 = 1 6 x 15.y2=40x
17. X 2 = - 3 5 19. X' = -89 21.y' = f x 23. ( X - 5)' = 8 ( y - 1) 25. (x - 1)2 = 8(y - 4) 27. ( x - 4)2 = - 8 ( y - 6 )
29. ( y - 4)' = - 7 2 ( x + 6 )

See. 10.2

1. ellipse, C = (1, 3 ) ; foci (-3, 3), (5, 3 ) ; vertices (-4, 3), (6, 3) 3. ellipse, C = (-2, 4); foci vertices (-2 k-
5

5. ellipse, c = (o,o); foci (0, k fi);


vertices ( 0 , 2 2 1 7. ellipse, c = (- 1,1); foci - 1 , l
6
k-); vertices (- 1, +), (-1, -+)
2

9. ellipse, C = (-4, 1); foci (-4, 1 2 7 \/2/3); vertices ( - 4 , 8 ) , (-4, -6) Z 1. hyperbola, C = (2,4); foci
5
vertices (2 k 5 , 4 ) 13. hyperbola, C = (1, 2); foci (1, 2 -C fi);
vertices (1, 3 ) , (1, 1) 15. circle, C = ( - 5 , 3 ) , r = 2 17. -
25
x2
+-
16
y2
= 1

19. (X - 4)'
(y - 6)' 21.--- (x - 3)2
(x - 1)2
-+-=l (Y - 3)2 = 1 23. ( y - 4)2 = 1
8 4 4 4 3

See. 10.3
-7rX
1. parabola, V = (0, 0), F = (1, l), axisy = x 3. hyperbola, center ( O , O ) , major axisy = ~ 5. hyperbola, center (0,0), major axis
6

y = 2 7. two straight lines parallel t o y = 2x 9. hyperbola, center (0, 0), major axisy = - ( g r 2 x 11. hyperbola, center (0, 0),
2
major axisy =
5
13. ellipse, center (O,O), major axisy = - x 15. ellipse, center (2, 2 ) , major axisy = -x +4 17. two vertical lines

x = k 3 19. ellipse, center (0, 0), major axisy = - x

Review Exercises
A-I8 ANSWERS TO ODD-NUMBERED EXERCISES/SELF-TESTS

21. 23.

25.
! t 27- -t
I
*
29. 31.

41. x 2 = 1 6 ~43. (X
(X - 3 ) 2
- 1) = 4y 45. ___ +- ( y - 7)2 = 1 47. ~
(x - 3)2
+-=( y + 1)Z y2
1 49. - - - =
xz
1
9 4 25 16 144 25

4) = S(X + 4) 2. ( a )

(-
ANSWERS TO ODD-NUMBERED EXERCISES/SELF- A-19

Chapter I 1
see. 11.1
1. -x cos x + sin x + C 5. -
3. 2 arctan(2) "' -
In 2
-
(In 2)2
f + $In($)
+ c 7. f[cos(ln x + sin)~nx)] + c 9. 3 lnjxl - -
~ '" x3
9
+c 2
ll.z-Lln(2+ f i ) - T +fin
- l n ( f i 1+ l ) 1 3 . ~ a r c t a n x + -1a r c t a n x - ~ + C 1 5 . B e 2 2 ( s i n x + 2 c o s x ) + C 1 7 . 4
6 4 4 2 2 2

19. he4$ sin(x/2) + ge4" cos(x/2) +


C 2 1. i(1 - e2) 23. 2x1l2 In x - 4x"' C 25. 3
tan 2x - 3x2 - - In/sec 2x1 + - + C
3x2
+ 2
2 4 2
27. x coshx - sinh x + C 29. x arcsinh x - d m + C 3 1. x sin2x + 4.x cos 2x - isin 2x + C
33. Qsin 3x cos x - $ cos 3x sin x + C 35. %(&sin 3x sin 5x + & cos 2x cos 5x) + C
37. sec3xtanx
4
+ -3 sec x tan x + -3 lnlsec x + tan x ( + C 4 1. e z ( x n - n x"-' + n(n - 1 ) ~ " -+~ . . . k n!) 43. 2?r 45. 1
8 0
47. -8.21 51. ( a ) A: $750,000, B : $1,050,000; ( b ) A: $226,520, B: $296,556

see. 11.2

2
25
I . -(arcsin(x/5) +
5 4(x2
X

+ 4)l'Z
+C 5. -In
4 ("+",
f i + 1 7.2(
X d r n + Inl- x + d r n
4 2
~

2
I) + C

9. 1 -( 3x
- arcsin(%)) +C 11. 4 x d n - Slnlx + 41
- +C 13. d= - 2 arcsec
27 d m
T ( 7 .d a ---In1 -2x+ -d
27 551/2 - 71/2 5!j3I2- 7 3 / 2 ) 17. 25 x
15.-(16 + 81 5 2 15 5
2 f i x+l
IS. -(arctan(T) + '&' + 1
25 .( + 1)2 + 5 ) - 2(x2 + 2x + 6) + C
21. L ( z a r c t a n ( : )
35.4 2
+ ___
3x +-.-.-.-
x2+9
1
2 d
x
m dFT3
-
x2+9
") +C 23. 9 25. d m - fiarcsec(x/fi) + C
27. ln(x2 + x
4 f l
+ 1) + __ arctan
3
("A')+
- C 29. iarcsin(2x - 1) +C 39. 4 [ 6 f i + l n ( 2 f i t 3)] 41. 377

arctan(=) +C 17. 4 Inlxl + f lnlx + 21 - 2 lnlx + 11 + K


1 1 8 2x+3 1 8
19. - - -.- arctan(-) - -*--
2(x2 + 3x + 6) 2 2*15V% fi 2 2.15fi 2(x2 + 3x + 6)
21. -1I n ( x 2 + 3 x + 5 ) 4+ - - . ~ a r c t a n ( ~ ) - 2 1 n / x + 1 / + K 2 3 . 2 + 6 x + l n l x - II-77In/x-21+42lnlx-31+K
3 3 f i f i 3 2
31. 1 +-
d
a-1

"-
Sec. 11.4
12(3
I . (a) u 7 (x - l)*l3; ( 6 ) 3 .( 7( - 11513 T)
+ .( - 112'3 +C 3. ( a ) u = 3 + x1/3; (b) 3(*i'i +
x1/3 7 / 2

7
1 -
+ x1/3)5/2

5
+ 6(3 + x +
~ / ~ ) ~C/ ~ )

5. ( a ) u = (x + 1)l"; (6) 2
3(x + 1)( - 5(x +~1)1'2 + 5 Inj(x + 1)1'2 + 11) + C 7. (a) u = (b) 6(T
x1/3
- x1l6 + ln1x1'6 + 11) + C
9. (a) u = (9- 4 . y . ( b ) 2 +x d m-8 I n l w +XI + C 11. (Q) u = &;( b ) In(&) - 2lnll - &I +C
13. (u) u = (e3' + 8)lI3; ( b ) - + -In
3
arctan (-)w+fl
1
- In [
1
; (e3 + 8 ) 2 / 3+ 2 m
9213 + 2 (6 + 4
+ 41
15. ( a) u = (1 + (1 + x1/2)1'2)1/2; 8u9 24u7
(b) - - __
9 7
16u5
+- + C, where u is as in ( a )
5
A-20 ANSWERS TO ODD-NUMBERED EXERCISES/SELF-TESTS

17. (a) u = e"; (6) arctan(2) + C 19. (a) u = (9 - x2)1/2; (6) <(ln
1
1 3 +
X
d n
I - 3 7 ) +C 21. (a) u = (2x + 1)1?
(b) \/2x+1- In( d m + 1) + C 23. (a) u = tan(x/2); (b) f - 4 in(1 + tan2(x/2)) + 4ln(1 + 2 tan(x/2) - tan2(x/2)) + C
25. (a) u 1 In(eD+ 1) - -
= e2; ( b ) -
3
1 ln(e2" - ex
6
+ 1) - -
1 arctan
6
("'G')+
~ C 27. (a) u = x1l3O;

( b ) 30(T
x39/30
- -+
x38/30 x37/30
- __
x36/30
+ y) x35/30
+C 29. u = tan(x/2); (6) - f i l n ( - l + fi) 37. (a) u =
(u9 + u6) du
(Q) (b) 6
38
~

37 36 u10 +2

S a . 11.5

1. (u) #27; ( b ) lnlx + 1- +C 3. (a) #36; (b) ~


v+ C 5. (Q) #17; (b) &4x - 1 0 ) d m + C 7. (a) #40;
9r

+C 9. (Q) #72; (b) -f(cot(x2) + x2) + C 11. ( a ) #98; ( b ) x2ex - h e x + 2e" + C 13. (a) #89;
2
3x
(6) f(x2 arcsin(x2) + d m+ C 15. ( a ) #105; ( 6 ) c ( 3 sin x
10
- cos x) +C 17. ( a ) #86;

( b ) -x3 cos x + 3x2 sin x + 2x cos x - 2 sin x +C 19. (Q) #88; (6)
sin3 x cos2 x
5
+-2 sin3
15
x
+c 21. (Q) #73;

(6)
-sin5 x cos x
-
5 sin3 x cos x
24
-
5 sin x cos x
16
+ -x5 + C
16
23. (Q) #45; ( 6 ) In
11+ I-; +C 25. (a) #73; (6) 3?r/16

27. (a) #73; ( 6 ) 3?r/32 29. ( a ) #27; ( b ) i l n l x


6
+ 2+
10
1-4 +C 31. (a) #35; ( b ) arcseclx + 11 + C 33. ( a ) #5;

(6) 4 lnll + sin(x2)1+ C 35. (a) #24; (6) x ln(x2 + 1) - 2x + 2 arctan x + C


S e e . 11.6
1. ( a ) 8.25; ( b ) 0.25; (c) 8; ( d ) 0 3. ( a ) 1.89612; (b) 0.16149; (c) 2.00455; ( d ) 0.00664 5 - ( a ) 1.72722; (6) 0.0M16; (C) 1.71832; ( d ) 0.01510
7. ( a ) 380; ( b ) 0; (c) 380; ( d ) 0 9. ( a ) -2.77148; ( b ) 0.3125; (c) -2.99740; ( d ) 0.16667 11. (Q) 1.09680; (b) 0.02946; (6) 1.08941;
( d ) 0.00052 13. (a ) 1.6439; (6) 0.00233; (c) 1.6421; ( d ) 0.000028 15. (Q) 0.65863; ( b ) 0.01042; (c) 0.65933; ( d ) 0.00115 17. ( a ) 0.96380;
( b ) 0.00570; (c) 0.96127; ( d ) 0.00089 19. ( u ) 20.64456; (6) 40.9486; (c) 17.35363; ( d ) 10.16132 2 1. (a) n 2 28; (b) n 2 4 23. ( a ) n 2 205;
(6) n 2 10 27. 35 29. 130,000~sq mi 3 1. 19.125 mph

Review Exercises
afi-2 2-4d5
1. -?(I - x2)3/2 +c 3. 4 e - 1 5. (x2 - 5)"' - fiarcsec +c 7. ~ + ____
5 3

+C 1 I. -xcscx - lnlcscx + cot X I + C 13. x - arctanx - l n / x + iln(x2 + 1) +C


15. x(arctan x)' - x 2 arctan x +x - arctan x +C 1
17. -( 1 - x ) ~ ' 1~ (+ x ) ~ '-~ - arcsin ( ( l
2
-2"')+ c 19. lnlsec xI + lnlsin xi + C
21. __ (9x - 2)(1
15 * 33
+ 3~)"' + C + csc x + C 25. 4In 2 + 3 27. 2x2 - ix sin 8 x - x 2 - & cos 8x + C
23. -cot x

29. - + l n ( f i +
I ,
1) 31. x2coshx - 2xsinhx + 2coshx + C 33. -x - 4x1/2 - 41nll - x1/2j + C
2"
35. -&InixJ + g l n / x - 41 + g l n l x + 21 + C 37. 3 +t 39.
In 2
9 (x2 + 9)5'2 17 (x2 + 9)3'2 3 1 3
41. 33(i) 35 + -3 33 +* (x2 + 9)1'2)
3
+C 43. --nix - 11 +
4 2(x - 1)
- -1nlx
4
~ + 11 + C
45. $ s e c 5 x - gsec3x +C 47. 3(e" + i)1/3 + Inl(e' + 1)1'3 - 11 - ln((e2 + 1)2/3 + ( e Z
- + 11113 + 1) - f i a r c t a n (2(e' + 1)1/3 + I
fi

49. sec x - tan x


1
+x +C 1 x
2(3x2 7)II2 + --(x2
5 1. -* +
7/3)'/' - -1nix (xz +
7/3)1/21 C 53. L!-- ( - Q ) + + + (7) -
1
+ b2 + b2 ( a )
~

6 f i 2 3.2 a2
55. -plnlx + +
11 $(x 1)-l Vlnlx - 21 + + +
K 57. ( x 2 - 9)1/21n/xl - (x2 - 9)'/' - 3arcsecl$l C +
59. 3(x + +
11113 Inl(x
1
+
1 y 3 - 11 - -1nl(x + 1 y 3 + (x + 1)1/3 + 1 - f i a r c t a n (
qx 111'3 1 + +
2 fi >+C

6 1. 2 lnll - 2(x XI +
3)lI2 - 8 arcsin + +23)1'2) + C 63. - ('"
( 2 4)1/2
~ C 65.
7fi 3
qln(l fi) + + ~ + +
4x 4
ANSWEKS TO ODD-NUMBERED EXERCISES/SELF-TESTS A-21

67. f i a 2 - g l n ( 2 a + *a) + a In a 69. 6- 71. &;nix - I1 - {lnlx + 41 + jlnlx + 21 + C 73. 4


2
1 7 7.5-3-1
75. x sec x - lnlsec x + tan X I + C 77. - cos78 sin 8
8
+-
8.6
C O S ~8 sin 8 + -
7 5 . 1 ~ 0 ~ ~ 8 s+i n___
8.6.4
7 8. 5 * 3 * 1c o s ~ s i n e+ 8 . 6 . 4 - 2
8.6-4.2
~ + C,

where cos 8 = ~

pq
1
sin8 =
q m
and 8 = arctanx 79. In
1 + ( x - 1)1/2
1 - ( x - 1)1/2
I+
C 81. - $ c o s ~ / ~ x C +
83. f x 3 / 2 In x - $ x 3 / 2 +C 27 I 3x 5
3 4 ( y ( ~- )
85. - +(Ty)+ 1
C 87. -In
4
2 - 1
l-
x2 1 + I+
C 89. $ eZ2(cosx2 sin x 2 ) + +C
5
91. t a n x - 2 +C zx
93. c ( 2 s i n 3 x - 3cos3x) +C 95. 3*-(1 + x1/3)14/5 - 6 * 2 ( 1 + x1/3)9/5 + y ( 1 + x1/3)4/5 + C
cos x 13 14 9
2 + (4 - 9 x y
97. h(i(1+ 9x)3/2 - (1 + gx)l/z) + c
1 1
99. 1 arctan(*)
-
fi 6
+c
28
101. (4 - 9x2)1/2 - 2 In
I3 3x
I + C 103.2

arcsinhx - - ( x 2 + 1)3/2+ - ( x 2
9 3
1)l+ +
C 105. ;earcsinr(x ( 1 - x2)/) +
C 107.
X
+
C 109. icosh8x - asinh 2x + C+
11 1. -cosxIn(cosx) cosx+ +
C 113. ( a ) 17; ( b ) 2; (c) 16; ( d ) 0 115. ( a ) 3.260805; ( b ) 0.15394; (c) 3.195605; ( d ) 0.00256564
117. ( Q ) 3.06870; ( b ) 0.01563; (c) 3.05924; ( d ) 0.00077 119. (a) 1.33790; ( b ) 0.0625; ( 6 ) 1.32594; ( d ) 0.00069 121. ( a ) 0,09092;
(6) 0.01198; (c) 0.08568; (d) 0.00052 125. 277 - 127. $2182.15 129. ( ) x arccoshx - 6
- +C
133.
16(xz
x + l
+ + 5)
2x
+ 3
128(x2 - 2 x + 5) + -
256
3
arctan(%) +C (2 9) + 2 k
135. - ln/3x + 11 + 3ln12x - 31
1+ C

Self-Test

-43lnlx + I/ 1 49InJ.r - 1) 1
+ 17 InJx + 41 + C 1 + (4x2 + 1)/2
5.
1
7 20
--
6: +6 + 1200
-~
lox - lo 225
6. --In
I 2x
8. - - ln(4 + sin x ) + -1 ln(1 + sin x ) + C 9. 6 + x16 - ln1x1l6 + 1I +C 10. (a) 1.39735; ( b ) 1.4052 11. n 2 183
3 3

Chapter 12

sec. 12.1

1. (a) ( y - x ) =
~ x;

7. ( a ) y = 2 - x ; ( b )

1
9. ( a ) y = 1 - x 2 ; (b)

t < 3n/2
3n/2 <t
11. (a) x =4 -y2; (b)

+ t
t = &Z

t = O x

13. (a) 2y = d-; (b) 17.

t = 3n/2
A-22 ANSWERS TO ODD-NUMBERED EXERCISES/SELF-TESTS

21.
4
(a) - = t; ( b ) y - 2 = 2(x - 5); (c) none 23. (a)
4 = 3t6; (6) y
- - 2 = 3 ( x - 1); (6) t = 0
= s,2 dx dx

t = 0, s,257

4 1 x 4 21
-2; (c) t = t l 29.
4 2t
( a ) - = -;
25. ( a ) - = e+; (b) y - - = -; (c) none 27. 3t2 - 3 ; ( b ) x =
( u ) - = ____
dx e e dx dx et
( b ) y - (In 2)2 - 1 = (In 2)(x - 2); (c) none

See. 12.2
1. 3/2 3. In(sec 1 + tan 1) 5. 4 In 3 7. -4(12)1/2 + 4 arcsin(l/2) 9. (24)l/' - (8)l/' 1 1 . 13. 4 - 15. 377
17. "[c
2 2
+ 41 ?[? - 4
4 4
19. In 2 - (In 2)') 2 1. 4 8 ~ / 5 23. 877

25. 277fi[T(y2
2
+-)21 '" + -41l n yI + (y 2 +-~~'211~ 27.
2
29. -
4771
16
(x - 3)2(x - 1)2
31. 2 n [ T + 81 33. ZTJ;@-~~ 35. q 3 ( x - 3) J C d x 37. 8 39. 1 6 d 41. (a) 1.20859; ( b ) 1.20800
0

S e e . 12.3

1. ( a ) 180; ( b ) 360; (c) ( 3 , 6) 3. X= f;J= 0 5. X= 515 '. r 128


~ 7. X = 1 ; y = o
105
- 9.
11. X= l;J=- 77

8
2e - 5
1 3 . ~ = 4 ; ~ = 15.~=2.>
-51
45 - -Y-- 19- V = 2 ~ y A= T Ji [ ( f ( x ) ) ' - ( ~ ( x ) ) ' dx
] 2 1. V = 2~ J ~ x [ ~ ( -
x g) ( x ) ] dw

n 1 3x [ x 3 - 3 x 2 -x + 31' d~
23. F = & 25. F=g 27.y- 29. x- = g 31.j=% 33.7=*- V 3,
?T[x[x~ - 3x2 - x + 31 dx

[(f(x) + 3)2 - 32]d x ; V = n [(f(x) + 3)2 - 32]d x , f ( x ) = x 3 - 3x2 - x + 3 35. X = J = 2/5

Mxz 89n 3 2
771111 ~3 [ 8 x 2 / 3 d ~ d x
53.y=-=
A
30
"mdx 57. r=
?[?
4 4
- 4 In 2 - (In 2)2
1 55-w= 74
271
x1/3

See. 12.4
1. (98,841.6)~ft-lb 3. 17,400 ft-lb 5. 19.5 ft-lb 7. 13,750 ft-lb 9. ( a ) ( 3 9 , 0 0 0 ) ~ft-lb;~ ( b ) 1 h, 57 min; (c) 17,403W 1 I . 20,000 ft-lb
13. ( a ) 40,435.2 Ib; ( b ) 22,464 Ib 15. ( a ) 64,000 Ib; (b) 444.44 Ib/sq in.; ( c ) 100,488.53Ib; ( d ) 100,573.87 Ib 17. 7,488,000 Ib
19. (98,841.6)~ ft-lb 2 1 (844,425)~ft-lb 23. ( a ) 40,435.2 lb; (b) 22,464 Ib 25. 64,000n lb 27. 7,488,000 lb 29. (450,000)~ft-lb
3 1. 211,200,000 ft-lb 33. 11,250 ft-lb 35. 10,625 ft-lb

Review Exercises
1. 4y 5.
tY
ANSWERS TO ODD-NUMBERED EXERCISES/SELF-TESTS A-23

9.

13.

I
x=I

21. (6) 2 sin(1); (c) y


I
15.

I.':.
+ cos2 (1) = 2sin (l)(x - sin (1))
t=O x
17.

+
( :) ( ;
23. y - - = - x - - 25.y-
19. ( b ) 2; (c) y = 2x - 2

'F) 1 = -3(x- 1)

27. y - (1 + 5) = - (x - $)29. horizontal tangcnts t = &2n, &n, 0 ; vertical tangents t = &?


2'
,371
- 2
31. &((37)3/ 2 - 11

33. 2((2)3'2 - 1) 35. In 3 - 3 37. fi(ec - 1) 39. - l n ( f i - 1) 41. In ___


2 -
( fi-1 ) \/?I
43. z m + L l n ( v
2 2
+d m )
45. s = S',-dt t3 + 2t2 + t + 4 47. -
24n 49. 27r[%- % I n 2 - @ 1 2 ) ~ ] 51. 2 ( ( 5 ) 3 / 2- 1 ) 53. 7r[5 a-
fi+In(-)] 5+ v%
1 4t 5 3 l + f i
55. 5/16 57. s = 4' q m d x 59. S = 2n
4l x2(1 + 4 ~ ~ ) ~ ' 61.
~ d xS = 271
6' x(1 +9~~)"~dx

63. S = 2" s1 (1 - x2)(1 +4 ~ / X~=


~ ~dx ) 65.
4' x( 1 - x2) dx

;v=
al'(1 - x2)2dx
67.X=
s1 x ( x 2 ) dx
;y=
4 i1
( x ~ dx
)~

0 A A A A

69. X=
4' x ( x 2 - x 3 ) dx
;J=
1
4 4 ( x 2 - x3)2 dx
x = -Mu2
71. - =
71 4 4x4 - x 6 ) dx M,,
73. j= -=
s, 1
4 1 - X2)*dX

A A V V V V

75.jT=-=
MS,
4' x7 dx
77. x=-=
M, 4' X( 1 + 9x4)Il2 dx -
;y=-=
M, l1 x3(1 +9 ~ ~ dx
) " ~
79. X = M " " =
2" l 1 x Z ( 1 +9 ~ ~ dx
) " ~

V V S S S S S S

2r['x4(l + 9x4)1/2dx +
2 ~ 4 ' ~ ~ ( (4 1~ ~ ) l / ~ d x 2" l 1 x 4 ( l +4 ~ ~dx ) ~ ' ~
- Mzz
y = - =4, 81. ,=My.= ; y-= - =
S S S S S S
83. ( u ) w = pm
-c' (x3)'(l - x ) dx = pn
4l P"
( x 6 - x 7 ) dx = -;
56
87. ( a ) 0 . 6 4 2 9 ~ ;( b ) 0 . 6 4 2 9 ~ 89. X = h / 3 9 1. ( u ) s = d(u - c)'
P"
56
P"
( b ) -; (c) - 85. ( a ) 0 . 2 2 5 ~ (~b ); 0 . 2 2 5 ~ (c)
8
(b - d)'; (6) s = d(a - 6)' + ( b - d)'
~ ; 0.375~"

+
Self-Test

(5x +3 -2 ~dx ~ ) ~
J3
x(5x + 3 - 2*2)2dx
4.y= , where A = (5x + 3 - 2 2 ) dx; X =
A
5. x= 0;
A
2" x(5x + 3 - 2x2) (5x + 3 + 2x2) dx
J= x(5x + 3 - 2x2)dx 6. s =
V
A-24 ANSWERS TO ODD-NUMBERED EXERCISES/SELF-TESTS

-
s3
x = - -Mv - -1/2
x(16x2 + 1)1/2dx
- -
; y = -MS
s3
-1/2
2x2(16x2 + 1)lI2 dx
7. 7275 ft-lb 8. ( a ) m[8m- 7&] lb;
S S S S 3fi
135n
( b ) *[203 fi- 175 fi]Ib 9. -CU ft 10. F = 18p(y)(6-y)(Zy) dy
3 f i 2

Chapter 13

See. 13.1

55. f 57. 4 59. 61.

1 7 = 2 csc B

63.

73.

r2 = sin2 28
83. ellipse, centered at (0, -9/8), with major axis alongy axis 85. r 2 = sec 28
ANSWERS TO ODD-NUMBERED EXERCISES/SELF-TESTS A-25

See. 13.2

1. (3, ~ / 6 ) 3. ( 1 , O ) 5. ( O , O ) , ( 1 , O ) 7. (1/2,a/3), (1/2, -71/3), (0,O) 9. (*I,


&77/6) 11. (*I, *71/12), (*I, *57~/12) 13- (t, 7 ~ )

P
15. (1,0), ( 1 , ~ ) 17. none 19. (f;fi,?r/8) 21. (2n - 1)z , - 1)-) for each integer n 23. ( a ) -5/fi;
(2n (6) - 2 / 6
( 4 2

25. (a) 1 ; (b) - 1 27. (a) -1/2; (b) -1/2 29. ( a ) 3 (6) 1/2 31. ( a )
4-3fi . (b) - fi/2 38. ( a ) - 1 ; (b) 1 35. ( a ) ~ -dT,
-1 + 2fi 4-Ih
(6) - fi 37.
2 ( a ) -1; (b) - 2 + 6 39. y = k x 41. (Y z 1.9 rad z 109 43. graphs coincide

See. 13.3

1 . 2 7 ~ - 1 3.1117+12 5.47~ 7.&eZw-1)


9.71 1 1 . 1 8 13.% 1 5 . 8 n / 3 - ~ 1 7 . ~ - 2 19.71
2
3w/ 2
a% a%
2 I. 2 - 2 arctan(l/fi) + 2 fi 23. A = (a - b sin 8) d8 25. (a) -(6)
4
-;
2
circle is traced twice as 8 varies from 0 to 271
w-oo
-

27. (a) J;(sinfi(8/3) + sin4(8/3) do; ( b )


C O S ~ ( ~ / ~ ) ) ~ / ~ - __ 29. ( a ) Ji(4(1 - cos 8)2 + 4 sin29)ll2 do; ( b ) 16 - 8 fi
2 8
3 1. ) ) ~ (/6~) 41a1[2/3 - cos(~/8)+ 4 cos3(~/8)] 33. ( a ) 2aJ;/4
( a ) J ; ~ ( U ~ sins(8/4) + a2 sinfi(8/4)C O S ~ ( ~ / ~ d8; sin28 d9; ( b ) 27
1471
35. ( a ) ~VJ;~ cos 8( 1 + sin 8)2do; ( b ) -
3

Review Exercises
1. B = s/2 3. B = n/2 s. I Y . R = n/2

17. +0 = n/2 19.

23. y = a 25. ( x 2 + y 2 ) 3 / 2= 2axy 27. ( x 2 + y 2 ) = a 3 2 26. (fi, n/6), (fi,


-71/6) 31. none
33. (a, 71/21 35. ( 3 , 71/21 37. ( a ) vert. tangent; ( b ) lirno,+(,/z)+tan$0 = 0 39- (0) - 2 / k (b) ~ / 24 1- ( a ) 1/2; ( b ) -1/3
97T T f i
45. E ~ +T fi-4 47. y 7 ~ c o s ~ 49.
2 271 51. v/4 53. 4 f i - $71 55. 18 - - 3 +-
4 57. - 2 59. T 61. 7~

63. I 9
+ - -fi 65. ( a ) f i J i T e c 0 do; (6) f i [ l - e ~ ~67.
] ( a ) 2J;cos(8/2) do; (6) 4 69. ( a ) J,27728(82 4)1/2d8; +
2 2 8
(6) y[(.2 + 1)3/2 - 11 7 1. (). J8/2 ~ 0 ~ 2 ( 8 / do;
1 37T
3) (b) -
2
(-2
- 1) 73. ( a ) 36J;l4 sec28 do; (6) 36 75. (a) 4J;lfi sin 8 d8;

( 6 ) 4(1 - fi/2) 77. (a) 4J;/4sinB do; ( b ) 4 - 2 f i


A-26 ANSWERS TO ODD-NUMBERED EXERCISES/SELF-TESTS

Self-Test

0 < a < 2, we get only (0,O); for the appropriate choices of a, every other point on the cardioid can be a solution 5. r = 1 + 4 cos 0

Chapter 14

See. 14.1
1. 1 3. - I 5. + w 7. 0 9. + w 11. 0 13. 4 15. - w 17. +co i f x + O + , - w i f x - + O - 19. 1 21. 1 23. 1
25. 4 27. 4 29. - I 31. 2 33. t 37. %7r/3,77

See. 14.2
1. 1 3. -2/v 5. - l / 2 7. 0 9. 4 11. 1 13. e2 15. 1 17. e 19. 1 21. 0 23. 1 25. 1 27. ea 29. +oo
31. e 33. 4 37. they are equal 39. 1

See. 14.3
1. 1 3. div. 5. div. 7. div. 9. 2 11. -1/4 13. 3 15. div. 17. 7 ~ / 2 19. - w 21. fi 23. div. 25. 1 27. 0
29. div. 31. ; 33. 7r/4 35. div. 37. 7r/2 39. div. (fn. not defined on [ - l , O ] ) 41. (a)improper; (b) impossible-not defined
on [-1,0] 5 1 . j = $ 5 5 . fi 59. $150=J012xe-0.0& dx +"

Review Exercises
1. -7r 3. 4 5. 0 7. 1 9. e4 11. 1 13. 0 15. 2 17. e2'5 19. e-lI2 21. 23. 1 25. 0 27. 0 29- + W 4
31. ln(3/2) 33. ?m/n 35. 3 37. div. 39. 1 41. 1 43. + w 45. 2 47. 2 49. 2 f i 51. div. S3. 4 55. -4
57. + w 59. 2
5
61. ( a ) I ; ( b ) 7r/2; ( c ) 7 r [ f i + ln(1 + fi)]
63. ( a ) yes; ( b ) 1.88895; 65. both = 1 67. a / b

Self -Test
1. ( a ) e ; ( b ) 0; ( c ) e-l; ( d ) 1 2. 1 3. (6) has 0 denominator at x = - 1 4. ( u ) T; (b) div. 5. yes 6. yes

Chapter 15

See. 15.1

1 3
4,
1> 6,
2 97
.3.- - - - 5.1,3,15,105 7.0,-1,0,- fi s . l , l , l , l 1 13 37 81 13.1,0,1,0
11.--,--,--,-
' (?2' e3' e4 2 2 3 4 5
( - l)n+l 2" 1 ( - 1 Y + ' ( n + I)
15. a, = 17. a =- 3" 19. a, = 5.5 + (0.5)(-1)n+' 21. a, = - 23. an =
5% 3+2n n! (12 Ion) +
+
25. a, = (-1)n(2n - 1)(2n 1) 27. decr. 29. decr. 31. incr. 33. decr. 35. incr. 37. decr. 39. decr. 41. bounded above
and below 43. bounded below 45. bounded above and below 47. bounded below 49. bounded above and below 51. -3
53. e f i 55. 0 57. 0 59. 1 6 2 . 1 63. does not exist 65. ee 67. 0 69. 0 71. 1 73. c7 75. 0 77. + w

See. 15.2

1. x- +
"
k=l
k2
1
2k
= 3/4 3. 3/4 5. div. 7. div. 9. div. 11. div. 13. div. 15. div. 17. 9/50 19. div. 2 1. 1/3 23. div.

- 627 2
25. 1/2 27. div. 29. div. 31. - 33. ? +
"
648
c
n=2 (.
1

+ 2)(n + 1)
35. 2
n=2
n2
-
37. 2
n=l
(2n - 1) 39. a 41. %
43. 10 m 45. 60 mi 47. $2500, $500 more 49. 5 min 27& past one 5 1. 10 times.
ANSWERS TO ODD-NUMBERED EXERCISES/SELF-TESTS A-27

See. 15.3
1. conv. 3. conv. 5. conv. 7. conv. 9. conv. 11. conv. 13. div. 15. conv. 17. conv. 19. div. 21. conv. 23. conv.
25. div. 27. conv. 29. conv. 31. div. 33. conv. 35. div. 37. conv. 39. conv. 41. div. 43. conv. 45. conv.
47. div. 49. conv. 51. conv. 53. conv. 55. 1/2187

Sect. 15.4
1. (b) < 0.162 3. (b) < 0.514 5. (b) < 0.00002 7. (6) < 0.028 9. (b) < 0.018 11. (b) < 0.390 13. abs. 15. div. 17. div.
19. abs. 21. cond. 23. abs. 25. abs. 27. div. 29. cond. 31. cond. 33. abs. 35. abs. 37. cond. 39. abs.

43. C" Tk1


k=l

Review Exercises
1 if n odd ( - 1y+1
if n even 11. a, =- 4"-1

13. a, = ( - l)n+'(2n + 1)(2n + 5) 15. a,


(-1)n
= 32n-1 17. a, = 19. dec., bounded above and below 2 1. dec., bounded above
2" +n
and below 23. inc., bounded below 25. 1/2 27. +w w
29. 0 31. 0 33. - 35. e3 37. ec5 39. e2 41. e-2 43. 0
3
45. 4
47. 0 49. 1 51. 0 53. 0 55. + w 57. 0 59. 0 61. + w 63. 65. & 67. 69. div. 71. 3 73. &
75. sin(1) 7 7 . div. 79. abs. 81. div. 83. div. 85. abs. 87. div. 89. abs. 91. abs. 93. abs.

Self-Test
1. ec3 2. 0 3. 1 4. 0 5. 0 6. 9 7. 4 8. conv. 9. conv. 10. conv. 11. conv. 12. cond. 13. 4 terms

Chapter 16

See. 16.1

-x 2
1. (a) p4(x)= 1 - x +-
x2
- - + -;
x3 x4
(6) Pn(x) =
(-1)iXi
C- 3. (a) P3(x) = -;
2
( b ) -0.005; (c) < 0.00000875
2 6 24 izo i!
2x3
5. (u) P3(x) = 1 +2- 2
+ -;x3 (6) 0.94869; (6) 1.4375 7. ( u ) P4(x)= 2x +-3
9. (u) Pz(x) = x ; (6) 1 11. ( u ) P4(x) = x + -;3 x3

8 16
k x2i-1 - - 1 ( x - 112 qX- 113 .( - 1)4
(') k' ()' = P2k-1(X) = C 13. (a) P4(x) = P5(x) = f ( x ) ; ( b ) P4(x) = P5(x) = --
12
- 2(x - 1) - ~

2
+ ~

3
+ -. 4 '
i=l

(6) p4(x)= P4(x); ( d ) -0.10338266. . , 15.


104

2 (-
k=l
~
l)k+l
k
17. ifl
k=0
k!
19. 4 - -- -
1
23
1
29
--=1
214
3.87299 21. n 26

8x4
23. P&) = x 2 - -
4!
+--32x6
6!

See. 16.3
A-28 ANSWERS TO ODD-NUMBERED EXERCISES/SELF-TESTS

23.x+-+-+-
x3
2.3
1.3x5
2-4.5
1.3.5x7
2-4.6.7
+ ...I xi<1 25. 2 ( -32nt1(2n
O0 ly(x
+ 1)
- 2)2%+l
,jx-22(<3 27. C ( -(2nl)n4nX2n+l
+ l)! , all x
n=O ?L=O

( - l)k(2k - l)!(x - 1)k


333! 344! k=l
2k - l)! k !

See. 16.4

1. ( a ) 7 z4P3(1) = c-
+
n=O
(-1)n = 2.8952; ( b ) n =: 4P5(1) =
2n 1 n=O
(-'In = 2.976; (c) n
2n 1
c ~

+ >5 lo9

1 1
3.---+---
2 Z5.5
1
2'*9 ZL3-13
z 0.4939576239 5. 1 + C (1/3) (O.l)n ~ z 1.032279 7. 2 + 2 (1/2) 22n-1(2n1 + 1) z 2.0804
n=l n n! n=l

9. ( b ) c n+l'
n=O
(-'In, (c) 0.8333.. . ; ( d ) 0.78333.. . 13. - + 1
2
1
-= 0.503125 15.
Z5*2*5 n=O
( - I)"
Z4""(4n
$ + 1) z 0.49395
17. ---- 1
Z3*1.3 Z7-3-7
0.04129

Review Exercises

1. (a) P4(x)= c
n=o
~
e ( x - 1)n
n!
; ( b ) <0.0014 3. (u) P4(x)= 2 (x -f i ./4y
n=O n!
; ( b ) <0.00002

5. (a) & - f i b - ./4) + ( z f i 2! +


- .I412 - (6 fi+ 8)(. - ./4)3 +
(30 fi + 40) (x - a/4)4; ( b ) <0.019
3! 4!

7. ( u ) 2 + (x - n / 2 ) -
- ~ ' 2 )-~ 3(x - 7m3
2.2! 3!
~ ( -x
2*4!
( b ) <o,116 9. ( a )
n=O
+
(2" + 1)x"
n!
m4;
; ( b ) <0.275 5
11. ( u ) 49 + 68(x - 2) + 38(x - 2)2 + 1O(x - + (x - 2)4; (b) 56.1901 13. r = 1; I = (3,5) 15. r = 1; I = (-1, 1) 17. r = 1;
I=[-l,l) IS.r=l;I=[-l,l] 2l.r=8;1=[-5,11] 23.7=1;1=[-1,3) 2 5 . r = + c o ; I = ( - c o , + c o ) 27.7- +CO;
* ( - 1)n2Zn+lx2n+2
I=(-co,+co) 29. r = l ; I = ( - l , l ] 31. r = 3 ; 1 = ( - 3 , 3 ) 33. 7 = 2 ; I = [-2,2] 35.
C
"=O
2n+1
> 1x1 <1

37. c
n=O
( - 1)n42n+lx4n+3
(2n + l)! , all x 39.
fix
fi+ - 2
+ fi >:
"=2
( - 1)"-I( 1 * 3 * 5 . . . (2n - 3)) xn
5"2" n !
,iXi<5 41. - ( - - l() Zn Pn+) Z!
2
n=O
, x > o

m X4n+2
zx zx2 ~ - 5 ~ 2 3- 1oX4
2-4n + 2 '
4 3 . ~ + 1 - 2 2 x n , I x l < l 45. 2+---+----++.41xI<8
8.3 829 8"81 84*243
47. 1x1 <1
ft=O TL=O

Self-Test

1. ( a ) I = (-1 - fi,-1 + fi);


(b) I = [-1/3, 1/31
2. 5 (-l)ntlnxn
5n+i
1
3.-+---+-
2
1
2510 29*72
1
213-208'
<210-8
n=O
ANSWERS TO ODD-NUMBERED EXERCISES/SELF-TESTS A-29

(-2,4,1),(0,4,-2),(0,-1,-2),(-2,4,-2) 7.(-4,6,-1),(4,6,1),(4,-1,1),(-4,6,~),(-4,-1,~),(4,-1,8) %yes 11.no


13. yes 15. yes 17. sphere, center (1, -2,4), r = 12 19. no graph 23.

~.

27-# _- y = 1/2x - 2

S e e . 17.2
1.

z = x

right circular cone

parabolic cyl.

11.
r 15.
!t

hyp. of 2 sheets
ell. of rev.

f I
ell. cyl.
17. 4

plane through (0, 0,O)


A-30 ANSWERS TO ODD-NUMBERED EXERCISES/SELF-TESTS

19. A" 21. 42 23.

--,--
/--1--.
X -l/&

X hyp. of 1 sheet
X
hyp. of 2 sheets
cyl. parallel
to x axis

,&
27. 4" 29. 4"

X Q
.)5
hyp. of X

rev. of surf. of rev. with


2 sheets y = k traces + 2 2 f 2 2 = s2/4as k + +oo

11.(12,-8) 13.(-l,-fi,-fi)
...
Y
15.(-9,-7) 17.(-4,2--$?,5)
A
X

19.(-2-~,fi-l,l-e) 21.A+B=(1,6);
A - B = (-5, -4);
fY
23- A + B = (-9, -4); A - B = (-5, -2)

29. A + B = (3, -2); A - B = (-11, -6) 31. (-S,O, 16) 33. & ( - 5 , 2 , 3 2 ) 35. + - 24- . . 4- - -4
a
37. (;.i>2 4fi 39. (" 5 ) 41.
fi'fi
(0,9)
fi l/m'l/m'v%

See. 17.4
9
1. (a) -22; (b) ~
-11
+iFT-7&7Tz5 5. (a) Y;(6) ~

&FiG
7 . (u) -16; (6) -8 9. (a) -11; (b) -6
ANSWERS TO ODD-NUMBERED EXERCISES/SELF-TESTS A-3 1

-11
1 1. (u) -; (b) ~
-11
13. (u) 2;
(6)
-7
f i f i
~

21. -2/5, 10 23. -1, 9 27. no 33. 600fift-lb 37. (18, -6,7)
15. ( u ) -1; (6) 5
fi
-18
17. ( u ) -.
fl'
39. 9(2,3, -1)
(6) ~
-18
7
19. (u)
*>
-'

41. 71 43. 36 45. 36


74
(6) ~
74
&i

(17,9,7) (-17, -9, -7)


47. j(B X C) 49. 51. It = 0
~

V$m'
m

See. 17.3
1. x = 1 - 4t;y = 2 + 2t; z = -6 + 8t 3. x = -2 + 6 t ; y = - 6 + t ; z = 3 + 2t 6. x = 0 + 13t;y = -2 + 19t; z = 6 - t
7. x = 1 - t ; y = - 6 - 91; z = -4 - 5t 9. 2(x + 6) + ( y - 3) - 4(z - 1/2) = 0 11. 51x + 18y - 312 = 303
13. 4(x - 6) - 22(y - 1) + IO(t + 7) = 0 15. 2 9 +
~ 10~ + 312 = 57 17. __ 19. ( l , O , -2) 21. x = t ; y = -14 + 5t;
fi
z = 10 - 3t 23. 2m

Review Exercises
1. 5. A= 7. 4"

r=5 i
rt. circ. cone Y

9. A= 1 1. sphere, ( - 1/2, -3/2, 1/2), r = 2 13.

,---c--

x surf. of rev.
ell. paraboloid

hyp. paraboloid
17. sphere, ( - 2 , 5, 8), r = 1 Id.
ANSWERS TO ODD-NUMBERED EXERCISES/SELF-TESTS
A-32

37. x2 = 1624
33.
t" 35.
t" +y2

cyl. CY'

xz 2 2 y2 6 fi,fi);
39. - + - + - = 1 41. x2 + z2 = y 3 43. x2 + z2 = e 2 Y 45. x2 + y 2 = l/z2 47. ( a ) 5; (6) -( (6) ifi
9 9 16 5

49. ( a ) 2
m (b)
55. ( a ) 3 m;
( -~
&I

( b ) (1/m3,/ f i )
~

;Pc)- i G
57. ( a ) (-9, 6,4); ( b )
1
51. ( a ) 1; ( b ) (-1,O);
59.
(6)

(a)
-1

(-2,
53. (a) m / 1 2 ; (6)
12, 6); ( b ) 2
m(-1/18,

6 1 . 14 63- 23
1/48)

65. 15(x - 2) + 3(y + 3) - z = 0 67. -1(x - 6) + 4(y - 2) + 8 ( t + 4) = 0 69. 38x + 1% + 292 = 262 71. x = 2 + t;
JJ = 1 + t ; z = -8 + 13t 73. x = 5 - 2t;y = 6 + t; z = -8t 75. __ 81. 4 2 m 83. 8 0 @
6
Self-Test
1. 2. x2 + z2 = ( y - 9)2/3 3. x - 5y + 3z = 6, 6/* 4. x = 2 + t ; y = 2 - t ; t = 6 + 2t; yes

elliptic paraboloid

5. ( a ) y<- $ %);(6) (-4, 20) 6. (u) 10 fi;(6) 42

Chapter 18

See. 18.1
I. SY 5. ty 7.

I_
9. 11. yes
ANSWERS TO ODD-NUMBERED EXERCISES/SELF-TESTS A-33

13. yes 15. F'(t) = (x,


\/t
cos
( 2t 8'3; F"(t) =
'
t s m d - cosfi
4t3'2
; 17. F'(t) = (-sin 2t, 2 sec2 t, 1/(1 + 1)');
F"(t) = ( - 2 cos 2t, 4 sec2 t tan t , - 2 / ( t + 1)3) 19. F'(t) = (tan t , 0); F"(t) = (sec' t , 0) 2 1. x = 1 + t;y = 1 - t ; z = fi+ t ;
-7rt
(x - 1) - (.y - 1) + (2 - fi)= 0 23. x = :
2
+ t ; y = -.
2 '
2 =
2
1
+ t + -,2 -
3
+ 2t - t3

See. 18.2
1
1. ( u ) T(t) =
(t' + 1)1'2 (cos t - t sin t, sin t + t cos t ) ; ~ ( t=
)
(t2 + 1)1'2 (-sin t - t cos t , cos t - t sin t ) ; (b)-(c)

3. ( a ) T(t) = ( 3 6 COS' 3t + 25)-'/*(6 cos 3 4 5 ) ; N ( t ) = (36 cos2 3t + 25)-'/'( -5, 6 cos 3t); (b)-(c)
N
T

7 - ( a ) T ( t ) = (sech 1, tanh t ) ; N ( t ) = (-tanh t, sech t); (6)-(c)


ty

1 -1 .
11. ( u ) T(t) = -(cos t - sin t , cos t + sin t , 1); N(t) = -(sin t + cost, sin t - cos t , 0);
fi dT
-1
B(t) = -(cos t - sin t , sin t + cos t , - 2 ) ; (b)-(c)
&
A-34 ANSWERS TO ODD-NUMBERED EXERCISES/SELF-TESTS

13. ( a ) T ( t ) = (t4 + t 2 + l)-'lZ(t, t2, 1); N(t) = (t' + 5t6 + 6t4 + 5t2 + 1)-ll2(1 - t4, 2t + t3, - t - 2t3);

1
15. ( a ) F ( t ) = (4 sin t , 2t - sin 2t, cos 2 t ) ; T(t) = (cos t , sin2 t , -sin t cos t ) ; N(t) = (-sin t, sin 2t, -cos 2t);
V T T Z z

17. k ( t ) = - 19. k ( t ) =
&et t(9t2 + 4)3/2 21. k ( t ) = 0 23. k ( t ) =

33.
2t3(t8
(t8

CY
+ 4t4 + i ) 1 / 2
+ t4 + 1)3/2
35.
6
25. k ( y ) =
(9

y = sin x
18
+ 4y2)3/2

37. C ( t ) = (-4P,?)

See. 18.3
ds ds
1. V ( t ) = (2t, 2 , 1); A ( t ) = (2, 0, 0); - = (4t2 + 5)'12 3. V ( t )= (cot t , -tan t ) ; A ( t ) = (-csc2 t, -secz t ) ; - = (cot2 t + tan2 t ) 1 / 2
dt dt
ds
5. V ( t )= (3 cos t , -4 sin t , 1); A ( t ) = ( - 3 sin t, -4 cos t , 0); -=
dt
(7 sin2 t + 7. V ( t ) = c t ( c o s t - sin t, -sin t - cos t ) ;
ds ds
A(t) = 2 e 8 ( -cos t , sin t ) ; - = $!?e+
dt
9. V ( t ) = ( e t - d ,e t + e+); A(t) = ( e t + e+, et - ect); - = (2eZt
dt
+ 2e-2t)1/2
+ el + ( + 2, t4 + 2t +
( 2 i f t +> 0 . +-
11. G(t) = 3t - t2 1,4t
3t2 - 1) 13. G ( t ) = (34 -4t2 +t) 15. G ( t ) = (-lnlt F ) t
2
e2t - e-2t
17. aT(t) = ( - 2 if < 0 2 aN(t) = 4t2 if # aT(t) = [ezt + e-2t11/2 ; aN(t) = [ezt +
2
e-2t11/2

2 1. G(t) = {(un cos a)t, (un sin a)t


2
g
- -t2 + k ) 23. -79" 25. -32" 27. A(t) = (-32 sin 4t)v(t) + (-40 cos 4t)u(t)

29. A(t) = (8 sin(2t2) + 32t2 cos(2t2))v(t) + (8 cos(2t2)- 32 sin(2t2) - 8t2 sin(2t2))u(t)


Review Exercises
1 21nt 2(1 - lnt)

, 2t csc(tZ)cot(t2)) F"(t) =((1 - t


t2)3/2,
2 csc(t2) Cot(t2). 4t2 csc(t2) Cot2(t2) - 4t2 csc3(tZ)
ANSWERS TO ODD-NUMBERED EXERCISES/SELF-TESTS
A-35

15.F(t)=

25.

29. 3 1. T ( t ) = (-sin 2t, cos 2); N ( t ) = (-cos 2t, -sin 2 t )

33. T(t) = ( 1
N(t) = 35. T ( t ) = et
( p + e-2t)1/2
et
( - 3 , 4 , 3 6 ) ; N(t) = 1 27fi -364 75
N(t) = 37. T(t) = ~

( p + e-2t)1/2 ( p + e-2t)l-2 d l l t +1 1 2 5 ( 7 7 3 )
4 m
B(t) = l(t)(4(25 + 9t), 3(25 + 9t), 0), where l ( t ) = 3
k ( 1 ) = - 39. T(t) = (cos t, sin2 t , -sin t cos t ) ;
fi(25 + 9t)ll2(411t + 1125)l/ 289
N(t) =
1
(-sin t, sin 2t, -cos 2 ) ; B(t) =
fi 4 1.
+ sin t ) ; k(?r/2) = -
d r n \/m
(sin2 t, C O S t,
~ sin t cos2 t
4
k(t) = 1

1 -7 cost sin t . 12
43. k(t) = 45. k ( t ) = - 47. k ( x ) = 49. k ( x ) = 2x3 51. aT = , aN =
2(1 +
t2)3/2 t (4x + 1)3/2 (x4 + 1)3/2 \/7 cos2 t +9 \/7 cos2 t +9
53. aT =
(32t3 - 1)
4 t 3 / 2 d r n >aN
,

= d m
1 ift>O.
-1 i f t < o
aN = it1 57. aT = - 6 5 s i n t c o s t . aN =
4 6 5 cos2 t + 17
[ +
+
1312 65 sin2 t
17 65 cos2 t

83. C ( t ) = (6t2 + 1, - 8 t 3 ) 85. C ( t ) =


3

Self-Test

t \/4t22 t-- 8t + 5 >;


N(t) =
d 4 t 2 - 8t +5 );

2
k(t) = 2. ( a ) V ( t ) = ct(
-sin t - cos t, cos t - sin t ) ; A(t) = 2&(sin t, -cos t ) ; ( 6 ) aT = - fie-i; UN =
(6)
\/4t2 - 8t +5
v?iPTi
3. k ( t ) = 4. ~ 3 2 8 8 . 3 8 f t / s 5. t = 3
(20t2 + 11312
A-36 ANSWERS TO ODD-NUMBERED EXERCISES/SELF-TESTS

Chapter 19

See. 19.1
1. all points outside or on the ellipse with equation x2/5 + y 2 / 3 = 1 3. all points (x,y) not on the x axis 5. all points above the line
y = -x 7. all points below the liney = 4x 9. all points in space except those on the plane with equationy = z
13. the traces in the x ; y , x-z,y-z planes arey = -x3, z = x 3 , and z = y ; in the plane = k the trace is y = k - x3

x2 y2
21. ellipses -
c
+-
c/9
= 1; if c = 0, level

curve reduces to the single pt. (0,O); no level curves for c <
0 23. if c # 0, then xy = c is a hyperbola with transverse axis the h e y = x if
c > <
0, a n d y = -x if c 0; if c = 0, level curve consists of the x a n d y axes. 25. if c = 0, the level curve is the straight line x = 2 ; if c # 0,
level curve is straight line with slope l/c, which goes through ( 2 , - 1) 27.

See. 19.2

m
3. f(.>Y) -+ -as (x,y) + (0,O) alongy = mx 5. f(x,y) -+ 1 + m as (x,y) 4 (0,O) alongy = mx2 7 . 15 9. 1 11. -14
m + l

15. lim f ( x , mx) =


*-0+
1
x,
+
lim f ( x ,
Iml r-0-
mx) = - +
1 - Iml
hence not cts. 19. l i m ~ ~ , ~ , ~ ~ - ~z )~= , ~L , if,
~ ~given
~ ( xany >
, y E, 0, there exists a 6 >0
such that 0 < <
d[(x,y, z ) , (a, b, c)] < + +
6 implies d [f(x,y, z ) , L ] E , where d[(x,y, z ) , (a, b, c)] = d(x - 2 ) ( y - b)2 ( z - c)*

See. 19.3

4s3t y2tz3 + tz3xy


A=--- t224
+ XZ3t2
.A = +
n; & = ;
7
dxy + x z t
15.f,= f, = 3 t z 2 d m xtz4
d m t3 2d 2d W ' 2 d W
17. f , = 2xy3zw c o s ( x 2 y 3 z ) ; f , = 3y2x2zwC O S ( X ~ ~ = +
Z )W ; ~ X, C O~ S ( X~ ~ ~ Z ) ;=~ ,sin(xzr3z) 19. f, = (4r +
4h) sec2(r2 2rh) tan(? 2rh); +
+
j i = 4rsec'(? + 2rh) tan(i2 + 2rh) 21. 7rT:-4(x - 2 ) + (-2)(y - 1) = z 1; LX:x = 2 - 41,y = 1 - 2t, z = 1 - I
23. ~~7 2 + + 1 = 0; L N : x = 1,y x -t, 2 = - 1 - t 2 5 . 7rT:-44(x + + 1) 36(y - 2 ) = - 22; LN:x = -1 - 44t,y = 2 36t,+
ANSWERS TO ODD-NUMBERED EXERCISES/SELF-TESTS
A-37

sin(x + y 2 )
(XdJ) # (0,O)
1
z = 22 - I 27. 4 29. for f(x,y) = ,f,(O, 0) = - 33. - 1260" per unit of displacement 35. f,, = 2 6;
( X d ) = (0,O) 2

39. f,,= -e2'sin(xeu);f,, = -x2eZy sin(xeY) + xeYcos(xeY);f,, =hL= -xe2Y sin(xe') + ey cos(xev) 41. f,,=
- 3xy2

S e e . 19.4
1. 0.001568 3. 0.0067996 5. 0.00003036 7. 0.00336051 9. 0.0004047 17. 29% 19. 2.98666667 21. 0.62923102 23. 4.02

See. 19.6
- 199 1+2fi 3 1 m
1. -58/(5*) 3. ~ 5. 4 8 f i 7. ~ 9. ~ 11. rnax.:(-54, 39); min.:(54, -39)
1690 lh6 12 783

13. max.:(-4/fl, l/fl); min.:(4/f11 -l/fl) 15. max.:(5,0,6); min.:(-5,0, -6) 17. 96a
~ 19. 14/5
13
21. (-*, -2*)

See. 19.7

11. x + y - 2 = 0 13. x + y - 1 = 0 1.5. 5~ - y - 4 = 0 1 7 . f ( x , y ) = '2 -y2, Vf(x,y) Vg(~,y)= 2 9 - 2x9 = 0


1 9 . ( u ) .rrT:12(x- 2) - 4(y - 1) + 4(z - 2) = 0; ( b ) L,:x = 2 + 12t,y = 1 - 44 2 = 2 4t +
21. ( u ) aT:-13(x + 2) + 12(y - 1) + 4(z - 1) = 0; ( b ) LN:x = - 2 - 13t,y = 1 + 1 2 , z = 1 4t 23. xy + J Z - xz2 = 1;+
( u ) .rrT:2(y- 1) + (-1)(z - 1) = 0; ( 6 ) L,:x = 1,y = 1 + 2t, z = 1 - t 25. (u) 7rT:3c2(x- 2) - eC2(y 1) eC2(r - 1) = 0; + +
( b ) L,:x = 2 + 3ec2t,y = -1 - c 2 t , z = - 1 + c 2 t 27. T:x = 1,y = 1 - t , z = a/2 - 2at

S e e . 19.8
1. (0, 0), rel. min. 3. Forf(x,y) = 2x2 - 4x3 + 2y2 + 2xy, (0, 0), rel. min.; (1/4, - 1/8), saddle pt. 5. (0, -1/2), rel. min. 7. (0, 0),
saddle pt. 9. (1, 3/2), saddle pt., (5, 27/2), rel. min. 1 1. ( - 1, I), rel. max.; ( - 1, - l), rel. max.; (0, 0), saddle pt.: f(h, k ) has nonconst.

sign near (0,O) 13. (1, l), (- 1, - 1) rel. min.; (0,O) saddle pt. 15. no crit. pts. 17. no crit. pts.
A-38 ANSWERS TO ODD-NUMBERED EXERClSES/SELF-TESTS

(-E,-%), rel. min. 21. max. = 2 at (- 1, 0) and (1, 0 ) ; min. = -$ at (0,a) 23. max. = 6 at (0, 2); min. = 0 at (0, 0)

25. max. = 12 at (2,O); min. = -8/5 at (2/5,8/5) 27. max. = 22 at (6, 2); min. = -29 at (0, 3 ) 29. min. = -1 at (-l,O), (1,O);
1235
max. = 5/4 at ( 2 4 , 2 fi/2) 3 1. min. = 1/2 at ( - 1/2,0); max. = __ at (1/2, 35/4) 33. i/i? x x 3 i/i;i 35. 1 x 1 x 1
4
Id1
37. D = 39. 2/3 by 2/3 by 2 4 1. no min. distance - (0, & 6)
are saddle pts. of dist. fn.
da2+ b2 + c2

Sec. 19.9
1. hamburgers, 5 milkshakes

k = 80

7.f(x,y) = 2 x 2 + 2y2 - x --y +1 so max. = 3 + $occurs


i at
(
1 -
--
\E'
)'\b 42875
9. min. = __ z 3572.9 at
12
(F+) .,
11. max. = f i a t ( 2 l / m , 2m)
13. max. = 3888 at (18,9,6) 15. min. = 72/7 at (6/7, 18/7, -12/7) 17. max. = 1 at
(-1, 1, l), (1, -1, l ) , (1, 1, -1) 19. min. = y f i a t (50/9, -200/9, -50/9) 21. min. = 3 at (t3,0,0) 23. min. = 4 f i a t
(5, 10, 16) 25. min. = 0 (passes through origin) 27. max. = 3/e at (1,O); min. = 0 at (0,O) 29. 12 X 12 X 12 31. fi/5 X 4*/5
35. ( a ) a = 3/4; ( b ) 1 - a = A; (c) h = 0 (no adjustment called for since min. occurs on boundary)

Review Exercises
1. 3. 5.

X32 m2
9. all pts. on or outside sphere of radius 4, center ( - 1, 0, 0) 1 1. limf(x, mx) = __2m2 13. forf(x,y) = -limf(x, m )=
1 + m2 x6 + yG ' r-0 1 + mG
~

x-0
ANSWERS TO ODD-NUMBERED EXERCISES/SELF-TESTS A-39

(d) 7''=1:x = 1,y = 1 + t , z = e + et;


21. ( u )
t" (b)
c , 2 fY
st;ight line
of slope e

c I

TY=l' . x = l + t , y = l , r = e + e t 23.f,=
-ll"IY3 Ixly2 + 2y arcsin(y/x) 25. f, = 2[cos(x + In(xy))]
x2 d m
sin(x + ln(xy))(l + I/x);& = 2[cos(x + ln(xy))] sin(x + ln(xy))(l/y) 27.f, = ev/92[-2y2/x + 2xy] 29-f, = eg2+Y2+z2[2x2y+J];

f, = er2+y2+22[2~y2 + XI; f , = 2 2 9 e'2+Y2+22 403 - 4rO -2(r - 82)2 -P - -;


1
3 1. f, = 2(r - 8) ; f , = ( z - r2)J 33. f, = -
m2 z
(p2) + arcsin(qp2) 37. fc = -. 3P2G2 -P2 ,
f, = 4p2 + -;&
W
= 8pz + -1 35. f , = q 1
4(2P'pq);& = 4
1
22 W m d F 7
39. ep'q[4x + $ + x2/y] 4 1. 0 43. 0 45. nT:x + $ - z = 1; L,:x = t , y = 1 + 2t, z = 1 - t
47. vr,:3x + 3y + z = 15; L,:x = 1 + (f)t, y = 1 + ( D t , z = 9 + (Q)t 49. let F(x,y,z ) = x arcsin(y / z ) - y arcsin(x/z2) + 77/12;
2 - fi
..:(=)(. f i 7 7 - 4 - 1) +y().,( 6-77 - 1) + ( 2 - v5 l - v5)) = 0; L,:x ( f=
= 1 + (--)t,y
ir-4 =
fi 4 f i
51. 5.000933 53. 10% 55. f(x/y,y/x) satisfies the eq. 57. 2x(to)y(to)- 3[y(to)12+ 4[x(to)l2- 4z(to) - %(to) 59. -48/5
4-3v5 5-2v5
61. ~ 63. ~ 85. ~ = m a x . ; - ~ = m i n 87. . \/1073=max.;-\/1073=min. 8 9 . ( 1 , - 2 ) = r e l . m i n .
10 27
71. (1, l), (-1, -l), saddle pts. 73. (0, k2n77) all saddle pts. 75. min. = -5/4 at (-3/2, -5/4), (1, -1/2), max. = 11 at (2, -3)
77. max. = 2 / f i a t ( 1 / & , 2 / f i ) , min. = - - / f i a t ( - l / f i , -2/&) 79. ( t l , O , O ) nearest, (O,O, t 3 ) farthest
81. ( a ) m a . = 77/2 at (7r/2, 0) (endpt.); (b) ; (c) x = -7r/6 is rel. mtn by second Deriv. Test 83. x = y = z = 4

85. ( 2 2 &,o, 0)

--
Self-Test

1. (u)f, = 2xy3 - 2x + ;;& = 3x5' - 2y + $; (b)&, a


= -(2xy3 - 2u + $) = 6xy2;& a
= -(3~2y - 2y + i) = 6-9'; (c)L(l, 1) = 1;
aY ax
&(I, 1) = 2; 7rT:(x - 1) + 2(y - 1) = z + 1; L,:x = 1 + t,y = 1 + 2t, z = -1 - t 2. to show that x- az + y - az = 0, let z =f(u) and
ax ay
u = u(x,y) = 2 3. ( u ) ____
x +Y
l2
6
(b) + v5; = ma., - = min. 4. min. = 3/4 at (- 1/2,0); max. = 13/4 at (1/2, * fi/2)
3 - m2
5. limf(x, mx) = - hence not cts.
x-0 1 + m2'

Chapter 20
See. 20.1
m+2 243
8 8 m - 41 m- 5 3 m + -m+
47
2
3841n(-)
f l + 2
m- 1 - yln(-)] 0- 1
5. lln(3)
77 In 2
7. & 9. - 11. 2
4
13. 0 15. ( a ) ss 5 2
( 2+
~ 3 y ) d y d x ; ( b ) 75 17. ( u ) s 2 s Y L d ~ d y( 6 ;)
x 2 +Y2
~
3 In 2
4
19. ( u ) ss
1 2x
C O S ( T X ~ )( 6~) -
~~X;
-fi
271

s s
-3 0 1/2 0

s
-1~. 1

2 1. (u) s7s"''
0 arctanWv)
sinx dx dy; ( 6 ) d m- 1 23. ( u )
1
4

x
28
G d y dx; ( 6 ) 15( fi - 1) 25. ( a )
0
71/2

0
Y
(sinx + cosy) dx d y ;

(6)
77-4
2 27. f ( 1 - cos(1)) 29. ss
o
2

r
2
4e-YZ dy dx = 2 - 2/e4 3 1. 33. -
-423
2
A-40 ANSWERS TO ODD-NUMBERED EXERCISES/SELF-TESTS

See. 20.2

1. +j 3. 9 5. $ 7 . 6 9. 18 11. 8/3 13. M ( L ) = 4, (11/15,4/5)


4'2"0' (
15- M ( L ) = -, -_ _
20'390
31,269'
+
2610
-4461)
2 f i + ln(3 2 f i ) ]
17. position

19.
triangle so that vertices are at ( - u , a), (0, O), and (u, a), M ( L ) =
I 3 1 f i + l n ( l + fi)
1023 k
21. ~-
13'568 384r = 1507.23 23. ~

5 15

See. 20.3
1. 0 3. 7 2 0 ~ 5. e - 5/2 7. -213 9. 3 13. 2 7 r 15. 32/9 17. 3/10
19. dydzdx 21. +"

See. 20.4

1. ( u ) 44
T/3 2
r2drdO; ( b ) 8n/9 3. (u) s s
o
71/2

n
2
r3sin26 d r d 6 ; (6) r 5. (a) 44
~ / 4 4se~a
drd6; (6) 4 1 n l f l 1 7- r 9. 4 r

11. e ( 8 - 3 f i )
3
13. ( 1 0 ~ ) / 3 15.
8r
-
3
17. 0 19. 54rk 21. - g p k < 2 , 0 > 23. z(fi-
3
1) 25. 3

Review Exercises
18
(h) fi- 1 3. (u) (b) 5- (a)
v5
~

X
ANSWERS TO ODD-NUMBERED EXERCISES/SELF-TESTS
A-4 1

17. - + -ln(2)
7 r 1 2fi
19. (esincl) - 1) 21. ( a ) (b) J I J z d m d y d x ; (c) fi--3
16 8 0 0

23. ( a ) Y (6) 4'


~-sz"zy dy dx; (c) 8/15 25. (a) (b) 4""lSin2 ecos z dydx; (6) e -1

27. e2-e+ 1 -ln(4) 29. 4 31. Q 33. 3~ 35. 8 1 ~37. T 39. -7 +r-
3 2
f i 41. (%,%)43.6 45. --3&
4
< 1,1>
47. (4 - 2fi)n 49. "[I
8
-+] 51. ba2m 53.
3
~ 55. 5 57. 59.
( 6 ~ -5 In 2)"
40
61. 1/8 63. 0 63. 47r 65. 5
32

7 1. (0, 0, 1/3)

+" 3.
47r
( a ) m/10; (b) T( ~ - 1); (6) &(25'3 - 1); 4. 27k/20

--1--

__-.-
r----
X ' ----A-..-
X
Y

Chapter 21
See. 21.1

1. 3. 5.
AY
f
A-4 2 ANSWERS TO ODD-NUMBERED EXERCISES/SELF-TESTS

5yZ
7. conserv.,f(x,y) = 2x2 - 2xy +-
2
+k 9. not conserv.
11. conserv.,f(x,y) =y2ex -y2x + y 3 + k 13. not conserv. 15. conserv.,f(u, u ) = u 2 sin u + eu cos u + k 17. conserv.,
f(s, I ) = t2es + s2es + k + x 2 + k 23.f(x,y, t) = xy sin t + k 25.f(x,y, z ) = x sin(y2) + sin(x2) + k
19. conserv., J(x,y) = t x 2cosy

See. 2 1.2

-53
1. 20 3. 0 5. ~ 7. e - 1 9. 217 13. 15. 3e - 3 17. & 19. &(lorn- 1) 21. Jcf(x,y)ds 23. JGxf(x,y)ds
20
25-Sc x 2 f ( x , - ~ds
)

See. 2 1.3
1. -11 3. 71 5. 1/45 7. does not exist 9. - 1

Sec. 2 1.4
1. -20

15.
f
3.0 5 . ( M ( x , y ) = 3 x 2 y 4 / 4 - x 2 y ) , 8
( Y + 1)dx
( x - 2)2 + +
( y 1)2
(2 -
<x
- 2)Z + ( y
+
+ 7 . ~ ~ ( x 2 + y 2 ) d x + ( y Z - x 2 ) d y = 09.0

1)2
equals -217 if (2, -1)
11.6
is inside C, 0 if outside
13.2471

See. 2 1.5
1. F = 84, C = 24 3. F = 0, C = - 1817 5. F = 6, C = - 15 7. F = - 210, C = - 168 9. F = C = 0
13. [curl(f F)] k = f(cur1 F) * k f , N - &M +
~~ ~~

Review Exercises
9 cos(1) 3 sin(1)
1. 4/3 3. - - -+ - 5. -9/2 7. 29/84 9. e - 1 11. conserv.,f(x,y) = xexy + k 13. conserv.,
10 2 2
f(x,y) = ex cos(y2) + x +y + k 15. Not conserv. 17. Not conserv. 19. conserv.,f(x,y) = y 2 ln(1 + e x ) - x 2 + k 21. -12
23. 1/2 25. -
71

6
+ ~
fir2

72
27. 0 29.
1 x
.
- 33. 617fi 35. 371 37. -
8
4517
2
39. 0 41. div F = 5, curl F = 0 43. div F = 1curl F = 0 45. div. F = - xyz sin(xy) + 9cos(xy); curl
m Y
k 47. F = 16, C = 8 49. F = 914, C = -258 51. F = C = 0 53. P = C = 0 55. F = C = 0
57. 0 59. 271

Self-Test
-5312
1. ~ 2. 0 3- - 194/3 4. 2592/5 5. 161,964/35 5 4627.5
15
i
Integration Tables

p+l
4. su"du
n + l
+
= - C (n # -1)

5. J-duU = In 1u1 + c

RATIONAL FORMS CONTAINING a + bu


6. s* = L[a + bu - a lnla + bull + c 2(a +" bu)2
--]+c
a
1
+ bu
7. s* + a bu = +[+(a + bu)2 - 2a(a + bu) + a2 lnla + bull +c
S m
du 1
- - -al n l-
a [ b ~ / + ~
8. s m udu = b2 -[-
l
a
a
+ bu + ln/a + bull + C 12. j"
9. ~
(a
~u 2 =
+
du L
bu)
[
b3
a + h - - a2-
a+bu
2alnla + bul + C
1 13. s u( a
du
+ bu)2
-
a(a
1
+ bu)
+ , 1l n l L (
a a +
bu
+ C

FORMS CONTAINING d z
14. S u w d u = *(36u
2 - 2a)(a +6 ~ ) +
~ 'C~
ifa>O
du
2 20.
15. J u ' m d u = 5(1562u2 - 12abu + 8a2)(a + bu)3/2 + C
ifa<O
16. s u " d x d u =
2u(a
b(2n
++
bu)3'2
3) - %
b(2n
z n s 3)
u n - l G d u+
17- s - =u=du( b u - 2 a ) a + C
a 3b2

: s
18.
du 2
= -(3b2u2
15b3
- 4abu +8 a 2 ) G+ C

~ -

FORMS CONTAINING u' f u'

24- S A a + u
1
= - arctan 2
a a
+C 26.
du

1
+C <a
25. s- - - arctanh if ( u /

FORMS CONTAINING d n
In formulas 27 through 38, we may replace

ln(u + d w ) by arcsinh?
a
lnlu + 1d
- by arccosh?
a In/
a+
u
1 by arcsinha

27. s- d F F 2 du = lnlu + 1d
- +c 29. s u 2 d m d u = ?(2u2 & a 2 ) d w
8

28. s d m d u = E2 d m & g l n2 l u + d m l +C -elnlu


8
+d
1
- +c
(tables continued on bock mdpapers)
CALCULUS WITH
ANALYTIC GEOMETRY
CALCULUS WITH
ANALYTIC GEOMETRY
DANIEL J. FLEMING
Department of Mathematics
St. Lawrence University

JAMES J. KAPUT
Department of Mathematics
Southeastern Massachusetts University

HARPER & ROW, PUBLISHERS


New York Hagerstown Philadelphia San Francisco London
Sponsoring Editors: Ann Ludwig, Sharmon Hilfinger
Project Editor: Eva Marie Strock
Production Manager: Laura Argento
Designer: Janet Bollow
Illustrator: John Foster
Cover Designer: Janet Bollow
Cover Illustration: Barbara Hack
Compositor: York Graphic Services
Printer and Binder: Kingsport Press

This work was published as a preliminary edition. Vol. 1 01976 by Daniel J.


Fleming and James J. Kaput; Vol. 1, part 2 01977 by Daniel J. Fleming and
James J. Kaput.
Calculus with Analytic Geometry 01979 by Daniel J. Fleming and James J. Kaput
Printed in the United States of America. All rights reserved. No part of this
book may be used or reproduced in any manner whatsoever without written
permission except in the case of brief quotations embodied in critical articles
and reviews. For information address Harper & Row, Publishers, Inc., 10 East
53rd St., New York NY 10022.

Library of Congress Cataloging in Publication Data

Fleming, Daniel J 1940-


Calculus with analytic geometry.
Includes index.
1. Calculus. 2. Geometry, analytic. I. Kaput,
James J., 1942- joint author. 11. Title.
QA303.F5835 515.15 78-24583
ISBN 0-06-382672-0
CONTENTS

Preface ix

Chapter 1 Preliminaries 1

1.1 Numbers, Inequalities, Absolute Value, and Completing the


Square 7
1.2 Review of Coordinate Geometry 9
1.3 Circles and Parabolas 17

Chapter 2 Limits, Derivatives, and Continuity of Functions 27

2.1 Functions: Ways of Describing and Combining Them 28


2.2 Limits: An Informal Introduction 39
2.3 The Derivative and Rates of Change 47
2.4 The Derivative and Slopes of Tangents 57
2.5 Limits, Continuity, and Differentiability 61

Chapter 3 Differentiation 73

3.1 Basic Rules of Differentiation 74


3.2 Differentiating Composite and Implicitly Defined Functions 85
3.3 Differentials 95
3.4 Higher Derivatives 102

Chapter 4 Applications o f Derivatives to Graphing 107

4.1 Describing the Behavior of Functions Using Special Limits 108


4.2 Describing the Behavior of Functions Using Derivatives-
Part 1 117
4.3 Describing the Behavior of Functions Using Derivatives-
Part 2 127
4.4 Extreme Values and the Mean-Value Theorem 133

Using Derivatives to Solve Problems and Understand


Chapter 5 Quantitative Relationships 143

5.1 Optimization Problems 144


5.2 Related Rates 155

V
5.3 Linear Motion 159
5.4 Using the Derivative to Help Understand Business and
Economics 162

Chapter 6 Integration 175

6.1 Antidifferentiation 176


6.2 Approximating Areas 185
6.3 The Definite Integral and the Fundamental Theorem of
Calculus 199
6.4 A Mean-Value Theorem and the Chain Rule for Integrals 210
6.5 Area Between Graphs of Functions, and Interpretations 216
6.6 Volumes of Solids of Revolution 223

Trigonometric, Exponential, and Logarithm


Chapter 7 Functions: Their Definitions and Derivatives 233
.._

7.1 Trigonometric Functions-Definitions and Properties 234


7.2 Derivatives of Trigonometric Functions 241
7.3 Inverse Functions and Exponential and Logarithm
Functions 25 1
7.4 Derivatives of Exponential and Logarithm Functions 260
7.4 Appendix: An Integral Approach to Logarithms 268
7.5 Applications of Derivatives of Transcendental Functions 269

Chapter 8 More on Transcendental Functions 279

8.1 Growth and Decay of Physical Quantities and Money 280


8.2 Integrating Transcendental Functions: Basic Formulas 288
8.3 Integrating Powers and Products of Trigonometric Functions 293
8.4 Inverse Trigonometric Functions 298
8.5 Integrals Involving Inverse Trigonometric Functions 305
8.6 Hyperbolic Functions and Inverse Hyperbolic Functions 309

Chapter 9 Important Ideas Revisited 32 1

9.1 The Formal Definition of Limit 322


9.2 Limits and Continuity 330
9.3 Why All the Fuss? 334

Chapter 10 A Brief Review of Analytic Geometry 339

10.1 Translation of Axes and the Parabola 340


10.2 The Ellipse and the Hyperbola 346
10.3 Rotation of Axes and the General Quadratic Equation 352

vi
Chapter 11 Techniques of Integration 359

11.1 Integration by Parts 360


11.2 Trigonometric Substitutions 365
11.3 Partial Fractions 370
11.4 Rationalizing Substitutions 378
11.5 General Strategies, Including the Use of Integration
Tables 382
11.6 Numerical Integration 386

Parametric Equations and More


Chapter 1 2 Applications of Integration 397

12.1 Parametric Equations 398


12.2 Arc Length and Area of Surfaces of Revolution 404
12.3 Centroids of Plane Regions, Curves, and Solids and Surfaces of
Revolution 413
12.4 Work and Fluid Pressure 422

Chapter 13 Polar Coordinates 433

13.1 Polar Coordinates 434


13.2 Polar Graphs: Intersections and Tangent Lines 443
13.3 Area and Arc Length in Polar Coordinates 448

Chapter 14 Indeterminate Forms and Improper Integrals 457

14. I Indeterminate Forms and LHospitals Rules 458


14.2 Other Indeterminate Forms 464
14.3 Improper Integrals 468

Chapter 1 5 Convergence of Sequences and Series 477

15.1 Sequences 478


15.2 Infinite Series-An Introduction 487
15.3 Positive-Term Series 496
15.4 Series with Alternating Positive and Negative Terms 505

Chapter 16 Power Series and Taylor Series 515


16.1 Taylor Polynomials 5 16
16.2 Power Series 525
16.3 Differentiation and Integration of Power Series-Taylor
Series 530
16.4 Miscellaneous Applications of Power Series 541

vii
Chapter 17 Vectors and Solid Geometry 549

17.1 Coordinates in Three-Space 550


17.2 Surfaces 554
17.3 Vectors in the Plane and in Space 563
17.4 The Dot and Cross Products 571
17.5 Lines and Planes 581

Chapter 18 Vector Valued Functions 59 1

18.1 Vector Functions 592


18.2 The Unit Tangent and Normal Vectors; Curvature 600
18.3 Motion 609

Chapter 19 Functions of Several Variables 62 1

19.1 Functions of Several Variables 622


19.2 Limits and Continuity 628
19.3 Partial Derivatives 633
19.4 Differentiability and the Total Differential 643
19.5 The Chain Rule 649
19.6 The Gradient and the Directional Derivative 656
19.7 Tangent Planes and Normal Lines 662
19.8 Extremes of Functions of Two Variables 666
19.9 Optimization Under Constraints and Lagrange Multipliers, with
Applications in Economics 672

Chapter 20 Multiple Integration 683

20.1 Double and Iterated Integrals 684


20.2 Applications of Double Integrals 693
20.3 Triple Integrals, with Applications 701
20.4 Multiple Integrals in Other Coordinate Systems 7 12

Chapter 2 1 The Line Integral 73 1

2 1.1 Vector Fields 732


2 1.2 The Line Integral 737
2 1.3 Independence of the Path 745
2 1.4 Greens Theorem 75 1
2 1.5 Applications of Greens Theorem: Divergence and Curl 757
Appendix A: An Introduction to Differential Equations 767
Appendix B: Tables 782
Solutions to Progress Tests S-1
Index 1-1

viii
PREFACE

I. THE GENERAL APPROACH


Origins Five years ago we started developing this text. Since then, preliminary editions
have been used at several different colleges and universities in instructional
contexts ranging from traditional lecture-discussion classes to independent
study situations and most everything in between, including several different
versions of PSI courses. We discovered that a text usable in such a wide variety
of circumstances required a number of features and a style that would makc thc
book even more effective in traditional lecture courses.

Level and Audience Our intended audience includes a broad range of students: those in science and
engineering, the life and social sciences, and economics and business. We
accommodate students with a wide range of backgrounds and abilities by
providing coverage of certain precalculus topics (e.g., trigonometry) that is
more complete than average and by providing a greater level of detail in our
explanations.

Style Among teachers of mathematics, the word intuitive represents almost any
attempt to present mathematical ideas in a nonaxiomatic manner, and our text
is surely intuitive in this sense. We also move beyond simple informality by
being systematically intuitive. For example:
1. We control the novelty at each stage, carefully avoiding (whenever possible)
introducing new ideas and new notation at the same time.
2. We emphasize how things tie together by extending old ideas into new
contexts and by taking old ideas back into familiar contexts.
3 . We deliberately examine each major idea from three points of view: nurncrz-
cal, graphical, and algebraic.

Geometric Growth Besides applying these three general principles in dealing with such notions as
Theme function, limit, derivative, differential, and integral, we also exploit as a basic
theme the concrete embodiment of all these ideas in the growth of geometric
objects. Each such instance is marked by the symbol 9 a t the left margin and
horizontal rules above and below such discussions. For example, we cover how,
as a function of its radius, a circle grows by circumferences, or a square
grows by the length of the sides in the direction of the growth. Such ideas are
discussed first in terms of rate of change, in relation to derivatives (Section 2.3),
then in terms of differentials (Section 3.4), and then in terms of the Funda-

ix
X PREFACE

mental Theorem of Calculus, which is seen to generalize all the concrete


geometric phenomena discussed to that point (Section 6.3). Similarly, we
motivate the proof of the Product Rule geometrically by examining the area of
a u ( x ) by u(x) rectangle (Section 3.1) and develop the method of Lagrange
multipliers as a consequence of geometric considerations (Section 19.9).

Rigor Complete and careful intuitive explanations cannot substitute for precision in
formal mathematical statements. We maintain precision in definitions and
theorem statements and in formal theorem proofs. In fact, Section 9.3 illus-
trates, in terms of the functions then available, Why All the FUSS?

Applications In keeping with the expansion of calculus into more and more of the under-
graduate curriculum, the current trend in calculus texts is to broaden the areas
of applications from science and engineering into other areas. Although we do
this, we concentrate our expansion in applications that students can relate to
without a great deal of special knowledge and, even more importantly, appli-
cations that illustrate how the ideas of calculus relate to everyday life. Such
applications are denoted by the symbol X and a vertical rule in the margin and
include (among other topics) an examination of
1. Why truckers drive so fast (Section 5.1)
2 . Why student apartment rentals can be so costly, even when vacancies exist
(Section 5.4)
3. The effects of various sorts of taxes and subsidies on rental costs (Section 5.4)
4. Present value of future income and the effects of inflation on such things as
the value of a winning lottery ticket or the real cost of a solar energy
installation (Section 8.1)
5. The effects of flashboards on a dam (Section 12.4)
6. The rationale behind government energy taxes (Section 19.9)
Furthermore, we show not merely that calculus is used to solve specific problems
but how calculus is used: as a conceptual tool for determining relationships and
revealing deeper symmetries. For example, the standard fence problem is
revisited to expose surprising symmetry (Section 5.1).

Calculators Calculators are helpful, but not necessary, when using this book. At several
important junctures we relate the mathematical idea under consideration to the
students calculator consciousness. For example, we discuss logical versus
numerical accuracy (Section 2.2) and the meaning of infinite limit (Section 4.1)
in terms of calculators.

History We value the history of calculus too much to relegate it to footnotes and
portraits. Thus we use the history of calculus at a deeper level, as a guide to the
organization of the subject (especially in the choice of geometric motivations)
and as the source of an occasional proof (e.g., the proof of LHospitals Rule,
Section 14.1).

11. ORGANIZATION AND SPECIAL FEATURES


Each chapter begins with a n introduction, which is a n overview of the chapter.
PREFACE xi

Objectives Each section begins with a set of objectives that tell the student what he or she
should be able to do upon completing the section. These objectives are partic-
ularly useful when reviewing a section for a n exam.

Progress Tests A very important feature of this text is one to four progress tests per section.
These tests give the instructor a collection of carefully constructed and
straightforward exercises with complete solutions, not just answers, and fur-
thermore, give
1. The student immediate feedback on whether what was just read was under-
stood-before the preceding material is combined with other material.
2. The instructor many convenient breakpoints, allowing for classes of different
lengths.
3. The instructor flexibility in level of coverage of topics. (Light or quick
coverage is achievable by assigning only the Progress Tests, for example.)

PSI Courses The combination of the objectives and progress tests is especially valuable in
any instructional format where students are expected to work through the
material without regular lectures. Coupled with the Instructors Resource Guide and
Test Bank, the objectives and progress tests eliminate much of the labor in setting
up such courses. (See the Instructors Resource Guide and Test Bank for further
details.)

Exercises Each section ends with a collection ofgraded exercises. Each chapter closes with a
collection of graded review exercises, along with miscellaneous exercises and a self-test.
The progress test problems and the many section exercises provide nearly
7000 exercises to choose from at virtually every level of difficulty. No one is
expected to assign or do all the exercises; this huge collection includes enough
exercises at any particular level and of any particular type to satisfy the great
diversity of needs that arise at different schools and even in a given course. The
miscellaneous exercises are more challenging, particularly beyond the early
chapters, when more material is available to draw upon. To ensure representa-
tive sets of both odds (with answers) and evens (without answers), we frequently
pair odd and even exercises of a given type. Students can buy a Student
Supplement that has complete solutions to odd-numbered section and review
exercises and self-tests. A Solutions Manual (separate from the Instructors Resource
Guide and Test Bank) has answers to evens and solutions to miscellaneous
exercises.

111. COVERAGE AND ORDER OF TOPICS


Although any instructor of calculus will recognize from the contents how the
chapters interconnect and can be rearranged, the following comments empha-
size and point out items not reflected in the contents. Throughout the book we
tried to be as flexible as possible in the reordering options because of the special
needs of different departments.
1. Student involvement in 6-6 work is not requested in the text before Chapter
9, but an instructor wishing to do so can move directly from Section 2.2 to
Section 9.1. (Note that Chapter 9 is not a throwaway theory chapter, as a
glance at the careful development of the idea of limit in Section 9.1 indi-
cates.)
xii PREFACE

2 . Special limits are informally introduced in Section 2.2 only to illustrate ways
that limits can fail to exist. Section 4.1 presents more complete treatment in
a more natural geometric context as the first step in a systematic approach to
describing the behavior of functions.
3. Chapter 6 can be used immediately after Chapter 3 if an early introduction
to antiderivatives and integration is desired. The approach to the definite
integral is based upon the monotonic case, which is more geometric and less
computational than the standard approach. This allows us to establish
integrability of all but infinitely wiggly functions, an example of which is
discussed in Section 9.3.
4. Derivatives of the transcendental functions can be included with the alge-
braic functions simply by following Chapter 3 with Chapter 7.
5. The exponential and logarithm functions are introduced in Section 7.3 by
building on students earlier experience, and the number e is introduced by
examining the slopes of 2 and 3 near x = 0. We then revisit e as a
limit-again as a consequence of geometric considerations. An appendix to
Section 7.4 then illustrates how the exponential and logarithm functions can
be developed from the integral of 1/ t , thereby illustrating the potency of the
Fundamental Theorem of Calculus.
6. Section 11.5 makes a strong case for learning the techniques of integration as
a means of making nontrivial use of integration tables.
7. Taylor polynomials are included in their natural context just before power
series; power series and Taylor series can be seen as direct generalizations.
8. Computer-generated graphics help illustrate the quadric surfaces in Chap-
ter 17.
9. Greens and Stokes Theorems are carefully developed in the two-dimen-
sional context to capitalize on students geometric experience.

Daniel J. Fleming
James J. Kaput
A book in the works as long as this one piles up more than its share of debts. We
first wish to thank the many students who helped improve earlier editions and
who taught us what is possible. Thanks to Ed Dubinsky, who contributed to an
early collection of class notes. We must express appreciation to several genera-
tions of typists, including those who married and bore children before we
finished the book.
We are also grateful to several generations of reviewers for their many helpful
suggestions: W. H. Beyer, The University of Akron; Jerald T. Ball, Chabot
Community College, Valley Campus; Robert A. Carman, Santa Barbara City
College; Bettye Anne Case, Tallahassee Community College; Arnold Dunn,
State University of New York at Potsdam; Garret J. Etgen, University of
Houston; William Hemmer, Cuyamaca College; Louis F. Hoelzle, Bucks
County Community College; Donald R. Kerr, Jr., Indiana University; An-
thony Peressini, University of Illinois at Urbana-Champaign; Evelyn Roane,
Northern Virginia Community College; Sister Madeleine Rose, Holy Names
College; Dwann E. Veroda, El Camino College; and Bert K. Waits, The Ohio
State University. We also thank Arnold Dunn for preparing the Student Supple-
ment and the Solutions Manual.
In addition, we thank Ted Sundstrom and Doug Kindschi of College IV,
Grand Valley State Colleges, for their help and cooperation, and Rick Miller
and Gerry Bradshaw of Clarkson College of Technology for their helpful
advice. Special acknowledgment to Paul Parente for his patient class testing; he
often received portions of text chapter by chapter, sometimes page by page,
usually late. Our greatest debt for direct help in putting together this book is to
our friend, colleague, and most vigorous and perceptive critic, Robert McCabe,
of Southeastern Massachusetts University. His constructive influence is absent
from few pages indeed.
Finally, we give tremendous thanks to Susan Kaput, our source of greatest
indirect help in producing this textbook. She fed, cared for, cleaned up after,
and otherwise served three little children (actually, there was only one when we
started) and two cantankerous and distracted authors.

xiii
TO OUR PARENTS
CHARLES AND LILLIAN FLEMING
AL AND EMILY KAPUT
1
2 CHAPTER 1 : PRELIMINARIES

Numbers, Inequalities, Absolute


Section 1.1: Value, and Completing the Square

Objectives: REAL NUMBERS

1 . Solve and graph inequalities in The set of real numbers, denoted R, contains the integers, the rational numbers,
one variable. and the irrational numbers. A rational number, a number that can be written
as the quotient of two integers, is always expressible by long division as a
2. Solve and graph equations and
nonending repeating decimal. On the other hand, a n irrational number is a
inequalities involving absolute
number whose decimal representation is nonending and nonrepeating. For
value.
example, 7~ = 3.1415926535 . . . .
3. Complete the square of The real numbers do not contain the complex numbers. As is common in
quadratic polynomials and apply calculus courses, we shall confine all our work to the real numbers R. As a result,
the quadratic formula. expressions such as fi or% will only be defined for nonnegative x .

IMPOSSIBILITY OF DEFINIKG DIVISION BY ZERO


The statements You cant divide by zero and its equivalent companion a/O
is not defined, which you have heard repeatedly since elementary school, are
not made as a matter of taste, simplicity, or convenience, but as a matter of
necessity. We have no choice f w e want the other basic rules of arithmetic to hold because
division by zero is inconsistent with these rules.
To see that this is true, we consider what happens if we attempt to define a/O
in the two cases a # 0 and a = 0.
First, suppose a # 0. If a/O were defined, that is, if a/O were a real number,
-
say b, then a/O = 6. But this would imply a = b 0 = 0, contradicting the
assumption that a # 0. Similarly, a contradiction occurs if a/O is defined to be
0. No matter what real number we try to assign the quotient a/O, we are led to
inevitable contradictions using the basic rules of arithmetic.

ORDER PROPERTIES OF THE REAL NUMBERS


Besides the numerical properties of R associated with addition, subtraction,
multiplication, and division, R also has a natural ordering. We use the standard
notation for this ordering: a <
b means a is less than b, a 5 b means a is less
than or equal to b, a >
b means a is greater than b, and so on.

1.1.1 Basic Properties of Order For any two .numbers a and 6 , either a < 6,
a = b, or a 6. >
1.1.2 a < + < +
b implies ( a d ) (b d ) for any number d. That is, adding or sub-
tracting any number to both sides of an inequality leaves the direction of the
inequality unchanged.

1.1.3 a <
b and c > <
0 implies ac bc. Multiplying or dividing both sides of a n
inequality by a positive number leaves the direction of the inequality un-
changed.

1.1.4 a <
b and c < >
0 implies ac bc. Multiplying or dividing both sides of an
inequality by a negative number reverses the direction of the inequality.

1.1.5 a < b and ab > 0 implies l / a > l / b .


1.1.6 a < b and ab < 0 implies l / a < l / b .
SECTION 1 . I : NUMBERS, INEQUALITIES, ABSOLUTE VALUE, COMPLETING THE SQUARE 3

INTERVALS
We can represent the real numbers by points on a straight line called the real
number line. We assign the number 0 to one point, which we shall call the origin,
and the number 1 to some other point to the right of 0, to be regarded as thc
positive direction. We place an arrow to the right of 1 on this line pointing in
the positive direction.
Using the distance from 0 to I as our unit of length, we determine the
positions of all the integers, which in turn determine the position of all the other
real numbers.

t I I I I I l l I w
-2-47 -1 O i l 4 2 3

We can now define and graph intervals of real numbers.

1.1.7 Ibefinition Suppose a <


b. Then
( a ) [a, b] is the set of all real numbers between a and b including a and b; that is
[a, b] contains all x with a 5 x 5 6.
It is called a closed interval and its graph is

( b ) (a, 6 ) is the set of all real numbers between a and b but not including a or b.
That is, (a, b) h a x < <
b. It is called an open interval and
its graph is

( c ) Analogously, [a, b ) contains its left endpoint but not its right endpoint, and
(a, 61 contains its right, but not its left, endpoint. These are called half-open
zntervals and their graphs are, respectively,

+
( d ) The znznite znterual [a, co) is the set of all numbers greater than or equal to
+
a. Hence, [a, co) contains all x with a 5 x. Its graph is

( e ) ( - co,a] is the set of all numbers less than or equal to a.


+
(a, co) and ( - 00, a ) arc defined similarly.
On occasion we shall refer to R as ( - 00, co). +
+
It is important to remember that the symbols 00 (plus infinity) and - cc
(minus infinity) do not represent real numbers and arc used solely as a nota-
tional convenience to indicate that the interval is unbounded in either direction.
We use the basic order properties listed earlier to simplify and solve ine-
qualities in much the same way as equations, but with special care when
multiplying or dividing by negative numbers. In the following example and
henceforth read * as implies.

Example 1 Solve - 2 x - 3 < 5 and graph the solution.


Solution -2x - 3 <5
* -2x < 8 by (1.1.2)
> -4 by (1.1.4)
JX

Thus the solution consists of all x in the infinite interval (-4, + co). Its graph is
a
-4 0
4 CHAPTER 1: PRELIMINARIES

PROGIkESS TEST 1

1. Test each property (1.1.2) to (1.1.6) using specific 2. Solve and then graph the solution o f
numbers. (a) x 4 + >
3 ( b ) -5x 10 <

-
(c) 3 <
2x - 5 ( d ) 6 - 5x 2x 9 < +

l a - hl
ABSOLUTE VALUE
>We base the idea of absolute value on the idea of distance. In particular, we
a b
assume that the distance between two points can never be negative.
Fig. 1.1
1.1.8 Definition Given two real numbers a and b on the number line, we define
I a - b I to be the distance between a and b. (See Fig. 1.1.)
1.1.9 Basic Properties of Absolute Value
( a ) I a + b I is the distance between a and -b [because I a +
b I = I a - ( - 6 ) I 1.
( b ) I a I 2 0 for all real numbers a, with I a I = 0 if and only if a = 0. ( I a I =
I a - 0 I is the distance between a and 0, hence is nonnegative; it is zero
precisely when a = 0, the only number that is zero distance from 0.)

i
aifa>O
(c) (a1 = 0 if a =0 so, in particular, I --a1 = la1
-aifa<O
[Note, for example, 1-31 = -(-3) = 3 = 131.1
( d ) 1 a b I = I a I I b I (This can be proved by examining all combinations of a
and b positive, negative, or zero-see Exercise 14, p. 8.)
(e) I a + bI 5 I a I+ I b 1 (Often called the triangle znequality.)
( f ) la1 = P [ F o r e x a m p l e , 1-31 = 3 = fl= d-:]
>
(9) For a 0, I x I 5 a if and only if -a 5 x 5 a, which is true if and only if
x is in the interval [ - a , a ] , pictured below.

-a 0 a

Test the triangle inequality using numbers of your choice.

INEQCALITIES INVOLVING ABSOLUTE VALUE


The key to solving most of the inequalities that we shall consider is to think in
terms o f distance on a number line.

Example 2 Solve Ix - 2 I 54 and graph its solution.


Solution We are looking for the set of numbers x whose distance from 2 is less than or
equal to 4. By simply counting off 4 units from either side of 2, we see that x
must satisfy -2 5 x 5 6; that is, the solution consists of all x in the interval
[ - 2, 61. Its graph is 0
-2 0 2 6

Example 3 Solve 12% + 3 I > 2 and graph its solution.


Solution We must reduce the given absolute value expression to one of the form I a - b 1,
so that we can interpret it on the number line in terms of distance. By [ 1.1.9(d)]
SECTION 1.1: NUMBERS, INEQUALITIES, ABSOLUTE VALUE, COMPLETING THE SQUARE 5

(2x + 31 = ( 2 ( x + +)I = 121 ( x + $1


= 2lx by [1.1.9(c)]

=2Ix-(-$)1 >2

Hence, by (1.1.3), I x - ( - 3 / 2 ) I >


1, so the solution consists of all numbers x
whose distance from - 3/2 is greater than 1. This includes all x in (- 00, - 5/2)
+
or (- 1/2, co). The graph of the solution is

-; 0

PROGRESS TEST 2

Solve the given inequality and then graph its solution.


1. I x + I1 2 5 2. 13x + 21 < 6 3. 1 2-
~ 61 < 9 4. 1%- 11 <o

COMPLETING THE SQUARE AND THE


QUADRATIC FORMULA
We shall see many occasions where it is useful to complete the square of a
quadratic expression. We review the process briefly. Its essence is to take half the
coefficient of the x term, square it, and add it to the expression, making
compensations to avoid changing the value of the expression.

Example 4 Complete the square of x 2 + 6x + 11.


Solution First write ( x 2 + 6 x + ) + 11. We then fill in the blank space so that the
resulting quadratic trinomial in the parentheses is a perfect square. Take half
4
the coefficient of x , = 3, square it, getting 9, and add it in the space provided.
Then subtract the 9 from the 11 to avoid changing the value of the total
expression:
(x2 + 6 x + 9) + 11 - 9 = (x + 3) + 2 0

If the coefficient of x 2 is not 1, we factor it out before applying the above


procedure, as indicated in Example 5.

Example 5 Complete the square of 3 x 2 - 6x + 1.


Solution We first write
3 x 2 - 6x + 1 = 3(x2 - 2x + ) +1
and then complete the square as before:
3(x2 - 2x + 1) + 1 - 3(1) = 3 ( x - 1)2 - 2
Note that we needed to subtract 3 because we had in fact added 3 1 when we -
added 1 inside the parentheses. 0
This technique can be applied to the general quadratic expression
ax2 + bx + c
to yield the quadratic formula (see Exercise 15).
CHAPTER 1: PRELIMINARIES

1.1.10 Quadratic Formula If ax2 + bx + c = 0, with a # 0, then


x =
- b t d n and conversely
2a

We refer to the values of the variable for which a n expression is zero as its
roots. By (l.l.lO),

>
( a ) If b2 - 4ac 0 , then ax2 + bx + c has two real roots.
( 6 ) If b2 - 4ac = 0, then ax2 + bx + c has a single repeating root.
(G) If b2 - 4ac < 0, then ax2 + bx + c has no real roots.
Note that our blanket agreement to deal only with real numbers leads us to say,
in the third case, that ax2 bx + +
c is never zero, by which we mean it is
never zero for real x. The number b2 - 4ac is usually called the discrimznant of
the quadratic expression ax2 bx + +
c.
Shortly, when solving more complicated inequalities, we shall need to
determine whether a given quadratic expression can be factored or whether it
is always positive or always negative. If we cannot factor the expression by
inspection, then we apply the quadratic formula. If b2 - 4ac 2 0, then we have
found the expressions real roots rl and r2 (with rl = r2 if b2 - 4ac = 0). In this
case we can factor ax2 bx + +c = a(x - rl)(x - r2). This is true because of the
fundamental fact from algebra that a polynomial has r as a root if and only if
it contains x - r as a factor. O n the other hand, if b2 - 4ac 0 , then the <
expression is never zero, is not factorable, and is always positive or always
negative.

Example 6 Factor the following expressions if possible. If the discriminant is negative,


determine whether the expression is always positive or always negative.
(a) x 2 - 5~ + 1 (b) 2x2 +x +2 ( c ) 3~ - X +4
Solutions ( a ) We cannot factor x 2 - 5x + 1 by inspection, so we apply the Quadratic
Formula (1.1.10):
f 5 + &

Hence
x=
+5* \/25--4
2
=I 5-$iii

(check this by
x -
2 2 multiplying)

( b ) Again 2 x 2 + x + 2 is not factorable by inspection, so by (1.1.10):


x =
-1 * d l -4.2.2
2.2

<
But 1 - ( 4 * 2 * 2) = - 15 0, so 2 x 2 x + +
2 is not factorable, hence
always positive or negative. But substituting x = 0 yields a value of 2, SO
+
2 2 x + 2 is always positive.
(c) We first put 3 x - x 2 + 4 in standard form: - x 2 + 3x +
4. But this can
be factored as ( - x + 4 ) ( x + 1). [The same factorization would result if we
used (1.1.10); check this.] 0
SECTION 1.l:
NUMBERS, INEQUALITIES, ABSOLUTE VALUE, COMPLETING THE SQUARE 7

SOLVING INEQUALITIES-THE CHART METHOD


In applying the calculus it is quite often necessary to determine when an
expression is positive or negative. We present a systematic chart method by
working through a detailed solution of the inequality
x2+x
x-2
<o
We factor the numerator and denominator of the expression as far as possible:
x2 + x -- (x)(x + 1)
x-2 x-2
and analyze each factor:
>
1. x is positive when x 0 and negative when x 0. <
2. x + 1 is positive when x >
- 1 and negative when x - 1 <
>
3. l/(x - 2) is positive when x 2 and negative when x 2. <
This information is summarized in a chart (Fig. 1.2), which contains a
horizontal line for each factor appearing in the numerator or denominator and
above the number line marked at each place where a factor changes sign. A
vertical line is drawn through each such point. For each factor we draw a
horizontal line dashed where the factor is negative, solid where the factor is
positive. We also circle points where the denominator is zero since they cannot
be part of the solution. [Note we wrote x - 2 rather than l/(x - 2).]
We then determine the sign of the original expression simply by counting
negatives: On an interval where a n even number of negatives (dashed lines)
appear, the expression is positive, whereas the expression is negative where an
odd number of negatives appear. This summary information is indicated at the
top of the chart by the plus or minus signs enclosed in parentheses.
In the interval (- 00, - 1) we have three negatives, so the product
1
x(x + l ) = x(x + 1)-
x-2 x-2
is negative. In the interval ( - 1,O) we have two negatives, so the product is
positive. Similarly the product is negative for x in (0,2) and positive for x in
(2, + a ) Thus
. the collection of numbers x for which
x2+x<o
x-2
is the set of all x in the intervals ( - 00, - 1) and (0,2).
You should get into the habit of checking your work. In this case a good
check is to insert a specific value from each interval and at least one not in these
intervals to see whether the result agrees with your answer. For example, in the
above problem substitute x = - 2 and x = 3, and observe that you get - 1/2
and 12, respectively, which is correct.

Example 7 Solve the inequality


6+x-xx2 >O
(x2 + x + l)(x + 4)(x - 6) -

Solution We factor as far as possible:


(3 - X)(X + 2)
(x2 + + x l)(x + 4)(x - 6)
8 CHAPTER 1: PRELIMINARIES

Since the sign of the entire expression is unaffected by any factor that is never
negative, a factor such as ( x - 6)' can be omitted from the sign analysis. Note,
however, that 6 cannot be part of the solution because the denominator is zero
for x = 6. Similarly, -4 cannot be part of the solution.
We are unable to factor x 2 x + +
1 by inspection; we therefore write its
discriminant 1' - 4 1 1 = -3- - <
0. Thus x' x + +
1 is always positive or
always negative. We substitute x = 0 a n d conclude that x 2 x + +
1 is always
positive. Hence we can ignore x 2 x + +
1 in the sign analysis. Thus the original
problem reduces to the chart analysis (Fig. 1.3) of

( 3 - ')(' + 1' with #6


x+4
[Do not be careless. T h e factor (3 - x ) is positive to the lgt of 3 a n d negative to
the right!]
Thus the solution is all x in the intervals ( - 00, -4), [ - 2 , 3 ] . Check this!
0

PROGRESS TEST 3

Solve the following inequalities using the chart method:

1. x ( x - 3)(6 - x ) <0

SECTIOS 1.1 EXERCISES

I n Exercises I to S, solue the inequality and graph its solution

I. ( 2 x + 61 ( 5 2. Ix -81 >$ 3. 15x - 101 2 0 4. Ix-41<4


5. I x + 31 5 6 6- 1 3 -
~ 11 2 0 7 . Solve the equations:
+
( a ) 1 3 ~ 41 = 2 (b) IX - 51 = 0

In Exercises 8 to 13, solve the given inequality.

(x + 1)(2x - 3 ) 2 0 ( 2 - 5 x ) ( x 2 - 3x - 4)
10.
(x2 + +
2x 1)(3x + 1) <O
8.
x+5
9.
+ +
( x 2 2x +
2)(x 2 )
5 0
( 2 ~ '- 6~ - 2O)(I -X) -

(xz + 6 x + 9)(x3 + x2) +


(x" 4 ) ( x 2 - 1) (x2 - 5 ) 2
12. >O 13. > O
11.
3x - 5
<O
(2x + 1)2(x - 4) (x4 - 4)(x + 3) -
14. Prove that I a b 1 = I a I I b I by showing equality for the 15. Derive the Quadratic Formula by completing the square
four cases involving combinations of positive and nega- of a x 2 + bx + c.
tive a and b. (The zero case is trivial.)
16. As a means of sharpening your algebra skills, prove that 17. Prove that 1 a * b I = I a I I b I using [ 1.1.9( f)]and the
(1.1.10) is valid by direct substitution of each of the roots properties of square roots.
into ax2 + bx + c.
18. The double inequality 4<
Ix - 2 I <
1 stands for the ( a ) Graph ;<
Ix - 21 <1
two inequalities 4<
Ix - 2 I and lx - 2 I <
1. Hence x ( b ) Graph 0 <
Ix - 21 <1
satisfies the double inequality if it satisfies both inequal- ( c ) GraphO< ( x - 2 1 <i
ities simultaneously.
SECTION 1.2: REVIEW OF COORDINATE GEOMETRY 9

Section 1.2: Review of Coordinate Geometry

Objectives: RECTANGULAR COORDINATES AND


THE MEANING O F GRAPHS
1. Use a coordinate system to plot
points in a plane and graph linear In a way similar to how we plot real numbers on a number line, we can plot
equations. orderedpairs of real numbers in a plane, as indicated in Fig. 1.4. We say ordered
pair because order does make a difference. That is, (a, 6) # ( 6 , a ) unless a = 6.
2. Determine the distance between
In Fig. 1.4 the horizontal line is called the x axis, the vertical line is called the
a pair of points in a plane.
y axis, and the intersection of these two lines, that is, the point labeled (0, 0), is
3. Determine the midpoint of a line called the origin. The four quadrants are labeled I to IV, respectively. We say that
segment. x andy are the coordinates of the point labeled by ( x , y ) , and the whole scheme is
called the rectan<qularcoordinate system f o r the plane or, more simply, the x;y plane.
4. Determine the slope of a straight
Notice that we use the same notation for the coordinates of' a point, say
line.
( - T , 2), as we do for the open interval ( - T , 2). The context will inevitably
5. Determine an equation for a make clear which meaning applies.
straight line, given enough Given an equation involving x and y , we can graph the set of points
information to define the line corresponding to the values of x a n d y that make the equation into a true
uniquely. statement. The graph of this set is called the graph of the equation, and such pairs
( x , y ) are said to satisfy the equation.

Example 1 Graph the equation y = I x I.


Solution An ordered pair ( x , y ) is part of the graph if the second coordinate (often called
t h e y coordinate) equals the absolute value of the first coordinatc (the x
coordinate). Hence the points ( I , l), (2, 2), (5,5), ( O , O ) , (-1, l), (-2, 2),
(-3, 3) are all part of the graph, whereas points such as ( I , - I ) , (0, l),
(-2, -2) are not. We plot the points of the graph in Fig. 1.5 and draw an
unbroken line through them. 0

IBJSTANCE AND iMlDPOINT


If (xl,yl) and ( x 2 , y 2 ) are points in the plane as in Fig. 1.6(a), recall that the
distance between these two points is given by the formula

1.2.1 Distance = d ( x l - + (y1 - y , J 2


This follows from the Pythagorean theorem.
The midpoint of the line segment connecting these points [see Fig. 1.6(b)] is
the point (X,y),whose coordinates are given by formulas

Y Y
I1
3. I
3.

Fig. 1.4 Fig. 1.5


10 CHAPTER 1: PRELIMINARIES

Y Y

t (Xl,Yl)
t

1.2.2 - X1+"2 - Y1 +YZ


x=- y =---
2 2

Example 2 Suppose that (xl,O) and ( x 2 , 0 ) are two points on the x axis. Show that the
distance between these points is the same as the distance used in defining the
absolute value I x1 - x2 1 .
Solution By (1.2.1) the distance between (xl, 0) and (x2, 0) is
d(X, - x 2 ) 2 + (0 - 0 ) 2 = d w
= 1x1 - ~ 2 1 by [1.1.9(f)] 0

PROGRESS TEST 1

1. Let A = (1,3) and C = ( 3 , 5 ) . Plot A , C, and the 6. Prove that the distance between the points (xl,yl)
midpoint B of line segment AC. Be sure to give the and (x2,y2)is unaffected if we interchange the roles
coordinates of B. of (xl,yl) and (x2,y2)in the formula (1.2.1).
2. Find the distance from A to C. 7 . Prove that, in general, the distance between the
3. Let R = (-1, 3) and T = (5, - 2 ) . Plot R, T, and midpoint given by (1.2.2) and the point (xl,yl) is
the midpoint S of line segment RT Be sure to give half the distance between the points (xl,yl) and
the coordinates of S. (X2,Y 2).
4. Find half the distance from R to T
5 . Find the distance from R to S. Does this answer
agree with that of Prob. 4?

SLOPES OF STRAIGHT LINES


1.2.3 Definition Suppose L is a straight line in the xy plane that is not parallel
to they axis, and suppose (xl,yl) and (x2,y2)are two distinct points on L (see
Fig. 1.7). Then by the slope of L we mean the number m given by

m = - Y2 -Y1
x2 - x1

Example 3 Determine the slope m of the straight line passing through the points ( - 3,2)
and (6, -1).
Solution Letting (- 3, 2) = (xl,yl) and (6, - 1) = (x2,y2),as in Fig. 1.8, we find that
SECTION 1.2: REVIEW OF COORDINATE GEOMETRY 11

-1 --z
m=
6 - (-3)

Notice that if we reverse the roles of (x,,y,) and (xz,y2)in Example 3 , letting
(-3,2) = (xz,y2) and (6, - 1) = ( X l , Y l ) , then

m=
2-(-1) -
--3 - _ -1
-3-6 -9 3
as before. It is irrelevant which point is chosen as (xl,yl) and which is chosen as
(x2,y2)as long asyou are consistent in applying (1.2.3). This is true because

Y2 -Y1 - - Y 1 -Yz
- -(Y1 - Y z ) -
Fig. 1.7 xz - XI -(xl - x 2 ) XI - x2

The word "straight" here has its zntuitiue meaning: the slope of a stra<ht line is ajixed
number independent of the points used to compute it. (See Exercise 32.) The idea of slope
is important in the calculus, so it is important that you get a good feel for what
it means geometrically. The line of slope - 1/3 in Fig. 1.8 slopes downward
from left to right; in fact, it decreases 1/3 unit vertically for each unit increase
in x. Put differently, it drops 1 unit whenever x increases by 3 units. In Fig. 1.9
are several graphs, each containing at least two straight lines whose slopes m are

I
Fig. 1.8

I
m undefined
iJ\
12 CHAPTER 1: PRELIMINARIES

indicated. Also indicated are points where these lines cross the x and y axes.
Check the slopes using these points.

1.2.4 Basic Properties of Slope


(a) In Fig. 1.9(a) we see that to have a negative slope means that the line slopes down to
the right; the more negative m is, the steeper the downward slope. (Compare
with m = -1/3 in Fig. 1.8.)
( 6 ) In Fig. 1.9(b) we see that to have apositive slope means that the line slopes up to
the right; the larger m is, the steeper the upward slope.
( c ) In Fig. 1.9(c) we see that two lines with the same slope areparallel. This is true
because for a given change Ax (read delta x ) in x,each line rises (or falls)
exactly the same amount Ay, so Ayy/Ax is the same for each line.
( d ) In Fig. 1.9(d) we see that a horizontal line has zero slope, and conversely. This is
true because, for any two distinct points (xl,yl) and (x2,y2)on a horizontal
line, y , = y,. This implies that
0
m = - Y2 -Y1 - = o
x2 - x1 x2 - XI
( e ) In Fig. 1.9(d) we also see that the slope of a vertical line is undefined. This is true
because for any two distinct points (xl,yl) and (x2,y2) on a vertical line,
x1 = x2. This implies that the quotient that would define the slope,

is not a real number.


(f)Suppose that L, has slope m, # 0 and L, has slope m,. Then L, is perpen-
dicular to L , (written L , IL,) if and only if m, = -l/m,. (Proof left as
Exercise 33.)

PROGRESS TEST 2

Give the slope of each of the straight lines in 1 to 3.


1. 2. 3.
Y Y

Y
X

4. Which of the lines in 1 to 3 has the most negative 7 . If a line slopes down to the right, with y decreasing
slope? at the same rate as x is increasing, what is the slope
5. Are any two of the lines in 1 to 3 parallel, and if so, of this line?
which two? 8. Give the slope of a line perpendicular to the line in
6. Are any two lines in 1 to 3 perpendicular, and if so, Prob. 7.
which two?

EQUATIONS OF STRAIGHT LINES


We saw that to graph an algebraic equation in x andy is to determine the set of
all points in the x-y plane whose coordinates (x,y) satisfy that equation. In
plotting these points we obtain a geometric picture of the algebraic equation. In
SECTION 1.2: REVIEW OF COORDINATE GEOMETRY 13

some cases it is possible to go the other way; that is, given a set of points in the
x y plane, or some geometric information that determines a set of points, we may
be able to derive an algebraic equation whose graph, in turn, equals the given
set of points.
In general, the connection between a graph and its equation is difficult to
make, but it is easy for straight lines because of the definition of straight line:
a line whose slope is always the same no matter which two points are used to compute it.
As you know from geometry, several kinds of information determine a
straight line: for example, a point and a slope, two points, and so forth. We now
list the familiar forms of the equation of a straight line L. Each of the forms ( a )
to ( d ) corresponds to a different way of uniquely determining a straight line.

1.2.5 Equations of a Straight Line


Point-slopeform: If the line L passes through ( x l , y l ) and has slope m, then L
Y has the equation
c Y -Y1 = m(x - x1)
[If L is to be straight, then the slope determined by ( x l , y l ) and any other
point ( x , ~must
) be m; that is 0, - y l ) / ( x - x l ) = m. This equation can be
put in the above form.]

Two-point form: If the line L passes through ( x l , y l ) and ( x 2 , y 2 )then


, L has the
equation

provided that L is not vertical, that is, provided x1 # x2. [This follows
from (a).]

Slope-interceptform: If L intercepts they axis at (0,b) and has slope m, then L


has the equation,
y=mx+b
[This is a special case of ( u ) with ( x l , y l ) = (0, b).] We shall refer to the point
Y (0, b ) as t h e y intercept. Sometimes we shall simply refer to b as t h e y
-f intercept.

( d ) Parallel-to-axes (special)forms: A vertical line through the point (G, d ) has the
equation
x=c
[A point ( x , y ) is on this line if and only if x = c, regardless of the valuesy
assumes.] Similarly, a horizontal line through (c, d ) has the equation
Y y=d
[A point ( x , y ) is on this line if and only ify = d.] Note that the equation of
they axis is x = 0, whereas the equation of the x axis isy = 0.

( e ) General form: Every equation that can be put in general form


ax + by + c = 0
so long as a and b are not both zero, is the equation of a straight line.
I x = c Conversely, every straight line has such a n equation. All the above forms
can be put in this general form.
14 CHAPTER 1: PRELIMINARIES

1.2.6 Definition Any equation that can be put in the form M by+ +c = 0 is
called a linear equation.
In Examples 4 to 9 we shall write an equation of a line that is determined by
some geometric information, leaving the equation in the general form, clearing
of fractions where necessary.

Example 4 Determine a n equation of the line with slope 3 passing through ( - 2, 7).
Solution Using the point-slope form, we obtain y - 7 = 3 [ x - (-Z)], so
3x-y + 13 = O . 0

p
p

Example 5 Determine an equation of the line passing through (5,Z) and ( - 6, 1).
Solution Using the two-point form, we obtain
1-2
y-2= -6 - 5 ( x - 5)

SO x - lly + 17 = 0.
Y

Example 6 Detcrmine a n equation of the line through (6, - 1) and (6,8).


Solution Both points have the same x coordinate, so the line is parallel to they axis, with
the equation x = 6. (Note that the slope of this line is undefined.) 0
Y

I
SECTION 1.2: REVIEW OF COORDINATE GEOMETRY 15

Example 7 Determine an equation of the line through (1,2) and (7, 2).
Solution Both points have the samey coordinate, so the line is parallel to the x axis, with
the equationy = 2. (Note that the slope of this line is zero.) 0
Y

Example 8 Determine an equation of the line through (1, -4) and perpendicular to L with
the equation x +
5y - 3 = 0.
Solution Solving fory to put L in slope-intercept form
1
y = --x+- 3
5 5
we see that L has a slope of - 1/5. Thus the line we are considering has a slope
of 5. Using the point-slope formula we obtain y - (-4) = 5 ( x - l), or
5x-y-990. 0
1'

Example 9 Determine an equation of the perpendicular bisector of the line segment joining
(-3,2) and (5,6).
Solution We begin by sketching graphs of the lines from the given information. Now L ,
has a slope of (6 - 2)/(5 + 3 ) = 1/2, so the perpendicular bisector has a slope
of -2. The midpoint of the line segment has coordinates (<y)where
16 CHAPTER 1: PRELIMINARIES

Examples 4 to 9 show that, as long as the given geometric information


uniquely determines a line, we can write a n equation for that line. Conversely,
a given linear equation can be graphed with little difficulty.

Example 10 Graph the equation 5 x + 3y = 10.


Solution We put the equation in slope-intercept form

y= 5
----)(.+-
10
3 3
Hence t h e y intercept is ( 0 , 10/3). Letting x = 3 , we find the corresponding
y = -5/3 3 + 10/3 = -5/3. We can now draw the graph through the two
6

points (0, 1 0 / 3 ) and ( 3 , -5/3). 0


Sometimes in graphing linear equations, as in Example 10, it is convenient to
determine the point where the graph intercepts the x axis by settingy = 0 a n d
solving for x . Given 5 x + 3y = 10, we would have 5x = 10, or x = 2, yielding
the point (2,O).
The point where a graph intercepts the x axis is called a n x intercept.
Another way to graph a linear equation is to put it in slope-intercept form,
read off they intercept, and then use the slope to draw the graph. I n the above
example, the slope -5/3 tells us that whenever x increases by 3 , y decreases
by 5.

PROGRESS TEST 3

1. Determine cquations for the lincs with the following 3. Graph the following equations:
properties: ( u ) y = 2x - 3
( u ) Through (3,5) with slope 4 + +
( 6 ) 2y 4x 5 =0
( 6 ) Through the two points (3,5) and (4,9) 4. Find a n equation for the line L , through (-5,3)
( c ) W i t h y intercept ( 0 , 2 ) and slope - 2 and perpendicular to the line L , that passes through
2. Find the slope a n d y intercept of the line with equa- ( 7 , O ) and (-8, 1). Leave your answer in point-slope
tion: form.
( u ) 6 x - 2y + 5 =0
- ~
( 6 ) 3x = 4y + 8

SECTIOX 1.2 EXERCISES

1. How far apart are: 2. Express your solution to I(b) in terms of absolute value.
(a) (4,;) and ( - 6 , -5) ( b ) (0, -2) and ( 0 , 7) (Note that they axis is a number line.)

W h a t is the midpoint of the line segment joining:


8. (bf) and (-6, -5) 4. (5,O) and (15,O) 5. ( 0 , 6) and (6,O) 6. (7, 7) and the answer to (5)
7 . ( a ) What is the slope of the line through the two points 8. ( a ) What is the slope of the straight-line through
in Exercise 57 ( - 1, - 1) and (3, 1 l)?
( b ) What is the equation of this line in slope-intercept ( b ) Does ( - 1, -1) lie on the straight line through
form? ( - 3 , -7) and (3, I l ) ?
(c) What is the slope of the straight line through
9. Which is the steepest upward sloping line in Exercise 8? (-1, -1) and (3, 15)?

For Exercises 10 to 1 4 j i n d the equation of the line as described.

10. With slope 3 through the point (4,5) 1 1. Through the points (4, 1) and (5,5)
13. With slope - 2 and y intercept ( 0 , l ) 1% Vertical through (3,4)
14. Horizontal through (3,4j 15. Write your answer t o Exercise 11 in:
SECTION 1.3: CIRCLES AND PARABOLAS 17

( u ) Point-slope form (6) Slope-intercept form 17. Find the equation of the line through (0, 1) with nega-
(c) General form tive slope whose x a n d y intercepts are the same distance
16. Write the equation of the line with y intercept 3 and from the origin.
parallel to the line with equation x +
4y - 12 = 0.

In 18 to 23 graph the equatzon.


18. In Exercise 10 19. In Exercise 11 40. In Exercise 12
2 1. In Exercise 17 22. 2 x - y = 4 , 23. y - 2 x + l = O
24. Write your answer to Exercise 10 in ( a ) point-slope form,
(6) slope-intercept form, (c) general form.
25. ( u ) Determine an equation of the line through (0, 1)
whosey coordinate increases by 1 whenever the x coordi- (c) Does part ( b ) prove that the midpoint of the hypote-
nate increases by 1. nuse is equidistant from the vertices?
(6) Arc thc lines in Exercises 17 and ( u ) perpendicular? 29. Determine three points on the line segment joining ( 2 , 8 )
(c) Graph each on the same coordinate system. and (10,2) that subdivide the segment into four pieces of
( d ) Give the equation in point-slope form of the line that equal length.
is parallel to that in part ( a ) and passes through the 30. Prove that the diagonals of a square are perpendicular.
origin. 3 I. T h e three points ( - 1, 7), ( 2 , 8 ) and (10,2) are three of
26. Show that the diagonals of a rectangle bisect each other. the vertices of a parallelogram.
(You may assume that one corner is at (0,O) and two ( u) Name a fourth vertex in the first quadrant.
sides are on the axes.) ( b ) Name another possible fourth vertex in the second
27. Given the three points (-3,4), (3, 2), and (12, - 1): quadrant.
( u ) Show they lie in a straight line using slopes. 32. Prove that the slope of a straight line is independent of
(6) Show they lie in a straight line using distance. the points used to compute it using similar triangles as
(c) Give an equation for this line. follows. Let (Iland Qz and PI and P2 be two pairs of
28. ( a ) The points (2, 8 ) , (10, 2), and (2, 2) form the vertices points on a line 1.
of a right triangle. Show that the perpendicular bisectors ( a ) Show that the right triangles whose hypotenuses are
of each side meet in a single point. What is this point? the line segments QIQz and PIPz are similar.
(6) Show that the situation in part ( u ) holds for any ( b ) Show that the appropriate ratios (giving the slope)
right triangle. (You may assume that the two legs lie on are equal.
the axes.) 33. Prove [1.2.4(f )I.

Section 1.3: Circles and Parabolas

Objectives: CIRCLES

1. Graph an equation of a circle, We now know how to write a n equation for a straight line and, conversely,
giving its center and radius. which kinds of equations have straight-line graphs. The next simplest kind of
geometric figure is the circle. The following example develops a n equation for a
2. Determine an equation for a circle with cgiven center and radius.
given circle.

3. Graph an equation of a
parabola, giving its axis of
symmetry, extreme point,
intercepts, and concavity.

Example 1 Determine a n equation for the circle with center (4, 7) and radius 3 .
Solution As shown in Fig. 1.10, a point ( x , y ) is on the given circle if and only if its
distance from (4, 7) equals 3. That is, by the distance formula (1.2.1),
d(x - 4)2 + ( y - 7 ) 2 = 3
Squaring both sides of the equation, we obtain a simplified equation of the
circle:
(x - 4)2 + ( y - 7)2 = 9 0
18 CHAPTER 1: PRELIMINARIES

Fig. 1.10

The same method applies in the general case to give the following theorem.

1.3.1 Theorem The equation of the circle with center (h, k ) and radius r(r > 0) is
(x - h)2 + ( y - k)2 = r 2
Note that the equation in Theorem (1.3.1) is equivalent to the distance equation
d(x - h)' + ( y - k)' =r
because r > 0. [This means that every solution of (1.3.1) is also a solution of the
distance equation.]
Given the geometric information describing a circle, we are now able to
determine its equation. We can also reverse the process, determining the circle
when given the equation.

Example 2 Describe the circle with equation ( y - 1)2 + (x + 5)' = 17.


Solution Regarding x + 5 as x - (-5), we see that this is an equation of the circle with
radius fiand center (-5, 1). 0
Often the information determined in Example 2 is less obvious. By multi-
plying out the factors of the given equation and combining all terms on the left
side, we obtain the equivalent equation
x2 + lox +y2 - 2y +9 =0
We could have been asked to describe the graph of this equation. However, by
separately completing the square of the x terms and then they terms, we get
(x2 + lox + 25) + (y' - 2y + 1) + 9 = 25 + 1
(We added 25 and 1 to the right-hand side of the equation to balance what
we added when we completed the square on the left.)
Finally, we write the expressions in parentheses as perfect squares and
subtract 9 from both sides to recover the original equation
(x + 5)' + (y - 1)' = 17
which we know is the equation of the circle with radius fl and center
(-5,l).
SECTION 1.3: CIRCLES AND PARABOLAS 19

This technique is quite limited in scope, giving the equation of a circle only if
the coefficients of x 2 andy2 are equal (in which case we can divide through by
this coefficient, making the coefficient of the squared terms one) and only if the
number resulting on the right side of the equation (after completing the
squares) is not negative. If the coefficients of x 2 a n d y 2 were different, then the
graph would not be a circle, but an ellipse (if they had the same sign) or a
hyperbola (if they had opposite signs). These cases are handled in Chap. 10.

PROGRESS TEST 1

1. Give the equation of the circle with 2. Describe the graph (if it exists) of the equation
( u ) Center (1,4) and radius 2 +
(u) x 2 4x + y 2 - 6y +
1=0
6
(6) Center ( - 3 , 3 ) and radius +
(b) y 2 x2 + 6x +12y 3 = 0+
(c) Center ( m , n ) and radius t +
(6) x 2 2x + y 2 - 5y +
40 = 0

PAICABOLAS
Our starting point is the graph of the equation
y = x2
In the absence of further information, we form the graph by first filling in a
table of data, including sufficient entries to reveal the general shape of the
graph. We then plot the data and draw a n unbroken curve through the
resulting points (Fig. 1.1 1).
We plotted additional points at x = t1/2 near where the graph seemed to
be changing direction.
The graph of
y=x2+L
is the graph ofy = x 2 slid c units vertically (up if c >
0, down if c 0). (See <
Fig. 1.12.) A fact visually obvious in all these graphs is the vertical symmetry
about they axis.

lo--

X y = x2 (-3,9) c 4 ( 3 . 9)
0b
-3 9 3
-2 4
* 5-- a
-1 1
0 0 1-2, 4) Q 6 (2,4)
1 1 4

2 4
3 9
4 16
1 41
-7 ~
20 CHAPTER 1: PRELIMINARIES

y=x2-5

4
-1
-4
-5
-4
-1
12 4
4 19 11

(a)

Fig. 1.12

We now graphy = 2x2,y = &x2,y = - x 2 , a n d y = -2x2, all on the same


coordinate axes. (See Fig. 1.13.)

X y = 2x2 y = +"XZ y = -22 y = -22

-3 18
~
9
10 -9 - 18
4 -8
-2 8 ~

in -4
-1 2 -
1
10
-1 -2
0 0 0 0 0
1 -2
1 2 ~

10 -1
4
2 8 ~

10 -4 -8
9 -9 - 18
3 18
~

10
I6 - 16 -3 2
4 32 10
SECTION 1.3: CIRCLES AND PARABOLAS 21

From these graphs, we can see that the graph o f y = ax2 opens upward if
a > <
0 and downward if a 0. The graph is flatter when a is small than when a
is large. Again, the graph of
y = ax2+ c
is merely the graph ofy = ax2 slid vertically by c units, and any such graph is
symmetric about t h e y axis (the line x = 0).
Algebraically, symmetry about they axis means that the same value ofy results when x
is replaced by -x.
As our last case we consider graphs in the general case
y = ax2 + bx + c

X y =x2- 5x +6
-3
-2
30
20
Since the graph ofy = x 2 - 5 x +
6 intercepts the x axis when y = 0, we
-1 12 could have determined its x intercepts by solving the quadratic equation
0 6 x - 5x +6 = 0 [by factoring or applying the quadratic formula (1.1.1 O)]. Of
1 2 course we could determine they intercept by setting x = 0. In fact both of these
2 0 remarks apply to graphs of any equation that can be put in the form
3 0
2
4
5 6 y = ax2 + bx +c
6 12
1
However, as yet we have no general technique that would guarantee that the
+
~

2 -4
graph of y = x 2 - 5 x 6 , for example, reaches its minimum at the point
(a) ( 5 / 2 , - 1/4). Although the ability to determine and deal with such critical
points is among the central topics of differential calculus, we can use the idea
of completing the square to handle not only this particular case but the general
case as follows:

1
(b)
(2, 0)

(25,
(3, 0)
tj\ATi+(;2)

-a,
~ c -
[ x + b/(2a)I2. For a

a <
+
The second term merely slides the curvey = a [ x b/(2u)I2 vertically by
b2/(4a) units. The variable term is the product of a with a perfect square
> +
0, the nonnegative term a [ x b/(2a)I2 reaches its
minimum, namely zero, when x + b/(2a) = 0 , that is, when x = -b/(2a). For
+
0, the nonpositive term a [ x b/(2a)I2 reaches its maximum, namely zero,
Fig. 1.14 again when x = -b/(2a). Proved most of 1.3.2:

1.3.2 Theorem The graph of the equationy = ax2 bx + + c reaches its extreme
point when x = -b/(2a). This is a minimum if a > 0, and a maximum if
a < 0. In either case, the point ( x , y ) with x = - b / ( 2 a ) is often called the vertex
of the parabola. Furthermore, the graph is symmetric about the vertical line
x 1-b/(2~).
ProoJ The symmetry statement follows because the graph o f y =
a [ x + b/(2u)l2 + [ c - b2/(4u)]behaves exactly as does the graph of the equa-
22 CHAPTER 1 : PRELIMINARIES

tion Y = a X 2 ,+[c - b2/(4a)] in the X - Y plane; here we relabel x + b/(2a)


as X .
Applying (1.3.2) to the equation y = x2 - 5 x +
6 of Example 3, we get
-b/(2a) = -( -5)/(2 -
1) = 5/2, which is the value of x we suspected yielded
the minimum point, namely (5/2, - 1/4).
1.3.3 Definition The graph of any equation that can be put in the form
y = ax2+ bx + c with a f 0 is called a parabola. It is said to be concave up if
and concave down if a 0.<
The line x = -b/(2a) is called the axis of symmetry of the parabola.
Notice that the axis of symmetry does not depend on the constant c; the
effect of c is only in the vertical height of the graph. Equations of the form
y = ax2 + c have axes of symmetry x = - 0 / ( 2 a ) = 0 since b = 0.

Example 4 Sketch the graph ofy + x 2 = x + 12.


Solution First we put the equation in the formy = - x 2 + x + 12. The axis of symmetry
is x = - 1/[2( - l)] = 1/2, they intercept is 12, and the x intercepts (using the
Quadratic Formula) are
- 1 k d r n -1*7
x= -
- = -3,4
2 -2
Finally, the graph (Fig. 1.15) reaches a maximum when x = 1/2, so

y = -
(3 ;
- +-+12,=12-
4
1

We can now sketch the graph. If we want more accuracy, we can plot more
points. 0
Y

Fig. 1.15
PROGRESS TEST 2

Sketch the graphs of the following equations using the


procedure of Example 4, indicating the axis of symmetry
with a dashed line and plotting two additional points as
a check.
1. y + 2x2 = 7x - 3 2. y = x 2 + 2x - 35 8. y = 2 x 2 + x + 1
SECTION 1.3: CIRCLES AND PARABOLAS 23

ADDITIONAL REMARKS
Notice in Prob. 3, Progress Test 2, that (1) the graph has no x intercepts,
(2) the equation 2 x 2 x + +
1 = 0 has no solutions, (3) the discriminant
1 - 8 = - 7 is negative, and (4)the graph is entirely on one side of the x axis.
All four facts abouty = 2x2 x + +
1 are equivalent and support one another;
they cannot disagree.
The contents of our entire discussion regarding the graphing of parabolas
can be turned on its side in the sense that the roles of x and y can be
interchanged to give a n analogous collection of statements.
1.3.4 The graph of x = ay2 + ly + c is a parabola with horizontal axis of symmetry
>
y = - b/(2a), concave right (opening to the right) if a 0 or concave left if
<
a 0, reaching its extreme point wheny = -b/(2a), having x intercept c, and
y intercepts
- b k d-
Y = 2a

Example 5 Sketch the graph of x = -y + 3y + 3, indicating the axis of symmetry with a


dashed line.
Solution As shown in Fig. 1.16, the axis of symmetry i s y = -3/2(-1) = 3/2; the
parabola is concave left, since a = - 1 <
0. The x intercept is 3, and t h e y
intercepts are
- 3 * - ~3 T f ~i
Y= -
-2 2
The extreme point has x coordinate x = -(3/2)2 + 3(3/2) + 3 = 21/4. 0

Y
t

Fig. 1.16
We shall use the phrase turn . . . on its side repeatedly in thefuture in exactly the same
way we have just done-to mean that we interchange the roles of x and y.
SECTION 1.3 EXERCISES

Graph Equations 1 to 3 using tables of data.


1. y = 3 x 2 +x -4 2. y = x 2 + 3 x + 5 3. x =y2 - 4y

For Exercises 4 to 13: ( a ) put the equation into an appropriate form and explain whether its graph is a circle, a parabola, or neither; ( b ) If it is
a circle, give its center and radius, and f i t is a parabola, give its axis f symmetry, its concavity (up or down, lt$ or right) and its extreme point (as
an ordered pair); (c) if it is a circle or parabola, sketch its graph.
4. x=y2+2y+l 5 . y - x 2 = x - 20 6. x 2 + y 2 - 9 = 0
7. x - 4 ~ + y ~ + 6 y - 1 2 ~ 0 8. x + y 2 - 4 y + 4 = 0 +
9. 4xz - 12x 4y - 6 = 0
10. y = x 2 + +
2x 2 11. x 2 + y 2 +
36 = 0 12. y = 4 x 2 +
52
13. x - y 2 +
2y = 1
24 CHAPTER 1: PRELIMINARIES

For the equations given in 14 to 21 that do not have graphs, explain why. Otherwise, graph the equation.
14. y = x 2 - 1 15. x = y 2 - 1 16. x2 = 4 - y 2
17. (x - 2) (y + + 1) = 49 18. x 2 - 4x + y 2 + 2y - 44 = 0 19. y - 3x2 - l l x +4 =0
20. x +
( y - 5)2 + 20 = 0 21. x2 +y= 0

CHAPTER EXERCISES

Review Exercises
Solve inequalities I to 15 and graph the solution on the number line.
1.4x- 1 5 2 2. x + 6 < 7 x - 3 3. 2x + 1 2(3x> - 10) 4. 4x - 5 2 2(x - 2)
5.3x + 7 -20 > 6. Ix - 51 < 5 7. Ix - 31 > 2 8. 12x + 61 2 3
9. 11 - 2x1 5 4 10. +
Ix 51 2 0 11. -lx+51 2 0 12. 14x - 71 12 >
13. 12 - 5x1 2 12 14. Ix- 11 2 -1 15. Ix- 1 ) < 0

Complete the square of quadratic expressions 16 to 24

16. x 2 - 6x+ 5 17. xi: + 4 x +8 18. xz-x-2


19. 2 x 2 - x - 3 20. - x 2 + x + 1 21. 2x2 - 2 f i x + 6
+
22. 8x2 2x - 15 23. 3x2 +2x - 1 24. 15x2 - 13x + 2

Solve inequalities 25 to 30 using the chart method, and graph your solution on a number line.

+ 1 ) ( 2 + l ) ( x - 1) 5 0
25.
( 2- x - 6)(2x2
+ +
( 2 2x l)(x 3)
-x)>O
(3 - 2 x ) ( - x 2 +
5x - 6)
+ 26.
(2x + 1 ) ( 9 - 1)(x - 4)

(.2
(x - 3)(2x - 1)
+
1)(x4 + 1) >
27.
(x3

x5(x2
2x - 3x2
+ l)(x2 - 4)2 > O
30.
28.
x ( x 2 - 4)(x 5) + <O
29. ( x 2 + 2)(x6 6) + -(x - 1)2

For the pairs of points in 3 1 to 34, determine ( a ) the distance between them, ( b ) the midpoint of the line segment joining them, (c) the slope of
the straight line through them, ( d ) the equation o f this straight line in general form, and ( e ) the equation of the perpendicular bisector of the line
segment joining the points.

31. (3,6), (5, 1) 32. (-4,5), (5,7) 35. In general form, give the equation of the line
33. (-4, -2), (-7, -6) 34. (-5, -4), (1,4) ( a ) With x intercept ( 3 , 0 ) ,y intercept (5,O)
( b ) Through (3,O) with slope 2
( 6 ) Parallel to the line in part ( 6 ) through (1, 5)

I n 36 to 3 9 give the equation o f t h e circle with:


36. Center (1, 2) and radius 4 37. Center (5,6) and radius 1/3
38. Center (-2, -4) and through the origin 39. Center (-2, -4) and tangent to the x axis

By completing the squares of the x and y terns in 4 0 to 44, describe the graphs of the circles with the given equations.

40. x 2 + y 2 - 8x +
2y - 24 = 0 41. x +y + 12x - 16y = 0
42. 8x2 + 8y2 - 72x - 32y 96 = 0 + 43. 2x + 2y - 12x + 16y = 0
44. 7x = 5y - x2 -y2 44 +
For each equation in 4 5 to 70, tell whether its graph is a straight line (ifso, give its slope andy intercept), a circle ((so, give its center and radius), a
If
parabola (zfso, give its axis of .ymmetIy, extreme point, and intercepts), or none of these. the graph is a straight line, circle or parabola, sketch it.

45. 5x +
2y2 = 36 46. y = - 2 x + 1 47. y = 3x 2 +
48. y = x(x - 5) 3 + 49. y +
2y - 6x - 20 = 0 50. y 2 - 2y +
8 x - 39 = 0
51. y - 4 = ~ + 4 52. y = 3x2 x-4 + 53. y = x 2 3x 5 + +
54. 3 +
x = 4 -y 59. x - 4x - 2y 10 = 0 + +
96. 5x2 5y2 - 7y - 26 = 0

57. -
y--l=2 58. y = $x - 5 59. x = 46.1
x-3
60. ~ 8~ - 1 = 0
1 3- 61. (x - 3) - ( y - 2) = 25 62. x2 + y 2 - 9 x +y + 2 = 0
1
63. - = y 2 64. y = v + 3 65. x = - ( y - 2) + 4
x?
SECTION 1.3: CIRCLES AND PARABOLAS 25

66. y = 4y2 - 3x +1 67. 3 x -y = 6y +9 68. 4x - 3 = y


69. x 2 +
2y2 = 49 70. 4 x - y + 2 = 0

Miscellaneous Exercises
Sketch the graphs of equations I to 4:
1. y = ) 3 x - 2) be proved later), determine an equation (in point-slope
2. y = 3 x - 2 form) for the tangent to the circle x 2 + y 2 = 25 at the
3. (X + +
- 3) ( J 5) - 25 = 0 point (3,4).
4. y - 4~ X+ + + 1 2 ~ 31 = 0 10. Repeat Exercise 9 in more generality. That is, determine
5 . Determine three points on the line segment joining anequationfor theline tangent tox + y 2 = r 2 a t (xo,yo)
(1, -1) and (13, 15) that divide the segment into four (where, of course, x i + y i = r 2 ) .
equal parts. 11. ( a ) For what value of b does x + by = 3 go through the
6. What is the smallest number reached by the expression point (4,5)?
x 2 - 3 x + 5? ( b ) For what value of c does x + + 2y c = 0 go through
7 . Solve: ( a ) 13x +
81 29 ( b ) 1x1 -1< the point (4,5)?
( x + 2)(x - 1)2(x2 + x - 6) 12. What is the slope of any line parallel to a line perpen-
O dicular to the line through ( - 3 , -7) and (3, l l ) ?
(c)
(x - 3)(x2 + 2x + 2)(x + 9) 13. What is the slope of any line with equationy = m ( m a
8. Prove that the diagonals of a rhombus are perpendicular constant)?
bisectors of one another. 14. Where do the linesy = 2 x + 3 and 2y = 2x + 3 cross?
9. Assuming that the tangent to a circle at (xo,yo)is per- 15. Do the linesy = 2x + 3 and 2y = 4x + 4 cross? Explain.
pendicular to the radius through (xo,yo)(a fact that will

SELF-TEST +
Chapter 2 %

27
28 CHAPTER 2 : LIMITS, DERIVATIVES, AND CONTINUITY OF FUNCTIONS

Functions: Ways of
Section 2.1: Describing and Combining Them
Objectives: DEFINITION OF FUNCTION
1. Describe a function Few mathematicians or users of mathematics would dispute that the most
algebraically, geometrically, and important and useful idea of all mathematics is that of function. To use
numerically. mathematics is to use functions. A function is used to describe how one quantity
2. Recognize when a given depends upon another, ranging from simple verifiable situations (the area A of
equation in two variables defines a square depends upon the length x of its sides according to the formula
one variable explicitly as a function A = x2), to complex hypothetical situations (the probability P of developing
of the other and when one variable lung cancer depends upon the quantity x of tar taken into the lungs according
is defined only implicitly as a to some formula). Much of the information (facts, relationships, laws) expressed
function of the other. by any science, be it a physical, biological, or social science, is expressed in the
3. Describe the natural domain and form of functions.
the range of a given function. We saw many functions in Chap. 1 given in the form of equations, such as
4. Form the sum, difference,
product, quotient and composition y=x2+2x+1
of a pair of functions. Here the quantityy depends upon x. However, not all equations describe
functions. The following operational definition will clarify the distinction.

2.1.1 Definition A function is a rule by which each number in a given set of


numbers is transformed into another UNIQUELY DETERMINED number. This
given set of numbers to which the function applies is called its domain. The set of
numbers rebulting when the function is applied to numbers in the domain is
called its range.

Consider the following function whose domain is R, the set of all real
numbers, and which is defined by the following rule applied to a real number:
square it, add twice the original number, and then add 1 to the result.
The number 3 is transformed by this function to the uniquely determined
+
number ( 3 ) 2 (2) * (3) +
1 = 16. The number -2 is transformed to the
number
(-2)2 + (2) - (-2) + 1 = 1
This function transforms any real number x to the unique number
x2 + 2x + 1
Obviously, this is the same function mentioned earlier in the form
y = x2 + 2x + 1
When a function is given with one variable, say y, equal to an expression
involving another variable, say x , we refer to x as the independent variable and y
as the dependent variable (becausey does depend upon x). Writing a function in this
way emphasizes that it relates two variables-x and y in this case.
However, as we shall see later, not every equation that givesy in terms of x
givesy as a function of x. The crucial matter is thaty be uniquely determined by x.
It is useful to view a function as a rule for transforming numbers. In such a case
the function is usually given a name, with the most popular single-letter name
for functions being simply $
Iffis the name of the function, we can describefcompletely by describing
what f does to a typical number x in its domain. The usual way of stating that
the function f is applied to a number x is f(x), read fof x, or fapplied to
x. If we call the original function previously introduced f, then the result of
applying f to a typical number x in its domain R is given by the equation
SECTION 2.1: FUNCTIONS: WAYS OF DESCRIBING AND COMBINING THEM 29

f ( x ) = x2 + 2x + 1
The results of applying f to the speczfic numbers we began with are

f ( 3 ) = 16 f(-2) =1
We shall often make statements such as consider the functionf(x) = 4x 1+
with domain R. Strictly speaking, the name of the function itself isf, andf(x)
is the result of applyingf to a typical number x in its domain. But since any
function is known by what it does to the numbers in its domain and since x represents any
number in its domain, there should be no confusion in referring to a function
by referring to what it does to a typical number x .
It may turn out that functions which look different are really the same. For
example,
f(x) = x2 + 2x + 1 g(x) = (x + 1)2 h(w) = (74 + 1)2
all with domain R, represent the same rule. Given any real number, the result of
applying all three functions is the same. We even could have written

f( 1 = ( l2 + 2( 1 + 1
requiring only that the number substituted into the parentheses be a real
number. The function is defined by the rule and not by the letters used to
represent it.

Example 1 Given that the area of a circle is a function of its radius r with A ( r ) = m2,show
that ( a ) A increases by a factor of four when the radius doubles and ( b ) A
increases by a factor of b2 when the radius increases by a factor of 6.
Solutions Substitute 2r into the area function to get
A(2r) = ~ ( 2 r =
) ~74r2 = 4.A
where A = v r 2 .
Similarly, A(br) = r ( b r ) 2
= .rrb2r2
=PA 0
Often when a function is given, its domain is not mcntioned.

2.1.2 Definition The set of all x for which f ( x ) is a real number is called the natural
domain o f j
When the domain offis not mentioned, we shall assume it to be its natural
domain. The main issues involved in determining the natural domain of a
function are division by zero and even roots of negative numbers.

Example 2 Determine the natural domain of each of the following:

Solutions ( a ) The denominator is zero-that is, x 2 - 4 = 0-if x = f 2 , so the natural


domain off is the set of all real numbers except +2.
+
( 6 ) 9 - x2 2 0 if 9 2 x 2 or - 3 5 x 5 3. Hence the natural domain of g is
the closed interval [-3, +3].
> >
(c) 1 - x 2 0 if 1 x 2 , so the natural domain of h is the open interval
(- 1, 1). Note that the numbers +1 are not part of the natural domain
because h ( f 1 ) would lead to division by 0. 0
When it is necessary to restrict the natural domain of a function artificially,
we specify the domain explicitly.
30 CHAPTER 2: LIMITS, DERIVATIVES, AND CONTINUITY OF FUNCTIONS

PROGRESS TEST 1

For each ofthefunctions in I to 3, compute ( a ) f ( t ) , ( b ) f ( x + h), 4. The circumference C of a circle is given as a function
( c ) f (2x1. Do not simplify your answers. of its radius r by C = 2ar. What happens to the
circumference when the radius doubles?
I. f ( x ) = 3x2 - 5 x +2 5. Determine the natural domain of these functions:
2. f ( x ) = ax b +
3. f ( s ) = J 3 1
(u) f(x) = dxz - x - 2 ( 6 ) y = ___
vG=T

THE NUMERICAL AND GRAPHICAL


DESCRIPTlONS OF A FUNCTION
All the descriptions of functions previously given were algebraic in that in each
case the rule has been given explicitly by an algebraic formula. However, since
equations can be graphed, and since our functions were given in terms of
equations, we can graph functions using what we already know about graph-
ing equations. Recall that an equation in x andy has a graph in the coordinate
plane consisting of all pairs ( x , y ) satisfying the equation. Thus we graph a
function f ( x ) by graphing the points ( x , f ( x ) ) .

2.1.3 Definition I f f is a function with domain D,then the graph o f f is the


collection of all points ( x , f ( x ) ) as x varies over D.In particular, to say that the
point ( x , y ) is on the graph offis equivalent to the statement t h a t y = f ( x ) .

In graphing a functionf(x) we have an option in labeling the vertical axis


eitherf(x) ory. It makes little difference, although if we usey, then we really
have the graph of the equationy = f ( x ) . We shall refer to the graph off(.) and
the graph ofy = f ( x ) interchangeably.
Often the functions we plot will involve familiar equations, so we shall not
need to plot many values off(.). For example, we know from (1.3.3), p. 22, that
the graph of

y = f ( x ) =x2 + 2x + 1
is a parabola, concave up, with vertical axis oTsymmetry x = - 2/(2 * 1) = - 1,
y intercept (0, l ) , and so forth.
O n the other hand, when confronted with a n unfamiliar function, it is most
useful to compute several values of the function and compile the results in a
table, particularly near points where the behavior of the function may be in
doubt. Near such points plot as many values as you need to understand the
function.

Example 3 Determine the natural domain and then graph the function

Solution The function is not defined at x = 3 because (32 - 9)/(3 - 3) = O / O , which is


not a real number. Otherwise,f(x) is defined for all other real numbers. We
compute and plot some values near x = 3 . See Fig. 2.1.
Notice that the points lie in a straight line. (We labeled as many points as is
practical.) We indicate that the function is not defined at x = 3 by placing an
open circle or hole in the graph off(.) at x = 3. 0
Obviously, except for the hole, Fig. 2 . l ( b ) gives the graph ofg(x) = x + 3, a
fact also apparent from an algebraic manipulation:
SECTION 2.1: FUNCTIONS: WAYS OF DESCRIBING AND COMBINING THEM 31

X f(X) = (XZ - 9)/(x - 3)

-1 2
0 3
1 4
2 5
2.5 5.5
2.9 5.9
2.99 5.99
3.01 6.01
3.1 6.1 X
3.5 6.5
4 7

I
Fig. 2.2

It is important to realize that (x - 3)/jx - 3) = 1 only if x # 3. For x = 3,


( x - 3 ) / ( x - 3) is not defined. These observations make it clear, however, that
no matter how close x is to 3 , f ( x ) is defined, and equals x + 3. Thus, for
example,
f(2.99999999) = 5.99999999
Nonetheless f ( x ) = (x2 - 9)/(x - 3) # g(x) = x +
3 because of the differ-
ence in the domains off(.) and g(x). In fact, it is worth stating the following
general principle:

2.1.4 Two functions f and g are equal if and only if


( u ) The domain off equals the domain of g.
( b ) f (x) = g ( x ) for each number x in the common domain off and g.
We can define yet another function, call it h(x), closely related to
f ( x ) = ( x 2 - 9 ) / ( x - 3) and g(x) = x + 3 by defining it to be equal to the
previous two functions for all x # 3 and defining h(3) = 1, for example. The
customary method for defining a function in this way is as follows:

h(x) =
1:,i ifxSt3
ifx=3
Thus the domain of h(x) is all of R, and its graph is as shown in Fig. 2.2.

Example 4 Graph the function


-1 ifx<O

1 ifx>O
Solution The solution is given in Fig. 2.3. Notice that the function is always -1 on
+ +
( - 00, 0) and is always 1 on (0, co). 0
We shall refer to a function that is constant for all real numbers as a constant
function. Thus the function defined by f ( x ) = 4 for all x is a constant function.
The graph ofy = f ( x ) is, of course, the horizontal liney = 4.
32 CHAPTER 2 : LIMITS, DERIVATIVES, AND CONTINUITY OF FUNCTIONS

f
t

I
Fig. 2.3 Fig. 2.4

Using the process of patching different algebraic formulas on disjoint


intervals, it is possible to construct almost any sort of function.
For example, the function
2 x + 12 for x < -4
x2 + 4x + 4 for - 4 < < x 0
f( X I = x3 for 0 < x < 1
Y 2 - x for 1 < x < 3
for 3 5 x

t
-2
is graphed in Fig. 2.4.

EQUATIONS AND FUNCTIONS

X
Not all equations in x a n d y when solved for, say,y automatically yieldy as a
function of x . By solving
x2 +y2 =9
for y , we get y = 5 d m (see Fig. 2.5 j . Thus each value of x yields two
+
values of y , so the equation x 2 y 2 = 9 does not define y as a function of x .
Fig. 2.5 Letting x = 2 yieldsy = 6
andy = - 6,
or, put geometrically, the vertical
line x = 2 hits the graph in two places. This suggests the following general rule.
Y If any vertical line hits the graph of an equation in x a n d y in more than one
20105
point, then the equation does not definey as a function of x (see Fig. 2.6).

On the other hand, an equation such as x 2 + y 2 = 9 (see Fig. 2.5) does define
more than one function of x zmplzcztly. For example, we can define the two
functions
I
I * X U ( X )= fl-7 and L(xj = - d m
X
(see Fig. 2.7). We shall have more to say on implicitly defined functions in
Chap. 3.
Fig. 2.6 Occasionally it is necessary or convenient to regard x as a function ofy.

Example 5 Use the equation x + 4 =y 2 and its graph ( u ) to define x as a function ofy and
( b ) to define two different functions of x (see Fig. 2.8).
Solution (a) Solving the equation for x in terms ofy, we get

x =y2 - 4
SECTION 2.1: FUNCTIONS: WAYS OF DESCRIBING AND COMBINING THEM 33

t
Y

tI

(a) (6) I
Fig. 2.7 Fig. 2.8

which clearly gives x as a function ofy: Each value ofy yields a uniquely defined
value of x. O n the other hand, if we solve fory in terms of x, we do not gety
as a function of x :
y=*\lx+4
Note also that any vertical line to the right of x = -4 hits the graph in two
places.
( b ) For x 2 -4, define s ( x ) = -\/x+4 and t ( x ) = - v ' x .

PROGRESS TEST 2

1. Forf(x) = ( 2 x - 2 ) / ( x - 1) ( a ) determine the natu- 4. Given the equation 4y2 = 8x +2


ral domain, (6) graph the function using a table of ( a ) Determine two distinct functions of x defined
data, (c) give a simple function that equals the origi- implicitly.
nal function everywhere except at one point, and ( b ) Give the domain of each.
( d ) define and graph a new function that equals the 5. ( a ) Connect the dots in the diagram below with a
previously defined functions on their common do- smooth curve that is to be the graph of a function
main but equals - 1 at the point where the original of x.
function was undefined. ( 6 ) Repeat ( a ) using a dotted line that will be the
2. Suppose 2 x - 3y + 6 = 0. graph of a function ofy.
( u ) Determine whether this equation gives y as a 6. Sketch the graph of
function of x .
(6) Determine whether this equation gives x as a
4 - x2 for x< 1
for x = 1
function of y .
x + 2 forx>1
3. Repeat Prob. 2 for the equation 4y2 = 8x + 2.
34 CHAPTER 2: LIMITS, DERIVATIVES, AND CONTINUITY OF FUNCTIONS

AN ALTERNATIVE DEFINITION OF FUNCTION


We saw that each function f has a graph consisting of all the ordered pairs
(x,f(x)), where x is in the domain of$ Many authors take this as a starting
point, defining a function as a collection of ordered pairs, no two of which have
the same first element. Then the set of numbers that appear as first elements of
these ordered pairs make up the domain of the function, and those that appear
as second elements make up the range. The condition that no two ordered pairs
have the same first element corresponds to our uniqueness condition.
The two definitions of function are logically equivalent, but we shall con-
tinue to use the rule-based definition except for Exercises 35 and 36.

THE ALGEBRA OF FUNCTIONS


We know how to add, subtract, multiply, and divide algebraic expressions and
can use this knowledge to define similar operations on functions.
How should we define the sum of the function f (x) = x 2 and the constant
function g ( x ) = 3? A natural answer would seem to be the function that
transforms any number x into the number x 2 + 3. In fact, we shall define the
+
sum off and g, which is denoted f g, in exactly this way:
( f + g)(x) = x 2 +3
+ +
(Remember, this is not tf g times x, but tf g of x.) This certainly makes
sense in terms of the graphs off, g, and f + g, shown in Fig. 2.9. The effect of
adding the constant function g to f is merely to slide the parabolafup by 3
units.
We define f - g similarly:
(f- g)(x) = x 2 - 3
We can use this example as the basis for a more general definition.

2.1 .(i Definition The sum of two functions f and g, denoted f + g, is defined by
( f + g)(4 = f ( x ) +dx)
and the dgerence of t w o functions f and g, denoted f - g, is defined by

(f- g ) ( 4 = A x ) - ).(g
The domain off ? g consists of those numbers common to the domains off and
P.
Graphs of two functions and their sum are shown in Fig. 2.10.
A function such as
p ( x ) = x4 - 3x2 + 31

Fig. 2.9
SECTION 2.1: FUNCTIONS: WAYS OF DESCRIBING AND COMBINING THEM 35

Fig. 2.10

is nothing more than the sum and difference of its separate terms, each of which
is a function of x in its own right.
We define the product of two functions f and g similarly.

2.1.7 Definition The product o f f and g, denoted f - g, is defined by


( f * g ) ( 4 =f ( x ) g ( x )
The domain o f f . g consists of those numbers common to the domains of J
and g.

2.1.8 Definition The quotient of two functions f and g, denoted f/g, is defined by

The domain off / g consists of those numbers x that are common to the domains
off and g and for which g(x) # 0.

Unfortunately, unlike the sum and difference, the product and quotient of
functions do not lend themselves to easy graphical interpretation.

PROGRESS TEST 3

For each pair of functionsf and g in Probs. 1 and 2 , find graph of the four resulting functions, compiling a table of
the sum, difference, product, and quotient. Note where data where necessary.
any of these is undefined. In Prob. 1 only, sketch the
~~ ~- ~~~
1. f ( x ) = 5, g ( x ) = 3 x +
1 2. f ( x ) = 2 x 2 , g ( x ) = x

TYPES OF PlJNCTIONS
It is useful to establish a vocabulary for the commonly occurring types of
functions that can be built up using the previously discussed operations.

2.1.9 Definitions
( a ) A function of the form f ( x ) = mx + b, where m and b are real numbers, is
called a linear function.
( b ) A function of the form f ( x ) = ax2 + + bx c, where a, b, and c are real
numbers ( a # 0), is called a quadratic function.
( 6 ) A function of the form f ( x ) = b,xn + bn_lxn-l + + +
. . . b,x b,, where n
is a positive integer and b,, b,, . . . , b, are real numbers, is called apolynomial
function and is said to be of degree n if 6, # 0.
36 CHAPTER 2: LIMITS, DERIVATIVES, AND CONTINUITY OF FUNCTIONS

( d ) Any quotient of polynomial functions, sayf(x) = p(x)/q(x),wherep(x) and


q ( x ) are polynomial functions, is called a rational function.
(e) Any function that can be built up from linear functions by adding, sub-
tracting, multiplying, dividing, or taking nth roots is called an algebraic
function.
BY [2.1.9(41,

is a n algebraic function.

T H E COMPOSITION OF FUNCTIONS
We shall now describe a more subtle way of combining functions. Let
f ( x ) = x3 and g(x) = 2x +1
Earlier we noted that we can substitute various symbols into a function. Thus
we can write
f(. + b) = ( a + 4 +
3 , ~ ( h~ ) = .( + h ) 3 , ~ ( 2 =~ )( 2 4 3 , . . .
The rule forfrequires us to cube any real number put into the parentheses.
Why not considerf(2x +
l)? According to the rule,
f ( 2 x + 1) = (2x + 1)3
We could have written this as

f(g(x)) = = (2x +
(Read this as f o f g of x equals g of x cubed, and so on.)
What we have done here is to apply thefunctionf to the result of applyingg. First g
works on x , yielding 2 x +1, and then f works on g(x) = 2x 1, yielding +
+
(2x 1)3. More generally, we make the following definition:

2.1.10 Definition Given two functions f and g, the composition off with g , denoted
f 0 g, is defined by

( f o g)(x) = f ( g ( x ) )
The domain off0 g consists of all numbers x in the domain of g for which g ( x )
is in the domain of J:

Note that the product off and g is shown by a dot, whereas the composition
is indicated by a small circle.
It does make a difference whether we say the composition off with g or the
composition of g with j ? Given the above two functions
f(x) = x3 and g(x) = 2x +1
the composition off with g is

(fOS)(X) =f(g(x)) =fW + 1) = ( 2 x +


whereas the composition of g with f is

(g o f)(x) = g ( f = g(x3) = 2 x 3 + 1
(4)
(The rule given by g is to double the number and add 1. In this latter case
the number happens to be the result of applying& namely x3.) Obviouslyf o g
and g of are not equal.
SECTION 2.1: FUNCTIONS: WAYS OF DESCRIBING AND COMBINING THEM 37

An algebraic function such as


h(x) = d4x2 + 2x + 3
can be regarded as the composition of

f (x) = 6 with g(x) = 4x2 + 2x +3


g
Here h(x) = f(g(x)) = f (4x2 2x + +
3) = d 4 x 2 2x 3. + +
X We can think of composition pictorially as in Fig. 2.11. Here x gets trans-
formed by g to g(x), which in turn gets transformed byftof(g(x)). Thus the
f8 composition, which we denote byf o g, transforms x tof(g(x)). The composition
Fig. 2.1 1 is really a chain of functions, one following another.
Remember, for the composition off with g to be dejined for an x , g ( x ) must be
in the domain off; that is, f ( g ( x ) ) must be defined.

Example 6 Let g(x) = x 2 - 4,f(x) = fi + 11. Determine ( u )f 0 g, (6) g of, and (c) the
domain of each composition.
Solutions (u) ( J og)(x) = f ( g ( x ) ) = f ( x 2 - 4)
= d F z + 11
( 6 ) ( g o f ) ( 4 = g ( f ( x ) ) = g ( f i + 11)
= (6 + 11)2 - 4
= x + 22
6 + 117
(c) The domain of ( f 0 g)(x) = d- +
11 is the set of x such that
x 2 - 4 2 0; that is, x 5 -2 or x 2 2. The domain of (g o f ) ( x ) = x +
22 & + 117 is the set of x such that x >_ 0. 0
The domain of composition o f f o g may sometimes be a proper subset
I of the natural domain of the rule for f o g. For example, if f ( x ) = x 2 and
g(x) = 6,then for x 2 0 ( f 0 g)( x ) = x . This rule has as its natural domain all
real numbers, although f o g applies only to nonnegative x.
Composition of functions is sometimes usefully expressed in terms of the
equation approach to functions. In this case the dependent variable of one
equation is substituted into the other equation.

Example 7 Suppose A = r r 2 and r = 3t + 1. Express A as a function of t.


Solution A = r r 2 = r(3t + 1)2 0

Example 7 may arise in a physical context. Since A = r r 2 gives the area of


circle as a function of its radius, an interpretation of r = 3t +
1 as the radius of
a growing circular oil spill as a function of time t yields an expression of the area
A as a function of time.

PROGRESS TEST 4

1. For each pair of functions f and g, determine f o g, 2. Let f ( x ) = ( x - 2 ) / 3 andf-l(x) = 3x 2 (readf +


g 0.Lf of, and g O g. as finverse). Note thatf-l does not mean 1/ Find
(a) f ( x ) = 3x - 2, g(x) = 7x f o f - l and f- oJ:
( 6 ) f ( x ) = fi,g(x) = 2x + 1 +
3. Giveny as a function x : y = x 2 5 x - 1; and x as a
+
function of w :x = (3w 2 ) / w . Expressy as a func-
(4 f ( 4= -, g(x) = x 2
x + l
+ 3x + 2 tion of w.Determiney when w = 1.
( d ) f ( x ) = -5, g(x) = x4
38 CHAPTER 2: LIMITS, DERIVATIVES, AND CONTINUITY OF FUNCTIONS

THE THREE POINTS OF VIEW


Besides being a lesson on functions, this section is intended to make another
point. A complete understanding of most things must involve more than one
point of view. To understand what a human being is, the point of view of
chemistry is surely not enough (we have all heard statements such as the
chemical constituents of the human body are worth approximately $3.50, yet
we certainly feel we are worth more than that!) We need to add the points of
view of biology, psychology, sociology, anthropology, literature, art, philoso-
phy, and so on to get a more complete idea of what a human being is. The same
kind of statement applies to functions.
We have developed three fundamental ways of describing a function:
1. The symbolic or algebraic description, usually as a formula
2. The numerzcal descrzption, usually as a table of data
3. The geometrzc descrzptzon, as given by the graph
In the following work we shall systematically use all three approaches to
develop and clarify new ideas as they occur. We hope you too will become
actively accustomed to using all three approaches.

SECTION 2 . 1 EXERCISES

For each function in Exercises I to S, give ( a ) its natural domazn, ( b ) its <
( c ) f ( x ) = 2 - x for x 0 andf(x) = x 2 + 2 for x 2 0.
range, and ( c ) its graph, using a table of data where necessary <
( d ) g(x) = x 2 for x >
2 and f (x) = x 3 for x 0.
1. f ( x ) = -102 2. g(x) = x4 ( e ) f ( x ) = a number that, when raised to the fourth
1 power, yields x.
3. h ( x ) = d u 4. ~ ( x = ) -
(f Y

x2 - 1
5. m(x) = __
x + l
45 ifx=O

I n Exercises 7 to I0 give the natural domain of the function.

7. f ( x ) = dx3 + 2x2 - 3x 8. g(s) = d F3-

9. h ( t ) = /= t-I
+
\/t2
1
- 5t - 14
12. Does the equation x - y 2 - 29 - 4 = 0
( a ) Determine y as a function of x?
( b ) Determine x as a function ofy?
10. W(X) = dlxl(x2 - I) 13. Determine two distinct functions of x implicitly defined
11. Which of the following descriptions gives a function of x? +
by the equation 16x2 y 2 = 25.
If a given description fails to give a function, explain 14. Repeat Exercise 13 for the equation x 3 = y 2 .
why. 15. ( a ) Using a solid curve, connect the dots to give y as a
( a ) Double the number x and then add 1 million. function of x
(b) Y (b) Repeat ( a ) using a dashed curve to give x as a func-
tion ofy.
Y
4
SECTION 2.2: LIMITS: AN INFORMAL INTRODUCTION 39

16. L e t f ( t ) = ( t 2 - 2t - 3 ) / ( \/t+l).Compute 26. f ( x ) = ( 3 - x) - 2(3 - x ) 27. f ( x ) = d m


( a ) f ( x ) (b) f (2x) (4f (x + h ) ( 4f ( h ) ( 4 f ( 3 ) 28. f ( x ) = - ( x - 1)2 29. f ( x ) = 2x + 1
17. Repeat Exercise 16 withf(r) = ( r - l)/(r + 1) + 27. 30. Find f o f when f ( x ) = 3 x + 2.
31. ( a ) Graphf(x) = 1x1. (b) Findfof.
In Exercises I 8 to 25 compute, in order, 32. Graph the function s ( x ) = largest integer n 5 x, some-
(a) f + g (b) f - g -
(c) f g (4f/g times denoted [ x ] . [Note s ( 1 / 2 ) = s(3/4) = s(.999) = 0,
(e) f g (f)g o f (9)f o f (h) g g and s(1) = 1.1
stating in (a) to ( d ) which numbers are not in the natural domains of
33. ( a ) Graphf(x) =
<
for x 1 and x> 1
these functions. for x = 1
(b) What shouldfil) be in part ( a ) so that the resulting
18. f ( x ) = 6+ 1, g(x) = x 2
graph can be drawn without lifting your pen from the
19. f ( x ) = x , g(x) = 2x + 1
paper?
20. f ( x ) = ax + +
b, g ( x ) = cx d 34. Graph f ( x ) = Ix + 1 I andg(x) = 1x1 + 1 on the same
2 1. f ( x ) = 3x -
x+4
4, g(x) = - coordinate axes.
3 35. Give the function f ( x ) = x 2 (domain R ) as a set of
1 ordered pairs.
22. f ( x ) = x, g ( x ) = -
36. Is the set of ordered pairs, the first of which is the
23. f ( x ) = 2x, g ( x ) = 3x absolute value of the second, a function? Explain. Graph
24. f ( x ) = x 2 , g(x) = x 3 this set of ordered pairs.
25. f ( x ) = x 3 + +
3x 1 , g(x) = 2 37. What is the domain o f f o g, where f ( x ) = fi and
g(x) = -x2 - I?
In Exercises 26 to 29 express the function as the composition of two
simpler functions. (More than one solution is possible.)

Section 2.2: Limits: An Informal Introduction

Objectives: THE CONCEPT OF LIMIT


1. Use informal methods, including Since the most important idea underlying the calculus is that of limit, we shall
compiling tables of data and present it in stages of increasing complexity and formality, beginning with a n
graphs, to determine the behavior informal discussion. All mathematical statements and conclusions in this section will be
of a given function near a given put on a solid logicalfoundation in subsequent sections, especially Sec. 9. I , Chap. 9, where
point. we revisit the formal dejinition of limit in detail.
2. Use informal algebraic methods, Loosely speaking, when dealing with limits, we are concerned with the
including careful cancellation and behavior of a function near but not at a particular number. Suppose a function f
rationalization, to evaluate the limit is defined for all x near a particular number a, but not necessarily at x = a. A s we
of a simple function and the limit choose values of x closer and closer to a, what hafipens to the values o f f (x)?
involving a combination of simple I n some cases your experience and knowledge of certain functions allow you
functions. to answer the question immediately.

Example 1 As we choose values of x closer and closer to the number 2, what happens to the
corresponding values off (x) = x2?
Solution O u r preliminary approach is to compile a table of data [Fig. 2.12(a)] for values
of x near 2 and compare these with the graph [Fig. 2.12(6)1 near x = 2.
It is clear from both the table and the graph that, as the values of x approach
2 (from either side), the values off (x) approach 4. I n fact, the values off ( x ) can
be made as close to 4 as we choose by taking values of x close enough to 2 . 0
The standard mathematical shorthand for the conclusion of the previous
example is
limx2 = 4
x-2

This is read as the limit as x tends to 2 of x 2 is 4 or the limit as x approaches


2 of x 2 is 4.
40 CHAPTER 2: LIMITS, DERIVATIVES, AND CONTINUITY OF FUNCTIONS

.f(X) = X2

4.0401

I 2

Pig. 2.12

Although the value off(.) = x 2 at x = 2 is 4, we did not use this fact to


answer the original question. Often the function is not defined at the point in
question, as in the following example. (Recall the discussion of holes in the
graphs of functions in Sec. 2.1 .)

Example 2 As x tends to 2 , what happens to the values of g(x) = (x3 - 2 x 2 ) / ( x - 2)?


Solution After factoring

x3 - x2(x
- 2x2 - - 2)
g(x) = -
x - 2
we see that the only difference between g ( x ) andf(x) = x 2 previously examined
is that g ( x ) is not defined at x = 2 : g ( 2 ) = 0/0 is not a real number; and the
graph of g ( x ) is a parabola with a hole at x = 2 (see Fig. 2.1 3 ) . Tables of data for
g are almost the same as those forJ
Even though g ( 2 ) is not defined, we see either from the table of data or the
graph that, as x tends to 2, the values ofg(x) are tending toward 4. The values of
g ( x ) can be made as close to 4 as we choose by choosing values of x close enough
to 2. So again we have
x3 - 2x2
limg(x) = lim =4
2-2 2-2 x - 2
even though g ( x ) is not defined at x = 2 . 0

g(.) = [xx - 2)]/(x - 2) A.) = [.2(x - 2)1/(. - 2)

3.9601 2.01 4.0401


X

undefined undefined

(a)

Fig. 2.13
SECTION 2.2: LIMITS: AN INFORMAL INTRODUCTION 41

Writing g ( x ) as
g(x) = x2 * x - 2
~

x-2
we can see why we cannot simply cancel the x - 2 factors leaving x2. For all
x # 2, g(x) is the product of x2 with (x - 2)/(x - 2) = 1, but at x = 2,
(x - 2)/(x - 2) is not defined, hence g(x) is not defined. Thus we can cancel
provided we are careful in so doing to note that the cancellation does not apply
when the factors being canceled are zero. We do this by writing

The reason we can then deal with the function that results from canceling is
that in determining the limit we are concerned only with the values of all x NEAR the point
in question and not AT the point (in this case, the point x = 2). The following
example illustrates this issue even more dramatically.

Example 3 Determine
limh(x) for h(x) =
x-2 [XSz ifx#2
ifx=2
Away from x = 2 this function is the same asf and g of the previous examples.
Solution
A t x = 2 it differs fromfin thatf(2) = 4 and h(2) = 6. It differs from g because
2 is not in the domain of g , whereas it is in the domain of h. In particular, then,
for x near but not at 2 the table of data and the graph for h (see Fig. 2.14) are
the same as those forfand g above. Since the behavior of h for all values of x
near but not equal to 2 is the same as that offand g, the limit as x approaches 2
of h(x) must also be the same. That is, limx,z h(x) = 4 even though
h ( 2 ) = 6. 0
The following example illustrates that the limit concerns the values of x on
both sides of the point in question.
Fig. 2.14

Example 4 1x1 near x = 0.


Examine the values off(x) = -
X

Solution We consider the positive and negative cases separately. For all x > 0,
1x1 -= 1
f(x) = -
x x
whereas for all x < 0,
f(x) = -
X
-x = -1
1x1 = -
X
(for x < 0, 1x1 = -x)
On the other hand,f(x) is not defined at x = 0. Hence we can sketch the graph,
as in Fig. 2.15.
Thus there is no single number which the values off approach as x ap-
proaches 0. In this case we say

X
1x1 does not exist
lim - 0
+-0 x

Notice the way this last example differed from the previous three by the way
you answer the following question: Given a value of x close to 0, can you tell
whatf(x) is close to?
42 CHAPTER 2: LIMITS, DERIVATIVES, AND CONTINUITY OF FUNCTIONS

Analogous versions of this question for the previous three examples would
have yes as a n answer. (The value of the function would be close to 4.)

ONE-SIDED LIMITS
Up to now we have used the designation
limf ( x )
+-a

to denote that we are taking the limit as x tends to a from both sides. Let us then
write
lim f ( x )
x+a+

to denote that we are taking the limit as x tends to a from the right side.
Similarly, we write
lim f ( x )
x+a-

to denote that we are taking the limit as x tends to a from the left side. These
latter two cases are commonly referred to as right- and left-hand limits, respec-
tively.
The following is then certainly reasonable:

2.2.1 Theorem limf ( x )


a-a

exists if and only if limx+a-f ( x ) and lim,,,+ f (x) both exist and are equal. In
this case, of course,
lim f ( x ) = lim f ( x ) = lim f ( x )
x- Q x- a+ x+ a

We can use the above language to reformulate the conclusions of Example 4


as follows:

. - Ix I
lirn 1x1
= -1 , lim-= 1
0-0- x 2-o+ x
and, since the left- and right-hand limits are unequal,

Ix I
lim - does not exist
32-0 x

FUNCTIONS GROWING OR DECREASING


WITHOUT BOUND
We now look at another way limits might fail to exist.

Example 5 Determine the values of

for x near 2 .
Solution We construct a table of data and sketch the function (Fig. 2.16). From both the
table and the graph we see that as x gets closer to 2,from both sides, the values of
SECTION 2.2: LIMITS: AN INFORMAL INTRODUCTION 43

f ( x ) are growing without bound and are certainly not approaching any fixed
number. Again we say:

lim ~ does not exist 0


2-2 (x - 2 ) 2
undefined undefined
Once again, in this last example, being told that x is near 2 would not enable
(a) you to tell what numberf(x) is close to. In both Examples 4 and 5 the limit
failed to exist, but for vastly different reasons. In Example 4 lima+ojx(/x fails to
exist because the left- and right-hand limits are unequal. In Example 5 the
function fails to have a limit at x = 2 because the values off grow large without
bound as x approaches 2. We often express this latter behavior by writing

1
lim ~ = $00
a-2 (x - 2)2
X

One must be very careful with this notation. The symbol + m does not
+
represent a number, and although we write = 00, lima+2 l/(x - 2)2 does not
Fig. 2.16
+
in fact equal anything! We are using the shorthand = 00 here to indicate the
behavzor of l/(x - 2 ) 2 for values of x close to 2. And no, we have not found a
way to divide by zero. In Example 6 we shall examine the behavior off(x) =
l/(x - I), which you will see combines the misbehavior of both Examples 4
and 5, decreasing without bound on one side of 1 and increasing without bound
on the other. It also indicates that it is not possible to solve the problem of
division by zero simply by adopting a fancy new symbol.
We are using the notation lim,,J(x) +
= co to mean that, as x tends to a
from both sides,f(x) increases without bound. Graphically, this means that the
graph off (x) will cross any horizontal liney = M drawn above a-no matter how
large M is. Numerically, we can think of the statement to mean that by choosing
values of x close enough to a we can getf(x) to exceed the capacity of your
calculator-no matter how large the capaczty ofyour calculator! (How near to 2 must x
be so thatf(x) = l/(x - 2)2 will exceed the capacity ofyour calculator?)
We similarly write

limf(x) = - 00
a-a

if the values off (x) decrease without bound as x tends to a from both sides. Thus, for
example,

limp- - 1 - -00
2 - 2 ( x - 2)2

Again, we emphasize that this latter notation expresses the behavior off(.) as
x tends to a and does not mean that the limit exists in the sense that the values of
the function can be made arbitrarily close to some specific real number by
choosing x close enough to a.
These ideas extend directly to left- and right-hand limits as illustrated in the
following example.

Example 6 Examine the behavior o f f (x) = l/(x - 1) near x = 1 by determining the


behavior as x + I + and as x + 1- separately. Sketch the graph off (x) near
x = 1.
Solution First we compute some numerical data:
44 CHAPTER 2 : LIMITS, DERIVATIVES, AND CONTINUITY OF FUNCTIONS

I(.) = l!(x - 1) X f(.) = I/(X - 1)

2 1
1.5 2
0.9 - 10 1.1 10
0.99 - 100 1.01 100
0.999 - 1,000 1.001 1,000
0.9999 - 10,000 1.000 1 10,000
undefined 1 undefined

It is fairly clear that as x -+ I + , the denominator is positive and tends toward


zero, so the quotient l/(x - 1) grows without bound. Hence we write

lim -- -+co
a-I+x-1
Similarly,
lim- 1 -
- -a
I-1- x-1
In particular,
1
lim ~

I-1 (x - 1)
does not exist. Also (see Fig. 2.17),

1
lim -# I
$00 and lim- #-a 0
I-1 x-1 X+l x-1
The following example shows that a limit may be difficult to ascertain even
Fig. 2.17 with a table of values and high accuracy.

Example 7 Determine whether

lim
(6-
fi)
X-2 (x - 2)
exists and, if so, determine its value.
Solution First we compute a table of data (using a calculator). From the table below it
would be difficult to guess the limit. However, by rationalizing the numerator
we can isolate the factor x - 2, which can then be carefully canceled in the
sense discussed earlier:

6 -a -- 6 - & + fi- &* x-2


x-2 x - 2 & + fi- ( x - 2 ) ( & + fi)
-
- 1 ifxf2
6+ &
Now it is surely reasonable to predict that 1/( 6+ fi)will tend to 1/(2 fi)
as x tends to 2 from either side. [Note that 1/(2
0.3535534, accurate to six decimal places.] 0
a) is approximately

X X (& - 4 M x - 2)

1 0.414214 3 0.317837
1.9 0.358087 2.1 0.349241
1.99 0.353996 2.01 0.3531 13
1.999 0.353598 2.001 0.353509
1.9999 0.353558 2.0001 0.353549
2 undefined 2 undefined
SECTION 2.2: LIMITS: AN INFORMAL INTRODUCTION 45

PROGRESS TEST 1

In Probs. 1 and 2, (a) make a table of values off near f ( x ) does not exist, examine the left- or right-hand lim-
x = a, ( b ) sketch the graph off near x = a, and ( c ) deter- its-if they exist.
mine limz+af (2).
3. f ( x ) = x - 1
~ ,a=l
1. f ( x ) = 9 - x 2 , a = 2 Ix - 11
1
x2 -9 4. f ( x ) = -, a = 2
2. f ( x ) = ~ ,a=3 2-x
x - 3
In Probs. 3 to 6 determine whether limz+af(x) exists, and 5. f ( x ) =
6-fi, a = 3
if so, determine its value by making a table of values near x - 3
a (using a calculator or table of square roots) or perform- -1
6. f ( x ) =
ing an appropriate algebraic manipulation. When limx+a ~

(x + 3)4 a = - 3

LIMITS AND CALCULATORS


The previous examples and problems illustrate both the strengths and weak-
nesses of using a numerical approach to determining limits of functions-in
particular, using approximations available through the use of a calculator. By
observing the values off (x)for specific values of x near to the point in question,
we can usually get a fairly good idea of what the limit may be; often this
amounts to observing the changes in increasingly accurate decimal approxi-
mations to some number. However, as Example 7 showed, this is not always
sufficient to determine the actual limit without further analysis. But, more
important, no amount of numerical data can provide a logical proof that the limit is a
spec@ number. A proof requires using the precise definition of limit, found in Sec.
2.5. Thus, no calculator or computer exists-or ever will exist-that can
calculate sufficient accuracy to prove that lim,,,f(x) is some particular number.
A proof can be provided only by a human being using precisely defined
mathematical concepts and logic. Numerical accuracy and logical accuracy are
entirely different matters. The calculator or computer can, however, lead us in
the direction of the limit when such exists. We encourage you to use a calculator
and perform frequent numerical experiments.

SOME FURTHER EXAMPLES


In all but Example 1 and Prob. 1, Progress Test 1, we determined limz,af(x) by
replacing f ( x ) by a function g(x) that equaled the original function f ( x )
everywhere except at x = a itself and hence had the same limit as x + a. We
obf?ined the function g(x) by carefully canceling a factor inf(x) or (in Exam-
ple 7) by rationalizing and then carefully canceling. We then determined (or
evaluated) limz+ag(x) by direct substitution; that is,
limg(x) = g(a)
x-a

Any such function g, whose limit at a can be determined by direct substitu-


tion, is said to be continuous at x = a. A detailed discussion of continuity appears
in Sec. 2.5.
We shall now apply the technique just mentioned to evaluate limits of the
type that will appear in the next section, which is on derivatives.
46 CHAPTER 2: LIMITS, DERIVATIVES, AND CONTINUITY OF FUNCTIONS

Example 8 Evaluate
lim S ( 3 + x) -f(3)
2-0 X

wheref(x) = x 2 + 1.
Solution We shall write out and simplify [ f ( 3 + x ) -f(3)]/x before attempting to
evaluate the limit. (Recall Sec. 2.1.)

f(3 + 2) -f(3) - [(3


-
+ x)2 + 11 - [32 + 11
X X

-
[9 + 6x + x 2 + I ] - [lo]
X

- 6x
- + x2
X

Now, for x # 0, (6x + x 2 ) / x = 6 + x. In evaluating the limit as x tends to 0,


we are interested only in values of x near 0, but not at 0. For all such x, we know
that
f(3 +4 -f(3) =6 +

X
Hence

=6
(Obviously, as x tends to 0 , 6 + x tends to 6; thus, in effect, we can evaluate this
limit by direct substitution.) 0

In the next example the variable in the limit is denoted by h, but our earlier
techniques still apply.

Example 9 Evaluate

wheref(x) = l/x.
Solution Heref(1) = 1 a n d f ( 1 + h) = 1/(1 + h). Thus
f(1 + h) - f ( l ) -- 1/(1 + h) - 1
h h
-
-
(I - 1 - h)/(l + h)
h
-
-
(1
-h
+ h)h (
A/B
Recall -= -
A
C BC
)
Hence, for h f 0,
f(1 + h) - f ( l ) - -1
h l + h
Therefore

lim f ( 1 + h ) -fP) = lim- -1


h-0 h h-0 1 +h
- --1
= -1
1
(Obviously, as h tends to 0, 1 + h tends to 1, so the fraction tends to - 1.) 0
SECTION 2.3: THE DERIVATIVE AND RATES OF CHANGE 47

PROGRESS TEST 2

Evaluate the following limits:

Hint: rationalize before taking the limit.

SECTION 2.2 EXERCISES

Evaluate the following limits in 1 to 12 where possible. In cases where the limit does not seem to exist, compile a table of data and graph the function
near the point in question.
5x3 + 8x2 d2x+3-x 3~ - 6 x - 9
1. lirn 2. lirn 3. l i m p 4. lim-
1-0 3x4 - 16x2 2-3 x - 3 x-2 Ix - 21 x-9 VG - 3

5 . lirn
G-6 x4 - 16
6. l i m p 7 . lirn (2x3 + x2 - 9)
1-0 2x 1-2 x2 - 4 x-1

lirn g(4 + h ) - g(4) , where g ( x ) = -


1
8. lim.f(2 -f(2) , where f (t) = 4t3 9.
t-0
+
t h-0 h x2 +4
10. lirn f ( 3 + h, - f ( 3 ) , wheref(x) = 16x2 11. limf() -f(3) , wheref(x) = 16x2
h-0 h 1-3 x-3
1
12. lirn g ( x ) - g ( 4 ) , where g ( x ) = 13. Compare the answers to Exercises 10 and 11. Can you
x-4 x -4 x2
~

+4 explain the correspondence? It may help to examine


respective tables of data.
14. Repeat Exercise 13 for the answers to 9 and 12,
respectively.

In Exercises 15 to 18 compile a table o f data and sketch a graph to determine the behauior of the giuen function near the giuen point a.

15. f ( ~=) -, a = - 2
1 1
16- g ( t ) =
1)3 a = - l
~

x + 2 (t +

17. = 1 1 1 1
S(X) 18. F ( t ) =
/2x + 11 a = - - + 1)2 a = - -2
~

2 (2t
19. For the largest integer function s ( x ) = [XI (see Sec. 2.1, Exercise 32),
( a ) determine lirn s ( x )
x-1-
( b ) determine lirn s ( x )
x-1+
(6) does lims(x) exist?
x- 1

Section 2.3: The Derivative and Rates of Change

Objeetives: AVERAGE AND INSTANTANEOUS R-ATES OF CHANGE


1. Compute derivatives of functions We remarked in Sec. 2.1 that perhaps the most important and useful mathe-
using the limit definition. matical concept is that of function, because it is through functions that we can
2. Interpret derivatives of functions make precise the quantitative relationships between one thing and another.
in general and in physically related However, really understanding and using functions usually amount to studying
situations as instantaneous rates of how the quantities related by the functions change. This is especially the case if
change. a quantity is given as a function of time. The barometric pressure changes with
48 CHAPTER 2: LIMITS, DERIVATIVES, AND CONTINUITY OF FUNCTIONS

time, but knowing what the pressure is at any given time is not as valuable as
knowing whether and how fast it is rising or falling. The same could be said of
the stock or commodity markets. For an outfielder to know the position of a fly
ball is not nearly as important as knowing how the position is changing, that is,
how the ball is moving. The understanding of physical, economic, social, or
biological systems often amounts to the understanding of how they change.
Two basic types of knowledge about rates of change are possible: the average
rate of change over a given intcrval and the instantaneous rate of change at a
given instant. Although it is sometimes sufficient to know the average rate of
change over an interval, in other situations the instantaneous rate of change
may be more valuable. Being told that the price of wheat would drop from
$4.20 per bushel to $3 between July and December (an average loss of 20C a
month for 6 months) is not particularly valuable, and, in fact, could be
downright fraudulent if, during September, the price hit $5. Similarly, knowing
that a car hit a tree 1 h after beginning a trip and 20 mi down the road does not
tell us how fast the car was going at the instant it hit the tree. All we know is
that the car averaged 20 miles per hour (mph) for the previous hour. The
driver could have hit the tree at 80 mph whilc trying to make up time lost
while drinking in a pub along the way.
To get a better idea of what is meant by the instantaneous rate of change at
a point, we shall examine a more precisely defined situation, one that played an
important motivating role in the creation of the calculus in the seventeenth
century.
Suppose an object is moving in a straight line andf(x) represents its position
at time x. We assume that some positive direction has been assigned, sof ( x ) can
in general be positive, negative, or zero, depending on where the object is
located with respect to the origin or reference point.
Now if h represents some change in time, then

f(x + h) - f ( 4
h

X f ( x ) = 16x2
gives the average velocity over the time interval from x to x +
h. That is, average
velocity is the change in position (displacement) divided by the change in time.
0 It is well known and can be verified by experiment that, ignoring air
16 resistance, an object falling freely for x seconds will fall approximately
64
144 f ( x ) = 16x2ft
256
(This is a special case of the general gravitation formulaf(x) = $gxz, where g
400
is the acceleration due to gravity. On earth g is approximately 32 ft/s2.) In
Table 2.1 Table 2.1 we show several values off(x),
One thing should be apparent: The object falls faster as time passes. It falls
16 ft in the first second and thus averages 16 ft/s for the first second (recall the
average rate of velocity is the distance traveled divided by the time elapsed).
During the next second (from time x = 1 to time x = 2) it averages
(64 - 16)/1 = 48 ft/s. The averages during the third, fourth, and fifth seconds
are, respectively, (144 - 64)/1 = 80 ft/s, (256 - 144)/1 = 112 ft/s, and
(400 - 256)/1 = 144 ft/s. This certainly agrees with our experience that the
longer the object falls, the faster it falls. This is why more people are willing to
jump off a chair than off a building.
Computing average velocities is a straightforward application of the basic
formula just noted:
change in position
Average velocity =
change in time
SECTION 2.3: THE DERIVATIVE AND RATES OF CHANGE 49

But now, can you state how fast the object is falling at the instant that 3 s have
x f(x) = 162
elapsed, that is, at x = 3? To answer this question precisely, we need more than
4 256 arithmetic and algebra; we need to apply the idea of limit. The strategy,
3 196 however, is simple. We compute the average velocities over smaller a n d smaller
time intervals containing x = 3 to see whether these are tending to some fixed
3 169
number as a limit. If so, we dejine this limit to be the instantaneous velocity at
% 156.25
x = 3. First, a more detailed table of values forf(x) near x = 3 is in order (see
3.1 153.76
Table 2.2).
3.01 144.9616
The average velocity from x = 3 to x = 34 = 712 is
3.001 144.096016
3.0001 144.00960016 f(7/2) --f(3) 196 - 144
-
-
= 104ft/s
3 144 ) 3
( 7 ~ 2- 1i 2
Tahle 2.2 Similar computations over smaller and smaller time intervals yield the average
velocities in Table 2.3.
We also could have calculated averages for intervals bejiore x = 3. For
example, from x = 2.9999 to x = 3, the average velocity is
f(2.9999) -f(3) - 143.9904002 - 144
- = 95.9984
2.9999 - 3 -0.0001
Thus we know that the instantaneous velocity at x = 3, say v(3), satisfies
95.9984 = ~(2.9999) 4 3 ) < < ~(3.0001)= 96.0016
It certainly appears that as x gets closer and closer to 3 from either side, the
average velocities get closer and closer to 96 ft/s. It would surely be reasonable
then to state that the instantaneous velocity at x = 3 is 96 ft/s.
We can use the ideas of the preceding section to clean up things, avoid some
of the messy calculations involved here, and make the whole approach more
precise. We let h stand for the small positive and negative numbers being added
to x = 3 and consider the limit as h + 0 of the resulting average.
Thus 3 + h can be thought of as a number near x = 3. If h 0, say >
h = ,001, then 3 + h = 3 + .001 = 3.001, a number slightly bigger than 3;
while if h < +
0, say h = -.001, then 3 h = 3 - .001 = 2.999, a number
slightly less than 3. We can now calculate the average velocity over a typzcal
small time interval between 3 and 3 + h:

Average velocity between 3 and 3 + h = f ( 3 + h) - f ( 3 )


(3 + h ) - 3
-
16(3 + h)2 - 144
-
96h + 16h2
h h

Time Interval Average Velocity ( j ? / s )

f(3# -A31 - 156.25 - 144


x = 3 tox = 36 - = 98.00
3; - 3 -
1
R
J'(3.1) - f ( 3 ) - 153.76 - 144
x = 3 to x = 3.1 - = 97.6
3.1 - 3 .1
~'"(3.01)- f ( 3 ) - 144.9616 - 144
x = 3 to x = 3.01 - = 96.16
3.01 - 3 .01
f(3.001) - f ( 3 ) - 144.096016 - 144
x = 3 to x = 3.001 - = 96.016
3.001 - 3 ,001
f(3.0001) - - f ( 3 ) 144.00960016 - 144
x = 3 to x = 3.0001 - = 96.0016
3.0001 - 3 ,0001

Tahle 2.3
50 CHAPTER 2: LIMITS, DERIVATIVES, AND CONTINUITY OF FUNCTIONS

The idea of average velocity means that h # 0 (average velocities only make
sense for nonzero intervals of time!) Hence we know that

represents the average velocity during the interval between 3 and 3 +h.
We define the instantaneous velocity at the instant x = 3 to be the limit of such average
velocities as h + 0. But, of course,
lim (96
h-0
+ 16h) = 96

Example 1 Find the instantaneous velocity under the previous circumstances at the instant
x = 4.
Solution The average velocity in the interval between time 4 and 4 h equals the+
distance fallen divided by the time elapsed. The distance fallen is
j ( 4 + h ) -f(4), and the time elapsed is (4 + h ) - 4 = h. Thus the average
velocity, with some simplifying, is
f(4 + h ) -f(4) - 128h
-
+ 1Gh2 = 128 + 16h (since h # 0)
h h
(We are dealing with average velocities over a nonzero time interval.) Thus the
instantaneous velocity at x = 4 is

lim ( + h, -(4) = lim (128 + 16h) = 128 ft/s 0


h-0 h h-0

In determining both of the above limits of average velocities, we knew that


h # 0 and canceled the h factor. Yet we were able to take the limit as h 4 0
because, as emphasized in Sec. 2.2, the limit reflects the behavior of the function
near h = 0 and not at h = 0.

PROGRESS TEST 1

1. With the same distance fallen function ( c ) Find the average velocity (in simplified terms) in
f ( x ) = 16x2, the interval between x = 2 and x = 2 h. +
(a) Find the average velocity in the interval from 2. Use l ( c ) to find the instantaneous velocity at x = 2.
x = 2 to x = 2.001. 3. Find the instantaneous velocity at time x = 1 using
(6) Find the average velocity in the interval from the limh+oidea.
x = 1.99 to x = 2.

THE INSTANTANEOUS-VELOCITY FUNCTION

Instantaneous Using the techniques just developed we can derive Table 2.4, which gives
Velocity at Time x instantaneous velocities for a freely falling object.
0
Can you name a function that satisfies the data? When x increases by 1 , the
32 instantaneous velocity increases by 32, so the function g(x) = 32x satisfies the
64 Table 2.4. We might be so bold as to conjecture that the function g ( x ) gives
96 the instantaneous velocity at any instant x. We shall now show that this is the
128
case.
160
In our previous discussions, to keep things simple and concrete, we calculated
Table 2.4 some instantaneous velocities at fixed integral values of x. Whynot calculate the
SECTION 2.3: THE DERIVATIVE AND RATES OF CHANGE 51

instantaneous velocity at any time x? The same ideas carry through. The average
velocity in the time interval between x and x +
h is (for h # 0)
Change in position -
-
f(x + h) -f(x) - 16(x
-
+ h)* - 16x2
Change in time h h

-
16(x2 + 2x15 + h2) - 16x2 - 32xh + 16h = 32x + 16h
h h
For any fixed x the instantaneous velocity at time x will be the limit a s h tends to
0 of the average velocities. But
lim (32x
h-0
+ 16h) = 32x
just as we expected! We have derived a new function that gives the instantane-
ous velocity at any time x. It is denotedf(x) (read fprime of x) and is called
the derivatzve o f f ( x ) = 16x2. We determined (and in effect defined)fl(x) a t an
unspecified point x by determining

Knowing f (x) = 32x at a typical x, we can now compute the velocity at, for
example, x = 3.76, by simple substitution:
,f (3.76) = 32(3.76) = 120.32 ft/s
In our previous discussion the motion under consideration was in one
direction only (taken to be the positive direction). Consequently, the average
velocities computed were actually average speeds, and the instantaneous
velocities were instantaneous speeds. For more general straight-line motions
velocity may be positive or negative. A positive velocity means that the motion is
in the positive direction, a negative velocity means that the motion is in the
opposite direction, and zero velocity means the object is at rest. In any case the
absolute value of the velocity gives the speed.

PROGRESS TEST 2

O n the moon, because of the moons smaller size and 2. How fast is a n object falling after 5.9 s have elapsed?
weaker gravitational pull, the distance a n object falls in 3. Compute m(6) using the limit definition applied at
feet as a function of time x in seconds is approximated by the particular point x = 6.
the function m(x) = 2.6~. 4. Assuming that the longest safe downward fall on
1. Using the technique developed, calculate the deriv- earth involves a terminal velocity of 24 ft/s (this is
ative function m(x) that gives the instantaneous a fall of 3/4 s and a distance of 9 ft), how long is a
velocity at any time x by: fall (in seconds) on the moon that gives the same
( a ) Finding the average velocity in the interval from terminal velocity? How many feet is this fall?
+
x to x h a n d simplifying this expression.
( b ) Finding the limit of the average velocities as
h + 0.

DEHTVA3TTVESIN GEKBRAL
Without regard for any physical interpretation for a functionf(x) [wheref(x) is
not necessarily equal to the function discussed earlier], we can compute the
average rate of change off(x) as x changes from x to x + h. This is merely a
quotient of the sort already computed several times previously:
52 CHAPTER 2: LIMITS, DERIVATIVES, AND CONTINUITY OF FUNCTIONS

f (x + h ) - f @ )
h
We can then go on to form the limit of such quotients as h + 0.
2.3.1 Definition The derivative OfJ; denoted f I, is a function defined at points in the
domain off by

f ( x ) = lirn f ( x + h) -f(4
h -0 h
provided that this limit exists. Iff(.) exists at a particular point x = a, we refer
tof(a) as the derivative off(.) at a and sayf is dflerentiable at a. Iff is differentiable
for all numbers in some set S, we say f is dzfferentiable on S. A function that is
differentiable on its entire domain is referred to as a dferentiable function.

As we saw earlier for the particular functions


f ( x ) = 16x2 and m(x) = 2 . 6 ~ ~
the derivatives existed for any real number x, and so each is a differentiable
function. We also saw that we were able to determine the derivative off at a
particular point in two ways. We found the derivative off at x = 3 by evaluat-
ing the limit

lim f ( 3 + h) - f ( 3 ) = 96
h-0 h
directly. O n the other hand, having found that the derivative o f f at an
arbitrary point x wasf(x) = 32x, later we could compute the derivative at
x = 3 by simple substitution: f ( 3 ) = 32 3 = 96. When a function is differ-
entiable at a point x = a, its derivative can be determined by either method.

Example 2 Forf (x) = x 2 + 1, compute (a)f(5) using the definition at the particular point
x = 5, ( b ) f ( x ) for a typical x, (c)f (5) by substitution intof(x).

Solution ( a ) f ( 5 ) = $m A 5 + h) --s(5)
-0 h

= lirn [(5 + h) + 11 - [25 + 11


h-0 h

= lim 25 + 10h + h2 + 1 - 25 - 1
h-0 h
= lim 10h + h2
h-0 h
= lim(l0
h -0
+ h) since h #0
= 10

( b ) f ( x ) = lim f@ + h) -f(4
h-0 h
= lim [(x + hI2 + 11 - [ x 2 + 11
h-0 h

= lim x2 + 2xh + h2 + 1 - x 2 - 1
h-0 h

= lim 2xh + h2
h-0 h
SECTION 2.3: THE DERIVATIVE AND RATES OF CHANGE 53

= lim (2x + h ) since h #0


h-0

= 2x
-
(c) Since f (x) = 2 x , f ( 5 ) = 2 5 = 10, which agrees with (a) 0

A MATTER OF NOTATION
It is possible to perform the entire previous analysis using other notations.
Instead of using h as the notation for the small numbers to be added to x , we
can use the symbol Ax (read delta x). The symbol Ax is a single indivisible
symbol and should be thought of as representing a small change in x. Then the
quotient

f(x + h) - f ( 4
h
that we have been using takes the form

f(x + Ax) - f W
Ax
We can take this delta notation a step further by letting Af denote the
change resulting in f when x is changed by the amount Ax. Thus
A f = f (x + Ax) - f ( x )
Then our quotient takes the form

In addition to the notation f ( x ) , we denote the derivative by d f / d x , so

df
- = lim- A f
du Ax-0 AX
Sometimes, when a function is given in the form y = f ( x ) as a functional
relationship between two variables, for example
y = 16x2
we write Ay to represent the change iny resulting from a change Ax in x. In this
case the limit looks like

AY
lim -
Ax-0

and is often denoted as dy/dx ory instead of df/dx.


In any of these notations, the resulting limit is still called the derivative with
respect to x. The only difference between the approaches is notation; the idea and
the result are exactly the same.
To summarize, if y = f ( x ) , then the derivative with respect to x can be
denoted by any of the following:

The whole concept of using different notations to represent the same thing is
certainly not foreign to you; for example, 0.5, 1 / 2 , and 2 / 4 all represent the
same number. In our situation each notation has its advantages and disadvan-
tages, and we shall choose among them according to our needs at a particular
54 CHAPTER 2: LIMITS, DERIVATIVES, AND CONTINUITY OF FUNCTIONS

time, just as we choose the appropriate notation for the number one-half
according to the needs of the situation.

Example 3 Compute df/dx forf(x) = 2x + 3.


Solution By definition,

-df
= lim
f(2 + A 4 - f ( x )
dx AX-0 Ax

= lim
+
[ 2 ( ~ AX) + 31 - [2x + 31
AX-0 Ax

= lim 2~ + 2 A x + 3 - 2~ - 3
AX-0 AX
lim -2 Ax
AX-o AX
= lim (2) since Ax #0
Ax-0

= 2 0

Example 4 Given thaty = x3, compute dy/dx.


Solution By definition,

d~ = lim -
-
AJ .
1im
[(x + ~ 4 3 -
1 [x31
dx AX-o Ax AX-o Ax
x3 + 3x2Ax + ~ X ( A X +) ~ AX)^ - x3
= lim
Ax-0 Ax
3x2Ax + ~ X ( A X +
) ~ AX)^
= lim
Ax-0 Ax
= lim [3x2 + 3xAx + AX)^] since Ax # 0
AX-0

= 3x2 0

YHOGRESS TEST 3

1. Determine df /dx given thatf(x) = 3 - x2. 4. Given g(x) = l / x (as in Prob. 2), compute
2. Determine g(x) given that g(x) = l/x. (Combine ( u ) g(2) applying the limit definition at x = 2.
terms of numerator.) ( b ) g(2) by substitution into the result of Prob. 2.
+
3. Determine dy/dx given thaty = x2 x.

MORE ON BNSTANTANEOUS RATES OF CHANGE


When introducing the derivative of a functionf(x) in general, we referred to the
quotient

f(x + Ax) -f ( 4
Ax
as the average rate of change off(x) over the interval between x and x + Ax.
Hence it makes sense to regardf(x), the limit of such averages as Ax + 0, as
the instantaneous rate of change offwith respect to x. That this makes sense
SECTION 2.3: THE DERIVATIVE AND RATES OF CHANGE 55

apart from any particular concrete interpretation forf is the source of its wide
usefulness. Whether f describes the distance fallen (sof gives the instantaneous
rate of change of distance with respect to time, that is, the instantaneous
velocity) or the profit in a certain economic situation ( s o f gives the so-called
marginal profit), we rely on the same fundamental idea underlying the inter-
pretation-the instantaneous rate of change of the quantityf(x) with respect to
the quantity described by the independent variable x.

Example 5 Given f(x) = 3 (constant function), compute and interpret f(x).


Solution Here,f(x) = 3 = f (x + Ax) for any x and any Ax. Therefore the change in&
+
Af =f (x Ax) -f(x), is equal to zero, so

Hence
f(x) = lim -Af = lim (0) = 0
A$-o AX Az+O

Naturally enough,f(x), being constant for all x, has zero change Af; hence
zero average change Af /Ax, and thus zero instantaneous change f (x). 0

ANOTHER RATE-OF-CHANGE PROBLEM:


THE GIkOWTH OF A CIRCULAR REGION
Material set ofby the 9 symbol and horizontal rules in the text denotes where the theme
ofgrowth ofgeometric objects is used to build understanding of change and rate of change.

9 The area A inside a circle is given as a function of its radius by


A = Tir2
Suppose that the length of the radius is increasing. When the circle is small, a
given change in radius, Ar, produces a relatively small change in area, AA,
whereas when the circle is large, the same Or produces a larger increase in area,
as can be seen in Fig. 2.18. (Here the delta notation turns out to be convenient.)
Obviously, the inside strip of width Ar has much smaller area than the
outside strip of the same width. You have experienced this difference whenever
AI
you mowed a lawn in a roughly circular pattern. In that case Ar is the width of
the lawn mower and AA is the amount you mow in a complete cycle. Certainly
it takes much longer to mow the outside strip than the inside one, and the
amount you mow is greater. From this we would expect that the rate of change
in area, with respect to radius, increases as the radius increases.
To make a more mathematical analysis, we compute

Fig. 2.18 AA = A ( r + Ar) - A ( r )


= a(r + Ar)2 - ar2
= 2arAr + a(Ar)2
Notice that AA is close to the area of a rectangular strip of width Ar and length
271r. The additional term a(Ar)2 represents the amount of area that would
theoretically have to be snipped out (in tiny wedges pointing toward the center)
to straighten out the circular strip. The average rate of change of area with
respect to radius is
56 CHAPTER 2: LIMITS, DERIVATIVES, AND CONTINUITY OF FUNCTIONS

AA
__ -
2arAr + a(Ar)2 = 2ar + a Ar
Ar
-
Ar
(Ar # 0)

Thus the instantaneous rate of change of area with respect to radius a t radius r
is

- = lim (2ar + m Ar) = 2mr


dr AT-o

For r = 2 , dA/dr = 2 4 2 ) = 47, and for r = 30, dA/dr = 2m(30) = 607. This
mathematically confirms our experience: The larger the radius the larger the
rate of change in area. In fact, the rate of change is nothing more than the
circumference, which again makes sense: The limit as Ar + 0 of the Ar strip is
a circumference! In effect, the circle grows by circumferences. In Progress Test
4 and then in the section and chapter exercises we shall see scveral additional
situations analogous to the above. The mathematics together with its physical
interpretation provide new insights. Underlying these situations are two fun-
damental ideas of calculus to be studied later: the idea of differentials and the
fundamental theorem of the calculus.

PROGRESS TEST 4

9 The area A of a square with side x is A = x 2 . 3. Compute U / A x in reduced terms.


1. Suppose x increases to x +
Ax. Compute the result- 4. Compute dA/dx ( a ) in terms of x, ( b ) when x = 2 ,
ing change in area, AA. and (c) when x = 200. ( d ) Discuss the growth of
2. Sketch the situation, labeling each part of the dia- area in terms of dA/dx and the two edges in the
gram, noting the area of each section so that each direction that the growth occurs.
term of AA is recognizable in the sketch. (Keep one
corner of the square fixed and let all the growing
take place in the other direction. Shade AA.)

SECTION 2.8 EXEllCISES

1. Near the surface of the sun the distance (in feet) fallen In Exercises 2 to I 5 use the dejnition to determine the derivatiue of
by a (rather warm) object in x seconds is approximately the given function at a typical point.
s(x) = 432~.
2. f ( x ) = 3x - 5 3. f ( x ) = 3 - 2x
( a ) Find the average velocity in the interval between
4. f ( x ) = 3x2 - 2 5. g(x) = 5x2 - 3
x = 2 s and x = 2 4 s .
6. g ( x ) = 6 - 2x2 7 . y = 3x2 - 2x
( b ) Find the average velocity in the interval between
x = 2 s and x = 2 i s . 8. y = x 9. r ( x ) = 3x3
( c ) Find the average velocity in the interval between
10. qx)= 2x3 + 5x 11. f ( w ) = w 2 + 3 w + l
x = 2 s and x = 2.01 s. 12. y = 5x3 - x 2 18. y = x 2 - x 3
( d ) Find the average velocity in the interval between 1 1
14. f ( x ) = - 15. f ( x ) = -
+
x = 2 s and x = (2 Ax) s. 3x 3x2
( e ) Find the instantaneous velocity a t x = 2 s. 16. Compute and interpret f (2) for a n arbitrary constant
( f ) Find the instantaneous velocity a t an arbitrary function f (x) = 6.
time x. 17. ( a ) Determine f (x) (using the definition) for f (x) = fi
(9) Find the instantaneous velocity after 5 . 4 s have at a typical point x. (You will need to rationalize.)
elapsed. ( b ) Apply the definition to computef(9) and compare
( h ) Given that the largest terminal velocity for a safe withf(9) obtained by substitution into the result of ( a ) .
j u m p is 24 ft/s (see Prob. 4, Progress Test 2), what height ( c ) T h e function f ( x ) = fi
is defined on [0, +co).
above the sun gives this terminal velocity and what is Where is it differentiable?
the duration of the fall? 18. Repeat Exercise 17 for f ( x ) = fi(defined on
(- 00, 11).
SECTION 2.4: THE DERIVATIVE AND SLOPES OF TANGENTS 57

21. T h e height (in feet) above the ground of a certain object


19. 9 The volume V of a sphere given as a function of its thrown vertically upward from ground level with a n
radius r is V = $rrr3.
initial velocity of 144 ft/s is given as a function of x in
(u) Compute AV (in terms of r and hr) and interpret
seconds by h ( x ) = 144x - 16x2 (ignoring air resistance).
geometrically.
( a ) Determine the instantaneous vertical velocity func-
( b ) Determine A V for r = 2 and Ar = 1 and compare
tion h(x).
with A K where r = 20 and hr = 1. (Thus the amount of
(6) When does the airborne object have zero velocity?
blowing required to increase the radius of a small balloon
(c) At what height is the object a t zero velocity?
by 1 in. is much less than that required to increase the
( d ) When does the object strike the earth and what is
radius of a large balloon by 1 in.-ignoring air pressure.)
its velocity at that time? (Note that positive height and
(c) Compute dV/dr.
velocity are in a n upward direction.)
( d ) Compare d V / d r and the formula for the surface area
( e ) What is the maximum height reached by the ob-
of a sphere. Interpret your results.
ject?
20. 9 Suppose a rectangle has a fixed width of 5 cm and ( f ) Graph the height function.
variable length x, so its area A is given by A = 5x. 22. Repeat Exercise 21 for a n object thrown vertically with
Determine and interpret geometrically (drawing a fig- an initial velocity of 96ft/s, whose distance above
ure): ( u ) AA (in terms of x and Ax) a n d ( b ) dA/dx. ground after x seconds have elapsed is h(x) =
96x - 16x2.

Section 2.4: The Uerivative and Slopes of Tangents

Objectwe: DERIVATIVES A N D SLOPES


Relate the derivative of a function Besides representing the average change off(%) in the interval between x and
to the graph of that function in x + Ax, the quotient
cases where the derivative is
computed using the definition.
f(x + Ax) - f ( x )
Ax

also represents the slope of the line through the points P = (x,f(x)) and
Q = (x + A x , f ( x + A x ) ) ; see Fig. 2.19. We simply apply the definition of slope
(1.2.3) to the line through P and Q. From the graphical point of view, taking
the limit to form the derivative off(x) amounts to taking the limit of such
slopes as Ax + 0. Graphically, as Ax tends to 0, the points Q are moving
along the curve toward P; see Fig. 2.20.

2.4.1 Definition If the derivative off at the point x exists, that is, if

4 I --

__

Fig. 2.19
58 CHAPTER 2: LIMITS, DERIVATIVES, AND CONTINUITY OF FUNCTIONS

f exists, we refer to the unique line through ( x , f ( x ) ) with slopef(x) as the tangent
line to the graph o f f (x).

This definition reflects exactly what we see in Figs. 2.19 and 2.20-namely,
that as Ax + 0, the lines through P and Qare approaching the tangent line to
the graph at ( x , f ( x ) ) ;see Fig. 2.21. The line Tis the tangent line to the graph of
y = f ( x ) at ( x , f ( x ) ) , with a slope off(.).
Let us now summarize the result of this discussion.

I
2.4.2 Derivatives and Slopes Whereverf(x) exists, it gives the slope of the
Fig. 2.21
uniquely defined line tangent to the graph offat ( x , f ( x ) ) . In this case we shall
also refer tof(x) as the slope ofthe graph O f f ( x ) , or merely the slope o f f ( x ) at x .

We shall examine the meaning of this general discussion for the familiar
functionf(x) = x 2 + 1, whose derivative we computed in the preceding section
to be
f ( X ) = 2x
In Fig. 2.22 we include a table of data forf(x) and the graph off(.) with
typical tangents. This new functionf(x) = 2x will give the slope of the tangent
to the graph ofy = x 2 + 1 for any given x .
The table of data fits the picture very well. Note that the tangent line is
horizontal, and thus has zero slope, only when x = 0.
f ( x ) = xz +1

X fX)

-3 -6
-2 -4
Positive
-1 -2 slope
0 0
1 2
t X
2 4
3 6

(a)
Fig. 2.22

Example 1 Interpret the derivative off(x) = 2x + 3 in terms of its graph. [In Example 3,
Sec. 2.3, we showed thatf(x) = 2.1
Solution The fact that the graph off(.) = 2x +
1 is a straight line with slope 2 (see
Fig. 2.23) is in perfect accord with the fact thatf(x), which gives the slope of
the graph at x , is the constant function 2. Notice that in this case, the tangent to
the graph at any point x is the graph itself. Notice also that the derivative, being
the constant function 2, agrees with the fact that this function changes at a
constant rate; any change Ax produces a change of 2 Ax in the function. 0

Example 2 Interpret the derivative ofy = x 3 in terms of its graph and give the equation for
the line tangent to y = x 3 at x = - 1 in point-slope form. (In Example 4,
Sec. 2.3, we found that dy/dx = 3x2.)
Solution The derivative dy/dx = 3x2 tells us that the slope of the graph is positive for all
x , except for x = 0, where the slope is zero. We compute tables of data fory and
dy/dx and then sketch the graph. (See Fig. 2.24)
SECTION 2.4: THE DERIVATIVE AND SLOPES OF TANGENTS 59

The graph certainly agrees with the derivative: the graph has positive slope,
except at x = 0, where the derivative dyldx = 3x2 is zero and the tangent line is
horizontal. Furthermore, as 1x1 increases, the graph gets steeper.
At x = - 1,y = ( - 1)3 = - 1 and dy/dx = 3( - 1)2 = 3, so the tangent line
passes through the point (-1, -1) and has slope 3. Hence its point-slope
equation is

Fig. 3.23
or
y - (-1) = 3[x - ( - l ) ]

y+1=3(x+1) .
Notice that the tangent to the graph ofy = x 3 at x = 0 is actually the x axis
and in fact crosses the graph at the point of tangency. O n the other hand, by
putting the equation for the tangent at ( - 1, - 1) into slope-intercept form
y=3x+2
we see that when x = 2,y = 8, so this tangent line crosses the curvey = x 3 at
(2,8).
It is important to recognize that our definition both of derivative and of
tangent is two-sided because, far the defining limit to exist, the left- and
right-hand limits

lim f ( x + *x) - f ( x ) and lim f ( x + Ax) - A x )


AX-0- ax Ax-O+ ex
must exist and be equal. The following example illustrates that this is not
always the case.

Example 3 Show that g ( x ) = 1x1 +


1 has no derivative, hence no tangent, at x = 0, and
then interpret this result graphically.
Solution We shall show that

lim do + a4 'do)-
and lim P(0 + Ax) - g(0)
AX-0- Ax Ax+O+ Ax
are unequal. Then we can canclude that the derivative

X y = x3

-3 - 27
-2 -8
-I -1
-7
1 -4
0 0
~ 1 ~ 1 -
3
2 8 4
I ?
1 1 3 I s
2 8 12 1 2
1 3
3 27 27 (-2, -8) 9
4 64 48

Fig. 2.24
60 CHAPTER 2: LIMITS, DERIVATIVES, AND CONTINUITY OF FUNCTIONS

lim g(0 + Ax) - d o )


AX-0 Ax
does not exist, and hence the tangent at x = 0 does not exist.
But

g(0 + A x ) - g(0) - [lo + Ax1 + 11 - [lo\ + 11


Ax ax
-
-
lQ4 +1-1
Ax
-
- !y
Ax
As we computed in Example 4, Sec. 2.2,

lax1 -AX
lim - - - lim - since [ A x / = -Ax
A X - ~ - AX A X - ~ - AX

= lim ( - I ) = -1
AX-0-

g(*) = 1x1 +1 Also,


/ lax1 = lim
d lim -
(1) = 1
B Az-O+ AX AX-O+
i
Thus there is no tangent at x = 0. Furthermore, for x >
0, g ( x ) =
Slope = - 1 Slope = +1 1x1 + 1 = x + 1 and for x <
0, g ( x ) = 1x1 + 1 = - x + 1, so the graph ofg for
x 2 0 is the straight line with slope 1 through (0, 1) and for x 5 0, its graph is
x a straight line with slope - 1 through (0, 1). The lack of a well-defined tangent
at (0, 1) is reflected in the existence of a sharp corner at (0, 1). (See
Fig. 2.25 Fig. 2.25.) 0
In Chap. 4 we shall see that derivatives provide a potent and efficient
technique for sketching graphs of functions.

PROGRESS TEST 1

1. For each of the following findf(x) using the defini- 2. +


Let f ( x ) = x and g ( x ) = x 2 c, where c is some
tion and determine an equation for the line tangent constant. Show that f ( x ) = g ( x ) = 2x. Interpret
to the graph at (a,f(a)). this observation graphically.
(a) f ( x ) = 3x2 a=l
(b) f ( x ) = x2 + x 1 + a =0
(c) , f ( x ) = 1 - x3 a=2

A REMARK ON OUH. APPROACH TO DERIVATIVES


Notice that our introduction to derivatives in Secs. 2.3 and 2.4 has used the
three basic ways of viewing functions. We began with the numerical point of
view, calculating numerical averages based on a physical interpretation. We
then switched to the more efficient algebraic point of view, computing the limits
of algebraic quotients. Finally, we looked at these limits from a geometric point
of view, recognizing them as slopes of tangent lines.

SECTION 2.4 EXERCISES

In Exercises 1 to 6 compare f (x) and the graph off (x), ( a ) giuing a sketch off (x), ( b ) describing when the slope is positive, negative, zero, and/or
undefined, and ( c ) giving the equation ( i n point-slope form) of the line tangent to the graph o f f when x = 1.
SECTION 2.5: LIMITS, CONTINUITY, AND DIFFERENTIABILITY 61

1. f ( x ) = 3 - x2 2. f ( x ) = 5 - 2x2 3. f ( x ) = 3x - 5
4. f ( x ) = 2 - 3x 5. f ( x ) = 1 - 2x3 6. f ( x ) = x - x3
7 . ( a ) Show that g ( x ) = /x- 21 is not differentiable at 16. Where is the slope o f y = x 2 - 3x +
2 negative?
x = 2. 17. Where is the slope ofy = x 2 - 6 x +
5 negative?
( b ) Interpret ( a ) in terms of the graph of g. 18. Determine the derivative of the largest integer func-
8. Repeat Exercise 7 for h ( x ) = )3x - 91 at x = 3. tion s ( x ) = [ x ] on the open interval (1/4, 3/4). (Recall
9. (Recall Sec. 2.3, Exercise 17.) Forf(x) = 6 interprat Sec. 2.1, Exercise 28, and Sec. 2.2, Exercise 18.)
the behavior o f f ( x ) near 0 ( x not negative) in terms of 19. Two types ofparallelism: recall Sec. 2.4, Progress Test 1,
the graph of f ( x ) near 0. In particular, examine Probs. 2 and 3.
lim,,o+f(x) and lim,,,+f (x). ( a ) What is the vertical distance betweenf(x) = x2 and
10. Repeat Exercise 9 forf(x) = near x = 1. (Re- g(x) = x2 +
1 when x = O?
call Sec. 2.3, Exercise 18.) ( b ) Repeat ( a ) for x = 10.
11. ( a ) Determine dy/dx, wherey = l/(x - 1). (c) What is the horizontal distance between the graphs of
(6) Compare dy/dx near x = 1 with the graph of f and g wheny = I ?
y = l/(x - 1). (Recall Example 6, Sec. 2.2.) ( d ) Repeat (c) fory = 10. Compare with (c).
z 2. Repeat Exercise 1I fory = 1/(2 - x) near x = 2 (Recall ( e ) If the graphs of f ( x ) and g(x) are defined to be
Sec. 2.2, Progress Test I , Prob. 4.) vertically parallel, suggest an alternative definition of
13. Interpret graphically the fact that f ( x ) = 0 for any horizontally parallel.
constant functionf(x) = 0. (f)Are the graphs o f f ( x ) = x 2 and h ( x ) = ( x - 1)2
14. Interpret graphically the fact thatf(x) = 1 for,f(x) = x . horizontally parallel? Are they vertically parallel?
15. Interpret graphically the fact that f ( x ) = 3 for any
function of the formf(x) = 3x + b, where b is a con-
stant.

Section 2.5: Limits, Continuity, and Differentiability


~ ___

Objectives: Anyone desiring a complete and detailed discussion of the material up to the following
Progress Test I m y turn directly to Sec. 9.1.
1. Apply the basic properties of
limits, as expressed in the Limit THE LIMIT THEOREM
Theorem, to evaluate limits
involving functions that can be The following definition states in precise language that
built up from quotients of
polynomials and nth roots. limf ( x ) = L
x-a
2. Recognize where functions are
continuous. if and only if we can makef(x) stay arbitrarily close to L by choosing x
3. Distinguish between a function sufficiently close to a.
that is continuous at a point and a
function that has a derivative at
that point.

2.5. I Definition Suppose we are given a function f ( x ) and numbers a and L , with
f ( x ) defined for all points, except perhaps a itself, of an open interval containing
a. We say that
the limit of f(x) as x tends to a is L

and write

limf ( x ) = L
a-a

if for every positive number E there is a positive number S such that

o < Ix - a1 <6 implies If(.) - L/ < E


62 CHAPTER 2: LIMITS, DERIVATIVES, AND CONTINUITY OF FUNCTIONS

4 The key element in this definition is that for every > 0 there must be a
E
6 > 0 such that x being within S units of a but not equal to a implies thatf(x) is
within e units of L. The requirement that x # a reflects the remarks made
earlier in this chapter that we are not concerned with the behavior o f f a t a, but
only near a. The limit is unaffected byf(a); in fact,f(a) need not even be
defined.
Graphically we are showing that there is a n interval about a such that if
x # a is in this interval, thenf(x) is in the (vertical) interval ( L - E , L + E ) ; see
Fig. 2.26.
In Chap. 9 we shall prove the following simple but useful fact:
Fig. 2.26
2.5.2 Theorem Suppose thatf(x) = mx + b is a given function, where m and b are
constants and x = a is any real number. Then
limf(x) = ma
a-a
+b
+
That is, limx+, (mx b) can be evaluated by direct substitution.
As special cases of (2.5.2) we have the following:

2.5.3 Special Limits


( a ) Iff(.) = b, a constant function (rn = 0), then limx+a (6) = b
(6) Iff(.) = x ( m = 1, b = 0), then limz+a (x) = a
The next theorem (proved in Chap. 9) justifies the steps required to decom-
pose complicated limits into much simpler limits involving only these special
functions, which can then be evaluated by direct substitution.

2.5.4 The Limit Theorem Suppose that lim,,,f(x) and limx-a g ( x ) both exist.
Then
) g ( x ) ] = [limx-af(~)]
(a) limx+a [ ~ ( x5 * [limx-,
( 6 ) limx+a[ f ( x )-g(x)]= [limX+,f(x)] * [lim,+,g(x)]. In particular, if 6 is a
-
constant, then limx+, [b g(x)] = b[lim,+, g(x)] (so constants can be pulled
outside limits).
(6) 1imx+, [f(x)/g(x)l = [~imx+,f(x)]/[1imx,, g(x)] provided that limx+a
A x ) # 0.
( d ) limx+a = vlimx-af(x), where n is a positive integer. If n is even, then
f ( x ) must be nonnegative.
The best way to appreciate this theorem is to see it a t work evaluating limits
of much more complicated functions than would be feasible using the definition
alone. Although we have stated parts (u) and (b) for only two functions, they
extend directly to sums, differences, and products of any finite number of
functions. A similar collection of statements holds for one-sided limits.
~

Example 1 Determine lim f/2x3 + 3x + 1


2-4

Limit Theorem [2.5.4(d)]

= Vlim 2x3
2-4
+ lim
x-4
3x + lim 1
2-4
Limit Theorem [2.5.4(a)]

= V 2 i i m x 3 + 3 i i m x + lim I Limit Theorem 12.5.4(6)1 \ ,>


1-4 x-4 x+4
SECTION 2.5: LIMITS, CONTINUITY, AND DIFFERENTIABILITY 63

= ~ 2 l i m x . l i m x . l i r n x+ s l i m x + lim 1 Limit Theorem [2.5.4(6)]


a-4 a-4 2-4 a-4 a-4

= v2(4)(4)(4.) + 3(4) + 1 special limits (2.5.3)

The following result is an immediate consequence of the fact that in the


definition of limx+uf(x),the behavior off at a is irrelevant. In particular, it
records formally our ability to cancel nonzero factors and determine the limit of
the reduced function. We have applied it dozens of times already.

2.5.5 Theorem I f f ( x ) = g ( x ) for all x # a in some open interval containing a, and


if lim,_,g(x) = L , then lim,,,f(x) = L.

Example 2 Determine
lim x 2 + x - 6
x+2 x2 - 4

Solution We cannot apply the Limit Theorem [2.5.4(c)] directly because


limx-z x 2 - 4 = 0. However, since
x2 +x -6 - (x + 3)(x - 2 ) - x + 3 for x # 2
x2 -4 (x + 2)(x - 2) x +2
we can first apply Theorem (2.5.5) to conclude that

NOW,since limx+2(x + 2) # 0, we have


x
lim --
+
3 - 1imx+2( x + 3) by Limit Theorem [2.5.4(c)]
2-2 x 2+ limx+2( x + 2)
-
lim2+zx + limz+23 by Limit Theorem [2.5.4(a)]
limx+zx + limx+22
- 2+3
- by (2.5.3)
2 + 2

The Limit Theorem also applies to justify the computations involved in


calculating derivatives.
The following theorem will be very useful in some of our later work. The
theorem says that if a functionf is squeezed between two functionsg and h, both
of which tend to L as x tends to a, thenfitself must tend to L as x tends to a.
Its proof is in Chap. 9.

2.5.6 Squeeze Theorem If h(x) < f ( x ) 5 g(x) for all x # a in some open interval
containing a, and if limx-u h ( x ) = limx+ug ( x ) = L , then limX-J(x) = L.
4 CHAPTER 2: LIMITS, DERIVATIVES, AND CONTINUITY OF FUNCTIONS

ROGRESS TEST 1

ztermine each of the following three limits, giving rea-


ns for each step.
, lim ( f i x 4 - 2x1 wheref(x) = 3 x 2 - 2x
xe-1
(combine steps-note the limit variable is h, and x
plays the role 01 a constant)

COhTTI?XITY
We all have a general idea of what a continuous process is: It has no sharp
breaks or changes. This same general idea applies to functions.

2.5.7 Definition We say the function f is continuous at a if


( a ) lim,,,f(x) exists ( b ) f(a) is defined (c) limz-,f(x) =f(a)

Furthermore, we say f is continuous on a set of numbers S iff is continuous at each


point of S. If S is the set of all real numbers, then we say f is everywhere continuous
and refer to f as a continuous function. Iff is not continuous at a, we say f is
discontinuous at a.

By our definition, a function fails to be continuous at a if any one of the three


conditions fails to hold. Since the idea of continuity at a is thus tied to the idea
of limit and to whether the function is defined at a, much of our earlier work
can be translated directly into the language of continuity.
In particular, a functionfmay fail to be continuous at a point a for any of
the reasons it fails to have a limit at a. For example, if either of the left- or
right-hand limits at a fails to exist or equalf(a), thenfis discontinuous at a. The
following graphically described examples (Figs. 2.27 to 2.31), were discussed on
pp. 40-44; s ( x ) , shown in Fig. 2.32, was discussed on p. 60.
The functiong(x) fails to satisfy condition ( b ) [hence (c)] of Definition (2.5.7)
at x = 2, since g(2) is not defined; see Fig. 2.28.
The function h ( x ) fails to satisfy condition (c) at x = 2, since 1imx-*h ( x ) = 4,
but h(2) = 6; see Fig. 2.29.
The function D(x) fails to satisfy ( a ) and ( b ) [hence (c)] at x = 0; see Fig. 2.30.
The function ~ ( x fails
) to satisfy ( a ) and ( b ) [hence (c)] at x = 2; see Fig. 2.31.

4
i
--9
I

x3 - 2 x 2 =
y = ___
x2:x -
L
2; \x2 for x # 2
x-2 h:x = 1 6 for =2
y:x1 = x 2 x - 2

continuous at Y = 2 discontinuous at x = 2 discontinuous at x = 2


g. 2 . 2 7 Fig. 2.28 Fig. 2.29
SECTION 2.5: LIMITS, CONTINUITY, AND DIFFERENTIABILITY 65

1x1
D(s)= - T(X) = ~
1
2)2
I
(x -
1
s ( x ) = 1x1 +
discontinuous at x = 0 discontinuous at x = 2 continuous at x = 0
Fig. 2.30 Fig. 2 . 3 1 Fig. 2.32

The function s(x), which has a corner at x = 0, is nevertheless continuous


at x = 0; see Fig. 2.32.
Geometricalb, as is evident from these illustrations, the idea of continuity at a
point has a relatively direct interpretation: The functionfis continuous at a if
and only if the graph has nojumps or breaks at a; that is, it can be traced near a
without lifting the pen from the paper.
Algebraicalb, the idea of continuity at a also has a relatively simple interpre-
tation: the function f is continuous at a if and only if limx+af ( x ) can be evaluated
by direct substitution. We can combine this fact with the Limit Theorem to obtain
ways of combining continuous functions to get new continuous functions. For
example, if we know thatfand g are continuous at a, then lim+,,f(x) = f ( a )
and limz-a g ( x ) = g(a). By Limit Theorem [2.5.4(a)], lim++, [ f ( x ) -t g ( x ) ] =
[lim,,,j~(x)] k [lim,,, g ( x ) ] = f ( a ) i:g(a), so we can conclude that f i:g is
continuous at a. Similarly, the remaining portions of the following theorem can
be deduced.

2.5.8 Theorem Suppose f and g are continuous at a. Then f tg and f * g are


continuous at a, as is f / g if g ( a ) # 0. Also, for n a positive integer is 3
continuous at a (fmust be positive if n is even.)

Now, we know by (2.5.2) that any function of the formf(x) = mx + 6 is


everywhere continuous because any limit of such a function at any point a can
be evaluated by direct substitution. Using (2.5.8),we can then apply this fact to
show that most familiar algebraic functions are continuous where defined.

The previous discussion overlooked a minor point, which deserves clarifica-


+
tion. A function such asf(x) = fiis defined only on (0, co),so, according to
our definition of limit, lims,,, 6
does not exist, hence f ( x ) = fi is not
continuous at 0. [Recall that (2.5.1) requiresf(x) to be defined on an open
interval containing 0.1 However, lim,,,+ 6
does exist and equals
f ( 0 ) = v5= 0.

2.5.9 If a function is defined on a closed interval for which a is an endpoint and


the appropriate one-sided limit exists and equals the value of the function at a,
we shall say that the function is continuous at a.
Hence we sayf(x) I fiis continuous at 0, and g ( x ) = f/G is continu-
ous at 1. [Here g(x) is defined on ( - 00, 11 and limz+l- f/G = 0 = g(l).]
66 CHAPTER 2: LIMITS, DERIVATIVES, AND CONTINUITY OF FUNCTIONS

To handle composition of functions, we first need a limit theorem.

2.5.10 Theorem If limx+ag(x) = b and iff is continuous at b, then

In effect, this theorem says that a continuous function preserves limits. An


immediate consequence is the following:

2.5.1 1 Theorem If g is continuous at a and f is continuous at g(a), then the


compositionfo g is continuous at a. Hence, provided that the domains match
up, the composition of continuous functions is continuous.

Our earlier statement that a continuous function has no jumps or breaks is


given formal expression in the following theorem, whose proof rests on the
corresponding fact about the real-number line: it has no breaks. The proof is
beyond the scope of this book.

2.5.12 The Intermediate-Value Theorem Suppose f is continuous on the


closed interval [a, b] and C is any number betweenf(a) andf(b). Then there
exists at least one number c in [a, b ] such thatf(c) = C.

As is evident from Fig. 2.33(a), there may be many c's in [a, b] such that
f ( c ) = C. However, ifffails to be continuous at just a single point in [a, 61, the
conclusion may fail as Example 3 illustrates.

Example 3 Show that the conclusion of the Intermediate-Value Theorem (2.5.12) fails to
hold for

2-x ifx<I
f(x) = (x + 2 ifx>l on P, 31

Solution As shown in Fig. 2.34,f(O) = 2 andf(3) = 5. If we let C = 24, we find that


there is no number G between 0 and 3 such thatf(c) = 23!

I , I

(a) I
Pig. 2.33 Fig. 2.34
SECTION 2.5: LIMITS, CONTINUITY, AND DIFFERENTIABILITY 67

PROGRESS TEST 2
+ 1.95 <
1
x for x 0
In Probs. 1 to 5 identify the points where the given
6. Letf(x) = 2 for x = 0
function is discontinuous and which of the conditions in
the definition of continuity fail(s) to hold.
x 2.1+ for x 0 >
( a ) Show that the conclusion of the Intermediate-
1. f ( x ) of Prob. 2, Sec. 2.2, Progress Test 1 Value Theorem fails for this function.
2. f ( x ) of Prob. 3, Sec. 2.2, Progress Test 1 ( b ) For the same function, name two nonoverlap-
3. f ( x ) of Prob. 4, Sec. 2.2, Progress Test 1 ping closed intervals where the hypothesis (hence
4. f ( x ) of Prob. 6, Sec. 2.2, Progress Test 1 conclusion) of the Intermediate-Value Theorem does
5 . The function in Sec. 2.1, Exercise ll(c) apply.

DIFFERENTIABILITY IMPLIES CONTINIXTY


Note the strong contrast between the functionf(x) = x 2 + 1, whose derivative
f ( x ) = 2x exists (Sec. 2.3, Example 2) and is zero at x = 0, and the function
g(x) = 1x1 + 1, whose derivative does not exist at x = 0 (Sec. 2.4, Example 3).
The graph off(.) has a horizontal tangent at (0, l), whereas the graph of g(x)
has a sharp corner and no uniquely defined tangent a t (0, 1). Nonetheless, the
graph of neither function has jumps or breaks at ( 0 , l ) . Each is continuous at
(0, l), but onlyf(x) is differentiable at (0, 1). The following discussion estab-
lishes the basic relation between differentiability and continuity for a general
function J:
The two conditions for a functionfon an interval I-(a)fis differentiable on
I and ( b )f is continuous on I-may be thought of as good-behavior condi-
tions on the graph of$ The first says t h a t f has a well-defined tangent at each
point in I , which implies that the graph is smooth without sharp corners. The
second says that the graph has no breaks or jumps on I. Now, since a graph that
cannot even have sharp corners certainly cannot have jumps or breaks, we
would expect condition ( u ) to imply ( b ) in general. To prove this we shall need
some useful equivalent ways of stating continuity at a point. Recall that two
statements are equivalent if each implies the other. This means that in practice
the two statements can be used interchangeably.

2.5.13 Theorem The following statements are equivalent:


1. f ( x ) is continuous a t x = a.
2. limf(x) = f ( a ) .
x-a
3. limf(a
Ax-0
+ Ax) = f ( a ) .
4. lim Af = 0, where
Ax-0
Af =f ( a + Ax) - f ( a ) .
Proof The equivalence of statements 1 and 2 is just the definition of continuity
at a. We shall show that statements 2 and 3 and statements 3 and 4 are
equivalent by systematically rewriting the limit statements. We first show that
statements 2 and 3 are equivalent. Suppose limx+uf(x) = f ( a ) and x = a + Ax.
Then Ax = x - a, so letting x + a is cquivalent to lctting Ax + 0. Thus

f ( a ) = limf(x) = limf(a
x m Ax-0
+ Ax)

and therefore statements 2 and 3 are equivalent.


To show that 3 and 4 are equivalent, we note that stating
68 CHAPTER 2: LIMITS, DERIVATIVES, AND CONTINUITY OF FUNCTIONS

f(a + Ax) +f ( a ) as Ax +0 (statement 3)

is equivalent to stating

Af = [ f ( a + A x ) - f ( u ) ] + 0 as Ax 4 0 (statement 4) W

We now can state a n d prove the promised result.

2.5.14 Theorem I f f i s differentiable a t x = a, t h e n f i s continuous a t x = a.


Prooj I f f i s differentiable a t x = a, then

lim
f(a + Ax) - f ( a )
Ax-0 Ax
exists. Thus we may write
lim
Ax-0
Af lim [ f ( u
AX-0
+ Ax) - f ( u ) ]

= [ lim
Ax-0
$- ") - f ( a )
Ax ]- [lim Ax]
Ax-0
(Limit Theorem [2.5.4(b)])

=f'(a) * 0 = 0
Thus by the fourth statement of (2.5.13),fis continuous a t x = a.
T h e results o n continuity stated in this section will be used repeatedly
throughout the book. In fact, the idea of continuity is basic in calculus. Other
important facts relating to continuous functions will be developed as we need
them.

SECTION 2.5 EXERCISES

In Exercises I to I0 evaluate the given limit, giuing reasons f o r each step.

1. lirn x 2 + x + 3 2. lim-
x2 +x 3. lim dZFj7i 4. lim-
x2 -4
1-1 3x2 + 9 a-0 2x2 +1 X+-2 a-2 x +2
5. lim-
&- 1
6. lirn
(2 +x ) -
~ 8
r-1 x - 1 X-0 X

I). lim
d m - 1
1Q. lirn
(x + AX)^ - x3
AX-0 Ax AX-0 Ax
11. Using f(x) = Ixl/x and g ( x ) = -Ixl/x, show that mine those numbers; zf a v , at which f fails to be continuous; and
limx-a [f(x) + g(x)] may exist when limz+,f(x) and ( c ) stute the reason(s) why it f u i l s to be continuous.
lim,-,g(x) do not.
12. Repeat Exercise 11 for limx+a[ f ( x ) * g(x)].
13. Show that limz+aI ,f(x)l may exist when lim,,,f(x) does \ 7 ifx=2
not. (See Exercise 11.)
14. Assuming that limz-af(x) = N, lim,,,g(x) = M ,
limx+ah(x) = L , limx+ai(x) = K , and that L + K # 0,
give an argument with reasons for each step, proving 1 5 ifx=1
that x2+x-2
ifxf1
lim [ f(x) + h(x)i(x) - g(x)]"5 = [N + LK -
x-a h(x) + i(x) L+K 1 3 ifx= 1

15. Is the combination function in Exercise 14 continuous at x4 - 16


19. f ( x ) =2
x = a?
x -4

20. f ( x ) = -x-2
In Exercises 16 to 21 ( a ) sketch the graph Ofthe functionsx ( b ) deter- Ix - 21
SECTION 2.5: LIMITS, CONTINUITY, AND DIFFERENTIABILITY 69

2x+3 X l l 25. Where is the largest integer function s ( x ) = [XI discon-


21. f ( x ) =

22.
[ 8-3x
x+3
1<x<2
x>2
Construct a function that is defined everywhere and is
tinuous? (Recall Sec. 2.1, Exercise 32.)
26. Show that the conclusion of the Intermediate-Value
Theorem fails for the largest integer function s ( x ) on $1. [i,
discontinuous only at x = 0, x = 2. 27. Define the functions f and g of Exercise 11 at x = 0 in
23. Construct a function that is continuous everywhere but such a way that they can be used to illustrate that the
at x = 0, x = 2 , where it is undefined. sum of functions discontinuous at a point may be con-
24. Use the Intermediate-Value Theorem (2.5.12) and Theo- tinuous at that point.
rem (2.5.9) to prove there is a number a such that 28. Repeat Exercise 27 for products.
a3 = 2.

CHAPTER EXERCISES

Review E x e r c i s e s
In Exercises 1 to 12 determine the values of the following functions, first at a typical x in the domain, and second at x = 1 2f 1 is in the domain:

(a) f + g, (b) (c)


f ( 4f g>
S (el g o x ( f )f o x (9) g O g.
1. f ( x ) = 3x - 1 , g ( x ) = x 2 2. f ( x ) = 2x + 3, g ( x ) = 3x2
1 1
3. f ( x ) = -, g(x) = x
X
+5 4. f ( x ) = -
+ 2 > g ( x ) = 3x

5. f ( x ) = 3x4 + 5, g(x) = x 6. f ( x ) = x, g ( x ) = x 3 - 2
7. f ( x ) = 4 , g ( x ) = x 2 - 3x - 2 8. f ( x ) = x 3 + 3x2 - 4, g ( x ) = 2
9. f (x) = m,g ( x ) = 2 9 10. f (x) = x4, g ( x ) = -\Jx+2
1
11. f ( x ) = -
x-2
1
> g(x) = -
x+2
12. f ( x ) = t x - 2 , g ( x ) = 3x +6
In Exercises 13 to 16 express the given function as a composition of twofunctions, neither of which is the originalfunction. (Note that the answer is
not unique.)

13. f ( x ) = ( x + 1)3 +
14. g ( x ) = ( x ~ ) 3~ ( ~ ) 2(x3)
~ +
15. h(x) = 3(x2)3 + 4(x2)2 + 2(x2) 16. f ( x ) = -\3J3x--5
17. Given thaty = 3x2 - 2x - 2, computey when
( a ) x = 2r ( b ) x = 3t 1 (c) x = t + + At
(a) x = w z ( e ) x = 3u
18. Repeat Exercise 17 fory = 5x2 - 3x 1. +
Which of the followang an 19 to 25 determane(s)y as a function of x? Gzve a reason f o r y o u r answer
19. Y 20. Y 22.

X X
1-6

id

22. y2 = x3 23. y 3 = x 2 24. G=x 25. l/y = x

In Exercises 26 to 29, ( a ) give a linearfunction that equals h(x) on the natural domain ofh(x), and ( 6 ) extend the domain of h to include all real numbers
in such a way that h ( x ) equals the linear function f o r all x.

26. h(x) = x2 x -2x1 +


- 27. h(x) = X 28. h ( x ) = 2x3 X-
2
x2
29. h(x) = x2 -
x -x 4- 12
70 CHAPTER 2: LIMITS, DERIVATIVES, AND CONTINUITY OF FUNCTIONS

In Exercises 30 to 35, ( a ) determine whether lim,+, f (x) existsfor the given function at the given point usin<?informal methods (giving reasonsforyour
answer) and ( b ) zf the limit exists, use the Limit Theorem and special limzts to just^ the steps associated with evaluating the limit.

80. f ( x ) = x 2 + 3x - 1, a = -3 31. g(x) = ___


&+=4

32. g ( x ) = x a=3

11
38. m ( x ) = -i/x2, a = 0

- 3x for x 81
34. r ( x ) = I/x3, a = O 35. f ( x ) = ,a= 1
for x = 1
36. Is the function in Exercise 35 differentiable at a = l ? 2x - 10 and lim -,2x - 10
41. Compute lim ___
87. Which of the functions in Exercises 31, 33, and 35 is c-5+ IX - 51 2-5- / X - 51

continuous at the given point a? 42. Compute lim- 1


38. Repeat Exercise 37 for the functions in Exercises 30, 32, and lim-
+-I+ 2x - 2 7-1- 2x - 2
and 34.
43. Give an informal argument to show that lim,,2
39. Determine limz+ah ( x ) for h ( x ) at the point where h ( x ) is
( 3 x - 4) # 5.
not defined in ( a ) Exercise 27 and ( b ) Exercise 29.
40. Repeat Exercise 39 for functions in ( a ) Exercise 26 and
( b ) Exercise 28.

In Exercises 44 to 55, ( a ) using the dejinition given in (2.3.11, determine the derivative of the giuen function at a typical number in its domain, and
( b ) determine the point-slope equation of the line tangent to the graph when the independent variable equals 1.

-1 1 1
44. J(x) = ~ 45. g(x) = x +- 46. h(x) = x - - 1
47. f ( t ) = - + 3
7x X X t2

48. y = 3x2 +x -2 49. y = fl 50. f ( x ) z-


1
51. g(x) =
1
-
& x3

+1 1
52. m ( x ) = 5x2 - 4x 53. f ( n ) = u3 - 5u2 34. g(w) = - 55. h ( r ) = 6r2 + 10
w + l

In Exercises 56 to 60 determine the point(s) ( Zf any) where the given function is discontinuous and state why the function is discontinuous there.
-1 ifx<2
1
87. f ( x ) = 2
x + I

59. f ( x ) =
3 ifx=2

60. f ( x ) =
[ x 2 - 5x
x - 3
1
+6 for #3
for x = 3

In Exercises 61 to 65 sketch the graph o f a fiinction that satzsjies the given conditions.
G I . f is defined for all x in [0, 21,fhas a limit at 1 , f i s not 6 5 . f i s defined for all x in [0, 21, lim,_lf(x) = +co.
continuous at 1. 66. Find lim,,,-f(x) and lim,+l+f(x), where
62. f is defined for all x in [O, 21, f ( x ) <
3 for all x , if x 2 1.001
limr,lf(x) = 3. if 1 < x < 1.001
G 3 . f is defined for all x in [0, 21, lim,,l-f(x) = - M,
limz+l+J(x) = 2.
+1 if .99 x< <1
x2 + 5x if x 5 .99
04. f is continuous on [ 0 , 2 ]but is not differentiable at x = 1
or x = 3 / 2 .

J1isr:ellaneous Exervises
1. As a preparation for Exercise 2, suppose that ( a ) Show that I(x + h ) is as much above l ( x ) as l ( x - h )
I(x) = mx + b is a linear function with, say, m 0. > is below I ( x ) for h 0.>
SECTION 2.5: LIMITS, CONTINUITY, AND DIFFERENTIABILITY 71

( b ) Interpret part ( a ) graphically. 7. Repeat Exercise 6 forf(x) = x 2 + 4.


(c) Determine the average of I(x - h) and I(x + h). 8. Determine the point where the line tangent toy = x 3 at
2. When examining the freely falling object problem x = 1 crosses the curvey = x3.
numerically, many students recognize that the average 9. ( a ) Determine where the slope o f f ( x ) = 3x - x 3 is a
velocity between x and x + >
h for a particular h 0 and maximum, that is, where the graph has steepest positive
x is as much above the proposed instantaneous velocity slope.
a t x as the average velocity between x - h and x is below ( 6 ) Determine where the tangent t o f ( x ) is horizontal.
the proposed instantaneous velocity. See Table 2.3 and (c) Sketch the graph off(.).
subsequent calculations. T h e students then go on to 10. Repeat Exercise 9 forf(x) = x 2 - 2x3.
conjecture that the instantaneous velocity at that point 11. We know that iffis differentiable a t a, thenfis continu-
is the middle or average of the average velocities on ous at a.
either side of x . (They are often told by the instructor ( a ) Is it necessary thatf be differentiable at a whenJis?
that although this happens to be true here, it is not true ( b ) Is it necessary that f be continuous at a when f
in general.) exists at a?
( a ) Prove that this conjecture is true forf(x) = 1 6 2 at 12. (a) Where is the largest integer function s ( x ) = [ x ]
x = 3 by comparing differentiable? (Remember that the definition of deriva-
tive is two-sided.)
f(3 + h) - f ( 3 ) and f(3 - h ) ( b ) What is s(x) when it exists? Give a geometric argu-
h -h
ment.
( b ) Prove that the conjecture holds for any x with 13. Define sl(x) = smallest integer n > x. [Thus s,(1/2) =
f ( x ) = 16~. 1 = sl(l), with ~ ~ ( 1 . = 1 ) 2.1
(c) Prove that the conjecture holds for any x with ( a ) Sketch the graph of sl(x).
+ +
f ( x ) = ax2 bx c any quadratic function ( a , b, c are ( b ) Where is sl(x) discontinuous?
constants). 14. Refer to Exercise 13.
( d ) Prove that the conjecture fails for g(x) = x 3 at ( u ) Sketch s ( x ) - sl(x).
x = 1. ( b ) Where is s ( x ) - s l ( x ) continuous?
( e ) Explain the difference between g ( x ) and the quad- 15. A malfunctioning rocket (weak thrust) is shot vertically
ratic function in part (c) by regarding the various aver- upward with an initial velocity of 144 ft/s and is at
ages as functions of h and/or noting which are linear and height h(x) = 144x - 3x2 after x seconds.
which are not. (See Exercise 1.) ( a ) Determine the function that gives the instantaneous
(f)Determine those functions for which the conjecture vertical velocity at time x.
holds and those for which it fails. ( b ) When does the airborne rocket have zero velocity?
(c) What is its height at this point?
3. 9 T h e volume V of a right circular cylinder of radius r ( d ) When does the rocket crash and what is its down-
and height h is given by V = m2h. ward (negative) velocity at this time?
( a ) Suppose that r is fixed and h is allowed to grow by ( e ) What is the maximum height reached by the
Ah. Compute AV, AV/Ah, and sketch the situation. rocket?
( b ) Compute dV/& and relate this result to the area of (f)Graph 4).
the top of the cylinder. (g) Compare ( a ) t o ( f ) with Sec. 2.3, Exercise 21, to
(c) Suppose that h is fixed and r is allowed to grow by determine the effect of the weak thrust in terms of
Ar. Compute A K AV/Ar, and sketch the situation. maximum height and when the rocket strikes the earth.
( d ) Compute dV/dr and relate this result to the lateral 16. Repeat Exercise 15 for a n initial velocity of 96 ft/s and a
area of a right circular cylinder. (If you do not know the height function of h(x) = 96x - 8 x 2 . In part (9)compare
formula, guess it.) with Sec. 2.3, Exercise 22.
17. With reference to Exercises 15 and 16, what is the effect
4. (An alternative limit notation) If the standard notation of strengthened rocket thrust on the constant a in the
for the two-sided limit limr-af(x) = L is replaced by function h(x) = uox - ax2 (where un is the initial vcloc-
lim,p-a,+of(x)= L , then give the appropriate analogs ity)?
for the right- and left-hand limits. (These will not use 18. ( a ) Does your car have an instantaneous-velocity meter?
absolute value notation.) ( 6 ) Which devices on your car would you use to calcu-
5 . (An alternative definition of derivative) Using argu- late average speeds? (Hznt: One of these is the clock.)
ments similar to those used in proving (2.5.13) and 19. ( a ) Givenf(x) = (l/x) + I , determine d f / d x using the
(2.5.14),show that the definition of the derivative off at definition.
x = a can be rewritten as ( b ) Where does the tangent to the graph off(x) have a
slope of -1/9?
r-a x -a 20. Given f ( x ) = d m , find f(1) using the definition.
21. Letf(x) = mx + b be any linear function.
6. Using the definition in Exercise 5, determine the deriva- ( a ) What is its tangent at any point?
tive off(x) = 6 x 2 - 3x at ( u ) x = 2 and ( b ) x = a. ( b ) What is the slope of this tangent?
72 CHAPTER 2: LIMITS, DERIVATIVES, AND CONTINUITY OF FUNCTIONS

(c) Findf(x) using the definition of derivative. 26. Give reasonable definitions of left differentiable and
( d ) Do your answers to ( a ) , ( b ) , and (c) agree? right differentiable at x = a. (See Exercise 25.)
22. ( a ) Determine where the slope off(x) = x 2 - 2x +
5 is
negative, zero, and positive. 27. 9 T h e volume V of a cube of side x is given by V = x 3 .
( b ) Determine the lowcst point on the graph ofJ: (u) Suppose x increases by Ax. Compute AT AV/Ax,
23. (Tangent to a circle) and, if possible, sketch the situation-with one corner
( u ) Givenf(x) = d m , whose graph is the upper fixed and all growth in the directions of the other
half of x 2 + y 2 = 1, determinef(x) by rationalizing the corners of the cube.
numerator of Af/Ax before taking the limit as Ax -+ 0. ( b ) Compute dV/& and relate this result to the fact that
( b ) Determine f(fi/2) and sketch the graph of f ( x ) the growth takes place in the directions of three faces of
the cube.
and the tangent line.
(c) Determine the slope of the radius line through the 28. (Recall Sec. 2.4, Exercise 19.) Let H ( f , g ) ( x ) represent
point of tangency and show that it is perpendicular to the horizontal distance from the point ( x , x 2 ) on the
the tangent line whose slope is given by f ( x ) . (This graph off(x) = x 2 to the graph of g(x) = x 2 + 1. As x
shows that the tangent line determined by the derivative grows without bound, what happens to H ( f ,g)(x)?
agrees with the classic tangent to a circle.) 29. (See Exercise 28)
( d ) Show the tangent at x = 0 is horizontal. ( u ) Given the two functionsf(x) = x 2 andg(x) = x 2 + 1
24. Determine where the function g(x) = lx2 - 11 is with P = ( x , x 2 ) (x # 0), let Q be the point on the graph
(u ) continuous and ( b ) differentiable. (Sketch its graph.) o f g where the line perpendicular t o the tangent t o f a t P
25. By determining left- and right-hand limits separately, crosses the graph of g. Determine the distance between P
determine whether and Q.
2x - 1 for x <1 ( b ) Repeat part ( a ) , beginning with the line perpendicu-
for x 21 lar to the tangent at Q = ( x , x 2 + 1). Let p be the point
where the perpendicular crosses the graph off.
is differentiable at x = 1. (c) Compare H ( f , g ) ( x ) and the results of ( u ) and ( b ) .

SELF-TEST
74 CHAPTER 3: DIFFERENTIATION

Section 3.1: Basic Rules of Differentiation

Objeetives: DERIVATIVES OF POLYNOMIAL FUNCTIONS


Use the rules of differentiation to In Chap. 2 we introduced the derivative of a functionf(x) as a means of
determine derivatives of sums, studying how the values of the function change as x changes. Recall the
products, quotients, and (integral) following.
powers of polynomial functions.
3.1.1 Definition Given that f is defined in some open interval containing x , we
define the derivative .ff with respect to x as

lim f ( x + Ax) -fW


AX-0 Ax
and denote it by f ' ( x ) , df/dx, or, i f y = f (x),y' and dy/dx. The derivative off
with respect to x is also denoted by D, f or, ify =f(x), Dxy. Determining the
derivative of a function is commonly referred to as dfeerentiating the function.
We have interpreted the derivative as the instantaneous rate of change off
with respect to change in x as well as the slope of the line T tangent to the graph
off(.) at the point (x,f(x)). The instantaneous rate of change was defined as the
limit of average rates of change over smaller and smaller intervals Ax, whereas
the slope of the tangent line was defined as the limit of slopes of "secant lines"
through P = (x,f ( x ) ) and Q = ( x +
Ax, f(x) +
Ax)), as shown in Fig. 3.1.
Table 3.1 collects for reference some of the results of our examinations of
specific functions in Chap. 2. However, direct application of 3.1.1 gives us our
first general rule (see Progress Test 1, Prob. 9):

3.1.2 T h e o r e m The derivative of a constant function is zero.


The pattern evident in functions 3 and 5, Table 3.1, suggests that
Iff(.) = x4, thenf'(x) = 4x3
We shall prove a general rule that establishes and formalizes this pattern.

3.1.3 T h e Power R u l e for Positive Integers If n is a positive integer and


f(x) = xn, then f ' ( x ) = nx"-l.
Prooj We need to examine ( x + Ax),. Although the Binomial Theorem tells us
how to expand (x + Ax), completely, the first three terms tell us all we need to
know:
(x + Ax). = xn + nx"-l(Ax) + A x " - ~ ( A x ) ~+ Bx"-~(Ax)~
+ . . . + (Ax).

Y Q= ( x + Ax, f ( x + A x ) )
4
f

I I I
X x + Ax
Fig. 3.1
SECTION 3.1: BASIC RULES OF DIFFERENTIATION 75

Function Deriuutiue Remarks


-~

1. f(x) = 3 f'(x) = 0 Zero rate of change, horizontal


graph: slope = 0
2. , f ( x ) = 2x +3 jyx) =2 Constant rate of change, graph
is straight line of slope 2
3. f(x) = x 2 f'(x) = 2 x Parabola concave up, horizontal
tangent at x = 0
4.f ( x ) = x 2 +1 f'(x) = 2x Parabola concave up, horizontal
tangent at x = 0
5. f(x) = x3 f ' ( x ) = 3x2 Slope never negative, with horizontal
tangent at x = 0
6. f ( x ) = 3 - x 2 f'(X) = -2x Parabola concave down
7. f(x) = 16x2 f ' ( x ) = 32x I f f i s distance fallen, thenf' is
instantaneous velocity
8. f ( x ) = x 2 +x f'(X) = 2x +1 f' is sum of derivatives of x 2
and of x
9. f(x) = l / x f'(x) = -1/x2 f(0) andf'(0) undefined, butf'(x)
< 0 otherwise
10. f(x) = G f'(x) = 1/(2&) f(x) = x1'2,f'(x) = 1/2x-"2

Table 3.1

Here we used the letters A and B, respectively, to denote the coefficients of the
third and fourth terms since they are unimportant to the final result. Now, by
the definition of the derivative,

(x + Ax)" - x"
= lim
Ax-0 AX

[x" + nx"-l(Ax) + A X " - ~ ( A X )+~ terms of higher degree in Ax] - x"


= lim
AX-0 Ax
nx"-l(Ax) + A x " - ~ ( A x ) ~+ terms of higher degree in Ax
= lim
4x-0 AX
lim (nxn--l + A X " - ~ ( A X+) terms of higher degree in Ax)
4,-0

= nxn-l H

The last line of the above proof made typical use of the Limit Theorem
[2.5.4(a)].
The Power Rule agrees with our earlier results; for example, D,(x2) = 2x.
By item 7, Table 3.1, we know
D X ( l 6 x 2 )= 3 2 x = 16DX(x2)
The following states more generally that the derzvatzve of a constant tzmes afunctzon zs
the constant tzmes the derivative.

3.1.4 The Constant-Multiple R u l e If k is a constant andfis differentiable at


x, then
DXW(X)l = kD,[f(X)l
ProoJ By the definition of D,[kf(x)]
76 CHAPTER 3: DIFFERENTIATION

) -f(x)
= k lim f ( x since constants can be pulled out
AX-0 Ax
of limits; see [2.5.4(b)]
= kD,[f(x)] by definition of k D x [ f ( x ) ] W

Combining the Power and Constant-Multiple Rules we have


3.1.5 Dx(axn) = anx-l (a is a constant and
n a positive integer)
All we need now to differentiate a typical polynomial (using rules instead of
the definition) is the ability to differentiate term by term. We shall show that the
derivative of a sum is the sum Ofthe derivatives.

3.1.6 The Sum Rule Suppose f ( x ) = u(x) + v(x), where u(x) and v(x) are differ-
entiable. Then
f ( x ) = u(x) + u(x)
Prooj Let us separate the usual limit quotient into a sum of u(x) and v(x) parts
and then apply the Limit Theorem [2.5.4(a)].

f ( x ) = lim f ( x + Ax) - f
Ax-0 Ax

= lirn
+
[ ~ ( x Ax) + ~ ( +x Ax)] - [ ~ ( x +) ~(x)]
Ax-0 Ax

= lirn
Ax-0 [
u(x + Ax)
Ax
- u(x)
+ v(x + Ax)
Ax
- v(x)
I
= lim
U(X + AX) - U(X) + lim U(X + AX) - U(X)
Ax-0 Ax Ax-0 Ax
= u(x) + v(x)
Although (3.1.6) is a statement about the sum of two differentiable functions,
it also applies to the sum of 3,4, 10, or any number of differentiable functions.
Furthermore, it extends directly to differences of functions because

f (4= 4 x 1 - V(4
= u(x) + (- l)v(x)
Now apply the Sum Rule and then the Constant-Multiple Rule to obtain
f ( x ) = u(x) + (- l)v(x)
= u(x) - v(x)
As a result of (3.1.5), (3.1.6), and our discussion, we know that:

3.1.7 Theorem Any polynomial function is differentiable.

Example 1 Differentiate f (x) = 1lx7 + 6x5 - 3x2 + 8 x - 1.


Solution We can differentiate term by term and add the results as we go along:
f ( x ) = 11 * 7 x 6 +6 . 5 -~ 3 ~* 2 x 1 + 8 . 1 e x o -0
= 77x6 + 30x4 - 6 x + 8 0

In Exercises 29 and 30 you are asked to interpret the following special cases in
terms of graphs and rates of change.
SECTION 3.1: BASIC RULES OF DIFFERENTIATION 77

(Linear and Quadratic Functions) Let a, b, c be constants. Then:


+
( A ) I f y = ax 6 , then dy/dx = a.
+
( B ) I f y = ax2 bx + c, then dy/dx = 2ax + b.

PROGRESS TEST 1

In Probs. 1 to 10 differentiate the given function using


the rules previously developed.
1. f ( x ) = x41 2. f ( x ) = x
+ +
3. f (x) = 4x3 + 3x2 3x 2 4. g ( x ) = 51x4 + +
26x2 432.21
5. f ( x ) = + + +
x 8 7 x 7 6 x 6 g(x) 6. s ( t ) = 4t3 - 3t/5 1 +
[where g ( x ) is differentiable]
+ +n
7 . p ( 7 ) = nr2 2nr 8. f ( ~=) ( x 5 - 3)(x2 + 4 ~ )
9. Show that iff(.) = b is a constant function, then
f ( x ) equals zero.
10. Where does the graph off(.) = x 3 - 6x2 have a
horizontal tangent?

PRODUCTS AND QtOTIENTS


Note that in Prob. 8 of the previous progress test it is necessary to multiply the
two factors before differentiating. Attempting to differentiate the product by
taking the product of the derivatives leads to a n incorrect answer, as can be seen
more vividly by treating f (x) = x 2 as the product f ( x ) = x x . -
Since the derivative of each factor is 1, the product of the derivatives is 1. But
of coursef(x) = 2x # 1. The derivative of a product is certainly not the product
of the derivatives!
We shall state the rule for products and then apply it to Prob. 8 before
proving it.

3.1.8 The Product Rule Supposef ( x ) = u(x)v(x), where u(x) and v(x) are differ-
entiable. Then
f ( X ) = u(x)v(x) + v(x)u(x)
Thus the derivative of the product of two functions is the sum of thejirstfunction times
the derivative of the second function plus the secondfunction times the derivative of thejrst
function.

Example 2 Use the Product Rule to differentiatef(x) = ( x 5 - 3 ) ( x 2 + 4x) and check the
result against the previously obtained derivative.
Solution Deriv. Deriv.
1st of 2d 2d of 1st
I - I -
f ( x ) = (x5 - 3)(2x + 4) + (xz + 4x)(5x4)
+
= 2x6 + 4x5 - 6 x - 12 5x6 20x5 +
= 7 x 6 + 24x5 - 6x - 12
which agrees with the solution obtained to Progress Test 1, Prob. 8. 0

9 Proof ofthe Product Rule: In applying the definition of derivatives to the


productf(x) = u(x)v(x)it is helpful to regard u(x) and v(x) as the lengths of the
78 CHAPTER 3: DIFFERENTIATION

sides of a rectangle, s o f ( x ) represents the area of the rectangle. Then, changing


x to x + A x changesf(x) to
f(x + Ax) = U(X + Ax)v(x + Ax)
andf(x + Ax) -f(x) may be regarded as the area of the shaded region in
Fig. 3.2(a). To simplify notation we label.
u(x + Ax) - u(x) as Au + Ax) - v(x) as Av
and v(x

as shown in Fig. 3.2(b). In particular, then, we can regardf(x + Ax) -f(x) as


the total area of the three regions labeled I, 11, and I11 in Fig. 3.2(b):
I I1 I11
f(x + Ax) - f ( x ) = u(x) Av + u(x) Au + Au Av.
By definition off (x) we have

f(x) = lim f ( x + Ax) - f ( x )


Ax-0 Ax

= lim
u(x) Av + v(x) Au + Au Av
(1) AX-0 Ax

AX-0

= lim u(x)-
AX-0 [ :I] [u(x)-i:] + lim [t:
+ Ax-0
lim -Av ]
Ax-0 see 25.4(a)1

[
= lim u ( x ) ]
AX-0
[lim kAx ]+ [ lim v(x)] [lim AX1
Ax-o Ax-0 AX-o

+ [dih Av] see [2.5.4(6)]


(2) = u(x)v(x) + v(x)u(x) + u(x) * 0
= u(x)v(x) + u(x)u(x)
Note: Both u(x) and v ( x ) are differentiable, so
AV Au
lirn - = v(x) and lirn - = u(x)
Ax-0 AX 01-0 AU
Furthermore, limAx,OAv = limAx,O[v(x Ax) - u(x)J = 0 because u(x), +
being differentiable, is Continuous [by (2.5.14)], and SO [2.5.13(d)] applies. W
Note that although the diagrams of Fig. 3.2 aided in the organization of the
above proof, the actual proof is independent of the diagrams. [See Progress
Test 2, Prob. 5, for another approach to line (l).]

u(x + Ax)
(a)
Fig. 3.4
SECTION 3.1: BASIC RULES OF DIFFERENTIATION 79

9 We can apply the Product Rule tof(x) = x 2 regarded as the product


f(x)= x * x
to obtain
- -
f ( X ) = x 1 + x 1 = 2x
Here we had u(x) and u(x) = x , so in this case the rectangle of Fig. 3.2 reduces
to the square associated with Sec. 2.3, Progress Test 4. In fact the idea that a
circular disk grows by circumferences or that a square grows by its two sides
(the two sides in the direction of the growth) has a n interesting extension to the
u(x)~(x) rectangle of the Product Rule. The rectangle in Fig. 3.2 grows by the
side of length u ( x ) times the rate of growth in that direction, namely u(x), plus
the side of length u ( x ) times the rate of growth in the other direction, namely
u(x).
We now turn to quotients of functions, beginning with a simple case.

3.1.9 Theorem Iff(.) = l / u ( x ) for u(x) differentiable and ~ ( x #


) 0, then

Prooj As usual, we apply the definitions off(x) andf(x):

f ( x ) = lim f ( x + Ax) - f ( x )
Ax-0 Ax

= lim
[l/u(x + Ax)] - l / u ( x )
AX-0 AX
+ Ax) (1/B) -
= lim
AX-0
u(x) - u(x
AX(V(X+ Ax)u(x))
1
(1/A) -
-
(B - A )
CAB 1
= lim -
AX-0 1 v(x + Ax)u(x)
1
= -1im see [2.5.4( a),( b)]
AX-0 Ax

Note that u(x), being differentiable, is continuous, so [2.5.13(c)] applies to


guarantee that
lim u(x
AX-0
+ Ax) = u(x) W

Example 3 Use (3.1.9) to differentiatef(x) = 1/x2.

Solution
-2x - -
f(x) = - -2 0
(x2)2 - x3

Note that we can rewrite l/x using a negative exponent as x - l and its
derivative (see Table 3.1, function 9) as - 1 * x - ~ Similarly,
. we can rewrite 1 / x 2
as x - ~and its derivative as -2xP3.
This suggests that the Power Rule may extend to all integers, a fact we shall
now formally state and prove.
80 CHAPTER 3: DIFFERENTIATION

3.1.10 Power Rule for Any Integer n


Iff(.) = P,thcnf'(x) = nxnpl
Prooj This result already holds for positive integers (3.1.3).
For n = 0 and x $ 0, f ( x ) = 1 so f ' ( x ) = 0. Hence suppose f ( x ) = x n ,
where n is a negative integer. Then f ( x ) = 1 / ~ - ~where
, -n is a positive integer
(for example, xp3 = l / ~ - ( - ~=) l/x3). By (3.1.9),

- ( -n)x-n-l
-
x-2n

-
- 12X2n-n-1

= nxn-1
Note that D,(x-") = -nx-n-l by the Power Rule for positive integers. W

We shall now use the Product Rule and the fact that any quotient u ( x ) / v ( x )
can be written as a product u ( x ) [ l / v ( x ) ]to prove:

3.1.1 1 The Quotient Rule Suppose f ( x ) = u ( x ) / v ( x ) , where u(x) and v(x) are
differentiable and v(x) # 0. Then

The derivative of a quotient is the denominator times the derivative OJthe numerator,
minus the numerator times the derivative of the denominator, all divided by the denominator
squared.

Proof Ofthe Quotient Rule: By the Product Rule applied to u ( x ) [ l / u ( x ) ] ,


-u'(x) 1
f ' ( x ) = U(.)- + -u'(x) (3.1.9)
[v(x)12 44
-u(x)v'(x) + v(x)u'(x)
-
[4x)I2

Exaniple 4 Differentiate f ( x ) = 3x4


4x7
+ +x 3 5x5
+ 2%
den. deriv. of num. num. deriv. of den.
- 0
(4X7 + 5x5)(12x + 3x + 2)
-/--
(3x4 + x 3 + 2x)(28x6+ 2 5 ~ ~ )
-
Solution f ' ( 4= [4x7 + 5x51I2
0
2 - L

den. squared

PROGRESS TEST 2

1. Find the derivatives of the following functions. Do (6) Y =


x2 +1
not simplify your answers.
+
( a ) j-(.) = (.3 +
2x)(5x4 +
2 0 ~ 3 7) 14x3 + 1
+ + +
( b ) f ( t ) = ( t 2 9t 3)(2t 1)
(dl t(x) =
xz-x-2
SECTION 3.1: BASIC RULES OF DIFFERENTIATION 81

ax +
(el f ( x ) = ___ all letters in (e) and ( f ) ,other
4. +
Keeping in mind thatf(x) = (x3 3x2 + 4)/x2 =
cx + than x, denote constants (x3 + +
3x2 4 ) ( ~ - ~ )differentiate
, f ( x ) using (a)
(f) 4 + +
.) = (ax2 6% c)(mx d ) + the Quotient Rule and (6) the Product Rule.
(c) Show that your results are equal.
2. Use the Product Rule to differentiate f ( x ) = xx2.
+ +
3. Take the derivative off ( x ) = 7 ( x 3 5 x 2 2x - 1) 5. Replacing Au and Av by u(x +
A x ) - u ( x ) and
by means of: u(x +
Ax) - u(x), respectively, in line 1 of the proof
of the Product Rule, show that the numerator of the
( a ) The Constant-Multiple Rule (3.14)
( b ) The Product Rule, using 7 as the first factor quotient equals u(x + +
Ax)u(x Ax) - u(x)v(x).

MIXIKG THINGS CY A BIT


Often we need to use the derivative rules in combination. We shall not simplify
our following answers unless there is a reason for doing so.

Example 5 Differentiatey =
(3d0 + 2x2 + 1)(4x3 + x 2 ) with respect to x.
2x + 1

Solution We can think ofy as a quotient whose numerator is also a product.

-
derivative
density derivative of number number of density
n
dx
- - (2x +
l ) G + 2x2 + 1)(12x2 + 2 ~ +) (4x3 + x 2 ) ( 3 0 x 9 +2
- &+ 2x2 + 1)(4~3>@j 0
4 + 1y
(2x
W
density squared density squared

Example 6 Determine D,f : f(x) = ( x 3 + 3x2 + 2)(x5 + 4x4 + 5)(3x10 + 1lx9 + x),
Solution This is a product of three factors, and to take the derivative, we can group the
first two together and apply the Product Rule to the resulting two factors:
1st factor 2d factor

D,f = (x3 + 3 x 2 + 2)(x5 + 4x4 + 5)(30x9 + 99xs + 1)


+ (3x10 + 11x9 + x ) [ ( x 3 + 3x2 + 2)(5x4 + 16x3) + ( x 5 + 4x4 + 5)(3x2 + 6x)]
(The derivative of the first aggregate factor is in brackets.) 0

As you may have already realized, most problems of this type are subject to
more than one approach. Example 5 could have been done as a product:

f =
3x10 + 2x2 + 1 (4x3 + 2 )
2x+1
Similarly, we could have grouped the last two factors in Example 6 and
proceeded accordingly. Although it usually does not matter which approach
you choose in such problems, different approaches may very well lead to
answers radically different in appearance.
We present one last example, which illustrates a typical problem.
82 CHAPTER 3: DIFFERENTIATION

Example 7 Give a n equation of the line tangent to f ( x ) = (x + 1)/(2x - 3) a t x = 5.


Solution We shall use the point-slope form of the equation of a line, b u t first we need the
slope at x = 5.

f(4=
( 2 x - 3)(1) - ( x + 1)(2) -- -5
(2x - 3)2 (2x - 3)2
Thus,

is the slope. Now we need the second coordinate of the point o n the graph of
f ( x ) . Substituting x = 5 into the function (into f(x)-not f ( x ) ! ) ,we get
f(5) = 6 / 7 . Thus the point o n the curve is (5,6/7), the slope is -5/49, a n d so
the equation of the line is
r--=
6 --(
x-5) 0
7 49

SECTION 3.1 EXERCISES

In Exercises I to 28 dzffeerentiate the giuen,function with respect to the indicated variable using the previous rules.
1. f ( x ) = - 2 x - 3 2. f ( x ) = 5 - 4x2
3. g ( x ) = x 3 + 5x2 + 1 4. g(t) = t10 + t9 + t s + 100
5. f ( x ) = 0.01x - 0.2 6. y = vr2
7. t ( x ) = ( x 2 +
1)(7x3 + 3x2 + x - 3) 8. A ( t ) = ( t 5 + 4t3)(t2 - 5t - 4)
10. y(t) = -
5 - 2t
1-t

12. f ( r ) = L -
r + l
+-r 2r2+ 1

13. y =
-72 +2 14. m(x) =
( 4 ~ - 2 ) ( 3 ~ I) +
4x3 +x x-1

15. f ( x ) =
(x + + 2) 16. u=---
u2 + 7u6 3
x2 +3 4u5 - 5 u2 +I
1
17. g ( x ) =
+ 3x2)(7x - 2)
18. f ( z ) = + 25 - 23
(23 z + l

19. m(y) =
(9- 5Y)(Y4 + 2Y - 1)
69 - 9
t
+ + + 2t + l ) ( t 2 - 5t - 4)
x2
21. r ( x ) = 2-4 - 22. x(t) = - (t3
+ 2x + 1
x3 t3 3

23. u = ( x + l)(x4 + x3 + 1)(3x2 - 7x)

25. y =- 2x + 4 - z x 3 + I 26. f(w)= (w5 + 1 ) ( d + 2~ + 1) - -1


9 - 3 xZ-x+l zW3

27. f ( ~=) -
UZ - u3
+ -6 u2
28. f ( x ) = -
1
x2+1
+-+-1
x 2 + 2
1
x2+3
29. Interpret remark A before Progress Test 1 ( a ) graphi- ( 6 ) Write the equation of that line in point-slope form.
cally, (6) in terms of rate of change. (c) Write the equation of the line perpendicular to the
30. Repeat Exercise 29 for remark B. Compare with the graph o f f a t x = 2.
algebraic approach of (1.3.3). For what x is the tangent 32. Repeat Exercise 31 forf(x) = x4 - 4x3 at x = 1.
line horizontal? 33. Where is the slope of the tangent to the function
31. ( a ) Find the slope of the line tangent to f ( x ) f ( x ) = 1/(x3 +
1) negative?
3x9 + 2 x 2 + 1 at x = 2. 34. Where is the slope of the tangent toy = l/x2 negative?
SECTION 3.2: DIFFERENTIATING COMPOSITE AND IMPLICITLY DEFINED FUNCTIONS 83

85. An object is thrown upward with initial velocity of 4 1. Use (3.1.9) and Exercise 39 to find a formula for the
120 ft/s, so its height at time t is given by h(t) = derivative of h ( x ) = l / [ u ( x ) 1 2 .
120t - 16t2, Determine: 42. Apply the formula developed in Exercise 41 to
(a) Its vertical velocity at time t. f ( x ) = l/(mx +
b)?
( b ) The maximum height it can attain. 43. Apply the formula developed in Exercise 41 to
(G) The time it hits the ground. jy.1 = 1 4 . 4 + x 3 + x2 + 312.
86. Repeat Exercise 35 but with the object thrown from a 44. Assuming that f ( x ) = u ( x ) / u ( x ) is differentiable, derive
96-ft-high platform. Thus its height is given by the Quotient Rule using the Product Rule applied to
+
H ( t ) = 120t - 16t2 96. (Assume that the object does f(x)u(x) = 4.).
not hit the platform on the way down.) 45. Using the Product Rule, derive a rule for differentiating
~-
f ( x ) = [dx)&)lm(x).
37. 9 The surface area of a cube of edge x is composed of six
squares of area x2. Knowing that the rate of change of 46. Q Use a rectangle of height 1 and width u(x) +
~ ( x to
)
area of a square with respect to change in its sides is 2x interpret geometrically the numerator of the fraction
tells us that the rate of change of a cubes surface area occurring in the derivation of the Sum Rule. (Let u and u
with respect to x is 12x because of at least one of the facts grow in opposite directions.)
about derivatives. Cite at least one property of deriva- 47. Use a rectangle of height k and widthf(x) to interpret
tives to justify this analysis. the derivation of (3.1.4).

38. Let V = (4/3)vr3. Find ( u ) dV/dr, ( b ) the derivative of 48. +


Determinedf/dxforf(x) = x3 g(x)h(x) - l/u(x) 4x,+
4ar2, and (c) the derivative of 8 w . where g, h, and u are differentiablc functions of x.
39. Using the Product Rule, assuming that u(x) is differen- 49. Write the list of differentiation rules in fractional (df/dx)
tiable, find a simple formula for the derivative of notation.
f ( x ) = 14x)I2.
40. Apply the formula developed in Exercise 39 to
( u ) f ( x ) = (x3 +
3x2 + +
5x 1)2
+
( 6 ) f ( x ) = (mx 6 ) 2 (m and b constants)

Differentiating Composite
Section 3.2: and Implicitly Defined Functions

Objeetives: ANOTllER POCVER RULE


Use the Chain Rule to: We can differentiate
1. Differentiate compositions of
functions.
f ( x ) = (3x2 + 5x)2
2. Differentiate functions raised to using the P r o d u c t R u l e :
rational powers.
f ( x ) = (3x2 + 5x)(6x + 5) + ( 3 x 2 + 5x)(6x + 5)
3. Differentiate imp Iicitly defined
= 2(3x2 + 5 x ) ( 6 x + 5)
functions.
4. Determine equations of More generally, using t h e idea of Sec. 3.1, Exercise 39, if u(x) is differen-
(nonvertical) lines tangent to tiable and
graphs of equations.
f ( 4= [4x)I2
then
f ( x ) = u(x)u(x) + u(x)u(x)
= 2u(x)u(x)
Iff(.) = [u(x)I3,we may regardf(x) as [u(x)I2u(x),a n d apply t h e P r o d u c t R u l e
again using t h e a b o v e result:
f(X) = [u(x)]2u(x) + U(X)[2U(X)U(X)]
= 3[u(x)]%(x)
T h i s is beginning to look like a Power Rule for functions. We n o w take it o n e
m o r e step t o see if the p a t t e r n continues. Suppose
84 CHAPTER 3: DIFFERENTIATION

Apply the Product Rule tof(x) = [u(x)I3u(x), using the previous result:

We shall now formalize the pattern that has emerged.

3.2.1 Power Rule for Functions If u(x) is differentiable and

f ( x ) = [u(x)ln (where n is an integer with u(x) # 0 for n 5 0)


then
f ( x ) = n[u(x)]-lu(x)
This is a special case of the far more general Chain Rule to be stated and
proved shortly, so we shall not prove it here. Of course, the earlier Power Rule
for integers is a special case of (3.2.1), with u(x) = x, making u(x) = 1:
D , [ ( x ) n ] = n(x)-1(1) = nxn-1.

Example 1 Differentiatef(x) = ( x 4 + 7x2 + 2x + 3)5.


Solution f ( x ) = 5(x4 + 7x2 + 2x + 3)4(4x3 + 14x + 2) 0

Example 3 Differentiatef(x) = l/(x3 + x ) ~ first


, using the Power Rule for negative expo-
nents and then using the Quotient Rule and (3.2.1) for the positive integer 7.

Solution

Thusf(x) = -7(x3 x)-(3x2 + + 1).


By the Quotient Rule,
+ x ) ~ ( O )- (1)7(x3 + ~ ) ~ ( +3 Ix) ~
(x3
f(XI =
+
(x3 x)14

(-7)(x3 + x)6(3xZ + 1) - (-7)(3x2 + 1)


- -
( x 3 + x)14 ( x 3 + x)

which equals the previous result. Using the Power Rule directly yields the
quickest answer. 0

Example 3 Differentiate g ( x ) = (x4 + 3x3)11(7x2 + 5x + 2).


Solution We shall use the Product Rule, using the Power Rule on the first factor.

9
dx
= + 3X3)11(14x + 5 ) + ( 7 2 + 5x + 2)11(x4 + 3x3)10(4x3 + gx2)
(x4 0

Example 4 Determine D,h, where

Solution We use the Power Rule on [u(t)I3 and use the Quotient Rule on u ( t ) :
41 +1 41 +1
( t 2 - 1)(4) - (4t + 1)(2t) 0
( t 2 - 1)2

You are encouraged to compute Dth for h(t) in Example 4 regarded as


h(t) = (4t +
1)3/(t2 - 113.
SECTION 3.2: DIFFERENTIATING COMPOSITE AND IMPLICITLY DEFINED FUNCTIONS 85

PROGRESS TEST 1

Differentiate 1 to 6. (You need not simplify your answers.)

1. f ( x ) = (x9 + 2x4 - x 3 + 41)7 2. g(x) =


1
(3
. + 7 9 + 2x + 115
3. U(X) = ( x 2 + 2x)(50x2 + 6 x - 3)4 4. f ( x ) = ( x + 4)
(3x5 + x2)8
(x7 + 6x5 - x2)4
6. g(x) =
( 5 x 2 + 2x)7

THE CIIAIN RULE


The function [u(x)]" is actually the composition of two functions g and u,
g(u(x)) where g(u) = un
(You may wish to review the discussion of composition.) The Power Rule for
functions says that the derivative of this composition is the derivative of g with
respect to u [which is ~ ( U Y -times
~ ] , the derivative of u(x) with respect to x. In
other words, the derivative of the composition is

g'(.(x)).'(x>
which can be written in the easily remembered form

-dg_du
du dx
The following rule gives the derivative of a composition of functions in
general, including the above as a special case. Instead of calling it the Compo-
sition Rule, we follow tradition and call it the Chain Rule.

3.2.2 The C h a i n Rule Suppose that u is differentiable at x a n d g is differentiable


at u(x). Then the derivative of the composition of g with u, g(u(x)), is
dg du
g'(u(x))u'(x) or --
du dx
Before beginning the proof of this most important rule we examine its
meaning in a few situations.
Thinking of the derivative in terms of rate of change, the Chain Rule says
that the rate of change of the composition with respect to x is the rate of change of g with
respect to u times the rate of change of u with respect to x.
Consider, for example, the composition of

g(u) = 3u + 5 with u(x) = 2x + 1


g(u(x)) = 3[u(x)]+ 5 = 3[2x + 11 + 5 = 6x +8
Here u changes twice as fast as x and g changes three times as fast as u, so g
changes six times as fast as x. See Exercises 51 and 52 for further numerical
illustration of this crucial fact.

9 For a more concrete example, recall Sec. 2.1, Example 8. Suppose the radius
r of a circular oil spill is increasing at the rate of 3 m/min; that is, the radius as
a function of time t in minutes is given by r(t) = 3t. The area of the oil spill is
m2,so as a function of time, the area is

A ( t ) = T[[r(t)]2= T[3t]2
86 CHAPTER 3: DIFFERENTIATION

The Chain Rule tells us that the rate of increase in area with respect to time is
the rate of change of area with respect to radius times the rate of increase of
radius with respect to time; that is,

4
dt
= &%*
d7 dt
= 2mr(t)r'(t) = 2m(3t)(3) = 18n-t

Because the functions are simple, with A ( t ) = 9.irt2, the first Power Rule
(3.1.2) applies to yield the same result.
To prove the Chain Rule we apply the definition of derivative to the
composition g(u(x)).A small change Ax in x produces the change Au in u, which
in turn produces the change Ag in g. The rate of change of g with respect to a
change in x is the product of the change in g with respect to u, Ag/Au, and the
change in u with respect to x, Au/Ax. This veans that the basic quotient Ag/Ax,
whose limit as A x -+ 0 is dg/dx, may be written as

Ag
--
-
Ax
_.-
Ag Au
Au Ax
This statement makes sense only if Au # 0 near x, so we divide our proof into
two cases.
Case 1: Au # 0 for small Ax
Case 2: Au possibly = 0
We prove Case 1 here, postponing the more complicated proof of Case 2 to
Chap. 9.
By definition,

A,
'd = lim -
-
dx AX
AX-o

Ag Au
= lim---
AX-o AU AX

= [ lim 3
Ax-o
1[ lim "1AX
AU AX-0
Limit Theorem [2.5.4(b)]

(because u is differentiable
with respect to x )

Looking at limAx+,o-
Ag more closely, we note that u being differentiable
hu
means that u is continuous, by (2.5.14).Thus, as Ax + 0, we know that Au + 0,
by (2.5.13). Therefore,

But g is differentiable, so by definition

Hence we can conclude that the derivative of the composition of g with u is

lim -
. --
*g = lim Ag Au
A X - ~ AX A X - ~ AU AX
dg _
du
-_
du dx
As we have noted, the Power Rule for Functions is an immediate conse-
quence of the Chain Rule with g(u) = un. In this case
SECTION 3.2: DIFFERENTIATING COMPOSITE AND IMPLICITLY DEFINED FUNCTIONS 87

dg dg du
-- _-
dx du dx
du
= n [ ~ ( x ) ] "-
-~
dx
The general Chain Rule is heavily used in the differentiation of trigono-
metric, logarithmic, and exponential functions (Chap. 7). Right now we
present another application.

IMPLICIT DIFFERENTIATION
Recall from Chap. 2 that an equation in, say, x andy may not be solvable for
one of the variables to yield a uniquely defined function of the other. Such
equations arise frequently in applications. For this reason it is useful to extend
the idea of differentiation to deal with such equations. For example, the
+
equation x2 y2 = 9 has a graph, and it is certainly reasonable to seek the
slopes' of lines tangent to its graph. Yet, as we have seen, solving for
y = I+ d n does not give y as a function of x. However, the two functions
y1 = d w and yz = - d m
are defined implicitly by the original equation. (Their graphs are the upper and
lower semicircles, respectively.)
For more complicated expressions in x and y it may be very difficult to
determine whether there arc any functions defined implicitly by the given
equation. Determining such functions by actually solving fory in terms of x can
be very tedious or even impossible. For example, the equation
x55 + 3y3 + "5 - 12 = 0
defines many functionsy = f ( x ) implicitly, but to solve fory in terms of x would
involve solving a fifth-degree equation in y. The question of whether or when
the implicitly defined functions are differentiable is normally addressed in more
advanced courses. We shall be content to determine a formula for the derivative
of the implicitly defined functions, assuming throughout that the implicitly
defined functions arc differentiable wherever the formula is defined. We use the
Chain Rule to do this. The technique is known as implicit dgerentiation.

Example 5 Assuming that y is implicitly defined as a function of x by the equation


x2 +
y2 = 9 , determine dy/& in terms of x and y.
Solution We are assuming that y = f ( x ) satisfies the given equation, so

x2 + [ f ( x ) ] 2= 9
Thus, differentiating both sides with respect to x, we have

or
-[x21
d + &d[ f ( x ) I 2 = 0
dx
d
Now -[x2] = 2x, whereas by the Chain Rule
dx

Hence we know that


88 CHAPTER 3: DIFFERENTIATION

2x + 2 [ f ( x ) ]-
df = 0
dx
so

Sincey = f ( x ) , we conclude that


dy
--
-x
-- 0
dx Y
We replacedy byf(x) in Example 5 to emphasize thaty is a function of x
and that differentiation is with respect to x . Normally we differentiate each side
of the equation in its given form.

Example 6 Determine &/dx given thaty is implicitly defined as a function of x by


2x3 +y3 +~ ~ = 419 ~ 2

Solution We differentiate each side with respect to x . Using the Sum Rule, we differenti-
ate the left side term by term:
-[2x31
d = 6x2
dx
and, by the Chain Rule,

To differentiate 5y4x2we must use the Product Rule, regarding 5y4x2as the
product of 5y4 and x 2 :
d
-[5y4~2] = 5y4(2x) + ( 2 )
dx
Pulling all these derivatives together, wc have

6x2 + 3y2 -4 dY = 0 = -
+ 10xy4 + 20x2Y3 dx d(19) -
dx dx
Hence, solving for dy/dx, we conclude that
dy
--
- 6 x 2 - 10xy4
- 0
dx 3y2 + 2Ox2Y3

Example 7 Determine the slope of the line tangent to the graph of 2x3 +
2y3 - 9xy = 0 at
the point (1, 2). Then give the equation of this tangent line in point-slope form.
Solution First we differentiate implicitly with respect to x ,
6x2 + 6y3 - 9xy - 9y = 0
and solve fory:
9y - 6x2 - 3y - 2 x 2
y = -
6y2 - 9x 2y2 - 3 x
The slope of the tangent line at the point where x = 1 a n d y = 2 is found by
substituting these values into y:
SECTION 3.2: DIFFERENTIATING COMPOSITE AND IMPLICITLY DEFINED FUNCTIONS 89

4
3(2) - 2 ( 1 ) 2 - -
y = -
2(2)2- 3(1) 5
Thus the point-slope form of the equation of the tangent line is
4
y-2=-(x-1) 0
5
In cases where x is not the independent variable, it is probably best to use the
dJd- notation (often called the Lcibniz notation, after its inventor).

Example 8 Assuming that x is a function ofy defined implicitly by the equation


3 + 3y2 - 9x2 = 4
find dx/dy in terms of x and y.
Solution The important thing to remember here is that the dferentiution is with respect to
y . Thus 3y2has derivative (with respect toy) 6y, whereas the derivative of - 9x2
is (through the use of the Chain Rule) - 18x(dx/c/dy). The derivative of 4 is 0,
and using the Product and Chain Rules, we differentiate x3:
.3(1) + y3x2-dx = x3 + 3 2 j -dx
4 dy
Therefore
x3 + 3 x 5 -dx + 6y - 1 8 ~dx- = 0
dy dy
so
dx -x3 -
-
-- 6y 0
dy 3 x 5 - 18x
Although it seems that we are differentiating equations, we should note
that we are assuming the equations hold for all values of the independent
variable in some interval (where, in fact, these equations are identities). See
Exercise 53.

PROGRESS TEST 2

1. Assuming thaty is a function of x defined implicitly 3. The following equation defines x implicitly as a
by the given equation, determine dy/dx. function of t. Find dxldt.
(6) x y = 1
( u ) xy = 1 3x2 + 5t2 - xt = 3
( c ) x2 + y 2 - 2xy + xy3 = 4 x +Y
( d ) -= 1 4. Find the point-slope equation of the line tangent to
-Y = 1 at the point ( 2 , i ) .
the curve
2. For equations ( a ) to ( d ) , find dx/dy.

HATIONAL EXPONENTS
We now enlist the help of both the Chain Rule and the principle of implicit
differentiation to extend our power rules to rational exponents. Up to now we
have dealt only with integer exponents.
Recall that fractional exponents are used merely as another way of writing
expressions involving radicals. Thus
6 512, y5 = 513, $5 = 5lq, ?JjF = ( G ) P = 5Pq
90 CHAPTER 3: DIFFERENTIATION

Also, = = ( f,6)3
= 5 or, using rational exponents,
12.5113 = (53)1/3 = 5l = 5
Similarly,

or
(125)-2/3 = [(125)'/3]-2 = 5-' = -1
25
The same sorts of manipulations apply to algebraic expressions:
f/m= (1 - x4)2/3

f/m= (x - 3)6/5 = (x - 3)(x - 3)1/5


In general, u P / q is defined to be $%. When q is even, uP/q is defined only for
u 2 0.
By extending the Power Rule to rational exponents we can differentiate
expressions involving radicals. We shall state the rule, apply it in a pair of
examples where we have obtained the derivative by other means, and then
prove it using implicit differentiation.

3.2.3 The Power Rule for Rational Exponents Assume thatf(x) = [ U ( X ) ] ~ ,


where u(x) is differentiable and r = p / q , withp and q both integers and q # 0.
Then
f '(x) = r [u(x)]'-V(x)

Example 9 Use the Power Rule to differentiatef(x) = fi.


Solution Writingf(x) = x1l2, we have [with U(X) = x]

--
- l o
2 6
This result agrees with that obtained using the definition of derivative in
Chap. 2 (see Table 3.1). Note that without any overriding reason for a choice,
the derivative can be left in either the radical or the rational exponent form.

Example 1 0 Use the Power Rule to determine dy/dx fory = d m .


Solution Writingy = (9 - x2)lI2, we have [with u(x) = 9 - x2]

-
- --x
(9 - x2)1/2
- -X
0
d n
Compare with Example 5. There we found that, assumingy as an implicitly
defined function of x (one example of which is y = dw),
SECTION 3.2: DIFFERENTIATING COMPOSITE AND IMPLICITLY DEFINED FUNCTIONS 91

4-
- -x
--
dx Y
This agrees with the result of Example 10, wherey = Notice that d m .
dy/dx is undefined at x = ?3. The tangent lines to the circular graph are
vertical a t x = *3.
Proof of the Power Rule for Rational Exponents: First, note that we need only
establish this kule for positive rationals, and then it holds for negative rationals
by the usual argument using the Quotient Rule. Furthermore, we need only
deal with a rule forf(x) = xP/q, because then the result for [u(x)]P/Qfollows
automatically by the Chain Rule.
Assumey = xP/q, with x # 0. Raise both sides of the equation to the qth
power to get
yq = ( X q q = xp
We now implicitly differentiate with respect to x to obtain

We used the Power Rule for integers, and on the left side we used the Chain
Rule, becausey is a function of x.
Solve for &/dx to get
dy - pxp-1 - p x P - 5
- - _ _ - _~
dx qyq-1 4 Yq
But we know that y = xP/q and y Q = xp, so

dy--
-
p xP-l
(xP / q 1
dx q xp

P
- -xP-l-P(xP/q) = -Px ( P / q ) - l
4 4
completing the proof

Example 11 Determine f (x), wherc f (x) = $?


Solution Sirice $P= x2/3 , we apply (3.2.3):

Notice thatf(x) is undefined ifx = 0, sof ( x ) is not differentiable at x = 0, even


thoughf(0) is defined (= 0). [In fact,f(x) = x2l3 is continuous at x = 0.1 0
Nott that for x = 0 andp/q 2 1, the definition of derivative applies to show
dy/dx = 0. If x = 0 and p / q <
1, then dy/dx is undefined, as in Example 11.

Example 1 2 Determine dy/dx i f y = d x 5 - 3x2 + 1.


Solution Herey = (x5 - 3x2 + 1)lI2, so, applying (3.2.3) again,

or
5x4 - 6~
, - 4 -- 0
dx 2dx5- 3 x 2 + 1
92 CHAPTER 3: DIFFERENTIATION

Example 13 Determiney wheny =


x3 +1
Solution We regardy = [e]. Then

-45 (x3 + 1)(2x) - ( 2- 5 ) ( 3 x 2 )


(x3 +
or
-~
-x4 + 15x2 + 2 x ] [note that%) : ( = (+r]
PROGRESS TEST 3

Differentiate the given functions in 1 to 5, leaving your answers in a form using only positive exponents and no radicals.

1. f ( x ) = x12 + + (G)3
3. g ( x ) = 4x23 5 . m(t) = [q-]-2

1
2. f(x) = 3 6 +- 4. h(x) = dx2 - 3 x + 1
$G
-

SECTION 3.2 EXERCISES


-

In Exercises I to 32, dafferentiate the given function:


1. f ( x ) = 3x213 + 6 2.
3. g ( t ) = -\J2/t 4.
+
5 . h(z) = 3/z2 t 1 / 3 / 2 6.
7. t(w) = 9 - 3 W G + 2/w2 8.
9. f ( x ) = 4 f / m 10.

12.

13. f ( x ) = dTT2 14. y = f/m


16.

17. h ( x ) =
1
18. A = d x / ( x 2 - 3x + 1)
v2Ti
~

19. f ( t ) = (t4- 32 - 2)6 20. f (x) = ( x 2 - 2x + 5)( 1IX + 4)


21. f ( y ) = ( y 3 + 1)4(5y2 - y + 2)7 22. f(.) = d ( 5 +~x 4
- 2x + 113

24. y = (x2 + x + 4)-2/3 + rn

26. f ( x ) = Jx + d5TP

28. y = d(x2 6 x + 1) +
-

30. g ( x ) = -/?,

32.

I n Exercises 33 to 44, using implicit dzfferentiation, ( a ) Jind dy/dx and ( 6 ) Jind dx/dy.
33. x 2 + y 2 = 25 34. x z + ( y - 2)2 = 9 35. 3 x y + y 3 - x 2 + 2 = 0 36. x y = x + y

37. x y = 4 38. \lxy=1 39. f i + x = x 40. ~- G=qi


Y x +Y
41. (x + = 3x9 42. (x + J ) ~ + =3 43. x2 + 2xy +y2 =1
Y
SECTION 3.3: DIFFERENTIALS 93

44. (x + J ) = ~ 1 (compare with Exercise 43.) (c) Solve the equation x2 + y 2 = 25 fory and differenti-
45. Determine dy/dx, given the equation x = y 2, and+ ate the positive function with respect to x. Compare with
interpret dy/dx in terms of the graph of x = y + 2. the result of Exercise 33.
46. ( a ) Determine the equation in point-slope form of the 47. Repeat Exercise 46(a) and (6) for the equation
line tangent to the circle of Exercise 33 at the point x 2 + y 2 - 6x - 8y = -20 at (5,5). Note that you will
(3,4). need to determine the center of the circle by completing
( b ) Is this line perpendicular to the radius through the square.
(3,4)?

In Exercises 48 and 49 determine dy/dx for the given equation at the given Foint.
48. +
x 2 xy + y 3 = 11 at ( I , 2) 53. Repeat Exercise 51 with u(x) = 2x2 - 3 , g(u) = u2 + 1,
49. 4y3 - x? - 2x + 5y = 0 at (0,O) x = 2, A x = 1.
50. Given 3x +2y - 4 = 0: S 3 . For a a nonzero constant, show by differentiating that
( u ) Differentiate this equation implicitly to find dy/dx. (x + a)3 = x 3 + 4x2u + 6 x 2 + a3 is not an identity
( b ) Solve the original equation for y and find dy/dx over any interval.
explicitly. 54. For y = - d m , compute dy/dx using (3.2.3), and
(c) Interpret dy/dx in terms of the graph of the equation.
compare with the result of Example 5. (Be careful.)
5 1. Suppose u(x) = 3x + 1 and g(u) = u2 - 5. Let x = 1 and
A x = 1. Compute Au and Ag.

Section 3.3: Differentials

Objectives: COMPI-TTNG DIFFEIkENTIACS


1. Compute and interpret Supposey = f ( x ) is a differentiable function. We have used the notations dy/dx,
differentials. d f d x , D,y, D,f, y', andf'(x) interchangeably to denote the derivative of the
2. Use differentials to approximate function with respect to x. Although it is very useful, the fractional Leibniz
quantities. notation suggests that the derivative is somehow a quotient of two objects, one
called dy and the other dx. This interpretation is reinforced by the definition of
the derivative as a limit of a genuine quotient a n d by statements such as the
Chain Rule, which written in Leibniz notation is

dy dy _
du
-_
--
dx du dx
It turns out that it Zs pmsible to define quantities dy a n d dx in such a way that
the quotient of these quantities dy/dx does exist a n d equals the derivative. These
quantities are useful both in calculations of approximations a n d later in the
integral calculus.

3.3.1 Definition Let y = f ( x ) be a differentiable function of x .


( a ) T h e dgerential of x, denoted dx, is defined to be any change A x in x , that is,
any real number A x = dx.
( b ) T h e dfferential ofy, denoted dy, is defined by 4 = f ' ( x ) dx. (If y is not
mentioned, we use df instead of 4.)

With this definition, if dx # 0, then the quotient of differentials equals the


derivative:

We rigged the definition of differentials so that the quotient of differentials,


which looks just like the Leibniz notation for the derivative, turns out to be equal
to the derivative when A x = dx # 0. This duality parallels the situation that
occurs for fractional numbers. We can regard $ as a symbol for a number (dy/dx
94 CHAPTER 3: DIFFERENTIATION

as a derivative), or we can regard it as the quotient of two numbers ( d y / d x as the


quotient of differentials). Our point of view makes no difference, because they
are equal! Which point of view we use at any given time depends upon what we
happen to be doing a t that time.
Calculating differentials amounts to calculating derivatives and multiplying
by the appropriate dx.A particular value of dy depends upon two quantities: dx
and x.

Fory = x3 1 determine:
+-
Example 1
X
(u) dy for arbitrary x and dx
( 6 ) dy for x = 2 and A x = 4
(c) dy for x = 2 and Ax = 0.01

Solution (u)
(
dy = 3 x 2 --
;Jdx
(b) dy = ( 3 22 + -$)(+) = 5.875
(c) dy = -(0.01)
47 = 0.1175 0
4

Although in Definition ( 3 . 3 . 1 ) dx = Ax, dy is not in general equal to Ay, but


rather is an approximation to Ay in the following sense. When defining the
derivativef(x), we used Ax to represent a small change in x , which in turn
produced a change

in y = f (x). We then defined

AY = f ( x )
lim -
AX-o AX
But this means that for small Ax,

(=: means is approximately equal to or is close to.) As a result, for small A x ,


Ay = : f ( x ) Ax = dy
Although the change Ay in the value of a function resulting from a change Ax
in its independent variable is often of interest, Ay is usually considerably more
difficult to compute than dy. Hence being able to replace Ay by dy for small
dx = Ax proves to be quite advantageous. Compare the calculation of dy in
Example l(a) with just the first two steps necessary to compute Ay for
y = x3 + l / x :

(X + Ax)3 + 7 1 ] - [x3 +
x + Ax
$1
SECTION 3.3: DIFFERENTIALS 95

PROGRESS TEST 1

1. Fory = (x + 1)2 - (l/x) compute: ( e ) (for calculators) Ay - dy for x = 2, dx =


( a ) dy at a typicalx and dx. a x = 0.01.
( b ) dy for x = 2 and dx = 0.1. 2. +
Forf(x) = (1 - x3)/(1 x2) compute:
(c) dy for x = 2 and dx = 0.01. (a) dJ:
( d ) Ay for a typical x and Ax. (Do not simplify.) ( b ) dffor x = -1, dr = 0.01.
~~ __ ~

INTERPRETATIONS OF DIFFERENTIALS
We can interpret the approximation of Ay by dy graphically (see Fig. 3.3). T is
the line tangent toy = f ( x ) at the point P = (x,f(x)) and Ax = dx is a change in
x. (We took Ax to be positive in the picture, but could just as well have
used Ax 0.)<
Now (length of %)/(length of 5 )is the slope of T, which isf(x). But the
len-gth of @ is Ax = dx, so, substituting, we have

Length of RS
= f(4
dx
that is, length of RS
= f ( x ) dx. Butf(x) dx is our definition of dy. Thus we can
interpret dy graphically as the change iny along the tangent line due to the change
Ax = dx in x. We already have seen, in interpreting the derivative graphically,
that Ay is the change iny along the curve due to the same change in x. Thus in Fig.
3.3.
A, - dy = length of @
Hence we can readily see that for small Ax, the tangent line is close to the curve,
and hence Ay - dy is small.

Q Besides its graphical interpretation, the differential dy has an interpreta-


tion in concrete geometric terms for those cases where y = f ( x ) represents the
area or volume of some geometric object. We examined Ay and dy/dx for several
such cases in the narrative, progress tests, and exercises of Chap. 2. Perhaps the
simplest involves
y = x2
regarded as the area of an x-by-x square. (See Chap. 2, Sec. 3, Progress Test 4.)
The various constituents of Ay resulting from an increase Ax in x are labeled
in the shaded portion of Fig. 3.4(a).

Q= ( x + Ax,f(x + A X ) ) - Y =f(.)
Ax

! ! X AX
X x + Ax (a)
96 CHAPTER 3: DIFFERENTIATION

A,? In particular,
Ax
Ay = ( x + AX)^ - x2
= 2 x A x + AX)^
But with Ax = dx, 2 x A x = dy, so
Ay = dy + AX)^
As a result,
Ay - dy = (Ax)
is the area of the upper right corner. This quantity, corresponding to the length
of @ in Fig. 3.3, is extremely small when Ax is small. For such Ax, the area Ay
is made principally of the area of the two long rectangles whose area is 2x Ax =
2x dx = dy [Fig. 3.4(6)].That Ay - dy approaches 0 much faster than Ax can
also be seen numerically in Table 3.2, where x is taken to be 100:

AY I-lY- 41
200 20 1 1
20 20.01 0.01
0.01 2 2.0001 0.0001
0.001 0.2 0.20000 1 0.000001
0.0001 0.02 0.02000001 0.0000001
0.0000 1 0.002 0.0020000001 0.000000001

Table 3.2

When dx decreases by a factor of &, Ay - dy decreases by a factor of &.


In this example the faster than statement can be written more precisely as

- dY = 0
AY _
lim _ _
Ax-0 Ax
The statement is true in this case because

AY
~ - dY -
- ( A x_
) 2 = AX _
AX Ax
More important, we can apply this analysis to a differentiable function in
gencral. In so doing, we shall use differentials to clarify a phenomenon that
occurred each time we applied the definition to determine the derivative of a
specific function y = f ( x ) . In each instance the quotient A y / A x separated into
two parts, one of which was the derivative, while the other part + 0 as Ax + 0.
We can write this separation algebraically using differentials:

dY
=-+- AY - dY
AX AX

But, by Definition (3.3.1), dy = f ( x ) dx and dx = Ax, so dy/Ax = f ( x ) . As a


result,

3.3.2
AY
Ax = f ( x )
- + AY Ax
~
- dY

[Compare (3.3.2) with any derivative computation of Chap. 2 ! ]

We can rewrite (3.3.2) as


SECTION 3.3: DIFFERENTIALS 97

Then

because lim,,,, (AylAx) = f ' ( x ) by definition. Thus we have proved in general


that Ay - dy +0 "faster than" Ax + 0:

3.3.3 Theorem If y = f ( x ) is differentiable at x, then

lim AY~
- dY =
Ax-o Ax

PROGRESS TEST 2

1. 9 The area of a circular disk of radius x is given by sum of the areas of triangles I1 and 111. ( f ) Inter-
y = f ( x ) = mx2. Suppose the radius is increased by pret geometrically the difference between A A and
Ax. Compute ( a ) Ay, ( b ) dy, and (c) AJ - dy. dA .
( d ) Interpret each of the quantities in ( a ) to (c) using
a diagram. (Hint: The area of a rectangular strip of
length 2mx and width Ax is 27ix Ax. How much does
this differ from a strip "bent" to fit around the
outside of the circle of radius x ? See Chap. 2.)
2. 9 Forx = 10inProb. 1,dx = O.Ol,calculate ( a ) Ay,
( b ) dy, and ( c ) Ay - dy, rounding off in the fifth
decimal place.
3. 9 The area of an equilateral triangle as a function
ofthe length of its sides x is A ( x ) = ( f i / 4 ) x 2 , which
is the product of half its base, x / 2 , and its height,
(fi/2)x. Increasing the sides by dx = Ax increases X AX
the area by AA, which is the area of the lower part of
~

2
the accompanying diagram (partially enclosed by
dashed lines). Algebraically, calculate ( a ) A A , 4. In the proof of (3.1.2), that D,(x") = nx+l for posi-
( b ) dA, and (c) A A - dA. Then from the diagram, tive integers n, identify the two terms on the right
compute ( d ) the area of rectangle I and (e) the side of (3.3.2).

THE CHAIN RNJLE FOR DIFFERENTIALS


Each rule for computing derivatives translates directly to a rule for computing
differentials because to compute the dzfferential of afunction with respect to some variable,
we take the derivative of the function with respect to that variable and then multiply by the
dy@rential of that variable.
The validity of this statement rests on the following:

3.3.4 Chain Rule for Differentials Let y = g ( u ) and u ( x ) be differentiable


functions of u and x , respectively. Then
dy = g'(u(x))u'(x) dx or dy = g'(u) du
The essence of this Chain Rule is that we can regard u both as an independent
variable (in computing d y ) and as a dependent variable whenever u does depend
upon another variable x.
98 CHAPTER 3: DIFFERENTIATION

Example 2 + 9 , u = 9x4 + x 3 - 7 , compute dy.


I f y = u5
Solution Now dy = 5u4 du and du = (36x3 + 3x2)dx, so
dy = 5u4 du = 5(9x4 + x 3 - 7 ) 4 ( 3 6 ~+
3 3 ~ )dx by (3.3.4). 0

This Chain Rule is also behind an alternative method for computing


implicit derivatives:

Example 3 Use differentials to find dy/dx, given that


2x2y5 + x3 - y 4 =9
defines y implicitly as a function of x.
Solution d(2x2y5 + x3 -y4) = d ( 9 ) = 0
d(2x2y5) + d(x3) - d ( y 4 ) = 0
2x2(5y4)dy + y 5 ( 4 x )dx + 3x2 dx - 4y3 dy = 0
Divide by dx:

dy + 4y5x
l o x 2y 4 - + 3x2 - 4y3 2
d =0
dx dx
Solve for dy/dx:
dy
--
-4y5x - 3x2
- 0
dx - 4y3

A common error in using implicit differentiation to find dy/dx is to differen-


tiate a term such as y4 to get 4y3 instead of 4y3 dy/dx, forgetting that y is a
function of x. This sort of error can be avoided using differentials because we
/-7 always dgerentiate with respect to whatever variable is present and then multiply by the
i diferentzal of that variable.
(a + A x , f ( a ) + df) /
NUMEIUCAL APPROXIMATIONS

a + A x , f(a + A x ) ) We saw that d f provides a good approximation to Af in the sense that not
only does (Af - d f ) + 0 as A x + 0, but (Af - d f ) / A x --+ 0 as well. Further-
more, d f is easier to compute than Af: To apply this approximation, we return
to the graphical interpretation of d f (see Fig. 3.5) for a particular x = a and
a x=a+Ax Ax = x - a.
Fig. 3.5 The slope of the tangent line T is f ( a ) , so the point-slope form of the
equation of T is

Y - f (a) = f ( a ) ( x - a)
or

3.3.5 Y =f (a) + f (4Ax


For small Ax, T approximates the graph ofJ; and since equation (3.3.5)
(linear in Ax) can be rewritten as

Y =f(a) + df
the differential d f is often said to provide a linear approximation o f f near a.
In particular, from Fig. 3.5 and (3.3.5) we obtain the useful approximation

3.3.6
SECTION 3.3: DIFFERENTIALS 99

Example 4 Approximate $% using differentials.


Solution We choosef, a, and Ax = dx so that the right-hand side of (3.3.6) is relatively
easy to compute and near $%, and the left side equals m.
Let f ( x ) = x1/5, a = 32, and Ax = dx = 1. Then f ( a Ax) = +
f(32 + 1) =f(33) = -\5J33 is the left-hand side. Now the right-hand side is

f ( a ) + f ( a ) dx = (32)ll5 + 5(32)-4/5(1)

1
=2+-
5 16
=2 +-
80
= 2.0125

Hence, by (3.3.6), -\5J332.0125.


z 0

The foregoing example embodies the heart of the procedure for approxima-
tions using differentials:
To approximate the number b, choose a differentiable functionf, a number a,
and Ax = dx such that:
( a ) f l u ) is relatively easy to compute and near b.
+
( b ) f ( a A x ) = b.
Then b z f ( a ) + f ( a ) A x , by (3.3.6).
Nowdays most calculations such as approximating fh? are done by using
electronic calculators. (Unless only simple decimals are involved, of course
calculators are not really calculators, but approximators.) Nonetheless, we
can use differentials to extend our capacity to approximate even in the presence
of calculators.

Example 5 Approximate dl 00.0000001.


Solution O n most calculators we would get 10 as our answer, that is, as our approxi-
mation. However, by chooshgf(x) = 6,
a = 100, and Ax = 0.0000001

z
~100.0000001 m+ 1
___
~ (0.0000001)

1
= 10 + -(0.0000001)
20
x 10.000000005 0
The important question of the accuracy of differential approximations is
addressed first in Chap. 3, Miscellaneous Exercises 1 to 7, and then more
completely in Chap. 16. There the differential approximation will be seen as the
simplest case of a far more general approximation procedure.

PROGRESS +EST 3

1. + +
Given y = (4x 1)5 x3, find dy. 4. Determine the differential approximation for
Giveny = 3 x 2 - 2x andx = fh,find dy in terms oft.
2.
+
3. Given 3 w d m 2z3 - 9w4 = 16, find dw/&
(a) v%? (b) (m)5
5 . Because of a temperature rise, a 10 x 10 x 10 metal
using:
box increases its edge measurement by 0.1 unit.
( a ) Implicit differentiation. Approximate the resulting increase in volume of
the box.
( b ) Differentials.
100 CHAPTER 3: DIFFERENTIATION

SECTION 3.3 EXERCISES

1. Supposef(x) = ax + b. Let x and dx = A x be given. 2. Forf(x) = 6and a typical Ax = dx, compute


( a ) Find A j ( b ) Find d j (0) df
( c ) Show that Af - d f = 0. (Here the linear approxi- ( b ) 4-
mation is perfect since the function itself is linear.) and then compute for x = 1, A x = -0.1:
( d ) Show that dy = dx wheny = x [so the two parts of (6)df
Definition (3.3.1) are consistent]. ( 4A.f
3. 9 Given a cube whose edge measurement increases from
x to x + Ax, describe, using a diagram, Ay, dy, and
lAy - dyl, where y = x 3 gives the volume of the cube.

In Exercises 4 to 7 determine dy: ( a ) for a tyiaical x and dx, and ( b ) for x = I, dx = 0.I .

4. = (3x2 + zx - 416 - 2x3 5. = x3-x+2


x + l
6. y = 5x 7. y = -
1
x3

8. 9 Suppose that the radius x of a circular oil spill is 9. ( a ) Determine df in terms of t and dt, where f ( x ) =
expanding at the rate of 3 m/min. ( a ) / ( x 5 - 1) and x ( t ) = t 2 + 3t - 7.
( a ) What is the approximate increase in area during the ( b ) Compute @when t = 5 and dt = 0.2.
interval from t = 10 s to t = 11 s?
( b ) What is the exact increase in area during this time
interval?

In Exercises I0 to 17, f t h e given equations define y implicitly as a function o f x , using dzfferentials ( a ) j n d dy/dx and ( 6 ) j n d dx/dy.
10. x 3 - X y + @ = O 11. 5x2 - 2y2 +6 =0 12. &- & o 13. x 3 + xy2 -y3 =3
14. - - - l = 1
x2 y2
15. x 5 -y5 + ~3~= 34 16. x 2 + y 2 = 25 17. xy2 +v5+x3 =1

18. Assuming that both x andy in Exercise 16 are functions of 19. Repeat Exercise 18 using the equation in Exercise 17.
t, compute dy/dt in terms of x , y, and dx/dt.

Section 3.4: Higher Derivatives


Objectives: COMPUTING HIGHER DERIVATIVES

1. Compute higher derivatives. Most differentiable functions can be differentiated again.


2. Interpret these higher derivatives If
graphically and in terms of rates of f ( x ) = 3x4 + 2x2 + x - 1
change.
thenf(x) = 12x3 + 4x + 1 is itself differentiable. We denote the derivative of
f(4by f W ,so
f(x) = 36x2 + 4
Similarly, we denote the derivative of f ( x ) by f ( x )
f ( x ) = 72x
andf(x) = 72, andf(x) = 0. These are called, respectively, the second,
third, fourth, and fifth derivatives of J: Beyond the third derivative it is
customary to denote the nth derivative by

fcn)(x)
where the n appears in parentheses to distinguish the nth derivative off from
other operations on J:
Each notation for the derivative extends similarly as indicated in Table 3.3.
SECTION 3.4: HIGHER DERIVATIVES 101

-
d2y d$
- .. d"y
-
dx dx dx

Table 3.3

(sr
You must be careful to distinguish between

-
dlZy and
dxn
The first is the nth derivative with respect to x and the second is t h e j r s t
derivative raised to the nth power.

Example 1 Compute

d4y
( a ) - fory = x3
dx4

Solution 4
( a ) - = 3x2, ~
d2y = 6x, d3y
-= 6 , d4y
- =o
dx dx2 dx dx4

(6) D, [m
2x 1
+-
(XZ -x + 3)4]
- +
- ( 3 ~ 5 ) ( 2 ) - (2x - 1 ) ( 3 )
+ 4 ( x 2 - x + 3)3(2x - 1 )
(3x + 5)Z
13
+ 512 + 4(x2 - x + 3)3(2x - 1 )
-
(3x
+
( 3 ~ 5)' * +
0 - 1 3 ( 2 ) ( 3 ~ 5)(3)
DB(f(x)) =
(3x + 5)4
+ 4[(x2 - x + 3)'(2) + ( 2 -~ 1)(3)(x2- x + 3 ) ' ( 2 ~- l)]
- 78
-
(3x + 5)3
+ 8(x2 - x + 3)3 + 12(x2 x + 3)2(2x - 1 ) 2 0 -

Example 1 shows that simplification may be helpful between successive


computations. Furthermore, ( u ) illustrates that taking the derivative of a
polynomial lowers its degree by 1 and taking the nth derivative lowers the
degree by n. In particular, if the degree of the polynomial is n, then the nth
derivative will be a constant and all subsequent derivatives will be 0.
All the functions we have differentiated in this chapter have nth derivatives
for any positive integer n (except at points where denominators are zero). Such
functions are said to be inznitely dfferentiuble.

We can also find higher derivatives of functions defined implicitly by


equations.
102 CHAPTER 3: DIFFERENTIATION

Example 2 Given x2 - 9y2 = 9, find d2y/dx2.


Solution Assuming y to be a function of x, we have

2~ - 1 8 ~4- = 0
dx
and thus

Then, by the Quotient Rule,

9(9y2 - x2)
-
(9YI3
Since x2 - 9y2 = 9, we have

The trick in this sort of implicit differentiation is to simplify the resulting


expressions using the original equation. Exercise 30 asks for a computation of
d2y/dx2 using differentials.

PROGRESS TEST 1

In Probs. 1 to 3 find the first four derivatives of the given 2. f(x) = x2 - 5x


function. 3. y = l / x
1. f(x) = x 5 + + + + +
x4 x3 x2 x 1 4. Find d2y/dx2 given that x2 + 2.9 + 3y2 = 1.

INTERPRETATIONS OF HIGHER DERIVATIWES


Since the derivative of a functionf(x) with respect to x describes the change in
the function with i-espect to a change in x, the derivative off(.) with respect
to x must describe the change inf(x) with respect to x. Similarly,f describes
the change i n f with respect to x.
If s(t) is a position function (a function of time t ) , then s(t) is the rate of
change of position with respect to time, more commonly called velocity. But
then s(t) is the rate of change of velocity with respect to time, more commonly
called acceleration. It is even possible to interpret 5 ( t ) , the rate of change of
acceleration with respect to time. This has been called jerk. (Sitting in a
smoothly accelerating automobile we would experience jerk if we were hit from
behind by a faster moving vehicle.)

Example 3 Suppose that s(t) = 50 + 64t - 16t2 is a position function. [For example, we
can think of s ( t ) as giving the vertical distance of an object thrown upward at
time t = 0, where the distance is in feet and time is in seconds. Thus the object
is 50 ft above the ground at t = 0 (on a platform, perhaps).] Analyze the height,
vertical velocity, and acceleration of this object.
SECTION 3.4: HIGHER DERIVATIVES 103

Solution Its velocity is given by


~ ( t=
) 64 - 32t
Thus, for t < >
2 , s(t) 0 (the object is moving upward). For t = 2, s(t) = 0
(object has stopped-at its highest point). For t > <
2 , s(t) 0 (object is moving
downward; its distance above the ground is decreasing).
Moreover,
~ ( t=
) -32
gives the acceleration of the object. This acceleration is, of course, the accelera-
tion due to gravity, which when we were introducing the derivative in Chap. 2,
was associated with a freely falling object. Notice that s(t), being negative,
means that the acceleration is in a downward direction. 0

Since the first derivative of a function can be interpreted as giving the slopes
of the tangent lines to its graph, the second derivative can be interpreted as
giving the change in the slopes of these lines with respect to x. This tells us
something about the shape of the graph. We shall examine this idea and its
important consequences in the next chapter.

PROGRESS TEST 2

Suppose that the distance between the ground and a 2. What is the velocity when the acceleration is zero?
particle moving vertically is given by the function 3. When is the velocity zero?
+
f ( t ) = t 3 - 6t2 32. 4. What is the height above the ground when the
1. Where is the acceleration positive and where is it velocity is zero?
negative?

SECTION 3.4 EXERCISES

In Exercises I to I 2 compute thejirst three derivatives of the given function.


1. f ( x ) = x 3 + x2 2. y = x8 1
3. g ( x ) = - 4. h(x) = x - ~
X2

5. f ( t ) =fi 6. f ( x ) = T 7. f ( x ) = 23 8. g ( x ) = (3x + 1)
9. h(r) = ( r + 4)5 10. y = -
X2
11. m ( y ) =-
Y 12. A = ( x + 11)
x+l 3y 1 +
13. What is the kth derivative of y = x for n k? ( n a > 14. What is the kth derivative of y = xn for n < k? (n a
positive integer) positive integer)
15. What is the third derivative o f y = l / x n ? ( n a positive 16. What is the third derivative ofy = xpq? ( p and q posi-
integer) tive integers and p / q not an integer)

In Exercises I7 to 20 describe the acceleration o f a particle moving in a straight line whose directed distance from a given point as a function o f time
t is given by:
17. s ( t ) = 2t4 - 4t2 - 5 18. g ( t ) = t3 - 3t2 1 + 19. f ( t ) = 4t3 - 3t2 20. d ( t ) = 16t
21. Explain the concavity of the graph ofy = ax2 bx c + + 22. What is the millionth derivative o f f ( x ) = x999,999?
[as defined in (1.3.4)] in terms of the sign of the second 23. If the velocity of a particle moving in a straight line is
derivative ofy. given by v ( t ) = 36 - 32t, what is the acceleration?

In Exercises 24 to 29 determine d 5 / d x 2 , where y is given implicitly as a function of x by:


24. X + y - y 3 = 4 25. y + x3Y3 + x3 = 1 30. Using differentials at each stage, recompute d2y/dx2 for x
26. 2y3 - xy + 5y = o 27. ( x + J ) = ~ 4x and y in Example 2. Let dy/dx = y before working on
29. x2 - xy +y2 = 1 the second derivative. Note that dy/dx = d 5 / d x 2 .
28. y 2 = x
3 1. Repeat Exercise 30 for x2 - y 3 = 1.
104 CHAPTER 3: DIFFERENTIATION

CHAPTER EXERCISES

Ekeview Exercises
In Exercises I to 35 dzfercntiate the given function with respect to thc indicated variable (a, b, c are constants):
1. f ( x ) = vT=7 2. qX)= vx5- 3x4 - 2x3 + x i - 5x + 7
3. g ( t ) = ( t - 5 ) 1 1 f m Z 4. f ( t ) = t10( 1 - 3t)5
1 6. g ( x ) = 2
5. f ( x ) = ~

6G x-1

8. h ( x ) =
(x"l)"
~

9. = (2. + 1)50(x2 - 10. f ( x ) =


L:
~

1 U X 2-
x + l
-
1)
11. f ( x ) = (ax' + bx + G ) ~ 12. m ( x ) = do+ (4.31)3
13. f ( x ) = \ / 1 + ( 3 x - 5)6 14. f ( x ) = (ax + b)l0
15. qx)= [(.3 + 4x - 1 1 ( ~ 5- x 2 + x + 2114
s + l *- s 2 + 1
16. r(s) = -
s-1 s'+2
17. p ( s ) = (s4 + s3 + s ' ) ~ ' ~
1 9 . t ( x ) = (3.14)"

2 I. g ( r ) = ( 1 12 - 4 4 2 ' 3

24. w ( t ) = \/(t4 + 1)5 + 6


26. f ( x ) = d x w
I l l
28. h ( t ) = - + - + - (s - 4)9
t t2 t3
30- f ( g ) = g4 + g3 + g - 6 31. f ' ( x ) = 3x2

32. h ( x ) = x . 2

34. f ( x ) = vl+x
In Exercises 36 to 4 5 determine ( a ) &/dx and ( b ) the point-slope equation of the tangent line at the indicated point (where given) using implicit
differentiation when y is given implicitly us u funclion o f x by:

36. xy + x?' =1
+() 37. 3x2 + 4xy -y2 = 0 (1,2(1 + fi))
38. fi+G =2 (1,l)

39. y 5 + 3 x 3 +
~ ~7 x 2 = 4 (use differentials),

40. fi=1 (1, 1) 41. x?'+ ~3~= 1


42. x3 + y 3 = 8 (-1, s) 43. 5 - 4y = x + 4y
Y
(4, *)
1 1
44. -+-=1
X Y ($ 4,

A s Exercises 4 6 to 55 determine dx/& using dgerentials, where x is In Exercises 66 to 75 use differentials to approximate the given quantity.
given implicitly as afunction ofy by the respective equations in Exercises
36 to 45.
66. -\4J815 67.
As Exercises 56 to 65 determine d 3 / d x 2 in terms o f x andy, wherey 68. (1.01)4'5
is given implicitly as a function of x by the respective equations in +
69- (0.99)3 (0.99)' + 0.99 - 2
Exercises 36 to 45. 70. 71. $-'
SECTION 3.4: HIGHER DERIVATIVES 105

7 2 . T h e empty weight of a spherical water tank 30 ft in upper bound on the approximation of Af by @, should be
diameter, 0.02 ft (approximately 1/4 in.) thick, made of reasonable. (This formula will be revisited in Chap. 16.) Sup-
a metal weighing 500 Ib/ft3. pose f ( x ) exists and is continuous on I = [a, a +
Ax] (or
73. T h e change in f ( x ) = x 3 + +
5x2 x - 4 when x in- +
[ a Ax, a ] for Ax <
0), with I f(x)I bounded above by M on I.
creases from 2 to 2.01. (That is, If(x)I 5 M for all x in I.) Then
74. T h e change iny wherey = 1/x2 + &when x decreases
from 1 to 0.99.
75. T h e amount of paint in gallons needed to paint the side
of a cylindrical water tank 5 0 f t tall with a radius of 1. Use this formula to determine an upper bound for the
25 ft. T h e paint will be about 0.04 in. thick, and 1 gal approximation of found in Sec. 3.3, Example 4,
equals 231 in.3 by:
76. What is the acceleration of a particle whose directed ( a ) Computingf(x) forf(x) = ~ l ~ .
distance from a givcn point (as a function of time t ) is ( b ) Determining an upper bound M forf(x) on [32, 331.
+
s ( t ) = 3t 5? (The upper bound will occur at one of the endpoints.)
77. Suppose a particles directed distance from a fixed point (c) Applying the above formula.
P is given as a function of time t by the function 2. Repeat Exercise 1 for the approximation of Review
s ( t ) = 3t2 - t 3 . Exercise 66.
( a ) When is its velocity increasing and when is it de- 3. Repeat Exercise 1 for the approximation of Review
creasing? Exercise 67.
(6) When is the particle traveling away from P and 4. Repeat Exercise 1 for the approximation of Review
when toward P? Exercise 68.
(c) What is the maximum positive directed distance 5. Repeat Exercise 1 for the approximation of Review
from P the particle reaches? Exercise 69.
78. Repeat Exercise 77 for s ( t ) = 5t - lot2. 6. Repeat Exercise 1 for the approximation of Review
79. Determinef a n d f forf(x) = d m . Exercise 70.
80. Use the definition of derivative to prove the difference 7. Repeat Exercise 1 for the approximation of Review
rule for derivatives; i.e., if f ( x ) = u(x) - u ( x ) , then Exercise 71.
f ( X ) = u(x) - u(x). 8. Use the Chain rule to show the graph off(.) = g(kx), for
81. Where does the graph o f y = l/x have tangents whose k > 1, is k times as steep at each point as is that of g(x).
slopes are increasing as we move from left to right, and 9. Suppose you attempt to estimate the height of a bridge
where are they decreasing? by measuring the time it takes for a stone to fall to the
82. Repeat Exercise 81 fory = 1/x2. water below. If you use the distance-fallen formula
88. An object is hurled upward, and its distance above the s ( t ) = 16t2 (in feet), and the stone takes 5 s to fali,
ground in feet as a function of time t measured in approximate your potential error if your time measure-
seconds is given by h ( t ) = 144t - 16t2. ment may be off by as much as 0.2 s.
( a ) Describe its velocity as a function of time. 10. Repeat Exercise 9 with the stone falling only 3 s
(6) When does it reach its highest point and what is its 1 1. Given g a differentiable function of u and u a differentia-
maximum height? ble function of x withf(x) = g(u(x)), determine a for-
84. Suppose that an attorney general announces that the mula for d2fldx2. (You will need to use the Product rule.)
increase in crime is decreasing. What does this mean in 12. Given g a differentiable function of u, u a differentiable
terms of the graph of the crime function? Does this mean function of L, and u a differentiable function of x, develop
that crime is decreasing? (The variable here is time.) a formula for d f / d x wheref(x) = g[u(u(x))]
85. Given a rocket whose distance above the earth as a func-
tion of time t is s ( t ) = -(1/3)t3 + 96t2 + lOOt - 2, 13. 9 Given f ( x ) = u(x)u(x),
with u and u differentiable.
when does the rocket reach maximum ueloczty? (Its accel- Write the formula for the differential @of the product
eration will be zero at that time.) and interpret it geometrically. (Recall Fig. 3.2.)
~

86. 14. 9 (Recall Chap. 2, Miscellaneous Exercise 3.) T h e vol-


ume V of a right circular cylinder of radius 7 and height
h is given by V = rrr2h.
87. ( u ) Compute and sketch dV, where r is regarded as a
constant and h is the variable.
88. Write out all the derivative rules in D, notation
(6) Compute dV, where is regarded as a constant and r
is the variable. What is the difference between dV and
Miscellaneous Exercises AV? If AV were slit parallel to the axis of the cylinder
and flattened out (by distorting its volume), what would
(Dzfferential Approximation Error) We remarked that for a be the volume of the nearly rectangular plate resulting?
differentiable functiony = f ( x ) , A f - @represents the amount
of bend in the graph. We now know that the shape of the 15. ( u ) Assuming that the curvature of a given circle
curve is reflected in the values off(x), so the appearance of should be a constant, show that the second derivative
the second derivative in the following formula, which gives an (which measures how the slopes of tangents are chang-
106 CHAPTER 3: DIFFERENTIATION

ing) is not a good measure of curvature for the upper (Use the result of Exercise 19.)
semicircley = d-. ( b ) Conjecture the nth derivative off(x) = u(x)v(x).
( b ) What geometric figure does have constant second
derivative curvature? 2 1. 9 ( a ) Generalize Exercise 17 by computing the differen-
1 6 . (Recall Chap. 2, Miscellaneous Exercise 23.) Show that tial o f f = uvw, where u, v, and w are all differentiable
the tangent to the upper semicircle y = d m is functions of x. (Introduce Au, Av, Aw, as in the proof of
perpendicular to the radius for each x in [ -r, r]. (Note: the Product Rule.)
The endpoints need special treatment.) ( b ) Interpret Af - df geometrically.

22. Compute D,[ f ( x + g(x))].


~

17. 9 (Recall Chap. 2, Miscellaneous Exercise 27.) Inter-


preting V = .x3 as the volume of a cube of edge x, com-
23. Compute D,[f((~(x))~)].
pute dV and interpret A V - d V geometrically. In how 24. (Alternative Proof of Power Rule for Rational Expo-
many dimensions does A V - dV shrink as Ax -+ 0, and nents) Iff(.) = xP/Q, where p and q are integers, then
in how many dimensions do the remaining parts of A V f(x) = (p/q)x(P/q)--l.We can assume p = 1 because the
general case follows using the Power rule for integers
(these are the constituents of dV) shrink?
(since X P / Q = ( x l / q ) ~ ) .
18. Given that the acceleration of a moving particle is a ( a ) State the derivative quotient, writing Ax as
linear function oft, a(t) = ct +
d, what sort of function (x+ Ax) - x.
describes the directed distance traveled by this particle? ( 6 ) Rewrite the identity (aQ - bQ)/(a - b ) = aQ-l +
19. Given thatf(x) = u(x)v(x), show that + +
aQ-2b aQp3b2 . . . + + abQ-2 bQ-l, which holds for
dZf d2u du dv d2v any integer q, in order to rewrite the derivative quotient
- = -v(x)
dxz dx2
+ 2--dx dx + u(x)-dxz +
Af/Ax in the form ( a - b)/(aQ - b Q )for a = (x Ax)l/Q,
b = xl/Q.
20. ( a ) Given thatf(x) = u(x)v(x), show that (c) Take the limit of the resulting quotient to obtain the
derivative.
d3f d3u
= -v(x)
dx
+ 3--d2u dv
dx2 dx
du d2v
+ 3 -dx
-
dx2
+ u(x)- d3v
dx3

SELF-TEST +
Chapter 4 X

107
108 CHAPTER 4: APPLICATIONS OF DERIVATIVES TO GRAPHING

Describing the Behavior of


Section 4.1: Functions Using Special Limits

Objectives: LEFT AND RIGHT LIMITS AND VERTICAL ASYMPTOTES

Determine vertical, horizontal, and Recall from Sec. 2.2 that the statement
oblique asymptotes of the quotient
limf(x) = L
of two polynomials and the x+a
quotient of roots of polynomials.
means that the difference betweenf(x) and L can be kept as small as we please
merely by keeping x close enough to a. Although this statement involves x
approaching a from both sides, the analogous left-hand limit statement
lim f ( x ) = L
x-a-

means the same thing except that x approaches a only from the left; that is,
<
x a. Similarly, we write the right-hand statement
lim f ( x ) = L
x+a+

if x approaches a only from the right; that is, x > a.


Our interest here is in describing the behavior offnear points a where such
limits do not seem to exist. In most instances we examine situations where the
values off grow or decrease without bound as x approaches a from one or both
sides, as summarized in Fig. 4.1 with graphs accompanied by equivalent
algebraic notation. (Recall Chap. 2, pp. 42-44.)
In all the situations described in Fig. 4.1 we can forcef (x)to stay larger than
+
any given number A4 (if 00 is involved) or less than any given number N (if
-00 is involved) simply by requiring that x be close enough to a. More
precisely, we present the following definition.

x = a

(a)

Fig. 4.1
SECTION 4.1: DESCRIBING THE BEHAVIOR OF FUNCTIONS USING SPECIAL LIMITS 109

4.1.1 Definition (Infinite Limits) Assume,that f ( x ) is defined at all points,


except perhaps a, of a n open interval containing a. We write
limf(x) = +co
x-a

if for every A4 > 0, there is a number 6 > 0 such that


0 < Ix - a / < 6 impliesf(x) > M
Similarly, we write
limf(x) = -co
a-a

if for every N < 0, there is a number 6 > 0 such that


0 < Ix - a1 < 6 impliesf(x) < N
Of course, this definition can be modified to include the one-sided cases
illustrated in Fig. 4.l(c) and ( d ) . If any of these situations prevails for a function
f a t a point a, then we say that f has a vertical asymptote at x = a.

In practice, as we soon illustrate, showing thatfhas a vertical asymptote at


x = a amounts to writingf(x) as a quotient u ( x ) / v ( x ) and then showing that as
x tends to a, u(x) tends to some nonzero number k and v(x) tends to zero through
either positive or negative values. For k >
0, if v(x) tends to zero through
positive values, then the quotient is positive and grows without bound. If v(x)
tends to zero through negative values, then the quotient is negative and
decreases without bound. The reverse conclusions hold, of course, if k 0. <
Notice that the Limit Theorem for quotients [2.5.4(c)] applies only when
limx-a v(x) # 0, so it does not apply to any of these situations.

Example 1 Show that the following functions have vertical asymptotes at the indicated
point a. Construct a table of data forf(x) and sketch the graph of both
functions near a.

Solutions (a) We compute the data forfnear a = 3 ; see Table 4.1. Here limx,3 2x = 6,
and as x + 3-, ( x - 3) approaches 0 through positive values. Hence
+ +
lim,,3-f(x) = co.Similarly, limx+3+J(x) = 00. Thusf(x) has a vertical
asymptote at a = 3, with lim,,J(x) = +co; see Fig. 4.2(a).
(6) For g(x) = x2 + we note that
2+x-x

(x - 3)2

X
0.0001 60,200
3.001 0.000001 6,002,000

0.01 580

(a)
59,800
5,998,000 ,
Fig. 4.2 Table 4.1
110 CHAPTER 4: APPLICATIONS 0: DERIVATIVES TO GRAPHING

while 2 + x - x2 = (2 - x)(l +
x ) . For x near 2 (on either side), 1 + x is
near 3 > 0. However, as x tends to 2- (through values less than 2), ( 2 - x )
tends toward zero through positive valves, hence (2 - x)(l + x ) tends
toward zero thraugb positive values, so

grows without bouqd. That is,


lim g(x) =
x-2-
+ 03

On the other hand, as x tends to 2' (through values greater than 2), (2 - x)
tends to zero through negative values; hence (2 - x)(l +x) does likewise.
As a result,

decreases without bound. That is,


lim g ( x ) = - c
a
x-2f

Hence g ( x ) has a vertical asymptote at x = 2, and we obtain the graph


shown in Fig. 4.2(6) to the left. 0
As we know, when examining the behavior of a functionf near, but not at,
a point a, it is possible to perform an algebraic manipulation that replacesf by
a function that equals f everywhere except a t a. We shall use this idea in the
second half of Example 2. Note that the natural place to look for vertical
asymptotes is where the expression describing the function has a zero in the
denominator.

Example 2 Identify any vertical asymptotes far the functions:

Solutipns

The obvious candidate for a vertical asymptote is x = 0. Now, as x + 0


4 through both positive and negative values, x 2 + 0 through positive values.
On the other hand,
lim (x - I) = -1
x-0

Hence liv,+of(x) = - 03, and f ( x ) has a vertical asymptote at x = 0.


(b) Since g(x) is undefined for x 2 4, we consider what happens to g ( x ) as x
tends to 4 only from the left side. Both the numerator and denominator of
4-
= x-4
tend to 0 as x + 4-, leading to the expression O / O , which has no meaning.
However, rationaliFing the numerator and canceling common factors yields
SECTION 4.1: DESCRIBING THE BEHAVIOR OF FUNCTIONS USING SPECIAL LIMITS 111

g(x) =
JS
~~
Now limz+4-(- 1)(4 + x) = -8. As x + 4-, d m + 0 through positive
x-4
values, so
lim g ( x ) = - 00 establishing a vertical asymptote at x = 4
x-4-

Again note that g ( x ) is not defined for x 2 4. 0


x = 4

PROGHESS TEST 1

In Probs. 1 to 3 evaluate the given limit.


4. f ( x ) =
x6 + x5 + 2
x2-x-6

5 . g ( x ) = --
2 ~
7
3x - 7 x-1 (x-1)2
2. lim-
2x3 6. In Chap. 1 we noted that limz+af ( x ) = + 00 if the
x2 -1
values of f ( x ) grow beyond the capacity of your
3. lim- calculator, regardless of its capacity. How near must x
r-1+ (x - 1)2
>
be to 4 (x 4) forf(x) = x / ( x - 4) to exceed the
In Probs. 4 and 5, determine any vertical asymptotes capacity of your calculator?
for the given function and sketch the function near
any vertical asymptotes.

LIMITS AT INFINITY A N D HORIZONTAL ASl2MPTOTES


Thus far we have examined the behavior of functionsf(x) only for relatively
small x. We now turn to the question of what happens when x increases or
decreases without bound. Consistent with our earlier uses of the symbols co +
and - co,we write
x+ + co to mean x increases without bound
and
x + - 00 to mean x decreases without bound
In these terms we may rephrase our current interest as an investigation of
1imz++-f(x) and limx+--f(x). For example, given the function

f ( x ) = -7j
X
1
+5
it is fairly clear that as x + + co, 1/x2 + 0, s o f ( x ) + 5. That is,

We are in effect saying that we can make l/xz +


5 stay arbitrarily close to 5
by choosing x large enough.
The following general definition expresses this more precisely.

4.1.2 Definition (Limits at Infinity)


( a ) We say that f ( x ) tends to L as x increases without bound and write
112 CHAPTER 4: APPLICATIONS OF DERIVATIVES TO GRAPHING

limf(x) = L
X-+m

if, given any E > 0, there exists a positive number M such that
x > M implies If(.) - LI < E

( b ) We say that f(x) tends to L as x decreases without bound and write


lim f(x) = L
X++m

if, given any E > 0, there exists a negative number N such that
x < N implies I f ( x ) - L J < E

If either ( a ) or (b), or both, applies, we say thaty = L is a horizontal asymptote of


the function f(x).
Part ( a ) says that we can keep the difference If(.) - LI arbitrarily small
(close to zero) by keeping x large enough; ( b ) states that we can keep ).(fI - L/
arbitrarily small by keeping -x large enough. Geometrically, (4.1.2) means
that the horizontal liney = L is a horizontal asymptote off(x) if we can keep
f(x) close to the liney = L by keeping x far enough to the right or far enough to
the left. Far enough to the left means that -x will be large enough in the
sense above. This is the sense in which we shall later use the phrase large
negative x. The limits of (4.1.2) obey a limit theorem exactly analogous to the
Basic Limit Theorem (2.5.4), with a replaced throughout by 200. In each
following case, determining a horizontal asymptote will eventually reduce to
applying the following obvious but useful fact. Its proof is left as Exercise 27.

4.1.3 Theorem If p is a positive integer and k is a constant, then


k k
lim - = 0 = lim -
x++m xp x+-m xp

The key to applying (4.1.3) to the quotient of polynomialsf(x) = p(x)/q(x) is


the following: Before taking limx+km,divide the numerator and denominator t y x, where
n Zs the larger of the degrees ofp(x) or q ( x ) [or equal to the degrees ofp(x) and q ( x ) ifthey
are equal].
Since in effect we are replacingfby a function equal toffor all x # 0, this
does not affect the limit, which depends only upon the behavior of the function
for x far away from 0.

Example 3 Show thaty = 3 is a horizontal asymptote of

Solution First we divide the numerator and denominator by x2 since 2 is the degree of
+
both 3x2 4 and x2 1. +
Now

lim f(x) = lim


3X2/X2 + 4/x2 for x f 0
x-kffi x-km x2/x2 + 1/x2
SECTION 4.1: DESCRIBING THE BEHAVIOR OF FUNCTIONS USING SPECIAL LIMITS 113

Notice that the function of this example can be rewritten as

from which it can also be seen that limz+2J(xj = 3. Table 4.2 gives some
values off(.) (rounded off in the last place) for large positive and negative x .
The graph o f f ( x ) is sketched in Fig. 4.3.
This technique for determining horizontal asymptotes extends to rational
functions in the general case, which we now investigate.

4.1.4 Suppose we have


a,xn + a,-pn--l + . . . + alx + a.
f(x) = 6,xm + brnplxm--l + . . . + 6,x + b,
where rn and n are positive integers and a,, 6, # 0. If n rn (so m - n is a <
positive integer), we divide the numerator and denominator by x m , to get
Y an
-+a,_,+ . . . +-a0
xm-n xmpn+l Xrn
lim f ( x ) = lim
I-tirn I+f_a;
b,+-+ 'm-1
. . . + -6,
X Xrn

- o+o+...+o =o
bm +0 + ... +0
If n = rn, we then divide numerator and denominator by xn, as in Example 3,
to get

an
lim f ( x ) = -
X+?CC
b?l
Finally, if n >
rn (so n - m is a positive integer), we divide the numerator and
denominator by x n to get

Now, as x grows or decreases without bound, the numerator approaches a,


while the denominator approaches zero. Hence the quotient,f(x), either grows
or decreases without bound. This proves the following theorem for f ( x ) , as
in (4.1.4):

4.1.5 Theorem
Oifn<m y = 0 is a horizontal asymptote
y = an/b, is a horizontal asymptote
X - i m
&co i f n > r n no horizontal asymptote

f(x) = 1 / ( 2 + 1) + 3
3.00990099 ... 3.00990099 ...
3.0001 - 100 3.0001
1000 3.000001 - 1000 3.000001
10,000 3.00000001 - 10,000 3.00000001

(a)
Table 4.2
114 CHAPTER 4: APPLICATIONS OF DERIVATIVES TO GRAPHING

You will be asked to apply (4.1.5) in Progress Test 2. The same division-
by-powers-of-x technique works for quotients of roots of polynomials. Rather
than work out another general theorem, however, we shall handle such func-
tions on a case-by-case basis.
An expression consisting of the rth root of a polynomial of degree n will be
said to have relative degree n/r.

Example 4 Determine any horizontal asymptotes for


d3x3 + 2x + 1

Solution The relative degree of the numerator is 3 / 2 , as is the relative degree of the
denominator. Hence we divide both the numerator and denominator by x 3 l 2 .
Note that neither g ( x ) nor x3I2 is defined for large negative x ; hence we shall
examine the behavior of g(x) for large positive x only. Now
1
d 3 x 3 + 2x + 1 =
--p + 2x + 1

and so the limit of the numerator is

Also, writing

we have

Thus the limit of the denominator is

so g(x) has a horizontal asymptotey = -


fi
VT- .
The function g ( x ) had a horizontal asymptote only in the positive direction
and was undefined for large negative x . It is even possible for a function to have
two different horizontal asymptotes.
SECTJON 4 . 1 : QESCRIBING THE BEHAVIOR OF FUNCTIONS USING SPECIAL LIMITS 115

Example 5 Determine the horizontal asymptotes of

h(x) =
3x +2
+PT
Solution Since 3x +2 has degree 1 and d m has relative degree 1, we shall divide
numerator and denominator by x. We shall consider positive x first.

On the other hqnd, for negative x,

Y
, . 1 ,..

Therefore,

lim h(x) = lim


(1/x)(3x + 2)
++--m
x--m (l/x) d G x
= lirn
3 + 2/x
x--m - d7p&P=T
3 + 2/x - 3+0 - 3
= lim
x+-m - d v - - v G - 5- - -2
Thus for positive x, h(x) has the horizontal asymptotey = 3/2; for negative x,
the horizontal asymptot? isy = -3/2. 0

Fig. 4.4 Note that h(x) also happens to have vertical asymptotes at x = ? 1/2, and is
not defined in [-1/2, 1/21, Its graph is sketched in Fig. 4.4; its function is
h(x) = [(3x + 2)/(dm-=q].

PROGRESS TEST 2

For the functions in Probs. 1 to 3 use (4.1.5) to determine


any horizontal asymptotes. In Probs. 4 to 6 determine any horizontal asymptotes.

1. f(x) = -
4 - 2x +
7x2 + x3 4. f(x) =
2x +1
fix3+x-7 m
2. g(x) = 4x3 +
x2 - 9
5. f ( x ) =
x2+x+l
8x2 - x + 1 + +1
3
d3x9 4x2
4x5 +
x3 - 9x 2x2 + x + 1
3. h(x) = 6. f ( x ) =
1 + 2x2'+ 3x7 3x + 2

OBLIQUE ASYMPTOTES
If f(x) is a quotient of polyqomials
116 CHAPTER 4: APPLICATIONS OF DERIVATIVES TO GRAPHING

where the degree of p ( x ) exceeds that of q ( x ) by 1, thenf(x) has no horizontal


asymptotes but does approach a line of nonzero slope as x -+ 500in the
following sense. By long division,

where r ( x ) , the remainder, has degree less than that of q(x). Now, as x -+ + 00,
or x -+ - 00, r ( x ) / q ( x ) -+ 0 by (4.1.5), so the difference betweenf(x) and the
straight line with equationy = cx d +
f ( x ) - (cx + d) = -
r(x)

approaches zero. If either situation prevails for a function f(x), we say f ( x )


has the oblique aymptotey = cx +
d. It turns out that c , the slope of the asymptote,
is the quotient of the coefficients of the highest degree terms in the numerator
and denominator.

Example 6 Determine the oblique asymptote of


3

f(4=
15x3 + 5x2 + 5x + 1
5x2 + 1
By long division, we have
f ( x ) = 3x + 1+ ~ 2x
5x2+ 1
Thus, for large positive and negative x, the graph off(x) is near the line
y = 3x + 1. Hence we can describe the graph off(x) away from the origin as
follows. We know that for x >
0 the graph off(x) is above the oblique asymp-
>
tote [because 2x/(5x2 + 1) 01 and, similarly, for x <
0 the graph is below the
asymptote. Methods for determining the behavior off(x) on the remainder of its
domain await the systematic use of derivatives to be developed in Sec. 4.2. 0

SECTION 4.1 EXERCISES

For each of the functions in Exercises I to 26 determine any vertical, horizontal, or oblique asymptotes.
X2 X2
1. f ( x ) = - 2. g(x) = -
x - 2 x2 - 4
( x - 1)2 2x2 - 3x - 1
3 . y=- 4. y=
2x - 2 xZ+x-l

5. f ( x ) =
3x2 + 2x - 9 6. u ( x ) =
2x - 1
4x2 +7 v57-T-i
7 . v(x) =
4 7 7 (Be careful!) 8. t ( x ) =
(x - 1)2
3x +1 ~

V F X 3x- d!iFE
9. f ( x ) = 10. g ( x ) =
x + l X

(treat as the difference of functions)

11. h ( x ) =
dF=E 12. f ( x ) =
x 3 - 3x2
x - 2 x2 + +
2x 1
SECTION 4.2: DESCRIBING FUNCTIONS USING DERIVATIVES-PART 1 117

13. g ( x ) =
x2+x+1 14. f ( x ) =
3x2 + 2x + 1
2x 9 + 6x2 + 3x - 9
1 1 16. f ( x ) = x2+x+ 1
15. J = - - - -
x2 x3 l/x + 1/(2x2)
t2 - 4 4x2
17. h ( t ) = 18. m ( ~ =
) -
t4 - t3 +
t2 - 1 9 - x2

19. f ( t ) =
4 t 3 - 2t2 +t 20. y =
11x2 +
2x - 1
2t - 1 x - 3

21. f ( x ) =
x4 - x3 +
2x - 1
22. r(x) =
x4 + 3x
x3+x2+x+I
~

x2 +1
23. s(x) =
x6 - x4 5 + 24. f ( x ) =-
1/x2
(x2- 3x - 1 ) 2 1/x3

x2 - 1 1/x2
2 5 . f ( ~=) - 28. y = -
x2 1+ 1lX4

2 7 . Prove ( 4 . 1 . 3 )

Describing the Behavior of


Section 4.2: Functions Using Derivatives-Part 1

Objectives: SOME TERMINOLOGY

1. Understand the geometric To use derivatives efficiently to graph functions, we first establish a working
meaning of the following words as vocabulary. The essence of each definition can be seen in the graphs that
they apply to the graphs of accompany it.
functions: increasing, decreasing,
relative maximum, relative 4.2.1 Definition We say that the function f is increasing on t h e interval I if
minimum, concave up, concave
down, and point of inflection.
x1 < x2 in I implies f ( x J 5 f(x2)
We say f is decreasing on I if
2. Understand the algebraic
meaning of the above terms as x1 < x2 in I implies f ( x J 2 f(x2)
reflected in the first and second
(See Fig. 4.5.) We use the words strictly increasing or strictly decreasing when strict
derivatives.
inequalities hold in these statements.
3. Determine relative extremes
using both the First- and Second-
Derivative Tests. These are left-to-right definitions in thatfis strictly increasing if it goes up
as you move from left to right, and strictly decreasing if it goes down as you
move from left to right.

fix,)
-1 c---+---i I
I
I
I
-=a

h X
22 X1 X2
V
I I
Increasing on I Decreasing. on I
Fig. 4.5
I18 CHAPTER 4: APPLICATIONS OF DERIVATIVES TO GRAPHING

4.3.2 Definition We say that the function f has a relatzve mznzmum at xo iff(xo) is the
minimum value off on some open interval (a, 6) containing xn. That is,

f(xn) <f(x) for all x in (a, 6)


Similarly, we say that f has a relative maxzmum at xo if f(xo) is the maximum
value off on some open interval (a, 6) containing xo. That is,

f(xo) >f(x) for all x ill (a, b )


Relative minima and relative maxima are often referred to as relatzve extremes of
the function.
In Fig. 4.6 there are relative mznima at xo and x2, and a relative maxzmum at xl.
Notice that f(xn) > f ( x , ) even though xo is a relative minimum of J: This
explains the use of the word relative. There does exist an open interval (a, b )
containing xo (and which does not containx,) such thatf(xo) is the smallest value
off on thzs interval. The graph of the function is lower at xo than at any other x
near xo.
Our definition of relative maximum and relative minimum at xo allows the
possibility that f is constant on an open interval containing xo [ s o f ( x ) =f(xo)
there], and our definition of increasing and decreasing functions likewise allows
this possibility. However, when graphing functions this is of little importance
because, where the function is constant, the special terms are irrelevant: To state
the function is constant on some interval and to give the value of that constant
completely describes the behavior of the function on that interval.
The next theorem is the first among several to relate extremes of a function
to its derivatives.

4.3.3 Theorem If xo is a relative extreme ofJandf(xo) exists, then

f(xo) = 0
That is, the tangent line at xo is horizontal.
In Fig. 4.7, f ( x o ) = 0 =f (XI) are horizontal tangents, and the relative
extremes are at xo and xl.
Caution: The converse of (4.2.3) is not necessarily true! The function
f ( x ) = x3 + 1 shows a case in whichf(xo) = 0 without f having a relative
extreme at xo; see Fig. 4.8.
hofof(4.2.3): We use the fact that the limit definition off(xo) is two-sided.
We consider the case off (x,,) a relative maximum. (The relative minimum case
is similar; see Exercise 17). The key is that for small ]Ax/,

I I * 2
20 Xl

Fig. 4.6 Pig. 4.7


SECTION 4.2: DESCRIBING FUNCTIONS USING DERIVATIVES-PART 1 119

becausef(xo) is a relative maximum. We know that the left and right limits
defining f ( x o ) are equal:

4.2.4

Y However, since Ax < 0 on the left side, the bracketed left-hand quotient is
positive, and since Ax> 0 on the right side, the bracketed right-hand quotient
is negative. Thus, as Ax -+ 0, the left-hand quotient approachesf(xo) through
positive values, and the right-hand quotient approaches the same number f (xo)
through negative values. The only way this can happen is iff(x,) = 0! W

Iff(x,) does not exist,fmay still have a relative extreme in any of several
X
ways:
(a) The left- and right-hand limits appearing in (4.2.4) may exist but fail to be
equal, as indicated by the dashed one-sided tangents in Fig. 4.9. Under
Fig. 4.8 such circumstances we have a corner.
(6) The left and right limits may grow or decrease without bound, yielding
vertical tangents and cusps7as indicated in Fig. 4.10.
Combinations of the phenomena in (a) and ( b ) may also occur.
We shall now examine the shape of a graph more closely. Suppose we know
that the graph of a functionfstarts at (1,2), increases to ( 3 , 5 ) , has a relative
maximum at x = 3, then decreases to (4,O). Do we have enough information
for a fairly accurate sketch? The continuous functions sketched in Fig. 4.9(a)
and (6) and Fig. 4.10(a), all of which satisfy the above, show that the answer to
this question is no! (We assume you also thought of the case withfdifferentiable
at x = 3.) We need to be more specific about the way in which a curve increases
or decreases.

t
I
I
I

X
120 CHAPTER 4: APPLICATIONS OF DERIVATIVES TO GRAPHING

-
Fig. 4.1 I
I
Concave U p
I
I
Concave Down

4.2.5 Definition We say that the function f is concave up on an interval I if the


graph o f j is above its tangent line at each point of I. Similarly, it is concave down
on I if its graph lies below its tangent line a t each point of I; see Fig. 4.1 1.
If we return to the previous example and include additional information on
the concavity off, we can in fact get a fairly accurate sketch. For example,
supposef starts a t (1, 2), increases in a concave down fashion to (3,5), and then
decreases in a concave up fashion to (4,O). Then a correct sketch appears in
Fig. 4.9(a).
Suppose we are told that a function which describes the unemployment rate
is increasing over an interval of time. Certainly we do not know enough to make

/\
any kind of economic judgment based upon this information until we know at
1 YO. f,XO))
(XO, J (XO)) least whether it is concave up (increasing at a n increasing rate) or concave down
(leveling off).
The following definition proves to be useful in describing the shape of a
Fig. 4.12 graph.

4.2.6 Definition We say the function f has a poznt of inzectzon a t ( x o , f ( x o ) ) if the


tangent line to the graph off exists at this point and iff has opposite concavity

/\
on either side of xo. (In Fig. 4.12 the curves are concave up to the leJt and concave
down to the rzght, in Fig. 4.13 they are concave down to the left and concave up to the
rzght.)
1 20, f(%)1
(20, S,ho)J You should not worry about memorizing these definitions; merely be certain
that you can picture what they mean.
Fig. 4.13 Note that we sometimes slightly abuse language in cases where confusion
does not result. In particular, we often may not distinguish between the
function and its graph.
PROGRESS TEST 1
-~ ~ -

1. Suppose f( x ) has the following graph: d

I
I
iI

( a ) In which intervals is f decreasing? Increasing? ( d ) At which relative extremes doesf have a deriva-
(6) In which intervals is f concave up? Concave tive? What is its value a t those points?
down? ( e ) List all points of inflection.
(c) List all relative maxima and minima f o r j 2. Sketch the following functions in the interval
SECTION 4.2: DESCRIBING FUNCTIONS USING DERIVATIVES-PART 1 121

[ - 2,5], appropriately connecting the given points. ( - 2,4) to (2,0), increases in a concave down fash-
(The ordered pairs here refer to points, not inter- ion to (3,2), is constant to (4,2), and then increases
vals.) in a concave up fashion to (5,4).
(a) f increases in a concave down fashion from 3. Sketch the graphs of four different functions that
( - 2,2) to (2,4), decreases in a concave down fash- satisfy the following conditions: The function de-
ion to (3,0), and then decreases in a concave up creases from the point (-2, - 1) to (0, -4), has a
fashion to (5, -1). relative minimum at (0, -4), and increases to
( b ) f decreases in a concave down fashion from ( I , - 1). It is constant on the interval (1, + 00).
~~

THE MEANING OF THE FIRST DERIVATIVE


In Sec. 4.4, we shall prove the following reasonable and fundamental fact
relating a function and its first derivative.

4.2.7 Increasing-Decreasing Theorem Suppose f is differentiable on a n


open interval I = (a, b).
(a) Iff'(x) > 0 for all x in I, then f is strictly increasing on I.
<
( b ) Iff ' ( x ) 0 for all x in I, then f is strictly decreasing on I.
(c) Iff'(.) = 0 for all x in I , then f is constant on I (that is, its graph is a
horizontal straight line).

Sincef'(x) is the slope of the tangent to the graph at the point (x,f ( x ) ) , and
sincef'(x) >
0 means this tangent tilts up to the right, the function must be
>
strictly increasing precisely when f '(x) 0. Similarly, it must be strictly de-
creasing where f ' ( x ) <
0 because there the tangent tilts down to the right. (See
Fig. 4.14.)
The next theorem enables us to use the first derivative to determine relative
extremes of a function.

4.2.8 First-Derivative Test for R e l a t i v e Extremes Supposef is differen-


tiable on an open interval containing the point x,, but not necessarily at xo, If
f is continuous at x,, and f ' ( x ) has opposite signs on either side of x,,,thenf has a relative
extreme at x,,. In particular,
( a ) Iff ' ( x )> <
0 on some interval (a, x,,) and f '(x) 0 on some interval (xo,b),
then f has a relative maximum at x,,.
< >
( 6 ) Iff '(x) 0 on some interval ( a , xo) and f ' ( x ) 0 on some interval (xo, b),
then f has a relative minimum at x,,.
Figure 4.15 illustrates some of these possibilities. Note: We can combine (4.2.3)
with (4.2.8) to conclude that iff'(x,,) does exist andf'(x) has opposite signs on
either side of x,,, then f '(x,,) = 0, as in Fig. 4.15(a) and ( 6 ) .

J'(x) > 0 Y(x) = 0 J"x) < 0


f i x ) strictly increasing
~ ~~
J i x ) constant
~~~ ~~ f i x ) strictly decreasing
Fig. 4.14
122 CHAPTER 4: APPLICATIONS OF DERIVATIVES TO GRAPHING

XO

Relative maximum withf(xo) = 0 Relative minimum withf(xo) = 0

(a) (b)

I
I
I

Relative maximum at a corner Relative minimum at a cusp


f ( x o ) does not exist f ( x o ) does not exist

(c) (d1
Fig. 4.15

Example 1 Determine any relative extremes off(x) = 3x2 - 6x + 1.


Solution Since f ( x ) is everywhere differentiable, to apply the First-Derivative Test we
determine and then factor the derivative:
f ( x ) = 6x - 6 = 6(x - 1)
<
Now f ( x ) = 6(x - 1) 0 for x <
1 and f ( x ) = 6(x - 1) 0 for x 1. > >
Hence we have a relative minimum at x = 1 with f ( 1 ) = - 2 . Of course,
f(1) = 0 (horizontal tangent). See Fig. 4.16. 0
For functions with more complicated derivatives it is helpful to determine
x the sign of the first derivative using the Chart Method developed in Sec. 1.1.
In the following example and henceforth we shall cease using the word
strictly when dealing with increasing or decreasing functions, treating the
constant case explicitly when it arises.
Fig. 4.16

Example 2 Determine where the following function is increasing and where it is decreasing.
Find all relative extremes.
xz
x4
f(x) =- --
4
+9 2
Solution We determine and then factor f ( x ) :
f(x) = x3 - x
= x ( x 2 - 1)
= x ( x - l)(x + 1).
SECTION 4.2: DESCRIBING FUNCTIONS USING DERIVATIVES-PART 1 123

We then use the Chart Method to determine the sign off '. Notice that to aid in
determining extreme points, we marked the axis below the chart using positive
>
slope lines where f ' 0 and using negative slope lines where f 0. <
Sign off':

----t---w
-1 0 1
i-1,yj
From the Increasing-Decreasing Theorem it follows that f is increasing on
+
( - 1 , O ) and (1, co) and decreasing on ( - co, - 1) and (0, I). By the First-
Derivative Test, f has relative minima at x = - 1 and x = 1, and has a relative
* X maximum at x = 0. We present the accompanying sketch at the left to show the
function's behavior near x = 0. 0

PROGRESS TEST 2

Find where each of the following functions is increasing


and decreasing. Determine all relative extremes. It is not 3. f ( x ) = x 2 - 2x 4. f ( x ) = x3 + 5x
necessary to sketch the graphs. 5. f ( x ) = x 3 / 3 - x2 - 3x +3
1. f(x) = 3 x + 1 2. f ( x ) = 3 (Use a chart)

THE MEANING OF THE SECOND IDERIVATIVE


The derivative of a function tells how that function is changing. Thus, given
f ' ( x ) , we know the derivative off ' ( x ) , namely f "(x), tells how f ' ( x ) is changing.
That is, f " ( x ) tells how the slopes of lines tangent tof(x) are changing. Contrast
the graphs of the two functions in Fig. 4.17, each of which increases from (a, c)
to ( b , d ) . Of course, curve A is above the tangent lines and hence concave up,
and curve B is concave down. As we move from left to right on A the slopes of
the tangent lines are increasing. That is, the first derivative is increasing-
which, by (4.2.7)) occurs when its derivative is positive. Thus the second derivative
of the concave-upfunction gzven by curve A is positive. Similarly, the slopes of tangents
of curveB are decreasing, so the first derivative is decreasing. But by (4.2.7), this
occurs when its derivative is negative. Thus the second derivative of the concave-down
function given by B is negative. It can be shown that the converses of these observa-
tions are true.

* x -

Fig. 4.1 7
124 CHAPTER 4: APPLICATIONS OF DERIVATIVES TO GRAPHING

4.2.9 The Concavity Theorem Supposef(x) exists on an interval I.


>
( u ) Iff(x) 0 on I , thenfis concave up on I.
(6) Iff(.) < 0 on I , then f is concave down on I.
Note: In the case thatj(x) exists and has opposite signs on either side of xo,
then the Concavity Theorem implies thatfhas a point of inflection at xo [see
(4.2.6)]. Furthermore, carrying the first-derivative note following (4.2.8) one
step further to the second derivative, we can conclude that iff(xo) does exist at
a point of inflection, thenf(x,) = 0. However, just as the first derivative being
zero does not imply a relative extreme, the second derivative being zero at xo
does not imply a point of inflection at xo. This is illustrated by
x
f ( x ) = x4 + 1 at xo = 0
Heref(x) = 4x3 andf(x) = 12x2,sof(0) = 0. Yetf(x) >
0 on both sides of
Fig. 4.18 0, and hence there is no change in concavity and thus no inflection point at 0.
The graph is concave up for all x . (See Fig. 4.18.)

4.2.10 Second-Derivative Test for Relative Extremes Supposef(x,) = 0


andf(x) exists in an open interval containing xo (see Fig. 4.19).
(u) Iff(x,) < 0, then f has a relative maximum at xo.
(6) Iff(x,) > 0, thenfhas a relative minimum at xo.
When the First- and Second-Derivative Tests both apply, we have an
excellent chance to check our work for errors.

~-
Example 3 Use the Second-Derivative Test to confirm that

has relative minima at x = i- 1, and show that it does not apply to confirm the
relative maximum at x = 0. (Recall Example 2.)
Solution Sincef(x) = x 3 - x 2 , we know that
f ( X ) = 3x2 - 2x
= x ( 3 x - 2)
Sincef(0) = 0, the test does not apply at x = 0. O n the other hand,
f(1) = (1)(3 - 2) = 1 >0
and
f(-1) (-1)(3)(-1) - 2 = 5 >0
so [4.2.10(b)] does apply to confirm the relative minima at x = t l . 0

If both the first and second derivatives are zero at xo, then the Second-Deriva-
tive Test does not apply and x,, may be either a maximum, a minimum, or

I
XO

(a)
Fig. 4.19
SECTION 4.2: DESCRIBING FUNCTIONS USING DERIVATIVES-PART 1 125

neither. Furthermore, x,, may or may not be a point of inflection; see Figures 4.8
and 4.18. Note thatf(x) = x 3 +
1 has a point of inflection at x = 0 because
f ( x ) = 6 x changes sign at x = 0.
The Chart Method again proves useful in determining concavity and points
of inflection for those functions with more complicated second derivatives.

Example 4 Describe the concavity of the following function and determine its points of
inflection:

Solution Since

we know that

f ( x ) =
(x2 + 1)2( - 2 x ) - (1 - x2)(2)(x2 + 1)(2x)
(x2 + 114
-
-
2x(x2 + 1) [ - x 2 -1 -2 + 2x21
(x2 + 114

We now construct a chart, noting that 1 / ( x 2 +


1 ) 3 is always positive and
thus need not be included in the sign analysis.
From the Concavity Theorem we conclude thatfis concave up on (- fi,0)
+
and (3, 00) and is concave down on ( - 00, - fi) and (0, fi). We note this
symbolically below the chart. Furthermore, f has points of inflection at
x = k f i a n d x = 0. 0

A bit of silliness that may help you remember the significance of the second
derivative: A function is happy to have positive second derivative, so it smiles
-. A function is sad to have negative second derivative, so it frowns -.

PROGRESS TEST 3

Determine the concavity of the given functions and find


the x coordinates of all points of inflection.
x3
1. f ( x ) = - - x2
3
+2 2. f ( x ) = x 4 - 18x2 +1

SECTION 4.2 EXERCISES

1. Sketch the speed as a function of time for a train that slow down at a n increasing rate, until t = 7, when its
moves as follows: At time t = 0 it begins moving down rate of decrease becomes more gradual until t = 8, when
the tracks at an increasingly accelerating rate until it reaches a very slow but constant rate of speed. Identify
t = 3, when, although it is still increasing speed, the rate any points of inflection in the resulting graph.
of increase slows down, so at t = 5 it is traveling at a 2. Sketch four different graphs that satisfy the following set
constant rate. It does so until t = 6, when it begins to of conditions: T h e function starts at ( 2 , O ) and decreases
126 CHAPTER 4: APPLlCATlbNS OF DERIVATIVES TO GRAPHING

until it gets to (4, -2). I t has a relative minimum at


(4, -2) and then increases until it reaches the point
( 6 , 1). It is coristant thereafter.
3. Supposef(x) has the graph shown.

( a ) O n which intervals is f increasing? Decreasing? ( d ) At which relative extremes doesfhave a derivative?


( b ) On which intervals isf concave up? Concave down? ( e ) List all bdt one point bf inflection.
(c) List all relative maxima and relative minima. (f)Where is the unlabeled point of inflection?
4. Explain why the sign chart shown could apply to the
derivative of the function in Exetcise 3 .

For Exercises 5 to 10, ( a ) determzne where the gzven function zs zncreaszng, ( b ) detefmzne where zt zs decreaszng, and ( c ) j n d all relatzve extremes
and the values of the functzon at these extremes.

5 . f ( . ) = 2x3 + 3 x 2 - 12x - 4 6. f ( x ) = 72.9 - x 4

7. f ( x ) =
2x 8. f ( k ) = ~ ( -6 x) +1
&FTT
9. g ( x ) =z ( x 2 - 16) 10. h ( t ) = t 3 - 6 t 2 + 91 - 5
For the functzonr zn Exerczses 1I to I5 determzne the concavzty and all poznts of znJectzon.
11. f ( x ) = 3 x 4 - 8 x 3 - 12x2 + 12x + 10 12, f ( x ) = x 4 - 12x3 + 542 - 108x + 81
14. g(x) = x23 +2
1
15. f ( x ) = - ( x - 5)(x - 2)2
3
16. Where is the derzvatzve of f ( x ) = ( 3 x 5 - 15x4 + 19. I n terms of second derivatives explain why the graph of
20x3 +
2)/8 increasing and where is it decreasing? a lihear function has zero concavity ( a straight face ).
17. Prove Theorem ( 4 . 2 . 3 ) for reiative Minima,
18. Give an example illustrating the necessity for continuity
at xo in the First Derivative Test.
SECTION 4.3: DESCRIBING FUNCTIONS USING DERIVATIVES-PART 2 127

Describing the Behavior of


Section 4.3: Functions Using Derivatives-Part 2

Objectives: A STEP-BY-STEP PROCEDCTREFOR GRAPH SKETCHING


Use the increasing-Decreasing and We now pull together the information from Secs. 4.1 and 4.2. Of special interest
Concavity Theorems, First- and are the points listed in the next definition.
Second-Derivative Tests, and
asymptotes, to sketch graphs of
functions in a systematic way.
4.3.1 Definition A number xo in the domain off is called a critical number or critical
point of the function f if any of the following is true: ( a ) f (xo) = 0 , ( b ) f (xo)
does not exist, (c) f ( x o ) = 0 , ( d ) f ( x o ) does not exist.
In such circumstances we also refer to ( x o , f ( x o ) )as a critical point.
Critical points are obvious candidates for relative extremes and points of
inflection. The following procedure organizes the data we obtain using the
machinery now in hand.

PROCEDURE FOR GRAPHING A FUNCTION P(x)


1. Note any information from the function itself that may be helpful in
sketching the graph (domain, intercepts, symmetry, asymptotes).
2. Determinef and write it in factored form. Note all critical points deter-
mined by f .
3. Determine f and write it in factored form. Note all critical points deter-
mined by f.
4. Using a sign chart forf if necessary, determine where f is increasing and
decreasing and determine relative extremes.
5. Using a sign chart forf if necessary, determine concavity and points of
inflection, and check your choices of the relative extremes in step 4.
6. Sketch the graph.
We suggest that you use this procedure in your initial attempts at graphing.
A few comments on steps 1, 5, and 6 arc in order:
Step I : In most cases the analysis involved in this step is quite simple, but
nonetheless can be very helpful in consolidating your information a t the end.
For example, suppose that
f(x) = p = T
You would certainly note that f is only defined for values of x satisfying 1x1 21
and is symmetric with respect to they axis [ f ( x ) = f ( - x ) ] .
Similarly,
x- 1
f(x) =-
x - 2
is defined only for x # 2 and has a vertical asymptote a t x = 2. It also has the
horizontal asymptotey = 1.
Given the function
f ( x ) = x3 + x2 - 2 x
we can factor, getting
f ( x ) = x(x - l)(x + 2)
128 CHAPTER 4: APPLICATIONS OF DERIVATIVES TO GRAPHING

and hencef(0) = O , f ( 1) 0, andf( - 2) = 0. Thus the graph meets the x axis


a t x = 0, 1, and -2.
However, for another function, such as
f(x) = 3x5 + x3 - x + 9,
no significant information is provided by step 1.
Step 5: It is important to remember that to use the Second-Derivative Test for
relative extremes, the first derivative must be zero at the point. If this is the case,
then you can check your work in step 4.

.p, \ /.*
r y *-*
I ., Step 6: It is also possible to pick up errors here if your information is not
/*
I **
I
I
- consistent. For example, suppose you have determined that f has a relative
II - * - : I maximum at x,, and that the tangent there is horizontal (that is,f(x,,) = 0).
J
I ! When using the Second-Derivative Test you observe that it is concave down to
XO x0 the left of xo and concave up to the right. The concavity results in a graph as
(a] (b) in Fig. 4.20(a). O n the other hand, f (x,,) = 0 means the graph should be as in
Fig. 4.20 Fig. 4.20(6). At this point you would certainly go back and check your work!

THE GRAPHING PROCEDURE: EXAMPLES

Example 1 Sketch the graph off(x) = x 3 - 3x + 5.


Solution (We shall number our steps in the early examples.)
1. This cubic expression is not easily factorable, so after noting thatf(0) = 5,
we proceed to step 2.
2. f ( X ) = 3 x 2 - 3 = 3 ( x 2 - 1)
= 3 ( x + l)(x - 1) critical points x = - 1, x = 1
3 . f ( x ) = 6x critical point x = 0
4. Sign off:

/// \\\\ ////


1 1

By the Increasing-Decrcasing Theorem,fis increasing on ( - 00, - 1) and


(1, +a), and by the First-Derivative Test, f has a relative maximum at
x = - 1, and relative minimum a t x = 1. Also, the tangents a t these
extremes are horizontal since f (-1) = f ( l ) = 0 (no cusps or corners).
5. Without using a chart we see that f ( x ) = 6 x is positive for x >
0, so f is
+
concave up in (0, co),and in (- co,O ) f ( x ) = 6 x is negative and hencef
is concave down. Furthermore x = 0 is a point of inflection. Finally, we
check our work in step 4:
f(- 1) < 0, so a relative maximum occurs a t x = -1
f(1) > 0, so a relative minimum occurs at x = 1
This checks, so we proceed.
6. We first plot our critical points
f(-1) = 7,f(O) = 5 , f ( l )= 3
Fig. 4.2 1 and then piece together our information in Fig. 4.21. 0
SECTION 4.3: DESCRIBING FUNCTIONS USING DERIVATIVES-PART 2 129

Example 2 Sketch the graph off(x) = x3 + x z + 6x - 5.


Solution 1. Again, little seems to be apparent from the function itself except that
f(0) = -5.
+
2. f ( x ) = 3x2 + 2x 6. By determining the discriminant (recall Chap. 1,

f P. 61,
b2 - 4ac = 4 - (4)(3)(6) = -68 <0
we conclude that f(x) cannot be factored and thus has no real roots.
Therefore it is always positive or always negative. By substituting a number
intoj(x), for example, x = 0, we see thatf(x) is always positive,
3. +
f ( x ) = 6x 2 = 6(x 1/3) + (critical point x = -1/3)
4. Without using a chart, the Increasing-Decreasing Thcorem tclls us that f
is always increasing and has no relative extremes.
5. f ( x ) = 6(x +1/3) 0 > for x - 1/3 >
f ( x ) = 6(x +1/3) 0 < for x -1/3 <
sof is concave down to the left of x = - 1/3, concave up to the right, and
has a point of inflection at x = - 1/3.
6. f(-1/3) = -1/27 +
1/9 - 6/3 - 5 = -1%7/27
Fig. 4.22 f ( 0 ) = -5 (see Fig. 4.22). 0
[n neither of the previous examples did we bother to determine the x inter-
cepts precisely. However, we now know enough aboutf ( x ) to have confidence in
the approximation reflected in Fig. 4.22. It turns out that Newton devised a
systematic technique using first derivatives to approximate x intercepts (some-
times called the Newton-Raphson method).
Step 1 becomes especially important when asymptotes are involved.

Example 3 Sketch the graph off(x) =


x 2 - 3x +6
x + 2

Solution 1. Here we note a vertical asymptote x = -2 with

Also, by long division,f(x) = x 2 - 3 3 x + 6 = x - 5


16
x+2 x + 2 so
l6 =
lim [ f ( x ) - (x - 5)] = lim -
2-k- ZkP x +2 o
Hencey = x - 5 is a n oblique asymptote. Finally, we note that the graph
has y. intercept (0, 3) and no x intercept. (Discriminant of the quadratic
numerator is negative.)
(x + 2 ) ( 2 x - 3) - ( x 2 - 3x + 6)(1)
2. f (x) =
(x + 2)2
-
x2 + 4x - 12 - (x + 6)(x - 2)
(x + 2)2 -
(x + 2y
Critical points are at x = 2 and x = -6.

f(x) =
(x + 2)2x + 4) - ( 2+ 4x - 12)2(x + 2)(1) - 32
3.
.( + 2)4 (x + 2)3

There are no new critical points, butf changes sign a t x = -2


30 CHAPTER 4: APPLICATIONS OF DERIVATIVES TO GRAPHING

4. Sign off:

-6 -2 2

By the First-Derivative Test, f has a relative maximum at x = -6 and a


relative minimum at x = 2. At both these relative extremes f has a hori-
zontal tangent. Note also thatfis decreasing on either side of the vertical
asymptote x = -2.
5. Sign off :
-

-_ _ _ _ _ -

-2

f is concave down to the left of the vertical asymptote x = - 2 and concave


up to the right.
6. f ( 2 ) = 1,f ( - 6 ) = -15; see Fig. 4.23. 0

J
k = -2

/
LJ
/
/
/
0.3 /
/
2. 1 /
* //

J r; =
,x2 - 3v +6
x + 2

Fig. 4.23
SECTION 4.3: DESCRIBING FUNCTIONS USING DERIVATIVES-PART 2 131

PROGRBSS TEST 1

In Probs. 1 and 2 sketch the graph of the given function 3. Use the first derivative to confirm the sketch of
by following the step-by-step procedure. Indicate clearly
all relative extremes and points of inflection. h(x) =
3x +2
-
1

in Fig. 4.4.
X
2. f ( x ) = ~

3x - 1

SOME FURTHER EXAMPLES: FELACTIONAL EXPONENTS


The following pair of examples illustrates two points:
1. Relative extremes can occur where f ( x ) does not exist.
2 . When dealing with fractional exponents, try to factor, leaving one factor
with an integer exponent.

Example 4 Graphf(x) = 6x113 + 3x413


Solution 1. f ( x ) = 3x1I3(2 + x ) , s o f ( 0 ) = f ( - 2 ) =0
2. f y X )= 2x-2/3 + 4x1/3

Critical numbers are x = - 1/2, x = 0 [ f ( 0 ) is not defined].


3. f ( x ) = -Lx-513 + kx-213
3 3
-- 4 x-5/3 4
- (-1 + x ) =-(-l +x)
3 3x513
Critical numbers are x = 1, x = 0 [f(O) is not defined].
4. Sign off:

~~+~~ ~

_ _ _ ~

\2/3

- 3I 0

(We circle any points on the number line for which any of the factors is
undefined.) Note that the sign off is positive on both sides of x = 0 since
1/x213 is always positivc. Hencef is increasing on both sides of x = 0, so the
First-Derivative Test does not imply an extreme at x = 0 but does imply a
relative minimum a t x = -1/2 with a horizontal tangent since

___-

____
I
tangent at x = 0 is vertical.
5. Sign o f f :
(+I (-)
_ -_
+

-~
xj/3
1+x ~
+
f(- 1/2) = 0. Because limz+,, 1/x213 = 00, we can conclude that the

X I
0 --- is
1 --
SECTION 4.3: DESCRIBING FUNCTIONS USING DERIVATIVES-PART 2 131

PROGRESS TEST 1

In Probs. 1 and 2 sketch the graph of the given function 3. Use the first derivative to confirm the sketch of
by following the step-by-step procedure. Indicate clearly
all relative extremes and points of inflection. h(x) =
3x +2
+iF=T
4
1. f(x) = x4 + -43x 3 - 4x2 - -
3 in Fig. 4.4.
X
2. f ( x ) = ~

3x - 1

SOME FI-RTHEH. EXAMPLES: FRACTIONAL EXPONESTS


The following pair of examples illustrates two points:
1. Relative extremes can occur wheref(x) does not exist.
2. When dealing with fractional exponents, try to factor, leaving one factor
with an integer exponent.

Example 4 Graphf(x) = 6x1I3 + 3x4I3


Solution 1. f ( x ) = 3x1I3(2 + x ) , sof(0) =f(-2) =0
2. f y x ) = 2x-2/3 + 4~
= 2x-2/3(1 + 2 x ) = x2i3(1
2
+ 2x1
~

Critical numbers are x = -1/2, x = 0 [f(O)is not defined].


3, ff(.) = -Lx-j/3+ AX-2/3
3 3
4
- Ax-5/3(-1 + x ) =-(-1 +x)
3 3x513
Critical numbers are x = 1, x = 0 [ f ( 0 )is not defined].
4. Sign off:

----

. p 3

1
-1 0

(We circle any points on the number line for which any of the factors is
undefined.) Note that the sign off is positive on both sides of x = 0 since
l/x23 is always positive. Hencef is increasing on both sides of x = 0, so the
First-Derivative Test does not imply a n extreme at x = 0 but dots imply a
relative minimum a t x = -1/2 with a horizontal tangent since
f(- 1/2) = 0. Because limx+ol/x2I3 = 00, we can conclude that the +
tangent at x = 0 is vertical.
5. Sign off :
+I
- -.- -
1 i-
- --
+I , +~
132 CHAPTER 4: APPLICATIONS OF DERIVATIVES TO GRAPHING

f is concave up for x 0 and x < >


1 and is concave down in (0, 1). Thus by
the sign off, we have points of inflection (PI) at x = 0 and x = 1.
6. See Fig. 4.24. 0

Example 5 Sketchf(x) = x5l3 + 5x2l3


Solution (We shall not bother to number our steps.)
f(x) = x 5 / 3 + 5x23 = x2/3 ( x + 5) f(0) = O,f(--5) =0
f(X) = -x2/3
5 + 1Ox-1/3
3 3
- 5(x
-
+ 2) critical numbers x = -2, x = 0
3x1/3

f(X) = 1Ox-i/3 - 1 O x - 4 / 3
9 9
10(x - 1)
- critical numbers x = 1, x = 0
9x43

Fig. 4.24
-- x +2

-
~ * _ U ?
0 4-1v- 1 %*&

f(- 2 ) = (lo)( - 3)/(9)( m)4


<
0, so there is a relative maximum at
x = - 2 by the Second-Derivative Test. Also, negative concavity on either side
of x= 0 agrees with the existence of a cusp there. There is a point of inflection
wig. 4.45 at x = 1. (See Fig. 4.25.) 0

PROGRESS TEST 2

1. Graphf(x) = 3x513 + 15x2l3,giving the coordinates of all relative extremes and points of inflection.

SECTION 4.3 EXEHCISES

Sketch the graphs of the functions zn Exercises I to 40, gzvzng all pertznent znformatzon.
1. f ( x ) = x 5 +1 2. f ( x ) = x4 - 1 3. g(.) = 3 - x - x 2 4. h(x) = x 2 + 2x - 1
5. f ( x ) = x 3 + 3x2 9x + 10
- +
6. f ( x ) = x 3 - Gx2 9x - 1
7. f ( x ) = x 3 + Gx2 + 12x 5
- 88: f ( x ) = x 4 - 2 x 2 + 1
SECTION 4.4: EXTREME VALUES AND THE MEAN-VALUE THEOREM 133

9. f(x) = -
2x
10. f(x) =
x + l
I l l . f(x) z-+-
1 dx 12. f ( x ) =
x-2
x2+ 1 X2+X+1 6 9 x2 - Gx +5
, . 3 2 2x 4x
13. f ( x ) = xJ + - 14. f(x) = 15. g(x) = 16. h(x) = -
+4
~

6Ti
~

X 4x - 3 x2

17. J J = x ~ ~ 18. y = x ~ 19. f(.) = 2Xfi/5 - 12x1/5 20. g(x) = fl


2x2 x3
*21.f(x) = - (421 g(x) = 23. f(x) = 24. f(x)=x d n
L - 1-x
J

m ~

x2 +1
.25. h(x) = x4I3 + 4x1I3
X
26. J(.) = x(x - 115 27. y = 5x23 - 3x53 28. f(x) =
+c7
29. f(x) = f/m 30. g(x) = 4
- 31. y = (2 - x)lI3 32. y = (x + 1)1/5
33. f ( t ) = d n 34. g(t) = t- 35. qt)= 2t1/3 + ~t-113 36. f(x) = 1 2 ~ -
~ 3 ~~ ~ ~
2t2 -4 12 - x2
37. f(t) = 38. f(x) =
dFF 6 T T
( x - 1)2
39. g(x) = ~ 40. y 3 (- ~ 3)3 - 1 2 ( ~- 3)2 + 3
~ 3)4 - 4 ( -
x3
41. Use first and second derivatives to confirm the sketch of 44. Repeat Exercise 41 for the partial sketch of
f(x) = 1/(x2 + 1) + 3 in Fig. 4.3.
42. Repeat Exercisc 41 for the partial sketch off(x) =
( x - 1)/x2 in Sec. 4.1, Example 2(a).
43. Repeat Exercise 41 for the partial sketch of g(x) = in Sec. 4.1, Example 6.
d l n / ( x - 4) in Sec. 4.1, Example 2(b). x i 2
45. Sketchf(x) =
X>2

Section 4.4t Extreme Values and the Mean-Value Theorem

Objectives: THE EXTREME VALIJES OF A FUXYCTION ON A N INTERVAL


1. Determine if and how Rolles More often than not, when applying derivatives to solve concrete optimization
Theorem and the Mean-Value problems such as those in the next chapter, we are seeking the extreme values of
Theorem apply to a given function a function on a finite interval. Because the theoretical issues underlying this
on a given interval. practical problem have wide-reaching implications throughout the calculus, we
take time here to examine them in some detail.
2. Begin to appreciate the
theoretical depth of the calculus
studied to date.
4.4.1 Definition We say that the function f attains a maximum on a given interval at
c if c is in that interval andf(c) 2f ( x ) for all x in that interval, and we say that
f attains a minimum on that interval at c if c is in that interval and f (c) 5f ( x ) for
all x in that interval. At such points c we say that f attains its extreme values.

Thus f attains a maximum on a n interval Z at c if c is in Z and no value of


f ( x ) on Z is larger than f (6). A function may attain a maximum or a minimum
on I at more than one point of I. Furthermore, such points may or may not
yield relative extremes and may or may not be endpoints of I. See Fig. 4.26 for
some possibilities. In Fig. 4.26(a) the function attains maxima a t c1 and c2 and a
minimum a t c3 (all are relative extremes). In Fig. 4.26(b) the function attains a
maximum a t cI (a relative maximum) and minima at c2 (relative) and endpoint
b. I n Fig. 4.26(c) the function attains maxima a t infinitely many points com-
prising the interval [cl, c2] and minima at c3 and endpoint b. In Fig. 4.26(d) no
extreme values are attained on [a, b].
From Fig. 4.26 ( d ) it is apparent that a function may fail to attain extreme
values on a n interval, so we need some idea of the conditions guaranteeing that
a function does attain extreme values on an interval. We take as our starting
134 CHAPTER 4: APPLICATIONS OF DERIVATIVES TO GRAPHING

I c2
b

Fig. 4.26

point the Extreme-Value Theorem, which gives us the conditions we need. AS


with its companion, the Intermediate-Value Theorem for continuous functions,
[(2.5.12)], we shall note that although it can be proved from more basic
principles about the real numbers, the proof is beyond the scope of a first
calculus course. (To investigate this further, consult your instructor or any text
in advanced calculus.)

4.4.2 The Extreme Value Theorem Suppose [a, b] is a closed interval and f
is continuous on [a, b ] . Thenfattains both a minimum and a maximum value
on [a, b ] .

Perhaps the most noteworthy thing about this reasonable theorem is that its
conclusion may fail if either the interval is not closed or the function is
discontinuous on the interval.
For example, if we define
1
for x #I in LO, 21
(x - 1 ) 2 2
f (4= for x = 1
thenfis defined on [0, 21 but discontinuous at x = 1. (See Fig. 4.27.) Although
fattains the minimum value of 1/2 at each endpoint,f(x) grows without bound
+
as x approaches 1 from either side [lim,,,f(x) = 001. No matter how large we
attempt to redefinef( 1) to be, it is possible to find an x near enough to x = 1 so
thatf(x) > f ( l ) . [For exampleJ(1 +
lo-") =: 10'' >lo1'.] In particular, the
functionfdefined above does not attain a maximum on [0,2]. If you still feel
thatf(1) can be defined so thatfattains its maximum on [0,2] at x = 1, name
f(1); but remember,f(l) must be a real number!
On the other hand, the function
Fig. 4.27 g ( x ) = 2x +1
SECTION 4.4: EXTREME VALUES AND THE MEAN-VALUE THEOREM 135

is continuous on the nonclosed interval (0, 11 (see Fig. 4.28) and attains a
maximum at x = 1; namely, g(1) = 3. But g(x) fails to attain a minimum on
(0, 11. Although g ( 0 ) = 1 and 1 5 g ( x ) for all x in (0, 1 ] , 0 is not in the interval
(0, 11, so it is not true that g attains its minimum on (0, 11 at x = 0. Nor could g
attain a minimum on (0, 13 at any number a > 0 in (0,13 because, for example,
g(a/2) < g(a). Thus there is no number in (0, I ] at which g attains a minimum
value on (0, 11. Again, if you think g attains its minimum at some number in
(0, 11, name that number!
Of course, it can turn out that the conclusion of the theorem may hold even
though its hypotheses do not; see Probs. l(c) and ( d ) in Progress Test 1.

I
Pig. 4.28

PROGRESS TEST 1

1. Sketch examples (an algebraic definition is not nec- ( d ) A discontinuous function on [ l ,41 that has a
essary) to illustrate each of the following: maximum and minimum value.
( a ) A continuous function on [ 1,4] whose minimum 2. Define algebraically a functionf(x) that satisfies the
value occurs at two different points. given condition.
( b ) A continuous function on [ 1,4] whose maximum (u) f ( x ) is continuous on (0,4) and has a maximum
and minimum values are distinct and such that each value but no minimum value on (0,4).
occurs at infinitely many points. ( b ) f ( x ) has a minimum value but no maximum
(c) A continuous function on (1,4) that has a maxi- value on [0,4].
mum and minimum value.

THE MEAN-V,QCITETHEOREM
As we saw in Fig. 4.26(a) to (c), a function may attain on an interval I a
maximum or minimum that is not a relative maximum or relative minimum. If
the function is continuous on I (no breaks or jumps) and f attains, say, a
maximum value on I at a point c strictly between the endpoints of I (whether or
not these endpoints are part of I ) , then there is an open interval containing c
and wholly contained in I such thatf (c) > f ( x ) for all x in this open subinterval.
Thusfhas a relative maximum at c. Similarly, iffattains a minimum value at
d o n I where d is strictly between the endpoints of I , then f has a relative minimum
at d. Furthermore, by Theorem (4.2.3) we know that f f h a s a derivative at such a
relative extreme, then its derivative is zero there.
Our next step is a natural one: to couple the Extreme-Value Theorem, which
guarantees that a functionfcontinuous on [a, b ] attains its extreme values there,
with Theorem (4.2.3) about zero derivatives at relative extremes to conclude the
existence of a point c in the open interval (a, b), wheref(c) = 0.
Putting this all together, we have proved Rolles Theorem, named after the
French mathematician Michel Rolle, who published it in 1691.

4.4.3 Ekolles Theorem Suppose thatfis continuous on [a, b] and differentiable


on ( a , b). Iff(.) = f ( b ) , then there exists a point c in ( a , 6 ) withf(c) = 0.

1. To guarantee that an extreme occurs at c between a and b, we assumed that


f (4 = f ( b ) .
2. To guarantee thatf(c) exists, we assumed that f is differentiable on the
open interval (a, 6).
136 CHAPTER 4: APPLICATIONS OF DERIVATIVES TO GRAPHING

Fig. 4.29

Rephrased geometrically, the conclusion of Rolles Theorem says there is at


least onc point on the graph off between a and b where the tangent line is
horizontal, that is, parallel to the line segment joining (a,f(a)) and (b,f(b)).
Ifwe were to slide the horizontal liney = f ( a ) = f ( b ) vertically (either up or
down) through the graph of the function, its last contact with the graph would
have to be as a tangent line-provided, of course, that the graph is smooth
enough to have a tangent at that point, which Zs the case sincefis differentiable
a- on (a, b ) ; see Fig. 4.29. Note thatfmay have a vertical tangent a t an endpoint,
s*,-:- (2,-1)
as in the second part of Fig. 4.29 a t x = b.
Rolles Theorem is also valid in the one case not represented above, wheref
is constant on [a, b]. I n that case every point c of (a, 6 ) satisfiesf(c) = 0.
Fig. 4.30 Figure 4.30 is a sketch of the function
f(x) = y m-2
for which no horizontal tangent line exists on [0,2] althoughfreaches a relative
minimum of - 2 at x = 1. Of course, Rolles Theorem does not apply, because
x _ 1 J = 3-2/(3
f has a cusp and is not differentiable] at x. =) 1. [Note:f(x) \ -
Our main result, the Mean-Value Theorem, is a generalization of Rolles
Theorem to the case wheref(a) is not necessarily equal to f ( b ) . Geometrically,
its conclusion states that there is a t least one point [ c , f ( c ) ] o n the graph off
between a and b where the tangent line is parallel to the line segment joining
( a , f ( a ) ) and (b,f(b)); see Fig. 4.31.
As with Rolles Theorem, this conclusion is geometrically clear because if we
C b
were to slide the line through A and B vertically (either up or down) without
Fig. 4.3 1 changing its slope [ f ( b ) - f ( a ) ] / ( b - a), then its last contact with the graph
would have to be as a tangent line parallel to the original.

4.4.4 The Mean-Value Theorem Iffis continuous on [a, b] and differentiable


on (a, b), then there exists a t least one point c in ( a , b) such that

Before formally deriving the Mean-Value Theorem (MVT) from Rolles


Theorem, we note that iff(.) (>O) gives distance traveled as a function of time
x over the interval from time a to time 6, then b - a represents the total time
elapsed, and f ( b ) - f ( a ) represents the distance traveled during that time
interval. Hence

.f(b) -.f(a)
b-a
represents the average speed over the time interval. But then the assertion of the
M V T is that there is a t least one point c in the time interval where the
instantaneous speed f (c) equals the average speed.
SECTION 4.4: EXTREME VALUES AND THE MEAN-VALUE THEOREM 137

As a result, you could be issued a speeding citation for exceeding 55 mph


without witnesses and without ever having your speed checked if you were
known to have driven more than 55 mi in 1 h. Having had a n average speed
greater than 55 mph, the M V T proves that a t least a t one point you had a n
instantaneous speed greater than 55 mph. (Of course you might demand in
court that the prosecution prove the M V T from first principles and prove the
Extreme-Value Theorem along the way. See Miscellaneous Exercise 3 for a n
even more subtle case.)
Proof of Mean-Value Theorem: The idea is to subtract a n appropriate quantity
from the function f so that the resulting function g satisfies the hypotheses of
Rolles Theorem-in particular, g(a) = g(b). We subtract the linear functiony
whose graph is the line through A and B. The point-slope form of the equation
of this line is

so, as a function of x,

Thus the function g ( x ) = f ( x ) - y is given by

Sure enough,

and

Furthermore, g is continuous on [a, b] and differentiable on ( a , b), with


derivative

Hence, by Rolles Theorem, there exists a c in (a, b) with

But then we have a c in ( a , b ) such that

which is what we needed to prove. W

We shall now examine two particular cases to see if and how the M V T
applies.

Example 1 Show if and how the M V T applies to f ( x ) = x 2 - 4x on [ 2 , 6 ] .


Solution Sincef is everywhere differentiable, the hypotheses hold. Hence there exists a t
least one c in [ 2 , 6 ] with
138 CHAPTER 4: APPLICATIONS OF DERIVATIVES TO GRAPHING

Nowf(x) = 2x - 4, so we know there is a c in [Z,61 such that


2c-4=4

Of course c = 4 works (see Fig. 4.32). 0

Example 2 Show if and how the M V T applies tof(x) =


Fig. 4.30 for the graph of$)
v m- 2 on [0, 13. (See

Solution VG),
Since f ( x ) = - 2 / ( 3 we see that f is differentiable at all points of
[0, 11 except x = 1. However,fis continuous at x = 1, so the hypotheses of the
M V T are satisfied. (The function need not be differentiable at the endpoints.)
Now

f(1) - f ( O ) - -2 - (-1) = -1
1-0 1
so by the M V T there is a c in ( 0 , l ) such that

Solving for c, we find c = 19/27. 0

Fig. 4.32

PROGRESS TEST 2

1. Determine whether the Mean Value Theorem applies 2. Graph the functionf(x) = /2x - 1I - 3 on [ - 1,2].
to the given function on the indicated interval. If SO, Note that f ( - 1) = f(2) = 0 but there exists no
find all possible values for c. Sketch the situation. number c where ,f(c) = 0. Does this contradict
x3 Rolles Theorem?
( u ) f ( x )z - - x - 1 [-1,2]
3
( b ) f ( x ) = x23 [-1,81
+
( 6 ) .f(x) = x 2 2.x [-2,11
SECTION 4.4: EXTREME VALUES AND THE MEAN-VALUE THEOREM 139

THEORETICAL APPLICATIONS
OF THE MEAN-VALUE THEOREM
Although the M V T has practical applications (for example, in determining
bounds on complicated functions over finite intervals), its most important use is
as a theoretical tool. The following two theorems are widely used in the
remainder of the text.
We already know that the derivative of a constant function is zero. The
converse is also true.

4.4.5 Theorem If the derivative of function is zero on a n interval ( a , b), then the
function is constant on (a, b).

4.4.6 Theorem If two functions have the same derivative for all x in an interval
(a, b), then the two functions differ by a constant on (a, b).
That is, iff(.) = g(x) for all x in (a, b), then there is a constant C such that

f(.> = g(x) + c on ( a , b )
Proofof(4.4.5): Suppose thatf(x) = 0 for all x in (a, 6 ) . It suffices to show that
if x1 and x2 are any two distinct points in (a, b ) with, say, x1 x2, then <
f(xl) = f ( x 2 ) .But f is differentiable on [xl, x 2 ] , so the M V T applies to guarantee
there is a c in (x1,x2) such that

Butf(c) = 0, s o f ( x 2 ) -f(xl) = 0; that is,

Proof of (4.4.6): We apply (4.4.5) to the function


h(x) = f ( 4 - g ( 4
whose derivative
h(x) = f ( x ) - g(x)
is zero on (a, b), to conclude that

h ( x ) =f(x) - g(x) = C (constant)


on (a, b). Thus
f ( x ) = g(x) + C on (a, b ) .
We promised to prove the Increasing-Decreasing Theorem, and now we shall
prove part (a). The proof of ( b ) , which is similar, is left as a n exercise.
Proof of the Increaszng-Decreasing Theorem (4.2.7): We need to show that if
f(x) > 0 on (a, b), thenfis (strictly) increasing on (a, b). Hence given x 2 > x1 in
( a , b), we need to show that

Butfis differentiable on [xl, x 2 ] , so the M V T applies to guarantee c in ( x l , x 2 )


such that

But f (c) > 0 by hypothesis and x2 - x1 > 0, so


140 CHAPTER 4: APPLICATIONS OF DERIVATIVES TO GRAPHING

that is,

as was to be shown. W

SECTION 4.4 EXERClSES

For each of the functions in Exercises 1 to 5, f ( a ) = J ( b ) but there exists no c in (a, b)for whichf(c) = 0. Which hypothesis ofRolles Theorem 2s
violated?
O j x l l x= 1
1. f ( x ) = a = 0, b = 2
(*,L 1 5 x 5 2 a=l,b=3

8. f ( x ) = 1x1 - 2 a = -1, b = 1 4. f ( x ) = 1x1 a = -1, b =1


5. f ( x ) = fl a = -1, b = 1
6. ( a ) Which hypothesis of Rolles Theorem fails for the 7. By subtracting the vertical distance between the line
function f ( x ) = lx2 - 41 on [ - 3 , 3]? with slope [ f ( b ) - f ( a ) ] / ( b - a ) through the point
( b ) Does the conclusion of Rolles Theorem hold in this ( a ,f ( a ) ) (instead of the line through (6,f ( b ) ) , devise an
case? alternative proof of the MVT.
(c) Is there a contradiction in this situation?

I n Exercises 8 to I 5 determine whether the M V T applies for the given function f on the given interval [a, bl, and zfso, determine each point c such
that f ( 6 ) = [ f ( b ) - f ( a ) ] / ( b - a).
X 3
8. f ( ~ ) = - - ~ + 1 , [ - 2 , 3 ]
3
9. f ( x ) = x 3 - +
1 2 ~ 3, [ - 3 , 3 ]

10. . f ( x ) = x4, [0,2] 11. f ( x ) = x2, [O, 21


12. f ( x ) = 6,[O, 11 13. f ( x ) = G,
[ - 1 , 11
14. f ( x ) = 6,[0, 11 15. f ( x ) = d25 - x, [ -5,5]
16. Rephrase the proof of the M V T by showing that the 17. Interpret the M V T for linear functions.
function (of x ) which gives the vertical distance between
the graph off(.) and the secant line through A and B (see
Fig. 4.31) has a maximum and satisfies the hypothesis of
Rolles Theorem.

CIIAPTER EXERCISES

Review Exercises
In Exercises 1 to 9 determine the given limit and describe any asymbtotes thus determined,

1. lim ___
fi 2. lim
+
x2(1 x )
3. lim-
x2 - 4
z-+m &+ 1 r-+m 1 -x r-0 2x2 +x
5. lim-
x3
6. lim-
x-2 7. lim
3x3+ 2x 8. lim- X2

x-2+ x - 2 r-1 (x - 1 ) 2 2--m 2x2 + 2x - 1 2-0 x - 1

9. lim
x2(x 1) +
z-0 x -1

In Exercises I0 to 15 determine where the given functzon is increasing and decreasing and indicate all relative extremes.
10. f ( x ) = 12x3 - 6x2 + 3x - 1 11. g ( x ) = (1 - x ) ( l + x)3
12. f ( x ) = -
(:I ;I3 2 - 3x
13. f ( x ) = -
2+x

14. h(x) =
x2
x - 3
+ 4x + 5 15. f ( t ) = 4 t 3 + 27t - 18t2 - 3
Chapter 5

143
144 CHAPTER 5: USING DERIVATIVES

Section 5.1: Optimization Problems

Objectives: DETERMINING EXTREME \ALI-ES ON ,4N ISTERbAL


Use derivatives to assist in We know that a function continuous on a closed interval [a, b] reaches its
1. Determining the extreme values maximum and minimum values on that interval. If such values occur between
of a function on an interval. a and 6, they are also relative extremes, and if the derivative exists a t such
2. Translating certain problems points, then it is zero there. This suggests the following:
calling for the maximizing or
minimizing of a given quantity into 5.1.1 Procedure for Determining Extreme Values
appropriate extreme-value of a Function on a Closed Interval [a,61.
problems.
( a ) Determine the values off at any critical points arising from f between a
and b.
( b ) Compare these withf(a) andf(b).

If the choice of extreme values in (5.1.1) is not obvious, then apply the
graphing machinery of Chap. 4. In the special case thatf exists and does not
change sign on [a, 61, we know thatfis either increasing or decreasing on all of
[a, b ] . Thus:

5.1.2 Theorem Iff exists and does not change sign on [a, 61, thcnfattains its
extreme values at the endpoints.

Example 1 Find the extreme values off(.) = x/(x + 1) on [ - 1/2, 11.

Solution fW
(x + 1)(1) - 4)-- 1
=
(x + (x + 1)2
Sincef(x) > 0 for all x in [ - 1/2, 11, the function is increasing. It follows
t h a t f ( - l / 2 ) = -1 is the minimum value andf(1) = 1/2 is the maximum
value. 0

Example 2 Determine any extreme values o f f ( x )= x 4 - 2x2 + 1 on [ -3,Ol.


Solution f(X) = 4x3 - 4x = 4 4 x 2 - 1) = 4x(x - l)(x + 1)
Here it is convenient to use a chart to determine the sign off.

//// \\\\ ///


///
*
-1 0 1

Thus we have relative minima a t x = - 1 and x = 1 and a relative maxi-


mum at x = 0. Since we are only concerned with values of x in [ -3,Ol we can
disregard the relative minimum at x = 1.
+
NOW f ( - 1 ) = 1 - 2 1 = 0, f ( 0 ) 1,
and f ( - 3 ) = 81 - 2(9) 1 = 64.+
The maximum value is 64 and occurs when x = -3.
The minimum value is 0 and occurs when x = - 1. 0
SECTION 4.4: EXTREME VALUES AND THE MEAN-VALUE THEOREM 141

In Exercises 16 to I 9 determine the concavity and points of injection of the given function.
x4 x2
16. y = - - - 17.
4 2
18. f ( x ) = (1 - ~)(1+ x ) ~ 1 9.

I n Exercises 20 to 54 grabh the given function-include all pertinent information.

20. f ( x ) = x 5 - 5 x 2 + 21. y = x3 - 1 5 ~
22. f ( x ) = xZ(1 x)3 + 23. f ( x ) = 3x4 - 4x3
24. f(,.)= - lox 25. y = x4 - 3x3 + 3x2
26. g ( t ) = t 4 +
2t3 - 3t2 - 4t 27. f ( u ) = u 4 - 4u3 - 2
28. y = w 4 - 12w3 +
36w2 - 10 29. f ( x ) = x 4 - 16x2
30. g ( x ) = x 3 - 3x2 - 9x + 10 31. f(x) = 2x6 - 3x4
2x
32. t = 3x5 - 5 x 3 33. f ( x ) = 6x115 + x615 34. f ( x ) = 3x513 - 1 5 ~ ~ 35. f ( x ) = -
x3 +1
3x X2 2x2
36. y = - 37. y = 22- 38. f ( x ) = 39. f ( x ) = ___
4x - 3
4x - 3

40. g(x) = -
2x 41. f ( x ) = x2(1 + 42. f ( x ) = x - 43. f ( x ) =
2x
I - x
~

(x + 1)2

x-1
45. f ( x ) = 3 + -
2x2
44. f ( x ) = - 46.
x2 +2 x+2
50.

53.

54. g(x) = r2 +
x + l
<
for x
forx>O
0
55.

56. ( a ) Show that the M V T hypotheses fail forf(x) = f,& 58. Do the hypotheses (hence does the conclusion) of the
on [-1,8]. M V T hold forf(x) = d g o n [- 1, l ] ? If so, what
( b ) Does its conclusion fail? Explain. is c?
57. Repeat Exercise 56 forf(x) = q m on [ - 3 , 31. 59. Repeat Exercise 58 forf(x) = lx31 on [ - 1, 11. (Be careful
at x = 0).
Miscellaneous Exercises
1. Use the MVT to determine a linear function that is an 7. Using the MVT and/or its consequences, show that if
upper bound forf(x) = Zx3 on [O, 11. f ( x ) is a polynomial of degree n, thenf(x) is a polyno-
2. Use the M V T to determine a linear function that is mial of degree n 1.+
parallel to the upper bound of Exercise 1 but is a lower 8. Develop some conjectures regarding the degree of a
bound forf(x) = 2x3 on [0, I]. (Its graph will be tangent polynomial and the number of relative extremes it can
to that off(.).) have. (If the graph of a polynomial has as many peaks
3. Provide the prosecution case that a driver clocked at and valleys as your open hand held in front of you,
45 mph in a 55 mph zone 1 h and 55 mi down the road what is the minimum degree of this polynomial?)
from a point where he was clocked at 55 mph should be 9. (Third-Derivative Test for Inflection Points) Suppose
convicted of speeding. f ( x ) exists on a n open interval containing x = a. Prove
4. Translate the above argument to the case where the that iff(.) = 0 andf(a) # 0, then f has a point of
initial clocking was at 45 mph and the clocking 1 hr later inflection at a. (Apply the Second-Derivative Test t o f
was at 55 mph. and interpret the results.)
5. By deriving a contradiction using Rolles Theorem, show 10. Show by example that the Third-Derivative Test for
that p ( x ) = ax2 bx + +
c has at most two real roots. Inflection Points may fail iff(a) = 0.
(Assume it has three and work backward.)
6. Show that a cubic polynomial has at most three real
roots. (Build on Exercise 5.)

( T h e next several exercises extend to higher derivatives the relatiom among zero derivatives, extremes, and injection points. T h i s covers cases where the
Second- and Third-Derivatiue Tests f a i l because the second or third derivative, respectiveb, is zero.)
11. Suppose that k >
2 is an integer andf(k)(x) exists in an given outline, prove that iffk)(x) has a relative maxi-
open interval containing the point x = a. Following the mum equal to zero at x = a andf(k-l)(a) = 0 also, then
142 CHAPTER 4: APPLICATIONS OF DERIVATIVES TO GRAPHING

f(k-2)(~) also has a relative maximum at x = a. 17. Apply Exercises 13 and 14 to determine extreme and
(a) Show thatf(k)(x) is decreasing in some open interval inflection points forf(x) = 5x6 - 6x5. Sketchf(x).
containing a. 18. Supposef(x) = (x - a)ng(x), where g(a) # 0 and n > 1
( b ) Show thatf(k-l)(x) changes sign at x = a. is an odd integer with g("'(x) defined in an open interval
(c) Show that the conclusion follows. containing x = a. Prove that the graph off(.) is tangent
12. Repeat Exercise 11 for a relative minimum. to the x axis with a point of inflection there.
13. Suppose the first n derivatives o f f exist in an open 19. Repeat Exercise 18 but with n even to conclude an
interval containing the point x = a, where n 2 2 extreme point at x = a.
is an integer. Prove that, for n an even integer, 20. Apply Exercises 18 and 19 to sketch the graph of
o
if f ' ( a ) = f " ( a ) = f " ' ( a ) = . . . =f("-1)(a) = and f ( x ) = (x - 1)2(x + 2 ) 3 near where it touches or crosses
f'"'(a) # 0, then f has a relative maximum at a if the x axis.
f'"(a) < 0 and a relative minimum at a iff'"'(.) > 0. +
21. Repeat Exercise 20 forf(x) = x3(x - 1)2(x 2 ) 2 .
(Apply Exercises 11 and 12 to "back up" to the given 22. Prove that f(x) = 4x3 + 3x2 - 6x + 1 has a root be-
conclusion, stepping backward two derivatives at a tween 0 and 1. (Hint: Apply Rolle's Theorem to any
time.) function whose derivative isf(x) on [0, 11.)
14. Under the same conditions as Exercise 13 but with n odd 23. Given that If'(x)I 5 1 for all numbers x , show that
and without the condition thatf'(a) = 0, prove that f ).(fI - f ( y ) I 5 Ix -yI for all numbers x,y.
has a point of inflection at x = a. (Use same approach as 24. Let P(x) = a + +
alx +
. . . anx". Show that between
in Exercise 13.) any two consecutive roots of the equation P'(x) = 0,
15. ( a ) Apply Exercise 13 tof(x) = x4. there is at most one root of the equation P(x) = 0.
( b ) Generalize part ( a ) to any positive even power of x.
16. ( a ) Apply Exercise 14 tof(x) = x5 to prove thatfhas a
point of inflection at x = 0.
(6) Generalize part ( a ) to any positive odd power of x.

SELF-TEST .t)
SECTION 5.1: OPTIMIZATION PROBLEMS 147

Fig. 5.4 for a graph ofA(x) on [0,50].)


Thus the dimensions of the pasture with
maximum area are x = 25 by y = 50, and the area is 1250 m2.
This example embodies the essence of a general procedure.

mentioned.
.Ic = 1 0 0 ~- 2u2 2. Select the quantity to be maximized or minimized and express it in terms of
I the remaining quantities.
I
I 3. Use the given information to eliminate all but one of the quantities in the
I expression in order to express the quantity to be maximized or minimized as
I
I a function of a single variable.
;0 25 50
4. Note any restrictions on the domain of this function,
5. Use the methods developed earlier to determine the maximum or minimum.

Example 4 A cylindrical tin can is to have a volume of 128 cm3. What are the dimensions
that require the least amount of metal?
Solution We assume the can is a right circular cylinder with both a top and bottom.
1. We draw a figure (see Fig. 5.5) and denote the radius of a typical such can

?
[JI
by r and the height by h.
2. The least amount of metal will be used when the total surface area is a
minimum, so we are to minimize surface area. The lateral area of the can is
h +
27~rhand the total area of the top and bottom is r r 2 m2 = 2 m 2 . Hence the
total surface area A is given by
A = 2vrh + 2m2
3. Now the volume of the can is 128 cm3, so using the formula for the volume
Pig. 5.5 of a right circular cylinder, we have
.rrr2/i = 128
Solving for h (which is a bit easier than solving for r), we get

h = - 128
m2

which allows us to express A in terms of the single variable r :

256 + 2n1,2
I
=
r 4. The physical restriction on r is simply that r > 0. (There is no can with
I r = 0.)
I
I 5. Now
I
I
I
A(r) = -
-256
r2
4n-r +

I1
I
= -4[ m 3 - 641
r2
and so the only critical value on (0, + 00) is r = = 4/f/-. 7~ Now

Fig. 5.6
512 + 47T
A(r) = -
r3
>0
148 CHAPTER 5: USING DERIVATIVES

for all r> 0, so the Second-Derivative Test tells us that r = 4/v77 yields a
relative minimum. Since it is the only extreme value of A(r) on (0, a), we +
can be assured by (5.1.3) that A ( r ) reaches its minimum value on (0, co)at +
r= 4 / w ; see Fig. 5.6. Note that as r gets close to zero, A(r) gets large
without bound, (an extremely tall can). Also, as r gets large, A(r) gets large
without bound (an extremely wide can).
6. Thus the dimensions that use the least amount of metal are

r=- =:2.73cm and h = -128 -


- 128 ~ 5 . 4 6 c m0
fi m2 T ( 16/77 23)

Example 5 The federal government is to fund both the building of a sewage plant and its
connection to towns A and B, which will share the plant. The federal govern-
ment wishes to locate the plant so as to minimize total connector costs. Town A
is 2.5 km downriver from the river port B and 2 km away from the river on the
same side as B. ( a ) The larger town As connecting installation costs are $4
million/km and Bs costs are $4
million/km. Where should the plant be
located? ( b ) During the approval procedure a demographic study is made that
indicates the population of B will remain stable, but town A will grow,
requiring connectors costing $4
million/km. Where now should the plant be
located?
Solution (a) 1. We sketch the situation as in the marginal figure and assign variables to
the lengths along the river associated with the plants location.
2. We wish to minimize the total cost, which (in millions of dollars) is

G x
C(X) =
4
+ -y5
3. Sincc x+ y = 2.5, we know thaty = 2.5 - x and thus can write C as a
function of x :

X Y C(x)=
dx2+ 4 +-2.5 5 - x

+ 2.5
4
4. The situation dictates that 0 5 x 5 2.5.
5. We compute C(x) and set it equal to 0:

C(X) =
X
- -1 =o
4 d i T Z 5
Solving for x , we obtain x = +(8/3). Since neither of these is in the
interval [0, 2.51, the minimum must occur at an endpoint. Now
C(0) = 4+ 2.5/5 = 1 and C(2.5) = ( d w ) / 4 =. 0.8.
6. Thus the minimum connecting cost occurs when the plant is located at
town B.
( b ) In this case the function to be minimized is

C(X) =
\/x2 + 4 +-2.5 - x , O I x I 2.5
3 5
Now
1
C(X) =
X
-- =o
3dW 5

when x = -t(3/2). We discard x = -3/2 since it is not in [0, 2.51. Now


SECTION 5.1: OPTIMIZATION PROBLEMS 145

I I /
I I I
I I I

Of course, not all intervals are closed, nor are all functions everywhere
continuous. On a n arbitrary interval I , one that is not necessarily closed or even
bounded, a continuous function need not attain any extreme values. However, if
f is differentiable on an arbitrary interval I except perhaps at one or more
endpoints and has a szngle nonendpoint relative extreme, then this relative
extreme is a n extreme value on I. Suppose, for example,fhas a single relative
maximum at xo as in Fig. 5.l(a). Iff(xo) is not the maximum value o f f o n I
(meaning thatf(E) >f(xo) for some other X in I ) then the graph would have to
turn upward elsewhere on I to reachf(F). This would give rise to another relative
extreme x1 on I (a relative minimum, in fact), as in Fig. 5.l(b).
More precisely, we have:

5.1.3 Theorem Supposef is differentiable on an interval I, except perhaps at one


or more endpoints, and is continuous on all of I. Iff has a single relative extreme
at xo not an endpoint of I , then f ( x o ) is an extreme value for f on I.

Example 3 Determine the extreme values if any of the functionf(x) = v x 2 + 8/x on the
interval (0, +a).
Solution Since lim+-o+f(x) = limr++mf(x)= +CO, f has no maximum value on the
interval (0, +m). Now
8
J(\) = KX2 + ~

8
f(x) = 2TX --
X2

2
= -[71x3 - 41
X2
I
I and so the only critical value in the interval is x = (4,~)~~.
I
1
+
Since f ( x ) = 271 > +
16/x3 0 for all x in (0, co), and in particular
I >
f ( ( 4 / 7 ~ ) ~ ~ ) 0, we know by the Second-Derivative Test thatfhas a relative
I minimum at x = ( 4 / 7 ~ ) ~ It~follows
. from (5.1.3) thatfattains its minimum on
I
,+x +
(0, CO) at (4/7)ll3. (See Fig. 5.2.) 0
x = (4/K)13

Fig. 5.2

PROGHESS TEST I

Determine the minimum and maximum values off on 2. f ( x ) = x5 + 2x3 on [0,3]


the indicated interval (if they exist).
2x2
8. f ( x ) = -on (1, + a )
1. f ( x ) = 2x3 + 3x2 - 12x + 4 on [-3,3] 1-x
146 CHAPTER 5: USING DERIVATIVES

SOLVING SPECIFIC OPTIMIZATION PROBLEMS


Using mathematics to solve concrete problems inevitably involves the practice
of mathematical modeling. Simply stated, mathematical modeling is con-
cerned with translating a real-world problem into a mathematical problem,
solving the mathematical problem, and then using the solution in turn to solve
the original problem.
The mathematical model for many problems leads to finding the maximum
or minimum value of a function on an interval. Such problems often identify
themselves by including such superlatives as least, greatest, strongest.
For example, suppose we have a large field adjacent to a river and 100
meters of fence with which we are to enclose part of the field to form a
rectangular pasture. Assuming that one side of the pasture will be the unfenced
river bank, what are the dimensions of the pasture with largest area that we can
enclose? What is this area? This is a maximizing problem; that is, we are
maximizing area with a fixed amount of fence (perimeter) and a given shape
(rectangle). Perhaps the first thing to recognize is that many different pastures
will satisfy the given conditions, and it is our job to determine which one will
have the largest area. See Fig. 5.3. (The wavy line denotes the river bank.)

We start by sketching a typical pasture (see figure at left) and labeling the
lengths of its sides by x andy. The area A to be maximized is, of course, given by
A = xy. However, all our differentiation and maximizing techniques apply to
functions of one variable. But we can use the given information, that we have
only 100 m of fencing, to eliminate one of the variables, say,y. Since we have
100 m of fence to use, we know
A = xy
2x + y = 100
Thus y = 100 - 2x and so A may be written as a function of the single
variable x :
A ( x ) = x(100 - 2x) = lOOx - 2 x 2
The physical situation gives us natural restrictions on x, namely 0 5 x 5 50.
(The two extremes yield the trivial minimum, namely, no area whatsoever!)
Thus our problem is reduced to finding the maximum value of
A ( x ) = 100 x - 2 x 2 on the interval [0,50]
But
A@) = 100 - 4x
= 4(25 - X)
= 0 when x = 25

Thus the only critical point is x = 25, and it is a relative maximum. Since the
endpoints x = 0, x = 50 yield A = 0, the maximum occurs when x = 25. (See
4

40

: J4 40
*O 30 I I30 90

192 m 2 800 m 2 1200 m 2 450 m 2


Fig. 5.3
SECTION 5.1: OPTIMIZATION PROBLEMS 149

2 +2
C(0) = -
3
25
5
- 1.17
N

C(2.5) =
* 3
z 1.07
4
Thus the plant should be located km upriver from A and 1 km downriver
from B. 0
As a practical matter, it is wise to convert to decimals while working
problems of this type only to assist in comparing magnitudes and perhaps in
giving the final answer. Premature punching of the calculator may obscure
useful simplifications and clarifying relationships.

PROGRESS TEST 2

We suggest you follow the six-step procedure, especially 3. Repeat Prob. 1 but with the subdividing fenceparal-
in Probs. 4 and 5. lel to the river.
4. The top and bottom margins of a page in a tiny
1. Given the same field along the river as in our intro- code book are each 1.5 cm, and the side margins are
ductory discussion, determine the dimensions and each 1 cm. If the area of the printed part of the page
total area of the largest pasture that can be enclosed is fixed a t 30 cm2, what are the dimensions of the
using 120 m of fence, with the additional condition page of least area? (Let x a n d y denote the height
that the pasture is to be subdivided into two parts and width of the page, respectively.)
(using the same fencing) with a fence perpendicular 5 . You are visiting a gold mine where each visitor is
to the river. Note that the location of the internal given a 12 x 24-cm piece of cardboard to be made
fence is irrelevant. by the visitor into an open-top box by cutting out
2. Determine the pair of numbers whose product is as squares from the corners and folding up the sides.
large as possible, where the sum of one number and The visitors box is then filled with gold dust as a
triple the other is fixed at 120. Have you seen this free sample to take home. What are the dimensions
problem before? of the box of largest volume you can construct?
-

THE DERIVATIVE AS A TOOL FOR DEEPENING


INSIGIiTS INTO QlTANTITATIVE RELATIONSHIPS
You undoubtedly recognized that Probs. 1 and 2, Progress Test 2, were essen-
tially the same; that is, they each translated into the same mathematical
formulation: maximize xy, where 3x + y = 120. Each problem had the same
mathematical model involving the same mathematical solution based upon the use
of first derivatives.
Historically, especially in physics, the derivative model has been used to
reveal and express dynamic relationships between variables. Now we shall
examine a few situations where money plays an essential role. We use money
because it is a quantity which, perhaps not commonly possessed, is commonly
understood.
The symbol X and the vertical rule in the text signal an application designed to illustrate
ways that calculus relates to the everyday world.
X The Fence Problem Revisited We first consider some variations on
the original fence problem (maximize the area of a rectangular pasture one
side of which is a river bank-no subdivisions), but instead of being given a
fixed 100 m of fence, we are given $100 to purchase fence. If all the fence costs
$1 per running meter, then this problem has the same mathematical model as
150 CHAPTER 5 : USING DERIVATIVES

the original with the same 25 x 50-m, rectangle solution. Notice that the total
length of the fence in one direction is equal to the total length in the other
( 2 x = 50 = y ) .
Suppose instead we are given the same $100 but told that, because there is
a road along the side of the fence parallel to the river, a more substantial fence
is required on that side, costing $2/m. What then are the dimensions of the
rectangular pasture of largest area that can be enclosed? We proceed as before,
drawing a diagram but labeling each side by its total cost rather than its length.
Of course, the sides perpendicular to the river of length x cost 1 * x = x dollars
apiece, and the side parallel to the river of lengthy costs 2y dollars. We are still
trying to maximize area A = q ~but , now the given condition ($100 to spend)
translates to
2x + 2y = 100
We solve for one variable in terms of the other-say,y in terms of x (we could do
the opposite to get A as a function ofy and obtain the same solution)-to obtain
~ = 5 0 - x
Thus
A = A ( x ) = 50x - x2 (0 5 x 5 50)
Now, if x = 50, then all the money is spent on x fence, leaving no money fory
fence and zero area. Similarly, x = 0 means all the money is spent fory fence,
resulting in zero area. Now
A(x) = 50 - 2x = 0 when x = 25
Since A(x) = -2 < 0 for all x, including x = 25, we know that A reaches its
maximum at x = 25. T h e n y = 50 - 25 = 25, and the largest pasture is a
25 x 25-m square of area 625 m2.
y = 25 The effect of doubling the cost of they fence was to cut the area in half and,
perhaps even more interesting, to change the shape of the largest possible
pasture. In effect, the benefit of a free fence along the river (which accounted
for the lengthening in the direction parallel to the river in the original situation)
was neutralized by the doubling of the cost on the opposite side along the road.
625 m 2 Suppose now the cost of the fence perpendicular to the river stays at $1/m
but the fence along the road costs $5/m and there is still only $100 for fencing.
Then the given condition becomes

2x + 5y = 100 or y = 20 - -2x
5
and

y = 10 (
A ( x ) = x 20 - 2
5)
2x2
= 20x - -
5
(0 < x < 50)
Now
4 -4
A(x)=20--x=O andA(x)=-<O
5 5

implies a maximum area a t x = 25 with y = 20 - (2/5)(25) = 10 m and


250 rn2 A = (25)(10) = 250 m2.
Is it an accident that the length of the x side is 2.5m in each of the three cases? One way
to check is to let the price of they fence be k dollars per meter, so the given $100
constraint is
2x + ky = 100
so
100 - 2x
y= k
SECTION 5.1: OPTIMIZATION PROBLEMS 151

1 ksmaller
A ( % )= ~
1
k
for various k
(1OOx - 2x2)
Then

and
1
A ( x ) = --(loox - 2 2 )
k (0 5 x 5 50)

1
A(x) = -(lo0 - 4 x ) = 0 when x = 25
k
Sure enough, whatever the cost of the fence along the road, the largest
pasture results when each of the other two sides is 25 m. The only effect of k
is in the constant l / k in the formula for A(x), which does not affect the location
of the relative extreme, as can be seen in Fig. 5.7.
Fig. 5.7 O n the other hand, if we also allow the cost of the x fence to vary, letting its
cost per meter b e j dollars, then the $100 constraint becomes
1
2jx + /y = 100 or y = -(lo0
k
- 2jx)

Thus
1
A ( x ) = -(loox - 2jx2)
k
and
1
A(x) = -(lo0 - 4jx) = 0 25 50
when x = - a n d y = -
k J k
<
since A 0 and 25/ is in [0, 50/j], x = 25/ is a maximum.
Notice that the length of each fence depends only upon its cost per meter and
not upon the cost of the other! Moreover, the total length of the x fence is
2(25/j) = 50/j, so the total amount of money spent on the x fence is
50 .
-(I) = 50 dollars
j
and the total spent on they fence is
50
-(k) = 50 dollars
k
The underlying symmetry of the solution to the original problem (same total
length in each direction) prevails here in the general situation: we spend the same
total amount in each direction, namely $50, no matter what the respective cost per unit length
of thefences! Physically, different relative costs per unit length lead to pastures of
different shapes and areas, but our analysis has revealed a deeper underlying
symmetry, which, in fact, is based upon the symmetry of the parabolic graphs
of the various area functions.

Physical, economic, or even legal restrictions often heighten the importance


of endpoints in optimization problems.
X Minimizing Transportation C o s t s A trucking firm estimates that
+
the approximate cost per mile for a given type of truck is [20 (1/5)v] cents
per mile, where v is the velocity in miles per hour and the 20 cents represents
fixed cost (licenses, insurance, depreciation, and so on). Given that the driver
gets $8/h, what average velocity for a 500-mi trip will minimize the total cost?
The cost per hour of the truck at velocity v is the product of the miles per
hour with the cents per mile, or

(
v 20 + s>
- cents/h

Adding in the labor cost of 800 cents/h, we conclude that the total hourly
cost is
152 CHAPTER 5: USING DERIVATIVES

2Ov +-
U2
5
+ 800 cents/h
Hence the total cost of the trip C(u) is the product of this hourly average with
the total hours of the trip
500 mi -
~- - 500
vmph u
That is,
C(u) =
500 (200 + U 2
~
+
800
U

= 500 20 ( + + 8:)
- ~ cents

Now, we need to minimize C(u) where u >


0 and u is less than somc reasonable
velocity. As usual, we search for relative minima by setting C(u) = 0 and
solving for u :

C(v) = 500 ($ - )
:t =0
when
u2 = (5)(800)
or
u z 63.25 mph
This is a relative minimum by the Second-Derivative Test, so the answer to
the original question is: As close to 63.25 mph as possible.
X Discussion of Truck-Velocity Example-Why Do Truckers
Hurry? In terms of United States traffic laws the legal interval over which
we minimized C ( u ) is (0,551, but, as anyone who has driven on major high-
ways knows, this is not the interval in which most truckers drive. The legal
solution to our problem is 55 mph since C(v) is positive-for 0 < < u 63.25.
Actually, by the Mean-Value Theorem (and common snese), the trucker would
need to spend some time at more than 55 mph to average 55 mph. But now we
know why truckers, most of whom own their own rigs, prefer higher speeds: to
make up for fixed costs. Note that we kept the 500 factored out of our calcula-
tions: The length of the trip did not affect the solution. In practical situations,
of course, the length of the journey would affect the solution because of the
regulated length of the driving day for truckers. (The 500-mi trip would take
almost 8 h at the optimal speed.)
As we saw, the mathematical analysis of a situation is sometimes less
concerned with a particular set of data than with how the elements of the
situation relate to one another. Let us determine how changes in fixed costs,
especially labor, would affect optimal speed. Suppose we denote the labor costs
(in cents per hour) by L. Then we can retrace our earlier solution to see that

I so

making C(u) = 0 when u = a - a relative minimum.


Solving this relationship for L, we get L = v2/5. An hourly pay rate corre-
sponding to a 55 mph optimal speed is thus $6.05. On the other hand, a trucker
averaging 65 mph justifies $8.45/h.
Notice that the 20-cent-per-mile nonlabor fixed costs do not enter the optimal
velocity issue. However, the (1/5)u term does. It says that the cost per mile
SECTION 5.1: OPTIMIZATION PROBLEMS 153

increases proportionally with the velocity. If a trucks proportionality constant


increased to, say 1/ 3 (because of increased fuel or tire costs, or a loss of efficiency
of the engine), then higher speeds would be more expensive. In fact, a t a labor
cost of $8/h, the optimal velocity would then become
u = \/30 z 49 mph
We could even consider a situation where the cost per mile increases with
velocity much faster than linearly, as is the case with a tugboat pulling a barge,
for example (because of the resistance of the water the tug and barge must plow
through). Suppose the cost per mile for our tugboat is

20 + -51u 2
~ Assuming $8/h labor,

U
making
C(u) = 500 (+u - 9) =0
when
u = q m z 12.6mph
Here the increased length of time for the 500-mi trip (almost 40 h) amounts to
a decrease in productivity of the driver. This would act to hold wages down-
unless, of course, productivity in terms of tonnage-moved increased to com-
pensate, which is the case for barges.

PHOGRESS TEST 3
~

1. (a) Determine the dimensions and area of the four- ( b ) How much money is spent on fencing in either
sided, rectangular pasture of largest area that can be direction?
enclosed by 100 m of fence (same type of fence on all 3. Suppose two captives are given 800 m of fence and
four sides). told to make two separate plots in which they must
( b ) Compare the symmetry of this example with the spend the next 20 years. However, they are not told
original three-sided example discussed on p. 146. that they must spend these years in different plots.
(c) How much area can be enclosed if we allow a How should they utilize their fence to achieve maxi-
circular pasture? mum area under the given conditions (endpoints are
2. (a) Repeat Prob. l ( a ) , but with the total amount of important here)? Explain your result in non-
money available for fence at $100 and the fence in algebraic terms.
one direction costing $2/m and the fence in the
perpendicular direction costing $5/m.
-

SECTION 5.1 EXYEIICTSES

In Exercises 1 to I0 determzne the minimum and maximum values o f f ($they exist) on the indicated interval.

1. +
( u ) f ( x ) = -on [-3,01
x-1
2. ( a ) f ( x ) = x2 + -1 on [A,21
( b ) Repeat ( u ) for the interval [-3, 11 ( 6 ) Repeat ( a ) for [ - 1, 21
x2 2x2 + 1 on [ - 3 , 0 ]
3. f ( x ) = - + - - 2x on [-3,4]
x3
4. f ( x ) = x 4 -
3 2
30x
5 . f ( x ) = 3x +-
x--2
on (2, +co) 6. f ( x ) =
2 4
on n 4 1
I54 CHAPTER 5: USING DERIVATIVES

7. f ( x ) = x4 + 5x3 + 6 x 2 on [0,5] into the shape of a circle. How should the wire be cut so
8. f ( x ) = x 4 1 3 + 4x113 on [-2,5] that the sum of the enclosed area is a minimum?
22. Repeat Exercise 21, but muxzrnzze area under the condi-
9. f(x) = x d n on [ -3,4] (Be careful.)
tions that the wire rnusl be cut.
10. 23. Find the rectangle of largest area that will fit inside the
top semicircle of x2 + y 2 = r 2 with one side on the
11. A box with a square base is to have an open top. T h e area diameter.
of the material available to make the box is 300cm2. 24. Determine an equation of the line tangent to
What should the dimensions be so that the volume be as + +
y = x 3 3x2 3 that has minimum slope. (Be careful.)
large as possible? 25. ( u ) A rancher wants to construct a rectangular corral
12. Repeat Exercise 11 with the conditions that four layers of divided into three compartments by two parallel wooden
the material are required for the base of the box. separators that are parallel to one end of the corral. She
18. An open-topped, rectangular, square-bottomed aquar- has $1200 to spend on this corral. Given that the separa-
ium is to hold 1 m3of water. Given that the material for tors cost $20/m and the outside fencing costs $lO/m with
the bottom costs twice as much as the sides, what are the requirement that no compartment be less than 8 m
the dimensions of the least expensive aquarium under long or wide, how should she lay out the corral to achieve
these conditions? maximum total area?
14. Repeat Exercise 13, given that material for the bottom (6) Repeat part ( a ) with all conditions the same except
costs: with the separators at $30/m. (Be careful.)
( u ) four times as much as the sides. (c) Given all conditions as above, at what price of the
( b ) k times as much as the sides. separators per meter does the 8-m requirement affect the
15. Given an aquarium, built under the conditions of Exer- solution?
cise 14(b), that is half as tall as it is wide at the base, 26. A straight river is 120 m wide. A man standing on one
determine the relative costs of the material for the sides bank wishes to arrive at a house 500 m upriver and on the
and bottom. other bank. If he can run twice as fast as he can swim,
16. A horizontal gutter is to be made from a long piece of how should he proceed to reach the house in the least
sheet iron 80 cm wide by turning up equal widths along possible time? (Ignore the current.)
the edges into a vertical position. How many centimeters 27. T h e strength of a rectangular beam is a constant k (which
should be turned up at each side to yield a maximum depends upon the material) times the product of its width
carrying capacity? (The carrying capacity is propor- and the square of its depth. Determine the dimensions of
tional to cross-sectional area.) the strongest such beam that can be cut from a cylindrical
17. At midnight ship B was 90 mi due south of ship A . Ship A log of radius R.
sailed east at 15 mph and ship B sailed north at 20 mph. 28. T h e stzffiess of a rectangular beam is a constant G (which
At what time were they closest together? (Draw a dia- depends upon the material) times the product of its width
gram giving all appropriate distances t hours after mid- and the cube of its depth. Determine the dimensions of
night.) the stiffest such beam that can be cut from a cylindrical
18. ( a ) A cylindrical can that must hold exactly 1000 cm3 is log of radius R.
to be constructed from two materials. T h e material for 29. Give the dimensions of the isosceles triangle of largest
the top and bottom costs twice as much as the material to area that can be inscribed i n a circle of radius R.
be used for the side. What are the dimensions of the least 30. ( u ) Suppose a person travels by boat (bringing along a
expensive such can? (Hznt: Let the cost per square centi- bicycle) across a 1-mi-wide body of water from A to a
meter of the material for the sides be k.) landing point L (see diagram), and then bicycles to C at
(6) Without working through the details, speculate on 12 mph. Given that she can row at 3 mph, where should
the shape of the least expensive can that uses the same her landing point be to minimize the total traveling time?
materials for all parts.
C
1s. T h e amount of heat received from a heat source is k l / x 2 ,
where Zis the intensity, x is the distance from the source,
and k is a proportionality constant. (a) Where should we
put a control panel in the 10-m space between a furnace
and a cooling vat (which is not as hot as the furnace) so mi
that the total amount of heat received by the control
panel is a minimum, knowing that the heat intensity of
the furnace is twice that of the vat? ( b ) Where will the
amount of heat received from each source be equal?
20. Repeat Exercise 19 for a furnace: A 1mi B
( a ) three times as hot as the vat. ( b ) m times as hot as
the vat.
( b ) Repeat ( u ) for a rowing speed of 5 mph.
(c) Repeat ( u ) for a rowing speed of K mph. (Let x be
21. A piece of wire of length w is cut into two parts, one of
which is bent into the shape of a square and the other distance from B to L.)
SECTION 5.2: RELATED RATES 155

( d ) Suppose she can get a n outboard motor. What speed 37. Find the length of the longest pole that can be carried
of the boat is necessary for her to avoid bringing the horizontally around a corner from a corridor 4 m wide.
bicycle? (Note that this length turns out to be the minimum value
( e ) Is there any rowing speed that would justify L = B ? of certain lengths.)
31. ( a ) A tank is to have volume 100 m3 and is to be made in 38. A Norman window is i n the shape of a rectangle sur-
the form of a right circular cylinder with hemispheres mounted by a semicircle. Such a window being built into
attached at each end. If the material for the ends costs a church is to be bordered by a narrow strip of gold leaf
three times as much per square meter as that for the sides, 12 m in length. Find the dimensions of the Norman
find the most economical proportions. (Let c be the cost window of largest area that can be so bordered.
per square meter for the sides.) 83. Determine the dimensions of the right circular cone of
( b ) Given a tank exactly as in part ( a ) except that the maximum volume that can be fit inside a sphere of
material for the ends costs K times as much as that for the radius 30.
sides, compare the length of the cylindrical part of the 40. If the largest package acceptable to the post office has
tank with its diameter in terms of K . cross-sectional girth and perimeter totaling at most
32. A grain elevator is to be built in the form of a right 72 in., give the volume of the largest cylindrical package
circular cylinder surmounted by a hemisphere. T h e that can be mailed.
material for the floor costs $2/m2. T h e material for the 4 1. Towns A and B decide to share costs of a water purifica-
sides costs $ l / m 2 and the material for the top costs $3/m2. tion plant along a river. Town A is 2.5 mi upriver from B
If the grain elevator is to hold 1000 m3, what would be and 2 mi from the river. Town B is 3 mi from the river
the most economical dimensions? (on the same side as A ) . Where along the river should the
33. Repeat Exercise 32 but with all materials costing the plant be located to minimize the total pipe involved to
same amount per square meter. connect the towns to the plant?
34. Repeat Exercise 32 but with $1200 to spend on materials 42. In Exercise 41 it turned out that the plant divided the
and the volume to be maximized. distance along the river into pieces, the ratio of whose
85. Repeat Exercise 32 but with $1200 to spend on materials, lengths equaled the ratio of the distances of the towns
all of which cost the same amount per square meter, and from the river. By letting the distances from A and B to
with the volume to be maximized. the river be a a n d b, respectively, determine if this holds
36. What are the relative dimensions of a cylindrical can of in general.
fixed surface area S and maximum volume?

Seetion 5.2: Related Rates

O bjeetive: A YHOCEDUHE FOR SOLWNG RELATEID RATE PROBLEMS


Translate the given information and We n o w t u r n f r o m applications involving t h e extreme values of a f u n c t i o n t o
a given question regarding rates applications based u p o n t h e following f u n d a m e n t a l fact: r f x zs a quantzty that zs
into derivative statements and changzng with tzme, then the znstantaneous rate ofchange of x at tzme t zs gzven by dx/dt.
questions, and then answer the We are concerned with situations in which several quantities, all v a r y i n g
given question by using relations w i t h time, a r e related by s o m e e q u a t i o n or equations. Given t h e rate of change
among the given and asked-for of certain of these quantities at a particular instant, w e shall use the e q u a t i o n s
rates. relating t h e m t o d e t e r m i n e t h e rates of change of other quantities a t this instant.
In each case the c r u x of the problem will be t o u n d e r s t a n d t h a t the rates of
change, w h e t h e r given or asked for, are derivatives.
We outline a procedure for handling such related rate problems and t h e n
illustrate w i t h a few examples.

5.2.1 P r o c e d u r e for Solving R e l a t e d R a t e problems


1 . D r a w a diagram and label any fixed quantities.
2. Express all quantities in terms of t i m e (explicitly or implicitly) and deter-
mine any relationships between these quantities.
3. Differentiate w i t h respect t o t explicitly or implicitly.
4. Insert t h e given quantities and solve for t h e u n k n o w n q u a n t i t y .
156 CHAPTER 5: USING DERIVATIVES

Example 1 A ballpon is being inflated a t the rate of 15 m3/min. At what rate is the
diameter increasing when the diameter has reached 10 m? Assume that the
diameter is zero a t time zero.
Solution Letting V be the volume at time t and w its diameter a t time t , we know
V = (4/3).rr3, where r is the radius. Since r = w / 2 , we have

Remembering that V and w are functions oft, we want to interpret information


given and asked for in terms of derivatives with respect to t. We are given that the
rate of change of the volume is 15 m3/min and we wish to find the rate of
change of the diameter when w = 10. Thus in terms of derivatives, the problem
can be restated: V and w are functions o f t related by V = (.r/6)w3. Given that
dV/dt = 15, find dw/dt when w = 10.
Differentiating with respect to t, we obtain (using the Chain Rule)

Solving for dw/dt, we have


dw
-
-~
2dV/dt
dt Tw2

Now dw/dt depends upon both dV/dt and w.But dV/dt = 15 and w = 10, so our
final answer is
dw 2 dV/dt 2( 15)
- - z 0.095 m/min 0
dt Tw2 T(10)2
A common error in solving problems of this type is to substitute the given
values for t or another quantity into the expressions before differentiating. This
leads to a dead end because, of course, all the derivatives will be zero. The
substitutions should be made after differentiating.

Example 2 A conical tank has a depth of 12 ft and a radius at the top of 6 ft. If water is
pumped into the tank at the constant rate of 4 ft3/min, how fast is the radius of
the surface of water in the tank changing a t the instant that the depth of water
in the tank is 6 ft?
Solution Let V be the volume of water in the tank at time t, and let r and h, respectively,
denote the radius of the surface and the depth of the water in the tank at time t.
Now V, r, h are related by

We are given that dV/dt = 4 and we wish to find dr/dt a t the instant that h = 6.
Since we are neither given nor asked to find the rate at which h is changing, it
is convenient to eliminate h from the problem. To do this we examine a cross
section of the tank (see the figure) and use similar triangles to conclude that

J J
SECTION 5.2: RELATED RATES 157

Since r = 3 when 12 = 6, we can restate the problem as: given V = (2n/3)r3 and
dV/dt = 4, find dr/dt when r = 3. Now
dV dr
- = 2vr2-
dt dt
so
dr
--
dV/dt
--
dt 2nr2
Substituting the given information, we have

dr - -
- =. 0.071 ft/min 0
dt 2 ~ ( 3 ) ~97

Example 8 Two automobiles start from a point A at the same time. One travels west at
80 mph and the other travels north a t 45 mph. How fast are the distances
between them increasing 3 h later?
Solution Let x be the distance the car traveling west has gone after t hours a n d y the
distance traveled by the car going north (see diagram). Using our right triangle
we see that s, x, a n d y are related by
s2 = x 2 +y2
We can differentiate implicitly with respect to t to obtain

ak dx dy
s- =x- +y-
dt dt dt
We are given dx/dt = 80 and dy/dt = 45, and we wish to find ds/dt when t = 5.
When t = 5,
) 80 5 = 400
~ ( 5= -
) 45 * 5 = 225
~ ( 5=
and
s(5) = d x ( 5 ) 2 +J(5)2 = v5iipz
Thus
ds - 400(80)
--
+ 225(45)
dt dmijT5
-
- -.210,625 q
427125 m
(42,125)(25) \/337
- = 5 m z 9 1 . 8 mph 0
2 10,625
PROGRESS TEST 1

1. A ladder 15 m tall leans against the side of a house. (c) Differentiate this relationship with respect to
If the bottom is pulled away at the rate of 4 m/s, time.
determine how fast the top is sliding downward ( d ) Insert the given information into the expressions
when the bottom is 9 m from the side of the house determined in ( b ) and (c) to find dy/dt.
by following the given outline: 2. One airplane flew over an airport a t the rate of
5 mi/min. Ten minutes later another flew over at the
( u ) Draw a diagram and label the vertical distancey rate of 4 mi/min. If the first plane was flying west
and horizontal distance x . and the second south (at the same altitude), find
( b ) Express all quantities in terms of time and give how fast they were separating 20 min after the sec-
the relationship between x and y. ond plane flew over.
158 CHAPTER 5: USING DERIVATIVES

3. W a t e r is p o u r i n g into a conical t a n k (vertex d o w n ) (Hint: D r a w a vertical cross-section diagram and


a t t h e r a t e of 16 m3/h. The t a n k is 18 m tall, w i t h a e l i m i n a t e r from the expression for V using similar
r a d i u s on t o p of 6 m . triangles.)
H o w fast is t h e w a t e r level rising w h e n t h e w a t e r is
3 m deep?

WVHEN IS A PRORLEM A C.4LCULITS PROBLEM?


C o v e r t h e solution t o t h e following e x a m p l e and a t t e m p t its solution yourself.

Example 4 A scientist finds a 200-cm-tall conical s t a l a g m i t e whose vertical h e i g h t is 10


times its base d i a m e t e r . H e r evidence indicates t h a t t h e s t a l a g m i t e w a s formed
by a c o n s t a n t dripping of a 1 percent limestone solution a t t h e r a t e of
10,000 cm3/yr. H o w old is the stalagmite?
Solution The volume of the s t a l a g m i t e is
1
-v 102(200)z 20944 c m 3
3
The a m o u n t of limestone deposited each year is (O.Ol)(lO,OOO) = 100 cm3.
Since t h e r a t e of increase of v o l u m e is constant, t h e s t a l a g m i t e is a p p r o x i m a t e l y
20944/100 = 209.4 yr old. 0

Despite t h e appearance of t h e word rate, this was not a calculus p r o b l e m .


If all significant rates in t h e problem (as above) are constant, this is a clue
t h a t you can probably solve i t w i t h o u t calculus.

SECTION 5.2 EKEHClSES


___ ~

1. Given the conical stalagmite situation of Example 4, edge of his shadow (on the ground) moves a t 120 mph.
find the annual rate of increase in height a t the time the (Be careful with units.)
measurements were taken. Assume that the shape of the ( 6 ) Is the shadows speed constant if his is?
stalagmite remains the same. 8. At a certain instant the lengths of the legs of a 3-4-5
2. Repeat Exercise I for the same stalagmite when it was right triangle are changing. If the longer one is increas-
only 1 yr old. ing at the rate of 0.4 unit per second, how fast must the
3 . A tank in the shape of a right circular cone (vertex shorter one decrease to maintain a constant area? (Of
upward) of radius 7 m and altitude 18 m is being filled course, it is a 3-4-5 triangle for only a n instant.)
with water a t the rate of 4 m3/min. 9. Repeat Exercise 8, but with the shorter leg increasing at
( u ) How fast is the water level rising a t the instant when 0.4 unit per second and the longer leg decreasing.
the water level is 9 m ? 10. When gas is compressed without loss or gain of heat (as
( b ) How fast is the radius of the surface increasing a t the in an insulated piston compressor) i t obeys the law
instant when the water level is 1 5 m ? = k , where k is a constant,p is the pressure, and u is
4. Repeat Exercise 3 with the same data except that the the volume. This is called adiabatic compression.
radius of the new tank is only 3 m. Suppose the volume of a certain gas is decreasing at
5. At a given instant a small balloon is released from 2 in3/s while its pressure is 36 I b / h 2 a n d its volume is
ground level a t a point 100 m away from an observer on 54 in.3. At what rate is the pressure changing?
ground level. If the balloon goes straight up a t the rate of I I. If a n ideal gas is compressed nonadiabatically, it obeys
3 m/s, how fast is the balloon receding from the observer the law pu = k, where k is a constant. Answer the ques-
after 35 sec? tion of Exercise 10 given the same data but obeying the
6. A light is located on the ground 50 m from a building. A pu = k law.
man 2 m tall walks from the light toward the building a t 1a . ( u ) T h e volume of a sphere is increasing at the rate of
the rate of 5/3 m/s. How fast is his shadow growing 1 m3/min. What rate is its radius increasing when the
shorter when he is 18 m from the building? volume is 27v m?
7 . ( u ) How fast must a man who is 5 ft 9 in. tall walk out ( b ) At what rate is the radius increasing 9 m i n later?
from under a 6-ft-high light bulb so that the leading 13. ( u ) A vertical line slides horizontally across the graph of
SECTION 5.3: LINEAR MOTION 159

y = x 2 (for x 2 0) at the rate of 1 unit of distance per side of the band as the band slides along. Answer parts
unit of time. How fast is the point P of intersection of ( u ) and (6) of Exercise 13 assuming that x is measured at
the line and the graph rising vertically when x = 5? the right side of the band.
(6) When x = loo? (c) How fast is P rising vertically at a typical x ?
(c) How fast is the point P moving away from the origin ( d ) What would happen to the answer of (c) i f w e meas-
at a typical x? ured x from the left side of the band?
14. ( u ) Given a differentiable function of x and the condi- 19. If P (as in Exercise 18) is moving along the curvey = x 2 ,
tions of Exercise 13(u), determine how fast P is rising x 2 0 in such a way that itsy coordinate is increasing at
vertically at a typical x. the rate of 1 unit per unit of time. How fast is its x
( b ) Determine how fast P is moving away from the coordinate increasing at a typical x?
origin at a typical x . 20. Repeat Exercise 19 for the curvey = x 3 for x 2 0.
15. Suppose the vertical line of Exercise 13(u) is moving 21. ( u ) Repeat Exercise 19 for thc curve y = 5x + 3 for
horizontally at the rate of 3 units per unit of time. x 2 0.
( u ) How fast is the point P rising vertically at x = 5? (6) Supposey = mx +
b is a typical linear function and
(6) How fast is the point P rising vertically at a typi- repeat Exercise 19.
cal x ? 22. Repeat Exercise 19 for the curvey = x 3 x for x 2 0. +
(c) How fast is P moving away from the origin at a 28. At the instant a melting snowball is 16 cm in diameter it
typical x? is melting at the rate of 2 cm3/min. How fast is its
16. Suppose the vertical line of Exercise 13(u) is moving diameter decreasing at that instant?
horizontally at the rate of k units per unit of time. 24. A 5-m-long horizontal eavestrough has an equilateral
Answer parts ( a ) to (c), Exercise 15. triangle cross section 0.1 m across the top.
17. Given that the vertical line of Exercise 13(u) is moving to ( u ) Given that the level of the water is increasing at the
the right at u ( t ) units per unit of time, answer parts ( u ) to rate of 0.01 m/min when the water is at a depth of
(c), Exercise 15. ( d ) What rule are you using in your 0.05 m, at what rate is water flowing into the trough?
analysis? (6) Later, water is flowing out of the trough at
18. Suppose there is an opaque vertical band 5 units wide 100 cm3/min. At what rate is the water level dropping
that slides across the coordinate plane horizontally, be- when the level is a t 0.08 m?
ginning with its right side at x = 5. Let P be the point (c) Repeat (6) with the water level at 0.02 m.
on the graph o f y = x 2 that is first to appear on the left

Section 5.3: Linear Motion

Objectives: DESCRIBING THE MOTION OF A PARTICLE

Describe the motion, velocity, and Suppose s = f ( t ) denotes t h e position of a particle m o v i n g along a straight line
acceleration of a particle moving in a s a suitably differentiable function of t i m e t. If s >
0, t h e n the particle is s units
a straight line when given a t o the right of the origin. If s <
0, t h e n t h e particle is s units t o t h e left of t h e
differentiable function describing origin; see Fig. 5.8. The p o i n t i n time chosen as t = 0 is merely a reference
its position as a function of time. point. I n particular, negative values of t represent time before t = 0.
For example, suppose t h e position of t h e particle at t i m e t is given by
s = f ( t ) = t 2 - 3t - 5
When t = O,f(O) = -5, so initially the particle is located 5 units t o t h e left
of t h e origin. When t = 5,
f(5) = 25 - 15 - 5 = 5
a n d so t h e particle is 5 units to t h e right of t h e origin; see Fig. 5.9.

Position of particle at time t = 5

I I *S
I
-5 0 5

Position of particle at time t = 0


Pig. 5.8
160 CHAPTER 5: USING DERIVATIVES

I I I 1
-4 -5 0 5
Fig. 5.10

We now investigate what inrormation about the behavior o f this particle can
be deduced from the first derivative of s = f ( t ) . As we know,f(t) gives the
velocity at time t. This quantity can be either positive or negativc. I f f ( t ) 0, >
then the particle is moving in the positive direction (to the right), whereas
<
f ( t ) Omeans that the particle is moving in the negative direction (to the
left). I f f ( t ) = 0, then the particle has stopped. In particular, a t the instant a t
which the particle changes direction its velocity, f ( t ) must be zero.
Now consider the previous example. Since f ( t ) = t2 - 3t - 5 , we have

(
f ( t ) = 2t - 3 = 2 t - -
3
Now 2(t - 3 / 2 ) 0 for t < <
3 / 2 and 2(t - 3 / 2 ) 0 for t > >
3 / 2 . Thus the
particle is moving to the left for time t <
3/2, stopped at time t = 3 / 2 , and
then moving to the right f o r time t >
3 / 2 . N o w f ( 3 / 2 ) = -29/4, and so the
behavior of the particle can be indicated diagramatically on the s axis, as shown
in Fig. 5.10.
Acceleration is the rate of change of velocity with respect to time. Thus the
derivative of the velocity-that is, the second derivative of f(t)-gives the
acceleration.
s =f(t) v =f(t) a =f(t)
The acceleration tells us whether the particle is speeding up or slowing down.
The key is to compare the sign of the velocity and the acceleration at a given time. I f they
have the same sign, then the particle is speeding up. I f they have opposite signs then the
particle is slowing down. Acceleration in a given direction can be thought of as the
effect of a pull in that direction (as with gravity, which provides a constant
acceleration). Thus when the direction of motion and the direction of acceler-
ation are in agreement, the particle is speeding up in that direction. If these
directions are opposite, then the acceleration is in the direction against the
motion, so the particle is slowing down. In the above casef(t) = 2 > 0 for all
t. Thusf(t) andf(t) have opposite signs for 0 t < <
3 / 2 and the same sign for
t > 3 /2 . Hence the particle slows down from t = 0 to t = 3/2, and stops at
t = 3 / 2 . It then accelerates in the positive direction for t >
3/2.

The next example illustrates a systematic way to extract and organize


information regarding the linear motion of a particle.

Example 1 Describe the motion of a particle whose position s at time t is given by


f ( t ) = t3 - 3t2 - 9t - 2.
Solution We computef(t) andf(t):
f ( t ) = 3t2 - 6 t - 9 = 3(t - 3)(t + 1)
f ( t ) zr 6t - 6 = 6(t - 1)
SECTION 5.3: LINEAR MOTION 161

We use a chart to determine the sign off(t) and note the sign off(t)
directly below the chart and with the same t scale; see Fig. 5.1 1.

Sign ofJit):

t = 1
Fig. 5.1 1

We now evaluate the function at each of the critical values and usef(t) to
diagram the motion parallel to a n s axis, as in Fig. 5.12.
f(-l) =3 f(3) = -29 f(1) = -13
Moving right t < - 1; stopped at t = - 1; moving left - 1 < <
t 3 ; stopped
a t t = 3 ; and moving right t > 3.
When t <
- l,f(t) > 0 andf(t) < 0, so the particle is moving to the right
and slowing down. It stops a t t = - 1. For - 1 < <t 1 we havef(t) < 0 and
<
f ( t ) 0, so the particle is moving to the left and speeding up. For 1 t < < 3,
<
f ( t ) 0 andf(t) > 0, so the particle continues to the left but is slowing down
until it stops when t = 3 . Fort > 3 both derivatives are positive, so the particle
continues to move right and speed up. 0

I I I I *S

- 20 -13 0 3
Fig. 5.12

It is important to remember, when using this procedure, that the time scale is
used in the derivative sign charts and the distance scale is used in the motion
diagram.

PROGRESS TEST 1

In the following problem s ( t ) = t 3 - 9t2 + 24t gives the 2. Evaluate s at each critical point.
position of a particle moving along a straight line as a 3. Diagram the motion of the particle along the s axis.
function of t. Describe its motion following the given 4. Compare the signs of s and s to determine when it
outline. is speeding up and slowing down.
I. Find s(t) and s(t) and diagram their signs.
162 CHAPTER 5: USING DERIVATIVES

SECTION 5.3 EXERCISES


~ ~~

In Exercises I to 12, s ( t ) gives the position Of a particle moving along a strazght line. In each case describe in detazl the motzon of the particle.
1. s(t) = t 3 - 3t2 - 24t - 2 2. s ( t ) = 8 - 4t + t2
4t s(t) = t 4 + 5t + 6t2
3. s ( t ) = 4.
~

t2 +4
5. s(t) = t 4 + 2t3 - 3t2 - 4t 6. ~ ( t=
) 5 - 2t2 - 3t3

7 . ~ ( t=) 7 - 6t2 + t 4 8. s ( t ) = t + -1t (t >0)


1
9. s ( t ) = 1 - t --
t (t > 0) 10. s ( t ) = d r n
11. s(t) = dic7
13. Graph the function of Exercise 1 on the t-s axis, using the length of your trip, one above the other with the
the sign of s to determine whether s is increasing or same t scale. T h e upper axis will plot velocity against
decreasing; use the sign of s to determine the graphs time, and the lower axis, the distance with respect to
concavity. time. Time t is horizontal.
14. Repeat Exercise 13 for the function of Exercise 4. ( b ) Plot your velocity and distance in either order or
15. X Graph your automobile trip home (or to another place simultaneously. (Use pencil because erasing will likely
along a familiar route) on the t-s axis, measuring t in be necessary.) Label each important point in time A , B ,
minutes and s in miles according to the following direc- C, . . . , (for example, a stoplight), and attach a narra-
I .
tions. (Do a careful job on a large sheet of paper. This tive that is keyed to these labels and explains the entire
exercise will be useful as a reference later. You can do this trip in terms of both derivatives and descriptive lan-
as a thought experiment using careful estimates, or guage.
with actual measurements, using a clock, speedometer, (c) Check that your three items correspond accurately;
and odometer.) this is the key to the exercise.
( a ) Label two sets of axes with the appropriate scale for

Using the Derivative to


Section 9.4: Help Understand Business and Economics

Objective: INTROD1JCTION
Devise and optimize functions The differential calculus, which has proved itself to be such a powerful tool for
describing business and understanding dynamic processes and relationships in the physical sciences and
economics situations. engineering, has for the same reasons become an increasingly important tool in
most other sciences as well. We shall illustrate its use by exploring some basic
economic concepts, first in general terms, and then more concretely.
In economics and business the word marginal means rate of change.
Often the independent variable involves units other than time, such as units of
production, sales, price, and so on. We now know that if P ( x ) = 400x -
x2 - 5000 represents profit as a function of x, the number of units produced,
then the marginal profit (with respect to units produced) is dP/dx. The deriva-
I (ZOO, 35,000) tive arises as naturally here as it does anywhere.
Increa3e *?/ Decrease
ii production
At a given level of production x, we may ask, does it pay to increase
production? The answer, of course, depends upon whether P ( x ) happens to be
increasing at that level; that is, whether dP/dx is positive. Here dP/dx =
2(200 - x ) is positive for x <
200, negative for x >
200, and is zero at x = 200,
which is a critical point. Since we know that x = 200 is a relative maximum,
the question is answered. (See Fig. 5.13.)
More often than not, the variables in economics and business functions make
realistic sense only at integral values ( x equals the number of television sets
Fig. 5.13 produced, and so on) within some range determined by circumstances. Fur-
SECTION 5.4: USING THE DERIVATIVE 163

thermore, the accounting data and surveys from which the functions are
defined represent only samples and approximations. The usual practice is to
define a smooth, differentiable function that best fits the data, and round off
answers to the appropriate unit. Henceforth we shall assume that functions are
defined and differentiable on a n appropriate domain and that

marginal (. . .) means the derivative of (. . .)

We now define and examine a few of the more important issues and func-
tions occurring in economics and business. Until further notice, x 2 0 stands for
the number of units of whatever is being produced (or sold, manufacturcd,
mined, and so forth). Of course, the following discussion, being elementary,
makes several simplifying assumptions to avoid complex issues that would only
obscure the role played by the derivative. In particular we avoid complexities
involving endpoint extremes for most functions.

MAXIMIZING PROFIT
Dehote by C ( x ) the cost of producing x units and by R ( x ) the reuenue resulting
from the sale (or rental, lease, whatever) of these items. Then, in general,
P(x) = R(x) - C(x)
is the projt function, which normally is to be maximized. As we know, to
maximize P ( x ) we find
P y x ) = R(x) - C(x)
set P ( x ) = 0, and look for relative maxima. This leads to the equation
R(x) = C(x)
which signifies the classic necessary economic criterion for maximal profit:

margznal reuenue equals margznal cost.

Graphically, a typical situation is given in Fig. 5.14. Profit is the directed


vertical distance between the revenue and cost curves, and is a maximum where
) this distance is greatest.
Now C ( x ) is normally composed of fixed (nonproduction, overhead) costs Go
pius variable (production) costs C,(x). Hence
X

I
P ( x ) = R ( x ) - [CO + CI(X)l
so
Fig. 5.14
P y x ) = R(x) - C1(x)
This tells us that fixed costs do not enter into the profit maximizing equation, so
actually marginal revenue wzll equal marginal production cost. This means ideally that
a n increase in overhead costs does not affect the production level decision; it
simply means that total profit is decreased by that amount. In Fig. 5.15 we see
several different situations, differing only in fixed costs. Optimal production
level xo remains the same in each case, although with fixed costs eo,
the business
operates at a loss a t all production levels. There xo minimizes the loss. Often
when making business decisions, it is useful to decompose C,(x) further when
possible into component parts, such as material costs CM(x)and labor costs
C,(x).
Fig. 5.15
164 CHAPTER 5: USING DERIVATIVES

COMPETITION VERSUS MONOPOLY


To examine revenue R ( x ) more closely, we need to know how R ( x ) relates to
price, which in turn depends upon how much the consumers are willing to pay
for the items being produced and how the number of items produced affects the
price. As we all know, an increase in price normally leads to a decrease in
demand. That is, at a higher price, fewer items can be sold.
If the producer does not sell enough to affect the quantity demanded, as is
the case when the producer accounts for only a small part of the market in that
product, the demand and price are out of the producers control; so the price is
assumed to be constant, independent of x. Economists normally refer to this as
perfect competition. Letting p be the price, we know that
R ( x ) = px
so in a perfect competition situation, the equation R(x) = C(x) reduces to
C(X) = p
Hence to maximize profit, we find the production level x for which the
marginal cost equals the price. This is the production level where the cost of
producing one more item is approximately equal to the selling price of that
item.
The opposite, monopoly situation, occurs when the producer sets the price
p ( x ) . However, even though the price is under control, the demand usually is
not, so if the price is too high, the number of units the monopolist will be able to
sell will be too low. Under the reasonable assumption that the producer wishes
to sell all that is produced, the question of maximizing profit reduces to setting
a production level x so that the resulting pricep(x), at which all x items are sold,
p = plKC yields a maximum profit
P(x) = xp(x) - C ( x )
The demand-price function is often assumed to be linear with negative slope
j - - - s * *-*_ L-
within the bounds provided by the situation. Slope is negative because as x
~

increases the price must be lowered to sell the x units produced (see Fig. 5.16).
This general discussion not only makes clear the value of the derivative as a
x = n u m b e r of unity
conceptual tool in understanding fundamental economic relationships but also
Fig. 5.16 provides a ready means for dealing with specific situations.

Example I (Monopoly Situation) Suppose a large coal mining firm produces q carloads of
coal per day at a daily cost of C ( q ) = g3 - 150q2 + 5800q + 500. If it can sell
q carloads at a price ofp(q) = 1000 - 6q per carload, how many carloads per
day should it produce to ( a ) maximize revenue, ( b ) minimize marginal cost,
(c) maximize profit? ( d ) What is maximum profit? (The firm is in an area
where alternative fuel sources are scarce.)

Solution ( a ) The revenue R(q) = (1000 - 6g)q is maximized when


R(q) = 1000 - 129 = 0 or when q = 83i carloads
(6) The marginal cost is C(q) = 3 q 2 - 300q + 5800. C(q) is minimized when
C(q) = 6q - 300 = 0 or when q = 50 carloads. We know this is a relative
minimum since C(q) = 6 > 0 for all q. [Note that it does not usually make
sense to ask when costs are a minimum since in general C ( q )will be a strictly
increasing function. A quantity more appropriate to minimize is the
SECTION 5.4: USING THE DERIVATIVE 165

approximate cost of producing the next carload, which, as we remarked


before, is approximately equal to C(q) for q large enough.]
(c) The variable pricep(q) = 1000 - 6q indicates this is a monopoly situation.
We have profit:
P(q) = (1OOOq - 6q2) - (q3 - 150q2 + 5800q + 500)
so P(q) = 1000 - 127 - 3q2 + 300g - 5800 = 0
when
q2 - 96q + 1600 = 0
or when
q 21.5 and 74.5
Since P(q) = -6q + >
288, with P(21.5) 0 and P(74.5) <
0, we know
that maximum profit is achieved when about 74.5 carloads per day are
produced. At this production level, the price per carload is $553.
( d ) The profit is P(74.5) = $41,198 - $13,556 = $27,642. 0

Example 2 (Competitive Situation) Suppose the above coal producer is not a monopolist
but faces a purely competitive situation with the going price per carload
constant at $553 and other conditions as in Example 1. What production level
yields maximum profit? (Alternative fuel sources are readily available.)
Solution Setting marginal cost equal to $553, we have
3q2 - 300q + 5800 = 553
so
100 t d3004
4= =: 77.4, 22.6
2
with q z 77.4 yielding the maximum profit since p (77.4) < 0. 0

Note that it pays to produce a few more carloads per day in the competitive
situation, reflecting the ability to sell more coal at $553 than before.
Progress Test 1 is the base for most of the discussion that follows, so you are
urged to work it through in complete detail.

PROGRESS TEST 1 APARTMENT RENTAL AND THE EFFECT OF COLLEGE STI-DENTS

X Part 1 (Lower Demand) An owner of a 120-unit apartment complex


knows from experience that every unit can be rented a t $200/month but that
each $10 increase in rent causes a loss of four tenants.
1. Letting x = the number of units rented, calculate a linear price functionp(x)
that describes this demand. [Hint:Assume p( 120) = 200, note that Ap = 10
for Ax = -4, and use the fact that p ( x ) is linear.]
2. Calculate a revenue function R ( x ) for this situation.
3. Determine the
( u ) Number of units to be rented to maximize revenue.
( b ) Rental price at this point.
(c) Total revenue at this point.
4. Given that the monthly cost of maintaining a rented unit is $60, the cost of
maintaining a vacant unit is almost as high at $40 (because the landlord
must pay utilities on unrented units), and the fixed monthly costs are
$1 7,000, determine the resulting cost function C ( x ) .
166 CHAPTER 5 : USING DERIVATIVES

5. ( u ) Determine the profit function P ( x ) and the number of units to rent to


achieve maximum profit.
( b ) What is the maximum profit?
(c) What is the rental rate?
Part 2 (Higher Demand) Now suppose that a college in the neigh-
borhood expands enrollment and demand for the apartments increases so that
all 120 units can be rented at $240/month and that each $10 rent increase
causes only two vacancies.
1. Calculate the corresponding price function p 2 ( x ) .
2. Find the new revenue function R,(x).
3 . Determine the
(u) Number of units to rent to achieve maximum revenue.
(6) Rental price at this point.
(c) Total revenue a t this point.
4. Given the same cost function as before, determine the
( u ) New profit function P2(x).
( b ) Number of units to rent to maximize profit.
(c) Maximum profit.
( d ) New rental rate.

DISCUSSION OF APARTMENT RENTAL


X Although the figures in Progress Test 1 were chosen to exaggerate an effect
slightly (especially regarding demand), the point of maximum profit is often
less than full occupancy. It can pay to have vacant apartments, because the
demand situation can change after the buildings are constructed, so that, in
effect, the buildings become too big. In the lower-demand situation the profit
a t full occupancy is P(120) = (480)(120) - $( 120)2 - 2 1,800 = - 200, a loss of
$200. Furthermore, it can happen, as in the preceding examples, that a sharp
increase in demand (perhaps even leading to a housing shortage) can lead to a
shift downward in the optimal operating level. This is the sort of thing that can
lead to six students sharing a three-room apartment a t exorbitant cost.
For the remainder of this discussion we shall assume apartment demand to
be at the lower level of part 1. Hence
p ( x ) = 500 - (5/2)x and R(x) = 500x - (5/2)x2
We also assume that costs are as before, and thus we obtain the following profit:

P ( x ) = 500x - -x2 +
- ( 2 0 ~ 21,800)
( 25 ,

= 4 8 0 ~- -5x 2 - 21,800
2
I n particular, then, our starting point is an occupancy level of x = 96, a rental
rate of $260, and a profit of $1240. Finally, for simplicity, we shall assume that
all quantities are given per month.

EFFECTS OF EXCISE VEHSITS REAL ESTATE TAXES


ON LANDLORD .\NIB TENANTS

x Suppose now the local government decides to levy a 10 percent excise tax on
all rentals above $200. The landlord must now recompute the cost function,
adding in the excise tax:

+
C,(X) = C ( X )
10

~ - + 21,800
X2
=5 0-
4
SECTION 5.4: USING THE DERIVATIVE 167

Before Tax After Tax


Fig. 5.17

1 The new profit function becomes


5 X2
pt(x) = (500x - -xz)
2 - (501 - -
4 -+ 21,800
and, setting Pt(x) = 0, we see that maximum profit of $700 occurs when
x = 100 at a rental rate of p ( l O 0 ) = $250; see Fig. 5.17. Here the effect of the
excise tax is to change C(x) to Ct(x), which moves the point of maximum profit
to the right, from x =96 to x = 100. The net return to the government is

(100) [ 1 ( 2 5 0 - 200)] = $500


10
and the loss in profit to the landlord is $540. From the tenants point of view,
there are four more tenants and the monthly rental rate drops from $260 to
$250, a net savings due to the tax.
From the landlords point of view, it would have been preferable by $40 for
the government to have raised the $500 tax revenue through a $500 increase in
real estate tax. This would simply add to fixed costs and decrease profit by that
amount, but not change anything else (see Fig. 5.18).
Quite clearly, by adjusting tax type and rate, the government is in a position
to influence the housing situation considerably. By raising the excise tax to 20
percent of the rent over $200, the cost, hence profit, functions change so that the
occupancy level that yields maximum profit becomes 105 units and the rental
rate $237.50. The total tax revenue increases to $787.50, and the landlords
profit drops to $250.
An interesting (but algebraically complicated) exercise is to calculate the
excise tax rate that yields maximum tax revenue.

96 120

Fig. 5.18
168 CHAPTER 5: USING DERIVATIVES

THE EFFECT OF SC-RYIDIES ON LANDLORD AND TEKAYTS


x For the following discussion suppose we are at the starting point again with
no excise tax, but the local government raises its real estate tax rate, so the
increased cost to the landlord is $1000, bringing fixed costs to $22,800and
lowering total profit to $240.We compare two types of subsidies.
Landlord Tax Rebate: The landlord, claiming his $240 profit to be insufficient
return on his investment, petitions the government for a tax rebate based upon
the number of vacancies in the apartment complex. In particular, noting his
current 24 vacancies as grounds for tax relief, he asks for and receives a n
agreement guaranteeing a rebate of $20 for each vacancy. The reasoning in the
tax office is that this will lower the $1000 increase by about (24)(20) = $480
and increase the landlords profit by that amount.
However, what the tax office does not take into account is that the demand
for apartments is not affected, whereas the landlords revenue and profit
functions are. H e will readjust his rental and occupancy rates accordingly so as
to maximize profit. The cost function, including the $1000 real estate tax
+
increase, is 20x 22,800.
The revenue function increases from

500x - -5x 2
2
to
( 5 0 0 ~- 2 ~ 2
5 ,
+ 20(120 - X)
making the new, subsidized, profit function

P,(x) = 500~
- -x2
( 25 ,
+ 20(120 - X) - + 22,800)
(20~

= 460~ 5
- - x 2 - 20,400
2
Then Ps(x)= 460 - 5 x = 0, which implies that only x = 92 units should be
rented. The rental rate is

500 - -
5(92) = $270
p(92) 2
and the total rebate from local government is (20)(28) = $560,so only $440
of the $1000 tax increase goes to the city. Furthermore, the new profit is
P,(92) = $760,which is $520 more than before the rebate.
What is the net result of thzs subsidy? The city loses $560 in tax revenue, the
landlord gains only $520 in profit, but four fewer units are rented, and the rent
for the remaining 92 tenants goes up by $10, meaning that the tenants end up
paying a n additional total of $920.This is obviously a n inefficient and unfair
sort of subsidy.

Rent Subsidy: Beginning with the same conditions, including the new $1000
real estate tax increase, suppose a federal rent subsidy program is begun.
Although most such apply to lower-income families and are a function of
income, we will simplify matters by assuming that all tenants get a $20 rent
subsidy. This affects the demand-price situation as follows.
Assuming that economic, housing, and population conditions are un-
changed, the linear demand-price function has the same slope ( - 5/2) because
a $10 increase in rent still drives out four tenants. With the $20 subsidy, the
landlord can now rent all 120 units at $220 (rather than $200 as before). Let
p , denote the new rent linear subsidy price function since p , ( x ) has the form
SECTION 5.4: USING THE DERIVATIVE 169

-5
p,(x) = - x
2
+b
with p,(120) = 220 = s ( 1 2 0 )
2
+b
5
we conclude that p , ( ~ )= 520 - -2
2
It is your turn to compute the other effects of the subsidy in Progress Test 2.

PROGRESS TEST 2

1. Determine the new rent subsidy profit function PT(x). 5. What is the net amount each tenant must pay after
2. Determine the number of units rented to maximize the subsidy?
profit. 6. Compute the total net subsidy to tenants and com-
3. Determine the new profit. pare this with the increase in profit to the landlord.
4. Compute the amount the landlord received for each
unit rented.

X RENT SVBSlDY AND BENT CONTROL


As a direct consequence of the rent subsidy, $1000 went to the tenants (a $10
savings per tenant), and the landlords profit increased by $1960, almost double
that amount. Thus to call this a rent subsidy is somewhat misleading. (The
increase in profit makes room for an additional rise in real estate taxes of
perhaps $1000. This would leave the landlord with $1200 profit and make the
rent subsidy a form of federal revenue sharing.) However, four more units are
rented than before. Because the supply-demand situation was altered by the
$20 subsidy, the landlord was able to raise rents from $260 to $270, meaning
that the $20 subsidy only saved $10 for each tenant.
This naturally raises the issue of whether the supply-demand situation
should be controlled in some way. For example, suppose the rent subsidy is
coupled with a rent freeze on the landlord, so the $20 goes directly toward the
lowering of the rent for the 96 tenants, from $260 to $240. This lower net rent,
since demand is still the same, results in attracting more potential tenants. As
far as the potential tenants are concerned, the rental price is now $240, so that
p ( x ) = 240 = 500 -(5/2)x, which implies x = 104. This means that eight
additional tenants are willing to move in. However, with the rent freeze, the
landlord receives $260 per unit, making the total revenue (104)(260) =
$27,040, while costs amount to (20)(104) + 22,800 = $24,880. Hence profit is
27,040 - 24,880 = $2160.
Again, the landlords profit rises almost as much as without the freeze.
Furthermore, the total rent subsidy of (20)(104) = $2080 goes toward lowering
the rent of the 104 tenants. Hence the landlord and the tenants both have about
$2000 more to spend even though the net subsidy was only $2080. What makes
the difference between this and the uncontrolled rent subsidy is the increased
number of tenants. The net effect of the controls is to make the apartments
affordable for eight more tenants, who then contribute, as do all the others, to
a share of the profit. This situation contrasts sharply with the tax rebate scheme,
which encouraged lower occupancy and higher rents. Note, however, that if the
landlords costs were rising more rapidly than revenue (260x) beyond the
x = 96 point, the additional eight tenants who were able to afford rental at a
price of 240 (to them, 260 to the landlord) would not be admitted if the

landlord wanted to maximize profit. This same sort of situation can lead to
shortages when prices are frozen: Producers strike, refusing to produce beyond
the maximal profit point (although the word strike is seldom used).
170 CHAPTER 5: USING DERIVATIVES

Remarks on Our SimplzJcations: Besides being rigged to make calculations


simple at each turn, the previous discussion assumes a simple linear price-
d e m a n d function. This makes assumptions a b o u t the willingness of tenants to
move in response to rent changes. These elasticity assumptions are not
entirely realistic. Also, we ignored the impact of taxes and subsidies on d e m a n d
i n the larger rental market. Since d e m a n d a t a particular apartment complex
depends u p o n the larger market, some of our assumptions regarding constancy
of d e m a n d are also not entirely realistic. Nonetheless, the analysis overall is
reasonable enough to reflect the ways that derivatives c a n b e used to analyze
the effect of economic policies. Furthermore, a more sophisticated analysis
based o n a more complicated mathematical model c a n account for those
phenomena ignored i n the above discussion. Incidentally, we shall put the
apartment complex up for sale in Sec. 8.1.

SECTION 5.4 EXERCISES

1. Suppose a firm can sell any number of its products at some constant. The government wishes to impose an
+
$12 apiece, but it costs 0.001x3 0 . 3 ~ + 50 to produce x excise tax on this product. However, if the tax rate t is
i tem s. too high, the quantity sold will fall, and thus tax reve-
( u ) Calculate the revenue function for this situation. nues will diminish.
( 6 ) Calculate the profit function. ( a ) Find the quantity saleable Qt as a function of tax
(c) Determine which production level yields maximum rate t.
profit and the actual profit at that level. (6) Determine the tax revenue function R.
( d ) Determine the level at which reuenue is maximized ( 6 ) At which tax rate t will revenue be maximized?

(be careful!). (Note that your answer will involve p and k.)
2. Repeat Exercise l(a) to (c) for a new selling price of $15. 9. (Average Cost) In certain situations where profit is not
3. A company charges $100 per item for the first 200 items, important, it is still important to minimize cost per item.
but in an effort to encourage larger orders and increase (A production run of free samples, a government press
revenue it makes a special offer to lower the price by run of tax forms, a run of license plates at the prison, and
$0.25 per each item of the entire order for each item so on.) This average cost is C ( x ) / x .
ordered beyond 200. What size order will yield maxi- ( u ) Analyze average cost for C ( x ) = 5x + 4.
mum revenue for the company? (This is the maximum (b) Analyze average cost for any linear cost function
size order the company would accept under the special C(x) = ax + b.
offer.) ( c ) What is the effect of large fixed costs b?
4. ( u ) Kepeat Exercise 3 but with the price of only those ( d ) Find the point of minimum average cost for
items beyond 200 lowered by $0.25. + +
C ( x ) = 0 . 0 0 0 2 5 ~ ~x 1000.
(6) Repeat part ( u ) with the price lowered by k dollars. ( e ) Note that the average cost is the slope of the line
(c) When k = $0.50 in part ( b ) , the maximum-size order from the origin out to the cost function. Graph the
turns out to be 200. Explain. Use a graph. function in part (c) accurately and analyze the slope of
5. Suppose that a wholesaler has purchased 180 identical this line.
television sets for $150 apiece. She finds she can sell them 10. A monopolistic enterprise has a price (demand) function
all at a price of $160, but for each additional dollar p ( q ) = a - bq and a cost function C(q) = A q 2 +
added to the price, one fewer set can be sold. Bq + C, where a, b, A , B , C are positive constants with
( a ) How many sets should she sell to maximize sales u > B. Given that the company will attempt to maxi-
revenue? mize net profit (after taxes), what rate r should the
( b ) Give the profit function for this situation. government charge on each unit q to maximize tax
(c) How many should she sell to maximize profit? T = rq?
( d ) Compare the profit at the point of maximum reve- 11. Given that the company in Exercise 10 will ignore taxes
nue with the maximum profit. in its cost function and production level decision and
6. ( a ) Suppose you were hired to sell the television sets of merely pass them on to consumers, what should be the
Exercise 5 at a commission of 10 percent. How many governments optimal tax rate? [Note that the tax will
would you sell to maximize your commission? change the price-demand function by adding rq to p ( 4 )
( 6 ) What is your maximum commission? without changing demand.]
7. Kepeat Exercise 6 with a 25 percent commission. Ex- 12. Bill Ashley estimates that his August 10 peach crop will
plain your decision. be 2000 bushels, with a profit of $2/bushel. H e also
8. Let Q represent quantity of a product sold. Suppose Q is estimates that the crop will grow by 80 bushels/day and,
related to its price p by Q = k [ 4 - ( l / p ) ] , where k is because of increasing market glut, his profit per bushel
SECTION 5.4: USING THE DERIVATIVE 171

drops by 16 cmts/day. Given a prohibitive frost threat ( a ) Daily price drop that would justify picking on Au-
after 30 days, when should he pick the crop to maximize gust 10.
profit? ( b ) Daily price drop that would justify picking on Sep-
13. Given the same basic conditions as in Exercise 12, except tember 10.
variable marketing estimates, determine the

CHAPTER EXERCISES

R e v i e w Exercises
In Exercises 1-20 determine ( a ) the maximum and ( b ) the minimum values of the given function on the given interval
(zf these values exist).

x3 x2
1. f ( x ) = - + -
3 2
+ x + 3 on [ 1 , 4 ]
3. f ( x ) = - x- 3- -
3
7x2
2
18x - 1 on [-3,101 4. f ( x ) = x ( x 2 + 9) on [ 0 , 5 ]
x4
5. g ( x ) = - - 3x3 + 6x2 - 4 x on [-1, 31
2
X2
7 . h ( x ) = -on [-1,0]
2x +1
9. y = x - f i o n LO, + m ) 10. f ( x ) = x2 - 2x + 5 on [-3,4]
1 3
11. f ( x ) = x 3 - - + + x - 4 o n [ 1 , 2 ] 12. f ( x ) = -x4 - -x2 on [I, 21
4x2 4 2
13. f ( x ) = x413 - 4x113 on [ -5,Ol 14. f ( x ) = x 3 - x 2 + x - 1 on [ 1 , 4 ]
4x
15. f ( x ) =- on [-3,4] 16. f ( x )
( x 2 4) +
17. f ( x ) =*on
x + l
[-3,4] 18. f ( x ) = ( x + 2 ) ( x - 2 ) 3 on [ - 2,0]
19. 20. f ( x ) = ( 8 - x ) ~ 'on
~ [0, 161

21. T h e product of two numbers is 16. Determine two such ( a ) Determine in which direction the particle is moving
numbers so that the square of one plus the cube of the when t = 0, t = 3.
other is as small as possible. ( b ) Is the particle slowing down or speeding u p when
22. Repeat Exercise 21 with the square of one plus the cube t = 3?
of the other as large as possible. 28. A particle is moving along a straight line with its posi-
23. A fence 6 m high is 2 m from the side of a house. What is tion at time t given by
the length of the shortest ladder that will rest on the
t-2
level ground outside the fence and lean on the side of the f ( t ) = 2t2 3
~

+
house?
24. What are the dimensions of the base of a rectangular box ( a ) Determine in which direction the particle is moving
of greatest volume that can be constructed from 100 in.2 when t = 0, t = 2.
of cardboard if the base is to be twice as long as it is ( b ) Is the particle slowing down or speeding up when
wide? t = 2?
25. At noon, ship A , steaming east at 16 mph, is due south of 29. Describe the motion of the particle whose position at
ship B , which is steaming south at 12 mph. They are time t is given by f ( t ) = 2t3 3t2 - 361+ 1.+
100mi apart at noon. At what time are they closest 30. Describe the motion of the particle whose position at
together? time t is given b y f ( t ) = 2t3 - 12t2 - 30t + 2.
26. A metal cup in the shape of a right circular cylinder is to 31. A conical tank, full of water, is 12 ft high and 20 ft in
hold exactly 100 i n 3 of liquid. What are the dimensions diameter at the top. If water is let out at the bottom at
of such a cup which would be the least expensive to the rate of 4 ft3/min, find the rate of change of the depth
make? of the water 10 min after the water starts running out.
27. A particle is moving along a straight line with the 32. A man starts walking eastward at 5 ft/s from a point A .
position at time t given by Ten seconds later a man starts walking west at 2 ft/s
from a point B, 3000 ft north of A . How fast are they
2t - 3
f (9 = separating 2 min after the second man starts?
172 CHAPTER 5: USING DERIVATIVES

33. A light is on the ground 40 ft from a building. A man How fast is the radius of the surface increasing a t the
6 f t tall walks from the light toward the building a t instant when the depth is 4 m?
6 ft/s. How rapidly is his shadow on the building grow- 37. Repeat Exercise 36, but determine the rate a t which the
ing shorter when he is 20 ft from the building? height h is increasing. Is either Exercise 36 or 37 a
34. At the rate of 8 ft3/min, water flows into a n initially calculus problem?
empty conical tank that is 10 ft high and 10 ft in diame- 38. A man 53 ft tall walks a t the rate of 3 ft/s toward a
ter at the top. How fast is the water level rising after street light that is 18 ft above the ground. At what rate is
4 min? his shadow changing when he is 8 ft from the base of the
- Let the cost of selling x items be S ( x ) , so the average
*#a. light?
marketing cost per item is S(x)/x. Show that the average 39. Sand is pouring from a spout a t the rate of 3 m3/min
marketing cost per item is minimized whcn the marginal and forming a conical pile in such a way that the radius
sales cost equals the average sales cost. a t the base is always one-third the altitude. How fast is
36. A tank in the shape of a right circular cylinder of radius the altitude of the pile increasing at the instant when the
6 m is being filled with water at the rate of 1/3 m3/s. radius of the base is 2 m ?

A particle moving along a straight line has its position at time t given by s ( t ) . In Exercises 40 to 44 giue a detailed description
o f the motion o f the particle:
t3 t2
40. s ( t ) = - - - - 6t
3 2
41. s(t) = t 4 - 32t + 48
t
43. s(t) = t3 + 3t2 + 3t + 2 43. s ( t ) = ~

t2 +5
4
44. s(t) = t4 - - t 3
3

Miscellaneous Exercises

1. ( a ) We are given a 200-111 stone wall and 400m of ( a ) How many urns would we produce t o maximize
fencing. Using the stone wall as one side of the pasture, revenue?
what are the dimensions and area of the largest rectan- (6) How many urns would we produce t o maximize

-
gular pasture that can be enclosed? profit?
(6) Repeat part ( a ) , but with the rectangular condi- ( c ) How many urns would we produce if we also had to
tion weakened into the trapezoidal condition indi- add to our cost the commissions paid in part ( b ) of
cated in the accompanying diagram. Exercise 3?
200 ( d ) Would the production level need to be changed from
part ( c ) if the commission rate were lowered from 20 to
10 percent? Explain your answer.
5 . A manufacturer will make rectangular open-topped
boxes from square sheets of cardboard 1 m on a side by
I I cutting out squares from each corner and folding up the
sides.
(c) What, in terms of area gained, is the value of the ( a ) What are the dimensions of such boxes having max-
weaker trapezoidal condition over the rectangular imum volume?
condition? ( b ) In a n effort to utilize waste, the manufacturer de-
2. A wire of length L is cut into two pieces, one of which is cides to make similar boxes from the squares cut out, in
bent to form a circle and the other a n equilateral tri- such a way that the total volume of the five boxes
angle. constructed from a single sheet is a maximum and the
( a ) How should the wire be cut so that the area enclosed smaller boxes are twice as tall as they are wide. What are
by the two pieces will be a maximum? the dimensions of each type of box? (Approximate to
( b ) Is there a solution to the corresponding minimum two decimal places.)
problem? ( c ) Compare the total volume in parts ( a ) and ( b ) .
3. ( u ) Find the point of maximum sales commissions given 6. Suppose that a seller asks $100 for a n item and a buyer
that x dozen urns can be sold at 50 - x dollars apiece, offers $80. O n e way for the seller to split the difference
and that the commission rate is 10 percent. and settle on a price p is to maximize the product
( b ) Is this point different if the commission rate is 20 (100 - p ) ( p - 80).
percent? ( a ) Find the price p that maximizes this product.
4. Suppose we are producing the urns sold in Exercise 3 ( b ) Discuss the result i n ( a ) graphically and determine
and our cost function is C ( x ) = x 3 + +
2x 100 (x is whether it really splits the difference.
dozens of urns).
SECTION 5.4: USING THE DERIVATIVE 173

7 . Repeat Exercise 6(a) in the general case where the sell- (m + n)p3 - (mp? + 2mnb,p, + npi)p + 1O,OOO
ers price is p, and the buyers offer is p,.
where p , is your last offer and p a is managements last
8. Suppose the unions last wage request was for $10/h,
offer. How would you pick m and n under the restriction
managements last offer was $7/h, and negotiations are
that rn and n cannot differ by more than 2?
stalemated. An arbitrator comes in with a settlement, as
9. What are the dimensions of the rectangular pasture that
she puts it, to maximize the function 3p3 -
can be enclosed at a fixed cost C, given that the costs per
576p + 10,000,
unit length of the four sides are, respectively, a, b, c, and
( a ) One side claims that this settlement skews the aver-
d? Examine this situation for underlying regularities.
age in favor of its opposite. Which side makes the claim?
( 6 ) Suppose you are in the union and the arbitrator
offers to let you pick whole numbers rn and n to maxi-
mize the formula

SELF-TEST +
Chapter 6

175
176 CHAPTER 6: INTEGRATION

Seetion 6. I: Antidiff erentiation

Objectives: AN INTRODUCTION TO ANTIDIFPERENTIATION


1. Solve simple differential Given a functionf, we can in most cases routinely determine its derivative by
equations, including those applying the appropriate formula.
involving acceleration and velocity Suppose now we consider the inverse problem: Given a functionf, determine
functions. a function F whose derivative is$ More precisely, we have:
2. Use substitution and the Power
Rule to determine antiderivatives.

6.1.1 Definition A function F is called an antiderivative off on the interval I , if


F(x) =f(x) for all x in I. Determining such functions F is called antidfferen-
tiation.

Consider, for example, the function


f ( x ) = 3x2
+
defined on ( - co, m). Since D,(x3) = 3x2, we know F ( x ) = x 3 qualifies as an
antiderivative of f(x) = 3x2. But so do

It is fairly obvious that any function of the form


F ( x ) = x3 +C
when C is any real number, is an antiderivative of f(x) = 3x2. Using that the
derivative of a constant is zero, we can conclude that a similar statement holds
for antiderivatives in general.

6.1.2 Theorem If F is a n antiderivative of a function f on an interval I and if C


+
is any constant, then F ( x ) C is also an antiderivative off on I.

From a geometric point of view this fact is also quite plausible, since the
+
graph of F ( x ) C is vertically parallel to that of F ( x ) in the sense that lines
tangent to any two antiderivatives at points with the same x coordinate must be
parallel. (See Fig. 6.1.)
Furthermore, as a consequence of the Mean-Value Theorem in Chap. 5, we
showed that all antiderivatives off(x) differ by a constant. This converse to
(6.1.2) can be restated as follows:

Theorem If G ( x ) and F(x) are both antiderivatives off ( x ) on Z,then there is


a constant C such that
G(x) = F(x) +C for all x in I
Thus, for example, even thoughf(x) = 3 x 2 has infinitely many antideriva-
tives, we can be assured they are all of the form x 3 + C for C a real number.

SOME NOTATION AND BASIC A&-TIDERIV-ATIVERULES


We use the notation

to indicate the process of antidifferentiating the functionj Thus we can write

Fig. 6.1
J3xzd.x = x3 +c
SECTION 6.1: ANTIDIFFERENTIATION 177

to denote that x3 +
C is a typical antiderivative o f f ( x ) = 3x2.
In effect, J(-) dx indicates the operation of determining a n antiderivative in
the same way that

all indicated the reverse operation of determining the derivative.


In what follows, when an interval is not mentioned in a n antiderivative
statement, it is tacitly assumed that the functionfand the antiderivative F are
both defined on some interval with, of course, F' =f on that interval.

Example 1 Determine a typical antiderivative in each of the following:


( a ) Jx2 dx ( h ) J4x8 dx (c) J f i d x

Solutions (a) s x 2
x3
dx = -
3
+ C because D,
4x9
( 6 ) ~ 4 x ' d x = - + C because D,
9

(c) Here it is helpful to write &asx1/2. Then s fi sdx = x1l2 dx =


x3/2

s& 3/2 -I- "


~

or dx = (2/3)fl+ C.Here

As Example 1 illustrates, antidifferentiation often involves reverse use of


derivative formulas, combined with a bit of creative manipulation. To help in
this process we shall state some derivative rules in their antiderivative form:

6. I .4 (Basic Antiderivative Rules)


(a) Jkf(x) dx = k J f ( x ) dx for k a constant.

(c) (Power Rule) If r # -1 is a rational number, then

Each of these result from applying the corresponding derivative rule. For
example, the derivative of the right-hand side of ( b ) is the sum of the deriva-
tives:
(1) D,[Jf(X)dx + Jd-4dxl = D,[Sf(x) dxl + D,[Sg(x) dxl
But, by definition of the antiderivative,

D,[Sf(X) dxl = f(x) and D,[Sg(x) dxl = g ( x )


so the right-hand side of (1) isf(x) + g(x). However, again by definition of the
antiderivative, this is the derivative of the left side of ( b ) , proving (b).
Notice that ( a ) means that constants can be pulled in front of an antideriv-
ative sign, as in

s 3x2 dx = 3 s x 2 dx = 3 [$+ C] (by the Power Rule)

= x3 +3C
Since the constant C added to an antiderivative represents any arbitrary real
178 CHAPTER 6: INTEGRATION

number, so does 3C. It is customary to write the arbitrary constant for a typical
antiderivative simply as C. In fact, we normally do not write the constant until
we reach the last step of the antiderivative process as in the next example.

Example 2 Evaluate: ( u ) J (4x5 + 9x) dx (b) s ( 3 fi + -$) dt


Solutions Check the result by differentiation.

( u ) J(4x5 + 9x) dx = J4x5 dx + J9x dx by [6.1.4(b)]

=4
J x 5 dx +9s x dx by [6.1.4(u)]

x6
= 4-+
6
9-
x2
2
+c by [6.1.4(c)]

= -2x 6
3
+ -9
9 +c
2

= 3st1/2dt+ 2Jtr2dt
t3/2 t-1
= 3-+
3/2
2-
-1
+C
2 + C
= 2t3/2 - -
t

= 3t1/2 - 2( - l)tP2 0

The definition of the antiderivative provides an unusual opportunity for


checking potential answers. As usual, the form in which we leave our answer
depends upon what, if anything, we plan to do with it. Generally, we shall leave
our answers in forms involving positive exponents and simplified fractions.
You may have wondered why the differential appears in antiderivative
statements. In fact, the underlined part of
n

f ( x ) dx = F ( x ) + C
.I--
is the differential of F ( x ) because
dF(x) = F(x) dx = f ( x ) dx
Hence, taking the antiderivative of both sides, we have

6.1.5 JdF(x) = F(x) + C

Although later we shall see theoretical reasons for including differentials in our
notation, at this point it is mainly a convenience whose value will become more
apparent as we go along.
SECTION 6.1: ANTIDIFFERENTIATION 179

PROGRESS TEST I

Evaluate 1 to 4. Check your answer.

1. 2. J(3t2 + 4fi) dt

3. J 6Z&,:li3 dt (break into a sum) 4. J ( w 2 + 1)2dw (expand first)

5. Show that variables cannot be pulled in front of


antiderivative signs, by showing Jx3 dx # xJx2 dx.
(Two antiderivatives are guaranteed to be unequal if
their derivatives are unequal.)
--

APPLICATIOYS -DIFFERENTIAL EQITATIONS


A dgerential equation is a n equation involving unknown functions and their
derivatives. To solve a differential equation is to determine a function or class of
functions that satisfy the given equation. Usually the solution of a differential
equation involves antidifferentiation. For example,

is a differential equation that we can rewrite as dy = 3x2dx. Its solution is


determined by antidifferentiation [applying (6.1.5)]:

Y
y = Jdy = J 3 x 2 d x = x 3 +C
In this case the solution is a n infinite class of functions, one for each real
number C. If we are given a n additional condition, we can usually specify a
particular function from among all those satisfying the original equation. If we
are told, for example, that y = 2 when x = 1, then we know that
2=13+c
so C = 1. Thus y = x 3 + 1 is the unique solution of the differential equation
dy/dx = 3 x 2 under the additional condition thaty = 2 when x = I . Geometri-
cally, functions satisfying dy/dx = 3 x 2 have graphs such as the family of parallel
curves in Fig. 6.2, and the effect of the additional condition is to single out the
wig. 6.2 unique function whose graph (solid curve) passes through (1, 2 ) .

A differential equation arises most frequently as the mathematical expression


of some given fact or relationship. We could arrive a t the same differential
equation just studied by stating in terms of derivatives that the slope of the
graph of the function at the number x is 3x2.
Most commonly, differential equations are used in physics and engineer-
ing-for example, in describing motion with acceleration due to gravity. Recall
that if we measure distance positively in the upward direction, then acceleration
and velocity in the downward direction will be negative. In particular, ignoring
air resistance, the downward acceleration due to gravity on earth is approxi-
mately 32 ft/s2. Since acceleration is the derivative of velocity v, we have a
differential equation

Thus
v ( t ) = -f - 3 2 dt = -32t +C
180 CHAPTER 6: INTEGRATION

Hence, knowing the velocity at a particular time to, (say, t, = 0), we have a
completely determined function that gives the velocity. Commonly, the velocity
at t = 0 is called the initial velocity and is denoted vo, so

6.1.6 v ( t ) = vo - 32t

Example 3 Given that an object is thrown vertically upward a t 96 ft/s, determine the
velocity at t = 2 , t = 3, and t = 4.
Solution By (6.1.6),
v(t) = 96 - 32t
so
~ ( 2=
) 32 ft/s 4 3 ) = 0 ft/S 4 4 ) = -32 ft/S 0

Just as we were able to differentiate more than once, we can antidifferentiate


more than once. Knowing that (instantaneous) velocity is the derivative ds/dt of
the position function, we have the differential equation

ds = v, - 32t
- or ds = (uo - 32t) dt
dt
Hence we have the function giving the height above the ground at time t :
s(t) = S ( u o - 32t)dt

= vot - 16t2 +C
Again, given the znztzalposztzon, that is s(O), usually denoted so, we have the
uniquely determined function

6.1.7 s(t) = so + vot - 16t2

Example 4 Determine the height above the ground after 2 , 3 , 4 s, respectively, of an object
thrown vertically upward from a platform 10 ft above ground at 96 ft/s.
Solution We are given that s(0) = so = 10, so by (6.1.7)
s(t) = 10 + 96t - 16t2
Thus
~ ( 2=) 138 ft 4 3 ) = 154 ft ~ ( 4=) 138 ft 0

We constructed the function in (6.1.7) in two stages, beginning with the


constant gravity function. We could have presented the original problem as a
second-order differential equation:

d2s
6.1.8 -
dt2 = g
(g is the constant gravitational acceleration)

To solve (6.1.8) for a particular function, we need two additional conditions,


one on velocity and one on position-usually given as initial conditions.

Example 5 (The acceleration due to gravity in metric units is 9.8 m/s2.) Suppose an object
is thrown upward from 10 m below ground level at an initial velocity of 50 m/s.
What is the maximum height in meters reached by the object, and what is its
velocity in meters per second when it reaches ground level again?
SECTION 6.1: ANTIDIFFERENTIATION 181

Solution First we must determine its velocity and height functions.

ds = v(t) = J (-9.8) dt = -9.8t


- + C,
dt
) (- 9.8)(0)
NOW~ ( 0= + C, = 50, SO v(t) = 50 - 9.8t. Hence
) l(50
~ ( t= - 9.8t) dt = 50t - 4.9t2 + GI
and
s(0) = c, = -10
so
s ( t ) = 50t - 4.9t2 - 10
By both common sense and the work of Chap. 5, the object reaches maximum
height when ds/dt = u ( t ) = 0.
But 50 - 9.8t = 0 when t z 5.1 s, and
s(5.1) z 117.6 m
The object reaches ground level when
s(t) = 50t - 4.9t2 - 10 = 0
or when

t=
50 t 48
9.8
- 0.2 or 10
The object thus reached ground level the second time 10 s after being thrown
) 50 - (9.8)(10) = -48 m/s. 0
upward. Its velocity is ~ ( 1 0 =

PROGRESS TEST 2

The downward acceleration on the moon is approxi- 4. ( a ) How long would it take a ball hit vertically with
mately 5 ft/s. an initial velocity of 150 ft/s to return to the surface
1. Give a formula for the vertical velocity of an object of the moon?
hurled upward from the surface of the moon with an ( b ) What would its velocity be at that time?
initial velocity of 20 ft/s. 5 . Solve the second-order differential equation
2. Give the height function for this object. d2y/dx2 = 12x + 2 given thaty = 3 when x = 0 and
3. Supposing the greatest initial vertical velocity of a +/dx = 10 when x = 1 .
hit baseball is about 150 ft/s, give the minimum roof
height of a domed baseball park on the moon (so
that pop flies will not hit the roof ).

THE POWER RCJLE FOR PIJNCTIONS;


ELEMENTARY SlTBSTIT~JTION
After some reflection, we can compute
J(2x3 + 5)l06x2dx
+
as follows. We are looking for a function whose derivative is (Zx3 5)I06x2,and
we know that by the Power Rule for Functions (3.2.1), the derivative of
+
(2x3 + 5)%is n(2x3 5)np1(6x2),for n an integer. If we take n = 11, we shall
have solved the problem, provided that we divide through the 11 and add the
constant C:

s ( 2 x 3 + 5)10(6x2)dx =
(2x3 + 5)11
+C
11
182 CHAPTER 6: INTEGRATION

In effect we recognized the original antiderivative to be of the form

J u r o du with u = (2x3 + 5), du = 6x2 dx


From this we concluded that

Antiderivative problems are seldom this neat; more typically, we need to


solve a problem such as
Jx2(2x3 + 5 ) I o dx
However, our procedure remains the same. We must change the form of the
function we are dealing with into one that we recognize as the derivative of
some other function. Often the best way to do this is to make a substitution and
take the differential as follows:
Let u = 2x3 t 5 , so du = 6 x 2 d x
We then compare du with x 2 d x by solving du = 6 x 2 d x for x 2 d x to obtain
x 2 dx = du/6. Thus the original antiderivative problem can now be stated as

= -1- d o d u by [6.1.4(a)]
6
1_
-_ ull
6 11
+C by the Power Rule [6.1.4(c)]

-
(2x3 + 5)11 + C
66

Check: D,
+
~ ( 2 ~ 5110
3
(6x2) + 0 = (2x3 + 5)lox2
66 66
In the preceding problem we actually used the Power Rule for Functions
(3.2.3), which we now state in its antiderivative form.

6.1.9 Power Rule for Functions If u is a differentiable function of x and


r# -1 is a rational number, then
Ur+l
Ju'du = r+l+ C
Most applications of (6.1.9) are of the preceding type: We find a factor of the
form
(expression in x)'
inside the antiderivative sign. If such exists, we try the substitution
u = expression in x

That is, we let u stand for the expression in x inside the parentheses, and compare
du with the factors of the integrand other than u'. We shall succeed i f d u turns out
to be a constant multiple of these other factors.

Example 6 Determine s( ~2
x2 +9
) and check the result.
SECTION 6.1: ANTIDIFFERENTIATION 183

Solution Rewrite the original problem as J(2x2 +


9)-'"xdx. Let u = 2x2 9 [not+
+
( 2 2 9)-1/2!!]. Now du = 4x dx, so x dx = (1/4) du. Thus we can substitute:

= 1
4
-su-1/2 du

1 u1/2
- --
4 1/2
+C by the Power Rule for Functions

+ 9)1/2 + C + 9)-1/2(4x) + 0

Note that when using substitution methods you should always give the
answer in terms of the original variable. The substitution variable should not
appear in your answer.

PROGRESS TEST 3

Using (6.1.9), determine


1. l x ( 1 - 5 2 ) 7 dx 2. J(x + 1)(x2 + 2x + 7)1'3 dx

MORE ON SUBSTITUTION

Example 7 Determine .f(x2 + l)3x3dx.


Solution Letting u = x 2 + 1, we know that du = 2x dx so x3 dx = ( x 2 / 2 ) du. Unfortu-
nately, x 3 dx and du do not differ by a constant factor. However,
u = x2 + 1 implies x 2 = u - 1
and so we can replace the x 2 factor on the right side of
X2
x3dx = -du
2
by u - 1, getting x 3 dx = (u - 1)/2 du. Thus

J(xz + 1)3x3dx = s (q)


u3 du

=-
1 J (u" - u3) du
2
-(-
= 1 us - $) + c
2 5
184 CHAPTER 6 : INTEGRATION

W e hasten t o note that m a n i p u l a t i o n s such as t h a t in Example 7 will not


always solve an antidifferentiation problem. Consider, for example,

SdFZTdX
Here, letting u = xz - 1I , we find du = 2x dr,so w e are missing a factor of x.
We cannot solve this problem by an elementary substitution. We shall devote
Chap. 11 t o solving antidifferentiation problems of this and even more com-
plicated types.

SECTION 6.1 EXERCISES

In Exercises I to 8 j i n d the indicated general antiderivative (check your answer).

1. J(x2 + x)& 2. J ( 3 x - 5) dx 3. Jx-2d.X 4. Jfidx

5. J(&+ 5) dx 7. J ( $ + x ) d x 8. J(1 +L+


6 ) d x
G
9. I n Exercises 1, 3, 5, and 7 determine the unique antide- 10. In Exercises 2, 4, 6, and 8 determine the unique antide-
rivative passing through the points (0, 1 ) (for Exercise l), rivative passing through the points ( - 1 , 8 ) , ( 1 , 3/4),
( - 1 , 3) (for Exercise 3), (1, 2 0 / 3 ) (for Exercise 5), ( - 2, l ) , and (4, 18), respectively.
(2,577 + 3/4) (for Exercise 7), respectively.

In Exercises 11 to 14 solve the second-order dzferential equation using the given information:

1I . d 3 = x ;y
dx2 4 = 3 when x = 2.
= 1 when x = 0, and - 12.
dx
when x = - 1 .
ds = 25 when t = 1 , s(0) = 0. d2s
13. _
d2s - 12t
dt
= 0; -
dt
14. -
dt
+ 6 0 t 4 = 12t2; d.= 0
dt
when t = 0, s ( 1 ) = -100.

15. ( a ) Find the maximum height achieved by an object the bridge in Exercise 16, how long would it take the
that has an initial vertical velocity of 100 ft/s at ground stone to reach the water?
level and a vertical acceleration of - 10 ft/s2 (it is par- 18. Suppose the greatest initial vertical velocity of a hit
tially self-propelled). baseball is about 150ft/s on the planet with gravita-
( 6 ) Find the length of time needed for the object in ( a ) tional acceleration of - 100 ft/s2. (The baseball and bat
to reach the ground again. would be heavier, but the players stronger.)
16. A rock dropped from a bridge hits the water in 4 s . ( a ) How high should be the roof of a domed stadium?
Ignoring air resistance as usual, calculate the height of ( b ) How long would it take the ball to rcturn to the
the bridge above the water. surface after being hit with an initial velocity of 150 ft/s?
17. Suppose you are on a large planet where the acceleration
due to gravity is - 100 ft/s2. From the same height as

In Exercises 19 to 34 determine the indicated antiderivative.

19. J(x5 + 3)7x4dx 20. Jx2(6 - x 3


21. J(x3 + x2 + 1)4(3x2 + 2x) dx 42. J ( x 2 - 2~ + l)(~ - 1 ) dx

23. Jd=(x2 - 1 ) dx 24. s(fi+ +)d~ 25.


dx

29. J(X2 - 1)ZdX 30. J q m d x

(In Exercises 32 to 34 it may be helpful t o refer to Example 7.)


32. ~ ( +~2 1 73~ 5
dx 33. J ( x 2 - 5 ) 5 ~ 3 d ~ 34. J(x + 1)dx
35. Determine the function whose graph has slope mx ( m a
real number) at any x and passes through the point
(0,O).
SECTION 6.2: APPROXIMATING AREAS 185

Section 6.2: Approximating Areas

Objectives: INTHODUCTION
1. Approximate areas of regions Suppose R is a bounded region in a plane enclosed by some curve C, as in
bounded by simple functions using Fig. 6.3. Although we may not have a ready formula that would give us the
rectangle approximations. area of R, we can be reasonably sure that as long as its boundary is not too wild,
it has a well-defined area. In fact, we shall assume this to be the case and
2. Take limits of these
concentrate on determining the area. Later, in (6.5.1), we use the theory devel-
approximations to determine exact
oped to define the notion of area precisely.
areas for linear or quadratic
We can determine the area of a straight-sided (polygonal) region relatively
functions.
easily by decomposing it into disjoint triangles and summing their areas;
3. Understand the relationships however, the problem of determining the area of a region with curved sides is
among the various types of much more challenging. The ancient Greeks, especially Archimedes, used what
approximations and the limits of was called the method of exhaustion. They approximated the curved region
these approximations. using smaller and smaller straight-sided regions-obviously a type of limiting
process. Even though the Greeks achieved remarkable accuracy, particularly in
special cases such as the circle, more than 2000 years elapsed before the essential
idea of limit was sufficiently developed to provide the means for a more
systematic and complete solution to the area problem.
The solution we shall develop evolved slowly through the seventeenth and
eighteenth centuries and was finally put in the form we shall use in the middle
of the nineteenth century by the German mathematician Georg Riemann. This
approach will use rectangles as the approximating figure.
But why study area? Aside from the fact that the area problem is of consider-
able intrinsic interest, it provides the concrete realization of mathematical ideas
of much wider utility-in much the same way as the notion of instantaneous
velocity provided a starting point in our study of instantaneous rates of change
and derivatives. In fact, it will turn out that many problems that on the surface
have nothing to do with area will have solutions that amount to finding the
area of a given region. See p. 189 for further discussion of this issue.

Fig. 6.3

APPROXIMATIONS
To apply our experience with functions and limits, we first assume that the
region R, whose area A we seek, is bounded by the graph of a continuous
functionf(x), the lines x = a and x = b (with a < b), and the x axis. (See Fig.
6.4.) Furthermore, to avoid complications with negative quantities, and to
ensure that the limit representing the area of R turns out to be nonnegative, we
Y assume thatf(x) 2 0 on [a,b ] . These assumptions are not particularly restrictive
l a b since most regions of interest can be broken into subregions of this type.
Fig. 6.4 We partition [a, b] into, say, n subintervals using the points
xo = a, x1,x2,. . . , xnP1,xn = b
and on each subinterval we base a rectangle whose height is determined by
t some value off(x) on that subinterval. Sincef(x) is continuous on [a, b ] , it is
continuous on each closed subinterval [xipl, xi] (1 5 z 5 n), and so by the
Extreme-Value Theorem (4.4.2), f ( x ) attains a minimum value mi and a
maximum value M ion [xi-1, x i ] . Thus we have two rectangles based upon each
such subinterval (see Fig. 6.5), one (shaded) of height mi and the other of
height Mi.
6 Denoting the width of the rectangle by Axi = xi - xi-1, the area of the
Fig. 6.5 shorter rectangle is mi Axi, and the area of the taller rectangle is Mi Axi. If we
186 CHAPTER 6: INTEGRATION

t t

l a b

Lower Sum, .S,(f)

(a)
Fig. 6.6

add the areas of all n shorter rectangles, we obtain an approximation &( f ) that
<
underestimates A ; that is, &(f) A . If we add the areas of all n taller rectangles,
we get an approximation Sn(f ) which overestimates A ; that is, A 5 $(, f ). (See
Fig. 6.6.) We refer to $,( f ) as a lower sum and s,(
f)as an upper sum. Thus,
SnU)IA I $n(f)
Our next step (following Archimedes) is to improve the approximations by
taking narrower and narrower rectangles. In Fig. 6.7, we shade the improve-
ment obtained by replacing one rectangle by two narrower rectangles.
The idea, now, is to form the limit ofsuch lower and upper sums as the widths of the
rectangles approach zero and thus close in on A from both sides:
6.2.1 U f1 + A + %(f)
Here is where we apply the ideas of limit and function and are thus able to
go beyond Archimedes, one of the greatest mathematicians of all time. (Newton
once noted, in explaining his own momentous achievements, that he stood on
the shoulders of giants. We are standing on top of a whole pyramid of giants.)
To get a feel for the types of issues and calculations that arise, we shall
examine some particular cases. We start by applying our technique to a case
where the answer can be confidently obtained by more elementary means. We
shall use rectangles to approximate the area of the triangular region R between
the x axis and the graph of
f ( x ) = 1 - x from x = 0 to x = 1 (see Fig. 6.8)
(R, being a triangle of base 1 and height 1, has area 4.)We shall first partition

More Area Included Less Extra Area Included


in Lower Sum in Upper Sum
(a) (6)
Fig. 6.7 Fig. 6.8
SECTION 6.2: APPROXIMATING AREAS 187

t t

4 x

(a1
Fig. 6.9

1 x3 = i,
10, 11 into four equal parts, with xo = 0, x1 = 4,x2 = 2, x4 = 1. Since
f ( x ) = 1 - x is decreasing on [0, 11, the maximum value on any subinterval
occurs at its left endpoint, and the minimum value occurs at the right endpoint.
Hence the lower sum based upon this partition [see Fig. 6.9(a)] is

U f )=f (+) (+) +f (+) (+) +f (i)(+) + f (+)(1)

-
-
-3. - +1- . - +1- . -1+ 0 1 1 .-=-
1 3 + -1+ - =1- 3
4 4 2 4 4 4 4 16 8 16 8
The upper sum based on this partition [see Fig. 6.9(6)] is

--1 . - + 1- . - +3- . - +1 - . - 1 1 1 1
4 4 4 2 4 4 4
-
-
-1+ - +3 - + -1= - 1 5
4 16 8 16 8
The error in each case (the area of the shaded region) is i.
Taking a finer partition, say 10 subdivisions, with resulting narrower rec-
tangles, should improve on this rough estimate. Let us take the evenly spaced
points (for convenience)
xO'='O x1 0.1 x2 = 0.2 ~j = 0.3 ... x9 = 0.9 XIO =1
Again using right endpoints for sl0(f)and left endpoints for zlo(f ), we
have:

(We kept the zero term visible to emphasize that we are dealing with a sum of
ten terms and put the sum in a form that will be comparable to the general case
later.) To total the numbers in the brackets we pair extreme terms together: the
first and last, the second and next to last, and so on, working toward the middle.
This way, the sum of each pair is the same, and the total is

(9 + 0) + (8 + 1) + (7 + 2) + (6 + 3) + (5 + 4) = 5 - 9 = 45
188 CHAPTER 6: INTEGRATION

Hence

SlO(f)= 45 - & = 0.45


Similarly,
-
1 1 1 1
S,o(f) =f(O)lo +f(.l)- 10 + f ( . 2 ) - 10 +. * * +f(.9)x

= [l 1
+ 0.9 + 0.8 + . . - + 0.11- 10
1
= [lo +9 +8 + * * * + 11-102
= 55 -
1 - 102 (by the pairing technique)

= 0.55
The error of each sum is reduced to 0.05.
To generalize this process and to take a limit, we now partition [0, 11 into n
subintervals using points, which for convenience we take to be distance l / n
apart along [0, 11. Hence we have

Here Axi = l/n for each subinterval [xi-l, x i ] . We shall parallel the technique
used for n = 10 exactly. Using right endpoints for the decreasiiig function
f ( x ) = 1 - x, we obtain

S , ( f ) = [f(+) ++) +f(+) + * * * +f(?) +f(;)]+


=[(1--$)+(1-;)+(1-+)+ . . * +(1-*)+(1-;)];

6.2.2 = [(n - 1) + (n - 2 ) + (n - 3) + . . . + 1 + 0171


Obviously, we need a formula for the sum of n consecutive integers. It is
useful to develop a general formula for such sums.
Suppose we have a sum of integers from 1 to m. We add the sum to itself,
pairing extreme terms as before but aligning them vertically, as follows:

1 + 2 + 3 +...+(m-l)+ m
+
rn + (m 1) + ( m - 2) + . . . + 2 + 1
-

( m + 1) + (m + 1) + (m + 1) + . . * + (m + 1) + (m + 1)
There are m such (m + 1) terms, so the total is m(m + 1). But since this is
twice the sum of 1 through m, we can conclude the following:

6.2.3 (Sum of Consecutive Integers)


1 + 2 + 3 + . . . + (m - 1) + m = [m(m + 1)]/2
Returning to &(f) and (6.2.2) we have the sum (in reverse order) from 1
to n - 1 = m, so the total will be
SECTION 6.2: APPROXIMATING AREAS 189

( n - l)(n - 1 + 1) 1
Sn(f)= 2[ 17

Thus, &(f) < 1/2 f o r all n. But, more important,

Similarly, using left endpoints for the taller rectangles,

= -+-+-
n
[n
n-1
n
n-2+...
n
+
= [n + ( n - 1) + (n - 2) + . . . + 13-n21
(using the sum formula with n = m )

Thus, zn(f)> 1/2for all n and, again,


-
lim Sn(f) = lim
n-+m n-+m 2
Hence, for this particular example, where we knew A = 1/2, we achieved our
aim (6.2.1). As the width l / n of the rectangles approaches zero, we were able to
conclude that

&(f) = + [ 1 I,- 1
+
1
t- -
" :l
2
1 +-
-
= Sn(f)

BlJT W H Y ARE,QP
We are now in a position to illustrate how the area of a region might arise in
dealing with a problem initially stated without any reference to area. Suppose
>
~ ( x ) 0 gives the instantaneous velocity of some object as a function of time x
from x = a to x = b, where the graph of u(x) is as shown. A typical rectangle of
height ~ ( x , and
) width x , - x , - ~= Ax, has a n area ~ ( x ,A) x $ . But what is the
'(') product of a constant velocity ~ ( x , and
) a time interval Ax,? A distance! (If you
1 travel at 30 mph for & h, you cover (30)(&) = 3 mi.) Hence the area of the
rectangle-which, by its flat top, assumes that the velocity is constant-is
approximately the distance traveled by the object over the interval from x , - ~to
x , . The sum of the areas of such rectangles approximates the total distance
traveled from time a to time b. How would we improve this approximation? We
Y
b would take more and more, but narrower and narrower, rectangles. As we saw
when defining derivatives, the smaller the interval, the better the constant
190 CHAPTER 6: INTEGRATION

velocity approximates the variable velocity. The limit of such approximations is


both the area under the graph of u ( x ) from a to b and the total distance traveled
by the object from time a to time b.
Thus in setting up the machinery to solve the area problem we also provide
the means for solving a much wider class of problems.

PROGRESS TEST 1

1. Suppose thatf(x) = 4 - x 2 on [ -2, 21 is subdivided 2. Suppose thatf(x) = 2x on [0, I ] and [0, I ] is evenly
by the points xo = -2, x1 = 0, x2 = 1, x3 = subdivided into 10 subintervals.
3 / 2 , xg = 2. ( a ) Compute slo(f).
(Note that f ( x ) = 2x is in-
( a ) Sketch and compute the corresponding lower creasing.)
sum & ( f ) . ( b ) Write out &(f) for [0, 11 partitioned into n
( b ) Sketch and compute the corresponding upper subintervals of length l / n , and express the sum in
sum s,(.f). brackets as a sum of consecutive integers.

A MORE COMPACT NOTATION


Before we go on to show how this rectangle approximation process applies in
much more generality, it is useful to introduce some shorthand notation for
sums.
If ai, a2, . . . , a, is a collection of n numbers, then their sum

a, + + ... +an
a2

can be written in the abbreviated form


n

i=l

which is read the sum from i = 1 to i = n of a sub i. We call this sigma notation
or summation notation. Thus, for example, if the typical ith term in the sum is
ai = 2i2, then

+ + + aq +
a2 a3 a5

5
= 2i2 = 2(1)2 + 2 ( 2 ) 2 + 2(3)2 + 2(4)2 + 2(5)2 = 110
i=1
The earlier sum of consecutive integers from 1 to m has its ith term equal to
i ; that is,
ai = i. Hence we may rewrite (6.2.3) as

xi=
m(m + 1)
i=i
2

Notice that the role of i in ai is merely as a n index; any other letter would do
as well. For example, in the preceding formula,

and the index letter does not even appear in the final sum. For this reason the
index letter is often called a dummy variable.
We can rewrite the sums appearing earlier in sigma notation. The partition
of [0, 11 into n subintervals of equal length may be written with its typical ith
point identified as follows:
SECTION 6.2: APPROXIMATING AREAS 191

xo=o XI=-
1
x2
2
=- x3
3
=- ... x. = -2
n n n n

Then the typical zth subinterval looks like

Since, as we noted,f(x) = 1 - x is decreasing on [0, 11, it reaches its minima


at right endpoints. Thus, on the zth subinterval, the height of the shorter
rectangle is

n
Its width Axi is l / n , so its area is

-.-=-
n-i 1 n-i
n n n2
Thus, adding these areas from i = 1 to i = n, we have the lower sum

" n - i " . I
= c T = c ( n - 2 ) ,
i=l i=l

Since the constant factor l / n 2 appears in each term and does not involve i, it
can be factored out, leaving

where the sum inside the brackets is merely the sum of consecutive integers from
n - 1 down to 0:
n
C ( n - i) = ( n - 1 ) + (n - 2) + ( n - 3) + . . . + [n - ( n - l)] + ( n - n)
i= 1 i= 1 i=2 i=3 ... , i=n-1 z=n

Hence we have arrived at exactly the same sum S , ( f ) achieved earlier in


(6.2.2).

PROGRESS TEST 2

1. Compute 2Fz2( 2 j 2 - 1) by substituting the four ( u ) RewriteS,,(f) in sigma notation.


values o f j into 2 j 2 - 1. ( b ) Determine &(f) in sigma notation for an even
2. For f ( x ) = 2x on [0, 11, as in Prob. 2, Progress partition of [0, 11 into n subintervals.
Test 1 :

MORE ON APPROXIMATING AREAS


We now move to a case where we do not have a ready area formula: Find the
area above the x axis and under

f ( x ) = x2 from x = 0 to x = 1
192 CHAPTER 6: INTEGRATION

As can be seen in Fig. 6.10, sincef is increasing throughout [0, 11, lower sums
will require left endpoints and upper sums will require right endpoints.
Using an even partition of [0, I ] into n subintervals, a typical subinterval
looks like

Hence, in sigma notation,

~
i=l i=l
n n
Fig. 6.10
i=l

It turns out that for n = 100 we get


S,,,(f) = 0.32835 and s,,,(f) = 0.33835
and for n = 1000 we get
-
S l o o o ( f )= 0.3328335
- and Slooo(f)= 0.3338335.
4,
These quantities seem to be closing in on the number a hypothesis even more
plausible when we note that the average of the upper and lower sums for
n = 1000 is 0.3333335. To confirm this, we need a formula for the sum of
squares of consecutive integers, because all our sums involve such squares. A
formula, whose proof is outlined in the exercises, is

6.2.4 Sums of Squares


n
i2=
n(n + 1)(2n + 1)
i=l
6

Applying this to the upper sum, after factoring out l / n 3 (which appears in
each term and does not involve z ) , we have

Now, it is clear that, as n grows without bound, the width Axi = l / n of the
rectangles approaches zero and

sn(f ) approaches -
1
3
The argument that
SECTION 6.2: APPROXIMATING AREAS 193

also approaches involves the same basic idea. However, it is more valuable to
examine the dzference between the upper and lower sums.
We expand the upper and lower sums and align common terms vertically to
facilitate the subtraction of &(f) from s,(f).
(Note that each has n terms.)

s,(f) = c-
n

i=l
i2

n3 =
12
,n+-+,+ n3
22 32
n
( 2 - 1)2

n
...+-
. . . +n,+T+
(i)2 (n -
n3
+ -n2n3

n2
+O + ... + 0 + 0 + ... + 0 +7
- - --1
n3 n
Thus, as n + +m,
1. The width of the rectangles approaches 0.
2. The difference between the-upper and lower sums likewise approaches 0.
3. One of the sums, namely S,(f), approaches $.
Hence, combining these facts, we have proved that the area is exactly It is 4.
of special interest to examine the difference between upper and lower sums,
?,(f) - &(f),in graphical terms. In Fig. 6.1 1 we shaded each region repre-
senting the difference between taller and shorter rectangles. We then slid each
shaded region to the left and stacked it along the vertical axis so the total area
would be easier to recognize. The regions stack up to form a single rectangle
of height
f(1) -f(O) = 1 - 0 = 1

and width l/n. This vertical error rectangle [whose area is - S,(f)] s,(f)
thus has area l/n. The previously derived algebraic result and its graphical
interpretation establish emphatically that

~ s)n ( f )
3%- + 0
as the width of the rectangles + 0.

ti

Fig. 6. I 1
194 CHAPTER 6: INTEGRATION

A GENERALIZATION TO MONOTONIC FUNCTIONS


The previous examples point the way to a more general analysis, which will
show that the difference between the upper and lower sums, and hence the
approximation error, approaches zero as the width of the rectangles approaches
zero. In several stages of increasing generality, we shall show that the rectan-
gle-approximation technique applies to regions bounded by a wide class of
functions.
Iff is either increasing or decreasing on all of [a, b], we shall say that f is
monotonic on [a, b]. Right now, we make the following assumption: f ( x ) is a
nonnegativefunction continuous and monotonic on [a, b].
We shall carry out the examination assuming thatf(x) is increasing and leave
the decreasing case to the next Progress Test. We are approximating the area A
of the region R bounded by the graph of& the x axis, and the lines x = a and
x = b.
We shall focus our attention on z n ( f ) - & ( f ) because this difference
includes both the errors involved in approximating by upper - sums as well as
errors involved using lower sums. As a result, if we can force S n ( f ) - &(f) to
zero by taking narrower and narrower rectangles, then we can be guaranteed
that the approximation to A , provided by either sum, can be made as accurate
as we please. This will establish the validity of the rectangle-approximation
technique for regions bounded by the type of function under consideration.
Suppose we have an even partition of [a, b] into n subintervals of length
A x i = ( b - a ) / n with the points
xo = a, x l , x 2 , . . . , x i , . .., xn = b

As before, we form the difference between the upper and lower sums, using left
endpoints for lower sums and right endpoints for upper sums. We align the
sums vertically, and factor out A x i = ( b - a ) / n to save space:

Now, since a and b are fixed, we know that

=o
Thus, by taking more and more, but narrower and narrower, rectangles we can
improve our area estimate to arbitrary accuracy. The graphical interpretation
of S , ( f ) - $,(f) is much the same as before. However, in this case (see
Fig. 6.12), the area of the error rectangle is
SECTION 6.2: APPROXIMATING AREAS 195

Furthermore, iff(.) happens to be constant on some subinterval of [a,61, the


maximum and minimum values o f f ( x )on that interval are the same, so the
taller and shorter rectangles are equal_ (see Fig. 6.12). Hence, on such
subintervals, zero error is contributed to S,(f) - & ( f ) , and so our analysis
applies to functions that may be constant over some subinterval.

JROGHESS TEST 3

Suppose f ( x ) 2 0 and is continuous and decreasing on 2. Determine lim [S,(f) - S,(f)].


n-+m
[a,61, and x,, = a, xl, x 2 , . . . , xi, . . . ,xnP1, xn = 6 is an
3. Draw a decreasing version Of Fig. 6.12.
even partition of [a,61 into subintervals of length
(6 - a)/..
1. Determine ?,(f)- & ( f ) algebraically as previ-
ously. [Be careful about endpoints, since f ( x ) is
decreasing.]

FIJRTIIER GENERACIZATIONS
In all our work we used partitions of [a,61 into equal-length subintervals for
convenience. Later, in applications, such even partitions are less convenient.

6.2.5 IPefinition Let P be a partition of [a,61 into n, not necessarily equal-length


subintervals, with P consisting of the points
a = xo, x l , x g , . . . , x i , . . . ,xnpl, x , = 6

We call the largest of the numbers Axi = xi - xipl (for 1 5 z 5 n ) the norm of
P and denote it by IlPIl. (Thus llPl1 is the width of the widest rectangle
determined by P.)

We can now make some important observations extending our earlier work.
The supporting argument follows.

6.2.6 Theorem Assuming thatf(x) is nonnegative, continuous, and monotonic on


[a,61, then the rectangle-approximation method yields arbitrary accuracy
provided that IlPIl is small enough.
196 CHAPTER 6: INTEGRATION

Pig. 6.13

Alook back at our earlier calculation of s n ( f )- &(f) shows that, whether


or not the partition is even, all terms of the form f ( x , ) Ax, cancel from
?%(f)- &(f) except for f ( x o ) Axa = f ( a ) A x , and f ( x n ) Ax, = f ( b ) Axn.
Graphically, we can still stack the differences between upper and lower rectan-
gles along the vertical axis (see Fig. 6.13) except that they do not form a n
error-rectangle as before. Nonetheless, the total error region (shaded) is
contained within a rectangle (with the heavy black border). We can thus see
that by letting IIPI/ + 0, we force j n ( f )- &(f) to zero.
Since the upper and lower sums really depend upon the choice of points that
make u p the partition P (and not on their number), we shall alter our notation
slightly and let ?(, f ) denote the upper sum and &(f) denote the lower sum.
Hence we can restate the above conclusion as

Heretofore we always used endpoints of the subintervals [ x i P l , x i ] in our


sums. Again, in future applications, the freedom to create approximating sums
based upon points other than the endpoints will prove to be most valuable. If x r
is any point in the subinterval [ x i - l , xi],thenf(x,T) satisfies

mi <f(x,T) 5 Mi
where mi and M iare the minimum and maximum values off(x) on [ x i - l , x i ] . As
a result, the area of the rectangle of heightf(xr) is between that of the shortest
and tallest rectangles based upon [ x i P l , x i ] .
Given such a n arbitrary choice of points x f , x i , . . . , x f , . . . ,x t in the re-
spective subintervals, we shall denote the total area E:=lf(x,) Axi enclosed by
rectangles of heightsf(xr) by S;(f). Thus we have

& ( f ) 5 S 2 f ) IW )
Consequently, knowing that the error involved in using upper or lower sums to
approximate A can be made as small as desired allows us to conclude that the
same is true for S;(,f). In particular,
n

This means that, as long as the width llPll of the widest rectangle approaches
zero, S;( f)approaches A independently of the way the points x f are chosen in the
subintervals.
SECTION 6.2: APPROXIMATING AREAS 197

We shall sayf is piecewise monotonic on [a, b] if there is at most a finite collection


of points c in [a, b] at which f goes from strictly increasing to strictly decreasing
or vice versa. (See Fig. 6.14 for a n example of a piecewise monotonic function.)
With minor extension of our arguments we shall have established the
following theorem, which summarizes our achievements on the area problem to
date.

6.2.7 Theorem Supposef is a function continuous and nonnegative on [a, 61, and
A is the area of the region between the graph off and the x axis from x = a to
x = b. If P is any partition of [a, 61 and x % is any choice of points in the
subintervals [ x i - l , x i ] (1 5 i In ) determined by P, then
(4 U f )I A I SP(f 1.
(6) Mf) I S X f ) I W ) .
If, further, f is piecewise monotonic on [a, 61, then
(6) lim Sp(f) = lim S;(f) = lim S,(f) = A .
IIPll-0 IIPII-0 IIPII-0

monotonic on [a, b]. However, iff is only piecewise monotonic on [a, b], then
b our earlier argument applies on each of the finite collection of subintervals to
I --% show that the difference between the upper and lower sums can be made as
~
f(x) small as we please on each subinterval. (There would be four such subintervals
I I for the function in Fig. 6.14.) It follows that the finite sum of these differ-
.9 I I I1 I 111
II 1v I
I ences-the total difference between the upper and lower sums on [a, b]-can be
I I I I +x made as small as we please by choosing IlPIl sufficiently small. Hence the
a b
arguments on the previous several pages apply to show that the indicated limits

There do exist continuous functions that are not piecewise monotonic


(infinitely wiggly) and are not covered by our theorem. (See Chapter 9,
Sec. 9.3. The arguments required to cover such cases are beyond the scope of
our present development.

LIMITATIONS OF THE
RECTANGLE-APPROXIMATION METHOD
Although we have indeed come a long way in attacking the area problem, we
have in Theorem (6.2.7) achieved a classic example of what mathematicians
call an existence theorem. It tells us that all the appropriate limits do exist
and do lead to the area of the region in question. I t does not give us any ready means
for computing the area.
For functions f satisfying [6.2.7(c)] we can exercise the freedoms to choose
regular partitions and points x r = xi to obtain

6.2.8

However, as is evident in the following exercises, the various techniques


needed to determine the sums and evaluate the limits restrict use of (6.2.8) to
relatively simple functions.
The real solution to the area computation problem is provided by the
Fundamental Theorem of Calculus, whose importance is reflected in its title. It
will be developed in the next section.
198 CHAPTER 6: INTEGRATION

SECTION 6.2 EXERCISES

In Exercises I to 10 sketch the function over the indicated interval. Thenfor the given partition P, ( a ) compute 8, ( f ) and ( b ) compute ip
( f ). For
the listed points xf in the subintervals determined ly P, ( c ) compute S i ( f ) and ( d ) compute IIPII.
I . f ( x ) = x + I on [ o , ~ ] ;= : $ , x ; = 5,
P {o,$, I,$, 2 ) ; ~ = 2. f ( x ) = x +
1 on [0, 21; P = even partition into 10 suh-
x ; = 2, x ; = 1.9 intervals. x : = midpoint of subinterval
3. f(x) = 3 - 2 x on [O, 11; P = {O,&$,$, I}; =X T A, 4. f ( x ) = 3 - 2x on [0, 11; P = even partition into five
x ; = i , x ; = & , x i = 0.9 subintervals; x : = midpoid of zth subinterval
5. f ( x ) = 9 - x 2 on [--2, 31; P = { - 2 , -1, 1, 18, 3 ) ; 6. f ( x ) = x 2 + 1 on [-1,2]; P={-l,-+,O,l,$,Z};
x * -- -32, x *p = 0 , x ; = 1, x ; = 2 x ; = -1, x f = -0.1, x; = x ; = 4, a
7. f ( x ) = 1 + x 3 on [ l , 41; P = (1, 2, 24,4}; x ; = 1, 8. f ( x ) = x 2 + x + l on [ - 3 , 3 ] ; P = ( - 3 , - 1 , 0 , 2 , 3 } ;
x *2 -- 21
4,x;=3 x*1 -
- -2, x ; = -1, x ; = 1, x ; = 3
9. f ( x ) = l/x on [1,5]; P = { I , 2, 3, 5 ) ; x: = I , = $, 10. f ( x ) = $ x 3 +
$ 2- 2 x on [-3,O]; P = ( - 3 , -2,
x; = 4
-1,O}; x ; = -3, x ; = -1, x ; = - 21
11. Describe and compare S p ( f ) , i p ( f ) and S ; ( f ) for a
constant functionf(x) = b on an interval [c, d ] for any
partition P of [c, d ] .

In Exercises 1 2 to 15 determine the value of the indicated sum.

12. 22
i=1
13. c 2 i 2
12

iXl

14. 2 (222 - 5)
R

i=1
15.
7
C(37
j=2
+ 1)

Using methods analogous to those used in this section, determine the areas ofthe regions R indicated in Exercises 16 and 17. [Take an euen partition,
determine either &( f ) or zp(J),and take the limit as 1IPl1 + 0, which is equivalent to the limit as n + + 001.

16. R bounded hyf(x) = x + 1, the x axis, x = 0 and x =2 17. R bounded byf(x) = 3 - 2 x , the x axis, II = 0, and x = 1

In Exercises 18 to 20, use that

to simplify thefollowing sums and to put them in a form to which (6.2.3) and (6.2.4) apply. Then apply theseformulas to obtain the sums entirely in
terms of n.

19.
(_ 1 ;
2 i _ 1)'
_ -
20. i;[I - (32];
i=1
iXl

In Exercises 21 to 31, use (6.2.8) to determine the area between the given function and the given interval [a, 61.
21. f ( x ) = x 2 , [O, 21 22. f ( x ) = x 2 +
1, [O, 21
23. f ( x ) = x 2 + x + 1, [O, 21 24. f ( x ) = 9 - x 2 , [0,2]
2 5 . f ( x ) = 9 - x2, 1-3, -11 26. f ( ~ ) 9 - x', [ - 3 , 3 ]
27. f ( x ) = 2 x 2 + +
x 3. [O, 21 28. f ( x ) = 2x' +
3x, [o, 11
29. f ( x ) = x - x 2 > [O, 11 30. f ( x ) = x 2 - 2x +
1, [O, 21
31. f ( x ) = x z - 4x + 4,LO, 31
82. Prove (6.2.4) using the following outline. Let S denote side when the vertical columns are added, whereas on
the sum of the squares of tha consecutive integers from 1 the right side 3s - 3( 1 + 2 + 3 + . . . + n ) + n results.
to n. ( d ) Solve the resulting equation for S to obtain
( a ) Prove the identity n3 - ( n - 1)3 = 3n2 - 3n 1. + + +
S = n 3 / 3 n(n 1)/2 - n/3 [using (6.2.3) along the
( 6 ) Successively substitute the numbers 1, 2, 3,. . . , n way].
into both sides of this identity (but do not evaluate the 33. Prove that Cr=l z 3 = [n(n +
1)/212 by an argument
resulting expressions), aligning the results vertically. analogous to that of Exercise 32, but using the identity
( c ) Show that all terms hut the term n3 cancel on the left n4 - ( n - lI4 = 4n3 - 6np 4n - 1 +
SECTION 6.3: THE DEFINITE INTEGRAL AND THEOREM OF CALCULUS 199

In Exercises 34 to 37, use the previous formula (together with any others already developed) to determine the area of the region between thegivenfunction
and the given interval [a, b].
34. f ( x ) = x 3 , [O, 11 35. f ( x ) = x 3 + 1, [O, 21
36. f ( x ) = - x 3 +
3 x 2 - 2 x , [1,2] 37. f ( x ) = x3 + x2, [0,4]
38. By examining an n by n +
1 rectangle of 1 by 1 squares,
devise yet another proof of (6.2.3).

The Definite Integral and the


Seetion 6.3: Fundamental Theorem of Caleulus

Objectives: DEFINITION AND MEANING OF THE DEFINITE INTEGRAL


1. Understand the meaning and In Sec. 6.2 we showed that if f(x) is nonnegative, continuous, and piecewise
properties of the definite integral monotonic on [a, b], then
n
and its connection with
antiderivatives as expressed in the
Fundamental Theorem.
2. Use the Fundamental Theorem
and the Power Rule to evaluate
definite integrals.
-
exists and equals the area of the region between [a, b] and the graph off (x).
Furthermore, as long as IlPll 0, this limit is independent of the way the
partitions P are chosen and of the way the points x, are chosen in the subinter-
vals determined by P.
Although we were ostensibly attacking the area problem, and so assumed
f (x) was nonnegative on [a, b], a careful reexamination of our arguments
leading to the limit will reveal that the nonnegativity off (2) played no essential
role. It simply guaranteed that the limit itself would be nonnegative. By
relaxing the nonnegativity condition we shall unlock a whole new world of
applications to work, energy, and economics and will be able to determine
averages and volumes, and solve a whole host of problems currently out of
reach. In fact, the applications of this limit are perhaps even more varied than
those of the derivative limit.
In the next definition, and often in future work, we shall write

n
c f ( x , ) Axi instead of f (x;) Axi
P i=1
to emphasize that the sum depends upon the nature of the partition P and not
simply upon the number of subintervals formed by P.

6.3.1 Definition of the Definite Integral Suppose the functionf is defined on


the closed interval [a, b]. If

exists and is independent of the choices of x, in the subintervals determined by


partitions P of [a, b], we denote it by

call it the deJinite integral off from a to b, and say that f is integrable on [a, b]. We
refer to a as the lower limit of integration, b as the upper limit of integration, and f ( x )
as the integrand. We often express this definition as

a .
200 CHAPTER 6: INTEGRATION

and refer to the sum appearing on the right as a Riemann sum for$

The precise definition of the limit in (6.3.1) requires that for any E > 0, there
is a 6 > 0 such that

IlPlj <S implies


IJ
b
f ( x ) dx - c
P
f ( x r ) Axi I< E

for any choices of xr in the subintervals determined by P.


It is apparent from (6.3.1) that a function unbounded on [a, b] cannot be
integrable on [a, b], because in such a case it is possible to construct arbitrarily
large Riemann sums. In Chap. 14 we shall extend our work to handle the
unbounded case.
Despite the apparent complexity of the limit defining Jkf(x) ak, there is an
obvious notational similarity to the antiderivatiue dealt with in Sec. 6.1. Fur-
thermore our remarks in Sec. 6.2, suggesting that by approximating the area
under the graph of a velocity function we are in fact-approximating the
distance traveled over the time interval from a to 6 , even strengthen the
temptation to connect the two ideas. Later the Fundamental Theorem of
Calculus will reveal that the connection indeed does exist. However, the two
ideas of definite integral J:f(x) dx and antiderivative If(.) dx are not the same.
After all, SBf(x)ak is a number, whereas Jf(x)dx is afarnily offunctions (any two of
which differ by a constant).
Our earlier work effectively established the integrability of functions that are
continuous and piecewise monotonic on [a, b]. The arguments required to
extend this result to arbitrary continuous functions (and thus to cover the
infinitely wiggly case) are beyond the scope of this text but can be found in
any advanced calculus text. Hence we assume the following:

6.3.2 Iffis continuous on [a, 61, thenfis integrable on [a, 61. That is, J:f(x) dr exists.

Actually, we can use (6.3.2) to cover cases in which f is defined on [a, b] and
has a finite number of jumps or breaks on [a, b] provided there is no point xo in
[a, b] with limz+zf ( x ) = k c o , that is, as long asf is bounded on [a, b]. We then
know the graph ofjwill look something like that in Fig. 6.15. Such a function is
n CI c2 *=% ~3 64 b sometimes said to be bounded and pzecewzse contznuous on [a, 61.
P
I We merely apply (6.3.2) to each subinterval of [a, b] on whichfis continuous.
J
1

Fig. 6.15 In the case of Fig. 6.15 this amounts to applying (6.3.2) separately to the
intervals [a, cl], [cl, cz], [cz, c 3 ] , [c3, c4], [c4, b] and then adding the results.
Our examination of the basic properties of the definite integral will be based
upon the interpretation that has carried us this far: area. Thus it is of some
x importance to interpret J B f ( x ) dx whenf(x) 5 0 on [a, b]. (See Fig. 6.16.)
But, of course, for x: any point in a typical subinterval [ x ~ - xz] ~ , determined
by a partition P of [a, b ] , f ( x : ) Axt is merely the negative of the area of the
rectangle based on [ x % - ~x z, ] . Passing to the limit of the sum of such negative
Fig. 6.16 numbers as liPll + 0, we have the following theorem.

6.3.3 Theorem Iff(%) 5 0 and is integrable on [a, 61, then J:f(x) dx = --A(R),
the negative of the area A(R) of the region R between the graph off and the
interval [a, 61.

For example, we can show by techniques used in Sec. 6.2 that


3
r4(x - 2)dx z
SECTION 6.3: THE DEFINITE INTEGRAL AND THEOREM OF CALCULUS 201

We see in Fig. 6.17 that the area of the triangular region I is A (I) = 4 (1)( 1)
f(X) =X - 2 4
= and that the area of triangular region I1 is A(I1) = 4(2)(2) = 2. Thus

.I4 (X - 2) dx = A(I1) + (-A(1)) 1


= 2 --
2
-3
2

x PROPERTIES OF THE DEFINITE INTEGRAL


Our definition of J l f ( x ) dx presupposes that a <
b. We shall extend our defini-
tions to cover situations where a >
b or a = b as follows. (Unless otherwise
noted, w e shall assume that functions are integrable over an appropriate interval I.)

I 6.3.4 Definition (a) If b < a, then we define


Fig. 6.17

( 6 ) If a is in the domain ofJ; then we define

lf ( x ) dx = 0

The first definition simply helps things work out in calculations. The second
makes sense geometrically because the only rectangles that fit over or under
the one-point interval [a,a] have zero area.
We summarize important properties of the definite integral in the following.

6.3.5 Properties o f the Definite Integral

(b) Jb kf(x) dx = k f ( x ) dx, where k is any constant


a

( d ) Iff(x) is bounded above by A4 on [a,b] and below by m on [a,b ] , then


,-Jb)
h
.--**
-4
m(b - a) 5 J f ( x ) dx 5 M ( b - a )
i(jni =
;id I
-* ~ ~ _ e -elx
I a
I
Rl I R, 1 b
5Jbm
I
n C b * (el Iff (4I d.1 on [a,61, J fI.(
a
dx dx
a

j,b J ( x ) dx = nrra of R Dzscussion of (6.3.5) In part (a) we make no assumptions about the relative
= n rra of R , + area of R , order of a, b, and c. If a c < <
b andf(x) 2 0 on [a, b ] , then it becomes a
= s: A X ) dx + s: f i x ) dx
perfectly reasonable statement about the area of a region R that is broken into
Fig. 6.18 two subregions R , and R, (see Fig. 6.18):
Parts ( b ) and (c) follow from the corresponding facts about the sums used in
defining the definite integral. Part (d) has a straightforward interpretation in
terms of area whenf(x) 2 0 on [a, b ] ; see Fig. 6.19. The area of the rectangles of
width b - a and heights, respectively, m and A4 clearly satisfy
b
m(b - a) 5 f ( x ) d x = A ( R ) IM(b - a) ( R is shaded)
X a
a Y

b-a
h Part ( e ) again has a direct interpretation in terms of area when 0 < f ( x ) g(x) <
Pig. 6.19 on [a,b]. Letting R, be the region bounded by the graph off(x), the x axis, and
202 CHAPTER 6: INTEGRATION

I I X X
X
a 6
Fig. 6.20 Fig. 6.2 1 Fig. 6.22

the lines x = a and x = b, and letting R, be the region similarly bounded by the
graph of g(x), which contains R,, it is clear from Fig. 6.20 that
b
Jbm
a
= A ( R f )5 4 R , ) = J g(.)
a
dx

We could also interpret part ( e ) iff(.) 5 g(x) 5 0 on [a, b ] by using (6.3.3) and
negatives of areas.

Example 1 Use [6.3.5(d)] to determine upper and lower bounds for

J' (1 + 2x - x2) dx
Solution (See Fig. 6.21.) Forf(x) = 1 +
2x - x 2 we have the minimum value off on
[0,2]achieved at both x = 0 and x = 2 withf(0) = 1 = f ( 2 ) .The maximum is
achieved at x = 1 withf(1) = 2. Hence, with b - a = 2 - 0 = 2 , we have
2
2 = ( 1 ) ( 2 )5 J (1 + 2x - x2)dr 5 2(2) = 4 0
0

We can improve on the lower bound of Example 1 by breaking the region


into trapezoidal pieces as in Fig. 6.22 and then determining the areas of the
trapezoids to obtain (using the formula for the area of a trapezoid of base b and
heights h, and h,, A = b(h, +
h2)/2. That is,
J2(1+2x-x2)dxtl -
1+2 1+2
( 2 )+1(T)=3
In Chap. 1 1 we shall return to elaborate on this trapezoid-approximation idea.

PROGRESS TEST 1

1. Use [6.3.5(d)]to determine upper and lower bounds (Draw a sketch when necessary.)
for J"t3 +
(4x3 $x2 - 2x) dx
2. Determine the following definite integrals, using the (4 J27 ( - 3 W ( b ) J3(1 - x)dx
interpretation of the definite integral as area where 0

f ( x ) is nonnegative, using (6.3.3) wheref(x) is nega-


tive, and using [6.3.5(a)]iff(.) changes sign on [a, b]. (d) S-l4dx
-2 -6
SECTION 6.3: THE DEFINITE INTEGRAL AND THEOREM OF CALCULUS 203

3. Determine the indicated definite integral for the


functions sketched in ( u ) and ( 6 ) .

THE PI"NDAMEKTAL TIIEOR &M:


A PRELIMINARY DlSCrSSION
So far, we are able to obtain definite integrals without using fancy manipu-
lations of sums and limits only for linear or step functions as in Progress Test 1.
But in such cases calculus was not needed to get thc answer! This situation
resembles that in Chap. 2, when we could compute derivatives only by apply-
ing the limit definition. We need simpler rules that will allow us to compute
definite integrals such as
X xdx
or J' (xz + 1)4x3 dx
0

Fig. 6.23 without resorting to the limit-of-sums definition.


We already remarked on the similarity of the antiderivative notation and
the definite integral notation. We shall now pursue this relationship.

~~ . ~~

Example 2 Determine the area under the graph off(x) = 3x from x = n to Y = b with
< <
0 n 6. (See Fig. 6.23.)
Solution Of course the definite integral J:(3x) dx will give the desired area, and this
integral could be computed using the definition and the appropriate finite sum
formula applied to the resulting Riemann sum. However, the area of this
trapezoidal region is just the difference of the areas of two triangles. (See
Fig. 6.24.)
Thus
3b2 3a2
J b (3x) dx = A ( R ) = __ - - 0
2 2

Now a typical antiderivative of 3x is F ( x ) = 3x2/2 + C for some constant C.


204 CHAPTER 6: INTEGRATION

Fig. 6.21

But then

F(b) - F ( a ) =
(3r
-+ ) C - -+
(3:
C
1
- 3b2
- 3a2
2 2

Thus for this example we could have determined the value of JB (3x) dx by
jinding any antiderivative F of 3x and forming the dzference F ( b ) - F(a)/
Similarly, we can compute the area under the graph of
f(t) = 32t from t = 0 to t = x
using either the definite integral limit-of-sums approach or by using an antide-
rivative F ( t ) = 16t2 evaluated at t = 0 and t = x. Either way we determine the
area to be

lx 32t dt = 16x2

Now, of course,f(t) = 32t is the familiar function that gives the velocity of a
freely falling object after t seconds. We know it is the derivative of the dis-
tance-fallen function F ( t ) = 1 6t2 or, in the terminology of Sec. 6.1, its antide-
rivative is a distance function. Hence the area 16x2 has two interpretations:
1. As the area under the graph of f ( t ) = 32t.
2. As the distance fallen after t = x seconds have elapsed.
Obviously, the relationship between antiderivatives and integrals is inti-
mate, but it needs further investigation.

9 Why not regard the right-hand side of the region bounded by the graph of
f(t)= 32t, the t axis, and the line t = x, as a variable, with the line t = x
sweeping out the area under the graph from left to right? Then the area Zs a
function of x:

ix 32t dt = F ( x ) = 16x2

Previously, when thinking of F ( x ) as a distance function, we computed its


derivative to obtain the velocity function F(x) = 32x. Let us now compute
and interpret the derivative of F ( x ) where F ( x ) is regarded as a n area function.
SECTION 6.3: THE DEFINITE INTEGRAL AND THEOREM OF CALCULUS 205

F(x + A X ) - F(x) = 16(x + AX) - 1 6 ~


= 32x A x + 16(Ax)
the difference between the areas of the larger and smaller triangles, is the area of
the shaded vertical strip (see Fig. 6 . 2 5 ) . Its two terms correspond to the two
J ~ X=)3 2 ~ parts separated by a dashed line. Now, dividing through by A x and forming the
limit, we have
t
F ( x ) = lim
32x A x + 16(A~)~
Ax-0 AX
F(x) = area = 16x2
= 32x
Fig. 6.25
Hence, geometrically, as the width of the strip approaches zero we are left with
the length f ( x ) = 3 2 x of the line sweeping out the region.

Many times before we have observed the connection between the rate of
growth of a geometric object and the size of its boundary in the direction of the
growth .
The circle, growing by its circumference: D,(7rx2) = 27rx
The sphere, growing by its surface area: D,($7rx3) = 4mx2
The square, growing by the length of the two sides in the direction of the
growth: B,(x2) = 2x
The cube, growing by the area of the three sides in the direction of the
growth: D,(x3) = 3 x 2
The cylinder, growing taller by the area of its roof: D,(7rr2h) = 7rr2
The cylinder, growing wider by its lateral surface area: D,(7rr2h) = 27irh
In each case the growth in the size of the geometric object (volume in three
dimensions, area in two dimensions) is the size of the boundary in the direction of the
growth (an area for a three-dimensional object, a length for a two-dimensional
object).
The derivative o f F ( x ) = 16x2 regarded as an area function is the length ofthe side in
the direction of the growth, namely 32x! That this result is not an accident peculiar
to this case, but rather holds for continuous functions in general, is the ex-
traordinary conclusion of the Fundamental Theorem of Calculus, discussed
after Progress Test 2 .

PROGRESS TEST 2

1. Evaluate the following definite integrals J B f ( x ) dx constant function f ( x ) = 5 from x = a to x = b


by determining an antiderivative F off and then (a < b)?
computing F ( b ) - F(a). (Each definite integral ap-
3. 9 (a) What is the area of the region under the
peared earlier, so compare your results with those
graph o f f ( t )= 5 from t = 0 to t = x?
obtained previously.)
( 6 ) Denoting your answer to (u) by A ( x ) , find A(x)
using the definition of derivative and interpret
(a) 1 1 x 2 d x (6) J 14 (x - 2) + +
A ( x A x ) and A ( x A x ) - A ( x ) geometrically.

(c) i2 + (1 2x L x2) dx
4. Find the area of the trapezoidal region under
< <
f ( x ) = 3x + 5 from a to b (0 a b ) using geo-
metric reasoning and compare your result with
2. What is the area of the region under the graph of the F ( b ) - F ( a ) , where F is any antiderivative of$
206 CHAPTER 6 : INTEGRATION

THE FITNUAMEKTAL THEOHEM OF CALCIJC~?S


We shall now state the theorem for arbitrary continuous functions and then
prove it for nonnegative continuous functions to take advantage of area
interpretations.

6.3.6 T h e Fundamental Theorem of t h e Calculus, Form I Suppose


f is continuous on [a, b ] , and for any x in [a, b] suppose F is defined by the fol-
lowing:

F(x) = J ' f ( t ) dt
a
Then
F'(4 =Ax)
In other words, J : , f ( t ) dt is an antiderivative off on [a, 61.

Note that to find the derivative of J : , f ( t ) dt you simply write down the
integrand with t replaced by x.Just as we did in the previous discussion, we use
one variable in the integrand ( t in this case) and another as the upper limit of
integration (x in this case). This helps keep separate the two roles played by the
function f .
We saw that for f ( t ) = 32t,

F(x) = ix 32t dt implies F ' ( x ) = 32x

Put more directly, we have

Similarly,

Proof of Fundamental Theorem, Form I, wzth f ( t ) 2 0 on [a, b ] ; We know that F ( x )


represents the area of the region R, below the graph ofy = . f ( t ) and above the
interval [ a , x ] ; see Fig. 6.26. That is,

F ( x ) = A(R,) = S ' f ( t ) dt
a

Js
Notice that F ( a ) = JEf(t) dt = 0 and F ( b ) = f ( t ) dt.
Let us now determine the derivative of this area function,
I n b
F(x + AX) - F(x)
Fig. 6.26 F'(x) = lim
Ax-0 Ax
Suppose A x > +
0. Then, as shown in Fig. 6.27, F ( x Ax) is the area under
fltj
the graph offout to x +Ax and F ( x ) is the area under the graph off out to x.
+
Thus F ( x Ax) - F ( x ) is the area of the shaded region.
We now turn our attention to the interval I = [x,x +
Ax]. Now by (4.4.2),
the Extreme-Value Theorem, since f is continuous on the closed interval Z,f
attains a minimum value on Z at, say, mx and a maximum value at, say, M,.
Thus, by [6.3.5(d)],with the width of the interval Z being Ax,we obtain (see
Fig. 6.28)
0 I i + A t b
Fig. 6.27 f (m, ) Ax IF(. + Ax) - F ( x ) If(4)
Ax
SECTION 6.3: THE DEFINITE INTEGRAL AND THEOREM OF CALCULUS 207

Thus, dividing through by A x , we have the limit quotient for F trapped:


F ( x + AX) - F ( x )
f(mJ I Ax I f (Mx)
We know that

x<m,<x+Ax and x<Mx<x+Ax

so letting Ax + 0 forces
mp + x and Mp+ x
But f is continuous, so

and

Since in turn, the quotient


F ( x + AX) - F ( x )
Ax
is always trapped between f (m,) and f ( M z ) ,we have by the Squeeze Theorem
(2.5.6),

F'(x) = lim
F(x + Ax) - F ( x )
AX-0 Ax =f (4
completing the proof for Ax > 0. A similar argument applies if Ax < 0.
We are now in a position to make the computation of definite integrals much
easier and to substantiate the underlying conjecture of Progress Test 2. The
Fundamental Theorem, Form I, says that S:f(t) dt is an antiderivative off on
[a, b]. Thus any antiderivative F o f f on [a, b] is of the form

F ( x ) = s ' f ( t ) dt
a
+C
Now

and
+C
F ( a ) = J- f ( t )dt
a
=o+c=c
208 CHAPTER 6: INTEGRATION

Thus
b
F(b) - F(a) = f ( t ) dt +C-C
U

which gives us:

6.3.7 The Fundamental Theorem of the Calculus, Form I1 Supposefis


continuous on [a, b] and F is any antiderivative off on [a, 61. Then

['f(x) dx = F ( b ) - F ( a )

This remarkable result says that to evaluate J B f ( x ) dx you simply find an


antiderivative off, substitute the number b and then subtract the result of
substituting the number a. The importance of Sec. 6.1 on antidifferentiation
should now be clear.
In fact, we shall often refer to an antiderivative as a n indejinite integral, t o f ( x )
in Jf(x) dx as an integrand (just as with definite integrals), and to the process of
determining either indefinite or definite integrals as integration.
We shall use the notation

F(x) 1 b

U
= F(b) - F(a) or [F(x)Ib = F(b) - F(a)
a
Thus
b b
J f(x) dx = F ( x ) ( = F(b) - F ( a )
a U

where F is any indefinite integral of$

Example 3 Evaluate Jg ( x 2 + 4)3x dx.


Solution Our first concern is to find a n antiderivative

J(x2 + 4)3xdx
We shall use the substitution technique with u = x 2 + 4, so du = 2x dx,making
x dx = (1/2) du. Thus we have

s(x2 + 4 ) 3 x d x = J u 3 2 d1u
1_
-_ u4
2 4
+C by the Power Rule

- (x2
-
+ 4)4 + C
8
Since we are interested in any antiderivative, we can take C = 0. Hence

--
- 84
8 - -484

= 512 - 32 = 480 0
SECTION 6.3: THE DEFINITE INTEGRAL AND THEOREM OF CALCULUS 209

Two remarks concerning Example 3 have general application:


1. Even if we had not let C = 0, the constant C in the antiderivative
+
( x 2 4)4/8 + C would disappear in the evaluation at the limits of integration
0 and 2. Hence we use the simplest antiderivative.
2. It was essential to write the antiderivative in terms of x rather than u. If
we had substituted the given limits of integration 0 and 2 into u4/8, we would
get a n incorrect answer. In the next section we shall devise a Chain Rule for
Integration allowing us to switch the given x limits to u limits.

PROGRESS TEST 3

Evaluate the following definite integrals:

l- s,
1
(4x3 - 3x2 - 5)dx 2. 4 1
x(x2 + 1)2dx 3.
(x2+ 1)dx
d x 3 + 3x + 1

IJSES AND LIMITATIONS


fBF THE FUNDAMENTAL THEOREM
Our main tool in using the Fundamental Theorem to evaluate definite integrals
is the Power Rule for antiderivatives. However, we emphasize that our new
technique for evaluating definite integrals J : f ( x ) dx is only as effective as our
ability to find a n associated antiderivative J f ( x ) dx. For example, since our
Power Rule does not hold for n = - 1, at present we have no way, beyond the
definition of the definite integral, to evaluate

' even though we are assured by (6.3.2) that Js


d x / x exists. In fact, it represents
the area of the shaded region in Fig. 6.29. In later chapters we shall resolve the
Pig. 6.29 problem of this particular definite integral in several different ways.

SECTION 6.3 EXERCISES

In Exercises I to 12 evaluate the given dejinite integral using the Fundamental Theorem of Calculus
-2 -2
1. Jo (4 - 2 x ) dx 2. J ( x - x 3 ) dx 3. [lo (2x2 + 1) dx 4. f1x3dx

sl/' Ji si
-2 -A

5. (3x - 9) dx 6. 5(x2 - l ) 4 x dx 7. \/%w--ldx 8. V m d x

sf
0 -2

10. (x3 + 6x - l)3(x2 + 2 ) dx


12. s: (x - l)3x dx

In Exercises I 3 and 14, use [6.3.5(a)]to evaluate the given dejinite integral.

13. 4' l x 2 - 11 dx 14. ll Ix2 - 4.1 dr

18. Sketch the region bounded by the curvesf ( x ) = x 3 , the x Show that J: f ( x ) dx and Jt(f(x) +
c) dx differ by the
axis, x = - 2 , and x = 2. Without evaluating, prove that area of a certain rectangle.
J"x3 dx = 0. Cite specific theorems. 18. Show that the area under f ( x ) = x n for rational n 2 0
16. Repeat Exercise 15 for f ( x ) = - x and J?,( - x ) dx. from x = 0 to x = b is l / ( n + -
I ) base height. Inter- -
17. 9 Supposef(x) 2 0 on [a, 61 and c >
0 is a constant. pret this result for n = 0 and n = 1.
210 CHAPTER 6: INTEGRATION

19. Suppose a ball is thrown upward from ground level at an sent? Explain first in terms of approximating rectangles.
initial velocity of 96 ft/s. Noting that upward velocity is 2 I.Repeat Exercise 20, iff(.) is the marginal profit function
positive and downward velocity negative, sketch the and x is the number of items.
velocity function on the time axis. Then use a definite 22. Differentiate F ( x ) = JE(3t2 + 2 t ) dt:
integral to determine the total distance traveled by the ( a ) Using Form I of the Fundamental Theorem.
ball in the air. Interpret your results. (This shows that ( b ) By first evaluating the definite integral.
care must be taken when negative velocities are involved.) 23. Repeat Exercise 22 for G ( x ) = JE(4t3 - 6t +
1 ) dt.
20. Suppose the acceleration a(t) is nonnegative from t = 0 to +
24. Compute G(x) for G ( x ) = 1: ( t 2 3 t ) dt. (Be careful.)
t = b. What does the area under the graph of a ( t ) repre-

For Exercises 25 and 26, note that properties of the dejinite integral [see (6.3.5)] were discussedprior to any mention ofderivatives and the Fundamental
Theorem. Show that the properties of derivatives in combination with application o f Form I (to an appropriate variation of JE f( t ) dtyield statements
consistent with the given property of dejinite integrals.
21. Property [6.3.5(b)]
26. Property [6.3.5(c)]

A Mean-Value Theorem
Section 6.4: and the Chain Rule for Integrals

Objectives: AVERAGE (MEAN) VALUE OF A FlJNCTION


1. Determine the average value of a X Suppose that the functionf(t) shown in Fig. 6.30 gives the temperature (in
function over a given interval. degrees Celsius) on a given afternoon as a function of time t marked off in
2. Evaluate definite integrals using hours. (t = 0 is noon. About 4 P.M. a thunderstorm hit.)
substitution by replacing the What was the average temperaturef o r the afternoon? It hit a high of 30 and a low of
original limits with the limits in +
16, so your first answer might be (30 16)/2 = 23. But this does not really
terms of the substitution variable. reflect the behavior of the temperature through the afternoon at all. We might
improve on this by taking an hourly average; that is, we average the tempera-
3. Use the Chain Rule for tures at the end of each of the 5 h, to get
Derivatives in combination with
Form I of the Fundamental 27 + 27 + 29 + 28 + 16
Theorem to differentiate 5
functions defined by
6.4.1
integrals.
= 25.4
Temper at ure

30
t
28

26
25

16
15

Timr
0 1 2 3 4 5
Fig. 6.30
SECTION 6.4: MEAN-VALUE THEOREM AND CHAIN RULE FOR INTEGRALS 211

Note that the right-hand side of (6.4.1) is one-fifth of the Riemann sum
5

with t; = i, i = 1, . . . , 5 and At, = 1.


But this average does not quite tell the story either, since the temperature
was above 25.4 for almost 90 percent of the afternoon (before the storm). We
could improve upon this average by taking a half-hour average (that is, 10
intervals of length 1/2 h):

28 + 27 + 28 + 27 + 28 10
+ 29 + 29 + 28 + 25 + 16 = 26.5
We can write the above quotient as

which again exhibits it as one-fifth of a Riemann sum


10
Cf( t Y ) A t ,
i=i

where t: = i / 2 , z = 1, . , . , 10, and A t , = 1/2.


But even this average omits some of the temperature changes during the
afternoon; it does not take into account the high temperature of 30, for
example. To get the best average, the one that takes into account all the
fluctuations, we let the length of the intervals approach zero and average more
and more temperatures, getting

This gives rise to the following definition.

6.4.2 Definition The average (mean) value of a continuous function f over an interval [a, b]
is defined to be

Example 1 Determine the average value of the functionf(x) = 3 x 2 + 1 on the interval


LO, 41.
Solution By (6.4.2) the average is given by

Now
( 3 x 2 + 1)dx = x 3 + XI 4

0
= 68

Thus the average value off(.) = 3 x 2 + 1 on [0,4] is (1/4)(68) = 17. 0


212 CHAPTER 6: INTEGRATION

If f ( x ) = 3x2 + 1 in Example 1 represents the velocity of a n accelerating


vehicle over the time interval from 0 to 4, then, by definition, the average
velocity (in appropriate units) over that interval is 17. Notice that this is not the
simple arithmetic average of the velocities a t x = 0 ( f ( 0 ) = 1) and a t x = 4
( f ( 4 ) = 49), which is (49+ 1)/2 = 25. Nonetheless, there Zs a time c with
0 < < c 4 such that a t that timef(c) = 17. This simply says that the graph of
f ( x ) = 3 x 2 + 1 crosses the horizontal line y = 17 between x = 0 and x = 4
when 3x2 + 1 = 17, or when x = 4 / f i z 2.3. (See Fig. 6.31.) Thus the accel-
erating vehicle attains its average velocity once during the interval.
Similarly, in reference to the temperature in Fig. 6.30, the average tempera-
ture is between its low of 16 and its high of 30 and the temperature must have
been a t that figure at least once during the day. As you may already have
guessed, we are working toward a Mean-Value Theorem for Integrals. It is, in
- fact, the M V T for derivatives stated in integral form.

The Mean-Value Theorem for Integrals Supposef is continuous on


[a, b]. Then there exists at least one number c between a and b such that

(0, 1 )

j5 Proof: If F is a n antiderivative off on [a, b], then the M V T for derivatives ( F


is differentiable on [a, b ] ) guarantees at least one number c such that
Fig. 6 . 3 1

But F is a n antiderivative o f f , so F ( c ) = f ( c ) , and by the Fundamental


Theorem, Form 11, we know that

J b f ( x ) dx = F ( b ) - F ( a )
a

Hence we can rewrite (1) as


1 b
f(4=-
6-u
J f ( x ) dx

Iff(x) 2 0 on [a, b], then we can interpret the M V T for Integrals geometri-
cally. (See Fig. 6.32.) We can rewrite its conclusion in the equivalent form

Sincef(c)(b - a ) is the area of the rectangle based on [ a , b] with heightf(c), we


interpret the conclusion of the M V T for Integrals as stating the existence of a
Fig. 6.32 rectangle on [a, b] whose area equals the area betweenfand [a, b].

YROGHESS TEST 1

1. Find the point on the interval 12, 71 where the func- measured in hours 0 5 x 5 6 (noon to 6 P.M.). What
tionf(x) = x 2 - 2x + 1 assumes its average value. was the average temperature for the afternoon?
(First compute the average value.) What was the high and low for the afternoon? (Note
2. The temperature in degrees F on a given afternoon that the total days energy consumption-for air
is given by T = T ( x ) = - x 2+ 4x + 70, where x is conditioning, heating, and so on-depends upon the
SECTION 6.4: MEAN-VALUE THEOREM AND CHAIN RULE FOR INTEGRALS 213

true average temperature for that day, and not just ( b ) Compare this with the arithmetic average of the
the average of the high and low.) velocities a t t = 0 and t = 5.
3. ( u ) What is the average velocity, in feet per second, (c) Interpret the result of ( b ) in terms of the graph of
of a stone dropped from a bridge if it takes 5 s for the velocity function.
the stone to hit the water below?

THE CHAIN RULE FOR INTEGRALS


In Sec. 6.3, Example 3, when computing

i2 +(x2 4)3x dx

using the substitution u = x 2 + 4 , x dx = (1/2) du, we arrived at


8
u4

We then remarked that to compute the definite integral using the given limits of
integration, we would need to convert the antiderivative to a n expression
strictly involving x , thereby reducing the problem to

4)4 it = 480

Sometimes it will be convenient to switch the given x limits of integration to u


+
limits. In this case, since u = x 2 4 we know that u ( 0 ) = 4 and u ( 2 ) = 8 , so we
could also evaluate the definite integral without switching back, as follows:

Our ability to change limits of integration in this way is based upon the
following rule.

6.4.4 The Chain R u l e for Definite Integrals If u(x) is differentiable on an


interval I containing [a, b ] , then

This rule in turn is based upon:

6.4.5 The Chain Rule f o r Antiderivatives If F is a n antiderivative off, then

f(u(x))u(x)dx = s f ( u ) du = F ( u ) + C = F(u(x))+ C
In each of the previous statements we are assuming that a n antiderivative off
exists on a n interval J such that u ( x ) is in Jfor x in I , and that f is continuous on
J . We now prove (6.4.5), from which (6.4.4) follows.
Proof of (6.4.5): We apply the Chain Rule for derivatives:

D,[F(u(x))l = F(u(x))u(x)
=f(u(x))u(x) (since F =f)
214 CHAPTER 6: INTEGRATION

Example 2 Use the Chain pule for Definite Integrals to evaluate

Solutiop Let
1
u = x2 +1 -du = xdr
2
Now
x =0 implies u = O2 +1=1
and
x =I implies u = l2 + 1 =2
Hence
l1(x2 + 1 ) 4 dx~ = - t2 u4 du

31
10
We can also use the Chain Rule for Derivatives to differentiate functions
defined by definite integrals. Form I of the Fundamental Theorem states that

Thus, for examAle, if

then

However, given
X2+5 dt

l f i + 1
we can regard this definite integral as F ( x 2 + 5), where

Hence

= F'(x2 + 5 ) -D,(x2 + 5) (Chain Rule for Derivatives)


-
- 1
(2x1
@G+1
- 2x
- V F T T + 1
We could even have been asked to differentiate
dt
G(x) =
SECTION 6.4: MEAN-VALUE THEOREM AND CHAIN RULE FOR INTEGRALS 215

The first step w o u l d be to apply [6.3.4(a)],rewriting G(x) as

We w o u l d then proceed a s before.

PROGRESS TEST 2
~ ~~~

1. D e t e r m i n e t h e given definite integral using (6.4.4).

4'n

dFTTX2dx

In Problems 2 to 4 use Form I ofthe Fundamental Theorem and any additional necessary properties of the dejinite integral t o j n d G'(x).

2. G(x) = [ 3t dt+ 2
~ 3. G ( x ) =
X
1
(t2 + 1)dt 4.

SECTION 6.4 EXERCISES

I n Exercises I to IO, (a) determine the average value of the function on the given interval and ( b ) determine where in the interval f achieves this
average value.
1. f ( x ) = 2x + 5, 10, 101 2. f ( x ) = x 2 x + +
1, [1,4]
+
3- g ( x ) = x 3 x - 2, [ 2 , 71 4. h(x) = 6+
1, [0,4]
5 . J J = 1 - X , [-2,3] 6. f ( x ) = 5x4, [0,21
7 . f ( x ) = 5x4, [-2,2] (compare with Exercise 6) 8. y = 1 - x 2 , [-1, 11 (sketch)
9. y = x 2 - 4, [-2,2] (sketch) 10. f ( x ) = x d m , [O, 31

In Exercises 11 to 16 use the Chain Rule f o r Dejinite Integrals to evaluate the given dejnite integral.

13. 1 1 x ( x 2 - l ) 5 d x

14. s7xf/u2+15dX
1
15. [: r d x 16. 4' (x3 + 3x + 4)5(x2 + 1) dx
Oil
17. Suppose the amount of oil on hand at a storage depot, in (thousands of gallons)
thousands of gallons, is given as a function of time t (in
weeks) by the graph shown. By determining the number
of gallons as an algebraically defined function of t for
each convenient interval, determine the average number
of gallons on hand during the 10-week interval from
(10, 2)
t = 0 to t = 10. Hint: a = 0 , b = 10; use [6.3.5(a)]. P t
1 2 3 4 5 6 7 8 9 10
In Exercises 18 to 25 determine F'(x).

18. F ( x ) = 4' $2 + s + 1 ds 19. F ( x ) =


39+1
( w 2+ 2w) dw

+ 1 dt
20. F ( x ) = ['G d t 21. F ( x ) J
1
2'-1 t

23- F ( x ) = lX X2

d m d t Hint: Let c be a number

between 2 x and x 2 , and apply [6.3.5(a)].


216 CHAPTER 6: INTEGRATION

24. F(x) =
L
5x
(w2 + 2w)dw (see Exercise 23) 25. F(x) = i(f2
+ t ) dt

26. Show that the average value of a linear function 28. Repeat Exercise 27 for the same ball thrown with the
J ( x ) = mx +b
on an interval [c, d ] can be determined same initial velocity but from a 48-ft platform.
arithmetically, without integration. 29. Show that J:(xz - 5 x + 10)3(2x - 5)dx = 0 by using
27. A ball is thrown vertically from ground level at an initial (6.4.4).
velocity of 96 ft/s. What is its average velocity for the
duration of its flight? (Note that positive direction is
upward.)

Area Between Graphs


Section 6.5: of Functions, and Interpretations

Objectives: INTEOD CJCTION


1. Use definite integrals to We already established that the area of a region in a plane may in fact represent
determine the area of regions a wide variety of quantities in problems whose original formulation does not
whose boundaries are the graphs involve area. We shall now concentrate on techniques for computing areas
of given equations. efficiently.
Another payoff from our work with areas will be our ability to move from
2. Be able to choose the
approximations of quantities based upon Riemann sums to computations of exact
appropriate independent variable
quantities based upon definite integrals. The ideas underlying our work with
for a given problem.
areas underlie virtually all applications of definite integrals.

AREA BETWEEN CURVES


Suppose R is the region bounded byf(x) and g(x) (both integrable on [a, b ] ) , and
<
the lines x = a and x = b ( a b ) withf(x) 2 g ( x ) on [a, b]. To approximate the
area of R we subdivide [a, b] with a partition P into n subintervals and sum the
areas of the approximating rectangles determined by each subinterval (see Fig.
6.33). We let x: be any point in a typical subinterval [xi-l, x i ] determined by P
x = a x=b
and sketch in the typical rectangle.
Fig. 6 . 3 3 We have
Y

a Riemann sum for f ( x ) - g ( x ) . Letting llPl1 + 0, the approximations ap-


x proach the exact area as a limit. That is,
(22: + 5) -
: . 1 - 9)

Of course, with g ( x ) = 0 we have our earlier result for area.

Example 1 Determine the area of the region bounded by the curves y = x 2 - 9, y =


2x + 5 and the straight lines x = -2 and x = 1.
Solution First we draw a sketch of the region (see Fig. 6.34). Since 2 x +
5 2 x 2 - 9 on
+
[ - 2 , 11, we know that 2x 5 plays the role of f ( x ) and x 2 - 9 the role of g(x).
We then draw in a typical rectangle and form the approximating sum:

= c [(2x,T + 5) - ( x r 2 - 9)] Axi


n
A (R )
i=l
SECTION 6.5: AREA BETWEEN GRAPHS OF FUNCTIONS 217

Y
Hence, by ( 6 . 5 . 1 ) ,

t A(R)= s1
-2
[(2x + 5) - ( x 2 - 9)] dx

s,
1
- (2x - x2 + 14)dK
-

= 36 0

In practice we shall apply (6.5.1) in a more streamlined fashion, omitting


subscripts and superscripts on x in describing the typical rectangle, and passing
directly to the definite integral. The important thing is to identify the region
Fig. 6.35 and the typical rectangle clearly. Here and henceforth we shall ignore specific
units unless they play a special role in the problem.
Often the straight lines x = a and x = b are not given but are determined by
finding the points of intersection of the bounding curves.

Example 2 Find the area of the region enclosed by the curves y = 3x and y = x 2 .
Solution To find the points of intersection we set 3x = x 2 and solve, getting x = 0 and
x = 3. Thus the curves intersect at (0,O) and ( 3 , 9); see Fig. 6.35.
A typical approximating rectangle has a width Ax and height 3x - x2, hence
a n area (3x - x 2 ) Ax. By ( 6 . 5 . 1 ) , we know that the total exact area is given by

r 3x2
(3x - x2)dx = - - -
2 3:
PROGRESS TEST 1

Sketch the region enclosed by the indicated curves and 1. R : y = x3, y = 8 , x = - 1, x = 2


find its area. 2. R : y = f i , y = x 2

M O R E ON AREAS
It is important to realize that to use (6.5.1) we needf(x) 2 g ( x ) on [a, b ] . If, for
example, the region R were as pictured in Fig. 6.3.6, we would break it up into
three subregions and apply (6.5.1) to each, getting

x=o x = r x = d u=b
Fig. 6.36 R = R , U R, U R,
218 CHAPTER 6: INTEGRATION

You can anticipate the possibility of this situation by settingf(x) = g ( x ) and


checking for solutions between x = a and x = b. Then use a sketch to determine
whos on top.

Example 3 Determine the area of the region bounded by the curves y = 1 - x 2 and
y = x 2 - 1, and the straight lines x = -2 and x = 2.
Solution Setting 1 - x 2 = x 2 - 1, we obtain 2x2 = 2, so x = +1. We sketch the curves
(see Fig. 6.37), obtaining three separate regions:
R,: x 2 - 1 2 1 - x 2 on [-2, -13
R,: 1 - x 2 2 x 2 - 1 on [-1,1]
R,: x 2 - 1 2 1 - x 2 on [l, 21
Hence

A(R) = s-2
-1
[ ( x 2 - 1) - (1 - x2)] dx + s
-1
1
[(l - x2) - ( x 2 - I)] dx

+ J 2 [ ( x 2 - 1) - (1 - x2)J dr
1

= s-
-2
(2x2 - 2) dr + s-1
1
(2 - 2x2) dx + [ (2x2 - 2) dx
2

=[ 3 3 - 2x];: + [2x - 3.1]1,+ [ 3 3 - 2x]2

= [(-$ + 2) (++ 4)] + - (2 -a) 1

- (-2 + $)]
Fig. 6.37 + [(y- 4) - ($ - 2)]

=d
In some situations, when given equations in x and y describing a region, it
is convenient to regard x as a function ofy and turn (6.5.1) on its side. (See
Fig. 6.38.)
Suppose q ( y ) 2 P(y) on [c, d]. Then the approximating rectangles are
horizontal and have area [ q ( y r )- P(y:)] Ayi,so we have the Riemann sum
= c n

2 [ d Y f )- P ( y f ) lAYi
i=l

as a n approximation of A(R). Thus we conclude the following:

6.5.2

Example 4 Find the area of the region R bounded by the curvesy2 = x + 1 andy = x - 1.
Solution Considering y as the independent variable, the two functions are

x = q(y) = y 2 - 1 and x = p(y) = y +1


SECTION 6.5: AREA BETWEEN GRAPHS OF FUNCTIONS 219

Setting q ( y ) = p ( y ) , we obtain
y2- 1 =y +1 or ( y - 2)(y + I) = 0
Hence the curves intersect when y = 2 a n d y = - 1. The corresponding values
of x are x = 3 and x = 0, respectively. (See Fig. 6.39.)
Now y + 1 > y 2 - 1 on [-1,2], so a typical rectangle has area
[(y + 1) - ( y 2 - 1)1 A?. BY ( 6 . 5 4 ,
2

Fig. 6.38
'w)= J -1
[(Y + 1) - ( Y 2 - 111 4

= J'(y
-1 -y2 + 2) dy

To determine the area of the region in Example 4 (see Fig. 6.39) using x as
the independent variable and vertical rectangles, we would need first to express
' 2,
all the boundaries of the region Lis graphs of functions of x. In practice this
usually means solving the given equations for y , being careful to distinguish
between the various functions of x defined zmplicitly by a given equation. For
example, solvingy2 = x +
1 fory yields two functions of x :

y1 = d n above the x axis


x=o y2 = - d n below the x axis; see Fig. 6.40
Fig. 6.40
It is important to recognize that there is no single typical vertical rectangle in this
example. Between x = - 1 and x = 0, a typical rectangle hasyl = d x
on top and y 2 = - d x on the bottom, and hence has area [ \/x+l-
( - d m ) ]Ax. Between x = 0 and x = 3, a typical rectangle has
y1 = d x on top and y = x - 1 on the bottom, and hence has area
~'n
[ - ( x - l ) ] Ax . As a result the total area of the region must be given
as a sum of two integrals, one for each type of typical rectangle:

J
A(R) = J0 [m-
( - d Z T ) , h+ J3 [ d X - ( x - l)] dx
t x =y 2 - 2y
-1 0

You are invited to show these two integrals yield A ( R ) = 9/2.


To determine the area of the region enclosed by the graphs of

i_

,o>0)
-8,m -*f V-_*
~ I
x + 2y=y2 and x+y2=6y
we would need to sketch each horizontal-axis parabola, determine where they
cross, and then decide whether to use horizontal or vertical rectangles. A glance
at Fig. 6.41 tells the outcome. How many integrals would be needed if we used
Fig. 6.4 I vertical rectangles?

PROGRESS TEST 2

Find the area of the region R bounded by the curves 1. Treatingy as a function of x .
x = 9 - y 2 and x = y+ 3: 2. Treating x as a function ofy.
220 CHAPTER 6 : INTEGRATION

INTERPRETATIONS OF AREAS
IN BUSINESS AND ECONOMICS
The basic idea of a definite integral as a limit of Riemann sums, especially
when connected to the area between two curves, applies in business and
economics whenever a total is sought. Hence the total ~ (cost, revenue,
profit, and so forth) associated with producing units from a to b ( a <
b ) is the
definite integral of the ~ function from a to b (this function being
appropriately integrable, of course.)

MAXIMIZING PROFIT OVER TIME


In operations where the revenue, cost, and hence profits all vary considerably
but predictably with time, the determination of the maximum profit point
requires an analysis of all these factors as functions of time rather than as
functions of unzts produced (as was the case in Sec. 5.4).
Suppose the coal producer of Sec. 5.4, Example 1, realizes that the local coal
reserves are running out, and therefore the amount of coal, hence the rate of
revenue flow R(t) will decrease, the cost rate C(t) of mining it will increase, and
so the profit rate P(t) = R(t) - C(t) will decrease with the passage of time.
The major issues for the producer are: (1) How long should the mine be
operated to maximize profit? (2) What is the total profit to expect? Under the
assumption that production per unit time is constact, the answers to these
t (time) questions are found by examining the area between the graphs of R(t) and
C(t); see Fig. 6.42.
Fig. 6.42 Our experience with area and integration tells us that the total profit is

6.5.3 it (R(t)- C(t))dt

where to is the point in time when the marginal revenue equals the marginal
cost (marginal in this case with respect to time).
In practice, of course, the rate of change functions are predictions given as
average rates per unit time (daily, weekly, monthly, and so on), which we treat
as instantaneous rates of change-that is, as derivatives-in order to use
calculus techniques.

CONSUMER AND PRODUCER SURPLUS


In Chap. 5 we remarked that to get a consumer to buy a certain quantity x of
some commodity, the price must be set accordingly. From this point of view,
price is a function of x ,

This function, inevitably decreasing (the larger the quantity x to be sold, the
lower must be the price), is called a demandfunction. Similarly, the price at which
a supplier is willing to sell a quantity x is given by the supply function. In a
competitive situation the market equilibrium occurs when demand equals
supply-that is, at (xo, Po), where the demand and supply curves intersect (see
Fig. 6.43).
However, not all consumers have the same desire or use for a given product,
so the market equilibrium price Po is a bargain to some consumers who are
willing to pay more than Po for a given product. The measure of the total
bargain to the consumers is called the consumer surplus (CS) and is represented by
SECTION 6.5:AREA BETWEEN GRAPHS OF FUNCTIONS 22 1

i
I x0 I .x
@

Consumer Surplus Producer Surplus


(a, [h)

Fig. 6.43 Fig. 6.44

the shaded area in Fig. 6.44(u). (Actually, in practice all the curves involved,
hence the regions, are defined only for strictly positive x.)
Similarly, in a typical situation some producers would be willing to produce
and sell. at a price lower than Po. The total benefit to producers is called the
producer surplus (PS) and is represented by the shaded area in Fig. 6.44(b). Hence
12

8 6.5.4 CS = 4 20
D(x) dx - p o x o PS = poxo - 4
$0
S ( x ) dx

From the integrals in (6.5.4) we see that the consumer surplus is the potential
total revenue received by producers (if each consumer paid what he/she felt the
product was worth) minus the amount that consumers actually spent. Similarly,
- X
4 n the producer surplus is the difference between the revenue actually received and
the lower revenue that some of the producers would have been willing to accept.
CS and PS thus provide a crude measure of the benefits of competition and the
Fig. 6.45 lack of a totally auction-based marketplace.

Example 5 Determine CS and PS if the demand function is D ( x ) = 16 - x and the supply


function is: ( u ) Sl(x) = x 2 / 8 before a merger of two large competitors; and
( b ) Sz(x)= 3x2/4 after the merger.
Solution We sketch all three functions on the same coordinate axes in Fig. 6.45.
(a) Before the merger, CS, = J:(16 - x ) dx - 8 * 8 = 32 and PS, =
64 - J:x2/8 dx = 423. ( b ) After the merger, CS, = Jt(16 - X ) ~ - ~ X4 -
12 = 8 and PS, = - J$3x2/4 dx = 32. Hence the result of the merger is that
the consumer surplus drops by 24, from 32 to 8 (a loss of 75 percent). While the
producer surplus drops by lo$, from 423 to 32 (a loss of 25 percent), the 50
percent increase in price and the lower cost of producing 50 percent fewer units
lead to a potentially dramatic increase in profit for the producer. Of course, the
consumer pays 50 percent more for each unit. 0

PROGRESS TEST 8
-

1. Determine the time to to shut down a coal mine 2. What is the maximum profit from the mine?
given that the revenues are decreasing linearly from 3. Detcrminc thc consumer surplus and producer sur-
a rate (in thousands of dollars) of 50 per month to plus given that the demand function is D(x) =
zero when the coal runs out in 10 years, and given 10 - x and the supply function is S ( x ) = x2/5.
that the monthly cost rate is climbing according to
the formula C(t) = t2/24, where t is measured in
months.
222 CHAPTER 6: INTEGRATION

SECTION 6.5 EXERCISES

In Exercises I to 20 determine the area of those regions which are bounded by the given graphs.
1. y = x2, y = x, x = 1 / 2 , x = 4 2. y = x' + x + I, x = 0 , x = 3,y = 0
3. y = x 3 , y = x2 4. x z y 2 , x = I , x = 2
5. y = x 2 , y = 18 - x 2 6. y = &,y = x 3
7 . y = x - 2x + 4 , y = 0 8. x = y 2 - 1,y = x - 1
9. y = x 4 , y = 2x - x2 10. y2 = 4x, y = 2x - 4
11. y =x' + 2x + 1,y x'+ + 2x = 9, x = 3 12. 2x = 7 - y , y = x2 - 4 x +5
13. 2y - 5 = 4x2, x = 0 , y = 0 14. 2y = 4x3 + 5, x = O,y = 0, x =1
15. 2y2 + y = x + 1 , y - x = 1 16. y2 = x - 1 , x = O,y = k 1
17. y = x3 - x,y = 0 18. 2x2=2y+ l , x + 2 = 2 y
19. y z l / V 7 = T , y = O , x = 5 , x = 2 20. 9 2= 1 , x = 1, x = 4
21. Graphy' = x 2 - x 4 by following this outline:
( a ) Note symmetries with respect to each axis.
(b) Solve fory in terms of x and note the possible values
of x.
( c ) Determine intercepts and use dy/dx to describe the
graph in the first quadrant.
( d ) Use symmetry to complete the graph.
22. Repeat Exercise 21 f o r y 2 = x 2 ( 4 - x').
23. Determine the total area enclosed b y y 2 = x 2 - x4.
24. Determine the total area enclosed by y' = x 2 ( 4 - x').

In Exercises 25 to 32 determine the area of the region enclosed by the graphs of the given equations.
2 5 . y' = x 3 a n d y = 2x - 3
27. y 3 = x , x = y
29. 4y' = x2(4 - x 2 )
31. y2 = ( 3 - x)", x = 0 , y = 0 , x = 2

In Exercises 33 to 36 assume that vehicle V, has velocity u1 and vehicle V . has velocity u2. I n the same way that the area between a nonnegative velocity
function and the interval [a, b] represents the distance traveled over the time interval [a, b ] , the area between two velocity functions nl(t) and v 2 ( t ) with
Vl(t) 2 v,(t) on the interval [a, b] represents the distance that the vehicle V, moues ahead of the vehicle V, over the time interval [a, b].
33. Suppose vl(t) = 15t and v2(t) = l o t over [O, 201. Com- 40. Suppose the marginal revenue and marginal costs are
pute the distance V, pulls ahead of V2 in the interval given ( x is thousands of items produced) as R ' ( x ) =
[O, 201 using: 5x +1 - x z and C'(x) = 4x - x 2 , determine the total
( a ) The idea just described. profit for the first 4000 items produced.
( b ) Elementary (noncalculus) means. 41. Given a demand function D ( x ) = 2 +
\/9-x a n d a
3 4 . Justify the statement made before Exercise 33 using supply function
interpretation of the areas of approximating rectangles.
3 8 . Suppose vl( t ) = 4t and v2( t ) = 3t on [ 1,4] with V, 200
units of distance behind V2at t = 1. Does V, catch u p to
S(x) = (a -I
for x 2 1
for 0 5 x 51
V, over the interval [ I , 4]? compute the consumer and producer surplus. (Be careful
36. Suppose ul(t) = t + 1 , v2(t) = ( 2 / 3 ) f i a n d at t = 0, V, at x = 1 ; separate integrals are required-perhaps.)
is 63 units of distance behind V,. 42. Suppose that the marginal revenues from an oil field are
( a ) How far apart are they at t = 4? +
given by R ' ( x ) = x 3 - 3 x 2 2x + 10 a n d the marginal
( b ) How far apart are they at t = 9? costs are given by C ' ( x ) = - x 3 +
4x2 - 3x + 10 ( x
( c ) How long does it take for V, to be 145 units ahead measured in thousands of barrels per day). Beyond a
of V,? certain point, when marginal cost exceeds marginal
37. What is the area of the triangle with vertices ( 1 , I ) , revenue (the shutdown point), the government steps in
(6, 3), (3,9)? Use a definite integral. and pays the difference between marginal cost and
38. Repeat Exercise 37 for the triangle with vertices ( - 1, 2), marginal revenue until they are equal again.
( 1 , 8 ) , ( 4 , -2). ( a ) What is the shutdown point and the total profit a t
39. Suppose the marginal cost for producing the xth item is that time?
2 + l/(x+ 1 ) 2 (dollars). What is the total cost of pro- ( 6 ) What is the total profit during the subsidized period?
ducing the first 100 items? ( c ) Sketch R'(x) a n d C'(x).
SECTION 6.6: VOLUMES OF SOLIDS OF REVOLUTION 223

Section 6.6: Volumes of Solids of Revolution

Objectives: THE DISK METHOD


Set up and evaluate definite We can think of a solid sphere as being generated by revolving the region
integrals that describe the volumes enclosed by a semicircle about its diameter. Similarly, a solid right circular cone
of solids generated by revolving results from revolving the region enclosed by a right triangle about one of its
plane regions about an axis, using legs (see Fig. 6.46). In this section we shall deal with more general solids
either the disk or the shell method, generated by revolving a given plane region about a given line. Such a solid is
as appropriate. known as a solid of revolution, and the line is called the axis of revolution. As in our
treatment of area problems, we assume the region to be bounded by the graphs
of functions.
Let us now determine the volume of the solid generated by revolving about
the x axis the region below the graph ofy = f ( x ) from x = a to x = b, wheref ( x )
is continuous and nonnegative on [a, b]; see Fig. 6.47.
As we know, the area under the graph can be approximated using vertical
rectangles of width Axi (see Fig. 6.47) based upon a partition P of [a, b]. When
each rectangle is revolved about the x axis, a disk is generated (see Fig. 6.48).
The volume of a disk of radius R and thickness h is rR2h, so the volume of a
typical disk generated by such a rectangle is

Since the sum of the rectangular areas approximates the area of the region
being revolved, the sum of the volumes of the disks approximates what we shall
define as the volume of the solid generated. Thus we say the approximate
volume of the solid is

But this is a Riemann sum, and if we take finer and finer partitions P of
[a, b ] , we get better and better approximations. Hence it makes sense to define
the actual volume V of the solid of revolution to be the limit, as 11 PI1 + 0, of
such Riemann sums. But, by definition, this limit is a definite integral.

Definition

4'
Y =f(x)
t /

Fig. 6.46 Fig. 6.47 Fig. 6.48


224 CHAPTER 6. INTEGRATION

Example 1 Determine the volume of a sphere of radius r generated by rotating the positive
semicircle determined by x 2 + y 2 = r 2 about the x axis, as shown in the sketch.
Solution We first solve for (positive)y in terms of x : y = f ( x ) = d m . By (6.6.1),
Y

The disk approach also works for regions revolved about the vertical axis. In
this case the generating rectangles are horizontal, so the given equations will
need to be solved for x in terms ofy.

Example 2 Determine the volume of the right circular cone generated by revolving the
following region R about they axis. R is enclosed by the x a n d y axes and the
line x + 2y = 4.
Solution We first sketch the region and draw in a typical horizontal rectangle, as in
Fig. 6.49.
A typical horizontal rectangle has width Ayi and length x = 4 - 2y,. Re-
volved about they axis, it generates a disk of volume 71(4 - 2yi)' Ayi. The sum
of such volumes

c
n

i=l
T ( 4 - 2y,)Z ayi

Fig. 6.49 is a Riemann sum that leads to a definite integral. Thus, by the horizontal
version of (6.6.1), we have

Example 3 shows that the volume of a solid of revolution depends not only
on the size of the region revolved but also on the axis of revolution. In part ( b )
we use disks with holes-that is, washer-shaped solids.

Example 3 Let R be the region bounded b y y = x 2 , x = 1, and the x axis. Compare the
volumes of the solids generated by revolving R about ( a ) the x axis, ( b ) the
y axis.
SECTION 6.6: VOLUMES OF SOLIDS OF REVOLUTION 225

Solution ( u ) Revolving R about the x axis, we can apply (6.6.1) directly (see Fig. 6.50) to
get

~ ( xdx ~= ) ~ 5 1
x 4 dx = T =-
77

5
(6) When rotated about they axis, a typical horizontal rectangle has its right
extremity on x = 4;
see Fig. 6.51. It is clear that a typical rectangle
generates a washer-shaped solid; see Fig. 6.52. The outside radius is 1 and
the radius o f the hole is 6.
Now the volume of the washer-shaped disk is
the volume o f the solid disk minus the volume of the hole:

Fig. 6.50 ~ ( 1Ayi


) ~- T ( 6)'
Ayi = ~ ( - 1y i ) Ayi
The sum of such volumes is a Riemann sum:

Thus

l ( . =v
' l - y ) dy

I The region generates less volume when revolved about the x axis than when
Pig. 6.5 1 revolved about they axis. The region is-in a sense to be defined precisely using
integrals in Chap. 12-"closer" to the x axis than to t h e y axis. 0

We can also use similar techniques to compute volumes of solids generated


by revolving regions about vertical or horizontal lines other than the coordi-
nate axes.

Example 4

Solution

A
1 = x-

x = J j x = l
t l I I + I

2 x X,Z +2

, = -2 L1-_-

Fig. 6.52
226 CHAPTER 6: INTEGRATION

washer is
a(xi2 + 2 ) 2 Axi - 7 7 ( 2 ) 2 Axi = a[xi4 + 4xi2] Axi
and the total volume of the solid is given by

a 6' (x4 + 4%) dx = a [I


+
-
4
--x
31,=za
23

PROGRESS TEST 1

1. Let R be the triangular region bounded by the lines revolving the following region R about the x axis: R
y = ( r / h ) x , x = h, and the x axis. Determine the +
bounded by 2y = 4x3 5, the x andy axes, and the
volume of the right circular cone of height h and line x = 1.
radius r generated by rotating R about the x axis. (6) Repeat ( a ) but with R revolved about the line
2. ( u ) Determine the volume of the solid generated by y = 5.

THE SHELL METHOD


Suppose a solid is generated by revolving about they axis the region between
the graph ofy = f ( x ) and the interval [a, 61 (u 2 0), withf(x) 2 0 and contin-
uous on [a, 61. Each vertical rectangle in this case generates a right circular
cylindrical shell (which is nothing more than a washer-shaped solid); see
Fig. 6.54.
But before working the problem in terms off(x), we note that the volume of
X
a right circular cylindrical shell of height h, inner radius r, and outer radius R
is the volume of the outer cylinder minus the volume of the inner cylinder. We
write this volume in a way that will eventually lead to a new integral formula
Fig. 6.54 for the volume of a solid of revolution:

6.6.2 aR2h - ar2h = 7i(R + r)(R - r)h = 2a- +


2
rh(R - r )

+
Now ( R r ) / 2 is the average of the inner and outer radii and R - r is the
thickness of the shell. This agrees with the approximate volume we get by
cutting the shell parallel to its axis and flattening it out to a rectangular solid
whose width is 277 times the average radius of the shell (which is its approximate
circumference). The height of the solid is h and its thickness is R - r (see
Fig. 6.55).
Translating back to our original situation (Fig. 6.54), where as usual we are
given a partition P of [a, b ] , we see that the outer radius is xi, the inner radius
Thickness +
xi-l, the average radius (xi xiPl)/2 = xs, the heightf(xs), and the thickness
Fig. 6.55 xi - xi-l = Axi. Hence, by (6.6.2), a typical rectangle generates a shell of
volume
6.6.3
Adding all these volumes, we get the Riemann sum
n
C 2ax,Tf(x,T) Axi
i=l

which approximates the volume of the whole solid. Thus the exact volume is
given by

n ,b
6.6.4
SECTION 6.6: VOLUMES OF SOLIDS OF REVOLUTION 227

We shall now determine the volume of the solid generated by revolving


about they axis the region of Problem 2 in the Progress Test above. (The disk
approach would require messy calculations and two separate integrals.)

_____________ ~ ~ ~ ~ _ _ _ _ _ _ _ _ _

Example 5 +
The region R bounded by 2y = 4x3 5, the x a n d y axes, and the line x = 1 is
revolved about they axis. Using the shell approach, determine the volume of
the solid generated. (See the Progress Test solution for a sketch.)
Solution Heref(x) = y = 2x3 + 5/2, a = 0, b = 1, so

The choice of whether to use the disk or shell approach on a problem will
usually be dictated by which is easier-either easier to set up or easier to
integrate. In any case, the first step should be to sketch the region to be
revolved.

Example 6 Let R be the region enclosed by the curves y = x 2 - 2x and y 2x = 9. +


Determine the volume of the solid generated by revolving R about the line
x = 4.

Solution We first sketch R, determining the intersection of the curves by setting


x 2 - 2x = 9 - 2x. Here we get x = 2 3 , s o y = 3 and 15, respectively. (See
Fig. 6.56.)
If we use the disk method, we need two separate integrals because two types
of typical rectangles are involved, one abovey = 3 and one belowy = 3. Hence
we shall use the shell method, which requires a single integral.
Ignoring subscripts and superscripts, a typical rectangle has height
(9 - 2x) - (2- 2x) and its center is at a distance 4 - x from the axis of
J
revolution. By (6.6.3) it generates a shell of volume
2~(-
4 x ) [ ( 9 - 2x) - ( x 2 - 2x)] Ax
The sum of volumes of such shells generated by rectangles from x = -3 to
x= 3 approximates the total volume. The exact volume is thus given by

J 32
-3
~ ( 4- x)[(9 - 2x) - (x2 - 2x)] dx = 2~
s
-3
( x 3 - 4x2 - 9x + 36) dx

= 2 ~ [ 1 4 4 ]= 2 8 8 ~ 0
Sometimes the heart of the matter in applications is setting up the region
Fig. 6.56 and the axis to generate the appropriate solid. (See Sec. 6.3 Exercises.)
228 CHAPTER 6: INTEGRATION

PROGRESS TEST 2

1. Determine the volume of a sphere of radius r using revolving a b o u t the line x = 3 the region bounded
the shell method. Revolve about the x axis. b y y = x 2 - 2x + 1 andy + x 2 + 2x = 9.
2. Determine t h e volume of the solid generated by

GENERAL REMARKS ON SETTING U P INTEGRALS


Just as with area problems, volume problems offer a choice of approach, which
automatically gives us the choice of independent variable. This c a n sometimes
b e confusing, but if we take the trouble to use a sketch and include a typical
rectangle, the rest will follow naturally. The width ofthe typical rectangle determines
the independent variable. For example, if the width happens to b e Ayi, then the
dominant variable isy, and the given equations will need to be solved for x in
terms o f p and the limits of integration determined accordingly.
Another caution: W e are using the phrase typical rectangle here in a
precise way; all others must be like it in terms of its extreme ends (upper and lower, or
left and right). Of course, some problems may need to be broken into parts with
one typical rectangle for each.

SECTION 6.6 EXERCISES

1. Determine the volume of the solid generated by revolv- 3. Determine the volume of the solid generated by revolv-
ing about the x axis the region bounded by the curves ing, about the x axis, the region bounded byy = x2 7, +
x = y and y = x2 using ( u ) the disk method, and y = 16,using ( a ) the disk method, and (6) the shell
( 6 ) the shell method. method.
2. Repeat Exercise 1 revolving about the line x = -2. 4. Repeat Exercise 3 revolving about the liney = 1.

In Exercises 5 to 36 determine the volume V of the solid generated by revolving the given region R about the given axis.
5 . R bounded byy = x + 1 and the x a n d y axes, about x 28. R bounded by x2 = 4y,y = 4, about x axis.
axis. 29. R bounded by y = x 2 - 2x,y axis, y = 3, about y axis.
6. R as in Exercise 5, about y axis. 30. R bounded by y 2 = x, x = y3, about x axis.
7 . R bounded by y = 2x, x = 4,and y = 0, about y axis. 31. R bounded by y 2 - 8x = 16,y 2 +
16x = 16,above x
8. R as in Exercise 7, about x axis. axis, about x axis.
9. R as in Exercise 7, about line x = 5. 32. R bounded by f(x) = l/(x - 1)3, x = - 1, coordinate
10. R as in Exercise 7, about liney = 6. axes, about x axis.
11. R bounded byf(x) = x-~~, x = 1, and x = 2, about x 33. +
R bounded by y = x 2 3x - 6 and x +y + 3 = 0,
axis. about x axis.
12. R as in Exercise 11, about y axis. 34. R as in Exercise 33, about line x = 4.
13. R bounded by y = x and y = 4x - x2, about y axis. 35. R bounded by y = and x and y axes, about x
14. R as in Exercise 13, about x axis. axis.
15. R as in Exercise 13, about line x = -5. 36. R bounded by y = 3x2 - x3 and x axis, about y axis.
16. R as in Exercise 13,about line y = -6. 37. Determine the volume of the torus generated by revolv-
17. R bounded by x = y 2 andy = x2,abouty axis. +
ing the region enclosed by the circle (x - 5)2 y 2 = 9
18. R as in Exercise 17, about x axis. about they axis.
19. R as in Exercise 17,about line x = 2. 38. Derive a general formula for the volume of a torus of
20. R as in Exercise 17,about line x = 200. cross-sectional radius 7 and central radius R. ( R is dis-
2 1. R bounded by y = x3, x = 3,and y = 0, about x axis. tance from the axis of rotation to the center of generat-
22. R bounded by y = 4 - x 2 and x axis, about x axis. ing circle.)
23. R bounded by y +
4x2 - 8x = - 1, x = 0,y = 0, and 39. Determine the volume remaining of a sphere of radius a
x = 1, about x axis. with a cylindrical hole of radius 7 centered on a diameter
24. R as in Exercise 23, abouty axis. of the sphere drilled through the sphere ( 7 < u).
25. R bounded b y y = x and 2y = x 2 , about x axis. 40. Suppose a sphere of radius r is cut by a plane leaving a
26. R as in Exercise 25 abouty axis. cap-shaped piece of maximum thickness h. Show that
27. R as in Exercise 25 about line x = 4. the volume of this cap is (7rh2/3)(3r - h). (This formula
SECTION 6.6: VOLUMES OF SOLIDS OF REVOLUTION 229

is useful in determining the quantity of liquid in a 42. Repeat Exercise 41, revolving about the x axis.
spherical tank.) 43. Repeat Exercise 41 for the vertices ( 2 , -4), (5, -7), and
4 1. Determine the volume of the ring generated by revolving ( 1 0 , -4), about they axis.
the triangular region with vertices ( 5 , 6), ( 5 , l ) , and 44. Repeat Exercise 43, about the x axis.
(9,3) about they axis.

CHAPTER EXERCISES

Review Exercises
In Exercises 1 to 40 determine the indicated antiderivative.

1. J(-$ + x 3 - f&)& 2. J(2x2- x5

5. J ( t + 2)(3t - 1 ) d t 6. J ( 2 w + 2)2dw 7. JZ3 +$- dz 8. J(4x + l)lodx


9. j ( x 2 + 2 ) 2 / 3 xdx 10. J(w + l ) ( w z + 2w + 6 ) 5 d w
dt
11- J(3t + 5)3 12. f(3 - x2)4 5 x dx 13. J(T + 1)3t dt 14. J ( x 3 + 1)lI3x5dx
15. J(x2 - 4x + 4 ) l S dx
21. ~ ( - 31)4dx
~ 22. J 5 1 d x

23. jx2(x3 + 6)
dx 24.
x dx

dx
27. Jtq-dt 28. J v 2 m d v 29. J(4x3 - 5 ) 5 x 2 d x 30.

31. J V t 3 - t 2 +1 32. J[(w2 - 2w)& - ( w 2- 2 ~ )dw~ ]

33. J d m ( 1 - -$) dx 34. Jdx4 - 4x3 + 12x2 - 1 ( 8 x 3 - 24x2 + 48x) dx


35. JT2dy 36. S(-x21 - x33 + x 3 ) d x
-
+ 12x3 - 5
d~4
38. J x 3 q m d x

39. Jx5 + x4 + x3 - 3 & 40.


x3 dx
x3
41. ( a ) Sketch the graph off(x) = x 2 + x + 1 on [0,5]. 42. Follow the same format as in Exercise 41 forf(x) =
( b ) Find the Riemann sum associated with the parti- +
x 3 - 3x2 2x on [0,4] with P = (0, 1 , 2 , 3 , 4 }
tion P = (0, 1 , 2 , 3 , 4 , 5 } and (x:) = (0, 1.5, 3, 3.5,5}. (x;) = {$, 1 , 2, 3.5). What is the area of the region
Sketch. bounded byy = f ( x ) andy = O?
( c ) Find s,
and gp for this partition. Sketch.
( d ) Use the Fundamental Theorem to determine the
area betweenf(x) and [0,5].

In Exercises 4 3 to 46 evaluate the given limits.

43. lim
n-m c7
i=1
32
44. limz
n-m .
a=1
2i2+i+3
5n3 n-m
i=1
46. limz- 8i+6
n-m.
1= 1
n2

In Exercises 47 and 48 use DeJinition (6.3.1) and thejnite-sum formulas to evaluate the given integral.

47. l4(3x2 + 1 ) dx 48. s: ( 1 - 2x2) dx


230 CHAPTER 6: INTEGRATION

In Exercises 4 9 to 64 use the Fundamental Theorem (and the Chain Rule fz necessary) to evaluate the given dejnite integral.

49. iv8dt 50.


-1
1
x3dx 51. s6
0
(3x2 + 2x + 2 ) dx 52. 4 (22 +
54. l5 (.3 -32 +2 ~ dx) 55. s 4 x m d x 56- 4 4x2-x+2
x4 dx

dw 58. lb + (3x 1 ) dx 59. i5 Ix - 21 dx 60. s x x v m d r


0

63. J 2 x 2 d m d x 64. j4+- l~


0 -4

In Exercises 65to 72 solve the given differential equation with the given conditions on the solution.

65. dx
k X
- 2 + X + 2 y(0)=1
d2y = 3x
66. -
dx
+7 y(0) = 3,y(O) = 4

d2y = 4
68. - y(1) = 2 , y ( l ) = 3
dx

71. f (x) = 3 x 2 f ( 0 ) = l , f ( l ) = 3/4 72. f ( x ) = x - 1 f ( 0 ) = I, f ( 1 ) = 3/4


78. The acceleration of a ball rolling down a certain 74.Repeat Exercise 73 for a ball rolling with acceleration
curved inclined plane is given by a(t) = 8t - a(t) = 1 +
2t - 6t2.
3t2 cm/sz. Assuming that the distance (in centimeters) 75. Derive a formula (ignoring air resistance] for the
is measured positively from the starting point at the height of a bridge as a function of time t in seconds it
top of the plane, how far does the ball roll after - takes a rock dropped from the bridge to hit the water
( a ) 2 s, ( b ) 3 s, ( 6 ) 4 s, and ( d ) 5 s? Verbally de- below.
scribe ( e ) the velocity and (f)the distance rolled
for this ball.

In Exercises 76 to 79, detemzine F(x).

76. F ( x ) = 4;2 d m d t 77. F ( x ) = i3,+ (t3 1)Sdt

79. F ( x ) = i7 + dx4
dx
x2 +9
In Exercises 80 to 85, determine F(s).

80. F(s) =
dx
81. F ( s ) = J d--s
m d x 82. F ( s ) = i4 f / m d t

83. F(s) = s3
0
d m d x 84. F ( s ) = J
s2+1 dx2 +x + 1
dt

In Exercises 86 to 89, ( a ) determine the average value ofthe givenfunction on the given interval and ( b ) determine where on that interval the function
achieves that average value.
86. f ( x ) = x 3 +1 on [ - 2 , 2 ] 87. g ( x ) = 8 - x 2 on [-3, 31
88. h ( t ) = f i o n [ l ,91 89. f ( t ) = 3t2 - 1 on [ - 1 , 3 ]
90. ( a ) Sketch the function f ( x ) = w o n [-1, 11.
( b ) Determine J w d x .
-1
(c) What is the average value off (x) on [ - 1, l ] ?
( d ) Where does f ( x ) achieve this average value?

In Exercises 91 to 98 determine the area of the region bounded the given curves.
92. y = / x 3 - 4 x 1 , ~= 0
94. y = x3,y = 3x2 - 2x
SECTION 6.6: VOLUMES OF SOLIDS OF REVOLUTION 23 1

95. y = 6 , y = x / 2 96. x = 1 -y2, x = y 2 - 1


97. y2 = 4 - x , x +
4y = 4 98. y = Ix/,y = 1 - x 2

In Exercises 99 to 119 set up and simpl& but do not evaluate, the dejinite integral giving the requested quantity. The accompanyingjgure (used in
Exercises 99 to I I9), involving the lines x = 0, x = I, y = 0, y = I, y = x2, a n d y = x3, describes several regions as indicated, including I, II,
and III. By the regions I + 11 and 11 + 111 we mean the enclosed regions outside o f 111 and I respectively. The region I + 11 + 111means the whole
one-by-one square.
Y
99. Area o f (a) I ( b ) 111 ( c ) I1 + 111 (d) I + I1 ( e ) 11.
t

I
In Exercises I00 to 109, by the disk method, the volume of the solid generated by revolving the given region.
100. I about the x axis 101. I1 + 111 about the x axis
102. I1 about the x axis 103. I + I1 about they axis
104. I about the liney = 1 105. I11 about the x axis
106. +
I I11 about the x axis 107. I about they axis
108. I11 about they axis 109. I11 about the h e y = 1

In Exercises I I O to Il9, by the shell method, the volume .f the solid generated by revolving the given region.
110. I about they axis 11 1. I1 + I11 about they axis
1 12. I1 about they axis 113. I about x = 1
114. I about the x axis (compare with Exercise 100) 115. I11 about the x axis (compare with Exercise 105)
116. I11 about they axis 117. I + I1 about they axis
118. 111 about x = 1 119. I1 about x = 1

In Exercises I20 to I25 determine the volume of the solid generated by revolving the given region R about the given axis.
120. R bounded by 4 x 2 + 9y2 = 36, about the x axis (Be 122- R bounded by y 2 + 1 = x , y +
x = 7, and x = 10,
careful not to compute double the volume.) about x = 10.
121. R bounded by y/2 = m,
the x and y axes, and 123. R bounded byy2 = l/x, x = 1, x = 2, and the positive
y = 2, about they axis. x axis, about x = 1.
( a ) The smaller region R bounded by x 2 + y 2 = 4 and 124. R bounded by x = y 2 1 andy + +
3 = X , about they
x + y + 2 = 0 about they axis. axis.
( b ) Repeat for the larger region (be careful to avoid 125. Same R as Exercise 124, abouty = -1.
duplication).

Miscellaneous Exercises
1. If f ( x ) = - f ( x ) for all x show that J b a f ( x ) dx = 0. 4. A particle is falling with a velocity v(t) = 1 + 2t.
2. Given that f ( - x ) =f ( x ) for all x and Ji
f ( x ) dx = 3, ( a ) Find the average velocity with respect to time on
find JZ7f ( x ) dx. the interval 0 5 t 5 4.
3. Find ( b ) Assuming that s(0) = 0, find the position function
s = s ( t ) of the particle.
1 6 6
lim E T
n-m .
2=1(n 1
5. Suppose the second derivative of a function of x is 6 x
and that the function passes through the point (1,4)
(Hint: Consider f ( x ) = 6and express the limit as a with slope 4. What is the function?
definite integral.) 6. Repeat Exercise 5 with second derivative equal to
32 CHAPTER 6: INTEGRATION

- 12x + 2 and the function passing through (0;4) with enclosed by the curves y = 4 - x 2 and = x2 -
slope 1. 3x + 2, what is its total mass?
. Given F ( x ) = 1; 5 t dt; determine ( u ) F(x) and [J;(t2~1)d t l
( b ) F ( x ) (c) Sketch F ( x ) from Y = 1 to x = 4. 17. Determine G(x). where G ( x ) = f ds.
1. +
Repeat Exercise 7 for F ( x ) = S16(t4 t?) dt. J3

1. Suppose that a vehicle decelerates at the constant rate 18. Determine F(x), Lvhere F ( x ) = G(J;(2t2 + 1) d t ) .
of 8 ft/sz from a velocity of 88 ft/s (60 mph). 19. Suppose J is continuous everywhere, with

b.
( u ) How many seconds are needed to stop?
( b ) How far does the vehicle travel over this period?
Repeat Exercise 9 for a deceleration of 4 ft/s2 from
i.!.i.)
dx = 0

60 ft/s2. for all choices of a and b. Show7 thatJ(x) = 0 for all 7


I. Repeat Exercise 9 for a nonconstant deceleration of in R.
4t ft/s2 from 88 ftis. 30. Given
!. Suppose vehicle V, has acceleration a, and vehicle Vz
has acceleration a2 in appropriate units of distance per
second-squared. Given that V, is 100 units of distance
behind V, traveling at the same velocity as V2but tvith
acceleration u l ( t ) = 4 t j xvhile u,(tJ = t . ho\\T long does
it take V, to overtake V,?
1. Repeat Exercise 12 but with V, traveling at 10 units/s
faster than V, and a l ( t ) = t = a z ( t ) ,
L. Repeat Exercise 12 but with u,(t) = t / 2 , a z ( t ) = t. (Be
careful.) 22. A tank in the shape of a hemisphere of radius 6 m is
i. Interpret the functions and results of Exercises 1 2 to 14 being filled with Mater at the rate of 113 m3/s. How
graphically. fast is the radius of the surface increasing at the instant
i. Given a uniform plate whose density (mass per square \vheA the depth is 4 m ? (See Sec. 6.6, Exercise 40.)
unit) is rn and has the same shape and size as the region

CLF-TEST

1. Evaluate the following indefinite integrals:

(a) J d-(1 + 2)dx (6) J- (G+ 1I3j dx


X2 fi
2. Use the Fundamental Theorem to evaluate the following definite integrals:

4 dS-TT
4 3u2du
(6) s1d
-1
xdx
m
3. Find the area of the region bounded by the curvesy = x 3 - x 2 andy = 2x.
t
4. For x in (--a, co) define F ( x ) =
0 dFTTdt.
(u) Determine F(x).
+
( b ) Let g(s) = F( fi s2). Find dg/ds.
5 . An object is thrown upward with a n initial velocity of 36 ft/s from a height
80 ft above the ground.
( u ) How high does the object rise?
( b ) At what velocity does it strike the ground?
6. ( a ) Determine the average value of the functionf(x) = 3 x 2 + 2 x 1 on +
the interval [0,4].
( b ) Where in [0,4] does f achieve this average value?
7 . Suppose R is the region enclosed by the graphs ofy = 2x2,y - 3 = 5 x , and
x = 0. Determine the volume of the solid generated by revolving R about
(u) the x axis, ( b ) they axis, and (c) the line x = 4.
233
234 CHAPTER 7: TRIGONOMETRIC, EXPONENTIAL, LOGARITHM FUNCTIONS

Trigonometric Functions-
Section 7.1: Definitions and Properties

Objectives: MEASURING ANGLES


1 . Convert degree measure to We take as a starting point the notion of a half-line (we shall call it the generating
radian measure and vice versa. line) fixed at one end and generating an angle by sweeping in a plane from some
initial position to some terminal position. For convenience we often assume the
2. Define and graph each of the six
initial position is the positive horizontal axis of a given s-t coordinate system for
trigonometric functions, noting the
period and any points where such
the plane. We shall call this initial position the initial side of the angle and the
functions fail to be defined.
terminal position the terminal side of the angle. Following custom we call a
rotation of the generating line in the counterclockwise direction positive and a
rotation in the clockwise direction negative. (See Fig. 7.1.)
Afull positiue rotation, one in which the generating line rotates in a counter-
clockwise direction until it coincides with the initial side, is a unit of angle
measure. Another, more common unit of angle measure, the degree, is defined
as 1/360 of a full positive rotation. The degree was handed down to us from the
Babylonians, who did their calculations in base 60.
Far more important for our and most scientific purposes is radian (rad)
measure. For reasons that will become clear as we go along, radian measure is the
most practical measure for the calculus.
To define radian measure, we assume that a unit circle is centered at the
origin of the s-t plane. We shall denote by P = (s, t ) the intersection of the
generating line and the given unit circle. As the generating line sweeps out a n
angle, the point P moves along the circle beginning on the initial side at (1,O).

7.1.1 D e f i n i t i o n The directed length of the path traveled by P on the unit circle as
the generating line sweeps out the angle is the radian measure of that angle.
Length along the perimeter of the circle is measured positively from (0, 1) in a
counterclockwise direction, and negatively in a clockwise direction.

Since the circumference of the unit circle is 277, the directed distance traveled
by P in a full positive rotation is 27r. Thus a full positive rotation measures
4
27r z 6.28 rad. The directed distance traveled by P in of a full negative
rotation is i(-27r) = -a/4, so 4of a full negative rotation measures
-7r/4 z -0.785 rad. In Fig. 7.2 are some sample angles, with both the radian
measure and the degree measure given for each.
If we rotate the generating line 1 2 full positive rotations, the directed
distance traveled by P along the circle and the radian measure of the angle
generated is
5 5
-(2~)
4
-T
2
z 7.85
Terminal
side

side
Initial
side (Negative angle) Initial
side

(0)

Fig. 7.1
SECTION 7.1: TRIGONOMETRIC FUNCTIONS 235

t t t

t
S

1 / 2 full positive rotation 114 full neg-ative rotation 1 / 1 2 full positive rotation
radian measure = (1/2)(2n) = K I3.14 radian measure = 1/4 ( - 2 ~ )= -xi2 % -1.57 radian measurr = (1/12)(2n1 = n/6 I0.52

180 degrees -90 degrees 30 dcgrees


Fig. 7.2
t
Since there is no restriction on the size and direction of the rotation of the
generating line, the directed distance traveled by P and hence the radian measure
t of an angle, can be any real number. In Fig. 7.3 we have marked off angles of
measures 1 to 6 rad.
Given any angle, its degree and radian measures must be the same propor-
tion or multiple of the measure of a full positive rotation. Hence, letting D be
the degree measure of the angle and R the radian measure, we have
S

which we can simplify a bit to get

Fig. 7.3

This simple relationship lets us convert one measure to the other through
simple substitution. For example, given a n angle of 65", & = R / v , so
R = ( 1 3 / 3 6 ) ~is its measure in radians. Often the measure of an angle in
radians is not reduced to decimal form but is left containing 77- as a factor.
We often identify an angle by its measure and use abbreviated phrases like
"consider the angle 77-/4" instead of "consider the angle whose radian measure
is 77-/4."

PROGRESS TEST 1

4
3. Give the radian and degree measure of the angle
9
consisting of rotations.
4. Convert to radian measure: ( u ) 45", ( b ) 30",
( c ) 60, ( d ) 250, ( e ) -75". (Leaver as a factor.)
5 . Convert to degree measure: ( a ) 77-/4 rad, ( 6 )
77/12 rad, (c) 0.35 rad, ( d ) -41 rad, ( e ) 62,000
rad. (Put your answer in decimal form.)
236 CHAPTER 7: TRIGONOMETRIC, EXPONENTIAL, LOGARITHM FUNCTIONS

THE TRIGONOMETRIC FUNCTIONS


Assume we have an angle of x rad whose terminal side intersects the unit circle
at the point (s, t), as in Fig. 7.4, which contains the usual labels for the four
quadrants of the plane as well.
We can use the coordinates (s, t ) as the basis for the six trigonometric
functions of x defined as follows, with their respective standard abbreviations
given parenthetically.

7.1.3 Definitions of the Trigonometric Functions


1
sine (x) = t (sinx) cosecant (x) = - (csc x)
t
1
cosine (x) = s (cosx) secant (x) = -- (sec x)
S

t S
tangent (x) = - (tanx) cotangent (x) = - (cot x)
S t
We shall now examine and graph the function sin x for 0 5 x 5 271. For
reference we recall some useful right triangles in Fig. 7.5, each of whose
hypotenuse is length 1. (The radian measure of the indicated angle appears
inside each triangle.)
In Fig. 7.6 we give the values of sin x in decimal form, approximating when
irrational numbers arc involved. Now sin 71/2 = 1, since P is at (0, 1).
As x continues to increase from 71/2 to T , the point P continues around the
circle in the second quadrant, with its t coordinate decreasing from 1 when
x = 71/2 and P = (0, 1) to 0 when x = 71 and P = (- 1 , O ) ; see Fig. 7.7.
As x continues to grow from 71 to $ T , the t coordinate of P (now in the third
quadrant) continues to decrease from 0 at x = 71 to - 1 at x = & T see ; Fig.
7.8(a). As x continues to grow from #71 to 271, the t coordinate of P (now in the
fourth quadrant) increases from - 1 at x = $71 to 0 at x = 271; see Fig. 7.8(6).

1
~

i 2
Pig. 7.4 Pig. 7.5

sin 0 = 0
6
n = 1 = 0.5
sin ~- ~

2
sin ~ * a
4
= ~

2
2 0.71 sin -??
3
=! !Y
2
2 0.87

Fig. 7.6 Fig. 7.7


SECTION 7.1: TRIGONOMETRIC FUNCTIONS 237

(a')

Fig. 7.8

We now plot the function y = sin x on the x;y coordinate system, labeling
certain of these points. (See Fig. 7.9.)
Obviously, as x increases beyond 2m, the sine function begins to repeat itself
because the same (s, t ) coordinates recur. The sine function continues the cycle again
from x = 277 to x = 477, and then again from x = 477 to x = 677, and so on.
Similarly, as x decreases from x = 0 to x = -2v, the same (s, t ) coordinates
occur. For example, sin( - a / 6 ) = - 1/2 = - 0.5 = sin( 1lm/6). (See Fig. 7.10.)
Again, sin x repeats its cycle as x decreases from x = - 2 m to x = - 4 m , and
so on. Putting all this information together, we get a more complete graph of
sin x. (See Fig. 7.1 1.)

7 . I .4 Definition Any function that repeats itself in every interval of fixed length p
and in no smaller interval is said to beperiodic ofperiodp. That is,f(x + p) = f ( x )
for all x in the domain of$
As we have seen, the sine function is periodic of period 27~.

Fig. 7.9

j' = sin Y

Fig. 7. I 0 Fig. 7.1 1


238 CHAPTER 7. TRIGONOMETRIC, EXPONENTIAL, LOGARITHM FUNCTIONS

PROGRESS TEST 2

1. Using the familiar right triangles given earlier, fill in X cos x cos X

the values of cosx in the table to the right:


~ ~
~ ~

0
2. Graph all the points listed in the table on the xy
plane, draw a smooth curve through them, and
-
-
6
extend the graph
- - in either direction beyond the
interval [0, 2771.
3. What is the period of cosx?

MORE T R l G O K O M E T R l C FUNCTIONS
The sine and cosine functions behave very similarly. In fact, one is just the other
+
slid horizontally by 7 ~ / 2 units; that is, cos x = sin(x 77/2). (See Fig. 7.12.)
The tangent function, defined by
t
tanx = -
Y tan x S
~
~

0 0 behaves somewhat differently, because for certain values of x, namely, when-


-
77 ever the point P = (s, t ) is on the t axis, s = 0, so tan x is undefined. In
0.577
6 particular this is true for x = k77/2. The tangent function is defined for x
~
- 1.000
arbitrarily close to %7~/2.We list some values of tan x for 0 5 x 5 71/2 in
4 Table 7.1. As x -+ ( 7 7 / 2 ) - , t + 1 and s + O+. Hence
-
- 1.732 lim t a n x = +a
3 z-(7r/2)-
1.25 3.009
1.30 3.602
T h a t is, the quotient t / s grows without bound. Similarly, as x + (-77/2)+,
t + - 1, while s -+ O+. Hence
1.40 5.798
1.50 14.101 lim tan x = - co
z--(W/Z)+
1.55 48.078
1.56 92.620 We also have
1.57 1255.765 lim t a n x = -a and lim t a n x = +co
-
,. a+
d 7 J / z+(37r/ 2)-
undefined
2
This pattern repeats itself in the intervals ( - 7 7 / 2 , 7~/2), (77/2,377/2),
( 3 ~ / 2 , 5 ~ / 2and
) , so forth. Clearly tan x is periodic of period 7 ~ (See
. Fig. 7.13.)

t 1' = cos n

Fig. 7.12
SECTION 7.1: TRIGONOMETRIC FUNCTIONS 239

I II II
f
I I
JJ = tan x

I I
I
I

* X

I I I /
I
I
Fig. 7.13

PROGRESS TEST 3

1. Following the procedure above use the definition of 5. Describe all x for which sec x is undefined.
the cotangent function, cot x = s / t , to analyze and 6. Describe all x for which csc x is undefined.
graphy = cot x for 0 < <
x 277. 7. What is the smallest possible value of lsecxl?
2. What is the period of cot x? 8. What is the smallest possible value of lcscxl?
3. Describe all x where cot x is undefined.
4. In terms of the terminal side of the angle, in which
quadrants is cot x positive and which negative?

THIGONOMETRIC IDENTITIES
We shall now recall the basic trigonometric identities but shall not concentrate
on proofs.

7. Z .5 Trigonometric Identities
sin x cos x 1 1
tanx = - cotx = - sec x = - cscx = -
cos x sin x cos x sin x
cos2x +sin2 x = 1 tan2 x 1 = sec2x + cot2 x 1 = csc2x +
cos x = sin ;( - x) sin x = cos - - x
(; ) (;
tanx = cot - - x
)
sin(a 2 b) = sin a cos b 2 cos a sin b
cos(a 2 b ) = cos a cos b 3 sin a sin b
tan a 5 tan b
tan(a -+ b ) =
1 T.tan a tan b

sin2x =
1 - cos 2x cos2 x = 1 + cos 2x
2 2
tanx = sin2x -
- 1 - cos2x
1 + cos2x sin 2x
Some of the above identities, such as
sin2x + cos2x = 1
240 CHAPTER 7: TRIGONOMETRIC, EXPONENTIAL, LOGARITHM FUNCTIONS

follow from the fact that we used the unit circle to define the trigonometric
functions:
sin2x + cos2x = t 2 + s2 = 1
Others require much fancier manipulations. [Recall that customarily we write
sin2 x instead of (sin x ) ~ and
, so on.] Later we shall show how derivatives can be
used to relate these identities to one another.
Also, for convenience, we give some values of the sine, cosine: and tangent
functions in Table 7.2.

We can use the identities and Table 7.2 to compute additional values of the
trigonometric functions.

Pig. 7.14

Example 1 Determine sin T/ 12, accurate through three decimal places.


. T
Solution
12
7 T 7 T 7 T . T
= sin - cos - - cos - sm -
4 6 4 6
Values of Sin, Cos, Tan at Popular x - fifi f i 1
2 2 2 2
sin x cos X tan x -
$2
=-(&- 1 ) ~ 0 . 2 5 90
0 1 0 4
~
1 fi
- 1
-
2 2 fi ADDITIONAL REMARKS
@ fi
- 1
2 2 Although we used the unit circle in defining radian measure and the trigono-
metric functions, we could just as well have used a circle of arbitrary positive
@ 1
- 6
2 2 radius R. In this case, one radian would be defined as the angle obtained when
the point P on the defining circle travels a distance equal to R in the counter-
1 0 undefincd
clockwise direction. See Fig. 7.14, where we have superimposed a unit circle
over the generating line to show that the size of the angle is the same, whether
@ -1
-&
2
~

2 or not we use a unit circle. Analogously, we could define sin x = t / R instead of


sin x = t / l , and so forth. The same functions result.
$ -1 It is important to remember, however, that the variable x stands for numbers
2 2
-1
and not angles. The domain of each trigonometric function consists of a set of
1
- -
real numbers, which are the radian measures of angles. In fact, we used angles
2 2 fi
to introduce the trigonometric functions mainly to take advantage of your
0 -1 0
earlier experience with geometry and trigonometry. In later chapters we shall
Table 7.2 see how these functions can be defined entirely independently of geometric
considerations. Finally, when working with a calculator, be sure that it is set to the
degree mode or radian mode as appropriate.

SECTION 7.1 EXERCISES

1. Using methods similar to those used to describe tan x, 2. Repeat Exercise 1 for cscx.
describe and graph secx for - v / 2 x 3r;/2.< < 3. What is the period of ( a ) sec x and ( b ) csc x ?

In Exercises 4 to I 1 convert the given degree measure to radian measure (leaving a factor of TI in your answer.)

4. 50 5. 15 6. 110 7 . 400
8. -2 9. 107 10. 3.14 11. -3.14
SECTION 7.2: DERIVATIVES OF TRIGONOMETRIC FUNCTIONS 24 1

In Exerckes 12 to 18 convert the given radian measure to degree measure.

12. 1 13. 0.62 14.


2
-71 15. -
77
5 10

16. -77
-6 2
17. 20077 18. -
7 3

In Exercises 19 to 29 determine the value ofthe given function at the indicated number using Table 7.2 and, gnecessav, the trigonometric identities.

71
19. sin 20077 20. cos(-;) 21. cos(+), 22. csc-8

23. sin (+) 24. cot(G77) 25. s e c 3s m 26. tan ($v)

27. tan (+n) 28. CO~(+T) 29. cos5Orr

30. Determine sin(lOloOa)- sin(v/2). (Note that your cal- 35. Use the identity in Exercise 34 to derive the identity
culator cannot do this one.) sin2x = ( I - cos 2 x ) / 2 .
81. Derive the identity t a n 2 x + 1 = sec2x from sinzx + 36. Use the identity in Exercise 34 to derive the identity
cos2x = 1. 6 0 9 2 x = (1 +
cos 2 x ) / 2 .
32. Repeat Exercise 31 for cot2 x +
1 = csc2 x.
3 3. Show how the identity sin 2x = 2 sin x cos x follows from
the identity for sin(a +
b).
3 4 . Analogously to Exercise 33, derive the identity
cos 2x = cos2 x - sin2x.

Section 7.2: Derivatives of Trigonometric Functions

Objectives: THE DERIVATIVES OF THE SINE


A N D COSIRE FUNCTIONS
1. Differentiate each of the six
trigonometric functions and By carefully e x a m i n i n g t h e g r a p h s of t h e sine a n d cosine functions in Fig. 7.15
combinations of these functions w e can learn some i m p o r t a n t facts about these functions.
with other differentiable functions.
J
2. Determine limits involving
trigonometric functions. J = sin x

Fig. 7.15
242 CHAPTER 7: TRIGONOMETRIC, EXPONENTIAL, LOGARITHM FUNCTIONS

I
Fig. 7.10

+
Each function appears to be continuous on ( - M, M)-a fact that can be
proved rigorously, but which we shall assume.
The sine function is odd, that is, sin( - x ) = -sin x ; and the cosine function is
even, that is, cos( - x ) = cos x .
Each function appears to be differentiable in that the graphs appear to be
smooth; that is, they have well-defined tangents at each point.
Let us examine the slopes of the tangents to the sine function to see whether
we can guess what its derivative might be (see Fig. 7.16). Near x = 0 the slope is
+
about 1, and it decreases to 0 (horizontal tangent line) at x = a/2. It then
becomes negative and decreases to about - 1 at x = a.It then increases again,
reaching 0 (horizontal tangent line) at x = 3 ~ / 2 ,and continues increasing,
reaching about 1 at x = 277.
We sketch this slope function-that is, the derivative of sinx between 0 and
2a-in Fig. 7.17. This certainly looks like the cosine function between 0 and 277.
Furthermore, this same pattern is repeated with a period of 277 for all x .

Fig. 7.17

We shall now prove that the derivative of sin x is cos x . We shall handle this
derivative carefully because the derivatives of all the other trigonometric
functions will be based upon it.
We begin by rewriting the standard derivative quotient:
f(x + Ax) - f ( x ) -
-
sin(x + Ax) - sin x
Ax Ax
-
-
(sin x cos Ax + sin Ax cos x) - sin x
Ax
Now we break the quotient into two terms and regroup:
sin Ax cos x
+ sin x cos Ax
Ax - sin x
= cosx
sin Ax
+ sinx [ cos Ax
Ax
- 1
1
Ax
Below, we shall prove the following:
SECTION 7.2: DERIVATIVES OF TRIGONOMETRIC FUNCTIONS 243

7.2.1 sin Ax COSAX- 1


( a ) lim
AX-o
-
AX
-1 and ( b ) lim
AX+n Ax
=o
Then using (7.2.1),we can prove:

7.2.2 D,(sinx) = cosx


Proof of ( 7.2.Z), using (7.2.1);
Dx(sin x ) = lim cos x -
sii:x
AX-0
( [ 1+ [ ;Y,-
sin x 0s 1 I)
= cosx lim sin A x [
TI+ sinx lim
Ax-0 Ax4

= (cosx)(l) + (sinx)(O)
= cosx
We shall now derive [7.2.l(b)] from [7.2.l(a)],leaving as our last step the
proof of [7.2.l(a)].
Proof that [7.2.I(b)]followsfrom [7.2.I(a)]:
First,

- COSAX- 1 COSAX + 1
COSAX- 1 -
Ax Ax cosAx + 1
- cos2Ax - 1
-

Ax(cos Ax + I )
-sin2 A x
-
A x ( c o s A x + 1)
(since sin2 Ax + cos2Ax = 1)
-
-
Hence
lim cos A x - 1 -
[-- AX 1
- lim sin A x -sin A x
Ax4 AX Ax-0 cosilx +1
sin A x lim
= lim -
AX-0 AX
- AX-0
-sin Ax
cos AX +1
= 1 *- O assuming [7.2.l(a)l
1+1
1 0
Besides using the Limit Theorem (2.5.4) in these arguments, we also used our
assumption that the sine and cosine functions are continuous when we evalu-
ated the limits in the last step by substitution. The same will be true in the
proof of [7.2.1(a)].
Thus we have reduced the proof that D,(sinx) = cosx to a proof of
[7.2.l(a)]. This argument is of special interest because it will be the first limit
argument that does not boil down to the rationalizing and/or canceling
routines that sufficed so far. Before moving to the proof, we notice that in
Table 7.3 (in which the values were computed using a calculator) and in
Fig. 7.18,
sin Ax zAx
for small Ax. (The values in the table are rounded off to the relevant digits.) In
Fig. 7.18we- see thatfor Ax small, the length ofthe arc A x and the length ofthe strazght
lzne segment PA are almost equal; hence their ratio (sin A x ) / A x must be very close to 1.
-. r 3 r . 1 .. 11 - . -i-~---~:--. -r -L --__._
A:-m
244 CHAPTER 7: TRIGONOMETRIC, EXPONENTIAL, LOGARITHM FUNCTIONS

t
~
sin AX
AX sin AX AX

0.2 0.198 0.993


0.1 0.09983 0.9983
0.01 0.0099998 0.999983
0.001 0.0009999998 0.99999983
0.0001 0.0000999999 0.999999995

B = il, 0) Table 7.3


Pig. 7.18
Proof of [7.2.1(a)]: sin =
lim - I: First, we need only deal with Ax > 0
1 AX-0 AX
because the sine function is odd, making
sin( -Ax) - -sin Ax - sin Ax
-Ax -Ax Ax
Now, suppose we have a circle of radius 1 with an angle of Axrad, as
indicated in Fig. 7.19. Using the geometry of this situation, we trap the quotient
in question between 1 and COSAX,and apply the Squeeze Theorem (2.5.6).
Now,
< area of sector POB < area of triangle COB
area of triangle POB
Using the facts that the area of a triangle is 4the base times the height and the
area of the Ax sector of the circle is Ax/(271) times the area of the whole circle,
7~ l 2 = T , the above inequality translates to

1--
-
2
OB. PA < Ax
27
- < -120-B - CB
- 7~
-

Multiplying through by 2 and using that = 1, we get


-
PA < Ax < ?%
-
But sin Ax = PA by definition, and, by similar triangles,
- - -
tanax = = CB = -
PA = = CB = -
CB
OA OB 1
so we have sin Ax < Ax < tan Ax
Since sin Ax > 0, (0 < Ax < 7r/2), we can divide through by sin Ax to get
Ax tanAx -
- using tan Ax = -
1 < ~ < ~ cos Ax
Finally, we can invert each term of the inequality, reversing inequality signs
(remember, 2 <
3 implies 4>
8) to get the promised trap of (sin Ax)/Ax
between 1 and cos Ax:

Since this holds for all Ax with 0 < Ax < 7/2, we know that
sin A x
1 2 lim -2 lim cos Ax = 1
Ax-0 AX Ar-0

Thus, by the Squeeze Theorem, the only possibility is that


sin Ax =
lim -
A+-0 AX

and the proof is completed.


SECTION 7.2: DERIVATIVES OF TRIGONOMETRIC FUNCTIONS 245

Since the derivative of sin x is cos x, the Chain Rule implies the following:

If u ( x ) is a differentiable function of x , then


7.2.3 D,[sin u(x)] = cos u ( x ) u(x) -
In stating and applying this rule, we follow custom and use as few paren-
theses as are necessary to avoid confusion. Hence we have written sin u ( x )
instead of sin ( ~ ( x ) )and
, cos u ( x ) * u(x) instead of [cos ( u ( x ) ) ] * u(x).

Proofof (7.2.3): By the Chain Rule (3.2.2), with g equal to the sine function, we
have

D,[sin (44)1= D, [g(u(x))l


= g(u(x)) * u(x)
But g is the cosine function, so
g(u(x)) * u p ) = cos(u(x)) u(x)
a

Henceforth, in establishing additional derivative rules, we shall use the


phrase by the Chain Rule repeatedly as we just did above but not carry out
the argument, which is the same in every case.

Example 1 Compute: ( u ) D,[sin(x2 + 3)] and ( 6 ) D,[x3sin(5x + l)].


Solution By (7.2.3):
( a ) D,[sin(x2 + 3)1 = (cos(x2 + 3 ) ) ( 2 x )
= 2 x cos(x2 + 3)
(Again, it is customary to omit the parentheses and reorder factors to avoid
ambiguity.)
+ + +
(b) D,[x3 sin(5x l)] = x3(cos(5x 1))(5) (sin(5x 1))(3x2) +
+ +
= 5x3 cos(5x 1) 3 x 2 sin(5x 1) 0 +
We shall also apply (7.2.3) to compute D,(cosx). But first we need:

7.2.4 (u) (;
cos x = sin - - x ) ( 6 ) sin x = cos

These statements follow by applying the identities (7.1.5) regarding the sine
and cosine of a difference, respectively, with a = 7 / 2 , b = x .
Now, by (7.2.3), we have

by [7.2.4(u)]

= (,,, ;( - .))( - 1) by (7.2.3)

= (sin x)( - I ) I> by [7.2.4(b)]


- -sin x

Hence, by the Chain Rule, for u(x) a differentiable function of x :

7.2.5 D,[cos u ( x ) ] = -sin -


u ( x ) u(x)
246 CHAPTER 7: TRIGONOMETRIC, EXPONENTIAL, LOGARITHM FUNCTIONS

Example 2 Compute but do not simplify: ( u ) D,[cos(x3 - 2x 4)] +


( b ) D,[(cosx)(l - sin5x)I (6) D,[sin3(7x I)] +
Solution ( u ) B , [cos(x3 - 2x + 4)] = - (sin(x3 - 2 x + 4))(3x2 - 2)
+
( b ) D,[(cosx)( 1 - sin 5x)] = (cos x ) ( - cos 5x)(5) (1 - sin 5x)( - sin x )
(c) Remember, sin3(7x + +
1) = (sin(7x l))3.
D, [sin3(7x + 1)] = 3 sin2(7x + 1) - D, [sin(7x + 1)]
= 3 sin2(7x + 1) - (cos(7x + 1))(7) 0

In part (c) we used both the Power Rule and the Chain Rule.

PROGRESS TEST 1

In Probs. 1 to 8 differentiate the indicated function.


+
1. f ( x ) = sin(x4 3x3 + 2x + 1) 2. f ( x ) = cos(x5 - 4x3 - 3 x + 1)
3. g ( x ) = sin5(3x2+ x - 1) 4. t ( x ) = dcos(x2 x)+
5. r(x) =
+
sin2(% 1)
6. s ( t ) = cos4(t2 + 1) sin3(7t) + sin 4t
cos3(5x - 2)
7. m ( p ) = 2 sinpcosp 8. f ( x ) = s i n ( d m )

DERIVATIVES OF THE TANGENT, COTANGENT, SECANT,


AND COSECANT FUNCTIONS
We shall now employ the Quotient Rule in combination with (7.2.3) and
(7.2.5) and trigonometric identities to differentiate some of the remaining
trigonometric functions:

D,[tanx] = D, -
k: :I
-
- cos x(cos x ) - (sin x)( -sin x )
cos2 x
+
- cos2x sin2x - 1 = sec2x
-
cos2 x cosz x
Also,
D,[secx] = D, -
.I
- (-l)(-sinx) - --
1 sinx
cos2 x cos x cos x
= sec x tan x
Parallel arguments establish the derivatives of cot x and csc x . (See Progress
Test 2.) Assuming that u(x) is a differentiable function of x , we have by the
Chain Rule the following derivative rules:

7.2.6 D, [tan u(x)] = sec2u ( x ) * u(x)

7.2.7 D, [cot u(x)] = -csc2 u(x) * u(x)

7.2.8 D, [sec u(x)] = sec u(x) tan u(x) * u(x)

7.2.9
SECTION 7.2: DERIVATIVES OF TRIGONOMETRIC FUNCTIONS 247

Example 3 Differentiate the following functions:


(u) f ( x ) = tan(5x - 3)
( b ) f ( x ) = sec(4x3 - x )
(c) y = csc3(5x)
( d ) g ( x ) = vot(x-1)
( e ) f ( x ) = tan4(sin 7x)

Solution We shall employ a variety of derivative notations.


(u) f ' ( x ) = sec2(5x - 3)(5)
df
(6) - = sec(4x3 - x)(tan(4x3 - x ) ) B,(4x3 - x )
dx
= sec(4x3 - x)(tan(4x3 - x))(12x2 - 1)
4 d(csc(5x))
(6) - = 3 cscZ(5x)
dr dx
= 3 csc"5x)( -csc(5x))(cot(5x))(5)
= - 15 csc3(5x)cot(5x)
( d ) g'(x) = D,[(cot(x - l))lI3]
1
= -(COt(X - 1 ) ) - 2 / 3 ~ e [ ~-~111
t(~
3
1
= -((cot@ - 1))-2/3( -cscZ(x - 1))(1)
3
cscyx - 1)
-
- -
3Vcotyx - 1)
( e ) f ' ( x ) = 4 tan3(sin 7x) D,[tan(sin 7x)]
= 4 tan3(sin 7x)sec2(sin7x) D,[sin 7x1
= 4 tan3(sin 7x)sec2(sin7x)cos 7x * 7 0

In part ( e ) we applied first the Power Rule and then the Chain Rule three
times. Most errors in differentiating functions of that type (for example,
sin3(cos 5x)) occur because the Chain Rule is not completely applied.
First and second derivatives of the trigonometric functions are of course
useful in graphing. We show here how the graph of the tangent function
(Fig. 7.13, p. 239) agrees with the information provided by its derivatives.

Example 4 Show that f ( x ) = tan x has the following properties:


( u ) It is increasing on its natural domain.
(6) It is concave up where it is positive and concave down where it is negative.
(c) It has a point of inflection at each point where it crosses the x axis.
Solution (a) Now f ' ( x ) = sec2x and f ( x ) = tanx have the same natural domain,
namely, all x for which cosx # 0 [secx = l/(cosx) and t a n x =
(sin x)/(cos x)]. But since f ' ( x ) = seLzx is positive for all such x , tan x is
increasing on its natural domain.
( b ) Sincef"(x) = 2 sec x(sec x tan x )
= 2 sec2x tan x
and 2 sec2x >
0, we know that f ( x ) and f "(x) have the same sign on their
(common) natural domain. In particular, assertion ( b ) follows.
(c) Where f ( x ) = tan x crosses the x axis, it, and hence its second derivative,
changes sign, from which assertion (c) follows.
248 CHAPTER 7: TRIGONOMETRIC, EXPONENTIAL, LOGARITHM FUNCTIONS

The successively higher derivatives of certain trigonometric functions tend to


repeat or change in a systematic way (see Progress Test 2, Prob. 8 and section
exercises 49 to 52).

PROGRESS TEST 2

In Probs. 1 to 5, differentiate the given function. (It is not


necessary to simplify your answers.)
1. J J = CSC[(X - 4)5] + +
2. f(x) = tan3(3x7 2x4 3x - 1)
3. h(x) = dcot(x3 2x) + 4. f(x) = sec(3x + -
1) tan( 1 - x)
5 . g ( y ) = sec5[(3y2 - J ) ~ ] 6. Prove that D,[cot x] = - a c 2 x.
7. Prove that D,[csc x] = -cscx cot x. 8. Describe the 100th derivative off(x) = sin ax.

MORE LIMITS
Variations of the following limit (established 7.2.1) occur frequently

sin t =
7.2.10 lim ~

t-0 t

sin 4x
Example 5 Evaluate lim -.
2-0 x
Solution We wish to use the above limit, so we multiply and divide by 4, getting

But now letting t = 4x, t + 0 as x + 0, so

This example obviously generalizes to:

sin kx = k,
7.2.11 For any constant k, lim-
a+O x

RADIANS VERSIJS DEGREES

i Suppose that at the beginning we had decided to use degree instead of radian
measure of angles. An angle A , which yields a point P = (s, t ) on the defining
circle, has a larger degree measure than radian measure. Nevertheless, we would
want the sine of the given angle A to be fixed-namely, as t. For example,
suppose A is the angle whose size in radians is 7r/6 and whose size in degrees is
30. (See Fig. 7.20.) In terms of radians, we already have sin v/6 = 0.5. O n the
other hand, in terms of degrees we want the sine of 30 to be 0.5 also. Hence we
can define a new function sindeg with sindeg(30)= 0.5. This new function
sind,,(.) has period 360 with, for example,
sindeg(0)= 0 sindeg(30)= 0.5 sindeg(90)= 1
Our new function sindeg,although defined for numbers (and not angles),
Pig. 7.20 accounts for the different way of measuring angles. This is described schemat-
SECTION 7.2: DERIVATIVES OF TRIGONOMETRIC FUNCTIONS 249

ically by the dashed arrow in Fig. 7.21, with the conversion formula connecting
the two measures.
The graph of y = sindeg(.) in Fig. 7 . 2 2 , even with the horizontal scale
squeezed in drastically, is much flatter than our previously defined sin x.
We can translate the degree measure x of an angle into radians, getting
(.ir/l80)x, so that

sinde,(x) = sin
(-
li0
This can also be seen directly by recognizing sindegto be the composition of
sin with (.ir/l80)x (see Fig. 7.21). By the Chain Rule,

In terms of slope this shows again that the graph of sinde,(x) should be much
flatter than that of the radian-based sine function. Its slope never exceeds
.ir/180 =: 0.0175.
These same arguments apply to all the other trigonometric functions and
should help explain why we said early in Sec. 7.1 that radian measure is the
most practical measure for the calculus. We do not need to worry about that
~ / 1 8 0factor every time we take a derivative.

R
radian

Angle Same number

of A
Fig. 7.2 1 D

Y = Sind,,(.)
250 CHAPTER 7: TRIGONOMETRIC, EXPONENTIAL, LOGARITHM FUNCTIONS

SECTION 7.2 EXERCISES

In Exercises I to 30, dfferentiate the indicated function:


1. f(x) = cos(x - 2) 2. f(x) = sin[(x3 - 3 ~ ) ~ ]
3. g(x) = tan(2x - 1) 4. m(x) = csc(1 - x)
5. y = COtZ(3X - 2) 6. y = m sin2x b +
7. f (t) = sin t cos t+ 8. +
h(8) = A sin(a8) B cos(b8)
9. f(x) = siqxcot2(3x) 10. f(x) = sin3(cos(x))

11. f(x) = 12. r(t) = m(sin2 t)(t3 + 1)


13. f(8) = a s i n 8 bcos8 14. g(t) = sect t a n t
\/sec x
15. f (x) = sec2 x 16. y = -
X

17. u(x) = tan2( m)


- sec2( m)(simplify) 18. ~ ( x =
)
sin(llx2 + 3x + 4) (simplify)
cos(llx2 + 3x + 4)
19. f(x) = sin(cos 3x) 20. f (x) = cos(sin 3x)

21. y = tan5(xz + 1) 22. y = cot5(x2 + l)tan5(x2 + 1) (simplify)


23. g(x) = d--\3Jcsc3x 24. y = (1 lx + csc x ) ~
sin5(3x + 2)
25. g(x) = 26. f (x) = (x + sin2 3x)( 1 - cos' 5x)
tan(2x - 1)
\/tan x
27. l(x) = 28. f(x) = (:)sinx
CSCZ(4X - 1)

29. f (x) = x sin 30. y = csc7(cot(x2))

I n Exercises 31 to 34 differentiate the given function with respect to t. Assume that x, y, 0 are all functions oft.
3 1. f ( t ) = cos x + 2t sin t 32. y = d x v + tan x
33. f ( t ) = x sin 0 34. y = x cos0 + y sin 0
In Exercises 35 to 40 determine dy/& using implicit diferentiation.
35. sin2x + cos2y = 1 36. 2 sin x cos x = y
37. v-'= tany 38. sin(x + y ) = cos(x - y )
39. cos(x - y ) = x - y 40. tan(x +y ) = x +y
41. By employing the trigonometric identity cos(a +
b) = 42. Give a formula for the derivative off (x) = tann x that is
cos a cos b - sin a sin b and (7.2.1) prove that the deriv- entirely in terms of tan x (n an integer).
ative of cos x is -sin x using the definition of derivative.
43. Give a formula for the derivative off(x) = secn x ( n a n 44. What is the equation of the line tangent to f(x) =
integer). tan(ax) at x = O?
45. Given cos(x - y ) = 1; 46. Prove the standard identity sin2x cos2x = 1 using +
( a ) Find dy/dx implicitly if it exists (be careful!). that a function with zero derivative everywhere must be
(b) Interpret ( a ) and give a sketch of the equation a constant function.
cos(x - y ) = 1 anyway. (Note that there is more than
one line in the graph.)
47. Repeat Exercise 46 for t a n 2 x +
1 = sec2x. 48. Repeat Exercise 46 for cotZx + 1 = csc2x.
In Exercises 49 to 52 determine the given derivative.

49. f'loo'(x), wheref(x) = A cos x 50. f ( 4 ) ( ~ )where


, f(x) = cos x sin x +
51. fs(x), wheref(x) = A sin ax 52. d15y/dx15, where y = cos 3x
53. Use the first and second derivatives to describe the 54. Repeat Exercise 53 for y = csc x, for (a) 0 <x <m
graph of y = secx, for (a) -m/2 x < <
m/2 and and (b) m < x < 271.
(b) m/2 x < <3m/2.
SECTION 7.3: INVERSE FUNCTIONS AND LOGARITHM FUNCTIONS 25 1

In Exercises 55 to 59, evaluate the given limits:

5 5 . lim x sec x 56. lim- tan Ax 63. ( a ) Using the degree-based approach, show that the
a-0 Ax-o AX
crucial limit necessary to compute D,[sindeg(x)]from the
lim- x 2 + x
cos 5 x definition
57. lim - S8.
x-0 cot 3 x X-o sinx
x sin k
S9. 1im
x-0 1 - cosx

60. Calculate the derivative of tandeg(x) using the Chain


Rule, expressing the result in terms of degree func- ( b ) Evaluate the limit in ( u ) using (7.2.11).
tions. 64. Modify the geometric argument used to prove [7.2.l(a)]
61. Repeat Exercise 60 for cotd,,(x). to evaluate the limit in Exercise 63(a). (The area of a
62. By employing an analysis analogous to that involved sector changes.)
with Figs. 7.16 and 7.17, show that it is reasonable that
D,(cos x ) = -sin x.

Enverse Functions and


Section 7.3: Expohential Logarithm Functions

Objectives: INVERSES OF FUNCTIONS


1 . Determine inverses of simple T h e graph of a function that is either strictly increasing or strictly decreasing on
functions. I has the property that a horizontal line will hit the graph in at most one point.
(See Fig. 7.23.) For such functions, when we are given a value of the function,
2. Compare graphs of ?unctions
that is, a number X on the vertical axis, we can tell exactly which x satisfies
and their respective inverses.
f ( x ) = X. For the functionf(x) = x3,which is increasing on [ - 10, lo], given a
3. Graph exponential and logarithm number, say X I64, we know exactly which x satisfies
functions.
f ( x ) = x 3 = 64
4. Use the properties of namely, x = 4. This contrasts sharply with the function g ( x ) = x 2 , which
exponential and logarithm changes direction on [-lo, 101. Given the number 64, we cannot tell which x
functions t6 manipulate
satisfies
expressions involving these
functions. g(x) = x 2 = 64
because it could be either x = 8 or x = -8. I n the case of f ( x ) = x 3 , we can
undo f by taking the cube root and get a unique answer. I n the case of
g ( x ) = x 2 , there are two possible square roots a n d no unique answer.

7.3.1 Definition Suppose the functionf has a given domain and range. We say f
has an inuerse, denoted f - l , if, for every Xin the range ofJ; there is a unique x
in the domain o f f with f ( x ) = X. I n this case f -
is a function defined by:
-
f-l(F) = x where x is the unique number with f ( x ) = x

(a)
Fig. 7.28
252 CHAPTER 7: TRIGONOMETRIC, EXPONENTIAL, LOGARITHM FUNCTIONS

The domain off- is the range off, and the range off- is the domain off.
We will consistently use the notation f - l (pronounced y inverse) for the
inverse off, and so f will not mean 1/J:
The following theorem (whose proof is outlined in Exercise 34) gives us
conditions on f which guarantee that f exists.

7.3.2 Theorem (Existence of Inverses) Iff is continuous and is either


strictly increasing or strictly decreasing on an interval I, then f has an inverse
f- (whose range is I ) .

Example 1 Givenf(x) = x3, determinef-l.


Solution We know f is continuous and increasing on ( - co, a), + so f -I exists. By the
definition,f-l( X) is that unique number for whichf(x) = X; that is, f -I( X) is
that unique number which when cubed is X. But of course that unique number
is $5, so
f-(7) = 0

-
We used Fin definingf both in (7.3.1) and in the example to distinguish it
from the variable x that was used to denote numbers in the domain off. The
letter we use as the variable makes no difference since a function is known by
what it does. Thus we shall often use an Fin the construction off- fromf, but
then relabel the resulting functionf-l using the more customary x . Hence for
f (x) = x 3 we can write
f-l(x) = $G

Example 2 Show thatf(x) = 3x + 2 has an inverse on R and determinef-I.


Solution f ( x ) = 3x + 2 , being a nonconstant linear function, is continuous and strictly
increasing on its domain R. Given an X in the range, we can algebraically
compute the unique x withf(x) = X by setting 3x + 2 = X and solving for x:

4
Thusf -(X) = (X - 2)/3, again a linear function, with slope and intercept
( 0 , -8). Finally, relabeling, we have
x-2
f-(x) = -
3
It is useful to think of the inverse functionf-l as undoing what the
original functionf(x) did. The above example, f ( x ) = 3x + 2, triples x and
then adds 2, so to undo whatf (x) did to x, common sense tells us first to subtract
the 2, getting 3x, and then to divide by 3, getting back to x . (A bit like
unwrapping a package-work from the outside in.) But of course, this is exactly
what f -1 does: it subtracts 2 and then divides by 3.
f
- x In general, the composition of f-l withfshould leave x unchanged. In terms
of Fig. 7.24, x gets taken on a round trip back to x.
ux f- Algebraically,

Fig. 7.24
SECTION 7.3: INVERSE FUNCTIONS AND LOGARITHM FUNCTIONS 253

f o r each x in the domain off; and similarly,

(fof-')(x) = f ( f - W=) x
f o r each x in the domain o f f (the range off ).

In the special case of f ( x ) = 3x + 2,f -'(x) = ( x - 2 ) / 3 , we get


(f-' Of)@) = f - l ( f ( x ) ) =f-'(3x + 2)
-
(3x + 2) - 2 -
-
3x
-= x
3 3
and
x-2
( f Of-l)(X) =f(f-W
=f(,-)
=3 (q)
+ 2 =x - 2 +2 =x
for all real numbers x .

Example 3 Show that for a # 0,


X-b
f ( x ) = ax +b (a # 0) implies f-l(x) =-
a

Solution To undof, we first subtract b, leaving ax, and then divide by a, so f-'(X) =
( X - b)/a. We can accomplish this construction algebraically by setting
ax + b = X and solving for x to get
-
x = -X - b
a

Hence f -'(X) = (X - b ) / a , or, using x as the independent variable,


x-b
f-'(x) = -
U

Note that this is a linear function of slope l / a through the point (0, - b / a ) . 0
Here are three simple applications of Example 3 (for all x in R ) :
(a) Given f ( x ) = 2x, ( a = 2, b = 0 ) ,f - ' ( x ) = x / 2 .
( b ) Given f ( x ) = x +
4, ( u = 1, b = 4), f-'(x) = ( x - 4)/1 = x - 4.
(c) Given f ( x ) = - x , ( a = -1, b = 0 ) ,f - ' ( x ) = x/(-1) = - x .
This last example is its own inverse! But this makes sense: How would you
undo a function that reverses the sign of x ? You reverse it again, getting
- ( - x ) = x.

GRAPHS AND DERIVATIVES OF INVERSE FUNCTIONS


The relationship between the graph off and the graph off both on the same
coordinate system is very revealing. In Fig. 7.25 we sketch the examples
considered so far, including the diagonal linep = x (dashed). In Fig. 7.25(u) we
include a pair of tangent lines for future reference.
That all this symmetry about the dashed h e y = x is no accident can be seen
from the following observation and Fig. 7.26. The set of orderedpuirs determined by
f are precisely those determined by f but with their order reversed. The diagonal line
acts like a mirror with the point ( r , s) being "reflected" across the diagonal to
the point (s, r ) . Of course, a point such as ( 2 , 2 ) on the diagonal is its own
reflection.
254 CHAPTER 7: TRIGONOMETRIC, EXPONENTIAL, LOGARITHM FUNCTIONS

Notice that in Fig. 7.25(r) the graph of the function that was its own inverse,
f ( x ) = -x =f-(x), is perpendicular to the diagonal; hence it is its own
reflection across the diagonal. On the other hand, f ( x ) = x +
4 and
f - ( x ) = x - 4, having the same slope, are parallel, equidistant from the
diagonal, as shown in Fig. 2.25(d).
The fact that the graphs .ff and f are symmetric across the diagonal enables us
to provide a convincing, but informal, argument for the following important
theorem. We leave its formal proof to more advanced courses.

+ ;x) = 3x +2
/

.1i
Fig. 7.25

Fig. 7.26
SECTION 7.3: INVERSE FUNCTIONS AND LOGARITHM FUNCTIONS 255

7.3.3 Theorem (Derivatives of Inverse Functions) Iffis differentiable at


x with dfldx = a # 0, then @-I/& exists at f ( x ) and equals l/a.
If the graph offhas a nonhorizontal tangent at P = ( x , f ( x ) ) ,then the graph of
f-l has a nonvertical tangent at Q = (f(x), f - l ( f ( x ) ) ) = ( f ( x ) ,x ) . [See Fig.
7.25(a), for example.] Furthermore, if the tangent to f has slope a # 0, then
Example 3 guarantees that the corresponding tangent tof-l has slope l / a since
the reflection across the diagonal of the tangent to f at P is the tangent toy-'
at Q.
The importance of a nonhorizontal tangent to f can be seen graphically in
Fig. 7.25(a) by examining the graphs off(.) = x 3 andf-l(x) = f& at their
common point (0,O). The tangent tofis horizontal (slope = 0), and the tangent
to f - l is vertical (undefined slope). Algebraically, D, [ f - l ( x ) ] = D, (6) =
( 1 / 3 ) ~ - ~=/ ~1 / ( 3 w ) obviously is undefined at x = 0.

PROGRESS TEST 1

InProbs. 1 and 2 ( u ) determinef-I, (6) computefof-l 3. Givenf(x) = x y - 1, what are the coordinates of the
andf-l O J and ( c ) sketchfandf-l on the same coor- point on the graph off-'(x) where the slope of the
dinate axes, using a dashed line to represent y = x. tangent line is undefined?
1. f ( x > = 2x - 1 2. f ( x ) = x3 - 1

EXPONENTIAL AND LOGARITHM FUNCTIONS


Many phenomena, especially those involving natural growth or decay, are best
described by what are called "exponential" functions.
Suppose we monitor the population of an idealized bacteria culture (no
deaths, food or oxygen shortages, and so on) in which the bacteria reproduce
once every minute. Begin at time x = 0 with 1 bacterium. After 1 min it divides
in two, yielding 2, each of which divides when x = 2 to yield a total of 4, each
of which divides when x = 3 to yield 8, and so on, yielding successively, 16 at
x = 4,32 at x = 5,64 at x = 6, . . . . The number of bacteria present at time x is

x repetitions

The function that gives the population as a function of time x is therefore


~ ( x=
) 2" (x an integer)

Similarly, a quantity that tripled in each unit of time would yield the
amounts (beginning with 1 at x = 0): 3 at x = 1 , 9 at x = 2, 27 at x = 3,81 at
x = 4, . . . . The amount present at time x is

g(x) = 3"

On the other hand, if a quantity were cut in half for each unit of time x (with
1 unit of this quantity at the beginning point, x = 0), at x = 1,; unit remains,
4
at x = 2 one half of this remains, that is, remains; at x = 3,
At time x, the amount remaining is
-4
= remains.

In general, we shall be dealing with functions of the form

f(x) = 6" where b >0


256 CHAPTER 7: TRIGONOMETRIC, EXPONENTIAL, LOGARITHM FUNCTIONS

Note that for b <


0,b" is defined only for some x and not others, and generally
is a colossal mess. In fact, we shall confine our attention mainly to the case
b > 1, because for b = 1 , l " is a constant function; and for 0 b < <
1, we know
1 < 1 / b . By (7.3.6), (l/b)" = (b-l)" = b&", so functions such as (1/2)" can be
handled in terms of 2-".

7.3.4 Definition A function of the form f ( x ) = b", where b >0, is called an


exponentialfunction of base b. More specifically, we make the following definitions:
( a ) For x = rn a positive integer, we definef (rn) = bm to be the product of b with
itself rn times.
( 6 ) For x = 0, f ( 0 ) = bo = 1.
( c ) For x = rn a negative integer, f ( m ) = bm = l/b-" (where -m > 0, of
course).
( d ) For rational x = rn/n (rn, n integers, n # O ) , f ( m / n ) = bmIn = = (fi)m.

The problem of defining b" for irrational x is somewhat more subtle, involv-
ing the use of limits. We shall illustrate the principle involved by examining the
case 1 0 a in detail.
The number fi,being irrational, has an infinite, nonrepeating decimal
representation, which to nine places is
fi=.
1.414213562
It is important to recognize that the various decimal approximations of f i a r e
+
rational numbers: for example, 1.414 = 1 # = #, a quotient of integers.
Thus 101.414 is already defined. Furthermore, the sequence of decimals

(1) 1.4, 1.41, 1.414, 1.4142, 1.41421,. . .


provides an increasingly accurate approximation of fi.Now, using the
sequence ( l ) , which has fi as its limit, we can obtain the sequence
101.4 101.41 101.414 101.4142 101.41421
(2) , > > , ,.'
each term of which is 10 raised to a rational power and hence is defined in
[7.3.4(d)]. We dejne lo* to be the number approximated by this sequence. In
effect, we know lo* as well as we know fi.Furthermore, when 1 O f i is defined
in this way, we are guaranteed that as x approaches fi,h(x) = 10" approaches
l O f i , that is,

Hence h(x) is not only defined at fi,it is continuous at fi.


The above very informal argument applies to any other irrational x as well
as to any other base b >
0. It certainly supports the following two basic
assumptions:

7.3.5 (a) Exponential functions have domain R.


( 6 ) Exponential functions are continuous on R.

In addition, we make the following two assumptions familiar from high


school algebra:

7.3.6 (a) (b"i)(b"z) z b"i+"z


( 6 ) (b"1)"~ = b"1"z.

In Fig. 7.27 we graphed several exponential functions that will guide us in


formulating further properties of the exponential functions.
SECTION 7.3: INVERSE FUNCTIONS AND LOGARITHM FUNCTIONS 257

Pig. 7.27

The functions graphed suggest the last two assumptions we shall need for
further development (we assume b # 1):

7.3.7 ( u ) Each exponential function is either strictly increasing ( b > 1) or strictly


<
decreasing ( b 1) with domain R and range ( 0 , +m).
( 6 ) The larger the base b > 1, the steeper the slope of the graph for x 2 0.
As we shall illustrate later in the appendix to Sec. 7.4, these assumptions are
used only to provide a convenient starting place that takes advantage of your
experience with exponential and logarithm manipulations.
By [7.3.7(u)] and (7.3.2) we know the following:
7.3.8 Theorem Every exponential function of base b # 1 has an inverse whose
domain is (0, +a)and whose range is R.

Consider the function r(x) = 2, for example. Its inverse is defined as fol-
lows:
7-I(X) is the unique number x with r ( x ) = 2 = X
Thus
~ ~ (is 8the) unique number x with 2 = 8-that is, rP1(8) = 3 sincc 23 = 8
7-l(4) = 2, since Z2 = 4
rP1(32) = 5, since 25 = 32
r-1(+) = -4 since r ( - 4 ) = 2-4 = -
1 =-
1
24 16
As you likely know, there is a traditional name for 7-l, namely log,, the
logarithm base 2. More generally, we have:
258 CHAPTER 7: TRIGONOMETRIC, EXPONENTIAL, LOGARITHM FUNCTIONS

7.3.9 Definition For any base b >


0 (b # 1) the inverse of the exponential
function b is denoted log,, x, and is referred to as the logarithm base b.
By (7.3.8) log, x has domain (0, +a)and range R. Also by definition,
log,(.) is that unique numbery with by =x
It is convenient to determine log, of a number x by answering the question:
b to what power gives x?
Thus we find logln(100) by asking 10 to what power gives loo? Of course
the answer is 2, so logln(10O) = 2. Similarly, log,,(O.OOOl) = -4 since
= 0.0001. We know log,(b) = x because the answer to b to what power
gives b is obviously x . (Who is buried in Grants Tomb?) This is just another
way of saying that the logarithm and exponential functions are inverses.

7.3.10 Theorem For all x, log,(b) = x; and for all x > 0, b(logbr) = x.

Each of the properties of exponential functions implies a companion prop-


erty of the logarithm functions. For example log,( 1) = 0 because bn = 1, and
log,(b) = 1 because b1 = b. We list some algebraic properties of the logarithm
functions below. We assume x , xl, x , are positive.

7.3.1 1

These properties are useful in simplifying more complicated expressions.

Example 4 +
Evaluate [1og2(4)I3 10g,(8~) + log, (5).0.1

Solution We examine each term separately:


[1og,(4)l3 = 23 = 8, 10g,(8~)= 3 log, 8 = 3 3 = 9, and -
log, (F) = 10g,(20~)- 10g2(25)= 0.1 - 5 = -4.9

Hence the value of the original expression is


8 + 9 - 4.9 = 12.1 0

Example 5 Solve for x : log,(x2) + log,(fi) = 6.


Solution We combine terms on the left using [7.3.11(b)] to get
log3(x2* fi)= 1 0 g ~ ( x ~=/ ~6)
Hence x 5 l 2 = 36, by the definition of log,.
Squaring both sides, x5 = 312, so
= 312/5 - 13.97
N 0

We can use the graphical symmetry of functions and their inverses to help
graph the logarithm functions. We shall continue to concentrate on the case
SECTION 7.3: INVERSE FUNCTIONS AND LOGARITHM FUNCTIONS 259

Fig. 7.28

b > 1. For base b > 1, all the exponential functions have pretty much the
same shape, varying only in steepness, so we will graphy = logzx (see Fig.
7.28) knowing that the logarithm functions of other bases b > 1 look very
similar.
+
The domain of log,(x), being (0, a), means that its graph lies entirely to
the right of the vertical axis. Its range is all of R, even though the curve
appears to be leveling off as we move to the right. (The domain of 2 is R.)
In general, y = log, x has no horizontal asymptotes because, given any y , there
exists some x > 0 such that log, x = y . Furthermore, for x 5 0, log, x zs not
dejined. To ask for log,( -3) is to ask b to what power gives - 3 ? No power of
b > 0 can yield a negative number.

PROGHESS TEST 2

1. Evaluate:
(4 logd9) - 10g,(1/3)5 ( b ) log#) + logdl)
(4 10g,(45)
2. Solve for x:
(a) log& - 1) +
log& +
1) = 0
(b) 3+l = 81
(c) 2 loglo x - log,, l / x = 3
3. Graph y = log, x and y = 3 on the same axes.
4. Find f o g and simplify to remove fractional expo-
nents wherever possible:
dxz + x + 4 ( 3 x + 1)
)(. f(.) = log, x , !?(I
= (x2 +
+
(6) f ( x ) = b , g ( x ) = logb(4~ 5)
260 CHAPTER 7: TRIGONOMETRIC, EXPONENTIAL, LOGARITHM FUNCTIONS

SECTION 7.3 EXERCISES


-
In Exercises I to 5 determine f - l and then graph f and f - l on the same coordinate axes. State the natural domain and the range off and f-l.
1. f(x) = x - 1 2. f ( x ) = 3 x 3 8. f ( x ) = GTT
4. f ( x ) = -
1 3x 4
5. f ( x ) = -
+
x3 2

In Exercises 6 to I7 evaluate the giuen numerical expression.


6. log,(4 * 16) 7 . (log3 9)3 8. log3(g3) 9. 10g,(5-~)
1
10. log6 1 11. log,49 12. log,7 13. log,-
243

14. log,,( 1 010) 15. log2(256,) 17. log,, 2

In Exercises I 8 to 29 solve the giuen equation f o r the indicated uariable using the dejinitions and properties of exbonential and logarithm functions.
18. 28+1 = 232-2 19. 321+1 = 34x-5

20. log3(8x - I) = log3(3x + 4) 21. log5(4x + 5) = 2


22- 43-1 = 232+l 2.3. 45x-2 = 64
1
24. log5(4x + 1) = log,(5x - 7) 2 5 . -10g(2-~) = 0
2
1
26. logb 10 = - 27. logb8 = 1
2

1
30. Write as a single term: 2 log, 4 - -log, 64 - log, 2. ( a ) Choosing a n arbitrary X in the range o f f and using
2
the Intermediate-Value Theorem to showf -l( X ) exists in

(+r
31. Write as a single term: log,,[log3(log, 125)].
I.
:S2. Graphy = a n d y = log,,, x in the same coordinate ( b ) Showing there cannot be two different numbers x1
system. and xp in I with f -,(X) = x, and f -l(X) = x2. (Use the
strictly increasing hypothesis.)
3.3.

system.
Graphy =
(9a n d y = log,,, x in the same coordinate 35. Explain why the graphs offand f -1 can cross only on the
diagonal y = x.
34. Suppose f is continuous and strictly increasing on an
interval I. Prove (7.3.2) thatf has a n inverse f whose -
range is I by:

Derivatives of Exponential
Section 7.4: and Logarithm Functions

Objectives: DERIVATIVES OF EXPONENTIAL FUNCTIONS


1. Differentiate exponential and We have all heard statements about the exponential growth of energy
logarithm functions of any base consumption, population, and what not, with the usual implication that the
b > 0. growth rate is not only large, but increasing. Our best tool for dealing with rates
2. Differentiate combinations of
of change is the derivative, so we shall now calculate the derivative of a n
these functions with other
arbitrary exponential functionf(x) = b of base b >
0. For reasons to become
differentiable functions. apparent shortly, we first attempt to calculate the derivative at x = 0.

3. Understand the meaning and


importance of the number e as a
base for the natural exponential
and logarithm functions.
SECTION 7.4: DERIVATIVES 261

= lim- bh - 1
if this limit exists
h-0 h
This, like the (sin Ax)/Ax limit, is not obvious. It is reasonable to assume that
f (x) = b" has a tangent a t x = 0. Thus we now assume that f '(O)(the previous
limit) exists.
Computingf'(x) at a n arbitrary x, we have

= lim b"bh - 6" by [6.3.6(a)]


h-0 h
b"(bh - 1)
= lim
h-0 h
bh - 1
= 6" lim ___ since b" does not involve h
h-0 h
= b'f'(0) by our earlier calculation
Thus we have:
7.4.1 Theorem If f ( x ) = b", thenf'(x) = 6" sf'(0).

Geometrically, we have shown that the derivatzve of a Qpical exponentzal function


f ( x ) = b" at any x zs the product of itself with the slope at x = 0. Since all the
exponential functions satisfy b0 = 1 and hence pass through (0, I ) , and since the
derivative of each depends upon its slope at (0, l), it is natural now to examine
slopes of exponential functions at or near x = 0.
We saw that for most scientific purposes, the radian-based trigonometric
functions arc used because otherwise all derivatives involve a "correction
factor" (which is ~ / 1 8 0if degree-based functions are used). The same is true
here: T h e "correction factor" in this case is f '(O), the slope at x = 0. Thus the
simplest derivative will be provided by that basefor which f ' ( 0 ) = 1. Our task then is
to determine which base b yieldsf'(0) = 1; that is,
for Lerhich base b does f ( x ) = b" have slope 1 at x = O?
Recall our earlier statement [7.3.7(b)1:
(1) The larger the base b >
1, the steeper the graph of f ( x ) = b", x 2 0. Al'so,
( 2 ) iff'(0) exists and is nonzero, then the graph off(x) = b" is concave upward.
The second statement follows from (7.4.1) becausef"(x) = b " [ f ' ( 0 ) ] 2 .[Note
6" > 0 for all x so thatf"(x) 0.1 >
As we know, statement ( 2 ) means that as x increases, so does the slope. Our
search for the "best base" begins with the slopes of
r(x) = 2" and g(x) = 3" near x = 0
At this point, we do not know the slopes of these functions at x = 0. However,
since each is concave upward, the slope of the tangent T, to r(x) = 2" at x =
0 [r'(0)1 is less than the slope of the line L, through (0, 1) and R = (0.1, 2O.l);
see Fig. 7.29(a). Similarly, the slope of the tangent T, tog(.) = 3" at x = 0 [g'(O)]
is greater than that of the line L, through S = (-0.1,3-0,1); see Fig. 7.29(b).
Using a calculator, we find that the slope of L , is:
r(O + 0.1) - r(O) - 2 O . l - 1
=: 0.72
0.1 0.1
262 CHAPTER 7: TRIGONOMETRIC, EXPONENTIAL, LOGARITHM FUNCTIONS

A r(x) = 2"

I
(a)

Fig. 7.29

O n the other hand, the slope of L, is


g[O + (-O.l)] - g(0) - 3-0.1
-
- 1
z 1.04
-0.1 -0.1
Thus we know:
r'(0) < 0.72 < 1 < 1.04 < g'(0)
Hence 3" is too steep and 2" is not steep enough at x = 0. Whatever the "best
base" zs, it must be between 2 and 3.
We now assume that such a base between 2 and 3 exists and make the
following definition.

7.4.2 Defiaition We define the number e to be that number for which the slope of
f ( x ) = e x is exactly 1 at (0, I), that is for which f '(0) = 1. The function
f ( x ) = ex is often called the natural exponential function (see Fig. 7.30).

1 :i
f ( x ) = ex
This number is called e after Leonhard Euler, a n eighteenth-century Swiss
mathematician who first recognized and exploited its wide-ranging significance.
It is one of the most important numbers in all mathematics. It is irrational and,
,' to 15 places,
/ Tangent line
e z 2.718281828459045
We shall see it many times in the coming pages and later show also how it can
be defined in terms of a limit (see p. 266).
Now, armed with definition (7.4.2) we recall from (7.4.1) that withf(x) = e x :
/
I
Fig. 7.30

Hence the derivative of f ( x ) = ex is itself! By the Chain Rule, if u ( x ) is a


differentiable function of x , we have

Example 1 Compute ~ ~ [ e 5 ~ ~ + 3 ~ ] .

Solution Dx[e5X2+32]= e522+3X . (lox + 3) 0

Because the natural exponential function has such a simple derivative, we


use it to complete the task we began earlier, finding the derivative of a n arbitrary
exponential function. To do this we need:
SECTION 7.4: DERIVATIVES 263

7.4.4 Definition The inverse of the natural exponential function ex is called the
natural logarithm function and is denoted In x (read "ell n of x " ) .

Hence we write ln(x) instead of log, x. This means


(3) ln(ex) = x for all x and elnx = x for positive x

because ex and In x are inverses. As we know, the graph of the natural logarithm
function is the mirror image of the graph ofy = e x across the diagonal? = x.
From Fig. 7.31 we can see that l n x <
0 for 0 x < <
1. Moreover,
In1 = 0 and lne = 1
Also, sincey = ex is asymptotic to the x axis,y = In x is asymptotic to they axis
(limx,o+ In x = - co).

Now, suppose we are given an arbitrary exponential functionf(x) = 6" for


some base b >
0. Applying the right side of (3) with x = b, we have
b = elnb

Raising each side to the xth power, we have b" = (ebb)", or, by [7.3.6(6)],

7.4.5 bX = e(ln b ) l

This means that we can express any exponential function in terms of the
natural exponential function. But then, knowing that 6" is merely ekx for the
constant k = In b, we can apply (7.4.3) to conclude, for ~ ( x a) differentiable
function of x, that

D,[ b U ( " ) ] = ox[&


b)u(z)I = b)u'(x) = bU(")u'(x)lnb
e('n b)U(P)(ln

or

7.4.6 D,[bU(")]= bU(")* u'(x)ln b

Example 2 Compute DP[2x3+5x-7].

Solution = 2x3+5x-7(3x2+ 5)ln 2


Dx[2z3+5x-7]

Y Y = ex

Fig. 7.3 I
264 CHAPTER 7: TRIGONOMETRIC, EXPONENTIAL, LOGARITHM FUNCTIONS

As with the trigonometric functions, successively higher derivatives of


exponential functions are often regular. For example, each higher derivative of
y = e3' merely introduces a factor of 3 :

PROGRESS TEST 1

1. Differentiate the following functions on their natu- 6. (Requires calculator) For f ( x ) = b" we know
ral domains: f ' ( 0 ) = bo In b = In b, so we can compare the slopes
( u ) f ( x ) = e-" ( b ) f ( x ) = e2-"* of various exponential functions at their common
+
( c ) g ( x ) = e x 2 (4x3 x) ( d ) g(t) = 2t3+2t point (0, 1). Fill in the following tables to four deci-
(e) y = T ' (fly= ma1 places. Note that table (6) agrees with the value
ex of e given earlier.
(g) s ( x ) = - (simplify your answer)
2"
( h ) t ( x ) = ex--(' In 2)
2. Compare the answers to Prob. l(g) and (h) and
(a)
b
1 Slope f b X
al (0,l)
(b)
b
___
~

explain using (7.4.5).


1/2 2.5
3. Given that e x Y = x , determine &/dx using implicit 1 2.6
differentiation. 2 2.7
4. Name the one nonexponential function that is iden- 3 2.71
tical to its derivative. 5 2.72
10 e
5. Determine the 100th derivative of g ( x ) = ekzfc.

DERIVATIVES OF LOGARITHM FUNCTIONS


Sincey = In x meansy = log, x , we know by definition of logarithm functions
(as inverses of exponential functions) that
eY =x
Also, since e x is differentiable with no horizontal tangents, we know by (7.3.3)
that l n x is differentiable. Thus we can differentiate the above equation im-
plicitly to obtain:

But eY = x , so for y = In x we have

4 ---1
-
d x x
For u(x) differentiable and positive [so In u(x) is defined], we apply the Chain
Rule to get:

7.4.7

Example 3

Solution
Compute D,[ln (4x5 + x 2 + 3x + 2 ) ] .
D,[In (4x5 + x 2 + 3x + 2)] =
2 0 ~ 4+ 2x + 3
4x5 + + 3x + 2 x2
.
SECTION 7.4: DERIVATIVES 265

Example 4 Givenf(x) = In (l/x), computef(x):


( a ) Directly.
(b) Using thatf(x) = In (1) - In ( x ) .

Solution

(b) f ( x ) = 0 - In (x) [In (1) = 0 because e0 = 11


Thusf(x) = -1/x.
The natural logarithm function is defined only for positive numbers, but
In 1x1 is defined for all nonzero x . In practice we often need the composition of
the natural logarithm function with other, not necessarily positive, functions
<
u(x). So that the composition be defined for those x making u(x) 0, we use
lnlu(x)l instead of ln(u(x)).

7.4.8 Theorem If u ( x ) is differentiable and nonzero, then

Prooj We shall prove the case u(x) = x , from which the general case will follow
via the Chain Rule.
If x> 0,IxI = x, and so the derivative has already been computed [in (7.4.7)]
to be l/x.
<
If x 0, then 1x1 = -x > 0, so
lnlxl = In( -x) = In[( - l)x]
By (7.4.7),

(-1) - 1
Dx[ln [(-1)x]] = ~ -
-x x

Hence Bx[lnlxl] = 1/x in both cases. W

We can repeat for general logarithms the implicit differentiation that gave us
the derivative of the natural logarithm:
y = log, x implies by =x
Hence by (7.4.6), bu((dy/dx)ln b = 1, so

dr 1 - 1
- - ~

dr bylnb xlnb
Again, by the Chain Rule, if u ( x ) is a differentiable function of x , then

7.4.9

Example 5 +
Givenf(x) = 10g2(x3 x ) , findf(x).
3x2+ 1 1 .
Solution f ( ~=) ___-
x 3 + x In 2
266 CHAPTER 7: TRIGONOMETRIC, EXPONENTIAL, LOGARITHM FUNCTIONS

PROGRESS TEST 2

In Probs. 1 to 5 differentiate with respect to x.


In x
1. f(x) = ln(3x4 + 2x2 + x + 1) 2. h(x) = -
X

3. f(x) = ln(ln(x)) 4. T(X) = 2("'n")


5. f(x) = log,(4x3 + x2 - X) 6. Findy' given ln(xy) = 1.

DERIVATIVES AND THE GRAPHS OF EXPONENTIAL


AND LOGARITHM FUNCTIONS
We can reexamine the graphs of exponential and logarithm functions in terms
of their derivatives. (Since these graphs were the basis of our assumptions in
determining the derivatives, we are merely showing that these results are
consistent with the assumptions.)

Example 6 Show thatf(x) = 6" is everywhere increasing for b > 1, constant if b = 1, and
decreasing if 0< <
b 1. Recall Fig. 7.27, p. 257.
Solution Sincef(x) = b" > 0 for all x, we know that the sign off'(x) = bx In b is the same
as thesignofln b.Butln b > Oforb > 1 a n d l n b < OforO < b < 1. Forb = 1,
In b = 0, S O ~ ' ( X
=) 0 and thusf(x) = 1" is a constant, namely 1. 0

Example 7 For b > 1, show f(x) = log, x is increasing and concave down on (0, + co).
Solution Since both factors of f'(x) = (l/x)(l/ln 6) are positive on (0, + a),f(x) is
increasing there. Since

Y y = b" -
- -.-
lnb
-' < o
x2
for all x in (0, +a)

I we know that f(x) is concave down there. Recall Fig. 7.28, p. 259. 0

A LIMIT DEFINITION OF e
As we remarked earlier, the number e can be described as a limit. Originally,
we searched for a base b such that the line tangent t o y = b" at x = 0 would
have slope 1. But then the equation of the tangent line at (0, 1) isy = x 1. +
X Suppose for a moment that b >
1 is an arbitrary base not equal to e. Then the
graphs of
y=b and y = x + l
Wig. 7.33 intersect when 6" = x + 1. We already know they intersect when x = 0; but for
b # e, the liney = x + 1 is not a tangent line and intersectsy = b" at another
point Q. (See Fig. 7.32. For b <
e, Qis as shown. For b >
e, Qwould be to the
left of they axis.) Raising both sides of b" = x + 1 to the l / x power ( x # 0) we
have

7.4.10 b = (x + 1)""
SECTION 7.4: DERIVATIVES 267

Figure 7.32 is similar to the figures used in Chap. 2, Sec. 2.4 to interpret the
derivative as the slope of a tangent line. There, we took the limit as x + 0 and
let the point Q "move down" the given curve approaching P as the limit. Here,
we wanty = x + 1 to be the tangent line, so we hold it fixed, and consider what
happens as b approaches e. By definition of e, as b approaches e, the different
curves (for different bases b) intersect the liney = x +
1 a t points Q that move
alongy = x + 1 closer and closer to (0, 1). This means that the x coordinate of

1
2
3
2
2.25
2.37037
-
Qapproaches 0. But, looking again a t (7.4.10), this implies by the definition of

7.4.1 1 e = lim ( I + x)IiZ


+
e that as the left side approaches e, the right side is the limit of (1 X ) I /as~
x 0. Hence:

4 2.44 141 a-0


5 2.48832
This limit can be rewritten using the substitution u = l / x ( s o x = l/u). Since
10 2.59374
50 2.69 159
the limit in (7.4.1 1) exists, limZ+o-and lima+,,+ both equal e. But letting x 4 O+
100 2.70481 is equivalent to letting u -+ + co.Hence we can conclude from (7.4.1 1) that
1,000
10,000
50,000
2.71692
2.71815 9.4.12 e = lim (I
U++m
+ i)'
100,000
1,000,000
We can use this limit representation of e to check the 15-place value of e given
earlier and determine e to any specified degree of accuracy. You are invited to
Table 7.4 fill out and extend Table 7.4.

SECTION 7.4 EXERCISES

In Exercises I t o 36 determine the derivative of the given function.


1. f(.) = e(10~3+3z-2) 2. f(x) = ln(4x - 3x5)
3. g(x) = 104" + 3 4. y = ln(2x - 1) + e-"
5 . h(x) = er2/(4zp1) + 31x 6. y = (1nx)e"
7. y = In(e+) (simplify and explain) 8. y = e l n z (simplify and explain)
9. y = exz 10. y = (compare with D,[e2"])
11. f ( x ) = e Z 2 ln(5x - + 1) +
12. f ( x ) = e e s iog3(x4- x )
e 3x 14. f ( t ) = e t 2 l n t
13. g ( x ) =
+ 2x - 3
4x3

15. f ( x ) = 1n(x3+ 23s +


16. f ( t ) = sin(et)
ln(x - 1)
17. f ( x ) = ln[sin2(5x2- 7x)] 18. g ( x ) = logI0(cot4x)
19. ~ ( t =) &in t + cos t ) 20. f ( x ) = f ? T 2 + 6 W
21. m ( x ) = ln(sec x tan x ) + 22. y = e(eS)
23. y = (e")" (compare with Exercise 22) 24. f ( x ) = ex cos x

25. g ( x ) = e" 'OS '. 26. m ( t ) = In -


(1:
27. n(z) = ln(zP) 28. N ( z ) = p l n z (compare with Exercise 27)
29. f ( x ) = sin2(e") 30. g ( x ) = cos2(e")
31. y = sec(eZz) 32. y = e x 2 csc x
33. f ( t ) = e c o s 2 t 34. g(t) = esin2t

1 36. y = ln(1nx)
35. y = ~

In x

In Exercises 37 to 40 determine the indicated higher-order derivative.

37. dny
- fory = 2" 38. dny
- fory = 32"
dx fl dx"
40. g'4'(x) for g(x) = e("')
268 CHAPTER 7: TRIGONOMETRIC, EXPONENTIAL, LOGARITHM FUNCTIONS

In Exercises 41 to 46 determine &/dx using implicit dferentiation.


4,l. ex+Y = e x 42. x2ey = 2
43. In
;( ) + q = 3
- 44. h ( x 2 + y 2 )= 2 l n x + 2 lny
45. (lnx)2 + (1ny)2= 1 40. eSing =x
47. Where is the line that is tangent toy = x - l n x hori- 48. Where is the curvey = e?' concave downward?
zontal? 50. Graphy = ln(x2).
49. Graphy = lnlxl.

SECTEON 7.4 APPENDIX AN INTEGRAL APPROACH TO LOGARITHMS

Our informal development of the exponential and logarithm functions began


with a n informal definition of 6" (especially regarding irrational x), assumed
their basic algebraic properties (rules for exponents), and also assumed their
continuity and differentiability. We then defined the logarithm functions as
inverses of the exponential functions. We shall now outline a more mathemati-
cally complete development that is almost the reverse of this and that will
illustrate the tremendous power of the Fundamental Theorem of Calculus.
Recall that Form 1 (6.3.6) says: Iff is continuous on [a, b] with x in [a, b],
then we can define a new function F by F ( x ) = J i f ( t ) dt. Furthermore,
F'(x) =f(x).

7.4.A. 1 Definition For any x > 0, define l n x = [-


+
Since l / t is continuous on (0, a),
:. +
we know In x has domain (0, co) and
is differentiable (hence continuous) with derivative l/x. We know In 1 =
J : ( d t ) / ( t ) = 0 by [6.3.4(6)]; it follows that In x of (7.4.A.1) is the same as In x
informally defined in Sec. 7.4 because the two functions have the same domain,
the same derivative, and are equal at x = 1. All the properties of In x (including
its graph-using the argument of Example 7) can be established from definition
(7.4.A.1). (For 0 < < x 1, In x is in the negative of the area of the shaded region
in Fig. 7.33.)
The algebraic properties of Inx now follow. For example, we shall prove
ln(xlx2) = In x1 + In x2 xl,x2 >0
by regarding x1 as a variable, x2 as a constant, and differentiating:

Dxl [ln(x,x,)] = -x2


=-1 = Bxl[In xi]
XlXZ x1
Pig. 7.38
Hence In(xlx,) = In x1 + C by (4.4.6). But for x1 = 1, ln(1 x2) =
In x 2 = In 1 + C = 0 + C, so C = In x2, finishing the proof. See the exercises
for further development.
Sinceln xis a strictly increasing differentiable function with domain (0, + co),
range R (see Exercise 1) and no horizontal tangents, we know by (7.3.3) that In x
has a differentiable inverse. Because In x is the same as in Sec. 7.4, we would
expect it to have the same inverse, namely, e x . However, we must still show that
this inverse-call it ex-has the properties attributed to it in Secs. 7.3 and 7.4.
But its derivative follows from that of In x : Set y = ex, so lny = x. Then
( l / y ) ( d y / d x ) = 1, which implies dy/dx = y = ex, as expected. The algebraic
properties follow from those of l n x as well. For example, to prove
exifxz = exiexz

we note that x1 = lnyl and x, = lny, for someyl,yz > 0 (the range of In x is
SECTION 7.5: APPLICATIONS OF TRANSCENDENTAL FUNCTIONS 269

R). Since e x is the inverse of the logarithm function,yl = exl, y z = e'z. Then
e"1+"2 = e'Jn %+lnh) = eh(vib) = y I y 2 = exlexz. Other properties follow similarly.
Finally, we can use the fact that t h e (natural) exponential function is defined
for all real numbers to define b" to be e x h b . T h e properties of b' now follow
from those of e x .

SECTION 7.4 APPENDIX EXERCISES

1. Prove lim,,,, In x =+ co as follows (a similar argument ( b ) Use the continuity of lnx, the definition of e" as the
shows lim,,o+ In x = - 00 from which it follows that the inverse of lnx, and either (7.4.11) or (7.4.12) to give an-
range of In is R.) other proof that E = e , where e is as defined in Sec. 7.4.
(a) Suppose M > 0 is given. Show there is a positive 6. ( a ) Using that X' = e""' by definition for all r (including
>
integer k such that k In 10 M (note In 10 0). > irrational r), prove that D,(x') = r x r - l .
( b ) Show that for x > >
l o k , lnx M . ( b ) Use ( a ) and Exercise 2 to prove that [7.3.11(c)] holds
2. Using a derivative argument analogous to that in the for all p .
previous discussion, prove [7.3.1l(c)] for rational p . 7. Use the corresponding property for e x and the definition of
3. Use [7.3.11(c)] with p = - 1 to prove [7.3.11(d)]. >
bx as e Z h b ( b 0) to prove [7.3.6(a)].
4. ( a ) Using the Intermediate-Value Theorem (2.5.12) and 8. Repeat Exercise 7 for [7.3.6(b)].
Exercise 1, prove that there exists a unique number E such 9. ( a ) Use (7.4.A.1) and the substitution u = t / a to prove
that 1nE = 1. that In b = JEb(dt/t)(a. b >0).
( b ) Use ( u ) and the fact that ex is the inverse of the strictly +
( b ) Using part ( u ) and J;b(dt/t) = J ; ( d t / t ) Jib(dt/t),
increasing natural logarithm function to conclude that prove that ln(ab) = In a + In 6.
E = e.
5. ( a ) Forf(x) = In x , computef'(1) using the definition of
derivative limh,o[(f(x + h ) - f ( x ) ) / h ] at x = 1.

Applications o f Derivatives of
Section 7.5: Transcendental Functions
-~

Objectives: LOGARITHMIC DIFFERENTIATION


1. Use logarithmic differentiation to Differentiating a function such as
compute derivatives of certain
complicated functions.
(3x7 - 2x2 +x ) " m
f(x) =
q x 2 - 7x +1
2. Use derivatives of trigonometric,
exponential, and logarithm involves complex a n d repeated applications of virtually all the differentiation
functions to graph transcendental rules. Just as common logarithms (base 10) a r e used to simplify complicated
functions, and solve optimization numerical computations, natural logarithms can be used to simplify compli-
and related rate problems. cated algebraic calculations. We illustrate how to use the properties of natural
logarithms to take the derivative of the above function.

First, take the natural logarithm of both sides and break down the right side to a simple
sum and dgerence:
+ x)"-
W f ( x ) l = In[
(3x7 - 2x2
vx2 - 7x + x3+ 'I
+ x ) ~ +] l n d m - In v x 2 - 7 x + 1
ln[(3x7- 2x2
1 1
= 2 ln(3x7 - 2 x 2 + x ) + ln(x3 + 1) - - ln(x2 - 7x + 1)
-
2 3

Second, dferentiate both sides, getting

f-=2.
'(XI
21x6 - 4x + 1 +-.---.
1 3x2 1 2x-7
f (4 3x7-2x2+x 2 x3+l 3 x2-7x+l
270 CHAPTER 7: TRIGONOMETRIC, EXPONENTIAL, LOGARITHM FUNCTIONS

Finally, solve f o r f '(x) by multiplying by f(x):


+ 2 +-- 3x2
42x6 - 8x
3x7 - 2x2 + x 2x3 + 2
2x - 7
3x2 - 21x +3 1
(3x7 - 2x2 + x-")

This three-step procedure is called logarithmic dgerentiation.


The above function, although messy, could have been differentiated using
conventional means, but the following cannot.

Example 1 DifferentiateJ(x) = x"(x > 0)


Solution lnf(x) = ln(x") = x In x
so

Hence f '(x) = x"(1 + In x). 0

Example 1 illustrates the following general principle:


To determine the derivative ofanyfunction ofthef0rmf(x) = U(X)~(", where u(x) and
v(x) are dgerentiable (v(x) > 0) use logarithmic dgerentiation.

Example 2 Differentiatef(x) = (3x + l),'".


Solution lnf(x) = (x2e")ln(3x + I), so, applying the Product Rule twice,

It turns out that logarithmic differentiation is also handy in proving and


extending some of the standard differentiation formulas. We can now confirm
that the Power Rule works for any real-number exponents, not just rational
exponents. The argument is ultimately based upon the assumption that the
exponential functions are defined and continuous for all real numbers. This
assumption was shown unnecessary in Sec. 7.4, Appendix.

7.5.1 (Power Rule for any Real Exponent) Given that u(x) is differentiable
andf(x) = [u(x)IS (s any real number), then
f'(X) = s ' [u(x)]S-1 * u'(x)
ProoJ
In f(x) = s In u(x)
so
SECTION 7.5: APPLICATIONS OF TRANSCENDENTAL FUNCTIONS 271

Hence

PROGRESS TEST 1

Find the derivatives of the following three functions:


d7FTVFTG
*. f(.) = p=mm
qiG-7
1. y = 3. f(x) = x@)
.( - 1117

USING DERIVATIVES TO GRAPH


TRANSCENDENTAL FUNCTIONS
The derivative can be extremely useful in graphing transcendental functions.

Example 3 +
Graphf(x) = cos(2x) 2 cos x on [77/4, 2771 and determine its extreme values.
Ignore inflection points.
Solution f ( x ) = - 2 sin 2x - 2 sin x = - 2(sin 2x + sin x)
= - 2(2 sin x cos x + sin x) (by a double-angle formula)
= -2 sin x(2 cos x + 1)
Hence f (x) = 0 when sin x = 0 and when cos x = - 1/2, that is, when
x = 0, 77, 277,2~/3,and 477/3. We ignore x = 0 since it is outside [~/4,277].We
now chart the sign off :

I,;;,l,;;,I,;io,/,;;,
Z(c0s x) +1
~

_-_ ____ sin x


--___ _- --- -_ -_

_-_ -_-
0

We conclude that there are minima at x = 2m/3, 477/3, and a maximum at


x = T .f(277/3) = -3/2, f(.) = - l , f ( 4 ~ / 3 ) = -3/2.Alsof(a/4) = f i a n d
f (27) = 3. Hence the function achieves its maximum at x = 277 and its mini-
mum at the two relative minima x = 27/3, 477/3; see Fig. 7.34. 0
273 CHAPTER 7 : TRIGONOMETRIC, EXPONENTIAL, LOGARITHM FUNCTIONS

To graph exponential and logarithm functions, it is useful to assume the


following pair of limits. These will be proved in more general form in (14.1.5) of
Chap. 14.

7.5.2 Theorem ( a ) lim


x-frn
h=
x
0
X
( b ) lim - = 0
x-+m ex
Each limit may be regarded as a statement about the relative growth rates of
the functions involved. Notice that, for example, letting x = we have

lnx ln(lO1OOO) 1000 In 10 In 10 2.3


X
-
101000 -
- 101000 - 10997 - 10997 = 0.00.. .023
996 zeros

Example 4 Determine the maximum value of f ( x ) = (In x ) / x .


Solution Note thatf(x) is defined only for x > 0 and is negative in (0, 1).
x(l/x) - (lnx)(l) - 1 - lnx
f (x) = -
X2 X2

Now, In e = 1, with
1 - lnx 2 0 when x 2e
1 - lnx 5 0 when x 2e
+
Hencef(x) is increasing in (0, e) and decreasing in (e, 0 0 ) with the single
+
relative maximum in (1, co) at x = e. By (5.1.3), f ( e ) = l/e must be the
maximum value of J: 0

PROGRESS TEST 2

1. Graph f ( x ) = xe, noting all pertinent data. Then graphy = x +sinx. (We could call this the
2. Determine where y = x + sinx is increasing and step-with-a-thick-rug function.)
decreasing and locate any and all relative extremes.

OPTIMIZATION AND RELATED RATES


Transcendental functions arise in optimization problems as naturally as the
functions considered in Chap. 5.

Example 5 Suppose a 10-ft-long symmetric V-shaped watering trough is to be made from


two boards, each 1 ft wide. What vertex angle between the boards will yield the
maximum capacity?
Solution It is most convenient to break the cross section into two right triangles. (See
Fig. 7.35.)
Volume = length times cross-sectional area
1
= (lO)[(sinx)(cosx)] (sinx = - base and cosx = height)
2
= 5 sin 2x (double-angle formula)
Clearly, we are maximizing V ( x ) = 5 sin 2x, with 0 5 x 5 ~ / 2 . Now
Fig. 7.35 V ( x ) = 10 cos 2x = 0 when 2x = n/2, that is, when x = ~ / 4 . Since
SECTION 7.5: APPLICATIONS OF TRANSCENDENTAL FUNCTIONS 273

V"(x) = -20 sin 2x and V"(7~/4) = -20 <


0, we know that x = 77/4 is a
relative maximum.
The two endpoints x = 0, x = 77/2 yield zero volume, so the best vertex angle
is 2(7~/4)= 7 ~ / 2rad, which yields a volume of 5 ft3. 0

Trigonometric functions are, of course, useful in situations where related


rates involve changing angles, as indicated in Prob. 2, Progress Test 3.

PROGRESS TEST 3
~

A horizontal gutter is to be made from a long piece 2. A boat is passing in a straight line 300 ft offshore
of sheet metal that is 0.8 m wide, by symmetrically from a n onshore radar installation a t the rate of
turning up 0.2 m on either side to a n angle that 50 ft/s. At what rate must the radar turn when the
maximizes the carrying capacity. If x is the angle the boat has traveled 300 ft past its closest point to the
edge makes with the vertical, what x gives the maxi- radar?
mum carrying capacity? (Assume that the carrying
capacity is directly proportional to the cross-sec-
tional area.)

SECTION 7.5 EXERCISES

I n Exercises I to 13 use logarithmic dzferentiation to determine the derivative of the given function.
(x - 5)* + 3
1. y =
( 2 - 4)4
3. f ( x ) = (x + 1)" 4. f ( x ) = (2x + 1)"

5. k(x) = q z d w - q x 4 + 4x + 4 6. m ( x ) =

(x - 5 1 4 ( 3 ~ 3+ gX - 2)7dGTT X9ellx
7. t(x) = 8. g ( x ) =
q 3 x 7 - 2x +1 (3x7 - 2x - 7)4

9. f ( x ) = x($) 10. f ( x ) = (2x)(lnh)


2x(1n x)
11. g(x) = ~ 12. h ( x ) = e~ (use Example 1)
e 5x 14. Determine D x [ x " + l ] ( a ) directly and ( b )
13. g(x) = (x")2 using x"+l = x x x and the Product Rule.
15- Repeat Exercise 14 for g ( x ) = ( X " ) ' using that 16. Use logarithmic differentiation to reprove
(x")= ~ x2" and the Power Rule. the Product Rule.
17. Use logarithmic differentiation to reprove the Quotient 1 8 . Determine the concavity of g ( x ) = e'.
Rule.

In Exercises 19 to 30 graph the given function on the indicated interval.


19. f ( x ) = x l n x on (0, +co) 20. f ( x ) = e-"' on (-00, +co)
2 1. y = (In x ) on ~ its natural domain 22. g(x) = cos 4x on [0,277]
28. h ( x ) = 3 sin(2x - 77) on [ - a , 27771 24. f ( x ) = 2x +
sin 2x on [O, 27r]
25. y = x" on (0, +co) 26. y = x x 2 on (0, w ) +
27. f ( t ) = e p t sin t on ( - 00, + co) 28. y = sin-
277
t
on (0,2)

277
29. y = t cos - on (0,4) 80. y = esin on [ - ~ , 4 7 r ]
t
3 1. A searchlight sweeps the sky at a rate of I rad/min. If 32. Given a sphere of radius 10 units, find the altitude of the
the cloud level is 1/4 mi above the ground, how fast (in right circular cone of maximum volume that can be
radlmin) is the searchlight beam sweeping the clouds: inscribed in the sphere. Let 2x be the vertex angle of the
( a ) Directly overhead? cone.
( b ) 1/4 mi past the point directly overhead?
274 CHAPTER 7: TRIGONOMETRIC, EXPONENTIAL, LOGARITHM FUNCTIONS

33. What is the narrowest alley a 64-ft-long hook-and-ladder 36.


Given an 8-ft-high prison wall and a 16-ft ladder, what is
truck can turn into from a 27-ft-wide street ignoring the the largest horizontal distance an inside ladder can reach
width of the truck? outside the wall. (Let 0 be the angle between the ladder
34. (A rocket tracking problem) A rocket is fired 200ft and the ground.)
exactly downwind from an observer who measures that 37. What is the natural domain off(x) = In(sinx)?
the angle the rocket makes with the horizontal is in- 38. If x increases from 100 to 101, what is the approximate
creasing at the rate of 77/21 rad/s. The wind is blowing change in f (x) = x 2 In x ? (Use differentials.)
at 20 ft/s. What is the rate of departure of the rocket 39. Find 4 in terms of the independent variable:
from its launching point after 7 s have elapsed? (Note: To +
(a)y = sec3(5x l), where x = x(0) = e s
maintain the constant angle increase with the horizon- (b) y = 4e3x*+2, where x(t) = t5 +
t4
tal, the rocket must be accelerating.)
35. Find any extreme values of f(x) = sin x
on
1 - 2 cos x
[O, 2771.

CHAPTER EXERCISES

R e v i e w Exercises
(Note that Exercises 1 to 21 involve only precalculus ideas.)
I n Exercises I t o 4 determine f -,(x) zf it exists. r f f does not have an inverse, name two numbers x1 and x, with f (x,) =f (x,).
1. f(x) = x j - 7 2. f ( x ) = (x - 5 ) 3 - 4 3. f(x) = 4x2 +
3x - 1
4. f(x) = -\"Jx-5 5. Which is larger, 6. For which x is
(2.5), or 32.5? tan 20x undefined?
7 . In which quadrant is the terminal side
of an angle of - 1000 rad?

I n Exercises 8 to 13 evaluate the given expression.

8. log,[( m ) ( x + 1)5] if x = 2 9. 510g,($%) 10. log,( 1/4)-3'7

11. log,(3fi) 12. 3(2 log,(41)) 13. (log, 4)3 - 10g,(4~)

14. If log,, 51 = a : 15. Simplify logb(b(logb(x)).


(a) What is log1,(5.1)?
(b) What is log(0.51)?
(c) What is log,,(51,000)?

I n Exercises 16 to 21 solve the given equation f o r x.


16. log3(x 1) + +log,(.) =I 17. log2(256) = x 3
18. log4(x - 1) = 2 19. log,(x + +
1) logz(x + 2) = log,(12)
20. log,9 = 3 21. log,(log,(x)) = 0

In Exercises 22 to 66 determine the derivative of the function with respect to the indicated independent variable.
22. g(x) = sin(x3 - 7x2 - 6x + 2) + cos(x5 - 4) 23. h(x) = er2+2++1 - ln(x2 - 3x - 5)
24. f ( z ) = (tanr)sec(z2 1) + 25. m(x) = cot-
etan(4x)
26. f(x) = sin2(2x + 1) * cos2(2x - 1) 27. s(x) = -
sec3x

28. f ( t ) = [iog2(t2 + t ) ] - 24t xe '


29. f(x) =
+
COtS(X2 9x)
I I

dx-1 dx-1
30. g(x) = + 31. h(s) = d-
V G T
~ ~

e m
SECTION 7.5: APPLICATIONS OF TRANSCENDENTAL FUNCTIONS 275

38. (u) f(z) = sin(cos2(z - 1))


( b ) g(z) = sin2(cos(z - 1))
4 1. ,f(x) = ln(csc x cot x)
+
43. m(x) =

45. ( u ) ~ ( x =
In(x3 2x - 4)
+
x5 4x4 - 6
) sins(cosx)
1
( b ) h(x) = sin(cos5x)

48. m(x) = tan d m - ln(x2 + 8)


50. t ( u ) = ln(tan2(v4 I)) +
52. p ( y )= esin2(y-4)
54. g(x) = loglO(x4- x + 12)
55. f ( x ) = In(cscx + cot x) 56. r(x) = T ~ - ~ ~

log2(9x - 1)
* (x2 - 6)7 58. g(x) = sinx2cot(x2 + I)
Z9. ~ ( x )= ( s i i i ~ ) ~ ~ ~
x
60. f(.) = 4
+ tan3x
- sin23x
62. f (x) = 7 sin 2x + 5 cos 3x

In Exercises 67 to 78 delemine dyldx, where y is an implicit function .f x dejned by the given equation.
67. ( u ) sinxy + secx = 1 68. ( u ) eZC"yJ =
(b) h ( x 2 +y2)= e
(Can you give the
graph of this equation?)
69. e' +
e" = xex 70. sin(x + y ) = cos(x - y )
71. xsiny + y c o s x = 0 72. e'+Y +x +y = 1
73. 10"+U = 2"-y 74. tanxy + xy = 0
75. csc(x + y ) - sec(x - y ) = x 76. sin(x + y ) = y c o s x

In Exerci.te.c 77 to 85 sketch the giuen,function (ignore injection points).


77. f(x) = sin 2x + 2 on [0, 2771 78. f(x) = x2ex on ( - 00, +w)
I
79. f ( x ) = el" on its natural domain 80. y = on [0, 2571
1 - sinx
81. y = 4cos8x on [ - v , T ] 82. y = lnlcosxl on [0, 3 ~ ]
83. y = Inlsinxl on [0, 2771 84. y = e c o s x on [ - 2 ~ , 2 ~ ]
85. y = p S l n 2x on [0, 2771
86. A conical tent is made from a circular piece of canvas
of radius 10 ft b y cutting out a sector and sewing up
what remains. Under the condition that the sector cut
out be at least 7r/2 rad (to make a rain flap), what is
the largest-volume tent that can be made?
87. Differentiate g(x) = ln[ln(ln(x))]. 88. Differentiate g(x) = e(es)L.
89. Differentiatef(x) = vec2. S O . Find the maximum off(.) = e x sin x on [O, 6771
+
91. Differentiatef(x) = (x9 2 ~ ) ' ~ " . 92. Determine g'(x) and g"(x), where g(x) = xe.
276 CHAPTER 7: TRIGONOMETRIC, EXPONENTIAL, LOGARITHM FUNCTIONS

I n Exercises 93 to 95 a particle movin,q in a stratqht line has its position at time t given by s(t). Describe the velocity and position of this particle.
93. s ( t ) = e-t sin t notes that the planes angle of elevation is 77/3 rad and
94. s ( t ) = /sin ti (Note especially what happens as multi- is increasing at the rate of (1/120)n rad/s.
ples of v.) ( a ) How fast is the plane moving at this instant?
95. s ( t ) = t +
sin t ( b ) Suppose instead that the observer finds the same
96. The force necessary to pull a weight W along a rate of angle change when the plane is directly over-
horizontal plane with a cable is given by head, what is the planes speed?
F = kW/(cos 8 + k sin 8), where W is the weight, k is 98. Describe the dimensions of the smallest isosceles tri-
the coefficient of friction, and 8 is the angle the cable angle (in terms of area) that circumscribes a circle of
makes with the horizontal. radius 1. (Let 28 be the angle between the sides of
( a ) Given that k = fi,at what angle is the least force equal length.)
required to pull the weight? 99. ( a ) Give the acceleration function for a particle whose
(6) For a smaller coeficient of friction, k = l/*, distance from a given point is s ( t ) = et e8. +
what is the bcst anglc at which to pull thc weight? ( b ) Show that s ( t ) satisfies the differential equation
07. An airplane is flying at an altitude of 2 mi in a path dZOs/dt2o - s ( t ) = 0.
that will take i t directly over an observer. T h e observer

In ExerciseJ 100 to 102, if x increases,from 10 to 10.1, determine the approximate change in the given expression.
100. 2 101. lnx 102. sin(10x2 + x)

I. Use logarithmic differentiation to develop a formula 2. Compare the derivatives of ex+ and eey (both with
that gives the derivative off(.) = ~ ( x ) ~ ( [Both
) . func- respect to x and under the assumption that y is a
tions are differentiable and v ( x ) # 0.1 function of x).
3. Find the equation of the line tangent to the curve 1. Repeat Exercise 3 forf(x) = In(log2(x))at x = 4.
xy - esiny = n a t ( 1 , ~ ) .
5 . Given a sphere of radius r, what are the dimensions of 6. At what rate is the angle between the minute and hour
the right circular cone of maximum surface area that hands of a clock decreasing at 3 oclock (in radians per
can be inscribed in this sphere? (Let the cross-sectional hour)?
vertex angle of the cone be 28.)

In Exercises 7 to 12 sketch the given function, ignoring injection points

X
7. f ( x ) = on [O, 2771 8. y = on [ - n , r ~ ]
1 - sinx 1 - cosx
9. f ( x ) = sin(cos x) on [0, 4n] 10. f ( x ) = cos(sin x) on [0,4v]
cox x
11. y = 12. y = sin(e) on ( - co,21
1 - 2 sin x
1:I. Name at least one function g(x) such that
gY.1 = (1/2)dx).
14. For the measure of an angle in revolutions (so an angle
generated by one complete counterclockwise rotation
of the generating line has measure 1)
( a ) Give a formula analogous to (7.1.2) that converts
between radian and revolution measure.
( b ) Define siq,,(x) analogous to sindegand evaluate
sin,,,(x) at the measures of the same angles used in
illustrating sindegx.
(c) Graphy = sin,,,(x).
( d ) Determine D,[sin,,,(x)] using the Chain Rule and
the formula in (u).

( e ) Determine lim- sinrev(x) using (7.2.11) and com-


+-0 x
pare with D,[sin,,,(x)] at x = 0.
SECTION 7.5: APPLICATIONS OF TRANSCENDENTAL FUNCTIONS 277

sin(x + Ax) - sinx


IS. [Proof that for small Ax > 0, AX
N

cosx.] In the accompanying sketch, suppose that x


increases by A x >
0, so the intersection of the generat-
t
inglinewith theunitcirclemovesfromB = (cos x , sin x )
to P = (cos(x +
Ax), sin(x +
A x ) ) . Then % is the A
growth of the sine function and the (negative)
growth of the cosine function. T h e previous argument,
that limArr-O(sin A x ) / & = 1 when rotated by x radi-
ans, shows in effect that as A x + 0, the difference
between the straight-line segment betweenP a n d B and
the circular segment between P and B approaches zero.
For small Ax-that is, A x z 0-we shall assume that
they are approximately equal and thus regard the figure * J

0
DPB with circular side PB as a triangle and refer to it
as A DPB and refer to the circular side as PB.
( u ) Prove A DPB is similar to U B O .
sin(x + Ax) - sinx
(6) Conclude cos x.
AX

SELF-TEST
Chapter 8 %

279
280 CHAPTER 8: MORE ON TRANSCENDENTAL FUNCTIONS

Section 8.1: Growth and Decay of


Physical Quantities and Money

Objeetives: THE BASIC EQUATIONS


1. Define and use exponential We saw that the derivative of a typical exponential function is the product of
functions to describe simple itself with a constant, its derivative at zero. If this function is describing a
growth and decay phenomena, variable quantity A at time t , then d A / d t = A ( t ) * A'(0). We shall now show
including continuously conversely that if d A / d t is proportional to A ( t ) , then A ( t ) is an exponential
compounded interest. function. We begin by assuming
2. Set up and solve differential
equations describing growth/decay the rate of growth (or decay i f d A / d t <
0 ) at time t
phenomena in the presence of is proportional to the amount on hand at time t
constraints, including variable
constraints. This statement is often expressed as the differential equation

8.1.1 = kA
dt
where k is the proportionality constant. This situation occurs commonly enough
to be called natural growth (if k > 0) or natural decay (if k <
0). Below, when we
use these phrases, we mean that equation (8.1.1) applies. We can solve (8.1.1) by
first noting that the derivative d A / d t can be regarded as a quotient of differen-
tials d A and dt. We can thus separate the variables A and t by multiplying both
sides by dt and dividing both sides by A to get

8.1.2 -= k d t
A
The next step is to antidifferentiate, but to antidifferentiate the left side we first
rewrite the differentiation formula (7.4.8) in terms of A and t :
d dA/dt
-[lnlAl] =-
dt A
But this tells us that lnlAl is an antiderivative of
dA / d t
A
That is,

But d A / A = k d t by (8.1.2), so we can conclude that

8.1.3 1nlAJ= J k d l = kt +C
Then by definition of the natural logarithm,
IAl 1 ekt+C = e'ekt
Hence
A = *ecekt

Since +-eC is a constant, we can replace &eC by C, where IC,l = ec, to get the
natural growth/decay function

8. I .a A = COekt
SECTION 8.1: GROWTH AND DECAY OF PHYSICAL QUANTITIES AND MONEY 28 1

As with any general solution of a differential equation, this is really a n infinite


family of solutions, and additional information is required to determine a
particular solution. For example, if we know the amount on hand at a particu-
lar time, say at t = 0, then we can conclude that
A(O) = Coek'O = C,, * 1 = C,
Hence, in (8.1.4), C, is the amount present initially. Of course the amount A ( t )
present at time t depends not only upon the amount present initially but also
upon the rate k. Usually, k must be inferred from other given information.

Example 1 Suppose that the number of bacteria in a culture grows naturally from 10,000
to 800,000 in 20 h. Give the function that describes the number present as a
function of t and determine how many bacteria will be present after 30 h.
Solution Knowing that the growth is natural means that (8.1.4) applies, and so the
problem reduces to finding C, and k. If we take t = 0 to be the initial time, at
which 10,000 bacteria are present, we know A(t) = 10,OOOekt. Measuring t in
hours, we are also given that A(20) = 800,000, so
10,000ek'20= 800,000
or
-
ek'20 - 80

Taking the natural logarithm of both sides, we get 20k = In 80, making

Thus we know that


~ ( t=) 10,000e0~2~9t
We also know that
A ( 3 0 ) = 10,000e(0~219"30'
= 10,000e6.57z (10,000)(713.3698) = 7,133,698 0
Note that Example 1 makes the standard assumption necessary to apply
calculus: Although the number of bacteria is a n integer, we assume that A ( t ) is
a differentiable function oft. Of course, whenever we make such an assumption,
we should be certain that the answer is rounded off to the nearest integer.
If a quantity A is growing naturally, that is, according to the formula
A ( t ) = Coekt, it is often of interest to determine doubling time: Given
A(t,) = Coekti and A(t,) = Coektz (tz >tl)
how large must t, - t , be so thatA(t,) = 2A(t,)? We must solve Coektz = 2C,ekt1
for t, - t,. Divide through by C,, and take the natural logarithm of both sides:
kt, = In[2ektl] = In 2 + kt,
Hence
kt, - k t , = In 2
so
In 2
t, - t, = __
k
Because the right side of this equation involves only In 2 and k, we can conclude
that doubling time t, - t , depends only on the magnitude oft, - t , and not on:
( u ) C,, the amount at t = 0
( 6 ) The choice of time t, on which the calculation is based
282 CHAPTER 8: MORE ON TRANSCENDENTAL FUNCTIONS

( c ) The quantity A(t,) at t = t,


Since In 2 =: 0.69, we can conclude a useful rule of thumb for approximating
doubling time: Divide 0.69 bJ, k.
Clearly, tripling time will be given by t, - t, = (In 3 ) / k . This argument
suggests the following more general theorem:

8.1.5 >
Theorem For any b 0, if A = Coekt,then the amount of time A t required
for A to change by a factor of b is given by
In b
At = -
k

What sort of function A ( t ) = COektchanges by a factor of b in each unit of


time, that is, for A t = l ? By (8.1.5) we need k = In b, from which it follows that
A(t) = = Cobt by (7.4.5)
In other words, the exponential growth functions we arrived at through the assumption that
the growth rate is proportional to the amount on hand as rejected in the dgerential equation
d A / A = k, is exactly the same as that sort of growth we discussed when introducing
exponential functions in Sec. 7.3!
Of course, if the quantity is decaying naturally with 0 < < b 1, then
In b < 0, so k < 0. In such an instance the rate is often expressed in terms of the
length of time required for the amount to be cut in half, commonly called its
half-lfe.
An important application of this occurs in radioactive decay because the
amount of a radioactive element decreases at a rate proportional to the amount
remaining.

Example 2 Carbon 14, a radioactive isotope of carbon widely used in dating organic
materials, decays naturally with a half-life of 5670 years. Give the decay
function for carbon 14.
Solution We know the function has the form A ( t ) = Coekt, where k remains to be
determined.
Regardless of the amount of carbon 14 present initially [where Co = A ( 0 )
and t is measured in years], we know that

= T1C o
A(5,670) = COek'5G70
Thus
,5670k - 1
2
making

so
k z -0.00012225
Hence
A ( t ) = COe-.00012225t

The practice of carbon 14 dating rests on the assumption that the proportion
of carbon 14 present in any naturally occurring carbon compound is constant
(being produced by the impact of cosmic rays on standard carbon). Thus when
an organism that has been assimilating carbon with this given proportion of
carbon 14 dies and stops assimilating new carbon, the amount of carbon 14 in
the remains decays to lower and lower fractions of the original proportion.
SECTION 8.1: GROWTH AND DECAY OF PHYSICAL QUANTITIES AND MONEY 283

Example 3 Approximately, how old is a bone that contains only one-tenth the amount of
carbon 14 it cohtained originally?
Solution A(t) = COe-0.00012225t

where C, is the original carbon 14 (at t = 0). We know the amount decreased by
a factor of &, and we know k , so we can use (8.1.5) and solve for the time
elapsed directly:

At = ln() 18,835 years 0


-0.00012225

PROGRESS TEST 1

1. Suppose the population of a city is growing expo- forth, doubling each additional day you do not
nentially arid increasing, from 20,000 twenty years smoke.
ago to 50,000 today. ( a ) Write the formula A = 2t-1 (where t - 1 is the
( u ) Give the citys population as a function of time t number of smokeless days) in the form of (8.1.4).
measured in years with t = 0, 20 years ago. (6) How many smokeless days before you become
( b ) When will the population reach 100,000 if it (i) a millionaire, (ii) a billionaire, (iii) a trillionaire?
continues to grow according to this formula? 3. If C, = 5, give two descriptions of the natural decay
2. Suppose you talk a rich uncle into giving you 1 cent function that corresponds to a half-life of 1 unit of
for the first day you do not smoke, 2 cents for the time.
second, 4 cents for the third, 8 for the fourth, and so

GROWTH AND DECAY OF MONEY


We shall now examine the idea of continuously compounded interest to clarify
the meaning of rate in reference to rates of natural growth and to arrive at
the exponential function in an alternative way, a way compatible with the limit
descriptian of e ; see (7.4.12), page 267.
Suppose we have a given amount of money (often referred to as principal)
C, in a bank on which interest is paid, say at a rate i, where i is expressed as a
decimal. Compounded annually, this yields C, +
ZC, after 1 year. If it is
4
compounded twice a year, then after year has elapsed, the interest earned on
C, for the first half-year, (2/2)C0, is added to the principal and the total,
C, + (i/2)C0, earns interest for the second half of the year. The interest for the
+
second half is (2/2)[C0 (z/2)Co],which yields a total principal after one full
year of

(C, + +C0) + +(GO + +C0) = Co[ (1 +) + + ( I


+ + +)I
= C,(1 +$)(1 ++)

= Co(l + $)
2

A similar analysis applies if interest is compounded n times per year (after


each period of length l/n), yielding a total principal after one full year of

If this process is repeated f o p t years, the resulting total principal is


284 CHAPTER 8: MORE ON TRANSCENDENTAL FUNCTIONS

(the number of compoundings is merely increased by a factor o f t ) .


If we let the frequency of compoundings grow without bound (monthly,
weekly, daily, every hour, every second, and so on), compounding the interest
continuously, the total principal at the end of t years is the limit

The limit inside the brackets is almost the same as the limit we arrived at in
(7.4.12). In fact, if we let u = n/i, we get (omitting the constant C),

n++m n++m

= lim (1 + +)ui

++r]
u 4 m

+ 00)
[
= u-+m
lim (I
(z does not vary as u 4

= [eli by (7.4.12)
Hence, the resulting total principal becomes, after t years of continuously
compounded interest,
P ( t ) = C,(ei)t = C,eit
Of course, this is exactly the same as the formula that results if we assume the
money grows naturally; in this case the constant of proportionality is z, the
interest rate. The rate i is the rate at which each dollar is growing at any instant.
It is the derivative of P ( t ) = C,,eit divided by P(t):

However, this rate should be distinguished from the effective annual rate,
which is the amount that each dollar grows over a whole year of compounding.
For example, a t a rate of i = 0.06 compounded continuously, after one year,
one dollar grows to
P(1) = 1 * e(o.06)(1) =: $1.06184
The effective annual rate in this case would likely be advertised as 6.18 percent.
For optimistic investors, our rule of thumb for doubling time says that money
paying at the rate i compounded continuously will double in about 0.69/i
years. For z = 0.06 this is about 11.6 years.

X Inflation can be thought of as a decay of money. Since inflation occurs


continuously, it can be thought of mathematically as natural decay, acting as
a negative intcrest rate. For example, by (8.1.5) we know that the half-life of
money when the inflation rate is 7 percent is:

At =
W/2)
~ 9.9 years
-0.07
(The general halving-time formula is both too obvious and too depressing to
state here.)
As expected, the interest rate and inflation rate have opposite effects on net
return. If C, dollars are invested at an interest rate z during a period when the
inflation rate is d, then after t years these dollars are worth (in todays dollars)
p ( t ) = C,,e(i-d)t
SECTION 8.1: GROWTH AND DECAY OF PHYSICAL QUANTITIES AND MONEY 285

PROGRESS TEST 2

1. What compound interest rate yields an effective 3. What growth rate will cause the population of the
annual return of 100 percent? United States to double in about 60 years? (This is
2. What compound interest rate will yield a n effective approximately the current growth rate.)
annual return of approximately 27 1.83 percent?
-~~~

PRESENT A S D FlTTURE LALIJE OF IKVESTMENTS


Using that the future value (neglecting inflation) of C, dollars invested at a
continuously compounded interest rate i is P ( t ) = COeit,we can determine how
much money C, needs to be so invested now to return P dollars in t years: we
merely solve the previous equation for C, getting

8. I .6
c0 - - pe-it

Economists refer to C, as the present value of P dollars to be paid in t years


discounted at rate i.

Example 4 Determine the present value of a paid-up insurance policy guaranteed to pay a
lump sum of $50,000 to the beneficiary in 20 years, given that money could be
equivalently invested during this period a t 7 percent compounded continu-
ously. (Note that the policy's market value or loan collateral value is based
upon this present value.)
Solution By (8.1.6)
C, = (50,000)e-(0."7)20= (50,000)ep1.4 $12,330 0

Suppose again that money can be invested at the rate i compounded


continuously during a period. However, instead of receiving P dollars in a lump
sum b years from now, we are to get the P dollars in n equal installments of P / n
a t the times to = 0, t,, . . . , t i , . . . , t, = b, where ti - tjp1 = At = b/n. (Hence
n = b/At.) Then the present value of a typicaljth payment at time ti is

Thus the sum of all such payments has present value

" P .
-e p z t , At
j=o
b

which is clearly a Riemann sum based upon the given partition of [0, b]. If we
let the frequency of payments increase without bound, with A t + 0, the income
approaches a continuous constant stream and so its present value approaches
the definite integral [whose value can be determined by (8.2.2), page 2891

ib5 e-it dt

In this case the annual rate of payment is a constant P / b dollars per year, but
a n analogous argument applies when the income stream varies continuously
with time. If the annual rate of income a t time t is given by the continuous
functionf(t), then the present value of this income stream over the next b
years is

4-b
f ( t )ePit dt
286 CHAPTER 8: MORE ON TRANSCENDENTAL FUNCTIONS

Since integration techniques necessary to handle such integrals will not be


available until Chap. 11, we confine ourselves to the constant case.
The market value of income producing property is based upon the present
value of the income stream it can be expected to produce.

Example 5 x (Recall the Apartment-Rental Uiseussion of Sec. 5.4.) The


local college, in need of dorm space and tired of hearing complaints of student
ripoffs, decides to purchase the apartment building. The battle-weary owner
asks $230,000 for the building. Assuming that the monthly income is $2000, the
expected life of the complex is 20 years, and money could earn 8 percent during
this period, is this a fair price?
Solution The annual income rate is (12)(2000) = 24000, so we compute (8.2.2)

The answer is yes because the value of the income exceeds the asking price. 0

The previous analysis of income-producing investments also applies to other


types of investments as well. For example, the true value of a solar hot water
system is the present value of the income stream not spent over the expected
life of the system. A similar remark applies to any other money-saving invest-
ment; see Exercises 28 and 29.

PROGRESS TEST 3

X Congratulations! You have won a $1 million lottery, 2. Determine the actual present value of the after-tax
which entitles you to $50,000 per year for the next 20 income assuming an income tax of 30 percent.
years in approximately a continuous stream.
1. Determine the present value of your ticket given
that the interest rate over this period will be 8 per-
cent.

CONSTRAINEU GHI~W~TH-SOiVIE
DIFFERENTIAL EQUATIONS
Although the word natural is used to describe the growth-decay phenomena
studied so far in this section, exponential growth of a population occurs only
when growth conditions are perfect. This occurs in nature, if at all, for only a
short period of time, after which the growth becomes constrained by such
limiting factors as access to nutrients and space, declining fertility, increasing
toxicity of the environment due to waste products, etc. We shall now examine
two ways such constraints can be incorporated into a more sophisticated and
realistic mathematical description of the growth situation.
Suppose that the growth rate k = k ( t ) is a function of time t (instead of being
+
constant as earlier). The simplest case is k ( t ) linear, say k ( t ) = a bt. In this
case, separating the variables as before, we get the differential equation

@ = (a
A
+ b t ) dt
that leads to the solution.
8.1.7 A = Ceat+bt2/2
SECTION 8.1: GROWTH AND DECAY OF PHYSICAL QUANTITIES AND MONEY 287

>
In this case, if a, b 0 , A is a strictly increasing function whose graph is
concave upward, b u t if a and b are of opposite sign, the graph has a relative
extreme a t t = - a / b (see Exercise 19).
T h e second case w e examine assumes t h a t the population has an upper
A
b o u n d determined by the limits of the environment to support growth. In this
case we assume t h a t the rate of growth is proportional to the product of the current
population A and its dzj5erencejom the maximum supportable population A.T h a t is, we
assume t h a t

8.1.8 dA = kA(A - A )
-
dt
This represents the effects of b o t h forces-the tendency to grow opposed b y the
environmental limitations. Note t h a t as A gets closer to A,d A / d t approaches 0.
We omit the details proving t h a t t h e solution of this differential equation is
given by :

8.1.9 The Logistic Function: A ( t ) = CA/(C + ePkAt)


T h e behavior of this function a n d t h a t i n (8.1.7) will b e explored i n the
exercises. Notice that by allowing either k o r 2, o r both, to vary with time, we
c a n develop functions even more sensitive to the subtle influences o n population
growth.

SECTIOS 8.1 EXERCISES

1. Suppose an exponentially growing population doubles in 10. Carbon 14 dating is only accurate up to about 70,000
3 years. How long will it take to triple? years. Below approximately what percentage of carbon
2. Suppose an exponentially growing population triples in 14 would the age estimate be undependable?
3 years. How long did it take to double? 11. Given $10,000 to invest at 9 percent compounded con-
3. ( u ) Suppose a naturally growing population increases tinuously for 5 years or 10 percent compounded annu-
from 20,000 to 30,000 in 5 years. How long will it take to ally, which situation will yield the larger return?
increase from 30,000 to 45,000? 12. Repeat Exercise 11 for a period of 10 years.
( b ) Compare the function A ( t ) = 20,000ekt, where k 13. (a) Suppose that a $25 U.S. savings bond maturing in
is as determined in part (a), with the function 10 years can be purchased for $18.50. What is the growth
B ( t ) = 20,000(1.5)t. rate (interest) of these bonds?
4. Suppose the population of a city growing exponentially ( b ) What is the growth rate if the price is lowered
at the rate of 8 percent per year is 100,000. When was to $15?
the population 75,000? (c) Neglecting patriotism, which is a more profitable
5. ( u ) If the half-life of radioactive catatonium is 20 years, 10-year investment, the $15 savings bond or a bank
how long will it take for the substance to decay to deposit paying 51 percent compounded continuously?
one-third of its original amount? ( d ) What would your answer be to part (c) if the pre-
( b ) What is the third-life of catatonium? vailing bank interest rates look like they are likely to
(c) Does catatonium have a ninth-life? And if so, drop soon?
what is it? 14. Given that the United States population grows expo-
6. What is the effective annual rate for money that accu- nentially and was 205 million in 1970 and 225 million in
mulates interest at a rate of 12 percent per month com- 1975:
pounded continuously? ( u ) What will the population be in the year 2000?
7. Suppose the population of a city grows by 20 percent (6) When will the population reach 500 million?
every 10 years. How long will it take the population to 15. Exercise 14 led to a growth rate of about 0.0186. Answer
double? the corresponding questions if the growth rate is reduced
8. A piece of organic material is found to have only 1 to 0.015.
percent of its carbon 14 remaining. Approximately how 16. ( u ) Given that the half-life of a certain radioactive sub-
old is it? stance is k years, write a function of the form
9. Repeat Exercise 8 for a piece of material with 2 percent A = C(, 1/2) (where u is a linear function of time t ) that
carbon 14. describes the amount on hand at time t .
288 CHAPTER 8: MORE ON TRANSCENDENTAL FUNCTIONS

( b ) Convert your answer in ( a ) to the natural exponen- proportional to T - T I ; that is, d T / d t = k ( T - TI).
tial form. Show that, as a function of time T ( t ) =
17. Given that the government charges $650 for a 10-year +
(To - Tl)ekt TI, where To = T(O),the initial temper-
$61000savings bond, what is the implied rate of continu- ature.
ous compounding? 24. (See Exercise 23.) Suppose a thermometer reading 80F
P 8. ( a ) Which is more profitable, a n investment that guar- is brought into a freezer set at 10F and its temperature
antees to triple your money in 20 years, or a return of 54 falls t o 45F in 5 min. Determine how long it will take to
percent continuously compounded? cool t o ( a ) 30F, (6) 15F. T h e n (c) determine the ther-
( b ) Given a 5 percent inflation rate through the next 20 mometer's temperature after 1 h i n the freezer.
years, reevaluate your above decision. Explain your 25. (See Exercise 23.) A pot of boiling water (100C) is
conclusion. removed from the stove i n a room at 20C and cools 5 C
1s. Graph (8.1.7) for a = 1, b = -4. in 5 min. With t = 0 being the time it was removed from
20. ( a ) Where does equation (8.1.9) have a point of inflec- the stove, how long will it take to cool down to 30"C?
tion? 26. (See Exercise 23.) ( a ) Repeat Exercise 25 for another pot
( b ) Graph (8.1.9) for C = 1, 2 = 1000, and k = 0.05. that cools to 20C in 20 min.
21. Suppose that the air resistance on a falling body is ( b ) Which pot in Exercises 25 and 26 is better insulated?
proportional to its downward velocity, so the downward O r are they equally well insulated?
acceleration is 32 ft/s2 minus a positive constant k 27. Repeat Example 5 under the assumption that (because of
times u. expected inflation) money will return 10 percent during
( a ) Write a n appropriate differential equation involv- the period.
ing v . 28. X Assuming that money earns 7 percent over a given
( b ) Solve this equation. period of 10 years (compounded continuously), what is
(c) Where does the initial velocity uo appear i n the the present value of a solar hot water system that will
solution? save $40/month over these 10 years? What is the actual
( d ) What happens to v as t grows large? number o f dollars saved by this system in these 10 years?
( e ) Describe the effect of a parachute i n terms of k. 29. Suppose a door-to-door insulation salesman offers to
22. An infectious disease is usually assumed to spread insulate your house for $5000. Assuming money earns 8
through a susceptible population at a rate proportional percent and that a n average of $50/month will be saved
to the product of the number infected times the number over 20 years, is it worth the investment?
not yet infected. (The same assumption has been applied 30. Give the present value version o f the interest-inflation
to the spread of rumors.) formula immediately preceding Progress 2 [see (8.1.6)].
( a ) Give a differential equation that models this as- Then interpret its meaning i n the three cases: ( a ) z >
d,
sumption. (b) i < d, (c) i = d.
( b ) Solve this equation.
23. (Newton's Law of Cooling) The temperature T o f a body
in a medium at temperature TI changes at a rate directly

Integrating Transcendental
Section 8.2: Functions: Basic Formulas

Objectives: LOGARITHRlIC AND EXPOKESTIAL FCNCTIONS


1. Integrate logarithmic, By t h e F u n d a m e n t a l T h e o r e m of t h e Calculus (see Chap. 6) w e k n o w t h a t if F
exponential, and trigonometric is an antiderivative off (so F' =f),t h e n
functions.
2. Integrate combinations of these
u;
functions.
The P o w e r R u l e for derivatives led t o t h e Power R u l e for antiderivatives:

However, in Chap. 7 we d e t e r m i n e d derivative f o r m u l a s for a wide class of


functions. E a c h derivative f o r m u l a likewise gives rise t o an antidifferentiation
f o r m u l a . F o r e x a m p l e , if u ( x ) is a differentiable function of x , t h e n
SECTION 8.2: INTEGRATING TRANSCENDENTAL FUNCTIONS: BASIC FORMULAS 289

It follows that

J- dx = lnlu(x)/ + C

Since l / u ( x ) = u ( x ) - l , we have extended the Power Rule to cover the case


r = -1:

for r # -1
8.2.1
for r = -1
In (8.2.1) and frequently below, to keep the formulas brief we write u(x) simply
as u, and (using the definition of differential) we write u'(x) dx as du.

Solution Here we recognize that the numerator is almost the differential of the denomi-
nator. That is, letting u = x3 + 1, we know du = 3x2 dx, so x 2 dr = (1/3) du.
Hence
x 2 dx (1/3)du 1
U
1
= -1nlul
3
+C

We can also write this antiderivative as In1I-"$ + C. 0

Henceforth in this section we assume that u = u(x) is a d$erentiable&nction of x .


The derivative formula,
D, [eU(Z)] = eU(2)
U(X)

implies the antiderivative formula

8.2.2 Jeu(x)u'(x) dx = eu(') +C

Example 2 Determine 41
ez2+2x(x + 1) dx.
1
Solution Let u = x2 + 2x, so du = (2x + 2) dx. Thus ( x + 1) dx = -
2
du and

1 1
Jex2+2x(x + 1) dx = Jeu- 2 du = -
2 J e u du
290 CHAPTER 8: MORE ON TRANSCENDENTAL FUNCTIONS

Hence, by the Fundamental Theorem,

In (7.2.3) we concluded that


D,[sin u(x)] = [cos u(x)] u'(x)
from which it follows that J cos u du = sin u + C. Similarly, each of the other
derivative formulas developed in Chap. 7 yields an antiderivative formula.
These are now summarized.

8-2.3 ( a ) Jcosudu = sinu +C (6) Jsinudu = -cosu +C


(6) J sec2 u du = tan u +C (d) J CSC' u du = -cot u +C
(e) J sec u tan u du = sec u +C (f)J c s c u c o t u d u = -CSCU +C
(g) JbUdu = -bu1
In b
+C

Example 3 Evaluate s- sec2 v x


fi
dx.

Solution Since we have a formula involving sec2 u [(8.2.3(c)], we try the substitution
u = 6 = x1I2. Then du = (1 /2)xP1l2 dx = (dx/2) so dx/ 6, 6
= 2 du. Hence

= 2 s s e c 2 u du
= 2 tan u C + from [8.2.3(c)]
= 2 t a n f i + ~

Example 4 Determine s dx.

Solution Let u = In x , so du = dx/x. Then

s?? dx = s s i n u du

= -cosu + C = -cos(lnx) + C 0

Example 5 Determine sd m e x dx.

Solution Note that V-' = (e+ - 1)i'2 and that for u = e x - 1, we have du = e x dx.
Hence
SECTION 8.2: INTEGRATING TRANSCENDENTAL FUNCTIONS: BASIC FORMULAS 291

PROGRESS TkST 1

In Probs. 1 to 4 evaluate the given integral.

1. s cos 3x esin3x dx 2. s eZx csc eZxcot e Z x dx 3. s ( 1 - sin2x)cosxdx dx

5. Determine the area under one arch of the sine curve


from 0 to 7 .

MORE ANTIDERIVATIVE FORMCLAS


Among the six basic trigonometric functions, antiderivative formulas for only
the sine and cosine appear in (8.2.3). We shall determine the integrals of the
four remaining trigonometric functions using (8.2.1).

8.2.4 (u) J tan u du = lnlsec uI + C


(6) J cot u du = lnlsin UI + C
(c) J sec u du = lnlsec u + tan u / + C
(d) J csc u du = lnlcsc u - cot UI + C
Prooj We shall derive (a) and (6). Formulas (6) a n d (d) are obtained in a similar
way and are left as exercises.
( u ) J tan x dx = (sin x/cos x) dx. Let v = cos x, then dv = -sin x dx, so - dv =
sin x dx. Hence J tan x dx = -1dv/v = -1nlvl +
C = -1njcos XI C= +
Inl(cos +C = In1 l/(cos x)i +
C = Inlsec xi C. +
The formula follofvs from the Chain Rule for Integrals.
(c) We shall evaluate J sec x dx by first rewriting the integrand as a fraction
whose numerator is the derivative of the denominator:

secx =
secx(secx +
tanx) - secx t a n x
-
sec2x +
secx +
tanx secx tanx +
Now
s s e c x dx = s (sec x tan x
sec x
+ sec2x) dx
+ tan x = J$
where v = secx + tanx. Thus
Jsecx dx = lnlvl + C = lnlsecx + tanxl + C
Again the general formula follows by the Chain Rule. 0

Example 6 Determine sm. x dx

Solution Since 1/cos(x2) = sec(x2), we know

s* = s (sec x2)x dx

= s s e c u T1d u (u = x2,du = 2xdx)

= -1nlsec
1
2
u + tan uI + C
1
= -1nlsec
2
x2 + tan x2/ + c = IndJlsecx2 + tan x21 + c
292 CHAPTER 8: MORE ON TRANSCENDENTAL FUNCTIONS

PROGRESS TEST 3

Evaluate each integral in Probs. 1 to 3.

1. J'2"csc 2" dx 3. S e x tan e x dx

2. J' tan3 x dx (use 1 + tan2 x = sec2 x) 4. Prove [8.2.4(6)].

SECTIOS 8.2 EXERCISES

I n Exercises 1 to 36 evaluate the given integral.


du
4. J1 ~

2/5 5x -

, ,s
5.
xdx
, ,s
6-
x5 dx
8- s-&
12. ?x
J ~ 'esec sec x tan x dx
0

16. s e e cos e4' dx

17. s0
v/4 &an x
~

cos2x
dx
18. seesex
sin2 x
cos x
dx 20.
n

J ex tan e x dx

21. s1
-2
eX+ldx 22. J 7 +sec2 2x
3tan2x
dx
24. J sin x eSec
cos2 x
dx

25- 1,- ex
dx 26. s c o s 2x tan(sin 2x) dx 28.

29. s + 66+
csc( l)cot( 1) dx 31. s- dx
ex

32. s * COSZX
dx

[In Exercises 33 to 36 divide before integrating.)


4 - 3x
33. 35.
36- s8--5xdu
37. Determine the area under the tangent curve from 44. Evaluate Jsec2x t a n x d x in two ways: ( a ) by letting
x = 77/6 to x = 77/3. u = secx and (b) letting u = t a n x . (c) Explain the
38. Determine the area under the curvey = l/(x + 3) from difference in the appearance of your answers. ( d ) As in
a=-2tob=6. Exercise 43(d), evaluate sec2 x tan x dx in two ways.
39. Determine the volume of the solid generated by revolv- 45. Prove [8.2.4(d )].
ing the region under the curve y = e'* from x = 0 to 46. Compute 1eln ' dx.
x = 1 about t h e y axis. 47. Compute J ln(ex)dx.
40. Determine the volume of the solid generated by revolv- 48. Compute D,[J; eln dt].
ing the region bounded by the lines x = - 1, x = 1,
y = ex, and the x axis about the x axis.
41. Determine the average value off(%) = e x on the interval
[O,11.
42. Determine the average value off(x) = (In X)/Xon [ l , el.
43. Evaluate J sinx cosxdx in two ways: ( a ) by letting
u = sin x and (b) by letting u = cos x . (c) Explain the
difference in the appearance of your answers. ( d ) Eval-
uate Jl sin x cos x dx by using the two antiderivatives
obtained in ( a ) and (b) and applying the Chain Rule for
Integrals in each case.
SECTION 8.3: INTEGRATING POWERS, PRODUCTS OF TRIGONOMETRIC FUNCTIONS 293

Integrating Powers and


Section 8.3: Products of Trigonometric Functions

Objectives: SINES AND COSINES


Evaluate integrals of the following: We shall make frequent use of the following identities.
1. Certain powers of sines and 8.3.1 ( a ) sin2 t + cos2 t = 1 ( 6 ) sin 2 t -
-
1 - cos2t
cosines 2
(c) cos2 t -
-
1 + cos2t
2. Products of certain powers of
2
sines and cosines
Our use will generally be one of two types. We shall use either ( a ) in dealing
3. Certain powers of the remaining with odd powers of sine and cosine or the double-angle formulas ( b ) and (c) in
trigonometric functions dealing with even powers.
4. Certain products of powers of
tangents and secants or cotangents 8.3.2 Procedure for Evaluating Integrals of the Form
and cosecants fsinn u cosm u du
Case I: If either m or n is a positive odd integer, use the identity
sin2 u +cos2 u = 1 in order to apply the Power Rule to a power of either sines
or cosines. (See Example 1.)
Case 11: If both m and n are nonnegative even integers, use double-angle
formulas, repeatedly if necessary, to lower the powers of sine a n d cosine.
The simplest situation has either m or n equal to zero.

Example 1 Evaluate s sin3 x dx.


Solution Here n is odd and m = 0, so the Case I procedure applies. Rewrite the integral
as indicated:
Ssin3xdx = Ssin2xsinxdx

= s (1 - cos2x) sin x dx (using [8.3.l(a)])


= s s i n x d x - Scos2xsinxdx
- -cos x - s u z ( -du) (u = cos x, du = -sin x dx)
-
- -cosx +-
u3
3
+ c = -cosx + -
3
x
S C
COS~

Example 2 Evaluate s cos4x dx.


Solution Here n = 0 and m is even, so the Case I1 procedure applies. Rewrite the integral
as indicated:
Sc0s4x dx = 1(cos2x)* dx
= s( ys2x)2 dx + (using [8.3.l(c)])

=-
4
1
s (1 + 2 cos 2x + cos2 2x) dx

+ ~ ( c o s 2 ~ j 2 d+x + c0s4x
= L4[ s d x 2 s 4 (using [8.3.1(c)])

-
--
sin 2x + -
x +- x + iscos4xdx
4 4 8
3x sin2x
=-+-+- sin4x
8 4 32
+c
294 CHAPTER 8: MORE ON TRANSCENDENTAL FUNCTIONS

In the last step we did not explicitly write out the substitution u = 4x, du = 4 dx.
More generally, using the substitution u = kx, du/k = dx, we see that

Scos kx dx = Scos u duk = k1 sin u + C = k1 sin kx + C


- - -

Similarly,
-1
S s i n kx dx = -cos kx +C
k
In using procedure (8.3.2), where neither n nor m is zero, the computations
are very similar to those in Examples 1 and 2.

Example 3 Evaluate S sin4x cos2x dx.


Solution Here Case I1 applies.

J
f sin4 x cos2 x dx = " r [sin2.I2 - cos2x dx
+ cos 2x
=S[
1 - cos 2x
2 I[ 1
2 1 dx

= -1S ( l - 2 cos 2x + cos2 2x)(1 + cos 2x) dx


8
=
1
-S
8
(1 - cos 2x - cos2 2x + c0s3 2x) dx

=' S d x -
8
~ S C2xO S ' S dx - -
8
cos2 2xdx +-
8' S
cos3 2x dx

1 1
(1)
Now
=-x--sin2x--
8 16 'S 8
cos22xdx+-
8 'S cos32xdx

S cos2 2x dx S
1
= - [ l + cos 4x1 du
2
1 1
= - x +-sin4x
2 8
and
S C O S ~2x dx = S (1 - sin2 2x) cos 2x dx
= - 1s ( 1 -u2)du ( ~ = s i n 2 x , -du
=cos2xdx)
2 2
=1
2
(u - $) = -sin
1
2
2x - -
1 sin3 2x
6
Substituting these results into (1) and combining like terms, we obtain
1 1 1
S s i n 4 2x cos2 2x dx = -x
16
- -sin
64
4x - -sin3
48
2x +C 0

It is important to realize that Case I applies even when one of m or n is not a


positive integer, as in Prob. 3, Progress Test 1.

PROGRESS TEST 1

Evaluate each of the following indefinite integrals:


1. Jsin4xdx 2. Jcos5xdY 3. J d=c0s3 3x dx 4. J sin2 2 cos2 -
X dx
2 2
SECTION 8.3: INTEGRATING POWERS, PRODUCTS OF TRIGONOMETRIC FUNCTIONS 295

TANGENTS A S D SECAXTS
We shall now consider integrals involving the remaining four trigonometric
functions. We shall give procedures for tangents and secants, but a n analogous
analysis applies for cotangents and cosecants. As is explained in the remarks at
the end of this section, our procedures are by no means all-inclusive. We shall
use the following identities.

8-33 (u) 1 + tan2 t = sec2 t (b) 1 + cot2 t = C S C 2 t

Example 4 Evaluate 1tan2 u du.


I
Solution Use [8.3.3(u)] to rewrite the integral as follows:
J tan2 u du = J (sec2 u
- 1) du
=tanu-u+C 0

We can extend the idea of Example 4 to handle integrals of the form

s tanm u du where m > 2 is positive integer


s tanm u du = s t a n m - 2u(sec2 u - 1) du

= s tanmp2u sec2 u du - s tanmp2u du

- tanmp1u
m-1
- s tanmp2u du

We have thus lowered the degree by 2. Repeated applications will reduce the
degree to 2 if m is even (so Example 4 applies) or to 1 if m is odd, in which case
[8.2.4(u)]applies: J tan u du = lnlsec ul C. +
We shall now apply the cotangent version of the above procedure.

Example 5 Evaluate 1cot3 u du.


Solution We apply [8.3.3(b)]to rewrite the integral as follows:
1cot3 u du = J cot U(CSC' u - 1) du
= J cot U(CSC' u du) - J cot u du
- cot2 u
- ~- 1n)sinul +C 0
2
The first term of the answer is a result of the Power Rule with v = cot u,
-dv = csc2 u du;and the second term is a result of applying [8.2.4(b)].Note that
-1nlsin UI
= lnlsin u1-l = lnlcsc u l , so the answer could also be presented as
co t2 u
lnlcscuJ - -+ C
2
We could even rewrite (cot2 u ) / 2 as
csc2u - 1 - csc2 u 1
2 2 2
and absorb the -1/2 into the constant C, leaving our answer in the form
csc2 u
ln/cscul - -+ C
2
296 CHAPTER 8: MORE ON TRANSCENDENTAL FUNCTIONS

Manipulations such as these are common in applications, so you should be


on the alert for various forms of a n answer.
We now move on to more general situations.
8.3.4 Procedure for Evaluating Integrals of the Form
Jsecnu tanm u du
Case I: If n is a positive even integer, factor out sec2 u du and replace the
remaining secant factors by tangents using the identity 1 + tan2 u = sec2 u.
Case 11: If m is a positive odd integer, factor out sec u tan u du and replace
the remaining tangent factors by secants using the identity tan2 u = sec2 u - 1.
To apply the procedure of Case I, it is not necessary that m be a n integer.

Example 6 Evaluate 1=sec4 x dx.


Solution Here n = 4, so Case I applies.
Jw sec4 x dx = 1 x sec2 x sec2x dx

=1 x [ 1 + tan2 x]sec2x dx
= 1u1I3[l + u 2 ] du (u = tan x, du = sec2 x dr)
= 1( u / ~ + u7I3) du
3u4/3 3u10/3
--
-
4
+- 10
+ C

Again, to apply Case 11, n need not be a positive integer.

Example 7 Evaluate Jrtan5 x dr.


sec x
Solution Here m = 5, so Case I1 applies.

JS- - sec-lI2 x tan5 x dx

=1 x tan4 x sec x tan x ak


= 1s e ~ - x~[tan2 / ~ xI2 sec x tan x dx
= 1s e c ~x[sec2~ / ~x - 112sec x tan x dx
= 1u-3/21u2 - 1l2 du (u = sec x, du = sec x tan x dx)
= J ( u 5 / 2 - 2 u 1 / 2 + u - 3 / 2 ) du
zu7/2 4u3/2
- 2u-112 + c
7 3

PROGRESS TEST 2

Evaluate the following integrals:


1. Jtan7xdx 2. J sec2I3x tan3 x dx 3. 1csc4x dx
SECTION 8.3: INTEGRATING POWERS, PRODUCTS OF TRIGONOMETRIC FUNCTIONS 297

A CAUTIOK
In Secs. 8.2 and 8.3 we cataloged some combinations of transcendental functions
that can be handled using substitution and trigonometric identities. At this
point we cannot handle integrals such as S x sin x dx, 1xex dx, or integrals of odd
powers of the secant (or cosecant) function. These all await integration parts in
Chap. 1 1, Sec. 11.1. In Sec. 1 1.5 we shall show how tables of integrals can also
be used to evaluate integrals of trigonometric functions.
1
Other integrals, resembling those dealt with previously-for example, e2' du
and S cos2x dx-can be determined only by using infinite series and
numerical techniques (Chap. 16).

SECTION 8.3 EXERCISES

I n Exercises 1 to 30 evaluate the given integral.


1. ssin3xdx 2. ~ C O 2xdx
S ~

3. !sin2 x cos4 x dx 4. s ( s e c t tan t)'dt

5. S c o t 7 8 d8 6. s dsin(3x + 1) cos(3x + 1) dx
7. s t a n 2 ( 2 x - 5) dx 8. lT'3 sec4 2x dx

9. i T 1 4tan5 x dx 10. iT" C O S ~3x sin 3x dx

11. ~ ( c o s x s i n ~ ) ~ d x 12. S *sin3 x dx

13. s cot5 x csc2 x dx 14. s s i n 2 x cos3 x dx

15. s cot5 x csc4 x dx 16. s sin4 x cos4 x dx

sin3 u
17. J m d u 18. ~ c o t 2 x c s c 4 x d x

19. s tan3 8 sec3 8 dB 20. s c o s z i 3 t sin3 t dt

21. sezsin2e"dx
22* s dx

23. stan3x G d x 24. [;:2 csc2 x cot4 x dx

JO
26. iv'6 cos4 x dx

27. sT/'
-T/ 2
sin3 lOOx dx

29. s T I 2cot3 x dx
T/ 4

3 1. Determine the volume of the solid generated by revolv- 33. Determine the average value ofy(x) = sin2 x between 0
ing about the x axis the region bounded by the x axis and and a / 2 .
one arch of the sine curve. 34. Determine the average value of g(x) = t a n x on
32. Repeat Exercise 31 for one arch of the cosine curve. [-44,441.
Would you expect your answer to be different from that
of Exercise 31?
298 CHAPTER 8: MORE ON TRANSCENDENTAL FUNCTIONS

In Exercises 35 to 40 apply the following identities:

sin 1
= -[sin(a + sin(a + b ) ]
a cos b - b) 38. Jsin 3x sin lox dx 39. J cos 7x cos 5x dx
2 40. J cos 3x cos x cos 5 x dx
sin a
1
sin b = -[cos(a - b ) - cos(a + b ) ] 4 1. Evaluate and compare J sin 2x dx and 2 J sin x cos x dx.
2 42. Evaluate and compare Scos 2x dx and J (cos2x -
cos a cos b = [cos(a - b ) + cos(a + b ) ] sin2x) dx.
2
35. J sin 5x cos 2x dx 36. Jsin 4.x cos 2x dx
43. Evaluate and compare
s tan 2x dx and dx.

37. J (sin 5x + cos 3 ~dx ) (Hint:


~ Expand and then let
a = 5, b = 3 in the first identity.)

Section 8.4: Inverse Trigonometric Functions

Objectives: IXTRODUCTIOS

1. Define and evaluate inverse Recall that when introducing the idea of inverse function we contrasted the
trigonometric functions. functionsf(x) = x 3 a n d g(x) = x2. The first, being strictly increasing on its
domain R, had a n inverse.
2. Differentiate inverse
O n the other hand, a n attempt to define g - l ( x ) for x in the range of g-the set
trigonometric functions.
of nonnegative real numbers-runs into difficulty because there are two possi-
3. Use the skills in (1) and (2) in ble square roots of x: +6 and - 6.
T h a t is, a nonzero x in the range of g
applications and in determining came from two different numbers.
relationships between functions We can avoid this difficulty by trimming our ambitions. Rather than define
involving inverse trigonometric a n inverse for the whole function g(x) = x2, we shall define a n inverse for part of
functions. g, that portion of g defined on the nonnegative real numbers. This restricted
+
function, denoted gp, with domain [0, 0 0 ) is strictly increasing and thus has
an inverse:

+6
g;l(x) = for all x in range of gp
which also happens to be [0, + 00).
Graphically, we formed gp from g by lopping off the left side of the parabola;
see Fig. 8.1(a). T h e graph of g;l is the mirror image of the graph of gp across the
(dashed) diagonal; see Fig. 8.l(b).
We shall follow this same procedure in determining inverses for the trigono-
metric functions, none of which is strictly increasing on its entire domain.

A g&) = xz /

(a)
Fig. 8.1
SECTION 8.4: INVERSE TRIGONOMETRIC FUNCTIONS 299

I
I
I
I
t
I I I I

I I
I I I I
- 3n 3n
~ ~

2 2
Fig. 8.2

*p TltE INVERSE SINE FUNCTION


y = arcsin x
a
At As evident from the graph of the sine function (Fig. 8.2), there are many
i d

ways we could restrict the domain to get a strictly monotonic piece of the sine
# +x function. Following tradition we choose the restriction of sin x to the interval
p
[ -a/2, a/2]. The graph of this restricted sine function is the thickened portion
#* IF in Fig. 8.2. The function
f ( x ) = sin x x in [ -a/2,a/2]
c
B is strictly increasing on its domain [ -a/2, a/2] and has range [ - 1, 11. Hence it
1(-1,-7r/2) has a n inverse whose domain is [ - 1, I] and whose range is [ - a / 2 , ~ / 2 ] . More
Fig. 8.3 precisely, we have:

8.4.1 Definition For each x in [ - 1, 11 we define the inverse sine of x, denoted


arcsin x, to be the unique y in [ -.1/2,a/2] such that sin y = x .

Some authors write the inverse sine of x as sinp1 x, but we prefer arcsin x. It
is easier to say and, more important, it avoids the possible confusion with
l/(sin x). The graph ofy = arcsin x, the reflection across the diagonaly = x of
our restricted sine function, appears in Fig. 8.3.

Example I Compute ( a ) to (e):


(a) arcsin ( I / fi) (6) arcsin( - I / &) (c) arcsin(sin a/6),
( d ) arcsin(sin 5a/6) (e) sin(arcsin 1/2)

Solution (a) arcsin (1/fi) is that number y with - a / 2 < y 5 a / 2 such that
siny = l/&. Since sin a/4 = l / f i and - a / 2 5 a/4 <
a / 2 , we know
that arcsin ( l / f i ) = a/4.
( b ) arcsin( - 1/ &) is thaty with -a/2 5 y 5 a/2 such that siny = - 1/ &.
But sin(-a/4) = - l / & and -a/2 5 -a/4 5 a/2, so
arcsin( - 1/ &) = -a/4.
(c) arcsin(sina/6) is t h a t y in [-a/2, m/2] such that s h y = sin a/6. Thus
y = a/6. (Who, we ask again, is buried in Grants Tomb?)
( d ) arcsin(sin 5n/6) is thaty in [ -a/2,7~/2] such that siny = sin 5m/6. Now
5 ~ / is6 not in [ -a/2,77/2]. However, sin 5a/6 = 1/2 = sin ~ / 6 and
, a/6 is
in [ -a/2, a/2], so arcsin(sin 5m/6) = arcsin( 1/2) = a/6.
( e ) Now arcsin 1/2 is that y in [ - a / 2 , m / 2 ] such that siny = 1/2, so
arcsin 1/2 = a/6. Thus sin(arcsin 1/2) = sin(a/6) = 1/2. 0
300 CHAPTER 8 : MORE ON TRANSCENDENTAL FUNCTIONS

Recall that, in general, we have the inverse relationshipsf-'(f(x)) = x for x


in the domain offandf(f-l(x)) = x for x in the domain off-'. Parts (c) and
( e ) , Example 1, are special cases, respectively, o f

8.4.2 x in [ -m/2, .rr/2], arcsin(sin x) = x .


( u ) For
(6) For x in [-1, 11, sin(arcsinx) = x.

Part ( d ) illustrates that if x is not in [ -a/2, .rr/2], then there i s a unique x1


that is in [ -.1/2,~/2] such that
arcsin(sin x ) = arcsin(sin xl) = x1

PROGRESS TEST 1

Compute the indicated quantity.


1. arcsin 1/2 2. arcsin( - 1/2) 3. arcsin(sin ~ / 8 ) 4. sin(arcsin 0.012)
5. arcsin(sin 2m/3)

DEFINITIONS O F THE OTHER INVERSE


TRIGONOMETRIC FUNCTIONS
We shall follow the same pattern and define inverses of the remaining trigono-
metric functions by first appropriately restricting their domains.
We restrict the cosine function to [O,.rr], the heavy part of the graph in
Fig. 8.4, where it is strictly decreasing and hence has an inverse.

8.4.3 Uefinition For each x in [ - 1, 11 we define the inverse cosine of x, denoted by


urccos x, to be the uniquey in [O,.rr] such that cosy = x . (See Fig. 8.5.)

To define the arctangent function, we first restrict the tangent function as


indicated by the heavy part of the graph in Fig. 8.6. Note that the domain of
this strictly increasing restricted tangent function is (- .rr/2,~/2),and its range
is R.

8.4.4 Uefinition For each real number x we define the inverse tangent of x, denoted
by arctan x, to be the uniquey in (-~/2,.rr/2) such that tany = x. (See Fig. 8.7.)

Because its domain is all of R, the arctangent is perhaps the most widely used
of the inverse trigonometric functions. As is evident from Fig. 8.7,

Fig. 8.4 Fig. 8.5


SECTION 8.4: INVERSE TRIGONOMETRIC FUNCTIONS 301

I i
I /
I

Fig. 8.6 Fig. 8.7

-3n -n -n I n n 3i7 211 511

Fig. 8.8

8.4.5 ( a ) lim arctanx = m/2 ( b ) lim arctan x = - ~ / 2


x-+x iC+--Co

The previous definitions of inverse trigonometric functions are based upon


rather standard restrictions. No such consensus exists on how to restrict the
secant function to obtain a strictly increasing or strictly decreasing function. We
choose the restriction to [O,m/2) and (n/2,77],as in Fig. 8.8. The range of this
restricted secant function consists of the disjoint intervals ( - co,- 11 and
[ I , +m).

8.4.6 +
Definition For each x in ( - 00, - 11 or [ l , co),we define the inverse secant of
x, denoted by arcsec x, to be the uniquey in [O,T] such that secy = x. (See
Fig. 8.9.)
Note thaty will never equal 7 ~ / in
2 (8.4.6) because sec m/2 is undefined. From
Fig. 8.9 it is apparent that:

8.4.7 ( a ) lim arcsecx = n/2 ( b ) lim arcsecx = m/2


x++m X+--P

The inverses of the cotangent and cosecant functions are seldom used in
practice and will be defined later in this section in terms of those inverses
already defined.

PROGRESS TEST 2

Compute those of the following that are defined.


I . arctan(- ~ ' 3 ) 2. arcsec( - 2) 3. arcsec[sec ( - ~ / 4 ) ] 4. arctan[tan (19n/6)]
5. arcsec(l/2)
302 CHAPTER 8: MORE ON TRANSCENDEWTAL FUNCTIONS

DERIVATIVES OF INVFQSE TRIGONOMETRIC FUNCTIONS


We shall compute formulas for derivatives of inverse trigonometric functions
similarly to the way we computed D,[In XI-namely, by using implicit differ-
entiation.

8.4.8 Suppose u is a differentiable function of x. Then


(a) D,(arcsinu) = 1 du
1 - u2 dx
1 du
( b ) DJarccosu) = -
di-=-pdr -> IUI <1
1 du
D,(arctanu) = ~-
(6)
1 u2 dx+
1 du
(d) DJarcsec u ) =
/uI d
->
m dx
1ul >1
pro^ As usual, we shall prove the formulas for u ( x ) = x, from which the stated
formulas follow by the Chaip Rule.
( a ) Lety < <
= arcsin x , - 1 x 1. Then by definition siny = x. Moreover, by
the Theorem on Inverse Functions (7.3.3) dy/& exists, so we can differenti-
ate implicitly with respect to x , getting

cosy-d y = 1 and 4- 1
---
dr dr cosy
Since sin2y + cos2y = 1, we know that cosy = t d-. But fory in
[-77/2,1~/2], cosy 2 0, so cosy = + v'l - sin2y. Thus

- 1

- di--=7 since x = siny

( b ) See Exercise 59 below,


(c) Lety = arctan x, so x = tany and

1 = seczy- 4 or 4-
- --1
dr a!i sec2y
Since sec2y = 1 + tan2y = 1 + x2, we know that
4 --
_ 1
dr 1+x2
(d) Let y = arcsec x, where we know 0 <y < IT. Now secy = x , so

dy 1
(1) secy tany- dy = 1 or --
dr dr secy tany
But sec y = x and tan y = k d n = ? d m , so

What is the sign of dy/ak? Well, for 0 < y <


7/2, secy and tany are both
positive, and for 71/2 < y <
IT, secy and tany are both negative. Thus the
SECTION 8.4: INVERSE TRIGONOMETRIC FUNCTIONS 303

product secy tany is always positive, and therefore dy/dx = 1/(secy tany) in (1)
must be positive for all values of y. We can guarantee this by writing
1
_
dy --
dr IxldiTx
[Notice that simply choosing the + sign in ( 2 ) would not suffice.]
Notice that the derivatives ofy = arcsin x,y = arctan x, andy = arcsec x are
all nonnegative on their respective domains, a fact that agrees with their
respective graphs: Each function is increasing on its natural domain. O n the
other hand, y = arccos x, which is decreasing, has a negative derivative,
The derivatives of the remaining inverse trigonometric functions are seldom
needed in practice. This will be clearer after we establish a simple relation
between arcsin x and arccos x. Since
1
D,(arccos x) = - = -D, (arcsin x )
d
-
we know that
D,(arccosx + arcsinx) = D,(arccosx) + D,(arcsinx) =0
Hence arccos x + arcsin x = C, a constant [by (4.4.6)]. For .r = 0, arccos 0 +
arcsin 0 = 77/2 + 0 = C, so C = 7 / 2 . Thus we can conclude:
8.4.9 Theorem arccos x = 7 / 2 - arcsin x

Following (8.4.9), we define the inverse cotangent and cosecant:

8.4.10 Definition ( u ) arccot x = 77/2 - arctan x


( b ) arccscx = 7 / 2 - arcsecx (1x1 2 1)

Example 2 Differentiatef(x) = arcsin (2/x2).

Solution

-
-
1 -4
drqq7q.T-
- 1 .--4
d(x4 - 4)/x4 x3

Example 3 Differentiatef(x) = arcsec (In x )


304 CHAPTER 8: MORE ON TRANSCENDENTAL FUNCTIONS

Example 4 A balloon is released (from eye level) 300 m from an observer and rises a t the
rate of 240 m/min. Find the rate (in radians per minute) at which the angle of
elevation of the observer's line of sight is increasing when the balloon is 400 m
high.
Solution Let 0 be the angle of elevation at time t and x the height of the balloon at time
t. Then (see sketch)

0 = arctan ($)
dB -
- 1 A(L)
dt 1 + ( ~ / 3 0 0 )dt~ 300
- 90,000 _1_du _ \ /
+
90,000 x 2 300 dt 300

Letting x = 400 and &/dt = 240, we have


dfl
--
dt
90,000
90,000+ 160,000
.-.300
1
240

72,000
-
- = 0.288 rad/min 0
250,000

PROGRESS TEST 3

Find a j / d u :
1. y = arcsin(2x2 - x + 3) 2. y = arcsec(d7T-i)

SECTION 8.4 EXERCISES

In Exercises 1 to 20 evaluate the given expression.


1. arctan( - I / fi) 2. arcsec( - 2/ fi)
3. arcsin( f i / 2 ) 4. tan[arcsec( - 3)]
5. cot [arctan(2/3)] 6. arccos 1/2
7. arccot 1 8. arccot(tan 377/4)
9. arccsc(2) 10. arcsin(csc 77/6)
1 I . csc[arccsc (- 2)] 12. arcsec( - 2/ fi)
13. cot[arctan ( - l)] . 14. sin[arccos (fi/2)1
15. arctan(cot 77/3) 16. arctan[tan ( 9 ~ / 8 ) ]
17. arcsec(sec 2 ~ / 3 ) 18. arcsinlsin (29~/4)1
1 9 . sin[ lim arcsec (%)I 20. csc[ liA arctan (xj3)1
X-.+o, I-.--?]

In Exercises 21 to 40 differentiate the given expression.


21. y = arcsin 6 22. y = arctan fi
23. y = arcsec d n 24. f ( x ) = x z arcsin(2x)
25. f ( x ) = arctan(3/x) 26. f(x) = x arctan(x2)
27. f ( x ) = arcsec(l/x) 28. f(x) =arcsin(2x/dm)
29. g ( t ) = t arccos(t2 + 1) 30. g ( x ) = darctan ~7
3 1.y = arccsc ( x 2 - 1) 32. f ( u ) = arccot d m
33. h(u) = arctan(1nu) 34. ~ ( x= ) arcsin(x/dm)
35. f ( x ) = e' arctan(e") 36. f ( x ) = arcsec[(4x - 5)/7]
SECTION 8.5: INTEGRALS INVOLVING INVERSE TRIGONOMETRIC FUNCTIONS 305

arctan(3x)
37. f ( x ) = 1 + gx2
38. f ( x ) = x arcsin(1 - x)
h ( w ) = earctan[ln(i2-l)l
39. g ( v ) = lnlarcsin e" 1 40.
41. Determine and simplify D,[arcsin(sin x)]. Explain your

42.
result.
Determine and simplify D,[sin(arcsin x)]. Explain your
52.

53.
Compute and simplify D, arcsin-

Sketchffx) = arctan(x +
[ :;I,
1) - arctan(x - 1).
result. 54. Sketchf(x) = arcsin 2 6.
43. Determine and simplify D,[arcsin(cos x)]. Compare with 55. An isosceles triangle has a base of 4 m. Suppose the
the result of Exercise 41 and explain. height is increasing a t the rate of 1/2 m/s. How fast is
44. Repeat Exercise 43 for D,[cos(arcsin x)] and compare the vertex angle changing when the height is 2 m?
with Exercise 42. 56. What is the domain of the restricted cotangent function
45. Prove arccos l / x = arsecx for 1x1 2 1 by showing that that yields the same inverse cotangent as defined in
they differ by a constant C (show their derivatives are [8.4.1O( u ) I?
equal), and then show that C = 0. 57. Repeat Exercise 56 for the inverse cosecant function
46. Prove arcsin l / x = arccsc x for 1x1 2 1. (See Exercise 45.) given i n [8.4.10(b)].
47. Prove arctan x + arctan l / x = ~ / (x 2 > 0). 58. A 10-m tall billboard is to be on a n 11-m tall scaffolding
48. For x in [0, 13 rewrite cos(arcsinx) as a n algebraic ex- near a highway. At what distance from the highway
pression in x by first letting y = arcsin x and showing should the sign be placed so that the angle between the
that cosy = +d n . lines of sight of the bottom and top of the billboard is a
49. For 1x1 2 1 rewrite tan(arcsecx) as an algebraic expres- maximum for people riding by whose eyes are approxi-
sion in x. (See Exercise 48.) mately 1 m above ground level?
59. Prove [8.4.8(b)] directly.
50. Differentiate and simplify arctan ~

(1 ;:1*nx)

51. (x # -1).

( b ) Conclude from (a) that arctan- x - 1 - -


x + l
arctanx - 7i/4 on (-1, + m ) .

Integrals Involving
Section 8.5: Inverse Trigonometric Functions

Objeetive: THE BASIC FOR,MULAS


Compute definite and indefinite The differentiation formulas for t h e inverse sine, t a n g e n t , and secant functions
integrals leading to antiderivatives give rise t o t h e following antidifferentiation formulas:
involving the inverse sine, tangent,
and secant functions.
8.5.1 (a) J du = arcsin u +c (b) JA= a r c t a n u + c
diT7 1 +u2

Example 1 Evaluate J d7.dx


4-x

Solution

L e t u = x/2, t h e n du = ( 1 / 2 ) dx and dx = 2 du, so

2du - du
= arcsin u +c
= arcsin(x/2) +C 0
306 CHAPTER 8: MORE ON TRANSCENDENTAL FUNCTIONS

Using the substitution u = x / a and a n argument similar to that in Exam-


ple 1, we can generalize the formulas in (8.5.1):

8.5.2 Let a > 0. Then


du
(4 J
= arcsin
,n (:> + c -

Example 2 Determine the area of the region bounded by the graph ofy = 1/ and ~'m
the straight lines x = 0, x = 1/2 a n d y = 0; see accompanying sketch.

0 1
Y

j = -
= arcsinx

R
-
- _77 _ o = - 77 0
6 6
x = o x = 112 x = l

Often it is necessary to complete the square to apply the formulas of (8.5.2.)


because each formula involves a sum or difference of squares.

Example 3 Evaluate J dx
X 2 + X + l '

Solution We first complete the square in the denominator, writing

Now
x2+x+1=
- (x 2 + x + - '1+ ( 9
4
1--
( :)2 + (&)2
= x+-

dx -
- dx
Jx2+x+l J (x + 1 / 2 y + ( m ) z

Let u = x + 1 / 2 ; then du = dx, and a = a= G / 2 , so


du arctan(L) +c
vm
+c 0

PROGRESS TEST 1

Evaluate:

1.
dx
2- s, dx
1 - 2x - x2
SECTION 8.5: INTEGRALS INVOLVING INVERSE TRIGONOMETRIC FUNCTIONS 307

dx
Example 4 Evaluate ~~

5 - 7x
Solution We can regard ~ d(
m
as &)2 - ( and then apply [8.5.2(a)] with
a = fi,u = f i x . Then du = f i d x , so dx = du/ fl and

You need to be especially careful in choosing integration formulas, because


similar appearing integrals can have vastly different solutions. For example,

which differs from the integral in Example 4 only by the factor of x in the
numerator, is handled using the Power Rule. (Let u = 5 - 7x2, which leads to
du = - 14x dx, and thus ( - 1/14) Ju- du14.)
Occasionally an integral may require manipulation in addition to substitu-
tions.

Example 5 Evaluate s dx
+ 4(ln x ) ~ ]
x[16

Solution The appearance of 16 + 4(ln x ) in


~ the denominator is a clue that the arctan-
gent formula may be appropriate. Let u = lnx, so du = dx/x. Then

.f x[ 16 + 4(ln x)*] = s 16 + 4u2


dx du

du

-[ L a r c t a n -u1
=4 2 2
+c (a = 2)

= -arctan
8
- +C 0

The arctangent function sometimes occurs when integrating quotients of


polynomials. Whenever the degree of the numerator exceeds that of the de-
nominator, always divide.

Example 6 Evaluate s x4 + 3 x 2 + 8x + 9 dx
x2 + 4

Solution Following our own advice, we use long division to get


308 CHAPTER 8: MORE ON TRANSCENDENTAL FUNCTIONS

PROGRESS TEST 2

Evaluate:
eZx dx
J d P

SECTIOY 8.5 EXERCISES

In Exercises I to 30, evaluate the given integral,

1.
s 1x31
x2 dx
~'m

7. s "+'
Xd-
dx x>O

dx
9.

3x dx
dx 14.

17. s cos x dx
sin2x + 3 sinx + 15 18. s1 +
-1 x2
dx
2x +5
20.
dx x< -1
ex + e-"
22. Jx3 - 4x2 + 3x + 1 dx
x* +4

25- J di(1+ x)
dx
26. JV'l
0
y7e:z dt

dx dx
27. 28.
ex V
-'

29. s csc x cot x dx


1 + csc2x 30- s(m) arcsinx 112
dx

In Exercises 31 to 33 prove the gzven formula by dtferentiating.


31. [8.5.2(a)] 32. [8.5.2(b)] 33. [8.5.2(c)]
34. Determine the area of the region enclosed by the x andy 37. Determine the volume of the solid generated by re-
axes, the curvey = 1/(1 +
4x2), and x = 2. volving about the x axis the region enclosed by y =
35. Determine the area of the region enclosed by the x andy 1/( qn), x = 0, x = 1, and the x axis.
axes, the curvey = 1/( d m ) ,and the line x = 3.
36. Determine the volume of the solid generated by re-
volving about the y axis the region enclosed by y =
l / ( x 4 d n ) , x = 1/4, x = 3/4, and the x axis.
SECTION 8.6: HYPERBOLIC FUNCTIONS AND INVERSE HYPERBOLIC FUNCTIONS 309

Hyperbolic Functions
Section 8.6: and Inverse Hyperbolic Functions

Objectives: INTRODL-CTIOS AND DEFINITIOSS


1. Define and differentiate the Certain combinations of exponential functions occur frequently enough, par-
hyperbolic functions. ticularly in applications, to get names of their own. It turns out that the
relationships among these functions and among their derivatives parallel
2. Extract the hyperbolic functions
relationships among the trigonometric functions. Their names reveal this
from combinations of exponential
parallel.
functions.

3. Define inverse hyperbolic 8.6.1 D e f i n i t i o n For all x, we define


functions.
( b ) coshx =,
ex + e-'
4. Interpret inverse hyperbolic 2
functions as logarithms.
These are called the hyperbolic sine and hyperbolic cosine functions, respectively.
5. Use inverse hyperbolic functions (Sinh x is pronounced "cinch x," and cosh x is pronounced the way it looks.)
in evaluating integrals.
The graphs of these functions can be obtained by simple addition of ( 1 / 2 ) e x
and ( - 1/2)e-" in the case of sinh x, and addition of ( 1 / 2 ) e z and (1/2)e-' in the
case of coshx; see Fig. 8 . 1 0 .
We define the remaining hyperbolic functions in terms of sinh x and cosh x.

8.6.2 (u)
sinhx - ex - e-"
tanhx = --
coshx - ex e-'
( b ) coth x = --
+
coshx ex +
e-" sinhx ex - e-"
1 - 2 1 -- 2
( c ) sech x = -- ( d )csch x = -
coshx e x + e-" sinhx e x - e-'
These functions obey identities analogous to the trigonometric identities. We
state three and give the computational proof of the first.

8.6.3

4 4

a , J = sinh x ( b ) 1 = cosh x
Fig. 8.10
310 CHAPTER 8: MORE ON TRANSCENDENTAL FUNCTIONS

t WHY THE WORD eHYPERBOLICf7?

t The ordinary trigonometric functions are also often called circular functions
because of the role of the circle in defining them. Given the unit circle in the s-t
plane we defined sin x = t and cos x = s, where x is the radian measure of the
angle illustrated in Fig. 8.1 1. We then defined the remaining trigonometric
functions in terms of this circle. Because cos x and sin x are the coordinates of P
on the unit circle, we know that cos2x +sin2 x = 1.
For purposes of continuing and deepening the parallel between trigonomet-
ric and hyperbolic functions, we shall reinterpret x in terms of the area of sector
OAP. The area of the whole unit circle is 7 r 2 = T . Since x/(27) is the fraction
Fig. 8 . 1 1 of the whole 277 radians of a full rotation, the area of sector OAP is the same
fraction of the total area T . That is,
X X
Area of sector OAP = - * T =-
277 2
Thus the variable x for the trigonometric functions is twice the area of sector
OAP.
If the generating line rotated in a clockwise direction, we would interpret the
area as negative, so the variable x can take on all real values.
But if we begin with the set of points (s, t ) satisfying s2 - t 2 = 1, we have the
so-called unit hyperbola, pictured in Fig. 8.12. This particular conic section is
examined in detail in Sec. 10.2.
This graph has two branches and we use the right (solid) one. As the point P
moves upward and to the right along this branch of the hyperbola, the area of
the shaded region OAP increases from 0 without bound, and if P moves
downward and to the right below the horizontal axis, we interpret the area as
negative. Hence, if we let x be double the area of OAP, x can take on all real
values. We could then redefine
sinh x = t cosh x = s
I where P = (s, t ) , and define the remaining hyperbolic functions in terms of
Fig. 8 . 1 2 these as before.
In Exercise 64 we shall show that the x serving as the variable is really twice
the area of OAP. Notice that since Pis on the hyperbola, we know that [8.6.3(a)]
holds.
You may wish to examine the behavior of cosh x and sinh x in terms of the
motion of P. (For example, cosh x 2 1 since P is on the right-hand branch of the
hyperbola.)

DERIVATIVES OF HYPERBOLIC FUXCTIOSS


The analogy with trigonometric functions continues among the derivatives of
the hyperbolic functions, although we need to be careful with minus signs.

8.6.4 For u a differentiable function of x :


( a ) D,(sinh u) = cosh u duldx ( b ) D,(cosh u) = sinh u du/dx
(c) D,(tanh u ) = sech2 u du/dx ( d ) D,(coth U) = -csch2 u duldx
( e ) D,(sech u) = -sech u tanh u duldx (f)D,(csch u) = -csch u coth u du/dx

Proof q(u): D,(sinh x ) = D,


tx2
~

ez - P (- 1)
e-x)

- = coshx
2
so ( a ) follows by the Chain Rule. Proofs of ( b ) through (f)are left as progress
test problems and section exercises. W
SECTION 8.6: HYPERBOLIC FUNCTIONS AND INVERSE HYPERBOLIC FUNCTIONS 31 1

HANGING CABLES
Hyperbolic functions are useful in describing natural phenomena. Whereas the
trigonometric functions are periodic and are thus useful for describing cyclic
phenomena, the hyperbolic functions are not periodic and thus they describe
different sorts of phenomena.
Suppose a homogenous flexible cable is hanging between two points sym-
metric with respect to they axis such that its lowest point is (0, a). An analysis of
the forces on each segment of cable to determine its shape leads to the differen-
tial equation

A straightforward calculation shows t h a t y = a cosh (./a) satisfies this differ-


ential equation, and therefore the shape of this curve (often called a catenary
curve) is the graph of the hyperbolic cosine function. Hence we see graphs of
hyperbolic cosines every time we look at telephone or electric cables suspended
between poles.

ASTIDIFFEREXTIATION OF HYPERBOLIC FUNCTIONS


As we know, every derivative formula gives rise to an antiderivative formula.
Here are the six we have so far:

8.6.5 (a) J sinh u du = cosh u C+


( b ) Jcosh u du = sinh u + C
(c) J sech2 u du = tanh u + C
(d) J csch2 u du = -coth u +
C
(e) 1 sech u tanh u du = -sech u +C
(f)Jcschucothudu = -cschu +C
The techniques for handling combinations of hyperbolic functions are
analogous to those for trigonometric functions, including the use of identities.

Example 1 Evaluate sinh3 x dx.


Solution sinh3 x = (sinh x)(sinh2 x) = sinh x(l + cosh2 x)
by [8.6.3(a)],so
J sinh3 x dx = 1sinh x dx + J cosh2 x sinh x dx

Identities analogous to double-angle formulas are developed in the section


exercises.

PROGRESS TEST 1

In Probs. 1 and 2 prove the given statement.


1. [8.6.3(6)] 2. [8.6.4(b)]
+
3. Show that D,(cosh x + sinh x) = cosh x sinh x.
4. Given that tanhx = 1/2, determine the values of
the remaining hyperbolic functions.
5. Evaluate 1sech4 x dx.
312 CHAPTER 8: MORE ON TRANSCENDENTAL FUNCTIONS

DEFISITIOXS OF INVERSE HYPERBOLIC FCKCTIOSS


The hyperbolic sine, being strictly increasing on its domain R has an inverse
defined on its range R ; see Fig. 8.13. O n the other hand, we are required to
restrict the hyperbolic cosine to its right half-the solid part of Fig. 8.14(u).

8.6.6 Definition
( u ) For all x, arcsinhx is that uniquey such that sinhy = x .
(6) For all x 2 1, arccoshx is that unique y 2 0 such that coshy = x.

Figures 8.13 through 8.18 contain graphs of each hyperbolic function in ( u )


and the respective inverses in ( b ) . For functions that are not strictly increasing,
their restriction to a strictly increasing piece is given as the solid part of (u).
Asymptotes are dotted. Included is any significant information regarding
domains.

Since the hyperbolic functions are defined in terms of exponential functions,


it is reasonable to expect that their inverses can be expressed in terms of
logarithm functions. For example, we have:

8.6.7 (u) arcsinhx = ln(x +d m )


(b) arccoshx = ln(x + vR) x 2 1

Y Y

:a)

Fig. 8.13

4 4

I Ir; t
y = cosh x 4 = arccosh x
x t l
(Q) jb)
Fig. 8.14
SECTION 8.6:HYPERBOLIC FUNCTIONS AND INVERSE HYPERBOLIC FUNCTIONS 313

Note that (a), for example, can be proved by showing the left and right sides
have the same derivative and then noting each side is 0 when x = 0. (A
standard implicit differentiation argument shows that D,(arcsinh x ) =
1/ m.,
Of course, each derivative formula for inverse hyperbolic functions gives rise
to an antiderivative formula. Since the antiderivative formulas are more
commonly used, we list these only.

I
y = tanh x y = arctanh x
1x1 < 1
ia) (b)
Fig. 8.15

-1 I

y = coth x y = arccoth x
x # O 1x1 >1
(a) (b)
Fig. 8.16

Y Y

y = sech x y = arcsech x
O < x < l
(0) (b)
Fig. 8.17
314 CHAPTER 8: MORE ON TRANSCENDENTAL FUNCTIONS

Y J

y = csch x J = arccsch x
X # O X # O

(a) (b)
Fig. 8.18

8.6.8
du = arcsinhz + C = ln(u + d m ) + c a >0
U

(b) sd du
n = arccoshz + C = ln(u + d n )+ C
U
u >a >0
if /uI < a
!
1
- arctanh + C
U a

= -1n
1
1
- arccoth -
a

1-
U

a f u
U
+C
+c
I if IuI >a
~ u l #a , a +o
2a a-u

Example 2 Evaluate s dx
d ( x - 2)2 +5
Solution Here we use [8.6.8(a)] with u = x - 2 , a = 6to get
dx
= arcsinh- x - 2

The answer to Example 2 could also have been expressed as a natural loga-
rithm.
Virtually all integration problems of this section will yield to other methods
developed in Chap. 1 1 , so one should be especially alert to the variety of forms
an answer can take.

PROGRESS TEST 2

In Probs. 1 to 3 evaluate the given integral. Express your


answer to Prob. 1 as a simplified logarithm.
dx - dx
1.
d x 2 + 6x +3 2* J 9x2 - 12x
SECTION 8.6: HYPERBOLIC FUNCTIONS AND INVERSE HYPERBOLIC FUNCTIONS 3 15

SECTION 8.6 EXERCISES

1. Given that csch x = 2 , determine the values of the re-


maining hyperbolic functions a t x = 2.
2. Repeat Exercise I , given that coshx = 3/2.

In Exercises 3 t o 20 dzfferentiate the given function.


3. f (x) = sinh3(x4) 4. y = tan3-
5. y = dl + 2 sinhxcoshx 6. t(x) = [coth2(x l)]cschx +
7. u = tanh ( l l x ) 8. y = ln(coth eE)
9. f (x) = ln(tanh x - csch x) 10. g(x) = tanh2(sechx)
11. y = XSinhs 12. y = XCOSha
13. f(x) = ln(sinhex) 14. y = sech3(x - 1)
15. s(t) = t 2 cosh 2t 16. g(x) = x(coshx) - d m
sinh x
17. y = - 18. h(x) = cosh(cscx)
arccos x
19. y = arcsin(sinhx) 20. y = cosh(cosx)

In Exercises 21 to 31 prove the given statement.


21. [8.6.3(c)] 22. [8.6.4(d)] 23. [8.6.4(e)] 24. [8.6.4(f ) ]
25. [8.6.7(6)] 26. [8.6.7(c)] 27. [8.6.7(a)] 28. [8.6.4(c)]
29. [ 8 . 6 . 7 ( f 11
30. [8.6.8(a)], by differentiating. 3 1. [8.6.8(b)], by differentiating.
32. (a) Show that coshx +
sinhx = e x and coshx -
sinhx = e-'.
+
( b ) Show that sinh(a b ) = (eaeb - e-'e-*)/2.
(c) Combine ( a ) and ( 6 ) to prove that sinh(a b) = +
sinh a cosh b +
cosh a sinh b.
33. Prove that cosh(a +
b ) = cosh a cosh b +
sinh a sinh b
(see Exercise 32).
34. Use Exercise 32 to prove that sinh 2a = 2 sinh a cosh a.
35. Use Exercise 33 to prove that cosh 2a = cosh2 a +
sinh2 a.
36. Prove that sinh' a = (cosh 2a - 1)/2.
37. Prove that cash' a = (cosh 2a 1)/2. +
38. Determine lim tanh x.
x-+=
39. Determine lirn sech x. 40. Determine lim cschx.
x-** X+?W

41. Determine lirn coth x. 42. Determine lirn csch x.


a-0 2-0

In Exercises 43 to 63 evaluate the given integral. (Note that some integrals may involve the use of hjperbolic identities, including those in Exercises 33
to 37.)

43.
J csch &cot h
&
fidx
44. s tanh2 u sech2 u dx

45. l: cosh3 x dx 46. s1


-1
sinh5 x dx

47. 4' sinh2 x dx 48. s cosh2 x dx

49. s x tanh(x2)ln(cosh x2) dx 50. s c s c h 4 x dx


316 CHAPTER 8: MORE ON TRANSCENDENTAL FUNCTIONS

dx dx
55.
J dTTG 58.
J d 9 x 2 + 6x + 3
dt
57. 58.
.f 8t + 2 - 4t2
59.
l3z dx
80.
dx
J d x 2 - 6x + 3
sin x dx
62.
61.
J* J- V G T T T
63.
i1 + d z z - 8z
Gz
!
13
66. Evaluate the integral J sinh x cosh x dx in two ways and
64. Recall Fig. 8.12.
(a) What is the area of triangle OQP in terms of sinh and show that your results are equal.
cosh? 2 tanh x
67. Prove that tanh 2x =
( b ) The area of the shaded region is the answer to ( a ) 1 + tanh2 x

minus the area of region AQP, this latter area is 68. Reprove [8.6.3(a)] using derivatives.
Jg sinh x d(cosh x). Evaluate this integral. 69. Reprove [8.6.3(b)] using derivatives.
(c) Show that x is twice the area of OAP. 70. Solve the differential equation (k2/32) dv/dt = k 2 - u 2 .
65. (a) A body is falling through air. T h e air resistance is 71. ( a ) Compute and simplify D,[cosh(ln x)]
proportional to the square of the velocity and the pro- ( b ) Antidifferentiate the result of ( u ) and then prove that
portionality constant is k . Then the distance fallen in cosh(1nx) = ( x + 1/x)/2.
feet as a function of time t in seconds is s = 72. Use first and second derivatives to confirm the graph of
(l/k)ln(cosh a t ) . Find the velocity as a function of y = cosh x , given in Fig. 8.10(b).
time. 73. Repeat Exercise 72 fory = sinh x , given in Fig. 8.10(a).
( b ) Show that the velocity tends to a constant, and 74. Repeat Exercise 72 fory = tanh x, given in Fig. 8.15(a).
determine this constant as t becomes large. Interpret 75. Repeat Exercise 72 fory = cschx, given in Fig. 8.18(n).
your results.
(c) Would k be larger for a person with a parachute or
without a parachute?
( d ) Find the acceleration as a function of time.

CHA4PTEREXERCISES

R e v i e w Exercises
1. How long will it take $100 to grow to $10,000 if ( b ) How many will there be i n 60 days if there were
interest is compounded continuously at 10 percent? only five flies to begin with?
2. Repeat Exercise 1, but with the original principal (c) If the population doubled every 10 days and there
doubled and the interest rate cut in half. were 10 flies to begin with, how many fruit flies will be
3. ( u j Suppose the population of a colony of fruit flies present in 60 days?
doubles every 20 days. Beginning with 10 flies, how
many fruit flies will there be in 60 days?

In Exercises 4 to 21 compute the given quantity Ifdefined.


4. arcsin(sin 1 1 ~ / 4 ) 5. arcsec(cos 77/2)
6. arcsin 1 7. arctan 1
8. arcsec 1 9. arcsin I / fi
10. arctan 100 11. arcsec 10
12. arcsin(tan 77,$4) 13. arctan(sin 77/2j
14. cos(arctan 1 +
arctan(l/fi)) 15. tan(arcsin 0)
16. sin(arctan 0) 17. arcsin(arctan( - 1))
18. arcsec( 1/2 j 19. arcsec v5
20. arcsec(2/ $1 21. arctan( - 1)

In Exercises 22 to 53 differentiate the given function.


22. f(x) = (arcsinx)3 23. f(x) = d z
24. g(x) = arcsin(x3) 2 5 . g(x) = arctan &
SECTION 8.6: HYPERBOLIC FUNCTIONS AND INVERSE HYPERBOLIC FUNCTIONS 3 17

26. h ( x ) = ln(arcsec(x + 1)) 27. ~ ( x )= cot(ln(arcsec x ) )


28. = e z arctan z 29. m ( x ) = x 2 arctan 2 x
arctan x 3 1. p ( x ) = arctan(logz x)
30. y =
arctan( l / x )
32. y = sin(arcsin x 2 ) 33. g ( x ) = arcsin(cos3x)
34. f ( x ) = arcsin(3x + 1) 35. f ( x ) = arctan(1nx)

36. f ( x ) = [arcsin(3x + I)l4 37. f ( x ) = +--


+ a x2
arctan x

38. f ( x ) = lnlarctanx, 39. f ( x ) = arcsin


(O:;nt )
40. f ( x ) = earcsec 41. f ( x ) = arcsecjcsc x)
1
42. f ( x ) = arcsec(e) 43. f ( x ) = -arctan fi
6
y =~~arccoshx 45. g ( x ) =
44.
1 +coshx
sinhx
46. J ( x ) = coshx - cosh(x2) 47. y = cosh(1n sec x)
48. f ( x ) = e P s i n h x 49. y = X t a n h x
cosh 2 x
50. h ( t ) = csch(1nx) - sech(1nx) 51. f ( x ) =
1 - cash 2 . ~

52. m(z) = tanh(zcot2)

In Exercises 54 to I45 evaluate the given integral.


54. J4 sin(m) dw 5 5.

56. s sec(e-)tan( e-) dx


ex
57.
sGT
dx

58. J n;b;x dx 59.


s (3x
dx
+ 9)2
60. [ + 3x2
x4
x-1
- x + 7 dx 61.

62. s + (3x
dx
5)12
63.

64- s dx
(2x - 7)2
65.

s& 67. J c0s3 3x dx

68. s tan3(:) dx

etan x
70. j - z
72. J cos2 x sin3 x dx

[In( 1 + cos .)I3


74. J (csc x ) ( l + cos x)
dx

76. J - x csc(3x2) cot(3x2) dx


78. J tan2(lnx) dx
79.
f (1 + 4x) v1n(3x3 + 9x4) dx
X J x + 3x2
80. J - e z tan(1 + e) dx

sin3(3 x ) dx
83. S T d x
82- J f/G@q
318 CHAPTER 8: MORE ON TRANSCENDENTAL FUNCTIONS

84. 4 .r sin4 2x dx 85. hr sin 3x cos 5x dx

+ 1) dx
88.

88.
J ( t a n x sec x
J cot5 x csc4 x dx
) dx~ 87.

89. Je
J x sin2(x2

90. J cos x cos 3x dx 9 1. Ssin3 3x cos5 3x dx


92. J (sin3x ) G d x 93. J cos3 x G d x

94. 1::: tan3 x cos x dx 95. J, r


COS,
x
7dx
98. J cot 2x csc4 2x dx 97. J (cot x)ln(sin x ) dx
-4 cos 2x
98. dx 99. J cot 3x csc4 3x dx

100. s- dx 101. J tan1I2 x sec4 x dx

102. dx
J 6 2 +x +2x4 - x2 dx 103.

104.
s 4- dx
105. J 5 dx
X V 5 7 7

106.
In(arcsin x )
dx 107. s1
,,
dx
4 x 2 + 12x + 14
x 2 dx dx
108. 109.
( x - 3 ) d x 2 - 6x +5
110.
L,,
a/3

66x - 1-x2
dx
111. s e4x
dx
+ e-4x

112.

dx
114.
J-6
3 dx
118.
J (3x + 1)6iFTG 117. J dx
x 2 arctan x arctan x +
118. 119. J X G

120. 121. s dx
x1/2 + x3/2

122. 123. s7d T s % zcos x dx

124. 125. J-d sin x dx


sec x t a n x dx
126. 127,
J 1 + sec2x

( M o s t of t h e integrals in Exercises 128 to 145 involve hyperbolic functions.)


129. J t a n h x dx
131. J c o s h 3 4 x d x

dt
133- J(,t + e-t)2

134. SqFy dt 135. J 5 - 9x2dx - 12x


SECTION 8.6: HYPERBOLIC FUNCTIONS AND INVERSE HYPERBOLIC FUNCTIONS 3 19

137. s dx
dlOx - 25x2

138. s dx
dxz - 2x + 26
139. J(x2 + 1)-lIz dx
141. s dx2 dx
- 6x + 10
In(1/e )
142. J x sinh3(x2)dx 143.
L,, cosh 2x dx

144. J coth2 t dt 145. 4 tanh3 x dx

146. Prove that (cosh x +


sinh x ) 3 = cosh 3x +
sinh 3 x .
149. Determine the area of the region bounded by y =
147. Solve the differential equation d 3 / d x 2 = k2y.
148. Determine the area of the region bounded by the curve
3/(x2 +
1) and the x axis from x = 0 to x = 1.
150. Show that y = ear sinh bx satisfies the differential
y = 1/(x2 +
l ) , the lines x = -1, x = 1, and the x
equationy - 2 9 + ( 2 - b21y = 0.
axis.

Miscellaneous Exercises
1. A painting masterpiece is 4 ft tall and will hang with 12. Evaluate J cos x cos 3x cos 5 x dx.
its bottom edge 8 ft above floor level. How far from the 13. Evaluate $ sin 2 x sin 4x sin 6x dx.
wall on which the painting is hanging should a person
(sech2 6)
d-dx
stand so that the angle subtended by the top and
bottom of the painting is a maximum, assuming that
the persons eyes are 5 ft above the floor2
14.

15.
Evaluate

(u)
s 6
Given a mixture of radioactive substances contain-
2. Assuming that a cube of ice melts at a rate propor- ing 6 g of A and 3 g of B , where the half-life of A is 10
tional to its surface area and decreases from 15 cm on years and the half-life of B is 20 years, how long will it
an edge to 12 cm in 1 h, how long will it take to melt take for the mixture to contain equal parts of A and B ?
completely away? (Note:T h e rate of melting is a rate of ( b ) How long will it take for the mixture to contain
decrease of volume.) twice as much B as A ?
3. ( u ) T h e half-life of radium is about 1600 years. How 16. ( u ) A porous material dries at a rate directly propor-
long will it take a chunk of radium to decay to 99 tional to the amount of moisture remaining (depend-
percent of its original mass? ing upon the material and the relative humidity). If a
( b ) Without using a calculator or tables, determine wet washcloth loses a quarter of its moisture in 30 min,
how long it will take the radium to lose 75 percent of its how long will it take to lose 90 percent of its moisture?
mass. ( b ) How long will it take to lose 99 percent of its
4. Strontium 90, a product of hydrogen bomb explosions, moisture?
has a half-life of about 27 years. If the amount in a 17. ( u ) If the worlds population continues to grow expo-
particular place is four times the safe level, how long nentially at its current rate of 1.8 percent and was 3.9
will it be before that place is safe? billion in 1975. when will it reach 6 billion?
5. Repeat Exercise 4 if it is discovered that what was ( b ) When will it reach 10 billion?
18. ( u ) Using the data given in Exercise 17(a) and the
originally regarded as safe is double the really safe
level. statement that the current estimate of the upper limit
G . Graph the equationy = x - 2 arctan x . for the human population on this planet is 10 billion.
7 . Suppose a telescope is mounted on a 100-m-tall build- how long will it take to reach 6 billion assuming (8.1.9)
ing 100 m away from a 200-m-tall building. This tele- holds?
scope will follow the flight of a freely falling object as it ( b ) When will it reach 8 billion?
drops to the ground from the top of the 200-m-tall (c) When will it reach 9 billion?
building. At which height of the object is the telescope 19. Solve Exercise 18 assuming that the growth rate falls
turning at the fastest rate? linearly 0.0002 per year beginning in 1975.
8. Repeat Exercise 7 with the object dropped from the 20. Solve Exercise 17 assuming that the growth rate falls
150-m level instead of 200 m. linearly 0.0002 per year beginning in 1975.
9. Prove a difference formula for cosh(u - 6). 21. Solve Exercise 17 assuming that the growth rate in-
10. Prove arcsin(tanh x) = arctan(sinh x ) . creases linearly 0.0002 per year beginning in 1975.
11. ( u ) Determine the inverse of the function
f ( x ) = cosh(3x + 1).
22. Evaluate s sech2 x dx
dtanh2x - 4
( b ) Calculate the derivative o f f - l found in part ( u ) .
320 CHAPTER 8: MORE ON TRANSCENDENTAL FUNCTIONS

23. Prove that (cosh x - sinh x)" = cosh nx - sinh nx. 26. Prove that arcsin x = arctan
( d b ) .
24. Evaluate [
[arccos(coth x)]csch2 x
dl - coth2x
dx.
27. Determine the voIume of the solid generated by re-
volving about the x axis the region enclosed by
25. Prove that arctan x = arcsin y = - , the x axis, and the line x = In 2.

In Exercises 28 to 30 graph the given function ( b e careful u i t h do-


mains).
28. f ( x ) = sin(arcsin x ) 29. f ( x ) = arcsin(sinx) 30. f ( x ) = sin(arccos x)
31. Sketch the graph of ( u ) y = arctan(e-") and ( b ) y = 34. When a capacitor discharges, the rate of change of
arcsec(x/2v) (on its natural domain). voltage (in volts per second) is proportional to the
32. Using implicit differentiation, graph the equation voltage, with a negative proportionality constant. Give
arcsin(x - y ) = 0. If you are unsuccessful, try using the the voltage as a function of time t.
definition of arcsin and common sense. (Be careful, this 35. Given that y 2 + x sinhy + cosh2 x = 1, determine
problem is trickier than it looks.) dyldx.
33. ( a ) Set up a differential equation describing the
change in temperature T o f a hot object brought into a
cool room where the temperature is rising a t the rate of
1C every 5 min.
( b ) Solve this differential equation.

SELF-TEST +
322 CHAPTER 9 IMPORTANT IDEAS REVISITED

Section 9.1 : The Formal Definition of Limit

Objectives: THE IDEA OF SEARNESS


1. Interpret as nearness In Chap. 2 , Sec. 2 . 2 , we repeatedly discussed the idea of limit in terms of closer
conditions and graph absolute and closer, nearer, and so on. The following function (which first appeared
value inequalities of the form in Sec. 2.5, Progress Test 2) illustrates the inadequacy of these informal state-
0 < lx - -
a1 < 8 and I f ( x ) L1 c E. ments and the need for the more precise definition of lim,,,f(x) = L given in
(2.5.1) and repeated below.
2. Apply the definition of limit for
<
certain simple, mainly linear,
functions. f(x) =
i
x
2
x
+ 1.95
+ 2.1
for x

for x >
0
for x = 0
0
The graph off(x) is given in Fig. 9.1. Quite clearly, as x gets nearer and nearer
to 0 from either side,f(x) gets nearer and nearer to 2. Just as clearly, however,
lim,,of(x) # 2 .
The problem is, that althoughf(x) does get closer and closer to 2, no value of
f ( x ) exceptf(0) is within 0.05 unit of 2. As a result, no matter how close we take
nonzero x to zero, we cannot force the values off(x) to be within 0.05 unit of
2. We need a more precise definition of limit. But before we give this definition
we review the idea of nearness) as it can be expressed using absolute value.
Recall from Chap. 1, Sec. 1.1, that I x - a ( = distance between x and a;
that is, X I /a +
I
+ I

I x - a1

Fig. 9 . 1 Suppose we wish to insist that x be a number within 0.5 of the number 2 .
Then we want the distance between x and 2 to be less than 0.5 unit. But the
distance between x and 2 is Ix - 21, so our condition will be satisfied for any
number x for which lx - 21 < 0.5. Graphically, x must be in the interval

-
0.5 0.5
AA (1.5, 2.5); see Fig. 9.2. Certainly any such x in this interval is less than 0.5 unit
L
1.5 2 2.5
from the number 2.
Now suppose 6 represents some small positive number (6 is the lowercase
Fig. 9.2 Greek letter delta). How do we specify that x be within 6 units of the number a?
In light of the above, this is equivalent to specifying that x satisfy the
inequality
Ix - al <6
a - 6 a a f 6 or, again equivalently, that x be in the interval ( a - 6,a + 6):
Fig. 9.3
6 6

i a - 6 a a + 6

Now, I x - a1 = 0 only when x = a, so the condition that x be within 6 units of


a, but not equal to a, can be given by
< Ix - a / < 6
0
The left-hand side of the inequality, 0 < /x - al, simply guarantees that x # a,
whereas the right-hand side, Ix - a / < 6, guarantees that x be within 6 units of
a. This gives rise to the interval ( a - 6, a + 6) with the point a removed; see
Fig. 9.3. We refer to such as a deleted interval.
Similarly, the condition thatf(x) be withine units of the number L (e >
0) is
equivalent to the condition that f ( x ) satisfy the inequality
Fig. 9.4
SECTION 9.1: THE FORMAL DEFINITION OF LIMIT 323

or, equivalently, thatf(x) be in the interval ( L - E , L E ) ; see Fig. 9.4. We +


pictured the interval vertically since values off(.) are measured on the vertical
axis.

Example 1 Show that when x is within 0.01 of 1, it follows thatf(x) = 2x + 3 is within 0.02
of 5 . Sketch the situation.
Solution <
We wish to show that when Ix - 11 0.01, it follows that If(.) - 51 < 0.02.
Evaluating the function and simplifying, we see that

If(.) - 51 = 1(2x + 3) - 51
= )2x - 21
= 21x - 11
< 2(O.O1) (since Jx - 1) < 0.01)
= 0.02
Thus we cah conclude that when 1x - 11 0.01, <
- 5) If(.) < 0.02, which is
what we wanted to show.
A slightly exaggerated sketch is given in Fig. 9.5. 0

Example 2 shows that the calculations and reasoning of the previous exam-
Fig. 9.5 ple apply even when the small numbers involved are not specified.

Example 2 Supposef(x) = 2 x + 3 and E >


0 is any small positive number. Show that if x
is within ~ / units
2 of 1, thenf(x) is within E units of 5 .
Solution We wish to show that if JX - 1 I < ~ / 2 then
, if(.) - 5) < E. As before,
+
i f ( x ) - 5 ) = / ( 2 ~ 3 ) - 51
= 21x - 11

<2 (since IX - 11 <~ / 2 )


= E

SO 1 f(x) - 51 < E , as we needed to show. 0

Notice that, having done Example 2 , we could answer the following ques-
tion: How near must x be to 1 to guarantee thatf(x) be within E of 5? (Answer: x
must be within 6 = ~ / of
2 1.) Suppose we had not done Example 2 and had
been asked: How near must x be to 1 to guarantee thatf(x) be within 0.5 of 5?
We could answer this question by writing out the stipulated condition and
simplifying. We need
if(.) - 5 1 0.5 <
that is,
I(2x 3 ) - 51 0.5 + <
or
12x - 21 0.5 <or 21x - 1) 0.5 <
Now the answer can be recognized. Divide both sides of the inequality by 2 to
get

Ix - 11 <-
05
2
= 0.25
324 CHAPTER 9: IMPORTANT IDEAS REVISITED

Thus x must be within 0.25 of 1 to guarantee thatf(x) be within 0.5 of 5. In


exactly the same way we could answer the question: How near must x be to 1
to guarantee thatf(x) be within E of 5? (Answer: x must be within S = ~ / of
2 1;
here we replace 0.5 in the above argument by E.)

PROGRESS TEST 1

1. The inequalities in (a) and ( b ) gives rise to a n inter- 3. (a) Show that if x is within 0.1 unit of 2, then
val. Sketch the interval. f ( x ) = 4x - 1 is within 0.4 unit of 7.
(a) lx - 21 <
1/2 <
(6) 0 I X - 91 3 < (6) Draw a sketch of the situation in (a).
2. Write down a n inequality that has as its solution the 4. For each of the following, determine how near x

-
given interval. must be to a to ensure thatf(x) will be within E units
(deleted interval) of L. Write your answer as a n inequality involving
(a) -1 2 3
absolute value.
( b ) (1.99,2.01) (a) f ( x ) = 3 x + 1, a = 3 , E = 1, L = 10
(6) f ( x ) = 2 x - 2, a = - 1 , E = 1 / 2 , L = -4
(deleted interval) (c) f ( x ) = (1/2)x + 1, a = 1 / 4 , E = 1/5, L = 9 / 8
-8 -3 +2

THE DEFIYITION OF LIMIT


We now recall the formal definition of limit.

9.1.1 Definition Suppose we are given a functionf(x) and numbers a and L with
f ( x ) defined for all points, except perhaps for a itself, of an open interval
containing a. We say
the Lzmit of f ( x ) as x tends to a 2s L
and write
limf(x) = L
x-a

if for every positive number E, there is a positive number S such that


0 < 1x - a1 < 6 implies if(.) - LI < E

$.
This definition requires tl& for every E >
0 there must be a 6 0 such that >
x being in the deleted interval (a - 6,a +
6) implies that f ( x ) is in
( L - E , L + E ) . The definition formally records our often repeated remarks
regarding indifference to what happens at a and the concern only with x near a.
The limit is unaffected byf(a)-and, in fact,f(a) need not even be defined.
To prove that limz.af(x) = L in a particular instance, we assume that E 0 >
is a number out of our control, given in advance. Our task then is to show that
there exists a S >0 such that 0 <
Ix - a / <
S implies - LI E .If(.) <
The logic of this approach is simple. Having assumed E to be an arbztrary
positive number, if we then show that an appropriate S 0 exists, we can >
conclude that such a S exists for every E >
0, as required by the definition,
Graphically we are showing that there is a deleted interval about a such that if
x is in this interval, thenf(x) is in the (vertical) interval ( L - E , L + E). See Fig.
9.6. Another illustration appears in Fig. 2.26, page 62.

Example 3 Prove, using (9.1.1), that limE*l ( 3 x + 1 ) = 4.


Solution Here L = 4 and a = 1. Suppose E > 0 is given. We must show that there exists
>
a 6 0 such that
0 < lx - 11 < S implies l(3x + 1) - 41 < E
SECTION 9.1: THE FORMAL DEFINITION OF LIMIT 325

But
+
1 ( 3 ~ 1) - 41 = 1 3 -
~ 31
= 3/x - 11 < E if lx - 11 < ~ / 3
That is, by choosing 6 = e / 3 , we know that

0 < 1x - 11 < 6 implies I(3x + 1) - 41 < E


Hence, by the definition, limx+l (3x + 1) = 4. 0

In Example 3 the function was defined at a. In Example 4 , f ( a ) is not defined.


(The function here is essentially twice the function in Sec. 2.2, Progress Test 1,
Prob. 2, page 45.)

2x2 - 18 = 12
Example 4 Prove, using (9.1.1), that lim
2-3 x -3
Solution Here L = 12 and a = 3. Suppose E > 0 is given. It is now our task to show there
>
exists a 6 0 such that

However, since 0 < / x - 31, we know that x # 3. For suchx,


2x2 - 18
-
-
2(x - 3 ) ( ~ 3)+= 2(x + 3)
x - 3 (2 - 3)

Thus the condition to be satisfied,

I 2x2 - 1 8 -
x-3
reduces to 12(x + 3) - 121 = 21x - 31< E . But this condition is satisfied when
<
Ix - 31 ~ / 2 .Hence choose 6 = ~ / 2 ,so then
0 < 1x - 31 S = -< &

2
implies
2x2 - 1 8
x-3 I
In Example 4 the requirement that x be in the deleted interval about 3
enabled us to replace (2x2 - 18)/(x - 3) by 2(x 3). +
We can now apply Definition (9.1.1) to prove our contention in the lead-off
discussion of this section that the limit as x tends to 0 of the function given there
does not equal 2 even though the values of the function get closer and closer to
2 as x gets closer to 0.

+ 1.95 for x < 0

1
x
Example 5 Supposef(x) = 2 for x = 0
x + 2.1 >
for x 0
Prove that
limf(x) #2 (see Fig. 9.1)
x-0

Solution Here a = 0 and the alleged L = 2. To prove lim,,of(x) = 2, we would need to


show that for every E >
0 there exists a S 0 such that >
O < 1x - 01 < 6 implies ).(fI - 21 < E
Hence to prove limx-of(x) # 2 , we need to find an E >
0 for which there does
not exist such a 6. A clue to finding such an E is given by a close-up view of the
326 CHAPTER 9: IMPORTANT IDEAS REVISITED

graph off (x) near x = 0 (see sketch). There are no values off (x) within 0.05
unit of 2 except f ( 0 ) = 2. Choose an E with 0 E < <
0.05, say E = 0.03. We
>
claim there is no 6 0 such that

0 < 1x - 01 = 1x1 < 6 implies If(.) - 21 < E = 0.03


If there were such a 6,we would have 0 < <
1x1 6,implying that all the values
off (x) are within 0.03 unit of 2 . But there are no values of f ( x ) within 0.03 unit
of 2 (in fact there are none within 0.05 unit of 2) exceptf(0) = 2 ! But x # 0 by
the condition 0 < 1x1. Thus the statement

0 < 1x1 < 6 implies 1 f ( x ) - 21 < 0.03


which is supposed to holdfor all such x [in the deleted interval ( - 6,+a)], holds
f o r no such x.
Hence there is no 6 that works for E = 0.03, proving our claim that
lim f ( x ) #2 0
x-0

ONE-SIDED LIMITS
We frequently needed to use left- and right-hand limits. Our formal definition
(9.1.1) is two-sided in the sense that the condition

0 < Ix - al < 6
determines a symmetric deleted interval about a : ( a - 6,a + 6). We shall break
the absolute-value statement into a pair of statements and manipulate these to
show how each of the two resulting statements determines its half of the
deleted interval.
The absolute-value statement 0 < <
1x - a / 6 is, by (1.1.9), equivalent to
the pair of statements
0 < (x - a) < 6 and 0 < -(x - a) <6
But by multiplying the second of this pair through by - 1 and appropriately
reversing inequality signs, we see that the inequality 0 <
1x - a1 6 is equiv- <
alent to the pair of inequalities
0<x-a<6 and -6<x-a<O
Finally, we add a to each set of inequalities to get
a<x<a+6 and a-6<x<a
Hence we can associate the two halves of the absolute-value statement with the
two halves of the deleted interval ( a - 6,a +
6). (See Fig. 9.7.)
We can now make formal definitions of left- and right-hand limits. (We leave
f ( x ) = L as a Progress Test problem.)
the definition of limx+aL
SECTION 9.1: THE FORMAL DEFINITION OF LIMIT 327

x- - x-- -x
a - 6 a a + 6 a - 6 a a a + 6

0 < lx - %I is a - d < x < a a < x < a + b


Fig. 9.7

9.1.2 Definition Supposef(x) is defined for all x in some open interval (c, a). We
say the limit of f ( x ) as x ten& to afrom the l ~ ist L, and write
limf(x) = L
x-a-
if for every E > 0, there is a number 6 > 0 such that
a - 6 <x <a implies 1 f ( x ) - LI < E

Example 6 Show, using (9.1.2), that limx-,o-f(x) = 1.95, wheref(x) is as defined in Exam-
ple 5.
Solution Here L = 1.95 and a = 0. Suppose E > 0 is given. It is our task to show that
there exists a 6 0 such that >
-6 <x <0 implies If(.) - 1.951 < E
However, for x <
0,fis defined byf(x) = x 1.95 (hence is defined in (c, 0) +
for any negative number c), and thus for such an x
if(.) - 1.951 = (.I + 1.95) - 1.951 = 1x1
Since any positive number 6 for which -6 < x < 0 will satisfy 1x1 < 6, we take
6=E. 0

PROGRESS TEST 2

In Probs. 1 and 2 prove, using (9.1.1), that 3. Give a formal definition of right-hand limit analo-
lim,+J(x) = L. Draw a sketch of the function near the gous to (9.1.2).
point in question. 4. Use your definition in Prob. 3 to prove that
1. f ( x ) = 1 - 2x, a = -1, L = 3 lim,,,,+f(x) = 2.1 wheref(x) is as defined in Exam-
x2 - 4 ple 5.
2. f ( x ) =
~ , a = 2, L = 4
x-2

THE LIMIT IA' A GEA'ERAL CASE


We can now use a generalized version of the above limit arguments to prove the
following theorem, which was originally stated as (2.5.2).

9.1.3 Theorem Suppose thatf(x) = mu +


b is a given function, where m and b are
constants and x = a is any real number. Then
limf(x) = ma
x-u
+b
pro^ As usual, assume that E> 0 is given. We must find a 6 > 0 such that
0 < 1x - a/ < 6 implies I(mx + b ) - (ma + b ) / < E
But
I(mx + 6 ) - (ma + b)l = jm(x - a)l
= Iml Ix - a/ <E
328 CHAPTER 9: IMPORTANT IDEAS REVISITED

when

If m = 0, then any 6 works sincef(x) is a constant function with (mi lx - a / = 0.


Thus, choosing 6 = E/iml, form = 0, or S arbitrary form = 0, we can conclude
that
0 < Ix - a1 < 6 implies ).(fI - (mu + b)l < E

thereby proving the theorem. W

All our formal limit arguments to date, including the proof of the last
general theorem, have involved linear functions. The E-8 arguments for
nonlinear functions are less routine.

Example 7 Use (9.1.1) to show that limz+, l / x = 1 1 2 .


Solution We have L = 1 / 2 , a = 2 , with E > 0 given. We need to show that there exists
>
6 0 such that
0 < lx - 21 < 6 implies I? - < E (see Fig. 9.8)

For E < 1 / 2 , the linesy = 1 / 2 + E a n d y = 1 / 2 - E intersect the graph of


f ( x ) = l / x when l / x , = 1 / 2 + E and l/x, = 1 / 2 - E , respectively; that is,
when
2 2
x1 = and xp =
~

1 + 2E ~

1 - 2E
respectively.
y = 1/2 +6 We shall select S to be the smaller of the distances between 2 and x, and
between 2 and x,. But
X
4E 4E
Ixl - 21 = ___
1 2E + and Ix, - 21 = ____
1 - 2E
> 1x1 - 21
Fig. 9.8 so we choose 6 = 4 e / ( l 2~).+
In this argument, ifwe allow E 2 1 / 2 , theny = 1 / 2 - E lies below the graph
of f ( x ) = l / x for positive x, so If(.)
- 1/21 E for all x 2. Hence the < >
+
same S = 4 ~ / ( 1 2 ~ will ) suffice. 0

After making the effort to show that a particular function has limit L as x
tends to a, it is helpful to know that the function cannot tend to any value other
than L. That is, limits, if they exist, are unique.

9.1.4 Theorem (Uniqueness of Limits) If lim,,,f(x) = L , and lim,+,,f(x) =


L,, then L , = L,.
ProoJ Assume that lim,,af(x) = L, and lim,,,f(x) = L,, with L , # L,. We
shall derive a contradiction, showing the assumption to be untrue. Let
k = IL, - L,l, so k >
0. Then there exist 6, 0 and S , 0 such that > >
k
0 < lx - a1 < 6, implies If(.) - L,I < -
2
and
O < Ix - ul < 6, implies If(.) - L,I < 2k
SECTION 9.1: THE FORMAL DEFINITION OF LIMIT 329

Let 6 be the smaller of the numbers 6, and 6,. Then 0 < Ix - a1 < 6 means that
< < <
0 lx - a1 6, and 0 lx - a1 6,. Thus <

Hence L , must equal L,.

SECTION 9.1 EXERCISES

In Exercises I to 6 use znformal methods to determine L and then use the dejinition of limit to prove that lim,,, f(x) =L
f o r the given function f (x) and point a.
1. f ( x ) = 5x + 2, a = 3 2. f(x) = 2x + 5, a = 1
1 1
3. f ( x ) = - x - 2,a = 6 4. f ( x ) = - x - 1, a = 5
3 5
x2 -9 x2-x-6
5 . f ( x ) = -, a = -3 6. f ( x ) = ,a=3
x + 3 x-3
7. Prove, using the definition of limit, that lim,,gx2 # 7 . 13. Show that lim,,4 fi = 2 using the definition of limit.
8 . Prove, using the definition of limit, that 14. Show that lim2+o( $ + 1) = 1 using the definition of
limE+ ( l x - 21)/(x - 2 ) does not exist. limit.
9. Prove, using the definition of limit, that 15. Prove, using the definition, that limz+af ( x ) = b for any
limz+l (5x2 - 5x)/x = 0. constant function f (x) = b.
10. Prove that lim2+af ( x ) = L if and only if 16. Prove that lim,,,+ ( l / x ) # L for any real number L.
lims+a+f(x) = L and lims+a-f (x) = L. 17. Repeat Exercise 16 for limx+o-( l / x ) .
11. ( a ) How near to a = 2 must x be so that l/(x 1) be + 18. Prove that limx+l- s ( x ) = 0 for the largest integer func-
within 0.2 of L = 1/3? (Draw a sketch near a = 2.) tion s(x) of Sec. 2.1, Exercise 28.
( b ) How near to a = 2 must x be so that l/(x + 1) be 19. Prove that lim,,,+ s ( x ) = 1. (See Exercise 18.)
within 0.01 of 1/31
( c ) How near to a = 2 must x be so that l/(x + 1) be
within e ( E >
0) of 1/3? [Having given your answer, you
have proved that limx+2l/(x + 1) = 1/3.]
12. Repeat Exercise 11 for f (x) = x 2 with a = 2, L = 4.

In Exercises 20 to 23 show that the indicated limits do not exist.

1
20. lim-
0-1 (x - 1)2 x-2+ x -2

X2 for x 5 2 23. lim f ( x ) , where f ( x ) =


22. limf(x), wheref(x) =
0-2 (x +1 for x >2 2-1

24. Show that limf(x) = 10, where 25. Show limf(x) = 1, where
x-3 t-2

26. Using an argument analogous to the proof of (9.1.4),


prove using the definition of limit at infinity [see
(4.1.2)] that lirn,,,, f ( x ) is unique.
330 CHAPTER 9: IMPORTANT IDEAS REVISITED

Section 9.2: Limits and Continuity


Objectives: PROOF O F THE LIMIT THEOREM
1 . Understand the meaning of Having explored the meaning of limx,af(x) = L , we are now in a position to
abstract limit statements. prove selected portions of the Limit Theorem (2.5.4).
2. Use this understanding to prove
simple facts about limits. 9.2.1 The Limit Theorem Suppose that limx-a f(x) and limx+a g(x)
both exist. Then

and in particular, if b is a constant, then


lim [b * g(x)] = b [lim g(x)]
x-a .x-n.

- ~ provided that limg(x) #0


x-a
x-a
( d ) lim
x-a
m=d m x-a
where n is a positive integer. If n is even, thenf(x) must be nonnegative.

To facilitate our proofs, we assume that


limf(x) = L and limg(x) = M
x -a x-a

Proof that limx+a [ f ( x ) + g(x)] = L + M:


Suppose that E > >
0 is given, so ~ / 2 0 also. Since limx-af(x) = L, there exists
a 6, >
0 such that

0 < Jx - < 6, implies ~ f ( x-


) LI < z2
and since limx-a g(x) = M , there exists a 6,> 0 such that
o < (x - aj < 6, implies Ig(x) - M I < 4 2
Letting 6 be the smaller of 6,and 6,,we know that 0 < (x - a1 < 6 means that
both 0 < /x - a1 < 6, and 0 < 1% - a / < 6,. Thus

-
Proof that lim [ f ( x ) g(x)] = L M
x-a
We shall first prove that

0) If M = 0, then lim [ f ( x )* g(x)] = 0.


x+a

Given E> 0, we need a 6 > 0 such that


0 < 1x - a1 < 6 implies If(x)g(x) - 01 = If(&o)l< E
SECTION 9.2: LIMITS AND CONTINUITY 331

But since limX,,f(x) = L, there is a 6, > 0 such that


< lx - a1 < 6, implies
O If(.) - LI <I
For 0 < lx - a1 < a,, we thus know

so

Since limx+ag(x) = 0, there is a 6, > 0 such that

Letting 6 be the smaller of 6, and 6,, we know that 0 < Ix - a1 < 6 means that
both 0 < <
lx - a1 6, and 0 Ix - a1< 6,. Thus <
If(x)g(x)l = If(x)l I g ( 4

which completes the proof of (i).


(ii) We shall now use (i) to prove the general case. (The following argument has
a geometric interpretation; see Section Exercise 5 .)
+
Rewrite f(x)g(x) as ( f ( x ) - L)g(x) L(g(x) - M ) LM. Then, by +
[9.2.1(41,
limf(x)g(x) = lim [ ( f ( x ) - L)g(x)] + lim [L(g(x) - M ) ]
x-a x-a x-a
+ lim
x-a
LM

But lim,,,f(x) = L and limx-a g(x) = M , so limz+a ( f ( x ) - L ) = 0 and


limx,a ( g ( x ) - M ) = 0. Also, limx-a ( L M ) = LM by (9.1.3), with m = 0.
Hence, applying (i),
lim [f(x)g(x)] = 0
x-a
+ 0 + LM
proving the general case. W

Of course, if f ( x ) is the constant function, f ( x ) = -1, we know that


limx-a [ - lg(x)] = - llimz+ag(x). Using this fact and [9.2.l(a)] for sums, we
can conclude [9.2.l(a)] for differences.

Proof that lim f(4 L


-= -, provided that M # 0:
M
We shall first prove (assuming M # 0)
.
lim- 1 =-1
0)
x-a g(x) M
We know that IM1/2 > 0 and limx-a g(x) = M , so there is a 6, > 0 such that
0 < /x- a1 < 6, implies 1M - g(x)l < /MI
But JMI - / g ( x ) (5 IM - g(x)l, so if 0 < Ix - a1 < S,, then
332 CHAPTER 9: IMPORTANT IDEAS REVISITED

Also, there is a 6, > 0 such that

Letting 6 be the smaller of 6, and 6,, we know that 0 < Ix - a1 < 6 means
that both
0 < Ix - a1 < 6, and 0 < Ix - a / < 6,
Thus

proving (i). Now

The general proof that limx+a = flis too difficult to be handled here,
so we omit it. In Progress Test 1, however, we outline the proof for n = 2 and
f ( x ) = x.

PROGRESS TEST 1

1. Using that /la( - /bll 5 la - bl, prove that (c) Let k = v'&?? 6,+ < 6+ 6
so that k
limx-af(x) = L implies that limx+alf(x)l = lL1. < <
by (b). Use ( u ) to show that 0 1x - a / 6, implies
2. Using Prob. 1, prove that if lim,,of(x) = 0, then l f i-6 <
1 Ix - al/k.
limx+a lf(x)I = 0, and conversely. < <
( d ) Let 6, = Ek and show that 0 1x - a1 6, im-
3. Prove that limx-a & = 6 ( a 2 0) by following plies l f i - 61< E.
the given steps: ( e ) Use the results of (c) and ( d ) to conclude that
( a ) Show that & - 6 = (x - a ) / ( & + 6). limx+a = 6.
( b ) Given E> 0, suppose 6, = 4 2 . Show that
0 < <
Ix - a / 6, implies that 6 > v'&??.

COMPOSITION AND THE SQUEEZE THEOREM


The following important fact about composition of functions was stated as
(2.5.10) but was not proved.

9.2.2 If lim,,,g(x) = b and i f f i s continuous at 6 , then


limf(g(x)) = f limg(x) = f ( b )
x-a (*-a 1
Proof: We need to show that lim,,,f(g(x)) = f ( b ) , so let E 0 be given. We >
need to show that there is a 6 0 such that 0 > <
Ix - a1 6 implies <
If(g(x)) -f(b)I <
E . Sincefis continuous at b, we know that limt,J(t) =f(b)
by definition (2.5.7). Thus there is a 6, 0 such that>
It - bl < 6, implies l f ( t ) -f(b)l < E
SECTION 9.2: LIMITS AND CONTINUITY 333

Letting t = g(x), we conclude that

Thus we have shown that the necessary 6 exists, completing the proof. W

The last item on our unfinished-business agenda is the proof of the impor-
tant Squeeze Theorem, stated as (2.5.6) without proof and then used in the
proof of the Fundamental Theorem of the Calculus (6,3,1), and again in
showing that limAz+o(sin I x ) / A x = 0.

9.2.3 Squeeze T h e o r e m If h(x) 5 f ( x ) 5 g(x) for all x # a in some open interval


I containing a, and if limz+a h(x) = limE+ag(x) = L , then lim,+,f(x) = L.
Prooj Suppose E >
0. We need to show there is a 6 0 such that >
0 < < <
Ix - a1 6 implies / f ( x ) - L1 E . But since 1imz+, h ( x ) = L and
1imz+, g(x) = L, there are 6, 0 and 6, > >
0, respectively, such that
(i) O < /x- a1 < 6, implies lh(x) - L / < E

and
(ii) O < Ix - a1 < 6, implies / g ( x ) - Ll < E

Now for x # a but close enough to a so that x is in I , we know that


(iii) h(x) I f ( 4I
Choosing 6, small enough so that 0 <
Ix - a / <
6, implies that x is in I , we
know that 0 < 1x - a1 <
6, implies (iii).
Let 6 be the smallest of S,, a,, a, and assume 0 <
Ix - a1 6.Then (i) to <
(iii) all hold. Thus by (ii), g(x) - L <
E , by (iii), h ( x ) - L f(x)-
L 5 g ( x ) - L , and by (i), - E <
h ( x ) - L. Hence, putting these statements
together, we have
< h(x) - L If(.) - L 5 g ( x ) - L <
-E E

) . ( f i - LI < e , completing the proof.


from which we can conclude that W

SECTIOS 9.2 EXERCISES

1. Prove using the definition of limit that if limo-af(x) = L L < f ( x ) and M <
g(x). (c) Relate all four rectangular
and lim,-a g ( x ) = M , then IimE+, [ f ( x ) - g ( x ) ] = pieces to the proof of [9.2.l(b)]. ( d ) Shade all but the L
L -M. by M part of the rectangle and describe what happens
2. Prove using the definition of limit that iff(.) = b, a geometrically as x + a.
constant function, and limz+ag(x) = M , then 6. Use a geometric approach analogous to that in Exercise
lirnz-, [ b g ( x ) ] = bM. 5 to illustrate the argument requested in Exercise 2.
3. Prove, using the definition of right-hand limit, that if 7 . Prove that if 0 < f ( x ) 5 g(x) for all x # a i n a n open
limr-a+f(x) = L and limE-a-g(x) = M , then interval I containing a, and if lim,,,g(x) = 0, then
+
1imz-,t [ f ( x ) g(x)] = L + M. lim,,,f(x) = 0.
8. Prove that if 0 < f ( x ) 5 g(x) for all x # a i n a n open
4. Repeat Exercise 3 for limZ.+,-,
5 . Illustrate the proof of the second part of the Limit interval I containing a, and if lim,+,f(x) >0, then
Theorem, [9.2.l(b)], geometrically as follows: ( a ) Draw >
limz+.a g(x) 0.
anf(x) by g ( x ) rectangle. ( b ) Inside this rectangle and
based on one corner, draw a n L by M rectangle, where
334 CHAPTER 9: IMPORTANT IDEAS REVISITED

Exercises 9 to 21 involve the application and elaboration of dejnitions 0s injnite limits (4.1.1) and Yimits at injnity ( 4 . 1 . 4
9. Prove that lim,,,+ l / ( x - 2) = + 30 using Definition 18. Prove that if lim,,,f(x) = 0 and f ( x ) 0 for x in <
(4.1.1) as follows: Let M > 0 be given. Show that there is ( -Sl, +a,) >
for some 8, 0, then lim,,a l/f(x) = - oc.
>
a S 0 such that x - 2 < 6 implies l/(x - 2) > M . 19. Using an argument analogous to that for [9.2.l(a)],prove
(Name such a S in terms of M . ) that if limr,+,f(x) = A and lim,-+,g(x) = B , then
10. Prove that lim,,,, 2/(x - 3) = + oc using (4.1.1) as in lim,,,, +
[f(.) &)I = A B . +
Exercise 9. 20. Using an argument analogous to that used in the proof of
+
11. Prove that lim,,3 5 / ( x - 3), = 03 using (4.1.1). [9.2.l(b)], prove that if lim,,,, g(x) = k # 0, then
12. Provethatlim,,,- (x + 2)/(x2 - 4) = -musing(4.1.1). 1 =-1
13. Prove that lirn,,,, l/(x - 5) = 0 using Qefinition lim -
g(x) k
(4.1.2) a8 follows: Let E > 0 be given. Show that there is
a-+r

an M >
0 such that x > M implies jl/(x - 5)l <
E. (M 21. Using a n argument analogous to the proof of the Squeeze
is given in terms of E . ) Theorem, prove the following using Definition (4.1.2): If
14. Prove that lirn,,,, 2/(x - 3) = 0 using (4.1.2) as in >
h(x) < f ( x ) 5 g(x) for all x &for some number& 0, >
Exercise 13. and lirn,,,, h ( x ) = L and lirn,,,, g(x) = L , then
15. Prove that lirn,,,, 4/(x - 2), = 0 using (4.1.2). lim,,+,f(x) = L.
16. Prove that limz,-, 2/(x + 3), = 0 using (4.1.2).
17. Prove that if lim,-u If(x)l = 0, then lim,,u l/lf(x)l =
+ co.

Section 9.3: Why All the Fuss?

Objective: INTRODUCTION
Appreciate the necessity for careful When confronted with complex-appearing definitions or detailed proofs of
mathematical statements in theorems that, on the surface, seem obvious, many students have legitimate
defining and evaluating limits and doubts about their usefulness. This is especially true because often these
i n proving theorems . definitions and theorems are given when the sorts of functions and situations
that would render simpler statements inadequate are not yet available for
exploration. We shall now briefly examine a few functions whose behavior
illustrates the need for care in stating definitions of limit and infinite limit and
for care in dealing with derivative quotients and the integrability of functions.
Functions such as these literallyforced mathematicians of the nineteenth century
into the definitions we have been using.
For example, letf(x) = sin( 1/x) for x >
0, h(x) = (1/x) sin( 1/x) for x 0. >
(See Fig. 9.9.) We first examine lim,,o+f(x). As x + 0+, l / x + co, so +
+
sin(l/x) oscillates between 1 and - 1 as many times in, say the interval (0,l ) ,
+
as sin x oscillates between 1 and - 1 in the interval (1, a). +
Notice that we
can make sin(I/x) as close as we please to 0 by choosing x su8ciently close to 0. However,
we shall show that lim,,o+ sin( 1/x) # 0: Given any small E >
0, say E 1, and <
>
any 6 0, there are some values of x in (0,6)for which sin(l/x) is within E of
0 and others for which sin(l/x) is not withiq E of 0. In fact, whenever l/x is an
+
odd multiple of a / 2 , sin(l/x) is either 1 or - 1 and hence outside ( - E , E ) . No
>
matter how small 6 0, we can find integers k large enough so that

(Note that odd integers are those integers of the form 2k + 1, where k is an
integer.) For
1
x=
(2k 1)(77/2) +
l/x = (2k + l)(a/2), so sin(l/x) = ? 1.
SECTION 9.3: WHY ALL THE FUSS? 335

1
f ( x 1 = sin -

(a)

Fig. 9.9

The lesson of this discussion is that the loose paraphrase just emphasized
does not suffice as a limit statement.

We can use h (x) = (l/x) sin( l/x) to make a similar point about infinite limits.
By choosing x suiciently close to 0, we can make h ( x ) = ( I / x ) (sin I / x ) as large as we
please. That is, given any M > 0, we can find S > >
0 such that h ( x ) M for some
x in (0, S). (See Sec. Exercise 1.) However, no matter how small we choose S,
<
there will be other values of x in (0, 6)for which h(x) M . [See Fig. 9.10(b) and Sec.
Exercise 2.1 In fact, we can find x in (0, 6) for which h ( x ) = (l/x) sin(l/x) is an
arbitrarily large negative number.
Hence lim,,,+ (l/x) sin(l/x) # + 00 and lim,,,+ (l/x)(sin l/x) # - 00.

THE CHAIN RULE


Recall the formal statement of the Chain Rule:

9.3.1 If u is a differentiable function for all numbers in an open interval containing x,


and g is a differentiable function of u in an open interval containing u(x), then
the derivative of g (u (x)) is (dgldu) (duldx)

In Sec. 3.2 we gave a partial proof of (9.3.1) by introducing the expression


(Ag/Au) (Su/Ax), where Ax is a small change in x, Au the corresponding change
in u, and Ag the resulting change in g. We assumed, because of the Au in the
denominator above, that Au # 0. However, for

the statement Su # 0 is not true near x = 0. (See Fig. 9.10 and Exercise 3.) Now
u ( x ) is everywhere differentiable (see Exercise 4), but Au = u(Ax) = 0 whenever
l/Ax is an integral multiple of T [which is the case for Ax = l / ( k T ) , k an
integer.] A different proof is required to cover functions such as this partic-
ular u ( x ) .
336 CHAPTER 9: IMPORTANT IDEAS REVISITED

Fig. 9.10

Proof of Chain Rule: [For general g and u, as in (9.3.1.)]


Define Q = Ag/Au - dg/du for Au # 0 and define Q = 0 for those Ax
making Au = 0. Since
lim Au = 0 ( u is continuous)
Ax-0

and since g is differentiable at u(x),


lim Q = lim Q = 0
Au-0 AX-0

Rearranging the equation defining Q, we get a n equation that is true for all AX,
including those for which Au = 0:

Ag = -AIL
dg
du
+ AU Q
Then upon dividing this equation by Ax and applying the limit theorem, we
get
dg
- AU + AU Q
Ag . du
lim - = lirn
Ar-o AX AX-O Ax
dg Au Au
= lim -- + lim -Q
AX-o du Ax AX-o Ax
dg du du
- -- + - S O
- (lim Q = 0)
du dx dx AX-0

A DEFINITE INTEGRAL
The function

being differentiable, is also continuous. However, in the terminology of Chap. 6,


it is infinitely wiggly; that is, there is no closed interval [a, b ] containing 0 on
SECTION 9.3: WHY ALL THE FUSS? 337

which u(x) is monotonic. Recall that in our analysis of area and the definite
integral we demonstrated that S i f ( x ) dx exists for continuous functions that are
piecewise monotonic on [a, b ] , but we had to assume that a general (possibly
infinitely wiggly) continuous function is integrable. Hence u(x) is a function
to which our earlier analysis did not apply and for which the more general
assumption was necessary.

SECTION 9.3 EXERCISES


-

1. Show that given any M > >


0, there is a 6 0 such that 4. ( a ) Show that ~ ( x (as) in Exercise 3) is differentiable for
h(x) = (l/x) sin (l/x) >
M for some x in (0,s). x # 0 by applying the derivative formulas and examining
2. For Exercise 1 show that, for some x in (0,S), h ( x ) <
-M. the results.
3. Show that for ( b ) Show that u ( x ) is differentiable at x = 0 by applying
the definition of derivative at x = 0.
sin (l/x) for x # 0 (c) Is u(x) continuous at x = O? Explain.
u(x) =
for x = 0
5. By considering the function u(x) (as in Exercise 3) show
that the following does not suffice as a paraphrase of
>
and for any 8 0, there is a A x in (0,s)with A u = lim,,,, U(X) = +co: We can make U(X)arbitrarily large
u(0+ Ax) - u ( 0 ) = u ( A x ) = 0. by choosing x sufficiently large.

Miscellaneous Exercises

1. State and prove a uniqueness theorem for limits at in- 5. ( a ) Show that
finity.
2. Assume thatf(x) 2 0 on ( a , 6) and a c< <
b. Prove that
if lim,,,f(x) exists and equals L , then L 5 0.
3. Suppose
if x is rational has a continuous first derivative at x = 0.
( b ) Show thatf(0) does not exist.
6. Let [ u ] denote the greatest integer less than u. Sketch
Is f continuous at x = O? (Note: every open interval con-
f ( x ) = [ l / x ] and prove that lim,,,,+f(x) = + crc.
tains both rational and irrational numbers.)
7 . Let g(x) = l/f(x), wheref(x) is as in Exercise 6. ( a ) De-
4. Suppose
termine if lim,,og(x) exists. ( b ) Determine if g(x) is
if x is rational continuous on any interval containing 0 when we define
g(0) = 0.
Is g continuous at x = O?

SELF-TEST .).
Chapter 10 %
340 CHAPTER 10: A BRIEF REVIEW OF ANALYTIC GEOMETRY

Section 10.1 : Translation of Axes and the Parabola

Objectires: TRANSLATION OF AXES


1. Identify and sketch equations We can often simplify the process of graphing an equation in x and y by
whose graphs are parabolas with introducing a new coordinate system. We refer to this process as a transformation
axes parallel to one of the of coordinates. If the new coordinate system is an X Y system, then the effect of
coordinate axes, using translation transforming the coordinates is to transform the original equation in x and y
of axes if necessary. to a new, in general simpler, equation in X and Y We do this in such a way that
the equation changes but the graph does not. The easiest transformation of
2. Determine the focus, vertex,
coordinates to work with is called a translation of axes. In this case the X Y -
directrix, and axis of a parabola
coordinate system is obtained from the xy-coordinate system by a parallel
from its equation.
displacement as in Fig. 10.1. In effect, we slide a new coordinate system under
3. Determine the equation of a the graph.
parabola from geometric Let us assume that the origin of the X Y system is located at the point with xy
information that uniquely coordinates (h, k ) . Now any point P in the plane has two sets of coordinates (x,y)
determines a parabola. and ( X , Y ) . From Fig. 10.1 it is not difficult to see that these coordinates are
related by the equations:

10.1.1 x=X+h y=Y+k


or, equivalently,

10.1.2 X=x-h Y=y-k


Given the X Y coordinates we can use (10.1.1) to obtain the xy coordinates. To
go the other way we use (10.1.2).

Thus the graph is a circle of radius 4 and center (3. -4). Now let X =x -3
A
-----t P i >J
x.Y, and Y = y +4, and the equation becomes
X 2 + Y 2 = 16
h Y
J = k h ? I
0 >I *x which in the X Y system is a circle of radius 4 centered at the oragzn.
Y=O k
e x By (10.1.2) the substitution X = x - 3 , Y = y +4 is equivalent to translat-
ing the origin of the xy-coordinate system to the point (3, -4). We now sketch
SECTION 10.1: TRANSLATION OF AXES AND THE PARABOLA 341

Y Y

t t

(b) (Cj

Fig. 10.3
Fig. 10.2

THE CONIC SECTIONS


The conic sections consist of ellipses, parabolas, and hyperbolas. They are so
named because they can be realized as the intersection of a plane with a right
circular cone. (See Fig. 10.3.)
In Fig. 10.3(a) a plane parallel to an element of the cone (an element is
a straight line on the cone passing through the vertex) intersects the cone in a
parabola. In Fig. 10.3(b) a plane parallel to the axis of the cone intersects the
cone in a hyperbola (degenerate if the plane contains the axis). In Fig. 10.3(c) a
plane not parallel to either the axis or an element intersects the cone in a n
ellipse (a circle if the plane is perpendicular to the axis).

THE PARABOLA
10.1.3 Definition A parabola is the set of all points in a plane that are equidistant
from a fixed point F and a fixed line L. The point F is called thefocus and the
line L is called the dzrectrzx. In addition, the line L through the focus and
perpendicular to the directrix is called the axzs of the parabola. The point V on
the axis midway between the focus and the directrix is called the vertex. (See
Fig. 10.4.)

By definition a point P is on the parabola if and only if lPAl = IPFI, where A


is the base of the perpendicular from P to L. The vertex, being midway between
the focus and the directrix, is on the parabola. Furthermore. if P is the
reflection of P through L, then from Fig. 10.4 it is clear that JPFI = /PA/,so
the parabola is symmetric with respect to its axis.
If a parabola is positioned so that the vertex is at the origin and the axis
coincides with one of the coordinate axes, we say the parabola is in a standard
posztzon. There are four possible standard positions for a given parabola, de-
pending upon whether the focus is located to the right or left of the vertex on
Fig. 10.4 the x axis or above or below the vertex on they axis.

Equations of a Parabola We shall derive an equation of the parabola in


one of the standard positions. It will then be an easy task to derive equations for
the parabola in any of the other standard positions.
>
Let 2p represent the distance ( p 0) from the directrix to the vertex and
assume that the axis of the parabola is the x axis with the focus to the right of
++-
342 CHAPTER 10: A BRIEF REVIEW OF ANALYTIC GEOMETRY

V = (0,0; F = p, 0,
x = -p
Fig. 10.5

J, the vertex. (See Fig. 10.5.) It follows that F = ( p , 0) and the directrix L has
4 equation x = - p .
b----J--,* ,*,,,'l P =
"JY' (X.J) Let P = (x,y).Then P is on the parabola if and only if IAP/ = IPFI, where
A = ( - p , y ) ; see Fig. 10.6. Now
(API = d(x + p ) 2 + ( y -y)2 = 12 + PI
and
IPFI = d(x - p ) 2 + (3 - 0)2 = d(x - p ) 2 +y2
Thus P = ( x , y )is on the parabola if and only if
d(x - p ) 2 +y2 = Ix + pl
or, equivalently,
Fig. 10.6
(x - p ) 2 +y 2 = (x + p)2

Squaring the terms ( x - p ) 2 and ( x + p ) 2 and simplifying, we obtain the


equivalent equationy2 = 4px.
Thus we have proved the following:

10.1.4 Theorem If p > 0, then a necessary and sufficient condition for the point
( x , y ) to be on the parabola with focus ( p , O ) and directrix x = - p is that
y2 = 4px

Example 2 Describe the curvey2 = 3x.


Solution Here 3 = 4 p , so p = 3 / 4 and y 2 = 3x = 4 ( 3 / 4 ) x . Thusy2 = 3x is a parabola
with vertex ( O , O ) , focus ( 3 / 4 , 0 ) and directrix x = - 3 / 4 . The axis of the
parabola is the x axis. (See Fig. 10.7.) 0

If in our development above we had placed the focus at ( - p , 0) and taken


the line x = p as the directrix, then we would have in effect replaced p by - p .
This would lead to the equation
y 2 = -4px
The axis of this parabola is again the x axis, but now the parabola opens to the
J left.

If we interchange the roles of x a n d y in the previous discussions, we obtain


the equation x 2 = 4py for the parabola with focus (0, p ) and directrixy = - p .
F Similarly we obtain the equatiop x 2 = -4py for the parabola with focus
01 (0,- p ) and directrix y = @. We summarize these results in Table 10.1.
By a translation of axes we can derive the equations for a parabola with a
vertical or horizontal axis. For example, suppose the parabola has its vertex at
+
V = ( h , k ) and has the liney = k 8s its axis. If the focus is at ( h p , k ) , then the
directrix will be x = h - p . Now let X = x - h , Y = y - k. In the XY-coordi-
Fig. 10.7 nate system V = (0,O) and the axis is the X axis. Since in this coordinate system
SECTION 10.1: TRANSLATION OF AXES AND THE PARABOLA 343

I
1 Equation Vertex Focus Directrix Axis Sketch
~
I
~

I p
x axis

x2 = 4py ~ (0,O) y axis


P>O I

I I

Table 1 0 . 1 Parabolas in Standard Form

the focus is at ( p , 0), we can apply Theorem (10.1.4) to conclude that the
parabola has the equation
x2 = 4pY
Thus in the original 9-coordinate system, the equation becomes
(x - h)2 = 4p(y - k )

Example 3 Identify and sketch the graph of the equation x 2 - lox - 8y + 53 = 0.


Solution We begin by putting the x terms on one side and they term and constant on the
other:
x2 - lox = 8y - 53
Complete the square in x :
+
x2 - 1 0 ~ 25 = 8y - 53 + 25 = 8y - 28
or
344 CHAPTER 10: A BRIEF REVIEW OF ANALYTIC GEOMETRY

jjX
Fig. 10.8

This is of the form X 2 = 4pY, which from Table 10.1 has its vertex a t the
origin of the XY-coordinate system and opens upward. The focus is located p
units up on the Y axis, and the directrix is the line Y = - p . Since X = x - 5 ,
Y = y - 7 / 2 , and p = 2 , in the original xy system we have a parabola with
vertex (5, 7 / 2 ) , focus (5, 1 l/Z ), axis x = 5, and directrix y = 3 / 2 . (See
Fig. 10.8.) 8

A reverse version of Example 3 might ask for the equation of the parabola
given that its focus was located at (5, 1 1 / 2 ) and that its directrix had equation
y = 3/2.

PROGRESS TEST 1

In Probs. 1 to 3 determine the vertex, focus, and directrix 4. Find a n equation for the parabola with vertex at
of the parabolas with the given equations and sketch. (4, - 3 ) and focus at (4,O) and sketch.
1. y = 2x2 2. 4 x = y 2
3. y 2 - 4y + 8 x +
28 = 0

PA4RABOLICREFLECTORS
7 Let Po = (xo,yo)be a point unequal to the vertex on the parabolay2 = 4px and
let T denote the line tangent to the parabola at Po. Let N denote the angle
~ /;\ between T and the line through Po parallel to the x axis. (See Fig. 10.9.) Finally,
let ,8 denote the angle PoFmakes with T. We shall show that N = p, from which
we can deduce the reflective properties that make the parabolic shape so useful
(X 0 .v
in applications.
c(
A0
X Now (Y is also the angle between T and the x axis. Since dy/dx = Zp/y, T has
9. F slope 2p/yo and equation y - y o = (2p/yo)(x - xo). Letting y = 0, we have
Fig. 10.9 2
2P
-yo = -(x - xo) or -yo + xo = x
-
Yo 2P
But (xo,yo) is o n y 2 = 4px, so

x = - -4Pxo + xo = -xo
2P
Thus Q has coordinates ( -xo, 0). Now
SECTION 10.1: TRANSLATION OF AXES AND THE PARABOLA 345

\
Light source at focus

Fig. 10.10

and
PoFl = d ( x 0 - PI2 +YO2
+ +
= dX"2 - 2px,.~ p 2 Yo2
+
= d x o 2 2pxo p 2 + since yo2 = ~ P X ,
+
= d(X0 PI2 = xo p +
Therefore triangle FQPo is isosceles, with equal base angles; that is, a = p.
Now, if a parabola is revolved about its axis, a surface, called aparaboloid, is
generated whose cross section parallel to the axis is a parabola. When light is
reflected from this surface, the angle of incidence equals the angle of reflection,
so if a light source is placed at the focus of the parabola generating the
paraboloid, then light rays will be reflected outward in rays parallel to the axis,
providing a directed beam of light. (See Fig. 10.10.)
The same principle is used in the design of a reflecting telescope, except that
now the parabolic surface causes parallel rays coming in to concentrate at the
focus. The only difference is the direction of the light rays. (See Fig, 10.11.)

SECTIOS 1 EXERCISES

In Exercises I to 12, ( a ) determine the uertex, focus, and directrix, and ( b ) sketch the parabola, zdenttfying these items in the sketch.
1. y2 = 20x 2. 3y2- 18x = 0
3. y + x = o 4. 2 - 2y=o
5. 5 x 2 - 400y = 0 6. x z - 2x + y = 0
7. 3y2 = 4x 8. y 2 - 6~ + 3~ = 0
9. x2 +l o x - 3y = -1 10. 3x2 + 12x - 5 + y = 0
11. y2 - 5y - 3 = lox 12. 4x2 - 12x - y = - 3

In Exercises I3 to 22 determine an equatzon of the parabola zn standard position with uertex at (0, 0 ) and.
13. Focus at (4,O) 14. Focus at ( - 6 , O )
15. Directrix x = -10 16. Directrixy = 5
17. Focus at ( 0 , -8) 18. Focus at ( 0 , 7)
19. Passing through (4, -2), opening downward 22. Passing through (-2, -3); two parabolas are possi-
20. Passing through (2. 3), opening upward ble-give each
2 1. Passing through (3, 5); two parabolas are possible-give
each

In Exercises 2 3 to 30 ( a ) determine an equation of the indicated parabola, and ( b ) sketch the parabola.
23. Vertex at (5, l ) , focus at (5, 3) 24. Vertex at (2, -3), focus at (2, 1)
25. Vertex at (1,4), focus at (1, 6) 26. Focus at ( - 1, - l ) , directrix x = - 3
27. Focus at (4, 6), directrixy = 8 28. Vertex at (8, 2), directrixy = 4
29. Vertex at (-6, 4), directrix x = 12 30. Focus at (5,8), directrixy = - 2
346 CHAPTER 10: A BRIEF REVIEW OF ANALYTIC GEOMETRY

Section 10.2: The Ellipse and the Hyperbola

Objectives: THE ELLIPSE


1. Identify and sketch equations 10.2.1 Definition An ellz$se is the set of all points in a plane the sum of
whose graphs are ellipses or whose distances from two fixed points, Fl and F,, is constant. (See Fig. 10.12.)
hyperbolas with axes parallel to the The points Fl and F , are called the foci of the ellipse. The straight line through
coordinate axes. Fl and F, is called the major axis of the ellipse, and the perpendicular bisector of
the line segment FIFz is called the minor axis. The intersection of the major and
2. Determine the foci, vertices, and
minor axes is called the center of the ellipse.
major and minor axes of an ellipse
from its equation.
Let P be an arbitrary point on the ellipse. We denote the sum of the distances
3. Determine the foci, vertices, IPFII and IPF,I by 2a. Thus P is on the ellipse if and only if
transverse and conjugate axes, and
asymptotes of a hyperbola from its IPFII + /PFz)= 2a
equation. If we flip the triangle F,PF, (see Fig. 10.13) about its base FIF,, we see that P,
4. Determine the equation of an
the reflection of P through the major axis, is also on the ellipse. By turning the
ellipse or hyperbola from given
triangle over, we see that P, the reflection of P through the minor axis, is also
information.
on the ellipse. Thus a n ellipse is symmetric with respect to both of its axes.
The definition can be used to draw an ellipse. Attach the ends of a string of
length 2a to the foci. If the tip of a pencil is used to hold the string taut, it will
trace out the ellipse as it is moved around since clearly the sum of the distances
from the tip to the points Fl and F, will always equal 2a. (See Fig. 10.13.)
With respect to a given xy-coordinate system, the ellipse is said to be in
standardposition if the center is at the origin and the foci are on either the x or
y axis.
To obtain an equation of an ellipse in one of the two standard positions, we
\ P let 2c denote the distance FIF, and we place the foci at (- c, 0) and (c, 0), so the
major axis is the x axis.
Note from Fig. 10.14 that 2c < 2a since 2c is the length of the base of triangle
F,PF, and 2a = IPFII + IPF,I is the sum of the lengths of the other two sides. In
<
particular, c a. Now P = (x,y) is on the ellipse if and only if
IPFII + lPFzl = 2a
Using the distance formula, we see that this equation is equivalent to
d(2 + c)2 + y2 + d(x - + y2 = 2a
c)2

\ \ Minor axis
Now if we square both sides, rearrange, and square again, we obtain

Fig. 1 0 . 1 2

Fig. 1 0 . 1 3 Fig. 1 0 . 1 4
SECTION 10.2: THE ELLIPSE AND THE HYPERBOLA 347

or

Now a 2 - c2 > 0 since a > c. Let b2 = a2 - c2 and divic.. both sides by b2


x2 y2
-+-=1
a2 b2
This shows that if P is on the ellipse, then the coordinates of P will satisfy (1).
Unlike the parabolic case, it does not follow immediately that each previous
step is reversible:. The content of Exercise 24 is to show that each point (x,y)
which satisfies (1) is in turn on the ellipse.
Thus we have proved the following:

10.2.2 Theorem Let 8 be an ellipse with foci located at ( - c , 0) and (c, 0) (c 0) >
and let 2a ( a > c) denote the sum of the distances from any point ( x , y ) on the
ellipse to the foci. Then E has equation

From the equation it is algebraically clear that the ellipse is symmetric with
respect to the x a n d y axes and the origin. Ify = 0, then x = * a . The points
( - a , 0), (a, 0), where the ellipse intersects its major axis, are called the vertices of
the ellipse. The distance 2a between the vertices is called the length of the major
axis. The ellipse intersects the minor axis at (0, - b ) and (0, b). The distance 26
between these points is called the length of the minor axzs.
Since c # 0 and b2 = a2 - c2, a is always larger than b. That is, the ellipse is
elongated in the direction of its major axis.

Example 1 Determine an equation for the ellipse with foci at (-3,0), (3,O) and a = 7.
Solution Since a = 7 and c = 3, we have b2 = a2 - c2 = 49 - 9 = 40, so b = 2m.
Thus the ellipse (see Fig. 10.15) has equation
4

t (0,2
4101
x2

49
y2
- + - = l o
40

If in our development we had placed the foci at ( 0 , -c) and (0, c), then in
effect we would have interchanged the roles of x andy, leading to the equation
x2 y2
-+ - = I
b2 a2

Again, the major axis (now they axis) would have length 2a, and the minor axis
( x axis) would have length 2b. The vertices would be the points (0, i a ) on they
axis. Since a > b, we can recognize from the equations which is the major axis
Fig. 10.15 by following these steps:
1. Write the equation in the form

2. If ( ) [ > 1, then ( ) = a and the x axis is the major axis.


3. If [ ] (> ), then [ ] = a and they axis is the major axis.
We summarize in Table 10.2.
348 CHAPTER 10: A BRIEF REVIEW OF ANALYTIC GEOMETRY

Equation '
I
Major
Axis ~

I
Minor
Axis ~

I
Foci '
I
Vertices Sketch
1

I lo. -51
Fig. 10.16 Table 10.2 Ellipses in Standard Form (n > 6, 8' = a
' - cl)

Example 2 Sketch the equation 25x2 + 16y2 = 400, labeling foci and vertices.
Solution Divide by 400. Then
25x2 16y2 x2
-+-
400 400
=1 or -+-=1
16
y2
25
Since 25 > 16, a2 = 25 and the major axis is they axis. Now b2 = 16 and so
c2 = 25 - 16 = 9. Thus the foci are at (0, -3) and (0, 3); see Fig. 10.16. 0

As with the parabola, it is easy to obtain any translation of an ellipse for


which the axes are horizontal and vertical by separately completing the square
in x and y .

PROGRESS TEST 1

1. Determine the equation of the ellipse with center at 3. Determine the center, vertices, and foci, and then
(0, 0), a focus at (4,0), and major axis of length 8. sketch the ellipse with equation
2. Repeat Prob. 1, but translated so that the center is
at (8, -6). +
16x2 - 9 6 ~ 25y2 - 2 0 0 ~= -144.

REFLECTIVE PROPERTIES OF THE ELLIPSE


An ellipsoid is obtained by revolving an ellipse about its major axis. Like the
paraboloid, the ellipsoid also has a n interesting reflective property. In this case
a light or sound source emanating from one focus will be reflected to the other
focus. (See Fig. 10.17.) We omit its development.
4 F2

-C, OJ IC, 0
Fig. 10.17 THE HYPERBOLA
10.2.3 Definition A hyperbola is the set of all points in a plane such that the absolute
value of the difference of the distances from these points to two fixed points, Fl
and F2, is constant. The points Fl and F2 are called the focz of the hyperbola.
SECTION 10.2: THE ELLIPSE AND THE HYPERBOLA 349

(See Fig. 10.18.) The straight line through Fl and F2 is called the transverse axzs of
the hyperbola, and the perpendicular bisector of the segment FlF2 is called the
conjugate axzs. The point where the transverse and conjugate axes meet is called
the center of the hyperbola.

Let P be a point on the hyperbola and let 2a denote the absolute value of the
difference of the distances IPF,/ and /PF2(.Then P is on the hyperbola if and
only if
IPF,I - IPF2l = * 2 a
We shall derive an equation for the hyperbola in a standard position by
placing the center at the origin and the foci on the x axis at ( - c, 0) and (c, 0);
Fig. 10.18 see Fig. 10.18. Now
lPF2/ - /PF,/ = k 2 a
so
d(x - c ) 2 + y 2 - d(x + c)2 + y 2 = t 2 a
Proceeding as with the ellipse, we rearrange, square, simplify, and then square
again to get
x2
---=I y2
(1) a2 c2 - a2
Since the sum of the lengths of two sides of a triangle exceeds the length of
the third side, we observe from Fig. 10.19 that
X

26 + IPF,I > IPF21 and 2c + 1PF21> IPF,/


or
2c > /PF2/- IPFII and 2c > /PF,/ - JPF2/
These latter two inequalities imply

Fig. 10.19
I
2c > lIPF,I - IPF2/l = 2a

>
and hence c a. In particular, c 2 - a2 > 0. Thus, letting b 2 = c 2 - a2,we can
rewrite (1) as

As with the ellipse the steps used in deriving (2) are reversible. Lettingy = 0
in (2), we see that x = t a and thus the hyperbola intersects its transverse axis
at (-+a, 0). These points are called the vertices of the hyperbola.
In summary, we have proved the following:

10.2.4 > >


Theorem Let c a 0. The hyperbola with foci ( k c , 0) and vertices
(&a, 0) has the equation

The transverse axis of this hyperbola is the x axis, the conjugate axis is they
axis, and the center is at the origin.

If in the previous development the vertices were on they axis at (0, &a) and
the foci at (0, k c ) , then essentially the same computations would yield

where again b2 = c 2 - a2.


350 CHAPTER 10: A BRIEF REVIEW OF ANALYTIC GEOMETRY

Transverse axis = x axis (length = 2a)


Conjugate axis = p axis (length = 26)

Center = (0, 0)
i a2 + b2 = c E

I I I I
\ I I

Note that, unlike the ellipse, the relative sizes of a and b are not instrumental
in determining the axes. However, the quantity appearing below the variable with the
minus sign is always taken to be b2.
The distance 2a between the vertices is called the length of the transverse axis.
The number 2b is called the length ofthe conjugate axis. Of course it is clear from
the equations that a hyperbola does not intersect its conjugate axis. It is equally
clear that in either of the standard positions the hyperbola is symmetric with
respect to the x a n d y axes and with respect to the origin.
Table 10.3 gives a summary of the pertinent information and the appropri-
ate sketch.

Example 3 Identify and sketch the graph of the equationy2 - 3 x 2 = 9, giving all relevant
J
information.
We divide by 9 to obtain
X2
y"--
9 3
=1
Thusa2=9,b2=3,andc2=a2+b2= 12,andsoa=3,b= fi,c=2fi.
Since the equation has the form of (ii) in Table 10.3, the transverse axis is they
axis and the conjugate axis is the x axis, The foci are at (0, 5 2 f i ) and the
vertices are at (0, k 3 ) ; see Fig. 10.20. 0

ASYMPTOTES AND THE CENTRAL RECTASGLE


The central rectangle of the hyperbola
x2
--- = 1
y2
Fig. 10.20 a2 b2
SECTION 10.2: THE ELLIPSE AND THE HYPERBOLA 351

y = --x
b
f

Fig. 1 0 . 2 1

is obtained by drawing parallel lines through the vertices ( t a ,0) and parallel
lines through the points ( 0 , t b ) on the conjugate axis. (See Fig. 10.21.) The
diagonals of the central rectangle lie along the lines y = t ( b / a ) x .
Y
Solving for y in terms of x , we have
h

Now

= lim -ab =o
x-+w dPT7+x
Similarly, one shows

t
Thus y = +(b/a)x are oblique asymptotes to the graph of
y = ( b / a ) d p , the upper half of the hyperbola x 2 / a 2 - y 2 / b 2 = 1; see
X Fig. 10.22(a). Analogously, y = t ( b / a ) x are oblique asymptotes of
y = ( - b / a ) v-', the lower half of the hyperbola x 2 / a 2 - y2/b2 = 1; see
Fig. 10.22(b).
For the hyperbolay2/a2 - x2/b2 = 1 a similar argument shows that the lines
y = t ( a / b ) x are oblique asymptotes.
As an aid in sketching the graph of a hyperbola it is a good idea first to
(b! sketch the central rectangle and then use the extensions of the diagonals as the
Fig. 10.22 oblique asymptotes.

Example 4 Sketch the hyperbola


16x2 - 9y2 = 144
Solution Dividing by 144, we have

Thus a 2 = 9, b2 = 16, and c2 = 9 +16 = 25. The transverse axis is the x axis
with the foci at (t5,0) and the vertices at ( t 3 , O ) . The conjugate ( y )axis has
352 CHAPTER 10: A BRIEF REVIEW OF ANALYTIC GEOMETRY

length 2b = 8. Hence the oblique asymptotes a r e y = +(4/3)x, and we can


sketch the hyperbola. (See Fig. 10.23.) 0

If a hyperbola in standard position is translated so that its center is at the


point (h, k ) , then, by separately completing the square of the x andy terms, any
equation for such a hyperbola can be put in one of the forms
(x - hI2 (Y - N2 - ( y - kI2 - =1
-- -1 or
b2
a2 b2 a2

Fig. 10.23

PROGRESS TEST 2

Determine the central rectangle, foci, and vertices and


then sketch the hyperbola 9 a x 2 - 36x - 4y2 - 8y = 112.

SECTION 1 0.2 EXERCISES

In Exercises I to I 6 sketch the graph of the given equation, labeling the center, foci, and certices.
1. 9x2 + 25y2 - 18x - 150y 9 =0+ 2. 16x2 - 9y2 - 160x - 72y = -112
3. 25x2 + 169~' + lOOx - 1352y = 69 4. x2 + 4y2 - 4 = 0
5. 4x2 +y2 - 4 = 0 6. +
x 2 4y2 - 6x +
5 =0
7. +
9x2 4y2 + 18x - 8y + 4 = 0 8. +
x 2 + 4y2 + 2~ 4 0 ~= -97
9. 3x2 + y 2 + 24x = - 9 10. 4y2 - 9x2 + 16y +
18x = 29
11. 9 x 2 - y z - 36x + 8y = 5 12. - 4 ~ ' + 9y2 - 1 6 -
~ 18y = 43
13. 9y2 - x 2 - 36y + 2x = -26 14. 25x2 + 4y2 - 5 0 - ~ 1% = 59
15. x 2 + y 2 - $ + l o x = -18 16. x2 + 2x -y2 - 4y 4 = 0)+
In Exercises I7 to 20 determine the equation of the ellipse satisfying the given conditions.
17. Vertices (i5,0),center (0, 0), b = 4 19. One focus a t ( 6 , 6 ) , b = 2, center ( 4 , 6 )
18. Center (0, 0), one vertex (10,0), a = 3 20. Foci ( 3 , 2 ) , ( 9 , 2), a = 4

In Exercises 21 to 24 determine the equation of the hyperbola satisfyzng the given conditions.
21. Vertices (5, 3 ) , ( 1 , 3), c = 2, b = 2 24. Foci ( 1 , 2), ( 1 , 6 ) , a = 1.
22. Center ( 0 , 4 ) , foci o n y axis a = 4 , c = 5 25. Show that each point ( x , y ) that satisfies
23. Center (1, 2 ) , foci (4, 2 ) , ( - 2 , 2 ) , b = 2 +
x2/a2 y 2 / b 2 = 1 is on the ellipse with foci ( k c , 0).

Rotation of Axes and


Section 10.3: the General Quadratic Equation

Objective: ROTATION OF AXES


Determine the graph of a general Any equation of a conic section with wesparallel to the coordinate axes can now be
second-degree equation by easily identified. If it is not in standard form, then we appropriately complete
performing an appropriate rotation, the square to reveal its graph as a translation of one of the standard forms. We
if necessary, to obtain an shall now develop the means for identifying the equation of a conic section
equivalent equation in standard whose axes are not parallel to the coordinate axes, a conic section that has been
form. rotated through a n angle a , as in Fig. 10.24, or rotated and translated, as in
SECTION 10.3: ROTATION OF AXES AND THE GENERAL QUADRATIC EQUATION 353

Y Fig. 10.25. We shall concentrate on a rotation of one of the standard forms,


since once this has been obtained, the translation can be handled as before.
Let us assume that the xy-coordinate system has been rotated through an
angle a to obtain an XY system. The rotation is counterclockwise if a >0 and
clockwise if a < 0. If P is any point in the plane, then P has coordinates (x,y) in
the xy system and coordinates (X, Y) in the XY system. (See Fig. 10.26.)
Given the coordinates in one system, we would like to derive the coordinates
in the other. If 6' measures the angle of inclination of the segment O P with the
+
positive X axis, then 6' a gives the angle of inclination of OP with the x axis.
From Fig. 10.26 we know that
(1) x = IOPlcos(6' + a ) y = IOPlsin(6' a)+
Fig. 10.24 and
(2) X = IOP~COSB Y = IOPlsin6'
Using trigonometric identities for cos(a + 6) and sin(a + b), we have
cos a - IOPlsin 6' sin a
x = JOPJCOSB
and
y = IOPJsinOcosa IOPJcosOsina +
From equations (2) we can thus conclude:

x = X c o s a - Ysina
10.3.1
y = X s i n a + Ycosa

Now if a rotation of the v system through an angle a is used to obtain the X Y


system, then a rotation of the XY system through an angle -a will yield the xy
Fig. 10.25
system. Thus, interchanging x with X,y with Y, and replacing a by -a in
(10.3.1), we obtain
X = x cos( - a ) - y sin( - a )
Y = x sin( - a ) + y cos( - a )
But cos( - a ) = cos a and sin( - a ) = -sin a , so we have derived the following
equations that enable us to obtain the XY coordinates when given the xy
coordinates:

X = X C O S+ y~ s i n a
10.3.2
Y = -xsina + y c o s a
Fig. 10.26

Example 1 Identify the graph of


x2 + 2fiv -y2 = 4
by deriving its equation in the XY system obtained by rotating the xy system
through an angle of m/6.
Solution Since cos 77/6 = f i / 2 and sin m/6 = 1/2 we have from (10.3.1) that

Thus in the X Y system the equation


x2 + 2 f i . 9 -y2 = 4
becomes the equation
354 CHAPTER 10: A BRIEF REVIEW OF ANALYTIC GEOMETRY

Y which, if we multiply out and simplify, reduces to


Y 4

This is a hyperbola in the XY-coordinate system with a2 = b 2 = 2 and c2 =


a 2 + b2 = 4. Thus the foci are a t ( t 2 , O ) and the vertices are at ( t
fi,0). The
transverse axis is the X axis and the conjugate axis is the Y axis.
Using (10.3.1) with X = 2 2 , Y = 0, we get that the foci in the xy system are
located at ( fi,1) and (- fi,- 1). The vertices are located at ( $/2, */2)
and ( - f i / 2 , - */2). Since the transverse axis is the X axis ( Y = 0), and
since Y = ( - 1/ f i ) x + ( fi/2)y in the xy system, the transverse axis is the
straight line x = f i y . Similarly, the conjugate axis has equationy = - f i x .
(See Fig. 10.27.)
2 + 2J3 XJ -y = 4
Fig. 10.27

PROGRESS TEST 1

1. Give the new coordinates of the points (1,0), (0, l), 3. Apply the same a = 77/6 rotation to the equation
( fi,l), (1, fi)
and (1, 1) if the axes are rotated a n x2 - flay + 2y = 10 and simplify the result,
angle of ~ / rad.
6
2. Give the new equation of the line
( fi - 4 ) y - 2 = (1 + 4 f l ) x after applying the
above rotation.

THE GENERAL SECOKD-DEGREE EQCTATIOS


The equation

10.3.3 Ax2 + Bxy + Cy2 + Dx + Ey + F = 0


where A and C are not both zero, is called the general equation of the second degree. If
B = 0, then by completing the square (if necessary), we can identify the graph
as one of the conic sections (possibly degenerate).
In particular,
Ax2 + Cy2 + D x + Ey + F = 0
has as its graph:
(a) a circle if A = C
( b ) a n ellipse if A , C # 0 and have the same sign
(c) a hyperbola if A , C # 0 and have opposite signs
( d ) a parabola if A or C is zero
Of course in the degenerate case ( a ) or ( b ) may reduce to a single point or no
locus whatsoever, and (c) may result in two intersecting straight lines.
We shall now show that if B # 0, then a rotation of axes will transform
(10.3.3) into the equivalent equation

10.3.4 A f X 2 + CY2 + D X + EY+ F = 0


As before, this can be easily graphed in the XY system and hence in the xy
system.
Starting with the original equation

(3) A x 2 + Bxy + Cy2 + DX + Ey + F = 0


we apply (10.3.1) and let
SECTION 10.3: ROTATION OF AXES AND THE GENERAL QUADRATIC EQUATION 355

x = Xcosa - Ysina y = Xsina + Ycosa


where a is to be determined. It is tedious but straightforward to show that the
above substitution transforms ( 3 ) to
10.3.5 AX2 + BXY + CY2 + DX + EY + F = 0
where

B = B(cos2 a - sin2 a ) + 2(C - A ) sin a cos a

Since
cos 2a = cos2 a - sin2 a and sin 2a = 2 sin a cos a
we can write
B = (cos 2a)B + (C - A)sin 2a
If B = 0, then no rotation is necessary. If B # 0, then B will equal zero for any
choice of a that makes
(cos 2a)B + (C - A)sin 2a = 0
This is the case if

A-C
10.3.6 cot 2a = ___
B

O n the interval (0,T) the cotangent function has as its range (- co,co).Thus
if B # 0, there is a unique a with 0 a < < ~ / for
2 which (10.3.6) is satisfied. It
is not necessary to remember the formula given in (10.3.5.) because in practice
we use (10.3.6) and standard trigonometric identities to determine cos a and
sin a. We then use the transformation given in (10.3.1).

Example 2 Identify and graph the equation


8y2 + 6 9 - 9 = 0
Solution Now A = 0, B = 6 , and C = 8 so, by (10.3.6), cot 2a = - 8 / 6 = - 4 / 3 . It

3
sina = r=+c-=&
follows that cos 2 0 = - 4 / 5 , so
1 - cos 2a
and cosa =
1 + cos 2a
Thus a z 7134. With these values of s i n a and cos a we apply (10.3.1) to
+
t i obtain x = (l/m)(X - 3 Y ) and y = ( 1 / ~ % ) ( 3 X Y ) . Now in the X Y
system our original equation becomes

1
2
(3X + Y ) ] + 6[1(X - 3 Y ) ][ L ( 3 X + Y ) - 9 = 0
m V%
which simplifies to
X

A?--= Y2 1
9
Thus the graph is a hyperbola with a = 1, b = 3 and c = m.
The transverse
axis is the X axis, which in the 9-coordinate system has equation y = 3x, and
the conjugate axis has equation x = - 3y. The foci are
and the vertices are at ( l / m , 3 / m ) and
Fig. 10.28 Fig. 10.28.) 0
356 CHAPTER 10: A BRIEF REVIEW OF ANALYTIC GEOMETRY

Using the procedure illustrated, one can always obtain a rotation of axes that
transforms the general second-degree equation (10.3.3) into one of the form
given in (10.3.4). Then, using a translation of axes if necessary, its graph can be
identified as a conic.
It is even possible to identify the graph of a second-degree equation directly
from the equation itself. We shall give the criteria without proof.

10.3.7 Theorem The graph of the equation


Ax2 + Bxy + Cy2 + D x + Ey + F = 0
is
(u) a hyperbola if B2 - 4AC 0 >
( 6 ) a parabola if B2 - 4AC = 0
( c ) an ellipse if B2 - 4AC 0 <
I I\
In this we are using the terms H$@rbola, parabola, and ellipse in the loose sense
that allows for possible degeneiacy.

PROGRESS TEST 2

Reduce 8 x 2 + 5y2 - 4 9 - 36 = 0 to a standard form


and sketch.

SECTION 10.3 EXERCISES

Identifv the graph o f each of the following equations and give its essentialfeatures. Check applying (10.3.7).

1. xz +y2 - 2xy - 4 f i x - 4fiy = 0 2. +


x 2 xy + y 2 = 6
3. x2 - 2*xy -y2 = 1 4. x2-xy + y 2 - x - y = 3
5. 3x2-4xy+8x= 1 6. +
7x2 8xy + y 2 = 3
7. 4x2 - 4xy + y 2 = 6 8. 2x2 - f i x y + y 2 = 8
9. xZ+xy+ 1 = o 10. 2x2 +4xy - y 2 - 2x +
3y = 12
11. +
6x2 5xy - 6y2 = 1 12. 3x2 +3y2 - 2 9 - 4 6 x - 4 6 y = 12
13. 25x2 + 14xy +
25y2 = 288 14. gX2 - 24xy + - 50x - lOOy = 175
15. +
5x2 + 6 ~ y 5y2 - 3 2 - ~ 3 2=
~ -32 16. +
3x2 4y2 5 = 0 +
17. x2-9=0 18. 9x2 - 24xy + 16y2 - 5 6 - ~ 9 2=
~ -688
19. +
5x2 8xy +
5y2 = 36 20. 3x2 + 1 0 9 + 3y2 + 8 f i x - 8fiy = 0

CHAPTER EXERCISES

R e v i e w Exercise@
In Exercises I to 30 sketch the graph o f the given equation and note all pertinent data (foci, vertices, center, and the like).
1. 9x2 -y2 = 9 2. y2 - x2 = 9
3. X' + ' 2~ - 8~ - 1
4 ~= 4. x2/2 +
2y2 = 8
+
5. 4x2 - 9y2 3 6 ~ 8~ 4 = 0 + + 6. y 2 - 4x2 = 16
7 . y2 - 12 = x 8. y 2 = 16 4x2 +
9. y = 4 0 -~ 4x2 - 97 10. x2 = -12y
11. x = 8y2 16y + +10 12. y 2 = - 1 6 ~
13. 9x2 - 4y2 - 5 4 -~ 1% +
29 = 0 14. 2 x 2 = 4 - y 2
15. 2x = y 2 8y + +22 16. (X + +
5)2 4(y - 3)2 = 1
17. 25x2 + 4y2 - 2 5 0 ~= 16,~- 541 18. x - 2 = q m
19. 4x2 = 2y -y2 20. y + +
3x2 6x - 1 = 0
21. 9x2 - 8y2 = -72 +
22. 2~ 4y2 = 24y - 39
SECTION 10.3: ROTATION OF AXES AND THE GENERAL QUADRATIC EQUATION 357

23. y + 64x = 4x2 + 254 24. 3x2 + 12y2 = 36


25. +
2x = 4y2 12y +8 26. 4 ( x +
3 / 2 ) 2 - 12(2 - J =
) ~ 48
27. ( X - 12)' = 4 - ( J + 3)' 28. x = -3y2 - 30y - 7 5
29. 2x2 - 4~ = 3yz + 69 + 7 30. ( x i 2 + 3)' + (3y + 6)' =9

In Exercises 31 to 40 determine the rotation that eliminates the xy term, rewrite the equation in the XY-coordinate system determined by this rotation,
and sketch.
31. 32r2 - 72xy + 53x2 - 80 = 0 32. x 2 + 3y2 = 2y - 2 fiq - 2 fix
33. 5x2 - BXY +
5v2 - 9 = 0 34. = 2xy - x - y
x2 + y 2
35. 11x2 + y 2 = 20 +
lo@& 36. x 2 + y 2 = 4 +
3xy
37. 3x2 +
4xy = 4 38. 7 x 2 + 7y2 = 16 +
18xy
+
39. 8x2 5y2 = 36 4 9 + 40. 16y2 - 24xy + 9x2 = 60y + BOX - 100
In Exercises 41 to 50 use the given information t o j n d an equation for the indicated conic section.

41. Parabola, focus ( 0 , 4 ) ,vertex (0, 0 ) 42. Parabola, focus (2, - l ) , vertex (2, 3 )
43. Parabola, focus ( 1 , l ) , directrixy = - 1 44. Ellipse, foci ( - 2 , - 1 & 3 fi), minor axis has length 6
45. Ellipse, foci (3 k fi,7), major axis has length 6 46. Ellipse, foci ( k l ,0), major axis has length 12
47. Ellipse, a focus (6, - l ) , center ( 3 , - l ) , major axis has 48. Hyperbola, foci ( t 5 , 0), a vertex ( 3 , O )
length 10 30. Hyperbola, foci ( 1 5 , 3), ( - 1 1 , 3 ) , a vertex ( 7 , 3)
49. Hyperbola, a focus ( 0 , 13), a vertex ( 0 , 12), center (0,O)

Miscellaneous Exercises

1. A line tangent toy2 = 4px at P meets the directrix at Q. If 6. Show that the tangents to an ellipse and a hyperbola with
the focus of the parabola is F , then show FQ 1.FP. the same foci are perpendicular at the points of intersec-
2. +
Given an ellipse x2/a2 y 2 / b 2 = 1 , show that the length tion.
of the chord passing through a focus and perpendicular to 7. Let F , and F2 be the foci of a hyperbola. Show that the
the major axis has length 2b2/a. tangent line to the hyperbola at a point P bisects the angle
3. Is it possible for lines from the foci of an ellipse to a between F,P and F2P.
common point P on the ellipse to be perpendicular? Ex- 8. Find the area of the largest rectangle with sides parallel to
plain. the coordinate axes that can be inscribed in the ellipse
4. Find an equation for the parabola with directrix $/a2 + y 2 / b 2 = 1.
y = ( 3 1 4 ) and ~ focus ( 2 , 3). 9. Show that any line distinct from but parallel to an as-
5. +
Show that the parabola y = ax2 bx + c has vertex ymptote of a hyperbola will intersect the hyperbola in
( - b / 2 a , ( b 2 - 4ac)/4a). exactly one point.

SELF-TEST +
Chapter 11
ECHNIQUES OF INTEGRAT I
COXTENTS Differentiation usually is the relatively routine
applying a formula. Antidifferentiation seldo
gration by Parts 360
direct application of a formula and can som
rigonometric Substitutions 365
considerable ingenuity. Our basic use of an
Partial Fractions 370 as a shortcut in evaluating definite integrals
Rationalizing Substitutions 378 by the Fundamental Theorem of Calcul
eneral Strategies, Including the chapter we shall extend considerably our a
Use of Integration Tables 382 evaluate integrals, both indefinite and defin
umerical Integration 386 Sections 11.I through 11.4 provide straightf
illustrations of the most commonly used tec
integration, and Sec. 11.5 addresses th
to do when confronted with an integral
signaling which method to use. We inc
general strategy the use of tables of integrals.
we point out in Sec. 11.5, to use such a table
often perform the same sorts of manipulating,
and classifying as is involved in Secs.
11.4 when evaluating integrals without tables. Hen
techniques are important whether or
integration tables. lastly, fairly simple c
exist ( v ' m , for example) whose an
expressible as finite combinations of the fun
to date and therefore cannot be determined
these techniques of integration. Section 11.6
numerical approximation techniques,
and Simpson's Rule, that enable us t
integrals when we cannot determine t
hence cannot use the Fundamental Theorem.

To fit the special needs of this chapter, the end o


chapter is organized somewhat differently tha
others. We have not distinguished between Revi
Miscellaneous exercises. The chapter exercises
especially important because, as a source of
integrals, they help you learn the subtle skill
which manipulations and techniques to use on a give
integral.

359
360 CHAPTER 11: TECHNIQUES OF INTEGRATION

Section 1 1.1 : Integration by Parts

Objective: INTRODUCTIOS
Recognize integrands that yield to In Chap. 6 we introduced the substitution technique for evaluating integrals,
the method of integration by parts both definite and indefinite. It is based upon the Chain Rules for Integrals and
and then apply this method to Antiderivatives, (6.4.4) and (6.4.5), respectively. In Chap. 8 we applied the
obtain the indefinite integral. substitution technique to a wide variety of integrals involving exponential,
logarithmic, trigonometric, and inverse trigonometric functions. Nonetheless, a
relatively simple integral such as
sxcosxdx
fits none of our available formulas no matter what sort of u substitution is tried.
(This antiderivative arises, for example, when we use the shell method to
determine the volume of the solid formed by revolving about they axis the
region under y = cos x from x = 0 to x = 77/2.) We need a function whose
derivative is the product x cos x . A good start toward such an antiderivative is
x sin x because, by applying the Product Rule to x sin x , we get x cos x +
sin x .
This is close, but contains the extra term sin x . However, if we add cos x to the
proposed antiderivative, we have y = x sin x +
cos x , and

4 = x cos x
- + sin x - sin x = x cos x
dx
We have found it!
s x cos x dx = x sin x + cos x + C
A more systematic process for undoing the Product Rule is the topic of this
section.

THE FORMULA FOR ISTEGRATIOS BY PARTS


Suppose u and u are differentiable functions of x . Then, by the Product Rule,

- d( u v ) = u- dv
du
+ v -dx
dx dx
O n the other hand, by rearranging terms,

u-
d v = - d( u v ) - u-
du
d x d x dx
and then multiplying through by dx and taking the antiderivative of both sides,
we obtain the following:

11.1.1 Formula for Integration by Parts


s u d u = uu - s u d u
We apply this formula to an integral J f ( x ) dx by thinking of J f ( x ) dx as Ju dv
and replacing J u dv by uv - J v du in the hope that the new integral will be
easier to evaluate than J f ( x )dx. Since most integrals J f ( x ) dx can be written in
the form J u dv in several different ways, we have a choice in deciding which
part off(x) dx to call u and which part (necessarily containing dx) to call dv. We
illustrate the technique by applying it to the integral considered earlier.
SECTION 11.1: INTEGRATION BY PARTS 36 1

Example 1 Use (1 1.1.1) to determine


Sx cos x dx.
Solution Here we can let u = cos x and dv = x dv, which together imply
X2
du = -sinxdx and u = sxdx =-
2
Then
s x c o s x d x = s u d v = uv - s u d u

But now the integral appearing on the right side, containing the integrand
(x2/2) sinx, is even worse than the original. Let us try instead u = x and
dv = cos x dx, SO
du = dx and v = J cos x dx = sin x
Then
u dv u u u du
1- I- - 1
s x cos x dx = x sin x - S sin x dx
= xsinx - (-cosx) + C
= x sin x + cos x + C
which is the same result achieved earlier. 0

Two comments on this technique are in order:


1. Although we added a n arbitrary constant C a t the end of the above
computation, we did not do so when integrating to get v from dv. Any constant
C, added to v will drop out of the final result, as can be seen if we replace u
by u +
C, in (11.1.1). (See Exercise 49.)
2. The integral of dv must be computable; otherwise we cannot even write
uu - 1v du.

Example 2 Evaluate In x dx.


Solution We have two possible choices of u and dv: ( u ) u = 1 and dv = In x dx or
( b ) u = In x and dv = dx. However, since we have no formula for J In x dx,
which is the original problem anyway, choice ( u ) is unworkable. For choice ( b )
we have du = dx/x and v = Sdx = x; so, by (11.1.1.),

S l n x d x = (1nx)x - S X P
n

= xlnx -
J dx
=xlnx-x+C 0

PROGRESS TEST 1

In Probs. 1 to 3 evaluate the given integral.


1. Jxe'dx 2. Jxsec2xdx 3. J arcsin x dx
4. Differentiate and simplify to check your result in
Prob. 3.
362 CHAPTER 11: TECHNIQUES OF INTEGRATION

MORE ISTEGRA4TIOXBY PARTS


Some problems will involve repeated applications of integration by parts.

Example 3 Evaluate J ( x 2 + 2x)e" dx


Solution Let u = x z + 2x, and du = ex dx. Then
du = ( 2 x + 2 ) dx u = ex
so
(1) J(x2 + 2x)e'dx = ( x 2 + 2x)e" - 2 J ( x + l ) e Ddx
Note that the integral on the right-hand side is similar to the integral we started
with, except that the polynomial factor in the integrand has degree one less
than the original polynomial factor. Applying the by-parts method to this latter
integral with U = (x +
l ) , dc = e x dx, we obtain
J(x + l ) e x dx = ( x + l ) e x - S e x dx
= ( x + l ) e x - ex + C
This in turn can now be substituted into (1) to get
J(x2 + 2x)exdu = ( x 2 + 2x)e" - 2 [ ( x + l ) e x - e x ] + C
= x2ex +C
It should be clear that ifp(x) is a polynomial, then any problems of the form
Jp (x)ex dx or Jp (x)eaxdx
can be done in this fashion. The latter case would involve various constant
factors involving a. We would, of course, have to repeat the process as many
times as the degree of p ( x ) . Also, in each step we would choose u to be the
polynomial and du to be the exponential. Such problems as
J p ( x ) sin ax dx and Jp (x)cos ax dx
can be handled in a similar way.
Sometimes successive integration by parts will result in a constant multiple
of the original integral appearing on the right-hand side. In this case we can
then combine this multiple with the original integral to solve the problem.

Example 4 Evaluate s e Z x sin 3x dx.


Solution Let u = e Z x and dv = sin 3 x dx. Then

du = 2e2"dx u ;
= -- cos 3x
and

(2) s 1
e2' sin 3x dx = -- e Z x cos 3 x
3
+ 23 s e Z x cos 3 x du

Again we apply the by-parts method to the integral on the right, with
-
u = 821 dfi = cos 3 x dx

d E = 2e2" dx V= sin 3x
3
We obtain
s 1
eZx cos 3~ dx = - eZx sin 3x - 2
3 3
s e Z x sin 3x dx
SECTION 11.1: INTEGRATION BY PARTS 363

We now substitute this result into (2) to get

s 1
e2' sin 3x dx = --e2'
3
cos 3x
2
+-
9
e2' sin 3 x - 4
9
s e2' sin 3x dx

This is simply an equation of the form

I=A+B--I 4
9
where I is the original integral. It can be solved for I = J e2' sin 3x dx by adding
(4/9) J e2' sin 3x dx to both sides:

2
9
s e Z x sin 3 x dx = --1 eZx cos 3x
3
2 e2'
+-
9
sin 3x
Thus
J e2' -3
sin 3x dx = -e2' cos 3x
2 e2'
+- sin 3x + c
13 13
We could have done Example 4 just as easily by choosing u to be the trig
factor and dv to be the exponential factor in each step. In either case, onceyou have
decided onyour choicesfor u and dv in thejrst step, you must then maintain this same format
in the second step, or the second step will simply undo thejrst and no solution will result. If
you do not believe this, try it.
One other piece of practical advice: Write out your solutions completely and
systematically. Leave yourself plenty of space. Messy, cramped calculations are
not particularly reliable.

PROGRESS TEST 2

Evaluate the following:


1. J sin(1n x ) dx 2. 1( 3 x 3 + x)ln x dx 3. J x 2 sin 2x dx

INTEGRATING ODD POWERS OF SECANT OR COSECANT


In Sec. 8.3 we noted that the technique for evaluating J sec" u du or J csc" u du
for n a positive odd integer required integration by parts.

1
Example 5 Show that Jsec3 x dx = - [sec x tan x
2
+ In lsec x + tan X I ] + C.
Solution We begin by factoring out a sec2x dx to serve as dv.
J sec3 x dx = J sec x sec2x dx
Now let u = sec x and dv = sec2x dx; so
du = secx tanxdx and v = tanx
Then
J sec3 x dx = sec x tan x - J sec x tan2 x du
= sec x tan x - J sec x(sec2x - 1 ) dx
= sec x tan x - J sec3x dx + J sec x dx
Transposing -J sec3x dx, we obtain
364 CHAPTER 11: TECHNIQUES OF INTEGRATION

2 J sec3x dx = sec x t a n x + J sec x dx


= sec x t a n x + In lsec x + t a n X I
and so
1
J sec3 x dx = -[sec
2
x tan x + In /secx + t a n X I ] + C 0

SECTION 1 1.1 EXERCISES

In Exercises I to 30 evaluate the given integral.

1. s x s i n x d x 2. J x c s c 2 x dx 3. 1'
I
arctan x dx 4. i e x l n x d x

5. Jx2" dx 6. J"x3exZdx 7. J cos(1n x ) dx 8. Jx2cosxdx

9. J x 2 1 n x d x 10. J x arcsec x dx 11. sTi6


T iS
x sec 2x tan 2x dx 12. ~ ~ " x t a n x d x
0

13. J x arctan x dx 14. x csc2 3x dx 15. J-e*"cosxdx 16. J e3x sin 2x dx

17. l e $ d x 18. ie x4 1n dx 20. J(x' + 1)sin 3x dx

21. s1
0
( x 2 - l)e2" dx 22. Je
1
(In x)' dx
In x
23. s - d x
6
24. s arct>G dx

x 3 dx
25. J 3x tan2 2x dx 27. Jx sinh x dx 28. J x cosh x dx

29. J arcsinh x dx 30. J arccosh x dx

In Exercises 31 to 36 determine the trigonometric or hyperbolic integral using integration by parts instead of a trigonometric or hyperbolic identity.
31. Jsin2xdx 32. J cos' Y dx
33. J sin 3x sin x dx 34. J cos 2 x cos x dx
35. J sin 3x cos 5 x dx 36. s cos x sin 4x dx
37. Evaluate J sec5 x dx. 38. Evaluate J csc5 x dx.
39. Given J d x / ( x l n x ) , what is wrong with letting 44. Repeat Exercise 43 for the region bounded by the x axis
u = l/(lnx), dv = d x / x ? Suggest a correct alternative. and y = sin x from x = 7i to x = 27i.
40. Use integration by parts to derive the general reduction 45. Determine the area of the region enclosed by y = In x
formula J(1n x)" dx = x(ln x)" - n J(1n x)"-l dx for any from x = 1 to x = e.
positive integer n. 46. Determine the area of the region bounded b y y = x2e"
41. Use integration by parts to derive a general reduction from x = -1 to x = 1 .
formula for J xnex dx for n a positive integer. 47. Determine the average value of f ( x ) = l n x from
42. Try two different choices of u and dv for J[(sin x ) / x ] dx. = 10-4 to = 104.
[Neither will work because J[(sin %)/XI dx has no ele- 48. Determine the average value off ( x ) = x 2 arctan x from
mentary antiderivative! See Sec. 11.5.1 x=Otox=l.
43. Determine the volume of the solid generated by revolv- 49. Show that any constant C, added when computing v in
ing about they axis the region bounded by the x a n d y integrating by parts drops out of the final integral,
axes,y = e x , and x = 1.

50. X (Recall Sec. 8.1, Example 5.) Suppose the monthly grows according to B ( x ) = 15 - ( x / 3 0 ) ( x - 60) ( x is the
income from an apartment complex grows linearly from years since graduation, money is measured in thousands
$1500 to $2500 over the 20-year expected life of the of dollars). A and B both start at $15.000 and finish at
complex during a period when money returns 8 percent. $45,000.
Determine the market value of the complex based upon ( a ) Determine the total money earned by each over the
the present value of its earning power. Compare this 30 years. (Sketch each earning graph.)
value with the $239,431 based upon a constant ( b ) Assuming that money pays 8 percent compounded
$2000/month income. continuously over this period, determine the present
51. X (Present Value of Personal Earning Power) In the 30 value of the two income streams.
years after graduation, person A's income grows accord- (c) Does the 8 percent present value discounting exag-
ing to A ( x ) = (x2/30) + 15, and person B's income gerate the difference between A and B ? Discuss.
SECTION 11.2: TRIGONOMETRIC SUBSTITUTIONS 369

PROGRESS TEST 2
-

In Probs. 1 and 2 evaluate the given integral.


dx
dx 2. J d m d x 3. Transform into a trigonometric inte-
gral.

SECTION 1 1.2 EXERCISES

In Exercises I to 30 evaluate the given integral.

x 2 dx
8. $ x 3 d n d x
7. S d n
x 2 dx dx
11. $ d m d x 12. J
S ( 4 - 9x2)3/2 d6 - 4 x - 2x2

X
dx 15. s4d x 3 dx
m
16. J d m d x

17. $d-dx 18. $ d m d x 19. s (x


(x2
+ 5 ) dx
+ 2x + 6)2 20. J(a2 - x2)3'2 dx
dx
24. J (2( +2 2x-~ -3 ) dx3)3'2
21* .f (.2 + 9)3
25.
J
[ ~
mdx
x
26. s d2FT.T
xdx
27. J 2x
XZ+X+l
+3 dx 28. J
x2
'(
+ 2x + 5
+ 3, dx

29.
dF=P
dx 30. s + -
(6x2
x3 dx
12x 5)3'2
31. Develop a formula (which will be useful in the next
section) for
(Ax + B ) dx
S ax2 + bx + c
( a ) by completing the square and ( b ) by breaking
the integral into parts. (Assume that b 2 - 4ac 0.) <
In Exercises 32 to 34 use a trigonometric substitution to re-prove the given formula.

32. s daK-7 du = a r c si n x
a
+c du --
1 arctan2
a
+c

34. J du 1 arcsec 2
=- +c
/UI d m a a

In Exercises 35 to 37 use the appropriate hyperbolic substitution to prove the given formula.

35. J du = a r c si n h x
a
+C 36. [ d u 2 d u n = '2 u d-t -
a2 arccosh 2 -t C
2 a
\lFv

38. Use integration to determine the area inside the upper


+
half of the circle ( x - 2 ) 2 ( y - 3)2 = 25.
39. Determine the area betweeny = d m a n d the x axis 40, Determine the area enclosed by the ellipse
from x = 0 to x = 2 f i . 9x2 + 4y2 = 36.
41. Repeat Exercise 40 for x 2 9y2 = 9. +
42. Develop a general formula for the area enclosed by the
ellipse a2x2 b2V2 = a2b2. +
370 CHAPTER 11: TECHNIQUES OF INTEGRATION

Section 11.3: Partial Fractions

Objective: INTRODUCTIOK
Integrate quotients of polynomials Suppose you need to integrate
(rational functions) by first
1 -
- 1
breaking them into sums of simple f ( 4= ~

1 - x2 (1 - x)(l + x)
parts-their partial fractions.
If you recognize that
1/2 1/2
f ( 4= -
1-x
+-
l + x
then

If we were confronted with a more difficult rational function to integrate,


such as
4x2 14x - 2 +
= x 3 + 3x2 - x - 3
an analogous decomposition of g ( x ) into easily integrated "partial fractions"
(whose sum is g ( x ) ) is harder to come by; but it does exist, as can be seen by
combining the following over a common denominator as indicated:
3 1
+---2
x + 1 x-1 x f 3
- 3(" - l)(x
- + 3 ) + 2(x + l)(x + 3 ) - (x + l ) ( x - 1)
(x + l ) ( x - l)(x + 3 )
-
- 4x2 + 1 4 -~ 2 =g(4
x3 + 3x2 - x - 3
As is probably apparent from this calculation, the method for writing the
partial fractions of a given quotient of polynomials amounts to undoing what
we do when we combine fractions over a common denominator. Since the last
step in combining fractions over a common denominator is to multiply the
factors of this common denominator, it is reasonable that the first step in going
the other way is to factor the denominator. But before we go too far, it is helpful
to review some basic algebraic facts and settle on some common terminology.
Recall that f( x ) is called a rational function iff (x) = p ( x ) / q ( x ) forp(x) and q ( x )
polynomials. Analogous to the distinction between proper and improper
fractions for rational numbers, we say the ratzonalfunction f( x ) = p ( x ) / q ( x ) is proper
if the degree of the numerator is strictly less than that of the denominator.
Otherwisef(x)is said to be improper. All comments in this section regarding procedures
for integrating rational functions will apply to proper rational functions only. Of course by
dividing the denominator into the numerator, any improper rational function
can be written as a polynomial plus a proper rational function. Since polyno-
mials can be routinely integrated, any method that allows us to integrate proper
rational functions automatically extends to improper rational functions by long
division. Also, to simplify matters somewhat we shall assume that the denomi-
nator q ( x ) has leading coefficient 1. Nothing is lost by this assumption because if
the leading coefficient is a # 1, we can divide numerator and denominator by a.
SECTION 11.3: PARTIAL FRACTIONS 371

Example 1 Express
4x5 + 3x4 - 9x2 + x + 1
2x3 + 4 2 + 3
as a polynomial plus a proper rational function with leading coefficient 1 in the
denominator.

Solution By long division we have


4x5 + 3x4 - 9x2 + x + 1 - 2x2 - -x5 + 5 - (55/2) x 2 - x + 14
2x3 + 4 2 + 3 2 2x3 + 4x2 + 3

which in turn equals


5 (55/4)x2 - (1/2)x + 7
- -x +5 - 0
2x2
2 +
x3 2x2 3/2+
Now that we have assumed for f(x) = p ( x ) / q ( x ) that the degree of q(x) is less
than that of p ( x ) and that the coefficient of the highest power of q(x) is 1, we
must factor the polynomial q(x). The Fundamental Theorem of Algebra says
that q ( x ) can be factored into a product of linear factors, such as (x - al),
(x - a2), and so on, and irreducible quadratic factors, such as ( x 2 blx c,),+ +
+ +
(x2 b,x c2), and so forth. By irreducible we mean simply that the factor
x2 + + bx c cannot be further factored using real numbers, We saw in Chap. 1
that a general quadratic expression ax2 bx + + c is irreducible precisely when
b2 - 4ac < 0.
If q(x) contains the factor (x - a)k,such as q(x) = (x - 2 ) 3 ( x - 5), we say
(x - a ) is a k-times-repeated linear factor. If k = 1, as for the factor (x - 5 ) above,
we say x - a is a nonrepeated linear factor. Similarly, we refer to the k-times-
repeated, or nonrepeated, irreducible quadratic factor x 2 bx + + c according to
whether q ( x ) contains ( x 2 bx+ + c ) ~for k > 1, or whether q(x) contains
+ +
( x 2 bx c), respectively.
One last preliminary fact we need from algebra is the following.

11.3.1 If pl(x) and p 2 ( x ) are polynomials such that el(.)


= p , ( x ) for all x except
possibly x = a,, a2, . . . , a,, then p,(x) = p,(x) for all x. That is, two polynomi-
als which are known to agree on all but a finite collection of numbers must be
identical.

We shall outline our procedures for decomposing f ( x ) = p ( x ) / q ( x ) into a sum


of partial fractions according to the four types of factorizations of q(x) given in
Cases 1-4.

Case 1: q ( x ) Is a Product of Nonrepeated Linear Factors


We are assuming q(x) can be written in the form
q ( x ) = (x - al)(x - a2) . . . (x - a,)
where the numbers al, a2, . . . , a, are all different.
Then there exist constants A,, A,, . . . , A , such that
P ( x )-_ A , + & + A . . . +- An
q(x) x - a1 x - a, x - a,
We are now going to illustrate the method for determining the constants
A,, . . . , A n with a specific example before commenting on the general
procedure.
372 CHAPTER 11: TECHNIQUES OF INTEGRATION

Example 2 Determine the partial fraction decomposition for the rational function

Solution We first factor q(x):


q(x) = ( X - 1)(x2 - x - 6 ) = ( X - 1 ) ( ~+ 2 ) ( ~- 3)
We now determine constants A,, A,, A, such that
x2+x+1 - A1
(~-1)(~'-~-66) X-1
+-+-
A2
~ +
A3
2X - 3
We now recombine the right-hand side over the common denominator
( x - l)(x + 2)(x - 3) and leave everything in factoredform to obtain

x 2 + x + 1
(X - 1)(x2 - x - 6)
-
- A,(x + 2)(x - 3) + A,(x - l)(x - 3) + A ~ ( x- I)(x + 2)
(2 +
- l)(x 2)(x - 3)
These two rational functions are defined for all x # 1, - 2, 3. They have the
same denominator and hence their polynomial numerators must be equal for
all x # 1, -2, 3. But then, by (11.3.1), they must be identical. Thus
x2 + x + 1 = AI(x + 2)(x - 3) + A ~ ( x- l)(x - 3) + A3(x - l)(x + 2)
is valid for all x. Now successively substitute the roots I, -2, 3 ofq(x) into both sides of
this equation. Note that with each substitution all terms except one on the
right-hand side reduce to zero. For x = 1, 3 = Al(3)(-2); for x = -2,
3 = A2(-3)(-5); and for x = 3, 13 = A,(2)(5). Thus

.
A - - -1 A,=- 1 A , = - 13
l - 2 5 10
Hence

(2 -
x2+x+1
1)(x2 - x - 6)
-
- --.-
1 1
2 x-1
+-.-
1 1 +-.-
5 x+2
13 1
lo x - 3
Having achieved the partial-fraction decomposition of
x2+x+1
(X - 1)(x2 - x - 6)
in Example 2, we can now integrate routinely as follows:

+ --1 1 + --]33 1 dx
x2+x+1
J ( x - 1)(x2 - x - 6) d x = J [ -2 Lx -L1 5x+2 102-3

Ix - 1) + -1n 1x + 21 + -1n
= --In1 1 13 Jx - 31 + C
2 5 10
All the algebraic calculations pay off by changing a rather difficult integral into
a routine application of

The Substitution Procedure used in Example 2 can be outlined as follows:

11.3.2 ( a ) Factor q ( x ) into distinct linear factors.


( 6 ) Write the partial-fraction decomposition for p ( x ) / q ( x ) in terms of A,, A,,
. . . , A,.
SECTION 11.3: PARTIAL FRACTIONS 373

( c ) Combine the partial fractions over the common denominator q(x), leaving
everything in factored form, and set p ( x ) equal to this numerator.
( d ) Substitute the n distinct roots of q ( x ) into both sides of the equation
determined in step ( c ) to obtain A,, A,, . . . , A,.
PROGRESS TEST I

Integrate each of the following by first finding the par-


tial-fraction decomposition.
- 8x3 - 8x + 1 dx (42 + 2) dx (Note that the coefficient of x 2
Sx5 x3 - g x
(Be sure to divide!) 2.
S 2x2 + 4x - 6 is not l . j

C a s e 2: q ( x ) Is a Product of Linear Factors,


Some of Which Are Repeated
In this case each factor (x - a)k gives rise to k partial fractions of the form

A1
- ~
A, ... A,
___
x -a ' ( x - a),' ' (x - a)k
Then p ( x ) / q ( x ) is equal to the sum of all the partial fractions arising from each
of the possibly repeated factors ( x - ai)"i.
Thus, for example, the partial-fraction decomposition of
X 2 + X + 1
( x - i)(x + 3)2(. - 213
is
A1
-1 +-+-A,+ 3 ) A3
(x + 312
+-+-+-
'44
(x - 2 )
A5
(. - 2 1 2
A,
.( - 213
Note that here and always the number of unknowns to be determzned in the partzal-
fraction decomposition must always equal the degree of q ( x ) .

Example 3 Evaluate 1 +-1 )1)3d x .


(x
x(x

Solution First we write


x+ 1 --
-
A,
+- A-, 1 +-+- A3 A,
x ( x - 113 .( - 112 .( - 113
As before, we combine the right-hand side over the common denominator
x(x - 1)3 and then equate numerators.

(4) +1= - 113 + A ~ +( A ~ ~ - 1)


~ - 112 ~ + (A , ~~
Letting x = 0 and 1, we obtain A , = - 1 and A, = 2 .
We now have two more unknowns to determine and no more roots to exploit,
so we simply substitute any two numbers, distinct from the values 0 and 1
already used, into both sides of equation (4). Let x = - 1, x = 2 (these equal A,
and A, only by coincidence) and we obtain, respectively,
0 = -8A1 - 4A2 + 2A3 - A,
3 = A, + 2A, + 2A3 + 2A,
But A , = -1 and A, = 2 so we have the equations
-4Az + 2A3 = -6
A, + 2A3 = 0
374 CHAPTER 11: TECHNIQUES OF INTEGRATION

Solving, we get
6 3
A -- and A - --.
2 - 5 3 - 5
Thus

The only variation in the Substitution Procedure outlined previously is in


step ( d ) . In this case there will not be n distinct roots to substitute. However, we
can substitute n distinct numbers, always remembering to include among them
all roots of q ( x ) . This is what we did in Example 3.

PROGRESS TEST 2
~ ~~ ~~~ _______________ ~

1. Evaluate the following indefinite integral: x4 + 1 &.

Case 3: q ( x ) Is a Product of Linear (Perhaps Repeated) and


Nonrepeated Irreducible Quadratic Factors
We assume henceforth that the discriminants of the quadratic factors are
negative; hence the quadratic factors are irreducible. Suppose q ( x ) factors as
q(x) = (X - U I ) ~ ,. * * (X - + b,x + cl) . (x2 + bjx + c j )
(x2
L Z ~ ) ~ , * *

Each nonrepeated quadratic factor ( x 2 + b , x + ci) of q(x) gives rise to the


partial fraction
+
A i x Bi
x2 + bix +
ci
Then p ( x ) / q ( x ) is equal to the sum of all the partial fractions arising from each
of the factors of q(x).
For example, the partial-fraction decomposition of
x2 + x
(x - 1)2(x2 + x + 1 ) ( 9 - 3x + 5 )
1s

A,
+-( x - 1 ) 2 +
A2 A3x + A, Agx A , +
x-1 x2+x+1 + x2-3x+5
Again, the number of unknowns in the partial-fraction decomposition equals
the degree of q ( x ) . Also, by completing the square in the denominator, any
integral of the form

can be transformed to
(Au + B) du
.f u2 + d 2
which in turn can be broken into two integrals, the first yielding a logarithm
and the second an arctangent.
SECTION 11.3: PARTIAL FRACTIONS 375

Example 4 Evaluate J ( x -x 21)-( 92x+ -2x3+ 2) dw.


Solution x 2 - 2x - 3
-- A ,
- A ~ x A3+
(x - 1)(x2 2x + + 2) x - 1 + x2 + 2x + 2
-
- A1(X2 + 2x + 2) + (A,x + A ~ ) ( -x 1)
(x - 1)(9 + 2% + 2)
Thus
x2 - 2x - 3 = A,(x2 + 2x + 2) + (A,x + A3)(x - 1)
For x = 1, - 4 = 5A,, so A , = -4/5; for x = 0, - 3 = 2A, - A , , so
A, = 7/5; and for x = -1, 0 = A , +
2A, - 2A3, so A , = 9/5. As a result,
(9- 2x - 3)
dw
J(x - 1 ) ( 2 + 2x + 2)
= J[- 4 - 1
5 x - 1
+-
5 X 2 +9 2x X+ +7 2 1.
= --ln/x
4 - 11 + -J
1 9x+7 &
5 5 (x +I),+ 1

= ---nix
4 - 1 + -J-
9 udu - - 11 - du
5 5 u2+1 5Jn2+1
= -- 4 lnlx - 11 + -lnju2
5
9 I/ --11 arctan u
10
+ 5
+C
= --h+
4
5
- I1 + -ln(x2
9
10
+ 2x + 2) - -
11 arctan(x + 1) + C
5

PROGRESS TEST 3

C a s e 4: q ( x ) Is a Product of Linear and Repeated Quadratic


Factors
In this case q(x) is of the form
q(x) = ( X - al)1 * * . (X - uk)n, ( x 2 + b,x + ~ ~ -
* ) * (~x 2 + bjx + cj).21
1

Each repeated quadratic factor of the form ( x 2 + bx + c ) ~gives rise to the


partial fractions:
A,x + A, A3x + A4 +
. . . ~-
x2 + bx + c (x2 + bx + c) (x2 + bx + c ) ~
For example,
x2 + 3
x(x2 + q y X 2 + + 113
has the partial-fraction decomposition
A, A ~ x + A ~A ~ +x A , AGX+AT
x + (2 + 1) + (x2 + 1)2 + (x2 + x + 1)
A ~ x A, + A1oX +All
+ (22 + + 112 + (x2 + + 113
376 CHAPTER 11: TECHNIQUES OF INTEGRATION

Before going on we should consider the problem of evaluating an integral of the


form

s (ybxB$
There are essentially three steps:

(u) Complete the square of the term x 2 + bx + c and write the integral in the
form
(Eu + F ) du
s ( u 2 + d2)

( b ) Break this into two integrals. The first,

can be evaluated using the Power Rule.


(c) To evaluate

s (.Zy&
make the trigonometric substitution u = d tan 8, and the problem will
reduce to integrating an integer power of cos 8.

Example 5 Evaluate s 2x4 + 3 2 + + 1 dx.


x ( x 2 + 1)2

Solution Now
2x4 + 3x2 + + 1 --
- A, + A,x + A3 A,x + A,
x(x2 + 1)2 x x2+1 + ( 2+ 1)2

- A,(X2
-
+ l), + ( A z x + A3)X(X2 + 1) + (A,x + A&
x(x2 + 1)2
Thus
2x4 + 3 9 + + 1 =~ ~ + 112
( +~ ( A 2, +~ ~ ~ ) +~1) (+ (A,%
~ 2+ A , ) ~

In this example we shall modify the Substitution Procedure for determining


the unknowns A,, A,, A,, A,, A,. We multiply out the right-hand side and
combine like powers of x to get
2x4 +32+ +1
= (A, + A 2 ) x 4 + A 3 x 3 + ( 2 A , + A , + A,)x2 + (A3 + A,)x + A ,
We now equate coefficients of like powers of x to obtain
A, + A, = 2
A3 =o
2 4 +A, +A, =3
A, +A,=1
A, =1

Thus A , = 0 and A , = 1. From the fourth equation, A , = 1 - A , =


1 - 0 = 1. Similarly A , = 2 - A , = 2 - 1 = 1, and A , = 3 - 2 A , - A , =
3 - 2 - 1 = 0. Therefore
SECTION 11.3: PARTIAL FRACTIONS 377

(2x4 + 3x2 + x + 1)dx


s x(x2 + 1)2

= lnlxl + T1l n ( x 2 + 1) + s c o s 26' d6' 2

= In/xl + -ln(x2
1
2
+ 1) + arctanx + x2 + 1 ) + c
~

In some problems the integrand is already in the form provided by a


partial-fraction decomposition. In such cases applying the procedure will lead
to constants that merely reproduce the original integral. For example, given
3x+1 &
.f (x2 + 4x + 6 ) 2
we would note that x 2 + 4x + 6 is irreducible, and so the integrand is already
of the form
Ax + B
(x2 + bx + c)"
Thus no partial-fraction decomposition is necessary.

PROGRESS TEST 4

1. Evaluate s x4 + 5 x 3 + lox2 + 8x + 1 dx,


(x- 1)(x2 + 2x + 2 ) 2

SECTIOhT 1 1.3 EXERCISES

In Exercises I to 24 evaluate the given integral.

s (2x - 1) dx
x2--x-6 2* s -+
(3x
x2
4)dx
4x - 5

5- s,?'" d,:"
378 CHAPTER 11: TECHNIQUES OF INTEGRATION

s ( x - 1) dx
( 2+ 3x + 5 ) ( x + 1)
( x 4 + x ) dx
22- J x4 + 4x3 + 5x2 dx 23* .f ( x - l ) ( x - 2 ) ( x - 3 )
24.

In Exercises 25 and 26 compute the given integral ( a ) using partial fractions, and ( b ) using a trigonometric substitution. Compare your answers.
dx
25.
J F 7 IL6- j x dxn 29. Use a method analogous to that of Exercise 28 to recal-
culate J sec3x dx.
27. Name a third (and easiest) method for Exercise 26. T/6 cos x dx
28. Use the partial-fraction method to rederive the formula 30. Compute
for J sec x dx using that
sin2x - 5 s i n x +5
1
sec x = --
cosx
cos x
1 - sin2x
31. Compute
6,T /2
sin x dx
cos2 x - 2 cos x +1

Section 1 1.4: Rationalizing Substitutions

Objective: INTEGRALS ISVOLVISG TERMS (ax + b)P/9


Transform certain integrands The substitution u = (ax +
b ) l / q can often be used to transform an integral
involving radicals and/or involving (ax + b ) P / q ( p and q integers, q >
0) into a rational function of u. The
transcendental functions into key is not to compute du directly, but to solve for x , getting
rational functions.
U' -b
x=-
a

from which we can conclude that


quq-1
&=- du
U

Example 1 Evaluate s (2x - l ) &


(.- 21213 1 * +
Solution We let u = ( x - 2 ) l l 3 ; then u 3 = x - 2 , so x = u 3 + 2 and dx = 3u'du. Thus
(2x - l)&

[ 2 ( u 3 + 2 ) - 13 3u'du
=s (6u5
U'+1
+ 9 u 2 ) du

=s(6u3 - 6~ + 9 +-u 2 +- '1) d u

- 3(x - 2)4/3 - 3(x - q2l3+ 9(x - 2)II3 - 9 arctan[(x - 2)1/3] +c


2
It may be that the integrand contains two (or more) terms of the form
(ax + b ) P / q , (ax + b ) P i / q 1 . We can then eliminate both radicals simultaneously
+
by letting u = (ax b ) l I T where r is the least common multiple of q and ql.
SECTION 11.4: RATIONALIZING SUBSTITUTIONS 379

Example 2 Transform [ x3/4 - x2/3


into an integral of a rational function

Solution Let u = x1l1*. Then u12 = x , 1 2 u l l d u = dx, u3 = ~ 1 1 4 , and ~4 = ~ 1 1 3 ,SO


u 9 = x314 and u s = x 2 1 3 . As a result,

We shall refer to manipulations such as these that change the integrand into
a rational function, as rationalizations of the integral. In much of this section we
shall concentrate on rationalizing rather than evaluating. Earlier methods
apply to the resulting rational integrand.

PROGRESS TEST 1

Rationalize the following integrals:


dx
1. 2. J v2x+ dx
1 + q%TT

INTEGRALS ISVOLVING A RADICAL TERM v-'


OR v ' n AND xp, p AN ODD ISTEGER
We can rationalize such integrals, where p is positive or negative, by letting u
equal the radical term and then taking the differential of u2. In each case u du
will be either x dx or -x dx. Thus we shall want to factor an x out of x p to pair
with dx. Since p - 1 is even, we can replace x p - l with a polynomial in u, as
indicated in Example 3.

Example 3 Rationalize J ":+ dx.

Solution Let u = d m .Then u 2 = x 2 + 9,so 2u du = 2x d x ; that is, u du = x dx. As a


result,

d m( x dx) u(u du)


X X2

=J [ l + (u - 3 ) ( u + 3) ] d u 0

Notice that evaluating the resulting rationalized integral in Example 3


obviously involves partial fractions. Using the trigonometric substitution
x = 3 tan 8 , a rational integrand eventually results, leading again to partial
fractions before a final answer is possible.
In examples where you have a choice between a rationalizing substitution
and a trigonometric substitution, the particular approach you begin with is not
crucial. In practice, if one method seems to lead to a rather complicated
integral, you might want to give up on it and try the alternate method. More
often than not, the work involved in either approach is the same, and so it
becomes a matter of personal preference.
380 CHAPTER 11: TECHNIQUES OF INTEGRATION

PROGRESS TEST 2

Evaluate the following:


(x3 - x) dx
2. s x 3 d n d x

SOME REMARKS O S SIMILAR SITUATIOSS


Other types of integrands not discussed previously also yield rational functions
of u if we let u equal the radical term. For example, to evaluate
S ( f / m ) / x 4 d x we try u = f/m. +
Then u4 = 1 x3, 4u3du = 3x2dx,
and so

dx = s v m ( x 2dx) -- su(4/3)u3 du = AJ
X6 (u4 - 1)2 3
u4 du
(u4 - 1)2

This can now be handled by the partial-fraction method.


We may also, on occasion, rationalize an integrand involving an exponential
term such as e x by letting u = e". Now, rather than computing du directly, we
solve for x getting In u = x, so du/u = dx. For example, consider
dx
J e6x + 3e4x + e2" + 1
Let u = e Z x . Then In u = 2x, so du/u = 2 dx and
dx du
J e6x + 3e4" + e Z x + 1 =Ju(u3 + 3u2 + u + 1)
which can be handled by earlier methods. (Which?)

ISTEGRALS OF RATIONAL FUNCTIOIYS OF


sin x A S D cos x
A somewhat technical but useful substitution

u = tan- X
2
can be used to transform an integrand which is a rational function of sin x and
cos x into one which is a rational function of u. Then x = 2 arctan u, so
dx = 2 du/(l + u2). From Fig. 11.3 we see that sin(x/2) = u / d m and
cos(x/2) = 1/ d m . Now, using the identities sin 28 = 2 sin d cos 8 and
cos 28 = cos2'6 - sin2 8 with d = x/2, we obtain
sin x = 2 sin-cos-
x x
2 2
U 1 -- 2u
= 2
dP-TT & q T - u2 + 1
and

Fig. 11.3
SECTION 11.4: RATIONALIZING SUBSTITUTIONS 381

Example 4 Rationalize s 1
tan x
+ cosx dx.
Solution Let

u
X
= tan-,dx =- . x =-
2du , sin 1 - u2
2u , cos x = -
2 u2+ 1 u2 +1 u2 +1
Then
tan x dx [sin x/cos x ]
S 1 + cosx =S +
1 cosx

=s 1 + (1 - u2)/(u2 + 1)
u du
0

PROGRESS TEST 3

Rationalize the following:


dx
J3 - 2cosx

SECTION 1 1 . 4 EXERCISES

In Exercises I to 30 (a) rationalize the given integral and then ( b ) evaluate the resulting integral.

1. 2. 3.

5. %!a 6. 7.

9. s(xz+3 ) d n d x 10. 11.


12. J- d G - 2 dx

13. s1 v m d x 14. 15.

.
0

17.
dx
18.
J- WdX 19. di
s2q7 X

dx dx
21. 22. 23.
J 2 s i n x - cosx +1 J 1 + tanx

25.
,J
e x dx
26.
dx
27. 28. sT +
0 1 2cosx
dx

29.
V/2

sinx
dx
+ cos-x 30.
s- ex dx

31. Compute J cos x dx using the substitution u = tan(x/2). 34. Repeat Exercise 31 using the substitution u = cot(x/2).
32. Repeat Exercise 31 for J sin x dx. 35. Repeat Exercise 32 using the substitution u = cot(x/2).
33. Repeat Exercise 3 1 for J cot x dx. 36. Recompute J sec3x dx using u = tan x/2. (Recall Sec.
11.1, Example 5 . )

I n Exercises 37 to 40 rationalize the given integral.


dx
38.
Jm
38. J-(sin2x +3 +2 sin x + 1) dx
cos x
40. s,/.1-x
CHAPTER 11: TECHNIQUES OF INTEGRATION

General Strategies, Including


Section 11.5: the Use of Integration Tables

Objectives: INTRODUCTION
1. Establish a general plan when Having studied the basic integration formulas of specific functions in Chap. 8
dealing with integrals when the and the more general techniques of the previous four sections, you have one
technique to use is not obvious. more integration skill yet to master: what to do when confronted with a n
2. Transform given integrals into integral without a section title indicating which method to use. Generally, the
forms appearing in integration idea is to find some formula that fits the integral. Right now, while your
tables, and then use the working knowledge of the specific formulas and techniques is fresh, the most
appropriate formulas to evaluate efficient approach is to spend some time manipulating the integral into some
these integrals. form recognizable in terms of a basic formula. Later, in months and years to
come when time has eroded your memory of the specifics, the table of integrals
becomes more important.
A reasonable question, then, is why learn zntegratzon technzques zn thejirst place?
The answer, to be illustrated in a few pages, is that to use integration tables in
any but the most routine cases we need to apply virtually every technique of
integration studied earlier. Most integrals arising in practice need to be modi-
fied before they can be matched to a formula appearing in a table, and it is in
this modifying and matching that we need to apply the skills developed
previously.

AS OV-ERVIEW-NONELEMENTARY FUNCTIONS
Before moving on it is useful to put our work with antiderivatives in perspec-
tive. Our main interest in searching for antiderivatives is in using these to
evaluate the sorts of definite integrals that occur in most applications. Of course
1;
our ability to evaluate an integral such as f( x ) dx by first finding an antide-
rivative F off and then computing F ( b ) - F ( a ) is based on the Fundamental
Theorem of Calculus. Our methods for finding such an antiderivative are based
upon relatively simple ideas. O n one hand, much of what is involved is merely
algebraic manipulation. This includes, for example, completing the square,
multiplying by constants, using long division, applying trigonometric identities,
and breaking rational functions into their partial fractions. O n the other hand,
since antidifferentiation is merely differentiation in reverse, the calculus part of
our techniques can be classified in terms of differentiation ideas. First, each
specific integration formula is a derivative formula written in reverse. More-
over, our two basic techniques of integration, elementary substitution and
integration by parts, are simply reversals of the Chain Rule and the Product
Rule, respectively.
However, even our best attempts to find antiderivatives are doomed to fail in
some cases because relatively simple functions exist whose antiderivatives
cannot be written as finite combinations of the functions studied to date.
Strange, but true! For example, antiderivatives of 1sin(x2)dx exzst, but, unfor-
tunately, they cannot be written as a finite algebraic combination or composi-
tion of rational, algebraic and transcendental functions considered so far. To
write a n antiderivative of sin(x2) we need a n infinite series (see Sec. 16.4).
Finite combinations of the functions we have studied so far are called
elementary functzons. The function whose derivative is sin(x2) is a nonelementary
function. However, if needed, we can approximate the dejinzte integral
Jr2 sin(x2)dx (the area under y = sin(x2) from x = 0 to x = 7 7 / 2 ) , using its
definition as a limit of Riemann sums. Since sin(x2) is continuous on [0,7i/2],
we know by (6.3.3) that this limit exists. More efficient techniques for approxi-
mating such definite integrals are the subject of Sec. 11.6. Furthermore, no easy
SECTION 11.5: GENERAL STRATEGIES, INTEGRATION TABLES 383

criteria exist that separate those functions whose antiderivatives are elemen-
tary-and that can thus be determined using techniques of integration or tables
of integrals-from those whose antiderivatives are nonelementary.

A GENERAL STRATEGY
When confronted with an integral to evaluate, it is best to proceed deliberately,
with a general plan in mind. Try simple approaches first. Does the integral
resemble a known formula? If so, how is it different? What kind of manipula-
tion is needed to get the integral into the right form? If a substitution looks
+
promising, try it; but write it out completely. For example, S ( x dx)/( 1 x4) involves
a quotient of rational functions, but plunging into the messy partial-fraction
+
approach will involve a difficult factorization of 1 x4. Maybe a trig substi-
tution will help. But again, we should look for a simpler approach first. Except
for the fourth power of x, the integral resembles the integral leading to the
arctangent function

dx
S7T-2-
- arctanx +c
+
But by letting u = x2, the denominator becomes 1 uz. Now u = x 2 implies
du = 2x dx, so x dx = du/2. Thus we can rewrite S ( x dx)/(l +
x4) as follows:
du/2 - 1 du -
1 arctan u +C
1
=-
2
arctanx +C
By writing out the substitution completely we were able to determine whether
the substitution led to a n integral that matched a formula ex&. In applying
formulas, close does not count. There must be an exact jt.

USING INTEGRATIOS T-4BLES


Lists of worked-out integrals exist in various lengths, running from a few dozen
formulas as on the front and back endpapers, to hundreds of pages in hand-
books. Although virtually every integral appearing in the book to this point
could be worked out using such tables, using tables in practice involves much
the same sort of classifying, manipulating, and substituting as was involved in
the techniques presented earlier.

Example 1 Evaluate d m d x using the table of integrals.


Solution We shall apply formula [28], which states that

Here we have u = f i x , a = &, so du = f i d x and dx = du/fi. Hence


384 CHAPTER 11: TECHNIQUES OF INTEGRATION

Many formulas occurring in integral tables take the form of reductionformulas


that allow us to replace integrals by succeedingly simpler integrals in a system-
atic way. Most of these reduction formulas are especially useful in cases where
repeated integration by parts would be required.

Example 3 Transform s G d x into a familiar form handled earlier.


Solution A search turns up no formula resembling the given integral, so we try a
substitution, u = tan x , du = sec2x dx. Thusdx = du/(sec2 x ) = d u / ( l + u 2 )and
J G a k = J( f i / ( l +
u 2 ) du. This last integral does not appear in a table
but can be rationalized using v = 6, dv = 1/2 6du, du = 2v dv and

We now have a rational integrand, which, again, does not turn up in the table.
But by using a partial-fraction decomposition based on the factorization of
+
1 v4 = ( v 2 + fiv + 1)(v2 - f i v +
1) we obtain, finally,

2 J 3 -- -*J2 v 2 + fv dv
iu+1 +?J u dv
v2-&v+1
We complete the square and let w = v & f i / 2 . Then, for example,
v dv - (w + ~?/2)dw
Jv2 + f i v + 1 - J w2 + ( l / f i ) Z
which breaks down into a natural logarithm and an arctangent (formulas [5]
and [24].) 0

Note that Example 3 required completing the square, two substitutions,


including a rationalizing substitution, and a partial-fraction decomposition.
Formulas also exist for definite integrals and, in particular, for products of
powers of sine and cosine. See Wallis' formulas [125-1261.
SECTION 115:GENERAL STRATEGIES, INTEGRATION TABLES 385

T/2
Example 4 Evaluate C O S x~ dx.

Solution By formula [ 1 2 5 ] ,with n = 5 (odd), we have

PROGRESS TEST 1

Using integral tables, evaluate:


dx
2. Ssin4xdx 3. S$/2C O S x~ sin2 x dx
J 5x v?G-TT

SECTION 1 1.5 EXERCISES

(Note: The Chapter Exercises provide exercises relating to thejrst part ofthis section.) In Exercises I to 30, ( a ) give the number of an appropriate
integrationformula in the table of integrals (the answer is not necessarily unique) and ( b ) apply that formula to evaluate the given integral. (For some
exercises a reduction formula may apply and be used more than once, leading to an integral that may be evaluated using another formula.)

s- dx 2. S d m d x
3. s x 2 d
dx
m , ,s
4.
2x dx

9. J x cot2 x 2 dx 10. J x 2 cos2 x 3 dx 11. J x 2 e x d x 12. s c d x


x3
I

14. arcsec v x - 1
13. J x a r c s i n x 2 d x dx 15. $ e 3 s i n x dx 16. J cos 5x sin 3x dx
V z - 7
17. J x 3 s i n x a k 18. { x 3 cos x dx 19. J sin2x c0s3 x dx 20. Jcos2 x sin3 x dx

21. J s i n j x d x 22. J c o s 6 x dx 23. dx 24. J ( d m ) dx


3
x d m
25. sC sin4 x a% 26. rC cos4 x dx
JO
27. sin4 2x
Jrl4
0
dx 28. ST/?sin5x dx
0

s
0
dx 30.
dx
29.
d5X2 + 3x + 2 xz+3x+ 1

In Exercises 31 to 36 some manipulation is required before the given integral can be transformed into a form t h a t j t s the table. Evaluate the given
integral.

32- s e3x
dx
- e-3x

34. $eZx d w d x

36. s d cosy vxd x

(Integrate by parts.)
386 CHAPTER 11: TECHNIQUES OF INTEGRATION

Section 1 1.6: Numerical Integration

Objectives: INTRODUCTION
1. Approximate the values of As noted earlier, iff(.) is continuous on [ a , b], thenJBf(x) dx exists. However, in
definite integrals using either some cases the antiderivative Sf(.) dx may not be a n elementary function and
trapezoidal or parabolic hence not computable using the methods studied in Chaps. 6, 8, or 11. Such
approximations of the integrands. integrals occur quite naturally in practice and do not necessarily involve
2. Where appropriate, determine especially complicated functions. For example, JB ePx2 dx arises as the area from
bounds on the errors involved in x = a to x = b under the widely used probability "normal curve." Although this
these approximations. definite integral exists (it is a real number), it simply cannot be computed using
the antiderivative shortcut.
This is a situation analogous to what we confront with a n irrational num-
ber-say, fi.It exists (arising, for example, as the length of the hypotenuse of
a 1, 1, f i r i g h t triangle), but can only be approximated using decimals. Just as
with the d&nition of *(a number that, when squared, yields 2 ) , the dejinition of
definite integral fails to provide a systematic procedure for determining the
approximation. The purpose of this section is to introduce two numerical
methods for approximating the definite integral SBf(x) dx, where we assume
that f ( x ) is continuous on [a, b]. We shall describe our methods for positive
functions, which frames our discussion in the language of area under the graph,
but our results hold for any continuous functions. In fact our numerical
methods (except for the discussions of approximation error) make sense even
when only a finite table of data is given rather than a completely defined
function (see the Section Exercises).
Wide availability of computing machines has made numerical integration a
common and very useful practice, and many particular techniques are in use. In
fact, the study of such numerical techniques is the subject of many books in
itself. Here we shall examine two of the basic techniques, the Trapezoid Rule
and Simpson's Rule.
Even these rules are available on most programmable calculators, so that
your becoming proficient in the actual calculations is not the whole purpose
here. Perhaps even more important is your becoming familiar with the sorts of
issues that arise in numerical integration: When is it appropriate or necessary?
How accurate are the estimates? How does the accuracy relate to the integrand,
the limits of integration, or the number of subdivisions? When might one
method be preferable to another?
In Sec. 16.4 we briefly examine another method for approximating definite
integrals using infinite series.
Xi- 1 Xi

(b)
Fig. 11.4 THE TRAPEZOID RULE
Instead of using flat-topped rectangles [Fig. 11.4(a)] as in the definition of
definite integral, we use trapezoids [Fig. 11.4(b)].
The area of a trapezoid of base b and heights h, and h, is b(h, h,)/2. +
Suppose the interval [a,b] is decomposed into equal subintervals by the points
+ +
xo = a, x1 = a Ax, x2 = u 2 A x , . . . , xi = a +
AX, . . . , xn = b. AS
shown in Fig. 11.5, the trapezoid associated with a typical i th subinterval has
area
Xz-1
Y
Ax
Xi
+ [ f P i-1 1 + f(.i 11 Ax
Fig. 11.5 We simply add these to get an approximation of Jif(x) d x ; that is,
SECTION 11.6: NUMERICAL INTEGRATION 387

11.6.1 The Trapezoid Rule The nth trapezoidal approximation is given by

where for each integer i between 1 and n, xi = a + i ( b - a)/n.


As has become our habit, we shall first apply the rule to a case where we
know the answer, 4s; dx/( 1 + x2). (Four times the area underf(x) = 1/( 1 + x 2 )
from 0 to 1.) We already know that

= 4 arctan 1 - arctan 0 = 4
1 +x2

Example 1 Determine the tenth trapezoidal approximation of J; (4 dx)/( 1 + x2).


Solution Here a = 0, b = 1, n = 10, andf(x) = 4/(1 + x 2 ) with A x = 1/10. In Table
1 1.1 we shall arrange our calculations in a n orderly way, and we suggest that
you do likewise. We round off to four decimal places.

4
xo = 0 A.0) = 1 + 02
~
= 4.0000

1
XI =-
10 = 1 + (1/10)2 z 3.9604
2
x* = -
10
f(4 =
1 + (2/10)* 3.8462

3
x3 = - z 3.6697
10 ('") = 1 + (3/10)*
4
xq = - z 3.4483
10 s(x4) = 1 + (4/10)*
5
xs = - = 3.2000
10 f(x5) = 1 + (5/10)*
6
X6 = -
10 f(x6) = 1 + (6/10)* z 2.9412
7
xi = -
10 f(x7) = 1 + (7/10)2 z 2.6846
X8 =-
a 2.4390
10 f(xs) = 1 + (8/10)2
9 z 2.2099
xg = -
f(x9) = 1 + (9.10)2
10
4
XIO = 1 mlo) = 1+12 = 2.0000

Table 1 1.1
388 CHAPTER 11: TECHNIQUES OF INTEGRATION

Thus, from Table 11.1,

=: 3.1399

This approximation is in error by


n - 3.1399 =: 0.0017 (again, rounded to four places)
In this example, we knew the antiderivative and hence the value of the
definite integral. Consequently we also knew the error. Numerical integration is
usually used when this is not the case, and as a result, having some idea of the
accuracy is quite important. We now give an upper bound for this error and
then discuss it.

1 1.6.2 Trapezoid Rule Error Suppose that f I exists and is continuous on [ a , b]


with If(x)i 5 M on [a, b]. If the nth trapezoidal approximation is denoted T,,
then

Although we shall not prove this error statement, we note that it makes
sense: The error bound increases with the size of the interval [a, b ] , decreases as
the number of subdivisions increases, and depends on the size of the second
derivative on [a, b]. This last fact occurs because the deviation off(x) from the
slant-line top of the trapezoid depends on the bend in the graph, which
depends upon how much the slopef ( x ) changes, which in turn depends on the
size of If(x)I on [ a , b].
The graph and the error bound both imply that the Trapezoid Rule gives
the exact value of the definite integral whenf(x) is a linear function. In such a
case f I 0, so the error is 0.

Example 2 Compute an upper bound for the error of the approximation in Example 1 and
compare it with the actual error.
Solution Here b - a = 1, n = 10, so it remains to determine an upper bound for the
second derivative off(x) = 4/(1 + x 2 ) on [0, 11. Nowf(x) = -8x/(1 + x 2 ) 2
+
andf(x) = 8(3x2 - 1)/(1 x 2 ) 3 . To determine an upper bound forf(x) on
[0, 11 we determine whether it is increasing or decreasing on [0, 11 by checking
the sign of its derivative. After simplifying, we find
96x(1 - x 2 )
f (x) = 20 on [0, 11
(1 + x2)4

Thusf(x) is increasing on [0, 11 and hence reaches its maximum at x = 1. By


(1 1.6.2),

16/8 - 1
-- --
12(100) 600
=: 0.0017 (rounded to four places) 0
SECTION 11.6: NUMERICAL INTEGRATION 389

Example 3 Determine the number of subdivisions required in a trapezoidal estimate of


1;(4 dx)/(l + x 2 ) to guarantee accuracy to within 0.00005 (accuracy through
four decimal places).
Solution Using the data accumulated in Example 2 , we need to find a positive integer n
such that
4dx (1)31~"(1)1 2 1
12(n2)
- -
12n2 6n2
< 0.00005
We have a choice here either to solve the inequality systematically for n, or to
use trial and ermr to determine an n that will suffice. Since the numbers here
are easy to deal with, we use the former approach, obtaining

<n2
6(0.00005)
Solved for n, this inequality tells us that n > 57.7. Thus any n 2 58 will yield
the desired accuracy. 0

PROGRESS TEST 1

1. Using that In x = 2. What value of n is necessary to provide accuracy


through five decimal places?
( a ) Estimate 2 using the Rule, with 3. ( a ) EstirnateS; e-$ dxusing theTrapezoid Rule, with
n = 5. n = 10.
( 6 ) Determine an upper bound for the error of this (6)
. , Determine an upper
_ . bound for the error of this
estimate. estimate.
(c) Compare the error bound with the actual error
through four places.

J = axz + bx + c SIMPSOSfS RULE


We shall improve on the trapezoidal approximation of a definite integral by
replacing the straight-line top of the trapezoid with a parabolic curve that fits
the graph of the integrand more accurately. Other types of curves could
be-and sometimes are-used, leading to different types of approximations,
each of which has its particular advantages and disadvantages. Some of these
are explored briefly in the section exercises. Simpson's Rule is based upon the
-h 0 h parabolic approximation and has the advantages of being both simple and
Fig. 11.6 amenable to machine calculation.
The formula for the area of the region under a typical parabolic curve
y = ax2 + bx + c over the interval [ -h, h ] , labeled as in Fig. 11.6, is

A=---2ah3 + 2ch (see Exercise 24)


3
We can rewrite A in terms of the heightsyo,yl, a n d y z as

h
11.6.3 A = -3[ y o + 4y, +y2] (see Exercise 25)
I
* x Now, using the fact that three noncollinear points determine a unique parab-
L A i
xo x1 x2
Ax Ax ola, we suppose that ( x o , f ( x o ) ) ,( x l , f(xl)), ( x 2 , f ( x z ) ) are three points on the
Fig. 11.7 graph of a continuous function as in Fig. 11.7. By (11.6.3), we know that
390 CHAPTER 11: TECHNIQUES OF INTEGRATION

a = xo hl x2 x3 x4

Fig. 11.8

11.6.4 A = + [f(xo) + 4f(x1) + f ( x 2 ) 1


We can now use (11.6.4) to approximate J B f ( x ) dx. First we break the
interval [ a , b] into an even number n of subintervals, each of length A x =
( b - a)/n. An even number is required because each parabolic arc covers two
adjacent intervals. The n / 2 parabolic arcs cover double intervals, as indicated
in Fig. 11.8. We now simply add the areas under the arcs to approximate
JBf ( x ) dx:
AX
4
3
[f(Xo) + 4f(x1) + f ( x 2 ) 1 + 7 [ f ( X 2 ) + 4f(x3) + ~ ( X J I

AY
+ ... + [ f ( ~ , - ~+) 4f(x,-,) + f ( x d 1
Except for the endpoints, each even point appears twice, so, noting that
Ax = ( b - a)/n, we recombine the above sum to obtain the following:

11.6.5 Simpsonfs Rule For each even positive integer n,

where, for each 0 5 i 5 n,


xn = 0.0 J ( 0 ) = 4.000000
X I = 0.1 4J(0.1) z 15.841384 xi = a +i ( 5 )
x 2 = 0.2 Zj(O.2) 2 7.692308
x3 = 0.3 4s(0.3) z 14.678899 We call this the nth parabolic approximation of 1
:f ( x ) dx and denote it by S,.
x4 = 0.4 ZJ(0.4) z 6 896552
A simple way to remember Simpsons Rule is as
x j = 0.5 4J(0.5) = 12.800000
x6 = 0.6 Z~(0.6) 5.882353
xi. = 0.7 4J(0.7) 2 10.738255
b - a [(ends)
3n
+ 4(odds) + 2(evens)]
x 8 = 0.8 ZJ(0.8) z 4.878049
x g = 0.9 4j(0.9) 8.839780 We will use Simpsons Rule on the same integral as in Example 1, whose
xIn = 1.0 J(1) = 2.000000
precise value we know to be 7 .
Table 11.2

Example 4 Use Simpsons Rule with n = 10 to approximate Ji(4 dx)/(l + x2).


Solution Again, we arrange our calculations in a table (see Table 11.2). Adding the
numbers of the right-hand column, we get

4- 4dx 1
-(94.247787)
30
= 3.141592667 0
SECTION 11.6: NUMERICAL INTEGRATION 391

Since 1 T - 3.141592667 I z 0.000000187 (rounded off in the last place), this


estimate is accurate through six decimal places.
The following formula provides bounds on the error in Simpsons Rule
approximations. We omit its proof.

1 1.6.6 ~ ) and is continuous on [a, b] with


Simpson%Rule Error S ~ p p o s e f exists
lf4(x)12 M on [a, b]. Then

Example 5 Determine a n upper bound on the error in Example 4.


Solution We need f4(~), where f ( x ) = 4/(1 +
x 2 ) . In computing a bound for the
trapezoid-approximation error (Example 2), we found
96x(1 - x 2 )
f(x) =
(1 + x2)4
so

f4(x) =
96(1 - lox2 + 5x4)
(1 + 4 5

A maximum f ~ r f ( ~ ) (on x ) [0,1] occurs at x = 0 ~ i t h f ~ ( 0=) 96 = M . (The


proof requires using f 5 ) ( x ) ; see Exercise 26.)
Since b - a = 1, n = 10, we have
(1)5 * 9 6 8
3.1415926671 5 180. 104 - -
15 =: 0.0000533 0

Notice that (rounded off in the last digit) the possible error for 10 subdivisions
with the Trapezoid Rule was 0.0017 compared to a possible error of only
0.0000533 for Simpsons Rule.
Notice that the error bound tells us that Simpsons Rule gives the exact value
of the definite integral whenf(x) is a polynomial of degree 5 3 . In such cases
f4 = 0, so the error is 0.

PROGRESS TEST 2

(We parallel the problems of Progress Test 1 so that the ( b ) (For calculators) What value of n is necessary to
accuracy of the two approaches can be compared and provide accuacy through 15 places (thereby exceed-
you can use some calculations done earlier.) ing the accuracy of most hand-held calculators)?
3. ( a ) Estimate 1;ePx2 dx using Simpsons Rule with
1. Using In x = , carrying calculations through n = 10.
1
five decimals: ( 6 ) Determine an upper bound for the error of this
( a ) Estimate In 2 using Simpsons Rule with n = 4. estimate, using that lf(4(x)l <
12 on [0, 21.
( b ) Determine a n upper bound for the error of this (c) Compare with the corresponding result for the
estimate. trapezoid approximation.
(c) Compare the error bound with the actual error.
2. ( a ) What value of n is necessary to provide accuracy
through five decimal places?
392 CHAPTER 11: TECHNIQUES OF INTEGRATION

SECTIOS 1 1.6 EXERCISES

In Exercises I to IO, ( a ) estimate the dejinite integral using the Trapezoid Rule with n = 4, (bj determine an upper boundfor the approximation
error, ( c ) estimate the integral using Simpsoni. Rule with n = 4, ( d j determine an upper bound for the approximation error, and ( e ) compare the
results of ( b ) and ( d ) with the actual error.

1. i23x2dx 2. s 01 6 x 5 d x 3. iTsinxdx 40 4.t.3 d . Y

5. L1ex& 6. ~v'2cosxdx 7. d 20
(2x - 1 ) d x 8. d 20
(4x + 1) dx
9. s1
0
(5x4 - 4) dx 10. l1 (x4 - 4) dx

For Exercises II to I9 follow instructions ( a ) to ( d ) .

11. 4' d m d x 13. s0' d - d x 14. sT"


0
G d x

15. i 2 % d x 16. s l d v d x
0
17. sl-
0
2x dx
x + l
18. ST'' dl - 2 sin2x dx
0

19. 4' ex' dx

20. How many subdivisions are necessary to determine distance traveled using the Trapezoid Rule. (Assume
JA G d x through four decimal places using ( a ) the that time is measured in hours, with xo = 0, x1 = 1/12,
Trapezoid Rule and (bj Simpson's Rule? (See Exer- . . . , x12 = 12/12 = 1 and the total distance is approxi-
cise 14.) matelyJAf(x) dx when f ( x ) is the velocity at time x ; here
21. Repeat Exercise 20 for Jh d m d x (See Exercise 13.) we only knowf(x) at the given values.)
22. Show that the Trapezoid Rule provides the average of 28. Using ( a ) the Trapezoid Rule and ( b ) Simpson's
the Riemann sums based, respectively, on left and right Rule, approximate the total area of a cornfield whose
endpoints of [xi-l, x, 1, associated with the definite inte- dimensions in meters are listed in the accompanying
gral of a continuous function. sketch at 10-m intervals.
23. How many subdivisions are needed to approximate
ln(3/2) through six decimal places using ( a ) the Trape-
zoid Rule and ( b ) Simpson's Rule? (Hint: ln(3/2j =
I n 3 - ln2.)
24. Prove that the area under y = a x 2 + bx + c from
x = -h to x = h is 2ah3/3 + 2ch. (See Fig. 11.6.)
25. Prove (11.6.3j as follows:
( a ) Expressyo,yl, y 2 in terms of h. 29. Using the Trapezoid Rule, determine the approximate
( 6 ) Use the equations in ( a ) to show that 2ah2 = volume of the solid generated by revolving the figure in
Yo +Y2 - 2Yl. Exercise 28 about its left end.
(c) Use (bj to complete the proof. 30. Suppose a cyclist were clocked at 10-min intervals with
28. Complete Example 5 by showing t h a t f ' 4 ) ( ~ )has a rela- the nine speeds resulting corresponding to the numbers
tive minimum at x = l / f i and then comparing in Exercise 28. (Note: The ninth is implied to be 0.)Use
I f 4 ) ( l / f i ) l , If'4'(0)l, and lf(4)(l)l. the Trapezoid Rule to approximate the total distance
27. Suppose a motorist noted the following speeds in miles traveled over that span.
per hour at 5-min intervals of a 1-hr trip: 0, 30, 40, 45, 31. Repeat Exercise 30, but instead compute the average
55, 25, 35, 0, 25, 55, 60, 50, 0. Approximate the total velocity over the measured interval.

CHAPTER EXERCISES

In Exercises I to I 1 I evaluate the given integral. Consult with your instructor on whether or not to use tables. These integrals are not graded with respect
to diflculty.
1. J x d m d x 2. J cot x csc3 x dx 3. J A x V 2 dx

4*
dx
S dx2- 4x + 13 s dx
(2 - 3 x 2 ) 3 ' 2
SECTION 11.6: NUMERICAL INTEGRATION 393

7. s l x f i d x
0 s (x2
dx
+ 9)2
10. iTsin5xsinxdx 11. J x c o t x c s c x d x 12. J cZx
sin 2x dx

x3 - 1
13. s-dx
x3 x +
14. sd dx
w 15. J(arctan x ) dx~

16. s dx
x3 vx-
17. J J 1E - dx x 18. J d-cos3xdx

19. J ( t a n x + cot x ) dx 20. J ( x In x ) dx~ 21. S x d m d x

22. S2sinx
dx
+ 2cosx + 5 23. s 1
dx
+ cos x 24. s ak
tanx - sinx

26. J s s e c x d x 27. J x sin2 4x dx

28. s 4- x2dx

31. J x 2 s i n h x d x 33. J"";dx


1 - 6

34. J sin2sinx x-coscos2x x dx 35. J x33x2--x22x-+12x1 36. J x arctanh x dx

37- 4 (1
dx
+ .2)2 38- kq-4
dx
39. J ( x 2 + 2)2' dx

40. 4'x3 d m d x 42. sv/*


0
e a x sin bx dx

43- s(x ( X - 2 ) dx
- 1)2(x 1 ) + 44. J
(x2
( x 2 + x + 1) dx
+ x - 2)(x2 + 3x + 2 ) 45. J ( t a n x sec x ) dx~

47. s f / m d x dx
48- J(x2 + 1)(x* + x + 1)
50. J x csc x cot x dx

53. s T l beax cos bx dx


0

xlnx
56. J ( x 2 + x - 2 )dx( x 2 + 5x - 14) 57. J-
V F Z T dx

59.
J___ WdX dx
X 60- SX(,Z + 3x + 4 )
61. s 2+ m
dx 62. s x2 +
(x2 - 1)2
dx

64. s*dx 65. fT" sec5 x dx


x3 - x2 J-v/4

67. s2a
a
d n d x 68. J x 3 d m d x

71. s (x -
( 3 -~ 2 ) dx
1)(x2 + 6x + 8 )
72. x 2 cos
ivl2 x dx

73. J ~ / sin~x dx 74. J ( a r c s i n x ) dx


~ 75. Jx t a n x sec x dx
(1 + cos x)2
394 CHAPTER 11: TECHNIQUES OF INTEGRATION

Xdx
78. J e d x
76- J ( x - 1)*(X2 + I )

dx 81. Jsin2x a d x
80.
( 4 x 2 - 24x + 27)3/2
82. J ( (x22x++63x )+dx 9 ) 4 83. J G l n x d x
( 4 - 9x2)3'2
dx

85. J ( 4 - 9 x 2 ) 3 / 2 x 4dx 87. sfi


88. $ x 5 d n d x 89. J x e x 2 cos x 2 dx

91. J- dx 92. J sin3 x cos2 x dx 93. JeZx sin 3x dx


1 + sinx
dx dx
96.
95- J f/fTzG J1- cosx + sinx
dx
99- s x z + 4x + 7
100. J cos x cos 3x cos 5x dx 101. Jy- dx 102. J-
d
dx
n
103. J x 2 arcsinh x dx 104. J x z arctan x 2 dx 105. S e a r c s i n s d x

106. 1e dx 107. J(4 - f)eZzdx


108. J x1 - xd x

109. Jsinh 3x cosh 5 x dx 1 10. JTe -I


x2cos x dx 1 1 1. Jln(cos x)sin x dx

In Exercises 112 to 1 15, ( a ) estimate the dejnite integral using the Trapezoid Rule with n = 4, ( b ) determine an upper bound for the approximation
error, ( c ) estimate the integral using Simpson's Rule with n = 4, ( d ) determine an upper boundfor the approximation error, and ( e ) compare the
results of ( b ) and ( d ) with the actual error.

112. r1 2x d m d x 113. i 2 4 x 3 d x 114. ile-zdx

115. (1e2xdx

For Exercises 116 to 121 follow instructions ( a ) to ( d ) appearing before Exercise 112.

117. i2:dx 118.


r yx sin. x dx
119. 127=7 dx

( c ) Repeat ( b ) to approximate 1;(4 A)/(


1 +
x 2 ) with
122. (Midpoint Rule) For f continuous on [a, b], let Ti =
(xi-l + x i ) / 2 where we have a division of [a, b] into n n = 10.
equal-length subintervals and n is an even number. By 123. ( a ) How can we use the previous calculations of f ( F i )
the Midpoint Rule, we mean the approximation of to refine a Midpoint Rule approximation when we
JBf ( X I dx by double the number of subdivisions from n to 2n?
( b ) Compare or contrast this situation with that for,
first, the Trapezoid Rule and, second, Simpson's Rule.
( c ) What if we use the average of the highest and
( a ) Interpret the Midpoint Rule graphically. lowest values off(x) on each subinterval [xi-l, xi I?
( b ) Use this rule to approximate In 2 with n = 10 and 124. (For calculators) Estimate J: x" dx using Simpson's
compare with the earlier results of applying our other Rule, with n = 10.
two rules. 125. Determine the volume of the solid generated by re-
SECTION 11.6: NUMERICAL INTEGRATION 395

volving about t h e y axis the region bounded by the


x a n d y axes, x = 1, a n d y = e-".
126. Repeat Exercise 125 for the region bounded by the x
2n - 3 du
and y axes, y = cos x, and x = n/2.
127. Determine the present value of a continuous income
stream expected to grow linearly from $1000 to
$6000/year over the next 10 years, assuming that the
prevailing rate of return on money during this period is
133. Use Exercise 132 to evaluate
s (x2
dx
+ 2x + 5)3 '

8 percent compounded continuously. 134. Derive, for bc # ad,


128. Repeat Exercise 127 but with the income dropping

129.
linearly from $6000 to $1000 over the next 10 years.
Evaluate Jln(x + d n )dx using a hyperbolic s (ax
Xdu
+ b)(cx + d)
- 2 [Lln(ax + b ) - -d ln(cx + d)
bc - ad a 1,

substitution. 135. Apply the formula in Exercise 134 to evaluate


130. Repeat Exercise 129 for J ln(x + d m )du.
131. Show that area of the sector of a circle of radius r .f 6x2 :$ -3 '

subtended by an angle of 0 radians is (r2/2) 8 . [Hint: 136. Which one of the following is elementary?
Base the sector on the positive x axis, break the region
into a (right) triangular piece and a second piece
having the circular part of the region as part of its
boundary.]
132. Use integration by parts to derive the reduction for-
mula (top, next column)

SELF-TEST
Chapter 12 x

397
398 CHAPTER 12: PARAMETRIC EQUATIONS AND MORE APPLICATIONS OF INTEGRATION

Section 12.1 : Parametric Equations

Objectives: GRAPHISG PARAMETRIC EQUATIONS


1. Sketch the graph of a pair of Thus far we have considered curves in the plane that arise as graphs of
parametric equations either directly functions of the formy = f ( x ) , x = g ( y ) ,or as graphs of equations of the form
by plotting points or by eliminating F ( x , y ) = 0. It is sometimes useful to regard a curve as consisting of points (x,y),
the parameter. where x andy are both functions of a third variable called the parameter. More
formally:
2. Determine the slope and
equation of a line tangent to the
1 2 . 1 . 1 Definition If x andy are both functions of the same independent
graph of a pair of parametric
variable t, and if D is a set of real numbers that is common to the domains of
equations.
both functions, then the equations

x = x(t),y =y(t) t in D
are calledparametric equations and t is called the parameter. By the graph of a pair of
parametric equations we mean the collection of all points (x(t),y(t))as t varies
over D.

If a pair of parametric equations is given without specifying D, then we shall


assume that D is the intersection of the natural domains of x(t) andy(t).

Example 1 Graph the parametric equations x = fi,y = t3I2 + 1.


Solution The intersection of the two domains is all t >_ 0. We construct a table showing
the points corresponding to a few values o f t .
I I
t 0 1/4 1 16/9 2 9/4 4

4t) 0 1/2 1 4/3 f i z 1.41 3/2 2


Y (4 1 ~ 9/8 1 2 91/27 + 1 z 3.83
(fi)3 I 35/8 9
Y

Sometimes, but not always, we can sketch parametric equations by eliminating


the parameter. In Example 1, if we solve x = fi for t, we get t = x 2 . We can
then substitute t = x 2 in the expression fory to get
y = t3/2 + 1 = (x2)3/2 + 1 = x3 + 1
We must be careful when eliminating the parameter to keep in mind the
possible values for x a n d y as the parameter ranges through its domain. In this
particular case, x = fi implies x 2 0, so although we eliminated the parame-
ter to gety = x 3 +1, the graph of the original parametric equations is only
Fig. 12.1 that part of the graph o f y = x 3 +
1 for which x 2 0.
In the special case where x = x(t) = t , the parameter amounts merely to
another letter used in place of x , soy = y ( t )is equivalent to givingy as the same
function of x .

Example 2 Compare the graphs of the following three pairs of parametric equations:
(a) x = 2t,y = 3t, ( b ) x = 2t2,y = 3t2, (c) x = 2 cos t , y = 3 cos t.
SECTION 12.1: PARAMETRIC EQUATIONS 399

(a]

Fig. 12.2

Solution The domain in all three pairs is R. Eliminating the parameter in ( u ) gives
t = x/2, soy = 3(x/2) = (3/2)x. Since t is any real number, the graph of ( u ) is
the entire straight h e y = (3/2)x.
In (b), x 2 0,y 2 0. To eliminate the parameter in (6) we solve for t2, to get
t2 = x/2. Theny = 3(x/2) = (3/2)x. Again the graph is a straight line, but this
time only x 2 0 is part of the graph.
In (c), we know - 1 5 cost 5 1, so 2(-1) 5 x 5 2(1) and 3(-1) 5
y 5 3(1). Here we can eliminate the parameter by solving for cos t ; that is,
cos t = x/2 and y = 3(x/2) again.
The graphs of ( a ) to (c) are given in Fig. 12.2. 0

Another way to eliminate the parameter is to exploit some relationship


between the functions for x and y .

Example 3 Identify and sketch the graph of x = 2 cos t , y = 3 sin t , 0 5 t 5 271.


Solution Here it is not particularly convenient to solve one of the equations for t or an
expression involving t and then substitute directly into the other. It is easier to
use that cos2 t +
sin2 t = 1. We solve the two given equations for cos t and sin t ,
respectively, getting

We can then conclude that

The graph of this latter equation is a n ellipse, so it remains to determine which


part of this ellipse is included in the graph of the original parametric equations.
To do this, we monitor what happens to the points (2 cos t , 3 sin t ) as t increases
through its domain. As t increases from 0 to m/2, x steadily decreases from 2 to
0 and y steadily increases from 0 to 3. We tabulate this first-quadrant data as
follows:
400 CHAPTER 12: PARAMETRIC EQUATIONS AND MORE APPLICATIONS OF INTEGRATION

Y A similar analysis on [7/2, m], 37/21


[T, and [3m/2, 2771 yields

Thus, as t increases from 0 to 2m, the points (2 cos t , 3 sin t ) trace out the
entire ellipse in a counterclockwise direction, reaching (0, 3) when t = 7/2,
( - 2 , O ) when t = 7 , (0, - 3) when t = 3m/2, and finally returning to ( 2 , O )
when t = 27. (See Fig. 12.3.) 0

It should be realized that the foregoing example generalizes.


Fig. 12.3
12.1.2 For 0 5 t 5 27, the graph of x ( t ) = a cos t,y(t) = b sin t is an ellipse centered at
the origin. If a = 6 , then the graph is a circle of radius a.

This last example also suggests that we can use parametric equations to
describe motion in the plane. This is indeed the case, and we shall return to
explore this idea in much more depth when we study vectors in Chaps. 17
and 18.
Lest our eagerness to eliminate parameters in the above graphing examples
persuade you that the important thing about parameters is to get rid of them,
we now present an application that shows how parameters are useful in
describing curves that would be difficult to describe using x and y alone.
Furthermore, the parametric equations resulting in this example provide a good
illustration of a case in which the elimination of parameters is quite diffi-
cult-and not especially valuable. (See Exercise 31.)

Example 4 Determine a pair of parametric equations whose graph is the curve traced out
by a fixed point P on the circumference of a circle of radius a as the circle rolls
without slipping along the x axis.
Solution We assume that the point P starts at the origin and then moves clockwise as the
circle rolls to the right. (See Fig. 12.4.)
Let (x,y) be the coordinates of P and let I9 be the angle 4 A C P measured in
4' radians. Now
t x = length OB = length OA - length P D
Since there is no slippage, the length OA is the s a E as the length of the arc PA'
of the circle. (Think about this!) Now the arc PA has length u.6' and
length P D = (length PC)sin I9 = a sin I9
so
x = a6 - a sin I9 = a(I9 - sin I9)
Also
B A y = length PB
= length CA - length CD
Fig. 12.4 = a - acose = a(i - case)
Thus the curve has parametric equations

12.1.3 Cyeloid x = a(I9 - sin 0) y = a(1 - cos 6) 0


SECTION 12.1: PARAMETRIC EQUATIONS 40 1

+I
J'

AR

I x,8, = a 8 - sin 8 1 , ~0 = a 1 - cos 8 '+* x-"


B
e
Fig. 12.5 Fig. 12.6

A curve given by (12.1.3), called a cycloid, is sketched in Fig. 12.5. Cycloids


have fascinating properties. Suppose a frictionless bead is to slide down a wire
from A to B , as in Fig. 12.6. What shape wire allows the quickest slide? Not a
straight wire, but a wire in the shape of an inverted cycloid with A in the
position of the "point" of the cycloid. Furthermore, if B is at the lowest point of
the cycloid, then the length of time required for the slide from A to B is the
same as the length of time for the slide )om any intermediate point to B !
Other types of cycloids can be generated by following the path of a point
inside a rolling wheel or the path of a point outside the rolling circumference of
the wheel (as on the flange of a railroad train's wheel).

PROGRESS TEST 1

Graph the given parametric equations.


1. x = $ , y = t - 4 2. .Y = 2 sec t , y = 4 tan2 t , O 5 t 5 7i
3. x = t - l l , y = 2 t + 3

TANGEKT LIKES AND DERIVATIVES


In Example 2 we saw by eliminating the parameter that the graph of x = 2t,
y = 3t is the straight liney = (3/2)x-a line through the origin with slope 3 / 2 .
The slope of this line could be reckoned from the parametric equations them-
+
selves. A given change A t in t induces the change 2(t A t ) - 2t = 2 At in x and
+
3(t A t ) - 3t = 3 At iny. Hence the slope of the line (they change divided by
the x change) is (3 At)/(:! A t ) = 3 / 2 no matter what t is. Even if we did not know
that the graph is a straight line, this last observation that the slope is the same
for all t would guarantee it.
Suppose now we have a pair of parametric equations (not necessarily linear)
x = x(t),y = y ( t ) t in I
where I is an interval.
We shall now compute the slope m,,of the line tangent to the graph of these
equations at t in I (provided this tangent exists). We proceed as we did in
Chap. 2 ; that is, we examine the slope of the line through P = ( x ( t ) , y ( t ) )and
+ +
Q = ( x ( t At),y(t At)) as At + 0. (See Fig. 12.7.)
Provided x ( t + A t ) - x ( t ) # 0 for small (Ail, the slope of this line is

At + At) - A t )
x(t + At) - x(t)
which we can rewrite as
[At+ A t ) -y(t)l/At - Ay/At
-
Fig. 12.7 [x(t + At) - x(t)]/At Ax/At
402 CHAPTER 12: PARAMETRIC EQUATIONS AND MORE APPLICATIONS OF INTEGRATION

Now, if x ( t ) a n d y ( t ) are differentiable at t with dx/dt # 0, then


12.1.4

Example 5 Determine a n equation for the line tangent to the cycloid


x = 2(t - sin t ) , y = 2(1 - cos t )
at the point corresponding to t = m/6.
Solution Now Po has coordinates (m/3 - 1, 2 - fi).
Since
x ( t ) = 2 - 2 cost and y(t) = 2 sin t
we know that
2 sin(m/6) - 1
\
\ mo = -
2 - 2 cos(m/6) 2 - fi
Thus, using the point-slope formula, the tangent line Toto the curve a t Po is the
graph of the equation

y - ( 2 - 6 ) =
2 - 6
[.-(;- l)] 0

The slope formula in (12.1.4) must be used with care. In places where cusps
or corners exist there is no well-defined tangent. O n the other hand, if a curve
happens to cross itself, with P = (x(to),y(to)) = (x(tl),y(tl))for to # t,, then it
is possible to have two or more different but well-defined tangents, with
Fig. 12.8 different slopes, at P. (See Fig. 12.8.)
We should also like to be able to discuss vertical tangents where such
tangents exist, because they occur frequently when parametric equations are
graphed. It is not enough simply to require that x(to) be zero because, for
example,y(t,) may also be zero. T o avoid these and other potential problems
we shall usually restrict our attention to curves that have a t least one well-
defined tangent a t each point-curves that are smooth in the following sense:

12.1.5 Definition A curve given parametrically by x = x ( t ) , y = y ( t ) ,for t in some


interval I , is said to be smooth on I if

( u ) x ( t ) , andy(t) exist and are continuous on I , and


+
( b ) [x(t)12 [y(t)12# O for t in I.

Now from our earlier discussion we know the following:

12.1.6 Theorem Suppose Cis a smooth curve given parametrically on I by x = x ( t ) ,


Y = Y (4.
( u ) If x(to) = 0, then C has a vertical tangent at (x(to),y(to))with equation
x = x(to).
( b ) Ify(to)= 0, then C has a horizontal tangent at ( x ( t o ) , y ( t o )with
) equation
Y =y(to).

Example 6 Determine those values o f t in [0,277] and those points in the plane for which the
graph of x = 2 cos t , =~ 3 sin t has ( u ) vertical tangents and (6) horizontal
tangents. (See Example 3 and Fig. 12.3.)
SECTION 12.1: PARAMETRIC EQUATIONS 403

Solution x ( t ) = -2 sin t,y(t) = 3 cost, so condition [12.1.5(a)] holds.

(a) x ( t ) = 0 when t = 0, 7 , 257.


For such t,y(t) = 3, -3, 3, respectively, so condition [12.1.5(b)] holds.
Thus vertical tangents exist at t = 0, T, 257. Now x ( 0 ) = 4 2 5 7 ) = 2,
x ( ~= ) -2. Alsay(0) = 0 = y ( ~=y(257).
) Hence there are vertical tan-
gents to the elliptical graph at ( 2 , O ) and ( - 2,O).
( b ) y(t) = 0 when t = 57/2, 3m/2.
+
For such t , x ( t ) = - 2 and 2, respectively, so condition [ 12.1.5(b)] holds.
Thus horizontal tangents exist at t = 77/2, 3 ~ / 2 .Now x(a/2) = 0 =
x ( 3 ~ / 2 )a n d y ( ~ / 2 )= 3,y(3n/2) = -3. Hence there are horizontal tan-
gents to the graph at (0, 3) and (0, - 3). These conclusions that the curve is
smooth with the indicated tangents fit Fig. 12.3. 0

PROGRESS TEST 2

1. Determine a n equation for the line tangent to the 3. Show that condition [ 12.1.5(a)] does not hold for the
graph of x = e t , y = e c t at t = In 3 without elimi- parametric equations in Prob. 2 at t = 0.
nating the parameter.
2. Determine any values o f t for which there are hori-
zontal or vertical lines tangent to x = 2 cot t ,
y = 2 sin2 t with t in (0, 57).

ADDITIONAL REMARKS
Although the cycloid example given earlier indicates that parametric equations
arise naturally in solving specific problems, we have only introduced the topic
of parametric equations here. We shall see parameters used in many different
contexts in the coming chapters-for example, as angles in polar coordinate
equations and as time in motion and projectile problems in Chap. 18.

SECTIOS 1 2 . 1 EXERCISES

In Exercises I to IS, ( a ) eliminate the parameter and ( b ) sketch the graph.

1. x = t 2 , y = t 2 +t 2.
1
x=-,y=-
t2 - 1
t t2

3. x = t2 - 1,y = 4t 4. x=-
1
(t - 1)
2 , y = 3t 1 +
5. x = e+,y = et 6. x = cos 2 t , y = sin 2t, 0 5 t 5 7i
1 1
7. x = l - - - - , y = l + - 8. x = 3 t a n t, y = 4 sect
t t

9. x = s e c t , y = -tan2 t , -
-7i
2
< t < -2 ?7
10. x = f i t , y = 2t - 16t2

12. x = 2 t , y = d P ,- 1 5t5 I
14. x = d m , y = d m
16. x = 3cos2t,y = 2sin2t

In exercises I7 to 20, ( a ) sketch the given equations by plotting a table of data, and ( b ) check the result in ( a ) by eliminating the parameter.
17. x = c o s 2 t , y = c o s t , 0 5 t 5 277 18. x = sin t , j = sin 2t
19. x = s i n t , y = 2 + 3sint 20. x = 1 +
cos t , y = 2 cos t
404 CHAPTER 12: PARAMETRIC EQUATIONS AND MORE APPLICATIONS OF INTEGRATION

In Exercises 21 to 30, without eliminating the parameter ( a ) determine dy/dx, ( b ) give an equation of the line tangent to the curve at the given value
o f t , and ( c ) determine the values o f t where vertical tangents exist and the curve is also smooth.
21. x = 2 t + l , y = t 2 - 2 , t = 2 22. x = t 4 + 1,y = t2 - 1, t = 0
23. x = t 3 , y = t 9 +1, t = 1 24. x = tan t , y = sect, t = ~ / 4
25. x = 1 - t , y = e d , t = 1 26. x = e t , y = eZt+l, t = 1
27. x = t 3 - 3 t , y = t2, t = 1 +
28. x = t 3 , y = t 2 1, t = 2
29. x = e t , y = t 2 + 1, t = I n 2 30. x = sec 2t, y = tan 2t, t = m / 6
31. Eliminate the parameter from the equations for a
cycloid (12.1.3). (Solve t h e y equation for 8.) 1,

32. Suppose a circularjxed spool of radius a >


0 is centered
at the origin and is wound with thread. Find parametric
equations for the curve traced out by the end of the
thread as the thread is unwound from the spool. Assume
that the end starts at (a, 0) and that the thread is kept
taut. (See Fig. A , ) Use 8 as the parameter and note that
the segment RP is tangent to the spool at R. This curve is
called an involute of a circle.
33. Triangle ABP is a rigid equilateral triangle of side a with
A sliding on they axis while B slides on the x axis. (See
Fig. C.) Determine parametric equations giving the x A
a n d y coordinates of P in terms of 8 as indicated.
34. Let C be a circle of radius a centered on t h e y axis a t
(0, a ) , and thus tangent to the x axis at the origin. (See
Fig. B.) Let L, be the horizontal liney = 2a and let L be
an arbitrary line through the origin. Denote the inter-
section of L with L , by A , and the intersection of L with
the circle by B. Let P = (x,y) be the point of intersection
0,al
of the vertical line through A and the horizontal line
through B . Find parametric equations for the curve
traced out by such points P using 8, the angle from the x X X
axis to the variable line L , as the parameter. (This curve I I B
is called the witch of Agnesi.) B C

Section 12.2: Arc Length and Area of Surfaces of Revolution

Objectives: LENGTH OF CURVES


1. Use definite integrals to Suppose we are given a piece of curve-usually called a n arc-in the xy plane.
determine the length of plane This arc may be the graph of a function of x , a function ofy, or the graph of
curves. a pair of parametric equations. Since the parametric case includesy = f ( x ) and
x = g ( y )as special cases, we shall first determine the length of a n arc that is the
2. Use definite integrals to
graph of
determine areas of surfaces
generated by revolving given plane x = x(t),y= y ( t ) a 5t5b
curves about the coordinate axes.
First, for convenience in our discussion, we assume that this arc does not cross or
overlap itself by assuming that

M t o ) ,Y(to)) # (X(tl), Atl)) for to # tl in [a, b )


Such an arc is called simple. We are allowing the possibility that ( ~ ( a ) ,
y ( a ) ) = ( x ( b ) , y(b)). In this case the arc is called closed. Our discussion thus
applies to such simple closed arcs as circles and ellipses.
One last assumption, stated out of necessity rather than convenience, is that
the arc be smooth on [a, b ] . Our plan for determining the length of this smooth
simple arc is:
SECTION 12.2: ARC LENGTH AND AREA OF SURFACES OF REVOLUTION 403

Fig. 12.9

1. Approximate its length using straight-line segments based on a partition of


see Fig. 12.9.
[a, b ] ;
2. Rewrite the resulting sum to get a Riemann sum.
3. Take the limit as the norm of the partition approaches zero to obtain a
definite integral.

Thus we let P be a partition of [a, b] into subintervals

[to, t l ] , . . . > [ t i - l , t i ] , . . [tn-l, t n ]


3 to = a, tn = b
Using the distance formula, the length of the curve from (x(ti_,),y(ti -1)) to
( x ( t i ) , y ( t , ) is
) approximated (see Fig. 12.10) by
asi = V " x ( t i - x(ti - 1 1 1 ~ + [y(ti -y(ti -111'

We multiply Asi by

Each of the functions x ( t ) andy(t) is differentiable on [ti-,, t i ] , so the Mean-


Value Theorem (4.4.4) applies to each. Hence there is a q in ( t i - l , ti ) such that

and a $ in (ti-l, t i ) such that

Thus the length of a typical approximating line segment can be written as


Asi = + [y'('&)I2Ati
d[x'(%)]'
which then implies that the total length of the arc is approximated by
II II

i=l i= 1

The next natural step is to take the limit as the norms of the partitions of [a, b]
tend to zero and arrive at a definite integral formula for the length of the arc.
406 CHAPTER 12: PARAMETRIC EQUATIONS AND MORE APPLICATIONS OF INTEGRATION

Unfortunately, a typical term of the sum may possibly involve distinct points?
and? from the subinterval [ti-1, t i ] . Using arguments beyond the scope of this
text (but which can be found in any book on advanced calculus) it can be
shown that because x ( t ) andy(t) are continuous, we can let =& and thus
arrive at a Riemann sum
n

Furthermore, the continuity of x(t) and y(t) guarantees that


v[x(t)I2 + [ y ( t ) I 2 is continuous on [a, b ] , so the limit will be the definite
integral

Hence we may define the length as follows:

12.2.1 Definition The length of a simple smooth arc

x = x(t),y = y ( t ) a 5 t 5b
is given by

s = lim
IIPII-0
[,k
.l=l
=
b
d[x(t)I2 + [y(t)I2dt
In the case that the curve is the graph of a functiony = f ( x ) ,a 2 x 5 6 , with
continuous first derivative on [a, b ] , we can regard x = x ( t ) = t. Then x(t) = 1
andy(t) = f ( t ) , so we have the following:

12.2.2 Theorem The length of the graph of a continuously differentiable function


y = f ( x ) , a 5 x 5 b, is

Similarly, we have:

12.2.3 Theorem The length of the graph of a continuously differentiable function


x = g(y), a I y Ib, is

Example 1 Use (12.2,l) to determine the circumference of a circle of radius 7.

Solution We center the circle at (0, 0) and use (12.1.2) to represent it parametrically by
x = rcost,y = rsint 0 5 t 2 277
Then
x(t) = -7 sin t,y(t) = r cos t
so
+
~ [ x ( t ) 1 2 [y(t)12 = d r 2 sin2 t + r 2 cos2 t = r f i =r
Thus
s=l 247
rdt = r t
SECTION 12.2: ARC LENGTH AND AREA OF SURFACES OF REVOLUTION 407

Example 2 Show that using (12.2.2) to find the length of the straight line graph of
f ( x ) = mx from x = 0 to x = b gives the same result as the formula for the
distance between two points.
Solution f ' ( x ) = m,so the length by (12.2.2) is

Y
p- 1 + m2& = ( d m 1 . x I b

0
=b d m
As shown in Fig. 12.11 this equals the result obtainable by the distance formula:
d w = b d m
[Notice that if m = 0, then our result says that the length of the segment of the
x axis from (0,O) to (b, 0) isb d m = b.]
The arc in Fig. 12.12 can be described in many ways, three of which are:
Fig. 12.11 (a) y = x2/3, 1 5 x <
8; (6) x =y3l2, 1 < y <
4; and (c) x = t3, y = t 2 ,
<
1 5 t 2. (Check by eliminating the parameter.)
The length should be the same no matter which arc length formula we
Y use-even though the integrands and the limits of integration will differ in each
case. The three integrals are:

(a) l8Jm dx using 12.2.2

(b) s4im&
1 using 12.2.3

Fig. 12.12
(c) s'
1
d[3t2I2 + [2tI2dt using 12.2.1

We leave the proof that these three integrals are equal ( ~ 7 . 6 3 4 as


) Exercise 44.

PROGRESS TEST 1

Determine the length of the following arcs: 2. 2 4 9 = x4 + 48 from x = 1 to x =3


1. x = etcost,y = etsint, 0 t< 7~

3. x = - " + e-' fromy = -1 t o y = 1


2
(use a hyperbolic function if you wish)

AREAS OF SURFACES OF REVOLUTION


In Sec. 6.6 we saw that by revolving a region about a line a solid of revolution is
generated. We shall now examine the outer surface of such a solid.
If a plane curve is revolved about an axis, the curve sweeps out what we shall
call a surface of revolution. If the curve is a straight-line segment of length L
parallel to the axis of revolution at distance r, the surface swept out is the lateral
surface of a right circular cylinder of area 2arL. If the curve is a straight line of
length L with one endpoint on the axis of revolution and the other at distance r
from the axis of revolution, it sweeps out the lateral surface of a right circular
cone of area arL. (See Fig. 12.13.)
The surface area of both the cylinder and the cone can be made more
obvious by slitting either along the generating curve and flattening out the
surface, The cylinder yields a rectangle of height 277r and width L, whereas the
cone yields the sector of a circle of radius L.
408 CHAPTER 12: PARAMETRIC EQUATIONS AND MORE APPLICATIONS OF INTEGRATION

a , b)
Fig. 1 2 . 1 3

Fig. 1 2 . 1 4

To develop a general technique to determine the surface area of such things


as a parabolic reflector, we shall consider the surface of revolution generated
when the graph of a functiony =f ( x ) from x = a to x = b is revolved about the
x axis. [As we have seen, a given plane curve may also be described as the graph
of a pair of parametric equations, and after we have completed they = f ( x ) case
we shall give analogous formulas for curves given parametrically.] Since our
approach to determining the surface area will involve our arc length formulas,
we assume that f has a continuous first derivative on [a, 61.
Again, we begin by approximating the graph with straight-line segments
based on a partition P of [a, b] as in Fig. 12.14.
Each segment of length L , whose endpoints are at distances rl and r2 from
the axis, generates the frustum of a cone (a chopped-off cone) with lateral
surface area determined by L and the average of the two radii:

12.2.4

This formula (a circular version of the area of a trapezoid) can be derived from
the earlier formula for the surface area of a complete cone using the cutting and
flattening idea (see Exercise 43 below). As with the formula for arc length, a
typical segment as pictured in Fig. 12.14 has a length that can be put in the
form

where x, is some number in [xi-l, x i ] .


Since the two ends of the segment are at distancesf(xj-l) andf(xi) from the
x axis (by 12.2.4), this typical segment generates a surface of area
SECTION 11.2: TRIGONOMETRIC SUBSTITUTIONS 367

arcsin 0 = 0. Also, x = v 5 / 2 implies 6' = arcsin [ f i / ( 2 fi)]


= arcsin 1/2
= n/6. Returning to the original integral, we have
-fi/2 x4& 25 sin4 0 f i c o s 6' dd
d 0
d c 7 =S ficos 0

= 25 4 ~ / 6
sin48dB

25
[l - cos 2dI2 dd

[l - 2 cos 28 + COS~ 281 dd

25 1
1 - 2 cos 28 + -(1
2
+ cos 40) 1
d9

=4
25
[ ~ / 6
[t- 2 cos 20 + 2
25 3
(3) = -[-e
4 2
- sin 20 + -sin
8
40
25 7 1.257
sin - + -sin -
3 8 3
25

If we had not changed the limits of integration, we would need first to


rewrite the antiderivative appearing in (3) in terms of 19, and then switch to x,
a two-step procedure because of the 40. The former approach is more efficient.

PROGRESS TEST 1

Evaluate the following:

2. sz
0
d4T;"dx

THE SUBSTITUTIOYS u =a tan 0 AND u = a sec 0


The next two trigonometric substitutions follow the same pattern as u = a sin 8,
so we present all three in tabular form. In each case it is understood that the
Power Rule does not apply directly.

For Ja' - u' For .iu2 + a' For \m


use u = n sin 8. use u = n tan 0. use u = a sec 0,
du = a cos fld8 dzi = n sec' 8 de du = a sec 0 tan 6 dB

I sin2 e + cos' e = 1 I 1 + tan' e = sec' e I


368 CHAPTER 11: TECHNIQUES OF INTEGRATION

Example 4 Evaluate s d-. +


x3 dx
x2 9
Solution Let x = 3 tan0. Then dx = 3 sec2 0 d0 and 4- = d 9 tan2 0 +9=
d m = 3 sec 0. Thus
x 3 dx tan3 0 * 3 sec20 d0
3 sec 0 -5l
= 27 J tan3 6' sec 0 dB
= 27 J tan2 O(tan 0 sec 0 do)
1" 3
= 27 J (sec20 - l)(tan 0 sec 0 do)
=9sec30 - 27secO + C

Example 5 Evaluate s (x2 - 4 ) 3 / 2 du


X

Solution Let x = 2 sec 0, then dx = 2 sec 0 tan 0 d0 and v ' m = d4 sec20 -4 =


dGZE? = 2 tan 0. NOW
( x 2 - 4)3/2dx = J ( d m ) 3dx
s x X

=s 8 tan3 0 2 sec 0 tan 0 d0


2 sec 0
= 8 J tan4 0 d0
2

= 8 J tan2 0 tan2 0 d0
= 8J(sec20 - 1) tan20d0
= 8[ J tan2 0 sec2 0 d0 - J tan2 0 do]
--8 tan3 0 - 8 t a n 0 + 80 + C
-
3
- (2 - 4 ) 3 / 2 - 4 d m 8 arcsec(x/2)
- + +C 0
3
As indicated in Progress Test 2, a trigonometric substitution can sometimes
be useful even when the integrand does not contain a radical term. Further-
more, when a quadratic expression ax2 bx + +
c appears inside a radical, it is
often useful to complete the square and rewrite the expression as a sum or
difference of squares so that one of the substitutions in (1 1.2.1) is appropriate.
For example,
J d w d r = 1d 9 - ( x - 3)' dr = J d m d u
with u = x - 3 . This latter integral can be evaluated using the trigonometric
substitution u = 3 sin 0. Be certain, when using such two-stage substitutions, to
return to the original variable. (See Prob. 2, Progress Test 2.)
In addition, if we are confronted with a radical of the form d m , for
example, we let u = 2x, regard v ' m as d m , and make the substitu-
tion 2x = 3 sin 0. Then dx = ( 3 / 2 ) cos 0 do, and so on. Alternatively, we could
factor the coefficient of x 2 out of the radical and write d w =
2 d v .We then let x = (3/2) sin 0 and proceed as usual. (See Exercises 9
and 10, for example.)
SECTION 11.2: TRIGONOMETRIC SUBSTITUTIONS 365

Section 11.2: I
Trigonometric Substitutions

Objective: INTRODUCTION
Transform integrals involving Certain frequently occurring integrals involving a radical term of the form
radicals containing quadratic
terms, v G o r &FZ,into
d 2 - 3 diFT-2 or d2T?
trigonometric integrals by replacing will not yield to a direct power-rule substitution. Contrast
u by either a sin 8, a tan 8, or
a sec 8.
Sxdmdu and S d m d x
The first integral, after the insertion of the constant factor 2, is a routine
application of the Power Rule:

O n the other hand, the integral J d m d r is missing an x factor, which


cannot be inserted because x is a variable. However, by making the substitution
x = sin@,so du = d(sin0) = cos0 do, we arrive at the trigonometric integral
d m c o s 0 dB
which can now be handled using the techniques of Chap. 8. In this section we
shall systematically examine three substitutions of this type to cover the three
types of radicals appearing above.
THE SWSTITUTION u = a sin 8
If an integral contains the term
@=-2 a>O

JF7 and does not yield easily to the Power Rule, we make the following substitution
Fig. 1 1 . 1 and draw the triangle in Fig. 11.1: u = a sin 0, so du = a cos 0 do.
The labeling of the right triangle follows from the Pythagorean Theorem
and the fact that sin 8 = ( u / a ) . In addition, we assume that -a/2 5 0 5 r / 2
because, in returning from B to the original variable after integrating, or in
changing limits of integration in definite integrals, we need to regard B as
arcsin ( u l a ) .

Example 1 Evaluate s du x 2 .
~gx3-

Solutlon Clearly the Power Rule does not apply directly, so we let x = 3 sin 0 and draw
the right triangle in Fig. 11.2. Then du = 3 cos B dB and
v ~ T Fd=9 - 9sin20 = 3 d K Z F i J = 3dZ7T
= 3 lCOSO1
= 3 cos 0
(Note that since - a / 2 5 0 5 a / 2 , cos0 2 0 so lcos6l = cos8.) Hence
x3 du 27 sin3 O(3 cos 0 do)

= 27 sin3 0 d0
= 27 S(1 - cos20) sin 0 d0
C
Fig. 1 1 . 2
366 CHAPTER 11: TECHNIQUES OF INTEGRATION

To switch back to the original variable x we refer to Fig. 11.2 and notice that
=
C O S ~ ( d n ) / 3 . Thus
x3 du
= 27[
- d m + ( @ c 7 ) 3

3 27.3

Often, in evaluating the trigonometric integral that results after the substi-
tution, we need to use double-angle formulas, This complicates the switch back
to the original variable, as indicated in Example 2.

Example 2 Evaluate J d Tdx. X2


9 x

Solution Except for the degree of x in the numerator, this integral is the same as that in
Example 1, so the same substitution and the same triangle (Fig. 11.2) lead to
the integral

..f d
X2du
m= s 6 3 cos 6 d6)
sin 3(cOsQ
= 9Jsin2 6 d6
= gJ'- c0s26
2
d6 [double-angle formula (8,3.1), p. 2931

Now to express our answer in terms of x using the triangle in Fig. 11.2 we need
functions given in terms of 6, not 26. Thus we use the identity sin 26' =
2 sin 6 cos 6 to get

6 - sinBcos6

Since x = 3 sin 6, or sin 6 = x/3, we conclude that 6 = arcsin (x/3). Also,


1+ C

cos 6 = ( d m ) / 3 , so putting all this together, we have


9
-[6 - sin 6 cos 61
2
+ C = -g[2 arcsin -3 - -3 3
9
- _ arcsin-
-
x
-
xdc7
S C
2 3 2
Similar care in using identities applies when using definite integrals. We can
either switch the limits of integration or express the antiderivative in terms of
the original variable and use the original limits of integration.

Vw2 x4&
Example 3 Evaluate
0 d m
Solution Let x = f i s i n 8, so ak = f i c o s 6 d6 and

We shall evaluate the integral by expressing the integration limits in terms of


6. Since 6 arcsin(x/&), we know that x = 0 implies 6 = arcsin (O/fi) =
SECTION 12.2: ARC LENGTH AND AREA OF SURFACES OF REVOLUTION 409

[We use absolute-value signs to allow for the possibility thatf(x) < 0.1 By the
Intermediate-Value Theorem (2.5.12) there is an Ti in [ X ~ - ~ at which f
, X ~ ]

attains the average off(xi-l) andf(xi); that is,

Hence
AS, = 2.lf(x,)l d1 + [f(.,*)]Z Ax,
and Xrz1 ASt is the total surface area generated by all the line segments based
on the partition P of [a, b]. As with the arc length development, the detailed
arguments leading from Z:=, AS, to a genuine Riemann sum and then to a
definite integral are beyond the scope of a first calculus course. Note that a
typical term ASt involves possibly distinct numbers T, and x,* in each subinter-
val. Nonetheless, it is geometrically reasonable that such sums provide increas-
ingly accurate approximations to the actual surface area as the norms of the
partitions P get smaller. Furthermore, as 11 P 1) decreases, so does the difference
between F, and x : in a typical subinterval, and since bothf(x) andf(x) are
continuous, it is reasonable to expect that the sums tend to a definite integral.
Hence we make the following:

12.2.3 Definition Supposef(x) exists and is continuous on [a, b]. Then the area of
the surface of revolution obtained by revolving the graph of y = f ( x ) ,
a 5 x 5 b, about the x axis is

To generalize the formula in (12.2.5) it is helpful to reexamine our arc length


formulas in terms of differentials. For a functiony = f ( x ) as in (12.2.5), the arc
length formula (12.2.2.) also applies. In particular, if we let u be a dummy
variable and x be any number between a and b, we can think of arc length s
as a function of x :
s = s(x) = s
a
dl + [f(u)I2du
where s ( x ) is the length of the arc from a to x . But then, by the Fundamental
Theorem of Calculus,
dJ -
-
dx - d1 + [f(4l2
so
ds = d 1 + [f(4I2
dc
It is especially useful to regard ds as the length of a short piece of arc. Let us
assume the graph o f y =f(x) lies in the upper half plane. We can then re-
interpret the surface area formula as
b
12.2.6(a) S = 2.s yds
a

which geometrically supports the intuitive idea that the total surface area is the
sum of the areas 2y. ds of short cylinders of radius y and length ds. (See
Fig. 12.15.)
If instead the curve is revolved about they axis and the graph lies in the right
half plane, then a similar argument yields:

b
12.2.6(b) S = 2.s xds
a
410 CHAPTER 12: PARAMETRIC EQUATIONS AND MORE APPLICATIONS OF INTEGRATION

!a)
.,--------
I
(bj
Fig. 12.15

These formulas are equally valid if the curve is given in the form x = g ( y ) ,
a <y <b. In this case

ds = d1 + [S(Y)I24J
and [ 12.2.6(a)] and [12.2.6(b)] are interpreted to mean, respectively,
b
s = 271 Ja Y d1 + k ( Y ) l 24J
and
b
s = 277 Ja g(Y) d1 + [g(y)I2dy
Finally, if the curve happens to be the graph of the parametric equations
x = x ( t ) , y = y ( t ) ,where, again, x ( t ) andy(t) exist and are continuous on [a, 61,
then we can revolve the graph about either the x or t h e y axis to get two
different surfaces.

12.2.7 (a) Revolved about the x axis, the graph generates a surface with area

271 [ y ( t ) ds where ds = f l x ( t ) I 2 + [y(t)I2dt

( 6 ) Revolved about they axis, the graph generates a surface with area

271 (
a
x ( t ) ds where ds = d[x(t)I2 + [y(t)I2dt
In fact, each formula (12.2.5 to 12.2.7) is a special case of the following:

12.2.8 General Surface Area Formula The area of the surface generated by
revolving the arc C about the axis L is

271JbadS
a

where D is the distance of the differential of arc length ds from the axis L.
Depending upon the axis L and the form that C is given in, the general
formula will be filled out in any of several ways. You are encouraged to start
with this formula in setting up surface area integrals. In each case the sum-
of-cylinders idea applies.
SECTION 12.2: ARC LENGTH AND AREA OF SURFACES OF REVOLUTION 411

Example 3 Determine the surface area of a sphere of radius r with its opposite ends sliced
off to a depth of E units, 0 E < <
r. (See Fig. 12.16.)
Solution We shall revolve the graph off(.) = v m a b o u t the x axis from x = -r E +
to x = r - E. Nowf(x) 0 on [ - r > +
E , r - E ] , so l f ( x ) i = f ( x ) , andf(x) =
-x/( d m ) ,so by (12.2.5) the total surface area is

a7Jr-d
-r+a T 4r - K
dx = 277 Sr--r+a
d z T J y- - x d x

r--E
= 2 a S rdx
-r+E
T--E

= 27i [ r x ] = 47Tr(r - E)
-r+a

Fig. 1 2 . 1 6 Notice that becausef(x) = d m is not differentiable at x = t r , we


could not use (12.2.5) to determine the surface area of the entire sphere gener-
ated by revolving the whole semicircle about the x axis.

Example 4 Use (12.2.7) to determine that the surface area of a sphere of radius r is 4 m 2 .
Solution We revolve about the x axis the upper semicircle, which is the graph of the
parametric equations.
x = r cos t , y = r sin t with 0 5 t 5 TI
(Note that for t in [ T ,2 , get the lower semicircle.) Now
~ 1 we
ds = d[r(-sin t)I2 + [r cos tI2 dt = dr2(sin2t + cos2 t ) dt = r dt
Thus, by [12.2.7(a)] the surface area of the sphere is
lii

S = 277 ( r sin tlr dt


0

= 2rr2 4 ii

sin t dt (sin t 2 0 in [0, T I )


ii

= 2ar2[-cost] = 2Tr2[-(-1) - (-111 = 4Tr2 0


0

Compare the results of Example 4 with that of Example 3 as e + 0.

Example 5 Determine the area of the parabolic surface generated by rotating y = x 2


3/4 < y 2 6 about they axis. (See Fig. 12.17.)
Solution First solve for x, using the positive solution: x = g ( y ) = Now dx/& = 6.
J g(y) = 1 / ( 2 $ )is continuous on [3/4, 61, so by [12.2.6(b)] the area is

T 117
= - [ 1 2 5 - 81 = -T 0
Fig. 12.17 6 6
412 CHAPTER 12: PARAMETRIC EQUATIONS AND MORE APPLICATIONS OF INTEGRATION

PROGRESS TEST 2

Determine t h e areas of t h e surfaces generated by revolv-


ing t h e given curves a b o u t the given axes. 2. x =y3, O < y 5 2,y axis
1. y2 = 4x, 1 5 x 5 3, x axis
3. x = 2 +
cost, y = sin t , 0 2 t 5 i7,y axis

SECTIOK 12.2 EXERCISES

I n Exercises I to Is, ( a ) set up a definite integral that equals the length of the given arc and ( b ) evaluate that integral.

1. y = (1 - x2/3)3/2, 0 5x 5 1 2. y = lnx, 1 5x 5 fi
Y2 1 1
3. x = In(cosy), 0 < y 51 4. x = - - -lny, -
e
<y 5 1
4 2
5. x = t 2 , y = 3t, - 2 5t50 6. x =t + s i n t , y = 1 - cost, 0 5 t 5 7

7 . x = arcsin 4 ,y = ln dEZT, o 5 t 2 2 8. x = 3 sin3 t , y = 3 c0s3 t, 0 5t5 v


4
9. y = ln(x + d n ) ,3 5 x 55 10. gy2 = 4(x - 113, 1 5 54
11. y = -
X 64x+ 3 , 1 5 X 5 2
12. y = ln(sec x ) , o 5x 51
4

13. y = - x+4 - 1 1 1 x 5 2 14. y = arcsin(eP), 0 5 x 5 @


4 8x2
13. x = 3 s i n t c o s t , y = 3 s i n 2 t , 0 5 t < r r 16. y = t , x = ln(sec t ) , 0 5 t 5 v/4 (compare with
Exercise 12)

I n Exercises 1 7 to 31, ( a ) set up a dejnite integral that equals the area of the surface generated by revolving the given curve about the given axis and
( b ) evaluate that integral.

17. y = -(e
2
1
+ e c 5 ) from x = 0 to x = 2, x axis 24. x = 3t2,y = 2t3, 0 5 t 5 l , y axis
25. x 2 - y 2 = 1 between ( 1 , O ) and (4, fl),
y axis
18. y = sinx from x = 7i/4 to x = ~ / 2 ,x axis (a ring 26. 18x2 = y(6 - J ) ~between (0,O)to ( 0 , 6 ) , y axis
shape) 27. y = x4I3, 1 5 x 5 8, x axis
19. 4x = y 2 - 2 lny fromy = 1 t o y = 2,y axis 28. y = e-, 0 5 x 5 1, x axis
20. y = 2 c f r o m x = -1 to x = 0,x axis 29. x = (1/6)y3 + 1/(2y), 1 < y 2 2 , y axis
21. x = 2 c o s 3 t , y = 2 s i n 3 t , 0 ~ t ~ v , x a x i s 30. 8y2 = x 2 - x4, 0 2 x 5 1, x axis
22. x = 2t,y = t 2 , 0 5 t 5 4, x axis 31. x = t2/2,y = (1/3)(2t + 1)32: 0 5 t 5 4 , y axis
23. x = (1/2)t2 + l / t , y = 4 6 ,1 2 t 2 4, x axis

In Exercises 32 to 36 use the general surface area formula (12.2.8) to set up a dejnite integral giving the area o f t h e surface generated lp revolving the
given curve about the given axis.
32. y = x 2 / 1 6 - 2 lnx, 4 5 x 2 8 , y axis 42. Determine the area of the surface generated by revolving
33. 2x = y 2 - 2, o < y 5 2 . ~ 5x, axis y = lnx, l/e 5 x 5 e, about they axis:
34. y = x 3 / 3 +
(4x)-l, 1 2 x 5 2, the limy = - 1 ( a ) First, by using x as the independent variable.
33. 9y2 = x ( x - 3)2 between (0,O)and (3,0), the line x = 3 ( b ) Second, by usingy as the independent variable. (You
36. y = x 3 / 6 +
(1/2)x, 1 5 x 5 2, y axis will need to invert the equation, determiney limits of
37. Determine the length of one arch of the cycloid x = integration, and use a trigonometric substitution.)
t - s i n t , y = 1 - cos t. 43. A cone of radius T and slant height 1 has area vrl, so a
38. Determine the area of the surface generated by revolving frustum of slant height L as pictured has a lateral surface
one arch of the cycloid in Exercise 37 about the x axis. area that is the difference between the area of the whole
39. Repeat Exercise 38, but revolve about they axis. cone and that of the smaller cone of slant height l and
40. Use ( a ) the Trapezoid Rule and ( b ) Simpsons Rule radius r ; that is,
with 10 subdivisions to approximate the length of one
arch of the sine curve.
v R ( L + I ) - v r l = v(RI - rl + RL)
41. Repeat Exercise 40 to approximate the length of the arc +
( a ) Show that ( L l ) / R = l/r. ( b ) Use ( a ) to prove
y = x3/6 + x/2 from x = 0 to x = 1. (12.2.4). Use (12.2.5) to show that the right circular
SECTION 12.3: CENTROIDS 413

44. Show that the three definite integrals associated with


Fig. 12.12 are equal.
45. ( a ) Given a sphere of radius r generated by revolving
y = + 4- about the x axis, a spherical zone of
<
height h (h 2r) is generated by revolving the portion
ofy = d m t h a t lies above the interval [ - h/2, h/2].
Determine the surface area of this zone.
cone of radius r and height h generated by the line (6) Show that this surface area is independent of which
y = (r/h) x from x = 0 to x = h has lateral surface area interval of length h is chosen in [ -r, r]. (Think of what
7ir d m ; here v m is the slant height. [This this means if you pass parallel planes 1 cm apart through
shows that (12.2.5) is consistent with the assumption the earth and compare the zone near the equator with
about the area of a cone upon which it was based.] that near a pole.)

Centroids of Plane Regions, Curves,


Section 12.3: and Solids and Surfaces of Revolution

Objectives: MOMENTS
1. Calculate moments and centers Although most of the geometric applications in this section originated in
of mass for discrete point masses physics and engineering, special knowledge of these areas is unnecessary. Fur-
in the plane. thermore, we shall deemphasize this aspect of the topic as we progress (often
ignoring the units involved, for example). Instead, we shall stress the mathe-
2. Calculate moments and
matical essence of these ideas, which will then apply to other areas including
centroids of regions in the plane,
population distributions and statistics.
plane curves, solids of revolution,
Suppose a uniform bar is oriented along the x axis and balanced on a
and surfaces of revolution.
fulcrum situated a t the origin. If we hang a 25-lb mass at x = -4, where should
we hang a 10-lb mass to balance the bar? The answer, as most any schoolchild
would determine (such balance scales are often used in teaching arithmetic) is
at x = 10. We balanced the tendency of the bar to twist in a counterclockwise
direction about its fulcrum.

12.3.1 Definition Given a mass m concentrated at directed distance X from a given


point P, we define the moment of that mass with respect to P as the product mX. If
several masses are placed at different points, then the total moment of this system
with respect to P is the sum of the moments with respect to P.

In our example we needed to place the mass so that the sum of the moments
with respect to the origin was zero. Conversely, we could begin with a collection
of masses hanging from the bar and ask where the fulcrum should be moved so
that the bar balances.
Suppose a 20-lb mass is at x = -2 and a n 80-lb mass is at x = 3 (see
Fig. 12.18). Where is the balance point? This is the point where all the mass
could be concentrated without affecting the total moment of the system with
respect to the origin.
The sum of the moments with respect to the origin is
(20)(-2) + (80)(3) = 200
-3 -2 -1 0 1 2 3
.nce the total mass is 100 lb and the moment is the product of mass and
I I I I I I I .
directed distance from the origin, we have 1007 = 200, so X = 2, and the system
20 113 80 Ib behaves as if all its mass were concentrated at X = 2. This also means the system
Fig. 12.18 could be balanced a t X = 2.
414 CHAPTER 12: PARAMETRIC EQUATIONS AND MORE APPLICATIONS OF INTEGRATION

Quite obviously this discussion extends to any finite number of masses placed
along the x axis (the role of the bar is superfluous).

12.3.2 Definition Suppose we are given a system of point masses along the x axis.
The point X at which we could concentrate all the mass of this system without
changing the total moment of the system with respect to the origin is called its
center of mass and is computed by
- total moment
x =
total mass

Example 1 Given the system of masses 5, 25, 50, 10, and 30 at the points - 10, -2, 1, 5,
and 6, respectively, determine the center of mass of the system.
Solution The total moment is
(5)(-10) + (25)(-2) + (50)(1) + (10)(5) + (30)(6) = 180
and the total mass is
5 + 25 + 50 + 10 + 30 = 120
Hence the center of mass is

x = - =180
-
-
120
3
2
.
The above discussion regarding masses along a straight line generalizes to a
system of n masses in the plane, whose respective masses and positions are listed:

ml, ( ~ 1 , ~ 1m2, . . . ; m i , ( x i , ~ i ) ;. . . ; mn, ( X n , y n ) .


); ( ~ 2 > ~ 2 ) ;

The moment of a typical mass mi [located a t (xi ,yi)]about they axis is


defined to be the product of its mass mi and its directed distance xi from they
axis. Similarly, its moment about the x axis is defined to be miyi.

12.3.3 Definition Given the system of point masses in the plane as above, we define
the total moment of the system with respect to they axis to be
n
(a) M , = mixi
i=l

and the total moment with respect to the x axis to be


n
( b ) ME = C miyi
i=l

Given the total mass M = Z;=, m i , we define the center ofmass ofthis system to be
the point ( G , j ) where

Just as in the linear case, the center of mass is the balance point-the point
at which all the mass could be concentrated without changing the total moment
of the system. (A mobile with all its objects in one plane could be hung from its
center of mass.)
SECTION 12.3: CENTROIDS 415

PROGRESS TEST 1

1. (a) Determine the total moment with respect to the 3. Repeat Prob. 1 if a mass of 10 lb is added at x = 60.
origin of the four masses distributed as follows: 4, 6, 4. ( a ) Determine the center of mass of a system of four
7, 9 lb at x = - 6, - 2, 0, 8, respectively, 1-lb masses at the points (1,4), (2,5), (4, 7), ( 5 , 16).
( b ) Where is this system's center of mass? ( b ) Repeat ( a ) for a system of four 2-lb masses at the
2. Given the same masses in the plane with the same same locations.
respective x coordinates as in Prob. 1, and with y (c) Repeat ( a ) for a system of four k-lb masses at the
coordinates -1, 3, 2 , and 4, respectively, deter- same locations.
mine: ( a ) M y , ( b ) M E ,and (c) the center of mass
in the plane.

CESTROID OF A PLANE REGIOK

This work with discrete masses extends to thin plates of uniform thickness and
density. We assume that a plate is imposed on a region in the xy plane between
c, = (x: f *:I the graph of a continuous, nonnegative functiony = f ( x ) and a n interval [a, b ] .
We approximate the region with rectangles based on a partition P of [a, b ] ,
where in this case a typical rectangle over the subinterval [xZpl,xz] has height
f ( x : ) with x: = + x,)/2, the midpoint of [ X ~ - ~ , X , ] see
; Fig. 12.19.
If p is the density of the plate in mass per unit area, then the mass of a
rectangle is the product of p with its area. In this case the area isf(x:) Ax,, so
the mass is p f ( x : ) Ax,. We assume the mass of a typical rectangle to be
X
a 7,- x: x, b
concentrated at its center C, = (x:,f(x:)/2), so the moment of the rectangle
with respect to they axis is x:pf(x:) Ax,. We approximate the total moment of
Fig. 12.19 the plate with respect to they axis by the sum
n n

i =1 i =1

As usual, we take the limit as llPII + 0 to get a definite integral formula


for M y :

12.3.4 Definition M y = p sba


xf(x) dx

Under the same assumptions, the moment of a typical rectangle with respect to
the x axis is the product of the mass p f ( x f ) Axi and the distancef(xf)/2 of the
center from the x axis, ( f ( x f ) / 2 )p f ( x f ) Axi. We approximate the total moment
with respect to the x axis with the sum

and take the limit as IlPll + 0 to obtain:

12.3.5 Definition M , = -sb


P
2 a
[f(~)]~dx

The total mass M of the plate is the product of the density and area, and the
area is S : f ( x ) dx, so

M = p f f ( x ) dx = pA
a
416 CHAPTER 12: PARAMETRIC EQUATIONS AND MORE APPLICATIONS OF INTEGRATION

Using the above formulas we can now define the center of mass of the plate to
be the point (X,y)where:
b

M P xf(4 dx J k X ) dx
12.3.6 Definition X= = -

and

a
Since the constant density p cancels from the above formulas, just as the
constant mass did in Problem 4 of Progress Test 1, it is clear that the point ( K 7)
depends only upon the shape of the plate. The constant density, hence mass, are
unimportant to the discussion. Henceforth in this section, unless otherwise
noted, we shall ignore density and mass. (An essentially equivalent assumption
is to assume that p = 1 instead.) Thus the discussion focuses on the plane
region R defined by the graph of the function and the role of the physical plate
disappears. In these circumstances, and henceforth, we shall use common
terminology and speak of (F,Y) as the centroid ofthe region R. However, we shall
continue to speak of ((momentsand use the notation M y and M,.

Example 2 Determine the centroid of the region bounded by the positive semicircle
f(x) = d m and the IC axis (see sketch).
4

t Solution My = s r x d m d x
--r

The area A of the region is m r 2 / 2 , so


- 0 - 2r3/3 4
x=-=O and y=--- - r
A 71r2/2 371

(As we expect from the symmetry of the semicircle, the centroid is on t h e y


axis.) 0

The principles responsible for (12.3.6) can be used to compute centroids of


regions for which horizontal rectangles are more convenient.

Example 3 Determine the centroid of the region bounded by a n isosceles triangle of base 6
and height h.
Solution We center the region on they axis and base it on the x axis as indicated in the
accompanying sketch. To avoid breaking each integral into two parts, we use
horizontal rectangles with opposite ends on the legs of the triangle. Symmetry
with respect to the vertical axis tells us Mu = h- = 0, so it remains to deter-
mine M , .
SECTION 12.3: CENTROIDS 417

Y The equation of the right leg isy, = ( - 2h/b)x + h which, solved for x , gives
.c x,= -by/(2h) + b/2. The corresponding equation for the left side is xl =
by/(2h) - b / 2 , so the length of a typical rectangle is x, - xl = -by/h + b.
Hence the moment of a typical rectangle with respect to the x axis is
y:(-by:/h + b ) Ayi. Thus the total moment is

M, = d h y ( - h br + b ) + =bdh( -f+p)dy
= b [ - 3Yk3+ l ]Y2
, = b [ - 7h2 +I]
h2 = -bh2
6
Also, the area of the triangle is A = ibh. (This could also be computed using an
integral.) Therefore
- Mz bh2/6 bh2 2 h
Y =-
A
=- bh/2
-
6 bh
-- 0
3
--.--
You may recall from elementary geometry that the centroid of a triangle
is the intersection of its medians (a median joins a vertex with the midpoint of
its opposite side). You might wish to show that our centroid is in the same
place.

PROGRESS TEST 2

1. Determine the centroid of the region R bounded by


y = 4x - x 2 and the x axis

CENTROIDS OF SOLIDS OF REVOLUTION,


CURVES, AND SURFACES OF REVOLUTION
I
Having discussed one-dimensional moments about a point and two-dimen-
sional moments about a line, we use the same ideas to deal with three-dimen-
sional moments about a plane, beginning with the simplest type of three-
dimensional figures, solids of revolution. We shall define moments of such solids
with respect to planes perpendicular to the axis of revolution, which makes it
convenient to introduce a third coordinate axis called the z axis. It is perpen-
dicular to the xy plane, which means it comes out at you from the paper. Since
each pair of axes determines a plane, we have the xy plane, the x-z plane, and
they-t plane. (See Fig. 12.20.)

>
2
Now suppose the region R under the graph ofy = f ( x ) from x = a to x = b
Fig. 12.20
(wherey = f ( x ) > 0 and suitably integrable on [a, b ] ) is revolved about the x
J axis. The formula for the volume of the solid so generated is developed by first

I noting that a typical rectangle generates a disk of volume ~ [ f ( x 5 )Axi.


x r is the midpoint of [ X ~ - ~ , X ~ ] Again

xr *
. )
] ~ (Here
we assume the solid to be of uniform
density, so we can ignore density and mass. We define the moment of this disk
with respect to the y-z plane to be
v [ f ( x f ) I 2Axi
Note that the disk is parallel to they-z plane while the axis of revolution, the x
X
axis, is perpendicular to they-z plane. (See Fig. 12.21.)

12.3.7 D e f i n i t i o n The total moment of the solid about they-z plane is


?.
Fig. 12.21
418 CHAPTER 12: PARAMETRIC EQUATIONS AND MORE APPLICATIONS OF INTEGRATION

By symmetry the centroid will lie on the x axis. To determine X we need to


divide by the volume (instead of area as in two dimensions):

A similar formula can be developed using the shell approach. (See Exer-
cise 20.)

Example 4 Determine the centroid of the solid hemisphere generated by revolving about
the x axis the region under the positive semicirclef(x) = d
- from x = 0
to x = r.
Solution We can apply (12.3.8) directly:

The volume of the hemisphere is (2/3)mr3, so


J

t We shall now define the centroid of the plane curvey =f(x) from x = a to
x = b wheref exists and is continuous on [a, b]. (Think of the curve as a thin
wire of uniform thickness and density.) As we saw in Sec. 12.2, a typical
approximating chord has length d l +
[f(x:)I2Axi where x: is a point in
[ x i - l , xi]; see Fig. 12.22. We define the moments of this chord about they and x
Y , - ~ 2: xz b axes, respectively, to be
Fig. 12.22 +
x: ~ 1 [f(x:)] Axt and f(x8) d l + [f(x:)] Axi
Hence it makes sense to define the total moments about the two axes of the
curve y = f ( x ) from x = a to x = b as follows:

12.3.9 Definition M , =
s x d l + [f(x)I2 dx M, = l b
f(x)dl + [f(x)] dx
T h e centroid (F,?) is defined by

-
x=-
M, - M,
S
y=-
S
where s = 1dl + [f(x)I2 dx
If the same curve is rotated about the x axis, a surface of revolution results,
with area
b
A = 2m J f ( x ) d l + [f(x)] dx
a

It has its centroid on the axis of revolution so we need only define the moment
with respect to they-t plane-which, as you can surely predict, is
b
12.3.10 Definition M,, =

Hence the centroid is at


277
1 x .f(x)dl

(F,0) where X = M,,/A.


+ [f(x)I2 dx
SECTION 12 3: CENTROIDS 419

Example 5 Set up but do not evaluate simplified integrals giving the centroid of the curve
+
24xy = x4 48 from x = 1 to x = 3 . (The length of this curve was determined
in Sec. 12.2, Progress Test 1, Prob. 2, page 407.)
Solution Examining the solution to the arc length problem, we note that

and thus
M, = 1 3 x ( $ + -$)dx
and

Therefore,

Example 6 Set up but do not evaluate simplified integrals givihg the centroid of the surface
generated by revolving the curve of Example 5 about the x axis.
Solution We can apply ( 1 2 . 3 . 1 0 ) dihectly, using information noted in Example 5 :

M,, = 2 c 4 3 x ( $ x 3 + z)($ + $ ) d x
X

We note from Example 5 that the surface area is (2c)M,, so

PROGRESS TEST 3

i by revolving the region bounded by hy = rx from


h,
x = 0 to x = h about the x axis is at X = ( 3 / 4 ) h .
3. Show that the centroid of the conical surface gener-
ated by revolving the curve hy = rx from x = 0 to
x = h about the x axis is X = ( 2 / 3 ) h .
4. Define the centroid of the surface generated by re-
-
L X

volving the graph of a function x = g ( y )about they


axis fromy = c toy = d. (9 is continuous on [c, d ] . )

PAPPUS THEOREMS
It was stated earlier that moments and centroids have wide application in
science and engineering. in the life and social sciences, and in statistics. Here,
however, we shall consider a different sort of application, named after its third-
century A.D. Greek discoverer, Pappus of Alexandria.

12.3.1 1 Pappus Theorem for Volumes Supposef(x) 2 0 on [a, b ] and A is the


area of the region R bounded by f( x ) , the x axis, and the lines x = a and x = b.
420 CHAPTER 12: PARAMETRIC EQUATIONS AND MORE APPLICATIONS OF INTEGRATION

If (F,?) is the centroid of R, then R generates a solid of volume 27TA when


revolved about they axis, and a solid of volume 2.54 when revolved about the
x axis. More generally, if a region R is revolved about some axis, then the
volume of the resulting solid is the area of R times the circumference of the
circle traveled by the centroid of R.
Prooj The proof of this useful result is surprisingly simple. Consider 2.54. By
(12.3.6),

so

But this last formula is of course the disk formula (6.6.1) for the volume of
the solid generated by revolving R about the x axis. W

Other versions of Pappus Theorems are proved similarly. (See the Section
and Chapter Exercises.)

Example 7 Determine a formula for the volume of a torus with cross-sectional radius r and
<
central radius R, r R. (See Fig. 12.23.)
4 Solution Such a torus can be generated by revolving the circular disk D bounded by

t ( x - R ) 2 + y 2 = r 2 about they axis. The centroid of D is at its center ( R ,0) at


distance R from they axis. Since the area of D is v 2and the centroid travels a
circumference of 2aR, the volume of the torus is
(2vR)(ar2)= 2 7 ~ ~ R r ~
(This result agrees with those of Sec. 6.6 Exercises 37 and 38, page 228.) 0

Notice that Pappus Theorem could be used to determine the centroid of a


region of known area if we know the volume of the solid it generates when
Fig. 12.23 revolved about some axis. (See Exercises 49 to 51.)

MOMEKTS OF INERTIA
The product mx of a mass m and a distance x from a given point is sometimes
called the first moment of that mass about the given point. The product mx2
is called the second moment or moment of inertia about the given point. While we are
ignoring the units involved, the sum of second moments of a system of masses
measures the resistance of that system to rotation about the reference point as
well as its resistance to change in velocity when in motion. Note that a pair of
nonzero masses whosejirst moments add to zero has nonzero second moment. (A
perfectly balanced barbell resists rotation about the balance point.)
Most of the formulas developed in this section for uniform solids and surfaces
have analogs for second moments, with the idea of centroid replaced by radius of
gyration. However, care must be taken with the density constant, because it does
not cancel from all the formulas as it did for centroids. These ideas are explored
in Chapter Miscellaneous Exercises 10 to 24. Third and higher moments, used
in statistics, are defined analogously. In Chap. 20 we shall also describe situa-
tions where the density is a variable.
SECTION 12.3: CENTROIDS 42 1

SECTION 12.3 EXERCISES

1. Given the masses 10, 20, and 3 0 lb a t ( 2 , 3), (5,9), and 2. Repeat Exercise 1 for the masses 15, 10, 5, and 3 0 kg at
(2,5), respectively, determine ( a ) M,, ( b ) M,, and the points ( - 3 , 2), (4, 12), (1, I), and ( 3 , - 15), respec-
( c ) the center of mass. tively.
In Exercises 3 to 1 5 ( a ) set up integrals and quotients giving the centroid of the region R bounded by the given curves; ( b ) evaluate these expressions,
giving numerical answers.
3. x +
y 2 = 4 and t h e y axis 4. x + y 2 = 4 and x = y 2 +
5. y = x - 3 x 2 - x +
3 and the x axis from x = -1 to 6. y = x3 - 3 x 2 - x + 3 and the x axis from x = 1
x = l tox=3
7. y=x - 3x2 - x + 3 and the x axis 8. y = x 3 , the x axis, and x = 1
9. y = In x , the x axis, and x = e 10. y = x 2 and x = y 2
11. y = sinx and the x axis from x = 0 to x = 7 / 2 12. y 2 = x and 2y +
3 =x
13. +
y + 4 = x2 andy x 2 = 2x 14. y = ex + e+ and the x axis from x = -1 to x = 1
15. y = (lnx)/x, the x axis, and x = e
16. Prove that the centroid of a rectangle is its center. (Let axis, y = f ( x ) , x = a, and x = b (f( x ) 2 0 on [a, b ] ) is
the rectangle be bounded by x = 0.x = a , y = 0,y = b.) revolved about they axis. Thus a typical vertical rectan-
17. Prove that the centroid of a circular disk is its center. gle generates a shell whose centroid is at (0, f (xi )/2) and
18. ( a ) Supposef(x) 2 g ( x ) 2 0 on [a, b ] . Determine a for- whose moment with respect to the X-tplane is the prod-
mula for M,, the moment with respect to the x axis of uct of its volume (ignoring density as usual) and the
the region R between the graphs off ( x ) andg(x) on [a, b ] . distance of its centroid from the x-z plane.
( b ) Determine the moment M-, of the region R of part ( a ) Determine the total moment of the solid with respect
( a ) about the linep = -4. to the x - t plane.
19. Determine a formula for the solid generated by revolving ( b ) Determine the centroid of the solid.
the region R of Exercise 18 about the x axis. 2 1. Extend the formulas of Exercise 20 to the solid generated
20. (Shell Method) Suppose the region bounded by the x by revolving the region R of Exercise 18 about they axis.
In Exercises 22 to 33 determine the centroid of the solid of revolution generated lp revolving the given region about the given axis. You will need to use
the results of previous exerczses in some cases.

22. R as in Exercise 3 about t h e y axis 23. R as in Exercise 5 about the x axis


24. R bounded b y y = x 4 a n d y = 2 x - x 2 about t h e y axis 25. R as in Exercise 24 about the x axis
26. R as in Exercise 5 about t h e y axis 27. R as in Exercise 6 about t h e y axis.
28. R bounded b y y = x 2 a n d y = x 3 about the x axis using 31. R as in Exercise 28 about the y axis using the shell
the disk method method
29. R as in Exercise 28 about the x axis using the shell 32. R as in Exercise 4 about t h e y axis
method (see Exercise 20) 33. R as in Exercise 5 about the liney = - 3
30. R as in Exercise 28 about the y axis using the disk 34. Determine the centroid of a frustum of a right circular
method cone with smaller radius r, larger radius R, and height h

In Exercises 35 to 4 4 determine the centroid of the given curve segment. Exercise numbers refer to Section 12.2 Exercises, page 412.
35. Exercise 1 36. Exercise 2 37. Exercise 3 38. Exercise 4
39. Exercise 11 40. Exercise 12 4 1. Exercise 1 3 42. Exercise 14
43. 12x3 = 4y4 + 3 fromy = 1 t o y = 3 44. x2 = 4y from (-2, 1) to (2, 1)
45. Determine a formula for the centroid of the surface 48. Repeat Exercise 47 with the curve revolved about the
generated by revolving the graph ofy = f ( x ) from x = a liney = -2.
to x = b about the liney = k. Assume that f ( x ) 2 k and 49. Use Pappus Theorem for Volumes and the fact that the
fis continuous on [a, b]. area of a semicircular region of radius r is ( . ; i / 2 ) r 2 to
46. Determine a formula for the centroid of the surface determine its centroid.
generated by revolving the graph ofy = f ( x ) from x = a 50. State and prove a Pappus Theorem for surface areas that
>
( a 0 ) to x = b about they axis. Assume f is continu- will give the surface area in terms of arc lengths and the
ous and positive on [a, b]. distance traveled by the centroid of the curve.
47. Determine the centroid of the surface generated by 51. Use the result of Exercise 50 and the fact that the arc
revolving about the x axis the curvey = x 4 / 4 l/(8x2) + length of a semicircle is 7ir to determine the centroid of a
from x = 1 to x = 2. semicircular curve.

In Exercises 52 to 57 determine the centroid of the suface of revolution whose area is requested in the given exercise from Section 12.2 Exercises,
52. Exercise 18 53. Exercise 19 54. Exercise 20 55. Exercise 27
56. Exercise 28 57. Exercise 29
422 CHAPTER 12 PARAMETRIC EQUATIONS AND MORE APPLICATIONS OF INTEGRATION

Section 12.4: WORK AR-D FLUID PRESSURE

Objectives: WORK
1. Define a definite integral giving To do work is to use energy, so it is important to have a good quantified
the total work done by a definition of work. A starting point is provided by the following classical
continuously variable force acting definition from physics:
over an interval.
12.4.1 Definition The work W done by a constant force F acting
2. Define a definite integral giving
through a distance x along a straight line is defined to be the product Fx.
the total force exerted by a fluid on
a vertical submerged surface.
Generally, the work unit is the product of a distance unit and a force unit.
3. Use centroids to simplify or As long as the force is constant, such as in lifting a fixed weight a short
avoid the calculations involved in distance (compared to the radius of the earth), we do not need calculus to
objectives 1 and 2. compute the work done; arithmetic suffices. We are interested here in defining
and computing work done by a force that vanes as it acts over some given
distance. Hence we assume that the variable force acting from a to b is described
by a continuous function f ( x ) , where x varies from a to b. Since the definition
(12.4.1) only applies to a constant force, we make recourse to the now-standard
approach of breaking the interval [a, b ] into n subintervals based on a partition
P and approximating the force on each subinterval [xz-1, x, ] by a constant
f(x,). Then an approximation to the actual work done on the subinterval is
given by
Y = f ( x z ) Ax,
We add these approximations to get an approximation of the total work done
over [ a , b ] : B r = l f ( x z )Ax,. The smaller each subinterval, the less the continuous
function f ( x ) can vary on each subinterval, so the better the approximation.
Then, as usual, we take the limit of the Riemann sums as 11 PI1 + 0, resulting in
a definite integral.

12.4.2 Definition We define the total work done by the continuously varying force
f ( x ) from x = a to x = b by

W = lim f ( x i 1 Axi = J b f ( x )dx


IIPII-0 i=l a

The simplest and most commonly used illustrative example of a variable


force acting over a n interval is that required to stretch or compress a simple
Y
spring over a fixed distance. Suppose a simple spring is oriented horizontally
along the x axis fixed at its left end with its right end at x = 0; see Fig. 12.24.
Fig. 12.24 The seventeenth century English physicist Robert Hooke discovered that within
certain bounds (as long as the spring can return to its natural length), the force
necessary to stretch the spring is directly proportional to the distance it is
stretched. Thus for some constant k , the elasticig of the spring, the force is given
by kx.

Example 1 Suppose the elasticity of a spring is 4 lb/in. How much work is done in
stretching the spring 6 in. beyond its natural length? How much additional
work is done in stretching the spring an additional 6 in.?
Solution We orient the spring as in Fig. 12.24. The force necessary to hold the spring
stretched at x inches beyond its natural length isf(x) = 4x.Hence, by (1 2.4.2),
the work done in stretching it the first 6 in. is
SECTION 12.4: WORK AND FLUID PRESSURE 423

while the work done in stretching it the second 6 in. is

If
12
4xdx = 2x2 = 216in.-lb 0

Notice that the elasticity of a spring can be inferred from simple data, such as
the fact that it takes a force of 10 lb to hold a spring stretched 2 in. We could
conclude that the elasticity is k = 5 since we know from Hookes Law that
10 = k 2.

Another sort of work problem involves lifting. The following two examples
are intended to illustrate the general approach:
Break the work into simple parts, add these up, and take a limit to d@ne the integral.
The first shows that we can break the work into pieces by breaking what is being
lifted into pieces, and the second shows that we can break the work into pieces
by breaking the distance into pieces (as with the spring above).

Example 2 Determine the work done in pumping all the water from a vertical cylindrical
tank 6 ft in diameter and 10 ft tall to a height 8 ft above the top of the tank.
Solution The key is to realize that each thin horizontal slab of water must be lifted a
different distance. (See Fig. 12.25.)
Measuring x from the top of the tank downward, a typical slab of thickness

t
Axi has volume 7 ~ Ax 3 i~. The density of water is about 62.4 lb/ft3, so its weight
is 62.4(7~3~Axi). (In the following Progress Test 1 we shall consider a conical
I 8
tank, where slabs have varying radius.) This slab needs to be lifted (xi + 8) ft,
so the amount of work needed to do this is about

( 6 2 . 4 ) ( ~* 32 A x i ) [ x i + 81 = (62.4)9m(xi + 8) Axi
As usual, we add up these little works based on the slabs to approximate the
total work and then let the thickness of the slabs approach zero to get a definite
integral:

+ 8) dx = ( 6 2 . 4 ) 9 ~
(62.4)97~(~

= (62.4)9~(130)=: 229,361.4 ft-lb


Note that we added slabs from x = 0 (the top of the tank) to x = 10 (the
Fig. 12.25 bottom). Hence the limits of integration 0 and 10. 0

We should point out that this solution could have been organized a bit
differently, but with the same final answer. For example, instead of saying
x = 0 is a t the top of the tank, we could have chosen x = 0 to be the destination
of the water. Then a typical slab would be lifted xi feet, but we would add up
work based on slabs from x = 8 (top of tank) to x = 18 (bottom). This yields
the integral

~ ~= (62.4)97~
( 6 2 . 4 ) 9 dx

= (62.4)9~[162- 321
= (62.4)97~(130)
as before.
In general, the way we choose coordinate axes in work problems is unim-
portant provided we are consistent after making the choice.
424 CHAPTER 12: PARAMETRIC EQUATIONS AND MORE APPLICATIONS OF INTEGRATION

Example 3~
A 60-ft chain weighing 1.5 lb/ft is hanging freely off a bridge (see sketch), How
much work is done in pulling the chain up onto the bridge?
Solution The key is to realize that as the chain is pulled up, the amount of force needed
decreases. If x feet are hanging, the force necessary to pull the chain a small
distance A x is about 1 . 5 lb.
~ Hence the work thus accomplished is 1 . 5 Ax.
~ Now

1-
0 5 x 5 60, so the total work is
3 (60
f60 ~ = -x2
1 . 5 d~ = 2700 ft-lb 0
I JO 4 10

i This chain-pulling problem can be analyzed from another point of view to


yield the same result. Instead of thinking that we are pulling x feet of chain A x
feet, we can think of the chain as being made of short pieces (which, of course, it
is) of length Axi. Then such a piece, weighing (1.5) Axi is pulled xi feet, so the
work done in pulling each Axi length of chain to the bridge is 1.5xi A x i . We then
add up such little works from x = 0 to x = 60 to get 2 1 . 5 Axi,
~ ~ and pass to
the definite integral as usual to obtain the same integral as before.

Note that power is work per unit time. In United States customary units, for
example, 550 ft-lb/s is one unit of power-called the horsepower (hp). A unit
of power in the metric system is the watt (W). One horsepower equals 746 W.
See Exercises 8 and 9 for further discussion.

PROGRESS TEST 1

1. Find the work done in stretching a spring from 2


in. to 8 in. if it takes 30 lb to hold it stretched
2 in.
2. By following the indicated steps, determine the work
done in pumping the oil out of the top of a conical \ ! / I
tank 12 ft in diameter at the top and 18 ft deep. The
oil weighs 50 lb/ft3.
( u ) Determine the weight of a slab of thickness Ay,
which is at distance 18 - y from the top of the tank.
(See sketch.)
( b ) Determine the work done in lifting that slab of
oil.
(c) Determine the total work done in pumping out 3. Determine the work done in lifting a 400-lb weight
the tank. 100 ft with a chain that weighs 5 lb/ft.

FLUID PRESSURE
The total force on the horizontal flat bottom of a full 6-ft-deep7 10- by 20-ft
rectangular swimming pool is the total weight of the water; that is,
-
(62.4)(10 * 20 6) = 74,880 lb
Everything is constant in this problem, so arithmetic, not calculus, suffices to
determine the total force. The opposite is the case when we attempt to deter-
mine the force on one vertical end of the pool, because the force per square foot
varies continuously with the depth. We digress briefly.
Force per unit area is usually called pressure. It is, in effect, the weight of
10 a column of fluid above a square unit of area. Most liquids, such as water, are
incompressible, and thus they have a constant density, say p . This means that
Fig. 12.26 the pressure at depth d is merely the density p times the volume of the column:
SECTION 12.4: WORK AND FLUID PRESSURE 425

We shall use one more basic fact about fluid force first made explicit by the
seventeenth century French mathematician Blaise Pascal: Within any fluid, the
force exerted at each point is equal in all directions.
Returning to the pool, we can now conclude that the pressure at any level on
the vertical face of the pool is 62.4 times the depth at that level. As usual we
impose a coordinate system on the situation. (See Fig. 12.26.) The pressure at
depth 6 - y i is 62.4(6 - yi)and, although the pressure changes with the depth,
Ix
Fig. 12.27
we can assume that the pressure is constant on a narrow horizontal strip of
thickness Ayi straddling the depth 6 - yi. Under this assumption, the total force
on the strip is approximately the product of the pressure per unit area and the
total area:
[62.4(6 -yi)][lO Ayi] = 624(6 - y i ) Ayi
If we were to add all these approximations based on a partition P of the vertical
interval [0, 61 into n such subintervals, we would have a n approximation of the
total force on the end wall of the pool:
n

As usual, we take the limit as llPll + 0 to obtain a definite integral that


describes the total force exactly:
+20+ 6

Fig. 12.28 624 (6 - y ) 4 = 11,232 lb


0

Notice that this computation involved only the end wall of the pool and the
depth of the water, but no other physical features. Hence this could also have
been the computation of the force on the vertical face of a dam of the same size
a t the end of a 100-mi-long canal, or even the computation of the force on a
floodgate on top of a much larger dam.
But, dams or pools, we use the same routine: Take a problem whose crucial
element (pressure in this case) is a variable, break it into small enough parts where we can
assume the element is constant, add the parts to approximate the whole, and then take the
10, 10
limit as the site of the pieces approaches 0 to obtain a precise definite integral description.
This previous fluid pressure problem generalizes directly to the force exerted
:
by a liquid of constant density p on a vertical surface whose shape is defined by
x = -10

Fig. 12.29
I x = 10 functions x = g ( y ) and x = f ( y ) from y = c t o y = d with g ( y ) 2 f ( y ) and
continuous on [c, d ] , as in Fig. 12.27. Assume that the liquid reaches level I 2 d
on the superimposedy axis. Then the total force is given by

12.4.3

Example 4 Determine the total force on a dam whose inside vertical face is shaped as in
Fig. 12.28, with water reaching to its top.
Solution We impose a coordinate system with the bottom of the dam resting on the x axis
and with they axis on the vertical centerline. (See Fig. 12.29.) Fromy = 0 to
y = 10, the width of a typical horizontal strip is 20. Fromy = 10 toy = 15 the
boundary at the right has equationy = x , s o f ( y ) = y , and at the left it has
equationy = - x , so g ( y ) = -y. Hence a typical strip has area
426 CHAPTER 12: PARAMETRIC EQUATIONS AND MORE APPLICATIONS OF INTEGRATION

Over both parts of the dam, a typical strip has depth 15 - y , so the total
force is
10 15
F = 62.4 20(15 - y ) dy + 62.4 2y(15 - y ) dy
10

+-
875
3 1 = 143,000 lb 0

In doing problems of this type, we strongly suggest that you not attempt to
apply (12.4.3) directly but that you make a fresh approach to each problem,
beginning with a sketch and a n appropriate choice of coordinates.

PROGRESS TEST 2

1. Determine the total force on the vertical end of a across the top, and filled with a liquid of density
symmetric triangular trough that is 6 ft deep, 4 ft 50 lb/ft3.
2. I X Suppose 3-ft flashboards are added to the top of (c) What is the force added to the original part of the
the dam in Example 4, surmounting the dam with a dam because of the flashboards? What percentage
3- by 30-ft rectangle, and suppose water rises to the increase is this? (This problem illustrates why lower-
top of the flashboards. ing the water level behind a weakened dam is so
( a ) What is the total force on the flashboards? crucial.)
( b ) What is the total force on the whole dam, in-
cluding the flashboards? [Note that it is not the sum
of ( a ) and the result of Example 4.1

USING CENTROIDS IN WORK AND FLUID PRESSURE


PROBLEMS: PAPPUS-TYPE THEOREMS
From our study of centroids-or centers of mass when physical quantities are
involved-we know that a thin, homogeneous plate balances on its cen-
troid-or center of mass. A similar statement holds for solids. In essence, the
centroid point is a point that can represent the whole plate or solid; it is a kind
of average point that for some purposes can stand for all the others. Recalling
from Sec. 6.4 the relation between integrals and averages, the use of the word
and idea of averages should not be entirely surprising. It has some useful
consequences as well.
Consider the work done in pumping the water from the cylindrical tank in
Example 2 to a height 8 ft above the top of the tank. (See Fig. 12.25.) The tank
is 10 ft tall, so its centroid is 5 ft above the bottom. The radius of the tank is
-
3 ft and thus its volume is 7 ~ 310~ = 90m ft3, so the total weight of the water is
(62.4)(907~).If all the water were concentrated at the centroid, then the work
done in lifting it the 13 ft to 8 ft above the top of the tank is
(62.4)(904( 13) = 229,361.4 ft-lb
which is exactly the result of Example 2.
A similar analysis applies to Example 3, where we determined the work done
in pulling a hanging 60-ft chain onto a bridge. We could have regarded all the
weight of the chain as being concentrated at its centroid-which happens to be
its midpoint in this case: 30 ft below the bridge. The total weight of the chain is
(1.5)(60) = 90 lb and is thought of as being lifted 30 ft, yielding a total work of
(90)(30) = 2700 ft-lb, just as before.
We can use similar thinking to determine the total force on one side of a
submerged vertical surface. We think of the entire surface as represented by its
SECTION 12.4: WORK AND FLUID PRESSURE 427

centroid and compute the pressure at that depth. Then the total force is the
product of the total area and the pressure at the centroid.
The force on the 10-ft-wide, 6-ft-deep rectangular end wall of the swimming
pool discussed turned out to be 11,232 lb. Using centroids, we note that the area
is 60 ft2,the centroid is at depth 3 ft, and the pressure at 3 ft is (64.2)(3),and so
the total force should be, as expected,
(62.4)(3)(60) = 11,232 lb
In general, the choice whether or not to use the centroid approach in work or
pressure problems depends upon whether the centroid is known, which is the
case for most common symmetric shapes.

PROGRESS TEST 3

1. Use the centroid result of Sec. 12.3, Progress Test 3, 3. Use the centroid result of Sec. 12.3, Example 3, to
Prob. 2 to determine the work done in pumping the determine the total force on the vertical end of a
oil of density 50 lb/ft3 out the top of an 18-ft-tall 6-ft-deep symmetric triangular trough 4 ft across the
conical tank of radius 6 ft (vertex down). Compare top filled with a liquid of density 50 lb/ft3. Compare
with Sec. 12.4, Progress Test 1, Prob. 2. with Sec. 12.4, Progress Test 2, Prob. 1.
2. Repeat Prob. 1 with the tank upside down (vertex
UP).

SECTION 12.4 EXERCISES

I n Exercises I to I2 determine the work done in accomplishzng the given task.


1. Pumping the contents of a half-full cylindrical water (6) If a l/lO-hp pump does 55 ft-lb of work per second,
tank to a level 5 ft above the tank. T h e tank is standing how long will it take this pump to empty the trough?
on its end, has a radius of 6 ft and a length of 8 ft. (c) Assuming a 50 percent efficient electric pump (only
2. Pumping full of water a conical tank through a hole in half the power fed in is put out by the pump), what
the bottom. The tank is 15 ft deep and 10 ft in diameter power source must be provided (in terms of watts) to
across the top. empty the trough in 1 min?
3. Lifting a 200-lb crate a height of 60 ft with a 2-lb/ft 9. Repeat Exercise 8 for a hemispherical tank (flat on top)
chain, 15 ft of which are used in wrapping the crate. 20 ft in diameter that is to be emptied through a dis-
4. Pumping a cylindrical tank full of 60-lb/ft3 oil through charge pipe at the top of the tank.
the bottom of the tank. Its radius is 9 ft, its bottom is a 10. Lifting a 500-lb anchor from a depth of 50 ft to a deck
hemisphere, and its cylindrical height is 10 ft. (This will 10 ft above the water using a cable which weighs 2 lb/ft
require two integrals.) out of the water and 1.5 lb/ft in the water. The anchor
5. Stretching a spring from 5 in. to 8 in. if it takes 3 Ib to weighs 400 lb in the water. (Break this into several parts.
hold it stretched 3 in. You can assume that the entire anchor passes through
6. Filling the tank in Exercise 4 through a n opening at the the water surface instantly.)
top from a source at the level of the bottom of the tank. 11. Lifting a 2-lb/ft chain that hangs doubled from a build-
7 . Lifting a leaky pail of water 100 ft out of a well with a ing down to a level of 100 ft from the top by ( u ) pull-
2 lb/ft cable. The pail begins with 50 lb of water and ing on one side of the chain and ( b ) pulling on both
leaks at a rate of 1 Ib every 4 ft. sides simultaneously.
8. ( a ) Emptying the water from a full 12-ft-long trough 12. Stopping a 4000-lb automobile that is traveling at
through a pipe 3 f t above the top of the trough. T h e 88 ft/s (60 mi/h). T h e brakes provide a deceleration of
vertical cross sections are isosceles trapezoids 2 ft wide a t - 10 ft/s2. (Note: Force equals the mass of the vehicle
the bottom, 6 ft wide at the top, and 2 ft deep. times the acceleration.)

In Exercises 13 to I7 determine the total force on the indicated vertzcal surface.


13. ( a ) A 4-ft-wide, 6-ft-tall rectangular exit gate whose top 14. T h e end of a water-filled trapezoidal-cross-sectional
is 24 ft below the surface of water. water trough 2 ft wide at the bottom, 4 ft wide at the
( b ) Same as ( u ) but with the water lowered by 12 ft. top, and 3 f t deep.
428 CHAPTER 12: PARAMETRIC EQUATIONS AND MORE APPLICATIONS OF INTEGRATION

15. (a) The outer surface o f a 2-ft-diameter circular subma- 18. How much must the water behind the dam in Exercise
rine porthole whose center is 1000 ft below the surface of 17 be lowered to cut the force on the dam in half?
the ocean (assume that the density of seawater is 64
lb/ft 3).
( b ) What is the average pressure per square inch on this
surface?
+no+ +
(c) What is the total force on the top half of the win-
dow?
(d) What is the total force on the bottom half of the
window? [Use part (a).]
16. T h e end of a water trough, with a triangular (isosceles)
cross section, that is 4 ft deep and 4 ft across the top.
17. The end of a symmetric dam as pictured in the sketch.

I n Exercises 19 to 28, use the centroid approach to solve the corresponding exercise.
19. Exercise 1 20. Exercise 2 21. Exercise 4 (do each part separately and add them)
22. Exercise 6 23. Exercise 13 24. Exercise 14 25. Exercise 15(a)
26. Exercise 15(c) 27. Exercise 17 28. Exercise 16
29. Ignoring the weight of the tank, determine the work
done in standing a cylindrical tank on end if the tank is
horizontal, full of 60-lb/ft3 oil, and 30 ft long, and has a
diameter of 10 ft. force of ele2/n2, where x is the distance between them. If
30. From Newtons Law of Gravitation it can be shown that our particle of 3 units of charge is fixed at the origin,
the force of gravity exerted on a n object which weighs how much work is done in moving a particle of opposite
w lb at the earths surface is (4000)2m/x2,where x is the charge of 4 units from x = 1 to x = lo?
number of miles from the center of the earth. (The 33. What is the total work done by a 200-lb man who climbs
radius of the earth is 4000 mi.) Determine the work done a ladder to a 50-ft-high platform carrying the end of a n
(in foot-pounds) in lifting a payload of 10 lb to a height uncoiling rope (of length greater than 50 ft) that weighs
400 mi above the surface of the earth. 1 lb/ft?
3 1. Suppose the rocket doing the lifting in Exercise 30 be- 34. Same as Exercise 33, but here the rope is only 30 ft long.
gins with 200 lb of fuel and consumes this linearly (with 35. Same as Exercise 33, but here the rope is 50 ft long and
respect to distance x), exhausting the fuel supply at dangling from the platform. H e carries the loose end up
x = 4400. What amount of work was expended in lifting while the other end is attached to the platform.
the fuel. (Note x = 4000 at liftoff.) 36. A vertical a by b rectangular plate has its top edge at the
32. Assuming appropriate units, two particles of opposite surface of a liquid. Determine how much the plate
electrostatic charges el and e 2 attract each other with a should be lowered to double the total force on the plate.

CHAPTER EXERCISES

R e v i e w Exercises
I n Exercises I to I7 graph the given pair of parametric equations.

1. x = 3 + 2 s i n t , y = - 2 + 4 c o s t : 0 5 t 5 2 7 7 2. x = It1,y = t
3. x = sin 2t,y = 2 sin2 t, O 5 t 5 2 7 4. x = 2 + sect, y = 3 + 2 tan t, --a/2 <
t < 77/2
5 . x = 3et,y = 1 - e t 6. x = 1 - 3 t , y = 2 + 7t
7. x = 1 +
l/t,y = t - lit 8. x = -1 + 3 s i n s , y = 2 + ~ C O S S 0
, 5 s 5

9. x = 2 + t2/2.y = -1 + t3/8 10. x = t,y = t 4 + 1


11. x = t 2 + t + l , y = t 2 / 2 + t - 1 12. x = t , y = l / t
13. x = 2t +
1,y = h ( t 2 +1) 14. x = 3 sec t , y = 2 tan t, 7;/2 < t < 3 ~ / 2
15. x = t +
1,y = t3 16. x = In t , y = e4f
17. x = c o s t , y = - 3 +
s i n t , -77/2 5 t 5 37;/2

In Exercises 18 to 21, (a) show that the graph Ofthe given parametric equations is a subset Ofy = x 2 - 1, (b) determine the slope Ofthe line
tangent to the graph at t = I, and (c) give the equation of this line in point-slope form.
18. x = s e c t , y = tant 19. x = l / t , y = (1 - t 2 ) / t 2
20. x = e-t,y = e-2t - I 21. x = s i n t , y = -cos2t
SECTION 12.4: WORK AND FLUID PRESSURE 429

In Exercises 22 to 28 determine an equation for the tangent line to the graph at the indicated point
22. x = sec t , y = - tan2 t , t = 77/4 23. x = c0s3 t , y = sin3 t, t = 7 / 6
24. x = e t / ( l+et),y = 1 +
2et, t = 0 25. x = 2t2 + +
t 1,y = 1 - 3t, t = 0
26. x = 2 t / ( l + +
t 2 ) , y = ( 1 - t2)/(1 t2), t = 1/2 27. x = c os t , y = 1 +
sint, t = 7i/4
28. = t2 t + + i , y = ( t - 1)3, t = 3 29. Find all values o f t where the curve x = C O S t~, x = sin4 t
30. Repeat Exercise 29 for the curve x = sin3 t , y = c0s3 t , has vertical and horizontal tangents, - 2 a 5 t 5 27.
-2977 5 t 5 27.
In Exercises 31 to 4 5 determine the length of the given arc.
31. x = t , y = 2t3'2, 0 5 t 5 4 32. x = tZ,y = t - 1, -1 < t 51
33. o
= 2t3,y = 3 t 2 , 5 t 5 1 34. x = t , y = cosht, - 1 5 t 5 1
35. y = ln(1 - x 2 ) , 0 5 x 5 1 / 2 36. 169 = x 2 - 32 lnx, 4 < x 58
37. x = et sin t , y = et cos t, 0 5 t 2 'ii 38. x = 2 sin t , y = 2 cost, 0 < < t 7

39. x = ln(sint),y = t +
1 , 7i/4 5 t 5 a / 2 40. +
6 x = y 3 3/y, 1 < y < 3
41. f ( y ) = ln(cscy), a/6 < y <
7/4 42. <
f ( x ) = In[coth ( x / 2 ) ] , 1 5 x 2
43. x = t cos t, y = t sin t, 0 I :t 5 7 44. x = t / 2 , y = t4/4, 1 5 t 5 2
45. x = t2/4 +(1/2)t,y = 2 f i , 1 5 t 5 4

In Exercises 4 6 to 56 determine the area of the surface generated by revolving the given arc about the given axis.
46. x = t 2 , y = 2t, 0 5 t 5 fi,x axis 47. x = 2 C O S ~t , y = 2 sin3 t, O 2 t 5 7 / 2 , x axis
48. The arc in Exercise 3 6 , y axis 49. The arc in Exercise 36, x axis
50. 3y = x 3 , 0 5 x 5 2, x axis 51. 3y = x 3 , 0 5 x <
2 , y axis
52. xy = 1 from ( 1 , 1) to ( 2 , 1 / 2 ) , x axis 53. y = lnx, 1 5 x 5 5 , y axis
54. x = 2 In t , y = t 2 , I <t 5 2, x axis 55. x = cos20,y = ( 1 / 2 ) sin20,' 0 2 0 5 7/4, x axis
56. x = 3 t 2 , y = 2t3, 0 5 t 5 1,y axis

In Exerczses 57 to 8 7 rejeer to the diagram shown. Set up, but do not evaluate, integrals giving the requested quantity in Exercises 57 to 8I
57. T h e length ofy = x 2 from x = 0 to x = 1.
58. The length o f y = x 3 from x = 0 to x = 1.

The area of the surface generated by revolving [from (0, 0 ) to ( I , I )]:


59. y = x 2 about the x axis 60. y = x 2 about they axis
61. y = x 3 about t h e y axis 62. y = x 3 about the x axis
63. y = x 2 about the h e y = 1 64. y = x 2 about the line x = 1
I WX
In Exerczses 65 to 81 give each coordinate of the centroad by dzvidang the integral that gives the
appropriate moment by the approprzate symbol A, V, s, or S (area, volume, arc length, or surface area,
respectively).
65. Centroid of I 66. Centroid of I11 67. Centroid of I1 + I11 68. Centroid of I + I1
69. Centroid of I1

In Exercises 70 to 75 give the centroid of the solid generated bl; revolving the given region about the given axis.
70. I about x axis 7 1. I1 about x axis 72. I11 about x axis 73. I abouty axis
74. I1 abouty axis 75. I11 abouty axis

In Exercises 76 and 77 give the centroid of the given arc from (0, 0 ) to ( I , I ) .
76. y = x 2 77. y = x3

In Exercises 78 to 81 give the centroid of the surface of the corresponding exercise.


78. Exercise 6 0 79. Exercise 61 80. Exercise 6 2 8 1. Exercise 59
82. Suppose a tank in the shape of the solid in Exercise 70 is
full of a homogeneous liquid of density p. Determine the ( b ) using centroids. ( c ) How much work is done by the
work (in unspecified units) done in pumping out the liquid if it runs out through a hole in the bottom of the
tank through the top, ( a ) without using centroids and tank?
430 CHAPTER 12: PARAMETRIC EQUATIONS AND MORE APPLICATIONS OF INTEGRATION

83. Repeat Exercise 82 using the solid in Exercise 71. 89. Determine the centroid of a typical triangle of base b
84. Repeat Exercise 82 using the solid in Exercise 74. and height h. (Let its base sit on the x axis and its third
85. Repeat Exercise 82 using the solid in Exercise 75. vertex q p t h e y axis.)
86. Suppose a tank in the shape of the solid in Exercise 73 is 90. Deterqjne the area inside the ellipse enclosed by the
divided in half on its vertical cross section by a vertical graph of x = a cos t , y = b sin t, 0 5 t 5 277.
plate. Determine the total force on one side of the plate, 91. Compute the length of the line segment joining ( a , b )
( a ) without using centroids and ( b ) using centroids. and (c, d ) , using ( a ) the distance formula and ( b ) the
87. Repeat Exercise 86 using the solid in Exercise 75. arc length formula.
88. If a spring has a n unstretched length of 6 in. and re- 92. Qetermine the length of 9 ~ = * (x2 + from x = 0 to
quires a force of 4 Ib to stretch it to 8 in., determine the x = @.
work done in stretching the spring to 12 in.

Miscellaneous Exercises
1. Determine the work done in turning a conical tank area. Does this idea rel+te to the Mean-Value Theorem
upside down if the tank of height 60 ft and radius 30 ft for Integrals?
originally has its point in a n upward position and is full 7. Give a precise statement and a proof analogous to that
of gasoline of density 40 lb/ft3. Ignore the weight of the given for Pappus Theorerp for the fact that we can
tank. calculate the total force on g submerged vertical surface
2. Suppose the tank in Exercise 1 is entirely comprised of by using its centroid, its area, and the pressure at the
sheet metal weighing 2 lb/ft2. What is the work done in centroid.
turning it on end-ignoring its contents? (Hznt: Use 8, Use our formulas to determine the volume of the frus-
centroids and treat the flat bottom separately, totaling tum of a cone whose larger radius is R, whose smaller
the two parts of the tank for a n answer.) radius is r , and whose height is h. (This will show that
3. Determine the work done by gravity when one end of a our results are consistent with the hypotheses on which
doubled-up 200-ft (in total length) hanging chain is they are based.)
dropped to hang its full length. 9. An object was dragged 60 ft over uneven ground and the
4. What is the total force on one side of a vertical divider force used was measured at 10-ft intervals, beginning
that divides the tank in Exercise 1 in half ( a ) before it just as the object began to move. T h e measurements
is turned on end, and ( b ) after it is turned pp end? were, respectively 60, 30, 40, 50, 20, 10, and 30 Ib.
5. A hemispherical tank (flat side up) is full of a fluid of ( a ) What was the approximate work done if the Trape-
density p. What is the total force on one side of a vertical zoid Rule is used?
dividing wall in this tank if the wall is halfway between ( b ) Repeat ( a ) using Simpsons Rule.
the center and outside rim of the tank?
6. T h e average pressure on a vertical surface submerged in
a fluid of constant density is the total force divided by its

Exercises 10 to 24 concern the development and application offormulasf o r second and higher moments. Let R be the region betweenf (x) 2 0 and [a, b].
10. Suppose a homogeneous plate of constant density p has 18. Suppose f exists and is continuous on [a, b]. Determine
the shape of the region R. Define the total moment of the moments of inertia Zg and I, of a thin wire of uni-
inertia Z, of this plate about t h e y axis (use a definite form density p whose graph is in the shape o f y = f ( x ) .
integral). 19. For f ( x ) as in Exercise 18, determine the moment of
11. Repeat Exercise 10, but give I, (about the x axis). inertia ZUzof a uniform surface of constant density p
12. Define the radius of gyration of the plate analogous to generated when y = f ( x ) is revolved about the x axis.
the center of mass. (If stuck, see Exercise 20.) 20. T h e radzus of gratzon of a region with respect to a n axis 1
13. Determine the radius of gyration of a homogeneous plate is defined to be the nymber r,, where rL2 = ZL/A. Simi-
of constant density p that has the shape of the region larly, the radius of gyration of a solid about plane P is a
bounded b y y = e-, from x = 0 to x = 2. number rp where rp2 = I P / K Curves and surfaces of
14. Define the third moment of a homogeneous plate of revolution are handled similarly. Assume throughout
constant density p with respect to each axis (the plate that all densities are constant and equal p. Determine
has shape R as in Exercises 10 to 12.) the radii of gyration (with respect to the two axes) of a
15. Determine the third moments of the plate described in horizontal rectangle whose lower left corner is at (0, 0)
Exercise 10. and whose upper right corner is at (a, b) (a, b >
0).
16. Determine the radius of gyration of a rectangle of width 2 1. Determine ryz for the cylinder generated by revolving the
a and height d, oriented with its base on the x axis and above rectangle about the x axis. (See Exercise 20.)
lower left corner a t (0,O). 22. Determine the center of gratzon (using the two radii of
17. Determine the moment of inertia with respect to t h e y gyration) of the region under the first arch o f y = sinx
plane of a solid of density p generated by revolving the (on [O,771).
regisn R about the x axis (denoted I,,).
SECTION 12.4: WORK AND FLUID PRESSURE 431

0
total force on the plate (in terms of a definite integral).

SELF-TEST
433
434 CHAPTER 13: POLAR COORDINATES

Section 13.1 : Polar Coordinates

Objectives: INTRODUCTIOS TO POLAR COORDINATES


1. Plot points and graphs of We start with a horizontal half-line extending infinitely to the right. This
equations in polar coordinates. half-line is called the polar axis, and its initial point is called the pole. (See Fig.
13.1.) We now consider directed angles with initial side the polar axis and
2. Transform polar coordinates vertex a t the pole. As usual, positive measures are assigned to angles generated
and equations into rectangular
in a counterclockwise direction. The half-line starting at the pole that is the
coordinates, and vice versa.
terminal side of the angle 8 is called the ray 8. Examples appear in Fig. 13.2.
Of course any two angles differing by an even multiple of T determine the
+
same ray (see Fig. 13.3). The ray (8 T) is referred to as the opposite of the ray
8 (see Fig. 13.4).
We assume that the polar axis is a copy of the positive real-number line, as
is every ray.

13.1.1 Definition Suppose P is a point in the plane. We say that P has polar
coordinates (r, 8 ) if P is located J r Junits out along the ray 8 if r>0, and out along
<
the opposite of ray 8 if r 0. The pole has polar coordinates (0,8) for all real
numbers 8.
- -
Pole Polar ..\xis Figure 13.5 shows the location of the points with polar coordinates (2, .rr/4),
Fig. 13.1 (3, -a/6), ( - 1,77/2), and ( - 3, - 2 ~ / 3 ) .Note that if we add .rr to 8 and replace
Y by -r, we obtain another pair of polar coordinates for the same point. The
same is true if we subtract T from 0 and replace r by - r. Thus the previous four
points can also be realized, respectively, as ( - 2,577/4), ( - 3 , 5 ~ / 6 ) , (1, 3 ~ / 2 ) ,
and (3, m/3); see Fig. 13.6.
With this definition, every pair of real numbers (r, 8 ) uniquely determines a
point in the plane. However, a given point in the plane does NOT have a unique pair of
polar coordinates. To the contrary, there are infinitely many polar coordinates for
any point. Since ray 8 = ray (8 -i- 2 n ~ for ) n = 0, 1,2,. . . , it follows that the

Fig. 13.2 Fig. 1 3 . 3 Fig. 13.4

+II -1, 7-42,

(4
Fig. 13.5
SECTION 13.1: POLAR COORDINATES 435

J
(a) bI c d\
Fig. 13.6 point with polar coordinates (r, 0 ) will also have polar coordinates ( r , 8 k 2 m ) .
Furthermore, as in the previous examples, if P has polar coordinates (r, B ) , then
P also has coordinates ( - r, B t T). We could also add even multiples of T to
B ? T. Having so many different polar coordinates for the same point may seem
confusing, but it turns out to be one of this system's main advantages. It does,
however, require that you be careful.

POLAR COORDISATES VERSUS


RECTANGULAR COORDINATES
To See the relationship between polar and rectangular coordinates, we begin by
letting the pole coincide with the origin and the polar axis with the positive x
axis.
Now suppose P is a point in the plane. What is the relationship between the
rectangular coordinates (x,y) of P and any pair of polar coordinates (r, B ) ?
Let us assume initially that P is in the first quadrant and that ( r , 0 ) is a pair of
Fig. 13.7 polar coordinates for P with r, B >
0. Then from Fig. 13.7, the rectangular
coordinates may be given as follows:

13.1.2 Polar to Rectangular


I = rcosQ
y = rsinB

Furthermore, these equations hold for any other polar coordinates for P.

Example 1 Determine the rectangular coordinates of the points with polar coordinates
(a) P = ( 2 , ~ / 3 ) ( b ) P = (-5, 1 1 ~ / 6 )
Solutions (a) Since
x = r cos B = 2 cos (m/3) = 2(1/2) = 1
and
y = r sin B = 2 sin ( ~ / 3 )= 2( f i / 2 ) = ~'3
we know that P has rectangular coordinates (1, fi);
see Fig. 13.8(a).

:a,
Fig. 13.8
436 CHAPTER 13: POLAR COORDINATES

\\ ( - 2 , 2 )

X
~\\
F----+
I.@
.

13.1.3 Rectangular to Polar x2 + y 2 = r2 tan 0 = y / x

Example 2 Determine a pair of polar coordinates for the point with rectangular coordi-
nates P:(-2,2)
Solution +
Since x = -2 and y = 2, we know that r2 = x 2 y 2 = 8 and t a n 0 =
y / x = - 1. Of course, there are many angles 0 with tan 0 = - 1. But since
( - 2 , 2 ) is in the second quadrant, we take 0 = 3m/4 and hence r = @; see
Fig. 13.9(a). Therefore one pair of polar coordinates is ( fi,377/4). We could
have chosen 0 = -77/4. In that case we would have to go fi units along the
opposite of ray (-m/4) to get to P, and so we would choose r = - fi,
obtaining polar coordinates (- fi,-77/4) for P;see Fig. 13.9(b). 0

PROGRESS TEST 1

1. Suppose P has polar coordinates ( &, - 77/6) and Q ( d ) Give the rectangular coordinates for P and Q.
has polar coordinates ( - 2, 777/3). [Note that the answers to ( b ) and ( c ) are by no means
( a ) Plot the points P and Q. unique.]
( b ) Determine four additional polar coordinates 2. Determine two pairs of polar coordinates for the
for P. point R with rectangular coordinates (1, fi).
( c ) Determine four additional polar coordinates
for Q.

GRAPHS OF POLAR EQUATIONS


13.1.4 Definition By the graph Ofa polar equation in r and 0, F(r, 0 ) = 0, we mean the
collection of all points that have at least one pair of polar coordinates (r, 0 )
satisfying the equation. If r can be expressed uniquely as a function of 0, then
we write the equation in the form r = f(0) and refer to the graph of the polar
function J
SECTION 13 1: POLAR COORDINATES 437

Y The multiplicity of the polar coordinates for a given point may present some
problems that were not encountered with rectangular equations. For example,
let F(r,8) = r 2 - cos8. Since F ( 1 , a ) = l 2 - C O S T = 1 - ( - 1 ) = 2, the co-
ordinates ( 1 , n-) do not satisfy the equation F(r, 8 ) = 0. However, the point with
polar coordinates (1, a) also has polar coordinates ( - 1 , 0 ) , and F ( - 1 , O ) =
( - 1)2 - cos 0 = 1 - 1 = 0. Hence, by definition, the point ( 1 , ~ is) on the
graph of F(r, 8 ) = 0 since it has a pair of coordinates satisfying the equation.
In rectangular coordinates the basic equations
x = x,, and y =yo
I correspond, respectively, to vertical and horizontal straight lines. If ro is a
Graph of r = ro
constant, then the polar equation
1.i
r = ro
4 has as its graph all points of the form (yo, 8), where 8 can be any real number.
This is a circle of radius lrol centered at the origin; see Fig. 13.10(a). Thus we
can see how the choice of coordinate systems can dramatically change the form
of an equation. Note that if 7, = 0, then the polar coordinates for the pole are
of the form ( 0 , Q )where 8 is any real number.
X If B0 is a constant, then the graph of 8 = 8, is all points (r, 8,) where r can be
any real number. This is a straight line through the origin with angle of
inclination 8,; see Fig. 13.10(b).
To graph more general polar equations we substitute various values of 8 into
the equation, determine the corresponding values of r, and plot the resulting
I points (r, 8). To help simplify matters we use the various properties of the
Graph of 8 = 0,
trigonometric functions and note symmetries whenever possible, particularly
8:b)
the following:
Fig. 13.10
13.1-5 Symmetry The graph of F(r, 8 ) = 0 is

( a ) symmetric about the origin if F(r, 8 ) = F(- r , 8 )


( b ) symmetric about the x axis if F(r, 8 ) = F(r, -8)
( c ) symmetric about t h e y axis if F(r, 8 ) = F(r, 77 - 8)
The validity of (13.1.5) follows from the fact that (-7, Q),( r , -8) and
(r, a - 8 ) are the reflections of (r, 8 ) through, respectively, the origin, the x axis,
and they axis, as illustrated in Fig. 13.11.

Example 3 Graph the polar equation r =' 1 + cos 8.


Solution First note that the period of cos6' is 27, so we can restrict our attention to
0 5 8 5 277. Since cos( -8) = cos(8), the equation is unchanged ifwe replace 8
by -8. By [13.1.5(b)]the graph is symmetric with respect to the x axis, so we

Y
4

Fig. 13.1 1
438 CHAPTER 13: POLAR COORDINATES

I--r ? = 1
8: 0 + 744
cos 8 :

+ cos 8 :
1

2
+

+
1/\/2

1 + :1/\/5)

I
Fig. 13.12

B T / 4 + 7i/2 r / 2 +3 ~ / 4 3n/4 + 7 i

cos 8 . l/fi+O o+ -I/* -l/fi+ -1

= 1+cos~ 1 + l/fi+ 1 1+ 1 - l/v5 1- 1 d5+ 0

I I Ra) n
I
Fig. 13.13

can further restrict our attention to 0 5 8 5 T . Now, as 8 increases from 0 to


77/4, cos 8 steadily decreases from 1 to 1/ fi;thus r = 1 + cos 8 decreases from
2 to 1 + I / ~ ' 5O. n ray o we are two units out. AS we sweep from ray o to ray
* 7/4, the distances out along the rays decrease from 2 to 1 1/ fi.We indicate +
2,0' this piece of the graph in Fig. 13.12,along with the tabulated data.
A similar analysis on the intervals 7 / 4 5 8 5 m/2, 77/2 5 I9 5 377/4, and
3a/4 5 I9 5 77 yields the data in Table 13.1 and the corresponding graphs (see
r=I+cosB Fig. 13.13).
Putting the pieces together, we obtain the upper half of the graph in
Fig. 13.14 Fig. 13.14, from which the lower half follows by the symmetry established
earlier. 0

+
The graph of r = 1 cos8 is referred to as a cardioid because of its heart
shape. It is not difficult to see that for a positive constant a, the graphs of
r = a(1 ? cos8) and r = a(1 ? sinI9)
are all cardioids (see Fig. 13.15), each of which is a rotation of r = a(1 cos 8 ) . +
[Note that, for example, a(1 - cos8) = a(1 - (-COS(T - 8))) =
a(1 + cos(7i - 8 ) ) and a(1 +s i n e ) = a(1 - cos(8 - 3n-/2)).]
It is important to realize that if any of the three symmetry conditions of
(1 3.1.5) hold, then the corresponding symmetry is guaranteed. However, if one
of these conditions fails, the graph may still possess that particular symmetry. In
the next example we shall graph the polar equation r = 2 sin 28,which has all
three symmetries even though none of conditions (a), ( b ) , or (c) in (13.1.5) holds.
SECTION 13.1: POLAR COORDINATES 439

t t

r = a(l + cos 0" r = d l - cos 6:

r = all + sin 6 )
Fig. 13.15

There are, however, other tests for symmetry. For example, the reflection of
the point (r, 8 ) through the x axis can also be realized as ( - r , T - 8). Hence, if
F(r, 8 ) = F( -r, T - 8), the graph of F(r, 8 ) = 0 will be symmetric about the x
axis. Similarly, if F(r, 8 ) = F( -r, -8), then the graph is symmetric about the
+
y axis; and ifF(r, 8 ) = F(r, 8 T), the graph will be symmetric about the origin.
These latter conditions are satisfied by the equation r = 2 sin 28, and hence the
graph has all three symmetries.

Example 4 Graph the polar equation r = 2 sin 28.


Solution Since sin 28 has period T, we can restrict our attention to 0 5 8 2 T, As
observed, all three symmetries are valid so we will plot the graph for
0 2 8 2 ~ / from
2 the data in Table 13.2 and then use the symmetries to obtain
the entire graph. Starting a t the origin, we move further out along each ray as
we sweep from ray 0 to ray m/8, to ray a/4,where we are 1 unit from the origin;
see Fig. 13.16(a).
As we sweep from ray m/4 through ray 3m/8 to ray m/2, we begin to move
back along the corresponding rays until we are only out 1/ fiunits at ray 3 ~ / 8
and back to the origin at ray m / 2 ; see Fig. 13.16(b). Because of the three
symmetries the complete graph has a copy of this loop in each of the four
quadrants (see Fig. 13.17). 0
440 CHAPTER 13: POLAR COORDINATES

Ra) x/2

t t / 4 Ray 3n/8

-L I
Fig. 1 3 . 1 6

t The graph of r = 2 sin 28 is called afour-leafed rose. Due to the symmetries, we


needed only to plot the graph for values of Q between 6' = 0 and B = 77/2.
However, one should note that the values of Q between 7 ~ / 2and 7~ give rise to
the leaf in the fourth quadrant, whereas the values of 6' between 3 v / 2 and 277
produce the leaf in the second quadrant. This is because r = 2 sin 26' is negative
in the intervals n/2 < <
Q n or 3n/2 < <Q 277, and so points are plotted along
the opposite ray in each case. A similar construction shows that r = 2 cos 28 is
also a four-leafed rose, but with the leaves centered on the axes (see Fig. 13.18).
In general one can show that r = a sin nQ, r = a cos no, a >
0, will produce
r = 2 sin 28 roses with 2n leaves if n is a n even integer and n leaves if n is a n odd integer.
Fig. 1 3 . 1 7
In some cases, transforming from one coordinate system to another can make
the graphing easier. For example, to graph r = 2a sin Q we can multiply by r
to obtain r2 = 2ar sin 8.
But r2 = x 2 + y 2 and y = rsind, so the graph of the polar equation
f r = 2a sin 6' is the same as the graph of the rectangular equation x2 y 2 = 2 9 . +
But, completing the square i n y as follows,
x2+y2=2ayox2+y2-2ay=o
tj x 2 +J? - 2 9 + a2 = a2
tj x 2 + ( y - a)2 = 2
we get the rectangular equation of a circle centered on they axis at (0, a ) with
radius a ; see Fig. 13.19(a). Similarly, r = 2a cos Q yields a circle centered on the
x axis a t (a, 0) with radius a, as in Fig. 13.19(b).
O n the other hand, by changing rectangular equations to polar equations,
we can sometimes make the graphing easier.
I = 2 cos 28
Fig. 1 3 . 1 8

r = 20 sin 0 i = 2a cos 8
01 b
Fig. 1 3 . 1 9
SECTION 13.1: POLAR COORDINATES 44 1

Example 5 Graph the rectangular equation ( x 2 + y 2 ) 2= 4 ( x 2 - y 2 ) . (Its graph is some-


times called a lemniscate of Bernoulli.)
Solution Since x 2 + y 2 = r2, x = r cos 8, and y = r sin 8, we have
(r2)2 = 4 ( r 2 cos2 6 - r2 sin2 8 )
Dividing by r2 and using the trigonometric identity cos 28 = cos2 6 - sin2 8,
the equation becomes
r 2 = 4 cos 28
Since cos 28 has period 7 i , we need only consider 0 5 6 5 7 i . Also, we get the
same equation if we replace r by -r and 6 by -6, so the graph is symmetric
with respect to the origin and the x axis. Hence we shall plot r = + 2 v ' a .
and then use symmetry. O n [O,m/4] we have the following data:

cos 28: 1 130

This leads to the first-quadrant graph in Fig. 13.20, from which the third-
quadrant graph (actually the graph of r = - 2 d a )follows by symmetry
with respect to the origin.
By symmetry with respect to the x axis we obtain the dashed portion of the
graph in the second and fourth quadrants. Now, for 77/4 6' < <
3~/4,
cos 26 < 0, and so there are no points that satisfy r 2 = 4 cos 26 and we get no
additional points. O n [ 3 ~ / 4 , 7 r ]we have a table similar to the above, with
r =2d a increasing from 0 to 2 and r = - 2 d a decreasing from 0 to
- 2. This accounts for the second- and fourth-quadrant dashed graph in Fig.
13.20. Thus the polar equation r2 = 4 cos 28 has the graph shown in Fig. 13.21
(a lemniscate), which is also the graph in rectangular coordinates of
( x 2 + y2)2 = 4(x2 - y ) . 0

In general, the graph of any equation of the form


r = k ( a ib cos 8 ) or r = k ( a t b sin 8 )
is called a limacon. If a = b, the limacon is a cardioid (as in Example 3). If a b, >
<
then the basic shape is that of Fig. 13.22(a); if a b, there is an inner loop as
in Fig. 13.22(b).

4 Ra) ni4

Fig. 13.20 Fig. 13.21


442 CHAPTER 13: POLAR COORDINATES

I r=2+cose
a

Fig. 13.22

PROGRESS TEST 2

1. Using symmetry a n d a table, show t h a t the graph of


r =2 +
cosB is as in Fig. 13.22(a).
2. Determine a n d sketch a polar equation that has the
same graph as the rectangular equation
(9+y2)2 = 4xy.

SECTION 1 3 . 1 EXERCISES

In Exercises I to I5, ( a ) plot the point with the given polar coordinates, (b) name a pair of coordinates for the same point that uses the opposite of
the original ry, and ( c ) give a third pair of coordinates for the given point.
1. (2, m/4) 2. (3, -m/4) 3. (2, 7m/6)
4. (-2,a/6) 5. (-4,0) 6. (1,400~)
7 . (-2,m/3) 8. (1,49a/6) 9. ( fi,- 14a/3)
10. (2,O) 11. (0,m/9) 12. (5, - 2 ~ / 3 )
13. (1,2) 14. (1, -3) 15. (1, -2)

I n Exercises I6 to 24 determine a pair of polar coordinates for the points with the given rectangular coordinates.
16. (1,l) 17. (-4,O) 18. (0,m)
19. (2v5,2) 20. (3,O) 21. (3fi,-3$)
22. (fir*) 23. (3, -3) 24. (2,2)

In Exercises 25 to 3 3 f i n d the rectangular coordinates for the points with the given polar coordinates.
25. (6,3m/4) 26. (-2,1 l ~ / 6 ) 27. (-3,~/3)
28. (2,O) 29. (2 fi,- m/4) 30. (5,25m/4)
31. (2, 1) 32. (1, 2) 33. (1,l)

In Exercises 3 4 to 42, find an equatzon in rectangular coordinates for the given polar equation.
34. r = 4cos6' 35. 6' = m/3 36. r= 2
37. r2sin26' = 2 38. r3 = 3 cos 6' sin 6' 39. r2 = cos 26'
40. r = 1 - 2 sin 6' 41. r2 = 1 + cos6' 42. r= 2
2 + case
43. +
Show that the graph of r = 1 2 sin 6' is as in Fig. (c) Determine for which 6' in [m, 3a/2],
13.22(b) by following the given steps: r = 1 + 2sinB = 0.
( a ) Prove symmetry with respect to they axis. ( d ) Compile a table for m 5 6' 2 3n/2, sketch, and then
( b ) Give a table for 0 2 6' 2 n/2 and then use ( a ) to use ( a ) to complete the graph.
complete the graph for 0 5 6' 5 7;.
SECTION 13.2: POLAR GRAPHS: INTERSECTIONS AND TANGENT LINES 443

In Exercises 44 to 71 sketch the graph of the given polar equations. (You may use earlier remarks and discussion as a guide regarding the general shape
and dimensions of the graph.)
44. 0 = 14~/3 45. I = -8 46. r = 4sin28
47. r = cos48 48. r = 4sin30 49. r = 2 cos 38
50. r 2 = 9 cos 28 51. r 2 = 4 sin 38 52. r = 9 sin0
53. r = 3 sin20 54. r = 3 - 2 sin 8 55. r = 18 cos0
56. r = 1 - 2 sin 0 57. r =2 +-
cos6

(The graphs of the equations in Exercises 58 to 61 are called conchoids.)


58. r = 5 sec0 59. 7 = 2csc6 60. r = 2 csc 0 - 1
61. r=cscO - 2 62. r = 6 (Archimedian spiral) 63. r = -30
64. r = e s (logarithmic spiral) 65. = 2e@/(2T) 66. r s i n 0 = -1
67. r = 5 cos38 68. r 2 = 9 cos 20 69. r 2 = 9 sin 20
70. r = 2sec0 - 1 71. r= 2
2 - cos 8
In Exercises 72 to 75, transform the given rectangular equation into a polar equation and sketch the graph in polar coordinates superimposed on a
rectangular system.
72. +
( x 2 +y2) = 2x2 2y2 - 3 73. (x2 +y2)2 = x 2 -y2
74. x+2y=5 75. (x2 +y2)3 = 4x92
76. ( a ) Show that the graph of the equation r2 = 77. Transform the linear equationy = mx into polar coordi-
2ar cos(8 - +) +b is a circle with center ( a , +), provided nates.
+ >
b2 c 0. 78. Transform the equationy = 2 into polar coordinates.
( b ) Determine the radius of this circle.

Exercises 79 to 85 concern the polar equations f o r conic sections.


79. Show that the graph of r = 1/[1 - (1/2)cosO] is a n 83. Describe the conic section that is the graph of
ellipse. r = 9/(3 + sin 0 ).
80. Show that the graph o f r = 1/(1 - cos 0 ) is a parabola 84. Repeat Exercise 83 for r = 2/[(1/2) sine]. +
with a hole in the graph. Where is the hole? 85. Determine the polar equation for the hyperbola
81. Show that the graph of 7 = 1/(1- 2 cos 6) is a hyper- x2 - y 2 = 1.
bola. Describe this hyperbola.
82. Show that the graph of 4r cos 0 = r - 8 is a hyperbola.
Describe this hyperbola.

Polar Graphs: Intersections


Section 13.2: and Tangent Lines

Objectives: INTERSECTIONS OF POLAR GRAPHS


1. Determine intersections between Determining all the points of intersection of two rectangular graphs is the
graphs of polar equations. relatively straightforward (although not always easy) task of solving a pair of
simultaneous equations. Because a point has many different names in polar
2. Determine slopes and equations
coordinates, the situation for polar equations is not as straightforward. If we set
of lines tangent to polar graphs.
the polar equations
r = sine and r = cosQ

equal, we get sin Q = cos 8, or tan Q = 1. This implies


8 = 7/4, 7/4 +- 7 ,~ / I4+ 2 7 , . . .
For 0 5 Q 5 2 7 , we get the two solutions
(1/ fl,7/4) and ( - 1/ v?, 57/4)
444 CHAPTER 13: POLAR COORDINATES

Fig. 13.23

which are polar coordinates for the same point! However, from our work in
Section 13.1, we know that the graph of each equation is a circle through the
origin, as given in Fig. 13.23.
Thus the simultaneous solution of two polar equations need not yield all the
points of intersection! The problem is that the polar representations of certain
points of intersection may not be the same for both curves. In this case the
coordinates of the origin which satisfy r = sin Q are of the form (0,n ~ ) while
, for
+
r = cos Q they are of the form (0,7~/2 n ~ ) .
If we think of (r, Q)as the location of a particle moving along the graph at
time 8 , then simultaneous solutions of F(r, Q) = 0 and G(r, Q)= 0 would arise
=o when the two particles collide-that is, when they are both at the same point at
the same time 8. However, they may arrive a t the same point at different times 6.
This would lead to a point of intersection of the two graphs, hut such a point
would not be determined by solving the two equations simultaneously. There
are analytic procedures for determining nonsimultaneous solutions, but we shall
be content to solve the equations simultaneously and then graph the equations.
Fig. 13.24 Any nonsimultaneous solutions can then be readily observed.

Example 1 Determine the points of intersection of the circle r = cos Q and the four-leafed
rose r = cos 28.
Solution From Fig. 13.24 we can expect four points of intersection. Setting cos 28 = cos Q
and using the identity cos 28 = 2 cos2 19 - 1, we have
2 cos2 Q - 1 = cos 6'
or
2 COS' Q - cos Q - 1 = 0
or
(2 cos Q + l)(cos Q - 1) = O
Hence cos Q = - 1/2 and cos Q = 1. As a result, Q = 0 gives (1,0), Q = 2 ~ / 3
gives ( - 1/2, 2 ~ / 3 ) ,and 6' = 4 ~ / 3gives ( - 1/2,4~/3)-all on both curves.
The origin does not arise as a simultaneous solution but is clearly on both
graphs. For example, it is on r = cos Q as (0,7~/2), and on r = cos 26' as
(O,T/4).

PROGRESS TEST 1

Determine all points of intersection of the following pairs 1. r = 1 - cos Q (cardioid), r = cos 8.
of polar equations: 2. r = 3/2 - C O S Q (limacon), Q = m/3.
SECTION 13.2: POLAR GRAPHS: INTERSECTIONS AND TANGENT LINES 445

4 TANGENT LINES TO POLAR GRAPHS


A
We now turn our attention to equations such as r = 1 + cos8, where r is
uniquely determined as afunctzon of 8. More generally, we suppose that r = f ( O ) ,
withfdifferentiable with respect to 8, and examine what can be learned about
I, the function and its graph from the derivative. In particular, we ask: What zs
the slope of the lzne tangent to the graph at the poznt (f(8,), 8,)? That is, what
is tan 0, (where 4, is the angle of inclination of the tangent line with the x axis)?
Y X (See Fig. 13.25.)
/ \ The graph of the constant function r = f ( 8 ) = 1 is the unit circle, and
r=fO
/ \ f ( 8 ) = 0, so the answer to the above question in polar coordinates is certainly
\
Tangent line not provided byf(8) directly. We transform the given function to rectangular
Fig. 13.25 coordinates using the equations
x = rcosd and y = rsind
This gives the parametric equations
x = f ( 8 ) cos 0 and y = f ( 8 ) sin 8
whose graph is the same as that of r =f(8). By (12.1.4), or the Chain Rule, the
slope is given by

Since
- = f ( 0 ) cos 0 + f ( 8 )sin 8
d8
and
dx
- = - f ( 8 ) sin 0 + f ( 8 ) cos 0
d8
we have proved the following theorem.

13.2.1 Theorem If -
dx
do
I 8x8,
# 0 , then

tanqo =
f(0,)cos 8, + f(0,)sin 8,
-f(do)sin 0, +f(0,)cos 8,
gives the slope of a polar curve r = f ( 0 ) at the point (f(O,), 8,).

Example 2 Show that the cardioids


r =1 + sin0 and r = 1 - sin8
are perpendicular at the point ( 1 , ~ )see
; Fig. 13.15, page 439.
Solution Note that (1, T) is on both curves. Let M , = tan+, denote the slope of the
tangent line to r = 1 + sin 0 at (1, T) and M I = tan +1 the slope of the tangent
line to r = 1 - sin0 a t the same point. By (13.2.1),

M , = tan+, =
(1 + sin m)cos T + cos T sin T -1
--=-I
-
-(1 + sin m)sin T + cos T cos T 1
and
Ml = tan+1 =
(1 - sinT)cosT + (-cos~)sinm - -1
-1
-(1 - sinm)sinn + (-cos~)cos~ -1
Since MOMl = - 1, the tangent lines are perpendicular. 0
446 CHAPTER 13: POLAR COORDINATES

Suppose we let $o (see Fig. 13.26) denote the least positive angle between the
ray B0 and the tangent line to the graph at (f(0,),0,). Then, from Fig. 13.26,
$0 = 0, + $0

which, as we shall show, implies the following:

13.2.2

The simplicity of the formula for tan +,


makes it easier to work with the angle
between the ray B0 and the tangent line than with the slope of the tangent line
itself as in rectangular coordinates.
To prove (13.2.2) we divide numerator and denominator of (13.2.1) by cos 8,
to get

Now $0 = 00 + $0, SO

$0 = +o - 00
Taking the tangent of both sides and using the trigonometric identity
tan a - tan b
tan(a - b ) =
1 + tan a tan b
we obtain
tan Go - tan 0,
tan#, =
+
1 tan +o tan 0,

We then substitute the expression for tan +, given in (1) and simplify the
resulting fraction to get (13.2.2).

Example 3 Use (13.2.2) and the identity

tan(a + b) = +
tan a tan b
1 - tan u tan b
to find the slope of the tangent line to the polar curve r = sin2 30 at (1/2,77/4).

Solution Let $, be the angle between the tangent line and the ray a/4. Then
f ( 0 ) = sin2 30 andf(0) = 6 sin 30 cos 38, so, by (13.2.2),

tan+o = sin2(3m/4) --
- -1
6 sin(3~/4)cos(37;/4) 6
Now $0 = 0, + $ 0 , where 0, = a/4 and +o is the angle of inclination of the
tangent line (see Fig. 13.26). Thus
tanQo = tan(0, + +,)
- tan 8, + tan +,
-
1 - tan0, tan+,

-
-
+
t a n ( ~ / 4 ) (-1/6) 5
=- 0
1 - [ t a n ( ~ / 4 ) ]-
( 1/6) 7
SECTION 13.2: POLAR GRAPHS: INTERSECTIONS AND TANGENT LINES 447

PROGRESS TEST 2

1. Show that the angle # between the tangent line to 2. Determine the slope of the tangent to r = 1 - cos 0
the cardioid r = 1 - cos 0 and the ray 0 is 0/2 for at (1, m/2).
each point ( r , 8 ) on the curve. [Use the identity
tan(O/2) = (1 - cos 0)/sin 0.1

SECTIOS 13.2 EXERCISES

In Exercises 1 to 20 determine all points of intersection of the graphs of the given equations
1. r = 3,8 = m / 6 2. r = cos 8 , B = rr/4
3. r = C O S ~= , ~1 4. r c o s 8 = 2,r = 4
5. r = cos0,r2 = cos 28 6. r = cos 28, r = sin 8
7. r = cos 8, r = 1 - cos 8 8. r = 2, r = 2 cos 28
9. r2 = 2 cos 28, r = 1 10. r = sin 20, r = 1 - cos 28
11. r = 2 sin 28, r = 1 12. r = cos 38, r = sin 38
13. r = m/2,r = 8/2 14. r = 7/2,r = 28
15. r = 1 - s i n 8 , r = 1 + sin8 16. r = 1 + cos8,r = 1 - sin8
17. r = sin 8 , r = sec 8 18. r = cos 8, r = csc 8
19. r2 = sin 28, r2 = cos 28 20. r = tan 8, r = 2 cos 8
21. Determine where the graph of r = (5/2)8 crosses itself.
22. Determine where the graph of r = 1 + 4 sin 28 crosses
itself.

In Exercises 23 to 34 determine ( a ) the slope ofthe tangent line to the curve at (f(O,), 8,) and ( b ) the tangent ofthe angle +, between this tangent line
and the ray 8,.
23. r = 1 + 2 cos 8, (2, 7/3) 24. r = 1 + 3 sin 8, (5/2,7i/6)
25. r =2 +
2 cos 8, (2,m/2) 26. r = 3 cos 28, (3/2, ~ / 6 )
27. r = 2(1 - 2sin8),(2,0) 28. r = 2 cos 0, (1: 7 / 3 )
29. r = tan 8, 8, = 7i/4 30. r = 8, (7/3,7/3)
31. r = cos 28,8, = 7/12 32. r = 4 cos 38,8, = 3m/4
33. r = 1 - cos 8, 8, = 7 / 2 34. r = 4 sin2 8, (1, 5 m / 6 )
35. r = 1 + 2 sin 8 , (1 - fi,57/41 36. r = 2 - 4 cos 8, (4, (2/3) m )
37. r = 4(1 + cos 8 ) , 0, = 5 m / 6 .
38. Determine the lines tangent to the three-leafed rose 48. Repeat Exercise 46 for r = e 9 and r = e ~ ~ .
r = cos 38 at the pole. 49. ( a ) Give an informal argument that the maximum dis-
39. Repeat Exercise 38 for the four-leafed rose r = cos 28. tance from the pole to a point ( r , 8 ) on a polar graph of
40. Show that the angle IJ is a constant for the logarithmic r = f ( 8 ) (wheref '(8) exists) occurs at a point where the
spiral r = e 9 . tangent line is perpendicular to the ray 8.
41. Determine the angle a between the tangents to the ( b ) Test the statement in ( a ) on the cardioid
graphs of r = cos 8 and r = -sin 28 at their points of r = 2 - 2 cos 8.
intersection.
42. Repeat Exercise 41 for r = 1 + cos 8 and r = sin 8.
43. Use symmetry to determine the points of intersection
between r = 2 cos(8/2) and r = 2 sin(8/2).
44. Prove that the formula for the angle of inclination 0 of
the tangent to the circle r = 2 sin 8 in terms of 8 is
+
0 = 28, where 0 = 8 + and where )I is as in Fig.
13.26.
45. Repeat Exercise 44 to get the formula @ = 8/2 for
r = 1/(1 - cos8).
46. Show that r = 1 + cos 8 and r = 1 - cos 8 intersect at
right angles.
47. Repeat Exercise 46 for r = 4 sin 8 and r sin 8 = 2.
448 CHAPTER 13: POLAR COORDINATES

Section 13.3: Area and Arc Length in Polar Coordinates

Objectives: AREA IN POLAR COORDISATES


1 . Determine the area of a region In Chap. 6 we determined the area of a region bounded by continuous curves
bounded by polar curves. first by approximating the region with convenient-namely, rectangular-
2. Determine the length of a polar
subregions. Our approach to regions bounded by polar curves is essentially the
curve. same except that the "convenient" approximating subregion is a circular sector
rather than a rectangle.
3. Determine the surface area of A circular sector of a disk of radius r enclosed by an angle of A0 radians is
the surface obtained by revolving a the fraction (A0)/(2m) of the entire disk. Hence its area A is the same fraction of
polar curve about the x or Y axes. the area of the entire disk. Thus A / m r 2 = (A0)/(277). Solving for A , we get

13.3.1 A = - 1r 2 A0
2
Now suppose R is the region bounded by the graph of the polar equation
r = f(S) and the rays 0 = a and 0 = P, wheref(8)is positive and continuous on
[a,P ] and P - a 5 277. A typical region of this kind is sketched in Fig. 13.27.
Subdivide [a,P ] into n distinct subintervals

loo, '11, ' ' > [',-I, OaI, . . * > [On-I, On]
where 0, = a , On = P, and
AO, = 0, - for each 0 <i 5 n
Fig. 1 3 . 2 7 Let 11PII denote the length of the largest subinterval in this partition.
In each subinterval [S,-,, S,] choose a point 0:. The circular sector between
ray OaP1 and ray 0, of radiusf(0:) approximates the region bounded by these
rays and the graph of r = f ( O ) ; see Fig. 13.28. By (13.3.1), the area of this
sector is

r = J 8,
3[f(O:)l2 AO,

We sum the areas of these n sectors to approximate the total area of R :

= -2 c [f(0,")l2AOi
l n
A(R)
Fig. 1 3 . 2 8 a=,

Figure 13.29 depicts the situation when n = 4.


Now as IlPl/ + 0, these approximations will tend to A ( R ) . But
P I
83

is a Riemann sum for the continuous function G(0) = (1/2)[f(0)]2 on [a,PI, so


in the limiting case it will tend to the definite integral

Fig. 1 3 . 2 9 To summarize, we have the following:

13.3.2 T h e o r e m If f ( 0 ) is positive and continuous on the interval [ a ,,8] of


length 5 277, then the area of the region bounded by the polar graph of
r =f(O) and the rays 0 = a and 0 = P is given by
SECTION 13.3: AREA AND ARC LENGTH IN POLAR COORDINATES 449

Of course, the area of a more general region such as that in Fig. 13.30 is
obtained as a difference of integrals:
1 p 1 p
A ( R ) = 2 s [.@)I2d6 -Tl [g(e)l2d0
a
or

r = g(0) 13.3.3
a
Fig. 13.30

Example 1 Determine the area of the region enclosed by the four-leafed rose r = 2 cos 28
(see Fig. 13.31).
Solution We find the area of the first leaf, which occurs for values of 8 in [ -n/4, n/4],
and then multiply by 4.
7r/4
A(R)= 4 [AJ
2 -7r/4
(2 cos 28)2 do]
7r/4
=2 [ (4cos228)d8

Fig. 13.31

Example 2 Determine the area of each of the following regions:


( a ) Inside the circle r = 3 cos 8 and outside the cardioid r = 1 + cos 8.
( b ) Inside both the circle r = 3 cos 8 and the cardioid r = 1 + cos 8;
see Fig. 13.32(a) and (6).
Solution Setting 1 + cos 8 = 3 cos 8, we get points of intersection when cos 8 = 1/2, so
8 = fn/3.
( a ) The desired region lies between the rays 6' = +a/3 and is of the type in Fig.
\
13.30 withf(8) = 3 cos 8 and g(8) = 1 + cos 8. Because of the symmetry with

'a;

Fig. 13.32
450 CHAPTER 13: POLAR COORDINATES

respect to the x axis, we can find the area between the rays Q = 0 and Q = m/3
and then multiply by 2. Thus

A(R) = 2 [+J"l3 ([3 cos elz - [ l + cos el2) 1 dQ

= &7'3 (9 C O S Q~ - 1 - 2 cos 0 - COS' 19)dB


7 /3 7/3
= 8 rT'3cos2Q dQ - 2 f COSQ dQ - f dQ
JO JO JO

=4&
v/3
(1 + cos2Q)dQ- 2 COSQ & dQ -
7/3
d6'

=4 [e + - sin 28 1, - [2 s i n Q ] y 3 - [HITi3 0
= 7~

( b ) Here the desired region is comprised of two regions of the basic type
considered in Theorem (13.3.2).
We again use the symmetry and work in the upper half plane (see Fig.
- 1 +cos~ 13.33):
A ( R ) = W(R,) + A(R2)I
= 2[ti 7/3
(1 + cosO)2d0
I %/2
Fig. 13.33 (1 + 2 cos Q + COS' 0 ) dQ +9 (cos2 Q ) dQ

(
= Q + 2 s i n B + - 1 Q+-sin22Q)
2
ill3+ - (e + -
7 /3

sin22Q)
T/3
= 5m/4 0

PROGRESS TEST 1

1. Show that formula (13.3.2) gives the expected area


of a circle of radius a when the circle is described as I20
r = f ( Q )= a, 0 5 6' 5 2 7 ~ .
2. Determine the area A inside the right-leaf of the =o
four-leafed rose 7 = 2 cos 28 and outside the circle
r = 1 (see sketch). (First solve for a and p.)

MORE ABOUT AREA I K POLAR COORDINATES


In constructing our earlier formulas, we assumed thatf(Q) 2 0 on [a,PI. O n
the other hand, the formula

which involves [ f ( Q ) 1 2does


, not "notice" whetherf(0) <
0. This can sometimes
lead to errors resulting from certain areas being counted twice iff(Q) changes
SECTION 13.3: AREA AND ARC LENGTH IN POLAR COORDINATES 45 1

t sign on [a, PI.


The inner loop of the limacon r = 1 2 sin 6' occurs when +
7 ~ / 65 8 5 1l ~ / (see
6 Fig. 13.34 and recall Sec. 13.1, Exercise 43). Hence if we
were to compute

Ti
1 '=
(1
1
+ 2 ~ i n B ] ~ d=6 ' - i 7 ~ / 6
(1 + 2sin~)zdd
(1 + 2 sin 6')' d6' + - JzT (1 + 2 sin 6')' d6'
11~/6

then the three integrals on the right correspond, respectively, to the areas of the
/ shaded parts of Fig. 13.39ta) to (c).
/
0 = 7n/6 0 = 1ln/6 Thus the area of the entire limacon is
Fig. 13.34

+
while (1/2)1iT(1 2 sin 6')' d6' counts the area of the loop twice.
The moral of this discussion is clear: When computing the area of a region
enclosed by the graph of i. = f ( 6 ' )on [ a ,PI, be certain thatf(6') 2 0 on [a, b]. If
<
f(6') 0 for some 6' in [a,PI, break the interval into subintervals wheref(6') has
a constant sign and interpret the associated integrals carefully.

ARC LENGTH AND SURFACa AREA


IN POLAR COORDINATES
By (12.2.1), the length of the graph of the parametric equations
x = x(t) and y =y(t) a 5t5 b
is
b
J = d[x'(t)]'+ [y'(t)]'dt
We can describe any polar chrve
r=f(6') a 56' < P
parametrically by using x = r cos 6' and y = r sin 6'. This yields
x =f(6')cos 8 and y =f(O)sin 6' a 5 6' 5 P
with the polar angle 6' serving as the parameter. Now, assumingfhas a con-
tinuous derivative on [a,PI, we have
x'(6') = -f(6')sin 6' + f'(6')cos 6'
y'(6') =f(O)cos 6' +f'(O)sin 6'

t t t
0 = 0

!a)
Fig. 13.35
452 CHAPTER 13: POLAR COORDINATES

Thus [~(e)]+ [y(@)]


= [f(e)]2sinz6 - 2f(e)f(O)sin e cos 8 + [ f ( e ) ] 2 ~ 0 ~ 2 8
+ [f(S)]2cos26 +
2f(e)f(O)sin e cos 0 + [f(8)]2sin2 e
+ +
= [f(6)](cos20 sin2B) [f(e)]2(cos2e + sin28)
= [f(0)I2+ If(e)l2
This proves the following:

13.3.4 Arc Length

That is, s = Sf dr, where, for polar curves, ds = + [f(e)I2de.


d[f(19)]~

Example 3 Find the length of the polar curve r = sin2(B/2), 0 5 0 5 7.


Solution Lettingf(0) = sin2(O/2), we obtain
f ( 0 ) = sin(8/2)cos(8/2)
Hence
ds = 4[f(e)12+ lf(e)I2d@
= dsin4(8/2) + sin2(8/2)cos2(B/2)dB
+ cos2(B/2)]dB
= ~sinz(fl/2)[sinz(e~2)
=d m d 8
= lsin(8/2)1 d8
= sin(O/2) d8 because sin(8/2) 2 0 on [O,T]
Therefore
s = iT sin(O/2) d0 = -2 cos(8/2)
7i

lo =2 0

In a similar way we can use (12.2.7) to find the area of a surface obtained by
revolving the polar curve 7 =f ( S ) ,(Y 2 8 5 p about a n axis. We replace x by
f(S)cOsO,y byf(B)sinO, and ds by 4[f(e)l2 [f(e)I2do. +

Example 4 Find the area of the surface obtained by revolving the logarithmic spiral r = e e ,
0 5 2 77/2, about the x axis.
Solution By (12.2.7), page 410, S = 2771fyds, where y =f(e)sinO, and ds =
+
d(es)2 ( e e ) 2 d8 = G d O = fiee do. Hence
277 l
0
T/2
f(0)sin 6 ds

2 fi77
sin d - cos 6 )
IT (integration by parts)

5
(1 + 2eT) 0
SECTION 13.3: AREA AND ARC LENGTH IN POLAR COORDINATES 453

PROGRESS TEST 2

1. Compute the total length of the cardioid +


the first quadrant piece of r = 1 cos 0 abour they
r = 1 + cos 0. (Use x axis symmetry.) axis. Express the integrand entirely in terms of 0/2.
2. Set up, but do not evaluate, a definite integral 3. Same as Prob. 2, but about the x axis. Express the
giving the area of the surface generated by revolving integrand in terms of 0.

SECTIOS 13.3 EXERCISES

In Exercises 1 t o 24 determine the area of the region bounded by the graph of the given equations. Use symmetry whenever possible. Figures and exercises
given earlier in the chapter may also prove helpful.
1. Inside both loops of r = 2 sin 28 2. r = 2 +
cosB
3. r = 3 + 2 c o s 8 4. r = 5 + sin8
5 . r = 2 - sin28 6. r = 28, with 0 5 8 5 1, 8 = 0 , 8 = 1
7. r = e s , with 0 5 8 5 v , 8 = 0,8 = a 8. r = 2 + 2 sin 38, 8 = 0,8 = a / 3
9. Inside the three-leafed rose r = 2 cos 38 10. r = m, 8 = 0 , B = 7/4
11. r = 6 s e c 8 , 8 = 0, 8 = v/4 12. r2 = 4 sin 28
13. Inside one loop of r = a sin 48 14. Inside one loop of r = a cos 58
15. Inside r = 5 cos 8 and outside of r = 2 + cos 8 16. Inside both r = 5 cos 8 and r = 2 +
cos 8
17. Inside both r = 1 + cos 8 and r = 1 - cos 8 18. Inside the smaller loop of r = 1 - 2 cos 8
19. Inside the larger loop of r = 1 - 2 cos 8 20. Inside r = 2( 1 + sin 8 ) and outside r = 4 sin f?
2 1. Inside r 2 = 4 cos 28 and outside r = 2 sin 8 24. The area common to the two circles r = sin 8, r = cos 8
22. Inside the larger loop of r = 2 - 4 sin 8 and outside the 25. ( a ) Use definite integrals in polar form to compute the
smaller loop area of the circle r = a cos 8.
23. Inside the larger loop of r = a - b sin 8 and outside the ( b ) Compare the answer to ( a ) with+JgTa2cos2 8 d 8 and
smaller loop, where a >
b and arcsin ( a / b ) = 8, explain.

In Exercises 26 to 31, ( a ) set up and ( b ) evaluate a dejnite integral giving the length of the indicated arc,
26. r = 1 + sin 8 , 0 5 8 2 277
28. r = a8, 0 5 8 5 2 v , a 0 >
30. r = 3 sec 8,O 5 8 5 a/4

In Exercises 32 to 36, ( a ) set up and ( b ) evaluate a dejnite integral giving the area Ofthe surface generated by revolving the indicated arc about the
given axis.
32. r = sin 28,O 2 8 2 7 / 2 , x axis 33. r = 2 sin8,O 2 8 5 a / 2 , x axis
34. r z = 4 cos 28,O 2 8 2 7/4, x axis +
35. r = 1 sin8,O 5 8 5 7 / 2 , y axis
36. r = e e , 0 5 8 2 7 / 2 , y axis
37. Show that the circumference of a circle of radius a 0 is > 38. Show that the surface area of a sphere of radius a 0 is >
27a in two ways: 47r2 in two ways:
( a ) By applying (13.3.4) to r = a. ( a ) Based on r = a, 8 = 0 , 8 = a.
( b ) By applying (13.3.4) to r = 2a cos 8. (Be careful ( b ) Based on r = 2a cos 8. (Be careful about counting
about counting the same arc twice.) the same area twice.)
+
39. ( a ) Show that the graph of r = a s i n 8 b cos 8 is a 40. Show that the area formula (13.3.3) is consistent with
circle and describe that circle. the sector formula (13.3,1), on which the former is
( b ) For such a circle prove that ds is proportional to d8. based, by computing the area of the region enclosed by
r = a, r = b, 8 = 0 , 8 = A8, a <
b, 0 < A 8 2a. <

CHAPTER EXERCISES

R e v i e w Exercises
In Exercises 1 to 16 graph the given polar equation.
1. 8 = - 7 ~ / 6 2. r = - 3 / 2
3. r = 2 sec2(8/2) 4. r = 2 - sin8
454 CHAPTER 13: POLAR COORDINATES

5. rcos8 = 4 6. r = 2 csco
7. r = 2 + sin 20 8. r= d w
9. r = 1 + sin28 10. r2 = 3 cos 28
11. r2 = 1 6 ~ 020
s 12. r = -8,8>0
13. r = 4/(1 - cos8) 14. r = 4sin38
15. r(1 + 2cos0) = -4 16. r = 2(2 +
sin8)
17. r = cot 8, -7/2 8 < < 7i/2 18. r = 2 cos(8/2)

In Exercises 19 to 22, ( a ) transform the given equation into polar form and ( 6 ) sketch the graph
19. x2 + y 2 - 4y = 0 20. y 2 = 4x 4 +
21. 2xy = k2 22. (x2 + y 2 ) 2= 2a2xy

In Exercises 23 to 28 convert the given equation to rectangular coordinates.


23. r = acsc6 24. r = 1/(1 - s i n 8 )
25. r = asin28 26. r2 = 2a2 cos 28
27. r2 = a 2 s i n 2 8 28. r = 1/(2 - cos8)

In Exercises 29 to 36 determine all points of intersection of the graphs of the given equations.
29. r = 2 cos8,r = fi 30. rcos8 = 2 , r = 4
31. rsin0 = 2,r = 1 32. r = sin28,r = c0s28
33. r = asin8,r = acsc8 34. r = a cos 8, r = a sec 8
35. r = 3 sin0, r = 12/(3 + sin8) 36. +
r = 1 cos 8, r = C O S ( ~ / ~ )

In Exercises 37 to 42, ( a ) determine the slope ofthe tangent lzne and ( b ) determine the tangent ofthe angle I$, between the lzne tangent to the curve at
( f ( 0 , ) , 0,) and the ray 0,.
37. r = 1 - sin 0 , 0 , = 77/2 38. r = 2 - 3 cos 8 , 8 , = 7i/2
39. r = a0,8, = 7;/2 40. r = a 0 , 0 , = 7i
41. r=5sin38,Bo=7;/4 42. r 2 = 8 cos 28.8, = 4 6
43. Show that r = sec2(8/2) and r = 2 csc2(8/2) are perpen- 44. Repeat Exercise 43 for the cardioids r = 2(1 - cos 0 )
dicular at their points of intersection. and r = 4( 1 + cos 8 ).

In Exercises 45 to 64 determine the area of the region bounded ly the graphs of the given equatzons.
45. r = 2 - cos 8. 8 = 7;/4, 0 = 7; 46. Inside r = d m
47. r = 3 cos 2, 0 = 0, 8 = 4 6 48. Inside one leaf of r = sin no, n a positive integer
49. Enclosed by r = 2 cos 48 50. Inside r = 2 sin 8, outside r = sin 8 + cos 8
5 1. Enclosed by r = sin 58 52. Inside r 2 = 2 cos 28, outside r = 1
53. Inside r = 2 d m , outside r = fi 54. Inside r = 2, outside r = 2( 1 - cos 6' )
55. Inside r = 3, outside r = 3(1 - cos 0 ) 56. Enclosed by r2 = 2 cos 28
57. Inside r 2 = 2 cos 20, outside T = 1 58. Inside r = 2 sin 8 and r = 2 cos 8
59. Inside r = 3 cos 0, outside r = 1 + cos 0 60. Inside r = 2 c0s2(8/2) and r = 2 sin2(8/2)
6 1. Inside r = 2 cos 30 62. Inside r = 2 - cos0
63. +
Inside r = 1 sin 0 and the lower loop of r2 = 64. Inside one leaf of r = cos no, n a positive integer
(1/2)sin 6'

In Exercises 65 to 72, ( a ) determine a deJnite integral that gives the length Ofthe indicated arc and ( b ) evaluate that integral.
85. r = e-', O 5 0 5 27i 66. r = 2', 0 5 8 2 7i
67. r = c0s2(8/2), 0 5 0 5 2 7 68. r = 1 - sine, 0 5 8 5 277
69. r = 282, 0 5 8 5 27i 70. r = l/8, 7i/2 2 8 5 7i
71. r = cos3(8/3), 0 2 8 5 3 ~ / 2 72. r = 4 - 4cos8, 0 2 8 5 27i
SECTION 13.3: AREA AND ARC LENGTH IN POLAR COORDINATES 455

In Exercises 73 to 78, ( a ) determine u d4nite integral giving the area @the surface generated by revolving the indicated arc about the indicated axis, and
( b ) evaluate that integral.
73. r = 6 sec 8, 0 5 8 5 7 / 4 , y axis 74. r = 2(1 - c o s 8 ) , 0 2 8 2 7,x axis
75. r2 = 4 cos 28,O 5 0 5 7 / 6 , x axis 7 6 . Arc as in Exercise 75 about y axis
77. One loop of r 2 = 4 cos 28 about x axis 78. One loop of r 2 = 4 sin 28 about y axis

Miscellaneous Exercises
1. Graph r = -3 2. Graph the hyberbolic spiral r = 2a/8
+
3. Graph Freeths nephroid r = 1 2 sin(8/2) 4. ( a ) O n the same graph, sketch r = 2 + cos8,
5. ( a ) O n the same graph, sketch r = 2 +
4cos8, + +
r = 2 2 cos 8, r = 1 2 cos 8, r = 1/8 + 2 cos 8.
+
r = 2 2 cos 8, r = 2 + cos 8, r = 2 (1/8)cos 8. + ( b ) Describe the shape of the limacon r = a + 2 cos 8 as
( b ) Describe the shape of the limacon r = 2 + b cos 8 as a + 0.
b + 0. (c) Use the answer to ( b ) to explain why the circle
(c) Discuss the answer to ( b ) in terms of the graph of the r = 2 cos 8 is traced out twice as 8: 0 -+ 2 7
circle r = 2. 7. Describe the graph of l/r = 3 + 6 sin 8 .
6. (Oblique coordinate system) Suppose a rectangular xy 8. Determine the mean value of f ( 8 ) = 1 - sin0 on
coordinate system is given on the plane. We define the [O, 2771.
60 oblique uu coordinate system as follows: Let the u 9. Determine the area of the region shaded in the accom-
axis coincide with the x axis. The u axis is a line through panying sketch. (Each circle has radius a , )
the origin making a 60 angle with the positive u axis.
Then any point P in the plane is on the intersection of a
pair of lines u = uo and L] = uo, where u = uo is a line
parallel to the u axis and intersecting the u axis at uo, and 9 = n/2
u = uo is a line parallel to the u axis and intersecting the
u axis at uo. We refer to the pair (uo, uo) as the uv coordi- t
nates for P (see sketch).

Exercises 10 to 1 3 are locus problems in polar coordinates.


10. Obtain a polar equation for the cissoid of Chap. 12,
( a ) Prove that the equations relating xy and uu coordi- Miscellaneous Exercise 36.
nates arey = u d / 2 , x = u +
(u/2). 11. Suppose F, has (polar) coordinates ( a , 0), F, has (polar)
( b ) Determine the xy coordinates of a point with uu coordinates_ ( a , a-) . Show that the locus of all points P
coordinates (1, 1). satisfying (PF,)(PF,) = a2 is a lemniscate.
( c ) Determine the uu coordinates of a point with xy 12. (Ovals of Cassini) For F,, F2 as in Exercise 1 1, determine
coordinates (4, 2). an for the locus of all points P such that
_ equation
_
(4 Repeat (c) for (4,O). (PF,j(PF2) = b2. Show that for a = b a lemniscate re-
( e ) Repeat (c) for (0, d) sults.
(f)Translate the xy circle x 2 + y 2 = a2 into uu coordi- 13. Determine the locus of all points P equidistant from the
nates. pole and the line r = csc 8.
( g ) Translate the xy liney = mx into uu coordinates. 14. Determine the angle $ at an arbitrary point of the lituus
( h ) Translate the uu line u = mu into xy coordinates. r28 = a2,
(ij Derive a distance formula for the uu system.
( j ) Determine the uu distance between the point with xy
coordinates (1, 1) and the origin. ,
456 CHAPTER 13: POLAR COORDINATES

SELF-TEST
Chapter 1 4 X

457
458 CHAPTER 14: INDETERMINATE FORMS AND IMPROPER INTEGRALS

Section 14.1 : Indeterminate Forms and LHospitals Rules

Objective: LIMITS OF QUOTIENTS


Evaluate limits of quotients when Earlier in studying limits of quotients
both the numerator and
denominator tend to 0 or both tend
to f w .
we saw that a variety of things can happen, depending on the behavior off and
g near a. The simplest case occurs when lim,,,f(x) = k and limz-a g(x) =
j # 0, because then the Limit Theorem (2.5.4) applies to tell us that

However, if g(x) approaches 0 through positive values, say, while


limx-a f(x) = k # 0, then x = a is a vertical asymptote, with

lim-f (4 = k00
x-a g(x)

depending on whether k >0 or k <


0, respectively. (See, for example, Fig.
14.1.) O n the other hand, if limx-a g(x) = k c o , and limx+af(x) = k , a real
number, then

Fig. 14.1 (You may wish to review Sec. 4.1.)


Of special interest are the hard cases where both the numerator and
denominator tend to 0, or both tend to k00We . have special names for such
situations.

14.1.1 Definition If limX,J(x) = limx+ag(x) = 0, then the limit of the quotient,


limx+a [f(x)/g(x)] is said to have indeterminateform O/O. If limx-a f(x) = 2 00 and
limx-a g(x) = t00, then limx,a [f(x)/g(x)] is said to have indeterminate form
C O / C O . These definitions also apply for one-sided limits as well as for a = e00
(limits at infinity).

We shall deal with the 0/0 case first. Actually, this case is not new or rare
because, for example, it occurs every time we use the definition to calculate the
derivative of a differentiable function:

lim f ( x + Ax) -fb)


Ax-0 Ax
Here limAx,o [f(x +A x ) - f(x)] = 0. (Why?)
Through algebraic manipulations we were able to deal with such situations
quite handily until we came to the derivative of sinx, when we needed a n
elaborate geometric argument to show that
sinx =
lim -
a-0 x

We will now present a collection of techniques which are all versions of what
is known as LHospitals Rule, named after the seventeenth century French
mathematician G. F. de LHospital, who included them in his calculus text, the
first ever published (1 696). Actually, the techniques were invented by his
SECTION 14.1: INDETERMINATE FORMS AND LHOSPITALS RULES 459

teacher, Johann Bernoulli (the same man who brought us logarithmic differ-
s entiation), whose work was the basis for LHospitals text.
The basic idea of all the versions of LHospitals Rule is to evaluate the limit
of the quotientf ( x ) / g ( x ) by evaluating the limit of the quotients of the deriva-
tives f (x)/g(x).An oversimplified motivation for this approach is that when an
zndeterminate form is involved, the limit of the quotient is really determined by the relative
rates of change of the numerator and denominator as x + a. How do we compare their rates
of change? Using thezr derzvatives, of course! An informal geometric analysis helps
X
show us what form the rule should take.
Assume that a is a real number with limx.+u f(x) = limx+ug ( x ) = 0, so in Fig.
14.2 the graphs of f(x) and g ( x ) both cross the x axis at A = ( a , 0).
Fig. 14.2 By definition of the quotient of two functions,f ( x ) / g ( x ) is the quotient of the
vertical lengths FX and GX. We know that the respective slopes of the secant
lines through A and F and through A and G are F X / X A and G X I X A . We also
know that

14.1.2
FX - F X / X A
-~
G X - GX/XA
Now, as x + a, the point X + A , which means the two secant lines FA and
GA approach the respective tangent lines Tf and q.Thus the quotient of the
secant line slopes on the right side of (14.1.2) approaches the quotient of the
tangent line slopes; that is, they approach the quotient of the derivatives. Hence
the quotient on the left side of (14.1.2), which is really f ( x ) / g ( x ) , must likewise
approach the quotient of the derivatives at a. Although this is not the precise
conclusion of LHospitals Rule, it is the form most often used in practice. We
now state the rule in a more precise and general form. A complete proof can be
found in any book on advanced calculus.

14.1.3 LHospitals Rule: 0 / 0 Suppose f(x) and g(x) exist near a, and
g(x)# 0 near a with limx,u f ( x ) = lim,,,g(x) = 0. To determine
limx+a[ f ( x ) / g ( x ) ]calculatef ( x )andg(x) separately and evaluate the limit of the
quotient f (x)/g(x). The conclusion is as follows:
If limf(*) = L , then lim f-(2) = L

x - m g(x) x-a g ( x )

This rule holds whether a and L are real numbers or k co. It holds for left- and
right-hand limits as well.

Note that if a is a real number, the phrase all x near a means all x in some
open interval containing a but not x = a (often called a deleted neighborhood
+
of a). If a = co,this phrase means all x larger than some real number N, that
+
is, all x in ( N , co). Similarly, for a = - 00, it means all x in ( - co,N ) for some
real number N.
We first apply the rule where we are confident of the answer-before moving
to cases where the answer is not at all obvious.

Example 1 Compute lim -.sin x


2-0 x

Solution The hypotheses hold since numerator and denominator are everywhere differ-
entiable and limx,o sin x = 0 = limx,o x.D,(sin x) = cos x andD,(x) = 1. Now
cos
lim-= 1
x-0 1
462 CHAPTER 14: INDETERMINATE FORMS AND IMPROPER INTEGRALS

We can now prove a generalized form of ( 7 . 5 . 2 ) ,a very useful pair of limits,


the second of which generalizes Example 5 .

14.1.5 Tpeorem

In x
(u) If p is any positive pumber, lim -= 0.
x++m xp

XP
( b ) If p is any number, lirn - = 0.
X++X ex

ProoJ
( u ) The hypotheses of (14.1.4) hold. (Note that In x grows without bound as x
does.) Thus

Since p > 0, this latter limit is zero; hence the original limit is likewise zero.
( b ) We consider two cases, (i) p 5 0 and (ii) p > 0.
Case (i). If p 5 9, then -p 2 0, so we can conclude that
lime++ml/(x-Per) = 0 without L'Hospital's Rule.
>
Case (ii). If p 0, then L'Hospital's Rule applies, so we differentiate
numerator and denominator as many times as are necessary to lower the degree
of the numerator to zero or below, where case (i) applies. W

From (14.1.5), we can conclude that l n x grows slower than any positive
power of x and ex growsfaster than any power of x. (See Exercises 39 and 40.)
Often it is necessary to perform a variety of manipulations of the limit with
different indeterminate forms appearing in the same problem.

Example 6 Determine lirn -.In x


.+o+ cot x
Solution We know that lirn,,,+ In x = - co and lim,,,+ cot x = + co,so (14.1.4) applies.
Hence we differentiate and then evaluate

lim ~
1/x
3-0+ -cscax
But this limit is indeterminate, of the form co/m again, and the derivatives of
the numerator and denominator do not look as if they will help resolve the
problem. Hence we write csc2 x = l/sin2 x and rearrange:

lim - 1,' = lim --sin2x


2-o+-csc2x x-o+ X

Although this latter limit is of indeterminate form O / O , we can use (14.1.3) to


reduce it to the following:

lim -2sinxcosx 0 =
=- o
x-o+ 1 1
Hence
In x
lim- =0 0
r-O+ cot x
SECTION 14.1: INDETERMINATE FORMS AND L'HOSPITAL'S RULES 463

PROGRESS TEST 2

Evaluate the given limits.


1. lim
1 secx + 2. lim
ln(x - 1) 3. lim -
x+ lnx
~-h/z)- tan x s-1+ l/(x - 1) p-+m xInx

A CAUTION
If we use L'Hospital's Rule in an attempt to evaluate
X
lim
z-o+ x2 + cosx
we obtain
1
lim = fco
2-o+ 2x - sinx
However, looking again at the original limit, we find that

lim x ----0
-0
z+o+ x2 + cosx 0+ 1
L'Hospital's Rule does not apply, since the original limit is neither of the two
basic indeterminate forms! The moral: Don't use L'Hospital's Rule untilyou check to
see that its hypotheses are satisjied. T h e original limit must be an indeterminate form.

SECTIOS 1 4 . 1 EXERCISES

In Exercises I to 33 evaluate the given limit.

lirn 3x. - ta nx
,a: - e-"
1. lim L- 2. lim- 3. 4. limTsin x
X-o tan x x-o sinx 2-0 sinx - 3x 2-0 x
I

5. lim -
fi 6. limp
Vl - x
7. lim
cos x - 1
8. lim-
1 -ee"
X-o sinx 2-1- Inx 0-0 x x-o x

9. lim 3cosx +x -3 10.


e-'
lim - 11. lim-
In x 12. lim sec x
-
x-0 x3 x-+o x2 e-o cscx X-TIZsec 4x
xex -x sin2 x arccos x (In
13. lim 14. lim- 15. l i m p 16. lirn -
2-0 1 - cos 2x s-o sinx2 1-1 x - 1 a-+m $5
eZX - cos 2x ln(x/a) tanh x
17. lim 18. lim- 19. 1 i m X 20. lim-
Z-o 2xsin2x x-a x -a x-0 tan x 2-0 tanh 3x

21.
sinh x
lim - 22. lim
In tan x
- 23. lirn
1n(x3 + 24. lim-
2" + 3"
x-0 x x - ( ~ i ~ / 2 ) +In
sec x r-+m in(x3 - 5-5 sx - 1

25.
lim x - arctan x 26. lim arcsinx -x 27. lim ta nx - sinx 28. lim- In tan 2x
2-0 tan3 x x-0 x3 s-o tan3 x e - ~1nsin3x

x - xe" 1/x2 2 cos x - 2 + x2


29. lim - 30. lim ____ 32. lirn
0-0 sin2% z-+= sin2(I/x) x-0 x4

33.
lim ex - sinx - 1 37. Use L'Hospital's Rule to determine any horizontal tan-
1 - cos 2x
x-0
gents to the cardioid r = 2(1 +
cos 0 ) .
34. Determine the limit in Exercise 30 without using 38. Use L'Hospital's Rule to show that the cycloid
L'Hospital's Rule. x = t - sin t, y = t - cos t has a vertical tangent at
35. (a) Show that the hypotheses of L'Hospital's Rule t = 0. (See Chap. 12, page 401.)
(14.1.3) hold for 1imx.-- ( e " / b x ) , which is indeterminate (In x)Q - 0 for all positive p
of form 0/0 provided b >
1. 39. Extend [14.1.5(u)] to lirn --
and q. x-+a: xp
( b ) Show that (14.1.3) does not determine this limit.
40. Extend [14.1.5(b)] to lim - X P = 0 for any p and any
(c) Determine the limit using other means.
positive q. x-+= eqx
36. Draw a sketch analogous to Fig. 14.3 to confirm that
lim,,o(es - l ) / x = 1.
464 CHAPTER 14. INDETERMINATE FORMS AND IMPROPER INTEGRALS

Section 14.2: Other Indeterminate Forms

Objectives: THE INDETERMINATE FORMS 0 * cc AND cc - 00

1 . Transform indeterminate limits Consider the following limit of a product:


into forms considered in Sec. 14.1.
lim x /In xi
3-o+
2. Use the properties of the natural
logarithm function to evaluate As x + O+, the first factor gets smaller while the second grows without bound.
indeterminate forms involving The limit of the product cannot be determined without further analysis.
variable exponents. Similarly, we cannot determine the following without further analysis:

lim
0-0
(+ - 2)
sin x

14.2.1 Definition
( a ) If lim,,,f(x) = 0 and lima+ag ( x ) = t o o , then limx+a [ f ( x ) g ( x ) ]is said to
be of indeterminate form 0 * co.
( b ) If, as x + a, one o f f @ ) or g ( x ) grows without bound while the other
decreases without bound, then limx-a (f(x) + g ( x ) ) is said to be of indeter-
minate form co - co. (Roughly speaking, f ( x ) and g ( x ) pull in opposite
directions, and the winner is not obvious.)

Our approach in both of these situations will be to use appropriate manip-


ulations and/or identities to reduce these to indeterminate forms considered in
Sec. 14.1. In general, we handle the indeterminate form 0 co using -

The indeterminate form co - co may involve a variety of manipulations,


ending with a fraction whose limit is indeterminate of form 0/0 or co/co.

Example 1 Evaluate lirn xlln x


x-O+
1.
Solution Consider lim,-,+ [lnxl/(l/x), which is of form co/co. Hence we apply (14.1.4)
to get

lim = lim ( - x ) = 0
x-o+ - 1/ x 2 x-o+

Thus the original limit is 0. (You are invited to try the alternative ap-
proach.) 0

Example 2 Evaluate lim (a is any real number).


2-0

Solution A typical technique is to combine terms over a common denominator:


eax - 1
lim ~

a-o xeax
This limit is indeterminate of form O / O , so we differentiate. But

lim
aeax - --a - a
-
+-o xaeax eax 0 1+ +
so the original limit is a. 0
SECTION 14.2: OTHER INDETERMINATE FORMS 465

It is a mistake to assume that every limit can be handled as routinely as those


treated thus far. Sometimes a variety of indeterminate forms may arise in the
same problem, requiring a degree of ingenuity. Follow the next example closely.

Example 3 Evaluate lirn (cot x


x-OL
+ In x ) .
Solution This limit is of indeterminate form co - 00, so we rewrite it

(1) lim - [
'0' x + ln x ] =
X-o+ sinx
~l$ cos x +sinsinx x In x
We know that the denominator approaches 0 through positive values and cos x
approaches 1. But sin x In x is of indeterminate form 0 00 and needs special
treatment. Now
In x
lim (sin x In x) = lim ~

x-o+ x-o+ l/sin x

which is of indeterminate form 0 0 / c o , so we apply (14.1.4), differentiating


numerator and denominator to arrive at

lim
x-o+
[( - cos x/sin2 x)
] = lim
(1/x)sin2 x
cos x
x-O+
= lim
-
[(sin x)/x]sin x
-cos xx-O+

We already know that limx,o+(sinx)/x = 1, so

lim
[(sin x)/x]sin x
--1 * 0
- =o
2-o+ -cosx -1
Hence lim,,l+(sin x In x) = 0 also.
Return to the original limit rewritten on the right side of (1). As x + 0+, the
numerator approaches 1 + 0, while the denominator approaches O+, so the
fraction grows without bound. Thus
lim(cot x
x-o+
+ In x) = + co 0

PROGRESS TEST 1

Evaluate the given limits.


1. lim
x-(?r/2)-
(tan x - sec x) (rewrite in terms of sines 2. lim
x-i+
(A
x -1
- 1-)l n x
and cosines)
3. lim [tan x ln(sin x)] (use l/tan x = cot x )
x-(?r/2)-

THE INDETERMINATE FORMS O", lm, and coo

In some situations a limit of the form

may be indeterminate because the base may be approaching 1 while the


exponent grows without bound. This was the case in (7.4.11) and (7.4.12), page
267, where we described e as

e = lim (1
5-+m
++)" and e = lim(1
0-0
+ x)l/x
466 CHAPTER 14: INDETERMINATE FORMS AND IMPROPER INTEGRALS

These are said to be indeterminate of form I =. Other similar indeterminate forms


are coo and Oo.
Our strategy for dealing with such indeterminate forms that arise when
evaluating

is analogous to our use of logarithmic differentiation in differentiating f (x)dx):


Take the natural logarithm and then use the properties of the natural logarithm to simp1;fV
the situation. The original indeterminate form will thus be transformed into one
that we already know how to handle.
Our first case will be used frequently in later chapters.

14.2.2 Theorem lim (I


x-+x
+ -$= er
Prooj This limit is indeterminate of form 1". Lety = [l + (r/x)]' and take
natural logarithms. We know that

because the natural logarithm function is continuous. (This enables us to


interchange In and lirn as necessary.) Now

and this latter limit is indeterminate of form co * 0. Rewrite it as

lim
ln(l + r/x)
x-+- l/x
which is of form O / O . Apply L'Hospital's Rule (14.1.3):

Thus
In[ lim y ] = r
x-+r

Knowing this, we can conclude that


lim y = e" W
x-+CC

The last step of the above proof is important since up to that point we have
not found the original limit, but the logarithm of that limit. Hence we need
apply the exponential function to "undo" the logarithm.

Example 4 Determine lirn x x 2 .


a-O+

Solution This limit is of form Oo. Now


In [ lim xx'] = lim [In(xx211
x-o+ x-o+
= lim [ x 2 In x]
x-o+

(which is indeterminate of form 0 cc))

= lim- In x (form c o / o o )
x-o+ 1/x2
SECTION 14.2: OTHER INDETERMINATE FORMS 467

Hence we differentiate, getting

Thus ln(1im x x 2 ) = 0, so
x-O+

PROGRESS TEST 2

Evaluate the given limits.


1. lim
x-+ffi
(7)
x + 2 2. lirn Pina
a-O+

A S EXTENSIOS TO OCx
The logarithm strategy works on limits involvingf(x)g(') even when the limit is
not indeterminate, as the proof of the following theorem shows.

14.2.3 Theorem Suppose that limf(x) = 0 withf(x)


x+a
> 0 for all x near a.
(a) If limg(x) =
+-a
+ cc), then limf(x)g(x) = 0.
x-a

(6) If lim g ( x ) = - co, then limf(x)g(") =


x-a a-a
+ co.
Prooj We prove ( a ) and leave ( 6 ) as Exercise 35.
In[ l i m f ( ~ ) g ' ~ =
ria
) ] lim[ln(f(x)g(x))]=
c-a
b i [g(x)lnf(x)]
As x approaches a, the first factor g ( x ) grows without bound while the second
factor decreases without bound (sincef(x) + 0 ) . Hence

which implies

SECTIOS 14.2 EXERCISES

In Exercises 1 t o 34 evaluate the given limit.


1. lim x tan(l/x) 2. lirn x sin(l/x)
Z-+oC X-+a

1
3. lim(x - l)tan(.i;x/2) 4. lim - - -
x-1 a-o [ x ? xzsecx
l l
5. l i m 1[ r
Z-o e - 1
- 2 1
arctanx
6. lirn e-"lnx
X-+=

7. lim[_l__i]
Z-o sinx 8. lirn ['2 sec*x - 1 - sinx
x+lii/2

9. limxcsc 2x 10. lim xx


x-0 x-o-
I I. lim(cos 3 ~ ) ~ " 12. lirn csc x arcsin x
x-0+ 2-o+

13. lim ( e l
x-o-
+ x)"' 14. lim(1
a-0'
+ x)lna
468 CHAPTER 14: INDETERMINATE FORMS AND IMPROPER INTEGRALS

15. lim(1n x)' 16. lim (1 - x)Cnt


x-o- x-0-

17. lim xl'lns 18.


x-o+
19. lim(l/x)slns 20. lim (cos I/x)"
x-0' z-+r

21. lim(1 - ~ 2 ) 1 / ( 1 - ~ ) 22. lim (sec x)cot


x-1- x-o+
23. lim(1
x-1
+ In x)cos(l-x) 24.

25. lim (tan2 x ) c o s 2 26.


x-.T/2

27. lim(ax + l)cotx (a >0) cos 2 x + cos 3x 1/xZ


x-o- 28.
l2-0
im( )
29. lirn [In(x + d m ) - In x ] 3 0. lim(sec x - cos x ) Za ~ ~ ~
x-o- x-0

3 1. lim (cosh ~ ) l / ~ 32.


x-+,

33. lim [csc2x


x-0
- l/x2] 34. Iim (sec 2x)cot'
x-a+

35. Prove [14.2.3(b)J. 36. Compare lim - 2 cot x and lim 1 - 2 c o t x


x-o+ cscx 2-0- csc x
37. Compare lim(ex
x-o-
+ x)l/' with the limit in Exercise 13. 38. Prove that lim[e-1/x2/x] = 0.
2-0

39. Compute Iim 2[(l/r)-csc21


8-0

Section 14.3: Improper Integrals

Objectives: INTEGRALS WITH IKFIXITE LIMITS OF ISTEGRATION


1. Evaluate integrals with infinite This section deals with two types of integrals commonly called "improper." Our
limits of integration. first concern is in evaluating integrals such as
2. Evaluate integrals whose
integrands grow or decrease
without bound between the limits of
integration. which may be thought of as representing the area of the region to the right of
J x = 1 between the x axis a n d y = 1/x2. (See Fig. 14.4.)
Definite integrals have only been defined over finite intervals, but the basic

t \=$
tool of calculus, namely limit, allows us to extend the definition in a natural
way. For this case we have
1000 dx
1
(O.$ = 0.9 IIOO.$ = 0.99

and thus it is natural to define


I = 0.999
b

t = l

Fig. 14.4

14.3.1 Definition
( a ) Suppose f ( x ) is integrable in [a, b] for every b in [a, 00). +If
limb,+, J : f ( x ) d x exists, then we say Sa+"f(x) is convergent, and define
J i " f ( x ) dx = J : f ( x ) dx.
SECTION 1 4 3: IMPROPER INTEGRALS 469

( b ) Suppose f(x) is integrable in [ a , b ] for every a in ( - m , b]. If


lima+-x Jif(x) du exists, then we say JbPf(x) dx is convergent, and define
fbxf(x) dx = lima+-mJ l f ( x ) dx.
(c) Supposef(x) is integrable over all finite intervals [a, b] and c is any real
number. If both J?J'(x) dx and Jcf"f(x) dx are convergent, then we define
J?zf(x) dx = JEXf(x) dx fcf"f(x) dx. +
If any of the limits in ( a ) to (c) fails to exist and be finite, we say that the
associated integral is divergent.
By [14.3.l(a)],we see that 1:" dx/x2 is convergent and equals 1.

Example 1 Evaluate s
4
+-m
-.dx
6

Solution

= lirn [ 2 f i - 4 ] = +cc
b-+m

Hence
4
5 is divergent. 0

Example 2 Evaluate s
--s
+X

---.
x2
dx
+1
Solution By definition, we must evaluate
rc dx. r+x dx
J--a
W and Jc x2 +1
where c is any real number. For convenience we can choose c = 0. Now

S
--m
O d u
= a*-=
lim
dx
lo-
= lim arctanx

= lim [arctan 0 - arctan a ]


[ I,"
a--x

= lim (-arctan
a--m
a) = - lim arctan a = -
a+-%
(- ): 7l
=-
2
(We can expect a nonnegative answer here since the integrand is positive for
all x.) O n the other hand
+" dx dx
- lim
= lim [arctan b - arctan 01
b-+ X

t Hence
b-+m
?7
= lim arctan b = -
2

I Example (2) gives us the area of the region between the curvey = 1/(x2 + 1)
Fig. 14.5 and the x axis. (See Fig. 14.5.)
470 CHAPTER 14: INDETERMINATE FORMS AND IMPROPER INTEGRALS

Example 3 Evaluate i +3:

cos x dx.

+X

Solution
b-+ x

= lim [sin b - sin 01


b-fa

= lim [sin b]
b-+ x

This limit does not exist (oscillates between + 1 and - 11, so Jim
cos x dx is
divergent. 0

Example 4 Compare lirn


t-+m
s
-t
t
4x3 dx and s
-x
+a

4x3 dx.

Solution lim f t 4x3dx = lim r x 4 1 t

= lim 0 = 0
t-+r

O n the other hand,

1,
+X

4x3 dx = lim
a+-m
s 0
4x3 dx + >jyx i b
4x3 dx

r :ither of these limits is a real number, so the integral J?: 4x3 dx diverges.
Since
1,+.r

4x3dx # lim
t-+x
s -t
t
4x3dx

the latter limit cannot be used as a shortcut means of evaluating J?: 4x3 dx. 0

PROGRESS TEST 1

Evaluate these improper integrals:


+x 21nx
dx 2. s
-x
0
ex dx 3. 1,
+ X

~e-"~dx

ISTEGRALS WITH ISFIYITE ISTEGRANDS


Can thejrst quadrant region to the lej? o f x = I and belowy = I / x z be regarded as having
j n i t e area? (See Fig. 14.4, page 468.) Unfortunately, neither our definition of
definite integral based on limits of finite sums nor the Fundamental Theorem of
Calculus applies.
SECTION 14.3: IMPROPER INTEGRALS 47 1

The function is not bounded on [0, 11 and is thus not integrable on [0, 11.
However,y = 1/x2 is continuous on [ E , 11 for any 0 E 5 1, so J,' 1/x2 dx does <
exist. Again, we can use a limit to answer the above questions:

lim
E'OL J'$=
, lim[-t]:=!iF[-1
,-Of +$I= +w
Although in this case the region does not have a finite area, the technique used
to answer the question makes sense in general.

14.3.2 Definition
( a ) Supposef(x) is integrable on [a +
e , b ] for every a E in (a, b] ( E + >
0 ) , and
lim2-atf(x) = t w . If lim,,,+ JB+,f(x) dx exists, then we say J B f ( x ) dx is
convergent and define J;f(x) dx = limE-O+ J:+,f(x) dx.
( b ) Supposef(x) is integrable on [a, b - E ] for every b - E in [a, b ) ( E 0), and >
limz.b-f(x) = tw . If limE+o+ J;-'f(x) dx exists, then we say JBf(x) dx is
convergent and define JBf(x) dx = lim,+o+f-'f(x) dx.
(c) Suppose limz-,cLf(x) = t o o and/or lim2,c-f(x) = tw for u c < <
b, and
both Jgf(x)dx and J%f(x)dx are convergent. Then we say J:f(x)dx is
convergent and define J;f(x) dx = Jgf(x) dx J;f(x) dx. +
If any of the limits in ( u ) to (c) fails to exist and be finite, then we say the
associated integral is divergent.

By (14.3.2) and our earlier calculations, JA


dx/x2 is divergent. A similar
calculation shows that Soi dx/x2 is divergent. Thus, by [14.3.2(b)],JLi dx/x2 is
divergent. However, if we had somehow ignored or forgotten the discontinuity
at 0 in [ - 1 , 11 and tried to apply the Fundamental Theorem of Calculus, we
would get

x = l n = l + t x = 2

Fig. 14.6 which is nonsense because the integral of a positive function could never be
negative.

Example 5 Evaluate 1 (x -
dx
11213
. (See Fig. 14.6.)

Solution Here lim = +w,so


(. - 11213
dx dx
= lim
(. - 11213
2

It is best to use two different limit varizbles in dealing with integrals when
the point of discontinuity lies strictly between a and b.

2xdx
Example 6 Evaluate
2(. - 11213

Solution lim
z-l (x2
2x
- 11213
= + w , so we must use [14.3.2(c)].
472 CHAPTER 14: INDETERMINATE FORMS AND IMPROPER INTEGRALS

= lirn [3( -2e


e-O+
+ e 2 ) l 1 3 + 31 + lirn [6 - 3(28 + 82)1/3]
S-OC

= 3+6 = 9 0

In some cases it may be necessary to use LHospitals Rule in evaluating the


limit associated with an improper integral.

Example 7 Evaluate 4 In x dx.

Solution Since lim (lnx) = -00,


x-07

1 1
lnx dx = lirn lnxdx
E-o+

(integration by parts)

= l i m [ - 1 - & E ~ E + E=] -1 - lim(E1nE) (0.00)


E-0- E-O+

In E
It remains to evaluate the indeterminate form lim -, By LHospitals Rule,
E-o+ 1/E

1/& . In&
lim ~ = 0 = lirn -
E+oc - I/& &-0+ l/&
Hence

PROGRESS TEST 2

Evaluate the given improper integrals.

s2 d&
0
2. JS3
dx
(. - 11415
3. i4-$5
IMPROPER INTEGRALS INVOLVING POWERS OF X
Improper integrals involving the antiderivative

occur frequently enough, particularly in the study of infinite series, to warrant


systematic treatment. Their straightforward proofs are left as exercises.

14.3.3 Theorem
(a) p 5 1 implies i+w
$ diverges.

(b) p > 1 implies [+= $ converges and equals -.P -11


SECTION 14.3: IMPROPER INTEGRALS 473

14.3.4 Theorem
(u) p < 1 implies 11$ converges and equals -.
1-P
1

(6) p 2 1 implies il:


- diverges.

SECTION 14.3 EXERCISES

In Exercises I to 40 evaluate the given integral.

1. 2. s-?dx
0
dx dx
5. 7. 8.

9. 11. J l x I n x d x 12. J5sec x tan x dx


0 0

13. 15. 1; e- cos x dx 16. l +%

e-sinxdx

17. 20. SCrn


n
sin x dx

21.

xdx dx
25. 27. J_-+* -
x4 + 1

29.
-,-7/2
31. l+- x2
dx
- 2x + 10 32. J+*
x 2 - 6x
dx
+ 11
33.
1: 4x2
dx
+ 12x + 13
34. J+r e-x sin 3x dx
0
35. 1 7/2
secxdx

37. 39. J+- dx (Note: You will need to evaluate three limits.)
-1 fi
e-%
40. J
+I.

p dx (Note: You will need to evaluate four limits.) 5 1. Determine the centroid of the semispherical surface

41.
--m

(a) Is s:$ dx proper or improper?


generated by revolvingy = d m , 0 5 x 5 r , about
t h e y axis.
52. Determine the total first-quadrant area between thex and
( b ) Evaluate the integral in ( a ) . y axis a n d y = e-x.

42. Repeat Exercise 41 for J+v


-7/2
2 53. (A fake paradox) Let R be the region bounded by the line
x = 1, the positive x axis and the graph ofy = l / x , and
let S be the solid of revolution generated by rotating R
43. Prove (14.3.31. 44. Prove (14.3.4). about the x axis.
45. Generalize (14.3.3) to powers of x - c. ( a ) Prove that S has finite volume.
46. Generalize (14.3.4) to powers of x - c. ( b ) Prove that S does not have finite surface area.
47. ( a ) Generalize (14.3.3) by replacing the limit of integra- ( c ) If S were turned to a vertical position and filled with
tion by any a 0. > paint, then we would have a finite amount of paint
( b ) Generalize (14.3.4) by replacing the limit of integra- covering a n infinite surface area. Chapter 15 will
tion by any a 0. > help clarify this situation. Explain why this is a fake

48. Prove that i+=


$ diverges for all real numbers p
paradox.
54. Let R be the region bounded b y y = x - ~ , x = 1, and
y = 0.
49. Without using parametric equations or polar coordinates,
but rather using an improper integral, show that the
( a ) Show that R has infinite area.
circumference of a circle of radius r is 27ir.
( b ) Show that the volume of the solid generated by
revolving R about the x axis is finite.
50. Use the approach of Exercise 49 to show that the surface
area of a sphere is 47;r2.
55. Determine the length of the spiral r = e-O for 8 2 0.
474 CHAPTER 14: INDETERMINATE FORMS AND IMPROPER INTEGRALS

56. Compute the area between the curvesy = k l / & to continues to return $2000/month income with money
the right of x = 4. returning 8 percent. What is its market value (the
+m present value of its unending income stream)? (Note, of
57. (a)
, , Prove that f + X x k e - x d.x = k f xk-le-x dx. course, that the total income is infinite.)
JO JO
59. X (See Exercise 58 and Sec. 11.1, Exercise 50.) Suppose
( b ) Use (a) to show that for n a positive integer, the same apartment complex of Exercise 58 returns a n
4 +m
xne-xdx = n ! unending monthly income stream that grows linearly
from $O/month at a rate of $1 per year. What now is its
market value?
58. ' X Suppose the apartment complex of Example 5, Sec.
1 8.1, is assumed to have infinite life during which it

CHAPTER EXERCISES

R e v i e w Exercises
In Exercises I to 36 evaluate the given limit.

1. lim -
sin ~ T X
2. lim
x cos x - sin x
3. lim- secx - 1
2--1 1+x x-0 X a-o x2
l i m sec x tan x sin x cot 2x
4. 5. lim- 6. lim-
x-0 2x x-o+ & 5-0 cot 3x
In sin x + 3x)l/"
7. lim- 8. lim x tan-2 9. lim(e"
x-o In tan x X-+m X x-0
cos mx In x 2
10. lim sin x In x 11. lim- (m,n odd integers) 12. lim-
x-O+ X-7 cos nx a-o cot x 2
ex + 2 e -~ 3 ~ sinx + 3x
13. l i m L 14. lim 15. lim
2-0 COtX x-0 1 - cos 3x 2-0 2x
x ln(1 + x)
16. lirn cot x In cos x 17. 18. lirn
2- 1 a-0 1 - cosx
19. lim(cos
2-0
20. lim (I
x-+m
- x2)e-x'

23. lirn
+
sin x cos x - 1
24. lirn
1 - cosx - (x3/6)
x - ~ / z 1 + cos x - sin x x-0 x4

26. lim (sin x)sec" 27. lim(sin x In x)


"-k/2)- x-o+
28. lim-
sin x2 3x - 2"
29. lirn ___
30. l i m [ ln(1 + x)
2-0 xsinx z-+m x x-0 X2 x(l + x)
31. lim- 3" - 2 x 32. lim-
tan x
33. lim - sin mx
[m,n even integers]
2-0 x x-o+ x ~ - 7 1 2 sinnx

34. lim (sin x)tan20 35. lim-


In tan x lim ex - 2 sinx - e-"
36.
X-T/ 2 X-O+ In tan 3x x-0 sinx - x

In Exercises 37 to 56 evaluate the given integral.


dx
37. Ja'2tan2xdx
0
38.
I, d2 +
2

x - x2

xlnx
dx
47. 4' (In x)2 dx
SECTION 14.3: IMPROPER INTEGRALS 473

49. 50. J*sec x dx


0

52. 53. 4 +
e-2sinxdx 54. l+= e c Z x cosh x dx

a xdx
55. 56.

57. Compare lim(x - l)l(x*-l)with the limit in Exercise 22


x-1- ( b ) What is the volume of the solid of revolution gener-
above. ated by rotating this region about the x axis?
58. Compare lim x1I0 and lim x1I2. ( c ) What is the surface area of this solid?

59.
x+o+ 2-+r

Evaluate the following limit 63. ( a ) 1s ~1% dx improper?

lim 2x4 - iox2


+
12 ( b ) Use the Trapezoid Rule with eight subdivisions to
2-a +
x4 x2 - 6 get an estimate for the integral in ( a ) .
( a ) Using LHospitals Rule and ( 6 ) by some other
means.
64. ( a ) Evaluate . ( b ) Evaluate 4 +r
dx
-.e x - e - 2
60. Find the area between the graph of y 2 = x 2 / ( 4 - x 2 ) 65. Compare ( a ) lim(cos x ) c s c 2 and ( b ) lim(cos x ) c s c X ,
and the lines x = 2 2 . (Sketch the graph.) 2-o+ x-o-
61. ( a ) Find the area of the region bounded by the x axis, sin ax tan ax
66. Compute lim 7. 67. Compute lim -.
the negativey axis, and the graph o f y = In x. X-o sin bx X-o tan bx
( b ) Find the volume of the solid of revolution generated 68. Suppose R is the region bounded by y = l / & . the
by revolving this region about t h e y axis. positivey axis, and the linex = 1 . Determine the area of
( 6 ) Find the surface area of this solid. R thinking in terms of ( a ) vertical rectangles and ( b )
62. ( a ) Find the area between the graph ofy = x / ( x 2 1)2 + horizontal rectangles. (The two answers should agree,
and the positive x axis. but two different types of improper integrals occur.)
3liscellaneous Exercises
1. Prove that the integral J, d x / ( x ( l n x)p) converges if 5. ( a ) Using the opposite of the take logarithms strategy,
p > 1 and diverges if p 5 1. evaluate
2. What value o f x makes the vertical line through it divide
the following region into pieces of equal area? It is 2-0 [In
lim +]
+
bounded by the curvey = 1/( 1 x 2 ) and the positive x
axis. ( b ) W7hat hypotheses did you use in your argument?
3. T h e gamma function is defined by T ( x ) =
6. Repeat Exercise 5 for lim
Jizdt. (See Sec. 14.3, Exercise 57.) x-0
( a ) Show that T ( x ) is defined (integral converges) for e-l/x2

x > 0. 7. Prove that lim -= 0.


0-0 x
( b ) Show that T ( x ) is undefined (integral diverges) for
8. ( a ) Prove that if k is a positive integer, then
x 5 0. 1im2+o(e-lz2/xk) = 0.
( c ) Using integration by parts, demonstrate that ( b ) Use part ( a ) to show that
qx+ 1 ) = xr(x). for x # o
(6)Using part ( c ) , show that r(n) = ( n - l ) ! for n a f ( x ) = (;lcz for x = 0
positive integer. (Hence the gamma function generalizes
the factorial operation to any positive number.) has continuous higher derivatives of all orders a t x = 0.
4. (j: tU dt near u = - 1 ) Let f ( u ) = J: tU dt, where (This is clearly true for x # 0.)
O<a<x. Jg e t 2 dt Jg e f dt
( a ) Evaluate the integral and compute limU+-l f ( u ) , 9. Compute ( a ) lim ~ and ( b ) lim -,
2-0- Jg et dt X-LO: J; e t 2 dt
using LHospitals Rule.
( 6 ) Compare your result with J: t-I dt.
Exercises I0 to I8 concern the use o f Comparison Theorems in showing convergence or divergence o f Ja+
f ( x ) dx and in establishing bounds on
approximation error when the antiderivative is nonelementary.
10. Suppose f ( x ) and g ( x ) are continuous with
+
0 5 f ( x ) I g(.) on [a, 30). ( b ) Prove that if J, f ( x ) dx diverges. then J, g ( x ) dx
( a ) Prove that if J; g ( x ) dx converges, then J, f ( x ) dx diverges.
converges. [Use a Squeeze Theorem on 1 1 . Use Exercise 10 to show that J: e- dx converges.
limb-+, J 3 ( x , dx.1 (Note that for x 2 1 , l / e x z 5 l / e s . )
476 CHAPTER 14: INDETERMINATE FORMS AND IMPROPER INTEGRALS

+= dx 18. (See Exercise 17.) Use the idea of Exercise 13 to estimate


12. Use Exercise 10 to show that diverges.
J ~ ~ ( l / f i ) e - x * dx / 2 within 0.01. (This limit is actually
1, and ( l / f i ) e - z 2 / 2 is called the "unit normal density
13. Using the Trapezoid Rule with n = 10 in Sec. 11.6,
function." T h e closely related function erf(t) =
Progress Test 1, we estimated Ji e-" dx 0.88186 with
( 2 / f i ) J b e - x 2 / 2 dx is called the "error function." Both
a n error less than 0.000066.
are widely used in probability and statistics.)
( u ) Prove that J:" cX2 dx 5 Jf" e - 2 x dx.
19. M,, the first moment of the region under a probability
( 6 ) Using ( u ) to show that J$"e-"' dx 5 Ji G-" dx + density function, is also called the "expected value" for
J:" e c Z X dx and, computing Jim e-2r dx, conclude that
that function and is denoted p, Compute the expected
J$=e - x 2 dx is within ( - 1/2)( - e ~ ~ )0.0091578 of
value for the unit normal density function.
Ji e-" dx.
20. Compute the expected value (see Exercise 19) for the
(c) Use ( a ) and (6) to give and justify an estimate of
function in Exercise 17.
J;f"e - X 2 dx accurate to within 0.01.
21. T h e standard deviation associated with a probability
14. Use the idea of Exercise 13, but using a n appropriate c
density function is defined to be the square root of
in JE e-" dx and J,'" e-Cx dx, to approximate J;f*e-" dx
J'z(x - p)2f(x) dx. (It is actually a second moment.)
accurate through four decimal places.
Compute the standard deviation for the unit normal
IS. J ; f " d x / d m can be broken into a sum
function in Exercises 18 and 19.
Ji dx/ d m + J,'" dx/ d m T h e second interval 22. Compute the standard deviation for the density function
is bounded above by J,'"dx/x2. What value of c is
in Exercises 17 and 20.
required to ensure an approximation of J:% dx/ d w
accurate within 0.005?(Hznt: Use trial and error, begin-
ning with a n analysis of J:= dx/x2.)
16. How large must 6 be in order that we can replace
Jb+= dx/ d m by Jb+= as a means of approxi-
mating SO+"d x / d w within 0.01?
17. A probability density functionf(x) must be everywhere
defined and nonnegative with J ' z f ( x ) dx = 1. Show
that
(1/2)e-"/2 for x 2O
f(x) = [o for x <0
is a probability density function.

SELF-TEST +
Chapter 15

define the convergence of an infinite sequence of


usly to the conver-

(as n + +m). We
e notion of addition
tion of numbers) to an
la + . . .. The sum will

oted to methods for deter-


te sequences and infinite

egration. As was the

h technique to use.

477
478 CHAPTER 15, CONVERGENCE OF SEQUENCES AND SERIES

Section 1 5 . 1 : Sequences

Objectives: ISTRODKCTION
1. Translate between the general Unending or infinite sequences of numbers such as the following occur
term of a sequence and its throughout mathematics:
elements.
(i) -1, + 1 , -1, + 1 , . . . , (-1) , . . .
2. Determine whether a given
sequence is increasing or
decreasing, bounded or
(iii)-
3 -+-
3 3 -+-+-
3 3 3 . 3
,-+-+ 3 3
. . . +-,. ...
unbounded. ,
10 lo 100 lo 100 1000. lo 100 10
3. Determine, in simple cases,
Since all our work in this and the next chapter involves sequences, we give a
whether a given sequence is
formal definition.
convergent or divergent and, if
convergent, determine its limit.
15.1.1 D e f i n i t i o n A sequence is a function whose domain is the set of
positive integers. Instead of writing the value of the function at a typical integer
n asf(n), we write it as a,. The numbers al, u2, . . . , a,, . . . are called the elements
of the sequence, and the number a, is called a general element of the sequence.

A sequence can be described by giving (1)its first several elements, with the
understanding that the implied pattern is to continue, or (2)a general element
(usually as a n algebraic formula).

Example 1 Determine the first four elements of each of the following sequences.

(- 1)1-1 (-1)O
- -- -
1
Solutions ( a ) a, = ~ -
31 3 3

(b) b, = 1
b 2 = 1 + - = -1 3
2 2
b 3 -- I + - + - 1= - 1 11
2 3 6
b4 -- I + - + -1+ - 1= - 1 5 0 0
2 3 4 2 4
The problem of finding the general element when given the first few ele-
ments can be considerably more complex. For example, suppose you were
presented with the sequence in Example l(a) displayed as
1/3,- 1/9,1/27,- 1/81,.. .
The denominators are increasing powers of three and thus are given by
3,n = 1,2,. . . , whereas the numerators are alternating plus and minus ones
and thus can be given by ( - ly-l, n = 1,2,. . . . Hence a, = ( - l)n-1/3n.
SECTION 15.1: SEQUENCES 479

Usually the form of a general element is not unique. For example, we could
also have given the numerators above as ( - l),+l. Other helpful facts in dealing
with sequences are the following (test these on n = 1, 2, 3):

15.1.2 (a) The positive even integers may be written as 2n, n = 1, 2 , , . , .


( b ) The positive odd integers may be written as 2n - 1, n = 1, 2, . . . .
(c) The positive integers beginning with k +
1 (k a nonnegative integer) may
be written as n + k , n = 1, 2 , . . . .

Example 2 Determine a general element for the sequence 1/2, - 3/4,5/6, - 7/8,
Solution Temporarily ignoring the signs, we note that the numerators consist of succes-
sive odd integers 2n - 1, and the denominators consist of successive even
+ +
integers 2n. The signs alternate 1, - 1, 1, - 1, so
( - l),+l(2n - I )
a, = n=l,2, ... 0
2n

PROGRESS TEST 1

1. Write out the first four elements of each of the 2. Find the general element of each of the following
following sequences: sequences:
(a)- 4/3,5/4,6/5, . . .
( a ) a, = (1 - l/n)n ( b ) a, = 1 + (-l), ( b ) 1, 1/2, 1, 1/4, 1, 1 / 6 , . . .
n even (c) 1/4, 3/16, 5/36, 7/64,. .
2 - 1
( d ) a, = ___
n odd 2

LIMITS OF SEQUENCES
Sequence (iii), given on page 478, when rewritten as
0.3,0.33,0.333,0.3333,. . .
clearly represents a sequence of decimals that provides an increasingly accurate
approximation of the number 1/3.
By going far enough out in the sequence we can guarantee that a, is within
any specified distance of 1/3-that is, limn,+, a, = 1/3.

15.1.3 Definition We say that the sequence a, tends to L as n grows without bound
and write
lim a, = L
n-+m

if given any > 0, there exists a positive integer K such that


F

n > K implies la, - LI < E


We shall also write this as a, + L as n + + or, even more simply, as
cc)

a, + L, and say the sequence is convergent. If limn+, a, fails to exist, then we


say a, is divergent.

This definition closely resembles that of lim2++,f(x) = L in (4.1.2) because


+
by definition a sequence is itself a function defined on the subset of [ l , co)
consisting of the positive integers. Limits of sequences obey most of the laws
480 CHAPTER 15: CONVERGENCE OF SEQUENCES AND SERIES

with which we are now quite familiar. For example, the limit of a sequence,
when it exists, is unique (see Sec. 15.1, Exercise 81). For the record we state:

15.1.4 Limit Theorem for Sequences Suppose lirn a, = A , lirn b, = B.


n-+m n++m

(u) lim (a, k b,) = A iB


n++m

( b ) lim (u,b,) = AB and lim (cu,) = CA (c a constant)


ni+m n-+a

A
(c) lim
n++m
(5)
b,
= (b, # 0 and B # 0)
( d ) lirn
n-+m
= m,k a positive integer. If k is even, then a, 2 0.

( f )Iff is continuous a t A , then lim,++mf(u,) = f(A). Continuous functions


preserve limits of sequences.
The proofs are left as Sec. 15.1, Exercises 82-86.
To take advantage of our earlier work with limits at infinity, we often regard
a sequence as the restriction of a function defined on the entire interval
+
[ l , oo), even assuming, if necessary, that this function is differentiable. In
particular, if limx-++,f(x) = L andf(n) = a,, then limn,+, a, = L.

n2+n+1
Example 3 Evaluate ( u ) lirn -
n-+m 1 -
fi
+

6 and ( b ) lirn
n-fm ,n

Solutions (u) As in Sec. 4.1, we multiply numerator and denominator of a typical element
by l / f i and then evaluate the limit of the resulting expression:

lim
(l/fi)+l
-Of1 - -1
n-+m(l/fi) -1 0 -1
( b ) We shall apply LHospitals Rule to a differentiable function of which the
given sequence is a restriction,f(x) = ( x 2 x + +
I)/ex ( o o / c o ) .
lim x 2 +x + 1 = lim -
2x 1 +
X-+= ex x-+- ex

We apply LHospitals Rule again to get


2 =0
lim -
x-+mex
But then

lim x 2 + x + 1 =o
X++m ex
from which we can conclude that

lim n2 + + 1 = 0.
n++w en

In part ( a ) of Example 3 we used, without mention, the Limit Theorem for


Sequences as well as the fact that limn,+, 1/ = 0 (which follows from the
fact that lim,,,, l / & = 0.)
SECTION 15.1: SEQUENCES 48 1

The following Squeeze Theorem, whose proof is left as a n exercise, will be


valuable in using sequences with known limits to compute limits of new
sequences.

15.1.5 Squeeze Theorem If 6, 2 a, 2 c, for each n, and if limn-;+mb, =


lirn,,,, c, = L, then
an = L
Theorem (15.1.5) is most often applied when b, is the constant sequence, all of
whose elements are zero. In this case b, + 0 trivially, so showing that c, + 0
allows us to conclude that a, + 0.

Example 4 Determine limn,+, a,, where

Solution We first note that a, is a quotient whose numerator is the product of the odd
integers while the denominator consists of a product of multiples of 4. Hence
1.3. 5 - - ( 2 n - 1)
a =
. . . * 4n
4-8-12.

--
- 4
.- .-. . . . (2n - 1)
1 3 5
8 12
*
4n
In this latter form we see that a, is a product of fractions, each of which is less
than 1, so the product should be less than 1. In fact each element is less than its
predecessor. We rewrite the first few elements as follows, hoping to apply the
Squeeze Theorem:

In general,
a _.-._
= 1 3 5 ....
4 8 1 2
.-2n - 1
4n

because the initial factors indicated are each less than one, so their product is
as well. Consequently, for each n,

1
Since lim 0 = lim - = 0, we know that
It-+- n + + m 4n

lim a, = 0
n++m

by the Squeeze Theorem. 0

It is worthwhile to collect some of the more common limits that we shall


encounter in the pages to come. The first six follow from limits determined in
Chaps. 4 and 14.
482 CHAPTER 15: CONVERGENCE OF SEQUENCES AND SERIES

Recall that n! (read "n factorial") is shorthand for


n*(n- l)(n-2)* ... -3-2.1
the product of all positive integers less than or equal to n.

15.1.6 Theorem Special Limits


( a ) If p ( n ) and q(n) are polynomials whose highest degree terms are pknk and
qjni, respectively, then

'(')
lirn -
'

n-fx 4(n)

( b ) If p
= pk/qj

> 0,
I" 1
lim - = 0.
np
n-rim
if k < j
if k = j
if k > j

nP
(c) For all p , lim - = 0.
n-+m en

( d ) If p > 0, lim In n = 0.
n-im n
(e) If r is any real number,

(f)If k is any real number, lirn nkin = 1.


n++m

(9) If ir[ < 1, then lim rn = 0.


n++m

rn
( h ) If r is any real number, lirn - = 0.
n-+x n!

Proox
( u ) The statement follows from (4.1.5), p. 113
( b ) The statement follows from (4.L 4 ) , p. 113
( c ) The statement follows from 114.1.5(b)l, p. 462
( d ) The statement follows from 114.1.5(u)l, p. 462
( e ) The first limit follows from (14.2.2) p. 466 and the second is its reciprocal.
(f)(f)follows from ( d ) since ln(nkIn) = k(ln n)/n + 0. Hence nkin + e0 = 1.
(g) We shall show that limn,+, ( T I n = 0, from which it follows that
limn,+, r n = 0 by [ 15.1.4(e)].Suppose some small E >
0 is given, say E 1. <
By definition (15.1.3), we must show there is an integer K such that n 2 K
implies IrIn <
E . But, taking the natural logarithm of both sides of this last
inequality, we have
lnlrl" < In& or n lnlrl < In&
(Note that the natural logarithm, being an increasing function, preserves
the direction of inequalities. Convince yourself of this!) Dividing through
by the negative quantity lnlrl (recall that Iri <
1 so 1nJrJ 0), we get <

Thus, by choosing n 2 K , where K is an integer greater than (In E)/(ln Irl),


we know that the original inequality Irln E holds, as required.<
( h ) We omit the Squeeze Theorem-based proof.

Applications of these limits appear in Progress Test 2 . Although we have


concentrated on those limits that do exist, we should note that limits of se-
SECTION 15.1: SEQUENCES 483

quences can fail to exht in essentially the same ways that limits at infinity can
fail to exist.

Example 5 Show that the following sequences diverge:

(a) a, = (-l),

Solutions (a) +
This sequence alternates between - 1 (for n odd) and 1 (for n even) and
hence approaches no single number as n + + co.
( 6 ) The even elements approach 0 whereas the odd elements are all 1. Choosing
E <1/2, we See that neither 0 nor 1 can be the limit of this sequence
because no matter how large the integer K , there will be even integers n>K
<
such that both la, - 01 E and la, - 11 E . >
(c) We kndw (using L'Hospital's Rule, for example) that

Hence In(' + + O+ as n + C O OBut


. then

PROGRESS TEST 2

Evaluate the following limits:

1. lim
n-+m
(Lr
n -a
2. lim
n-+m
( - 1)" 2n
n!
3. lim
n-+m
( - l), 3"+2
4"

4.
In n
lim - 5. lim 2n2 - 6n 1 + 6. lim sinna
n-+m n.01 n-+- 3n3 + n + 2 n-+m

BOUNDED AND MONOTONIC SEQUENCES


15.1.7 Definition
( a ) A sequence a, is said to be increasing if anfl 2 a,, for all n.
( b ) A sequenct: a, is said to be decreasing if a,+, 5 a, for all n.
(c) A sequence that is either increasing or decreasing is called monotonic.

We use the modifier "strictly" when strict inequalities hold in the previous
three statemefits.
Example 6 illustrates two standard techniques for determining whether a
sequence is nionotonic.

Exatnple 6 Are the following sequences monotonic?

( a ) an =
1 . 3 . 5 * . . . .(2n-1)
(6) an =
ln(n
+
+33)
n!
Soluticrns (a) Note that for positive term sequences the conditions a,, 2 a, and
a,, 5 a, are equivalent, respectively, to a,,l/a, 2 1 and a,,,/a, 5 1.
484 CHAPTER 15: CONVERGENCE OF SEQUENCES AND SERIES

Now
1.3.5. *(2n-l)[2(n+l)-l]
an+, =
(n + I)!
- 1.3.5. ... e(2n- 1)(2n+ 1)
n!(n + 1)
Thus

- -
an+, - 1* 3* 5* .. + * (2n - 1)(2n + 1) n!
an n!(n + 1) 1.3.5. ... ( 2 n - 1)

- 2n
-- + 1>1 for all n
n+1
Hence a, is a strictly increasing sequence.
(b) An approach similar to ( a ) leads to
a,,
-- - ln(n + 4 ) . n +3
an n +4 ln(n + 3)
but it is not clear how this quotient compares to 1, so we examine the
original sequence a, = [ln(n + 3)]/(n + 3). We shall know whether an is
increasing or decreasing if we can determine the sign of the derivative of
+
f(x) = [In(% 3)]/(x +
3). Now

(x + 3)- + 3)(1)
1
- ln(x
xi-3 - 1 - ln(x + 3)
f (x) = (2 + 3)2 (2 + 3)2
Thusf(x) <
0 if and only if 1 - ln(x 3) 0 or ln(x + 3) + <
1 = In e . >
But since the natural logarithm function is strictly increasing,
> >
ln(x + 3) In e is equivalent to x + 3 e, which is certainly true for all
+
x 2 1. Hencef(x) = [ln(x 3)]/(x + 3) is decreasing on [ l , co),so the +
sequence a, = [ln(n + 3)]/(n +
3) is likewise decreasing. 0

15.1.8 Definition
( a ) A sequence a, is said to be bounded aboue if there exists a number U such that
a, 5 U for all n.
(b) A sequence an is said to be bounded below if there exists a number L such that
L 5 an for all n.
(c) A sequence that is both bounded above and below is called bounded.

Recall, for example, the increasing sequence of rational numbers


1, 1.4, 1.41, 1.414, 1.4142, 1.41421, 1.414213,. . .
which provide, at each stage, an increasingly accurate approximation of the
irrational number a.
This sequence is bounded above by 1.5, for example.
We need to assume the next fundamental theorem to guarantee, in effect,
that any sequence of real numbers that should converge actually does converge
(to a real number).

15.1.9 Theorem If a, is increasing and bounded above, or decreasing and bounded


below, then a, converges.

The proof of (15.1.9) rests on the basic completeness of the real numbers;
the real-number line has no holes. In this respect the set of real numbers is
essentially different from its subset of rational numbers: the above sequence of
rational numbers, although bounded above, does not converge to a rational
number because fi is irrational.
SECTION 15.1:SEQUENCES 485

Although (15.1.9) tells us that certain limits exist, it does not provide a means
for identifying the actual limit.

Example 7 nl
Show that a, = 2 converges.
n"

- ( n + I)! . -12" = (n + l)n! ._


a,+, - n"
Solution
a, ( n + l),+l n! (n +
l)(n +
I), n!

Thus a, is decreasing. Since all the elements of a, are positive, a, is bounded


below (by 0 for example). Thus a, is convergent, by (15.1.9). 0

We shall show in Sec. 15.3 that a, = n ! / n n converges to 0.


When discussing convergence of sequences or using theorems regarding
convergence, it is usually possible to disregard erratic behavior of early elements
in the sequence, because convergence depends only on the later elements-the
a,, for n > j , wherej is a fixed integer. Thus, for example, a, = 10"/2"' is not
<
decreasing, because al a2. However, for n 2, a, a,,.> >
Since each element
is positive, (15.1.9) tells us that a, converges. (See Exercise 62 next page.)

SECTION 1 5 . 1 EXERCISES

In Exercises I to I4 write t h e j r s t four elements of the given sequence.

1. a, =-
2n 1+ 2. a, =-
( - 1y-1
n+3 n+l

3. a, = ne-, 4. a, = n + 3
~

(n - l)!
5 . a,= . . . e ( 2 n - 1) n
1.3.5. 6. a, =
In(n 1) +
n!
7. a, = sin(3x/n) 8. a, =
1.3.5. ... * ( 2 n - 1)
2.4. ... a(2n)
10. a, = (-1)"+1 - 1
9. a, =
2,n!
n even
12. a, =
~~~~~ n odd
1 - (-l),
13. a, = 14. a, = (-1),(1 -
2

In Exercises I5 to 26 give a general element for each of the following sequences. ( T h e form of an answer is not unique.)
15. 2/5, -4/25, 8/125! - 16/625, 16. 1/2, -3/4, 5/8, -7/16, . . .
17. 3/5, 9/7, 27/9, 81/11, . . . 18. 5/7, -7/9, 9/11, -11/13, . . .
19. 6 , 5 , 6 , 5, . . . 20. 3,1,3,1) ...
21. 1, 1/2, 1/6, 1/24, 1/120,. . . 22. 1/2, 5/4, 9/8, 13/16,. . .
23. 2/22, -3/32, 4/42: -5/52, . I 24. 2 . 4 , 4 * 6 , 6 * 8 , 8 * 1 0 ., . .
25. - 1 . 3 , 3 * 5 , -5.7, 7 . 9, . . . 26. 1, 1/4, 1/27, 1/64, 1/3125,. . .

In Exercises 27 to 40 determine whether the given sequence is increasing, decreasing, or not monotonic.

27. a, = -
3, 28. a, =- 2" 3" 1 +
4" + 1

29.
(n
a, =---
+ 1)2 30. a, =
ln(n + 1)
en &
486 CHAPTER 15: CONVERGENCE OF SEQUENCES AND SERIES

2 " ( 1 . 3 * 5 . . . . ~ ( 2 n - 1)) n2
31. a, = 32. an = -
2 . 4 - 6 . . . . *(2n) n!
33. a, =In(), n+l fi+l
34. a, =-
n-1
3n
35. a, =A 36. a, =-
4n +1
n+l

37. a, = ~
&+ 1 38. a, = niln
n

40. a,, = c o s n a

In Exerczses 41 to 50 determine whether the given sequence is bounded above or bounded below.
2" 3n+1
41. a, =
2 * 4 * 6* ... - (2n) 42. an = 3 . 6 * 9 ~ .. . . * (3n)

43. a, =-
2n2 + n 44. a -- 3"
3n + 5 n-1+3n
45. a, = ne-" 48. a, = l n ( L )
n f l
(2n)' (2n)!
47. a, = - 48. a, =
n" 3 . 5 . 7 - . . . * ( 2 n + 1)
49. a, = (2" + 3,)lIn 50. a, = (1 + 2/n)2/"

In Exercises 51 to 78 deterrnzne the limzt, f i t exists, of the gzven sequence Give reasons fofyour conclusions.
3n3 + 2n2 - 1 + 6n + 9
a, = a, = 2n3
31.
1 + 2n - n3 52.
3n2 + 5
( - 1)"5"
54. a, =-
(n - l)!

55. a =-
2" +5 56. a, =
4G-T
" 3" n+6
sin n
57. a, =-- 58. a, =- en
n en - 1

59. a, = (1 + t) 2
60. a, = 6
2 + 3 6
61. a, = t a n 10"
62. a, = -
2D2
63. a, = cos(n + a/2) 64. a, = sin(n~/2)

( ny +) e 66. a, =
v m
65. an= 2n2 +9
2n2 + n - 1
67. a, =
4n4 + 7
(Hint: let m = n + 1)
2n+3
69. a, = ( - 1)"- 3,-1

71. a, = n 7 / ,

74. 1% n
a, = - 0001

6 1.3.5- ... * ( 2 n - 1)
75. a, =-- 76. a, =
en n!
1 , n arctann
77. a, = dt 78. a, =
etz
2n2 5 +
SECTION 15.2: INFINITE SERIES-AN INTRODUCTION 487

79. Show that a, = (1 + 2,)lIn is convergent. 83. Prove [15.1.4(b)]. (Translate the idea of the correspond-
(2n)! ing proof for standard limits of functions [9.2.l(b)].
80. Show that a, = - 2n
2 (n.1
,is convergent. 84. Prove [15.1.4(c)].
85. Prove [15.1.4(e)].
81. Prove that if limn,+, a, = L and limn,+, a, = L, then 86. prove [15.1,4(f)l.
L = L. 87. Prove the Squeeze Theorem.
82. Prove [15.1.4(a)] for sums.

Section 15.2: Infinite Series-An Introduction

Objectives: INTRODUCTION AND DEFINITION OF INFIXITE SERIES


Use the definition of convergence Although we know how to add anyjinite collection of numbers, we have no
and divergence and basic facts definition for the infinite sum
about infinite series, including
geometric series, to determine
convergence or divergence of
certain series and to determine the However, we can certainly form the partial sums as follows:
sum of convergent series.

Each new term seems to cut the distance to 2 in half. In fact it appears that for
each positive integer n,

15.2.1 ,s = 1 +-+
; (+y + (3,
+ . . . + (3 - = 1+ ~
2 - 1
2,
=2 --
1
2n
This can be verified as follows:
1 1
-s
2 n 5, -p
l
2
=[1+(+)+(+) + * * * +
--i[l + (+)+ ($) + 2

2
n+l

=1- (+) n+l

Hence

s, =2 1- [ (+>I =2 - (+>
Arithmetic and algebra brought us this far, but, as was the case with
- in Chap. 2, to complete the analysis we need the
instantaneous rates of change
notion of limit. We now define
488 CHAPTER 15: CONVERGENCE OF SEQUENCES AND SERIES

=2 by [15.1.6(g)]
This gives a meaning for the infinite sum based on the notion of limit of a
sequence. It is consistent with our earlier observation that the partial sums are
tending toward 2.
We shall use this same approach to define infinite sums in general.
It will be convenient to use the shorter sigma notation for sums (introduced
to write Riemann sums in Sec. 6.1). We denote the sum
a1 + + . . +an
a2

by XExl a k . The index letter k serves as a variable, usually referred to as a


dummy variable because it need not appear when the sum is written out. For
example,
4
2 ( 3 k 2 - 1) = (3 l 2 - 1) + (3 2 - 1) + (3 * 32 - 1) + (3 * 4 - 1) = 86
k=l

Here ak = ( 3 k 2 - l ) , but exactly the same sum could also be written as


z g , (3i2 - 1).
F o r j an integer, the sum
aj + aj+1 + aj+z + * * * + 0,

is written as ZEZj a k . Thus we can write the sum s4 appearing earlier as

s4 =
k=O (;y (-g+ (g+
= (+)2 + (f)3 + ($)l
15.2.2 Definition We refer to
x

x a k = a l + a 2 + a 3 + *
k=l
as a n injnite series, or more simply as a series, and refer to the numbers ak as its
terms. Associated with each infinite series is its sequence of partial sums
s1 = a,, s2 = a1 + a 2 , . . . , sn = a1 + a2 + * . + a,, * *

We say that the series converges to S, and write


P

x a k = S or al+a2+a,+ =S
k=l

if limn-+13(sn= S). In this case we often refer to 2 kp=l ak as a convergent series and
say that S is its sum.
If limn++msn does not exist, then we say that Z ;=l ak is a divergent series. We
also refer to
13

C ak = aj + aj+l + aj+z + . . . ( jan integer)


k=j

as an infinite series, with partial sums, convergence, and divergence defined


analogously.

Just as in the introductory example, Z;==, (y2)k,


we have defined the nu-
merical value of a general infinite series, if it exists, to be the limit of its
sequence of partial sums. Thus, by our earlier discussion 2 ;=o is conver- (y2)k,
gent with sum equal to 2 .
SECTION 15.2: INFINITE SERIES-AN INTRODUCTION 489

O n the other hand,

C k = 1 +2 +3+ .

k=l

is divergent, because, by (6.2.3), page 188,

lim sn = lim
n
k = lim
n(n + 1) = +60
n-+m n++m
k=l
n-+m 2
As is evident in the definition, a n infinite series can be written either in sigma
form or in expanded form. Depending upon circumstances, we may or may not
choose to include a general term in the expanded form. Thus we could also
have written the series in (1) as
m

C k = l + 2 + 3 + +k+
k=l

The last three dots (ellipsis) indicate that the terms of the series continue
without end.
Note that we use separate indices, one for a typical kth term of the series and
another for the nth partial sum-the sum as k runs from 1 to n. As usual, it is
not especially important which letters are used.

Example 1 Determine whether the series


- 1
k=l

converges or diverges. If it converges, find its sum.


Solution Note that
1 - 1
-
k=l k=l k(k + 1) k=l

Thus
s =- 1 +-+-+
1.2
1
2.3
1
3.4
... + (n -l 1) + n(n
1
+ 1)
=
(1 - f +) + (+t - +)
u + (3- $)
+ . . . + (--L)+
t
n-1
\np 1
n
p/
1
(---
1
n
\; n+1

The interior terms cancel one another in pairs as indicated, and we are left with
1
sn=l--
n + 1
Since
lim (1
n-+
--)n +1 l = 1,

we conclude that
1

That is, the series converges to 1. 0


490 CHAPTER 15: CONVERGENCE OF SEQUENCES AND SERIES

A series such as that in Example 1 is often referred to as a telescoping series


because of the way its interior terms collapse. (You may wish to compute the
first several partial sums of Example 1 numerically-its convergence to 1
becomes quite convincing.)

30

Example 2 Investigate convergence or divergence of the series ( - l)i+l.


i=l
Solution s1 = (- 1)1+1 = (-1)2 = 1
s2 = ( - l)l+l + ( - 1)2+1= ( - q 2 + ( 4 =o 3
s3 = ( - i ) l + l + (-i)2+1 + (-1)3+l = (-112 + ( - 4 3 + (-114 =1
It is not difficult to see that in general
1 if n is odd
sn = (0 i f n is even
As a result limn,+, sn does not exist, so Z;==, ( - l)i+l diverges. 0

Examples 1 and 2 show that determining convergence or divergence of a


series often amounts to manipulating its partial sums sn into a form that allows
us to recognize their limit as n + + 00.

PROGRESS TEST 1

Determine the convergence or divergence of the given


series. For convergent series, find the sum.
m 1 m

c
kzl (k + 4;(k + 5 ) (Use partial fractions.) 2.
k=l
(1 + (- l)k) (Write out the first six or seven
partial sums. It is not necessary
to give the nth partial sum in
general.)

BASIC FACTS ABOUT SERIES


Theorem 15.2.3 is obtained by combining the Limit Theorem for Sequences
with basic facts about ordinary addition.

m m

15.2.3 Theorem Suppose ak and b, converge and c is any number. Then


k=l k=l

m m
C ak * C b,
30 Ti

(a) C (ak i: bk) converges with C (ak -t bk) =


k=l k=l k=1 k=l
m m oc

(6) 2 cak converges with cak = c ak


k=l k=l k=l

Note that the above arithmetic is valid only for convergent series. However, we
do have:

15.2.4 Theorem
m X

(a) If c #0 and C ak diverges, then 2 cak diverges.


k=l k=l
SECTION 15.2: INFINITE SERIES-AN INTRODUCTION 491

X x. X

( b ) If uk converges and 2 b, diverges, then (ak +- b k ) diverges.


k=l k =1 k=l

If we add or subtract a finite number of terms from a n infinite series, we will


not affect the convergence or divergence of the series, because the convergence
or divergence of its sequence of partial sums is unaffected by the early
elements. If the series converges, this will, of course, change the sum by the sum
of the affected terms. (See Sec. 15.2, Exercise 52.)

15.2.5 Theorem L e t j be a positive integer. Then the series Z;=, ak and uk x;=l+l
either both converge or both diverge. Furthermore, if Zz=l uk = L , then

cz
Example 3 Find the sum of the series 1/2%.
i =3

Solution We know that Z:=.=, 1/2% = 2; thus 1 1/2 1/4 +


Z;==, 1/2%= 2, and so + +
1/2%= 2 - (1 1/2 + +
1/4) = 2 - 7/4 = 1/4.
Converge or diverge? That is the question to be asked of any series. Our first
armament in attacking it is simple enough.
X

15.2.6 kth-Term Test for Divergence If lim


k-+x
ak # 0 , then 2 uk diverges.
k=l

Prooj? We shall show the equivalent statement that if 2z==l


uk is to converge,
then lirn,,,, ak = 0.
X

Suppose, then, that 2 uk = L and k > 1. Thus


k=l

ak = (a1 + + 02 + ak) - (l + 2 -k * * + uk-l)

= lim sk - lim skPl [15.1.4(u)]


k-+x k-+m

=L-L=O
This theorem is sometimes more briefly called the kth-Term Test. It also
applies when limk-+= ak does not exist. Thus, for example, we know that
Z rZl ( - l)k diverges.

Example 4 Determine convergence or divergence of

i
m= 1 cos(*)

Solution lim cos ( m 2 ;


( u ) m-+m ~ ,)= coso = 1, so

diverges
m=l
492 CHAPTER 15: CONVERGENCE OF SEQUENCES AND SERIES

= e, by [15.1.6(e)], so
n-+m n

i + ir
n=l
(1
diverges 0

Beware: The above divergence test causes trouble in two ways:


1. It encourages a confusion between sequences and series. But remember,
even when we know that E;=l ak is convergent, its value is determined not
by a limit of ak7sbut as a limit of the sequence of partial sums sn. Thus part
( b ) , Example 4 tells us not that the series sums to e but that the series is
divergent.
2 . It tempts using the converse-which is not true, as Example 5 will illustrate.
If limk-+%ak = 0, then nothing can be concluded regarding the conver-
gence or divergence of 2rzl a k .

1
Example 5 (Harmonic Series) Show that - diverges.
k=l k

Solution We shall show that even though lirnk-+= ak = limk++%l / k = 0, the sequence of
partial sums grows without bound, so 2CZl l / k is divergent. To simplify the
arithmetic we shall confine our attention to the partial sums sl, s2, s4, s8, . . . .
Since the entire sequence of partial sums is increasing, showing these sums grow
without bound implies that the entire sequence of partial sums does likewise.
But consider the following sequence of inequalities. (We have underlined
those fractions that replace a larger fraction and thus yield the inequality.)

s 2 = l + - >1 - + -1= 2 *1 - 1
2 2 2 2

1
s16 > 2 (Check this!)

>n 1
-
2
Since the partial sums can be made larger than any given number, Z E=l 1/ k
diverges. 0

The above series, 2r=l l / k , is called the harmonic series. It illustrates how
common sense may be misleading when dealing with series. Although successive
partial sums are formed by adding ever smaller additional terms, the sum
nevertheless grows without bound. (215 terms are needed before the sum exceeds
10 = 20/2. How many terms are needed to exceed loo?)
SECTION 15.2: INFINITE SERIES-AN INTRODUCTION 493

PROGRESS TEST 2

Determine the convergence or divergence of the follow-


ing series, If convergent, find the sum.

(see Example 1)
k=3

GEOMETRIC SERIES
15.2.7 Definition For any numbers a and r, a series of the form
m

is called a geometric series.

Thus geometric series are those series whose ratio of each term with its
predecessor is a constant. O u r introductory series 2 zZ0
( 1 / 2 ) k was obviously a
geometric series with a = 1, r = 1/2. The proof of the following theorem is a
direct generalization of the argument that showed Z;=, (1/2)k = 2.
cc.
15.2.8 Theorem Suppose ark is a geometric series.
k=O

m
(a) If Iri < 1, then 2 ark = -
1-r
U

k=O

a.
( b ) If Irl 2 1, then 2 urk diverges.
k=O

Since the behavior of geometric series is so completely determined by the


above theorem, a standard strategy for analyzing a series involving powers of
constants is to manipulate it into a form that either reveals the given series to
be geometric or allows direct comparison with a geometric series.

Example 6 Determine the convergence or divergence of the following series. If convergent


find thg sum.

By (15.2.5), where we start the summation index cannot affect the conver-
gence or divergence of the series. Since cz=2(2/3)n-1 converges (irl =
2/3 < l), and 2$, (4/3)"-l diverges (lrl = 4/3 >
l ) , the given series
diverges, by [ 15.2.4(b)]. Alternatively, we could have observed that since
+
(2%-' 4%-')/3%-l >
1, the general term does not tend to zero, and thus
the series diverges, by (15.2.6).
494 CHAPTER 15: CONVERGENCE OF SEQUENCES AND SERIES

( b ) We first rewrite the general term in order to reveal that the given series is
geometric:
(-l)n3n+1
- (-1)"3n*3
4%-2 4%-2

- 4Q * ( - 3)" = 48
- 4"
(GT
Thus

Now
48
by [15.2.8(a)], with r = -3/4, a = 48
n=O
- -192
- 7
so

---81
- 7
We use an infinite series every time we use a decimal because a decimal zs an
infinite series:
* a1a2a3 ' . . = al - 10-1 + a2. 10-2 + 10-3 +
where uk is a n integer between 0 and 9, inclusive. Furthermore, an infinite
repeating decimal (which thus represents a rational number) is a geometric series.
This fgict, coupled with [15.2.8(a)],can be used to express a repeated decimal as
a quotient of integers in reduced terms.

Example 7 Find the rational fraction in lowest terms for the repeating decimal

*
0.61N212.. . .
Solution 0.6121212.. .
-
- -10
6 f -1,2200 + +
l2
10,000,000 + ...

- -6
- 12

6
=-
12
10 + 1,000 -[ c( 7 3 1
n=O

+-* 12 1 by [15.2.8(a)], with r = -1


- -6
10 1 - l/lOO
1,000 100
- 6 12
--+-----'- 100
10 1,000 99
- -6 +-- 12 - 6 . 9 9 + 12
10 10.99 lo * 99
101 -
- -101
- 606 -
10.99 5.33 165
SECTION 15.2: INFINITE SERIES-AN INTRODUCTION 495

PROGRESS TEST 3

In Probs. 1 t o 3 determine t h e convergence o r diver-


gence of the given series. F i n d t h e s u m of the convergent
series.
X "

4. Find the rational fraction i n lowest terms for the


repeating decimal 0.32424. . . .

SECTION 15.2 EXERCISES

In Exercises I to 31 determine the convergence or divergence of the given serzes. If Convergent, find the sum.

1. 5- 1 " 1 3. 2- 3
k=l k2 - 2k k=l k=2 k2 - 1

8. i;sin(:
k=l
-- 6. cm

(-l)n2 7.
m

i=1
(-l)Zj+I
"
*- =m
i=1
1

m = l 5m - 1 k=l

12

17- 2 3k-1
5k+2
+ 4k 18. kT (- l)k3k m .
I

k=l k=3 k=O

27. 2
n=l
n2In

32. For any series 2 z = l u k and k >


1, sk = sk-l + a k , with 35. Repeat Exercise 32 for S, = ln[2n/(n +l)].
s1 = a,. Hence a given sequence Sn can be regarded as 36. +
Repeat Exercise 32 for Sn = n / ( 3 n 1).
the sequence of partial sums of a series. Compute the 37. Repeat Exercise 32 for Sn = n2.
series Z;=, uk that has Sn = 1 - 1/4" as its sequence of 38. ( u ) Show that Z;=, +
1/(2n n ) converges by showing
partial sums. that its partial sums are bounded above.
n + 1 ( b ) Determine the sum of Z;=, 1/(2n + n).
33. Repeat Exercise 32 for Sn = -
n+2'
34. Repeat Exercise 32 for Sn = ln(n + 2). (Treat k = 1
separately.)

In Exercises 39 to 41 express the given repeating decimal in the f i r m p / q .


39. 0.2171717. . . 40. 0.123123.. . before it finally stops bouncing? O r does it ever stop
41. 0.120120. . . bouncing? Is there a lust bounce? Discuss this with
42. Is the decimal representation of 7~ a geometric series? someone.
43. A ball is dropped from a height of 2 m, and each time it 44. Repeat Exercise 43 for a ball dropped from a height of
strikes the floor it bounces to two-thirds its previous 2 m and which bounces only to one-third its previous
height. What is the total distance traveled by the ball height.
496 CHAPTER 15: CONVERGENCE OF SEQUENCES AND SERIES

45. ( a ) A (real) honey wagon is being drawn by a horse 47. Repeat Exercise 46 if holding requirements are lowered
traveling at 5 mi/h back to the aviary from a market to 40 percent. How much more capital per initial $1000
10mi away. A busy bee, determined to get its honey is created by the governments action?
back, travels back and forth between the wagon (which 48. Repeat Exercise 46 but with hypothesis (c) changed to
has some drippings on it) and its hive at 30 mi/h. As- reflect the fact that only 75 percent of the money loaned
suming that the rest time at either end of the bees trip is at each stage returns to banks as deposits.
negligible, what is the total distance traveled by the bee 49. At what time between 1 and 2 oclock is the minute
as the wagon returns home? hand of a clock exactly on top of the hour hand? [Hint:
( 6 ) Redo the problem assuming that you had never The hour hand moves 1/12 as fast as the minute hand,
heard of infinite series. (Hint: The bee flies for as long as so the hour hand reaches 1 + 1/12 when the minute
the horse walks. Dont be upset; John von Neumann, hand travels from straight up to 1. When the minute
one of the cleverest mathematicians of the twentieth hand reaches 1 + 1/12, the hour hand goes to
century, is said to have solved the problem using series.) (1 + +
1/12) (1/12 * 1/12).]
46. Banks create capital by loaning out a proportion of their 50. (See Exercise 49) When is the next time the minute hand
deposits, a portion of which are then deposited in banks catches up with the hour hand?
again. Then a proportion is loaned out and a portion 5 1. (See Exercises 49 and 50)
redeposited, and so on. The federal government requires (a) How many times strictly between noon and mid-
banks to hold a certain proportion of their deposits. night does the minute hand coincide with the hour
What is the total amount of capital created if ( a ) $1000 hand?
is deposited in a bank, ( b ) the banks are required to ( b ) If you have not solved (a), pretend you are 10 years
hold 50 percent of deposits, and (c) all the money old and try again.
loaned at each stage finds its way back into bank de- 52. Prove [15.2.3(b)]. 53. Prove (15.2.5).
posits?

Section 15.3: Positive-Term Series

Objectives: INTRODUCTION
1. Apply the Comparison and Limit Although we were able to put the partial sums of telescoping and geometric
Comparison Tests, and the Integral, series into a form whose limit was then straightforward to compute, in general
Ratio, and Root Tests to determine this is not so easy to accomplish. We shall now develop various tests designed to
the convergence or divergence of tell us whether or not a given series converges. Determining the actual sums of
series of positive terms. series will be among the topics of Chap. 16.
2. Choose the appropriate test(s)
Unless otherwise noted, we shall assume that the terms ofall series Z ak dealt
to determine convergence or mith in this section are nonnegative. For simplicity we shall refer to such as posi-
divergence of a given series.
tive-term series. Of course, any theory and technique devoted to convergence
will also extend to series that may have a finite number of negative terms, but
whose terms ak are nonnegative for all k 2 j for some positive integer j .
For positive-term series,

sn+l = sn + a,, 2 sn for all n


which implies that the sequence of partial sums is increasing. Then by (15.1.9)
we have the following:

15.3.1 Theorem If the sequence of partial sums of the positive term series Z rZl ak is
bounded above, then Zrm=l
ak converges. Otherwise Z ak diverges.

THE INTEGRAL TEST


The next test links our work with convergence of improper integrals to conver-
gence of series.

15-3-2 The IntegralTest Supposef(x) is nonnegative, continuous, and decreasing


on [l, + w). Then the series Z ; J ( k ) converges or diverges according as the
improper integral J: f ( x ) dw converges or diverges.
SECTION 15.3: POSITIVE-TERM SERIES 497

...
1 1
f(l)
2 3
tf(2) + .* .
n-1 n
t f ( n - 1) = s,-~

Fig. 15.1

Prooj Since the function involved is nonnegative, we can phrase our argu-
ment in terms of the areas in Fig. 15.1, from which, for each n 1, >

(Note that sn-l I sn because f ( n ) 2 0 for all n. Also, each rectangle has
width 1.)
Suppose first that S?"f(x) du converges with, say,

lim
n++m
lnf(x) dx = L

Then the sequence of partial sums sn is bounded above by L + f(l), and hence
2,kmXlf(k)converges, by (15.3.1); see Fig. 15,1(c).
O n the other hand, if S:"f(x) du diverges, then the partial sums sn are
unbounded, by the last inequality in ( l ) , so 2,km=lf(k)diverges, by (15.3.1).

We apply the Integral Test to a positive term series ZF=j uk ( j a fixed positive
integer) by associating with this given series a functionf(x) withf(k) = uk for
all k 2 j .

Example 1 Determine the convergence or divergence of the series A


k=2
k In k

Solution Letf(x) = l/(x In x ) ; thenf(k) = l / ( k In k ) and f is positive, continuous, and


decreasing on [2, co). +
498 CHAPTER 15: CONVERGENCE OF SEQUENCES AND SERIES

x =n
= lim [In u]
n++m X=2

n-+n

= lim [ln(ln n) - ln(1n 2)] =


n-+m
+ 00

Thus x-!-
k=2
k In k
diverges by the Integral Test. 0

Notice that I: Cz2 l / ( k In k ) diverges even though the kth term tends to zero
a s k - +a.

18.3.3 Definition A series of the form Z FZ1 l/kp, p any real number, is called a p
series. The p series for p = 1 is the harmonic series.

By Example 5 in Sec. 15.2, we know that the harmonic series diverges. The
Integral Test will be used to prove the following more general fact.

15.3.4 Theorem The p series I: l/kp converges if p > 1 and diverges if p 5 1.


pro^ Assume p < >
0. Then l/kp = k-p = kQ(q 0), and so limk++" l/kp =
limk++mkQ = + 00. I f p = 0, then l / k P = l/ko = 1. Thus forp 5 0 thep series
diverges by the kth-Term Test.
>
Now assumep 0. Thenf(x) = 1/ x p is continuous, positive, and decreasing
+
on [ 1, w), and Z,"=,f(k) = Zr=, l/kp. By the Integral Test, the convergence
or divergence of I:;Czll / k p depends on ST"
du/xP. But, by (14.3.3), page 472,
the improper integral, hence the series, diverges for p 2 1 and converges for
P>1.

Thus, for example, by (15.3.4),


" 1 " 1 " 1
C T
k=l
.%
n=WO
c,
j=1 J

are divergent p series, and


" 1
,zs CpT
n= 1
"
i=1
1
x p
"
m= 1
1

are convergent p series.


Naturally, the Integral Test will be most useful on series 2 gZl ak where the
function f ( x ) such that f(k) = ak is relatively easy to antidifferentiate.

PROGRESS TEST 1

Determine the convergence or divergence of each of the following series:

4.
k=l
2 cos (k2 + 1)
SECTION 15.3: POSITIVE-TERM SERIES 499

COMPARISON TESTS
Now that we have a store of series whose convergence or divergence is known,
we shall develop tests that allow us to determine the behavior of new series by
comparing them with the known series.

15.3.5 The Comparison Test Suppose C;=, a, and Zz=, b, are positive-term
series.
( a ) If a k 5 bk for all k and if Zr=, b, converges, then Zr=, uk converges.
( b ) If ak 2 b, for all k and if Zp=, b, diverges, then Z;=, a, diverges.
Prooj Let s, and t, denote the partial sums of Z;==, uk and ZCzl b,, respec-
tively. Since we are dealing with positive-term series, s, and t, are increasing
sequences.
( u ) If uk 5 bk for all k , then

S
, = U, + u2 + . . * + U, 5 b, + b, + . . . + b, = t,
for all n. If Zz=, b, converges, then limn,+, t, = L , so the increasing
sequence of partial sums s, for 2ZZl a, is bounded above by L. Hence
C;=, a, converges, by (15.3.1).
( b ) If uk 2 b, for all k , then s, 2 t, for all n. If 2:=, b, diverges, then
+
t, = co,so the sequence of partial sums s, for 2 uk cannot be zzl
bounded above. Hence 2zXla, diverges by (15.3.1). W

To apply the Comparison Test to a series Z = ;, a,, you must compare it with
a series 2 :=, b,, whose convergence or divergence you already know. Such a
series is often referred to as a test series, with the most frequently used test
series being geometric series and p series.
Since we know by (15.2.5) that for a fixed integer], Z uk converges if and
;, uk converges, it suffices to prove the appropriate inequality uk 5 b,
only if Z =
or uk 2 b, only for all k 2~ for a fixed integerj.

Example 2 Test the following series for convergence or divergence:

= *
X

k=l 3k
1

+
Solutions Note that the general terms of both series tend to zero, so the kth-Term Test
does not apply.
( u ) As our test series we take the convergent geometric series zcrl
1/3,. Now
3, + 6> 3,. Thus 1/(3, + <
fi) 1/3,, so the series C,& 1/(3, fi) +
is, term by term, less than the known convergent series 2 1/3, and hence
converges by the Comparison Test.
( b ) If k 2 3. then Ink 2 In 3 >
1 and hence (ink)/* l / f i . But >
Z;=, l / f i is a divergent p-series, and hence Z,& (ink)/* diverges
by the Comparison Test. 0

We should point out that the Integral Test would be difficult to apply in the
above example, especially part (u).
The following elaboration of the Comparison Test is useful because it
replaces the chore of establishing a n inequality with the often simpler task of
evaluating a limit. The comparison idea is still the same except that we focus
attention on the behavior of quotients of respective terms.
500 CHAPTER 15: CONVERGENCE OF SEQUENCES AND SERIES

15.3.6 The Limit Comparison Test Suppose Z r=l a, and Z b, are (strictly)
positive-term series.
( a ) If limk++ma k /bk = c >0,then either both series converge or both diverge.
( b ) If limk++ma, / b k = 0 and b, converges, then Z r x l a k converges.
(c) If limk-+a a k / b , = +
co and XkpZl b, diverges, then ZkpZl a, diverges.

ProoJ [We prove (a). Parts ( b ) and (c) are similar and are left as exercises.] If
>
the sequence ak/bk converges to c 0,then, by definition (15.1.3), for any E 0, >
there is a positive integer K such that k K implies>

This absolute-value inequality is equivalent to the double inequality

k
C - E < - < C S E
bk

or
(2) (c - E)bk < < (c + E)bk

Now if Zg=l 6, converges, then so does Z;==, (c + E)b,, by [15.2.3(b)]. Hence


2 ; = , ak converges, by the Comparison Test applied to the right side of ( 2 ) . O n
the other hand, if X ; = , b, diverges, then so does 2r=l(c - E)b,, and thus
2Zzl ak diverges as well.

The key to using any comparison test is to recognize an appropriate test


series Z r = l 6,. In the case of quotients of polynomials, a good test series is
2 i?=l l / k p , where p is the degree of the denominator minus the degree of the
numerator.

Test c=
k=l
3k5
2k2+k-1
+ k4 - k2 + 5 for convergence or divergence.

Solution We use the convergent p-series Zz=, l / k 3 as the 2zzl b, test series. Then
2kZ+k-l -. -1- 2k5 +
k4 - k3
+
3k5 k4 - k 2 +5 +
k3 - 3k5 k4 - k 2 5 +
But the limit of this quotient, by [15.1.6(a)], is 2 / 3 > 0, so the given series
converges by the Limit Comparison Test. 0

A similar approach works when roots of polynomials are involved, except


that we use relative degrees. For example, to test the series 2 ( c=l p m / k
+ 2)
we would use as our test series 2z=l l / k 2 I 3 .

Arguments parallel to those used in the previous example yield the following
theorem:

15.3.7 Theorem If, for each k , a, is either a quotient of polynomials or roots of


polynomials, then a, converges if the degree or relative degree of the
denominator exceeds that of the numerator by more than one. Otherwise the
series diverges.
SECTION 15.3:POSITIVE-TERM SERIES 501

PROGRESS TEST 2

Test each of the following for convergence or divergence.


m
Ink k+2
1. C 63/2 (Use C iik5 as a limit-test series.)
k=l k=l k=l

arctank
/vi as a limit-test
k=l k=l
series.)

THE RATIO AND ROOT TESTS


Our last two tests for positive-term series are based on comparing a series with a
geometric series.
m
15.3.8 The Ratio Test Suppose ak is a positive term series and lim
k-+m
akfl
ak
=L
k=l

or lim ak+l-- +cc =L


k++x ak
s

(a) If L < 1, then ak converges.


k=l
m

( b ) If L > 1 or L = + co, then 2 ak diverges.


k=l
m

(c) If L = 1, then the test is inconclusive and C ak may either converge or


diverge. k=l

Prooj
( a ) Suppose L <
1 . Choose r with L < 7 < 1. Since 1imk++= ak+l/ak = L , we
can find K such that, for k 2 K ,

That is,

In particular,

K+1 < raK

aK+2 < raK+l < r ( r a K ) = 72aK


< raK+2 < r ( r 2 a K ) = r3aK
and, in general,
aK+j < riaK for e a c h j = 1, 2 , ...
But .XTZ1 riaK = aK Z;==,r j is a convergent geometric series since
Irl = 7 <
1. Thus by the Comparison Test,
m

C
j=1
K+j = If+, f K+2 -k

converges and hence 2 ;=l ak converges.


502 CHAPTER 15: CONVERGENCE OF SEQUENCES AND SERIES

>
( b ) If L 1 or L = + 00, then in either case we can find K such that k 2K
implies
Uk+l
->I
k

that is, aktl >


a k . And since the aks are all positive, the sequence of ahs
cannot tend to zero. Thus Zr=, ak diverges by the kth-Term Test.
(c) Consider Z;=,l / k and ZTZl l / k 2 . For each series ak+Jak + 1 as
k -+ + co,but the first diverges and the second converges. Hence if L = 1,
anything can happen. W

Warning: Be sure you form the ratio in the correct order. Replace k every-
where in ak by k + 1 and then divide by a k .

Example 4 Test the series 2 2k(kk+! 1)


k=l

Solution ak+, -
- - 2k+1[(k + 1) + 11 k!
k (k + I)! 2k(k+ 1)
- +
2k+1(k 2 ) k ! - 2(k + 2 )
- -
+
2k(k l ) k ! ( k 1 ) + (k + 1)2
Thus

and so the series converges. 0

Example 5 Test the series


k=l

Solution ak+l - ( k + 1)l


-- . _k !
k (k + l)! kk
-
-
( k + l ) ( k + l ) k ._
k!
k!(k + 1) kk

-
kk
Then lim,,,, ak+l/ak = e by [15.1.6(e)], so the series diverges. 0

Knowing haw to use a particular test is not as easy as knowing when to use it.
As with techniques of integration, for some examples several different tests can
be applied successfully. However, it can be shown that if ak involves only
quotients of polynomials, roots of polynomials, or logarithms of polynomials,
then the Ratio Test will yield 1 and hence give no information. Thus, for
example, the Ratio Test fails when applied to any of the following:

k=1 ka;
, ,=
i :IZ1 k=l
m
=
n=l
1n(n
d7T-G
As a rule of thumb (as always there are exceptions), the Ratio Test can best
+ 1)

be applied to series involving factorial or factorial-like terms and/or terms of


the form ak.
SECTION 15.3:POSITIVE-TERM SERIES 503

15.3.9 The R o o t Test Suppose zrZla k is a positive-term series and


Iimk++m( u k ) 1 I k = L or 1imk+= = + w = L.
(u) If L <1, then XgZl uk converges.
( b ) If L > +
1, or L = w , then Ec=l uk diverges.
( 6 ) If L = 1, then the test is inconclusive and x;=l a k may either converge or
diverge.

The proof, involving a comparison with geometric series, resembles that of


the Ratio Test and we leave it as Exercises 57 to 59. T h e Root Test is particu-
larly useful if the kth term of the series zc=la k is itself a kth power.

Example 6 Test the series 5(z)"


k=l 3k+2 '

Solution
ukl/k = ~

+ - <1 l ask-++w,
3k+2 3

so the series
*
k=l
(-)3k k+ 2 k converges. 0

It should be clear by now that your ability to use successfully the various
series tests is directly related to your ability to compute limits of sequences.
When working the exercises below, refer whenever necessary to the special limits
developed in Sec. 15.1, especially those in (15.1.6), page 482.

PROGRESS TEST 3

Test each of the following series for convergence or diver-


gence:

2k

k=l

SECTION 15.3 EXERCISES

In Exercises I to 54 test the given series for convergence or divergence.

k=l

" 1
k=2

7. 5 sin(;/2k)
8. 5 tan(-) 9. 5 k4eck5
k=l k=l k=l

k=l

k=l
c,
"
k=l
1

5 +7
m
k! 3k 1
17.
k=l k=l
504 CHAPTER 15: CONVERGENCE OF SEQUENCES AND SERIES

" 1
19. 20.
k=l

k2
21. 22.
k=l

" Ink
23. 24.
k= 1

"
25. k2 sin( 1/k) 26.
k=l

27. 2
k=l
k/2k2 28.

" 1
29. 30.
k=2

31. 32.

33. 34.

35. 36.

37. 38.

39. i;=m !
m=1
( u a constant) 40.
Inn

41. 42.

43. 44.

45. 46.

47. 48.

49. 50.
5-
n=l

" Ink
51. 2 7
k=l 52.

1.3-5. ... ~(2k-3) 2


53.
k=2 -
2 4 6. ... -(2k - 2) (7)
54.

55. Use the Integral Test on Z Ezl l/k4 to determine an upper 56. Repeat Exercise 55 for the first 10 terms of Zc=z l/k3.
bound on the error involved in using the first nine terms 57. Prove [15.3.9(u)].
to approximate the sum. (Hznt: Compare 58. Prove [ 15.3.9(b ) ] .
c-- c-- c -
" 1 101 " 1 59. Prove [15.3.9(c)].

k = l k4 k=l k4 - k=ll k4
and the appropriate integral.)
SECTION 15.4: SERIES WITH ALTERNATING POSITIVE AND NEGATIVE TERMS 505

Series with Alternating


Section 15.4: Positive and Negative Terms

Objeetives: CONVERGENCE OF SERIES WITH NEGATIVE TERMS


1. Determine convergence of series Except for the kth-Term Divergence Test, we have yet to develop tests for
of alternating positive and negative convergence or divergence of series with negative terms. However, given any
terms. series 2 c=l
a k , we can take the absolute value of its terms to get the positive-
term series Z;Zz1 lakl, to which the tests of the last section apply.
2. Determine convergence or
divergence of 2;=1 la,l,
15.4.1 Definition If Z eX1lakl converges, we say that the series Z gZl ak is
3. Approximate to any given absolutely convergent.
accuracy the sum of a convergent
series of alternating positive and
negative terms.

Example 1 Show that the following series are absolutely convergent:


sink
k=l

Solutions 5 1 for all k , we have I(sin k)/k21 5 l / k 2 . But Zg=, l / k 2 is a


( a ) Since lsin kl
convergent p-series. Thus the positive-term series Z PXl I(sin k ) / k 2 /converges
by the Comparison Test, and so the original series 2g=l (sin k ) / k 2 is abso-
lutely convergent.

as k + + co. Hence
5 1 ( - 1 ) yk+ 1 ) 1 = I;,
k=l k=l

converges by the Ratio Test, so the original series is absolutely conver-


gent. 0

15.4.2 Theorem If 2 c=llak 1 converges, then zrX1a k converges. T h a t is, an absolutely


convergent series is convergent.
Prooj Assume that Z;==, lakl converges. Now
- /akI 5 ak implies 0 5 ]ak1 + ak
But ak 5 lakl in turn implies
(1) lakl f ak 5 lakl + iakl = 21ak/

Since 22==,lakl converges, we know that 2;==,2 1 a k ( converges. Hence, by (1)


and the Comparison Test, the series of nonnegative terms Zc=l ( i a k l ak) +
converges. Finally, since a k = ( a k +
lakl) - l a k l , and since BeZl (ak + l a k / ) -
iakl is the difference of convergent series, and hence converges by [15.2.3(a)], we
can conclude that B EZ1 ak converges. W

By (15.4.2), our work in Example 1 showed that the two given series are
convergent.
506 CHAPTER 15: CONVERGENCE OF SEQUENCES AND SERIES

15.4.3 Definition A series whose successive terms alternate in sign is called a n


alternating series. Such a series is of the form X;=l ( - 1 1 ~ a k or X;=l (- l)k+lak
where ak > 0 for all k. (In the first instance the initial term is negative and in
the second instance, positive.)

The following test is our main tool in dealing with convergence of alternat-
ing series. It expresses the notion that a certain type of alternating series may
converge because its positive and negative terms partly cancel one another
enough so that the sequence of partial sums converges.

1 5.4.4 Alternating-Series Test Suppose that for an alternating series


E;=l ( - l)k+lak both
lim ak = 0
k++m
and ak > ak+l for all k

Then X;=l ( - l)k+lak converges.


The essence of the justification is to be found in Fig. 15.2, where some
partial sums are plotted. At each stage as n increases, a term that is smaller in
absolute value than the previous term is added or subtracted from the previous
partial sum, so the partial sums jump back and forth, taking shorter jumps each
+
time. The length of the jumps approaches 0 as n + 00. As a result, all the
odd partial sums are on one side of L , converging to L , and the even sums are
on the opposite side, likewise converging to L. We leave the algebraic details to
more advanced courses.

The Alternating Series Test likewise applies to alternating series whose first
term is negative as well as to series for which its hypotheses hold for all k 2 j for
a fixed integerj.

co (_l)k+l
Example 2 Show that -converges.
k=l
k

Solution The hypotheses of the Alternating-Series Test obviously hold with ak = l / k , so


Ec=l ( - I)k+l/k converges.

Notice that

which is the harmonic series and is divergent. Thus Example 2 provides an


example of a series that converges but is not absolutely convergent, showing
that the converse of (15.4.2) is false. The situation in Example 2 occurs often
enough to merit a definition.
SECTION 15 4 SERIES WITH ALTERNATING POSITIVE AND NEGATIVE TERMS 507

15.4.5 Definition If Z;=,ak converges but X;=l lak I diverges, we say that XZxl ak
is conditionally convergent.
Thus

k=l

is conditionally convergent.

PROGRESS TEST 1

For each of the following, determine whether the given


series is absolutely convergent, conditionally convergent,
or divergent.

3- 5
m= 1 ( - l ) mml n m (See Sec. 15.3, Progress Test 1,
Prob. 3, page 498.)

2. (Hint: Use Limit Comparison


k=l Test for absolute convergence,
with b, = l / k l g i 1 8 . )

APPROSIMATISG ALTERYATIYG SERIES


WITH PARTIAL SCAMS
If Z;=,( - l)k+lak satisfies the hypotheses of (15.4.4)-and thus converges to,
bay, L-then, as noted earlier, the odd partial sums decrease toward L and the
even partial sums increase toward L. Thus for n even, s, < <
L s ~ + and
~ , for n
< <
odd, sn+l L sn. In either case,

lL - s,/ < I%+1 - % I = I(-l)n+2"n+ll


= %+l
This provides us with the following useful error estimate.

15.4.8 First-Term-Neglected Estimate for Alternating S e r i e s Suppose


that X r z l ( - l)k+lak = L with limk++=ak = 0 and ak >
for all k. Then

lL - cn

k=l
( - l)k+lUk I I an+,

That is, the sum of the first n terms is within a n f l (the first term neglected) of
the actual sum of the series.

Example 3 How many terms of Z;=,( - l)"l/k! should be added to obtain an approxi-
mation of the sum accurate through three decimal places? (The error must be
less than 0.0005.)
Solution Since

as k -+ f w , we know by the Ratio Test that the given series is absolutely


convergent.
508 CHAPTER 15: CONVERGENCE OF SEQUENCES AND SERIES

By (15.4.6), the absolute value of the error in using the first n terms is
bounded by a,,, = l/(n +
l)!. Hence we need n large enough so that

(n + I ) ! < 0.0005
For n = 5,

+ l)! - 720 z0.001389 > 0.0005


1
(a
For n = 6,
1
(n + l)!
-
5040
=: 0.000198 0.0005 <
Hence the first six terms of the series will yield sufficient accuracy. 0

PROGRESS TEST 2

1. How many terms of the series E;=, ( - l)k+1/42kf1


are required to ensure a n approximation of the sum
of the series within

A GESERAL PROCEDURE FOR CONVERGENCE TESTING


When confronted with a series 2 =;, ak whose convergence or divergence is not
obvious by inspection, the following three-step procedure may prove useful in
organizing your approach to the problem.
1. Try the kth-Term Divergence Test. If limk-+= ak = 0, then:
2 . Apply an appropriate positive term series test to Zr=, lakl. If ZF=, lakl
converges, you have finished since Zr=, ak converges absolutely. If
zg=, lakj diverges and if Zgzl ak is an alternating series, then:
3. Apply the Alternating-Series Test to determine whether Z kp=, ak is condi-
tionally convergent.
Series exist (but not in this text) that do not yield to any of our tests. For
example, if Zr=, lakl diverges but 2r=lak is not alternating (but contains
negative terms ak for k > j for any fixedj), then step 3 does not apply. Such
cases will yield to approaches developed in more advanced courses.
A collection of positive-term series to test for convergence or divergence in
the absence of context is provided in Miscellaneous Exercises 1 to 34.

SECTION 15.4 EXERCHSES

In Exercises I to I I , ( a ) show that the given alternating series is convergent, and ( b ) determine the error involved in using t h e j r s t j v e terms as
an approximation to the series.

6* 5
kzl
( - l)k+lk
3k
SECTION 15 4 SERIES WITH ALTERNATING POSITIVE AND NEGATIVE TERMS 509

In Exercises I2 to 39 determine whether the given series converges absohteb, converges conditionul&, or diverges.

12.
5%
k=l
13. cx

k=l
( - l/n)3 (Be careful!)
=

k=l
(-l)k

15.
m
-s(-1P- 2k +1

(-l)k
18.
k=2

21.

24.
n=l
27. 28. 5T
n=l

30. 31.
+ (- l)+larctan n
L1
n=l n

33.

36.

39. 5
k=1
(-l)k[l - cos(+)]

40. By rearranging terms we can make the sum of a condi- 46. Suppose that S = 1 - 1/2 +
1/3 - 1/4 + 1/5 -
tionally convergent series any number we wish. By fol- 1/6 + 1/7 - . . . and therefore (1/2)S = 1/2
lowing the given outline, show that E;=l ( - l)+l/k can - 1/4 + 1/6 - 1/8 +
- . .. Then S + (1/2)S = 1 +
be rearranged to yield 2 as its sum. 1/3 - 1/2 + 1/5 + 1/7 - 1/4 + = S (with its
( u ) Separate the given series into two (divergent) series 0 terms rearranged). But S + (1/2)S = S implies S = 0,
and E , one consisting of the odd terms and the other of whereas S = (1 - 1/2) +
(1/3 - 1/4) + (1/5 - 1/6)
the even terms. >
+ . . 0. Where is the flaw that leads to the contra-
(b) Choose just enough terms from 0 so that the sum diction?
exceeds 2.
(c) Add just enough terms from & to bring the sum
under 2.
(d) Add just enough terms from 0 to exceed 2 again.
( e ) Show that a series constructed in this way converges
to 2.
41. Show by the procedure given in Exercise 40 that
Bzz1 ( - 1)/3k can be rearranged to yield a sum of
- 1.8.
42. If B EZ1 ak is absolutely convergent and we form another
series, each of whose partial sums 3, is a rearrangement
-
of the original partial sum S,, does limn,+, S, =
Ez=l a,? State your result as a theorem.
43. Give an example of a convergent series Z ak such that
Z=
;, 6 >
diverges ( a k 0 for all k ) .
44. Give an example of a divergent series Z zZ1
ak such that
B zzl
uk4 converges.
45. (Note Exercises 43 and 44.) Prove that if Z z = l ak con-
verges absolutely, then for any p >
1, Bz=,akP con-
verges.
510 CHAPTER 15: CONVERGENCE OF SEQUENCES AND SERIES

CHAPTER EXERCISES

Review Exercises
In Exercises 1 to 8 determine al , a 2 , a 3 , a 4 , for each of the gzven sequences.

1. a, = n sin(a/n)

2n + 1
3. a, =-
4n - 3

5. a, = ~
( - 1)%!
2n+1 6. a, =
Ez
-

8. an = 3 + (- 1),3

In Exerczses 9 to I 8 determzne a general element a, f o r the given sequence.


9. 1. 1/2. 1. 1/4. 1, 1/6, . . . 15. -1/3, 1/27, -1/243, 1/2187, . . .
10. 2 / 5 . 5/2, 4/7, 7/4, . . . 16. 1/2, 1 / ( 2 f i ) , 1/(3$), 1/(4fi), . . .
11. 1, -1/4, 1/16, - 1 / 6 4 , . . . 17. 1/3, 1/3, 3/11, 1/5, 5 / 3 1 , 3/35, . . . (replace canceled
12. 1, 1 - 1/2, 1 - 1/2 +
1/4, 1 - 1/2 + 1/4 - 1/8,. . factors)
13. 3 * 7, -5 * 9, 7 * 11, -9 * 1 3 , . . . 18. 1, 1/4, 1/27, 1 / 2 5 6 , . . .

In Exercises 19 to 24 determine whether the given sequence is monotonic or bounded in a 9 1 way.


4n
19. a, = n 2 e r n , n = 2 , 3, ... 20. a, = (-11,- 21. a, =-
n2+ 1
22. a , = yInn n= 3 , 4 , 5
n
,... 23. a, = 11 - n 2 ) 24. a, = (2" + 3,)lIn
I n Exercises 25 to 62 determine the limit, ;f it exists, of the given sequence.

25. a,
2n + 1
=- 26. a, =- In n 27. a, = 3n2 +n+1
4n - 2 .\"/;2+j n+2

2%. a, =
n2 + 2n + 1 29. a, = 2n2 2n + +4 30. a, = ( - I), - 7,
1on+2
en 3h3 + n2 +1
31. a,=
rn 32. a, = ( n + 3)", 33. a, =
q 4 m -
f/G 3n 1 +
34. a, = (-1)"(~/4)"+~

38. a, = n even
37. a, = (1 - 5/n)" (Inn)/n n odd
n even In n
40. a, = (1'2 42. a, =-
n / d m n odd en
(In n)3
43. a, =-
n

46. a, = n sin(l/n) 47. a, = ~


( - l),3,
n!
48. a, = In
(
n2+n-1
n+3 1
49. a, = ( n + 50. a, =-
sin n 51. a, = (l/n)"*
n

52. a, = sin(n) 54. a, = (en + n)2/n


55. a, = (1 +&) nz
57. a, = 6-
SECTION 15.4: SERIES WITH ALTERNATING POSITIVE AND NEGATIVE TERMS 511

n2+n+1 ( - 1)"4"
58. a, = 59. a =- 60. a, = ( - 1 ) n G
en (1.1)%!

61. a, =
3n2 + n - 6
62. a, =
vn2+ 2n + 1
a $3n3 + n2 + 1
For those Ofthe following series in Exercises 63 to 77 that converge, determine the sum.

" 5 " 4 " 5


&Ei 64. - &i
k=l k=O k=l

2k + 5k-1
k=l

69.
x

x ( - l ) k T
4ktl
70. 2-
k=3
1
k2 - k
k=l

+ 2k
5
m
(-l)k4k
72. 5k
k=O

75. 2 (+)
n=l
[sin - sin (&)I 76. 5
n=l
[arctan n - arctan ( n + l)]
k=l
k + l

(See Miscellaneous Exercises 1 to 34 for practice in determining convergence or divergence of positive-term series.)

In Exercises 78 to 93 determine whether the given series is absolutely convergent, conditionally convergent, or divergent.

(- l ) k * 3 * 6 * 9 * . .. * (3k)
5
k=l .
4 8 1 2 . . . . (4k) k=l

81. " sin(-) r k + 3


k=l 4 2k +
84. 5
k=l
( - l ) , sin k
k2

5
k=l
(- l)k+12k
3kk

93. 2
k=l
( - l ) k + l Ink
k2 + 1

In Exercises 94 to 9 7 j i n d the series ZkTzl ak that has as its sequence of partial sums s, the given sequence.

94. s, n+2
=- 95. s, =- 2n +1 96. s, = In(n + 1) 97. s, = A
n+3 3n +2 6n + 36

Miscellaneous Exercises
In Exercises I to 34 determine whether the given series converges or diverges, citing the appropriate test(s) and comparison test series where appropriate.

1 + 2k " V r n
v5TTT
k=l
2.
k=l k2+ k + 1 4. k=l2 + k2 + 1
k3

+ l)! X 1

5. 5
k=l
2"+l(k
(k + 1)ktl
k=l +
512 CHAPTER 15: CONVERGENCE OF SEQUENCES AND SERIES

5 (k + 5)! 11.
m
1
9.
k=l
cos( 1/k)
lo* k=l 2(k + 2)! k=l ln(k + l)ln(k + 2)
log, k
12.
k=l k=l

15. 16. 2
k=l
4 k ( l * 3 * 5 ** *
(k
.

+ I)!
( 2 k - 1))
17. 2-
k=2
1
ek Ink

2k ln(k + 1)
18. 2
k=l
k . 3k

In n
21.

m
Inn
24. 26. kP1l2 arctan k
n= 1 k=l

27.

30.

33.

k=l

35. Show that


k2
$k6

1
+ 2k + 1
+ k3 + 1
c converges ifp
~
34.
k=l
2k(2k 5 )
jk(k + 1)

2 1 and diverges if
+

44. (Generalized Ratio Test) Suppose uk 2 0 for all k 2 1


k(ln k)P
p 1. < k=2 and that there is a number R <
1 and an integer K
Ink such that k 2 K implies ak+Jak 2 R. (Note that
36. Show that - converges if
kP
p > 1 and diverges if limk++- aktl/ak need not exist.) Show that 2 & ak con-
p 5 1. k=1 verges.

37. Show that if


k=l
c
rn

ak converges, then
m

k=l
l/ak diverges.
45. Apply the result of Exercise 44 to prove that
1 + 1/2 + 1/3 + 1/6 + 1/9 + 1/18 + 1/27 + . . .
m
converges. (Note that the Ratio Test fails.)
38. Show that if lim an exists, then
k-+m
2
(ak - ak-l) con- 46. Let S, = Z%=, l / k . Using an upper Riemann sum for
f ( x ) = l/x based on the integer partition of [ l , +co)
verges. k=2
39. T h e sequence a, is said to be Cauchy if for every E 0, > into unit width rectangles, show that S, >
ln(n + 1).
there is a positive integer K such that n,m 2 K implies (Draw a diagram.)
<
la, - am/ E . Prove that a, = l/n is Cauchy. (Choose K 47. (See Exercise 46.) By stacking the rounded-base trian-
so that n,m 2 K implies l / n ~ / 2and l/m < ~/2.) < gular errors along the vertical axis inside a rectangle of
40. Prove that if an converges, then a, is Cauchy. (See Exer- area 1 (as we did in Chapter 6), show that the increasing
cise 39.) (The converse, which is true, is considerably sequence a, = S, - ln(n + 1) is bounded above and
harder to prove, and reflects the fact mentioned earlier hence converges. (It is a transcendental number, ap-
in this chapter that every sequence that should converge proximately equal to 0.577, and is denoted by y. It is
does converge; the real-number line is complete and called the Euler constant.)
has no holes.) 48. Prove that if limn,+, a, = A and limn,+, b, = B , then
41. An infinite product is an expression n;=l (1 + a k ) = limn,+, (a,b,) = AB. (Hint: See the earlier arguments
+
(1 al)(l + +
a2)(l a,) . . . whose convergence is de-
for standard limits.)
fined as the convergence of its sequence of partial products
p, = (1 aJl + az)(l +
a3) . . . (1 + +
a,). If Uk 2 0
+
for all k 2 1, prove that p , 2 1 al + a2 + . . . + a,.
42. (See Exercise 41.) For ak 2 0, prove that the convergence
+
of n;=, (1 ak) implies the convergence of Z;=, ak.
43. (See Exercise 41.) Prove the converse of Exercise 42.
(Consider lnp,.)
SECTION 15.4: SERIES WITH ALTERNATING POSITIVE AND NEGATIVE TERMS 513

SELF-TEST +
Chapter 16 %

515
516 CHAPTER 16: POWER SERIES AND TAYLOR SERIES

Section 16.1 : Taylor Polynomials

Objectives: TAYLOR POLYNOMIALS


1. Construct Taylor Polynomial Because calculations involving polynomial functions are relatively easy, poly-
approximations for a given nomials are often used to approximate other functions. To evaluate a polyno-
function. mial we need only add and multiply numbers. O n the other hand, how would
2. Determine upper bounds on the we evaluate ex at x = 0.1? Use a calculator? A table? The values available from
error of such approximations. such sources are only approximations; more important, such approximations
are usually achieved by using polynomials to approximate the original func-
tions.
In (3.3.5) we used the differential df = f ( x ) dx to approximate the function
f ( x ) . There we approximatedf(x) near ( a , f ( a ) ) by the linear function
f ( a ) +f(a)(x - a) ~ f ( x ) (Recall Fig. 3.5, page 98)
Butf(a) + f ( a ) ( x - a ) is merely a first-degree polynomial in x whose graph is
the straight line tangent to the graph off(x) at ( a , f ( a ) ) .For x near a , f ( x ) is near
the tangent line.
We can improve on this approximation by using a second-degreepolynomial
whose graph is not only tangent to that off a t ( a , f ( a ) ) but also has the same
concavity at that point; that is, it bends)) the same way that f does a t x = a.
One such polynomial is

Pz(x) = f ( a ) + f ( a ) ( x - a) + f-7
Yj-a ) .( - ,)Z

It passes through the point ( a , f ( a ) ) because


P,(a) = f ( a ) + 0+ 0=f(a)
Its first derivative

evaluated at x = a is
q a ) = f ( a ) +f(a)(a - a) = f ( a )
which is the same as that of$ Finally, its concavity, given by
q x ) =f(a)

agrees with that off at x = a.


A third-degree polynomial would probably be a n even better approximation
if its derivatives equaled those off at x = a. Then the rate of change of its
second derivative would be the same as that of J: One such polynomial is

Each of the following derivatives evaluated at x = a equals the respective


derivative offat x = a since all but the appropriate term are zero when x = a :

P i ( x ) = 0 +f ( a ) + f0. 2 * ( x - a ) + f(a)
- 3( - a ) 2
2 3.2
+
Pi(%)= 0 0 + f ( a ) +f(a)(x - a )
Pi(x) = 0 + 0 + 0 +f(a)
The same reasoning carried n steps would give us the polynomial
SECTION 16.1: TAYLOR POLYNOMIALS 517

Note that the nth derivative of P%(x)is given by

-* n ! =f'"'(a)
P,'"'(x) = f'"'(a) (Check this!)
n!

16.1.1 Definition We say that

is the nth Taylor polynomialfor f at a.

(The English mathematician Brook Taylor worked with many of the ideas in
this chapter during the early 1700s in the generation after Newton.)

Example 1 Compute P4(x) at a = 1 forf(x) = In x .


Solution We must first compute the first four derivatives and evaluate each a t a = 1.
1
f'(X) = - f'(1) = 1
X

-1
f"(X) =- f"(1) = - 1
X2

z
f"'(X) = 7 f"'(1) =2

Hence

P4(x) = In 1 + - (1x - 1) + -(-1)


(x - 1 ) 2 + - 2( x - +-
(-6)(x - 1)4
l! 2! 3! 4!

APPROXIMATIONS OF S I S X USING
TAYLOR POLYXOMIALS
We shall examine Taylor polynomials for sin x at a = 0 algebraically, graphi-
cally, and numerically. In so doing, we shall see how a typical trigonometric
table can be generated using Taylor polynomials. But first we need derivatives
at 0:
f(0)= sin0 = 0
f ' ( 0 ) = coso = 1
f " ( 0 ) = -sin 0 = 0
f " ' ( 0 ) = -cos 0 = - 1
~ ( 4= ) sin o = 0
518 CHAPTER 16: POWER SERIES AND TAYLOR SERIES

Obviously, this cycle of four derivatives repeats indefinitely, so in particular,


every even derivative off(x) is 0. Thus, for example:

P,(x) =f(O) + f ' ( O ) ( x - 0) + f- ("(0)


x
2! - 0)2 + f-
"'(0).(
3!
- 013

x3 x5
= x - - + -
3! 5!
and
x,
Pg(x)= x - - + - - - + -
x3 x7 x9
3! 5! 7! 9!
We dealt with Pl(x) = x before, when examining lim,,o(sin x ) / x . We noted then
that for small x , sin x =: x , making (sin x ) / x z 1. Geometrically this means that
the graph of the first Taylor polynomial is close to that off(x) = sin x for x near
Fig. 1 6 . 1 0; it is the tangent line at (0,O); see Fig. 16.1.
In Figs. 16.2 and 16.3 we give sketches of

respectively. Between - 77/2 and + 77/2 we exaggerated the difference between


the approximation andf(x) = sin x so that the graphs could be visibly distin-
guished. Even so, the increasing accuracy of P'(x) as n increases is quite evident.
Further information regarding accuracy is provided by a close inspection of
Table 16.1, where all values are rounded off in the sixth decimal place. The R,
column on the right gives P5(x) - sin x , that is, the error involved in using P5(x)
as an approximation of sin x .
For x close to 0-say, x in [0, 0.11-all the approximations are quite good.
However, as reflected in the graphs, for larger x , higher degree polynomials are

I
Fig. 16.2 4'
t

-x / 2
I/

Fig. 16.3
SECTION 16.1: TAYLOR POLYNOMIALS 519

PI = x p3 ps p7 p9 sin x Rj

0.000000 0.000000 0.000000 0.000000 0.000000 0.000000 0.000000


0.100000 0.099833 0.099833 0.099833 0.099833 0.099833 0.000000
0.200000 0.198667 0.198670 0.198670 0,198670 0.198670 0.000000
0.400000 0.389333 0.389418 0.389418 0.389418 0.389418 0.000000
0.500000 0.479167 0.479427 0.479426 0.479426 0.479426 -0.000001
0.785398 z 17 0.704652 0.707 143 0.707 107 0.707107 0.707 107 -0.000036
0.900000 0.778500 0.783421 0.783326 0.783326 0.783327 - 0.000094
1.oooooo 0.833333 0.841667 0.841468 0.841471 0.841471 -0.000196
1.300000 0.933833 0.964774 0.963529 0.963559 0.963558 - 0.0012 16
1.400000 0.942667 0.987485 0.985393 0.985450 0.985450 -0.002035
1.500000 0.937500 1.000781 0.997391 0.997497 0.997495 -0.003286
1.570796 z 0.924832 1.004525 0.999843 1.000004 1.000000 -0.004525
2.000000 0.666667 0.933333 -0.907937 0.909347 0.909297 -0.024036
3.000000 - 1.500000 0.525000 0.09 1071 0.145313 0.141 120 -0.383880
4.000000 -6.666667 1.866667 - 1.388413 -0.661728 -0.756802 - 2.623469
6.000000 - 30.000000 34.800000 -20.742857 7.028571 -0.279415 -35.079416

Table 16.1

required in order to give accurate approximations. For x as large as 0.4 (about


23 degrees) each polynomial of degree 5 or more is accurate through six decimal
places, while P,(x) yields this accuracy for x as large as m/4. The best approxi-
mation shown is that of P9(x), which gives five-place accuracy for x as large as
m/2 and is within 0.1 even at x = 4. To get this close at x = 6 would require
P d x )*
In a standard trigonometric table, values of sine (as well as the other
functions) are only needed for x in [O,m/4], so P&x) can be used to give a
four-place table for the sine function.

PROGRESS TEST 1

1. Determine the eighth Taylor polynomial at a = 0 ( b ) Repeat for P2(x).


forf(x) = cos x . (c) Repeat for P4(x).
2. ( a ) Graph PI(.) (a constant function) and cosx on 3. Compute P5(x) a t a = 0 for f ( x ) = e".
the same coordinate axes.

TAYLOR'S REMAINDER FORiMULA


How accurate are the approximations provided by Taylor polynomials? Before
we answer this question we establish some convenient notation.

16.1.2 Definition: If P,(x) is the nth Taylor polynomial approximation off(x) at a,


we denote by R,(x) the difference betweenf(x) and P'(x) and call it the nth
+
remainder term. Thus f ( x ) = Pn(x) Rn(x).

In Table 16.1 the right-hand column consists of the fifth remainder term for
the sine function's Taylor polynomials at a = 0. The following important
theorem describes R,(x) in terms of the ( n + 1)st derivative offand will thus
enable us to give an upper bound for the error associated with a Taylor
polynomial approximation.

16.1.3 Taylor's Formula: Suppose f is a function whose ( n + 1)st derivative


exists on an open interval I containing a. Then for any number x # a in I , there
exists a number c strictly between a and x such that
520 CHAPTER 16: POWER SERIES AND TAYLOR SERIES

or equivalently

We adopt the common convention thatfob(,) = f ( a ) .This will simplify matters


when summation notation is used.
Notice that if we take x = b, with a <
b, and n = 0 in the above theorem,
then its conclusion states that there is a c strictly between a and b such that

f(b) =f(a) + f+b '(4 - a)


This equation can be rewritten as

Hence in this particular case the conclusion of (16.1.3) is the conclusion of the
Mean-Value Theorem (4.4.4). For this reason (16.1.3) is also referred to as the
Generalized Mean-Value Theorem. As with the original Mean-Value Theo-
rem, its proof is a n application of Rolle's Theorem to a specially designed
function.
Proof of(I6.1.3): We assume that x is a fixed number with a x; arguments for <
a >
x are similar. For each t in [a, x], define a function of t as follows:

f ' Y t ) (x -
F ( t ) =f ( x ) -f ( t ) - f ' ( t ) ( x - t ) - - t)2 - ...
2!

Now F ( a ) = f ( x ) - Pn(x) - R,(x) = 0 and F(x) = 0. Furthermore, F ( t ) ,being


the product and difference of functions differentiable on (a, x), is itself a
differentiable function of t on (a, x). Hence, Rolle's Theorem applies to guar-
antee that there is a c in (a, x) such that F'(c) = 0. But thus we need to compute
F'(t). We shall use the Product Rule on all but the first two terms (remember, F
is a function of t and x is fixed):
+ (x - t )f " ( t ) ]
F ' ( t ) = 0 - f ' ( t ) - [f ' ( t ) ( - 1)

- [?
f
2(x - t ) ( - 1) + (x - t ) 2 -
f"'(t)l
2!
- [f$ 3(x - t)2( - 1) + (x - t ) 3 -
1
f ' 43!
'(t)

n(x - ty-y-1) + (x - ty- n!


SECTION 16.1: TAYLOR POLYNOMIALS 52 1

Notice that thef'(t) terms canceled and that the second term in each bracketed
pair canceled out the first term in the next pair.
By Rolle's Theorem then, there is a c in (a, x) such that

Dividing through by ( x - cy (since x # c), we conclude that

f'"+1'(C)(X - a)"+l --
-
R&) =
(n + l)n! (n + l)!
An immediate consequence of Taylor's Formula is based on the simple fact
+
that if the approximation error equals [f("+l'(c)/(n l)!](x - a)n+l for some c
strictly between a and x, then its absolute value can be no larger than
[ M / ( n + l)!] Ix - @+l, where M is an upper bound on the possible numbers
I f'"+l'(c)i.
16.1.4 Error Bound for Taylor Approximations: If the hypotheses of
(16.1.3) hold and if If'"''(t)I 5 M on [a, x ] (or [x, a] if x a), then <

For n = 1, (16.1.4) reduces to an upper bound for the differential approxi-


mation of Chap. 3. (See Chap. 3 Miscellaneous Exercises 1 to 7.) For n = 1,
the upper bound for lR1(x)linvolves the magnitude of the second derivative of
f near a, which should make sense by the following argument. Pl(x) is the
tangent line approximation to the graph at ( a , f ( a ) ) . The error of this approxi-
mation depends on the amount of "bend" in the graph near ( a , f ( a ) ) ,which in
turn depends upon the rate at which the slopes of the tangent lines change near
( a , f ( a ) ) . But these slopes are given byf'(x) near ( a , f ( a ) )and, of course, the rate
at which f ' ( x ) changes is given by f " ( x ) near ( a , f ( a ) ) .
We now revisit the approximation of $%? originally given in Sec. 3.3,
Example 4 (accurate to three decimal places). Note that "n-place accuracy"
means an error less than 5 x

Example 2 Estimate fh?


through six decimal places.
Solution Letf(x) = x1l5 and compute Taylor polynomials forf(x) at a = 32. We shall
attempt a third Taylor approximation. Now

5 5
A.
1
f " ' ( ~ ) = 5.
5
Ax-14/5
5

We know that

where M is a n upper bound for


522 CHAPTER 16: POWER SERIES AND TAYLOR SERIES

on [32, 331. But If4(t)i reaches its maximum on [32, 331 a t 32 because
504/(625tl9I5) is decreasing on [32,33]. Hence

1f4(t)i 504 - 504


(625)(32)19/5 - 625
-.-- 219 ,0.00000154
Choosing M = 0.00000154, we know that

lR4(33)l 5 M z0.0000000641
Hence the third Taylor polynomial yields the desired accuracy. Now

f(32) = 2 f(32) = -1 f(32) = - - 1 f(32) = - 9


80 3200 5 12,000
and therefore
1 1
f(33) = 2 + -(33
80
- 32) - -*-(33 - 32)
3200 2!
+-.-. 9 1
- 32) =: 2.012347 0
512,000 3! (33

Notice that in finding M in Example 1 we found an upper bound for lf4(t)l,


n ~ t f ( ~ ) ( If
t ) we
. had not reached the desired accuracy with the third Taylor
polynomial, we would merely compute additional terms as necessary.
Before continuing, we gather together three of the Taylor approximations
computed at a = 0 so far (see Progress Test 1).

x3 x5 x7 ( - 1)nx2n+1
16.1.5 (a) s i n x = : x - - + - - - +
3! 5! 7!
... + (2n + l ) !
( - 1)nxzn
(b) c o s x z l - -x22!+ - - x4
-+
4!
x6
6!
+ (2n)!

It is important to realize that this polynomial approximation for sinx is not


P,(x) but P2n+l(x). [Pzn+l(x)happens to equal P,,,,(x)-show this!] Similarly,
the previous polynomial approximation for cos x is PZn(x)[which equals
P2,+l(x)]. The given polynomial for e is Pn(x).

APPROXIMATISG e
Using the earlier limit representation of e given in (7.4.12), page 267

u = 10,000 is required to reach a n estimate of e accurate through three decimal


places. As is often the case, Taylor polynomials provide a much more efficient
approximation method. By taking x = 1 in [16.1.5(c)] we know that
e=el= 1 + 1 + -12+ - +13 ... + - I+nR n ( l )
2! 3! n!

=1 + 1+ + + 1
-
2!
1
-
3!
* * * +-
1
n!
+ Rap)
SECTION 16.1: TAYLOR POLYNOMIALS 523

where R,( 1) is the error involved in using the sum of the previous terms as an
approximation of e. By (16.1.4) we know that

where M is a n upper bound for lffl+l)(t)1 = IetI = et on [ O , x ] . Since et [the


( n + 1)st derivative of et] is increasing on [0, XI, it reaches a maximum at x. In
+
particular then for x = 1, we know that the ( n 1)st derivative reaches a
maximum of e1 = e on [0, 11. But we are trying to approximate e, so we cannot use
M = e as an upper bound on the error of the approximation! However, we do
know that e < 3 (recall the original definition of e), so we can use M = 3 as our
upper bound. Hence

For example, with n = 6 we get

ezP6(l)zl + 1 + -21+ - +61 - + 24-1+ - 1


120
1
720
z 2.718056 and lR6(1)1 0.000595
and with n = 10 we get
e zPlo(l) z 2.7182818 and lRlo(l)l z 0.0000000751
This means that Plo(l) provides six-place accuracy. In fact n = 17 yields the
15-place accuracy of the estimate given earlier and n = 39 yields 45-place
accuracy.

BOUKDS ON THE APPROXIMATION ERROR FOR SIN X


Determining an upper bound for If'"+l)(t)l on a n interval, which can then be
used to yield a n upper bound for lRn(x)l,is sometimes a creative art. In Example
2 we used the value of If'"+l)(t)lat an endpoint, whereas in approximating e
the endpoint value turned out to be the number e we were approximating.
In approximatingf(x) = sin x using P5(x) = x - ( x 3 / 3 ! ) + (x5/5!) we used
prior knowledge of sin x to compute R5(x),from which we concluded that P5(x)
gives four-place accuracy on [0, 7/41 (see Table 16.1). We can now use (16.1.4)
to verify this fact. However, instead of using an upper bound for IR5(x)l we use
that

= P5M
becausef(6)(0) = -sin 0 = 0. Hence we are entitled to use a n upper bound for
1R6(x)I and so we use n = 6 in (16.1.4). Now if'6+1'(t)l = I -cos tl 5 1 on
[0, 7/41. Thus
1 * 17/41'
IR,(~)I I 7! z 0.0000366
which says that P5(x) gives the four-place accuracy claimed. Note that P7(x)
gives six-place accuracy on [0, 7/41.
524 CHAPTER 16: POWER SERIES AND TAYLOR SERIES

PROGRESS TEST 2

1. Determine f/% to within seven decimal places (c) What degree Taylor polynomial for ex at a = 0 is
(usingf(x) = x1/3). necessary to approximate e5 to within four decimal
2. ( a ) Determine eOsl to within six decimal places. places?
( b ) Determine fhto within four decimal places. 3. Calculate Pn+l(x) forf(x) = ln(1 x ) at a = 0. +

SECTION 1 6 . 1 EXERCISES

In Exercises I to If, part (a), compute P n ( x ) f o r the given f ( x ) , a, and n.


1.( a ) f ( x ) = e-', a = 0 , n = 4 9. ( a ) f ( x ) = arctanx, a = 0, n = 2
( b ) Give P n ( x ) for arbitrary n. (6) Use ( a ) to approximate a/4.
2. ( a ) f ( x ) = In(1 - x ) , a = 0 , n = 4 10. ( a ) f ( x ) = cosh x, a = 0 , n = 4
( b ) Give Pn(x) for arbitrary n. ( b ) Give Pn(x) for arbitrary n.
3. ( a ) f ( x ) = In(cos x ) , a = 0 , n = 3 11. ( a ) f ( x ) = sinh x , a = 0 , n = 4
( b ) Use ( a ) to approximate In(cos 0.1). ( b ) Give Pn(x) for arbitrary n.
(c) Determine an upper bound on the error of the ap-
12. ( a ) Compute P3(x) and P 4 ( x ) for f ( x ) = x 3 - x 2 -
proximation in (6).
4. ( a ) f ( x ) = ln(sinx), a = a / 2 , n = 3
+
9 x 9 at a = 0.
( b ) Repeat ( a ) at a = 2, denoting the results by p,(x) and
(6) Use ( a ) to approximate ln(sin a/3). -
(c) Determine an upper bound on the error of the ap- P4(x), respectively.
proximation in ( b ) . (c) Compare P3(x) and & ( x ) by expanding and simpli-
5. ( a ) ~ ( x = ) m, a = 0, n = 3
( b ) Use ( a ) to approximate m.
fying & ( X I .
( d ) Use ( b ) to compute f(2.01).
(c) Use ( a ) to approximate fi. 13. ( a ) Compute P 4 ( x ) and P 5 ( x ) for f ( x ) = (x4/4) -
6. ( a ) f ( x ) = 2", a = 0 , n = 3 (x3/3) - x2 + 1 at a = 0.
( b ) Use ( a ) to approximate d. (6) Repeat ( a ) at a = 1, denoting the results by &(x)
(c) Compare part ( b ) with Exercise 5(c).
( d ) Determine an upper bound on the error 6(b).
and z(x)
(c) Compare P4(x)and < ( x ) by expanding and simpli-
7 . ( a ) f ( x ) = In -, a = O , n = 4 fying <(XI.
(1 - X I
( d ) Gse b to compute f ( l . 0 1 ) .
(Use properties of In before differentiating,) 14. For f ( x ) = e x and a = 0, graph ( a ) Pl(x), ( b ) Pz(x), ( c )
( b ) Compare ( a ) with Exercise 2(a). P&)> ( 4P4ix).
8. ( a ) f ( x ) = arcsinx, a = 0, n = 2
(6) Use ( a ) to approximate 77/6.

In Exercises I5 to 20 compute the given quantity to the given accuracy using Taylor polynomials and (16.1.4) to guarantee the accuracy. (You may use
results of previous exercises and leave your answer as a j n i t e numerical sum.)

15. In 2 accurate within 16. ln(3/2) accurate within


17. l / e accurate within 18. e2 accurate with
19. fiaccurate through four decimal places 20. a accurate through four decimal places
21. Using a = 77/8 instead of a = 0, which Taylor polyno-
mials yield six-place accuracy for sin x on [O,a/4]?
22. Repeat Exercise 21 forf(x) = cos x .
23. Compute P 6 ( x ) for f ( x ) = sin2 x by first using a double-
angle formula. 25. Use Exercise 24 to show that

24. ( a ) If f ( x ) = - show that at a = 0, P,(x) = --1


1-x'
I+$-
1-xZ+x4-x6+ ... +(-l)nP
1 + x + x 2 + '.. +x".
xn+l ( - l)n+lX2n+2
( b ) Using ( a ) show that R,(x) = -.
1- x + 1 + X Z
SECTION 16.2:POWER SERIES 525

Section 16.2: Power Series

Objectives: INTRODUCTION
Determine the radius of In the previous section we saw how a (finite-degree) polynomial P'(x) can be
convergence and interval of used to approximate a given function. What happens if we attempt to achieve
convergence of a given power perfect accuracy in the approximation by letting n grow without bound? The
series. answer to this question will couple the ideas developed in Chap. 15 regarding
infinite series with the notion of "infinite-degree polynomial," whose formal
definition and official name are given in the following:

16.2.1 D e f i n i t i o n An expression of the form


bo + b,(x - U) + bz(x - a)' + - * * + b,(x - a), + * *

where b,, b,, . . . , b,, . . . and a are constants, is called a power series in ( x - a).
If a = 0, then the series
bo + blx + b 2 X 2 + * * * + bnx" + *

is called a power series in x .


We shall often denote the above series, respectively, using summation
notation: m
m

2 b,(x - a)n and 2 bnxn


n=O n=O

with the convention that ( x - a)O = 1 and x o = 1-even when x = a and


x = 0. (This convention enables us to avoid the cumbersome notation necessary
to separate out the n = 0 term, which technically would be required to avoid Oo,
an indeterminate form.)
That a power series is different from an infinite series can be seen by asking
the question of either power series in (16.2.1): Does this power series converge?
Since convergence has only been defined for series of numbers, the question
makes no sense. However, for each number substituted for x in a power series we
do get an infinite series. For example, in (16.2.1) if 6, = 1 for all n, then we get
lf;=o x", which is a geometric series for each x and converges, to 1/(1 - x), for
all 1x1< 1, by (15.2.8). The appropriate question to ask of a power series is: For
which values o f x does it converge? Notice that Zz=o b,(x - a)n always converges at
x = a. It will turn out that if there are other points at which the power series
converges, then the set of these points will always comprise an interval.

THE RADIUS OF CONVERGENCE AND INTERVAL OF


CONVERGENCE OF A POWER SERIES

As Example 1 shows, the basic tool in determining the numbers x for which a
given power series converges is the Ratio Test (15.3.8), page 501.

mn
Example 1 Determine those x for which 2-
2,
converges.
n=O

Solution We regard mn/2* as un and examine the series Ztz0 iunl. We form the ratio of
successive terms
526 CHAPTER 16: POWER SERIES AND TAYLOR SERIES

Since

n-fm

we know by the Ratio Test that the series converges whenever (1/2)1x/ 1, that <
is, when 1x1 <2. It diverges when (1/2)lxl >
1, that is, when 1x1 >
2. A t x = 2 2
the Ratio Test fails, and so we examine each case separately. If x = 2, the power
series becomes
m 13
n
CF.2n=Cn
n=O n=O

which diverges [nth-Term Test, (15.2.6)]. If x = -2, the power series becomes

which again diverges. Hence Z:==, nxn/2" converges for all x in (-2,2). 0

The approach of the above example suggests a proof of the following


theorem:
.a

16.2.2 Theorem Suppose b,(x - a)n is a power series, with


n=O

lim
n-.+ffi 1 bn+l
-
bnl
=L

(a) If L = 0, then 2:,"=o


b,(x - a)" converges absolutely for all x .
< <
( b ) If 0 L + co, then Z,"=o b,(x - a)%converges absolutely for all x such
that Ix - a1 < 1/L, diverges for all x such that lx - a1 1/L, and may>
either converge or diverge when Ix - a / = 1/L.
(c) If L = + co, then Z:,"', b,(x - converges only for x = a.

Pro$ We prove (b), leaving ( a ) and (c) as exercises. From the limit of the ratio
of successive terms as in Example 1:

By the Ratio Test, the series converges absolutely when

Llx - a/ <1 or Ix - a/ <-1


L
It diverges when L(x - a1 >
1, or Jx - a1 >
1/L. The Ratio Test fails when
Llx - a / = 1, and the examples will illustrate that the series can either converge
or diverge when lx - a1 = 1/L. W

18.2.3 Definition SupposeZ,"=, b,(x - a)"isasin (16.2.2).Ifcondition( a ) holds, we


say that Z lz0 b, ( x - a)" has a n injnite radius of convergence and interval of convergence
( - 00, + co). If condition ( b ) holds, we say that 2;=o b,(x - a)n has radius of
convergence r = 1/L. In this case we say that the interval ofconvergence is the set of
all x for which Z:=o b,(x - a)" converges. (This interval may or may not include
endpoints.) If condition (c) holds, we say that Z:,"==,b,(x - a)n has zero radius of
convergence and a n interval of canvergevce consisting of the single point a.

Note that the radius of convergence of the power series in Example 1 is 2, and
the interval of convergence is ( - 2,2).
SECTION 16.2: POWER SERIES 527

We should point out that although the hypothesis in the previous theorem
and definition holds for all power series in this text, it does not hold for all
power series. Nonetheless, it can be shown that the set of x for which any power
series converges is an interval of one of the above types. Notice also that, in
effect, the above theorem says that convergence at the endpoints of the interval
of convergence must be handled on a case-by-case basis. (See, however, Sec.
16.2, Exercises 41 and 42).

Example 2 Find the radius of convergence r and interval of convergence I for each of the
given power series:

m m ..n
(a) C n!xn
n=O

Solutions We apply the Ratio Test to the series X& lunl where u, = bnxn,

+
Ifx # 0, then limn.++x(n+ l)/xl = co,and thus the series diverges by the
Ratio Test for all x # 0. Hence r = 0 and I = (0).

Now
1 -
lim
n-+m
( A=
n+1
)e-l 'and lim -
n++-n+1
-0

Thus, for all x ,

Since
<1

for all x , the series converges absolutely for all x. Hence r = + co and
I = (-m, +a).

Now
528 CHAPTER 16: POWER SERIES AND TAYLOR SERIES

for 1x1 < 2, so r = 2. Letting x = t 2 we obtain the series


m
2"n! P ( - 1)%2"n!
and
n=l
1 . 3 - 5 - . . . - ( 2 n - 1) n= I
1.3.5. ... *(2n- 1)

But
2%n! - -
- 2 4 6 * + . * (2n)
1 . 3 . 5 . . . . * ( 2 n - 1) 1 - 3 . 5 . ... e(2n- 1) > I
since each of the factors
2 4
- 6 2n
1' 3'5' . '
' 2n - 1
is bigger than 1. Thus both of the endpoint series diverge by the nth-Term
Test, making I = ( - 2, 2).

as n -+ + co, making r = 1. When x = 1, the power series becomes


m
1
C In(n + 1)
n=l

Now ln(n + 1) < n + 1. Thus


1 1
In(n + 1) >-n + 1
so the series diverges by comparison with the divergent series
Xz=l l/(n +
1). When x = -1, we obtain

which converges by the Alternating-Series Test. Hence I = [ - 1, 1). 0

Example 3 Find the interval I and radius of convergence of


n(x + 2)"
C 3n(n + 1)
n=O

Solution Now

Therefore

Thus by the Ratio Test the series converges absolutely when Ix 2//3 1, that + <
is, when lx 21+ < 3. It diverges when / x + 21 >
3, so r = 3. Since a = -2 and
r = 3, we must examine the endpoints of the interval ( - 2 - 3, - 2 3) = +
(-5, 1). Substituting x = -5 and x = 1 we obtain, successively,
m
(-1)". n
n=O n=O

both of which diverge, by the nth-Term Test. Thus I = ( - 5 , 1). 0


SECTION 16.2: POWER SERIES 529

PROGRESS TEST 1

F i n d the radius r and interval Z of convergence for each


of t h e following power series.

3.
9 (X - 4)n

SECTION 16.2 EXERCISES

I n Exercises I to I8 determine the radius of convergence r and the interval of convergence I for the given power series.

1.
2. 5
n=O
( - 1)"2nxn
3n-1

15. 5
n=l

17. 18. 5
n=O
( - l)%qn+l(x- 3)%
32"

n!x" 1.3.5- ... * ( 2 n - l)xn


19. For the power series - 20. For
n=l nn n= 1 2nn!
( a ) Show that r = e. ( a ) Show that r = 1 .
( b ) Show that the series diverges at x = e by showing that ( b ) Show divergence at x = 1 by comparison with the
the ratio a,+,/a, always exceeds 1, where a, = n!en/nn. divergent series E:=l 1/(2n).
[Hint: Show that ( c ) Apply the Alternating-Series Test for the series at
x = - 1 by following the given outline:
(i) Show that a, > an+l for all n, where

by taking the natural logarithm and using the fact that


a, = 1 3 5 - . * (2n - 1 ) / ( 2 " n ! )
(ii) Show that an2 + 0 as n -+ + 00 (from which it
In(1 + l / n ) 5 l / n . ]
folIows that a, + 0.) Hint: Show that an2 <
(c) Show how the argument in ( b ) applies when x = -e. ( 2 n - 1)/(2n)' and apply the Squeeze Theorem,

In Exercises 21 to 36, ( a ) determine r and ( b ) determine I for the given power series.
530 CHAPTER 16: POWER SERIES AND TAYLOR SERIES

x"
31. c,
n=2
[Use the Root Test (15.3.9).]

33. c n!
n=l
nnx"
(See Exercise 19.)

35. c +
r

n=O
[l (-l)"]xn

In Exercises 37 to 40 determine the set of xfor which the given series converges. (Note these are not power series in x, but nonetheless the question makes
sense and can be answered. Note also that the set of such x need not be an interval.)

(sin x)"
38. C-(cos X)n
71x1

40.
m

2 ( x 2 = 2x + 1/4)"
n=1
(Consider first ca

n=l
u", where 43. (Root Test) Prove that if lim,,,,
radius of convergence of Z:=o b,x" is 1/L.
Ib,ll/n = L , then the

= x 2 - 2x + and then use a sketch Of 44. Show that the Root Test approach to the radius of
x 2 - 2x + 1 / 4 . ) convergence (see Exercise 43) applies to
41. Show that if Z;==, bnxn converges absolutely a t one end- Z=
;, ~ " / l O ~ - ( - l ) ~ whereas
, the Ratio Test fails.
point of its finite interval of convergence, then it does
likewise a t the other.
42. Show that if a power series Z,"=, b,x" has a half-open
interval of convergence, then convergence is conditional
at that endpoint.

Differentiation and
Section 16.3: Integration of Power Series-Taylor Series

Objectives: DIFFEREXTIATION OF POWER SERIES

Compute a power series If Z;==, bnxn has a nonzero radius of convergence 7, then for each x in ( - r , r ) ,
ZgG0b,(x -a), that equals a given ;, b,xn converges to a number. If we call this numberf(x), then ZE=o bnxn
Z=
infinitely differentiable function on defines a function on ( - r , 7 ) given by
-
an interval (a r, a + r), r > 0, by m

using differentiation, integration, or


other algebraic manipulation of a
f(x) = 2 bnxn
n=O
for x in ( - r , r )

given power series, or by using


In such a circumstance we shall say Z;==, bnxn represents f ( x ) on ( - r , r).
Taylor polynomial coefficients.
When introducing power series we noted that for each x in (- 1, l), Z=
;, xn is
a geometric series converging to 1/(1 - x). Thus Z:,"=oxn representsf(x) =
1/(1 - x) on (-1, 1) because

We can ask, in general, what are the properties of functions that can be
represented by power series? As remarked earlier, a power series can be thought
of as a generalized ("infinite-degree") polynomial, and polynomials have very
convenient mathematical properties. They can be differentiated and integrated
term-by-term as many times as we wish. It turns out that similar properties hold
for power series, provided we stay in (- r, r). Furthermore, as the following
SECTION 16.3: DIFFERENTIATION AND INTEGRATION OF POWER SERIES 53 1

theorem asserts, the derivative of the power series that represents f ( x ) will
representf'(x) on ( - r , r). We leave its rather involved proof to more advanced
courses.

16.3.1 Differentiation of Power Series Suppose 2z=ob,x" has a radius of


>
convergence r 0 andf(x) = Z:==, b,x" on ( - r , r). Thenfis differentiable on
(- r, r ) and
X

f'(x) = nb,xn-l for each x in ( - r , r )


n= 1

This theorem makes assertions about the radius of convergence and not the
interval of convergence, which may or may not contain any of its endpoints. For
the most part we shall confine our attention to the open interval and ignore the
case-by-case analysis of endpoints. A similar theorem holds for powers of
( x - a ) , a # 0.
As illustrated in the next example, the best way to differentiate a power series
is to write out its first several terms before differentiating. It is not advisable to
work only with the general term because confusions regarding the indices are
quite likely.

x"
Example 1 Prove that - = ex for all real numbers x .
n=O n!

Xn
Solution By the Ratio Test the series Z,"=07 converges for all x. (Check this!) If
n.

for all x , then by (16.3.1),

= l + x + - +X 2. . . + - p l + -X"+ ...
2! ( n - l)! n!

Which function equals its derivative? We showed in Section 8.1 that if


dy/dx = y , theny = cex. Hencef(x) = ce" for some constant c. But sincef(0) = 1
in ( l ) , we can conclude that c = 1 andf(x) = ex. 0

In Sec. 16.1 we constructed Pn(x),the nth Taylor polynomial forf(x) = ex at


a = 0 , using the first n derivatives of e x , getting

Since ex has derivatives of all orders, the same procedure that gives the Taylor
polynomials can be used to generate the power series in Example 1.
532 CHAPTER 16: POWER SERIES AND TAYLOR SERIES

16.3.2 Definition Let f be a function which has derivatives of all orders at x = a.


The series

is called the Taylor seriesfor f at a or the Taylor series expansion off about a. If a = 0,
then the Taylor series

is often referred to as the Maclaurin series. (The British mathematician Colin


Maclaurin was a younger contemporary of Taylor in the 1700s.)

In effect the limit, as n + + 00, of the Taylor polynomzal approximatzons of ex


is the Taylor series for ex. Furthermore, and most important, the power series
represents ex exactly for each x. Recall that (16.1.3) states that

f ( x ) = Pn(x) + Rn(x)
where R,(x), the nth Taylor remainder, is the error involved in using P,(x) to
approximatef (x). If a functionf has derivatives of all orders in an open interval
I containing a, then it is always possible to form the Taylor series for f at a.
However, Example 8 will show that the Taylor series for f need not represent f on I,
even when the power series converges for all x. The key is whether or not
R,(x) + 0 as n + +a.

16.3.3 Taylor's Theorem Supposef has derivatives of all orders on ( a - r , a + r)


>
for r 0. Then

if and only if limk-+mR k ( x )= 0.


Pro@ For each positive integer n we know that f ( x ) - Pk(x) = Rk(x) on
(a - r, a + r ) .
f(x)= c- f'"'(a)(x
MI - a), if and only if

f ( x ) = klim Pk(x) if and only if


++m

lim [ f ( x ) - Pk(x)]= 0 if and only if


k++m

lim Rk(X) =0
k++m

Example 2 Show that the Maclaurin series for sin x is given by


2n+ 1
sin,y=x--+--
3!
x3 x5
5!
. . . + (-IT (2n + l)!
+ ...

for all x
n=O (2n + l)!
Solution We have already shown in [16.ls5(a)]that the Taylor polynomials for sin x are
the partial sums for the series indicated above for all x. Hence it remains to
show that as n -+ + 00, the remainder tends to 0. But, by (16.1.3),
SECTION 16.3: DIFFERENTIATION AND INTEGRATION OF POWER SERIES 533

for some cn with 16%I <


1x1. But the only possibilities forf("+l)(c,) are t sin cn or
-C cos c,. In any of these cases lf(n+l)(cn)l5 1, so

However, by the Special Limit Theorem [15.1.6(h)], for any real number r,
limn-, rn/n! = 0. Hence
IXln+l
0 5 nlim IR,(x)l 2 lim -= o for all x
-+x n-+m ( n + l ) !

It follows by the Squeeze Theorem (15.1.5) that limn,+, IR,(x)l = 0, so


limn,+, R,(x) = 0. The conclusion follows by Taylor's Theorem. 0

Although we could develop the Maclaurin series for cosx similarly, we


choose instead to differentiate the series for sin x term by term. By the Differ-
entiation Theorem for Power Series (16.3.1),the derivative of the sine series will
give the cosine series:
x3 xj x7 ( - l)nx2n+l
D,[sinx] = D,[x - -
3!
+-
5!
--+ ... +
7! (2n + l)! + .**I
( n = 0 , 1 , 2 , .. . )
so

cosx= 1
3x2
--+ - - - +5x4 7x6
+ + 1)x2" + . . .
(2n + l)!
a * *

3! 5! 7!
-1
- + - - x4
- - x2 -+ x6
... + (-IYxzn + ...
2! 4! 6! (2n)!

Of course, the partial sums of this series are precisely the Taylor polynomials for
cosx recorded in [16.1.5(b)], page 522.

PROGRESS TEST 1

1. Show that the Maclaurin series forf(x) = 1/(1 - x)


is 8,"=, x n for x in ( - 1, 1). [Note that earlier state-
ments confirm that this power series represents
f(x) = 1/(1 - x) on (-1, l).]
2. Use Prob. 1 to give a power series representation for
g(x) = 1/(1 - x ) on ~ (-1, I).

COMPUTATIOSS OF POWER SERIES REPRESENT-ATIONS


Problems 1 and 2 of Progress Test 1 suggests that power series may arise or be
computed in more than one way. The following theorem will be very useful in
dealing with power series representations for functions because it guarantees, in
effect, that any power series representation of a function on an open interual is its Taylor
series-no matter how that series is computed.
534 CHAPTER 16: POWER SERIES AND TAYLOR SERIES

16.3.4 Uniqueness of Power Series Representations If f ( x ) =


2,"=0bn(x - a), for lx - a1 < r, then b, = f n ( a ) / n ! for each n = 0, 1 , 2 , , . . ,
Hence if
a. 30

2 b,(x - a), = c,(x - a), on ( a - r, a + r)


n=O n=o

then

cn'(4 for each n = 0, 1, 2 ,


b, = ,C = f- ...
n!
Any twopower series represelltations of a function f on an interval ( a - r, a + r ) must
be the same.
Pro$ We apply the Differentiation Theorem (16.3.1) repeatedly:
f ( X ) = bo - U ) + b2(x -
+ b,(x + b,(x - + b,(x - + . . 9

+ 2b2(x - a ) + 3b3(x - + 4b4(x - + . . *


f ' ( X ) = b,

f " ( x ) = 262 + 3 * 2b3(x - U ) + 4 * 3b4(x - + ...


- -
f "'(x) = 3 * 2b, + 4 ' 3 2b4(x - a ) + 5 4 * 3 ' 2b,(x - a)2 + . . 9

b, = f'"'(4
-
n!
.
Notice that in the above proof we used the fact, implied by (16.3.1), that if
+
f has a power series representation on ( a - r, a r ) , thenf has derivatives of all
orders on ( a - r, a + r ) . Some authors refer to a function with such a power
series representation as analytic on ( a - r, a + r).
We are now free to compute power series representations for functions
without resorting to the usually difficult procedure of computing the Taylor
series directly via nth derivatives and the often more difficult chore of showing
that R,(x) + 0. Notice, for example, how difficult it would be to compute
higher derivatives in the following example.

Example 3 Compute the Maclaurin series for f ( x ) = xe".


Solution By Example 1 , we know that e" = 2 z = O x n / n ! for all x . Hence

(Note that x acts as a constant with respect to the series X Z z 0 x n / n ! , so


[15.2.3(b)]page 490, applies.) 0

Another way of constructing power series representations amounts to a


"composition" with a known series. In such cases it is important to note how the
radius of convergence is affected. I n general, if the original representation is
valid for Ix - a1 <
r and we replace x by u ( x ) in this representation, then the
new representation is valid for all x such that Iu(x) - a/ r. Usually we solve <
this last inequality for lx - al.
SECTION 16.3: DIFFERENTIATION AND INTEGRATION OF POWER SERIES 535

Example 4 Construct power series representations for


1
l + x
Solution +
We regard 1/( 1 x ) as 1/( 1 - ( - x ) ) and replace x by - x in the power
series for 1/(1 - x ) at a = 0:
m X

The original radius of convergence is 1. But 1 - x / <


1 if and only if 1x1 1, <
so the radius of convergence of xz=o
(- 1 ) " ~ "is also 1.
We use the series constructed in ( a ) as the starting point and replace x by
4x2, getting
1
1 + 4x2
= cX

n=O
( - 1)"(4x2)" = 1 - 4x2 + 4 2 ( ~ 2 )- +
2 4 3 ( ~ 2 ) 3 4 4 ( ~ 2 )-4 *

m
= (- l)n4n~2n
n=O

The original series converges for 1x1 <


1, so the new series converges when
149 <
1, that is, when 1x1 <
1/2.
In Progress Test 1, we differentiated the series for 1/( 1 - x ) to get the series
for 1/(1 - x ) ~ :
m

We now replace x by x 2 to get the desired power series:


m

n= 1

The original series converges for 1x1 <


1, and since lx21 <
1 if and only if
1x1 <1, we know that the latter series has a radius of convergence equal
to 1. 0

INTEGRATION OF POWER SERIES


The Differentiation Theorem for power series (16.3.1) has a n equally useful
analog for integration. [We omit its proof, which is an application of (16.3.1)
and the Fundamental Theorem of Calculus.]

16.3.5 Integration of Power Series Suppose Z;=o bnxn has a radius of con-
>
vergence r 0 andf(x) = Z;=o bnxn. Then f is integrable on ( - r , r ) and

This theorem allows us to integrate power series term by term on the interior
of the interval convergence and to construct Taylor or Maclaurin series that
536 CHAPTER 16: POWER SERIES AND TAYLOR SERIES

could be difficult to construct using derivatives. A similar theorem holds for


power series in ( x - a ) with 0 replaced by a in the lower limit of integration.

Example 5 Determine a power series representation for


f ( x ) = arctanx for 1x1 <1
Solution Using Example 4(a), we replace x by x 2 in
*
1
= C(-l)"x"
1+x n=O
to obtain

But now, applying (16.3.5) to this latter series, we have

Notice that while arctan x is defined for all real numbers, the power series
representation is not. It can be shown using Taylor's Remainder Theorem that
the representation is valid on [ - 1, 11. However, for 1x1 >
1, the power series
diverges.
Our techniques also extend to power series at nonzero a.

Example 6 Determine the Taylor series for In x at a = 1.


Solution Writing
1-
- - 1
x 1+(x-1)
we then replace x by x - 1 in the power series representation of 1/( 1 + x ) ; that
is, Z;==, ( - l)nx" [see Example 4 ( a ) ] . We obtain

We now apply the Integration Theorem to get

(Compare with the Taylor polynomials for l n x in Sec. 16.1, page 517.) 0

PROGRESS TEST 2

Determine power series for each function given at a = 0.


Note the radius of convergence for this power series.
1. xe-' 2. sin2 (In Prob. 3, integrate the series for 1/(1 x) and +
1/(1 - x), respectively, and then subtract, using the fact
that the difference of convergent series is convergent.)
SECTION 16.3: DIFFERENTIATION AND INTEGRATION OF POWER SERIES 537

THE BINOMIAL SERIES


Most of our earlier manipulations with series have used the geometric series as
a starting point. We now introduce another "starting-point" series, which is a
generalization of the Binomial Theorem.

16.3.6 Binomial Theorem If m is a positive integer, and u and b are real numbers,
then

(a + b)" = am + (;")u"-'b + (y)am-2b2 + * * . + (m - 1 )ab" + bm


where
m! m(m - l ) ( m - 2 ) . . . ( m - n + 1)
)(: = n!(m - n ) !
-
n!
Our particular interest is in expanding (1 x ) ~ where
, +
m is not necessarily a
positive integer. We shall define a n appropriate series, show that its radius of
convergence is 1 , and assert that it does represent the function ( 1 + x ) ~ .

16.3.7 Definition The series

1 +mx+
m(m - 1 )
2! x2+ ' * * + m(m - 1 ) . .n.!( m - n + + ..,

for m any real number, is called the binomial series. The numbers
m(m - 1 ) . . . (m - n + 1)
= n!
are called the binomial coejicients.

We can use the Ratio Test to determine the radius of convergence of the
binomial series:

m(m - 1 ) . . . (m - n + l ) ( m - n) n!
(n + I)! m(m - 1 ) e e(m - n + 1)

Since lim,,,, 1x1 Im - nl/(n + 1 ) = 1x1, we know that the radius of convergence
is 1 .
For m a positive integer, the binomial series reduces to the polynomial
+
expansion of (1 x)" given by the Binomial Theorem. (Check this.) In general
we have the following:

16.3.8 Theorem For m any real number and any 3c in ( - 1 , l ) ,

(1 +X)m= 1 c (p
+ n= 1
538 CHAPTER 16: POWER SERIES AND TAYLOR SERIES

This theorem can be proven either using Taylor series or manipulations of


the derivatives of (1 + x ) and
~ the binomial series. An outline of the proof is
given in Exercise 45. In any case, the Uniqueness Theorem for power series
(16.3.4) tells us that the traditional binomial coefficients are also the Taylor
coefficients f(")(o)/n!.

Example 7 Determine the Maclaurin series representation for d G


Solution Let m = 1/2. Then, by (16.3.8),

Now

and, in general
1/2(1/2 - 1)(1/2 - 2) . . . (1/2 - n + 1)
n!
-
- 1/2(-1/2)(-3/2) + .((3 - 2n)/2)
n!
- 3 * 5 * . . . * (2n - 3))
- (- IY-l(I
2"n!
Hence

PROGRESS TEST 3

1. Determine a power series representation off(x) = 1/ d E for x in (- 1, 1).

MACLAURIK SERIES FOR e-lIx2


The following example promised earlier shows that a Taylor series developed
from a given infinitely differentiable function can converge for all x yet fail to
represent that function.

Example 8 Show that the Maclaurin series for


ifx#O
ifx=O
converges everywhere but represents f only when x = 0.
Solution First,f(O) = 0. To determinef'(0) and subsequent derivatives we shall use the
definition of derivative:
SECTION 16.3: DIFFERENTIATION AND INTEGRATION OF POWER SERIES 539

e-l/x2
1/ x
= lim- = lim-
x-0 x x-o e 1 / x 2

Since limz-o Il/xl = limx+o~ e l = +


~ ~ *co,~ we can apply 1Hospitals Rule
(co/co) to obtain
1/ x - 1/x2
f ( 0 ) = lim- = lim
z-o & / x 2 x - ~&x2( -2/x3)

and therefore

lo ifx=O
Now

f ( 0 ) = lim f ( X ) - f ( O ) = lim-f (XI


8-0 x -0 m-0 x
ze-1/x2 21~4
= lim- = lim- ( ~ / ~ )
x-0 x4 x-o e 1 / x 2
-8 / x 5 4
= lim = lim- =o
2-0 e1/a2(-22/X3) z-o x 2 e l / r 2

Similarly, f cn)(0)= 0 for all n. Thus the Maclaurin series for f is

But f ( x ) # 0 if x # 0 and thus, although the Maclaurin series converges for all
values of x, it only represents f at x = 0. By Taylors Theorem, Rn(x) 0 except +
for x = 0. 0

SOME STANDARD MMACLAURIN SERIES


We list for reference the Maclaurin series that have been developed to date or
can be determined in a similar fashion.
540 CHAPTER 16: POWER SERIES AND TAYLOR SERIES

+ 1) xn
(f)(1 + x)m = 1 + ccc

nrl
m(m - 1) + a

n!
(m - n
1x1 <1
x X2n+l
(9)sinhx =
n=O (2n + l)!
X X2n
( h ) cosh x = -
n=O
(2n)!

SECTIOS 16.3 EXERCISES

I n Exercises 1 to 14 give a power series representation (at a = 0 unless otherwise indicated) for the given function on the indicated interval. If the
interval is not given, identlfy the radius of convergence. You may use representations developed earlier unless specific directions call for another approach.

1. 2. f ( x ) = e32, all x

3. ~ ( x =) x2e-r2, all x 4. f ( x ) = (sin x ) / x , all x # 0


5. f ( x ) = ( x - l ) l n x , at a = 1 6. f(x) = sinx, at a = v / 4
7. f ( x ) = (1 - cos x ) / x , x #0 a. f ( x ) = In(1 + x2)
arctan(x3)
9. f(x) = xln(1 + x4)>1x1 < 1 10. f ( x ) =
x

11. 12.

13. 14.

15. ( a ) Use the definition of Maclaurin series to prove


( b ) Substitute ( - x 2 / 2 ) in the geometric series and de-
[1 6 . 3 . 9 ( g ) ] .
termine the radius of convergence.
( b ) Derive [16.3.9(g)] from [16.3.9(h)].
16. Derive [ 1 6 . 3 . 9 ( g ) ]using the definition of sinh x .
( c ) Replace x in the resulting representation by x 2 +
and multiply both sides by ( x + 2 ) / 2 to obtain a repre-
17. Derive [16.3.9(h)] using the definition of cosh x .
sentation of the first fraction term of ( a ) .
18. Use integration and Example 7 to give a power series for
ImTp. ( d ) Subtract the representation for 2 / [ ( x 2 2 ) 21 + +
that occurred in ( c ) and combine the resulting series by
19. Determine a power series representation for f ( x ) =
(1 - 2x2)/(1 +
2 ~ by ~using
) the
~ following outline: writing out the first several terms.
( a ) Replace x in the geometric series [which represents 22. Use the idea of Exercise 21 to determine a power series
1 / ( 1 - x ) ] by -2x2 and note the radius of convergence representation o f f ( x ) = ln(x2 4x + +
13) in powers of
of the resulting series. x + 2.
( b ) Multiply this series by x and differentiate both the 23. ( a ) Determine a power series for f ( x ) = arcsin x by
function represented and the series to obtain a series making an appropriate substitution into the series in
representation for f ( x ) . Progress Test 3 , Prob. 1 , and integrating.
Determine a power series expansion for fi arctan t 2 dt, ( b ) Write out and simplify the first four terms of the
20.
1x1 < 1, by the following given outline:
series in ( a ) .
24. Determine a power series representation of f ( x ) =
( a ) Make a n appropriate substitution into [ 16.3.9(e)].
1 / ( x 2 - 4 x + 5) in terms of x - a for a n appropriate a.
( b ) Integrate.
State the radius of convergence.
21. Determine a power series expansion for f ( x ) =
x/(x2 + 4x + 6 ) in powers of x
given outline:
+
2 by the following

( a ) Complete the square and break the fraction into two


25.

26.
Repeat Exercise 24 forf(x) = arctan
t3
-
Repeat Exercise 24 for f ( x ) = ln(2 - x - x 2 ) . (Hint:
2),

terms by writing its numerator as ( x + 2 ) - 2. Factor 2 - x - x2.)

I n Exercises 27 to 38 compute the Maclaurin series (except Exercise 2 8 )f o r the givenfunction using earlier results where appropriate. Give the radius uf
convergence, (Note that in some exercises trigonometric identities may be useful.)
27. sinxcosx 28. (x - l ) e - ~ * + 2 ~ - la, t a = 1
SECTION 16.4: MISCELLANEOUS APPLICATIONS OF POWER SERIES 54 1

29. sin2(2x) 30. J s l n ( l - t)dt


0
X sin(x2)
31.
(1 - x)2 32. -
X

33. 4 u 3 cos( 6)
du 34. cos2(x/3)

35. 46 + 3x 38.
qi=7
X

X
37. 38.
q .
39. Suppose that 42.Using the Taylor series for l n x at a = 1 developed in
Example 6, show directly that limn,+, R, (x) = 0 for x in
= (-l)nxzn+l
f(x) = c
n=O (2n + l)!
(1, 21. [R,(x) also tends to 0 for x in (0, 11, but this is more
difficult to show.]
43. Find the Taylor series for l / f i at a = 1 and show
for all x. Show that y =f(x) satisfies the differential directly that limn,+, R,(x) = 0 for x in [ l , 2).
equation y + y = 0. 44. The differential equation
40. Use the result of Exercise 4 to give another argument that
lim,,o[(sin x)/x] = 1. x -d2y
+-+ 4 xy = 0
dx2 dx
+
41. Prove that f(x) = In( 1 x) is represented by its Mac-
laurin series on (0, 11by following the given outline. (This is called Bessels equation of order zero. For x > 0 define
fact has, in effect, been proved earlier, and you may use
some of these earlier calculations as a check on your
work.)
( a ) Determinefn)(x) and thenf(n+l)(x)/(n + l)!.
( b ) Use ( a ) to determine R,(x) for x in ( 0 , 11. + (Jo(x)) + x(Jo(x))
Show that x(Jo(x)) = 0.
(c) Show that x/(l +c,) <
1. 45. Letf(x) = 1 + Z(T)xn, -1 < x < 1.
(d) Use (c) to prove that R,(x) + 0 as n + + co. [R,(x) (a) +
Show that (1 x)f(x) = mf(x).
also tends to 0 on ( - 1,0), but this is more difficult to ( b ) Show that f(x) = (1 x) by + showing that
show.] ln(f(x)) = ln[(l x)]. +

Section 16.4: Miscellaneous Applications of Power Series

Objectives: NUMERICAL APPROXIMATIONS


1. Use power series to approximate Our work with Taylor polynomials in Sec. 16.1 gave us methods to approxi-
the values of the functions they mate the value of a function, with bounds on JR,(x)l providing accuracy
represent. estimates. In some cases the calculations required in using R,(x) are quite
2. Use power series to represent complicated, but can be circumvented using (15.4.6), the First-Term-Neglected
nonelementary antiderivatives. Estimate for a n alternating series c,=~(
- l)na, whose terms are decreasing in
3. Use power series to approximate absolute value (recall page 507):
definite integrals.
the sum of the terms a. - al + a2 - . . . + ak is within lak+ll of the sum of
the entire series

Example 1 Approximate ln(3/2) through three decimal places using the power series
representation of ln(1 + x ) , 1x1 1. <
Solution Using [16.3.9(d)], for 1x1 < 1, we have

Thus, with 3/2 = 1 + 1/2,


542 CHAPTER 16: POWER SERIES AND TAYLOR SERIES

This is an alternating series with 1/[2n+1(n + l)] decreasing, so the sum


k

C (2+1)(n + 1)
n=O

+
is within 1/[2k+2(k 2)] of the sum of the entire series, namely, ln(3/2).
If k = 5, the error will be bounded by

-
1 --
Z7- 7
--
1
896
- 0.001
If k = 6, the error will be bounded by
1
--
28-8
- 1
2048
z 0.000488
yielding the desired accuracy (error < 0.0005). Hence
ln(3/2) =: -
1 --
2 4.2
1
1 +---
8.3
1 +---1
16.4 32.5
1
64.6
+- 1 2 8 4 0.406

In the previous example we used error when, to be precise, we meant the


absolute value of the error. We shall use error in this way throughout this
section.
Notice that the above technique is preferable to a Taylor polynomial
approach when it applies, because we merely compute additional terms of the
series to improve our accuracy and examine the size of the first term not added.
This is in contrast to the Taylor polynomial approach, where computation of
bounds on the ( k + 1)st derivative can be cumbersome and need be repeated
for each improvement in accuracy.
If we can manipulate a known series to fit a given situation, we can then
avoid calculating derivatives and compute numerical values directly. In Sec.
16.1, we revisited the approximation of $% using Taylor polynomials for
f ( x ) = x1I5 at a = 32, computing an approximation accurate through six
decimal places. Using the binomial series, only five terms yield an approxima-
tion accurate through eight places. (See Sec. 16.4, Exercise 8.)

APPLICATIONS OF POWER SERIES TO INTEGRATION


In Sec. 11.6 we developed two methods, the Trapezoid Rule and Simpsons
Rule, for approximating a definite integral for which the antiderivative was not
a n elementary function, a function expressible as a jinite combination of
functions studied up to then (powers, roots, quotients and compositions of
polynomials, trig functions, or log and exponential functions). We now have the
means to express such nonelementary functions more explicitly. By manipu-
lating such power series representations, we have yet another way to approxi-
mate specific definite integrals. However, the following example accomplishes
more than we were able to do in Section 11.6, namely, represent a n actual
nonelementary function.

Example 2 Compute an antiderivative of e-.


Solution (It was noted in Sec. 11.5 that such a n antiderivative is not a n elementary
function.) Substituting - x 2 in [16.3.9(c)], we have
SECTION 16.4: MISCELLANEOUS APPLICATIONS OF POWER SERIES 543

Thus, applying the Integration Theorem for power series (16.3.5), we obtain

(- 1 ) n p + 1
-
2
- n=O n ! ( 2 n + 1)
This gives us a power series representation of Jox ePtz dt, valid for all x. 0

As Prob. 3 of both progress tests in Sec. 11.6 we estimated


subdivisions and used derivatives to determine bounds on the errors involved;
4' ePx2 dx using 10

actually, we used an informal bound on the fourth derivative for Simpson's


Rule because the fifth derivative was too complicated to analyze directly.
Three-place accuracy resulted using either rule.

Example 3 (u) Estimate 12


ePx2 dx using

Give a bound for the error in (u).


Give a bound for the error when k = 20.
We merely substitute into the power series:

4' ePxz dx z2
(-1123
+-
l !* 3
(4225
+-+-+
2!.5 3! ' 7
(-11327
*..+
( - 1 )10221
-
l o ! 21
+---+ ...
=2-- 23
1!.3
25
2!*5 3!*7
27
+-lo! * 21
% 0,889045

We have a n alternating series, so it remains to show that its terms are


decreasing in absolute value. This can be done by showing that

I")>1 (- 1)" 22n+1


for lunl =
bn+l n!(2n + 1)
But

1% - -
22n+l (n + 122n+3
) ! ( 2 n+ 3 )
I%+ll ( n ! ) ( 2 n+ 1 )
- --.- +
2n+3 (n+1)
2n 1 P >-n +4 l >1

for n >
3. Thus, the terms beyond u3 are decreasing in value, so the error is
bounded by
I(- 1)11223j
la111 = z 0.00914
11!.23
844 CHAPTER 16: POWER SERIES AND TAYLOR SERIES

(c) For k = 20, the error is bounded by

l'211 = 21!.43 = 0.000000004


243 (eight-place accuracy) 0

Note that although we have leaned heavily on the alternating-series error


bound, constructions comparable to the above series approximations apply for
other types of series. However, determining bounds on the error involved may
be more difficult, perhaps requiring use of Taylor's Remainder.

PROGRESS TEST 1

1. Here we improve on the Trapezoid Rule and Simp- oped in Progress Test 2, Sec. 16.3, determine how
son's Rule approximations of In 2 that occurred as many terms are required to reach an approximation
Prob. 1 of both progress tests in Sec. 11.6. There, the accurate in the third decimal place, given the previ-
Trapezoid Rule with five subdivisions gave In 2 ous information.
accurate through two places, Simpson's Rule with (c) If In 2 z 0.693147 is accurate in the sixth deci-
four subdivisions gave In 2 z 0.693 accurate in the mal place, how many terms are required to exceed
third decimal place. To achieve five-place accuracy the accuracy of the Trapezoid Rule with 183 subdi-
the Trapezoid Rule required 183 subdivisions and visions and Simpson's Rule with 14 subdivisions?
Simpson's Rule required 14 subdivisions. 2. ( u ) Compute the (nonelementary) antiderivative
( a ) Determine a number x between 0 and 1 such J": sin(t2)dt.
+
that (1 x ) / ( l - x ) = 2. ( 6 ) Use ( u ) to determine J";
sin(t2)dt to within
( 6 ) Using the series for ln[(l + x ) / ( l - x ) ] devel-

SECTION 16.4 EXERCISES

In the problems below you may leave your approximation as a Jinite numerical sum.
1. ( a ) Use the Maclaurin series for arctan x with n = 3 to (c) Show that the nth term a, of the binomial series is of
approximate T. the form a, = u/(n!5"2jn-l), where u is the product of the
( b ) Repeat ( a ) with n = 5. numbers 5(k - 1) - 1 for k = 2, 3, , , . , n.
(c) What value of n is required for 10-place accuracy? ( d ) It can be shown that the la,l are decreasing. Use this
( d ) Compare the above results with the approximations fact to give a bound on the error involved in using the
of T given in Sec. 11.6 using the Trapezoid Rule and first five terms of the series a s an approximation.
Simpson's Rule. ( e ) Why does the series for (1 + x)l15 fail to yield an

2. Compute l1 e-x3 dx within


estimate for
9. (Another approximation of In 2) ( a ) Show that the
Maclaurin series for In( 1 - x ) on ( - 1, 1) converges at
3. Compute 4 112 dx
-
1
within
+ x4
its left-hand endpoint and represents In 2 there.
( b ) Use ( a ) to give another series for In 2.
4. Using Sec. 16.3, Exercise 4, determine Jil2(sin x ) / x dx (c) Use ( b ) to approximate In 2 with n = 2 .
within ( d ) Repeat ( c ) with n = 5 .
5. Use the binomial series to approximate f,h-f to five- 10. ( a ) Use the power series for -ln(l - x ) with x = 1/2 to
place accuracy. give another series for In 2. [Note: -1n(1/2) = In 2.1
6. Use five terms of the appropriate binomial series to ( b ) Use ( a ) to approximate In 2 with n = 2.
approximate $'% (c) Repeat ( b ) with n = 5 .
7. Approximate Jh d-dx through four decimal 11. Compare the three series approximations for In 2 from
places. Progress Test 1, Prob. 1 and Exercises 9 and 10, and the
8. By following the given outline, use the first five terms of two integration approximations from Sec. 11.6, and rank
the binomial series for [ l + ( ~ / 3 2 ) ] l /to
~ approximate them in order of convergence efficiency.
i)% and give a bound for the error involved.
( a ) Change the form of the given expression to 12. ( a ) Compute
(1/2)(32 + x ) l I s and show that the radius of conver-
dx
gence of its binomial series is 32. ( b ) Estimate -within
1 +x3
( b ) Write out and add the first five terms of this bino-
mial series for x = 1
SECTION 16.4: MISCELLANEOUS APPLICATIONS OF POWER SERIES 345

In Exercises I3 to 17 evaluate the given integral to within 10-4.

13. 4' d m d x 14. J1e-x3dx


0
15. $,'& 1 % x3
16. 4li3 +
17. 1'" arctan(x2)dx

18. ( a ) Determine a formal alternating series for dt to within 10-6.


(1 - c t ) / t dt (the "exponential integral") without
regard to convergence.
( b ) Show that

as defined in (a).

CHAPTER EXERCISES

R e v i e w Exercises
In Exercises I to 9, ( a ) determine the fourth Taylor polynomial for the given function at the given point, ( b ) determine an upper bound on the error
involved in using this polynomial to approximate the quantity given in brackets.
1. ex, a = I ; [&I 2. sinx, a = a/6; [sin(a/4)]
3. cosx, a = a/4; [cos(a/3)] 4. tanx, a = a/4; [tan(v/6)]
5 . cscx, a = v/4; [csc(v/6)] 6. 2", a = 0; [$5]
7. x sin x, a = ~ / 2 ;[(sin 0.1)/10] 8. x21nx, a = 1 ; [ e 2 ]
9. ex +
e2', a = 0;[e e 2 ] +
10. Use Taylor polynomials to approximate fiaccurate 12. ( a ) Determine the nth Taylor polynomial for f ( x ) =
through four decimal places. 1 / G a t a = 0.
11. Expand the polynomial f(x) = x4 + 2x3 + 2 x 2 + ( 6 ) Use the result of part ( a ) to estimate (0.9)-lI2
4x + 1 in powers of x - 2. through six decimal places.
( b ) Use part ( a ) to compute f(2.1).

In Exercises I3 to 34, ( a ) determine the radius of convergence and ( b ) determine the interval of convergence for the given power series.

13. c(x
m

n=O
- 4)"

c
OE

n=O
( - l)%(x- 3)"
23"+l(n2 + 1) 22. c
n=o
(-1)n &(x
+ 1)
- 1)"

25.
n!xn
26. c ( - 1)"2n(x + 4)n
"=l 4"n2
n=l
30. ( - l ) n 3 n ( ~- 1)'"
n=O n= 1 n2

33-
5
n=O
( - 1)"n!2"xn+l
4n++ +
2)!

In Exercises 35 to 50 using any appropriate technique, determine a Maclaurin series representation for the given function. Indicate the open interval on
which this representation is valid.
cos(ax)
35. x arctan(2x) 36. x2e-x/3 37. xsin4x2 38. -
X

39. l/?GT 40. d m 41. x2cos 6 42.


(1
X

+ x2)2
546 CHAPTER 16: POWER SERIES AND TAYLOR SERIES

x2 + 1
43. -
x - 1
44. x z sin(3x2) 45. @rTT 46. ln(2 + x2)
47. F ( x ) = fd"
1 - t4 48. F ( x ) = 4'"" +
3 t3

49. F ( x ) = Jxt 2 arctan(4t2) dt 50. sinh(x2)


0

In Exercises 5 1 to 5 4 approximate (as a j n i t e numerical sum) the given quantity to within the indicated degree of accuracy.

54. sl"
0
x 2 sinh x dx

Miscellaneous Exercises
1. Show that iff(.) has a continuous ( n + 1)st derivative on
an open interval containing a, then interval containing a and x. If F ( x ) = J E f ( t ) dt, then
show that the nth Taylor polynomial for F ( x ) is given by
limf(X) - P,(X) = 0
P,(x) = J-; P,(t) dt.
r-u ( x - a)"
3. Compute P,(x) forf(x) = ( b + x)" where m is a n integer
2. Suppose P,(x) is the nth Taylor polynomial forf at a . a n d b a constant, and compare the coefficients with the
where f has a continuous nth derivative on an open usual binomial coefficients.

In Exercises 4 to 14, ( a ) determine the radius of convergence, and ( b ) the interval of convergence for the given power series.
x

2n+p

6. 2
n=O
(-
n!(n + p ) ! (5) ( p a positive integer)

10. cr

n=O
b,xn where b, =
n

2- .

k=O 2k
.
I

n + 1
n=l

In Exercises 1 5 to 19 determine a power series representation for each ofthe given functions and giLe the open interval on which this representation is
valid.
cos2 x
15. -
X
16. F ( x ) = i x l n ( = ) d t 17. F ( x ) = J z d n d t 18. arcsinhx
2-t 0

19. F ( x ) = l ' l n ( 3 + 2 t ) dt
In Exercises 20 to 23 approximate the following to within the indicated degree of accuracy.

21. d 1 : 2 ~ a r c t a n ( x 2 ) d x (lop4)

22. 4' xZe-"' dx

24. Let i be such that i2 = - 1 and use the Maclaurin series


for ex to show Euler's formula eis = cos 8 i sin 8. +
SECTION 16.4: MISCELLANEOUS APPLICATIONS OF POWER SERIES 547

SELF-TEST + - ~~

1. Find the interval of convergence of each of the following series.

(1 C
Jc (-I)%(.
2n(n
+
+ 1)
(b) 2
n=l
(- l ) n 3 n ~
+
n2 2n
n=l

2. Find Maclaurin series forf(x) = x i ( 5 + x ) valid


~ for 1x1 5 . < I

3. Use the first four terms of an appropriate power series to approximate

- a \m&. How accurate is this approximation?


Chapter 17

549
550 CHAPTER 17: VECTORS AND SOLID GEOMETRY

Section 1 7.1 : Coordinates in Three-Space

Objectives: CARTESIAS COORDISATES

1. Plot points in Cartesian In Chap. 1 we introduced rectangular coordinates and noted the one-to-one
coordinates. correspondence between points in the plane and ordered pairs of real numbers.
This coordinate system has been a n indispensable tool in our study of the
2. Recognize the equation of a calculus to date. To further this study we shall introduce a three-dimensional
sphere. coordinate system that will allow us to identify points in space with ordered
3. Determine the distance between triples of real numbers in a similar way.
two points in space. We start with a horizontal plane which has the usual xy rectangular
coordinate system positioned so that the positivey axis extends to the right and
4. Determine the midpoint of a line the positive x axis extends toward us; see Fig. 17.1(a). A third real number line,
segment joining two points in called the z axis, is placed perpendicularly through the origin of the xy plane
space.
with its positive direction upward and its zero point at the origin of the xy
5. Recognize the equation of a plane. We refer to these intersecting number lines arranged in this fashion as the
vertical or horizontal plane. Cartesian coordznate system. If you were to grasp the z axis with your right hand
and curl your fingers from the positive x axis to the positivey axis, then your
thumb, if extended, would point in the direction of the positive z axis, as in Fig.
17.1(b). For this reason our system is often referred to as a rzght-handed system.
Each pair of axes determines a coordinate plane. These three mutually per-
pendicular planes are called the xy plane, the x-z plane, and they-z plane.
If P is a point in space, then the unique plane through P parallel to they-z
plane intersects the x axis at some point with one-dimensional coordinate xo. We
refer to this number as the x coordznate of the point P.In a similar way one
obtains they coordznateyo and the z coordinate zo by considering the intersection
z with they and z axes of the planes through P parallel, respectively, to the x-z

t plane and the x y plane. We thus label the point P as (xo,yo,t o and ) refer to this
triple of real numbers as the Cartesian coordinates of P (see Fig. 17.2). With this
convention, points on the x axis are of the form ( x , 0,O);those on they and z
axes have the form (O,y,0) and (0,0, z ) , respectively.
Conversely, any ordered triple (xo,yo,zo) of real numbers uniquely deter-
mines a point P as the intersection of three planes: the plane through (xo, 0,O)
parallel to they-z plane, the plane through (O,yo,0) parallel to the x-z plane,
and the plane through (0, 0, zo) parallel to the xy plane. Figure 17.3 shows the
location of some typical points in space.
In light of this identification we refer to the set of all ordered triples of real
numbers as Carteszan three-space. If the point P has Cartesian coordinates (x,y,z ) ,
then we will often write P = ( x , y , z ) .

i -2, 4, 2

-J

b
J 01 0.0
I
X
i 2 . -3, - 3 , I

Fig. 17.1 Fig. 17.2 Fig. 17.3


SECTION 17.1: COORDINATES IN THREE-SPACE 55 1

The three coordinate planes divide Cartesian three-space into eight subre-
gions called octants. Thejrst octant consists of those points with nonnegative x , y ,
and z coordinates. There is no standard numbering for the remaining octants.

THE GRAPH OF A N EQUATION


17.1.1 Definition The graph of an equation in x , y , and z consists of all those points
whose coordinates satisfy the given equation.

It is not difficult to determine whether a given point is on the graph of a


particular equation since one need only check that the coordinates satisfy the
equation. For example, the point (2, 3,8) is on the graph of x 2 + y 2 - z = 5
+
since (2)2 + (3)2 - ( 8 ) = 4 9 - 8 = 5 ; but the point ( 1 , 2, - 1 ) is not on the
+
graph since (1)2 (2)2 - ( - 1) = 6 # 5 . The more general problem of
sketching the graph of an equation F ( x , y , z ) = 0 will be dealt with in more
detail in the next section and in subsequent chapters.
The context plays an important role in graphing equations. For example, in rectan-
gular coordinates, the equation x = y has as its graph a straight line through
the origin; see Fig. 17.4(a). In Cartesian coordinates we interpret the equation
x = y as a n equation in x , y , and z where the t variable does not appear. A point
with coordinates ( x , y , z ) will be on the graph if and only if x = y, and hence z
can be any real number. The graph is a plane perpendicular to the xy plane
passing through the line in the xy plane with rectangular equation x = y ; see
Fig. 17.4(b).
An even simpler class of equations is of the form x = k , y = k , or z = k ,
whose graphs are planes parallel to the coordinate planes (see Fig. 17.5). As we

!b:

2 2

4
t

-Y

Fig. 17.5
552 CHAPTER 17: VECTORS AND SOLID GEOMETRY

shall see in the next section the chore of graphing a more complex equation can
often be simplified by examining its intersections with planes of this type.

THE DISTANCE FORMULA


17.1.2 The Distance Formula If Po = (xO,yO,zO) and Pl=
(xl,yl,zl) are points in space, then the distance between them is given by

d P 0 , PI)= d ( x 1 - .Ol2 + (y1 -Y0l2 + (21 -


Prooj If both points lie in a plane parallel to one of the coordinate planes, then,
I as was the case for the distance formula in the plane, the Pythagorean theorem
plane x = xo applies directly. Figure 17.6 depicts the situation when both points lie in the
Fig. 17.6 plane x = xo. Now

so

/7--------- .1
d P 0 , Pl)= dbJ1- Yo)2 + (21 - %)2
which gives the result since x1 - xo = 0 in this case.
If Po and PI do not lie in a plane parallel to one of the coordinate planes,
zo I then we consider the rectangular parallelepiped (box) with opposite vertices Po
1x1 - and Pl, as in Fig. 17.7. Applying the Pythagorean theorem to the triangle PoSQ,
we see that PoQ has length d(xl - xo)2 + (yl- yo)2.Now apply the theorem
again to the triangle PoQPl to obtain

d(P0, PI) =
=
d [ d ( q- .Ol2 + (A-Y0)212 + ( 1 %
4 x 1 - x0l2 + (Yl -Yo)2 + (21 - z0l2 .
- zol)2

Example 1 Find the distance between the points Po = ( - 1, 2,3) and Pl = (3, - 2,4).
Solution d(P0, PI) = d(3 - ( - 1))* + (-2 - 2)2 + (4 - 3)2 = 0

Example 2 Show that the points P = (3, - 1,2), (2 = (4, 1,6), and R = (5,3, 10) are
collinear.

z
Solution The points are pictured in Fig. 17.8. It suffices to show that d(P, Q) +
d(Q2,R)= V , R )
f which is the case since

ZIn
I ,

= ( 5 , 3, 101
d(P, Q) = d(4- 3)' + (1 - (-1))' + (6 - 2)2 = fi
+ (3 - 1)2 + (10 - 6)' = fi
d ( Q , R ) = d(5 - 4)'
P(P, R ) = d(5 - 3)2 + (3 - ( - 1))2 + (10 - 2 ) 2 = 2 fi 0

THE SPHERE
17.1.3 Definition A sphere ofradius r ( r > 0) and center (a, b, c) is the set of all points
whose distance from (a, b, c) is r.
*/
Fig. 17.8 As an immediate consequence of the distance formula (see Exercise 21) we
have:

17.1.4 The Equation of a Sphere Let r >


0. The sphere of radius r and center
(a, b, c) has equation ( x - a)2 + ( y - b)2 + ( z - c ) ~= r 2 . Conversely, every
such equation has as its graph a sphere.
SECTION 17.1: COORDINATES IN THREE-SPACE 553

Example 3 +
Show that x 2 + y 2 + z 2 4x - 2y + 62 + 5 = 0 is the equation of a sphere.
Determine its center and radius.
Solution Our objective is to show that this equation can be written in the form (1 7.1.4).
To do this we first regroup the terms ( x 2 4x ) + +
( y 2 - 2y ) +
+
( z 2 62 ) = -5; complete the square in x, y , and z ; ( x 2 4x + + +
4)
(y' - 2y+ + + +
1) ( z 2 62 9) = -5 4 + + +
1 9; and rewrite the term as
(x + +
2)' + +
( y - 1)2 ( 2 3)2 = 9.
It follows that the equation has as its graph a sphere of radius 3 centered at
(-2,1, -3). 0

As was the case for circles in the plane, it is not difficult to see that any
second-degree equation of the form
Ax2 + By2 + Cz2 + Dx + Ey + Fz + G = 0 with A = B = C (#O)
+
can be written in the equivalent form ( x - u ) ~ ( y - b)' +
( 2 - c ) ~= k and
<
thus will either have no graph ( k 0) or will have as its graph a sphere ( k 0) >
or a single point ( k = 0).

THE MIDPOINT OF A LINE SEGMENT


Using a n argument similar to the two-dimensional case it is routine to derive:

17.1.5 T h e Midpoint Formula The midpoint of the line segment joining


Po = (xo,yo,zo)and PI = (xl,yl,zl) has coordinates

Example 4 Give an equation for the sphere with diameter PQ, where P = ( - 1,2,4) and
Q = (3, 136).
Solutions The radius of the sphere is
7 = (1/2) d(P, Q) = ( 1 / 2 ) d ( 3 - (-1))' + (1 - 2)' + (6 - 4)' = fi/2
The center C of the sphere is located at the midpoint of the diameter

c=(-1+3
2 '-
2+1
2 ' -)4 +2 6 = (1,3/2,5)

From (17.1.4) a n equation for this sphere is

(x-1)2+
(y - - 3 + ( z - 5 ) 2 = 2 14 .
PROGRESS TEST 1

1. Draw a Cartesian coordinate system and plot the 3. Identify the graphs of the following equations:
points P = (-2, 1,3), Q = (1, -3, -5) and ( a ) x' f y ' + z2 - 4x + 6y - 22 + 14 = 0.
R = (3, 2 , 1). ( b ) 4x2 + 4y2 + 4 t 2 - 4x - 24y + 82 + 5 = 0.
2. Show that the points P = (-2, 3,4), Q =
(1, - 1,5), and R = (-4, 2, 6) are the vertices of
a right triangle and find its area.
554 CHAPTER 17: VECTORS AND SOLID GEOMETRY

SECTION 1 7 . 1 EXERCISES

In Exercises 2 to 7, P and Q are given to be the opposite vertices o j a rectangular parallelepiped with faces parallel to the coordinate planes. Gzve the
rernainzng vertices.
2. P = (2, 1, 6), Q = (3, 2,4) 3. P = ( - 3 , 2 , 1 ) , 4 = ( 4 , - 6 , 2 )
4. P = (4, - 1, 2), Q = ( 3 , 0 , 9 ) 5. P = (-2, -1, -2), Q = ( 0 , 4 , 1 )
6. P = (3, -1, -2), Q = (-1, - 2 , 3 ) 7. P = (-4, - 1, l ) , Q = (4, 6, -8)
8. Determine the length and midpoint of each of the diag-
onals PQ in Exercises 2 to 7 .

In Exercises 9 to 12 determine whether the three points are collinear.

In Exercises 13 to 16 determine which of the triangles having the given vertices 4 Q R are right triangles.
13. P = ( 1 , 2 , 8 ) , Q = (2, 1, 3), R = ( 1 , 4 , 5 ) 14. P = (1, -1,2), Q = (3, -1,6), R = (-2, 1, 3)
15- P = ( 5 , 0 , - 3 ) , 9 = ( 2 , - 1 , 4 ) , R = ( 6 , 4 , - 2 ) 16. P = (0,1, 2), Q = (2, 1, 3), R = (1, 3,O)

In Exercises 17 to 20 identi& the graphs (zj they exist) ojthe given equations.
17. x 2 - 2 X + y 2 + +
4~ '2 - 82 = 123 +
18. x 2 + y 2 '2 - x 2y + +
6z = 23/4
+ +
19. 3x2 3y2 3z3 + 6y - 122 = -39 20. X' +y' +
t 2 - 122 = -38
2 1. Prove Theorem (17.1.4). 22. Prove Theorem (17.1.5).

Sketch the graph in Cartesian three-space of each of the following equations.


23. x = -2 24. y = 3
2 5 . z = 3/2 26. 3x + y = 1
27. x - 2 ~ ~ 4 28. 2y +
3z = 5

Section 17.2: Surf aces

Objectives: INTRODUCTION
Identify and sketch the graphs of By a surface we shall mean the graph of any equation in three variables. We shall
equations of (1) planes, write such a n equation symbolically as F ( x , y , t ) = 0. The purpose of this
(2) cylinders, (3) surfaces of section is to establish certain basic surfaces and their equations, namely, planes,
revolution, and (4) quadric cylinders, surfaces of revolution, and the quadric surfaces. Later we shall
surfaces. frequently use these surfaces as a source of examples, so it is crucial that you be
able to recognize and sketch them. We shall assume that you are familiar with
the equations in two variables for the conic sections (see Chap. 10).

SYMMETRIES
Symmetries are even more useful in graphing surfaces than in graphing curves.
In addition to symmetries with respect to the origin and the axes, we are also
concerned with symmetries with respect to the coordinate planes. Table 17.1
summarizes the various types of symmetries possible for two points P and Q:
SECTION 17.2: SURFACES 555

I 11

(a) origin Q = ( - a , -b, -c)


( b ) x axis Q = (a, -b: -c)
( c ) y axis Q = ( - a , b, -6)
( d ) z axis Q = (-0, - b , c )
( e ) x l y plane Q = ( a , b, -c)
(j) x-z plane Q = (a, -b, c)
( 9 ) y - t plane Q = (--a, b, c)
Table 17.1 Symmetries Let P = (a, 6 , c). We say P and Qsatisfy the symmetry indicated in
column I if and only if Q has the coordinates indicated in column 11.

symmetry with respect to the origin, one of the coordinate axes, or one of the
coordinate planes. Some illustrations appear in Fig. 17.9.
A surface S is said to be symmetric with respect to a line L if and only if for
every point P on S,the point Q that is symmetric to P with respect to L is also on
S.A similar statement holds for the symmetry of a surface with respect to a
point or a plane. Using Table 17.1 it is easy to state necessary and sufficient
conditions for the graph of a n equation F ( x , y , t ) = 0 to obey a particular
symmetry. For example, the graph of F (x,y,z ) = 0 will be symmetric to the x y
plane if and only if F ( x , y , z ) = F ( x , y , -2). That is, the equation remains
unchanged when t is replaced by -2.

Example 1 Identify the symmetries of the graphs of the given equation.


( a ) x2 - 2y + y 3 - 6 = 0
(b) 2 = d w
Solutions ( u ) If we replace x by - X in equation (a), the equation is unchanged. From
Table 17.l(g) it follows that the graph of this equation is symmetric with
respect to they-t plane. None of the other symmetries holds.
( b ) Replacing either x by - x , ory by -y, or both, will not change equation
( b ) , and hence the graph is symmetric with respect to t h e y - t plane, the X-t
plane, and the z axis. None of the other symmetries holds. 0

SKETCHING SURFACES
In general the intersection of two surfaces will result in a curve. Of particular
help in sketching the graph of an equation are its curves of intersection with

t Q- A -a.b,cl

la; P, Qsymmetric with , b! P, Q s y m m e t r i c with


respect to t h e j - r plane respect to the-r axis
Fig. 17.9
558 CHAPTER 17: VECTORS AND SOLID GEOMETRY

either the coordinate planes or planes parallel to the coordinate planes. We refer
to these curves as traces of the surface. We begin with the simplest type of surface,
a plane.

17.2.1 Definition An equation of the form Ax + By + Cz + D = 0, where at least


one of the coefficients A , B , C is not zero, is called a linear equation in the variables x,
y, and z.

In Sec. 17.5 we shall show that the graph of a linear equation is a plane, and
conversely, every plane has a n equation of this form. In this section let us
assume this and concentrate on graphing such equations.

Solution Lettingy = z = 0, we see that the plane intersects the x axis at the point
(6,0,0). Similarly, they and z intercepts are ( 0 ,9 ,0 ) and (0, 0, 3). Joining these

We now examine a slightly more complex example.


Z

4 /

0)

+ z = 6

J'

k x
Fig. 17.11
SECTION 17.2: SURFACES 557

c 4

t t

J
X
J r = k

Fig. 17.12

Example 4 Sketch the surface t= d m by first examining its traces.


Solution We noted in Example l ( b ) that the graph will be symmetric with respect to the

i x-z andy-t planes. It is clear that z

If t = 0, then we have d q
2 0, and hence it suffices to sketch the graph
in the first octant and then use the symmetric properties to complete the sketch.
= 0, which implies x = y = 0. Thus the trace
in the x y plane is the single point ( 0 ,0,O).If z = k > 0, then v m
= k is
+
equivalent to x 2 y 2 = k2. This is the equation of a circle of radius k centered
at the origin. Since we are in the p l a n e t = k , the origin in this case means
the point (0, 0, k ) . (See the first part of Fig. 17.12 above.)
The trace in the x - t plane ( y = 0) is 2 = 1x1. In the planesy = k parallel to
the X-z plane, we have z = d m , which is the upper branch of the
hyperbola t 2- x 2 = k2. Similarly, in they-t plane the trace is t = Iy 1, and in
the planes x = k we obtain the upper branch of the hyperbola z 2 - y 2 = k2.
J The resulting surface is the upper nappe of a right circular cone. See Fig. 17.12
X for the remaining first octant traces. The graph is sketched in Fig. 17.13. 0
Fig. 17.13

PROGRESS TEST 1

1. Sketch the graph of the equation x2 + z 2 = 4y. Note


relevant symmetries and traces.
2. Sketch the graphs of the planes with equations
+
( u ) 3~ - 2,~ 22 = 6
+ +
( b ) - 2 ~ 3y t = 0

CYLINDERS
17.2.2 Definition Suppose C is a curve that lies in a plane n,
with L a
n
line not parallel to that passes through some point on C. (See Fig. 17.14.) The
collection of all points on all lines through C and parallel to L is called a cylznder.
The curve C is called the dzrectrix of the cylinder. The line L and all lines
through C parallel to L are called rulzngs or generators of the cylinder.

Cylinders with the directrix in one of the coordinate planes and rulings
Fig. 1 7 . 1 4 perpendicular to the plane of the directrix are particularly easy to recognize.
558 CHAPTER 17: VECTORS AND SOLID GEOMETRY

Example 5 Sketch the surface S with equation x 2 + z2 = 9.

circle in the x-z plane of radius 3 centered at the origin, then we know
P = (x,y, z ) is on S if and only if ( x , 0, t)is on the circle C. Thus S is a cylinder

17.2.3 Theorem The graph of a n equation F ( x , y ,z ) = 0 where one of the varia-


bles does not appear is a cylinder. The rulings of the cylinder are parallel to the
axis of the absent variable. The directrix lies in the plane of the variables that
do appear in the equation and is the graph of F ( x , y ,z ) = 0 considered as a n
equation in two variables.

Example 6 Graph the surfaces with the following equations.


(a) y = 2x2 (b) x2 2x + + 4 . 2 = 15
Solutions ( a ) The directrix is the parabola in the xy plane with equationy = 2 2 . The
surface is a parabolic cylinder with rulings parallel to the t axis; [see
Fig. 17.16(a)].
( b ) We complete the square and rewrite equation ( b ) as ( x + 1)2/16 +
z2/4 = 1. In the X-z plane this is the equation of a n ellipse. The surface is
a n elliptic cylinder with rulings parallel to they axis; see Fig. 17.16(b). 0

SWRFACES OF REVOLUTION
A surface obtained by revolving a plane curve C about a line L in the plane of
the curve is called a surface of revolution. The line L is called the axis of the
surface of revolution generated by the curve C (Fig. 17.17). The intersection of
a surface of revolution with a plane perpendicular to its axis is always a circle.
J = 2X2
Suppose the plane curve z =f(y) in they-z plane is revolved about they
axis to generate a surface of revolution S; see Fig. 17.18. Let P = (x,y, z ) be an
arbitrary point and consider the plane through P perpendicular to they axis.
J

L
I
Fig. 17.16 Fig. 17.17 Fig. 17.18
SECTION 17.2:SURFACES 559

Plane of Equation of Axis of Equation of


Generating Curue Generating Curue Revolution Surface of Revolution

x axis

y axis

z axis x2 + y2 = [h(2)]2
z
Table 17.2 Surfaces of Revolution

t
The circular cross section of S with this plane has its center on they axis a t
R = (O,y,0).The point Q = (O,y,f( y ) )in they-z plane is on this cross section.
-+I Now P will be on the surface if and only if it is on this circular cross section,
+
that is, if and only if IPRI2 = IQRI'. But lPRI2 = x 2 z 2 and IQRI2 =
[f ( y ) I 2 .Thus P is on S if and only if x 2 + y 2 = [f ( y ) I 2 .
A similar equation can be obtained for any other surface of revolution
whose axis of revolution is one of the coordinate axes. These results are sum-
marized in Table 17.2. Note that in each case the equation ofthe generating curve is
Fig. 1 7 . 1 9 expressed as a function of the variable associated with the axis of revolution.

Example 7 Determine a n equation for the surface obtained by revolving the curve
y = In x in the x y plane about they axis; see Fig. 17.19.
Solution Since the axis of revolution is they axis, we solvey = In x for x in terms ofy,
getting x = e u . Now from ( c ) in Table 17.2 we obtain x 2 + z 2 = e2u as the
desired equation. 0

Example 8 Identify and sketch the surface with equation 9 x 2 + y 2 + z 2 = 36.


Solution We write the equation in the formy2 + z 2 = 36 - 9 x 2 = [f ( x ) I 2 , wherey =
f ( x ) = 3 d-. The graph o f y = 3 d m in the x y plane is the upper
half of the ellipse x 2 / 4 + y 2 / 3 6 = 1 ; see Fig. 17.20(a). Thus the surface is the
surface of revolution obtained by revolving this semiellipse y = 3 d m
about the x axis; see Fig. 17.20(b). 0

iJ'

a)
Fig. 1 7 . 2 0
560 CHAPTER 17: VECTORS AND SOLID GEOMETRY

We refer to the graph of the equation in Example 8 as an ellipsoid of


revolution. It could also have been interpreted as the surface obtained by revolv-
ing the semiellipse z = 3 d m in the x-z plane about the x axis. Many
surfaces of revolution can be generated in more than one way.

PROGRESS TEST 2

Identify and sketch each of the following surfaces:


1. z = l n x
2. x2 - 4y2 = 16 4. Find a n equation for the surface obtained by re-
3. 3x2 + 3y2 = 122 volving the curve z 2 = x 3 about the z axis.

QUADRIC SURFACES
The graph of a second-degree equation in x,y, and z is called a quadric surface.
We shall not attempt a complete discussion of quadric surfaces, but rather shall
consider those quadric surfaces that are positioned symmetrically with respect
to the coordinate axes and have relatively simple equations. Of course, using
the material already developed in this section one can recognize the graph of a
second-degree equation that is a cylinder, surface of revolution, or a sphere. We
assume in the following discussions that a, b , and c are positive constants.

THE ELLIPSOID
T h e graph of the equation
x2 y2 22
17.2.4 -+,+-=1
a2 b c2
is called an ellipsoid (Fig. 17.21). The ellipsoid has all the symmetries noted in
Table 17.1. Each trace in the respective coordinate planes is a n ellipse, as is each
trace in the planes x = k, (lkl < a), y = k (lkl < b ) and z = k (Ikl <
c).
If any two of the constants are equal, then the ellipsoid is a n ellipsoid of
+ +
revolution. For example, x 2 / a 2 y 2 / a 2 t 2 / c 2 = 1 can be written as x 2 +
y 2 = [(a/c)d 1
- 2, which is the surface obtained by revolving the semi-
Fig. 17.2 1 (computer graphics
for Figs. 17.21 to 17.28 b y Dr. ellipsey = ( a / c ) d m in the y-z plane about the z axis. If a = b = c, then
L e e Estes, Southeastern Massa- the ellipsoid is a sphere.
chusetts University).

THE HYPERBOLOID OF ONE SHEET


The graph of the equation

17.2.5

is called a hyperboloid of one sheet; see Fig. 17.22. It has all the symmetries of Table
17.1. The trace in the x y plane is the ellipse x 2 / a 2 + y 2 / b 2 = 1 and the traces in
the x-z and y-z planes are the hyperbolas x2/a2 - z 2 / c 2 = 1 and y 2 / b 2 -
z 2 / c 2= 1, respectively. The traces in the planes z = k yield ellipses that increase
in size as Ikl increases, The traces in the planes x = k andy = k are hyperbolas.
If a = b , then the surface is a hyperboloid ofrevolution, obtained by revolving the
Fig. 17.22 hyperbola x 2 / a 2 - z 2 / c 2 = 1 about the z axis (the conjugate axis).
SECTION 17.2: SURFACES 561

Equations of the form x 2 / a 2 . - y2/b2 + z 2 / c 2 = 1 and -x2/a2 +y 2 / b 2 +


z 2 / c 2 = 1 also represent hyperboloids of one sheet, which envelop they and x
axes, respectively, rather than the z axis as does (17.2.5).

THE HYPERBOLOID OF TWO SHEETS


The graph of the equation

17.2.6 -x2_ - -y2- 22


=1
a2 b' c2

is called a hyperbolozd of two sheets; see Fig. 17.23. The traces in the planesy = k
and z = k are hyperbolas. There are no traces in the planes x = k if Ik 1 a, and <
the traces in the planes x = k when lkl >
a are ellipses that increase in size as
/kl increases.
If b = c, then the equation is that of a hyperboloid ofrevolution, obtained by
Fig. 17.23
revolving the hyperbola x 2 / a 2 - z 2 / c 2 = 1 about the x axis (the transverse
axis).

+
The equations - x 2 / a 2 - y 2 / b 2 z2/c2 = 1 and - x z / a y2/b2 - +
z z / c 2 = 1 are also hyperboloids of two sheets. The two separated pieces are
situated on the z axis for the former and on they axis for the latter.

THE ELLIPTIC CONE


The graph of the equation

17.2.7 -x2+ - - -y2= o 22

a2 b2 c2

is called an elliptic cone; see Fig. 17.24. The trace in the x y plane is the origin,
whereas the traces in the planes z = k are ellipses that increase in size as 1 k 1 does.
The traces in the planes x = k a n d y = k are hyperbolas that reduce to inter-
secting straight lines in the coordinate planes x = 0 and y = 0.
If a = b, then the cone is called a right circular cone, and it can be obtained by
Fig. 17.24 revolving the line x = (a/c)z in the x-z plane about the z axis. The equations
x 2 / a 2 - y 2 / b 2 + z2/c2 = 0 and - x 2 / a 2 + +
y 2 / b 2 z 2 / c z = 0 are also elliptic
cones, which envelop they and x axes, respectively.

THE ELLIPTIC PARABOLOID


The graph of the equation
z
17.2.8 -x2+ - =
y2
-
a2 b2 c
is called an ellzptzc paraboloid see Fig. 17.25. The trace in the planes z = k are
ellipses if k > 0. No trace exists if k < 0. The traces in the planes x = k and
y = k are parabolas. The graph is symmetric with respect to the X-z and y-z
planes.
If a = b, the equation is that of aparaboloid of revolutzon obtained by revolving
+
the parabolay2 = ( a 2 / c ) z about the z axis. The equations x 2 / a 2 z 2 / b 2 = y / c
Fig. 17.25 and y 2 / a 2 + z 2 / b 2 = x / c are also elliptic paraboloids, which open about the
positivey and x axes, respectively. If in the above discussion we allow c to be
negative, then the graphs will open about the appropriate negative coordinate
axis.
562 CHAPTER 17: VECTORS AND SOLID GEOMETRY

THE HYPERBOLIC PARABOLOID


The graph of the equation

17.2.9

is called a hyperbolic paraboloid; see Fig. 17.26. The graph is symmetric with
respect to the x-z and y-z planes. The trace in the X-z plane is a parabola
opening upward, whereas the trace in the y-z plane is a parabola opening
downward. The trace in the x y plane is a pair of intersecting straight lines. The
traces in the planes z = k are hyperbolas with transverse axes parallel to x axis
if k> 0 and parallel to they axis if k <0. The traces in the planes x = k and
y = k are parabolas opening downward in the first case and upward in the
second.

Equations of the form x 2 / a 2 - z 2 / b 2 = y/c and y 2 / a 2 - z 2 / b 2 = x/c also


<
represent hyperbolic paraboloids, as do any of the above with c 0. Because of
their saddle shape, they are the most difficult of the quadric surfaces to sketch.
>
The three situations, with c 0, are sketched in the three parts of Fig. 17.26.

SGMMARY AKD EXAMPLES


Ellipsoids, hyperboloids, and cones are called central conics. In each of our
preceding discussions the center was the origin. Elliptic and hyperbolic para-
boloids are examples of noncentral conics. In our previous development the origin
is called the vertex of the elliptic paraboloid; for the hyperbolic paraboloid it
is called the saddle point. If in any of the equations (1 7.2.4) to (17.2.9) or their
variations the variables x,y, and z are replaced, respectively, by x - h , y - k ,
and z - I, then the graph will be the same surface except translated so that its
center (vertex or saddle point) is located at the point ( h , k , 1).

Fig. 17.26

Example 9 Identify and sketch the graph of


( a ) 9x2 - 16y2 + z 2 - 144 = 0
+
( b ) x2 - 2x + 4y2 - 16y 1 = 162
Solutions ( a ) Equation ( a ) can berewritten asx2/16 -y2/9 +
z2/144 = 1,which has the
form of (16.2.5) with z andy interchanged. Thus the graph is a hyperboloid
of one sheet; see Fig. 17.27 (left).
( b ) We first complete the square in x and y, obtaining ( x 2 - 2x + +
1)
+
4(y2 - 4y + 4) + 1 = 162 + 1 16 or, equivalently, ( x - 1)2/16 +
( y - 2)2/4 = z + 1. This has the form of (17.2.8) with x,y, z replaced by
x - l , y - 2 and z + 1, respectively. The graph is a n elliptic paraboloid
opening upward with vertex at (1, 2, - 1); see Fig. 17.27 (right). 0

Being able to sketch the various quadric surfaces in space will be a great aid
to your learning and using the calculus of several variables which follows. Of
course, the only way to learn is to practice. However, more general surfaces can
be extremely difficult to sketch. An example of such is

/sin(a/8)(x - 16)l
z=
/ X - 161
+ lsin(a/8)(y - 16)l
lY - 161
which is sketched in Fig. 17.28.
SECTION 17.3: VECTORS IN THE PLANE AND IN SPACE 563

Fig. 17.27 Fig. 17.28

PROGRESS TEST 3

Identify and sketch the graph o f


+
1. x 2 - 4y2 z 2 = 16 2. y2 - x2 = 2t 3. x 2 + '2 - 2~ - 22 - 36y + 38 = 0

SECTION 1 7 . 2 EXERCISES

In Exercises I to 30 identzfy and sketch the graph ofthe given equation.


1. 2~ - 4y = 6 11. 4 ( x - 1 ) 2 + y 2 + 2y+ 22 = 4 21. 5 x 2 -y2 - 522 = 10
2. x -2 +
, ~ 32 = 4 12. x2 - 4 9 = 4y2 22. x2 - y / 3 + 22 = 0
3. x = t 13. x2 +y - (z - 114 = o 23. 3x2 + y 2 - 6z2 = 36

-
4. 3y2 + 3z2 - [ h ( x - I)]' = 0 14. 1x1+ lzl = 1 24. x* + 3y2 - 922 = 0
a. 15. x2 - 5 , + 2y2 + 4y = 2 25. 2 x 2 - 4y2 = 82

6. z-4x=l 26. x2 + y * + 5y = 22
7. x 2 - 82 = 0 17. 2x +
4y - 82 = 0 27. -y2
x2 - t 2+ 4 x
- 6y = 9
8. 5 x 2 + 25y2 +
102' = 50 18. x2 +
z 2 = /yl 28. + y 2 - 2y + 22 - 22 = 0
x2
9. 2y2 - 4x2 - 1222 = 1 2 19. y = cosz 29. 2 2 2 + 2x2 - [arctany]* = 0

10. x 2 / 4 +22' = 8y 20. 4x2 - 222 +


4y2 = 0 30. z2 = X' + 4y2 - 2~ + 8~+ 42

In Exercises 31 to 35 determine an equation for the surface obtained by revolving the given plane curve about the indicated axis.
31. y = 3 x , y axis 32. y = m , y axis 33. x 2 - t 2 = 1, z axis 34. y 2 = z 3 , z-axis
35. y = ln(t - l ) , y axis 36. Find an equation for the torus obtained by revolving circle ( x - 2)* + t 2= 1 about the t axis.

Section 17.3: Vectors in the Plane and in Space

Objectives: ISTRODKCTION
1 . Add, subtract and form scalar Many physical quantities, such as velocity, acceleration, force, or momentum,
multiples of vectors. possess both a magnitude and a direction. Such quantities are called vector
quantities or vectors and are quite different from such quantities as mass or
2. Interpret the operations of
temperature, which can be completely described by means of a real number.
addition, subtraction, and scalar
These latter quantities are usually referred to as scalar quantities.
multiplication of vectors
Vectors of the type mentioned above can be represented either geometrically
geometrically.
or analytically. The geometric approach has the advantage of visual clarity and
is very intuitive. However, actual computations with vectors are more readily
done using an analytic representation. Consequently, we shall discuss both
approaches. Our emphasis will be on the analytic approach, but we shall use
the geometric interpretation for illustrative purposes and in our applications.
564 CHAPTER 17: VECTORS AND SOLID GEOMETRY

PLASE VECTORS
i' If P and Q are distinct points in the plane, then t 3 d i r e c t e d line segment from
P to Q is called a geometric vector. We denote it by PQ and refer to P as the initial
point (also base or tail) of the geometric vector and Qas the termznalpoint (or head).

-
Vectors can be represented by means of geometric vectors. For example, a force of
50 lb in the northeasterly direction applied to a n object located at P can be
represented as in Fig. 17.29, where PQ has length 50 and points toward the
northeast. The s a s e force applied to a point R would be represented by a

-
geometric vector RS, which also has length 50 and points to the northeasLWith
this in mind we make the convention of identifying geometric vectors PQ and
I East + 4 -
RS that have the same length and the same direction, and we write P Q = RS.
Fig. 1 7 . 2 9
In print we usually denote a vector by a single boldface letter such as A
+ +
(when writing by hand we instead write A ) . If P Q is a geometric vector which
+ j
represents A, then we refer to PQ as a representative of A and write A = PQ.
Figure 17.30 shows several representatives of a vector A. T h e particular repre-
sentative of A that has its base at the origin is called the standard representa-
I
tive of A.
Using standard representatives, we can define a one-to-one correspondence
i
between vectors and points in the plane by letting A correspond to the point R,
which is the h e a 2 f its standard representative. In particular, if A has standard
i representative OR, where R = (al,a,), then the representative of A with base
--$
P = ( x , y )will be PQ, where Q = ( x a,,y + +
a,); see Fig. 17.31. Note that the
standard representatibe length of this vector is v'W. If a is the angle of inclination with the
of A
positive x axis (0 2 a < 2 ~ ) then
,

cosa = a1 and sina =


a2

Fig. 1 7 . 3 0 That is, the length and direction of A are completely determined by the numbers a, and a,,
the coordinates of the head of its standard representative. This observation
leads to our analytic definition of a vector.

Definition A two-dimensional (plane) vector is an ordered pair of real numbers


(al,a,). The numbers al and a, are called respectively, the x andy components or
coordinates of the vector. Two vectors are equal if and only if they have the same
x component and the samey component. That is, (a,, a,) = (b,, b,) if and only
if al = b, and a, = b,. The collection of all plane vectors is denoted by R,.

As noted, we shall use boldface letters to denote vectors. We usually use the
same letters (though always lowercase) with the subscripts 1 and 2 to denote the
respective components. Thus we write A = (al,a,), B = (b,, b,), and so on.
The use of pointed parentheses to denote vectors allows us to maintain a useful
distinction between points in the plane and vectors.
a1

Fig. 1 7 . 3 1 17.3.2 Definition The vector (0,O) is called the zero vector and is denoted by 0.

17.3.3 Definition The length of a vector A = (al,a,), denoted IlAIl, is given by

IIAII = m
11 All is also referred to as the magnitude or norm of A. If A # 0 , then the direction
of A is the unique angle a, 0 2 a < 2m, such that
SECTION 17.3: VECTORS IN THE PLANE AND IN SPACE 565

a1 a2
cos a = - and sin a = -
IIAII IIAII
We assign no direction to the zero vector.

In the future we shall simply refer to vectors using whichever interpretation


is most convenient. As we go along, new definitions concerning operations on
vectors will be given in terms of the components (that is, analytically), but we
-+ with the appropriate geometric interpretation. If A is given in the
shall follow
form PQ, then you will need to determine the components of A before applying
any of the definitions. To help in this we have:
4
17.3.4 Theorem If A = PQ, where P = (xl,yl) and Q = ( x z , y 2 ) ,then

A = ( x z - X l , Y 2 -A>
+
pro^ The idea is, simply, to check that OR, the standard representative of A
+ +
given in Fig. 17.32, has the s a s e length and direction as PQ. But 1IOR/I =
d(X2 - .112 + (Ya -Y1)2 = IlPQIl and

xz - x1
cos a1 = cos a z =
d(xz - + (Y2 -Y1I2
sina, = sina, = Yz -Y1
d(%- X 1 l 2 + (YZ - Y d 2
+ +
Thus OR and PQ have the same length and direction, and hence they are
equal.

4
Example 1 Let A = PQwhere P = (1,2) and Q = (-3, -2).
( u ) Determine the components of A.
( b ) D e t e r m i 3 the length and direction of A.
(G) Sketch P Q and the standard representative of A.

Solutions (u) By (17.3.4), a, = -3 - (1) = -4, u2 = - 2 -(2) = -4. Thus


A = (-4, -4).
( b ) By (17.3.3), IIA/I = d(-4)' + (-v=
m= 4fiand

Since 0 5 a 5 27~,we must have a = 5m/4.


(c) See Fig. 17.33. 0

4
I
t P = 1,2

0 = 0.O \ [ V
x2 - x1 (-4, -4)
Fig. 17.32 Fig. 17.33
566 CHAPTER 17: VECTORS AND SOLID GEOMETRY

PROGRESS TEST 1
-

1. Let A< ( -2, 3). If P = (1, -4), find Q such that


A = PQ. Sketch.
2. L e t P = (1, - 3 ) , Q = ( - 6 , 7 ) , R = (-4, -3).Find
+ - +
S such that PQ = RS.
3. Find the length and direction of A = 1, fi). <
OPERATIONS ON VECTORS
17.3.5 Definition Let A = (al, a,) and B = (b1, 6,) be two vectors. Then their sum
+
A B is defined by
A +B= (a1 + b1,a, + b,)
T h a t is, to add two vectors, simply add componentwise.

Example 2 Compute A + B when


( a ) A = ( - 1 , 0 ) , B = (3, 7).
-+ +
( b ) A = PQ,B = RS, where P = (1, - l ) , Q = (3,7), R = (-3,4), and
S = (8, 1).
Solutions (u) A + B = (-1,O) + ( 3 ,7 ) = (-1 + 3,O + 7) = (2,7)
( b ) Before applying (17.3.5) we must first use (17.3.4) to determine the compo-
nents of A and B.
+
A = PQ= ( 3 - 1 , 7 - (-1)) = ( 2 , 8 )
+
B = RS = ( 8 - (-3), 1 - 4) = (11, -3)
and thus
A +Bz ( 2 ,8 ) + (11, -3) = (13,5) 0

17.3.6 Definition If A = (al, a,), then the negative of A, denoted -A, is defined by
- A = ( --al, - a z ) . The dgerence A - B is defined by A - B = A + (-B).

we see that subtraction of vectors is also componentwise.

Example 3 Find A - B if A = (2, - 7) and B = ( -3,4).


Solution A - B = (2, -7) - ( - 3 , 4 )
= (2, -7) + (3, -4) = (5, -11) 0

The following properties of vector addition and subtraction are an immedi-


ate consequence of the corresponding properties of real numbers and the
componentwise nature of the definitions.

17.3.7 Theorem Let A, B, and C be vectors. Then


(a)A+B=B+A
(b) A + +
(B C ) = (A + B) + C
SECTION 17.3: VECTORS IN THE PLANE AND IN SPACE 367

(c) A + O = A
(d) A - A = 0
Pro6 We illustrate by proving (a). The proofs of ( b ) , ( c ) , and ( d ) are similar.

A + B = (a1, a,) + ( 4 ,b,)


=1'( + ' 2 + b,)
b,7

= ( b , + % b , + a,)
= (b1, b z ) + (a,, a,) =B +A w

When dealing with vectors it is traditional to refer to a real number as a scalar.

17.3.8 Definition If A = (a,, a,) is any vector and k is any scalar, then kA =
( k a l , ka,). We refer to kA as a scalar multiple of A.

An important relationship between the length of kA and the length of A is


given by the following:

17.3.10 Theorem Let A = (a,, a,) and let k be any scalar. Then
IIkAll = Ikl IIAll
Prooj

R = x + al + bl,y + a 2 + 5 , llkAll = I/(kai, kaz)lI


= d(k.1)2 +
(ka,)Z
= dk2(a12 a,,) +
= = Ik/ IIA/1 w
v'P~W
THE GEOMETRY OF VECTOR OPERATIOSS
A+)= x+al,j+a, Vector Addztzon: Let A = (a,, a,), B = (b,, b,), and let P = ( x , y )be any point in
+
the plane. The representative of A with base P is PQ, where Q = ( x al,y + a,); +
+
and the representative of B with base Q is QR, where R = ( ( x al) b,, + +
( y + a,) + b,); see Fig. 17.34. Now A + B = (al b,, a, +
b,), and so the +
+
representative of A + B with base P is PS, where S = ( x + (al + bl),
P = x,y y + +
(a, b,)). But x +
(al +
b,) = ( x a,) b, and y +(az +b,) = + +
Fig. 17.34 ( y + a,) + b,, so R = S. Thus we have:
568 CHAPTER 17: VECTORS AND SOLID GEOMETRY

A-B

A + B

B
B
A

P
Fig. 17.35 Fig. 17.36 Fig. 17.37

17.3.1 1 Tail-to-Head Addition A representative for A + B can be obtained by


placing a representative of B with its base at the head of some representative of
+
A and taking as the representative of A B the vector from the base of A to
the head of B.
Since representatives of A, B, and A + B comprise the sides of a triangle, we
can use the fact that the length of one side of a triangle is less than or equal to
the sum of the lengths of the remaining sides to obtain the following:

17.3.12

An equivalent realization of the vector sum can be obtained by taking


representatives of A and B with the same initial point P and then forming the
parallelogram with these representatives as adjacent sides (see Fig. 17.35). The
diagonal with base P i s then a representative of A + B. This is referred to as the
parallelogram law of addition.
If, in Fig. 17.35, A and B represent forces applied at P,then A + B is the
resultantforce. That is, the single force A+ B applied at P will have the same
effect as the two combined forces A and B.
Vector Subtraction: Since A - B is the vector that when added to B yields A, we
can use the tail-to-head method to obtain a geometric realization.
Fig. 17.38
17.3.13 Vector Subtraction If A and B have the same initial point, then the vector
from the head of B to the head of A is a representative of A - B. Thus A - B
points from B to A; see Fig. 17.36.
>
Scalar Multiplication: Since IlkAI/ = / k ( IlAIl, we know that if k 0, then kA
will have k times the length of A and the same direction; see Fig. 17.37. If k< 0,
then kA will have lkl times the length of A but the opposite direction; see Figs.
17.37 and 17.38 and the discussion on page 569.

PROGRESS TEST 2

1. LetA=(2,-4),B=(-3,1),C=(-2,3).De-
termine (A + B) + C, A - B, and C - A, and il-
lustrate each geometrically.
2. Let A = (-3, l ) , B = (4, -2), C = (-1, -1).
Determine
( a ) 3A +2B - C +
( b ) llClI(A B)
(c) 1 1 3-~ ~ B I I
SECTION 17.3: VECTORS IN THE PLANE AND IN SPACE 569

USIT VECTORS
17.3.14 Definition A vector of length 1 is called a unit vector.

17.3.15 Theorem Let A f: 0. Then

A is a unit vector having the same direction as A.


(a) /Al
( b ) A = IIAIl(cos a , sin a ) , where a is the direction of A.
Proof:
( a ) By (17.3.10) and (17.3.8),

( b ) This is an immediate consequence of (17.3.3). W

From the above it follows that if 1(A((= 1, then the x component of A is cos a
and they component of A is sin a , where a is the direction of A. For this reason
a unit vector is often referred to as a direction vector. In particular, we have:

17.3.18 Theorem: If A is a direction vector, then kA has length Ikl. The direction of
k A will be the same as A if k >
0, and opposite to that of A if k 0. <
+
Example 4 Determine a vector B of length 3/2 that has the same direction as A = PQ,
where P = ( - 1,2), Q = (3,4).
Solution A = ( 3 - ( - l ) , 4 - 2) = ( 4 , 2 ) and IIAI/ = d m =2 6 . Thus

is a unit vector in the direction of A. Therefore

is a vector of length 3/2 that has the same direction as A. 0

17.3.17 Definition The vectors i = (1,O) a n d j = (0,1) are called the unit coordinate
vectors.

Note that i and j are unit vectors in the directions of the positive x andy axes,
respectively. By our rules for addition and scalar multiplication, for a n arbi-
trary vector A we have

J
A = (a1, an)
+
= (a13 0) ( 0 , a z )
= al (1,O) + a2 ( 0 , 1) = ali + apj
If A and B are vectors and a and ,8 are scalars, then the vector a A PB is+
called a linear combination of A and B. Thus every vector can be wrztten (uniquely) as
a linear combinatzon of the unit coordinate vectors i and j . This is illustrated in
Fig. 17.39.

VECTORS I K SPACE
i a,i T h e concept of a three-dimensional vector (3-vector) is a direct extension of the
Fig. 17.39 notion of a plane vector. That is, a 3-vector is an ordered triple of real numbers. If
570 CHAPTER 17: VECTORS AND SOLID GEOMETRY

A = (al, a,, a 3 ) is such a vector, then the numbers al, a2, and a3 are called,
respectively, the x,y, and t components or coordinates of A. As with plane vectors
(2-vectors), the operations are defined componentwise. Specifically, if
A = (al, a,, a 3 ) and B = ( b l , b,, b3) are vectors, and if k is a scalar, then
+
A B = (a, +
bl,a2 b2,a3 +
b3) +
A - B = (al - bl,a2 - b2,a3 - b,)
kA = (ka,, kaz, ka,)
The properties of vector addition, subtraction, and scalar multiplication given
in Theorems (17.3.7) and (1 7.3.9) extend directly to 3-vectors by taking into
account the additional component. The collection of all 3-vectors is denoted
by R3.

If A = (al,a,, as) and if P = (x,y,z ) is any point, then\the directed line


segment from P to Q = (x + al,y +
a,, z + a3), denoted PQ,is called a repre-
sentative of A. As with plane vectors, this gives us a geometric realization of a
vector in R,. Ag%n, we do not distinguish between represen2tives of the same
vector, Thus if PQ is a representative of A, we write A = PQ The notions of
base, head, and standard representative of A are as for 2-vectors; see Fig. 17.40.
As with vectors in R,, we can obtain the components of a vector given in the
X
+
Fig. 17.40 form PQ by subtracting the coordinates of P from those of Q.
-+
17.3.18 Theorem If A = PQ, where P = (xl,yl, zl)and Q = (x2,y2,z,), then

A = ( x 2 - X l , Y 2 - Y1,tZ - 21)
The norm or length of A = (a,, a,, a 3 ) is again the length of any representative
of A. Thus, from the distance formula (17.1.2), we have

IIAII = da12+ + a3
As before, IIkA/l = Ikl /IA/I and A/IIAII is a unit vector.
For 3-vectors the unit coordinate vectors are given by
i = (1,0,0) j = (O,l,O) k = ( O , O , 1)
and every 3-vector can be uniquely expressed as a linear combination of i, j,and k
by means of
(al, a,, a s ) = ali + a2j + a3k
It is often convenient to think of R, as being a subset of R, by identifying a
J typical vector (al,a,) in R, with the vector (al,a,, 0) in R,. Thus, in effect,
X when dealing with linear combinations of the form ali + a 2 j , we can always
Fig. 17.41 treat i and j as being the unit coordinate vectors in R,.

SECTIOS 17.3 EXERCISES


+
I n Exercises 1 to IO, ( a ) determine Q such that P Q = A, ( b ) determine I/AII, and ( c ) sketch the standard representative o j A.
1. A = ( - 2 , 3 ) , P = ( 4 , - 1 ) 2. A = (-3/2, 1/3), P = (3, -2)
3. A = 4i + 2j, P = (2, 1/2) 4. A = (-3, - 8 ) , P = ( - 2 , g )
5. A = -2i + 6j, P = (-11, -6) 6. A = ( - 2 , - l , - ~ ) , P = ( l , l , l )
7. A = (5, -2, I ) , P = (-2, 1,2) 8. A = ( - l , - 2 , 3 ) , P = ( 3 , 1 , 4 )
9. A = (3, -1,O), P = ( 2 , -1, 1) 10. A = (-2, 1, I ) , P = (2, - 1 , 2 )

+
I n ExercisesII to 20 wrzte A = P Q in component form.
11. P = (-3,5), Q = (9, -3) 12. P = (4/3, l/Z), Q = ( 0 , -2)
13. P=(d2,1+ f i ) , Q = ( - l , l ) 14. P = ( - 1/2, 7/9), Q = ( 7 , -2/9)
SECTION 17.4: THE DOT AND CROSS PRODUCTS 57 1

15. P = (3,7), Q = (-6,O) 16. P = ( 2 , 1, - l ) , Q = (4, - 1: 7)


17. P = (3: $2, - l ) , Q = ( - 1 , 2 , 4 ) 18. P = ( - 2 , 4 , 13), Q = (7, -1, 12)
19. P = (T, I , el, Q = ( - 2 , fi,1) 20. P = (7, - 1, fi), Q = (3, 2, - 1)

In Exercises 21 to 30 compute A + B and A - B, and illustrate geometrically.


21. A = (-2, I), B = (3,5) 22. A = 3i - 2j, B = ( - 1 , 4 )
23. A=(-7,-3),B=(-2,-1) 24. A = (11,3), B = -8i + 2j
25. A = (6, -2, - I ) , B = (2, 1 , l ) 26. A = (5, - 1,2), B = (3,2, - 1)
27. A = (8: -4, 2), B = (4, -4,O) 28. A = i + 2j - k, B = 3i + 2j + 2k
+
29. A = PQ, P = (1, 6), Q = (-3, 2); B = 7i + 2j
+
30. A = PQ,P = (-1,4, I), Q = (-3, -5,2); B = 4i - 2j +k
In Exercises 31 to 35 determine the indicated vector or scalar quantity where A = (1, 0, -2), B = (-3, 1, 17), C = (6, 1, - 5 ) .

In Exercises 36 to 41 determine a plane vector k with the same direction as A and with the indicated length
36. A = ( - 3 , 4 ) , IIBII = 6 37. A = (1, fi), IIBII = 2
38. A = (2, -3), IIBII = 1 39. A = 4i - 4j, IIBII = 3
+
40. A = Po;P = (1, -2), Q = (3, 6); I/BII = 1/2
-+
41. A = PQ;P = (-1, -3), Q = (-1,6); 11BlI = 9
42. Prove Theorem (17.3.9), part (e).
43. Prove Theorem (17.3.9), part (g).

Section 17.4: The Dot and Cross Products

Objectives: INTRODUCTION
1 . Evaluate the dot product of two In this section we discuss two different types of vector multiplication. The first,
vectors. called the dot product, is defined for vectors in R, or R,. The second, called the
cross product, is only defined for vectors in R,. Both products are very useful in
2. Use the dot product to
many geometric and physical applications.
determine (a) the angle between
two vectors, ( b ) scalar and vector
projections, and (c) the work THE DOT PRODUCT
done by a force in moving an
17.4.1 Definition If A = ( a l , a,, a,) and B = (b,) b,, b 3 ) , then the dot
object along a straight line.
product of A and B, denoted A B, is defined by
3. Evaluate cross products and
apply the various geometric
A * B = a,bl + a,b, + a3b3
properties to compute areas and The dot product is also referred to as the scalar product or the inner product.
volumes.
It is important to note that the result of taking the dot product of two vectors is a
scalar. Of course, if A, B are in R,, then the dot product is given by
572 CHAPTER 17: VECTORS AND SOLID GEOMETRY

We summarize some properties of the dot product that follow directly from
the definition.

17.4.2 Theorem Suppose A, B, C are vectors in R, (or R,) and k is any scalar, then
(a) A - B = B O A
( b ) A * ( B + C) = A * B + A * C
(c) k(A*B) = (kA)*B = A*(kB)
(d) 0.A = 0
( e ) A9A = IIAII2
To explore a useful geometric interpretation of the dot product, we first
define the angle between two vectors. Notice that even when the vectors are in
R,, two vectors with a common base point lie in a plane.

17.4.3 Definition Suppose A and B are nonzero vectors. If A is not a scalar multiple
of B, then by the an$e 0 between A and B we mean the least positive angle
between representatives with the same initial point. If A = kB, then we define
the angle between A and B to be 0 if k >
0 and 7i if k <
0 (see Fig. 17.42).

Note that the angle between two vectors is by definition between 0 and 7i.

B
b*7- a=o
A B
8=n
Further, this angle is directionless in the sense that the angle between A and B
is the same as the angle between B and A.
We say A and B are parallel and write AI/B if 0 = 0 or t' = 7i. If 0 = ~ / 2 ,
then we say that A and B are perpendicular or orthogonal and write A I B. It is
Fig. 1 7 . 4 2 convenient to define the zero vector to be both parallel and orthogonal to every
vector. With this definition AIIB if and only if A = kB for some scalar k.
We now give the geometric significance of the dot product: which is a
consequence of the Law of Cosines.

17.4.4 Theorem Let 6' be the angle between the nonzero vectors A and B. Then

cose = A-B
IlAIl JIB11
Prooj? Choose representatives of A and B with the same initial point and
consider the triangle that has these representatives and a representative of
A - B as its three sides (see Fig. 17.43). By the law of cosines
IlA - B112 = IlA1I2 + IlBIl2 -- 2lIAlI IIBII C O S Q
Now, by [17.4.2(e)],
IJA- B1I2 = (A - B)*(A - B)
z (A - B)* A + (A - B)* (-B) by [17.4.2(b)]
Fig. 1 7 . 4 3 = A * A - B O A- A * B + B * B = 1IAII2 + IIBll2 - 2A.B
Comparing this with the above, we conclude that

and hence

cose =
A*B
IIAlI IlBll
.
As an immediate consequence of (17.4.4) we have:

17.4.5 Theorem A IB if and only if A * B = 0.


SECTION 17.4: THE DOT AND CROSS PRODUCTS 573

Example 2 Let A = (2, - 1) and B = (k, 3). Determine k such that A 1 B.


Solution A * B = (2, -1). ( k , 3) = 2k - 3
Thus A - B = 0 if and only if k = 3/2, so B = (3/2,3). 0

Example 3 Determine the angle between the vectors A = ( - 1 , 3 , 4 ) and B = (1, 2, -2).
7

Solution

= -l/mz -0.1961
~ f
Since 0 5 0 5 T, we have 0 =. 1.768, or l9 =: 101.3"; see Fig. 17.44. 0

0 4 101 3b If neither A nor B is zero, then from (17.4.4) we have


.J
A * B = llA41~
I/BII cos 6'
Since /cosQl5 1, we conclude:

Fig. 17.44 17.4.6 Cauehy-Sehw-arz Inequality P*BI I IlAIl IIBII

PROGRESS TEST 1

1. Compute the cosine of the angle between the vectors 2. For which choices of k will A = ( k , 3, -4) be per-
A and B. pendicular to B = ( 4 k , - 1, k)?
( a ) A = (2, - l ) , B I= (5, 10) +
3. Show that 1IA B1I2 = /IA1I2 2A.B + /1B/12 +
( b ) A = (2, 1, -6), 13 = (3, -1, 1)

SCALAR A X D VECTOR PROJECTIONS


Suppose A and B arc nonzero vectors with a the angle between them. Consider
representatives of A and B with the same initial point P and let L be the straight
line through this representative of B. Drop a perpendicular from the head of A
to the line L and let Q denote the point w h e 2 this perpendicular meets L (see
Fig. 17.45). The vector with representative PQ is called the vector projectzon of A
on B or the uector component of A zn the dzrectzon of B and is denoted by
ProjB A
Note that ProjB A has the direction of B if 0 5 a <
a/2, whereas it has the
P, \ I' B
opposite direction of B if a/2 <a 5 7 i . If a = m / 2 then ProjB A = 0.
A ' '
\,
fi
The scalar projectzon of A on B or the component of A in the dzrectzon of B is
(I. Q defined by
ProjB A = IIA(1cos a
FromFig. 17.45wenotethatifO 5 a < 7 / 2 , then IIProj, AIJ = IIA(I cos a =
\ ProjBA. O n the other hand, if m / 2 < a 5 7, then lJProjBAll=
\ Proj, A /IAIl cos(m - a ) = - IIAlI cos a = -ProjB A. Thus the absolute value of the
scalar projection is equal to the length of the vector projection. Since the sign of
the scalar projection reflects whether ProjB A has the direction of B or - B, and
Fig. 17.45 since B/JIBII is a unit vector in the direction of B, by (17.3.15) we know that
574 CHAPTER 17: VECTORS AND SOLID GEOMETRY

B
17.4.7 ProjB A = (ProjB A) -
IIBII

The following gives a n easy method for determining vector and scalar projec-
tions using dot products.

17.4.8 Projections and Dot Products Suppose A and B are nonzero vectors.
Then
Proj, A = ( - 3 / 2 ; 312, 3)
A*B
(u) ProjB A = -
IIBII .
( b ) Proj, A = (P)B.
*B

Prooj? By (17.4.4), cos (Y = and so


IlAIl IlBlI

Also,

B = --
ProjBA = (ProjB A) - A*B B = ( s ) B = ( g ) B W
Fig. 17.46 IlBlI IlBll IlBIl

Example 4 Determine ProjBA and illustrate geometrically, where A = ( - 1, 2, 3) and


B = (1, -1, -2).
Solution Referring to Fig. 17.46 above,
( - 1 , 2 , 3 ) * ( 1 , -1, -2)
ProjBA =
(1, -1, - 2 ) * ( 1 , -1, -2)
- --9 (1, -1, -2) = (-3/2,3/2,3) 0
- 6

The notion of a scalar projection allows a n interesting physical interpreta-


tion of the dot product. In our prior discussion of work (Sec. 12.4) we always
assumed that the constant force was applied along the line of the motion.
Because of this, the direction was irrelevant to the calculations and so we were
able to represent the force as a scalar. In this case, work was defined as force
times displacement. More generally, a constant force is a vector, and the
direction in which the force is applied need not be in the direction of the
motion. Specifically, suppose a constant force is applied to move an object from
a point P to a point Q along a straight line. Note in Fig. 17.47 that
F = Proj$F + B. However, since B is perpendicular to the line of motion, it
will make no contribution to the work done. Thus, we define the work done by
F in moving the object from P to Q to be the component of F in the direction of
Fig. 17.47 the motion times the displacement. That is,

17.4.9 Definition-Work

another interpretation of the dot product of two vectors: It is the work done by
the first vector (considered as a force) as the point of application moves along
the length of the second.
SECTION 17.4: THE DOT AND CROSS PRODUCTS 575

PROGRESS TEST 2

1. Let A = ( a l , a2, a,). Show that the components of A 3. A constant force of magnitude 10 lb and in the
in the direction of the positive x,y, and z axes are, direction of the vector (1, 1) is applied to move an
respectively, the x , y , and z components of A. object 30 ft along a straight line. Find the work done.
2. Determine the scalar and vector projections of
A = (2, 1, -6) into B = (3, -1, 1).

SOME GEOMETRY
We have not yet specifically defined what we mean by the direction of a
vector in R,. To do this we introduce the notion of direction angles. These will be
useful in our later study of lines in space.

i 17.4.10 Definition Suppose A is a nonzero vector in R,. The direction angles of A,


denoted a , p, and y , are, respectively, the angles between A and the coordinate
vectors i, j, and k. The numbers cos a , cos P, and cos y are called the direction
cosines of A (see Fig. 17.48).

If A has direction angles a , p, and y , then -A will have direction angles


7i - p, and 7i - y (check this). Since cos (7 - 0) = -cos0, it follows
- a , 7i
that the direction cosines of - A are precisely the negatives of the direction
cosines of A. From (17.4.4) we conclude the following:
J
X

Fig. 17.48 17.4.1 1

As a consequence of these observations, we have:

17.4.12 Theorem If A is a nonzero vector with direction cosines cos a , cos p, and
cos y, then
(a) A = I/A/l(cosa,cosp, cosy) ( b ) cos2a + COSP+ COSY =1

Note that [17.4.12(a)] emphasizes again the fact that a vector is completely
determined fy its Length (IIAll) and its direction (the direction cosines). It follows from
(17.4.1 1) that if A is a unit vector, then the components of A are the direction
cosines of A.
In particular, we have:

17.4.13 Theorem If A # 0, then A/IIA/I is a unit vector with the same direction as A.
576 CHAPTER 17: VECTORS AND SOLID GEOMETRY

+
Example 5 Find the direction cosines for the vector A = PQ, where P = (1, 2 , - 1) and
Q = (-1,4,6).
Solution We first use (17.3.18) to determine the components of A:
A = (-1 - 1 , 4 - 2 , 6 - (-1)) = ( - 2 , 2 , 7 )
NOW1JAl/= d 4 + +
4 49 = $ET, SO A/llAll = (-2/m, 2/m,7/$ET)
is a unit vector having the same direction as A. Thus cos a = - 2/ m,
cos ,B = 2/ m, and cos y = 7 / m. 0

Example 6 Prove that a parallelogram is a rectangle if and only if its diagonals have the
same length.
Solution Let P, Q, R , and S denote the vertices of the parallelogram in Fig. 17.49. Let
+ --+
A = PQ and B = PS.Now by our geometric realization of vector addition and
+ -+
subtraction, the diagonal PR is a representative of A + B and the diagonal SQ
is a representative of A - B. Thus the parallelogram has diagonals of equal
length if and only if IIA + BII = /1A - BII or, equivalently, IIA + BII2 =
I/A - B/I2.Now//A +
+ B/12= (A B)*(A + B) = A * A + 2A.B + B * B a n d
IIA - B/I2= (A - B)*(A - B) = A * A - 2 A - B + BOB. Equating these, we
obtain A * B = 0; that is, A IB. Thus the diagonals have equal length if and
+ +
B only if PQ i PS, which is in turn true if and only if the parallelogram is a
Fig. 17.49 rectangle. 0

The problem of finding the distance from a point to a line can be realized as
a minimization problem and solved using the methods of Chap. 5 . A much
simpler solution can be obtained using vector methods.

Example 7 Use vector methods to show that the distance from a point (xn,yn)
. -
to a line L
represented by ax + by = c is given by

Solution Let N = ( a , b). Then N is perpendicular to the line L (see Progress Test 3,
Prob. 3). Suppose PI= (xl,yl) is any point on L and place a representative of
--f
N with its base at P. Then the desired distance is ilProjN PIQl/ (see Fig. 17.50).
Note that this is the case regardless of which side of L the representative of N
happens to lie on. Now

Fig. 17.50
SECTION 17.4: THE DOT AND CROSS PRODUCTS 577

PROGRESS TEST 3

1.

2.
Determine the vector B of length 2 with the same
direction as A = ( - 2, 1 , 3 ) .
Compute the direction angles of the vector
A = (1, -1, fi).
ax
on
+ L.
+
Then
PlP2 IN.)
-
3. Show that N = ( a , b ) is perpendicular to the line
= c. (Hint:ChoosePl = (xl,yl), P2 = ( x 2 , y 2 )
PlP21(L, so it suffices to show that

THE CROSS PRODUCT


T h e cross product of two vectors differs markedly from the dot product in that
(1) the cross product of two vectors is itself a vector, and (2) some of the usual
rules of algebra are not valid for cross products.

17.4.14 Definition Let A = (al,a2, a,) and B = (b,, b2, b 3 ) be two vectors. T h e cross
product of A and B, denoted A x B, is defined by
A x B = (a2b3 - a3b2, a,b, - alb3, alb2 - a2bl).
A simple way to remember (17.4.14) is to express it in terms of the first row
expansion of a determinant.

17.4.15

Example 8 Determine A x B for A = (2, 1, - 3 ) , B = ( - 4 , 2 , 1).

Solution

+
= (1 6)i - (2 - 12)j + (4 + 4)k
= (7,10,8) 0

Although the following properties of the cross product are direct conse-
quences of the definition [see the proof of part ( d ) , for example], a real under-
standing of cross products will develop from the geometric interpretation.

17.4.16 T h e o r e m Let A, B, and C be vectors in R, and let k be a scalar. Then


( a ) A X B = -(B X A)
(b) k ( A x B) = (kA) X B = A X (kB)
(c) A x (B + C ) = (A X B) +
(A X C )
(d) A X A = O
(e) A x O = O X A = O
( f ) A * ( B X C ) = (A X B ) * C
(9) IlA x B/12= /lAl12llB1I2 - ( A e B ) 2
Proof of (d): By the definition,
AxA= (a2a3 - a3u2)i + (a3a1 - ala3)j + (a1a2 - a2al)k = 0 W
578 CHAPTER 17: VECTORS AND SOLID GEOMETRY

Fig. 17.51

We now turn to the geometric significance of the cross product.

i j k
ixj=(l,O,O)x(O,l,O)= 1 0 0
0 1 0
0 0 1 0 1 0
=I1 oli-Io olj+lo

17.4.18 Theorem Suppose A and B are nonzero vectors in R, and B is the angle
between them. Then I/A X BII = IIAlI IIBII sine.
Prooj Now A * B = IIAlI IIB//cosd. Thus, from [17.4.16(g)],

IlA x B/12= llAl12 IlBl12- (A*BI2


= lIAl12 IIB/l2- /IAl12IlBIl2cos2 6
= /IA1I2/1BII2(1 - cos26)
= IIA112 /IBI/2sin2 B
Since sin0 2 0, the theorem follows by taking square roots.

As an immediate consequence we have the following generalization of


[17.4.16(d)]:

17.4.19 A 11 B if and only if A x B =0


If A and B are not parallel, then IIA x B/I has a n interesting geometrical
realization. Suppose A and B have the same initial point and suppose 6 is the
SECTION 17.4: THE DOT AND CROSS PRODUCTS 579

parallelogram with coterminal sides A and B (see Fig. 17.52). Then the height
of 6 is 1/BII sin 8, and the base has length A. Hence the area of 6 is given by:

Example 9 Compute the area of the parallelogram having the standard representatives of
A = (3,2, 1) and B = ( - 1 , 4 , 3) as coterminal sides; see Fig. 17.53.
i j k
Solution AX B= 3 2 1 = ( 2 , -lO,14)
-1 4 3

Thus A ( @ ) = I/A X BI1 = d 4 + 100 + 196 = 1 0 f i .


There is also an interesting geometric interpretation of the triple scalar product
(A x B ) * C . Suppose A, B, and C form a right-hand triple and consider
representatives with the same initial point. Let 6 be the parallelepiped with
these representatives as coterminal edges (see Fig. 17.54). The face containing A
and B is a parallelogram with area IIA x BII. Since the volume of 6 is given by
the product of the area of its base and its height h, we have
Fig. 17.52 V ( 6 ) = IIA X Bllh
Now the height h is the scalar projection of C onto A x B; see Fig. 17.54(b).
Thus

By the same argument, but using different faces to represent the base, we
z obtain:

Volume of a Parallelepiped
V ( 6 ) = (A X B ) * C = (B X C ) * A = (C X A ) * B

Fig. 17.53

il

Fig. 17.54
880 CHAPTER 17: VECTORS AND SOLID GEOMETRY

17.4.22 T h e o r e m The vectors A, B, C form a right-hand triple if and only if

(A x B)*C>O
Note that C lies in the plane of A and B if and only if A x B is perpendicular to
C-that is, if and only if (A x B ) * C = 0. Thus we have

17.4.23 T h e o r e m If A, B, and C are vectors with the same initial point, then they are
all in the same plane (coplanar) if and only if
(A X B ) * C = 0

Example 1 0 The standard representatives of the vectors A = ( 1 , 2 , - l ) , B = (3, - 1 , 4 )


and C = (7,5, -3) form the coterminal edges of a parallelepiped 6 . Deter-
mine the volume of 6 .
Solution V ( 6 )= I(A X B ) * C /

= l(7, -7, -7)*(7,5, -3)l


= 149 - 35 + 211 = 35 0

In the previous argument we wrote h = Proj,,, C, which uses the fact that
C and A x B are on the same side of the plane determined by A and B. In other
words, A, B, and C form a right-hand triple. In general, if A and B are
not parallel and if C does not lie in the plane of A and B, then V ( 6 ) =
I(A x B) Cl. In fact, the sign of the triple scalar product tells us whether A, B,
and C form a right-hand triple.

PROGRESS TEST 4

Let A = ( 1 , 2 , - 3 ) , B = ( 2 , -4,5), C = ( 7 , 2 , -4). 3. Is A, B, C a right-hand triple?


1. Find the area of the parallelogram with coterminal 4. Determine a vector D of length 2 which is perpen-
sides A and B. dicular to the plane of A and C and such that A, C,
2. Find the volume of the parallelepiped with cotermi- D forms a right-hand triple.
nal edges A, B, and C.

SECTION 1 7 . 4 EXERCISES

In Exercises I to IO, (u) determine the dot product o f the given vectors and ( b ) find the cosine o f the angle between them.
1- (3, -17), ( 4 > 2 )
3. (7,7),(2T, -5)
5 . 2i + 5j, 6i - 1/2j
7 - ( 2 , 4 , 4 ) , ( -4,2, -4)
9. (3, -1, l ) , (-2, 1, -4)

In Exercises II to 2 0 j n d ( u ) ProjB A and ( b ) Proj, A.


11. A = ( 2 , 3 ) , B = (-4, -1) 12. A = (4, - 3 ) , B = 2i - j
13. A=(8,l),B=(-l,l) 14. A = (3,2), B = ( - 7 , l )
15. A = i - j , B = 5j 16. A = (1, - l , 2 ) , B = ( 2 , 1 , 5 )
17. A = (3,6, -2), B = ( 0 , - 2 , 3 ) 18. A = (4, -7, l), B = ( 6 , 1 , 5 )
19. A=3i+9j-k,B= -2i+9j+k 20. A = j + 2k, B = 4i + 2j
SECTION 17.5: LINES AND PLANES 58 1

In Exercises 2 1 to 24jind k so that the angle between the given vectors is n/4.
21. A = (3,2), B = (2, k ) 22. A = 3i + 2j,B = ki - j
23. A = (2, 1), B = (k, -3) 24. A = ki + j , B = 2i - j
25. If A and B are perpendicular and of unit length, and if 26. Show that A I B if IIA + BII = 1IA - B/I.
C = aA + PB, show that a = C * A , /3 = C O B . 27. Let A $ 0 and suppose A * B = A * C . Does it follow
that B = C?

In Exercises 28 to 3 1 use vector methods.


28. Show that the diagonals of a rectangle are equal. 3 1. Find the angle between the diagonal of a cube and one
29. Show that the medians of a triangle meet in a point, of its edges.
30. Show that the sum of the squares of two sides of a 32. Show that if A, B, C are nonzero and mutually perpen-
triangle is equal to the square of the third side if and dicular in R, and if a , P, y are scalars such that a A +
only if the triangle is a right triangle. PB + yC = 0, then necessarily a = ,I3 = y = 0.

In Exercises 33 and 34jind the work done by the illustrated constant force of magnitude 30 lb as the point of application moves from P to Q.

F /
33. 40 ft 34. 100 ft 35. Prove Theorem 17.4.1.

I n Exercises 36 to 49, suppose A = (1, 0, 2 ) , B = (2, 1, - 3 ) , C = (-1, 4, 6 ) and D = (1, 3, 0 ) and determine
36. AxB 37. D x C 38. (A - B) X (A + B) 39. A x (B x C)
40. AxD 41. ( A X B ) * ( C x D) 42. ( A X B) x C 43. A*(B x C)
44. The area of the parallelogram with coterminal sides A
and B.
45. The volume of the parallelepiped with coterminal edges
A, B, and C. 50. Show that, for any 3-vectors A and B,
46. A unit vector perpendicular to the plane of B and D. (A - B) X (A + B) = 2(A X B).
47. A vector of length 3/2 that is perpendicular to both B 5 1. Suppose that representatives of the 3-vectors A, B, C, D
and C. all lie in the same plane. What can you say about
48. The area of the triangle with sides B, C, and B - C. (A x 3) x (C x D)?
49. Two distinct unit vectors perpendicular to the plane 52. Show that the line segment joining the midpoints of two
through the three points ( l , O , l ) , (2, 2, -4), and sides of a triangle is parallel to, and one-half the length
( - 1,3,2). of, the third side.

Seetion 17.5: Lines and Planes

Objectives: LISES
1. Determine parametric equations In this section, if P = (x,y,z ) denotes a point i n three-space, t h e n P = ( x , y ,2)
of a uniquely determined line. will always represent t h e vector whose components a r e t h e s a m e as t h e coordi-
2. Give the equation of a uniquely nates of P. T h u s , geometrically, t h e s t a n d a r d representative of t h e vector P h a s its
determined plane. head a t the point P (see Fig. 17.55).
N o t e t h a t if P = (x,y,z ) and Q = (xl,yl, tl)a r e arbitrary points in space,
3. Determine whether lines t h e n (see Fig. 17.56)
intersect, are parallel, or are skew.
4. Determine whether planes are
parallel and, if not, determine the = ( x i , ~ i , t i )- ( z )Q -
~ 7 ~ 7= P
equation of their intersection. Let L b e a line i n space, P = (xl,yl, zl)a point o n L , and M = ( a , b , c ) a
5. Use the skills 1 to 4 in vector parallel to L. If R = (x, y,z ) , t h e n a necessary and sufficient condition for
+
combination. t h e point R to be on L is t h a t the vector PR b e parallel to M (see Fig. 17.57). B u t
582 CHAPTER 17: VECTORS AND SOLID GEOMETRY

,J /
Fig. 1 7 . 5 5 Fig. 1 7 . 5 6 Fig. 1 7 . 5 7

two vectors are parallel if and only if one is a scalar multiple of the other. Thus
+
R is on L if and only if P R = tM, for t some scalar, or R - P = tM; that is,
R=P+tM
Thus we have a vector equation for the straight line through the point P and
parallel to the vector M. It can be thought of as a three-dimensional analog of
they = mx + b equation of a straight line in the plane; the vector M is the
three-dimensional slope.
Since R = ( x , y , z ) and P + tM = (xl at,y, + bt,zl + +
c t ) , by equating
components we see that the vector equation R = P +
tM is equivalent to the
parametric equations
x = x1 + at y =y, + bt z = 2, + ct
Recall from Sec. 12.1 (you may wish to review Sec. 12.1) that the graph of
a pair of parametric equations x = x ( t ) , y = y ( t ) is a curve that consists of all
points in the plane of the form (x(t),y(t)).Similarly, a set of three parametric
equations x = x(t),y = y ( t ) ,t = t ( t ) will have as its graph a space curve consisting
of all points of the form ( x ( t ) , y ( t ) , t ( t ) ) .In this particular instance, by our
construction, the graph will be the straight line through P and parallel to L. In
summary, we have:

17.5.1 Theorem The straight line through P = ( x l , y l , t l ) and parallel to


M = ( a , b, c) has vector equation
L ( a ) R(t) =P + tM
and parametric equations
( b ) x = x1 + at, ~l =yl + bt, t = t1 + ct
The components of the vector M are called direction numbers for L. If we choose
any other vector parallel to L , we obtain a different, but equivalent, set of
7 parametric equations for L. In particular, any set of numbers proportional to
Fig. 1 7 . 3 8 the direction numbers for a line are themselves direction numbers, so direction
numbers are not unique.

Example 1 Determine parametric equations for the line L that passes through the points
PI = (1,2, - 1) and Pz = (3,4,2).
3
Solution Since L goes through the points Pl and Pz, it is parallel to PlP2 = ( 2 , 2 , 3 ) ; see
Fig. 17.58. Taking this vector as M and using PI as the point on L, we apply
[17.5.l(b)] to obtain x = 1 +
2t,y = 2 +
2t,z = -1 3t. +
SECTION 17.5: LINES AND PLANES 583

If the direction numbers of L-say, a, b, and c-are all nonzero, then we can
eliminate t in [17.5.l(b)] to obtain symmetric equations for L.

17.5.2 x - - 1 - Y - Y , - Z-- * ,
~ - -
U - b C

In the plane, a pair of straight lines will either be parallel or intersect. In


space, nonparallel lines need not intersect. In this case we say the lines are skew.
We can use the above algebraic descriptions of lines to determine whether a
given pair of lines intersect or are skew.

-
Example 2 Show that the following lines L , and L, are skew.
x-1 Y-2- 2-3
L,: --
- -
2 1 -4
L,: x =2 + 4t,y = -1,z =3 + 3t
2
Solution L , is parallel to M, = (2, 1, -4), and L, is parallel to M, = ( 4 , 0 , 3 ) . Since
4 M, is not a scalar multiple of M,, the lines L, and L, are not parallel. L, has
+
parametric equations x = 1 2s,y = 2 + s, z = 3 - 4s. They will intersect if
and only if there is a value for s and a value for t yielding the same point-that
+
i s , s a n d t s u c h t h a t 1 2s = 2+442 +
s = - 1 , 3 - 4s = 3 +
3t.Fromthe
second equation, s = - 3. Substitution of s = - 3 into the first equation yields
t = - 7/4, and substitution of s = - 3 into the third equation yields t = 4.
Thus there are no values of s and t that yield the same point. (The equations are
inconsistent.) Hence L, and L, do not intersect. 0

Fig. 17.59 EQUATIOXS OF PL-ANES


17.5.3 Definition Suppose N = (a, b, c) is a nonzero vector and PO = (xO,yO,
zO)is
--$
a point. The collection of all points P = (x,y, z ) such that POPis perpendicular
to N is called the plane through Po with normal vector N.
+
If POP is perpendicular to N, then it is perpendicular to any nonzero scalar
multiple of N, and conversely. Hence any nonzero scalar multiple of a normal
vector to a plane is itself a normal vector to that plane (see Fig. 17.59).

17.5.4 T h e o r e m The plane II through PO = (xO,yO,to)with normal N = (a, b, c)


+
has the equation a(x - xO) b(y - y o ) +
c(z - zO)= 0.
+
Prooj By definition P = (x,y, z ) is on II if and only if N POP = 0. The
equation follows since
+
N POP = (a, b, C) (X - x O , ~ -yo, z - zO)
= a(x - x g ) + b(y -yo) + ~ ( -z 20)

Some authors call the components of any vector normal to a plane its attitude
numbers.

Example 3 Determine a n equation for the plane through (2, -1,2) with normal
(3, -2,4).
Solution +
By (17.5.4), the plane has equation 3(x - 2) - 2 ( y 1) + 4(z - 2) = 0, or,
more simply, 3x - 2y + 4z = 16. The x,y, and z intercepts are, respectively,
584 CHAPTER 17: VECTORS AND SOLID GEOMETRY

(16/3,0,0), (0,-8,0), and (0,0,4). A portion of the plane is sketched in


Fig. 17.60. 0

The following connection between planes and their equations was promised
.: in Sec. 17.2, page 556.

t
I
17.5.5 Theorem Every plane has a n equation of the form ax + by + cz = d where
a, b, and c are not all zero. Conversely, the graph of a n equation of this form is
a plane with normal N = ( a , b, c).
Prooj From (17.5.4), every plane has an equation a ( x - x,) + b(y - y o ) +
c(z - z,) = 0, or alternatively, ax + by + cz = d, where d = ax, + by, + czo.
4
Conversely, let Po = (xo,yo,zo)be a fixed point on the graph of ax by+ +
cz = d and let P = ( x , y , z ) be any point on the graph. Then ax, + by, +
czo = d and ax + by + cz = d.
Subtracting these equations we have a(x - x,) + b ( y - y o ) + c(z - 2,) =
.J d - d = 0 , which, by (17.5.4), is the equation of a plane through Po with
Fig. 17.60 normal N = ( a , b, c). W

Example 4 A straight line L passes through the point (3, 1,2) and is perpendicular to the
plane II: x + 2y + 22 = 6. Find the point where this line intersects the plane.
Solution By (17.5.5), N = ( 1 , 2 , 2) is normal to ll.Since L is also perpendicular to II,it
follows that N is parallel to L. Thus L goes through (3, 1, 2) and has direction
numbers 1, 2, 2. From (17,5.1), L has parametric equations x = 3 t , +
+
y = 1 2t,z = 2 + 2t. If the point (3 + t, 1 + 2t, 2 + 2t) on L is also to be on
+ + + +
II,then we must have 3 t + 2(1 2 t ) 2(2 2 t ) = 6. Solving for t , we
have t = -1/3, which, when substituted into the equations for L , gives
(8/3, 1/3,4/3) as the desired point of intersection. 0

-
POP, x p,p, PLANES AKD CROSS PRODUCTS
---$ +
Suppose the points Po, Pl, Pz are not collinear. Then POPl and POPzwill not be
+ +
parallel, and hence the cross product N = POP, x POPz will be a vector per-
/ j L k Po
+ +
pendicular to both POPland POPz.By definition (1 7.5.3), the points Pl and Pz are
on the plane determined by Po and N. As we would expect, three noncollinear
Fig. 17.61 points determine a plane (see Fig. 17.61).

Example 5 Find a n equation for the plane through the points Po = (1,2, - l ) ,
Pl = (3,4, 2), and Pz = ( - 1 , 0 , 4 ) .
+ +
Solution By the discussion above, N = POP, x POPzis a normal to the plane. But
N = ( 2 , 2 , 3 ) X (-2, -2,5)

By (17.5.4), the plane has equation 16(x - 1) - 16(y - 2) + O(z + 1) = 0, or


y - x = 1. [In applying (17.5.4) we used the point Po. We could just as well
have used PI or Pz.] 0

We define planes to be parallel or perpendicular according to whether their


normals are parallel or perpendicular, respectively.
SECTION 17.5: LINES AND PLANES 585

Example 6 Determine a n equation for the plane II through the point (2,1, -6) and
perpendicular to both of the planes II,: x 2y 2 = 1 + +
and
n,: +
2x - 3y 2 = 4.
Solution The normals for II, and II, are, respectively, N, = (1, 2, 1) and
N, = (2, - 3, 1). Any vector N that is perpendicular to both N, and N, will be
a suitable normal for II.Thus we take

The equation for II is 5 ( x - 2) + ( y - 1) - 7(2 + 6) = 0, or 5 x + y - 72


= 53. 0

PROGRESS TEST 1

1. Find parametric equations for the following lines: 3. Find an equation for the following planes n:
( a ) Through(7, -2, 5) and parallel to (3, 1/2, - 1). ( a ) Through (2,6, -4) with normal (4, - 1, 2)
( b ) Through (2, 1,6) and (-5,3, 7). ( 6 ) Through (1,2, -3), (2,4,6), (-2, -1, 1).
2. Do the following lines intersect? If not, determine if
they are parallel or skew. x = 1 - 3t, y = 7 + t,
2 = 11 + 4t; ( X - 3)/1 = ( y - 9)/( - 1) =
(2 - 4)/3.

SOLME ADDITIOSAL GEOMETRY


Using vector methods it is relatively simple to determine the (perpendicular)
distance from a point Po = (xO,yO,tO) to a plane rI: ax + +
by cz = d. Let
PI = (xl,yl, 2 , ) be a point on II and place a representative of N = ( a , b, c) with
its base a t PI, From Fig. 17.62 it is apparent that the desired distance D is the
+
absolute value of the scalar projection of P,P, onto N.Note that the absolute
value is necessary since the representative of N need not be on the same side of
3 as the point Po.
J

Example 7 Show that the distance from the point Po = ( x o , y 0 , ~ , )to the plane
n: ax + by + cz = d is given by

D= lax0 + by, + c2, - dl


da2+ b 2 + c2

D = Proj, p,P,

Fig. 17.62
586 CHAPTER 17: VECTORS AND SOLID GEOMETRY

Solution From our previous discussion,

-
- I(X0 - X1,YO -A!20 - 21) (a, 4 C>l
Il(a, b, c>ll
- laxo + by, + 620 - (ax1 + by, + cz1)I -- lax, + by, + czo - dl
du2+ b2 + c2 du2+ b2 + c2
T h e last step follows because P, = (xl,yl,zl) is o n II a n d hence satisfies
ax1 + by, + cz, = d. 0

~~~ ~

Example 8 Suppose the straight line L has the vector equation R(t) = P tU, where U +
is a unit vector. Let Q be any point. Determine the point S o n L that is closest
to Q.
Solution T h e situation is diagramed in Fig. 17.63. Since S is o n L , we know
(1) s = P + tU
for some t. Now (S - Q) I U, so
(S - Q) U = 0

[(P + tU) - Q]* U = 0


so
*J (P- Q * U + t U * U = 0

/ But U U = llU1(2= 1, so we can solve this latter equation for t , getting

X
J t = (Q- P ) * U
Fig. 17.63 Substituting in ( l ) , we have S = P + [(Q- P) *U]U. 0

SECTIOK 17.5 EXERCISES

In Exercises 1 to 8 compute parametric equations for the line satisfying the given conditions.
1. Through ( 1 , 2 , -6) and (-3,4,2). 2.Through (8, - 1 , 4 ) and (-2,0, 7 ) .
3. Through ( - 2, - 6, 3) and parallel to (6, 1, 2 ) . 4. Through ( 1 / 2 , -8, e ) and parallel to ( 0 , 6 , 9).
5 . Through (0, - 2 , 6) and parallel to both the planes 6. The intersection of the planes x 2y - 22 = 5 and +
2x - y + 72 = 8 and 3x - 2y - 2 = 3. 5 x - 2y - 2 = 2.
7 . The intersection of the planes x + y - 22 = 3 and 8. Through the point ( 2 , 1 , - 1 ) and perpendicular to the
4~- y + 2 = 6. line x = 1 + 3t,y = 2 - t , z = 3 + 2t.
I n Exercises 9 to 1 6 j n d an equation for the plane satiflying the given conditions.
9. Through (-6, 3, l / 2 ) with normal ( 2 , 1 , -4). 16. Through ( 2 , - 1 , 7) and parallel to the plane
10. Through (4, -6, -1) with normal (3, -2, -6). 4x - 8y +
62 = 2 7 .
11. Through(1,2,-6)(3,7,9)and(4,-1,6). 17. Find the distance from the point ( 2 , 1 , - 6) to the plane
12. Through (-3, -4, 6) ( 2 , - 8 , 1 ) and (3, 7 , -4). 2x +
3y - z = 4.
13. Through (6, 1, -7) and containing the line x = 2 - t , 18. Let N be a unit normal for the plane IT, which passes
y = 4 + 3t, z = 8 + 7t. through the point Po = (x,,yo, t o )Show
. that the point S
14. Containing the intersecting lines x = 1 + 2 t , y = 4 - t , on II closest to the point Q satisfies S =
z = 3 + 4t and ( x - 3)/(-6) = ( y - 7)/(-1) = P0.N - Q * N ) N .
(2 - 8)/( - 13). +
19. Find the point on the plane x y - z = 3 closest to the
15. Containing the parallel lines x = 2 + t , y = 3 - 6t, point ( 2 , 1, -3).
2 = -1 + t and x = -4 + 2t, y = 8 - 12t, 20. The line L is obtained by projecting the line
z = 3 + 2t. L,: x = 1 +
2t, y = 3 - t, z = 2 t onto the plane +
SECTION 17.5: LINES AND PLANES 587

2x + y - t = 7. That is, through each point of L one 22. Find the distance from the plane 3x + y - z = 2 to the
drops a perpendicular to the plane 2x + y - t = 7. The point ( 2 , - 1 , 5 ) .
line L is made up of the points where these perpendicu- 23. Find the distance from the point = ( 1 , 2, - 1 ) to the
lars meet the plane. Find equations for L. line L: x = 3 +
t , y = - 2 + 2t, z = 1 + 2t.
2 1. Find equations for the line L of intersection of the planes
x + y + 22 = 6 and 2x - y - z = 4.

CHAPTER EXERCISES

R e v i e w Exercises
In Exercises I to 35 i d e n t ~(plane, cylinder, surface of revolution, particular quadric surface) and sketch the surface with the given Cartesian equation.
1. ~ ~ + y ~ + 2 ~ - 2 ~ + 6 y - 141 2 = 2. 9x2 +y' - 18x - 4y + 4 = 0
3. 9x2 + 9y2 - 42' - 18x + 18y - 162 2 =0 + 4. 92' + 18y2 - 4x2 = 0
5. y2 + 2' = e' 6. ~ ~ + y ~ + 2 ~ + 4 x + 81 1y =+O
7. y2 - 2y = t 2 - 62 8 + 8. 4y' - 8y - 9x2 - 36x - 32 = 362
9. 4x' - 92' = 36y 10. 3x2 + 32' = Y ' ' ~
11. 4x2 + 4 ~ ' t 42' + 4~ +
~ 42 = 5
1 2- 12. y = 4z2 - 242 +36
13. 9x2 + y 2 + 72x - 4y + 148 = 92 14. y 2 + 42' = 4x
15. x 2 + y 2 = (arcsin 2)' 16. +
x 2 + y 2 + 18y t 2= 0
17. X' + 4~ + y 2 - 104' + '2 - 1 6 ~ 92 = 0 + 18. 92' + 542 + 4y2 - 72y = -369
19. 4x2 -y2 + 2y - 2 9 - 82 = 14 20. 27x2 + 12y2 - 9 2 + 2 1 6 ~- 48y - 182 + 363 = 0
21. 12x2 + 4y' + 5z2 - 24x + 56y - 402 + 228 = 0 22. 9y' - 4x2 - 362' = 36
23. 3x - 2y +
72 = 0 24. +
9z2 36x2 - 4yz = 36
25. x + 2y + 32 = 18 26. 27x2 + 12y' + 9z2 = 108
27. x = y + % 28. x 2 + 3y2 = 9
29. x 2 = 21'4 - y 2 30. t = 6
31. y + 42 = 12 32. 1x1 + lyl = 4
33. t = lny 34. x2 + y 2 = (In z ) ~
35. x = t 3

In Exercises 36 to 4 5 j n d an equation f o r the surface of revolution obtained b~ revolving the given plane curve about the indicated axis.
36. x = 4 z 2 , x axis 37. x = 4 t 2 , z axis
38. x 2 / 9 + a Z / l 6 = 1 , x axis 39. x 2 / 9 + y 2 / 1 6 = 1,y axis
40. z 2 = y 3 ,z axis 4 1. z2 = y3,y axis
42. z = eY,z axis 43. z = eY,y axis
44. yz = 1,y axis 45. y z = 1 , z axis

In Exercises 46 to 5 5 j n d ( a ) the length of A, ( b ) a unit vector in the direction of A and ( c ) the cosine ofthe direction of A.

46. A (4, -3) 47. A = (m,v%)


48. A = (3,1/2) 49. A = (-2, -6)
--+
50. A = P Q , P = ( l , - l ) , Q = ( 2 , 4 ) 51. A = (-1,O)
52. A = (-1, -3) 53. A = (-2/3, 1/4)
+
54. A = (0,- 2 ) 55. A=PQ,P=(-2,-4),Q=(1,5)

In Exercises 56 to 60 determine ( a ) A x B and ( b ) the area Ofthe parallelogram with coterminal sides A and B .
56. A=(2,1,-l),B=(3,0,4) 57. A = (0,- 2 , 3 ) , B = ( 2 , 3 , 0 )
58. A = (1,2, -6),B = (-2,1,3) 59. A = (3,0,l ) ,B = (0,2, -4)
60. A = ( 0 , 2 , 0 ) , B = (5,0,0)

In Exercises 61 to 63jind the volume of the parallelepzped with cotermznal edges A, B , and C.
61. A = ( 1 , l , O ) , B = (0,2 , 3 ) , C = ( 4 , 0 , 1 )
62. A = ( 1 , 2 , - l ) , B = (0,2,0),C = ( 3 , 4 , 0 )
63. A = ( 3 , -2, - l ) , B = ( 1 , 3 , - 2 ) , C = ( 1 , 1 , 1 )
588 CHAPTER 17: VECTORS AND SOLID GEOMETRY

In Exercises 64 to 7 0 j i n d an equation for the plane satisfying the given conditions:


64. Through (1, 2, - l), (1, 3, 2 ) , (2, - 1, 5 ) . 68. Through (2, 1, - 2 ) and containing the line x = 4 + t ,
65. Through ( 2 , - 3 , 0 ) , (0,5, 7), ( 6 , 0 , - 4 ) . y = 2 - 3t, 2 = 4 + 2t.
66. Through ( - 2 , - 1, -4) and parallel to the plane 69. Containing the intersecting lines x = 3 + t , y = 2 - 6t,
3 x + 4y - 2z = 4. z =4 +
2t, and x = 7 - 4 t , y = 7 - 5 t , z = -4 8t. +
67. Through ( 6 , 2, -4) and perpendicular to the line 70- Containing the Parallel lines X = 2 - t, Y = 4 3t, +
x = 2- t , y = 3 + 4t: z = 8t. z = 7 - 8 t , and x = 4 - 2t,y = 3 + 6t, z = - 1 - 16t.

In Exercises 7 1 to 7 4 j n d equations for the straight line satisfying the given conditions:
71. Through (2, 1, -8) and ( 3 , 2, 5). 72. Through (-5, 1 , 4 ) and (0,8,-9).
73. Through (5, 6,0) and parallel to (-2: 1, -8). 74. Through ( 2 , 1, 1) and perpendicular to the line
75. Find the distance from the point ( - 1, 2,4) to the plane x = 4 - t , y = 5 + 7t, z = -1 3t. +
x + 3y - 2 = 5 . 76. Find the distance from the point ( 0 , 2 , -8) to the plane
77. IJse vector methods to show that the altitudes of a 3 x - y + z = 0.
triangle meet in a point. 78. Use vector methods to show that the midpoints of the
79. Use vector methods to show that the diagonals of a sides of a quadrilateral form the vertices of a parallelo-
parallelogram bisect each other. gram.

In Exercises 80 to 8 3 find the work done as the constant force with the ,<hen magnitude and direction is applied to move an object in the plane along a
straight line from P to Q.
80. /IF1/ = 30 lb, a = ~ / 6 P, = (1, - l ) , Q = (3,5) 81. //FI(= 21 lb, a = ~ / 4 P, = (2, I), Q = (4, 7)
82. /IF/I = 2 lb, a = n/3, P = ( 3 , 5), Q = ( 5 , 10) 83. IlFll = 40 lb, 01 = ~ / 6 P, = (4, - 2 ) , Q = (8, - 6 )
84. Force F, has magnitude 11FJ = 30 Ib and direction
a = ~ / 3 Force
. F2 has magnitude llF211 = 201b and force F, + F2 in moving a n object in the plane along the
direction a = ~ / 6 Find
. the work done by the resultant x axis from (0,0) to (40, 0).

Miscellaneous Exercises
1. Find a n equation for the sphere that is tangent to the 2. Find a n equation for the sphere that has its center on the
plane 2x + 3y + 22 = 12 and has its center at ( 3 , 2 , 1). y axis and passes through the points (O,O,5) and
3. An elliptical-cross-section torus is a surface of revolution (1, - 2 , O ) .
obtained by revolving a n ellipse about a line that does not 4. A surface is said to be ruled if through every point on the
intersect the ellipse. Find an equation for the elliptical surface there passes a straight line all of whose points are
torus generated by revolving the ellipse ( x - + on the surface. Complete the argument sketched below to
y2/b2 = 1 in the x-y plane about t h e y axis (c a). > show that the hyperboloid of one sheet, x2/a2 +
3. Suppose the v z coordinate system is rotated through an y2/b2 - z2/cZ = 1, is a ruled surface.
angle B about the z axis to obtain a n XYZ system: ( a ) Show that for every choice of 8 the straight line
x = a cos B + (a sin 8 ) t , y = b sin B - ( b cos 8 ) t ,
z = t lies on the hyperboloid.
( b ) Let A = (x,,yl,zl) be an arbitrary point on the
hyperboloid. It suffices to show that there exists a 8
such that the straight line in part ( a ) goes through A .
Since t must equal z , one can do this by showing that
x , = a cos 8 + ( a sin 8 ) z,,yl = b sin 8 - ( b cos 8 ) z ,
can be solved for 8. (Hznt: Solve these equations for
cos B and sin 8 and show that this suffices to deter-
mine 8.)
6. Find the angle between the diagonal of a cube and the
diagonal of one of its faces.
7 . Let L , and L , be lines parallel to the vectors v, and v2,
.I: respectively, and let a be the angle between v1 and v2.
( a ) Show that if a point P has coordinates (X,Y, Z ) in this T h e angle 8 between L , and L, is defined by 8 = a if
latter system, then it has coordinates (x,y, z ) in the 0 5 a 5 ~ / and 2 b y 8 = n - a i f ~ / 2< a 2 T. Deter-
former, where x = X c o s 8 - YsinB, y = mine the angle between each of the following pairs of
X s i n 0 f Ycos 0. and z = Z. lines.
( b ) Identify the quadric surface z = xy by first rotating (a) x = 2 - t , y = 3 + 2t, z = 4t;
through a n angle of 77/4 radians. x = 3 + 6 t , y = - 2 - 4t, z = 1 + 3t.
SECTION 17.5: LINES AND PLANES 589

(b) x = -2t,y = 4 + t, 2 = -1 - 5t; 10. Determine a n equation for the plane through the points
(X - 6)/1 = ( y - 5 ) / 3 = (t +8)/(-7). ( 1 , 2 , 5 ) and ( 3 , 6 , -2) and parallel to the line of inter-
By the angle between two planes we mean the angle between lines normal section of the planes x + y - 32 = 2 and 2x - y +
to these planes. In Exercises 8 and 9 determine the angle between each 22 = 4.
o f the following pairs o f planes. 11. Determine an equation for the plane through the point
8. 3 x + y - t = 4 , 2 ~ + 4 y + 3 2 = 12 ( 3 , - 1,4) and perpendicular to the line of intersection of
9. x --y - 52 = 1 , 2 x + y - 2 = 3 the planes x - y +
52 = 8 and 2 x - 3y z = 4. +
12. Show that three points P,
+ - +
a R in space are collinear if 13. Show that four points P, Q. R, S in space are coplanar if
+ - + +
and only if PQ x PR = 0. and only if PS ( P Q x QR) = 0.
15. Prove the converse of Exercise 14. T h a t is, if N = ( a , b, c)
14. Let II be a plane with normal N = ( a , b , c), and let and if d is any real number, then the collection of all
X = ( x , y ,2 ) be a n arbitrary point on II.Show that there X = ( x , y ,z ) such that X N = d represents a plane.
exists a real number d such that X N = d for all X. 17. Let L,, L , be skew straight lines through the points P, Q
16. Let L be the straight line with equation X = Po + t M and R , S respectively. Show that the distance from L , to
and let rI be the plane with equation X N = d, where L , is given by
IINI1 = 1. If N M # 0, show that L intersects II a t the
head of the standard representative for

19. Show that the standard representative of


18. Show that the distance from the point A to the line IIBIIA + IIAIIB
through the points B and C is given by IIAIIIIBII

d=
I1 a x 211 bisects the angle between the standard representatives of
I1 3
11 A and B.
21. Give examples with A # 0 to show that A C = A B
20. Show that if A is perpendicular to B and C = oA + PB, need not imply C = B.
thena=C*AandP=C*B. 23. Let A and B be nonzero vectors. Determine the scalar k
22. Show that ( ( A+ B(I2 - IIA - B1I2 = 4 A . B . such that A - k B is perpendicular to B.
+
24. Show that /IA/IB IIBllA is perpendicular to 25. When does 1 +
1.4 BI, = IiAII + /jBII?Justify.
IIAIIB - IIBIIA. 27. Show that IIA - B11 2 1IAII - IIBII. [Hint: Consider the
26. When does IA B! = I!AIIJJBII?Justify. equation A = B + (A - B).]
28. Show that (A +
B) (A - B) = IIA/1* - IIB/I2.

SELF-TEST
CHAPTER 18 X
OR I

nt and Normal

69 1
592 CHAPTER 18: VECTOR VALUED FUNCTIONS

Section 18.1 : Vector Functions

Objectives: IhTRODUCTION
1. Sketch the graphs of vector U p to this point we have studied the calculus of real functions of a real variable,
functions. that is, functions that transform real numbers into real numbers. Henceforth we
shall refer to such functions as scalarfunctions. Since many physical quantities of
2. Differentiate and integrate vector
interest are in fact vector quantities, it is important to consider vector valued
functions.
functions of a real variable or, more briefly, vector functions.
Vector functions have the form

18.1.1 F(t) = ( x ( t ) , y ( t ) , 4 t ) )
= x(t)i + y (t)j z(t)k +
where the component functions x ( t ) , y ( t ) and z ( t ) are scalar functions defined on
some common domain D.(i, j, and k are the unit coordinate vectors.) If the
domain of a vector function is not specified, then we assume it is the intersec-
tion of the natural domains of the component functions.
The graph of (18.1.1), called a space curve, is by definition the graph of the
parametric equations
x = x(t),y =y(t),2 = z(t) t <D
These are precisely the points traced out by the head of the standard repre-
sentatives of F ( t ) . When graphing a space curve, we usually place arrowheads
on the graph to indicate the direction that the points are traced out as t
increases (see Fig. 18.1).
If z(t) = 0 for all t in D,then the graph lies in the x y plane. In this case we
usually write

where now i = (1,O) and j = ( 0 , 1). We refer to the graph of F in this latter
case as a plane curve. In either situation we often refer to the curve F, when in
effect we mean the curve that is the graph of the vector function F.
Vector functions are particularly useful in describing the motion of a n object
in space (or in the plane). To accomplish this we assume that the mass of the
Fig. 18.1 object is located at its center of mass and refer to it as aparticle. If t denotes time,
then the head of the standard representative of F ( t ) gives the position of the
particle at time t. In this setting we often refer to F as the position function of the
motion and F ( t ) as aposition vector (see Fig. 18.2). O n occasion we shall make a
distinction between the position vector F ( t ) and thepoint F ( t ) located at the head
+ location of the particle
at time i
of the standard representative of F ( t ) .

SOME EXAMPLES
I
path of the particle 1 / Several plane curves were discussed in Sec. 12.1 in the context of parametric
equations. In particular, F ( t ) = ( 2 cos t , 3 sin t ) has as its graph a n ellipse
(Sec. 12.1, Example 3, page 399). The graph of F ( t ) = ( a ( t - sin t ) ,
a(1 - cos t ) ) is a cycloid (Sec. 12.1, Example 4, page 400). Similarly, we know
from work in Sec. 17.5, pages 58 1 to 583, that the graph of any vector function
of the form
F(t) = ( a t + ~ 1bt, + ~ 2 ct
, + ~ 3 )

Fig. 18.2 is a straight line in space.


SECTION 18.1: VECTOR FUNCTIONS 593

Example 1 Sketch the graph C of the vector function


F(t) = (a cos t , b sin t , t ) a, b >0
Solution The parametric equations are
x = a c o s t , y = bsint, z = t
We first eliminate the parameter from the first two equations. Since x / a = cos t
and y/b = sin t , we have

It follows that C lies on the elliptical cylinder (parallel to the z axis) x 2 / a 2 +


y2/b2 = 1. As t increases, the points move steadily up and around the cylinder
making a complete revolution around the cylinder on each interval of length
277. We refer to such a curve as an elliptical helix or, if a = 6, a circular helix. A
table of values on [0, 271 and a sketch is given in Fig. 18.3. 0

Example 2 Sketch the portion of the space curve F ( t ) = (cos t , sin t , 2 sin t ) that lies in the
first octant.
Solution First we eliminate the parameter from x a n d y and from x and z : x = cos t ,
y = sin t , so

Also, z = 2 sin t , so sin t = z / 2 and hence

x2 +);( 2
=1

Equation (1) is a circular cylinder parallel to the z axis and equation (2) is an
eIliptica1 cylinder parallel to they axis. The graph of F must lie on both, so it
must be the intersection of these cylinders. The portion in the first octant
sketched in Fig. 18.4 corresponds to the values 0 5 t 5 7 7 / 2 . 0

j
I ,/f?
i l , 0.0'
cylinder
J t = o t = O
2 + 2/4 = 1
Fig. 18.3 Fig. 18.4
594 CHAPTER 18: VECTOR VALUED FUNCTIONS

THE CALCULUS OF VECTOR FUNCTIONS


We define limits of vector functions componentwise. That is, we define
lim ( x ( t ) , A t ) , z(t)>= ( a , b, c>
t-to

lim x ( t ) = a, limy(t) = b, and lim z ( t ) = c


t-to t-to t-to

Our definition of continuity of a vector function is a direct analog of


scalar functions.

Fig. 18.5 18.1.2 D e f i n i t i o n The vector function F is continuous at to if F(to) is defined and
lim F(t) = F(to).If F is continuous at each point in a set of numbers I , then we
two
say F is continuous on I . A function F continuous on its domain will simply be
referred to as continuous.

With this definition, F is continuous at to if and only if each of the compo-


nent functions is continuous at to.
We define the derivative of a vector function using the familiar limits.

18.1.3 D e f i n i t i o n The derivative of a vector function F, denoted by any of F, D,F,


or dF/dt, is the vector function defined by
F(t + At) - F(t)
F(t) = lim
At-0 At
at all t for which this limit exists.

Let us analyze (18.1.3) geometrically. For small At, F(t + At) - F(t) has as a
representative the short vector from the head of the standard representative for
z \ +
F ( t ) to the head of the standard representative for F(t At); see Fig. 18.5. For
I t small (say, positive)
F(t + At) - F(t) = -[F(t
1
+ At) - F(t)]
At At
is a n elongation of this vector. In particular, the line through this representative
is a secant line.
As At + 0 these secant lines tend to the tangent line to the graph at the
point F ( t ) . Thus F(t) is a tangent vector in the sense that the representative of
F(t) whose base is at the point F ( t ) , lies in the tangent line to the graph at F ( t ) ;
see Fig. 18.6.
X It should not be surprising that the differentiability of F is completely
Fig. 18.6 determined by the differentiability of its components.

18.1.4 T h e o r e m Suppose F(t) = ( x ( t ) , y ( t ) ,z(t)>.Then F(t) exists if and only if each


of x(t), y(t), and z(t) exists. Furthermore,

F(t) = ( x ( t ) , y(4 z(t)>


ProoJ
F(t + At) - F(t) = ( ~ ( +t At) - x ( t ) , y ( t + At) - y ( t ) , z(t + At) - z(t)>
SECTION 18.1: VECTOR FUNCTIONS 595

Thus

F(t + At) - F(t)


1im
At-0 At
x(t + At) - x(t) y ( t + At) - y ( t ) z(t + At) -
= lim
At-0
(x(t
At
>
At
+ At) - x ( t ) , lim Y ( t + At) - y ( t ) ,
>
At

=(lirn
At-0 At At-0 At
lim
At-0 z(t + It)-
At
These latter limits exist and equal the derivatives of the component functions if
and only if the component functions are differentiable. W

Repeated applications of (18.1.4) yield higher order derivatives of F. Thus,


for example,
F(t) = ( ~ ( t )y(t),
, ~(t))
J provided, of course, that the second derivative of each component function
Fig. 18.7 exists.

Example 3 Determine parametric equations for the line tangent to the graph of the twisted
cubic F(t) = ( t , .t2, (2/3)t3) at the point P = (1, 1, 2/3).
Solution The point P = (1, 1, 2/3) corresponds to the value t = 1. Since F(t) =
(1, 2t, 2 t 2 ) , F(1) = ( 1 , 2 , 2 ) is a tangent vector to the curve at P.By (17.5.1),
the tangent line, sketched in Fig. 18.7, has equations
x =1 + t , y = 1 + 2t, z = 2 / 3 + 2t 0

PROGRESS TEST 1

Let F(t) = ( e t cos t , et sin t , e t )


1. Sketch the graph of F. 2. Compute F(t).
3. Give equations for the line tangent to the graph of F at the point (1,0, 1).
596 CHAPTER 18: VECTOR VALUED FUNCTIONS

One must be cautious when working with cross products since the order is important. Keep
this in mind when using [IB.I.S(e)].

Example 4 Let F ( t ) be a differentiable vector function on the interval I and suppose


I/F(t)lI = k for all t in I. Show that F ( t ) F(t) = 0 for each t in I.
Solution By (17.4.2), IIF(t)112 = F ( t ) F ( t ) = k2, SO
D , [ F ( t ) F ( t ) ] = Dt(k2)= 0
By [18.1.5(d)],
D , [ F ( t ) F ( t ) ] = F ( t ) F(t) + F(t) F ( t )
= 2 F ( t ) F(t) by ( 1 7.4.2)
Therefore 2 F ( t ) F(t) = 0 , which implies F ( t ) F(t) = 0. 0

Example 4 has a nice geometric interpretation. If F ( t ) = ( x ( t ) , y ( t ) ,t ( t ) )


+ +
is a curve such that IIF(t)I/ = k , then [ x ( t ) I 2 [ y ( t ) I 2 [ z ( t ) I 2= k 2 for each t
in I. That is, the graph of F lies on the sphere of radius k centered at the origin.
The reasonable conclusion of Example 4 is that the tangent vector F(t) is
perpendicular to the position vector F ( t ) for each t (see Fig. 18.8).
Part (c) of Theorem (18.1.5) is called the Chain Rule for vector functions and
X will be used extensively in the remainder of this chapter. For emphasis we
Fig. 18.8 restate it in Leibnitz notation.

18.1.6 Chain Rule If F = F ( s ) and s = h ( t ) , then

-_
dF -
- dF_
ds
dt ds dt

ISTEGRALS O F VECTOR FUNCTIOSS


18.1.7 Definition If F ( t ) = ( x ( t ) , y ( t ) , z ( t ) ) has integrable components on [a, b ] ,
then we define

Similarly, if each component has an antiderivative we define

Example 5 Let F ( t ) = (sin t , cos t , 2 t ) . Determine

( a ) J F ( t ) dt (b) JT14 F ( t ) dt
0

Solutions ( a ) J F ( t ) dt = (Jsin t dt,Jcos t dt, J 2 t d t )


= ( - cos t + el, sin t + c2, t 2 + c3)
= ( - cos t, sin t, t 2 ) + (cl, c2, c 3 )
= ( - cos t, sin t , t 2 ) + C where C = (cl, cp, c 3 )
SECTION 18.1: VECTOR FUNCTIONS 597

Note that whereas the indefinite integral of a scalar function involves an


arbitrary scalar constant, the indefinite integral of a vector function involves a n
arbitrary vector constant. The following result is a n immediate consequence of
(18.1.7) and the corresponding result for scalar functions.

18.1.8 Theorem If D,[F(t)] = G(t),then JG(t) dt = F ( t ) + C.

Example 6 Given that F(t) = (sin t , e t , 2 t ) and that F(0) = (2, 1,2), find F ( t ) .
Solution Since F(t) = (sin t , e t , 2t), by (18.1.8),

F ( t ) = J-(sin t , et, 2t) dt +C


= (s sin t dt, s e t dt, s 2t

= ( - cos t , e t , t 2 ) + (cl, c,, c3)


.>
+ (cl, c, cg)

Let t = 0, so

PROGRESS TEST 2

1. Determine F ( t ) if F(t) = ( 3 t 2 , 8t3, 2), F(0) =


(O,O,0), and F(0) = ( 4 , 6 , 8 ) .
2. Suppose F is differentiable and F(t) # 0 for all t.
Show that

ARCS AND ARC LESGTH


A vector function F whose domain consists of a closed interval I = [ a , b] is
called an arc. Following the language of Chap. 12, an arc F is said to be smooth
if the components of F have continuous derivatives on I and IIF(t)lI # 0 for all
t in ( a , b). If F is smooth, then the graph of F will have no breaks, corners, or
cusps. If F(tJ # F(t,) whenever t,, t, are in [a, b ) with t , # t,, then the arc is
said to be simple. Geometrically, the graph of a simple arc does not cross itself. A
simple arc for which F ( a ) = F ( b ) is said to be closed. These properties of a n arc
are illustrated in Fig. 18.9.
Definition (12.2.1) on the lengths of plane arcs can be routinely extended to
the three-dimensional setting.

18.1.9 Definition The length s of a smooth arc F(t) = ( x ( t ) , y ( t ) , z ( t ) ) , a 5t5 b , is


given by
598 CHAPTER 18: VECTOR VALUED FUNCTIONS

smooth and simple smooth, not simple


,graph crosses itself at P)

smooth? simple. and not smooth


closed corners at P and a
cusp at R
Fig. 18.9

If F is a smooth arc, then by the positive direction along the graph of F we


mean the direction that the head of the standard representative of F ( t ) moves
along the graph as t increases. The opposite direction is referred to as the negative
direction. If F is simple, then the positive direction can be unambiguously
indicated by placing arrow heads along the curve; see Fig. 18.10.
positive direction negative direction
Suppose F ( t ) , a 5 t 5 b, is a smooth simple arc and to is a fixed number in
along F along F [a, b ] . We define the arc lengthfunction on [a, b] by
Fig. 1 8 . 1 0

If t> to, then s ( t ) represents the length of the curve between the points F(t,)
and F ( t ) ; see Fig. 18.11. If we think of F as describing the motion of a particle,
then s ( t ) represents the distance traveled (along the graph of F ) by the particle
from time to to time t. From (18.1.10) and the Fundamental Theorem of the
Calculus, we have that
4

ds
18.1.1 1 -
dt = IIF(t)ll
In particular, since F is smooth, ds/dt >
0, so s is a strictly increasing function of
t . Consequently the functions is invertible. Hence, if t ( s ) denotes the inverse, we
can express the given function F in terms of arc length as follows:

= F(t(s))
The calculations involved in expressing a vector function in terms of arc
length are, in general, quite difficult. Nonetheless, as we shall see, it is of
theoretical value that it can be done. In particular, an interesting consequence
t of using arc length as a parameter is that the derivative with respect to s is
Fig. 1 8 . 1 1 always a unzt tqngent vector.
SECTION 18.1: VECTOR FUNCTIONS 599

18.1.12 Theorem Let F ( t ) be a smooth vector function and let s represent arc length
along the graph of F. Then d F / d s is a unit tangent vector.
ProoJ By the Chain Rule,

_
dF - d F ds
- --
dt ds dt
Now

and, since F is smooth,

Solving the above equation for d F / d s , we have

Clearly d F / d s has unit length, and since it is a positive scalar multiple of the
tangent vector, it is itself a tangent vector.

VELOCITY A S D ACCELERA4TION
A s noted earlier, one of the most useful applications of vector functions is in the
study of motion. If G ( t ) = ( x ( t ) , y ( t ) ,z ( t ) ) is the position function of a particle
moving in space, then G(0) is called the initialposition. Many motions of interest
Y
take place in a plane-including, for example, the motion of a planet about the
location at time
e sun. In cases such as these our position function has the form G ( t ) = ( x ( t ) , y ( t ) ) .
When using arc length as a parameter, we usually take s(0) = 0. Thus s ( t )
measures the distance traveled along the path of the motion from the point
(x(O),y(O),~ ( 0 ) )Of
. course, we measure time from some convenient starting
point, which may allow for negative t and hence negative s. For example, in
describing the plane path of a boat being tracked by radar, we may take t = 0
seconds to represent the instant it is directly in front of the radar installation.
installation Then G( - 2 ) , the head of the position vector G ( - 2 ) , represents its position 2 s
Fig. 18.12 prior to its arrival in front of the installation; see Fig. 18.12.
We shall assume that the position functions describing motions are twice
differentiable.

18.1.13 Definition Suppose G ( t ) is the position vector of a particle moving in space.


( a ) The vector V ( t ) = G(t) is called the velocity vector of the particle at time t.
( b ) The vector A ( t ) = V ( t ) = G(t) is called the acceleration vector.

By (18,1.11), l [ V ( t ) [ l= c!s/dt is the speed of the particle. Thus the velocity


vector points in the direction of the motion and has magnitude equal to the
speed of the particle. This is completely analogous to the situation in Sec. 5.3
when we studied straight-line motion. In that setting the absolute value of the
scalar velocity gives the speed, and the sign of the velocity determines the
direction. The analogy between the acceleration vector A ( t ) for a motion in the
plane or in space and the scalar acceleration a ( t ) for a straight-line motion is
not as direct. Both, of course, are obtained by differentiating the velocity. But,
in general, lIA(t)Il will not give d2s/dt2, the scalar acceleration in the path. A
more detailed description of motion and vector acceleration will be given in
Sec. 18.3.
600 CHAPTER 18: VECTOR VALUED FUNCTIONS

SECTION 1 8 . 1 EXERCISES

In Exercises I to 10 sketch the graph of the given vector function.


1. F ( t ) = ( 2 sect, 4 tan2 t ) 2. F ( t ) = ( t 2 ,t2 + t )
3. F ( t ) = ( - 2 + cos t , 3 + sin t ) 4. F ( t ) = ( 2 t + 1)i + ( 3 t - 6 ) j
5. F ( t ) = (4 - 4t4)i + t2j 6. F ( t ) = ( 2 cos t , 2 sin t , t )
7. F ( t ) = ( t cos t, t sin t, t ) 8. F ( t ) = ( 2 t 2 , 4t, t 3 )
9. + +
F ( t ) = (t 2)i tj + t2k 10. F ( t ) = eti + c t j + f i t k

In Exercises I I to I3 determine whether F is continuous at t = t,.


(t2 - 1,2t 7)+ O l t l L , -1 F(t) =
arctan t, 2t2 - t + 1, 2 ) t 5 1, -
11. F ( t ) = ((77/2 - arcsin t , 9 t )
t> 1
0 - 13.
(a/2 +arcsin t, 2 6 , 2 t ) t >
1 0 - ~

12. F(t) = t > o . t,=O


t = 0

In Exercises I 4 to 20 determine F(t) and F(t).


14. +
F ( t ) = (2$, t3 6 t ) 15. F ( t ) = (sin $, e t 2 )
16. F ( t ) = ( t In t, e3t, t 2 ) 17. F ( t ) = (cos2 t , 2 tan t, t / ( t + 1))
18. F ( t ) = ( C S C ~t, t 2 cos t ) 19. F ( t ) = ln(sec t ) i + 5j
20. +
F ( t ) = ( t - cos t ) i tj - ( l / d ) k

The line through F ( t o ) and parallel to F(t,) is called the tangent line to F at F ( t o ) . The plane through F ( t o ) with normal F(t,) is called the
normal plane to F at F ( t o ) . In Exercises 21 to 24jind equations f o r the tangent line and normal plane at the indicated point.
21. F ( t ) = ( e t , e - t , fi + t ) , (1, 1, fi) 22. F ( t ) = ( t 2 + 1 , 2 - t, t 2 - 2 t ) , (2, 1 , - 1 )
23. F ( t ) = ( t sin t , t cos t , f i t ) , (77/2,0, f i a / 2 ) 24. F ( t ) = ((2/3)t3, t , t ) , (18: 9 , 9 )

In Exercises 25 to 30 determine F ( t ) from the gzven znformation.


25. F(t) = ( 3 t + 1, t 2 + 2 ) , F ( l ) = ( 3 , - 1 ) 26. F(t) = (et sin t, et cos t ) , F ( 0 ) = (1, - 1)
27. F(t) = (cos t , sin t. 4nt), F(77/2) = (2,0,v 3 / 3 ) 28. F(t) = ( 6 t , 12t2 + l ) , F ( 0 ) = ( - 2 , l ) , F(0) = ( 3 , 2 )
29. F(t) = (0,- 3 2 ) , F ( 0 ) = ( 0 , 0 ) , F(0) = (150,150 fi) 30. +
F(t) = ( 1 2 t 2 8, 20t3 + 2, 12), F ( 0 ) = (0,I, 0 ) ,
3 1. Suppose that F is three times differentiable. Show that F(0) = ( 1 6 , 0 , 4 )
D,[F*(F x F)] = F * ( F X F)

32. Show that if a particle moves in space with constant


speed, then the velocity and acceleration vectors must be 34. Find a vector function whose graph is the intersection of
perpendicular. the surfaces z = d w
a n d y z = 2. +
33. A particle moves in space in such a way that the position 35. Suppose that the graphs of the two smooth curves F and
vector is always perpendicular to the velocity vector. G do not intersect. If IIF - GI/ has a minimum at t = to,
Show that the particle moves along a sphere centered at show that F(t,) - G(t,) is perpendicular to both F(t,)
the origin. and G(t,).

The Unit Tangent and


Section 18.2: Normal Vectors; Curvature

Objectives: INTRODUCTIOS
1. Determine the unit tangent and We saw in Chap. 4 the tremendous usefulness of first and second derivatives in
unit normal vectors for a vector graphing scalar functions. In Sec. 5 . 3 we used these derivatives to describe the
function. motion of a particle traveling in a straight line. We now wish to extend each of
these endeavors ( u ) to describe the geometrical characteristics of plane and
2. Determine the curvature for a
space curves and ( b ) to describe nonlinear motion of particles in a plane or in
plane or space curve.
SECTION 18.2: THE UNIT TANGENT AND NORMAL VECTORS; CURVATURE 60 1

space. In this section we concentrate on ( a ) and in the next section we concen-


trate on ( b ) . We begin with the simplest case, describing curves in the plane.

PLANE CURVES: KNIT TANGENT A N D NORMAL VECTORS


Given a plane curve, our aim is to construct a pair of perpendicular unit vectors
based on the curve-in effect, to act as coordinate vectors. These will better enable
us to describe the curve near that point. We assume throughout thzs discussion that the
vector function F is smooth with smooth derzvatives.

18.2.1 Definition If F is a vector function, then for each t the unit tangent vector at
F (t), denoted T ( t ) , is defined by

Note that T actually is a unit vector, that is, J1T/1= 1. Also, since T is a
positive scalar multiple of the tangent vector F, it is itself a tangent vector. As
noted in (18.1.12), if s represents arc length along the curve, then

18.2.2 Definition For each t the unit normal vector at F( t ) , denoted N ( t ) , is defined by
N(t2)

Since we are assuming that F has a smooth derivative, it follows that T is


smooth. In particular, T ( t ) exists and is # 0 for all t, and hence N ( t ) is defined
for all t. Also, since /IT//= 1, we know from Example 4 of Sec. 18.1 that
T IT. But N has the same direction as T, and so we conclude:

18.2.3 T ( t ) 1N ( t ) for all t


When sketching T ( t ) and N ( t ) we use representatives whose base is on the
I curve at the point F ( t ) . Thus T and N are orthogonal unit vectors that move
Fig. 18.13 along the curve as t moves through the domain of F (see Fig. 18.13).

Example 1 Compute T ( t ) and N ( t ) for the vector function F( t ) = ( t 2 , 2 t ) and sketch their
representatives at the points corresponding to t = 0 and t = 1.
Solution We have F(t) = (2t, 2), so liF(t)ll = 2 d m and hence

T(t)=

Similarly,

and
1
llT(t)ll =
so
N(t) =
602 CHAPTER 18: VECTOR VALUED FUNCTIONS

Thus F(0) = ( O , O ) , T(0) = ( 0 , 1) = j,N(0) = (1,O) = i, and F(1) = (1, 2),


T(1) = ( l / & , l/&), N ( l ) = ( l / & , - 1 / ~ ) . These representatives are
sketched in Fig. 18.14.

THE DIRECTION O F THE USIT NORMAL


1 = (l/v7,-l/&
Since T(t) is a unit vector, we know from (17.3.15) that

(1) T ( t )= (cos + ( t ) , sin + ( t ) )


where + ( t ) is the direction of T(t).Figure 18.15(u) shows a typical situation
where + ( t ) is increasing and Fig. 18.15(b) shows a situation where + ( t ) is
decreasing.
Differentiating (l), we have
T(4 = ((-sin +(t))+(t), (cos +(t))+(t)>
Fig. 1 8 . 1 4 = +(t)( -sin +(t), cos + ( t ) )
Now since ii( -sin + ( t ) , cos +(t))ll = 1, we know that IIT(t)ll = l+(t)l. Let
u(t) = ( -sin + ( t ) , cos + ( t ) )
Then
t
T

Since
-sin+(t) = cos(+(t) + 7/2) and cos+(t) = sin(+(t) + 7/2)
we can conclude that u is always a unit vector 90 in advance of T. That is, a
/ la,
counterclockwise rotation of T through 90 will coincide with u.It follows that
-u is 90 in retreat of T. Thus from (2) we can conclude:

18.2.4 Theorem Suppose F is a vector function and + ( t ) denotes the direction of


T(t).Then
>
( u ) +(t) 0 implies N(t) has direction + ( t ) 7 / 2 . +
<
( b ) +(t) 0 implies N(t) has direction + ( t ) - 7/2.

Theorem (18.2.4) is illustrated in Fig. 18.16. By applying (18.2.4) in a variety


of situations, it can be shown that the representative of N always lies on the
[b) concave side (inside) of the curve. And the representatives of N and T are
Fig. 1 8 . 1 5 always on opposite sides of the curve. The situation at F(t,) in Fig. 18.13 thus
could not occur.

(a) @(t) >0


Fig. 1 8 . 1 8
SECTION 18.2: THE UNIT TANGENT AND NORMAL VECTORS; CURVATURE 603

PROGRESS TEST 1

Determine T ( t )and N ( t ) for the given vector functions


and sketch their representatives at the point correspond-
ing to the indicated value of t.
1. F ( t ) = (cos 2t, sin 2 t ) , t = 7~/8
2. F ( t ) = ( t - sin t , 1 - cost), t = 7~

CURVATURE
If an object is in motion and if no force acts on this object, then according to
Newtons first law of motion its velocity is a constant; that is, it will move at a
constant speed in a straight line. Therefore, for a particle to have nonconstant
velocity, that is, to have either nonconstant speed or to move in other than a
straight line, it must be subject to a force. We are especially interested in
relating the shape of the path traveled by a particle and the forces necessary to
keep the particle on that path. We shall give a quantitative definition of what
we intuitively regard as the sharpness of a curve: the rate at which its
direction changes.

18.2.5 Definition If F is a plane vector function, then the curvature of F, k ( t ) , is


defined by

where +(t) is the direction of the tangent vector T(t) and s represents arc length
along the graph of F.

This definition expresses exactly what we would wish: It measures the change
in direction of the curve with respect to distance along the curve rather than with
respect to t. This ensures that we are measuring a property of the curve itself
rather than properties having to do with the way we move along the curve.
Now T = (cos +, sin @I),so by the Chain Rule

dT
- = (-sin
d+
+, cos +) -
ds ds
and hence

Thus an alternate formulation of curvature is given by


b

18.2.6

If F is a straight line, then the curvature is zero and the unit tangent does not
change direction as you move along the curve; see Fig. 18.17(a). If a curve is
turning gradually, then T changes direction gradually, so k will be small; see
Fig. 18.17(b). However, if the curve is turning sharply, then a small change in
arc length produces a large change in the direction of T and thus k will be large;
see Fig. 18.17(c).
C The next formulas for k ( t ) are in general easier to compute than (18.2.5) or
Fig. 18.17 (18.2.6) because they use the derivatives of F with respect to t.
604 CHAPTER 18: VECTOR VALUED FUNCTIONS

18.2.7 Theorem Suppose C is the graph of F ( t ) = ( . x ( t ) , y ( t ) ) . Then

Pro@ ( a ) By the Chain Rule

d T ( t ) - d T ( t ) dt
- T(t) - T(t)
- -- --
ds dt ds ds/dt F(t)
and hence

+
( b ) Since gives the direction of the tangent vector, tan + gives the slope of the
curve. Thus

Differentiating with respect to t, we have


d$ x(t)y(t) -y(t)x(t)
(3) (sec2+) - =
dt (X(t)I2

Substituting for sec2 qj in (3) and solving for d+/dt, we get

(4)
Now
dG -
- d@/dt -
- -- d+/dt
ds ds/dt d [ X ( t ) ] 2+ [y(t)]2
so we substitute d+/dt from (4) into this latter expression. After simplifying and
taking absolute values, we have

By [18.2.7(a)] and definition (18.2.2),

IITII T - T - d T / d t
kN=------ - d T dt - d T
- -_--
IIF/I IITll //FI/ ds/dt dt ds ds
and thus we have:

18.2.8 -=
dT(t) k(t)N(t)
ds
Although [ 18.2.7(6)] has the most complicated appearance of the various
formulas for curvature, it is usually the simplest to work with.
SECTION 18.2: THE UNIT TANGENT AND NORMAL VECTORS; CURVATURE 605

Example 2 Show that the curvature of a circle of radius r is constant and equals l/r.
Solution Since the position of the circle is irrelevant, we take as our vector function
F(t) = (7 cos t , r sin t )
which has as its graph the circle of radius r centered at the origin. Now
x(t) = -r sin t y(t) = r cos t
~ ( t=
) -r cos t y(t) = -r sin t
so, by [18.2.7(b)],

F I(-rsint)(-rsint) - (rcost)(-rcost)/
k(t) =
+
[( - r sin t ) 2 ( r cos t ) 7 3 / 2
$1

I
Example 2 suggests the following:
I/
I
3 18.2.9 Definition For k ( t ) # 0, we define the radius of curvature p ( t ) by p ( t ) = l / k ( t ) .
Fig. 18.18 The circle o f radius p ( t ) on the concave side of the graph of F and tangent to
the curve at F ( t ) is called the circle of Curvature.

The circle of curvature may be thought of as the circle that best fits the
curve at the given point (see Fig. 18.18).

Example 3 Find k ( t ) for the vector function F(t) = (2t, t 2 ) and sketch the circle of curva-
ture at the point (0,O).
Solution By [18.2.7(6)],

k(t) =
1W) - 2t(O)I -
-
1
[p)*+ ( 2 q 2 3 3 1 2 2dEF
Therefore k ( 0 ) = 1/2 so p ( 0 ) = 2. The center of curvature is thus 2 units in the
A circle of curvature direction determined by N(0) = j, that is, at the point (0,2) on they axis (see

AY
a t :O. 0)
Fig. 18.19). 0

If a curve is given in scalar functional formy = f ( x ) [or x = g ( y ) ] ,then we


can apply [18.2.7(6)] to obtain the curvature as a function of the independent
variable. (See Exercise 29.)

18.2.10 Theorem Iff exists, then the curvature of the graph offat the
point ( x , f ( x ) ) is given by
x
0,0, 1
graph of
Fit = ( 2 1 , t 2 )

Fig. 18.19 In Exercise 30 it is noted that the second derivative itself is an inappropriate
measure of curvature of the graph of$ In Exercise 31 it is illustrated that the
rate of change of the angle that the tangent line makes with the x axis is likewise
a n inadequate curvature measure.
For engineering purposes it is often of interest to know where a given curve
has its sharpest bend, that is, its maximum curvature.
606 CHAPTER 18: VECTOR VALUED FUNCTIONS

Example 4 Determine the point on the graph ofy = In x where the curvature is a maxi-
mum.
Solution L e t y = lnx. T h e n y = l/x a n d y = -1/x2, so

and

k(x) =
(1 + x 2 ) 3 / 2 - x(3/2)(1 + x2)ll22x
(1 + X2)3
- (1
-
+ x2)l2[1 + x 2 - 3x2] -- 2 [ ( 1 / f i ) - x][(l/fi) + x]
(1 + x2)3 (1 + X2)5/2
An analysis of the sign of k(x) shows that a relative maximum occurs a t
x = 1/ fi.Since this is the only extreme in (0, +
w), then (1, fi,In( 1/ fi))
must be the point where the maximum value of k ( x ) occurs. 0

PROGRESS TEST 2

1. Find the curvature for the function F(t) = ( t 3 , t 2 ) .


2. Determine the point on the curvey = ex where the
curvature is a maximum.

SPACE CURVES AND THE MOVING TRIHEDRAL


As in two dimensions, the unit tangent and unit normal vectors for space curves
are given, respectively, by

Since T and N are perpendicular, the angle between them is 77/2. Thus
B =T x N is also a unit vector since, by (17.4.18),
IIT X NII = IIT//IINII sin: =1

The vector B is called the unzt binormal. The vectors T, N, and B are mutually
orthogonal of unit length and form a right-hand triple. We refer to them as the
moving trihedral. In effect they yield a coordinate system that is similar to that of
the i, j, k vectors but moves along the curve.

Example 5 Determine T, N, and B for the circular helix F(t) = ( a cos t , a sin t , b t ) , and
sketch their representatives at the points corresponding to t = 0 and t = ~ / 2 .
Solution Now F(t) = ( - a sin t , a cos t , b ) , so
IIF(t)II = d a 2 s i n 2 t + a2cos2t + b2 = d m
Thus
- a sin t a cos t
T ( t )=
d F T F @TF &q-F
Similarly,
SECTION 18.2: THE UNIT TANGENT AND NORMAL VECTORS; CURVATURE 607

circular - a cos t -a sin t


cylinder T(t) =
d m d m
-a
- (cos t , sin t , 0)
a2 +b
so
a
lIT(t)ll = and N(t) = ( -cos t, -sin t, 0)
T &q7
I I hT A
__---I--_ Fin a 11y ,
I i j

I -cost -sin t 0 1

Fig. 18.20

The representatives at t = 0 and t = m/2 are sketched in Fig. 18.20. 0

We defined the curvature forplane curves as the absolute value of the rate of
change of the direction of the tangent vector with respect to arc length. Since
the direction of a vector in three dimensions cannot be expressed in terms of a
single angle, this definition is no longer convenient. However, in (18.2.6) we
showed that this definition is equivalent to (1 dT/ds (1. We use this formulation to
define curvature in R,.

18.2.1 1 Definition The curvature k of a space curve F is defined by

As before, we have

18.2.12 dF(t)
--
ds
- T(t) and -
dT(t) - k(t)N(t)
ds

The following gives us a computational version of curvature for a space curve.


We omit its proof.

18.2.13 Theorem Suppose F(t) is a space curve. Then

Example 6 Compute the curvature for the circular helix F(t) = (a cos t, a sin t , bt).
Solution F(t) x F(t) = ( - a sin t , a cos t, 6) x (-a cos t , - a sin t, 0)
= (ab sin t , -ab cos t, a)
Thus
il(ab sin t , -ab cost, a2)i1
k(t) =
~ I ( - a s i n t , a c o s t , b)1I3

- d2FF2-
- Iald2T-F - la1

(~m
(4- ) ~- a2 + b2
608 CHAPTER 18: VECTOR VALUED FUNCTIONS

Example 7 F i n d k ( t ) for t h e vector function F(t) = ( e t cos t, et sin t, e t ) .

Solution

i j k
F x F = et(cos t - sin t ) &(sin t + cos t ) et
- 2et sin t 2et cos t et

SECTION 18.2 EXERCISES

In Exercises 1 to 10.
(a) Find T(t) and N(t).
( b ) Sketch a portion of the curve near t = to.
( c ) Sketch representatives of T(to) and N(to) at F ( t o ) .
1. F(t) = ( t cos t)i + ( t sin t)j, to = 0 2. F(t) = (3 - 2t2)i + (4 + 5t)j, to = 2
3. F(t) = (2 sin 3t)i + (4+ 5t)j, to = a 4. F(t) = ( t - sin t , 1 - cos t ) , to = a / 4
5. F(t) = ( t - sin t , 1 - cost), to = 77/6 6. F ( t ) = ( t 3 , 3 t ) , to = 2
7. F(t) = ( 2 4 e t +e-), to = 0 8. F(t) = (ln(sin t ) , ln(cos t ) ) , to = a/4
9. F(t) = ( t 3 / 3 - t , t 2 ) , to = 1 10. F(t) = ( 2 - cost, 1 - sin t ) , to = a

In Exercises II to 15:
( a ) Find T(t), N(t) and B(t).
( b ) Sketch a portion of the curve near t = to.
( c ) Sketch representatives of T(to),N(to), and B(to) at F ( t o ) .
11. F(t) = ( e t cos t , et sin t , e t ) , to = a/2 12. F(t) = ( 3 t cos t , 3t sin t , 4 t ) , to = 7;/2
13. F(t) = (t2/2, t 3 / 3 , t ) , to = 1 14. F(t) = (cos t)i + etj + +
( t 2 2)k, to = O
15. F(t) = 4(sin)2ti + (2t - sin 2 t ) j + cos(2t)k, to = a / 2

In Exercises 16 to 2 5 j n d the curvature. The results g e a r l i e r exercises may be useful.


16. F(t) = (cos 2t, sin 2t) 17. F(t ) = ( e t sin t , et cos t )
18. F(t) = ( t 3 , 3 t ) 19. F(t) = ( t 3 , t 2 )
20. F(t) = ( t 3 / 3 - t , t 2 > 21. F(t) = ( a cos b, b sin t )
22. F(t) = ( e t , 2t, e c t ) 23. F(t) = ( l / t : t 3 / 3 , t )
24. f ( x ) = sinx 25. P(Y) = y 2 / 3
26. Show that the curvature for a polar curve r =f(0) is 29. Prove (18.2.10). [Hint; Express y = f ( x ) parametrically
given by with x = t , ]
30. Show that l f ( x ) l does not provide an adequate measure
of curvature. [Considery = f ( x ) = ax2 + bx + 6.3
31. ( a ) Let 0 = 0 ( x ) be the angle of inclination of the tan-
[Hint; Express r = f ( 0 ) parametrically with parameter gent line t o y = f ( x ) . Show
0.1 If ( X I 1
27. Use Exercise 26 to find the curvature of the polar curves.
(a) r = 2 + sin@ ( b ) r = 4 cos 20
Id
d0
= d l + [f(X)]2
28. For each of the following determine the point where the ( b ) Show that dO/dx is not an adequate measure of
maximum curvature occurs. curvature. [Considery = x 2 at (1, 1) a n d y = - &
(a) y = x2 ( b ) y = sinx, -a 5 x a < at (1, - 1); the curvature should be the same at these
<
( c ) F(t) = ( a sin t, b cos t ) , 0 t 5 277 points.]
SECTION 18.3: MOTION 609

I n Exercises 32 to 36 sketch the circle of curvature at the indicated point


32. F ( t ) = (8t, 2 - 2 t 2 ) , t = 1 33. F ( t ) = (t2/2, t 3 / 3 ) , t = 1
34. F ( t ) = ( e t cos t , et sin t ) , t = 57/2 35. y = sinx, x = ~ / 4
36. y = In x , x = 1
As the circle of curvature moves along the graph of the vector
37. Determine and sketch the evolute of the parabola
function Fits center traces out a curve called the evolute of F. By
F(t) = (t, t2).
(18.2.9), the evolute of F is the graph of the vector function C,
38. Using the computations from Progress Test 1, show that
where
the evolute of the cycloid F ( t ) = ( t - sin t , 1 - cos t ) is
C ( t ) = F(t) + P(t)N(t) the cycloid C ( t ) = ( t - sin t , cos t - 1).

Section 18.3: Motion

Objectives: INTRODUCTION
1. Describe the motion of a particle As mentioned when introducing vector functions, such functions provide an
when given the position function. excellent means for describing the motion of a particle in the plane or in space.
The position function of such a particle will be denoted by G and, as usual, the
2. Determine the position function
velocity and acceleration are denoted by V and A respectively. The graph of G
of a projectile from information
is usually referred to as the p a t h or trajectory of the particle. From Sec. 18.1 we
about its velocity and acceleration.
recall
3. Determine the tangential and
normal components of 18.3.1 V ( t )= G ( t )
acceleration.
18.3.2 A(t) = G(t) = V(t)
When using arc length, we shall assume it is measured from the initial position
G(0). That is,

t
18.3.3 44 = J llV(~)ll
0

Thus for t 2 0, s ( t ) acts as an odometer, giving the distance that the particle has
traveled in its path at time t. Furthermore,

ds
18.3.4 -
dt
= IlV(t)ll

gives the instantaneous speed of the particle. Now T = V/llVll is a unit vector
that points in the direction of the motion at any instant. By writing the velocity
vector in the form

18.3.5 V = ($)T

we emphasize that the velocity is completely determined by the speed and


direction of the particle.

Example 1 The position function of a particle moving in the plane is given by G ( t ) =


( 2 t 2 , t 4 / 4 - 1). Sketchthe pathandsketch therepresentativesofV(1) andA(1).
Solution Eliminating the parameter, we havey = x2/1 6 - 1 and so the path is along this
parabola, starting at (0, - 1) and moving up the right branch as t increases. The
velocity and acceleration are given, respectively, by
610 CHAPTER 18: VECTOR VALUED FUNCTIONS

graph of G ( t ) = ( 2 t 2 , t4/4 - 1)

Fig. 18.31

V ( t ) = ( 4 4 t3) and A ( t ) = (4, 3t2)


Thus V ( 1 )= (4, 1) and A(1) = (4,3).
The path and representatives of V(1)and A ( l ) are sketched in Fig. 18.21. 0

According to Newton's second law of motion the resultant (sum) of all the forces
acting on a particle of mass m that is moving along some path in space is related
to the acceleration of the particle by the vector equation

18.3.6 F =mA

Example 2 A particle of mass m has position function


G ( t ) = ( a cos w t , 6 sin a t )
where a, b, and c3 are positive constants.
( a ) Describe the trajectory.
(6) Determine the velocity, speed, and acceleration.
(c) Determine the force acting on the particle.
Solutions ( a ) The path is the ellipse x 2 / a 2 + ~ ' / 6 ~= 1. The particle starts at (a, 0) and
travels in the counterclockwise direction.
(6) Now V ( t ) = ( --a@ sin at, 6 0 cos w t ) and
A ( t ) = ( -aw2 cos o t , -6a2 sin a t ) = - a 2 G ( t ) .
The speed is given by
IlV(t)lI = w d a 2 sin2 wt + b2 cos2 at
(c) By (18.3.6), F = m A = - m u 2 G 0

In the above example the force is proportional to the position vector. Any
force of this type is called a central force. If, as in Example 2, the constant of
proportionality is negative, then the force is called a centripetal or attractive
force. The centripetal force of Example 2 is illustrated in Fig. 18.22. If a = 6
central force: (a circular trajectory), then this is the sort of force exerted on a ball being
F = -nw2G whipped in a circular path at the end of a piece of string held fixed at one
Fig. 18.22 end.
SECTION 18.3: MOTION 61 1

PROGRESS TEST 1

1. Deduce from (18.3.6)Newtons first law of motion: A 2. Show that the force acting on the particle with
particle subject to no forces will remain at rest or position function G(t) = ( u cosh at, u sinh a t ) , for a,
will travel in a straight line with constant velocity. w > 0, is a central force.

THE PROJECTILE PROBLEM


Suppose a projectile of mass m is fired with an angle of elevation of a radians
and with an initial speed of u, feet per second. We wish to determine the path of
the projectile under the assumptions that the path lies in a plane and that the
only force acting on the projectile is the force due to gravity. (We are neglecting
forces due to wind and air resistance.) This situation generalizes the vertical
Y velocity explored in Sec. 6.1.
i We choose coordinates so that the x axis coincides with ground level. Suppose
that G ( t )= ( x ( t ) , y ( t ) ) denotes the position vector of the projectile with
G(0) = 0. That is, the projectile is fired at ground level from the origin. O u r
path of projectile object is to determine G(t).If V ( t )denotes the velocity of the projectile, then
V, = V(0) is the initial velocity. Since the initial speed is u, ft/s and the initial
direction is a , by (17.3.15), page 569, we know that
V(0) = (u,, cos a, u,, sin a)
T h e force of gravity F ( t ) acting on the projectile is directed vertically down-
ward, in the direction of -j. It has magnitude mg, where g is the gravitational
I ground level constant. Thus F ( t ) = -mgj = (0,- m g ) ; see Fig. 18.23.
Fig. 18.23 By (18.2.6),
-mgj = m A
so

(1) A = -g j
But A = G, so this is a vector differential equation that can be solved for G.
Integrating (1) we have
V ( t )= -gtj + C,
Letting t = 0, we conclude that C, = V(0) = V,, so

(2) V ( t )= -gtj + V,
Integrating again, we obtain
-gt2
G(t) = -j 2 + V,,t + C,
But G(0) = 0 = C,, SO

-gt2
(3) G(t) = -j 2 + Vot
We know that V, = (u,, cos a , u, sin a),so we can write ( 2 ) and (3) in compo-
nent form to obtain:

18.3.7 G(t) = ((u,, cos a ) t , (u, sin a)t - ( g / 2 ) t 2 )


and
18.3.8 V ( t )= (u,, cos a , u,, sin a - g t )
612 CHAPTER 18: VECTOR VALUED FUNCTIONS

Suppose that a = m / 2 . Then physically we are describing a vertically projected


object. In such a case

G ( t ) = ((uo)(O)t, (uo)(l)t - (g/2)t2>


= (0, vot - (g/2)t2)
Hence G ( t ) is equivalent to a scalar function. In fact it is equivalent both in
form and interpretation to the scalar position-above-the-ground function in (6.1.7).
Similarly, the velocity function reduces to
V(t) = P , v o - g t )
which again agrees completely with the vertical velocity function (6.1-6).
Henceforth, unless otherwise noted, we shall assume 0 a m/2. < <
Example 3 Show that the path of the projectile given by (18.3.7) is parabolic.
Solution By (18.3.7), x ( t ) = (vo cos a)t andy(t) = (vo sin a ) t - (g/2)t2. Since cos a # 0,
we can solve for t in terms of x ,
X
t=-
uo cos a
and conclude that

Since a , uo and g are constants, this function has the formy = Ax - Bx2,which
is the equation of a concave-down parabola. 0

Example 4 A projectile is fired from ground level with a n initial speed of 1000 ft/s and with
a n angle of elevation of m/6 rad. Determine: ( u ) the time when the projectile
strikes the ground, ( b ) the range of the projectile, and (c) the speed of the
projectile at the instant of impact.
Solution Since uo = 1000 and a = m / 6 , by (18.3.7),
G ( t ) = ( S o o f i t , 500t - (g/2)t2)
(u) The time of impact will occur when
~ ( t =) t[500 - (g/2)t] = 0
But t = 0 corresponds to the initial position, so the projectile strikes the
ground after t = lOOO/g = 31.25 seconds; see Fig. 18.24.
( b ) The range of the projectile is the horizontal distance traveled during the
flight. Since the projectile will not strike the ground until t = 31.25, the
range is
~(31.25=
) SOOfl(31.25) = 1 5 , 6 2 5 f i z 27,063.3 ft
(c) V ( t )= ( 5 0 0 f l , 500 - g t ) , so the velocity at impact is
t = 31 25
V(31.25) = ( 5 0 0 f i , 500 - 32(31.25))
= (500 fi,-500)
The speed at impact is
Y 31 25 = rang-e
Fig. 18.24 [lV(31.25)11= d(500fi)2 + (-500)' = 1000 ft/s 0
SECTION 18.3: MOTION 613

PROGRESS TEST 2

1. A projectile is fired from ground level with initial (c) Determine the maximum height the projectile
speed uo feet per second a t a n angle of elevation of a attains.
radians. ( d ) If uo is fixed and a is allowed to vary, then
( a ) Determine the time of impact. determine the angle of elevation that will yield
( 6 ) Show that the terminal speed equals the initial the maximum range.
speed.

TASGENTIAL AND NORMAL COMPOKESTS


OF ACCELERATION
Since acceleration is the derivative of velocity, it must reflect any changes in
either the direction or the speed of the object. The next result shows very vividly
how the acceleration is affected by such changes. Since F = m A , these changes
must also be reflected in the force.

18.3.9 Theorem If G ( t ) denotes the position function of a particle moving in space,


then

pro^ Frdm (18.3.5),V ( t ) = ( d s / d t ) T ( t ) .Thus, differentiating with respect to t,


we have
ds
(i;;) +
A(t) = - T -T(t)
dt
But
dT =-
T(t) = - dT -
ds
dt ds dt

= k(t)N(t)k (by 18.2.12)


dt
Substitution of this latter expression into (4) gives the desired result.

The scalar functions

18.3.10

are referred to as the tangential and normal components of acceleration, respec-


tively.

Similarly, the vector functions

18.3.1 1 qT and a
are called, respectively, the tangential and normal acceleration. The tangential and
normal forces are obtained by multiplying each function in (18.3.11) by m .

Since A = a,T + aNN we have


+
l/A/l2= (UTT a) (aTT a) +
= q 2 T T + 2aTaNT N + aK2N N
614 CHAPTER 18: VECTOR VALUED FUNCTIONS

But T * T = N N = 1 and T *N = 0 , so IIA112 = aT2 + aN2, and hence


18.3.12

Equation (18.3.12) is an excellent aid in computing a, and ax.

Example 5 Determine % ( t ) and a N ( t ) for the motion with position function G(t) =
( e t sin t, et cos t ) .
Solution Since G(t) = et(sin t , cos t ) , we know that
V ( t )= G(t) = et(cos t , -sin t ) + et(sin t , cos t )
= et(cos t + sin t , cos t - sin t )
and

-
ds = liv(t)11= let1 d(cos t + sin t ) 2 + (cos t - sin t ) 2 = Get
dt
Thus
d2s
aT(t) = - = G e t
dt2
Now
A ( t ) = V(t)
= et( -sin t + cost, -sin t - cost)
+ et(cos t + sin t, cos t - sin t ) = e t ( 2 cost, -2 sin t )
Hence
liA(t)li = IetI d4 cos2 t + 4 sin2 t = 2 2
By (18.3.12),
a N (t ) = d ( 2 e t ) 2 - (fiet)2 = d- = ~ e t

x Good Driving: Understanding the Laws of Motion Our bodies


experience the above tangential and normal force vectors riding in an auto-
mobile whenever we change our speed or direction. As the automobile speeds
up, the back of our seat pushes us forward with a tangential force maTT, where
m is our mass. Similarly, as we turn, the interior of the car forces us in the
direction of the turn with a normal force ma.
Let us now turn our attention to the normal force

F = Ma = Mk - N
(3 ( M = mass of car)

acting on the car as we travel around a curve at constant speed. Assuming the
car does not skid, this force acts on the car via friction where the tires meet the
road. If the curve is sharp, then k and hence F must be large. If the speed ds/dt
is large, then again F must be large. If both the curvature and the speed are

, large, then F will be very large. However, if the normal force necessary to turn
the car exceeds the force available through friction, then the car skids. In
particular, if the available force decreases because of rain, snow, ice, or bald
tires, then, for a given curvature k , ds/dt must be kept small to prevent skidding.
Whether or not we choose to obey the statutory driving laws, we have no
choice but to obey the laws of motion.
SECTION 18.3:MOTION 615

Y POLAR COMPONENTS OF ACCELERATIOS


Theorem (18.3.9) gives a decomposition of the acceleration into components
that are tangent and normal to the path, that is, components that are parallel
and perpendicular to the velocity vector. In the case of a plane motion, by
working in polar coordinates we can obtain a n equally useful decomposition of
A into a pair of components, respectively parallel and perpendicular to the
position vector.
Suppose G ( t ) = ( x ( t ) , y ( t ) ) , and let ( r ( t ) , d ( t ) ) denote polar coordinates for
( x ( t ) , y ( t ) ) .Then u ( t ) = (cos e ( t ) ,sin d ( t ) ) is a unit vector parallel to G(t).Since
ilG(t)ll = ir(t)i, we can write G in polar form as G(t) = r(t)u(t);see Fig. 18.25.
If v ( t ) = ( -sin d ( t ) , cos 8 ( t ) ) ,then v ( t ) is a unit vector perpendicular to u ( t )
that points in the direction of advancing 8. Figure 18.26 shows the situation
with representatives of u, v on the path. Now
Fig. 1 8 . 2 5
du = (-sin$,cos$)
- =v and d v = (-cosB,
- -sin$) = -u
dB d8
so

_
du - d u dB = v g
- -- and d v--
- -- - -u-dB
d v dB -
(5 1 dt d8 dt dt dt d8 dt dt
Since G = ru,

V =-d G = r-d u dr
+ -u
dt dt dt
d8 + -u
= r-v dr [by ( 5 ) ]
dt dt
Differentiating and again, using ( 5 ) , we have
d28
A = r-v dr dB + r--d$
+ --v dv + -ud2r dr d u
+ --
dt2 dt dt dt dt dt2 dt dt
I
= r-vd2$ dr do - r
+ --v + -d2r dr db
+ --v
Fig. 1 8 . 2 6
dt2 dt dt ($)2
- U
dt2 dt dt
Grouping terms, we obtain

18.3.13

KEPLERS LAWS O F PLANETARY MOTIOK


Early in the seventeenth century the German astronomer Johann Kepler used a
detailed study of data collected by the Danish astronomer Tycho Brahe to
conclude that:
I. The planets move in planar elliptical orbits about the sun, with the sun
located at a focus of the ellipse.
11. A planets position vector based at the sun sweeps out area at a constant
rate. (See Fig. 18.27, where the areas of the shaded regions are equal. This
implies that the planet takes the same time in going from P to Qas from R
to S.)
orbit of a planet 111. The ratio ra3/T2is the same for all planets, where r, is the average of a
about the sun planets shortest and longest distances from the sun and T is the time
Fig. 1 8 . 2 7 necessary to complete an orbit.
616 CHAPTER 18: VECTOR VALUED FUNCTIONS

In the previous century the Polish astronomer Copernicus had switched the
roles of the earth and the sun as the center of the universe, thereby overthrow-
ing the 1500-year-old Ptolemaic view that the sun revolved about the earth.
However, it was Kepler who saw clearly (but without a telescope) the elliptical
motion of the planets. Then, approximately 60 years later, Newton followed
Keplers giant step by perhaps a n even greater step when he showed how these
laws follow from a universal inverse square law of gravitational attraction.
We shall show how (18.3.13) and the inverse square law imply conclusion I1
above.

We set up our coordinate system so that the sun is a t the origin. Let G ( t )
9
denote the position of the planet and assume Q(0)= 0. Then, as above, we have

\ /position
t
G(t) = (rit),Q ( t ) )

of planet at time t
G = ru (see Fig. 18.28). Now Newtons law of gravitational attraction asserts
that the force acting on the planet is a central force. Thus F = c G = cru. But
F = mA, so A is a scalar times u. By (18.3.13), the only way this can happen is
if the coefficient of v is zero. That is,

72-
d2$ + 2--dr dQ = 0
(6) dt2 dt dt
Now let w = dQ/dt. Then ( 6 ) becomes
dw
-
1(
x r-
dt
+ 2w-drdt =0
I /
position of planet so
at time t = 0
Fig. 18.28 rdw + 2wdr = 0
and

Integrating, we have

so

It follows that

Y (7) r2w = r 2 @ is constant


dt
4
Now if A = A ( t ) denotes the area (see Fig. 18.29) swept out by the position
vector from time t = to to time t and 8, = $ ( t o ) ,19 = Q ( t ) ,then, by (13.3.2), our
formula for area in polar coordinates, page 4 4 8 , we have

A = -s
l e
$0
r2d0

But dA/dQ = $r2 by the Fundamental Theorem of Calculus, so by the Chain


Rule

-
dA - dA dQ -
- -- r 2 - dQ
- -1
dt dQ dt 2 dt
area swept out from
time to to time t which, by (7), is constant. Since dA/dt is the derivative with respect to time of
Fig. 18.29 the area swept out, Keplers second law is established.
SECTION 18.3: MOTION 617

SECTIOS 18.3 EXERCISES

In Exercises I to I O j n d the veloci4, acceleratzon, and speed of the particle whose position function is given ly G ( t ) .
1. +
G ( t ) = ( t 2 1, 2t - 3 , t ) 2.G ( t ) = (eet, ezt)
3. G ( t ) = (In sin t , In cos t ) 4.G ( t ) = ( 2 sin 2t, 2 cos 2t, t )
5. G ( t ) = ( 3 sin t, 4 cos t , t ) 6.G ( t ) = ( t cos t , t sin t )
7. G ( t ) = ( e c t sin t, ect cos t ) 8.G ( t ) = ( t cos t +
sin t, sin t - t cos t )
9. G ( t ) = ( e t + ect, et - e c t ) 10. G ( t ) = ( 3 t , 7t - t2, 2 t )

In Exercises I I to I5 determine the position function G ( t )from the given information.


11. V ( t ) = ( 3 - 2t,4 + 3 t ) , G(0) = (1, -1) 12. V ( t ) = ( t 2 + 1, 3t3), G(0) = ( - 4 , 1)
13. A ( t ) = (0,- 8 ) , V(0) = ( 3 , l ) , G(0) = (0,O) 14. A ( t ) = ( 3 sin t , 3 c o s t ) , V(0) = (1, - l ) , G(0) = ( 3 , 2 )
15. A(t) =(&, 12+, V(0) = ( 2 , 2 ) , G(0) = ( 1 , 3 )

In Exercises I6 to 20jind the normal and tangential components of acceleration for the given motion.
16. G ( t ) = ( 3 sin t , 3 cos t ) hopes that he will land on a special cushion located 200
17. G ( t ) = (sin(t2), c o s ( t 2 ) ) ft down range. What angles of elevation are possible to
18. G ( t ) = ( 4 t , t 2 , 2 t 2 ) ensure Dumbo's safe return?
19. G ( t ) = ( e e t , e t ) 25. A building used to house Dumbo's circus (see Exercise
20. G ( t ) = ( e t cos t, et sin t, t ) 2 4 ) has a flat ceiling 75 ft high. Dumbo still wishes to
21. A projectile is fired from a point k feet above the ground land on his cushion 200 ft away, but he has no desire to
with a n initial speed of vo ft/s and a t a n angle of eleva- go through the ceiling in the process. What angle of
tion a with the horizontal. Determine the position func- elevation should he instruct his helpers to use?
tion for the projectile. 26. O n the 4th of July a special firecracker with a n adjusta-
22. J. N. is a n excellent golfer. When he swings full with any ble time fuse is fired from the top of a building 100 ft
iron he imparts an initial speed of approximately 176 high with an initial velocity of 480 ft/s and at a n angle
ft/s on the golf ball. When he uses a one-iron the angle of elevation of 7 / 6 rad with the horizontal. The fuse is to
of elevation at which the ball leaves the ground is be set so that the firecracker will explode at the instant it
approximately 3 0 " . How many yards does J. N. hit a reaches its highest point. For how many seconds should
one-iron? the fuse be set? If the fuse malfunctions and the fire-
23. Higher number irons are known to impart a higher cracker does not explode, how many feet from the base
angle of elevation. J. N. (see Exercise 2 2 ) can hit a wedge of the building will it land?
(10-iron) 120 yd. What angle of elevation does his wedge
impart the golf ball?
24. Dumbo, the human cannonball, is fired from his cannon
with a n initial speed of 80( n / 3 ) ft/s. H e has high

In Exercises 27 to 3 0 j i n d the polar components of acceleration for the motion of the particle with position function G ( t ) = ( r ( t ) ,8 ( t ) )
27. r ( t ) = 2 cos 4 t , O(t) = 2t 28. r ( t ) = 1 - cos 3t, 8 ( t ) = 3t
29. r ( t ) = 2 sin 2t2, 8 ( t ) = t 2 30. r ( t ) = e2t, 8 ( t ) = 2t

CHAPTER EXERCISES

R e v i e w Exercises
In Exerczses I to I0 determzne F ' ( t ) and F"(t).
1. F ( t ) = ( t 2 - In t, t - 3 t 3 ) 2. F ( t ) = ( t e c t , cos(t2))
3. ~ ( t =) ( 4 t - fi,(In t ) 2 ) +
4. F ( t ) = [ t / ( t 2 1)]i + (arctan t ) j
5. F ( t ) = (arcsin t ) i - csc(t2)j 6. F ( t ) = (sin Zt, cos fi, t2)
7. F ( t ) = ( d m ,l / t , e t 2 ) 8. F ( t ) = ( t + l/fi, 4 - 3/t, 4t3)
9. F ( t ) = ( G, ln(t2 + 2 ) , e f i ) 10. F ( t ) = ( t 2 - 4 / t ) i + ( l / t 2 - $)k
618 CHAPTER 18: VECTOR VALUED FUNCTIONS

In Exercises 11 to 20 determine F ( t ) from the given information.


11. F(t) = ( 1 2 t 2 - 4 t , - 2 5 t 4 ) , F(0) = ( - 3 , 5 ) 12. F(t) = ( l / d m , t d m ) , F ( 0 ) = ( 7 1 / 4 , 2 / 3 )
13. F(t) = ( l / ( t +
l ) , e t ) , F ( 0 ) = (4, - 3 ) 14. F(t) = ( 1 2 t , 24(1 - t 2 ) ) , F(0) = ( 3 , l ) , F(0) = ( 2 , 0 )
15. F(t) = (sin t , 1 8 t 2 ) , F ( 0 ) = ( 2 , - l ) , F(0) = (4, 3 ) 16. F(t) = ( 3 t 2 , sin t , sec t ) , F(m/4) = (0,1 , 2 )
17. F(t) = ( 4 t + 2, t 2 - 3t, 4 ) , F ( 0 ) = ( 2 , 1 , 4 ) 18. F(t) = ( l / ( t 2 +
I), 1 / I / n , d m ) ,
19. F(t) = ( 1 8 t , sin t , 1 2 t 2 ) , F(0) = (0,0,O), F(0) = (3,4,7)
F(0) = ( 2 , 3 , 1 )
20. F(t) = ( 3 t 2 - 2t, I / ( t + 1 ) 2 , 4 t ) , F ( O ) = (O,O,0 ) , F(0) = ( 2 , 1 , - 1 )

In Exercises 2 1 to 30 sketch the graph of F.


21. F ( t ) = ( t 2 , 3t - 6) 22. F ( t ) = ( t 2 ,t6)
23. F ( t ) = ( e t sin t , et cos t ) , 0 2 t 5 2 7 24. F ( t ) = (c0s3 t, sin3 t ) , 0 5 t 5 271
25. F ( t ) = (sect, tan2 t ) , - a / 2 < <
t m/2 26. F ( t ) = (4 sin 2t, 3 cos 2t, t ) , 0 5 t 5 277
27. F ( t ) = ( 2 cos t, 4 sin t , 2 t ) , 0 5 t 5 2 v 28. F ( t ) = ( t cos t, t, t sin t ) , 0 5 t 5 2~
29. F ( t ) = 2ti + t3j + 3 t 2 k , - 1 5 t 5 1 30. F ( t ) = 2(sin t ) i + e(cos t ) j + 4(sin t ) k , 0 5 t 5 7/2
I n Exercises 31 to 35 determine T ( t ) and N ( t ) and sketch their representatives at t = to.
31. F ( t ) = ( 3 cos 2t, 3 sin 2 t ) , to = a / 4 32. F ( t ) = (4 - 3t, t 2 ) , to = 1
33. F ( t ) = ( fi,t 2 ) , to = 1 34. F ( t ) = ( t 2 / 2 , t 3 / 3 ) , to = 0
35. F ( t ) = ( e c t , e t ) , to = 0

I n Exercises 36 to 40 determine T ( t ) , N ( t ) , and B ( t ) and sketch their representatives at t = to. Determine k(to).
36. +
F ( t ) = 6(sin 2 t ) i ~ ( C O S2 t ) j + 5 t k , to = a / 2 37. F ( t ) = ( 2 - 3t)i + 4tj + 2t3I2k, to = 1
38. F ( t ) = ( e t , et cos t , e t sin t ) , to = a 39. F ( t ) = ( 4 sin t , 2t - sin 2t, cos 2 t ) , to = 71/2
40. F ( t ) = ( t 2 , t3, t ) , to = 0

In Exercises 41 to 50 determine k ( t ) .
4 1. F ( t ) = (cos 3t, sin 3 t ) 42. F ( t ) = ( 3 cos t, 5 sin t )
43. F ( t ) = ( 2 t +
4, t 2 - 2 ) 44. F ( t ) = ( 2 t sin t, 2t cos t )
45. F ( t ) = (cos t +
t sin t , sin t - t cos t ) 48. y = x4/4 +
1/(8x2)
47. y = v5 48. y = e-
49. y = l / x 50. y = 2x2 +
x - 1

I n Exercises 51 to 57 determine the tangential and normal components of acceleration.


51. G ( t ) = ( 3 cost, 4 sin t ) 52. G ( t ) = ( 2 t + 4, t2 - 2 ) 53. G ( t ) = ( t 2 , fi) 54. G(t) = (tsint,tcost)
55. G ( t ) = (cos t + t sin t , sin t - t cos t ) 56. G ( t ) = ( t , t2, t 3 )
57. G ( t ) I= ( 4 cos t , 9 sin t , t )

I n Exercises 58 to 62 sketch the circle of curvature at the indicated point.


58. y = x 4 / 4 , x = 1 59. y = x3/3, x = 1 60. F(t) 1 ( 2 t 2 ,2 t ) , t = 0 61. F(t) = (l/t, t), t =1
62. F ( t ) = ( 2 cos t, 3 sin t ) , t = m/2

I n Exercises 63 to 65 determine the evolute ofthe indicated vector function. (For a dejinition of the evolute of a function see the discussion following
Exercise 3 6 in Sec. 18.2).
63. F(t) = (2t2,2t) 64. F(t) = ( l / t , t ) 65. F ( t ) = ( 2 cos t , 3 sin t )

Miscellaneous Exercises
1. Show that if F is twice differentiable, then [F F] = 3. Suppose that the graph of a smooth curve does not go
+
IIFI/* F F. through the origin. Show that F(to) I F(to), where F ( t o )
2. Suppose that F, G, H are differentiable. Show that is the point on the graph closest to the origin.
[F (G x H)]
-
-F*(GXH) +F*(GXH)+F*(GXH)
SECTION 18.3: MOTION 619

4. Express the curve 11. T h e space curve F ( t ) = (3 cos t , 3 cos 2t, 3 sin t ) can be
realized as the intersection of two quadric surfaces. Use
F(t) = ( ~ ( C O St + t sin t ) , 2(sin t - t cos t ) ) this observation to sketch the portion of the graph of F
using arc length as a parameter measured from the point in the first octant.
corresponding to t = 0. 12. Find parametric equations for and sketch the space
5. T h e motion of a particle in the plane is given by curve joining the points (0, 0,9)and (1, 3, - 1) obtained
F ( t ) = ( C O S ~t , sin3 t ) , 0 5 t 5 a/2. After how many by intersecting the surfaces with equations
seconds will the particle have traveled 3/8 of a unit x 2 + y 2 + z = 9 a n d y = 3x.
along the curve? 13. Find T, N, and B at any point on the space curve
6. At what point on the curvey = 6 x - x 2 is the curvature <
F ( t ) = ( 2 sin t , 2 cost, - 2 ln(cos t ) ) , It1 77/2.
a minimum? 14. Show that the acceleration vector of a particle moving
7. A hill has angle of elevation a/6 with the ground. A in space has length
projectile is fired with initial speed v,, feet per second
from the base of the hill (toward the hill) a t a n angle of
elevation a ( > a / 6 ) . How far down range will the pro-
jectile have traveled when it strikes the hill? 15. A projectile is designed to curve leftward with a n accel-
8. At what angle of elevation should the projectile in eration of L feet per second squared. Use a three-dimen-
Exercise 7 be fired in order to maximize the range? sional analysis to describe the motion of such a projec-
9. A particle moves along a curve in such a way that the tile.
tangent vector at each point on the curve is always 16. Let F be a vector function. Show that limt+ F ( t ) = L if
perpendicular to the standard representative to that and only if for every e > >
0 there exists a 6 0 such that
point. Describe the curve. llF(t) - LII <
E whenever 0 <
1 t - to 1 6. <
10. The vector function F ( t ) = ( t cos t , t sin t , 9 - t 2 ) lies
on a particular quadric surface. Use this observation to
sketch the graph of F.

T h e representatives of T(t) and N ( t ) with their bases at F ( t ) on the graph of a space curve F determine a plane called the osculating plane at F(t).
In Exercises I 7 to 19 determine the osculating plane at the indicated point.
17. F ( t ) = ( e t , et cos t , et sin t ) , t = 7i/2 18. F ( t ) = (cost, -sin t , t ) , t = a / 2
19. F ( t ) = ( 2 cos t , 2 sin t , t ) , t = a / 6 20. Discuss the behavior of the circle of curvature near a
point of inflection on a plane curve.

SELF-TEST +
Chapter 19 +
622 CHAPTER 19: FUNCTIONS OF SEVERAL VARIABLES

Section 19.1 : Functions of Several Variables

Objectives: FUSCTIOSS O F TWO VARIABLES


1 . Determine the domain of Often in geometry and in the physical, biological, and social sciences, because
functions of two or three variables. of the inherent interconnectedness of phenomena, one quantity depends upon
more than one other quantity. We shall extend our definition of function to
2. Sketch the graphs of functions
cover such situations.
of two variables.
As before, R denotes the real numbers, R, denotes the set of ordered pairs
3. Sketch the level curves of a ( x , y )of real numbers, and R, denotes the set of all ordered triples (x,y,z ) of real
function of two variables and the numbers. We assume the usual identification between R, and points in space.
level surfaces of a function of three Note that in this chapter we are interested in ordered pairs and ordered triples
variables. considered as points rather than as vectors. We begin with functions of two
variables.

19.1.1 Definition Afunction f of two variables is a rule that assigns to each point ( x , y )
in some subset D of R, a unique real number denoted by f ( x , y ) . The set D of
points to which f applies is called the domain of5 and the set of all numbers
f ( x , y )resulting when f is applied to these points [as (x,y)varies over D ] is called
the range of J: We shall sometimes denote the range and domain off by R (f)
and D (f), respectively.

In practice we often do not specify D ( f),but rather assume that f has its
natural domain. That is, D ( f ) is the set of all points ( x , y ) that yield a real
number when substituted into the rule for$

Example 1 Determine the domain of the function


f ( x , y ) = d 1 4 4 - 16x2 - 9y2
Solution A point ( x , y ) is in D ( f) if and only if 1 6 2 + 9y2 5 144, or equivalently,
x2 y2
-+-<1
9 16 -
Thus D ( f)consists of all points inside or on the ellipse with equation
x2 y*
s + = = 1 (see Fig. 19.1) 0

1 J(x,s) = J144 - 16x2 - 9~~

Fig. 19.1
SECTION 19.1: FUNCTIONS OF SEVERAL VARIABLES 623

x =

domain of J : x , r ) =
4 - 2x
~

Fig. 19.2

Example 2 Determine the domain of the function

Solution For f(x,y) to be defined we must have 9 - x 2 2 0 and y - 2x # 0. Now


9 - x 2 2 0 if and only if 1x1 5 3. Thus the points in D ( f ) must lie in the
vertical strip extending between and including the points on the lines x = - 3
and x = 3. In addition, sincey - 2x # 0, we must discard fom this set all points
on the liney = 2x. A sketch of the domain is given in Fig. 19.2. We have used
dashes to indicate that the points alongy = 2x are not to be included in
D(f)*

GRAPHS
19.1.2 Definition The graph of a function f of two variables consists of all the
4 points (x,g,f(x,y)) in R, as ( x , y ) varies over D(f ) in R,.
4
Note that the graph of the functionfis the same as the graph of the equation
t = f ( x , y ) .In
this setting the domain D(f)will be some subset of the x-y plane
and the graph offwill, in general, be a surface. If (x,y) is in D ( f ) ,thenf(x,y)
gives the directed distance from the x-y plane to the point (x,y,f(x,y)) on the
graph of$ Figure 19.3 shows a typical situation where f(x,y) 2 0 for (x,y) in
*(f 1.
When we write a function in the form t =f(x,y), we refer to x a n d y as
independent variables and z as the dependent variable. In many instances the
L D ( f ) problem of graphing a function of two variables is simplified by using our
Fig. 19.3 knowledge of equations in R, developed in Chap. 17, especially Sec. 17.2.

Example 3 Sketch the graph of the functionf(x,y) = d m .


Solution Letting z = VW
and squaring both sides, we have
2 2 = x2 + y2
624 CHAPTER 19: FUNCTIONS OF SEVERAL VARIABLES

+J

Fig. 19.4

which from (17.2.7) or Sec. 17.2, Example 4, we know is the equation of a


circular cone whose axis is the z axis. The graph of z = v ' m is the upper
nappe of this cone (see Fig. 19.4). 0

The following example does not lead to one of the familiar quadric surfaces,
but the techniques for obtaining the graph are much the same.

Example 4 Sketch the portion of the graph off ( x , y ) = ex + y that lies in the first octant.
Solution Restricting our attention to the first octant is equivalent to restricting the
domain o f f t o the first quadrant in the x y plane. Hence the graph off is the
+
graph of the equation z = ex y , x,y 2 0. The traces in the x-z andy-z planes
( y = 0 and x = 0, respectively) are the curves z = ex and z =y +
1, respec-
tively. Since x 2 0, we know ex 2 1, and hence there is no trace in the planes
t = k unless k 2 1. Some typical traces are given in Fig. 19.5. Using these we
obtain a sketch of the graph o f f i n Fig. 19.6. 0

!
-
z = ex z
1 d
X J t
trace in x-z plane trace in]-z plane

xx
+ .J
octant

trace in plane z = k ( k 1 1:
I-i
trace in plane j = k 1 k > 0) Fig. 19.6
Fig. 19.3
SECTION 19.1: FUNCTIONS OF SEVERAL VARIABLES 625

PROGRESS TEST 1

1. Determine and sketch the domain of

2.
f ( W )= m
Sketch the graphs of the following functions.
(4 f(X,Y) = 4 - 2x -Y
( b ) f(x,JJ) = d3x2 + 4y2 - 12

FUNCTIOSS OF SEVERAL VARIABLES


Definition (19.1.1) easily extends to functions of three, four, or, for that matter,
any finite number of variables. For the most part we shall be concerned with
functions of two or three variables. A function of three variables will be de-
noted by
w = f(X,Y,4
where the domain is now a subset of R,. In this setting, w is the dependent
variable and x, y, and t are the independent variables. For example, the
function

is well defined if and only if


x2 Sy2 + 22 <4
Hence the domain off is the set of all points inside the sphere of radius 2
centered at the origin. Note that none of the points on the sphere itself is in
D(f)because any such points will result in a zero denominator.
Whenever we have occasion to refer to functions of four or more variables,
we denote the independent variables by xi and write

w =f(xI: . . , xn)
~ 2 .,

The domain in this case will consist of some collection of ordered n-tuples of real
numbers. That is, D(f)is a subset of R,, where R, denotes the collection of all
ordered n-tuples of real numbers ( x l , x2, . . . , x , ) . Of course, for n 2 4, D (f)
has
no geometric realization.
You have at your command rather elaborate machinery for sketching graphs
of functions of one variable. However, sketching the graph of functions of two
variables is considerably more difficult. For functions of three or more variables
it cannot be done since the graph will be a subset of R,, where n 2 4.
The fact that we cannot picture a function of three variables often leaves
students rather ill at ease when dealing with them. Nonetheless, they are a
traditional part of the elementary calculus and arise very naturally in applica-
tions. Notice, for example, that the density of a nonhomogeneus substance
varies depending on location, and hence is a function of the three variables
necessary to determine the location.

LEVEL CURVES AND LEVEL SURFA3CES


As noted, the graph of a function of two variables is in general a surface in
three-space. We can obtain information about this surface (and hence about the
function) by examining the curves that result when we slice the surface with
planes parallel to the xy plane. Such curves, which are sketched by projecting
them into the xy plane, are called level curves of the function. More precisely we
have:
626 CHAPTER 19: FUNCTIONS OF SEVERAL VARIABLES

level c u r v e f ( x , j ) = k,
level c u r v e J ( x , j ) = k,
Fig. 19.7

Definition Letf(x,y) be a function of two variables and let k be a constant.


The graph in the x y plane of the equation f ( x , y ) = k is called a level curve off
(see Fig. 19.7).

Level curves are what cartographers use in drawing contour maps of a


region. Think of the terrain that makes up the region as a surface. The level
curves (contour curves) that appear on the map are marked to indicate the

@
3500 altitude. The curves marked 3000 represent the curve of intersection of the
terrain (considered as a surface) with a horizontal plane of altitude 3000 ft.
Level curves appearing very close together on the map indicate steep terrain,
and level curves that are far apart indicate flatter terrain. A typical situation is
pictured in Fig. 19.8(a) and ( b ) . In both cases the difference in altitude is 500 ft.
!a) The curves in (a) reflect the steepness of the mountain while those in ( b ) reflect a
Fig. 19.8 more gradually sloping hill.

Example 5 Sketch some level curves of the surface


f(x,r) =Y - 2x
The curves y 2 - 2x = c are parabolas opening to the right, and since
y 2 = 2(x + &), the vertices are on the x axis at (-&,O). Some of the level curves
corresponding to various values of c are shown in Fig. 19.9. 0

3'

(bj

f(x,y) = y- 2x
Fig. 19.9
SECTION 19.1; FUNCTIONS OF SEVERAL VARIABLES 627

Suppose a metal plate occupies a region in the xy plane, and suppose the
function T(x,y) gives the temperature distribution of the plate a t a given
instant. That is, at this particular instant the temperature at the point on the
plate with coordinates (x,y)is T(x,y).For a given c, the level curve T ( x , y ) = c
identifies those points on the plate having the same temperature c; these curves
are called isotherms, or isothermal curves. Similarly, the surface pressure (corrected
for different altitudes) over the United States varies as a function of location,
yielding a function of two variables whose level curves, commonly called isobars,
are the curves one normally sees on daily weather maps. These curves consist of
points having equal barometric pressure. The closer together the isobars, the
greater the differences in pressure, and, since air moves in such a way as to
equalize pressure, the greater the wind.
If a solid metal object is heated, then the temperature distribution is a
function of three variables, and we speak of isothermal surfaces T ( x , y ,z ) = k. In
general, for functions of three variables we have:

19.1.4 Definition Iff(x,y, z ) is a function of three variables and if k is a constant,


then the surface with equation

is called a level surface of$

Example 6 Examine the level surfaces of the function

Solution If k = 0, then the level surface is t = 0, the x-y plane. For k f O ,f ( x ,y ,z ) = k


becomes
z = k(x2 + y 2 )
which has as its graph a paraboloid with vertex at the origin. The paraboloid
opens about the positive t axis if k >
0 and the negative z axis if k <
0. The
larger lkl, the steeper the paraboloid. Some typical level surfaces are sketched in
Fig. 19.10, 0
2

k = 4

1 k = -4

level surfaces of ~ , r . j2 .) =
x2 + J2
Fig. 19.10
628 CHAPTER 19: FUNCTIONS OF SEVERAL VARIABLES

PROGRESS TEST 2

1. Determine the domain of the function 3. Sketch the level surfaces for the function
w = cos x + siny + a r c s i n d m Describe the IYI = z 2 - x2 - y 2 corresponding to c = 0, 1, - 1.
domain geometrically.
2. Sketch the level curves for the function t = x/yZ
corresponding to c = 0, 1, - 1.

SECTION 1 9 . 1 EXERCISES

In Exercises 1 to 1 0 j n d the domain of the given functions.


- x2
d3x2+ 5y2 - 15
-
y2
1. z = 2. z=- 3. z=?-l 4. z = f i
Y-" Y
5. z = ln(x + y ) 6. z = arcsin 7. z = ln(4x - y ) 8. z = d12 - x 2 - y

9. w = x
Y-Z

In Exercises 1 I to 20 sketch the graph of the given function.


11. z=x2+ 2y2 12. z = 9 - x 2 13. z = x 3 + y 14. z = xy
15. z = d w 16. z = - d9 - x2 -y2 17. z = y 2 - x2 18. z = dx2+ 2y2 - 1
19. z = d x 2 +y2 +1 20. z = 2% + eU

In Exercises 2 1 to 25 describe the level curves.


21. z = x 2 + 9y2 22. z = d9 - 3x2 - 2y2 23. z = xy 24. z=x2+y
x - 2
25. 2 =-
Y + l

In Exercises 26 to 30 sketch the level sugaces corresponding to the indicated values of c.


26. w = x 2 + y 2 + t 2 ;c = 0 , 1 , 4 27. w =x +y + 22; c = kl,t 4
28. w = x 2 + y 2 - 2z; c = 0,1, --I 29. w = x2 + 2y2 + 3z2; c = 0 , 1, -1
30. w = xyt; c = 1

Section 19.2: Limits and Continuity

Objectives: LIMITS
1. Determine limits of functions of Before introducing the concept of a limit for functions of several variables, it is
two variables. instructive to revisit the single-variable setting. Suppose the domain of f
contains a deleted open interval about the point a. We recall that
2. Determine points where a
function of two variables is limf(x) = L
X-0.
continuous
if for every E > 0, there exists a 6 > 0 such that
(1) 0 < Jx - a1 < 6 implies if(x) - Ll < E

Now in R, 1% - y / is the distance between x andy. If we denote this distance by


d(x,y), then we can rephrase (1) as

(2) 0 < d ( x , a ) < 6 implies d ( f ( x ) ,L ) < E

Since we also have a notion of distance between points in R, given by

d[(x,,y,), (x2,Ydl = d(x2 - .,I2 + (yz -Y1)2


SECTION 19.2: LIMITS AND CONTINUITY 629

it is fairly simple to formulate (2) for functions of two variables. The only
problem that remains is to specify the two-dimensional analog of the statement
that fcontains a deleted open interval about the point a.

19.2.1 Definition Suppose P = (a, b ) is a point in R, and r is a positive number.


The open disk S,(P) is the collection of all points ( x , y ) such that
<
d [ ( x , y ) ,(a, b ) ] r. A set of the form S,(P).with the point P removed is called a
deleted open disk about (a, 6 ) (see Fig. 19.11).

19.2.2 Definition Supposefis a function of two variables whose domain contains


some deleted open disk about (a, b). Let L be some real number. We say

if, given any E > 0, there exists a 6 > 0 such that


0 < d [ ( x , y ) ,(a, b)l < f3 implies d [ f ( x , y ) ,LI < E
As before, if
open disk S,(P) deleted open disk
lim f ( x , y ) = L
Fig. 19.1 1 y)-(a,
(8, b)

then we sayf(x,y) has limit L or tends to L as (x,y) tends to (a, 6). We sometimes
denote this byf(x,y) + L as (x,y) + (a, b ) .

Graphically, the functionfhas limit L as (x,y) tends to (a, b ) if, given any
open interval ( L - E , L +
E ) on the z axis, one can exhibit a deleted open disk
S,(a, b ) about (a, b ) in the xy plane with the property that every point in this
disk gets sent b y f i n t o the interval ( L - E , L +
E ) ; see Fig. 19.12.

Example 1 Show that

Solution Since we are taking the limit at (0,O) we can assume that (x,y) # (0,O). For all
such points, x 2 / ( x 2 + y 2 ) 5 1, and hence
630 CHAPTER 19. FUNCTIONS OF SEVERAL VARIABLES

f(x,y) = L and if C is any smooth curve through (a, b ) , then it can


If lim(z,u)+(o,o)
be shown that f ( x , y )will tend to L as (x,y)tends to ( a , b ) along C (see Fig. 19.13).
In particular, if letting ( x , y ) tend to ( a , b ) along two distinct curves through ( a , b )
points in the x - j plane tending leads to different values for L , then lim(z,u)+(u,b)f(x,y) does not exist. (This corre-
to the point :u, b ) along C sponds to the one-dimensional criterion that if lim,,,+ f( x ) # limz+u-f( x ) , then
Fig. 1 9 . 1 3 lim,,,f(x) does not exist.)

Example 2 Show that


x2 - y2
lim
x2 + y2
~

(z,2()-(0,0)

does not exist.


Solution Let L, be the straight line through the origin with slope m. Then (x,y)is on L,
Y if and only ify = mx. Also, ( x , m x ) + (0,O)along L, as x -+ 0 (see Fig. 19.14).
t Since we are investigating the limit at (0, 0), we can assume x # 0. Now

and so clearly f ( x , m x ) + (1 - m2)/(1 + m 2 ) as x + 0. Since this leads to dif-


,. x , mx, ferent results for different choices of m,fdoes not have a limit at (0,O). 0

aiib ,o, 0) The usual limit theorems remain valid for functions of two variables.
,i That is, if

Fig. 1 9 . 1 4

Iff is a function of a single variable whose domain contains the range of the
function g of two variables, then we can form the composition function f o g by
means of the rule
SECTION 19.2: LIMITS AND CONTINUITY 63 1

PROGRESS TEST 1

1. Let f ( x , y ) = -. x3 2. Use definition (19.2.2) to prove that


x4 + y2
( a ) Determihe the limit of f ( x , y ) as (x,y) tends to lim ( x ) = a
(2,Y)+(U,b)
(0,O)along any straight line through the origin.
( 6 ) Determine the limit off(x,y) as (x,y) tends to
(0,O) along the parabolay = x 2 ,
(c) Does lim(,,y)-(O,o)f(X,Y)exist?

CONTINUITY

A point P is said to be an interior point of a set D if D contains some open disk


about P. If all the points of a set are interior points, then the set is called open. A
point P with the property that every open disk about P contains at least one
point in D and one point not in D is called a boundary point of D , and the
collection of all such points is called the boundary of D. Boundary points of a set
need not be elements of the set. If a set does contain all of its boundary points,
then it is said to be closed. We illustrate several of the possible situations in Fig.
19.15, where we used dashes for those portions of the boundary that are not in
the set.

19.2.4 Definition Supposefis a function of two variables and ( a , b ) is an interior


point of D ( f ) , the domain of$ T h e n f i s said to be continuous at ( a , b ) if

A function f is said to be continuous on an open set D if it is continuous at each


point of D.

oundarv pomts

open set closed set neither open nor closed


Fig. 19.15
632 CHAPTER 19: FUNCTIONS OF SEVERAL VARIABLES

Note that this definition is essentially the same as that for the one-variable
case (2.5.7). That is, the function is continuous at a point if it has a limit at this
point that equals the value of the function at the point. The graph of a
continuous function of two variables is a surface that has no holes or breaks.
From the definition and the limit properties it can be shown that sums,
differences, scalar multiples, products, quotients (where the denominator is not
zero), and compositions of continuous functions will themse1ves:be continuous.
In particular, polynomial functions of two variables, such as f(x,y) = 4 . ~ 5 ~ +
2xy3 + 6y2 + 9, are everywhere continuous. A rational function of two variables
(that is, a quotient of polynomial functions) is continuous at all points where
the denominator is not zero. As a result, a function such as

is continuous everywhere since it is the composition of the continuous cosine


+
function with the continuous rational function ( x 2 2xy - y 2 ) / ( x 2 y 2 1). + +
In particular, limits of such functions can be obtained by direct evaluation.
Thus

FUNCTIONS OF THREE VARIABLES


All statements of this section can be routinely extended to functions of three
variables. In this setting, distance between points is given by

d[(xl,Yl, = d(x2 -
ZI), ( X 2 , Y 2 , Z211 + ( Y 2 - y J 2 + (22 - 2d2
and if P = (a, b, c), then S,(P), called the open ball about P, consists of all points
that are less than E units from P.

SECTIOK 19.2 EXERCISES

In Exercises I and 2 use the dqinition to establish the given limit.

1. lim
h,d-(l,-Z)
(3x - 4y + 1) = 12
f ( x , y )does not exist by exhibiting distinct curves through (a, 6 ) with the property that f (x,y ) tends to distinct
In Exercises 3 to 6 show that lim(c,u)+(a,b)
limits as (x,y) + (a, 6 ) along these curves.

3- f ( x , y ) = - ( a > 6 ) = (0,O)
x +y

5- f(X,.Y) = m>
x2 + y
(a, 6) = (0,O)

In Exercises 7 to 12 use the limit theorems to evaluate the given limits.

7. 8. lirn
x 3 + 9x + 6
(e,y)-(O,O) 3 + x* + y 4
9. 10. lim
(e,y)-(3,1)
3x2 + x/y
11. lim 8xy2 + 2y 12. lim
4x2 +5
(2,2/)-(2,-1) x + 3y (z,y)-(0,0)
~

xy +9
13. Show how (19.2.3) can be used to establish the continu-
ity of f ( x , y ) = sin(x + y ) at (0, 0).
SECTION 19.3: PARTIAL DERIVATIVES 633

In Exercises 14 and 15 determine whether f( x , y ) is continuous at ( a , b )

16%. Show that lim(c,y)+(2,5)


(3x + 4y) = 26. 17. Show that open disks are open sets.
18. Give a definition of continuity at the point ( a , b, c ) for
Hint:
the three-variable function f( x , y , t ) .[Parallel (19.2.4).]
(i) Use the triangle inequality for the absolute value to
19. Give a definition of limit for functions of three variables
show that
paralleling (19.2.2).
d[(3x + 4 ~ )261
, 23 ) -~ 21 + 4)y - 51 20. Use Exercise 19 to prove that lim(,,v,z)-(a,a,c) ( y ) = b.
(ii) Note that

= d [ ( x , y ) , (2,5)1
Obtain a similar inequality involving ly - 51 and com-
bine with (i) to show that
4(3x + 491, 261 I 7 d [ ( x , y ) ,(2,511

Section 19.3: Partial Derivatives

Objectives: INTRODUCTION
1. Determine the partial derivatives For a function of a single variable the derivative measures the rate of change of
of a function of two or three the function with respect to its independent variable.
variables. For a function of two variables it would thus seem reasonable to consider t w o
derivatives to measure the rate of change of the function with respect to each of
2. Determine equations for the
its independent variables. Such derivatives, called partial derivatives, are defined
tangent plane and normal line to
as follows:
the graph of a function of two
variables at a point. 19.3.1 Definition Suppose ( a , b ) is a point in the domain of the function$ T h e
partial derivative o f f w i t h respect to x at ( a , b ) , denoted h ( a , b ) , is defined by

provided that this limit exists. Similarly, the partial derivative off with respect t o y
at (a, b ) , denotedf,(a, b ) , is defined by

Note that these definitions give rise to two new functions f, andf U ,called the
partial derivatives of$ These are given by

19.3.2
634 CHAPTER 19: FUNCTIONS OF SEVERAL VARIABLES

,a, b , 0) la, b + A>, 01


v
curve of intersection of graph off with the plane x = a IAYI
Fig. 19.16 Fig. 19.17

The domains of these functions consist of all those points (x, y ) in the domain of
f for which their respective defining limits exist.

THE GE0,METRIC ISTERPRETATION


OF P,QRTIAL DERIVATIVES
Suppose that&(a, b ) exists and let C, denote the curve obtained by intersecting
the vertical plane x = a with the graph off(see Fig. 19.16).
We can compute the slope of the tangent line to C, at the point P =
(a, b, f ( a , 6)) by working in the plane x = a and proceeding as in Sec. 2.4.
If Ay denotes a change in y , then the secant line through P and Q =
(a, b + +
Ay,f(a, b Ay)) has slope

(see Fig. 19.17)

As Ay 0, the points Q slide along the curve C, toward P. In turn, the


corresponding secant lines tend to the line tangent to C, at P. The slope of
T, is given by

which is, by definition, equal to &(a, 6). Similarly, if cb denotes the curve
obtained by intersecting the graph of fwith the planey = b , then a completely
planeg = b analogous argument shows that &(a, 6 ) represents the slope of the line Tb
Fig. 19.18 tangent to C,, at P (see Fig. 19.18).

COMPUTING PARTIAL DERIVATIVES


If y is held fixed, then f(x,y) can be considered as a function of the single
variable x , and [ 19.3.2(a)] amounts to the derivative of this single-variable
function with respect to x in the sense of (2.3.1). Thus in most instances we can
use our knowledge of ordinary derivatives from Chaps. 2 and 3 to find&. We
simpb treaty as a constant and daj3erentiate with respect to x . Similarly, to computefU we
dzfferentiate with respect toy, holding x jxed.
SECTION 19.3: PARTIAL DERIVATIVES 635

Example 1 Determinef, and& for the function f ( x , y ) = 2x3y2 - 3x4 + 9y2 + 6 and find
their values at the point ( 1 , 2 ) .
Solution To find f, we treat y as if it were a constant. Thus f , ( x , y ) = 6x5 - 12x3.
Similarly, holding x constant and differentiating with respect to y , we have

f,(x,y) = 4 x 5 + 18y
Hence
& ( l , 2 ) = 12, & = ( 1 , 2 ) = 44. 0

Alternative derivative notations Alternative notations for& and& are

f,(,Y) = f , ( w
=) axaf and f,(X,Y) =f,(x,y)= -
af
%J
The latter notation is reminiscent of the Leibniz notation for functions of a
a
single variable, and we shall understand -to mean take the partial derivative
ax
d
with respect to x in much the same way that -was understood to mean take
dx
the derivative with respect to x. Thus, for example,
a
-(3x2y4 + 9x) = 6xy4 + 9 and
a
-(3x2y4 + 9x) = l2x2Y3
ax aY
If the function is given in the form z =f ( x , y ) , then

are also used to denote f , and f u , respectively.

Example 2
af and -
Determine - af for the function f(x,y) = y 2 sin(x3 + xy).
ax ay
Solution One must be careful in this example to use the appropriate derivative rules.

a
= y 2 -[sin(x3 + xy)]
ax
=y2[c0s(x3 + ~ y ) -]axa( x 3 + XJ)

=JJ~[COS(X~ + X J ) ] ( ~ X +~ J ) = (3x52 +J~)COS(X~ + v )


Note that in the second step we needed to use the Chain Rule.

=yzA[sin(x3 + xy)] + rsin(x3 + ,911 - (a( r 2 )


aY aY
=J~[COS(X~ + xy)]-(x3a + 9)+ [sin(x3 + 9 ) ] 2 y
aY
= J ~ [ C O S (+Xx~y ) ] ( x )+ 2y sin(x3 + xy)
=y[xy + xy) + 2 sin(x3 + x y ) ]
COS(X~
636 CHAPTER 19: FUNCTIONS OF SEVERAL VARIABLES

In this latter case the differentiation was with respect t o y and hence we were
required to t r e a t f a s the product of two functions. 0

In some cases it may be necessary to resort to the limit definition to deter-


mine the existence or nonexistence of a partial derivative at a particular point.

Determine f X ( x , y).
Solution If ( x , y ) $ (0, 0), then

[Since we were taking the limit as A x + 0, we assumed A x # 0. Thus


( A x , 0) # ( 0 , O ) and so, by the definition off(x,y),
AX)^ * 0
f ( A x , 0) = = 01 0
AX)^ + O2

PROGRESS TEST 1

In Probs. 1 and 2 determinef, a n d & .


1. f ( x , y ) = 3 x 3 4 + x y
2 . f ( x , y ) = xy ln(x2 + y 2 )
3 . Determine f,(0,O) for

P-ARTIAL DERIYATIVES AS RATES OF CHANGE


Suppose at a particular instant the temperature of a heated metal plate located
+
in the plane is given by T(x,y) = 9 - x 2 y4, where (x,y) represents a typical
point on the plate. The partials

measure the rate of change of the temperature with respect to a horizontal and
vertical change in direction, respectively. In particular, if a particle located at
SECTION 19.3: PARTIAL DERIVATIVES 637

( - 1, 2) (where aT/& = 2, ?T/ 3y = 3 2 ) were to be moved slightly to the right


>
(Ax 0), the temperature would increase by 2" per unit of displacement, and if
<
it were to move slightly downward (Ay 0), the temperature would decrease by
32" per unit of displacement.

9 For another example, recall that the volume of a right circular cylinder is
given as a function of two variables by V = m 2 h , where r is the radius and h
is the height. Thus

Note that 3 V / & is the formula for the lateral surface area of a cylinder of
radius r and height h, and a V / 8 h is the formula for the area of a circle of radius
r. This is. of course, what we saw in Chaps. 2 and 3 when we treated V as
functions of the height and radius separately. We review the situation in our new
terminology. Now a V / ar measures the rate of change of the volume with respect
to the radius. Suppose you leave the height of the cylinder fixed and make a
small change Ar in the radius. The thin shells pictured in Fig. 19.19 represent
the change in the volume AVinduced by this change in the radius. For Sr small,
I V / I r is approximately equal to the lateral surface area of the cylinder
+
(SV/Sr = 2viirh 7rh Sr).
Similarly, if 7 is fixed and h is changed by I h units, then the resulting change
in the volume A V is represented by a thin disk of radius 7 (see Fig. 19.20). In
fact, AV/Ih equals the area of a circle of radius r; that is, A V / I h = (vr2Ah)/Ah.
Thus we see that in the limiting cases (AT -+ 0 or Ih + 0), if h is fixed, then
the rate of growth of the volume with respect to r is '(by lateral surface areas." If
r is fixed, the rate of growth of the volume is "by areas of circular disks."

Ar >

Ah > 0
lAhl
J

/I---

(a)

Fig. 19.20
638 CHAPTER 19: FUNCTIONS OF SEVERAL VARIABLES

I :a)
z =f(x,y)
Fig. 1 9 . 2 1 Fig. 1 9 . 2 2

,normal line

tangent plane and i j k


normal line to
the graph off at P
= o 1 &,(a, b ) = (&(a, b),fy(a, b ) , - 1)
Fig. 1 9 . 2 3 1 0 &(a, 6)

19.3.3 Theorem Supposefis a function of two variables whose tangent plane IIT
exists at (a,b,f(a, b)). Then the vector

b),&(a, b ) , - 1)
is a normal to IIT, and nT has the equation
&(a, b)(X - 4 +?,(a, b)(y - b) = 2 - f ( G b )

If z = f ( x , y ) has a tangent plane at P = (a, b,f(a,b ) ) , then the line LA,


through P and perpendicular to the tangent plane is called the normal line to f
at P (see Fig. 19.23).
Since ( & ( a , b),&(a, b), - 1) is normal to IIT, we conclude the following:

19.3.4 Theorem The normal line to z = f ( x , y ) at (a, b,f(a, b ) ) has parametric


equations
x= a +&(a, b)t y =b + f,(a, b)t z = f(a, b) - t
SECTION 19.3: PARTIAL DERIVATIVES 639

Example 4 Find equations for the tangent plane and normal line to the surface
z = 3 . ~ 5-~2xy3 at the point (- 1, 3,81)
Solution Now&(x,y) = 12x3y2 - 2y3 andf,(x,y) = 6 x 9 - 6xy2, so&(-l, 3) = -162
and&( -1,3) = 72. Thus

nT: -162(~ + 1) + 72(y - 3) = t - 81


and

L,: x = -1 - 162t,y = 3 + 72t, t = 81 - t


It is possible for&(a, b ) and f u ( a ,b ) to exist and for the tangent plane at
P = (a, b,f ( a , b ) ) to fail to exist. Such a situation would occur, for example, if
you intersect the graph with a plane (distinct from x = a ory = b ) perpendic-
ular to the xy plane and get a resulting curve of intersection that either has no
tangent line at P or has a tangent line that does not lie in the plane determined
by T, and Tb.It can be shown that if the partial derivatives are continuous,
then this will not happen.

PROGRESS TEST 2

1. Determine equations for the tangent plane and nor-


mal line to the graph of the function f ( x , y ) =
~'m at the point (3,4,5).
2. The temperature distribution of a heated plate in a
+
plane is given by T ( x , y ) = 12 - 3x2 6y5. Deter-
mine the rate of change of the temperature in the
direction of the positive x axis at (3, -1).

FUNCTIORiS O F THREE B'ARIABLES


Partial derivatives of functions of three variables are defined in an analogous
way. In this setting, two variables are held fixed and the function is differenti-
ated with respect to the remaining variable.

19.3.5 Definition If IYI = f(x,y,t),then thepartial derivativesf,, f u ,and& are defined


by:

whenever the appropriate limits exist.


As before, we shall on occasion use the alternate notation

f=- af af
f-=- af
f,=-
ax aY at
640 CHAPTER 19: FUNCTIONS OF SEVERAL VARIABLES

Example 6 Determine&,&, a n d & for the function f(x,y, z ) = y 2 sin(xyr) + zezY.


Solution Holdingy and z constant and differentiating with respect to x, we have
+
f , ( x , y ,t)= y 2 [ c o s ( ~ z ) ] y z zexY * y = y3r cos(xyz) + yzexY
Similarly, using the Product and Chain Rules on the first term,
fy(x,y,z ) = xyt2 cos(xyz) + 2y sin(xyz) + xzexY
and
&(x,y,z ) = xy3 cos(xyt) + exY 0

HIGHER-ORDER PARTIALS
Iff is a function of the two variables x andy, then the partial derivativesf, and
& are themselves functions of two variables. The partial derivatives of these
functions (if they exist) are called second partials of$ In our notation they are

(f,),,( h ) y 7 (A),,(&I,
where, for example, ( f,)y means the partial derivative of the function f, with
respect toy. It is customary to drop the parentheses and write them as

f,x ,f,,,&z7fyy
where the order of differentiation is read from left to right. In the a notation
the four second partials are

which we collapse to
ay ay
----
ay ay
ax2 ay ax ax ay ay2

respectively. In this latter notation we must be careful to read the order of


differentiation from right to left.

Example 6 Determine the four second partials for the functionf(x,y) = - 3xy2

Solution Sincef,(x,y) = 2xy3 - 3y2 and&,(x,y) = 3x2y2 - 6 9 , we have

f,,(x,y) = 2Y3 f,,(xJJ) = 6Xy2 - 6Y


&,(x,y) = 6xy2 - 6y fy,(x7y) = 6x3 - 6x

The partial derivatives f,, and fyx are called mixed partials of$ In the above
example the mixed partials are equal. This is in fact the case for most of the
functions that we will encounter. The theorem telling us the conditions under
which mixed partials are equal follows. Its proof is usually given in more
advanced courses.

19.3.6 Theorem Iff,, and &, are continuous at (a, b), then &,(a, b ) =&$(a, b).

Examples of functions whose mixed partials are not equal are given in
Exercises 5 1 and 5 2 .
Third-, fourth-, and higher-order partials are defined in a completely
analogous way. For example,
SECTION 19.3: PARTIAL DERIVATIVES 64 1

indicates the fourth-order partial derivative obtained by differentiating first


with respect t o y , then x , and then twice more with respect toy.

Example 7 Determine
ay for f ( x , y ) = x3 + xeu.
ax ay ax
~

Solution 'f
- = 3x5
ax + ey
a2f a + eY) = 3x2 + ey
= -(3x5
ay ax ay
ay = -(3x2
a
+ eu) = 6x 0
ax ay ax ax
If all partial derivatives of a given order are continuous, then repeated
application of (19.3.6) shows that the order of differentiation is irrelevant. Thus,
for example,

In particular, each of these derivatives for the function in Example 7 would


equal 6x (check this!). In many instances the independence of the order of
differentiation can simplify our computations.

Example 8 Let f ( x , y ) = xy2 sec2(y3).Determine ~


ay
ax2 ay3 *

Solution If we differentiate using the indicated order, then we must begin by differenti-
ating three times with respect toy, which, in view of the products involved, is
rather tedious. However, since derivatives o f f of any order will result in
continuous functions, we have

Now af/ax = y 2 sec2(y3),so a 2 f l a x 2 = 0, and hence


ay - 0
-- 0
ay3 a x 2

Higher-order partials for functions of three variables are denoted in the same
way, and the corresponding theorem on mixed partials carries over. In fact, the
definitions and theorems hold for functions of more than three variables as well.

Example 9 Compute the mixed second partials for the function


f ( x , y , z ) = xeu +yeZ + ze"
Solution f, = eY + zex' f Y = xeu+ eZ,& = yeZ + ex. Thus
f,, = ey =hx h,= ex =f,, fyZ = ef =LY
(We used the continuity of the second partials to cut the number of computa-
tions in half.) 0
642 CHAPTER 19: FUNCTIONS OF SEVERAL VARIABLES

PROGRESS TEST 3

SECTION 19.3 EXERCISES

In Exercises 21 to 25 determine equations for the tangent plane and normal line to the surface at the given point.
21. t = 4 - x2 - y 2 ; P = ( 2 , 1 , - 1 ) 22. t = x2 +y + 3 ln(y); P = (1, 1 , 2 )
23. z = e cos T X ; P = ( 1 , 0 , - 1 ) 24. z = d m ; P = (3,4, fi)
25. 2 = 3x2y3 - 2 ~ ; P = ( - 1 , 2 , 2 2 )

In Exercises 26 to 29 determine the value of the indicated partial derivative at the given point.

33. The temperature distribution of a heated plate in the


if ( x , y ) = (0,O) plane is given by T(x,y) = ( 1 2 - 3x2 + 6 ~ ~What
) ~ .
( a ) Show that f ( x , y ) is not continuous at (0,O). change in temperature will result from a small upward
( b ) Show thatf,(O, 0) andf,(O, 0) exist. movement from the point (3, - l ) ?
(Thus the existence of partial derivatives at a point need 34. Determine f,, f,, a n d & if
not imply the continuity of the function at that point!) &I
+
31. Show thatf(x,y) = 4x4 - 2x2y2 3y4 satisfies the par-
tial differential equation

32. Show that w = x2y + zy2 + xz2 satisfies the partial


differential equation w, + w y + wz = (x f y + 2).
SECTION 19.4: DIFFERENTIABILITY AND THE TOTAL DIFFERENTIAL 643

In Exercises 35 to 44 determine all distinct second partials.

35. f ( x , y ) = x 2 6 + 3xly
37. f ( x , y ) = x 2 a r c t a n ( 9 )
39. f ( x , y ) = sin(xeY)
41. f(x,y) = x / m
43. f ( x , y , 2) = dx2 + y2 + z 2
In Exercises 45 to 50 determine the indicated partial of$

45. f ( x , y ) = x tan(y3LJ;,,,, 46. f ( x , ~ )= ~4~ -+ 3, a3f/?xi?y?z


47. J ( u , u , w ) = h ( u 2 +
3v - w 3 ) >?3s/?u2 ?v 48. f ( P . I ) = ln[sin(P - I)],fppI
49. f( r , 8 ) = r 2 e T B ,ay/a84 50. +
f ( x , y ,z ) = x e g +yeZ ze,, aY/?xay2 ?z
5 1. Let f be defined by 52. Using a n argument similar to that in Exercise 51 show
that S,,(O, 0) #S,,(O, 0) for the functionf(x,y)

if (.,y) = (0,O) =i x* arctan(y/x) - y 2 arctan(x/y)


0
if xy
ifxy=O
#0

( a ) Use definition [19.3.2(a)] to show thatf,(O,y) = -y 53. Verify that the function f ( x , y ) = arctan(x/y) satisfies
for ally. ?2f/2x2 + ?f/?J = 0.
(b) Use definition [(19.3.2(b)] to show that f,(x, 0) = x 54. Determine z2F/ax 2y if
for all x .
( c ) Using ( a ) and the fact that F(x,y) = a arctan(t) dt

55. Show that f ( x , y , z ) = ( x 2 + y 2 + z2)-1/2.


( x 2 + y 2 + z2 # 0), satisfies S, + &, + S,, = 0. This
show that f,,(O, 0) = - 1. latter equation is often called Laplaces equation in three
( d ) Using (6) and the fact that dimenszons.

show thatf,,(O> 0) = 1.

Section 19.4: Differentiability and the Total Differential

Objectives: ISTRODUCTIOS
1. Understand geometrically and How should we define differentiability for a function of two or three variables?
algebraically the notion of One might be tempted to define it in terms of the existence of the partial
differentiability of functions of two derivatives. However, in Exercise 30, Sec. 19.3, the function
variables in relation to the
corresponding notion for functions
of one variable.
2. Use differentials to approximate
the value of a two- or three-variable was shown to possess both first partials at (0,O) and yet be discontinuous at that
function near a given point. point! Certainly, differentiability should imply continuity. Figure 19.24 illus-
trates another weakness of the existence of the first partials as a condition for
differentiability. The curves of intersection with the planes x = 0 a n d y = 0
have horizontal tangent lines at (0,O). That is,f,(O, 0) =f,(O, 0) = 0. However,
the curve of intersection with the planey = x has a corner at (0,O) and hence no
tangent line there. Thus this function has no tangent plane at (0,O). Any
definition of differentiability for a function of two variables at a point should
also ensure the existence of a tangent plane at that point.
644 CHAPTER 19: FUNCTIONS OF SEVERAL VARIABLES

In Sec. 3.3 it was shown that if a function of a single variable is differentiable


at x, then

lim A f - d f
~ =o
Ax-0 Ax
where Af = f(x +
Ax) - f(x) is the change infresulting from the change Ax in
the independent variable. It is this intimate relationship between the increment
in f and the differential offthat we wish to preserve. We begin by extending
these notions to functions of several variables.

THE TOTAL DIFFERESTIAL


X
j = o 19.4.1 Definition Iff is a function of two variables, then the total
Fig. 19.24 dzferential OfJ; denoted df; ordf(x,y) is defined by

d f = f , ( x , y ) Ax +f,(.,V) 4J
where Ax and Ay are arbitrary real numbers and (x,y) is a point at which f
possesses first partials. Iff is given in the form t = f(x,y), then we also denote
the total differential by dz.

Note that dfis a function of the four variables x, y , Ax, and Ay. As in the
single-variable case, we define the differentials of the independent variables by

19.4.2 dx = Ax and 4 = Ay
If Ax and Ay are considered as increments in x andy, then the change from (x,y)
to (x + Ax,y +
Ay) produces a change Af(or Az) in the functionf, given by

19.4.3 Af=f(x + Ax,y + Ay) -f(x,y) (see Fig. 19.25)

Example 1 Compare the values of Afand @for the functionf(x,y) = x2 + xy2 if x = 1,


y = 3, Ax = 0.01, and Ay = -0.02.
Solution By (19.4.1),

df=f(w a x)+ f & Y ) A


,
4 +
= (2x f y 2 ) Ax ( 2 9 ) Ay
4 Letting x = 1,y = 3, Ax = 0.01,Ay = -0.02, we have
d f = ll(O.01) + 6(-0.02) = -0.01
From (19.4.3).
Af=f(l + 0.01,3 - 0.02) - f ( l , 3)
z f ( l . 0 1 , 2.98) - f ( l , 3)
= (1.01) + (1.01)(2.98) - 10 = -0.010696 0

Example 1 illustrates that the differential dfof a function of two variables is


a useful approximation of Afin the same way that it was a useful approxima-
tion for functions of one variable. In particular: ( a ) The computations in-
\ volved in computing df are considerably less tedious than those in computing

(x + Ax, + A] A$ (6) For small Ax and Ay, dfis close to A$ In this case dfis within 0.000696
Fig. 19.25 of A$
SECTION 19.4: DIFFERENTIABILITY AND THE TOTAL DIFFERENTIAL 645

The following theorem gives a condition on the partial derivatives offthat


ensures this good approximation property. It will also guide us in formulating
a definition of differentiability for a function of two variables.

19.4.4 Theorem Suppose the functionf(x,y) has continuous first partials in some
open disk S about (a, b). Then ,
A f ( a , b ) = df(a, b ) + Gl A x + G, Ay
where G, and G, are both functions of Ax and Ay, satisfying

Pro@ We choose Ax and Ay small enough that the rectangle centered at (a, b ) of
height 21Ayl and width 21Axl lies inside A . Now
Af = f ( a + A x , b + AJ) - f ( a , b)
or

(2) + AYJ)-f(a, b + A!)] + [f(a,b + Ay) -f(a, b ) ]


Af = [ f ( a + Ax, b
Now fix A x and Ay (both > 0) and let p ( x ) = f ( x , b + Ay), q ( y ) = f ( a , y ) .Then
p and q are differentiable on the intervals
[a, a +
Ax] and [ b , b Ay], respec- +
tively. Applying the Mean-Value Theorem (4.4.4) to each of p and q, we have
(see Fig. 19.26)

(3) b(a + Ax) - p ( u ) = p ( d ) A x a < d < a + Ax


and

(4 ) q(b + b)- q ( b ) = q(6) b < < b + Ay


6

N o w p ( ~ =&(x,
) b + Ay) and q(y) =fu(a,y),so (3) and (4) are equivalent to
(5) f ( a + Ax, b + Ay) -f(a, b + Ay) = f , ( d , b + $ ) A X
and

(6) f(a, b + A,) -f ( a , b ) = &(a, 6) $

Fig. 19.26
646 CHAPTER 19: FUNCTIONS OF SEVERAL VARIABLES

respectively. Substituting (5) and (6) into (2), we have

A f =&(d, b + 4)Ax +&(a, c) AY


Let

G, = f , ( d , b + Ay) -&(a, b) and G, =&(a, c) -&(a, b)


Then
A f = [&(a, b) + G11 Ax + [&(a, b ) + G,1
=&(a, b) Ax +&(a, b) Ay G, Ax +
G, Ay +
= d f + G,Ax + G,Ay
Now as A x and Ay tend to zero, we conclude that d -+ a, c -+ b, and
+
b Iy + b. Sincef, and f , are continuous, it follows that
GI = [ f , ( d , b + AY) -&(a, b)l + [&(a, b) -&(a, b)l = 0
and
G, = [&(a>6) -f,(a, b)I + [&(a, b) -f,(a, b)l = 0
The argument is essentially the same if we remove the restriction Ax,
>
Ay 0. W

DIFFERENTIABILITY
Note that the conclusion of (19.4.4) ensures that dfz Af for Ay and Ax small.
With this in mind we provide the following definition.

19.4.5 Definition of Differentiability Suppose that f is a function of two


variables whose first partials exists in some open disk about (a, b). We say f is
dgerentiable at ( a , b ) if there exist functions G, and G, such that

(a) Af (a, b) = d f ( a , b ) + G,Ax + G,A?


and

Iff is differentiable at (a, b), then Af - d f = G, A x + G, Ay. Thus

Since this latter sum tends to zero as (Ax, Ay) + (0,0), we have:

19.4.6

This is the two-variable analog of Theorem (3.3.3) mentioned in (1) above. In


fact, condition (19.4.6) is equivalent to f being differentiable at ( a , 6 ) ; see Exer-
cise 22.
Finally, to complete our analogy to the one-variable case, we have:

19.4.7 Theorem If a function f is differentiable at (a, b), then it is continuous at


( a , b).
Prooj From (19.4.5), If = d f + GI Ax + G, Ay. That is,
f(a + Ax, b + Ay) -f ( a , b ) =f 2 ( a , b) Ax +&(a, b) Ay + G, A x + G, Ay
SECTION 19.4: DIFFERENTIABILITY AND THE TOTAL DIFFERENTIAL 647

Thus
lim
(Aa,AyMO,O)
[f ( a + A x , b + Ay) - f ( a , b)] = 0
or, equivalently,
lim
(Aa,Ay)-(O, 0)
f(a + Ax, b + Ay) = f ( a , 6 )
Let x = a + A x , y = b + Ay. Then (x,y) + (a, b ) if and only if ( A x , Ay) +
( 0 ,0), and hence

As might be expected, it is in general rather difficult to establish differentia-


bility from either the definition or the equivalent condition (19.4.6). However,
(19.4.4) implies the next theorem, which provides a condition guaranteeing
differentiability and is much easier to check.

19.4.8 Sufficient Condition for Differentiability If f,(x,y) andf,(x,y) exist


and are continuous in some open disk about (a, b), then f is differentiable at
(a, b).

THE TOTAL DIFFERESTIAL ASD THE TANGENT PLANE


The geometric interpretation of the total differential of a function of two
variables again directly parallels the one-variable case. There, the fact that
df z Af for A x small was reflected geometrically by the fact that the tangent
line is near the curve-provided you are near the point of tangency. Further-
more, as indicated in Fig. 19.27, &is the change in the height of the tangent line
+
as we move from a to a Ax, whereas Af is the corresponding change in the
height of the graph of the function its$
Suppose now that f is a function of two variables. It can be shown that the
differentiability off at (a, b ) is equivalent to the existence of a tangent plane to
the graph of z = f ( x , y ) at P = (a, b , f ( a , b ) ) . We shall assume this result. By
(19.3.3), the tangent plane has the equation

(7) 2 - 20 =,Ma, b)(x - a ) +f,(a, b)(y - b )


where zo = f ( a , b).

Y
, tangent line

Fig. 1S.27
648 CHAPTER 19: FUNCTIONS OF SEVERAL VARIABLES

+ A x , y = b + Ay. Then (7) becomes


i
Let x = a
tangent plane (8) z - 2 0 =&(a, b) A x +&(a, b) AY
Now the left side of equation (8) represents the change ( A z ) ~in the z coor-
dinate ofthe tangent plane as you move from the point (a, b ) to ( a A x , b Ay); + +
see Fig. 19.28. O n the other hand the right side of equation (8) is by definition
dz,so we have
( A z ) T= dz
Now since z = f(x,y), we know that

X /-kj-- L----

a + A x , b + A))
92 = f ( a

+
move from (a, b ) to ( a A x , b ) +
+ Ax,b + AY) - f ( a , b)
represents the change in the z coordinate of thefunctzon itselfas we make the same
,A in the xy plane (Fig. 19.25). And sincef
is differentiable at (a, b ) , AZ z dz for Ax and Ay small. But dz = (Az),, so
Fig. 19.28 92 z (At),. In Fig. 19.29, lAz - (Az),~ measures the vertical distance from the
+ +
point on the surface above ( a A x , b Ay) to the point on the tangent plane
+ +
above ( a Ax, b Ay). Thus the fact that AZ =: dz is realized geometrically by
the fact that the tangent plane is near the surface-provided you are near the
tangent plane point of tangency.
i I
If w = f ( x , y , z ) is a function of three variables, then we extend (19.4.1) in a
natural way and define the total dzjerential d f by

19.4.9 d f = f,(X,Y, 4 Ax + fY(X,Y, 2) A! + f , ( x , r , 2) A 2


In turn the increment off is given by
Af=f(a + Ax,b + AJ, + Az) - f ( a , b, c)
and f is dzjerentiable at (a, b, c ) if the first partials exist in some open ball about
(a, 6, c) and
+
Af = d f + G,Ax G2Ay G3Az +
for functions GI, G,, and G, that tend to zero as (Ax, Ay, Az) + (0, 0,O). In this
case there is no simple geometric realization for the differential, but we still
have dfz Affor Ax, A , and AZ small. As with functions of two variables, the
continuity of the first partials ensures differentiability, and differentiability at a
point ensures continuity at that point.
In many computations we use the approximation Af z d f in the form
f(a + A x , b + Ay) z f ( a , b ) + d f
or
f ( a + A x , b + Ay, c + Az) z f ( a , 6, c) + df

Example 2 Use the total differential to approximate d(3.02), + (3.96),.


Solution Let f( x , y ) = d m ,with a = 3, b = 4, A x = 0.02, and Ay = -0.04. Then
f(a + AX,b + Ay) = d(3.02),+ (3.96),
Now

sof,(3,4) = 3/5 andf,(3,4) = 4/5. By (19.4.6),

d(3.02)2 + (3.96)2z5 + __
3 -
5(0.02)
~

5(0.04)
-- 4.98 0
SECTION 19.5: THE CHAIN RULE 649

PROGRESS TEST 1

1. Let z = 2 x 2 - 3xy + y 2 . Determine ~ A Z- if


x = -1,y = 3, A x = -0.01, and Ay = 0.02.
2. +
Approximate d(3.01)2 ( 2 . 9 7 ) 3 .

SECTIOS 19.4 EXERCISES

In Exercises I to I0 determine 1Af - dfl for the indicated values of x, I, Ax, and Ay
1. f ( x , y ) = 2xy2; x = - 2 , ~= 3, AX = 0 01, Ay = 0.04 7 . f ( x , y ) = xexv; x = 2 , y = - 2 , Ax = 0.2, Ay = -0.1
2. f ( x , y ) = xy +x 3 , x = l , y = 4, Ax = -0.03, 4~ = 0.3 8. f ( x , y ) = 4xZ - ~ X J+ y 2 ; x = 1, y = - 2 , A X = -0.02,
3. f ( x , y ) = xy2 - 2x2.x = 4,y = 5,Ax = 0.001,Ay = 0.04 ly = 0.03
4. ~ ( x , J = +
) 3x2 XY - 4 ~ ' ; x = 1 , y = -3, AX 0.06, 9. f ( x , y ) = xy + In(xy); x = 1, y = 3, AX = -0.02,
Ay = 0.08 Ay = 0.01
5 . f ( x , y ) = e-xu; x = 2 , y = 1.5, Ax = 0.03, Ay = -0.02 10. f ( x , y ) = x - J / X + y ; x = 3, y = 5, A X = 0.01,
6. f ( x , y ) = In(y/x); x = 4 , y = 3, Ax = 0.07, Ay = -0.04 Ay = 0.02

In Exercises I 1 to I5 use (19.4.6) to show that the functions are diferentiable f o r all ( x , y ) .
12. f ( x , y ) = 3x2 + 5y2
14. f ( w ) = x Y/X + (x $ 0)
18. Approximate the length of the third side of a triangle
with adjacent sides of lengths 15.8 cm and 30.2 cm if the
angle between these sides is 58".
+
19. Approximate 4 ( 0 . 9 8 ) 2 + (1.97)2 (2.02)*.
if ( x , y ) = (0,O) 20. Approximate d ( 2 9 7 ) 2 (402)2 +
( u ) Determine f, and f u for ( x , y ) # (0,0 )
2 1. Approximate sin 46" cos 29".
( b ) Use the definitions off, and f u to find fx(O, 0) and 22. Let f be a function of two variables such that

f,@ 0 ) .
( c ) Use Theorem (19.4.8)to show thatfis differentiable
a t (0,O).
17. When measuring the dimensions of a right circular Show that f is differentiable at ( a , b).
cylinder, maximum errors of 1 percent and 0.5 percent
23. A rectangular box is measured and found to have base
3 cm by 7 cm and height 18 cm. Approximate the maxi-
are possible in measuring the radius and height, respec-
mum error in the volume measurement if the maximum
tively. Approximate the maximum possible error in
error in measuring the dimensions was 0.02 cm.
calculating the volume.

Section 19.3: The Chain Rule

Objectives: ISTRODUCTIOX
Use the appropriate version of a T h e familiar Chain Rule for functions of a single variable enables us to
Chain Rule to compute partial (or differentiate the composition of two functions using the derivatives of the
ordinary) derivatives of the functions being composed. The situation for functions of two or more variables
composition of two functions. is more complex since several different types of compositions can be formed.
For example, if z = f ( x , y ) and if in turn x = h(r, s) a n d y = g ( r , s), then we
can compose f with the functions h and g to yield z as a function of r and s:
=f J), g(r, $1)
[Of course, for such a composition to make sense each ( r , ~ must ) be in the
domains of both h and g and the resulting pairs (h(r,s), g ( r , s)) must be in the
domain of$ We shall assume throughout this section that all functions consid-
ered have domains so that the compositions being discussed are well defined.]
650 CHAPTER 19: FUNCTIONS OF SEVERAL VARIABLES

O n the other hand, if we compose the functions x = p ( t ) a n d y = q ( t ) withA


then we have z as a function of the single variable t :
z =f(p(t), dt))
In the former case a Chain Rule is in order to express the partial derivatives
o f t with respect to r and s in terms of the partials ofA h, and g. In the latter case
we would obtain the (ordinary) derivative of z with respect to t in terms of
partial derivatives off and (ordinary) derivatives of p and q.
Even if we limit our attention to functions of one, two, or three variables,
there are several different ways that compositions can be formed, and so several
versions of a Chain Rule are needed. We shall begin by giving the statement
and proof of one particular version of the Chain Rule. The general Chain Rule
that encompasses all possibilities follows.

THE CHAIN RULE-A SPECIAL CASE


19.5.1 T h e o r e m Let z = f ( x , y ) be differentiable and suppose that the functions
x = x ( r , s) a n d y = y ( r , s) are continuous and possess partial derivatives with
respect to r and s. Then z considered as a function of r and s has partial
derivatives with respect to these variables given by

and

(b)

Prooj Since the proofs of ( a ) and ( b ) are similar, we shall prove (a). Now z , as
a function of r and s, is given by
z =f M y , s), A T , s))
By definition,

(1)
aZ
- = lim f ( x ( r + A T , s),y(r + AT, s)) - f ( x ( r , J),Y(r,s))
ar AT-o Ar
provided that this limit exists.
Now since s is fixed, we take A s = 0, and so

(2) Ax = x(r + Ar, s) - x ( r , s) Ay = y(r + Ar, s) - y ( r ,s)


Thus
x(r + Ar, s) = x ( r , s) + A x = x + A x
and
y(r + AT, s) = y ( r , s) + Ay = y + Ay
Substituting in (l), we have

Ar AT-o Ar
Since f is differentiable, by definition we know that

(3)

where GI, G, + 0 as (Ax, Ay) + (0,O).


SECTION 19.5: THE CHAIN RULE 65 1

We can assume GI, G, are zero if Ax = Ay = 0 because, if not, we can simply


redefine them to be zero at (0,O) as done in the proof of the single-variable
Chain Rule (9.3.1).
Now dividing ( 3 ) by Ar, we have
a'?
(4) -A'?- _
Ar
--
ax Ar
+-,+aroy 5r GI-
AX
Ar
t G' A-
AY
r
Let hr + 0. From (2) and the continuity of x ( r , s) andy(r, s), it follows that Ix
and Ay + 0, and hence G,, G, + 0. Thus, from (4),

'*
ar
. -
_ -- lim A'?
17.-0 Ar
aZ aZ AX
=-lim-+-lim-+
ax Ar-0 Ar ay Ar-0 hr-0 [
lim G - + G -
Ir

Example 1 Let 2 = + xy, x = res,y = re-'. Determine at/% and &/as.


Solution By (19.5.1.a),
a2
_ --_a*-ax + --
ay
a2

ar ax ar ay a r
= (2xy +y)eS + ( x 2 + x)e-'
= ( 2 r 2 + r e P ) e S + ( r 2 P + reS)e+ = 3r2es + 2r
Similarly, by (19,5.1.b),
a2 ax a* ay
a2
-
--
as
_ _
ax as
+ --
ay as
= ( 2 9 + y ) ( r e s ) + (x2 + x ) ( - r e - ' )
= ( 2 r 2 + r P ) ( r e s ) + (r2eZs + res)(-re-s) = r3es 0

In the previous example if we express t as a function of r and s directly, we


have 2 = r3es + r 2 . Thus
32
- = 3r2es + 2r and a'?
- = r3es
ar as
which agrees, of course, with the results of Example 1.

THE GENERAL CHAIS RULE


We can generalize (19.5.1) to deal with the situation where
* =f(+ x2,. . . , Xn)
is a function of the n variables xl, x,, . , , , x, and where for each z, 1 5 z 2 n,
x, = x t ( u 1 , u2, . . . > U r n )
is a function of the m variables u l , u,,. . . ,urn.
In this setting the x,'s are called zntermedzate variables and the u3's are called
zndependent variables. The following general version of the Chain Rule, which
has essentially the same proof as (19.5.1), tells us how to find the partials with
respect to the independent variables in terms of the partials with respect to the
intermediate variables.
652 CHAPTER 19: FUNCTIONS OF SEVERAL VARIABLES

19.5.2 The G e n e r a l C h a i n Rule Let z = f ( x , , x,, , . . , x,) be differentiable and


suppose that for each i = 1,2, . . . ,n, xi = xi(ul, u2, . . . ,urn) is continuous and
possesses partial derivatives with respect to each of the variables ul, u,, . . . , urn.
Then, for e a c h j = 1 , 2 , . . . , m ,
a~ ax,
aZ
-
auj
-
at ax,
- --
+--+
ax, auj ax, auj ... +--ax,at ax,
auj

Hence to find the partial derivative of z with respect to one of the inde-
pendent variables, say u I , we must:

(i) Compute the partial derivatives o f z with respect to all of the intermediate
variables.
(ii) Compute the partial derivative of all of the intermediate variables with
respect to the given uj.
(iii) Multiply the results from (i) and (ii) term by term and add.
Notice that there will always be one term in the sum for each intermediate
variable. It is understood in (19.5.2) that any partial derivatives become
ordinary derivatives whenever the function being differentiated a t any stage is
a function of a single variable.
For emphasis we state the conclusion of (19.5.2) in some of the commonly
occurring situations.

19.5.3

19.5.4

19.5.5

19.5.6

It is probably worthwhile for you to formulate the remaining five versions


that one can encounter when dealing with functions of one, two, or three
variables.

Example 2 Show that iff is differentiable, then z = f ( x / y ) satisfies the partial differential
equation
XE + y -a t = 0
A

ax ay
SECTION 19.5: THE CHAIN RULE 653

Solution Let z =f(u) and u = u ( x , y ) = x / y . Then, by (19.5.6),

and

Thus
X a t + y -az = --
xdz --
x-dz =0
ax ay y du y du

Example 3 A plate exposed to a heat source maintains the temperature distribution


T ( x , y ) = 9 - 3x2 - 5y2 - xy over the time interval [0, u ] . A particle moving
along a certain path on the surface of the plate has velocity ( - 4 , 3 ) at the
< <
instant t = to (0 to u ) that it passes through the point ( - 1, 1). Assuming
that the particle has the temperature of its location, how fast is the temperature
of the particle changing at the instant the particle is at ( - 1, l)?
Solution Let F ( t ) = ( x ( t ) , y ( t ) ) be the vector function that describes the motion of the
particle. Now V ( t ) = ( d x / d t , d y / d t ) , and since V(to)= (-4, 3) we have

By the Chain Rule in (19.5.3),


dT
-=--
dt
aTdx
a x dt
+--aTdy
ay dt

Since the particle is located at the point (-1, 1) at time to, we have
x = -1,y = 1 a t time to. Thus

dt I t=to
= (6 - 1)(-4) + (-10 + 1)(3) = -47
Hence if T is measured in degrees Celsius and t in seconds, then the temperature
is decreasing at the rate of 47C per second at the instant the particle passes
through the point ( - 1, 1). 0

PROGRESS TEST 1

1. Use the Chain Rule to find the indicated partial 2. The height of a right circular cone is decreasing a t
derivative. the rate of 4 cm/s and the radius is increasing at the
at rate of 2 cm/s. How fast is the volume changing at
(u) -, where z = x = 7cos8, the instant that the radius is 12 cm and the height is
ar d xT +Y
-7 14 cm?
y = 7 sin 8
z,
( b ) dz where z = xy21n 1

a'= t2 +t
654 CHAPTER 19: FUNCTIONS OF SEVERAL VARIABLES

IMPLICIT DIFFEREKTIATIOK
In Sec. 3.2 we developed a method for finding dy/dx ify is defined implicitly as
a differentiable function of x by the equation F ( x , y ) = k. Using the Chain Rule
we can arrive at a more direct means for determining dyldx. Let z = F ( t , y ) ,
where t = x andy = y ( x ) . Then z is a function of x alone, so if F is differentiable,
then
dz
- -_
--
dx
aF dt
at dx
+--aFay dxdy
- aF- - aF dy
--+
at 3y dx
aF aF dy
- - +--
-
ax ay dx
Thus if a F / a y # 0, we have
dy - - a F / a x - -F,
19.5.7 -- -
dx aF/ay
In a completely analogous way, if the equation F ( x , y , z ) defines z implicitly
as a differentiable function of x and y , then

19.5.8

provided that F, # 0.

Example 4 Assuming that the equation x ? ~ - 2xy4 = x2 + 1 definesy as a differentiable


function of x , find dyldx.
Solution Let F ( x , y ) = x?2 - 2xy4 - x 2 - 1. Then F, = 3x2y2 - 2y4 - 2x and
F, = 2 x 5 - %xy3.Thus, by (19.5.7),

& ---
- - -3x2y2
3 x 5 2 - 2y4 - 2x - 2y4 + 2x + 0
dx 2 x 5 - 8xy3 2 x 5 - 8xy3

As indicated in the above example, the idea is to bring all terms on one side
of the equation, regard that side of the equation as giving a function of
whatever variables appear, and then take partials in order to apply (19.5.7) or
(19.53).

THE TOTAL DIFFERENTIAL ASD THE CHAIS RULE


If z = f ( x , y ) , then the total differential is given by

In this case dx = A x and dy = .ly are arbitrary real numbers. Suppose now that
x = x(u, u),y = y ( u , u). Then z is a function of u and u, so the total differential is

If, in addition we agree to interpret dx and dy in ( 5 ) as total differentials when x


and y are intermediate variables, then
SECTION 19.5: THE CHAIN RULE 655

ax ax du
dx = - du
au + I
GU
and

Substituting into ( 5 ) , we get

at du
=- at dv
+- by the Chain Rule
8U av
Since we can also derive equation (5) from equation (6) by simply reversing the
previous steps, we conclude that ( 5 ) and (6) are equivalent. In light of this we
shall follow the convention of treating dx and dy as total differentials in ( 5 )
whenever x andy are intermediate variables. A similar convention applies when
w =.my,

SECTION 19.5 EXERCISES

In Exercises 1 to IOjind the indicatedpartial (or ordinary) derivative ( a ) by using the Chain Rule and ( b ) expressing w as a function of the
indicated independent variables before dzj'erentiating.

1. w = x ,x = rcos0,y = rsin0; aw/a0 7. w = e"Ysin(x + 2y), x =r + 3s - t, y = 2r + s + t;


x2 + y 2 zw/ a7
2. = 3 x 3 - y / x , x = t2, y = 3t + 4 ; dw/dt
w
3. w = x 2 + y 2 - 3 t ; x = r c o s 0 , y = r s i n 0 , z = r 3 ; z w / z r
, u =y + 22, v = y 2 , x = z?; awl&
rst x3z3
4. w = arcsinx, x =
dr2 + s2 + t2
: aw/at 9. w = -
U
+ -,2 u 3 1
x = t , y = -, z = t2, u = 2t; dw/dt
t

5. w=- xy x = sin t , y = cos t ; dw/dt 10. w = h ( x 2 + y 2 ) ,x = e r cos O,y = e r s i n 8 ; awl&, Zw/a8


x2 +y2'

In Exercises 11 to 20 compute the indzcated partzal (or ordznary) derivatives.


1 1. 2 =
ex--?l/e"+Y, x = I n ( s / t ) , y = In st; az/ as, az/ at 12. z = x 2 + y 2 , x = sin(s - t ) , y = cos(s t ) ; &/as, + &/at
13. = y 3 / x 2 , x = r2 - S,Y = 2rs; at/&, & / a s 14. z = e x y ' , x = r s , y = r/d'iT7;
az/ar, az/as
15. w = e - 3 2 cos z , x = t , y = 2t, z = 4 t ; dw/dt 16. z = arcsin xy, x = s+ t, y = s - t ; &/as, & / a t
17. z = ex + y , x = arctan(u/v),y = l n ( u + v); az/ au, az/ a v 18. w = x2 y z + + +z2 v2, x = r cos t, y = r sin t, z = rt;
u = r; a w l a t , aw/ar
19. z = X U , x = 1 + s2,y = et; aZ/as, at/zt 20. w = xUx,x = r,y = r + s , z = cos(rs); a w / a r , a w / a s

In Exercises 21 to 25 assume that the given equation dejnes y implicitly as a dzferentiable function cf x and determine dy/dx.
21. x 3 3 - 3xy4 x2 = 9 + 22. x4 + 5x53 +y4 =x 23. 2x1/3 + 4y2/3 = 2 24. x cos(xy2) + y 3 = 3.
25. x2+ f i = y 2 + 1

In Exercises 26 to 30 assume that the given equation dejnes z implicitly as a dzj'erentiable function of x and y and determine at/ax, at/ ay.

26. = z3 + 4
~ 2 -4y 2 x y 27. sin(xyz2) = xz + y
28. xe-?lr- zexY= 5 29. xdx2 +y2 + 22 =z + 2
30. x?z5 - xy2z3 = x? +9
656 CHAPTER 19: FUNCTIONS OF SEVERAL VARIABLES

31. Let w = f ( x 2 - y 2 , y 2 - x 2 ) , wheref(u, u ) is differentia- 36. Let w = f ( d x 2 y 2+ + z 2 ) ,where w = f ( t ) is differen-


ble. Show that tiable. Show that

32. Let w = f ( x , y ) be differentiable and let x = rcos8, 37.Suppose that the temperature a t the point ( x , y , z ) in
y = r sin 8. Show that + +
space is given by T ( x , y , z ) = x 2 y 2 z 2 - xyz. A
particle moving along the space curve F ( t ) =
( x ( t ) , y ( t ) , z ( t ) ) passes through the point (1, -1, 2) at
time t = to with velocity (3, - 2 , 4 ) . How fast is the
33. Let z = j ( x 2 -y), where f is differentiable. Show that temperature changing at that instant?
38. T h e radius of a right circular cylinder is increasing at
the rate of 4 cm/s, and the height is decreasing at the
rate of 2 cm/s. How fast is the volume changing a t the
34. Iff is differentiable and w = zf(.v, xz), show that instant when the radius is 20cm and the height is
40 cm?
39. A particle in the x y plane passes through the point (3,4)
with velocity ( -2, 5). Another particle travels along the
33. Let F be differentiable and let w = surface z = x 2 + y 2 in such a way that it is always
F(x, - x2, x2 - x g , x3 - xq, xq - x j ) . Show that directly above the particle in the x;y plane. How fast is
the height above the x y plane of this latter particle
changing at the instant that it passes through the point
(3,4,5)?

40. Let f ( x , y ) =
0
dt 3f and. ,;If
Find -
ax cy
41. Let f ( x , y ) = i,,,
du)
ax
Zf and -.af
k ( t ) dt. Find -
ay

Section 19.6: The Gradient and the Directional Derivative

Objectives: INTRODUCTIOS
1. Determine the gradient of a Iffis a function of two variables, thenf, measures the rate of change offwith
function of several variables. respect to a change in distance in the x direction, that is, in the direction of the
unit vector i. Similarly,& measures the rate of change offin they direction, the
2. Determine and interpret the
direction of the unit vector j. In this section we shall define a directional
directional derivative of a function
derivative, D,f, which measures the rate of change off with respect to a
of several variables.
change in distance in the u direction, where u is any unit vector.
If we place a representative of u with its base at (a, b ) , then the plan.: through
u and perpendicular to the xy plane intersects the surface in a curve that goes
through the point P = (a, b , f ( a , b)). Geometrically, D,A when evaluated at
(a, b), should yield the slope of the tangent line to this curve at P. In particular,
the special cases where u = i or u = j will then yield the familiar geometric
interpretations ofh(a, 6) and&(a, 6). These situations are pictured in Fig. 19.30.

THE DIREGTIOSAL DERIVATIVE


Suppose u is a unit vector in the xy plane. Then u = (cos 0, sin 0 > , where 0 is
the direction of u. If L, denotes the line in the x;y plane through (a, b ) parallel
to u, then L, has parametric equations.
x =a + (cos O ) t y =b + (sin 0 ) t
We can now regard L, as a t axis, where t = 0 corresponds to the point (a, b )
and the positive direction along L, is in the direction of u (see Fig. 19.31).
SECTION 19.6: THE GRADIENT AND THE DIRECTIONAL DERIVATIVE 657

slope D,J(a, 6 )
z slopeJJa, b ? slopef,l\a, b ) ? l

:a)
Fig. 19.30

Y
4

t i o,/
/
=
I, {
2
x = a
= 6
+
+
cos Pt
sin P t
Q= I
n+icosQ b+tsin8Sn+icos8,h+t~inQ
,I? 0
W X
/

19.6.1 Definition of Directional Derivative Let f be a function of two


variables. The directional derivative off at (a, b ) in the direction of the unit vector
u = (cos 0, sin 0 ) is defined by

D,f(a, b ) = lim
f(a + (cos R ) t , b + (sin R ) t ) -f(a, b )
t-0 t
provided this limit exists.

Notice that since t represents a change in distance in the u direction, D,f can
be interpreted as the rate of change off in the u direction.
658 CHAPTER 19: FUNCTIONS OF SEVERAL VARIABLES

The following theorem gives us an easy means for computing directional


derivatives.

19.6.2 Theorem Suppose u = ( u l , u z ) is a unit vector andf(x,y) is differentiable at


(a, b). Then

Duf (a, b ) = A(%b)Ul + .&(a, b)uz


Pro$ Since u is a unit vector, u1 = cos 8 and uz = sin 8, where 8 is the direction
of u.Let h ( t ) = f(u + ult, b + uzt). Then, by the definition of the derivative for
a function of one variable,

= lim
f(a + ~ l tb, + uzt) -f(a, b )
t-0 t
= D,f(a, b) by (19.6.1)
But h ( t ) is the composition of z = f ( x , y ) with the functions x = x ( t ) = a + ult
a n d y = y ( t ) = b + uZt, so by the Chain Rule (19.5.3)
dt afdx
h(t) = - = -- + --
af
dt ax dt ay dt
=&(a + ~ l tb, + uzt)ui +.&(a + ~ l tb, + u z t ) ~ ,
Letting t = 0, we obtain the desired result:
D,f(a, b ) = h(O) =A(%b b l +&(a, b)uz

Note that, as expected, if u = i = ( 1 , 0 ) , then


Dif (a, b ) = A ( a , b )
and if u = j = (0, l ) , then
Djf(a, b ) =.&(a, b )
If v is any nonzero vector, then we define D, f ( a , b ) , the directional derivative of
f i n the direction of v, by
V
19.6.3 D, f ( a , 6) = D, f ( a , 6) where u = -
llvll

Example 1 Determine the directional derivative at ( - 1 , 2 ) in the direction of v = ( 3 , 4 )


for the function f ( x , y ) = x2 + 2.9 - y 2 .
Solution Let u = v/llvll = ( 3 / 5 , 4 / 5 ) . Then u is a unit vector in the direction of v. Now
+
f , ( x , y ) = 2x 2y andfy(x,y) = 2x - 2y. Thus
D,f(-1,2) =Ouf(-1,2)
= & ( - 1 , 2 ) ( 3 / 5 ) +&/(-I, 2)(4/5)
= 2(3/5) + ( - 6 ) ( 4 / 5 ) = - 18/5 0

Suppose a thin plate occupies a region in the x y plane and suppose T ( x , y )is
the temperature distribution for the plate, where T is measured in degrees
Celsius, and x andy are measured in centimeters. Then D, T(a,6) gives the rate,
in deg C/cm, at which the temperature changes at (a, b) in the direction of u.
Suppose a heat-seeking particle is located at position (a, 6) on the plate. In what
direction will the particle travel? Certainly it would want to head in the
SECTION 19.6: THE GRADIENT AND THE DIRECTIONAL DERIVATIVE 659

direction of maximal increase in temperature. Thus, among all possible direc-


tions u, the answer is provided by the direction that yields the largest value for
D,T(a, 6 ) . Similarly, a heat-fleeing particle would travel in the direction that
yields the least value for D,T(a, 6). As an aid in answering questions of this type
it is useful to interpret the conclusion of Theorem (19.6.2) as a dot product.

THE GRADIENT
19.6.4 Definition If z = f ( x , y ) , the gradient Off at (x,y), denoted Vf or Vf(x,y),is
given by

Vf = (f,(x>Y)>f,(x!Y))
The symbol
0 using an upside-down delta or nabla, is read as delj?

Note that if u = ( u l , u 2 ) is a unit vector, then

m a , 6) u = (&(a, b),f,(a, 6 ) ) (u1, u z )

= f ( a , b)u1 + f,(% 61%


But this latter expression is, by (19.6.2), just D,f(a, 6). Thus we can restate
(19.6.2) as

19.6.5 Theorem Suppose f is differentiable at (a, b ) with u a unit vector. Then


D,f(a, 6 ) = W a , b ) u
If a: is the angle between Vf(a, 6 ) and u,then from one of the basic properties
of the dot product (17.4.4), ~ 7 eknow that

D,f(a, 6 ) = Vf(U! 6 ) u
= IIVf(a, 6111 IIuII cos a
But u is a unit vector, so IIuIl = 1, and we conclude that

D,f(a, b ) = llVf(a, b)ll cos


If (a, 6) is fixed and u (hence a ) is allowed to vary, then this expression will be
maximized when C O S N = 1. Since by definition 0 2 01 5 ii, the maximum
occurs when N = 0, that is, when u has the same direction as V f ( a ,6). The
minimum value for D,f(a, 6) will occur when cos ci = - 1 or a: = 7 . In this case
u has the opposite direction of Vf(a, b ) , that is, the direction of - Vf(u,6 ) . Note
that the actual maximum and minimum values for D u f ( a , 6 ) are IlVf(a, 6)11 and
- 1 1 Vf(a, b)ll, respectively. We summarize these observations in the following:

19.6.6 Theorem Supposefis differentiable at (a, 6 ) . Then


( a ) The maximum value of a directional derivative at (a, 6) is IIVf(f(a, 6)11, and
it occurs in the direction of V f ( a ,6 ) .
(6) The minimum value of a directional derivative at (a, 6) is - llVf((a,b)l1,
and it occurs in the direction of -Vf(u, 6).

Example 2 Determine the direction that yields the fastest rate of increase offat (1, 1) where
f(x,y) = x3 - xy2.
Solution Sincef, = 3 x 5 - y 2 andfy= x3 - 2xy, Vf(x,y) = ( 3 x 9 - y 2 , x3 - 2xy). Also,
since D,f(l, 1) measures the rate of change off at (1, 1) in the direction of u.
the fastest rate of increase will occur in the direction that maximizes D,f( 1, 1).
By (19.6.6), this is the direction of Vf(1, 1) = (2, -1). 0
660 CHAPTER 19: FUNCTIONS OF SEVERAL VARIABLES

Example 3 The temperature distribution of a thin plate is given by T ( x , y ) = 10 -


x 2 - 2y2. A heat-seeking particle is located at the point ( - 1 , 3) on the plate.
Determine the path C along which the particle will travel as it moves towards
the origin, the hottest point.
Solution We solve the problem by finding a parameterization for C and then eliminating
the parameter to obtain a n equation for C. If x = x ( t ) , y = y ( t ) are parametric
equations for C, then the vector ( x ( t ) , y ( t ) ) points in the direction of the
motion of the particle. By (19.6.6) this vector should always have the direction
of VT This will be the case if
(x(t),y(t)) = VT = (-2x, -4~)
Setting components equal, we have the two differential equations

which are equivalent to

dx
- = -2dt and
4
- = -4dt
X Y
respectively. From our work in Sec. 8.1 we know these have solutions
x = clecZt and y =~ ~ e - ~ ~
respectively. Since the particle starts at ( - 1 , 3 ) , we know that
-1 = x(0) = c1 and 3 = y ( 0 ) = c2
Hence x = -e-2t a n d y = 3e-4t provide a pair of parametric equations for C.
Eliminating the parameter, we observe that the particle will travel along the
Fig. 19.33 parabolay = 3x2 from ( - 1 , 3 ) to (0,O);see Fig. 19.33. 0

PROGRESS TEST 1

1. Determine D,f ( a , b ) for each of the following: ( 1 , 3 ) has value 2 while the directional derivative at
(a) f ( x , y ) = 3x2 + 4y2, v = ( 1 , 2 ) , (a, b ) = ( 3 , -4) this same point in the direction of ( 1 , 0 ) has value 4.
Determine the value of the directional derivative off
X
( b ) f(V)=
rnv = (I6- I ) , at ( 3 , - 2 ) in the direction toward the origin.
3. A particle is located on the surface z = 3 . ~ 3 ~10 +
(a, 6) = (3,4) above the point (2, 3) in the xy plane. In what
2. The directional derivative of the functionf(x,y) at horizontal direction should the particle initially
the point ( 3 , - 2 ) in the direction of the vector head in order to traverse the steepest path?

AN EXTENSION TO FUNCTIOSS OF THREE VARIABLES


Let w = f ( x , y ,z ) and let u = ( u l , u p , u 3 ) be a unit vector.

19.6.7 Definition The directional derizutiue off at ( a , b , c) in the direction ofu is defined
by

provided that this limit exists.


SECTION 19.6: THE GRADIENT AND THE DIRECTIONAL DERIVATIVE 66 1

As before, D u f ( a , b, c) gives the rate of change offat (a, b, c) with respect to


a change in distance in the u direction.
For functions of three variables, we define the gradient by

19.6.8

Theorems (19.6.5) and (19.6.6) can be routinely extended. Thus iffis differen-
tiable at (a, 6 , c) and if u is a unit vector, then

19.6.9

Furthermore, the maximal rate of increase off is livf(a, b, c)l/ and occurs when u
has the direction of Vf(a, b, c). Similarly, the minimum rate of - IIVf(a, b, c)lI
occurs when u has the direction of -V(a, b, c).

Example 4 Determine the directional derivative off(x,y,z ) = -x3 - xz + yz2 at ( - 1, 1, 2)


in the direction of the vector v = (2, 1,2).
Solution We first find a unit vector in the direction of v:

u=-= (2/3,1/3,2/3)
IIVII
Now
Vf(x,y, 2 ) = ( 2 9 - 2 , x 2 + 2 2 , --x + 2yz)
so
Vf(-l,l,2) = (-4,5,5)
Thus
D,f(-1,1,2) = Qlf(-1,1,2)
= Vf(-l, 1 , 2 ) * u
= (-4,5,5) (2/3,1/3,2/3)
= -8/3 + 5/3 + 10/3 = 7/3 0

Example 5 In what direction will the function f(x,y,z ) = x 2 + 2yz - z 2 experience the
greatest rate of increase at the point (2, - 1, 3)?
Solution Now Vf(x,y,z) = (2x, 22, 2y - 22), so Vf(2, - 1,3) = ( 4 , 6 , -8). Thus the
maximal rate of increase in f will occur in the direction of the vector
( 4 , 6 , -8). 0

SECTION 1 9 . 6 EXERCISES

In Exercises I to I0 determine the directional derivative o f f at the given point in the direction of v.
662 CHAPTER 19: FUNCTIONS OF SEVERAL VARIABLES

In Exercises I I to 15 determine the direction that will yield ( a ) the maximum and ( b ) the minimum rate of increase in f at the given point.
11. f ( x , y ) = "5 - 3y2; ( 3 , - 2 ) 12. f ( x , y ) = e x arctany; ( 0 , 1)
13. f ( x , y ) = m (-8,2) 14. f ( x , y , z ) = x 2 + 3xy - t 2 ;(1, -6, 3)
+
15. f ( x , y , z ) = ~3 xy2z3; ( 2 , 1, -1)
16. Let C be a smooth curve given by F(s) =
( x ( s ) , y ( s ) , z ( s ) ) , where s represents arclength and let
F(s,) = ( a , b, c). Show that iff(x,y,z) is differentiable at
( a , 6 , c), then

where T denotes the unit tangent to C at s = so.

In Exercises I 7 and I8 use the result of Exercise I 6 to determine the solution.


17. Find the rate of change with respect to distance along C 20. T h e temperature distribution of a metal plate located in
of the function f ( x , y ) = x 2 + 3xy2 a t the point ( 2 , 2), the xy plane is given by T ( x , y ) = 8 - 3x2 - 4y2. Find
where C is the curve with equationy3 = 2x2. a n equation for the curve that a heat-seeking particle
18. Let C be the space curve given by F ( t ) = ( e t , e-t, ~ t ) . starting at (3, - 1) will travel along toward the origin.
Find the rate of change with respect to distance along C 2 1. A metal plate in the shape of a circular disk of radius
for the function f ( x , y , 2) = 3 x 2 - 231, +
z2 at the point 5 in. is located in the xy plane with its center at the origin.
(1, 1 , O ) . T h e temperature distribution for the plate is given by
19. The function t = f ( x , y ) has, at the point (1, 2 ) , direc- T ( x , y ) = 5 2x + +
4y - x 2 - y 2 . A heat-fleeing parti-
tional derivatives that are equal to 2 in the direction cle starts at the origin. At what point on the boundary of
toward ( 2 , 2 ) and - 2 in the direction toward (1, 1). the disk will the particle exit the disk? (Use the fact that
What is the directional derivative a t (1, 2 ) in the direc- the differential equation du/dt +
k,u = k, has solution
tion toward (4, 6)? u = k,/k, +
Ce&').

Section 19.7: Tangent Planes and Normal Lines

Objectives: MORE ABOUT GRADIEN-TS


1. Determine the tangent plane and In Sec. 19.6 we showed that the gradient of a functionfof two or three variables
normal line to a surface always points in the direction of maximal increase in$ We shall now show that
F ( x , y, z ) = k at a point P. the gradient is always perpendicular to the level curves of a function of two
variables, or perpendicular to the level surfaces of a function of three variables.
2. Interpret the gradient as a
We shall assume throughout that the functions under consideration have
normal to the level curves or level
continuous first partials that are not all zero at the point in question.
surfaces of a function of two or
three variables.
Functions of two variables Suppose P = (a, b ) is a point in the domain
off(x,y) and C is a smooth level curve off through (a, 6). Then C has an
equation of the form

f(X,Y) = k
for some constant k. Let
x = x(t) and y =y(t)
be a parameterization for C, with the point P corresponding to t = to. It follows
that

f ( x ( t M t ) )= k
and so
SECTION 19.7: TANGENT PLANES AND NORMAL LINES 663

(x(to),~(to)) By applying the Chain Rule (19.5.3) to the left side of ( l ) , we have

f , ( w ) x ( t ) +f,(w)JJ(t)= 0
or, equivalently,
Vf(X,Y) ( X ( t ) , J(t)> = 0
When t = to, Vf(x(to),y(to)) = Vf(a, b ) , and thus we have

(2) W a , b) (x(to),y ( t o ) ) = 0
Now, by definition, a vector is perpendicular to a curve at a point if it is
perpendicular to any tangent vector to the curve a t that point. Since
(x(to),y(to)) is a tangent vector to C at (a, b), equation (2) implies the follow-
Fig. 19.34 ing:

19.7.1 Theorem Iff(x,y) has continuous first partials at (a,b ) and if vf(a, b ) # 0 ,
then Vf(a, b ) is perpendicular to the level curve offpassing through (a, b ) (see
Fig. 19.34).

Example 1 +
Sketch the level curve of the functionf(x,y) = 4x2 9y2 passing through the
point P = ( 1 , 4 ~ ? / 3 ) and sketch the representative of o f ( 1 , 4 ~ ? / 3 ) at P.
Solution The desired level curve C has the form

Y 4x2 + 9y2 = k

--
It vs:1>4v23,
Since the point P is on C, it follows that k = 4( 1)2
has equation
4x2 + 9y2 = 36
+ 9(4 f i / 3 ) 2 = 36. Thus C

or, equivalently,
--\
-- x2 y2
* x - 9+ q = l

Now Of = ( 8 x , 18y), so Vf(l,4 */3) = (8, 24 fi).


A sketch is given in Fig.
19.35. 0
C : 4x2 + 93 = 36
I
Functions of three variables Now let w =f(x,y, z ) and let P = (a, b, c)
Fig. 19.35
be a point on the level surface
s: f(x,y,z) = k
Suppose C is any smooth space curve lying on the surface S and passing
through P, with
x = x(t), y =y(t), z = z(t)
a parameterization of C. Since C lies on S, we know that

f ( 4 4 Y ( t ) , z ( t ) )= k
so
mwt),Y(t), 4t))l = 0
As above, by the Chain Rule (19.5.3), this is equivalent to
of(x(t),y(t),44) (X(t),Y(t), z(t)> = 0
In particular,
Vf(4 b, 4 (X(tO)dJ(tO), z(t0)) =0
664 CHAPTER 19: FUNCTIONS OF SEVERAL VARIABLES

where to is the value of the parameter corresponding to the point P. Thus


V f(a ,b, c) is perpendicular to the surface S at (a, b , c) because it is perpendicular
to every curve lying on S and passing through ( a , b, c); see Fig. 19.36. This
establishes the following:

Theorem If f ( x , y , z ) has continuous first partials at (a, b , c) and if


Vf(a, b , c) # 0, then Vf ( a ,b, c) is perpendicular to the level surface off passing
through (a, b , c).

In Example 3, Sec. 19.6, we determined the path followed by a heat-seeking


particle on a plate whose temperature at a point ( x , y ) is given by T ( x , y ) =
10 - xz - 2y2. We found that if the particle starts at ( - 1, 3), it follows the
(dark) pathy = 3x2 to the origin, the hottest point (see Fig. 19.37). In terms of
level curves, as mandated by (19.7.1): the particle travels in such a way as to cross any
level curves at right angles. At each instant, beginning on the level curve
T ( x , y ) = -9, it travels in the direction of the gradient. Also illustrated in Fig.
Fig. 19.36 19.37 are paths followed by heat-seeking particles originating at other points.
(In Exercise 31 you are asked to prove that all such paths not originating on the
x axis are of the formy = axz.) There are many other situations occurring in
nature that serve to illustrate theorems (19.7.1) and (19.7.2). For example, a
.Y
charged particle moving in an electromagnetic field will cross the equipotential

t surfaces at right angles; water flowing freely downhill will seek a path that runs
perpendicular to the contour curves of the terrain; and most insects moving
toward the source of a sex-attractive pheromone will travel in the direction of
the pheromone concentration gradient.

TANGENT PLAYES AND KORMAL LISES

t X In Sec. 19.3 we developed a means for finding the tangent plane to the graph of
z = f ( x , y ) at a point ( a , b , f ( a , b)). Suppose now we are given a surface S with
equation
F(x,y,z) = k

level curves i f
This can be considered as a level surface of the function w = F ( x , y , z ) . If
T\X,4) = 10 - X* - 242 P = (a, b, c ) is a point on S,and if F has continuous first partials, then V F ( a ,b, c)
Fig. 19.37 is normal to the surface at P,that is, normal to the tangent plane to 5 at P.

19.7.3 Theorem If F ( x , y , z ) has continuous first partials and if (a, 6 , c) is a point on


the surface F ( x , y , z ) = k with VF(a,b, c) # 0 , then the tangent plane to the
surface at this point has the equation

FJa, b, c)(x - a ) + F&a, b, c)(y - 6 ) + Fz(a,b, C)(Z - c) = 0


and the normal line has equations

x =a + Fz(a, b , c)t, y =b + F,(a, b, c)t, z =c + Fz(a, b, c)t

Example 2 Find the equations for the tangent plane and normal line to the surface
z 2 sin xy
TZ
+ 3y cos z +2=0

a t the point ( ~ / 2 ,1, T).


Solution Let

+ 3y cos 2
SECTION 19.7: TANGENT PLANES AND NORMAL LINES 665

Then the desired surface is the level surface F(x,y,z) = -2. Now
25 cos xy 22x cos xy
+ 3 cosz,
F, = T2
, Fy = 772

22 sin xy
F, = ~ - 3y sin z
772

and therefore F , ( T / ~ ,1, 7 i ) = 0, Fy(7i/2, 1, T) = -3, and F,(m/2, 1, T) = 2 / m .


By (19.7.3))the tangent plane has equation -3(y - 1) ( ~ / T ) (- +
z T) = 0
andthenormallinehasequationsx = 7;/2,y = 1 - 3t,andz = 7i ( 2 / ~ ) t .0 +

Example 3 Show that the formulas given in Theorems (19.3.3) and (19.3.4) for the tangent
plane and normal line to the graph of a function z = f ( x , y ) at a point
(a, b , f ( a , b ) ) are a special case of Theorem (19.7.3).

Solution If we let F ( x , y , z ) = f(x,y) - z , then the graph of the level surface F ( x , y ,z ) = 0


of F is the same as the graph ofJ Now

Letting c = f ( a , b ) and applying (19.7.3), we find that the tangent plane has
equation
&(a, b)(x - a) +&(a, b)(y - 6) - (2 - f ( a , b)) = 0
which agrees with (19.3.3), page 638. Similarly, the normal line has equations
x = a +&(a, b)t, y = b + &(a, b)t, z = f ( a , b ) - t, which agrees with (19.3.4),
page 638. 0

PROGRESS TEST 1

1. Supposef(x,y) = x 2 - 4y2. Sketch the level curves 3. Given the surfaces y - x2 z 2 2 = 0 and + +
o f f that pass through the points (3, &/2) and x2 + y 2 - z = 8, which intersect in a curve C, find
(- 2 m, l ) , respectively. Sketch Of at each of these equations for the tangent line to C at the point
points. ( - 2 , 2 >0).
2. Find equations for the tangent plane and normal
line to the surface x 3 + 4y3 + z 3 = 5xyz at the point
( 2 , 132).

SECTION 19.7 EXERCISES

In Exerczses I to 6 sketch the leuel curve oj'f that passes through the poznt P = (a, 6 ) and sketch the representatzue of vf (a, b ) at P.
1. f(x,y) = 9,P = (1,1) 2. f ( x , y ) = y 2 - x2, P = ( 2 , 1)
3. f ( x , y ) = x 2 + 4y2, P = ( 2 , 1) 4. f ( x , y ) = y2 - x3, P = (1, 1)
5. f ( x , y ) = x?, P = (3, 1/9) 6. f(x,y) = 2x2 + y 2 +
1, P = (1, 1)
7. Show that the tangent line at (a, b ) to the level curve of
f ( x , y ) passing through (a, b ) has the equation

f ( a , b)(x - a ) +&a, b)(y- 6) = 0


(Assume that f has continuous first partials and
V f ( a , 6 ) # 0.)
666 CHAPTER 19: FUNCTIONS OF SEVERAL VARIABLES

In Exercises 8 to IGJind an equation for the tangent line at P to the level curve off that passes through P. (Use the result of Exerase 7.)
8- f(x,y) = 9, (P = (1,2) 9. f(x,y) = y2 - 2xy + x2 - 2 x , P = ( 2 , 4 )
10. f(x,y) = 3x2 - 2xy +y2, P = ( 1 , l ) 11. f(x,y) = x3 + y 3 - 2 9 , P = ( 1 , l )
12. f(x,y) = x 2 +
4y2, P = (2, 1) 13. f(x,y) = xexy, P = (1,O)
14. f(x,y) = x3, P = (3, 1/9) 15. J(.,~) = x 5 - 2y3 + y ,P = ( i , i )
+
16. f(x,y) = x3 2xy + y 3 , P = (1, -1)
17. Show that the level curves off(x,y) = x3 + y 3 are per-
pendicular to the level curves of g(x,y) = xy for all
(a, b ) # (0,O).
18. Let C be a level curve o f f t h a t does not go through the
origin and let ( a , b ) be the point on C closest to the
origin. Show that the vector ( a , b ) is perpendicular to
the tangent line to C a t ( a , b ) .

In Exercises I9 to 25Jind equations for (a) the tangent plane and ( b ) normal line to the given surface at the indicated point.
+
19. 3x2 - 2y2 z 2 = 14, P = (2, 1 , 2 ) 20. x = ln(3y/4z), P = (0,4, 3)
21. 3 x 3 - x t 2 = 14, P = (-2, 1, 1) 22. ln(x2 + y 2 - t 2 ) = 0 , P = (1, -1, 1)
23. xy = y z - xz2 = 1, P = (1, 1, 1) 24. sin(xz2) = zy, P = (a/6, 1/2, 1)
23. yeQY = zexz, P = (2, - 1, - 1) 29. Prove that iff(x,y) and g(x,y) are differentiable func-
26. Let P = ( a , b , c) be a point on the sphere x 2 + y 2 + tions of two variables, then V ( f + g) = Of Vg. +
t 2= r 2 . Show that the normal line to the sphere a t P
passes through the point ( - a , -b, -c). 30. Suppose Vf(a, 6) = Vg(a, b ) # 0 and Vf(a, b ) Vg(a, b ) =
27. Find equations for the tangent line to the curve of 0. What can we conclude about the level curves at
intersection of the surfaces x sinyz = 1, zeyz-x2= 77/2 at ( a , b)?
the point (1, 1,77/2). 3 1. Show that any path traveled by a heat-seeking particle
28. Find the points on the surface on a plate with temperature distribution T(x,y) =
10 - x 2 - 2y2 is a parabola of the f o r m y = ax2 pro-
x2 y2 22
-+-+-=I vided that the path does not originate on the x axis (see
9 4 36 Fig. 19.37, page 664).
where the tangent plane is parallel to the plane
2x - 3y + z = 4.

Section 19.8: Extremes of Functions of Two Variables

Objectives: INTRODUCTIOS
1. Determine relative extremes of a In Chap. 5 we used derivatives to develop powerful and very useful techniques
function of two variables. for determining relative and absolute extremes of single-variable functions.
Although, as we might expect, the situation is somewhat more complex for
2. Determine absolute extremes of
functions of two variables, many of the underlying ideas and examples extend
a continuous function of two
our earlier one-variable work.
variables on a closed and bounded
set.
19.8.1 Definition We say thatf(x,y) has a relative maximum at (a, b) if the
3. Use objectives 1 and 2 to solve domain offcontains an open disk D about (a, b) such thatf(a, b) >f(x,y) for
applied maximum-minimum all ( x , y ) in D.
problems.
19.8.2 D e f i n i t i o n If (a, b) is in a subset S of the domain ofA then we say
thatfhas an absolute maximum on S at (a, b) iff(a, 6) >f(x,y) for all (x,y) in S.

Relative and absolute minimum are defined similarly.


SECTION 19.8: EXTREMES OF FUNCTIONS OF TWO VARIABLES 667

DETERMINING RELATIVE EXTREMES


In (4.2.3) we showed that if a function of one variable has a derivative at a
relative extreme, then this derivative is zero there. That is, the tangent line at
a relative extreme must be horizontal. This fact implies a similar theorem for
functions of two variables.

1 9 . 8 . 3 T h e o r e m I f f has a relative extreme at (a, b ) and if the first


partials of f exist at (a, b), then f z ( a , b ) =&(a, 6) = 0 or, equivalently,
Vf(a,b ) = 0.

Prooj We show &(a, 6 ) = 0 for a relative maximum. The other cases are
similar. Holdy = b constant and differentiate the one variable functionf(x, b ) :

O , [ f ( x , b)l = &(x, b )
I plane$ = b Butfhas a relative maximum at (a, b ) , s o f ( a , b ) >f(x, b ) for all ( x , 6) near (a, b ) .
Fig. 19.38 Thus g ( x ) =f(x, b ) has a relative maximum at x = a and so, by (4.2.3),
g(a) =&(a, b ) = 0. rn

Geometrically, the conclusion of (19.8.3) implies that the planes x = a and


z x = b intersect the graph o f f i n curves that have horizontal tangent lines at

t (a, b , f ( a , b ) ) (see Fig. 19.38). In particular, if the tangent plane exists at such a
point it will be horizontal.
The functionf(x,y) = y 2 - x 2 shows that the converse of (19.8.3) is false
since Vf = ( -2x, 2y) = 0 when x = y = 0, but f does not have an extreme at
(0,O). To verify this we note that f ( x , 0) = - x 2 <
0 for all x # 0. Since
f ( 0 , O ) = 0, f cannot have a relative minimum at (0,O). Similarly,
>
f(0,y) = y 2 0 for ally # 0, s o f cannot have a relative maximum at (0,O).
The sketch in Fig. 19.39 shows that the graph offhas a saddle shape at the
origin.
From our work in Chap. 4 we know that a function of one variable can have
a relative extreme at a point where the derivative fails to exist. Similarly,
,J f(x,y)= d m has a relative minimum at (0, 0), but neither
Fig. 19.39 X Y
f,(X!Y)= nor f&Y) =
@T7
exists a t (0,O). The conical graph (see Fig. 19.40) has a sharp point at (0,O).

i 19.8.4
Despite its limitations, (19.8.3) is quite helpful in identifying candidates for
relative extremes.

Definition A point (a, b ) in the domain off(x,y) is called a critical point off
if either V f ( a ,b ) = 0 or V f ( a , b ) fails to exist.

19.8.5 Definition A critical point offwhich is not a relative extreme offis called a
saddle point of J:

For the most part we shall deal with differentiable functions, so critical
points can be identified by simultaneous solution of the equations
-4
f,(x,y) = 0 and f,(.,r) = 0
The problem, then, is to distinguish saddle points from relative extremes. Our
starting point is analogous to the Second-Derivative Test for functions of one
variable, except that for functions of two variables the underlying issue of
Fig. 19-40
concavity is necessarily more complex.
668 CHAPTER 19: FUNCTIONS OF SEVERAL VARIABLES

19.8.6 Second-Derivative Test for Relative Extremes Suppose f has


continuous second partials in an open disk centered at (a, 6 ) and vf(a, b ) = 0.
At (a, b ) :
( a ) Iff,,f,, >
- f:, 0, then f has a relative extreme, which is
(i) a relative maximum iff,, 0 <
(ii) a relative minimum iff,, 0. >
<
( b ) Iff,,&, - f;, 0, then f has a saddle point.
(c) Iff,,f,, - f 3, = 0, then the test is inconclusive.

A proof of (19.8.6) is outlined in Miscellaneous Exercise 17. The idea of the


proof is based on examining concavity of curves formed by intersecting vertical
planes with the graph off at (a, b).
To apply (19.8.6) we must first determine the critical points by simultane-
ously solving the equationsf, = 0 a n d & = 0.

Example 1 Determine the relative extremes, if any, for the functionf(x,y) = x3 +y 3 + 3 9 .


Solution We first simultaneously solve
f, = 3x2 + 3y = 0 and f , = 3y2 + 3x = 0
From the first we gety = -x', which, when substituted in the second, yields
3x4 + 3x = 3x(x3 + 1) = 0. Hence x = 0 or x = - 1. The critical points are
thus (0,O) and ( - 1, - 1). Now f,, = 6x, f,,= 3 , and f,, = 6y, so at (0,O)

&,A, -f:, = -(312 <0


Hence by [19.8.6(b)],f has a saddle point at (0,O).
At (-1, -1),f,, f,, -f:, = (-6)(-6) - (3)' = 27 >
0. By [19.8.6(a)],f
has a relative maximum a t ( - 1, - 1) sincef,,( - 1) = -6 0. 0 <
When the Second-Derivative Test does not apply at a critical point ( a , b), we
must resort to other methods, one of which is simply to examine the values of
f near (a, b). The technique is to suppose that h and k are small numbers and
then examine f(a + h, b +
k ) - f(a, b).

Example 2 Determine the relative extremes, if any, for the function


f(.,y) = y 4 - x2
Solution Solvingf, = - 2 x = 0 and f , = 4y3 = 0, we obtain the single critical point
(0,O).Now

f,X& -f& = (-2)(12y2) - (0)' = - 2 4 ~ '


which is 0 at (0,O). Hence the Test is inconclusive. We examine
f(0 + h, 0 + k)-f (0,O) = f ( h , k ) = k4 - h2
Now, on the x axis we have f (h, 0) = -h2 2 0, while on t h e y axis we have
f ( 0 , k ) = k 4 2 0. Thusf has opposite sign for various (h, k ) near, but not at,
(0,O).Sincef(0,O) = 0, we conclude thatfattains neither a relative maximum
nor a relative minimum at (0, 0), and hence (0,O) must be a saddle point. 0
SECTION 19.8: EXTREMES OF FUNCTIONS OF TWO VARIABLES 669

PROGRESS TEST 1

Determine and classify all critical points for the given


functions.
1. f(x,y) = x 2 +y2 + 3+ 4
2. f ( x , y ) = x 4 +y4

ABSOLUTE EXTREMES
The Extreme-Value Theorem for functions of a single variable (4.4.2) guaran-
tees a n absolute maximum and minimum for a continuous function on a closed
interval [a, b ] . Determining such extremes amounts to comparing any relative
extremes in the interior of the interval with the values of the function at the
endpoints-the boundary of the interval. The situation for functions of two
variables is similar, but a bit more subtle. The definition of continuity on an
open set in the plane, (19.2.4), can be extended to the boundary of such a set,
yielding a definition of continuity on a closed set. A set in the plane is said to be
bounded if it can be enclosed by a circle of large enough radius. Now, given a
function f that is continuous on a closed and bounded set in the plane, an
Extreme-Value Theorem for functions of two variables guarantees thatfattains
a n absolute maximum and minimum on D.Such a theorem-and in fact a
corresponding theorem for functions of n variables-is normally developed in
more advanced courses. We shall assume that the extreme values we need
actually exist and concentrate on a procedure for finding them.

19.8.7 Procedure for Determining Absolute Extreme Values of a Con-


tinuous Function f on a Closed and Bounded S e t D
1. Sketch D and determine any critical points on the interior of D.
2. Describe the boundaries algebraically and describe the restrictions of the
function f on the separate pieces of the boundary.
3. Determine any extreme values off on the respective boundary pieces.
4. Compare the results of 1 and 3 to determine the extreme values off on D.
As illustrated in the next example, step 3 often consists of applying optimization
techniques from Sec. 5.1.

Example 3 Determine the maximum and minimum values of


f ( x , y ) = x2 + 4xy + y 2 + 6x + 1
on the closed triangular region

D = {(x,y)/O 5 x 5 4, x - 4 <y 5 O} (see Fig. 19.41)


1. We solve the equations
&=2x+4y+6=0 and &=4x+2y=O
to obtain the single critical point (1, - 2).
2. The boundary of D consists of the three line segments
c,: x = 0, -4 <y 5 0
Fig. 19.41 c,: y = 0, 0 5 x 4
c,: y=x-4, 0 5 x 1 4
670 CHAPTER 19: FUNCTIONS OF SEVERAL VARIABLES

+
O n C,: f ( O , y ) = y 2 1 = g(y), -4 < y 5 0.
O n C,: f ( x , 0) = x 2+ 6x + 1 = h ( x ) , 0 5 x 5 4.
O n C,: f ( x , x - 4) = x 2 + 4x(x - 4) + +
( x - 4)2 6x+1
+
= 6 x 2 - 18x 17 = k ( x ) , 0 5 x 5 4.
3. Now g(y) = 2y 5 0 for -4 < y 5 0, so the extremes of g occur at the
endpointsy = -4 andy = 0. The corresponding points on C, are (0, -4) and
(0,O).
4
Similarly, h(x) = 2x +
6 = 0 when x = - 3. But 0 5 x 5 4, so h ( x ) reaches
t its extremes at x = 0 and x = 4.The corresponding points on C, are (0,O) and
(4,O). Finally, k(x) = 12x - 18 = 0 when x = 3/2, so the extremes of k ( x ) can
occur when x = 0, x = 3/2, or x = 4. Since y = x - 4, the corresponding
points on C, are, respectively, (0, -4), (3/2, -5/2), and (4,O).
4. From steps 1 and 3 we have the points (1, -2), (0, 0), (0, -4), (4,O) and
(3/2, -5/2). We now compare:
f(l,-2)=1-8+4+6+1=4
f(0,O) = 1
+
f(4,O) = 16 24 + 1 = 41
f ( 0 , -4) 16 +
1 = 17
f(3/2, -5/2) = 9/4 - 60/4 + 25/4 + 18/2 + 1 = 7/2
.J The maximum value is 41 and occurs at (4,O).The minimum value of 1 occurs
Fig. 19.42 at (0,O). 0

Example 4 An automotive engineer is attempting to fit a rectangular map compartment of


maximal volume into a chopped-off corner under the dashboard. She measures
and then models the available space as in Fig. 19.42, where the constricting
plane has equation 5 x + +
2y z = 30. What are the dimensions and volume of
the rectangular map compartment of largest volume that can be built?
Solution We let the dimensions of the compartment be x,y, and t,so its volume is given
by V = xyz. Since z = 30 - 5 x - 2y, we can write V as a function of x a n d y :
V ( X , J J=) 3 0 -~ 5x3
~ - 2xy2
We are attempting to maximize V on the triangular region in the x y plane
bounded by the axes and the intersection of the plane 5 x + + 2y z = 30 with
the xy plane (see Fig. 19.43). However, both physical and algebraic considera-
tions imply that V ( x , y ) = 0 for ( x , y )on the boundary of this region. Hence it
remains to determine a positive relative maximum on the interior.
V, = 30y - 1 0 9 - 2y2 = 0 and V, = 30x - 5 x 2 - 4xy = 0
Since we are assuming ( x , y )is not on the boundary, we know x # 0,y # 0, so
we can divide the equations through b y y and x , respectively, to get
30 - lox - 2y = 0 and 30 - 5 x - 4y = 0
These equations yield the critical point (2,5). We use the Second-Derivative
Test: VzzV,, - V25, = ( - 1Oy)(-4x) - (30 - lox - 4 ~ )0 ~ >
at (2,5), with
Vzz(2, 5 ) = -50 <0. Hence (2,5) is a relative maximum for 7 i Hence the
dimensions of the compartment are 2 by 5 by 10 and its volume is 100 in.,. 0

Fig. 19.43 More applications will appear in the next section.


SECTION 19.8: EXTREMES OF FUNCTIONS OF TWO VARIABLES 67 1

PROGRESS TEST 2

1. The temperature distribution on a metal disk of Determine the hottest and coldest points on this
radius 1 centered at the origin is given by disk. (Usey as the independent variable for T o n the
boundary, describing the right and left halves sepa-
T(x,y) = 2y2 + 4x2 + 2y + 1 rately.)

SECTIOS 19.8 EXERCISES

In Exercises I to 20 iden@ all critical points of the given function and state whether each is a relative maximum, relative minimum, or saddle point.
1. +
f ( x , y ) = 2y2 - xy x2 2. f ( x , y ) = x2 + + + 3x - 9y
xy y2
3. + 18~y
f ( x , y ) = 21x2 - 4x3 + 2y2 - 4y3 4. f ( x , y ) = x 2 +y2 - 4xy
5. + + +
f ( x , y ) = 8 x 2 4y2 4y 3 +
6. f ( x , y ) = 6x2 6y2 + y 3
7. +
f ( x , y ) = x 2 - 24xy By2 8. f ( x , Y ) = x 2 + y 3 - 4xy - BX + 1 3 +~ 1
9. + + +2
f ( x , y ) = x 3 + y z - 6xy 6~ 3~ 10. f ( x , y ) = x y + x 3 3 - 12x32
11. f ( x , y ) = 2 x 3 - 3y4 - 69' 12. f ( x , y ) = x y - x y - x3y3
13. +
f ( x , y ) = x 4 - 4xy y4 14. f ( x , y ) = x 4 - X 3 ' + y 4
15. f ( x , y ) = ex cosy 16. f ( x , y ) = e-" s h y

17. f ( x , ~ =
X
) ---
Y 18. f ( x , y ) = 2x2 + 2xy + 5y2 + 4 x
Y X
19. f ( x , y ) = ( y + 2~)e-(~~+g') 20. f ( x , y ) = e Z y ( x 2+x +y)
In Exercises 21 to 31 determine the absolute maximum and minimum
32. A rectangular, open-topped box is to hold 256 m3. De-
values obtained by the function f on the closed domain R bounded by the
termine the dimensions of such a box that can be con-
given curves.
structed using a minimum amount of material.
21. f ( x , y ) = 2x2 + y 2 - y ; y = 1 - 2,x axis. 33. A rectangular box is to hold 36ft3. What dimensions
22. f ( x , y ) = x2 - xy + + 2y2 7y; x = * 3 , y = 2 3 . will yield the least expensive box if the material used to
23. +
f ( x , y ) = x 2 3y; x axis,y axis, x + y = 2 construct the top and bottom cost three times as much
24. f ( x , y ) = x 2 - xy + y 2 - 3x - 4 ; x axis, y axis, as that used to construct the sides?
x+y=4 34. Show that among all rectangular boxes having a fixed
25. f ( x , y ) = x2 + 4 x - 3xy + y 2 - 2y; x axis,y axis, x = 2, surface area, the cube will have the maximum volume.
y = 2 35. Find the dimensions of the rectangular box of maximum
26. f ( x , y ) = x 2 + y 2 - xy - x ; x = 2 1 , = ~ k 1 volume if the sum of the lengths of the edges is 24.
27. f ( x , y ) = 8x + 2xy - x 2 - 3y2 - 2;xaxis,yaxis,x = 10, 36. What is the volume of the largest rectangular box that
y = 3 can be inscribed in the ellipsoid x 2 / a 2 + y 3 / b 2 +
28. f ( x , y ) = e 2 2 t g 2 ~ r ; x = k1,y = k 1 z 2 / c 2 = l ? (Hint: Maximize the square of the volume.)
29. +
f ( x , y ) = y 2 - x 2 4 ~ 3 x 2~ +; y 2 = 1 37. Use the methods of this section to find the distance from
30. f ( x , y ) = eYe3'; x 2 + y 2 = 1 the origin to the plane ax + + cz = d. (Hint: Mini-
31. +
f ( x , y ) = 2 x 2 4y2 2x + + 1; x axis,y = 9 - x 2 mize the square of the distance.)

In Exercises 38 and 39 determine the dimensions of the rectangular box of largest volume situated with its bottom on the x y plane, one corner at the
origin, sides parallel to the axes, and its opposite corner on the given plane.
38. 6x + 4y + 32 = 36 39. 9x + 9y + 3z = 18
In Exercises 40 and 41 determine the points on the given surface where the distance from the origin to this surface is a minimum.
40.z 2 - ~y = 8 41. x2 - y z = 8
42. Give another version of [19.8.6(a)] with the sign of 43. ( a ) Show that (19.8.6) does not apply to f ( x , y ) =
&(a, b ) replaced by that of &,(a, b ) and f,, replaced by x4 - 3 x 3 + 2y2 a t the critical point (0,O).
fur. Why does this version follow from the original? ( b ) Show that (0,O)is a saddle point.
44. Repeat Exercise 4 3 for f ( x , y ) = 4x4 - 5 x 9 + y 2 .
672 CHAPTER 19: FUNCTIONS OF SEVERAL VARIABLES

Optimization Under Constraints and Lagrange


Section 19.9: Multipliers, with Applications in Economics

Objectives: INTRODUCTIOS
1. Betermine extreme values of Most applied extreme-value problems involve the determining of extreme
functions of more than one variable values of a multivariable function subject to some constraint. For functions of
under constraints using level two variables f ( x , y ) ,the constraint often takes the form of an equation in two
curves and using Lagrange variables g(x,y) = C. Solving a maximizing problem, then, amounts to finding
multipliers. the maximum value off(x,y) where the points ( x , y )are on the curve g ( x , y ) = C.
2. Apply these techniques to Since each value k off is associated with a unique level curve f ( x , y ) = k in the
problems in economics and plane, the idea is to search among the level curves o f f for a level curve
elsewhere. providing a maximum value for f on g ( x , y ) = C. As we shall see, this level
curve will be tangent to the curve g(x,y) = C in at least one point.
We can interpret the classic fence problem in these terms: What are the
dimensions of the rectangular pasture of largest area that can be enclosed by a
fixed perimeter C? Letting x and y be the dimensions of the pasture, the
function A ( x , y ) = xy is being maximized under the constraint 2x + 2y = C.
The graph of the constraint curve is a straight line of slope = - 1, while for
each possible area k, there is a unique hyperbolic level curve A ( x , y ) = xy = k.
The level curve that meets the constraint curve 2x + 2y = C and is associated
with largest area k is clearly that which is tangent to the constraint curve at
(xo,yo);see Fig. 19.44. The fact thaty, = x, (yielding a square xo byy, pasture)
agrees with our earlier work in Sec. 5.1.
On the other hand, if a fence is not needed on one side, say a fy side, then
the constraint is given by y + 2x = C, so the constraint curve y = C - 2 x is
tangent as indicated in Fig. 19.45 at a point whosey coordinate is double its x
coordinate. (Coincidentally, the rectangle with opposite corners (0,O) and
(xo,yo)in each case serves as a sketch of the optimal pasture.)
More generally, for a function f ( x , y )and constraint g ( x , y ) = C, the maxi-
mum occurs where the appropriate level curve offis tangent to the graph of

I l l I
I , \
I*/\\
, 18 \
\
, increasing

= k,

\
----
A ( x . y = ko
X
2x + 24 = c
Fig. 19.44 Fig. 19.45
SECTION 19.9: OPTIMIZATION AND LAGRANGE MULTIPLIERS 673

g ( x , y ) = C-provided
that the curves in question have tangents and we are not
on the boundary of D ( f ) . (See Fig. 19.46.)

PREFERENCE, UTILITY, AND INDIFFERENCE


The previous ideas are widely used in economics, where, as anyone who has
drawn up a budget can attest, optimization problems inevitably involve con-
straint conditions. Economists assume that we spend our money in ways that
maximize our satisfaction-or utdip, as they call it. They further assume that
our preferences, at least our relative preferences, are roughly quantifiable so
that a measure of total utility U is possible.

X Suppose on a given Saturday night that a student will divide available


funds between x hamburgers and y beers. Depending on personal preferences
and relative prices, this student will choose to buy a certain number x of
hamburgers and a certain numbery of beers in order to maximize utility
Fig. 19.46 U(x,y).Given fixed prices, say $1 for a hamburger and 504: for a beer, different
choices could quite likely lead to equal utility. For example, it is possible that
different combinations of hamburgers and beer yield equal utility. Each per-
sons utility function has level curves, called zndflerence curues by economists. In
Fig. 19.47 the student achieves the same utility U , from 1 hamburger and 5
beers as with 4 hamburgers and 1 beer. The cost of the former combination is
(1) + +
($)(5) = 3(4) dollars and the cost of the latter is (1)(4) (;)1 = 4;
dollars. However, given a fixed amount to spend, say 3 dollars, the student will
+
attempt to maximize U(x,y)according to the constraint (1)x (4)y = 3. The
optimal solution, as illustrated in Fig. 19.47, is 2 hamburgers and 2 beers.
However, given $5 to spend, the constraint curve moves out so that the
optimal point of 2 hamburgers and 54 beers is reached on the indifference
curve U , (see Fig. 17.47). This person obviously likes beer. However, if the
price of beer goes up to 754: then, given the same $5, this person will choose to
have 3 hamburgers and 3 beers on the U, indifference curve because the slope
of the constraint curve changes from - 2 to -4/3 (see Fig. 19.47). Further-
more, a different person will have different indifference curves. Does the
person, one of whose indifference curves appears in Fig. 19.48, have a drinking
problem? Note that, given the original hamburger-beer prices, this person will
spend $5 by buying 1 hamburger and 8 beers. It takes 10 hamburgers to make
this person as satisfied with one beer as with 8 beers and 1 hamburger.
y (beers)
y (beers)
f f
\

X (hamburgers)
1 1 2 3\4 5\6
I 1
x+-y=3
1
x f - y = 5
2 2
Fig. 19.47
674 CHAPTER 19: FUNCTIONS OF SEVERAL VARIABLES

PROGRESS TEST 1

1. Draw a plausible pair of spinach-hamburger indif-


ference curves for a 250-lb 12-year-old. (Plot ham-
burgers horizontally.)
2. Using the indifference curves in Prob. 1 and as-
suming that a plate of spinach and a hamburger
each cost $1, illustrate graphically how this 12-
year-old might divide $6 between spinach and ham-
burgers.

TAXES ON ENERGY COKSUMPTION


X In recent years the federal government has attempted to use a combination of
energy taxes and rebates in order to lower energy consumption. The basic idea
is to discourage consumption by raising the visible prices via specific taxes. But
to avoid overburdening consumers financially and to maintain comparable
utility, the government rebates much of the tax for people of average income.
To analyze this policy for a typical consumer, we let x be the annual consump-
tion of units of energy, say gallons of gasoline to be specific, and lety be the
quantity of all other goods purchased annually.
Before the tax a typical person is operating at the point of maximum utility
(xo,yo)where that persons budget constraint curve B is tangent to his/her
indifference curve of highest utility U , (see Fig. 19.49). The imposition of the
tax raises the price of gasoline relative to all other goodsy, making gasoline less
affordable and making other goods appear preferable-as reflected in the more
negatively sloped constraint curve B . Now, to return this typical consumer
(who is also a voter) to the original level of utility (satisfaction), the government
4 rebates the tax at income tax time. This rebate has the effect of sliding the
budget constraint curve out to B, allowing a tangent at (x2,y2)on the original
indifference curve U,. The net effect of the tax is to lower gasoline consumption
from xo to x2 gallons while maintaining equivalent utility; the consumer buys
more o f y (everything else) to compensate.

{all other goods)

(gallons of
B B B gasoline per year)
Fig. 19.49
SECTION 19.9: OPTIMIZATION AND LAGRANGE MULTIPLIERS 675

THE LAGRARGE MULTIPLIER TECHA-IQUE


U p to now our dkussions have focused on the geometric aspects of optimiza-
tion off(x,y) under the constraint g(x,y) = C. We can move to a powerful
algebraic technique by using the fact that if g(x,y) = C is tangent to a level
curve off(and all appropriate partials exist), then the normals to the respective
curves must be parallel at the point (a, b ) of tangency (recall Fig. 19.46). Since,
by (19.7.1), the normals have the directions vf(a,6) and Vg(a, b ) , we can
conclude that one gradient is a multiple of the other. That is, Vf(a, b ) =
A Vg(a, b). Assuming throughout that neither gradient is 0, this condition leads
to the three equations

19.9.1 f, = Agx &/ = Ag, g(x,y) = c


which can be solved simultaneously for x, y , and A. This will yield candidates for
extremes off(x,y), as Fig. 19.50 indicates, from which we then choose the
appropriate point(s).
The variable A in (19.9.1) is called a Lagrange mult$lier (after the French
mathematician Joseph Lagrange, who first developed this technique in the late
eighteenth century).

~~

Example 1 Determine the minimum, if it exists, of the function


f(x,y) = 2x2 + 3x + 2xy + 2y + y 2
subject to the conditiony2 - x = 1.
Solution Let g ( x , y ) = y 2 - x. Then
(1) & = 4x + 3 + 2y = Agx = A(-1)
(2) & = 2x + 2 + 2y = Ag, = h(2y)

J
increasingf

=c

Fig. 19.50
676 CHAPTER 19: FUNCTIONS OF SEVERAL VARIABLES

Set x = y 2 - 1 in (1) and (2) to obtain the simultaneous equations


4y2 - 4 + 3 + 2y + h = 0 and 2y2 - 2 + 2 + 2y - 2yh = 0
or
4y2 + 2y - 1 + h = 0 and y(y + 1 - A) =0
The equation on the right tells us that either y = 0 or y = h - 1. We use
y = h - 1 in the equation on the left to obtain a quadratic in A, which solves to
yield h = 1/4, 1. Now h = 1/4 implies y = -3/4, which yields x = -7/16
and the point ( - 7/16, - 3/4). Also, h = 1 implies y = 0 and x = - 1, which
yields the point ( - 1,O).
Sincef(- 1,O) = - 1 andf(-7/16, -3/4) = -155/128, weconclude that
the minimum value offis -155/128 and occurs a t (-7/16, -3/4). 0

To determine extreme values of a function on a region bounded by a con-


straint curve g(x,y) = C, we can use Lagrange multipliers on the boundary in
combination with the techniques of Sec. 19.8 on the interior.

Example 2 Determine the minimum value off(x,y) = 2x2 + 3x + 2 9 + 2y + y 2 on the


set of all (x,y) such thaty2 - x 5 1.
Solution Example 1 has given us a minimum value at (-7/16, -3/4) on the curve
y 2 - x = 1 , s o we now identify the region consisting of points (x,y) satisfying
<
y 2 - x 1. We solve this inequality for x to obtain x > y 2 - 1, whose graph
consists of all points inside the parabola x = y 2 - 1 as shaded in Fig. 19.51.
> Looking for critical points, we set

t 1 * - x = 1
f, = 4x + 3 + 2y = 0 fu = 2x + 2 + 2y = 0
which solve to yield the critical point ( - 1/2, - 1/2), which is inside the region.
By the Second-Derivative Test, ( - 1/2, - 1/2) is a relative minimum. Since
f(-1/2, -1/2) = -5/4 (= -160/128) andf(-7/16, -3/4) = -155/128,
x we conclude thatfreaches its minimum of -5/4 at ( - 1/2, - 1/2). 0

Although we have concentrated on functions of two variables, the Lagrange


technique extends directly, via a higher dimensional interpretation of the
equation Of = h Vg, to functions of three or more variables. The key, of
I course, is that there be enough equations to determine solutions. These exten-
Fig. 19.51 sions are treated in the section and chapter exercises.

PROGRESS TEST 2
-
1. Determine the extreme values off(x,y) = x? on the
curve 2x2 + y 2 = 12.

THE MEANING OF THE LAGRANGE MULTIPLIER


Suppose you are a division vice president asking the company president for an
increase in your divisions operating budget to be spent on labor costs x and
material costsy in order to increase productivityf(x,y). You must prove that a
modest budget increase will spur a large productivity increase.
Under your current budget constraint g(x,y) = C, you determined the point
(a, 6) of maximum productivity M [sof(u, 6) = MI by finding h so that the
following equations all hold:
SECTION 19.9: OPTIMIZATION AND LAGRANGE MULTIPLIERS 677

We shall now show that X is more than a technical help in computations and
can provide the basis for your argument for a larger budget. Foryou, C, ( a , b), A,
and M all have been determined-in effect, by the president. But for the
president, the budget C and hence ( a , b ) , A, and M are all variables.
By (3) we can write h ( g ( a , 6 ) - C ) = 0 , so
(4) M = f ( a , b ) - h ( g ( a ,b ) - C)
We shall compute d M / d C , the measure of the rate of change of maximal
productivity with respect to budget. Since a, b, and h all depend on C, we will
need the Chain Rule as well as the Product Rule in order to differentiate the
right side of (4). We differentiate the two terms on the right of (4) separately:

and

d b ) - hC1 = db
-dC
[Ada,

= hg,(a, b)-da
dC
+ hg,(a, 6)-dC
db + C-d h - X - C-d h
dC dC
[since g(a, b ) = C ]

Therefore, combining these derivatives we have

Hence the Lagrange multiplier is the rate of change of maximal output with respect to
change in budget. If, at a particular optimal operating point (a, b), h is large and
positive, then it makes sense for the president to increase your budget, whereas,
if h is small, then the budget probably should be held constant.
Since the mathematics of the above argument does not depend on the
particular interpretation given, we have proved the following theorem.

19.9.2 Theorem Suppose that f ( x , y ) yields a maximum M at (a, b ) under the


constraint g ( x , y ) = C when v f ( a , b ) = h v g ( a , b ) . Then X = d M / d C .

SECTIOS 19.9 EXERCISES

I. (Reversed Fence Problem) Draw several level curves and 2. Repeat Exercise 1 for ( u ) a four-sided rectangular pas-
the constraint curve that illustrates the solution of the ture and ( b ) a circular pasture.
following: Give the dimensions of the three-sided rec- 3. ( a ) Sketch a pair of plausible milkshake-hamburger
tangular pasture with an area of 10,000 m2 requiring the indifference curves for the 250-lb 12-year-oldof Progress
least amount of fence. Test 1. (Plot hamburgers horizontally.)
678 CHAPTER 19: FUNCTIONS OF SEVERAL VARIABLES

( b ) Given that hamburgers cost $1 and milkshakes cost 4. Sketch a hamburger-salad indifference curve for the
506, use your solutions to ( a ) to estimate how this 12- above-mentioned 250-lb 12-year-old who has decided to
year-old might spend $5 on hamburgers and milkshakes. lose weight. (Plot hamburgers horizontally.)

In Exercises 5 to 12 use Lagrange multipliers to determine the indicated extreme value for the given functions and constraints
5. f ( x , y ) = xy, x 2 + y 2 = 1, maximum 6. f(x,y) = xy2, x 2 + y 2 = 12, all extreme values
7 . f ( x , j ) = 2x2 + y 2 - x - y +
1,x2 + y 2 = 1,maximum 8. f(x,y) = x 2 + 3y2, 4 x + y = 6 , minimum
9. f ( x , y ) = x 2 + 2y2 + lox - 15y, 2x 2y+ + 150 = 0, 10. f ( x , y ) = 4x2 - 4xy + y 2 , x 2 + y 2 = 1, maximum
minimum 12. f ( x , y ) = 5x2 - 2xy +
3y2, 2x2 + y 2 = 12, all extreme
11. f ( x , y ) = x 2 + y 2 , x4 + y 4 = 1, all extreme values values

In Exercises 13 to 20 use Lagrange multipliers [solve thefour equations resultingfrom C f (x,y , t )= h V g ( x ,y , z ) and g ( x ,y , t ) = C ] to determine the
indicated extreme value for the given functions and constraints.
13. f ( x , y , t ) = Cxyz, x + 2y + 32 = 54, maximum 14. f ( x , y , 2) = x + y+ +
22, x 2 + y 2 z2 = 54, minimum
15. f(x,y,z) = x 2 + y 2 +z2, x + 3y - 22 = 12, minimum 16. +
f ( x , y , t ) = xy2z3, x + y z = 12, maximum
+
17. f ( x , y , z ) = x 3 + y 3 t3,x + y +
z = 1, maximum 18. f ( x , y , t ) = xyz, 4 x 2 + 9y2 + 16z2 = 432, maximum
+
19. f ( x , y , z ) = d x 2 + y 2 z2, x - 4y - t - 100 = 0, 20. f ( x , y , t ) = x 4 + y 4 + t 4 x, 2 + y 2 + 2 2 = 1, all extreme
minimum values

In Exercises 21 to 25 use Lagrange multipliers to determine the minimum distance between the given surface and the indicated entity. It su$ces to
minimize the square of the distance.
21. T h e surface x 2 - t2= 9 and the origin. 22. T h e plane 2% - 2y + 42 = 5 and the origin.
23. T h e plane x 2y+ +
3r = 7 3 and the point (1, 2,4). 24. T h e surface x 2 - y 2 - 22 = -2 and the origin.
25. T h e intersection of the planes x + y t = 1, + 26. Use Lagrange multipliers to show that the rectangle of
3x + 2y + t = 1, and the origin. maximum area and fixed perimeter C must be a square.

In Exercises 27 to 32 use the methods of this section to solve the given extreme-oalue problem.

27. Determine the extreme values of f ( x , y ) = ( a ) Determine how to allocate the budget to maximize
( 3 x 2 + 2y2)/eZz+~* on the disk x 2 + y 2 5 1. the production level.
28. Determine the dimensions of a right circular can of ( b ) What zs the maximal production level?
volume 16 i n 3 of minimum surface area. ( c ) If the budget is increased by 1 (thousand dollars),
29. A rectangular aquarium is to have an open top. Given what is the effect on the production level? Should
that the material for the bottom costs 2 c / i n 2 and the the budget be increased? [See (19.9.2).]
material for the sides costs lc/in.2, what are the dimen- 35. In Examples 1 and 2 we determined the minimum value
sions of the least expensive aquarium that holds of f ( x , y ) = 2x2 + 3x 2xy+ +
2y + y 2 for y2 - x 2 1.
1728 in.3? ( a ) Use the result of Example 2 to adjust the constraint
30. Determine the rectangle of maximum area and sides t o y 2 - x 5 a for an appropriate a so that the abso-
+
parallel to the axes inscribed in the ellipse 4x2 y 2 = 1. lute minimum occurs on the boundary.
31. Repeat Exercise 30, but maximize the perimeter. ( b ) Compare the necessary adjustment with the h used
32. Using that the volume of the ellipsoid x2/a2 + y 2 / b 2 + to compute the minimum on the original boundary.
z 2 / c 2 = 1 is (4/3).rrabc, determine the ellipsoid x 2 / a 2 + ( c ) Use Lagrange multipliers to recompute the mini-
+
y 2 / b 2 z 2 / c 2 = 1 with minimum volume that passes mum value off on the boundary and compare the h
through ( 3 , 1, 2). resulting in this solution with that suggested by the
33. A manufacturer producesf(x,y) units of a given product remarks preceding (19.9.2).
where x and y represent, respectively, the labor and
capital costs of the product. If the labor and capital per
unit are A and B dollars, respectively, and the total
budget is C dollars, prove that at the level (a, b) of
maximum production, &(a, b ) / A =&(a, b ) / B = A,
where h is the Lagrange multiplier. This equation is
called the law of equzmargznal productivity.
34. Suppose that a company has a labor cost x , capitaly,
production function (in thousands) f ( x , y ) = 12x33
where the labor cost per unit is $2 and the capital cost
per unit is 93. Suppose also that the total budget is 120
(in thousands of dollars).
SECTION 19.9: OPTIMIZATION AND LAGRANGE MULTIPLIERS 679

CHAPTER EXERCISES

R e v i e w Exercises
In Exercises I to I0 determine and sketch the region in R, or R, that is the domain of the given function.
1. f(x,y) = d4 - x2 - y 2 9. f(x,y) = y./*

5. f(x,y) = ln(3 - 9 ) 6. f(x,y) = arcsin(x + y )


7. f(x,y) = p
G+ 8. f(x,y,z) = ln(t - x2 - y 2 )
9. f(x,y,z) = d x 2 2~ + y 2 + + - 15 10. f(x,y, P) = arcsin x arctany + + In z
In Exercises I I to 15 show that the indicated limits do not exist.

2Y2 4y2 - x2 x94


11. lim - 12. lim 13. lim ___
(Z,U)40,0) x2 + y2 (T,,)40,0)
~

x2 + 4y2 (z,,)-(0,0) x6 + y 6

x3 +y3 ( x - 1)2
15. lim
14. lirn
(~,,)'(O,O)
~

x + y2 (,,U)41,2) (x - 1 ) 2 + ( y - 212

if (",V) = (0,O)
( a ) Is f continuous at (O,O)?
( b ) Determine iff, or f, exist at (0,0).

In Exercises 17 to 21, ( a ) sketch the graph ofx ( b ) sketch several level curves ofJ (c) sketch the curves of intersection with the planes x = a and x = b,
and ( d ) j i n d equations for the tangent lines to the curves in ( c ) at the point (a, b, f(a, b)).
17. f(x,y) = x2 + y ; a = 2 , b = 1 18. f(x,y) = 4 - x2; a = 1, b = 2
19. f(x,y) = 9 - x 2 - y 2 ; a = 1, b = 2 20. f(x,y) = x 2 + y 2 ; a = -1, b = 2
21. f(x,y) =ye"; a = 1, b = 1

In Exercises 22 to 28 determine f, and f,.


22. f(x,y) = e x 2 cos(xy) 23. f ( x , y ) = y 2 arcsin(y/x)
24. f(x,y) = x2 ln(x2 + y 2 ) 25. f(x,y) = cos2(x + In(xy))
26. f(x,y) = arctan(x/y2) 27. f(x,y) = x 2y e ,/x2

28. f(x,y) = szvX


e t 2 dt

In Exercises 29 to 37 determine the j r s t partials with respect to each of the independent variables.
29. f(x,y, z ) = xyexz+y2tz2 30. f ( u , v, w ) = w 2 arctan(u/v)

rt - st
33. f(w,z,p) = 4p2z +p/w - W/Z 34. f(r, s, t ) = -
r2 t2 +
35. f ( p , q ) = q arcsin(qp2) 36. f(A, C, D ) = D 3 d m
G3 dt
37. f ( G , I , P ) = P z s
d W
~

In Exercises 38 through 43 determine the indicated partial derivative.


38. f(x,y) = arcsin(x2 +y 2 ) , f , ,
40. f ( x J J ) = y arcsin(4f,,,,
42. f (x,y, z ) = arctan(xyt), f,,
680 CHAPTER 19: FUNCTIONS OF SEVERAL VARIABLES

I n Exercises 44 to 4 6 determine equations for the tangent plane and normal line to the graph off at the given point.
44. f(x,y) = x arcsin(x/(2y)), P = (1, 1,77/6) 45. f ( x , y ) = y ln(x2 + y 2 ) + e x , P = (0,1, 1)
46- f ( W ) = 5x P = (3,4,3)
PT7'
In Exercises 4 7 to 4 9 determine equations for the tangent plane and normal line to the given surface at the indicated point.
47. x1'2 +y1'2 +
9 ' 2 = 5, P = (1,1,9) 48. xy cos(y2z) = +/2, P = (1, 6,
1/3)
49. x arctan(y/z) - y arcsin(xlz2) = 77/12, P = (1,1: $7)

I n Exercises 50 to 5 4 use the total dgerential to approximate the given quantity.


50. f(- 3.04, 3.95), wheref(x,y) = x / d m 56. Let z = f(d m ) ,where f is differentiable. Show
51. v99.01 + 25.16 that
32.
53.
sin 44" cos 61"
T h e force of attraction between two particles of masses m
32
yaX-
xaz=,
ay
and n located r units apart is given by F = gmn/r2. If
there is a maximum possible error of 2 percent in the 57. T h e temperature in a particular fluid a t the point
measurements of the masses and 3 percent in the (x,y, z ) is given by T(x,y,z) = x 5 - yz + y 2 x . The ve-
measurement of r, then what is the approximate maxi- locity at time t = t,, of a particle moving along a path
mum percentage error in the measurement of F ? through the fluid is given by V ( t , ) = (-3,4, 7 ) . How
54. A can is measured and is found to have a height of 12 in. fast is the temperature of the particle changing at that
and a radius of 4 in. If there was a possible maximum instant? (The particle assumes the temperature of the
error of 0.02 in. in each of these measurements, then fluid at each point.)
what is the approximate error if the volume is computed 58. Let z = d m . Show that zEZzyZl = (z,,)'.
using these measurements?
55. Let z =f(x/y,y/z), wherefis differentiable. Show that

_x _az at
ax +&=O

In Exercises 5 9 to 63 determine the directional derivative off at the given point in the direction of the vector v.
59. f(x,y) = 3 x 5 - xy3; P = (1, -2), v = ( 3 , 4 ) 60. f ( x , y ) = xy ,/d- P = (-3,4), v = ( - 2 , 5 )
61. f ( x , y ) = ex siny; P = (0,~ / 6 ) v, = (4,- 3 ) 62. f ( x , y , z ) = x 2 arcsin(z/y); P = (-1, 2,0), v = (1, 1, 1)
63. f(x,y,z) = x y z / d F T j ? + 22; P = ( - 1 , 2 , -2), v = (1, 1, fi)

I n Exercises 6 4 to 68 determine the maximum and minimum value of the directional derivative off at the given point.

64. f ( x , r ) = x2d-, P = (3, -4) 65. f(~,v) ~5~


= - 3x/y, P = (1, 1)
66. f(x,y) = e m , P = (fi, - fi) 67. f(x,y,z) = x5z3 - 3zx2, P = (1, -1, 2)
+
68. f(x,y,z) = l n ( d x 2 + y 2 z2), P = (-1, 1, $7)
In Exercises 69 to 73 identify the critical points off and state whether each is a relative maximum, relative minimum, or saddle point.

69. f ( x , r ) = x2 + y 2 3r + + +
xy 1 7 7 . Use the method of Lagrange multipliers to find the
70. f(x,y) = y 3 - x3 + 3xy extreme values of the functionf(x,y) = y 3 2x3 on the +
71. f ( x , y ) = XJ' - 9 - ~5+x circle x2 + y 2 = 1.
72. f(x,y) = x 4 - 4xy + y 4 78. Use the method of Lagrange multipliers to find the
73. f ( x , y ) = xsiny maximum value of the functionf(x,y, z ) = ( x y ~on ) ~ the
74. Find the absolute maximum and minimum values of the sphere x2 + y 2 + z2 = 1.
functionf(x,y) = x2 - xy + y 2 - 3x - 4 on the closed 79. Determine the points nearest to and farthest from the
triangular region with vertices (0,O), (4,0),
and (0,4). +
origin and that are on the ellipsoid 36x2 9y2 4x2 = 36.+
75. Find the absolute maximum and minimum values of the 80. Suppose a manufacturer has the production function
function f ( x , y ) = 2x2 + y 2 x 5 + +
3x on the closed f ( x , y ) = 60xy, where x is labor cost a n d y is material
region bounded above by the parabolay = 1 - x 2 and cost. Suppose that the unit cost of labor is 2 and the unit
below by the line y = -3. (Note: This problem will cost of material is 1 (each in thousands). What will be
involve solving a cubic that clearly has as a solution the increase in production due to increasing the budget
x = -1.) from $600,000 to $601,000? [Use (19.9.2), noting that
76. Use the method of Lagrange multipliers to find the the budget constraint is in thousands of dollars.]
extreme values of the functionf(x,y) = 2x2 6y3 on the +
ellipse x 2 + 3y2 = 1.
SECTION 19.9: OPTIMIZATION AND LAGRANGE MULTIPLIERS 68 1

81. ( a ) Use Lagrange multipliers to obtain the maximum 83. Find three positive numbers that add up to 12 and such
value of f(x,y) = x - 2y on y = cos x, -a/2 2 that the sum of their squares is a minimum.
x 5 a/2. 84. Find three positive numbers that add up to 12 and such
( b ) Sketch the appropriate level curve tangent to that their product is a maximum.
y = cos x. 8 5 . Find the points on the surface x 2 - y t = 8 that are
(c) Check the solution to ( a ) using the single-variable closest to the origin.
maximizing technique of Sec. 5.1.
8 2 . Determine the maximum value off(x,y) = 9xy - x3 - y 3
for x + y 5 6 in the first quadrant.

Miscellaneous Exercises
1. Let f ( x , y ) be differentiable at ( a , b ) , and let F(t) = 12. Let
(x(t),y(t), z ( t ) ) be a space curve whose graph lies on the 2 2

surface z =f(x,y) and goes through the point f(w)= 2/ g ( z ) dz


P = ( a , b , f ( a , 6)). Show that the tangent line to F a t P
lies in the tangent plane to the surface z = f ( x , y ) at P. and let x = t 2 + 2t + 2 a n d y = t 3 - 2t + 2. Find
2. A functionf(x,y) has as its trace in the x-z plane the
+
curve z = x 2 2, and its trace in the plane x = 1 is given that g(4) = 2 .
z = 3 + y . Assuming thatfis differentiable at (1, 0), find 2
1
dt tZO

a n equation for the tangent plane a t the point on the 13. Letf(t) = Jf h ( w ) dw and let t = d x 2 + y 2 + z2. Find
graph off above the point (1,O) in the x y plane. af/ax at the point (1, 1, fi)given that h ( 8 ) = -4.
3. The tangent plane to the graph of a functionf(x,y) a t 14. T h e height and length of a rectangular box with square
the point (1,2, - 1) has equation 2x + 3y + 4z = 4. base are measured and these measurements are used to
Find f,(1, 2) and f,(1, 2 ) . compute the volume. If there was a maximum possible
error of 1 percent in measuring the length and 2 percent
in measuring the height, then what is the approximate
maximum percentage error induced by using these
if (.,y) = (030) measurements to compute the volume?
Determinef,(O, 0) andf,(O, 0). 15. Formulate a Chain Rule appropriate to compute &/dt
given that z = f ( x , y ) , x = x ( u , u), y = y ( u , u ) , u = u(t),
and u = u ( t ) .
16. Formulate a Chain Rule that gives az/ar given that
I o if (x,y) = (0,O) z =f(x,y), x = x(u, u ) , y = y ( u , u), u = ~ ( 7s,) , and
( a ) Show that f is not continuous at (0, 0). u = 4 7 , s).
( b ) Show that f,(O, 0) andf,(O, 0) exist. 17. Prove part (i) of [19.8.6(a)] by following the given out-
(c) .4ref, andf, continuous at (0,O)? line: Let u = (cos a, sin a ) be a unit vector of arbitrary
6. Use the definition of the limit to show that direction, II, a plane through (a, b ) parallel to u and
lim (x2 + y 2 ) = 5 perpendicular to the xy plane, and C, the intersection of
(B,YI-(l,21
nuwith the graph o f t =f(x,y).
(Hint:x2 + y 2 - 5 = (x - 1) +
( y - Z)2 2(x - 1) + + ( a ) Show that D,Zf(x,y) = [D,f(x,y)], cos a +
4(Y - 21.1 [~,f(x,4)l, sin 01.
7. Use the definition of the limit to show that ( b ) Use that f,, = f,, (why is this true?) to show that
lim
(s,,)-(-2,1)
(3x + 5y) = - 1 D,2f(x,y) = f,,cos2 a + +
2f,, sin a cos a f,, sin2 a.
(G) For a # 0, 7 , show that (l/sin2a)D,Zf(a, b ) is a
quadratic polynomial in cot a with no roots if and
>
only if f,,(a, b)&,,(a, b ) - f z s 2 ( a , b ) 0. Show that
D,Zf(u, b ) is likewise positive for a = 0 and a = a.
I 1 (X>Y)= (0,O)
continuous at ( X J J ) = (0,O)? ( d ) Interpret the sign of D,Zf(a, b ) andf,,(a, 6) in terms
9. Use the definition to find f,(2, 3) and f , ( 2 , 3), where of concavity to conclude part (i) of [19.8.6(a)].
f(x,y) = ln(2x 3y).+ 18. At a particular instant the radius of a right circular
cylinder is 12 cm and is increasing a t the rate of 5 cm/s
10. A function f(x,y) is said to be positively homogeneous of
degree n iff(tx, ty) = tnf(x,y) for all t 2 0. Show that iff and the height is 64 cm and is decreasing at the rate of
is differentiable and positively homogeneous of degree n, 3 cm/s. How fast is the surface area changing at this
then xf(x,y) +yfg(x,y) = nf(x,y). (Hint: First deter- instant? Is it increasing or decreasing?
mine a/atlf(tx, ty)] and a/at[tnf(x,y)]; then let t = 1.) 19. A particle is moving along the curve of intersection of
11. Show that iff(x,y) is differentiable and positively ho- the sphere x2 + y 2 + z2 = 17 with the plane
mogeneous of degree n, then f,(x,y) and f,(x,y) are 2x + y + 32 = 13. Find the speed at which the particle
positively homogeneous of degree n - 1 (see Exer- is traveling a t the instant it is a t the point (2, 3 , 2 ) if
dx/dt = 4 at that instant.
cise 10).
682 CHAPTER 19: FUNCTIONS OF SEVERAL VARIABLES

20. Suppose that z = f ( x , y ) ,x = e'cos 8,y = er sin 8. Show with the property that the sum of the squares of the
that vertical distances from the points to the line is called the

(+)'+ (+)'= eZr[ ($)'+ (-9'1 least-squaresline for the points. Since the vertical distance
from ( x i , y i )to the liney = mx + b is Iyi - (mxi
the problem is to find the value of m and b that mini-
+ b)l,

21. Show that the normal line a t any point o n the cone mizes the function
x2 +y2 = z2 intersects the z-axis.
n
22. Use the method of Lagrange multipliers to find the point
on the plane x + 3y - 22 = 4 which is closest to the
D(m, b ) = (yi- mxi - b)2
i=1
origin,
23. Use the method of Lagrange multipliers to determine Show that these values are given by
the points on the plane x + y + t = 4 with the property
that the sum of the cubes of their coordinates is a maxi-
mum.
24. A parallelepiped is to be inscribed in the ellipsoid
x 2 / a 2 + y 2 / b 2 + 9 / c 2 = 1 so that the edges are parallel
to the coordinate axes. Determine the dimensions if the m= b=
parallelepiped is to have maximum volume.
25. Determine three positive numbers with the property
that their sum is a minimum and the sum of their
reciprocals is equal to 1.
26. (The Method of Least Squares) Let (xl,yl),(x2,y2),. . . )

(xn,yn)be n points in the plane. T h e h e y = mx + b

SELF-TEST .).
I
Chapter 20 x

683
684 CHAPTER 20: MULTIPLE INTEGRATION

Section 20.1 : Double and Iterated Integrals

Objectives: ISTRODCCTION
1. Understand the definition of the Our introduction of the definite integral in Chap. 6 was based on determining
double integral as a limit of the area under the graph of a positive function and above some interval on the
Riemann sums. x axis. We now consider the analog of this problem for functions of two
variables. Specifically, let R be a region in the x-y plane and letf(x,y) be a
2. Express double integrals in
positive function defined on R. Our object is to determine the volume (if it
terms of iterated integrals.
exists) of the solid region S below the graph off and above the region R in the
3. Evaluate iterated integrals. x-y plane (see Fig. 20.1). We assume throughout two reasonable restrictions on the
region R :
(1) R is bounded.
( 2 ) The boundary of R is a closed arc that is composed of a finite collection of
smooth arcs.
Some typical such regions R are sketched in Fig. 20.2. In our following discus-
sion we further assume that the volume of S exists and concentrate on how to
determine its value. Later we shall discard this assumption and use our devel-
opment to dejine the volume.

VOLUMES
Since the region R is bounded, we can contain it in a rectangle D whose sides
are parallel to the coordinate axes. We begin by superimposing a grid over D
consisting of a finite number of lines parallel to the x a n d y axes. Consider all
the rectangles determined by this grid that lie wholly wzthzn the region R. Since
there is a finite number of such rectangles (say n), we can order them, getting
R,, R,, . . . , R,. We refer to this collection as a n znnerpartztzon of R and denote it
by P (see Fig. 20.3). By the norm ofP, denoted IlPll, we shall mean the length of
the largest diagonal of the R,s.
Now suppose (x,,y,) is an arbitrary point in R, and consider the parallelepi-
ped (rectangular box) whose base is the rectangle R, and whose height is
f(xz,yz).This parallelepiped has volume V, given by
v, =f(xz,Y,)3 4
X where AA, denotes the area of R,. Furthermore, this volume approximates the
Fig. 20.1 volume of the solid region above R, and below the graph off(see Fig. 20.4). If

0
Fig. 20.2 Fig. 20.3
Inner partition P of R
* X

Fig. 20.4
SECTION 20.1: DOUBLE AND ITERATED INTEGRALS 685

we now sum the volumes of each of the n parallelepipeds obtained in this


fashion, we shall have an approximation to V ( S ) ,the volume of S. That is,
n

i=l

To improve this approximation we can refine the grid by inserting more lines
parallel to the x a n d y axes. This has the effect of increasing n, the number of
rectangles in the inner partition P.We continue in this fashion, letting n grow
without bound. If we do this in such a way that IlPIl + 0, then the corre-
sponding sums will tend to V ( S )independently of how the (xi,yi) are chosen in
each Ri. We denote this by writing
n

Just as we dropped the condition that f be positive in Chap. 6 to define


the definite integral of a function of a single variable independently of area,
we now define the double integral independently of volume.

THE DOUBLE INTEGRAL


Supposef ( x , y ) is defined on the region R and suppose P is an inner partition of
R consisting of the rectangles R,, R,, . . . ,R,.

20.1.1 Definition Any sum of the form X&l f ( x i , y i )AAi, where (xi,yi)
is in Ri and
AAi is the area of Ri, is called a Riemann sum off over P.

Now, as in the single-variable case, f is said to be integrable (over R ) if the


Riemann sums tend to some fixed number. More precisely, we have:

20.1.2 Definition We sayf is integrable over R if there exists a real number Iwith the
following property: Given any E > >
0, there exists a S 0 such that whenever P
<
is an inner partition of R with IJPll S, then

I E j ( x i , y i ) AAi - I
i=l
I< E

for all choices of (xi,yi)in Ri. Iff is integrable over R , then the number I (which
is unique) is called the double integral off over R and is denoted by

We express (20.1.2) symbolically by writing

20.1.3

We shall assume the following result, which is usually proved in advanced


calculus.

20.1.4 Theorem: I f f i s continuous on R, then f is integrable over R.


686 CHAPTER 20: MULTIPLE INTEGRATION

In light of our earlier discussion, iff is positive and continuous on R, then


we define the volume of the solid region S Iring above R and below the graph o f f by

20.1.5

PROPERTIES OF THE DOUBLE IKTEGRAL


The double integral satisfies many properties similar to those of the definite
integral. We shall state these without proof, assuming throughout that f is
integrable over the given regions.

20.1.6 Theorem: Suppose R, and R, are two regions in the xy plane having no
points in common except possibly boundary points. Then

Iff is positive on R, and R,, then (20.1.6) is reasonable, since the volume of
the region S over R , U R, should certainly be the sum of the volumes of the
pieces of S that lie above R , and R,, respectively (see Fig. 20.5). Of course
(20.1.5) can be extended to any finite collection R,, R,, . . , , Rn of disjoint
(except for boundary points) regions in the xy plane.
The remaining basic properties of the double integral are given by:

k a constant

20.1.9 v) dA

Our restrictions on the types of regions R in the xy plane that we are


considering guarantees that each has a well-defined area. This area could be
determined using the methods from Chap. 6. However, we note that if
f ( x , y ) = I for all ( x , y ) in R , then
n n

2x1 i=l

Since this latter sum is just the sum of the areas of all the rectangles in a typical
inner partition P (see Fig. 20.3), it i s reasonable that in the lipit we shall obtain
the area of R.

20.1.10 Thgorem If R is a region in the xy plane, then the area of R is given by

A(R)= JJ dA
R

The evaluation of a double integralfrom the deJinition is in general a difficult


chore. Fortunately, we can draw uppn our knowledge gained in Chaps. 6 and
12 t o aid in this task. We begin by restricting our attention to special types of
regions in the xy plane.
SECTION 20.1: DOUBLE AND ITERATED INTEGRALS 687

TYPE I AND TYPE II REGIONS

20.1.1 1 Definition If h and g have continuous derivatives on [a, b ] , then a region of


the form
R = { ( w1 a)I x 5 b, 4 x 1 I y Ig(.))
is called a type Z region. Similarly, if p and q have continuous derivatives on
[c, d ] , then

R = {(x,JJ)I c 5 r Id,P(JJ) Ix I4 ( Y ) )
is said to be type ZI region.

J Typical type I and I1 regions are sketched in Fig. 20.6. By (12.2.2),page 406,
regions of either type will have boundaries of finite length. Any region R
satisfying our original restrictions (1) and (2) can always be decomposed into a
finite collection of type I and/or type I1 regions. Thus if we can develop a
means for evaluating a double integral over type I and I1 regions, we can
employ (20.1.6) to handle the general case.
X

EVALUATIOS OF DOUBLE ISTEGRALS


Suppose S denotes the solid region below the graph of the positive continuous
la, Tbpe I region
functionf(x,y) and above the type I region R (Fig. 20.7). Fix x between a and b
4 and let n, denote the plane through ( x , 0,O)and parallel to they-t plane. Then
n, slices a cross section from S in the form of a plane region below the trace of
n,
t = f ( x , y ) in and above the interval [ h ( x ) , g ( x ) ](see Fig. 20.8). Since x is
fixed, t = f ( x , y ) is a positive continuous function of the szngle variable y . We
= q J
know from Chap. 6 that the area of this cross-sectional slice is given by
X

(3) A(x) = I,)


g(x)
f ( x , r ) dr

If we now let x vary over [ a , b ] , then ( 3 ) yields a function that gives the areas of
the cross sections of S. It can be shown that A is a continuous function of x on
b T \ p e 11 reglon [a, b]. If we subdivide [a, b] in the usual way, then
Fig. 20.6
A(xA

2
Z
t
f

X Solid region S Cross section of S in plane II,


Fig. 20.7 Fig. 20.8
688 CHAPTER 20: MULTIPLE INTEGRATION

z can be interpreted as a n approximation to the volume of the cross-sectional slice


f of thickness Axi (at xi) as depicted in Fig. 20.9. Adding these volumes we have
n
V ( S )z A ( x i ) Axi
i=1

which is an ordinary Riemann sum of the type considered in Chapter 6. Since A


is continuous on [a, b ] , we conclude that

In light of (20.1.5), we have


J -
Cross-sectional slice of width Axi
X
(4)
with volume % A ( x , : Axr
Fig. 20.9 The integral that appears on the right side of ( 4 ) is called an iterated integral.
There are two types of iterated integrals, and they can be defined independently
of any volume consideration.

20.1.12 Definition If R is a region of type I and f is continuous on R, then the iterated


integral off over R, denoted

is defined by

In (20.1.12) it is understood that one first evaluates the inside "partial


integral"

i(,)
Q(X)

f(X,Y) 4

by holding x constant. The result will be a continuous function of x , which is


then integrated over [a, b].
In an analogous way we define the iterated integral over a type I1 region by

20.1.13

Example 1 Evaluate the iterated integral

I = J2J1(2x + 2y)dydX
0 -1
Solution By (20.1.12),

= a 2
4 x d x = 2x2
SECTION 20.1: DOUBLE AND ITERATED INTEGRALS 689

Example 2 Evaluate the iterated integral

I = JIJu+' (2x
-1 0
+ 2y) dx dy
Solution By (20.1.13),

PROGRESS TEST 1

Evaluate the given iterated integrals:

1. I = 6'lY + (12x2 4y) dx dy

2. I = J1J3'sin(ax2)dy dx
1/2 0

FURTHER EXAMPLES
Y As suggested in our earlier discussion, we can compute double integrals using
f y = 4 - x 2
iterated integrals:

20.1.14 Theorem Supposef(x,y) is continuous on the region R.


( u ) If R is of type I, then

Jp!W = JJ
a
b QQ)

h(x)
f(X!Y)dydX

x ( b ) If R is of type 11, then

Fig. 20.10

Example 3 EvaluateJJR(2x5 + 3x) dA, where R is the region bounded byy = 4 - x 2 and
y=x+2.
Solution Setting 4 - x2 = x + 2 and solving, we obtain x = -2, x = 1. Thus the
graphs intersect in the points ( - 2 , O ) and (1,3); see Fig. 20.10. Now R is a type
I region with description
R = { ( ~ , y )-2l 5 x 5 1 , +~ 2 < y 5 4 - x2)
so, by [20.1.14(~)],
690 CHAPTER 20: MULTIPLE INTEGRATION

s,
1

=
[x2(4 - x2), + 3x(4 - x2) - x2(x + 2), - 3x(x + 2)] dx

= s,
1
(x6 - 9x4 - 7x3 + 9x2 + 6x) dx

It is worth noting that the region R in Example 3 is equal to R, U R,, where


[see Fig. 20.11(a)]
R, = {(x,y)IO Ir 5 3 , - 4 7 5 x I r - 2)
and
R, = {(X7y)13< J 5 4, - -5 x I S}
Thus

= Ji, +(2x5
- ~ I
3x) dA + JJ (2x5 + 3x) dA
R,
[by (20.1.6)]

3 y-2 4 d G
(5) -
-
d s, ( 2 x 5 + 3x) dx dy +J J
3 -&q
(2x5 + 3x) dx dy
These integrals can in turn be evaluated using (20.1.13) and the results added to
obtain the value of the double integral. Of course this latter approach is less
efficient than the former.

A HELPFUL MXEMOSIC DEVICE


By thinking of a n iterated integral as a limit of a double summing process,
many students are better able to keep track of the variables and limits of
integration when performing iterated integration. Although it is possible to
develop iterated integrals in such a way as to make this notion precise, we shall
merely steal its style and use it as a mnemonic device. In reference to Example 3
and the type I description of R, we can think of

lll:r (2x3 + 3x) dy dx


as the limit of a double summing process that first adds the values of
2x5 + 3x along elements of the region in they direction, adding values along
a vertical strip of dys between the lower and upper boundary of R, as in Fig.
20.1 l(b). Since the next variable of integration is x, it is to be expected that the
lower and upper limits on the 4s should be expressed in terms of x. The second
iteration can then be thought of as summing the values on the resulting strips
SECTION 20.1: DOUBLE AND ITERATED INTEGRALS 69 1

Y Y

h;
Fig. 20.1 1

of thickness dx in the x direction from left to right-from the lower limit


x = -2 to the upper limit x = 1.
In a similar way, we can view the second (type 11) approach to the same
double integral as summing the values of 2 x 5 + 3x along a horizontal strip uf
dxs in the x direction from left to right, and then summing the values on the
dy strips in t h e y direction from bottom to top. This point of view makes
obvious the need in this case to use two separate pairs of iterated integrals when
adding values on dxs first: We get two dgerent kinds of horizontal strips
depending on whether we are below or above the liney = 3; see Fig. 20.1 l(c).
Below y = 3, we add dxs along horizontal strips bounded on the left by
x = - d-and on the right by x = y - 2. We then add over such strips (of
thickness dy) in they direction fromy = 0 t o y = 3, gettiiig the first iterated
integral in (5). Abovey = 3, we add values on dxs along horizontal strips
bounded on the left by x = - d-and on the right by x = d-; see Fig.
20.11(c). We then add over such strips in they direction fromy = 3 t o y = 4,
getting the second iterated integral in ( 5 ) .
Thus in general we can think of the Qpe I approach as generating and then summing over
vertical strips and the Qpe 11 approach as generating and then summing ouer horizontal
strips.

REVERSING THE ORDER OF ITERATION


By switching the order of integration, we can sometimes simplify a problem
greatly.

Example 4 Evaluate the iterated integral


. .
Y I = JIJ ex dx dy
O Y
To apply (20.1.13) directly (horizontal strips) we must first determine the
indefinite integral sex* dx. However, this is a nonelementary integral! Suppose
instead we use (20.1.14). By [20.1.14(6)] we can regard the given iterated
integral as a double integral:

where R is the type I1 region given by


Fig. 20.12 R = {(x,y)lO 5 y 5 1,y 5 x 5 l } (see Fig. 20.12)
692 CHAPTER 20: MULTIPLE INTEGRATION

Now R can also be described as a type I region:

(7) Jl ex' dA = JIJx ex' dy dx


0 0
(vertical strips)

Combining (6) and ( 7 ) , we obtain

l1J1
O Y
ex2 dx dy = JISxdy
0 0
ex' dx

= 6' [ e x 2 y ] xdx
0

Whether or not we use the mnemonic device involving strips, it should be


clear that the process of reversing the order of integration is not simply a matter
of interchanging differentials and renaming variables! The form of the new
iterated integral depends on the new description of the region involved. Always
sketch the region.

PROGRESS TEST 2

1. Suppose R is the region bounded by the graphs of


y = x2 a n d y = x.
( a ) Sketch and describe R as a region of type I.
(a) I = 6'; 6
r/2 sinx
f ( W )dy dx
4 Y+3
( b ) Find the associated iterated integral I of
f ( x , y ) = 4 x + 2y over R (vertical strips).
(b) I = JJ
-3 y2-9
f ( X , Y ) dx dy

2. Sketch the region R over which the iterated integral 3. Set Up an iterated integral (or integrals) equal to
is evaluated and give a n equivalent iterated integral f ( x , y ) d A , where R is the region depicted in Fig.
(or integrals) in which the order of integration is 20.13.
reversed.

SECTIOS 20.1 EXERCISES

I n Exercises I to I 4 evaluate the given iterated integral.

1. dz~''+' (60x - 40y) dy dx 2. J3J2z'+4


-1 2 +1
(2x3 - 1) dy dx

4. i T " i u y sin x dx dy
6. 4v/44sins
e u cos x dy dx

10. s4sE*'3
1 s
3y2&aj dx
i~/3iser I tan z
12. y 3 c0s4 x dy dx

14. sls'
0 0
(sin x + s h y ) dy dx
SECTION 20.2: APPLICATIONS OF DOUBLE INTEGRALS 693

I n Exercises I5 to 26, ( a ) express $sRf(x,y) dA as an appropriate iterated integral (or integrals) and ( b ) evaluate the resulting iterated integral.
15. f(x,y) = 2 x +
3y; R is the rectangle bounded by 16. f(x,y) = x 3 ; R is the region bounded by y = x2 and
x = -1, x = 5 , y = -3, andy = 2. y = 4x.
17. f ( x , y ) = xy/(x2 + y 2 ) ; R is bounded by x = 0, y = 1, 18. f(x,y) = 2; R is the disk centered at the origin of
y = 2, and x = y . radius 9.
19. f(x,y) = C O S ( T X ~ ) ; R is the region bounded by x = 1/2, 20. f(x,y) = x sin(y/2); R is the region bounded by j i = 0,
x = 1,y = 0, andy = 2x. y = v/2, x = 0, and x = 2y.
21. f(x,y) = sin3x siny; R is bounded by y = 0, y = T , 22. f(x,y) = x? - 2y3; R is the region above y = x 2 + 4
x = arctan(l/y), x = 7 / 2 . and belowy = 12.
23. f(x,y) = G;
R is the region below y = 2 x , above 24. f(x,y) = l/(x + Y ) ~ / ~R; is bounded b y y = 1 , y = 4,
y = x, and between x = 1 and x = 4. x = 0, and x = 4y.
25. f(x,y) = sinx + cosy; R is bounded byy = 0,y = ~ / 2 , 26. f(x,y) = lox +
109; R is bounded by y = x 2 and
x = 0, x = y . y = &.

I n Exercises 27 to 32 evaluate the given integral b y j r s t reversing the order of integration.

27. llil siny2 gJy dx

10 -

5 -

3 - X

I I I b X
2 5 7

Section 20.2: Applications of Double Integrals

Objectives: AREA AND VOLUME


Use double integrals to determine As in Sec. 20.1 we shall restrict our attention to regions R in the x-y plane that
such quantities as area, volume, can be decomposed into a finite number of subregions of type I or type 11. From
first and second moments, centers (20.1.10) the area of such a region is given by
of mass, and forces of attraction.

20.2.1 A(R)= JJ R

We shall in this and subsequent sections make frequent use of the integration
tables.
694 CHAPTER 20: MULTIPLE INTEGRATION

Example 1 Determine the area of the region R bounded by the curves x = 1yI and
x +y2 = 2.

Solution The region R is sketched in Fig. 20.14. We can decompose R into the two type I
regions R,, R, or the two type I1 regions R,, R,. By symmetry A(R,) = A(&)
and thus we elect to work with this latter decomposition. Now, by inspecting
Fig. 20.14(c), we conclude that
R3 = { ( W J ) l O I Y i 1,Y i x 52 -Y2}
so

1
=2 J
0
(2 -y2 -y) +
y3 y2 1 7
[
= 2 2y----
3 2],=3
-

Example 2 Determine the area of the region R bounded by the graphs of


1
h(x) = lnx and g ( x ) = -( x - 1).
e - 1

J
Solution Since h ( 1 ) = g(1) = 0 and h(e) = g(e) = 1, the curves intersect at the points
(1,O) and (e, 1); see Fig. 20.15. As a region of type I, R has the description
t
R = [(x,3)11 I x I e,- e - 1 (x - 1) < y 1nx

Now R is also a region of type 11. Solving


1
y=Inx and y = - e - 1 ( x - 1)

for x in terms ofy we have x = e, x = ( e - 1)y + 1. Thus


R = {(x,y)lO I y I1,e Ix 5 ( e - 1)y + I}
Fig. 20.15 Working with this latter description of R we obtain
SECTION 20.2: APPLICATIONS OF DOUBLE INTEGRALS 695

Also, from (20.1.5),iff(x,y) is continuous and positive on R, then the volume


of the solid region S lying above R and below the graph off has volume given
by :

20.2.2 W)
= JJfl(x,r)dA
R

Example 3 Find the volume of the solid region S in the first octant bounded on the sides by
the cylinder x 2 + y 2 = 4 and on the top by the plane 3t = 1 2 - 2x - 2y.
Solution T h e region S is sketched in Fig. 20.16. Note that it sits above the quarter-circle
R in the xy plane and below the plane t = ( 1 2 - 2x - 2y)/3. Now
R = { (x,y)l O 5 x 5 2,O <y 5 d-}
so

= JJ [
12 - 2x - 2y
W) R
1 2 & 2
3
1 dA

=yl l ( 1 2 - 2x - 2y)dydx

= ;i2[ 12y - 2xy -y2

=-
1
3 0
2
[ 2d
1 m - 2x d n - (4 - x 2)] dx

=
3
[(6xd w + 24 arcsin(x/2)) + 2(4 -3x2)3/2- 4x + 3
= 4m - 32/9 (We used integral formula 40.) 0

X 'Q)
Fig. 20.16
696 CHAPTER 20: MULTIPLE INTEGRATION

Example 4 Find the volume of the solid region S inside the paraboloid z = 4x2 + 9y2 and
below the plane t = 36.
Solution A sketch of the solid S is given in Fig. 20.17. We first determine the region R
in the x - y plane above which this solid lies. Setting 4x2 + 9y2 = 36, we see that
R is an ellipse centered at the origin. By the symmetry of the problem we can
find the volume in the first octant and multiply by 4. Let R , be the quarter of
the ellipse in the first quadrant. Then

Now V ( S )is the difference of the volumes V, and V, where V, is the volume of
the solid region above R, and below the plane t = 36, and V, is the volume of
the solid above R, and below the paraboloid z = 4x2 + 9y2. Thus

V ( S )= 4 JJ 36 dA - 4 JJ (4x2 + 9y2)dA
R1 Ri

= 4 J i [36 - 4x2 - 9y2]dA

[36 - 4x2 - 9y2]4 dx

=44
( 1 / 9 ) d w ( 1 0 8 - 1 2 2 - 36 + 4x2)dx
(72 - 8 x 2 ) d w d x

= Ti
64
( 9 - x 2 ) 3 / 2dx

=-[9
64 x(9 - x 2 ) 2 / 3
4 $
2 7 x d m
8
243
+ -arcsin
8
I):( 3

0
= 108n

(We used integral formula 47.) 0

Y
t
SECTION 20.2: APPLICATIONS OF DOUBLE INTEGRALS 697

PROGRESS TEST 1

1. Determine the area of the region bounded by the


curves y = x 2 and y = x .
2. Determine the volume of the solid region S in the
first octant below the graph of the cylinder
z 2 + y 2 = 1 and above the triangular region R in
the xy plane bounded b y y = x , y = 1, and x = 0.

MOMESTS AND CESTER O F MASS


J In Chap. 12 we used the definite integral as an aid in giving mathematical
t definitions for such physical quantities as the center of mass of a lamina of
constant density. By a lamina we mean a very thin flat sheet, and by
density we mean area density, that is, mass per unit area. We consider now a
lamina L with nonconstant density. If R is the region that L occupies in the x-y
plane, then we denote the density a t (x,y) by p(x,y)(see Fig. 20.18). We shall
X assume throughout that p is continuous on R.
We first examine the problem of determining M , the mass of L. We begin by
considering an inner partition of R into n rectangular subregions R,, R,, . . . , R,.
Now the mass of that part of L occupying R, is approximated by

Lamina L occupying the region R


Fig. 20.18 where (xi,yi) is some point in R,. Note that (1) is only an approximation since
p ( x , y ) need not be constantly equal to p(x,,y,) on the rectangle R,. However, if
IlPlI is quite small, then the continuity of p ensures that p ( x , y ) will not vary
much over Ri. Thus the total mass M is approximated by:
n
M z C P(xi,yi) M i
i=l

Since this latter sum is a Riemann sum for the continuous function p over R ,
we dejine the mass of L by

20.2.3

Suppose we wish in turn to determine the center of mass (<A for L. Our first
chore would be to determine the first moments of L about the axes. To do this
we think of each portion of L occupying the rectangle R, in the partition P as
having its mass concentrated at (xi,yi). Then (see Fig. 20.19)
(Mx)i z Y i p ( x i , y i ) ui (My), xiP(xi,yi)
t X where (Adx),and represent the first moments of Ri about the x andy axes,
respectively.
Since the moment of the sum is equal to the sum of the moments, we can
approximate the first moments of L by
n n
Fig. 20.19

Taking limits, we define Mxand M , by:

20.2.4
698 CHAPTER 20: MULTIPLE INTEGRATION

20.2.5

It follows that the center of mass is given by (<2,


where

20.2.6

Although the above work led to double integrals, we of course use iterated
integrals to evaluate the double integrals.

Example 5 A lamina L occupies the region in the first quadrant bounded by y = $ and
x = 4. Given that the density at each point is proportional to the square of the
distance from the origin, find the center of mass of L.
Solution The lamina is sketched in Fig. 20.20. Now p ( x , y ) = k ( x 2 + y 2 ) , so

Y where R is the region given by


A
Thus

1 *J
Fig. 20.20

Now

= k i ( x3
y + T x2
)dx

and
Mu = k S
0
s
4 v 5

0
x ( x 2 + y 2 )dydx

= k s 04 [ x > + 2 ] 3: d x
SECTION 20.2: APPLICATIONS OF DOUBLE INTEGRALS 699

Thus

In a similar way we can obtain defining integrals for moments of inertia. For
example, consider the portion of L occupying the rectangle Ri,whose mass we
consider as being concentrated at (xi,yi).Then (Zz)i, the moment of inertia of
this portion of L about the x axis (see Fig. 20.21), is approximated by
(I,)i = (distance from x axis)2 x mass
=Yi2P(xi,Yi) AAi
Hence I,, the moment of inertia of L about the x axis, is approximated by
Fig. 20.21

Thus the defining integral for I, is given by

20.2.7

In an analogous way the moments of inertia about they axis and the origin
(the polar moment) are defined by, respectively,

20.2.8

and

20.2.9

Example 6 Find the polar moment of the lamina occupying the region R bounded by
y = x2,y = 4, and x = 0 if the density is inversely proportional to the square of
the distance from the origin.
Solution The region is sketched in Fig. 20.22. Now
k
P ( X 9 ) = and R = {(x,y)/O5 x 5 2 , x 2 < y 5 4 )
Thus

2 16k
= kJ2J4@a!x = k J (4 - x 2 ) d x = -3 0
0 22 0

GRAV-ITATIONAL ATTRACTION
A particle of mass ml located at the point Q exerts a force of gravitational
attraction F on a particle of mass m2 located at P given by

where g is the gravitational constant. (This is a version of Newtons inverse-


Fig. 20.22 square law of gravitational attraction.)
7R0 CHAPTER 20 MULTIPLE INTEGRATION

Example 7 Assume L is the lamina occupying the triangular region R bounded by x = 1 ,


x = 2,y = 0, andy = x - 1, and assume that the density is proportional to the
distance from they axis. Set up iterated integrals to give the components of the
force of gravitational attraction F exerted by the lamina L on a particle of mass
p located a t the origin.
Solution A sketch of the lamina is given in Fig. 20.23. As usual, we partition R into
rectangular subregions. We approximate the mass of each of these and treat
each as if its mass were concentrated at a single point (x,,yz) in the rectangle R,.
The mass of this zth piece is given by
m, zkx, AA,
Thus
F, z gpkxa AAa(x, - 0,Ya - 0)
IIK ~ , - 0)113
- o,Y,
gives the force of attraction that this portion of the lamina exerts on the particle
located at the origin. The components of F, are, respectively,
gpkxa *A, and
gpkxtya '4
VFG? VFTZ
I If we add these and pass to the limit, we get the components of F, the force of
Fig. 20.23 attraction of L on the particle located at the origin. These are, respectively,

Hence

PROGRESS TEST 2

1. Determine the first moment about they axis of the 2. Determine the moment of inertia about the x axis of
lamina occupying the region bounded by x = 0, the lamina occupying the region bounded by
y = 0, and y = 1 - x if the density is proportional y = x 2 , y = 0, and x = 2 if the density is given by
to the distance from the x axis. p(.,y) = x + 2.

SECTION 20.2 EXERCISES

In Exercises I to 6 determine the area of the region R bounded by the given curves.
1. y = x3,y = x 2
3. y2 = 4x,y = 2x - 4
5. 2 x 2 = 2 y + l , x + 2 = 2 y

In Exercises 7 to 12 determine the volume Ofthe indicated solid region S.


7. The solid in the first octant below the graph of 8. +
The solid inside the sphere x 2 + y 2 t 2= 4 and above
2x y + + 3t = 6. = 1.2
9. T h e solid in the first octant bounded by the cylinders 10. The solid bounded by the planes x + y +t = 2, t = 2 ,
x 2 + y 2 = 9 and x 2 +
t 2= 9. and the cylinders x = y 2 and y = x 2 .
11. The solid in the first octant bounded by x 2 + y 2 = 4, +
12. The solid region bounded above by x 2 + y 2 t 2= 1
t = y , z = 0, and x = 0. and below by t = 0.
SECTION 20.3: TRIPLE INTEGRALS, WITH APPLICATIONS 701

In Exercises 13 to 18 determine the mass and the center of mass of the given lamina.
13. L is the triangular lamina in the first quadrant bounded 14. L occupies the region bounded by y = 4 - x 2 and
by the line x + y = 2 , given that the density function is y + x + 2 = 0; the density is proportional to the dis-
p ( x , y ) = x2 - 2y. tance from the liney = 5.
15. L occupies the region bounded by x + y * = 1 and 16. L occupies the region bounded by x = 4 - 3y2 and
y = x + 1; the density is proportional to the distance x = y 2 ; the density is constant.
from x = 7. 18. L occupies the region bounded byy = x 2 andy = x ; the
I?'. L has the shape of a n isosceles right triangle and the density is proportional to the distance from the x axis.
density is proportional to the distance from the vertex
opposite the hypotenuse.

In Exercises 19 to 23 determine the moment of inertia of the given lamina about the indicated line.

19. L occupies the region bounded by y = x 2 , y = x , 23. L occupies the region bounded by y = x , y = 1, y = 4,
p ( x , y ) = k x ; about the x axis. and x = 0, and the density is proportional to the dis-
20. L occupies the region bounded by x = y 2 + 1 and tance from the x axis; about t h e y axis.
y - x + 3 = 0, and p ( x , y ) = k y ; about the line 24. Set up (but do not evaluate) a n iterated integral that
x = -2. gives the first moment of the lamina occupying the
21. L occupies the quarter-disk in the first quadrant region bounded byy2 = x - 1 a n d y = x - 3 about the
bounded by x 2 + y 2 = 16, and p ( x , y ) = 2x + 2y; about +
liney = x + 3 if the density is given by p ( x , y ) = xy x.
the h e y = 6. (Hint: Use the formula for the distance from a point to a
22. L occupies the region bounded by x = y 2 and x = y , and line developed in Chap. 17, page 585.)
the density is proportional to the sum of the distances
from the x a n d y axes; about the liney = 4.

Section 20.3: Triple Integrals, with Applications

Objectives: INTRODUCTIOS
1. Evaluate triple iterated integrals. If w = f(x,y,z)is defined on some bounded solid region S, then we can define
the triple integral off over S , denoted
2. Apply the theory of the triple
integral to solve physical and
geometric problems.

in much the same way as we defined the double integral. Since S is bounded, we
can contain it in a parallelepiped D whose faces are parallel to the coordinate
planes. We begin by superimposing a grzd over D consisting of a finite number of
planes parallel to the faces of D.This has the effect of subdividing D into a
z
collection of smaller parallelepipeds (boxes), a finite number of which lie wholly
4 within S (see Fig. 20.24). We denote these by S,, S,,. , . ,S, and refer to them as
D an inner partitzon P of S. A few typical elements from an inner partition are
sketched in Fig. 20.25. By I1PI1, the norm of P, we shall mean the length of the
largest diagonal of the St's.
Now for each z let (x,,y,, 2,) be an arbitrary point in S,and let IV, denote the
volume of S,. A sum of the form
n

/.
Y

+
is called a Riemann sum off over P.If we now let n + 00 in such a way that
lIP1/ + 0, and if these Riemann sums tend to some fixed number indepen-
/
X ently of how the (xi,yi,zi) are chosen in Si,then we say f is integrable over S
Fig. 20.24 and we write
702 CHAPTER 20: MULTIPLE INTEGRATION

J J
X X

Fig. 20.25 Fig. 20.26

20.3.1 Definition

By a regular region in three-space we mean a region of the form

(1) S = {(X,Y,2 ) I (x,JJ) E R, H ( x , y ) I I G(x,y))


where R is a plane region of the type considered in Section 20.1 and where H
and G have continuous partials on R (see Fig. 20.26).
Similarly, regular regions can be of the form

(2) S = {(X,Y, 2 ) I (x,4 E R, w,I Y I 2) a x , 2))


or

(3) s = {(X,Y, 2 ) I ( Y ,2 ) E R, U ( y ,2 ) I x I V ( Y ,2 ) )
where R is a region in the x-z or y-z plane and the functions P, Q, U, V have
Y continuous first partials. (See Fig. 20.27 for a sketch of the former.)
Fig. 20.27 We shall assume the following:

20.3.2 Theorem If f ( x , y , z ) is continuous in the regular region S, then


Jllsf(x,y,z ) dV exists.
Iff(x,y,z) = 1, then the Riemann sums reduce to Zr==,A q , which approx-
imates the volume of S ; and thus we have another way to realize the volume
of a solid:

20.3.3

EVALUATIOK O F TRIPLE INTEGRALS


It can be shown that a triple integral over a region can be evaluated by means
of iterated integrals. In particular, if
SECTION 20.3: TRIPLE INTEGRALS, WITH APPLICATIONS 703

then

20.3.4 JJp,?[ J d.7 =


b o(x) G ( x , y )
J
h(x) &,Y)
f ( X A 2) * dy dx
The iterated integral is obtained by first holding x and y constant and
evaluating

s,,,,f ( u*
G(x,u)
2)

where H ( x , y ) and G ( x , y )are t limits. This reduces the triple iterated integral to
a double iterated integral of theform

ib[;>(x,Y) dy dx
which is evaluated as in Sec. 20.1.

Example 1 Evaluate

2 Solution

= i2ix (8x2y2 - 8 9 ) dy dx

= 4 [8x2$ - 4xy2

8x5

Most triple iterated integrals arise directly from geometric or physical


J applications, the simplest of which is the determination of a solids volume.
Fig. 20.28

Example 2 Determine the volume of the solid region S i n the first octant bounded below by
t= 2 and above by the plane x + y t = 6. +
Solution By (20.3.3), V ( S ) = llJN
dV. To evaluate this integral we shall first express S in
the form (1) and then use (20.3.4) to express the triple integral as an iterated
integral. A sketch of S is given in Fig. 20.28. Eliminating z from the two
equations, we see that S sits over the region R in the first quadrant of the x y
plane bounded by the x and y axes and the straight line x + y = 4. Thus
R = { ( x , J ) I O 5 x 5 4, 0 < y <4 - X )

Now ( x , y , t ) is in S if and only if ( x , y ) is in R and


2 52 5 6 - x - y
so
S = {(x,y,t)IO5 x 5 4, 0 5~ 5 4 - X , 2 5 2 5 6 -x -y)
904 CHAPTER 20: MULTIPLE INTEGRATION

From this description of S and the order of the differentials, dV = dz dy dx, we


can now apply (20.3.4) to evaluate SSSs dV:
4-+ 6-X-Y
V ( S )= J 4 J dzdydx
0 0 2

@ dx

=14[4y-xy-- Y2
2 1
4-x
0
dx

We evaluated the triple integral in Example 2 using an iterated integral


whose order of iteration and associated limits are dictated by (20.3.4).However,
this order of integration is by no means unique. Since there are six ways to order
the three variables x,y, and 2, there are, in general, six possible iterations; each
is associated with a particular description of S. For example, if a solid region S
were described as

The inside integral would be obtained by holding x and z fixed and integrating
with respect toy.
O n the other hand, given a particular triple iterated integral, we can, as with
the double iterated integral, read off from the limits of integration the bound-
aries of the associated solid, paying particular attention to the order of the
iteration. The outside differential tells you which variable is being limited by
the constant limits on the outside integral, the middle differential indicates the
variable being limited by the curves on the middle integral, and the inside
differential indicates the variable being limited by the surfaces on the inside
integral. For example, the solid S associated with the following iterated integral

may be described as (see Fig. 20.29)


z= Y
S = { ( x , y , z ) / OIx 5 1, 0 5 2 5 x2, 0 < y 2 l}
The different iterated integrals that can be used to compute a given triple
integral J J l s f ( x , y , z ) dV require different descriptions of S.The next example
illustrates how to form these descriptions using the projections of S into the
Fig. 20.29 coordinate planes.
SECTION 20.3: TRIPLE INTEGRALS, WITH APPLICATIONS 705

Example 3 Give the six iterated integrals for the triple integral of Example 2.
Solution O u r approach is to sketch the projections of the solid S, as in Fig. 20.28, in each
of the coordinate planes; see Fig. 20.30(a). Each such projection has two
descriptions-in effect, a type I and a type I1 description. The projection R, in
the xy plane is obtained by eliminating z from the equations of the top and
bottom of S, z = 6 - x - y and z = 2. This yields one of the boundaries,
namely, y = 4 - x. The others are x = 0, y = 0; thus [see Fig. 20.30(b)]
R, = {(x,y)IO Ix 5 4 ,0 Iv I 4 - x>
= {(x,y)lO <r <4,0 I x I 4 - Y >
Similarly, by lettingy = 0, we have the descriptions for R,:
R, = { ( x , z ) ~ O 2 x < 4,2 < z I 6 - X }

= { (x,z ) 12 z I 6, 0 I x I 6 -z)} [see Fig. 20.30(c)]


Finally, we let x = 0 to get R,:
R, = {(y,z)lO I V 5 4 ,2 5 2 I 6 - Y >
= { ( y ,z ) 12 2 z 2 6 , 0 i y 6 - z }< [see Fig. 20.30(d)]
Now, in turn, ( x , y , z ) is in S if and only if
( x , y ) is in R, and 2 z I6 - x -y
or
(x,z ) is in R, and 0 <y < 6 - x - z
or
( y , z ) is in R, and 0 5x26 --y -z

h X 4

b\ di

Fig. 20.30
706 CHAPTER 20: MULTIPLE INTEGRATION

Thus, putting all this information together, we have the following six iterated
integrals, given in the order corresponding to the previous descriptions:

4 4-y 6-x-y

=ii1 4 6-2 6-2-8


hdxdY

=il 6 6-2 6-2-t


dydzdx

=it J 4 6-11 6-Y-2


dydxdz

=iih
= s6s6-Z16-~-t
dxdzdy

dxdydz 0
2 2

We express the pattern developed in Example 3 to assist in more complicated


situations.

20.3.5 Guidelines for Setting u p Triple


Iterated Integrals O v e r a Solid S
( u ) The outer two differential variables determine the coordinate plane in
which we project the solid S (forming a plane region R ) .
( b ) The limits of the outer two integrals are then determined by the boundaries
of R, and depend on whether R is given a type I or type I1 description.
(c) The upper and lower limits on the inner integral are the boundary surfaces
of S in the direction of the inner differential variable expressed in terms of
the other two.

d MNEMONIC DEVICE FOR TRIPLE


ITERATED INTEGRALS
As with double iterated integrals, some students find it helpful to view the triple
iterated integral as the limit of a triple summing process over the solid S where
the values of the functionf(x,y, z ) being integrated are added in three stages,
with the order depending on how the solid region is described. We imagine the
values off being added over the solid S in a three-stage geometric process.
We begin by adding over boxes stacked in the direction of the inside differ-
ential variable. This yields a long prism-shaped box between the boundaries of
S in the direction of that inside differential variable. It is necessarily perpen-
dicular to the plane of the outer two differential variables in which S is
projected. We then add over such prisms in the direction of the intermediate
differential variable to form a slab that is perpendicular to the direction of the
outer differential variable. Finally we add over such slabs between the limits
of the outer integral.
Figure 20. 31 illustrates the process for
[ b);:l 4G(2,~)
f(X,Y > z ) dz 4dx
when S is a solid regular region based on the xy plane. The values offare added
first in the boxes in the z direction frsm the bottom surface ( z = H ( x , y ) = 0 in
SECTION 20.3: TRIPLE INTEGRALS, WITH APPLICATIONS 707

\z = H ; x , J ) = 0
4
Fig. 2 0 . 3 1
t
this case) to the top, G(x,y). The values o f f a r e then added on the resulting
prisms across a slab in they direction fromy = h(x) t o y = g ( x ) . Finally, the
values offare added on slabs ranging in the x direction from x = a to x = 6.

CHANGING THE ORDER OF ITERATIOR


x = -J* I x = .Ji4. -4.2
The next example involves switching the order of iteration, which requires that
Fig. 20.32 the solid S first be recovered from the original iterated integral.

Example 4 Given that

( a ) sketch the region S, and ( b ) express I as an iterated integral(s) with


dV = d y d x d z .

Solution ( a ) From the iterated integral, we have

S= {(x,y,z)~O<y<2,- d r n I x I q-. 2y<z<x*+y2)


To assist in sketching S,which we need do before applying [20.3.5(a)],we
project S into the xy plane ( z = 0) getting
R=((x,y)lO<yI2, --<XI 1-
If x = k d m , then x 2 = 2y - y 2 , or x 2 + y 2 - 2y = 0. Completing
the square i n y , we have x 2 + ( y - 1)2 = 1, which is a circle of radius 1
centered on they axis a t (0, 1); see Fig. 20.32. Now ( x , y ,z ) is in S if and only
< <
if ( x , y ) is in R and 2y z x 2 + y 2 . Thus S is the solid region bounded
below by the paraboloid z = x 2 + y 2 and above by the plane z = 2y (see
Fig. 20.33).
( b ) Since the order of integration is 4 dx dz,by [20.3.5(a)]we need the projec-
tion R, of the region S in the x-z plane. To do this we eliminatey from the
J equations
x
Fig. 20.33 (4) z = 2y and z=x2+y2
708 CHAPTER 20: MULTIPLE INTEGRATION

4
t

-1 I 1

Fig. 20.34

obtaining
z=x2+- 22
4
or, equivalently, by completing the square in z and rearranging, we have
(2 - 2)2
+x2=1
4
Thus R, is an ellipse in the x-zplane centered on the z axis (see Fig. 20.34).We
express x in terms of z to get
x = * d m
and so
R, = {(x, t)10 5 t 2 2 , - d T5x 2 d m }
Hence we know that

where the inner limits remain to be determined. By [20.3.5(c)] it remains to


solve the equations in (4) for y in terms of x and z. We get y = t / 2 and
X *
y = d-. Since S lies to the right of the X-t
plane, we choose the plus sign
Fig. 20.35 in the latter equation, and conclude that z/2 2 y 2 d-; see Fig. 20.35.
Hence

PROGRESS TEST 1

Evaluate the following integrals:

1. sss
1

-1 0
x

0
xz
xzdydtdx

2. JfJst d y where S is the solid region in the first


octant inside the cylinders x2 + y 2 = 1 and
xz+ z2 = 1. (Sketch S carefully.)
SECTION 20.3: TRIPLE INTEGRALS, WITH APPLICATIONS 709

APPLICATIONS OF THE TRIPLE ISTEGRAL


The principle involved in applications of the triple integral is the same as for
the double integral: Obtain an approximation of the desired quantity as a
Riemann sum and then pass to a limit that is a triple integral and describes that
quantity exactly. Throughout we shall have occasion to use the fact that the
mass of a substance of density k and volume V is kL! Because of the triple
integral, our ability to determine mass and moments of solids will extend
greatly our capability developed in Sec. 12.4, where we were limited to shapes
and densities that could be described using only a single variable.

Example 5 Find the mass M of a cylindrical solid S of radius a and height h if the density at
any point is proportional to the distance from the base of the solid.
Solution A right circular cylinder of radius a, whose axis is the z axis, has equation
x 2 + y 2 = a2. Thus

If we impose a grid on S, then a typical subregion Si has volume A K , If


(xi,yi,z,)is a point in Si and k is the constant of proportionality, then the
density at this point is kzi and so the mass of Si is approximated by
M iz kziA&; see Fig. 20.36. Since
n n

a'

c we can pass to the limit to obtain

,J
Fig. 20.36

Example 6 Set up a n iterated integral that gives the moment of inertia Zz about the z axis
of the homogeneous solid S bounded below by the upper nappe of the cone
32' = x 2 + y 2 and above by the plane z = 1.
Solution A sketch of the solid S with a typical subregion S, is given in Fig. 20.37. The
distance from the point (x,,y2,2,) in S, to the z axis is d m a n d the mass
of S, is approximated by k A V,. Thus the moment of inertia about the z axis of
this subregion is approximated by
1% =: ( m T ? ) " k
(Iz A V,1
and hence

=c
n
1,
I-=1
k(X,2 + y,2) A y
710 CHAPTER 20: MULTIPLE INTEGRATION

Passing to the limit, we have

To obtain the projection of S in the xy plane (and thus use dt as the inner
differential), we eliminate t from
t =1 and 3t2 = x 2 +y2
getting
4 x2 +y2 = 3
a circle centered at the origin of the xy plane of radius fi.This has the

Fig. 20.37 Now, as indicated in Fig. 2 0 . 3 7 , ( x , y , z ) is in the cone S if and only if ( x , y ) is


in R and
d(x2 +y2)/3 5 2 I 1

Although the integral in Example 6 can be evaluated, it is not particularly


easy to do so. We shall give a technique in the next section that simplifies this
chore considerably by switching coordinate systems.

PROGRESS TEST 2

1. Find the first moment about the y - t plane of the


homogeneous solid in the first octant bounded by
the cylindery = x 2 and the planes x = 1 and z = y .
2. Set up but do not evaluate a triple iterated integral
for the second moment about the t axis of the solid
bounded above by z = 12 - x 2 - y 2 and below by
t = 3, where the density at any point is inversely
proportional to the distance from the origin.

SECTION 20.3 EXERCISES

I n Exercises I to I 2 evaluate the indicated iterated integral.


SECTION 20.3: TRIPLE INTEGRALS, WITH APPLICATIONS 711

6. slsss
0 0 0
ex dx dz dy 6. (l(z~y 12x(x2 + y 2 ) dz dx dy

6162-i62-v-z
8. 24z dx dy dz

9. JTi2 J1J 12y cos x dz dy dx 10.


0 0 0

12.

In Exercises I3 to 17 use triple integration to find the volume of the given solid.
13. T h e solid region in the first octant bounded above by the 16. T h e solid region below the paraboloid
plane x + y +
t = 12 and on the sides by the cylinder z = 9 - ( x 2 + y 2 ) and above the xy plane.
2 S y 2 = 9. 17. T h e solid region bounded on the sides by the cylinders
14. T h e solid region above the plane t = 2 and below the y 2 = x and x 2 = y , below by the plane x + y z = 2, +
hemisphere z = \/18 - x 2 - y 2 . and above by the plane t = 2.
15. The solid region in the first octant above the p l a n e t = 1
and below the plane 2x 3y 42 = 12. + +
In Exercises I8 to 21 the iterated integral represents the volume of a region S. Sketch S and gioe the limits of integration on the indicated integrals.

18. V ( S )= JJ J
0
1 3-31

0 2
(24-12x-4~)/6
dz dy dx 19. V ( S )= s J
0
3

0
&2
J0
2y

dzdydx

(a) JJL dV, where dV = dy dz dx (a) JJi dV, where dV = dx dz dy

(b) JJ&dV, where dV = dxdydz (b) JsJ dV, where dV = d y d z d x

2 2-x 2-x--y 2 1/x 22

20. V(S) = JJ J dzdydx 21. V ( S )= J J dydxdz


0 0 0 1 0 0

(a) JJJ dV, where dV = dy dz dx (a) JJi dV, where dV = dx dy dz

(b) JJi dV, where dV = dx dy dz


( b ) Explain why JJJs dV cannot be expressed as a single
iterated integral if dV = dz dy dx.

22. Find the moment of inertia about the z axis of the 27. Determine ( a ) the mass and ( b ) the center of mass
homogeneous solid inside the cylinder x 2 + y 2 = 9, (F,y,F) of the solid S in the first octant bounded by the
above the plane t = 0, and below the plane z = 3y. plane x = 2 and the cylinder z2 + y 2 = 9, where the
23. Find the moment of inertia about they axis of the solid density a t a point is proportional to its distance from the
in the first octant inside the cylinder x 2 + y 2 = 1 and x axis. (You will need to find three moments.)
below the paraboloid z = x 2 + y 2 , where the density is
proportional to the distance from the z axis.
24. Determine the mass of the solid region above the x;y
plane and below the paraboloid t = 8 - x 2 - y 2 if the
density a t any point is proportional to the distance from
the base.
25. Determine the mass of the homogeneous solid bounded
above by z = 2 , below by z = 1, and on the sides by the
paraboloid z = x 2 + y 2 .
26. Determine the mass of the solid region bounded on the
sides by the cylindery = x 2 and the planey = 2x, on the
bottom by the xy plane, and on the top by the plane
z = x + y . The density is proportional to the distance
from the x-z plane.
712 CHAPTER 20: MULTIPLE INTEGRATION

MULTIPLE INTEGRALS IN
Section 20.4: OTHER COORDINATE SYSTEMS

Objectives: THE DOUBLE INTEGRAL IN POLAR COORDISATES


1 . Evaluate double integrals in (You may find it helpful to review polar coordinates at this point.) Our
polar coordinates. objective is to define the polar double integral
2. Evaluate triple integrals in
cylindrical and spherical (1) J p r l 6) dA
coordinates.
where R is a polar region of type I or type 11 as depicted in Fig. 20.38. As in the
3. Transform a multiple integral in rectangular case, we shall assume that the bounding curves have continuous
one coordinate system to an derivatives. We begin by imposing a polar grid over R consisting of concentric
equivalent integral in another circles centered at the pole and half-lines emanating from the pole (see Fig.
coordinate system. 20.39). This in turn determines a n inner partition P of R consisting of a finite
number (say n ) of polar rectangles R,, R,, . . . ,R, that lie wholly within R.
Again we let 1lPI1 denote the length of the longest diagonal of the R,)S.A
Riemann sum off over P is any sum of the form
10 = P
n
(2) Cf(r,,6,)4
i=1

where (r,, 6,) is an arbitrary point in R, and &ti denotes the area of R,. Now if

(3)

exists independently of how the (r,, 19,)are chosen in R,, then we sayf is integrable
over R and we denote the value of the limit by JJE f ( r , 6) dA. As before, we
Type I polar region assume the following:
(a:
20.4.1 T h e o r e m If f ( r , 6 ) is continuous on the polar region R , then f
is integrable over R.

If f ( r , 6) = 1 on R, then the sum ( 2 ) becomes Zyrl AA,,the sum of the areas


of the polar rectangles in the inner partition P of R. Thus, in the limit, we
obtain the area of R.

0 = 4

Type I1 polar region


(bj
Fig. 20.38

Inner partition P of R
Fig. 20.39
SECTION 20.4: MULTIPLE INTEGRALS IN OTHER COORDINATE SYSTEMS 713

20.4.2 A(R)= Ss dA
R
To gain some insight into how we can express the polar integral in (1) as an
iterated integral, we examine U,.Suppose (Ti,Si) is chosen as the center of R,
(see Fig. 20.40). Now Mi is the difference of the areas of two circular sectors of
+
"angle width" Mi. The larger sector has radius rz Ari/2, and the smaller has
radius ri - Ari/2. Thus, applying the formula for the area of a circular sector,

='[ri2
2
+ ri Ari +
= ri Ari ABi
Polar rectangle R, of area The factor ri in the expression for Mi leads to a factor of r in the associated
A A , = r, Arc All,
iterated integrals-one iteration order for each of the two ways of describing R.
Fig. 20.40
20.4.3 Theorem Supposef(r, 6') is continuous on the region R.
( a ) If R = {(r, 6') 101 5 6' 5 p, g(6') 5 r 5 h ( 0 ) } , then

( b ) If R = { ( r , 6') l a 5 r I b,p(r) 5 0 5 q ( r ) } , then

Example 1 Determine the area of the region R in the first quadrant that is inside the
+
cardioid r = 2(1 cos 0) and outside the circle r = 2.
Solution R is sketched in Fig. 20.41. It can be described as a type I region:
0 = np R= {(r,0)~OI0~a/2,2~r~2+2cos6'}

1 Hence, by (20.4.2) and [20.4.3(a)],

A(R)= JJ dA =
R
4T/212+zcos

r=2
dB
0
r dr d6'

=L
J2 (8 cos 0 + 4 cos2 0 ) d0 = 4 + a / 2 0

The next example illustrates that certain multiple integral problems posed in
Fig. 20.41 terms of rectangular coordinates are simpler to solve using polar coordinates.

Example 2 A lamina L occupies the annular region R in the first quadrant between the
semicirclesy = d nandy = d n .
Find the moment of inertia of L
about the x axis if the density at any point of L is inversely proportional to its
distance from the origin; see Fig. 20.42(a).
Solution In polar coordinates the two semicircles have equations r = 1 and r = 4,
respectively. Thus a description for R in polar coordinates is provided by
714 CHAPTER 20: MULTIPLE INTEGRATION

Y 4'

t e=rr'2 f

e=o
(a)
Fig. 20.42

R = {(r, 6 ' ) 10 5 6' m/2, 1 5 7 5 4)


(which can be regarded as either type I or type 11.)
If R, is a typical polar rectangle in an inner partition of R, then the distance
from ( T i , 0,) in R to the x axis is r, sin 0,; see Fig. 20.42(b). Since ri measures the
distance from the origin, the mass of the piece of L occupying R, is approxi-
mated by

Hence the moment of inertia of this piece about the x axis is approximated by

(I,), z (ri sin 6',)2


= kr, sin2 0, AA,
Thus
n
I, z k7, sin2 8, AA,
i=l

+ 0, we have
Taking the limit as ~~P~~

I, = k J&r2 sin26' dA

=k t r2 sin2 6' r dr d6'

=k t"'2[4 r3 sin2 6' dr d6'

=
k
r4sin2d 1 r=4

r=1
d6'

= ~i
255k "I2 255km
sin26'dB = - 0
16

If we had tried to solve Example 2 in rectangular coordinates, then I, would


have the form
SECTION 20.4: MULTIPLE INTEGRALS IN OTHER COORDINATE SYSTEMS 713

J To express this as an iterated integral, we must first subdivide R into the regions
f
I
R, and R, as in Fig. 20.43. Thus
1 VGi? ky2 diz2 b
2

4 = Jo J m mdydx + J 4 J 1 0
q-4 dx
Clearly, in this case the polar coordinate approach is to be preferred!
More generally, it is often possible to transform an iterated integral in
rectangular coordinates to an equivalent iterated integral in polar coordinates.
For example, if the region R has the description
R = { ( x , y )I a I x I b, 4 x 1 Iv I
4
in rectangular coordinates, and if the same region has description
J=o R = { ( r , 0 ) I a I0 I P,p(O) Ir Id6'))
J=-

Fig. 20.43 in polar coordinates, then

JJ;;./(x,Y) dy dx = ip[t:) f ( r cos 0, r sin 0 ) r dr d0

There are several other possibilities, but the procedure in each case is to describe
the region R in polar coordinates, replace f ( x , y ) byf(r cos 0, r sin O ) , replace
dy dx (or du dy) by r dr d6' (or r d0 dr) and form the new iterated integral(s) using
the polar description of R. Of course, the order in which the differentials appear
will depend on the description of R.

Example 3 Evaluate the iterated integral

I = 4i d16-y2 ( x 2 + y 2 )dx dy

by first transforming to an equivalent integral in polar coordinates.


Solution From the iterated integral we read off the region over which the integral is
evaluated:
Y
o=4
t 2
R = {(x,v)lO I y I 4 , O Ix I1-
Since R is just the portion of the disk x 2 + y 2 I 16 lying in the first quadrant
(see Fig. 20.44), it has polar description
R = {(r,O)IO I0 5 77/2,0 Ir I4)
Now
+
f ( r cos 0, r sin 0 ) = ( r cos 0 ) 2 ( r sin O ) ,
= r2(cos26' + sin20) = r2
and thus

Fig. 20.44 o=o

Example 4 Determine the volume of the solid S bounded below by the xy plane, on the
sides by the cylinder x 2 + y 2 = 1, and on top by the sphere x2 + y 2 z 2 = 9 +
(see Fig. 20.45).
716 CHAPTER 20: MULTIPLE INTEGRATION

Solution By (20.1.5),

V ( S ) = JJ d9 - x2 - y d A
t R

'2 +-i2 + z2 = 9 Now in polar coordinates, R is just the unit disk


R = {(r,0)10 5 r 2 1,O 5 6' 2 2m}
so
/-- ----
,/--I---
V ( S )= JJ d9 - ( x 2 + y 2 )d A
3
R

r = l =SJ D
1

O
27
d m r d O d r

.J
Fig. 20.45

PROGRESS TEST 1

1. Determine the area of the region


- R enclosed by one 3. Evaluate f f, elC2+UZ dA, where
leaf of the four-leafed rose r = cos 20.
2. Determine the volume of the solid region bounded
below by the upper nappe of the cone t = d m
and above by the plane t = 1.

CYLINDRICAL COORDINATES
We have seen that certain plane curves are more naturally expressed in terms of
polar coordinates rather than rectangular coordinates. In three dimensions
there are two commonly used generalizations of polar coordinates. The first of
these, called cylindrical coordinates, are particularly useful in physical or geometric
situations involving symmetry about the t axis.

In effect the cylindrical coordinate system is obtained by placing the t axis


P = I. 8.z perpendicularly through the polar plane at the pole. The Cartesian and
cylindrical coordinates are related by the equations

Example 5 ( a ) Determine an equation in cylindrical coordinates for the surface with


Cartesian equation x2 = y 2 2 9 . +
( b ) Determine an equation in Cartesian coordinates for the surface with
cylindrical equation r = 4 cos 8.
SECTION 2 0 . 4 : MULTIPLE INTEGRALS IN OTHER COORDINATE SYSTEMS f17

x!3-
2 2
4

I
/-i-Lr
/--I--\
L-- 3

,J
(a) ':C)

Fig. 20.47

2 +7

f 2,0.01

Fig. 20.48
,J
Fig. 20.49

(u) From Chap. 17 we know that x 2 = y 2 + 2 t 2 has as its graph an elliptic cone
with its axis the x axis. By (20.4.5),x 2 - y 2 = r2 cos2 6' - r 2 sin28 =
r2(cos28 - sin2 6 ' ) = r 2 cos 28. Thus the cylindrical equation for this cone
is r2 cos 26' = 2z2. (See Fig. 20.48.)
( b ) Multiplying r = 4 cos 6' by r, we have r2 = 4r cos,Q.But r2 = x 2 + y 2 and
7 cos 6' = x , so the Cartesian equation is x 2 + y 2 = 4x. Upon completing the
square in x we get ( x - 2)2 + y 2 = 4. Thus the graph is a right circular
cylinder of radius 2 whose axis is the vertical line through (2,0,0). (See Fig.
Fig. 20.50 20.49.) 0

PROGRESS TEST 2

1. Find cylindrical coordinates for the points with 2. Find cylindrical equations for the surfaces with
Cartesian coordinates Cartesian equations
(a) ( - 1, fi,4) d-2
( b ) (1, 1, (u) x2 - y + 2 2 = 1 (b) x2 +y2 = 9

SPHERICAL COORDISATES
The second generalization of polar coordinates to three dimensions is particu-
larly useful when working with problems involving a center of symmetry.

20.4.7 Definition A point P in space is said to have spherical coordinates ( p , & 6 ' ) (see
-+
Fig. 20.50) if ( a ) p = IlOP11, ( b ) 0 is the angle between the vertical coordinate
3
vector k and OP, and (c) 6' is any polar angle for the projection of P in the xy
plane.
718 CHAPTER 20: MULTIPLE INTEGRATION

Note that, by definition, p is always nonnegative and @ is always between 0


and TI.
If Qis the projection of P into the x-y plane, then, referring to Fig. 20.51(a),
we have

+
Figure 20.51(b) depicts the plane containing thez-axis and OP, from which we
observe that

Combining (4) and ( 5 ) we obtain equations for transforming points from


spherical to Cartesian coordinates.

20.4.8 x = p sin $I cos 0,y = p sin + sin 0, z = p cos $I


The equations in the other direction are given by

20.4.9

As with cylindrical coordinates we can gain some insight into this new system
by examining the graphs of the basic equations p = k , I$ = k , and 8 = k .
The graph of p = k >0 is a sphere of radius k centered at the origin, while
p = 0 has as its graph the origin itself. The graph of 8 = k is the same as in
cylindrical coordinates (a vertical plane). The graphs of 0 = 0 and @ = 'ii are
the positive and negative z axes, respectively, and the graph of @ = 7 ~ / is 2 the
+
xy plane. Otherwise, = k has as its graph the upper or lower nappe of a cone
depending on whether 0 k < < 77/2 or 77/2 < <
k 'ii, respectively. These vari-

ous graphs are shown in Fig. 20.52.

Example 6 Find the Cartesian coordinates for the point with spherical coordinates
(6, 77/6,3'ii/4).
SECTION 20.4: MULTIPLE INTEGRALS IN OTHER COORDINATE SYSTEMS 719

5 c

t A

/ ai
5
A

4
$=k,O<k<rr/2

:c ,d)
Fig. 20.52

Solution From (20.4.8),


x = p sin @ cos 6 = 6 s i n ( ~ / 6 ) c o s ( 3 ~ / 4=) - 3/ fi
a
y = p sin @ sin 19 = 6 s i n ( ~ / 6 ) s i n ( 3 ~ / 4=) 3/
z = p cos @ = 6 C O S ( T / ~ ) = 3 fi
so the point has Cartesian coordinates (- 3/ fi,3/ fi,3 fi),0

Example 7 Find spherical coordinates for the point P with Cartesian coordinates
(1, -1, a).
Solution By (20.4.9),

@= arccos( f i / 2 ) = n/4

In the xy plane the projection of P, (1, - l), is in the fourth quadrant, so we can
choose I9 = 57/4. Thus a set of spherical coordinates is (2,77/4,577/4). 0
Although all texts define spherical coordinates in the same way, some books
list the coordinates in the order ( p , 8, +), rather than ( p , +, 19)as we have done.
This will not affect the graph of a spherical equation, but it is important to
know which order is being used when you attempt to plot a point with a given
set of coordinates.
As before we can use (20.4.8) and (20.4.9) to transform equations for a
surface. For example, a sphere of radius 4 centered at (0, 0 , 5 ) has the Cartesian
+
equationx2 + y 2 ( z - 5)2 = 160r,equivalently,x2 + y 2 + z 2 - 102 = -9.
But x 2 + y 2 + z 2 = p 2 and z = p cos @, so the corresponding equation in
spherical coordinates is p 2 = lop cos - 9. +
720 CHAPTER 20: MULTIPLE INTEGRATION

Conversely, given a n equation in spherical coordinates, we can obtain the


corresponding Cartesian equation. For example, the equation
+ +
p sin tan sin 20 = 1 can be written as

( p sin + sin 0)( p sin + cos 0)2 = p cos +


by multiplying both sides by p cos + and using the identities
t a n @=
(sin +)/(cos $) and sin 20 = 2 sin 0 cos 0. Now, by (20.4.9), the corresponding
Cartesian equation is 2 9 = t.

PROGRESS TEST 3

1. Find spherical coordinates for the points with Car- 2. Find spherical equations for the surfaces with Car-
tesian coordinates tesian equations
(a) (-1, V R 4 ) d%
( b ) (1,1, ( u ) x2 -y2 + 2 2 = 1 ( b ) x2 +y2 = 9

THE TRIPLE INTEGRAL IN CYLISDRICAL


AND SPHERICAL COORDINATES
In Cartesian coordinates it is natural to use inner partitions made up of
parallelepipeds of the form
si = {(X,Y,t)lXi-l Ix I Xi,Yi-1 I Y I Y i , Z i - 1 It Iti>

with volume AE = Axi Ayi Azi. The corresponding integral JJJ,J(x,y, z ) dV,
when expressed as an iterated integral, leads to the differential of volume
d V = dxdydz
When working in cylindrical or spherical coordinates, the natural inner
partitions consist, respectively, of cylindrical wedges
Si = { ( r , Q , z ) ~ r ~I I ri,QiPl I 0 5 O i , t i - l I 2 I
- ~r zi)
and spherical wedges
si = {(P,+,0)lP&l IP IP i , Q i - l I + I h 0 i - l I 0 soil

The basic Cartesian, cylindrical, and spherical subregions are sketched in Fig.
20.53.
The corresponding Riemann sums in these latter coordinate systems have the
forms
n n

Parallelepiped C) lindrical wedge Spherical bvedge


:a:

Fig. 20.33
SECTION 20.4: MULTIPLE INTEGRALS IN OTHER COORDINATE SYSTEMS 721

- - -
where 6 , 4)
Oi, and (pi r&, O i ) are arbitrary points in the respective wedges Si,
~

and A 5 represents their respective volumes. If we now refine the partitions in


such a way that IlPIl + 0 (IlPIl = length of largest of the diagonals of the Si's),
and if these sums tend to some number independently of how the points are
chosen in each Si, then they give rise to triple integrals in the respective
coordinate systems. That is, we define:
n
20.4.10 JJJ h(r, 8 , z ) dV = lim IIPII-0
h f i , &, Z i ) A K
i=i

and

Cylindrical wedge of volume


Abi z ri Ari ABi Azi
Fig. 20.54
It can be shown that if h and g are continuous, and if the functions deter-
mining the bounded regions S are continuously differentiable, then these limits
will exist. That is, continuous functions are integrable over S.
To determine how to express these integrals as iterated integrals, we must
first determine A& in each case. Now from Fig. 20.54 we see that a typical
cylindrical wedge has volume approximately equal to the volume of the
parallelepiped with edges of lengths ri AO,, Ari, and hi,Thus in cylindrical
coordinates
AK ri Ari ABi Azi
Similarly, the volume of a spherical wedge is approximated by the volume of a
parallelepiped with dimensions pi sin Oi AQi, pi AGi, and Api, so in spherical
coordinates (see Fig. 20.55)
A & z p i 2 sin oi Ap, A$i AOi
It can be shown that the corresponding iterated integrals will involve
differentials of volume of the form

20.4.12 d V = rdrd8dz (cylindrical)


Fig. 20.55
and

20.4.13 dV = p 2 sin (I dp d$ d6' (spherical)


z
Of course, the order in which the differentials of the basic variables appear
will depend on how the integral is expressed as an iterated integral and how the
solid is described. For example, if S is described in cylindrical coordinates by
s = {(r,8,z)la IS I e , f c e , I r I g c e , , F ( r , 8 ) 1 z I G ( r , 8 ) )
as in Fig. 20.56, then

JJJ h(r,8 , t)dV =

As with Cartesian coordinates, the order in which the differentials appear


d = r allows you to read off which variable is being limited by the respective limits of
Fig. 20.58 integration.
722 CHAPTER 20: MULTIPLE INTEGRATION

describe and sketch S.


Solution From the order in which the differentials appear, we know that the outside
z
limits of integration are limits on r , the middle limits are on 8, and the inside
limits are on z . Thus
s = {(r,O,z)IO<r< 1,0~8~7r/2,O<Z< d2=7}
Now
R = {(r,8)10 r 5 1,0 5 8 5 7/2}
describes a quarter circle of radius 1 centered at the origin. Thus
~ )R
{ ( ~ , ~ , z ) l ( r , in )
Y is one-quarter of a vertical right circular cylinder of infinite extent. Since z 2 0,
S is bounded below by the xy plane. Now z = d m implies r2 + z 2 = 4.
+
But since x 2 y 2 = r2, the graph of r 2 + z 2 = 4 is a sphere of radius 2 centered
X at the origin. Since z 5 d m , S is bounded above by the upper half of this
Fig. 20.67 sphere. A sketch of S is given in Fig. 20.57. 0

Example 9 Find the volume of the region S bounded below by the cone @ = n / 4 and
bounded above by the sphere p = 2.
Solution As before, V ( S ) = {{ di7I s
The region S is sketched in Fig. 20.58. We shall use
the symmetry of its ice cream cone shape and multiply by 4 the volume in the
T first octant. In this octant 0 5 0 5 7/2. Denote this portion of the solid by S
4I.l
Since

114 = {(P, 0, 13) I 5 l3 2 71/2, < 0 < n / 4 , < P 5 2}


we have
Ti2 T/4 2
V ( S ) = 4V(Sli4) =4J J p 2 sin @ dp d@d8
0 0 0

sin 0 d+ dl3
-4 [
= 32 Ti2 -cos+] Ti4 dl3

3 0

,J
Fig. 20.58

Example 1 0 Set up, but do not evaluate, an iterated integral giving the moment of inertia
about the z axis of the homogeneous region bounded below by the upper nappe
of the cone 3z2 = x 2 + y 2 and above by the plane z = 1.
Solution Letting x = 0, we see that the cone meets they-z plane in the intersecting
straight lines z = +(l/fi)y. It follows [see Fig. 20.59(a)] that the cone has
equation -
@ = arctan($) =- 77

3
in spherical coordinates.
SECTION 20.4: MULTIPLE INTEGRALS IN OTHER COORDINATE SYSTEMS 723

(a)

Fig. 20.59

In the equations relating Cartesian and spherical coordinates, z = p cos 0, so


the plane t = 1 has equation p cos @I = 1 or, equivalently, p = sec 0. Thus a
description of S in spherical coordinates is given by
S = { ( p , 0,8) 10 5 3' 5 2 7 , 0 5 0 5 7 / 3 , 0 5 p 5 sec $1
Suppose now that S has been broken up into small subregions consisting of
spherical wedges, and let ( p i , $ i , B i ) denote a point in a typical subregion Si.
The mass of Si is given by
20.4.14 Mi = k A5 ( k is the density)
Since the distance from ( p i ,@Ii, ei) to the z axis is pi sin oi[see Fig. 20.51(b)],
the second moment of this piece about the t axis is
(I,)i = ( p i sin ~ ~ ) A2 k
Thus

-
n
4 kpi2 sin2 @Ii AV,
i=l
and hence
r, =
2- 7i/3 sec4

4 "4 4
i 2 ~i ~ / 3i s e c
k p 2 sin2 0 p 2 sin @I dp d o dQ

0
p4 sin3 0 dp d+ d8

TRA4SSFORMIKGINTEGRALS FRQM CARTESIAS TO


CYLINDRICAL OR SPHERICAL COORDIKATES
In many instances we can simplify the evaluation of a n integral

in Cartesian coordinates by transforming it to an equivalent integral in another


coordinate system.
From our equations relating the coordinate systems, we have

20.4.15
724 CHAPTER 20: MULTIPLE INTEGRATION

2
where represents the same region S described in cylindrical coordinates. In any
iteration of this latter integral, d P will be of the form dV = 7 dr $0 dz with the
order of the differentials depending on the description of Further, 5 should
2.
be described in such a way that r 2 0 and 0 varies over a n interval of length
2 277.
The corresponding transformation to spherical coordinates is given by

20.4.16

where in this case Srepresents the region S described in spherical coordinates and dV
is some permutation of p 2 sin 0 dp d o do.

Example 1 1 Express the integral

/
as an equivalent iterated integral in cylindrical coordinates.

I = SsL d 4 - x2 - y 2 d V

where
S={(x,y,z)I-l < x < l , - d c 7 < y < d m , X 2 + y 2 < Z < l }
+
Thus S is the solid region above the paraboloid2 = x 2 y 2 and below the plane
~~
z = 1 (see Fig. 20.60). In cylindrical coordinates, S can be described as
s = { ( 7 , 6 , z ) 10 0I2n,O Ir I 1 , r2 <z 1}
and since d4 - x 2 - y 2 = d n ,we have
J
X

Fig. 20.60

Example 12 Express the integral

I = Jo Jo (x2 +y2 +z ~ ) dz~4/ dx~

as an equivalent iterated integral in spherical coordinates.

Solution I = JJl + (x2 y2+ dV

where S is the quarter-hemisphere (see Fig. 20.61)


s = { ( x , y , z ) 10 Ix 5 l , o < y 5 q n , o I2 < d l - x2 - y 2 }
In spherical coordinates, S is given by

s = {(P, +,0 ) l O I 0 Ia/2,0 I 0 I77/2,0 I P I1 )


and since
( ~ 2+ J? + ~ 2 ) 3 / 2= ( p 2 ) 3 / 2 = p3
we have

I = i
r/2
14
T/2 1
p3*p2sin~dpd+dB
7

Fig. 20.61 =L L v/2 7/2 1


&p'sin+dpd$dO
SECTION 20.4: MULTIPLE INTEGRALS IN OTHER COORDINATE SYSTEMS 725

PROGRESS TEST 4

1. Use cylindrical coordinates to find the mass of the x2 + +


+ y 2 z 2 = u2 and x 2 + y 2 z 2 = b2 ( a 6 ) if >
homogeneous solid bounded on the sides by the density at any point is inversely proportional to
x 2 + y 2 = 1, on the bottom by the x y plane, and on its distance from the origin.
+
the top by x 2 + y 2 z 2 = 2 .
2. Use spherical coordinates to find the mass of the
solid in the first octant between the spheres

SECTION 20.4 EXERCISES

In Exercises 1 to 5, ( a ) transform the given iterated integral into an equivalent iterated integral in polar coordinates, and ( b ) evaluate the resulting polar
in tegra 1.

3. s 4 s d 3 y 2 dy dx
0 0

In Exercise 6 to 1 0 use a double integral (or integrals) in polar coordinates to determine the indicated area.
6. T h e area of the region between the concentric circles 8. T h e total area enclosed by the lemniscate r2 = 2 cos 28.
x 2 + y 2 = 9 and x 2 + y 2 = 1. 9. The area of one leaf of the three-leafed rose r = 4 sin 38.
7 . The area of the region inside the circle r = 3 cos 6' and 10. The area enclosed by the smaller loop of the limacon
outside the cardioid r = 1 + cos 6'. r = 1 - 2 sin 8.

In Exercises 1 1 to 1 5 determine the volume of the indicated solid region.


11. The solid inside the sphere x 2 + y 2 + t 2= 4 that is also 14. The solid bounded above by the cone z = d m , on
+
inside the cylinder x 2 y 2 = 1 . +
the sides by the cylinder x 2 y 2 - 3y = 0, and below
12. T h e solid bounded above by the cone z = r, below by by the x-y plane.
the x y plane, and on the sides by the cylinder 15. +
T h e solid inside the ellipsoid 4 x 2 4y2 t 2= 4. +
r = 4 cos 8.
13. The solid bounded by the paraboloid z = 4 - x 2 - y 2 ,
the xy plane, and the cylinder x 2 + y 2 = 1.

In Exercises 1 6 to 2 0 j n d the indicated moment f o r the lamina occupying the region R and having the given density.
16. R is the region inside the cardioid r = 1 +
cos 8 ; the 20. R is the region bounded by the graph of r = sin 8 ; the
density is proportional to the distance from the pole; density is proportional to the distance from t h e y axis;
find M,. find M,.
+
17. R is the region inside the circle x 2 y 2 = 4 and outside 21. A lamina that occupies the region inside the cardioid
the circle x 2 + y 2 = 1; the density is proportional to r =2 +2 cos 8 and outside the circle r = 2 has density
the distance from the x axis; find Mu. inversely proportional to the square of the distance from
18. R is the region inside the circle x 2 - 2x + y 2 = 0 ; the the pole. Find the force of attraction exerted by this
lamina is homogeneous; find I,. lamina on a particle of mass p located at the origin.
+
19. R is the region inside the circle x 2 y 2 = 9; the lam-
ina is homogeneous; find IL, where L is the line? = 3.

In Exercises 22 to 26 evaluate the given integral by Jirst transforming into an equivalent integral in cylindrical or spherical coordinates (whichever is
more convenient ).

m h
dz dy dx
726 CHAPTER 20: MULTIPLE INTEGRATION

27. Find the first moment about the x y plane of the solid 32. Find the mass of a right circular cone of height h and
bounded above by the plane t = 4 and below by the radius at the top a, if the density is proportional to the
+
paraboloid z = x 2 y 2 where the density at any point is distance from the axis of the cone.
proportional to the distance from the x y plane. 33. Find the force of attraction of the region in the first
28. Find the moment of inertia about the z axis of the solid octant between the concentric spheres x 2 + y 2 +
in Exercise 2 7 . t 2= a 2 and x 2 + y 2 = b 2 ( a >
b ) on a particle of mass rn
29. Find the moment of inertia about the t axis of the region located at the origin if the density of this region is
between the concentric spheres x 2 + y 2 t 2= a 2 and + inversely proportional to the distance from the origin.
>
x 2 + y 2 + t 2= b2 [ a b ) if the density at a point is 34. Find the moment of inertia about the x axis of the
proportional to its distance from the origin. region inside the sphere x 2 + j 2 t 2= 18 and the +
30. Find the mass of the homogeneous solid bounded below cylinder x 2 + y 2 = 9 if the density is proportional to the
by the cone $I = 7i/6 and above by the sphere p = 4. distance from the z axis.
3 1. Find the mass of a spherical solid of radius a if the 35. Find the mass of the homogeneous solid region bounded
density is proportional to the distance from the center. by the cylinder x 2 + y 2 = ax and the cone t 2= x 2 + y'.

CHAPTER EXERCISES

R e v i e w Exercises
In Exercises 1 to I0 each o f t h e iterated integrals represents an area or a volume of some region. ( a ) Sketch the region ( i n the appropriate coordinate
system). ( b ) Compute the iterated integral.

- y 2 ) aj dx

In Exercises 11 to I5 evaluate JJR f (x, y ) dA, where

In Exercises 16 to 20, evaluate the indicated iterated integral.

16. 4,mi XZ
xsinyajdx 18. 4i~ / 4t a n x
3y2secxdydx

In Exercises 21 to 25, ( a ) sketch the region over which the integral is being evaluated, ( b ) write an equivalent integral w i t h the order of integration
reversed, and ( c j evaluate this latter integral.
SECTION 20.4: MULTIPLE INTEGRALS IN OTHER COORDINATE SYSTEMS 727

In Exercises 26 to 30 determine the area Ofthe given region R.


26. R is bounded by y = 2x2 - 2x - 1, y = x2 - 2x + 3. 27. R is bounded by y = In x , y = ex, y = 1, y = 2.
28. R is bounded by y 2 = 4x, 2x2 = y . 29. R is bounded by y = x , y = 3x, and x = 2.
30. R is in the first quadrant and bounded by y = 0, y = x ,
and x 2 + y 2 = 4.

I n Exercises 3 1 to 35 use the double integral to determine the volume of the given region S.
31. S is bounded by the coordinate planes and x +y + 34. S is bounded by the cylinder x 2 + z 2 = 4 and by the
z = 1. planes y = 0 and x + y z = 3.+
32. S is bounded below by the x y plane and above by the 35. S is bounded above by z = 18 - x 2 - y 2 and below by
paraboloid z = 4 - x 2 - y 2 . z = x2 + y ?
33. S is bounded below by the x y plane, on the sides by the
cylinder x 2 + y 2 = 1, and above by the plane z =
3 - x.

In Exercises 36 to 4 0 determine the area of the given polar region R.


36. R is enclosed by r = 4 - cos 0. 43. Find the moment of inertia about the origin of the
37. R is enclosed by r = 2 sin 30. lamina bounded by y 2 = x and y = x if the density is
38. R is the region inside both the curves r = 2 cos 0 and given by p ( x , y ) = xy.
r = 1. 44. Find the moment of inertia about the origin of the
39. R is the region inside r = 2 cos 0 and outside r = 1. homogeneous lamina bounded by one loop of
40. R is enclosed by r2 = sin 20. r2 = cos 20.
41. Find the center of mass of the lamina bounded by 45. A lamina occupies the annular region in the first quad-
y = x2 +
2x a n d y = x +
2 if the density is proportional rant between r = 1 and r = 4. Given that the density is
to the distance from the x axis. inversely proportional to the distance from the origin,
42. Find the moment of inertia about t h e y axis of the determine the force of attraction exerted by this lamina
homogeneous lamina bounded by r = 1 + cos 0. on a particle of mass p located at the origin.

In Exercises 4 6 to 4 9 determine and evaluate an equivalent iterated integral in polar coordinates.

47.

48. s4sd4'-'*
0 0
( x 2 + y 2 )dx dy 49.
J0 J0 +
In Exercises 50 to 5 3 j n d the volume of the indicated region S ly using an appropriate double integral in polar coordinates.
50. S is the region inside both of the surfaces x 2 + y 2 + 52. S is bounded above by z = r and below by z = r 2 / 2 .
z 2 = 16 and x 2 + y 2 = 4. 53. S is inside the cylinder x 2 + y 2 - 4y = 0 and bounded
5 1. S is bounded below by the upper nappe of the cone above and below by the cone z 2 = x 2 y 2 . +
b2(x2 + y 2 ) = a2z2 and above by the plane z = b
(a, b >
0).

In Exercises 5 4 to 6 3 evaluate the given integral.


lv/21cos 0
1
: cos 8
54. r dz dr d0

56.

58.

60.

62.
728 CHAPTER 20: MULTIPLE INTEGRATION

In Exercises 64 to 68 determzne the volume of the solzd S


64. S is bounded by t = x2, x 2 + y 2 = 4. and t = 0. +
65. S is bounded by x 2 y 2 = 4 z and z = y .
66. S is bounded above by the paraboloid z = r 2 , on the 67. S is bounded by the cylinder x 2 + y 2 = 2x and the
bottom by the x-y plane, and on the sides by the cylinder sphere x 2 + y z + t 2 = 4.
r = cos 8. 69. Give all six iterations for the integral JJJs dV, where S is
68. S is bounded below by 0 = si/4 and above by the region in the first octant bounded below by z = 4
p = 2 cos 0. and above by 2x +
3y +
42 = 24.
70. Give the remaining 5 iterations for the integral 7 1. Find the moment of inertia about the z axis for the solid

s4ssx
0 0 0
dy dx dz
region between the spheres x 2 + y 2 t 2 = 1 and
+ +
+
x 2 y 2 t 2= 4 if the density is inversely proportional
to the distance from the origin.
72. Find the center of mass of the homogeneous solid
73. Find the moment of inertia about the x axis of the solid
bounded below by the x-y plane and above by
region bounded below by z = 2x, above by t = 4x, and
z = 1 - r2.
on the sides byy = 0 a n d y = 1 if the density is given by
P ( W ) = xy.

Miscellaneous Exercises
1. Express 8. Prove the parallel-axis theorem: The moment of inertia of
a lamina about any axis L in its plane equals Z, Mr2, +
L1im dxdy +
1
mZ/4

f
1;/4

i
dxdy where M is the mass of the lamina, r is the distance from
L to the center of mass C of the plate, and Z, is the
as a single iterated integral. moment of inertia of the lamina about a n axis parallel
2. Express to L through C.
G,
s4sv + sGs4 + s1s4
2 2
dx dy
4 2
dx dy
6 yi3
dx dy
9.

10.
Evaluate JJE dA/ where R is the triangle
bounded b y y = 0 , y = x, and x = 1. (Be careful.)
Show that the volume of the elliptic cylinder x 2 / a 2 +
as a single iterated integral.
>
y 2 / b 2= 1 ( a , b 0 ) between the x-y plane and the plane
3. Express the integral
t = cx + + dy h does not depend on the constants c and
d. (Hint: Use a polarlike substitution for x a n d y . )
121\/zl(12-2Z-2U)/3 dt dy dx 11. Use cylindrical coordinates and the fact that V ( S ) =
JJJs dV to develop the (shell method) formula
V =1 : 277xf(x) dx for the volume of the solid of revolu-
as a sum of iterated integrals with dV = dx d t dy. tion generated by revolving about t h e y axis the plane
4. Express the integral region between the graph o f y = f ( x ) and the interval
v/3 [a, 61, for f(x) 2 0 on [a, b ] . (You may wish to switch
$/, 4 r d r d 8
1

axes.)
12. Find the area outside the parabola r ( 1 + cos 8 ) =
as an equivalent iterated integral (or integrals) in rec- 2a and inside t h e cardioid r = 2 4 1 + cos 8 ) .
tangular coordinates. 13. Determine JJR + 6)
(6 dx dy, where R is bounded
5. Evaluate 6+ 6
by x = 0 , y = 0, and = 1. (Hint: Use a para-
- 7

6+
metric representation of $ = 1 to obtain a polar-
ln(x2 + y 2 )dx dy like substitution for x and y.)
14. Suppose C is a solid right circular cylinder of radius a,
6. The solid region in the first octant between the spheres length h , and uniform density p. Determine the gravita-
x 2 + y 2 + z 2 = 1 and x 2 + y 2 + z 2 = 4 has a density tional attraction on a particle of unit mass on the axis of
that is inversely proportional to the distance from the C at a distance b from one end of C. (Hint: First approxi-
origin. Set up an iterated integral that gives the first mate the attraction of a cross-sectional slice of thickness
moment of the region about the plane x + J z = 16. + A x t . To do this, use a polar representation of a n element
7 . The density of a bounded solid that does not contain the of the disk.)
origin is given by 15. Use the idea of Exercise 14 to show that the attraction of
f(x,y,Z ) = ( x 2 + y 2 + t 2 ) 3 / 2 a uniform-density solid sphere on a particle of unit mass
a t distance b from its center ( b> radius) is the same as if
Show that the force of attraction exerted by this solid on the total mass were concentrated at the center.
a particle of unit mass located a t the origin is directed
toward the centroid of the region occupied by the solid.
SECTION 20.4: MULTIPLE INTEGRALS IN OTHER COORDINATE SYSTEMS 729

SELF-TEST
Chapter 2 1
THE LIN

CONTENTS
21.1 Vector Fields 732 ps. 1 to 16 we were
transform real numbers
21.2 The Line Integral 737
roduced functions that
21.3 Independence of the Path
21.4 Greens Theorem 751
ations of Greens from R to R, R to R,,
and Curl 757

ctor functions are


d they arise quite

urface of a very useful

x analysis presents
ir natural setting. More

e key concept in this


ector field over a

line integral of a
over a curve to a double integral of the

this result in the context of problems

73 1
732 CHAPTER 21: THE LINE INTEGRAL

Section 8 1.1 : Vector Fields

Objectives: INTRODUCTION
1. Determine when a vector field is
conservative (the gradient of some Suppose a fluid is flowing in a stream whose surface occupies the region D in R,.
function). If the velocity of a particle in the stream depends only on its location and not on
time, then the flow is called steady state. If, in addition, the flow is horizontal and
2. Find the potential functions for a independent of the depth, then we can regard it as a t w o - d i m e n s i o n a l ~ u ~ d We
~o~.
conservative vector field. can now define a function, called a velocityjeld, that assigns to each point ( x , y )
in D the vector that gives the velocity v(x,y) of the fluid at that point. The
formal graph of such a function would be a subset of R, and hence could not
be sketched. However, we can choose selected points ( x , y )in D and sketch the
representative of v(x,y) whose base is at (x,y). A typical such velocity field is
sketched in Fig. 21.1.

VECTOR FIELDS
Functions of the above type arise quite frequently in applications ranging from
fluid flow to gravitational or electromagnetic fields, whenever it is necessary to
\elmit) field of a stream associate a vector quantity to each point in a given region. Such functions are
Fig. 21.1 referred to as vectorjelds. If the vectors under consideration represent velocities
(or forces), then the vector field is called a velocity field (or force field).
Specifically, we have:

21.1.1 Definition Suppose D is a subset of R,. A function F that assisns to each


point (xl,x 2 , . . . , x,) in D a unique vector F(xl, x,, . . . , x,) in R, is called a
vectorjeld. Such a vector field is often denoted by writing F: D -+ R,.

T h e above steady-state fluid flow gave rise to a vector field from a subset of
R, into R,. If the flow were not steady state, then the velocity of a particle in
the surface of the stream would depend on both the location and time. If we let

then v(x,y, t ) would denote the velocity of the particle located at (x,y)at time t.
Such a time-dependent velocityjeld is an example of a vector field from a subset of
R, into R,.
In this text we shall deal, for the most part, with two-dimensional vector
fields, that is, vector fields from a subset of R, into R,. Many of our conclusions,
with appropriate modifications, can be extended to three-dimensional vector
fields. Several of these extensions will be noted in the exercises.
If F: D -+ R, is a two-dimensional vector field, then F can be expressed in
component form as

or, equivalently,

where M and N are scalar functions of two variables defined on D.


Since the points ( x , y ) in D can be identified with the position vectors
X = ( x , y ) ,we can also consider F as a vector function of a vector variable. In
this case we write F in the form

As usual, we shall use whichever notation best fits the particular situation.
SECTION 21.1: VECTOR FIELDS 733

Example 1 Suppose a particle of unit mass is located at the origin. Determine and sketch
the force field that gives at each point X = ( x , y )the force of attraction exerted
by this particle on a typical particle of unit mass located at ( x , y ) .
Solution In Chap. 20 we noted that Newtons inverse-square law states that

where F is the force of attraction that a particle of mass ml located a t Q will


exert on a particle of mass m, l o c a t 3 a t P. In our setting, m1 = m, = 1,
Q = ( O , O ) , and P = X = (x,y). Thus PQ = (0 - x 7 0 - y ) = -X and
-+
llPQll3 = 11-
x 113 = lIXl(3= ( 2+y2)32

Y
Thus the force field is given by

t
5 ( + or, in component form,

The negative sign means that the forces are directed towards the origin. Also,
since

the magnitude of the force is small for points far from the origin (IIXll large)
and is large for points near the origin (IlXil small). Equation (1) explains why
Gravitational force field we use the terminology inverse-square law. A sketch of this gravitational force
Fig. 2 1.2 field is given in Fig. 21.2. 0

GRADIEST FIELDS-CO;YSER\TIOS OF ESERGY


An important class of vector fields arises from considering the gradient of a
scalar function of several variables. Specifically, i f f is a function of two
variables and if D denotes the set of points in R, at which the first partials of
f exist, then the gradzent j e l d , grad f. denoted by Vf: D -+ R,, is given in
component form by
21.1.2 V f ( % Y ) = (f,(x7Y)>f,(x,Y))

Definition Suppose F: D -+ R, is a two-dimensional vector field. If there


exists a scalar functionf, differentiable on D,such that

V f ( W J )= F ( w )
for each ( x , y ) in D,then F is called a conservative vectorjeld and f is called a
potential function for F on D .

The genesis of the terms conservative and potential in (21.1.2) is in


physics-in the law of conservation of energy. Specifically, suppose a particle of
mass m is moving along the plane curve X ( t ) subject to the conservative force
F = Of: As usual, V ( t ) = X(t) and A(t) = X(t) denote the velocity and
acceleration of the particle, respectively. By Newtons second law of motion,
F ( X ( t ) ) = mA(t)
734 CHAPTER 21: THE LINE INTEGRAL

Thus we can rewrite equation (3) as

for some constant C.


The quantity (1/2)mlIV(t))l12 is called the kinetic energy of the particle and
- f ( X ( t ) ) is called its potential energy. The sum of its kinetic and potential
energies is called its total energy. Thus equation (4) is a version of the law of
conservation of energy, which states that if the force field is conservative, then
the total energy must be constant.
Because of the above, iff is a potential function for the conservative force
field F, then -f is often referred to as a potential-energy function for F.
The problem of determining necessary and sufficient conditions for a vector
field to be conservative is a significant one that will require some preliminary
groundwork. However, in most specific instances we simply assume F is con-
servative and then systematically proceed as below either to determine the
potential functions or to derive a contradiction.

Example 2 Show that the vector field


F ( ~ , =~ (2xy3
) - 4x, 3+2 + 2r>
is conservative [for all ( x , y ) ] .
Solution To show that F is conservative we must exhibit a scalar function f(x,y) whose
first partials exist for all ( x , y ) such that
(51 f,(x,y) = 2xy3 - 4x and f,(x,y) = 3x2y2 + 2y
Sincef, is obtained from f by holdingy constant and differentiating with
respect to x, we can attempt to recapture f by integrating f, with respect to
x-again holdingy constant. In this case the constant of integration will be a
function ofy. (Think about this!) Thus

f ( X , Y ) = Jf,(.3,!
dx + C(Y)
(6) = s ( 2 x y 3 - 4x) dx + C ( y ) = x 5 1 ~- 2 x 2 + C ( y )
All that remains is to determine, if we can, C ( y ) . Now by differentiating the
above with respect t o y , we know that
SECTION 21.1: VECTOR FIELDS 735

f,(x,y) = - a[ x 5 3 - 2x2 + C ( y ) ]= 3 x 5 2 + C(y)


aY
O u r original assumption ( 5 ) was that&(x,y) = 3 x Y 2 + 2y, so, setting these
two expressions for& equal, we obtain
3x52 + C(y) = 3 x 5 2 + 2y
+
Hence C(y) = 2y, and so C ( y ) = y 2 k. Substituting this latter expression in
(6), we conclude that any function of the form
f ( x , y ) = ~5~- 2x2 + y 2 + k ( k a constant)
is a potential function for F, and so F is conservative. Of course, we can easily
check our computations by taking the gradient off to see if it equals the original
vector field F. (Check it,) 0

Thus, in general, to determine the potential functions of a conservative


vector field

F(x,y) = ( M ( x , y ) ,N ( x , y ) )
we proceed as follows:
(4 S e t h ( x , y ) = M ( x , y ) .
( b ) Integrate with respect to x , getting

(c) Differentiate with respect t o y , getting

( d ) Set this latter expression equal to N ( x , y ) and solve for C(y).


( e ) Integrate to determine C ( y ) and hence obtain, from (a), the potential
functions f.
Dually, we could just as well start by setting&(x,y) = N ( x , y ) ,then integrate
with respect toy to obtainf(x,y) = J N ( x , y )dy + K(x),and so on. If in step ( d )
we do not obtain C ( y ) as a function ofy alone [or K(x)as a function of x alone
with the dual procedure], then we can conclude that the vector field is not
conservative.

Example 3 Show that the vector field F(x,y) = ( y , - 2 ) is not conservative.


Solution Assume F is conservative and letfbe any potential function. Thenf,(x,y) = y
andf,(x,y) = - x . Integrating the first of these equations with respect to x and
the latter with respect t o y , we have
f ( x , y ) = xi + C ( Y ) = -XY + K ( x )
Thus
K ( x ) = 2xy + C ( y )
Now
a
K(2) = -[2g
ax + C ( y ) ]= 2y + 0 = 2y
and so
0 = - a K ( x ) ] a
= -[2y] =2
aY aY
Thus the assumption that F is conservative leads to the contradiction
0=2. 0
736 CHAPTER 21: THE LINE INTEGRAL

The following criterion is useful in determining when a vector field is not


conservative. It is an immediate consequence of Theorem (19.3.6) on the
equality of mixed partials.

2 1.1.3 Theorem Suppose D is an open subset of R, and F: D + R, is the vector


field given by

F(x7y) = ( M ( x , y ) ,N ( x 2 ) )
where A4 and N have continuous first partials on D.If F is conservative, then

for each ( x , y ) in D .
Prooj Since F is conservative, there exists a scalar function f such that F = Of
on D.But F = ( M , N ) and Of = (f,,f,), soM =f,and N =&. Now M a n d N
are continuous with continuous first partials, and hencef has continuous first
and second partials. By Theorem (19.3.6), page 640, f,, = f,, on D. The
conclusion follows since M , = (A), =A, and N, = (f,),=f,,. W

Since under the above hypotheses, conservative implies that M , = N,, it is


logically equivalent that M , # N, implies that F is not conservative.
It turns out that with a suitable adjustment on the hypotheses, the condition
M , = N, is both a necessary a n d sufficient condition for F = ( M , N ) to be
conservative. We will discuss this further following Green's Theorem in Section
21.4. Nonetheless, one should certainly check to see whether M , = N, when
determining if F is conservative. If not, there is no need to proceed any further;
the field is not conservative. If M , = N , , then even though there is nothing to
guarantee that F is conservative, you can utilize the procedure outlined earlier
to attempt to find a potential function.

PROGRESS TEST 1

Identify those vector fields that are conservative and 3. F(x,y) = ( 2 x ln(2 + y 2 ) - 6 x 2 , 2 x 3 / ( 2 + y 2 ) + 2y),
determine all potential functions. D = R,.
1. F ( x , y ) = (23- 2x,4xy2 +y), D = R,.
2. The inverse-square field of Example 1. [Dconsists of
all (X,Y) # (O,O).I

SECTION 2 1 . 1 EXERCISES

In Exercises I to 6 sketch the given vectorjield.

1. F(x,y) = (0,1)
3. F(x,y) = (Y/3, - x / 3 )
5. F(x,y = (x,x)

In Exercises 7 to 20 identtj, those vectorjelds that are conservative and determine the potential functions. Unless otherwise stated, assume D = R,.
7 . F(x,y) = ( 4 -
~2 ~ ,-5 2~ ~ )
9. F(x,y) = (x3- 3x, 4xy2 + 9y)

11. F(x,y) = ( y 2 e " - j 2 ; 2ye" - 2xy + 3y2)


+ s2sin
< -

13. F(x,y) = (xe"), +x) 14. F(r, s) = (2s sin r cos 7 I, sin2 r - 2s cos r )
SECTION 21.2: THE LINE INTEGRAL 737

15. F(u, u ) = ( 2 u sin u + eu cos v, u2 cos v - e" sin v )


17. F(s. t ) = ( e s ( s 2 t' + +
2s), 2tes)
19. F ( x , y ) = (2xe"' cosy + 2x: -8' siny)

20. +
F(x,y) = ( x 3 sinh(xy) 2 x cosh(xy) - 4x3,
x 3 sinh(xy) + 2y)
21. A three-dzmenszonal tectorjield F : D + R , is called con-
servatzve if F = Cf on D for some differentiable scalar
function f ( x , y ,2). Suppose D is a n open subset of R, and
F ( x . y , z ) = ( M ( x , y ,z ) , N ( x , y ,z ) , P(x.y, 2)) is conserva-
tive. Show that M,, = N,, N, = P,,, and P, = M,.

In Exercises 22 to 25 use the following procedure to determine the potential functions f o r the gir;en three-dimensional conservatirre vector fields.
( a ) Set f , = M . 2 2 . F ( x , y , z ) = (6x2y2 - 3 y z 2 , 4 x 3 - 3xz2, - 6 x y r )
( b ) Integrate with respect to x , getting f ( x , y , z ) = J M d x + 2 3 . F(x,y,z ) = ( y sin z , x sin z , xy cos z )
C ( Y >2). 24. F ( x , y , z )= (y'e' + 3z2, 2xye' + +
2y, xy2eZ 6xy)
( c ) Differentiate the result in ( b ) with respect toy and set equal +
23. F(x,y, z ) = (siny2 2x2 cos x2, 2x4' cosy', sin x')
to N , obtaining a n expression for C,(y,z).
( d ) Integrate C, with respect to y to obtain C =
s C,(Y>2) 4 + K k ) .
( e ) Substitute ( d ) into ( b ) , differentiate with respect to I, and
equate with P.

Section 2 1.2: The Line Integral

Objectives: CURVES
1. Evaluate the line integral of a If X ( t ) ,a 5 t 5 6, is a simple arc, then we can order the points on the graph by
vector field over a curve. insisting that X(t,) precede A'(t,) if and only if t , <
t,, where t , and t, are
allowed to vary over [a, b ) if the arc is closed, and over [a, b] otherwise. When the
2. Determine the work done by a
graph of a simple arc is given this ordering, we say it has the orzentatzon of
force field in moving an object
zncreaszng parameter. If X(t) is contznuous, then the graph together with this
along a curve.
ordering is called a curbe and X ( t ) is called a parameterztatzon of the curve. We
usually denote a curve by C. If we wish to emphasize the particular parameter-
ization we write:
C: X ( t ) ,a 5t 5b
It is important to note that two quite distinct vector functions can determine
the same curve C. For example, the simple continuous arcs
X(t) = (t,t2), 0 5 t 51
and
Y ( t ) = (sin(t - a/2), sin2(t - 'ii/2)), m/2 5 t 5 'ii
3
both have as their graphs the piece of the parabolay = 9, 0 5 x 5 1. Fur-
f thermore, in both cases the orientation of increasing parameter provides the
same ordering to this graph. That is, the same points precede one another in
each ordering. The curve having these parameterizations is sketched in Fig.
21.3, where as in Chap. 18 we have indicated the ordering by placing arrows
along the graph.
It can be shown that if a curve C has a smooth parameterization, then any
parameterization of C will be smooth. Hence we shall refer to a curve with a
smooth parameterization as a smooth curve. A curve Cis said to be piecewise smooth
if it can be formed by piecing together, end to end, a finite collection of smooth
Fig. 2 1.3 curves C,, C,, . . . , C., In this case we write the following equation:
738 CHAPTER 21: THE LINE INTEGRAL

(C)

I
iU)

Fig. 2 1.5

n
c=uci
i=l

Some typical piecewise smooth curves are sketched in Fig. 21.4.


Finally, if C is a curve, then by - C we shall mean the curve that has the
same graph as C but the reuerse orientation (see Fig. 21.5). It is worth noting that if
C has parameterization
C: X(t),a 5 t 5 b
then a particular parameterization for - C is given by

21.2.1 -C: X(a + b - s), a 5 s 5 b


(Note that as s varies from a to b, a + b - s varies from b to a.)
WORK
In Chap. 17 we noted that if a constant force F is applied to move an object
along a straight line from P to Q, then the work done by this force is given by

(1) w = (proj~F)IIZl/
We now wish to consider a much more general problem. Suppose that
F: D + R, is a force field defined on some open subset D of R,, and C is a
SECTION 21.2: THE LINE INTEGRAL 739

smooth curve contained in D joining the point P to the point Q (see Fig. 21.6).
How much work is done by this variable f o m in moving an object from P to Q along C?
To answer this question we shall apply what by now should be a familiar
process: We subdivide the curve C into small pieces, approximate the work done
on each of these segments of C, add the result, and then take the appropriate
limit to obtain a definite integral.
Now suppose C is given parametrically by the simple smooth arc

X(t) = (x(t),At)> a 5t 5b
Since C is smooth, V ( t )= ( x ( t ) , y ( t ) ) is continuous on [a, b ] . Let P be a
partition of [a, b ] into n subintervals

[to, 41,. . , [ t i - l , 41,


where to = a, tn = b , Ati = ti - ti-l, and IIPlI = max .Iti. Let t ; ~ [ t ~ti] - ~for
,
Fig. 2 1.8 each i, and assume that F takes on the constant value Fi = F(X(t;)) on the
segment of Cjoining X ( t i - l ) to X ( t i ) .An approximation to the work done by F
along this segment is obtained by taking the product of the scalar projection of
Fi in the direction of the motion with the displacement. T h a t is,
Wiz (Proj,! Fi) Asi
where Ti = T(t;)is a unit vector in the direction of the motion and Asi is the
length of the segment (see Fig. 21.7). Now recall from (18.3.4) that if s repre-
sents arc length along C, then ds = llV(t)l1dt, SO Asi z llVill &, where Vi =
X(t; V ( t t ) .Thus
Wiz (Proj, F i ) Asi
(q
IITi I1
IIV, 11 Ati

= (Fi *Ti)lJVi11 Ati (since lITi 1 1 = 1)


Xt,_,
= ( F i * ~ ) ~ l Ati
V i=l ~( F i * V i ) A t i
Fig. 2 1.7 IIVi I1
Hence
n n
Wz C (Fi*Vi)Ati = C F(X(t,T))*V(t,T)Iti
i=l i=l

If F has continuous components, then this latter sum is a Riemann sum on [a, b ]
for the continuous scalar function F ( X ( t ) ) * V ( t )Thus
. as n + x and +
IIPII --$ 0 , by (6.3.21,
n
2 F ( X ( t ; ) ) * V ( t f ) A t+
i
i=l

21.2.2 Definition Suppose F is a force field with continuous components on the


open set D and suppose C is a smooth curve in D given parametrically by the
simple smooth arc X(t), a t 5 b. The work done by F in moving an object
from X(a)to X ( b ) along C is defined by
b
W= F(X(t))*V(t)dt
a

It can be shown (see Exercise 16) that the value of the integral in ( 2 1.2.2)
does not depend on the particular parameterization of the curve C.
740 CHAPTER 21: THE LINE INTEGRAL

Example 1 Find the work done by the force F(x,y) = ( x 2 +


2y,x + y 2 ) as the point of
application moves from the point (0,O)to (1, 1) along the parabolay = x 2 .
Solution Let C denote the directed curve from (0,O)to (1, 1) along the parabolay = x 2 .
A parametric representation of C is given by X(t) = ( x ( t ) , y ( t ) ) = ( t , t 2 ) ,
0 2 t 5 1. NOW F(X(t)) = F(x(t),y(t)) = ( t 2 + 2t2, t + t 4 ) = (3t2, t 4 + t )
and V(t) = ( ~ ( t )y(t))
, = (1, 2 t ) . Thus

W= s 0
F(X(t))*V(t)dt

=11(3t2,t4+i)*(l,2t)dt

= l 1
[3t2(1) + ( t 4 + t)(2t)] dt
= l 1
(5t2 + 2 t 5 ) dt = 3

(If the magnitude of F is measured in newtons and distances are measured in


meters, then W is measured in joules.)

Example 2 Show that the work done by a variable force as the point of application moves
from P to Q along the smooth curve C is equal to the change in the kinetic
energy of the particle at the end points of C.
Solution Let X(t), a 5 t 2 b, be a parameterization of C and let m be the mass of the
particle on which the force F is being exerted. By Newtons second law of
motion, F = mA. As noted in Sec. 21.1,

and hence
b
W= F(X(t))*V(t)dt
a

= ( mA(t)*V(t)dt

1
= -mlIV(b)l12 - ~1 ~ / ~ V =( Kb
C Z- K,
) ~ ~ ~
2
where Kb is the kinetic energy of the particle at Q a n d K, is the kinetic energy
at P.

PROGRESS TEST 1

1. Suppose the force F(x,y) = ( 3x2 + lOy, 3y2 - 5 x 2 )


moves a particle from ( - 1, - 2 ) to (1,2) along the
cubic curve C given parametrically by
X(t) = ( t , 2t3), - 1 5 t 5 1. Determine the work
done.
SECTION 21.2: THE LINE INTEGRAL 74 1

THE LINE INTEGRAL


In Chap. 6 our attempts at finding the area of a plane region led us to the
definition of the definite integral, which in turn proved to be a very fruitful
concept in many contexts quite distinct from the original area problem. In
much the same way, our above development of a n integral definition for work
leads us to the fundamental and fruitful notion of a line integral. We merely
replace the force field by a general vector field.
Suppose F: D --f R, is the vector field

where D is a n open subset of R,. Let C be a smooth curve in D with the


parameterization C: X ( t ) ,a 5 t 5 b, and let P, 11 PI1 and t r be as in the devel-
opment of work on page 739.

21.2.3 Definition The line integral of F over C, denoted by

is defined by
n
F ( X )* d X = lim 2 F(X(t:))*X(t:)Ati
i=l

provided this limit exists independently of how the t:s are chosen in [ti-,, ti].

As noted in our discussion of work, if F has continuous components on D,


then the sums in (21.2.3) are Riemann sums for continuous scalar functions
on [a, b ] . This observation, together with (6.3.2), enables us to evaluate
sc F ( X )* d X .

2 1.2.4 Theorem Suppose F(x,y) = ( M ( x , y ) ,N ( x , y ) ) ,where M and N are contin-


uous on the open subset D of R,. Let C be a smooth curve in D with the
parameterization X ( t ) , a 5 t 5 b. Then l c F ( X ) * d X exists and
b

L F ( X )* d X =
l F ( X ( t ) )* X ( t )dt

Furthermore, the value of this integral is independent of the particular para-


meterization of C.

Our notations become clearer when we write the integrands in component


form:
F ( X ( t ) ) X(t) dt = ( M , N ) (x(t),y(t))dt = M x ( ~dt) + Ny(t) dt
= M d x + Ndy
This yields the computationally useful formula:

d M dx + Ndy = l M(x(t),y(t))x(t)
dt + N(x(t),y(t))y(t)dt
If a smooth curve C is decomposed into pieces C, . . . , C, as in Fig. 2 1.8(a),
then the additivity property of definite integrals implies that

Fig. 2 1.8 2 1.2.6


d Mdx+Ndy= 5 S M d x + N d y
i=l C,
742 CHAPTER 21: THE LINE INTEGRAL

Hence we extend the notion of a line integral to piecewise smooth curves


C= U rZlCiby insisting the line integral over C be defined as the sum of the
integrals over the smooth curves Ci; see Fig. 21.8(bj to the left.
If C has the parameterization X ( t j , a 5 t 5 b, then, by (21.2.1 j , - C has the
parameterization Y ( s ) , a 5 s 5 b, where Y ( s j = X ( a + b - s). By the Chain
4 Rule, Y ( s ) = X(a + b - s)( - 1 ) . Thus
c = u 1c ,
j =

lcMdx + Ndy = J b F ( Y ( s ) ) * Y ( s ) d s
a
b
b
= F(X(a + b - s)) X(a + b - s)( - 1 ) dJ
Now if t = a +
b - s, then dt = -ds. Furthermore, s = a implies t = b, and
s= b implies t = a. With this substitution we have

Mdx + Ndy = J a F ( X ( t ) ) X ( t j dt
s, b
b

= -l F ( X ( t j ) X ( t ) dt = - J M dx + Ndy
C

Thus we conclude

+ Ndy = -
2 1.2.7 Mdx
R Mdx + Ndy

Example 3 Evaluate xy dx + y 2 dy, where

(a) c: X ( t ) = ( t , t 2 ) , 0 5 t 5 1.
(bj c: X ( t j = ( t , t ) , 0 5 t 5 1.

Solutions (a) Sincex(t) = t,y(t) = t2, we havedx = x(t) dt = dt and dy = y ( t ) dt = 2t dt.


Thus
1
xy dx + y 2 dy = [ t ( t 2 )dt + (tz)2tdt]

( b ) Now x ( t ) = y ( t ) = t , so dx = dy = dt, and


1
xy dx + y 2 dy = ( t 2 dt + t 2 dt)
=l 1n i?dt=Tll=F 2 0

Note that in Example 3 the curves are, respectively, the piece of the parabola
y = x 2 joining (0,O)to (1, 1 ) and the straight-line segment joining the same two
points alongg = x ; see Fig. 21.9(a) and (b), respectively, top of page 743.
In situations of this type, where the curve C is given byy = f ( x j , a 5 x 5 b
[or x = g ( y ) ,c < y 5 d ] , one can work directly with the functional equation.
For example, in part ( a ) of Example 3 ,
SECTION 21.2:THE LINE INTEGRAL 743

9 Y

t t

(a
Fig. 21.9

thus dy =f(x) dx = 2x dx, and so

4
xY dx + Y 2 dy = l 1
[x x2 dx + (x2), 2x dx] = s
0
1
( ~+
7
3 2x5) dx = -
12

Example 4 Evaluate Jc 2 9 dx + x2 dy, where


(u) C is the curve obtained by traversing the x axis from ( - 1,O) to (1,O) and
then counterclockwise along the parabolay = 1 - x2 to (0, 1).
( b ) C is the directed straight-line segment jpining ( - 1 , O ) to (0, 1).
Solutions (u) Cis a piecewise smooth curve consisting of a directed line segment C, and a
parabolic arc C, (see Fig. 21.10). Now C, has the parameterization
J
c,: ( t , O ) , -1 5t5 1
.f If we try to parameterize C, by letting x = t,y = 1 - t 2 , 0 5t5 1, we obtain
the wrong orientation. We can use (21.2.1) to obtain
c,: (1 - t, 1 - (1 - t ) 2 ) = ( 1 - t,2t - P),O 5 t 5 1
Alternatively, we can work with
-c,: (t, 1 - t2), 0 5t5 1

(-1, 0) c, (1, 0;

= L, 2xydx + x2dy - s
-62
2xydx + x2dy

(-1, 0) X(t) = P + tPQ,O 5 t 5 1


744 CHAPTER 21: THE LINE INTEGRAL

c: X(t) = (-1,O) + t(l,1)


= (-1 + t,t), 0 5 t 5 1
Thus
2 9 dx + x 2 dy = 4 [2( - 1 + t)t dt + ( - 1 + t ) 2 dt]

Since both of the curves in Example 3 join (0,O) to ( 1 , l), we see that the
value of a line integral over a curve joining two points will in general depend on
the curve. O n the other hand, in Example 4 the integrals over two distinct
curvesjoining ( - 1 , O ) to (0, 1 ) have the same value. It appears that integrals of
certain vector fields might be independent of the path that one chooses to join
two points. We shall investigate this question thoroughly later.
We close this section by giving an example of a line integral over a closed
curve.

Example 5 Evaluate
X
L*dx+- x 2 +;y 2 4

where C is the unit circle oriented in the counterclockwise direction.


Solution Now C: (cost, sin t ) , 0 5 t 5 277, so dx = -sin t dt and dy = cos t dt. Thus

-sin t( - sin t dt) cos t(cos t d t )

i2.;:
[ 1
sin2 t +
sin2 t dt +
cos2 t
+ sin2
=
t + cos2 t
i 2.;:
1
-
- l*dt= 27i 0
1 1

PROGRESS TEST 2

1. Evaluate Jc x 3 dx + 2 x d y , where C is the square 2. Evaluate Jc(x - y 2 ) dx + +


( x 2 y) dy, where C is the
with vertices (0,O): (0,2), ( 2 , 2), (2,O) oriented in the piecewise smooth curve obtained by traversing the
clockwise direction. line y = x from (0,O) to (1, l ) , and then clockwise
along y = 2 - x 2 to ($i, 0).

J
SECTION 21.3:INDEPENDENCE OF THE PATH 745

SECTIOS 2 1 . 2 EXERCISES

I n Exercises I to I0 evaluate the line integral over the given curve.


1. J c ( 3 ~ 3- 2 ~ dx
) +(x3 +3')dy; C: X ( t ) = 7. JC(xeu) dx + (x*e,) dy; C : X ( t ) = ( t , t ) , 0 5 t 5 1
(2t +l,t2), 0 5 t 5 1 8. Jc x 2 cosy dx - y 2 sin x dy; C: X ( t ) = ( t , t ) , 0 5 t 5 1
2. + +
Jc(3x3 - 2 x ) dx (x3 3y2) dy; C is the directed line
segment joining ( 1 , O ) to (3, 1).
-y ak dy C is the ellipse -
x2
4
+ y2
- = 1 ori-
9
3. Jc(3x5 - 2 x ) dx + (x3 + 3y2) dy; C is the triangle with ented in the counterclockwise direction.
vertices ( - 2 , 0 ) , (0, l), and (3, 0) oriented in the coun-
2dx xdy
terclockwise direction. 10. 7 +7 ;C is the straight line segment joining
4. Jc siny dx + x cosy dy; C is the directed line segment
joining (1, a/3) to (2, a/2). the point (0, -3) to the point (1, - 1).
5. Jc(x + y ) dx + ( y + x 2 ) d y ; C is the curve joining 1 1. Let F(x,y,2 ) = ( M ( x , y ,21, N ( x , y ,21, P ( x , y , 2)) be a
( - 3, - 3) to (0,O)along the liney = x and then to ( 1 , 1) three-dimensional vector field defined on some open
alongy = x 3 . subset D of R, and let C be a smooth curve in D with the
6. Jc(2xy2 + 1) dx +(2x5 - 1) dy; C is the rectangle with parameterization X ( t ) = ( x ( t ) , y ( t ) , t ( t ) ) , a 5 t 5 b.
vertices ( - 2 , 0 ) , (1, 0), (1, l ) , and (-2, 1) oriented in Using(21.2.3) asamodel, defineJ,Mdx + Ndy + Pdz.
the counterclockwise direction. State the analog of (21.2.5).

In Exercises I 2 to I5 evaluate the line integral over the given curve (see Exercise 11).
12. JC + <
xy dx yz dy + xt dz;C : X(t ) = ( t , t 2 , t 3 ) , 0 t 5 1 meterizations for the same smooth curve C, then it can
13. Jc x2yz dx - v 2dy + t 3d z ; C: X ( t ) = ( t , 1 - t , t 2 ) , be shown that there exists a scalar function t = h ( s )
O<t<l defined on [a, b] with range [c, a'] such that h ( c ) = a and
14. Jcxz dx + y 2 dy +
z2 dz; C : X ( t ) = (cost, sin t , t ) , 0 5 h ( d ) = b , h'(s) >
0 on [c, d ] , and Y(s) = X ( h ( s ) ) ,
t 5 m/2 c5s < d. Using this information, show that
15. J c e Z d x + eYdy + e x & ; C : X ( t ) = ( t 2 , t , t 2 ) , 0 5 t 5 1 Jc F(X) d X is independent of the parameterization
16. If X ( t ) , a t < <
b, and Y(s) c 5 s 5 d, are both para- of c.

I n Exercises 17 to 20 use this dejinition to evaluate the given integral. Let f (x, y ) be continuous on D and let C: ( x ( t ) , y ( t ) ,a < <
t b, be a smooth
curve contained in D . T h e line integral of the scalar function f over C with respect to arc length is dejined by Jc f ( x , y ) ds = f ( x ( t ) , JB
A t ) )4[X'(t)l2 + b ' ( t ) I 2 dt
17. Jc x'y3 ds; C: (sin t , cos t ) , 0 5 t 5 a/2 18. Jc x 3 2 ds; C : (cos t , sin t ) , 0 5 t 7/4 <
19. <
Jc(y3 + x ) d s ; C : ( t 3 , t ) , 0 t 5 1 20. <
J c x y 2 / 3ds; C: ( t , t 3 l 2 ) , 0 t 5 1

A thin wire is assumed to occupy the curve C. Let f ( x , y ) denote the density (mass per unit length) of the wire at the point (x, y ) on C. In Exercises 21
to 25 derive an integral dejnition f o r the given quantity. (See the discussion preceding Exercises I7 to 20.)
2 1. M , the mass of the wire. 22. M,, the first moment of the wire about the x axis.
23. M,, the first moment of the wire about t h e y axis. 24. I,, the second moment of the wire about the x axis.
125. I,, the second moment of the wire about t h e y axis.

Section 2 1.3: Independence of the Path

Objectives: INTRODUCTION: CONSECTED REGIOSS


1 . Determine when a line integral In this section w e shall show t h a t t h e line integral of a conservative vector field
is independent of the path. over a c u r v e C d e p e n d s only on t h e e n d p o i n t s of C and n o t o n t h e curve itself.
T h i s implies t h e reasonable conclusion t h a t t h e work d o n e by a conservative
2. Evaluate a path-independent line
force field i n m o v i n g an object f r o m a p o i n t A t o a p o i n t B d e p e n d s only on t h e
integral using the Fundamental
points and n o t on t h e p a t h t h e object travels in going f r o m A t o B. To s t a t e
Theorem for Line Integrals.
these results precisely we shall need:

2 1.3.1 Definition An o p e n subset D of R, is connected if any t w o points i n D can be


j o i n e d by a piecewise s m o o t h c u r v e t h a t lies wholly w i t h i n D.
746 CHAPTER 21: THE LINE INTEGRAL

D = D, u D,

(6)

Fig. 21.12

This states mathematically what we normally mean by a connected set-one


which does not consist of disjoint pieces. Each of the sets in Fig. 21.12(a), ( b ) ,
and ( c ) is connected, whereas the region in Fig. 2 1 . 1 2 ( d ) is not.
In dealing with connected regions it is often necessary to distinguish between
those in (a) and (6) in Fig. 21.12, which do not have holes, and (c), which does.

2 1.3.2 Definition An open connected subset D of R, is simply connected if whenever C


is a simple closed piecewise smooth curve lying in D , then all points enclosed by
C also lie in D.

The region sketched in Fig. 21.12(c) is connected but not simply connected.

2 1.3.3 Definition Suppose D is a n open connected subset of R, and F = ( M , N ) is


a vector field defined on D.We say that the line integral of F is independent ofthe
+
path in D if, given any P and Q in D, then f c M dx N dy is the same for all
piecewise smooth curves C joining P to Q and lying in D.

If f c M dx + N dy is independent of the path in D and if P and Q are in D ,


then we shall often write

lQMdx + Ndy
to denote the common value of the line integral over any curve joining P to Q.

Ih-DEPENDENCE OF THE PATH


By the second form of the Fundamental Theorem of the Calculus (6.3.7),
SECTION 21.3: INDEPENDENCE OF THE PATH 747

This means that if the integrand is a derivative, then the value of the definite
integral depends only on the endpoints of the interval. The following is an analogous
version for line integrals.

2 1.3.4 The Fundamental Theorem of Calculus for Line Integrals Sup-


pose F = Vf, where f has continuous first partials on the open set D.If P and Q
are any two points in D and if C is any piecewise smooth curve in D joining P
to 9,then

ProoJ We shall assume that C is smooth. The extension to piecewise smooth


curves is routine (see Exercise 17). Suppose C has the parameterization X ( t ) ,
a 5 t 5 b. Then

k VdX = ibv f ( X ( t ) )* X ( t )dt

We are now in a position to state several equivalent conditions to the path


independence of a line integral. First we assume:
1. D is an open connected subset of R.
2 . F = ( M ,N), where M , N have continuous first partials in D.

2 1.3.5 Theorem The line integral M dx sC + N dy is independent of the path in D


if and only if F is conservative.
ProoJ Assume F is conservative with, say, F = Of: Then by (21.3.4),
+
J c M d x Ndy depends only on the endpoints of C and not C itself. Thus
S c M d x + Ndy is independent of the path in D.
Conversely, assume that the integral of F is independent of the path in D. Let
(a, 6 ) be some fixed point in D , and for each ( x , y ) in D define

f(x,y)= L M dx + Ndy
where C is any piecewise smooth curve joining ( a , b ) to ( x , y ) .Since D is open,
there exists some open disk ab2ut ( x , y ) lying entirely within D. It follows that
we can choose curves C and C in D , as in Fig. 21.13. Now

+ Ndy
= L,Mdx + Ndy +
c2
Mdx

Now the first integral depends only o n y , so


748 CHAPTER 21: THE LINE INTEGRAL

Fig. 21.13

Also, on C, dy = 0 , so

,.
Thus f, = M . By a similar argument we can work with C to conclude that
fy = N. It follows that F = of;
so F is conservative. W

2 1.3.6 Theorem The line integral Sc M dx + N dy is independent of the path in D


if and only if J,Mdx + Ndy = 0 for every piecewise smooth closed curve K
contained in D.
Pro@ Let P, Q be any two points in D and let C, C, be any smooth curves
joining P to Q; see Fig. 21.14(a). Then K = C, U -C, is a piecewise smooth
and closed curve [see Fig. 21.14(b)] and so

0= M dx +N&
-
- Mdx + Ndy

Thus
sCl
Mdx -+ Ndy = s
cz
Mdx + Ndy
and so the integral is independent of the path. The proof of the converse is the
content of Exercise 18.

In Sec. 21.1 we noted that if F = ( M , N ) is conservative, then My = N,.


Now if F happens to be the vector field

and if D = { ( x , y ) I ( x , y ) # (0,0)}, then Mu and N, are continuous in D and


M y = N, = ( y 2 - x 2 ) / ( x 2 + y2),. If Cis the closed unit circle, then by Example
SECTION 21.3: INDEPENDENCE OF THE PATH 749

(a)
Fig. 21.14

5, Sec. 21.2, page 744, JcMdx +


Ndy = 277 f 0. Hence, by (21.3.6) and
(21.3.5), F is not conservative in D.
It can be shown that if we strengthen our original condition (1) and insist
that D be simply connected, then the condition M u = N, is sufficient to ensure
independence of the path.

2 1.3.7 If D is simply connected, then Jc M G?X + N dy is independent of the path in D


if and only if M , = N,.

The problem with the above integral of F around the unit circle Cis that to
ensure the continuity of the component functions (and their partials), we must
exclude (0,O)from any connected region containing C. Since C encircles the
origin, any such region will have a hole and hence will not be simply connected
(see Fig. 21.15).
t X
Suppose we let 6be the simply connected open subset of R, consisting of the
xy plane with the negative x axis and the origin removed. Now, we noted above,
if M = - y / ( x 2+ +
y 2 ) ,N = x / ( x 2 y 2 ) ,then M , = N,. Furthermore, M and N
are continuous in 5, so by (21.3.7) and (21.3.5), F = ( M , N ) is a gradient. If
we proceed formally as in Sec. 21.1 to find a functionfsuch that ..,
F = Of,we

Fig. 21.15
I obtainf(x,y) = arctan(y/x). However, f is not continuous on D !To see this,
take any point on the positivey axis, say ( 0 , l ) , and let P = ( x , y ) tend to ( 0 , 1)
along the curves indicated in Fig. 21.16(a). Along one curve y / x -+ + cc), so
f ( x , y ) -+ m/2, while along the otherylx -+ - CC], sof(x,y) + -77/2. In Mis-

4
f
I

//I D

(a)
Fig. 21.16
750 CHAPTER 2 1 : THE LINE INTEGRAL

cellaneous Exercise 2 you are asked to show that the correct choice o f f is
given by

I
arctan(y / x ) x>0
T/2 x = 0,y > 0
f ( x , y )= +
'i; arctan(y/x) x < O,y > 0
- 5T/2 x = 0,y < 0
-T +
arctan(y/.x) x < 0,y < 0

Although this function may look gruesome, it is merely the function that assigns
to each ( x , y ) its unique polar angle 19between - 7 / 2 and ~ / 2 see
; Fig. 21.16(b).

PROGRESS TEST 1

Evaluate each of the following:

[ ( e 2 cosy) dx - (e2 siny) dy]

SECTIOS 2 1.3 EXERCISES

Eualuate each of the zntegrals in Exercises I to 10.

1. (4.5' + 3 x 2 ) dx + ( 2 x 3 - 3y2)dy 2. .T,':p' (2xe2"(x + 1) - c o s j ) dx + ( x siny + 2y) dy


(7,%/2)
3.
LO' (siny - y cos x ) dx + ( x cosy - sin x) dy

3.

7.
(a/ 6 , z / 3 )
( 2 sec x tan Y tany + 2 x ) dx + (secz x seczy) 4 8.
( x - 1) dx + Ydy
(0.0) ( x - 11, + y 2 .( - +y2

9. ( --y2e-Su ) dx + (1 - v)e-'U dy ( y 2 cosh x - 2x cosh y ) dx + (2y sinh x - x2 sinh y ) dy


11. Show that

(x2
Y
+ y"3'2 4
is independent of the path in D = { ( x , y ) 1 ( x , ~ ) (0,0)). +
I n Exercises 12 to 16 show that the integrals are not independent of the path in D using either (21.3.7) or (21.3.6).

12. (3x' - 2y) dx + (22' + 2.x) dy; D = R , 13. 4 (y' + y ) dx + (2xy - eU) dy; D = R ,
+1
14. +x+-
c ."I!
.Y

xy 2
d x ; D = { ( x . ~ (x:y)
) ~ f (0,0)) 15. ( y - y 2 ) dx + (2xy - x ) dy; D = R ,

.'; -' + ( x -x 1)2


- 1
dx 4;
16. (w -
1)' + y 2 +y2 18. Show that if Jc M dx +
Ndy is independent of the path
D= {(XLY) I( x d )#
(1,O)) in D, then J,Mdr +
Ndy = 0 for every piecewise
17. Prove Theorem (21.3.4) for piecewise smooth curves. smooth closed curve K contained in D.
SECTION 21.4: GREENS THEOREM 75 1

Section 2 1.4: Greens Theorem

Ob j e ct ive s: INTRODUCTION
Understand and use Greens Greens Theorem relates the value of a line integral over the boundary of a
Theorem to aid in evaluating line region in the plane to a certain double integral over that region. It has several
integrals or double integrals. interesting physical interpretations, some of which will be discussed in Sec. 2 1.5.
As we shall see shortly, it is also a multipurpose tool for evaluating integrals. We
begin our development by discussing the types of regions for which Greens
Theorem applies.

REGULAR REGIONS
We shall assume throughout this section that all curves are piecewise smooth and
simple.

2 1.4.1 D e f i n i t i o n A plane region D is called regular if it is closed and bounded, and


if the boundary of D consists of a finite number of nonintersecting closed curves.

If D is a regular region, then we denote the boundary of D by aD.The


boundary is said to have the standard orientation if each of the curves Ci making
up the boundary is oriented so that the region is always on the left as one moves
along Ciin the direction of increasing parameter. Some examples of regular
regions with their standard oriented boundaries are shown in Fig. 21.1 7.
Note that the standard orientation condition forces the outer boundary
curve to have the counterclockwise orientation, whereas all inner boundary
curves are oriented in the clockwise direction.
If D is a regular region with the standard orientation, then we shall denote the
sum of the integrals of each of the closed curves that make up all by

(c)

Fig. 21.17
752 CHAPTER 21: THE LINE INTEGRAL

GREENS THEOREM AKD SOME EXAMPLES


2 1.4.2 Greens T h e o r e m Suppose that M and Nhave continuous first partials in
the regular region D . Then

We shall defer a discussion of the proof of Greens Theorem until after we


have had a n opportunity to see how it is used in evaluating integrals. In
particular, the following examples illustrate situations where Greens Theorem
is used to:
1. Transform the problem of evaluating a line integral to that of evaluating a
simpler double integral;
2. Transform the problem of evaluating a double integral to that of evaluat-
ing a simpler line integral;
3. Transform the problem of evaluating a line integral over a closed curve C
to that of evaluating a line integral over a simpler closed curve.

Example 1 Evaluate

( x 2 -y3) dx + ( y 2 + x 3 ) dy
where D is the annular region of Fig. 21.18 with inner radius rl and outer
radius r2.
Solution Since D is a regular region, we apply Greens Theorem with
M ( x , y ) = x2 - y 3 N(x,y)=y2 + x3
M,(x,y) = - 3Y 2 = 3x2
N,(x,y)
Thus
( x 2 - y 3 ) dx + ( y 2 + x3) dy =
Evaluating this latter integral using polar coordinates, we have

If D is a region in the xy plane, then we know that the area of D is given by

A(D) = sf D
dA

In many instances, when D is regular, determining A ( D ) can be simplified by


Fig. 21.18 Greens Theorem.

Example 2 Show that the area of a regular region D is given by

Solution Let M ( x , y ) = -y and N ( x , y ) = x. Then N, - M , = 2 and so, by Greens


Theorem,

J L 2 d A = J J d A =A(D) 0
D
SECTION 21.4: GREENS THEOREM 753

Example 3 Use the result of Example 2 to find the area of the region enclosed by the ellipse
x2/a2 +y 2 / b 2 = 1.
Solution Let D be the region enclosed by the ellipse. Then D is a regular region and a
counterclockwise parameterization for aD is given by
( a cos t , b sin t ) , 0 5 t 2 271
Thus dx = - a sin t dt, dy = b cos t dt, and so

1
A(D) = -$
2 aD
[-ydx + xdy]
=
1
4ZT
[( - b sin t)( - a sin t dt) + (a cos t ) ( b cos t dt)]

If K is a piecewise smooth simple closed curve, then the region inside K is


simply connected. Hence if M y = N, in this region, then by Theorems (21.3.7)
and (2 1.3.6), we know that M dx sK +
N dy = 0. However, if K encircles a point
at which the partial derivatives are not continuous, then (as noted in Section
2 1.3) even though M , = N, for all other points, we can no longer conclude that
the line integral over K is zero. T h e problem, of course, is that any region
containing K and excluding the bad point P will necessarily not be simply
connected, and so we cannot apply Theorem 21.3.7 (see Fig. 21.19). Such a
region will, however, be regular and we can use Greens Theorem to transform
Region containing K but excluding P the problem of integrating over the curve K to the often simpler problem of
Fig. 21.19 integrating over a circle.
Specifically, let P be a point encircled by K at which the partials of M or N
are discontinuous. Center a circle r about P of small enough radius to avoid
intersecting K. Then D , the region inside K and outside r, is regular, so by
Greens Theorem

[N, - M y ]dA = $ M d x + Ndy = M dx + Ndy + M dx + Ndy


2D r
where K has the counterclockwise orientation and r has the clockwise orienta-
tion. If M , = N, in D , then JJD [N, - M,] dA = 0,
so
D

where -r now has the counterclockwise orientation. In particular, if P = ( a , b )


and r has radius r, then
Fig. 2 1.20 -r: (a + rcost,b + rsint) 0 5t5 ZT

Example 4 Evaluate

where K is a closed curve oriented in the counterclockwise direction consisting of


+
the piece of the ellipse x2/9 y2/16 = 1 in the third quadrant joining ( - 3,O)
to (0, -4), the straight-line segment from (0, -4) to (3, 0), and the semicircle
y = q n f r o m (3,O) to ( - 3 ,0 ) as in Fig. 21.21.
754 CHAPTER 21: THE LINE INTEGRAL

Solution We note that M, = N, = ( y 2 - x 2 ) / ( x 2 +y2)*for all ( x , y ) # (0,O). Let r be


the unit circle centered at the origin with the clockwise orientation. Then
-r: (cost, sin t ) 0 5 t 5 271
and so, arguing as above and referring to Fig. 21.21, we have
X
x2 + y
- dx +- X
x 2 + y 24
- sin t( -sin t dt) (cos t)cos t dt
cos2 t + sin2 t + cos2 t + sin2 t 1
=d 277
dt=2r

One can appreciate the advantage of the above approach by attempting a


direct calculation of the line integral in Example 4 using the definition.
Fig. 21.21

PROGRESS TEST 1

1. Let A ( R ) denote the area of the regular region R. 3. Evaluate


Show that

where K is the counterclockwise curve from (0,3)


, . to I

2. Use Greens Theorem to evaluate ( 0 , - 3) along x = y 2 - 9 and then to (0, 3) along


they axis.

where K is the rectangle with vertices (1, l), (2, l),


( 1 , 4 ) , (2,4) oriented in the counterclockwise direc-
tion.

GREENS THEOREM: d DISCUSSIOK OF THE PROOF


We shall prove Greens Theorem for a special type of region and then indicate
how the theorem can be extended to more general regular regions. Specifically,
we assume that D is simultaneously of Type I and Type 11, as discussed in Chap.
20. Thus D can be expressed in the form
(1) = {(.,Y)l. i X Ib , m I Y I @)I
and in the form

(2) D {(X,Y)lC <Y I d,p(y) IX < 4(Y))


Such a region is sketched in Fig. 21.22(a) and ( b ) .
Now assume M and N have continuous partials in D. It suffices to show that

(3)
and
SECTION 21.4: GREEN'S THEOREM 755

for then we can add these integrals to obtain the conclusion of Green's Theo-
rem. To establish (3) we work with D in the form (1)' where C, and C, are as
depicted in Fig. 21.23. Note that C, has the parameterization ( x , f ( x ) ) ,
a 5 x 5 b , and -C, has the parameterization ( x , h ( x ) ) , a 5 x 5 6. Now

-JJ My dA = -I4,)
b h(x)
MY(X'Y)dy dx

= -lb [ M ( x , y ) ] h ( idx
f (I)
) by (6.3.7)

I 1 * X
a b

Fig. 2 1.23

Thus (3) is established. The proof of (4) is essentially the same if one works with
the characterization (2) for D (see Exercise 17).
We can gain some insight into how the proof of Green's Theorem could be
extended to more general regions by considering the semiannular region D in
Fig. 21.24
Fig. 2 1.24. Now D is not a region of the type considered in our proof, but D can
be broken into the two regions D,and D,, which are of this type (see Fig. 2 1.25).
Applying Green's Theorem to D, and D,,we have

and
c1 c5
Fig. 21.25
756 CHAPTER 21: THE LINE INTEGRAL

Fig. 21.26

Now

However, -C, = C,,a n d so

sc3
M d , + Ndy + s
c1
Mdx + Ndy = s (73
Mdx + Ndy - J h f d x + Ndy = 0
c3

Thus

We can in turn now obtain a version of Green's Theorem for a n annular


region (Fig. 21.26) by breaking it into two semiannular regions a n d proceeding
much as above (see Exercise 18). In fact, a general proof of Green's Theorem
can be constructed by breaking u p a regular region into carefully chosen
subregions a n d proceeding in a like fashion.

SECTIOS 2 1.4 EXERCISES

In Exercises I to 5 uerlfv Green's Theorem bJ directly evaluatzng both gDM dx + N dy and JJD [N,- M y ]d A
<
1. M ( x , y ) = xy, N ( x , y ) = - x ; D = { ( x , y ) I O x 5 2 , 4- M ( x , ? ) = 3 x +
2 ~ N, ( x , y ) = 2 x + 9; D is the region
O<Y<j) enclosed by the ellipse x2 + ( y 2 / 4 )= 1.
2. M ( x , y ) = j . N ( x , y ) = 0; D is the annular region inside 1 4x?, , V ( X , ~ =
3. N ( x , y ) = 3 ~ 3 ~ - ) ~ 3D ~ = (;
(x,y)l
the circle x 2 + y 2 = 9 and outside the circle O<x<2,0<y52x}
x2 +y' = 1.
3. M ( x , y ) = e z siny, N ( x , y ) = e x cosy; D =
{(.,y)lO 5 x <T / 4 %0 < y 5 s j 4 )

I n Exercises 6 to 10 use Green's Theorem to evaluate the given line integral.

6. f K x 3 dx + 3x2y2 4,
where K is the rectangle with ver- 9. $?K
( x 2 + y 2 ) dx - x2 dy, where K is the circle
tices (0,0),(0, 3), ( 2 , 3), (2.0). x2 +y2 = 9.
7. $iK ( x 2 + y 2 ) dx + ( y 2- x') dx, where K is the circle 10. $2K (~34+ 3y - sinx) dx + (293 - 2eY) dy; K is the
x2 +y2 = 4. rectangle with vertices (0, 0), ( 3 , 0 ) , ( 3 , 4 ) , (0,4).
8. iGK
+ e" cosy dy,
ex siny dx where K is the circle
( x - 1)2 + ( y + 5)? = 9
SECTION 21.5: APPLICATIONS OF GREENS THEOREM: DIVERGENCE AND CURL 757

In Exercises 11 to 14, use Greens Theorem to compute the area of the given region (see Example 2 and Prob. 1, Progress Test I)
11. The triangle with vertices (0, 0), (4, 0), and (4, 3). 17. Let D be a region of the form D = { ( x : y ) l c < y 5 d,
12. The region bounded by the curves9 = x 2 andy = x + 2. p(y) 5 x 5 q ( y ) } .Show that ifM and N have continu-
13. The region inside the curve +y23 = 4. ous partials in D,then
14. The region bounded by the curves y = x 2 - 1 and
y = 0.
15. Let C be a piecewise smooth simple closed curve that
does not pass through the point (2, - 1). Show 18. Show how the argument that established Greens Theo-
rem for a semiannular region can be extended to yield
(Y + 1) dx ( 2 - X) dx
$ (x - 2)2 + (y + 1)2 + (x - 2)2 + (y + the conclusion of Greens Theorem on an annular region.
(See Fig. 21.26.)
257 if (2, -1) is inside C
0 if (2, -1) is outside C

16. Show that

for every piecewise smooth closed curve K that does not


pass through the origin.

Applications of Greens Theorem:


Section 2 1.5: Divergence and Curl

Objectives: INTRODUCTION
1. Determine the flux of a vector Greens Theorem has some interesting physical consequences depending on the
field over a closed curve. types of vector fields under consideration. Among the most common are
inverse-square fields and velocity fields. The former arise in problems from
2. Determine the circulation of a
electrostatics; the latter arise naturally in the study of fluid dynamics. We shall
vector field over a closed curve.
discuss a two-dimensional steady-state fluid flow. This is a classical problem
whose importance has been revitalized in recent years as the study of fluids,
especially blood, has come under serious mathematical study. O u r work is only
a first step because of the simplifying assumptions made. Subsequent steps take
the study into three dimensions and take into account such additional variables
as friction, viscosity, and time-dependent flow (as with a pulse).
Specifically, suppose a fluid of constant density 6 is flowing in a stream whose
surface occupies the region D in the plane. As in Sec. 2 1 , l we assume the flow is
horizontal and independent of the depth. In effect, this allows us to treat the
problem as two dimensional. We further assume the flow is steady state. In this
setting the velocity field of the flow has the form

V ( W ) = ( M ( x , y ) ,N ( x , y ) )
Fig. 2 1.27 A sketch of a typical such steady-state velocity field is given in Fig. 21.27.

Consider a region R in D whose boundary consists of a piecewise smooth


simple closed curve K. We wish to discuss two measures of the flow. One is
called thejux of Sv over K and measures the rate of flow of the mass of the fluid
out of the region R. The other is called the circulation of v around K and, in a
sense to be made more precise, it measures the net tendency of the fluid to flow
along K. We assume throughout that K has the counterclockwise orientation
and has parameterization
K: X(t) = (x(t),y(t)), a 5t5b
758 CHAPTER 21: THE LINE INTEGRAL

As an aid in determining the flux and circulation we shall use the unit
tangent T to K and n the outer unit normal to K. From Chap. 18 we know that

or equivalently [since d X = X ( t ) dt and ds/dt = lIX(t)il]:

where s represents arc length along K . Since K has the counterclockwise


t n
orientation, n is obtained by rotating T 90 in the clockwise direction (see Fig.
Fig. 2 1.28
21.28). Thus n is given by

(3)
or

(4) n =($,$)
We shall develop an integral giving the flux by summing flow in the
direction of n and then develop an integral giving the circulation by summing
flow in the direction of T.

THE FLUX ISTEGRAL


Since the dot product of two vectors has the same sign as the cosine of the angle
between them (1 7.4.4), we know that V . n > 0 at points on K where the fluid is
<
leaving R. Similarly, v * n 0 at points where the fluid is entering R. If
v * n = 0, then the flow is parallel to the tangent and so the fluid is neither
entering nor leaving R at that point (see Fig. 21.29).
To determine the flux we shall subdivide K into n pieces C, . . . , C, having
arc lengths Asl, . . . , Asn, respectively. We let v, and n, denote the values of v
and n at some point on C, and treat them as being constant throughout C,.
Since n, is fixed, we are in effect treqting C, as if it were a straight line segment.
In Fig. 21 30(a) we see the actual situation, whereas Fig. 21.30(b) shows the
idealized approximation. We further assume, for now, that the flow is out of R
over C, as in Fig. 21.30.
If n, is parallel to v,, then the area A , of the fluid that passes through C, in
time At, is equal to the area of the reGtangle with sides of lengths AJ, and

v-n = 0
D

Fig. 2 1.29 Fig. 2 1.30


SECTION 21.5: APPLICATIONS OF GREEN'S THEOREM: DIVERGENCE AND CURL 759

Ati 11 v, 11 ; see Fig. 2 1.31( a ) . If n, is not parallel to v,, then the area of the amount
of fluid that passes through C, is equal to the area of the parallelogram whose
base has length Ast and whose height is the scalar projection of Ativz onto n, (see
Fig. 21.31). In either case,

A , = (ProjnZAt,v,)As, = ~ asi = on, AS,

The amount of mass flowing through C, in time At, is approximated by


( 0' M , = SA,, so the rate of flow is approximately

(5)

Since we are trying to measure the net outward rate of flow, we regard -A , as
the flow into R over Ci.However, as noted earlier, in this case v, en, would be
negative and so our approximation in (5) would still be valid. We now sum up
Ativi the contributions over all such segments to obtain our approximation to the
(b) flux:
Fig. 2 1.3 1 n

This suggests the following definition of the flux of Sv over K .


n

2 1.5.1 Definition

= (M,N}*(dy,
-dx} = -Ndx + Mdy
and thus we have

21.5.2 Theorem Flux = L 8 v . n ds = S h - Ndx + Mdy

Example 1 Determine the flux of v over the unit circle x 2 + y 2 = 1 where v = ( y 2 ,x 2 } .


Solution A counterclockwise parameterization for K , the unit circle, is given by
K : (cos t , sin t } 05t 5 277

Thus, by (21.5.2))
Flux = -c, &ends = Sl -N& + Mdy
= S L -x2dx + y 2 d y

= q7' "_
- cos2 t( -sin t dt) + sin2 t(cos t dt)

It is customary to define the notion of flux for any vector field (not merely one
associated with a fluid flow).
760 CHAPTER 21: THE LINE INTEGRAL

2 1.5.3 Definition Suppose F(x,y) = ( M ( x , y ) ,N ( x , y ) ) is a vector field and K is a


piecewise smooth simple closed curve with counterclockwise oripntation. The
fzux of F over K is defined by

Flux = 4 F e n ds = 4 -N dx + M dy
where n represents the outer unit normal to K.

THE DIVERGESCE THEOREM


There are two types of derivatives associated with a vector field, the diver-
gence and the curl. The first of these is related to the flux integral via
Greens Theorem.

2 1.5.4 Definition The divergence of the vector field F = ( M , N ) , denoted div F, is


defined by div F = M , N,. +
21.5.5 Divergence T h e o r e m Suppose K is a piecewise smooth simple closed
curve and that F = ( M ,N ) has continuous first partials throughout the region
R enclosed by K. Then

s, F n ds = JL div F dA

where n represents the outer unit normal to K , and K has the counterclockwise
orientation.
Prooj? The region R is certainly a regular region, and since K has the counter-
clockwise orientation, it yields the standard orientation for aR. Thus

Fends = 4 -Ndx + Mdy


-
- tR
n

-Ndx + Maj

= Jl (M, - ( -Ny)) dA (by Greens Theorem)

= Sl (M, + N u )dA = Ss div F dA


R

Example 2 Determine the flux of the vector field F(x,y) = ( x 3 , y 3 )over the unit circle K.
Solution By the Divergence Theorem

Flux = F n ds = si div F dA (D is the unit disk)

=3 12vi1 377
r3 dr d0 = - 0
2
SECTION 21.5: APPLICATIONS OF GREENS THEOREM: DIVERGENCE AND CURL 761

In the case where F is a velocity field, div F(x,y) measures the rate of mass
flow per unit area from a point ( x , y ) . A point where div F >
0 is said to be a
<
source and a point where div F 0 is called a sink. If div F = 0 throughout the
stream, then, reasonably enough, the fluid is said to be incompresszble. A general
vector field for which d i v F = 0 is said to be solenoidal. Note that by the
Divergence Theorem, if the flux across K is zero, then either div F = 0 or else
the sources and sinks in R must cancel one another.

PROGRESS TEST 1

1. Let F(x,y) = ( x 3 + y 3 , y 3 - x 3 ) and let K be the


+
curve x2 y 2 = 4 oriented in the counterclockwise
direction. Use the Divergence Theorem to determine
the flux of F over K .

THE CIRCULATIOS IhTEGRAL AND STOKES? THEOREM


Suppose we again consider the two-dimensional flow problem, only now we
compare the velocity of the fluid along K with T, the direction of K . Now v T
measures how close the directions of v and T match one another; see Fig. 21.32.
If we envision the points on K as small beads strung along K , then along a small
piece of curve of length As the flow of the fluid will tend to move the beads in
>
the direction of K when v T 0 and in the opposite direction when v T 0. <
The beads will remain stationary at any points where v T = 0. If we sum
terms of the form v T As and pass to a limit, we will obtain an integral of the
form

I=Sv*TdJ
K

called the circulation of v around K.


Suppose now we return to our bead analogy and think of the beads as being
fixed to the curve. If I > 0, then the flow will tend to rotate the curve in the
counterclockwise direction. If I < 0, the fluid will tend to rotate K clockwise. If
I = 0, then K will not rotate. It is in this sense that we say the circulation
measures the tendency of the fluid to flow in the direction of K.
As with flux, we define circulation for an arbitrary vector field. Note from
( Z ) , page 758, that T d s = (dx,4).

v*T=O

t
T

Fig. 2 1.32
762 CHAPTER 21: THE LINE INTEGRAL

2 1.5.6 Definition Suppose F = ( M , N ) is a vector field and K is a piecewise smooth


simple closed curve with counterclockwise orientation. T h e circulation of F
around K is defined by

Circulation = s, F T u3 = 4 Mdx + Ndy.

STOKES THEOREM
21.5.7 Definition The curl of the vector field F = ( M , N ) , denoted curl F, is
defined by
curl F = (N, - M y ) k

When working with two-dimensional vector fields, one can just as well define
the curl of F to be the scalar quantity N, - M y . In higher dimensions it is
essential that curl F be defined as a vector quantity and so we elect this
convention (see Exercise 16).
Again, Greens Theorem provides a connection between the curl of F and the
circulation of F around K. This connection is usually referred to as Stokes
Theorem. As before, K has the counterclockwise orientation and R denotes the
region enclosed by K.

21.5.8 Stokes Theorem Suppose that K is a piecewise smooth simple closed


curve and F = ( M , N ) has continuous partials throughout the region R. Then

F*Tds = JL (curl F ) * k d A

ProoJ.:
F T ds = M dx + Ndy
= Ji [N, - M y ]dA (by Greens Theorem)

= Ji ( O , O , N, - M y ) ( O , O , 1) dA = ll R
(curl F) k dA

If F is a velocity field, then curl F measures the tendency of the flow to rotate
about k. Stokes Theorem provides a relationship between the circulation of F
around the boundary of a region to the curl of F on the interior of the region. If
curl F = 0 throughout, then F is said to be irrotational. Note that if F is a force
field, then the circulation integral JK F T ds = lK +
M dx N dy is just the work
done by F in moving an object along the closed curve K.

PROGRESS TEST 2

1. For F and K as in Progress Test 1, use Stokes Theo-


rem to determine the circulation of F around K.

SECTION 2 1.5 EXERCISES

In Exercises I to I0 determine t h e j u x of F over K and the circulation of F around K. Assume that K has the counterclockwise orientation.
1. F(x,y) = ( x 2 - 2y,y2 + 4n); K is the rectangle with 2. F ( x , y ) = ( n 2 + siny,yz - cos x ) ; K is the rectangle
vertices (O,O), (3, 0), (3,4), (0,4). with vertices (O,O), (77/2,0), (77/2, 77/4), (0,77/4).
SECTION 21.5: APPLICATIONS OF GREEN'S THEOREM: DIVERGENCE AND CURL 763

3. F(x,y) = ( y 3 + 2y, 3y2x); K is the circle x2 + y 2 = 9. 13. Let F = (M, N ) be a vector field a n d f a scalar function
4. F(x,y) = (ln(x2 + y 2 ) , ln(x2 + y 2 ) ) ;K is the boundary of two variables. Show that
of the semiannular region betweeny = d w
and [curl(fF)]*k = f(cur1 F ) * k + [curl(f,f)]*F
y = d m .
5. F ( x , j ) = (xy,y2 - x2); K is the triangle with vertices 14. Iff has continuous flrst and second partials, show that
curl O f = 0.
(O,O), ( 3 , 0 ) , ( 2 , 2 ) .
6. F(x,y) = ( x y z , x 9 ) ; K is the circle x 2 + y 2 = 25. 15. If F = ( M , N ) , where M and N have continuous first
7. F'(x,y) = (4x + 6y, 2x - 9y); K is a rectangle with area and second partials, show div(cur1 F) = 0.
42 square units. 16. If F = (M, N;P). curl F is defined by curl F =
8. F(x,y) = ( - y / ( x 2 + y 2 ) ,x / ( x ' + j 2 ) ) ;K is any closed (P, - iV,, M a - P,, N , - M y ) . This formula is more
curve that does not encircle or contain (0,O). easily recalled by means of the mnemonic device
9. F(x,y) = ( e x siny, e x cosy); K is any circle centered at
(0,O).
10. F(x,y) = ( x ' y + y 3 , -xyz - y 3 ) ; K is the circle
x2 +y2 = 1.
11. Let Fl = ( M l , N,),F, = ( M 2 ,N 2 ) be vector fields and
let f be a scalar function of two variables and k any real where the expansion is by elements of the first row and
number. Show that a/ ax "timesf" is understood to bey,, Assuming that all
( a ) div(F, + F,) = div Fl + div F2 partial derivatives that occur exist and are continuous,
( b ) div(kFl) = k div F,
( c ) div(fF,) = fdiv F, + v*
F,
show that
( a ) curl(Vf) = 0
12. Let F be a n inverse-square field of the form F(X) =
( b ) curl(F x G) = [(curl F ) * G ] - [F*(curl G)]
-X/llXl13. Show that F is solenoidal in the region
>
/lXl1 r, where r 0. > ( c ) curl(fF) = f(cur1 F) +
Ofx F

CHAPTER EXERCISES

R e v i e w Exercises
In Exercises I to I0 evaluate the given line integral over the indicated curve using (21.2.5).
1. J c y d x - x & ; C is the portion of the parabola 6. Jc xy2 dx - x d y ; C is the triangle with vertices (0,- l ) ,
y = 1 - x 2 from (0, 1) to ( 1 , O ) . (2,0), ( 0 , 3 ) oriented counterclockwise.
2. Jc e? dx + x 2 dy; C is the curve consisting of the line 7. Jc(x - y ) dx + xy dy; C is the closed curve obtained by
segment from (0,O)to (1, 1) followed by the horizontal traversing y = x 3 from (0,O)to (1, 1) and then in a
line segment to (2, 1). straight line back to the origin.
3. Jc x siny dx + xy dy; C is the portion of the parabola 8. Jc x dx - y dy; C is the unit circle centered at the origin
y = x from ( 0 ,0) to (1, 1) followed by the horizontal oriented counterclockwise.
line segment to (-2, 1). 9. Jcy e x y dx +
x e x y dy; C is the straight line segment from
4. Sc x dx - y 4 ; C is the portion of the graph ofy = sin x (0,O)to (1,1).
joining (0,O)to ( ~ 1 21)., 10. Jc(x - y 2 ) dx +
x dy; C is the rectangle with vertices
5. Jc 2xy dx + (x2+ 2y) dy; C is the line segment joining (0,0), (3,0), ( 3 , 2), (0,2) oriented clockwise.
(1, 1) to (2, - 1).

I n Exercises II to 20 identify those vectorjelds that are conservative (on R,) and determine the potentialfunctions.
11. F(x,y) = (e"Y(xy +l ) , x2exy) 12. F(x,y) = (6xy, 3x2 2y) +
13. F ( x , y ) = (e' cosy2 + 1, 1 - 2y e x s h y ' ) 14. +
F(x,y) = ( 2 x y / ( 1 y 4 ) , arctan(y2))
15. F(x,y) = (ye" cos(eS), -sin(e')) 16. F(x,y) = (2xeY +y2e", x2eY + y z e x )
17. F(x,y) = (sec x t a n x tany - cosy. secx sec'y + siny) 18. F(x,y) = ( 1 - 2 9 , ~ '- 1)
19. +
F(x,y) = (y2ex/(l e l ) - 2x, 2y In(1 e')) + 20. F(x,y) = (sin xy +
xy cos xy - 2xy, x2(cos xy - 1))

In Exercises 21 to 25 show that the given integral is independent of the path a n d j n d its value

22. (?,;u (2xe")) dx + ex' dy

25. lo:(:3,G16)
24. L;)') ( 3 x 9 - 2xy3) dx + (x3 - 3x3' + 2y) dy
(siny + y 2 cos x) dx + ( x cosy + 2y sin x ) dx
784 CHAPTER 21: THE LINE INTEGRAL

I n Exercises 26 to 30 show that the given integral is independent of the path in an appropriate region containing C and evaluate the integral over C.

C: X ( t ) = (cos t , sin t ) , 0 5 t 5 37/2


28. 4(+ - $)dx - (+4;
x* +
1
XY
29. 4($ cos f + 2 x ) dx - ($cos J)dy;
c: X ( t ) =( t , t2 + l), 1 5 t 5 2 C: X ( t ) = ( 7 i t / 2 , 1 + t), 0 2 t 2 1
30. 1+ (x
-Y
3)2 + y 2 dx + (x
x+3
+ 3)* + y * 4;
C: x ( t ) = ( s i n t , cos t ) , o 5 t 5 277
In Exercises 3 1 to 35 use one Ofthe area formulas deaeloped in Sec. 21.4, Example 3, or Progress Test I to determine the area Ofthe given region.
31. The region enclosed by the curve C: ( c o s ~ sin3
~ , t), 34. T h e region bounded by the curve y = x 2 , the x axis,
0 5 t 5 277. x=l,andx=4.
32. The region bounded above byy = 4 - x 2 and below by 35. T h e region above the x axis and below one arch of the
y x+ +2 = 0. cycloid x = t - sin t , y = 1 - cos t .
y2
33. The region inside the ellipse -
9
x2
+-
12
= 1

In Exercises 36 to 4 0 use Greens Theorem to evaluate the gzeen line integral.


36. Jc3ydx + 7 x 4 ; C : (cost,sint), 0 5 t 5 2 7 39. jcex siny dx + ex cosy dy; C: (c0s3 t , sin3 t ) ,
o j t 5 2 7
37* ( 2 x 3 - J 3 ) dx + ( x 3 + Y 3 ) d y ; D is the annular region 40. Jc dy + 3xy dy; C i s the rectangle with vertices (0,0),
(2,0), ( 2 , 3 ) , (0,3 ) oriented counterclockwise.
between x* + y 2 = 1 and x 2 + y * = 4.
38. J c ( x j + y 2 )dx + (2xy + 3x2) d x ; C: (cos t , sin t ) ,
o5t527

I n Exercises 4 1 to 4 5 determine diu F and curl F for the given vectorjelds


41. F ( x , y ) = ( 2 x , 3y) 42. F(x,y) = ( x 3 , -yx2)

45. F ( x , y ) = ( x sin(x/y),y2 cos(,y))

I n Exercises 46 to 55 determine ( a ) the circulation O f F around the curve K and ( b ) t h e j u x of F over K where K has the counterclockwise orientation.
46. F ( x , y ) = (3x2 + y , x + 5y2); K is the rectangle with 51. F ( x , y ) = ( - y / ( x 2 + y 2 ) ,x / ( x 2 + y 2 ) ) ; K is the circle
vertices (0,0), (2, 0), ( 2 , 3), (0,3). ( X - 16) + ( J - 9) = 1.
47. F ( x . y ) = ( 2 x --y, x + 2y); K is the rectangle with ver- 52. F ( x , y ) = ( y 3 > x 3 ) ;K is the unit circle centered at the
tices ( - 1 , O ) . (l,O), (1,2), (-1,2). origin.
48. F ( x , y ) = ( y 2 - x 2 %x y ) ; K is the rectangle with vertices 53. F ( x , y ) = ( e x siny, e x cosy); K is the unit circle centered
(-1, 11, (2. I ) , (2, 31, (-1, 3 ) . at the origin.
49. +
F ( x , y ) = (xy2 x 2 . y 3 - x ) ; K is the rectangle with 54. F ( x , y ) = ( x , y ) ; K is the unit circle centered a t the
vertices (2, 5 ) , (5, 5), (5, i ) ,( 2 , 7 ) . origin.
50. F ( x , y ) = (x?, 1 - 2 3 4 ; K is the rectangle with verti- 55. F ( x , y ) = (ex siny, ex cosy); K is the triangle with verti-
ces (-3. l ) , (2. l ) , (2,3), ( - 3 , 3 ) . ces (O,O), (1, 11, (0,1).

I n Exercises 56 to 60 use Greens Theorem to evaluate the given line integral.


x dx : C is the closed curve ob- 57. 4 x2
( x - 1) dx
- 2x +
1 +y*
Y 4
x* - 2x + 1 +y2
. C is the el-
+
tained by traversing y 2 = x +
16 counterclock- +
lipse ( x - 1)*/16 y2/9 = 1 with the counterclockwise
wise from (0,4) to ( 0 , -4) and then counter- orientation.
clockwise along the ellipse x 2 / 9 + j 2 / 1 6 = 1.
SECTION 21.5: APPLICATIONS OF GREENS THEOREM: DIVERGENCE AND CURL 765

*. (x -
(X
.)2
- a ) dx
+ ( y - b)* - a)*
+ (X
(v-
b)dy
+
( y - b)
. c is 60.
4 - ( y - b ) dx
+
( x - a)2 ( y - b)* -I- ( x
( x - a ) dy
- a) + ( y - b)2 C is
any simple smooth curve that encloses the point ( a , b). any simple smooth curve that encloses the point ( a , b).
-y dx ( x - 2) dy
59. C is the el-
x2 - 4 x + 4 + y 2 x -4
+ 4+y* +
lipse ( x - 2 ) , / 4 2 +y2/18 = 1 with the counterclock-
wise orientation.

Miscellaneous Exercises
1. Let D be the region inside the circle C,: x 2 + y 2 = 100 6. Letfand g have continuous partials in R, and let C be a
and outside both circles C,: ( x - 2), + y 2 = 1 and smooth closed curve in R,. Show that
C,: ( x + 2)2 + y* = 1. Suppose M , and N, are contin-
uous and equal in D.If JL(ofX V g ) * k d A= f f V g * d X
$Mdx+N&=4 and $Mdx+Ng$=5 where D is the region enclosed by C.
c3 c4

where C:, (x - 2)* + y 2 = 2, find


In Probs. 7 to 9 let C be a piecewise smooth curve (not necessarily
n
simple). T h e winding number of C about P = (a, b ) is dejned
$ Mdx+Ndy
c, 1 - ( y - b ) dx ( x - a) 4
5 be the simply connected region consisting of the w =-J ( x - a)2 + ( y - b),
2. Let 277 + (x - + ( y - b)
x;v plane with the negative x axis and the origin re-
moved, and let F ( x , y ) = ( - y / ( x 2 + y 2 ) :x / ( x 2 + y 2 ) ) . 7. If C is a simple curve, show that the winding number of
C about P is either 0, 1, or - 1.
Show that the function in the discussion followitg
(21.3.7), page 750, satisfies V f ( x , y ) = F ( x , y ) on all ofD. 8. Use Exercise 7 to give a precise definition of what it
+
3. Show that Sc x 2 dx y x 4 dy = 0 , where C is any ellipse
means for a simple smooth curve to have a counter-
clockwise or clockwise orientation.
centered a t the origin. Is this integral zero over every
piecewise smooth closed curve that encircles the origin? 9. What is the winding number of the curve
4. Let C be a smooth closed curve and let D be the region
C: (cos t , sin t ) , 0 5 t 5 2nn?
enclosed by C. I f f h a s continuous second partials, show 10. A spring has the shape of the circular helix
that (cos t , sin t , t ) , 0 5 t 5 277. Find the center of mass of
this spring if the density a t any point is proportional to
the square of the distance from the origin. (Hint: First
define what J c f ( x , y , z ) ds means, where f is a scalar
where n is the unit outer normal to C. function and s is the arc length along C.)
5. Let f and g have continuous first partials in R,. Show
that
J [ f o g + g of1 dX =0

for every smooth closed curve in R,.

SELF-TEST
1. +
Evaluate Jc(x2 - y 2 )dx x dy, where C is the counterclockwise oriented
curve obtained by going from (- 4,O) to (0, - 16) alongy = x 2 - 16 and
then along a straight line to (16,O).
2. +
Use Greens Theorem to evaluateJK(4xy - x 3 ) dx ( 2 x y 2 )dy, where K+
is the counterclockwise boundary of the region bounded by the curves
y = x2 a n d y = x .
3. Use the Fundamental Theorem for Line Integrals to evaluate
+
Jc(y - x 2 ) dx ( x + y 2 )dy where C: ( 2 t +
1, t2 - 6 t ) , 0 5 t 5 1.
4. Determine theflux of the vector field F(x,y) = ( 2 - y 3 , 2y2 - x4)) over
K , where K is the counterclockwise boundary of the region bounded by the
curves y = 9 - x 2 and y = 0.
5. Determine the circulation of the vector field F(x,y) = ( y - x4, 2x + y 3 )
around the counterclockwise oriented cardioid r = 1 cos8. +
Appendix A
AN INTRODUCTION TO
DIFFERENTIAL EQUATIONS

CONTESTS Virtually every discipline that uses mathematical models


A . l First Order Differential Equations 767 expresses important quantitative relationships in eauations
A.2 Second Order Differential Equations 775 involving functions and derivatives, that is, differential equations.
We saw examples of simple differential equations in Chaps. 6
(objects thrown vertically and being acted upon by the force of
gravity), 8 (natural growth and decay of populations, radioactive
elements, and money) and 18 (projectile problems). This Appendix
provides a brief glimpse of the methods used to solve
differential equations-to determine functions that satisfy the
differential equations. Extensions of some of these methods
appear in the exercises.

767
7638 APPENDIX A: DIFFERENTIAL EQUATIONS

Section A. 1: First Order Differential Equations

Objectives: INTRODUCTION
Recognize and solve a first order
Any equation involving functions and their derivatives is called a dzfferential equation. In
differential equation that is
(a) separable, ( b ) homogeneous,
particular, if F is a function of n +
2 variables, then
(c) exact, or ( d ) linear. (1) F(x,y,y,y, . . . J) = 0
is called an ordinary differential equation of order n. A function f is said to be a solution
of (1) if
F(.,f(,f(x),f(.), . . , P W )= 0
for all x in some interval I . For example, the equation
( 2 ) y + y = 0
is a second order ordinary differential equation. One can easily check thatf(x) = sin x
is a solution.
If g ( x , y ) = 0 defines implicitly some function that solves a differential equation,
then we refer to g ( x , y ) = 0 as an implicit solution of the equation. For example,
x2 + y 2 = 9 is a n implicit solution of the differential equation
y y + xy = 0
since upon differentiating x 2 + y 2 = 9 implicitly, we have

and so

y y + 9 = y z -- ( ;)+.= -xy+xy=o

It can be shown that any solution of Eq. (2) is of the form


(3) y = c1 sin x + c2 cos x
for suitably chosen constants c1 and c2. Conversely, everyy of the form of (3) is a
solution of Eq. (2). For this reason (3) is called the general solution of the differential
equation. In most instances the general solution of a n nth order differential equation
will involve n arbitrary constants cl, c2, . . . , 6%. If we assign n conditions t o y and its
derivatives, then we can select from all solutions the particular solution that satisfies those
conditions. Specifically, if A,, A , , . . . , An-l are fixed constants, then the problem of
finding the particular solution of
F(x,y,y, . . . ,y(n))= 0
that satisfies the initial conditions

y(x0) = A,,y(xo) = A , , .. , ,y(-l)(xo)= An-l

is called a n initial value problem.

FIRST ORDER DIFFERENTIAL EQGATIONS


We shall limit our attention to first order equations, which can be written in derivative
form as

or in dzfferential form as
M ( x , y ) dx + N ( x , y )dy = 0
APPENDIX A: DIFFERENTIAL EQUATIONS 769

SEPARABLE DIFFERENTIAL EQUATIONS

A. 1.1 Definition A first order differential equation is said to be separable if it can be written
in the form
(4) M ( x ) dx + N ( y )dy = 0
where M and N are continuous.
To solve (4) we first write it in the equivalent form
M(x) + N(y)y = 0
Now i f y = f ( x ) is a solution, then

M ( x ) + N(f(x))f(x) = 0
which, upon integrating, yields
JM(x) dx + JN(.f(x))f(x) dx = C
or equivalently,

A.l.2 J M ( x ) dx + J N ( y )4 = C

Example 1 Solve the differential equation


(y2+y)dx + xdy = 0
Solution Dividing through by x ( y 2 + y ) , we can write the equation in the equivalent form

Thus

Now

and

If we denote C by In lkl, then our solution can be written in the form

so

and

or equivalently,
xy = k ( y + 1)

Example 2 A tank contains 30 gal of brine in which 5 lb of salt have been dissolved. Pure water
flows in the tank a t the rate of 2 gal/min. T h e mixture, which is kept uniform by
stirring, runs out of the tank a t the same rate as the pure water flows in. How much salt
remains in the tank after 10 min?
770 APPENDIX A: DIFFERENTIAL EQUATIONS

Solution Let x denote the amount of salt in the tank a t time t . Since the rate of inflow is the same
as the rate of outflow, the tank contains 30 gal of mixture a t any time t. The concen-
tration of salt a t time t is Ib/gal, and since this mix is flowing out at the rate of 2
30
gal/min, we have
(5) -
dx = - x ( 2 ) = -LL lb/min
dt 50 25
Since the tank contained 5 lb of salt initially, we have the initial condition x ( 0 ) = 5 .
Separating the variables from ( 5 ) , we have the initial value problem
-d xf E = Odt x(0) = 5

Integrating and simplifying, we have


x = 5e-t/25

Thus after 10 min the tank contains 5 c 2 I 5 3.35 lb of salt. 0

HOMOGEXEOUS EQUATIONS

A.1.3 Definition A first order differential equation is said to be homogeneous if it can be


written in the form

To solve (6), we make the substitutiony = ux. Then


-4- = u + x - du and y=u
dx dx
so (6) becomes

u
dv
+x- = f ( u ) , or, equivalently, ~
du --
- dx
dx f(u) -
Since this latter equation is separable in the variables x and u, we can solve it by
integration and then replace u by y / x .

Example 3 Solve the differential equation


4 -- y2
_ + 2xy
dx X2

Solution We note that

Thus, lettingy = ux, we have

u + x -ddux = u 2 + 2u
or

We integrate (using partial fractions on the left side) to obtain


APPENDIX A: DIFFERENTIAL EQUATIONS 77 1

or equivalently,

--U - cx
v+l
Substituting v = y / x and simplifying, we have

y = - cx2
1 - cx
.
A function F ( x , y ) of two variables is said to be homogeneous of degree n if
F(tx, 9)= t " F ( x , y )
T h e following theorem is a simple way of identifying a homogeneous differential
equation given in differential form. An outline of the proof is in Exercise 22.

A. 1.4 Theorem T h e first order differential equation


M ( x , y )dx + N ( x , y )dy = 0
is homogeneous if and only if the functions M and N are both homogeneous of the
same degree.

PROGRESS TEST 1

Classify and solve the following differential equations:


1. 3xy dx + +
( 2 4)dy = 0 2. (y +d v ) d x - xdy = 0

EXACT DIFFERENTIAL EQUATIONS

A. 1.5 Definition T h e differential equation

(7) M ( x , y ) dx + N ( x , y ) dy = 0
is said to he exact in the region R if there exists a functionf(x,y) such thatf,(x,y) =
M ( x , y ) andf,(x,y) = N ( x , y ) for all ( x , y ) in R.

Note that the exactness of ( 7 ) is equivalent to the vector field ( M , A V ) ,being


conservative, and so by (21.3.5) and (21.3.7) we have:

A. 1.6 Theorem Let R he a simply connected region in the plane. The differential equation
M ( x , y ) dx + N ( x , y )dy = 0
is exact in R if and only if M , = N,.

If M d x + Ndy = 0 is exact, then it can be written in the equivalent form


f,dx + f,dy = 0, wherefis such that f, = M and fu = h: But f,dx + f,dy = 6 the
total differential off, and so this equation is equivalent to the equation df = 0, whose
solution isf( x , y ) = C. Thus solving an exact differential equation is exactly the same as
finding a potential function for the conservative vector field ( M ,N).

Example 4 Solve the differential equation

(3xZln(3 + y 2 ) - y 4 ) dx +
6x 5
Solution Since M , = 7 - 4y3 = N,,the equation is exact. Set
+Y
f, = 3x21n(3 + y 2 ) - y 4
772 APPENDIX A: DIFFERENTIAL EQUATIONS

Then, integrating with respect to x , we have


f ( x , y ) = x 3 ln(3 + y 2 ) - y 4 x + C(y)
Thus

Settingf, = N,, we conclude C ( y ) = 2y, so C ( y ) = y 2 . Thus


f ( x , y ) = x 3 ln(3 + y 2 ) -y4x + y 2 , and the general solution of the equation is
x31n(3 + y 2 ) - y 4 x + y 2 = C 0

FIRST ORDER LIBEAR DIFFERENTIAL EQUATIOXS

A. 1.7 Definition A first order differential equation is said to be linear if it can be written in
the form

where P and Q are continuous.


If in Eq. (8) Q = 0, then we can write the equation in the separable form

which can be solved to give


lny + J P ( x )dx = C
or, equivalently,
eJP(x)d2 =c

Note that, by the Product and Chain Rules,

(9) D , ( ~e JP ( x ) d x ) = dv
dx
elP(c.)da: + P ( ~ e JP
Z ( x~) d x

Thus if Q(x) is not the zero function, we can multiply (8) through by to obtain

2 eJP(x)dx + eJP(x) d x = Q(x) e l P ( r )d x


dx

which from (9) is equivalent to


D,(y ,JP(r)d $ ) = Q(x) e m ) dx
Integrating, we obtain,
eJP(x) d x = J Q(.) eJP(x) dx
+C
or, equivalently,

A. 1.8 y = e-JP(x)dx [J~ ( x e S) P ( x ) d x + C]

T h e next example, although similar to Example 2, results in a linear rather than a


separable differential equation.

Example 5 A tank contains 60 gal of brine in which 15 lb of salt have been dissolved. Brine
containing 2 lb of salt per gallon flows into the tank at the rate of 4 gal/min. The
mixture, which is kept uniform by stirring, simultaneously flows out of the tank at the
rate of 2 gal/min. How much salt is in the tank after 30 min?
Solution As in Example 2, we let x ( t ) denote the amount of salt in the tank a t time t. The rate at
which salt is flowing into the tank is

(2)
(g) = 8 lb/min
APPENDIX A: DIFFERENTIAL EQUATIONS 773

The concentration of salt in the tank at time t is x ( t ) divided by 60 + (4 - 2)t: and so


the rate at which salt is flowing out of the tank is
2 gal

dx
Now -, the rate of change of the amount of salt in the tank at time t, is rate in minus
dt
rate out, so

or, equivalently,

dx
-
dt
+ (-)x30 1+ t =8

This equation is linear, with P ( t ) = -and Q(t) = 8 . Now J P ( t ) dt = ln(30


30 + t
+ t),
+
so e l p ( t ) d t = e1n(30+t)= 30 t. Thus the general solution is

x(t)(30 + t ) = J 8 ( 3 0 + t ) dt + C = 8 ( 3 0 2+ t) +C

or
x ( t ) = 4(30 + t) + 0

Since the initial amount of salt in the tank was 15 lb, we have

15 = x ( 0 ) = 120
c
+-
30
so C = -3150 and the amount of salt in the tank at time t is

X ( t ) = 120 + 4t - 303150
+t
~

3150 = 187.5 lb.


Hence x ( 3 0 ) = 240 - - 0
60

PROGRESS TEST 2

Identify and solve the following differential equations.


1. dy/dx + (tan x ) y = cos x
2. ( 2 x tany - y sec x tan x + 2x) dx + ( x 2 sec2y - sec x ) dy = 0

SECTIOS A. 1 EXERCISES

In Exerczses I to I O j n d the general solution of the given diferential equation.


1. xcscydx+(x2-x-2)dy=0 2. +
( y 2 9 ) dx +
(x +
5 ) dy = O
3. (v +y + +
x 1 ) d x +ydy = 0 4. (x + y ) d x - xdy = 0
5. ( x eYIx - y ) dx + x dy = 0 6. ( 3 ~ -3 6~~ - y ) dx + ( 4 ~ +3 18y
~ - x ) dy = 0
7. (& -y3cosx
1dx + 8. y exu dx + ( x eEy + 1) dy = 0
(ln(1 + x 2 ) - 3y2 sin x + 1) dy = 0
9. dy/dx + 2xy/(x2 + 1 ) = 4 x 10. dy/dx + 2y = x 2 e-

In Exercises I 1 to 18 solve the given initial value problem.


11. ( 2 x cosy - y 2 cos x)dx + (1 - 2y sinx - x 2 siny)dy = 0 12. (3x2y2 - 3 x 2 ) dx + (2yx3 + 4y3) dy = 0 [y(O)= 31
[Y(O) = 21
774 APPENDIX A, DIFFERENTIAL EQUATIONS

13. ( yex + 2ex + y 2 )dx + ( 8 + 2xg) dy = 0 [ ~ ( o =) 11 14.

13. 2xy dx + ( x 2 + 9)d y = 0 [y(O)= 41 16. &/dx = -


2xy [y(O)= 13
x 2 -y2

17. (x tan(y/x) + y ) dx - x dy = 0 [y(2/7i) = 11 18. -


dx
+
d y -
Y =-
x2
1
x 2 [x(l) = 2e]

19. If p(x,y) M ( x , y ) dx +6 ( x , y )N ( x . y ) 4 = 0 is exact, 20. 4 P ( x ) y = Q(x)gn


A differential equation of the form - +
dx
then + ( x , y ) is called a n integratzngfactor for the differen-
is called a Bernoulli equation.
tial equation M ( x , y ) dx + N ( x , y )dy = 0. Solve each of
( a ) Show that if n = 0 or n = 1, then the Bernoulli
the following by first showing + is a n integrating fac-
equation is linear.
tor and then solving the equivalent equation
( b ) Show that the substitution v reduces the
+
+Mdx ONdy = 0.
Bernoulli equation to the linear equation
( a ) (2xy4 - y 3 ) dx + ( 3 . ~ - +
5 ~2xy2 4y4) 4 = 0
[+(.>V) =FI dv
;E; + 1 - n F ( x 1 1 v = 1 - n)Q(x)l
+
( b ) (x + y ) dx 4 = 0 [ + ( x , y )= ex]
(c) (2xg2 + y ) dx +(2g3 - x) dy = 0 [ Q ( ~ , Y=Y-])
21. Use the result of Exercise 20(b) to solve 22. Prove Theorem (A.l.4). (Hint: If M and N are both
homogeneous of degree n , then let t = l/x).
(a)
4 +Y =
- XY4 (b)
d y Y
-+G=?
dx dx Y
23. A large tank contains 50 gal of brine in which 8 lb of salt 24. By a chemical reaction, chemical A is converted into
have been dissolved. Brine containing 3 lb of dissolved chemical B . The rate at which A converts into B is
salt per gallon is flowing into the tank at the rate of 5 proportional to the amount of A present a t any time.
gal/min. T h e mixture, which is kept uniform by stirring, After 10 min, 60 g of A remains; a t the end of 40 min,
runs out of the tank a t the rate of 3 gal/min. How much 15 g of A remains. How many grams of A were present
salt is in the tank after 17 min? initially? How much of A will remain after 1 h?

Seetion A.2: Second Order Linear Differential Equations

Objectives: INTRODUCTION
1. Solve a second order linear
A second order differential equatiotl is said to be linear w i t h constant coeflcients if it can be
homogeneous differential equation with
written in the form
constant coefficients.
2. Use the method of undetermined
coefficients or the method of variation of
parameter to solve a second order linear wheref(x) is continuous on some interval Z and a,, a2 are constants. Iff(.) = 0 on I ,
nonhomogeneous differential equation then the equation is called homogeneous, and we refer to
with constant coefficients.

as the associated homogeneous equation of (1). (This is a use of the term homogeneous,
different from that used for first order equations in Sec. A.l.)

We shall concentrate first on solving the homogeneous equation. The following


theorem, whose proof can be found in any differential equation text, reduces the
problem to that of finding just two solutions.

A.2.1 T h e o r e m Ify,(x) andg2(x) are solutions of (2), and ifg, is not a scalar multiple of
y 2 , then the general solution of (2) is given by

Y = ClYl + CZYZ
where c1 and c2 are arbitrary constants.

For example, one easily checks thatyl(x) = sin x andy2(x) = cos x are solutions of
y + y = 0. Since sin x and cos x are clearly not scalar multiples of one another, the
general solution is given by
y = c1 sin x + c2 cos x
APPENDIX A: DIFFERENTIAL EQUATIONS 775

THE CHARACTERISTIC EQUATIOX


A.2.2 Definition The polynomial

(3) c(m) = m 2 + alm + az


is called the characteristic polynomial of the homogeneous differential Eq. (2). The
equation c(m) = 0 is called the characteristic equation.

If c ( m ) is an irreducible quadratic, then c(m) = 0 has no real roots. Otherwise


c(m) = 0 has either a twice-repeated real root m l or it has two distinct real roots, ml and
m2. By applying the quadratic formula we can identify the three possibilities as follows:
>
(4) If u12 - 4a2 0, then c(m) = 0 has the distinct real roots.

rn, =
-a1 +-d mz =
-a, - -d
2 2
(5) If u12 - 4a2 = 0, then c ( m ) = 0 has the repeated real root m1 = -a,/2
<
(6) If u12 - 4a2 0, then c(m) is irreducible.
We shall show that the solution of (2) will depend on which case occurs. As a first
step in that direction we have:

A.2.3 Theorem If m, is a real root of the characteristic equation of (2), theny,(x) = e"1" is
the solution of (2).
Prooj Lety,(x) = emix, Theny;(x) = mlemis andy;'(x) = ml2emiS, Thusy;' + sly; +
azyl = m12emiz + almle"i' + u2emix = emi"(m12 + alml + a2) = emiOc(ml) = 0. W

If m, # m2, then certainly P i " is not a scalar multiple of emz"', and so by (A.2.1) and
(A.2.2), we have:

A.2.4 Theorem If c(m) = 0 has the distinct real roots m,,m2, then (2) has general solution
y = cle"l" + c2emz'

Example 1 Solve the differential equationy" + 2y' - 4y = 0.


Solution The characteristic equation is c(m) = m 2 + 2m - 4. Now c(m) has the distinct real roots
mI = -1 + fi and m2 = -1 - fi.Thus the general solution is
y = c1 exp( - 1 + fi)x + c2 exp( - 1 - fi)x
= e-z[cI exp fix + c2 exp - f i x ] 0

-4.2.5 Theorem If c(m) = 0 has the repeated real root m,, then (2) has general solution
y = clemi' + c2xemiX
ProoJ By (A.2.3) emir is a solution of (2), and certainly emit and x emia are not scalar
multiples of one another, Thus the result will follow from (A.2.1) ifwe can show x e"1'
is a solution of (2). Let y = x em,+. Then
y" + a,y' + a,y = 2m1e"1" + m12x eml" + al(emlz + m I x e"1z) + a2x emlx
= (m12 + alml + az)x emir + (2m, + a,) emis
= c(ml)x em>" + (2m, + a,) emi"

But c(ml) = 0, and since by (5), m, = -a,/2, we know 2m, + a, = 0. Hence


y" + a,y' + a2 = 0 rn

Example 2 Solve the differential equation

- -
d2y 6 -4+ 9 y = O
dx2 dx
776 APPENDIX A: DIFFERENTIAL EQUATIONS

Solution Since c(m) = m 2 - 6m + 9 = (rn - 3)2, the general solution is given by


y = +
~ , e 3 ~ c2x e3c 0

-4.2.6 Theorem If c(m) is irreducible, then ( 2 ) has general solution


y = eax[cl sin bx + c2 cos bx]
where a = -a,/2 and b = 4-/2.
Proof: A direct calculation shows that i f y = eax sin bx, then
+ a,y + a2 = eas[(a2 - b 2 + ala + a2) sin bx + (2ab + a,b) cos bx]
y
Now a = -a,/2, so a, = - 2 a , and hence 2ab + a,b = 2ab - 2ab = 0. Similarly,
a2 - b2 + a,a + a2 = 0

Thus eax sin bx is a solution of (2). A similar argument shows that ear cos bx is a solution,
and so by (A.2.1) the general solution is as stated. H

Example 3 Solve the differential equationy +y + y = 0.

Solution Now c ( m ) = rn2 m + +


1 is irreducible. Since al = a2 = 1, we have a = -1/2 and
b = */2, and so the general solution is

PROGRESS TEST 1

Solve the following differential equations:


1. y + 4y - 12y = 0 +
2. y - 1 0 ~ 2 5 =
~ 0 3. y + y + 3y = 0

FINDING PARTICULAR SOLUTIONS FOR A


NOSHOMOGENEOUS DIFFERENTIAL EQUATION
Since our efforts to date allow us to determine the solution of the associated homoge-
neous equation ( 2 ) more or less by inspection, the following reduces the problem of
solving (1) to that of finding any partzcular solution.

A.2.7 Theorem Lety, be any particular solution of (1) and lety, = clyl c2y2 be the +
general solution of the associated homogeneous equation. Then (1) has general solution

Y = y p +YC
Proof: Let y be any solution of (1) and let z = y - y,. Then y = y p t and +
2 + a1z +
a22 = ( y -yp) + a,(y -&) +
a,(y - y p ) = (y +
a,y +
a,y) -
(& +
a,yj, +
a 2 y p )= f ( x ) - f ( x ) = 0. Thus z is a solution of the associated homoge-
neous equation, and so t = y c for some appropriate choice of c, and c 2 . Thus
+ +
y = y, yc. Conversely, let y = y p z , where t is any solution of the associated
homogeneous equation. Then

y + a,y + a2 = ( y , + 2) + a,(y, + t)+ a,(y, + t)


= (y;: + a,y; + azy,) + + a,z +
(2 022)
=f(x) + 0=f(x)
and so any function of the form y o + y c is a solution of (1). H
APPENDIX A: DIFFERENTIAL EQUATIONS 777

Example 4 Show thaty,(x) = 2x2 + Gx + 7 is a solution of the differential equation


( 7 ) y - 3y + 2y = 4x2
and find the general solution.

Solution Since
y: - 3yL + 2 ~ =, 4 - 3 ( 4 ~+ 6) + 2(2x2 + GX + 7 )
=4 - 1 2- ~ 18 + 4x2 + 1 2 + ~ 14 = 4x2
y , is a solution of ( 7 ) . Now the associated homogeneous equation has characteristic
polynomial c(m) = m 2 - 3m +
2 = (m - l ) ( m - 2). Thusy, = cleZ +
c2eZx,and so by
(A.2.7) the general solution of ( 7 ) is
y = clef + c2e2 + 4x2 0

VARIATION OF PARAMETERS
Lety, = clyl(x) + c2y2(x)be the general solution of ( 2 ) . In the method of varzatzon of
parameters we seek a particular solution to (1) having the form

(8) Y p = W l + UZYZ
where v, and v2 are functions to be determined. Since there are two functions to be
determined, we should expect to use two conditions. T h e first, of course, is thaty, be a
solution of (1). T h e second, which helps to minimize the computations, is that

(9) 4Y1 + 4Y2 = 0


Now
Y;, = y1 + 4 Y 2 + w;+ w;= w;+ w;
and
y; = v;J; + 0;y; + u11; + u2y;
It is routine to show that

y;: + a,& + a2y; = q[y; + a,y; + Wll + U2b; + a,y; + azy2l + 4 Y ; + v;v;
But y 1 and y 2 are solutions of ( 2 ) , and SO y; a& + + a2yl = 0 and
y; +
+ sly; a,y, = 0. Hencey, is a solution of (1) if and only if
(10) 4 y ; + 4J;= f b )
Solving (9) and (10) simultaneously for v; and a;, we obtain

A.2.8

We can now integrate to determine v, and v2 and then use ( 8 ) to formy,.

Example 5 Solve the differential equation


y + 4 = csc x
Solution Since c(m) = m 2 +
1, we have a, = 0, a2 = 1, and from ( A . 2 . 6 ) a = 0 , b = 1. Thus the
associated homogeneous equation has general solution
y, = c, sin x + c2 cos x
Let y , = sin x , y 2 = cos x . Then y I y ; - y 2 y ; = (sin x ) ( -sin x) - (cos x)(cos x ) =
- (sin2x + cos2 x ) = - 1. Thus
-cosxcscx sinxcscx = - l
u; = = cotx v; =
-1 -1
778 APPENDIX A: DIFFERENTIAL EQUATIONS

Thus u1 = S c o t x dx = In lsinxl and u2 = 1 -dx = -x (we take the constants of


integration to be zero), so
yp = (In Isin X I ) sin x - x cos x
and hence the general solution of the equation is
y = c1 sin x + c2 cos x + (In Jsinxi) sin x - x cos x
= (cl + In lsin xi) sin x + ( c 2 - x) cos x 0

UNDETERMINED COEFFICIENTS
T h e method of undetermined coejicients can be applied to find a particular solution of (1)
when f ( x ) is either
(1 1) e k x , x", sin cx, cos cx
(12) a finite product of functions from (1 1)
(13) a sum of constant multiples of functions from (1 1) or (12)
Thus a typical such function f would be
(14) f ( x ) = 3x2e2' + xsin4x + 3x2 - x + 5
T h e method consists of looking for a solution to (1) that has the form

Yp = c 4%
n

i=1

where the Ai's are constant coefficients to be determined and the ui's are all distinct
functions of the form (1 1) or (12) that appear either i n f o r in any derivative o f j For
example, in (14) the ui's consist of x2e2", xe2", e2", x sin 4x, x cos 4x, sin 4x, cos 4x, x 2 , x,
1. T h e Ai's are then determined by substitutingyp into the differential equation and
equating coefficients. To be assured of solving for the Ai's any ui that is a solution of the
associated homogeneous equation must be replaced by .doi, w h e r e j is the least positive
integer such that x3ui is not a solution of the homogeneous equation.

Example 6 Solve the differential equation


Solution y" - 4 y + 3y = 2e3x + xz - 5
The associated homogeneous equation has the characteristic polynomial c(m) =
m2 - 4m + 3 = ( m - I ) ( m - 3), soyc = Glen +
c2e3x. Sincef(x) = 2e3' xz - 5 , +
the ui's are e3', x 2 , x, 1. However, e3" solves the associated homogeneous equation
(take c1 = 0, c 2 = l), so we replace it by xe3", giving usy, = Axe3" Bx2 Cx D . + + +
Thus y;i = + +
Ae3" 2Bx + +
C and 9:; = 6Ae3" 9Axe3" 2B. Therefore +
y ; - 4yL + 3yp = (9A - 12A + +
3A)xe3" (6A - 4A)e3" + +
(3B)x2 (3C - 8 B ) x +
+
(2B - 4C 3 0 ) . Since 9A - 12A +
3A = 0 , the first term drops out. Equating
coefficients, we have 2A = 2,3B = 1 , 3 C - 8B = 0,2B - 4C + 3 0 = -5, and hence
A = 1, B = 1/3, C = 8/9. D = -19/9.
Thus

yP - xe3" + -1x 2 + -x8 19


--
- 3 9 9
and the general solution is given by

y = cles + c2e3' + xe& + -31x 2 + -x89 19


-- 0
9

PROGRESS TEST 2

1. Use the method of variation of parameters to solve 2. Use the method of undetermined coefficients to solve
y" - 4y' + 4y = x2e2" y" - 4y' + 4y = eZe + 1
APPENDIX A: DIFFERENTIAL EQUATIONS 779

SECTION A.2 EXERCISES

In Exercises 1 to 1 2 j n d the general solution of the given homogeneous differential equation.


1. y - 4y = 0 2. y + y - 3y = 0 3. y - y - 30y = 0
4. y + 8y = 0 5 . y + 4y +
3y = 0 6. y - 2&y + 2y = 0
+
7. y - 8y 16y = 0 8. y - 2y - 6y = 0 9. y - 2y + 4y = 0
10. y - 12y 3% = 0+ 11. y +
2y +
8y = 0 12. y + 4y + 4y = 0

In Exercises 13 to 1 7 use the method of variation ofparameters to solve the given equation.
13. y + y = tanx 14. y + y = s e c 3 x 15. y - y = e x s i n x
16. y - 2 y + y = e X / x , x > O 17. y - J = c o s x

In Exercises I 8 to 22 use the method of undetermined coeficients to solve the given equation.
f8. y + y = x 3e- + 19. y - y = e x s i n x 20. y --y - 2y = x 2 + xe-
21. y - 2y + y = xe 22. y - y = x s i n x

R e v i e w Exercises
In Exercises I to 1 0 ( a ) identiJ, all categories among S separable, H homogeneous, E exact, L linear into which each of the dtfferential equations f a l l s
and ( b ) use any method to solve the differential equation.
1. x(y + y ) d x + y ( x 2 - 1) dy = 0 2. ( y + e x ) dx + (x + eU) dy = 0
3. x -dy+ y = x a 4. 4 - 3y = e2x
-
dx dx
5. (2x tany + 8cos x + 2 x ) dx + 6. -dy+ x y + y = x + I
(x sec2y + 2y sin x ) dy = 0 dx
7. x c o s y dy - s h y dx = 0 8. (secy + 2n lny) dx +
+
(2x secy tany x2/y) dy = 0
9. xy - y = x 3 d c 7 10. (x - y tan(y/x)) dx + (x tan(y/x)) dy = 0
In Exerczses 1 1 to 20 solve the gzven znitial value problem.
11. xy + 2y = 6 , y ( l ),= 7 12. y 2 dx - (xy +
x2) dy = O , y ( l ) = 3
13. y + ( l / x b = l / e , y ( l ) = 2 14. 2xy dx = (x2 + y 2 ) dy,y(2) = 4
15. ( y + xy) dx + xdy = O,y(l) = 3 16. (1 - c o s x ) y + y s i n x = O,y(r/4) = 1

17. 9 - y = ~ x , J J ( ~=) 3

In Exercises 2 1 to 30 determine the general solution of the given second order differential equation.
21. y + 8y = 0 22. y + 3y+y =0
23. y + 3,~ - y
=0 24. y + 2y - 48y = 0
23. y +
- 2 G y 5y = 0 26. 2, + 4y+ 8y = 0
27. y - 1 6 ~ + 6 4 ~= 0 28. y + 3y = 0
29. 4y + 7y = 0 30. 2, + 3y + 8y = 0

In Exercises 36 to 4 0 use the method of undetermined coe3cients to solve the given dtfferential equation.
36. y - y - 6y = e x s i n x 37. y + 2y + 4y = cos 4x
38. y y = x + ex + 39. y - 4y + 4y = x e Z x
40. y + y = 3 - 4 COS X
780 APPENDIX A DIFFERENTIAL EQUATIONS

In Exercises 41 to 45 use the method of variation of parameter to solve the given dzfferential equation,
41. y - 3y +
2y = e / ( e + 1) 42. y + y = tanx
+
43. y y = sec3 x 44. y - 4y + 4y = x2eZx
+
45. y y = sec x tan x
46. A large tank contains 50 gal of brine in which 25 lb of 47. A large tank contains 200 gal of brine into which 30 lb
salt have been dissolved. Pure water flows into the tank of salt have been dissolved. Brine containing 3 lb of salt
at the rate of 1.5 gal/min. T h e mixture, which is kept per gallon flows into the tank at the rate of 4 gal/min.
uniform by stirring, runs out of the tank at the rate of T h e mixture, which is kept uniform by stirring, flows
1 gal/min. After how much time will the amount of salt out of the tank at the rate of 3 gal/min. How much salt
in the tank be reduced to 20 lb? is in the tank after 20 min?
48. An object that weighs 15 lb falls from rest in a straight 49. By a chemical reaction, chemical A is converted into
line toward the earth. T h e force due to air resistance is chemical B. T h e rate at which the conversion takes place
4 v pounds, where L is the speed in feet per second. Find is proportional to the amount of A present a t any time.
the speed and distance fallen by the object after 7 s. After 2 h, 25 g of chemical A remains; after 3 h only 10 g
is left. How much of chemical A was present initially?
How much of chemical A will be left after 5 h?

Miscellaneous Exercises

1. Show that if l / N [ M , - N,] is a function of x alone 2. Show that if 1 / M [ N , - M,] is a function o f y alone
[say = p ( x ) ] , then [Say = qLJJ)l,then
A x ) M ( x , y ) dx + P(X) N ( x , y ) 4 = 0 . ( y ) M ( x , y ) dx + .(y) N ( x , y )4 = 0
is exact where ~ ( y=) e J q ( v ) d ,

In Exercises 3 to 6 use the results of Exercises I and 2 to solve the given dzfferential equation.
3. ( 2 x 2 + y ) dx (~- +
y X)4 =0 4. ( x 2 + 2y2)dx xy + 4 =0
5. (x4 + y 4 ) dx - xy3 dy = 0 6. + (2x9~+ 1 ) 4 = 0
( y 2 x + y 2 + y ) dx
dY = 0 by first d2y 4
7. Solve the differential equation x? d2y
dx
+-
dx
8. Solve the differential equation 7+ y - = 0 by first
dx dx
4 db d2y making the substitutions p = dy/dx, - d? = p -d p and
making the substitutions p = - - = -and solving dx2 dy
dx dx dx2
the resulting first order equation. solving the resulting first order equation
( c ) Letting Co and C, be arbitrary, use (ii)to show that
9. (Power Serzes Solution) Solve the differential equation ( i )
+
y xy + y = 0 using the following procedure: CO 2% ! C,
( a ) Assume the series has a power series solution of the c2n -
- (-1Y- 2%! C2n+1 = (-1)n (2n + I ) !
formy(x) = Z:=, Cnxn. Substitute this series into ( i )
and adjust indices to conclude ( d ) Conclude ( i ) has solution
w

C [ ( n + 2 ) ( n + I)C,+~+ ( n + I ) C , ] X ~= o
n=O

( b ) Using the fact that Z$=, a,Xn = 0 only if a, = 0 for


all n, arrive a t the recurrence relation

(ii)

In Exercises I 0 to I5 use a procedure similar to that in Exercise 9 to solve the following differential equations.
10. y - xy = 0 (Airys equation) 11. y xy 2y = 0 + + 12. y - xy + y = 0
13. y - xy - 2y = 0 14. y + y = O 15. y - 2xy = 0
16. Show that the solution of Exercise 14 obtained using the
power series method agrees with the solution obtained
using (A.2.6).
APPENDIX A: DIFFERENTIAL EQUATIONS 781

17. (Orthogonal Trajectories) An orthogonal trajectory to a family Use this procedure to determine the orthogonal
of plane curves F(x,y, c) = 0 is any curve that intersects trajectories of the following families:
every curve in the family at right angles. To determine
17-1:y = cx2
the orthogonal trajectories, proceed as follows:
17-2: x 2 + y 2 = c*
( a ) Differentiate the equation F(x,y, c) = 0 implicitly
17-3:y = cx3
with respect to x .
( b ) Eliminate the parameter c from the two equations in 18. A heated metal plate has temperature distribution
( a ) to arrive at a differential equation 4ld.x = T ( x , y )= 10 - x 2 - 2y2. By (19.6.6) and (19.7.1) a
f(X>Y1. heat-seeking particle will travel along a trajectory or-
(c) Since two curves are perpendicular if and only if thogonal to the isotherms T ( x , y ) = C. Determine the
their slopes are negative reciprocals, the general so- trajectory a heat-seeking particle starting at ( - 1, 3)
lution of will travel along to get to the origin (compare with Sec.
19.6, Example 3).

will yield the orthogonal trajectories.

APPENDIX A SELF-TEST
1. Solve the following first order differential equations:
( a ) (XJ + 2 ~ dx) +
3 ( x 2 - 2 ) dy = 0
( b ) (sin? +2 x cosy) dx + ( x sin 2y - x 2 siny) dy = 0

(c)
4 + (tan X)J = C O S ~x
2. Solve the initial value problem

3. Solve the differential equationy - 4y = x 2 + ez


4. Solve the initial value problem y y = 2 csc x ; +
y(7;/2) = O,y(r/2) = 2
I
Appendix B
1 TABLES

I I

782
TABLE 1 TRIGONOMETRIC FUNCTIONS 783

Table 1 Trigonometric Functions

Degrees , Radians 1 Sin cos Tan cot 1


0 0.0000 0.0000 1.0000 0.0000 1.5708 90
1 0.0175 0.0175 0.9998 0.0173 57.290 1.5533 89
2 0.0349 0.0349 0.9994 0.0349 28.636 1.5359 88
3 0.0524 0.0523 0.9986 0.0524 19.081 1.5184 87
4 0.0698 0.0698 0.9976 0.0699 14.301 1.5010 86
5 0.0873 0.0872 0.9962 0.0875 11.430 1.4835 85
6 0.1047 0.1045 0.9945 0.1051 9.5144 1.4661 84
7 0.1222 0,1219 0.9925 0.1228 8.1443 1.4486 83
8 0.1396 0.1392 0.9903 0.1405 7.1154 1.4312 82
9 0.1571 0.1564 0.9877 0.1584 6.3138 1.4137 81
10 0.1745 0.1736 0.9848 0.1763 5.6713 1.3963 80
11 0,1920 0.1908 0.9816 0.1944 5.1446 1.3788 79
12 0.2094 0.2079 0.9781 0.2126 4.7046 1.3614 78
13 0.2269 0.2250 0.9744 0.2309 4.3315 1.3439 77
14 0.2443 0.2419 0.9703 0.2493 4.0108 1.3263 76
15 0.2618 0.2588 0.9659 0.2679 3.7321 1.3090 75
16 0.2793 0.2756 0.9613 0.2867 3.4874 1.2915 74
17 0.2967 0.2924 0.9563 0.3057 3.2709 1.2741 73
18 0.3142 0,3090 0.95 11 0.3249 3.0777 1.2566 72
19 0.3316 0.3256 0.9455 0.3443 2.9042 1.2392 71
20 0.3491 0.3420 0.9397 0.3640 2.7475 1.2217 70
21 0.3665 0.3584 0.9336 0.3839 2.605 1 1.2043 69
22 0.3840 0.3746 0.9272 0.4040 2.4751 1.1868 68
23 0.4014 0.3907 0.9205 0.4245 2.3559 1.1694 67
24 0.4189 0.4067 0.9135 0.4452 2.2460 1.1519 66
25 0.4363 0.4226 0.9063 0.4663 2.1445 1.1345 65
26 0.4538 0.4384 0.8988 0.4877 2.0503 1.1170 64
27 0.4712 0.4540 0.8910 0.5099 1.9626 1.0996 63
28 0.4887 0.4695 0.8829 0.5317 1.8807 1.0821 62
29 0.5061 0.4848 0.8746 0.5543 1.8040 1.0647 61
30 0.5236 0.5000 0.8660 0.5774 1.7321 1.0472 60
31 0.541 1 0.5150 0.8572 0.6009 1.6643 1.0297 59
32 0.5585 0.5299 0.8480 0.6249 1.6003 1.0123 58
33 0.5760 0.5446 0.8387 0.6494 1.5399 0.9948 57
34 0.5934 0.5592 0.8290 0.6745 1.4826 0.9774 56
35 0.6109 0.5736 0.8192 0.7002 1.4281 0.9599 55
36 0.6283 0.5878 0.8090 0.7265 1.3764 0.9425 54
37 0.6458 0.6018 0.7986 0.7536 1.3270 0.9250 53
38 0.6632 0.6157 0.7880 0.7813 1.2799 0.9076 52
39 0.6807 0.6293 0.7771 0.8098 1.2349 0.8901 51
40 0.6981 0.6428 0.7660 0.8391 1.1918 0.8727 50
41 0.7156 0.6561 0.7547 0.8693 1.1504 0.8552 49
42 0.7330 0.6691 0.7431 0.9004 1.1106 0.8378 48
43 0.7505 0.6820 0.7314 0.9325 1.0724 0.8203 47
44 0.7679 0.6947 0.7193 0.9657 1.0355 0.8029 46
45 0.7854 0,7071 0.7170 1.0000 1.0000 0.7854 45
I I I
cos Sin cot Tan 1 Radians 1 Degrees 1
784 TABLE 2 EXPONENTIAL FUNCTIONS

Table 2 Exponential Functions

X ex ecX

0.00 1.0000 1.oooooo 0.50 1.6487 0.60653 1


0.01 1.0101 0.990050 0.51 1.6653 ,600496
0.02 1.0202 ,980199 0.52 1.6820 ,594521
0.03 1.0305 ,970446 0.53 1.6989 ,588605
0.04 1.0408 ,960789 0.54 1.7160 ,582748
0.05 1.0513 0.95 1229 0.55 1.7333 0.576950
0.06 1.0618 ,941765 0.56 1.7507 .57 1209
0.07 1.0725 ,932394 0.57 1.7683 ,565525
0.08 1.0833 ,923116 0.58 1.7860 ,559898
0.09 1.0942 .9 13931 0.59 1.8040 ,554327
0.10 1.1052 0.904837 0.60 1.8221 0.548812
0.11 1.1163 ,895834 0.61 1.8404 ,543351
0.12 1.1275 ,886920 0.62 1.8589 ,537944
0.13 1.1388 ,878095 0.63 1.8776 ,532592
0.14 1.1503 ,869358 0.64 1.8965 ,527292
0.15 1.1618 0.860708 0.65 1.9155 0.522046
0.16 1.1735 ,852144 0.66 1.9348 .5 16851
0.17 1.1853 ,843665 0.67 1.9542 .5 11709
0.18 1.1972 ,835270 0.68 1.9739 ,506617
0.19 1.2092 ,826959 0.69 1.9937 ,501576
0.20 1.2214 0.818731 0.70 2.0138 0.496585
0.21 1.2337 .810584 0.71 2.0340 ,491644
0.22 1.2461 ,802519 0.72 2.0544 ,486752
0.23 1.2586 ,794534 0.73 2.0751 ,481909
0.24 1.2712 ,786628 0.74 2.0959 ,477114
0.25 1.2840 0.7 7880 1 0.75 2.1170 0.472367
0.26 1.2969 .77 1052 0.76 2.1383 ,467666
0.27 1.3100 ,763379 0.77 2.1598 ,463013
0.28 1.3231 ,755784 0.78 2.1815 ,458406
0.29 1.3364 ,748264 0.79 2.2034 ,453845
0.30 1.3499 0.7408 18 0.80 2.2255 0.449329
0.31 1.3634 ,733447 0.81 2.2479 ,444858
0.32 1.3771 ,726149 0.82 2.2705 ,440432
0.33 1.3910 ,718924 0.83 2.2933 ,436049
0.34 1.4049 ,711770 0.84 2.3164 ,431711
0.35 1.4191 0,704688 0.85 2.3396 0.427415
0.36 1.4333 ,697676 0.86 2.3632 ,423162
0.37 1.4477 ,690734 0.87 2.3869 ,418952
0.38 1.4623 ,683861 0.88 2.4109 ,414783
0.39 1.4770 .67705 7 0.89 2.4351 ,410656
0.40 1.4918 0.670320 0.90 2.4596 0.406570
0.41 1.5068 ,663650 0.91 2.4843 ,402524
0.42 1.5220 ,657047 0.92 2.5093 ,398519
0.43 1.5373 ,650509 0.93 2.5345 ,394554
0.44 1.5527 ,644036 0.94 2.5600 ,390628
0.45 1.5683 0.637628 0.95 2.5857 0.386741
0.46 1.5841 ,631284 0.96 2.6117 ,382893
0.47 1.6000 ,625002 0.97 2.6379 ,379083
0.48 1.6161 ,618783 0.98 2.6645 ,375311
0.49 1.6323 ,612626 0.99 2.6912 ,371577
0.50 1.6487 0.60653 1 1.00 2.7183 0.367879
TABLE 2 EXPONENTIAL FUNCTIONS 785

Table 2 Exponential Functions (Continued)

X ex e-x I x ex

1.00
1.01
2.7183
2.7456
0.367879
,364219
1.50
1.51
4.4817
4.5267
0.223 130
,220910
'
~

1.02 2.7732 ,360595 1.52 4.5722 ,218712 1


1.03 2.801 1 ,357007 1.53 4.6182 ,216536
1.04 2.8292 ,353455 1.54 4.6646 ,214381 1
1.05 2.8577 0.349938 1.55 4.7115 0.2I2248
1.06 2.8864 ,346456 1.56 4.7588 , 2 10136
1.07 2.9154 ,343009 1.57 4.8066 ,208045
1.08 2.9447 ,339596 1.58 4.8550 ,205975
1.09 2.9743 ,3362 I 6 1.59 4.9037 ,203926 '
1.10 3.0042 0.332871 1.60 4.9530 0.201897 1
1.11 3.0344 ,329559 1.61 5.0028
1.12 3.0649 ,326280 1.62 5,0531 ,197899
1.13 3.0957 ,323033 1.63 5,1039 ,195930 I
1.14 3.1268 .319819 1.64 5.1552 ,193980
1.15 3.1582 0.316637 1.65 5.2070
1.16 3.1899 ,313486 1.66 5.2593 ,190139
1.17 3.2220 ,310367 1.67 5.3122 ,188247 1
1.18 3.2544 ,307279 1.68 5.3656 ,186374
1.19 3.2871 ,30422 1 1.69 5.4195
1.20 3.3201 0.301 194 1.70 5.4739 0.182684
1.21 3.3535 ,298197 !.71 5,5290 ,180866 I
1.22 3.3872 ,293230 1.72 5.5845 ,179066
1.23 3.4212 ,292293 1.73 5.6407 ,177284 1
1.24 3.4556 ,289384 1.74 5,6973 ,175520 1

1
1.25 3.4903 0,286505 1.75 5.7546 0.173774 1
1.26 3.5254 ,283654 1.76 5.8124 ,172045
1.27 3.5609 ,280832 1.77 5.8709 ,170333 I
1.28 3.5966 ,278037 1.78 5.9299 ,168638 '
1.29 3.6328 ,275271 1.79 5.9895 ,166960
1.30 3.6693 0.272532 1.80 6.0496 0.165299
1.31 3.7062 ,269820 1.81 6.1104 ,163654
1.32 3.7434 ,267135 1.82 6.1719 ,162026
1.33 3.7810 ,264477 1.83 6.2339 ,160414
1.34 3.8190 ,261846 1.84 6.2965 ,158817
1.35 3.8574 0.259240 1.85 6.3598 0.157237
1.36 3.8962 ,256661 1.86 6.4237 ,155673
1.37 3.9354 ,254107 1.87 6.4483 ,154124
1.38 3.9749 ,251579 1.88 6.5535 ,152590
1.39 4.0149 ,249075 1.89 6.6194 ,151072
1.40 4.0552 0.246597 1.90 6.6859 0.149569
1.41 4.0960 ,244143 1.91 6.7531 ,148080
1.42 4.1371 ,2417 14 1.92 6.8210 ,146607
1.43 4.1787 ,239309 1.93 6.8895 ,145148
1.44 4.2207 ,236928 1.94 6.9588 ,143704
1.45 4.2631 0.234570 1.95 7.0287 0.142274
1.46 4.3060 ,232236 1.96 7.0993 ,140858
1.47 4.3492 ,229925 1.97 7.1707 ,139457 ~

1.48 4.3929 ,227638 1.98 7.2427 ,138069


1.49 4.4371 ,225373 1.99 7.3155
1.50 4.48 17 0.223 130 2.00 7.3891 0.135335
786 TABLE 2 EXPONENTIAL FUNCTIONS

Table 2 Exponential Functions (Continued)

I X ex e-= X ex e-x ~

I
2.00 7.3891 0.135335 2.50 12.182 0.082085
2.01 7.4633 ,133989 2.5 1 12.305 ,081268 1
2.02 7.5383 ,132655 2.52 12.429 ,080460 1
2.03 7.6141 ,131336 2.53 12.554 ,079659 I
2.04
2.05
2.06
7.6906
7.7679
7.8460
,130029
0.128735
,127454
2.54
2.55
2.56
12.680
12.807
12.936
0.078082
.078866
,077305
i
2.07 7.9248 ,126186 2.57 13.066 .0 765 36
2.08 8.0045 ,124930 2.58 13.197 ,075774
2.09 8.0849 ,123687 2.59 13.330 ,075020 1

2.10
2.11
8.1662
8.2482
0.122456
,121238
2.60
2.61
13.464
13.599
0.074274
,073535
11
2.12 8.3311 ,120032 2.62 13.736 ,072803
2.13 8.4149 ,118837 2.63 13.874
2.14 8.4994 . 1 17655 2.64 14.013 ,071361
2.15 8.5849 0.116484 2.65 14.154 0.070651
2.16 8.6711 ,115325 2.66 14.296 ,069948
2.17 8.7583 ,114178 2.67 14.440 ,069252
2.18 8.8463 ,113042 2.68 14.585 ,068563
2.19 8.9352 ,111917 2.69 14.732 ,067881
2.20 9.0250 0.110803 2.70 14.880 0.067206
2.21 9.1157 ,109701 2.71 15.029 ,066537
2.22 9.2073 ,108609 2.72 15.180 ,065875
2.23 9.2999 ,107528 2.73 15.333 ,065219
2.24 9.3933 ,106459 2.74 15.487 ,064570
2.25 9.4877 0.105399 2.75 15.643 0.063928
2.26 9.5831 ,104350 2.76 15.800 ,063292
2.27 9.6794 ,103312 2.77 15.959 ,062662
2.28 9.7767 ,102284 2.78 16.119 ,062039
2.29 9.8749 ,101266 2.79 16.281 ,061421
2.30 9.9742 0.100259 2.80 16.445 0.0608 10
2.31 10.074 ,099261 2.81 16.610 ,060205
2.32 10.176 ,098274 2.82 16.777 ,059606
2.33 10.278 ,097296 2.83 16.945 ,059013
2.34 10.381 ,096328 2.84 17.116 ,058426
2.35 10.486 0.095369 2.85 17.288 0.057844
2.36 10.591 ,094420 2.86 17.462 ,057269
2.37 10.697 ,093481 2.87 17.637 ,056699
2.38 10.805 ,092551 2.88 17.814 ,056135
2.39 10.913 ,091630 2.89 17.993 ,055576
2.40 11.023 0.090718 2.90 18.174 0.055023
2.41 11.134 ,089815 2.91 18.357 ,054476
2.42 11.246 ,088922 2.92 18.541 .053934
2.43 11.359 ,088037 2.93 18.728 ,053397
2.44 11.473 ,087161 2.94 18.916 ,052866
2.45 11.588 0.086294 2.95 19.106 0.052340
2.46 11.705 ,085435 2.96 19.298 .05 1819
2.47 11.822 ,084585 2.97 19.492 ,051303
2.48 11.941 ,083743 2.98 19.688 ,050793
2.49 12.061 ,082910 2.99 19.886 ,050287
2.50 12.182 0.082085 3.00 20.086 0.049787
TABLE 2 EXPONENTIAL FUNCTIONS 787

Table 2 Exponential Functions (Continued)

I X
ex ec X ez e-

3.00 20.086 0.049787 3.50 33.115 0.030197


3.01 20.287 ,049292 3.51 33.448 ,029897
3.02 20.491 ,048801 3.52 33.784 ,029599
3.03 20.697 ,048316 3.53 34.124 ,029305
3.04 20.905 ,047835 3.54 34.467 ,029013
3.05 21.115 0.047359 3.55 34.813 0.02 8 72 5
3.06 21.328 ,046888 3.56 35.163 ,028439
3.07 21.542 ,046421 3.57 35.517 ,028156
3.08 21.758 ,045959 3.58 35.874 ,027876
3.09 21.977 ,043502 3.59 36.234 ,027598
3.10 22.198 0.045049 3.60 36.598 0.027324
3.11 22.421 ,044601 3.61 36.966 ,027052
3.12 22.646 ,044157 3.62 37.338 ,026783
3.13 22.874 ,043718 3.63 37.713 ,026516
3.14 23.104 ,043283 3.64 38.092 ,026252
3.15 23.336 0.042852 3.65 38.475 0.025991
3.16 23.571 ,042426 3.66 38.861 ,025733
3.17 23.807 ,042004 3.67 39.252 ,025476
3.18 24.047 ,041586 3.68 39.646 ,025223
3.19 24.288 ,041172 3.69 40.045 ,024972
3.20 24.533 0.040764 3.70 40.447 0.024724
3.21 24.779 ,040357 3.71 40.854 ,024478
3.22 25.028 ,039955 3.72 41.264 ,024234
3.23 25.280 ,039537 3.73 41.679 ,023993
3.24 25.534 ,039164 3.74 42.098 ,023754
3.25 25.790 0.038774 3.75 42.521 0.023518
3.26 26.050 ,038388 3.76 42.948 ,023284
3.27 26.311 ,038006 3.77 43.380 ,023052
3.28 26.576 ,037628 3.78 43.816 ,022823
3.29 26.843 ,037254 3.79 44.256 ,022596
3.30 27.113 0.036883 3.80 44.701 0.022371
3.31 27.385 ,036516 3.81 45.150 ,022148
3.32 27.660 ,036153 3.82 45.604 ,021928
3.33 27.938 ,035793 3.83 46.063 ,021710
3.34 28.219 ,035437 3.84 46.525 ,021494
3.35 28.503 0.035084 3.85 46.993 0.02 1280
3.36 28.789 ,034735 3.86 47.465 ,021068
3.37 29.097 ,034390 3.87 47.942 ,020858
3.38 29.371 ,034047 3.88 48.424 ,020651
3.39 29.666 ,033709 3.89 48.91 1 ,020445
3.40 29.964 0.033373 3.90 49.402 0.020242
3.41 30.265 ,033041 3.91 49.899 ,02004 1
3.42 30.569 ,032712 3.92 50.400 .O19840
3.43 30.877 ,032387 3.93 50.907 ,019644
3.44 31.187 ,032065 3.94 51.419 ,019448
3.45 31.500 0.031746 3.95 51.935 0.019255
3.46 31.817 ,031430 3.96 52.457 .O19063
3.47 32.137 ,0311 1 7 3.97 52.985 ,018873
3.48 32.460 ,030807 3.98 53.517 ,018686
3.49 32.786 ,030501 3.99 54.055 .018500
3.50 33.115 0.030197 4.00 54.598 0.018316
788 TABLE 2 EXPONENTIAL FUNCTIONS

Table 2 Exponential Functions (Continued)

X ex e-"

4.00 54.598 0.0183 16 4.50 90.017 0.01 1109


4.01 55.147 ,018133 4.51 90.922 .o 10998
4.02 55.701 ,017953 4.52 91.836 .o10889
4.03 56.261 ,017774 4.53 92.759 ,010781
4.04 56.826 ,017597 4.54 93.691 ,010673
4.05 57.397 0.01 7422 4.55 94.632 0.010567
4.06 57.974 ,017249 4.56 95.583 ,010462
4.07 58.577 ,017077 4.57 96.544 ,010358
4.08 59.145 ,016907 4.58 97.514 ,010255
4.09 59.740 ,016739 4.59 98.494 ,010153
4.10 60.340 0.0 16573 4.60 99.484 0.010052
4.11 60.947 ,O 16408 4.61 100.48 ,009952
4.12 61.559 ,016245 4.62 101.49 ,009853
4.13 62.178 .0 16083 4.63 102.51 ,009735
4.14 62.803 ,015923 4.64 103.54 ,009658
4.15 63.434 0.015764 4.65 104.58 0.009562
4.16 64.072 ,015608 4.66 105.64 ,009466
4.17 64.715 ,O 15452 4.67 106.70 ,009372
4.18 65.366 ,015299 4.68 107.77 ,009279
4.19 66.023 ,015146 4.69 108.85 ,009187
4.20 66.686 0.014996 4.70 109.95 0.009099
4.21 67.357 ,014846 4.71 111.05 .009005
4.22 68.033 ,014699 4.72 112.17 ,008915
4.23 68.717 .O 14552 4.73 113.30 ,008826
4.24 69.408 ,014408 4.74 114.43 ,008739
4.25 70.105 0.014264 4.75 115.58 0.008652
4.26 70.810 .O 14122 4.76 116.75 ,008566
4.27 71.522 ,013982 4.77 117.92 .008480
4.28 72.240 ,013843 4.78 119.10 ,008396
4.29 72.966 .O 13705 4.79 120.30 ,008312
4.30 73.700 0.01 3.569 4.80 121.51 0.008230
4.31 74.440 ,013434 4.81 122.73 ,008148
4.32 75.189 ,013300 4.82 123.97 .008067
4.33 75.944 ,013168 4.83 125.21 ,007987
4.34 76.708 ,013037 4.84 126.47 ,007907
4.35 77.478 0.012907 4.85 127.74 0.007828
4.36 78.257 ,012778 4.86 129.02 ,007750
4.37 79.044 ,012651 4.87 130.32 ,007673
4.38 79.838 .O 12525 4.88 131.63 ,007597
4.39 80.640 .O 12401 4.89 132.95 ,007521
4.40 81.451 0.012277 4.90 134.29 0.007477
4.41 82.269 ,012155 4.91 135.64 ,007372
4.42 83.096 ,012034 4.92 137.00 ,007299
4.43 83.931 ,011914 4.93 138.38 ,007227
4.44 84.775 ,011796 4.94 139.77 ,007155
4.45 85.627 0.01 1679 4.95 141.17 0.007083
4.46 86.488 ,011562 4.96 142.59 ,007013
4.47 87.357 ,011447 4.97 144.03 ,006943
4.48 88.235 ,011333 4.98 145.47 ,006874
4.49 89.121 ,011221 4.99 146.94 .006806
4.50 90.017 0.01 1109 5.00 148.41 0.006738 ~
TABLE 2 EXPONENTIAL FUNCTIONS 789

X ex ecX X ex e-" 1 1
5.00 148.41 0.006738 5.50 244.69 0.0040868
5.01 149.90 ,006671 5.55 257.24 ,0038873
5.02 151.41 ,006605 5.60 270.43 ,0036979
5.03 152.93 ,006539 5.65 284.29 ,0035 175
5.04 154.47 .0064 74 5.70 298.87 ,0033460
5.05 156.02 0.006409 5.75 314.19 0.0031828
5.06 157.59 ,006346 , 5.80 330.30 ,0030276
5.07 159.17 ,006282 347.23 .oo28799
5.08 160.77 .006220 5.90 365.04 ,0027394
5.09 162.39 ,006158 5.95 383.75 .0026058
5.10 164.02 0.006097 6.00 403.43 0.0024788
5.11 165.67 ,006036 6.05 424.11 ,0023579
5.12 167.34 ,005976 6.10 445.86 ,0022429
5.13 169.02 ,005917 6.15 468.72 ,0021335
5.14 170.72 ,005858 ' 6.20 492.75 ,0020294
5.15 172.43 0,005799 6.25 518.01 0.00 19305
5.16 174.16 ,005742 6.30 544.57 ,0018363
5.17 175.91 ,005685 6.35 572.49 ,0017467
5.18 177.68 ,005628 601.85 .0016616
5.19 179.47 ,005572 632.70 .0015805
5.20 181.27 0.005517 665.14 0.0015034
5.21 183.09 ,005462 699.24 .0014301
5.22 184.93 ,005407 735.10 .OO 13604
5.23 186.79 ,005354 772.78 ,0012940
5.24 188.67 .005300 812.41 00 12309
5.25 190.57 0.005248 854.06 0.001 1709
5.26 192.48 ,005195 897.85 ,0011138
5.27 194.42 ,005144 943.88 ,0010595
5.28 196.37 ,005092 992.27 .0010078
5.29 198.34 ,005042 1043.1 .0009586
5.30 200.34 0.004992 1096.6 0.0009 119
5.31 202.35 ,004942 1152.9 ,0008674
5.32 204.38 ,004893 1212.0 .0008251
5.33 206.44 ,004844 1274.1 ,0007849
5.34 208.51 ,004796 1339.4 ,0007466 '
5.35 210.61 0.004748 1408.1 0.0007 102
5.36 212.72 ,004701 1480.3 ,0006755
5.37 214.86 ,004654 1556.2 ,0006426
5.38 217.02 ,004608 1636.0 00061 13
5.39 219.20 ,004562 1719.9 .0005814
5.40 221.41 0.0045 17 1808.0 0.0005531
5.41 223.63 ,004472 1900.7 ,000526 1
5.42 225.88 ,004427 1998.2 .0005005
5.43 228.15 ,004383 2100.6 .0004760
5.44 230.44 ,004339 2208.3 ,0004528
5.45 232.76 0.004296 2321.6 0.0004307
5.46 235.10 ,004254 2440.6 .0004097
5.47 237.46 ,004211 2565.7 .0003898
5.48 239.85 ,004169 2697.3 .0003707
5.49 242.26 ,004128 2835.6 ,0003527
5.50 244.69 0.004087 2981.0 0.0003355
790 TABLE 2 EXPONENTIAL FUNCTIONS

Table 2 Exponential Functions (Continued)

l x ex ec2

8.00 2981.0 0.0003355 9.00 8103.1 0.0001234


~ 8.05 3133.8 ,0003191 9.05 8518.5 .0001174
8.10 3294.5 ,0003035 9.10 8955.3 .0001117
~ 8.15 3463.4 ,0002887 9.15 9414.4 ,0001062
8.20 3641.0 ,0002747 9.20 9897.1 ,00010 10
I
8.25 3827.6 0.0002613 9.25 10405 0.0000961
8.30 4023.9 ,0002485 9.30 10938 .0000914
8.35 4230.2 ,0002364 9.35 11499 ,0000870
8 40 4447.1 ,0002249 9.40 12088 .0000827
8.45 4675.1 ,0002139 9.45 12708 ,0000787
8.50 4914.8 0.0002036 9.50 13360 0.0000749
8.55 5166.8 .0001935 9.55 14045 .0000712
8.60 5431.7 ,0001841 9.60 14765 ,0000677
8.65 5710.0 .0001751 9.65 15522 .0000644
8.70 6002.9 ,0001666 9.70 16318 ,0000613
8.75 6310.7 0.000 1585 9.75 17154 0.0000583
8.80 6634.2 .0001507 9.80 18034 ,0000555
8.85 6974.4 .0001434 9.85 18958 ,0000527
8.90 7332.0 ,0001364 9.90 19930 ,0000502
8.95 7707.9 ,0001297 9.95 20952 .0000477
I
l 9.00 8103.1 0.0001234 10.00 22026 0.0000454
TABLE 3 HYPERBOLIC FUNCTIONS 791

Table 3 Hyperbolic Functions

X sinh x cash x tanh x

0 0.00000 1.0000 0.00000


0.1 ,10017 1.0050 ,09967
0.2 ,20134 1.0201 ,19738
0.3 ,30452 1.0453 ,29131
0.4 ,41075 1.0811 ,37995
0.5 ,52110 1.1276 ,46212
0.6 ,63665 1.1855 ,53705
0.7 ,75858 1.2552 ,60437
0.8 ,8881 1 1.3374 ,66404
0.9 1,0265 1.4331 ,71630
1.0 1.1752 1.5431 ,76159
1.1 1.3356 1.&a5 ,80050
1.2 1,5095 1.8107 ,83365
1.3 1.6984 1.9709 .a6172
1.4 1.9043 2.1509 .a8535
1.5 2.1293 2.3524 ,90513
1.6 2.3756 2.5775 ,92167
1.7 2.6456 2.8283 ,93541
1.8 2.9422 3.1075 ,94681
1.9 3.2682 3.4177 ,95624
2.0 3.6269 3.7622 ,96403
2.1 4.0219 4.1443 ,97045
2.2 4.4571 4.5679 ,97574
2.3 4.9370 5.0372 .gaoio
2.4 5.4662 5.5569 ,98367
2.5 6.0502 6.1323 ,98661
2.6 6.6947 6.7690 ,98903
2.7 7.4063 7.4735 ,99101
2.8 8.1919 8.2527 ,99263
2.9 9.0596 9.1146 ,99396
3.0 io.oia 10.068 ,99505
3.1 11.076 11.122 ,99595
3.2 12.246 12.287 ,99668
3.3 13.538 13.575 ,99728
3.4 14.965 14.999 ,99777
3.5 16.543 16.573 ,99818
3.6 18.285 18.313 ,99851
3.7 20.21 1 20.236 ,99878
3.8 22.339 22.362 ,99900
3.9 24.691 24.711 ,99918
4.0 27.290 27.308 ,99933
4.1 30.162 30.178 ,99945
4.2 33.336 33.351 ,99955
4.3 36.843 36.857 ,99963
4.4 40.719 40.732 ,99970
4.5 45.003 45.014 ,99975
4.6 49.737 49.747 ,99980
4.7 54.969 54.978 ,99983
4.8 60.75 1 60.759 ,99986
4.9 67.141 67.149 ,99989
5.0 74.203 74.210 .99991
792 TABLE 4 NATURAL LOGARITHMS

Table 4 Satural Logarithms

1x1 0 1 2 3 4 5 6 7 8 9 1
1.0 0000 0100 0198 0296 0392 0488 0583 0677 0770
1.1 0953 1044 1133 1222 1310 1398 1484 1570 1655 1740
1.2 1823 1906 1989 2070 2151 2231 2311 2390 2469 2546
1.3 2624 2700 2776 2852 2927 3001 3075 3148 3221 3293
1.4 3365 3436 3507 3577 3646 3716 3784 3853 3920 3988
1.5 4055 4121 4187 4253 4318 4383 4447 451 1 4574 4637
1.6 4700 4762 4824 4886 4947 5008 5068 5128 5188 5247
1.7 5306 5365 5423 548 1 5539 5596 5653 5710 5766 5822
1.8 5878 5933 5988 6043 6098 6152 6206 6259 6313 6366
1.9 6419 6471 6523 6575 6627 6678 6729 6780 6831 688 1
2.0 6931 6981 7031 7080 7129 7178 7227 7275 7324 7372
2.1 7419 7467 7514 7561 7608 7655 7701 7747 7793 7839
2.2 7885 7930 7975 8020 8065 8109 8154 8198 8242 8286
2.3 8329 8372 8416 8459 8502 8544 8587 8629 8671 8713
2.4 8755 8796 8838 8879 8920 8961 9002 9042 9083 9123
2.5 9163 9203 9243 9282 9322 9361 9400 9439 9478 9517
2.6 9555 9594 9632 9670 9708 9746 9783 982 1 9858 9895
2.7 9933 9969 1.0006 0043 0080 0116 0152 0188 0225 0260
2.8 1.0296 0332 0367 0403 0438 0473 0508 0543 05 78 0613
2.9 0647 0682 0716 0750 0784 0818 0852 0886 0919 0953
3.0 1.0986 1019 1053 1086 1119 1151 1184 1217 1249 1282
3.1 1314 1346 1378 1410 1442 1474 1506 1537 1569 1600
3.2 1632 1663 1694 1725 1756 1787 1817 1848 1878 1909
3.3 1939 1969 2000 2030 2060 2090 2119 2149 2179 2208 ~

3.4 2238 2267 2296 2326 2355 2384 2413 2442 2470 2499
3.5 1.2528 2556 2585 2613 2641 2669 2698 2726 2754 2782 ~

3.6 2809 2837 2865 2892 2920 2947 2975 3002 3029 3056
3.7 3083 3110 3137 3164 3191 3218 3244 3271 3297 3324 ~

3.8 3350 3376 3403 3429 3455 3481 3507 3533 3558 3584 ~

3.9 3610 3635 3661 3686 3712 3737 3762 3788 3813 3838
4.0 1.3863 3888 3913 3938 3962 3987 4012 4036 4061 4085
4.1 4110 4134 4159 4183 4207 4231 4255 4279 4303 4327
4.2 4351 4375 4398 4422 4446 4469 4493 4516 4540 4563
4.3 4586 4609 4633 4656 4679 4702 4725 4748 4770 4793
4.4 4816 4839 4861 4884 4907 4929 4951 4974 4996 5019
4.5 1.5041 5063 5085 5107 5129 5151 5173 5195 5217 5239
4.6 5261 5282 5304 5326 5347 5369 5390 5412 5433 5454
4.7 5476 5497 5518 5539 5560 5581 5602 5623 5644 5665
4.8 5686 5707 5728 5748 5769 .5790 5810 5831 5851 5872
4.9 5892 5913 5933 5953 5974 5994 6014 6034 6054 6074
5.0 1.6094 6114 6134 6154 6174 6194 6214 6233 6253 6273
5. 1 6292 6312 6332 6351 6371 6390 6409 6429 6448 6467
5.2 6487 6506 6525 6544 6563 6582 6601 6620 6639 6658
5.3 6677 6696 6715 6734 6752 6771 6790 6808 6827 6845
5.4 6864 6882 6901 6919 6938 6956 6974 6993 7011 7029
TABLE 4 NATURAL LOGARITHMS 793

Table 4 Natural Logarithms (Continued)

1 2 3 4 5 6 7 8 9

5.5
~ 1.7047 7066 7084 7102 7120 7138 7156 7174 7192 7210 ~

5.6 7228 7246 7263 7281 7299 7317 7334 7352 7370 7387
1
~

5.7
~ 7405 7422 7440 7457 7475 7492 7509 7527 7544 7561
I 5.8 7579 7596 7613 7630 7647 7664 7681 7699 7716
5.9 7750 7766 7783 7800 7817 7834 7851 7867 7884
1.7918 7934 795 1 7967 7984 800 1 8017 8034 8050 8066 1
~ 6.0
6.1 , ~
8083 8099 8116 8132 8148 8165 8181 8197 8213 8229
I 6.2 8245 8262 8278 8294 8310 8326 8342 8358 8374 8390
8405 8421 8437 8433 8469 8485 8500 8516 8532 8547
6.3
~

~ ~

1 6.4 8563 8579 8594 8610 8625 864 1 8656 8672 8687 8703
6.5 1.8718 8733 8749 8764 8779 8795 8810 8825 8840 8856
6.6 8871 8886 8901 8916 893 1 8946 8961 8976 8991 9006
~ 6.7 ~ 9021 9036 905 1 9066 908 1 9095 91 10 9125 9140 9155
', 6.8 1 9169 9184 9199 9213 9228 9242 9257 9272 9286 9301
~ 6.9 I 9315 9330 9344 9359 9373 9387 9402 9416 9430 9445 1
1 7.0 ~ 1.9459 9473 9488 9502 9516 9530 9544 9559 9573 9587 1
9601 9615 9629 9643 9657 967 1 9685 9699 9713 9727
9741 9755 9769 9782 9796 9810 9824 9838 985 1 9865 I

9906 9920 9933 9947 9961 9974 9988


7.4 2.0015 0028 0042 0055 0069 0082 0096 0109 0122
0176 0189 0202 0215 0229 0242 0255 0268
0281 0295 0308 0321 0334 0347 0360 0373 0386 0399
' 7.7 0412 0425 0438 045 1 0464 0477 0490 0503 0516 0528
0567 0580 0592 0605 0618 0630 0643 0656
0694 0707 0719 0732 0744 0757 0769 0782
0819 0832 0844 0857 0869 0882 0894 0906 1
0943 0956 0968 0980 0992 1005 1017 1029 ,
1066 1078 1090 1102 1114 1126 1138 1150
1187 1199 1211 1223 1235 1247 1258 1270
1306 1318 1330 1342 1353 1365 1377 1389
1424 1436 1448 1459 1471 1483 1494 1506
1541 1552 1564 1576 1587 1599 1610 1622
1656 1668 1679 1691 1702 1713 1725 1736
1770 1782 1793 1804 1815 1827 1838
89 1861 1872 1883 1894 1905 1917 1928 1939 1950
90 2 1972 1983 1994 2006 2017 2028 2039 2050 2061 2072
1 9 1 2083 2094 2105 2116 2127 2138 2148 2159 2170 2181
92 2192 2203 2214 2225 2235 2246 2257 2268 2279 2289
93 2300 2311 2322 2332 2343 2354 2364 2375 2386 2396 ~

2428 2439 2450 2460 2471 248 1 2492 2502


2534 2544 2555 2565 2576 2586 2597 2607 ~

2638 2649 2659 2670 2680 2690 2701 2711 1

2742 2752 2762 2773 2783 2793 2803


2844 2854 2865 2875 2885 2895 2905
2946 2956 2966 2976 2986 2996 3006 3016
~

Use In 10 = 2.30259 and the properties of in x to find logarithms of numbers greater than 10 or less than 1
794 TABLE 5 COMMON LOGARITHMS

Table 5 Common Logarithms


1

I N 0 1 2 3 4 5 6 7 8 9 1
1
10 0000 0043 0086 0128 0170 0212 0253 0294 0334 0374 I
11 0414 0453 0492 0531 0569 0607 0645 0682 0719 0755 1
12 0792 0828 0864 0899 0934 0969 1004 1038 1072 1106 1
13 1139 1173 1206 1239 1271 1303 1335 1367 1399 1430
14 1461 1492 1523 1553 1584 1614 1644 1673 1703 1732 1
~ ;i 1761
2041
2304
1790
2068
2330
1818
2095
2355
1847
2122
2380
1875
2148
2405
1903
2175
2430
1931
2201
2455
1959
2227
2480
1987
2253
2504
2014
2279
2529
18 2553 2577 2601 2625 2648 2672 2695 2718 2742 2765
19 2788 2810 2833 2856 2878 2900 2923 2945 2967 2989

~ ;; 3010
3222
3032
3243
3054
3263
3075
3284
3096
3304
3118
3324
3139
3345
3160
3365
3181
3385
3201
3404

1 ;;
3424 3444 3464 3483 3502 3522 3541 3560 3579 3598
3617 3636 3655 3674 3692 3711 3729 3747 3766 3784
24 3802 3820 3838 3856 3874 3892 3909 3927 3945 3962
25 3979 3997 4014 4031 4048 4065 4082 4099 41 16 4133
26 4150 4166 4183 4200 4216 4232 4249 4265 4281 4298
27 43 14 4330 4346 4362 4378 4393 4409 4425 4440 4456
28 4472 4487 4502 4518 4533 4548 4564 4579 4594 4609
29 4624 4639 4654 4669 4683 4698 4713 4728 4742 4757
30 4771 4786 4800 4814 4829 4843 4857 4871 4886 4900
31 4914 4928 4942 4955 4969 4983 4997 5011 5024 5038
32 505 1 5065 5079 5092 5105 51 19 5132 5145 5159 5172
33 5185 5198 5211 5224 5237 5250 5263 5276 5289 5302
34 5315 5328 5340 5353 5366 5378 5391 5403 5416 5428
35 5441 5453 5465 5478 5490 5502 5514 5527 5539 5551
36 5563 5575 5587 5599 5611 5623 5635 5647 5658 5670
37 5682 5694 5705 5717 5729 5740 5752 5763 5775 5786
38 5798 5809 5821 5832 5843 5855 5866 5877 5888 5899
39 5911 5922 5933 5944 5955 5966 5977 5988 5999 6010
40 6021 603 1 6042 6053 6064 6075 6085 6096 6107 6117
41 6128 6138 6149 6160 6170 6180 6191 6201 6212 6222
42 6232 6243 6253 6263 6274 6284 6294 6304 6314 6325
43 6335 6345 6355 6365 6375 6385 6395 6405 6415 6425
44 6435 6444 6454 6464 6474 6484 6493 6503 6513 6522
45 6532 6542 655 1 6561 6571 6580 6590 6599 6609 6618
46 6628 6637 6646 6656 6665 6675 6684 6693 6702 6712
47 6721 6730 6739 6749 6758 6767 6776 6785 6794 6803
48 6812 682 1 6830 6839 6848 6857 6866 6875 6884 6893
49 6902 691 1 6920 6928 6937 6946 6955 6964 6972 698 1
I 50 6990 6998 7007 7016 7024 7033 7042 7050 7059 7067
, 51 7076 7084 7093 7101 7110 7118 7126 7135 7143 7152
52 7160 7168 7177 7185 7193 7202 7210 7218 7226 7235
7243 7251 7259 7267 7275 7284 7292 7300 7308 7316
7324 7332 7340 7348 7356 7364 7372 7380 7388 7396
TABLE 5 COMMON LOGARITHMS 795

Table 5 Common Logarithms (Continued)

N 0 1 2 3 4 5 6 7 8 9 1
55 7404 7412 7419 7427 7435 7433 745 1 7459 7466 7474
56 7482 7490 7497 7505 7513 7520 7528 7536 7543 7551
57 7559 7566 7574 7582 7589 7597 7604 7612 7619 7627
58 7634 7642 7649 7657 7664 7672 7679 7686 7694 7701
59 7709 7716 7723 7731 7738 7745 7752 7760 7767 7774
7789 7796 7803 7810 7818 7825 7832 7839 7846
7860 7868 7875 7882 7889 7896 7903 7910 7917
793 1 7938 7945 7952 7959 7966 7973 7980 7987
63 7993 8000 8007 8014 802 1 8028 8035 8041 8048 8055
64 8062 8069 8075 8082 8089 8096 8102 8109 8116 8122
65 8129 8136 8142 8149 8156 8162 8169 8176 8182 8189
8202 8209 8215 8222 8228 8235 8241 8248 8254
8267 8274 8280 8287 8293 8299 8306 8312 8319
8331 8338 8344 8351 8357 8363 8370 8376 8382
8395 84Q1 8407 8414 8420 8426 8432 8439 8445
70 8451 8457 8463 8470 8476 8482 8488 8494 8500 8506
71 8513 8519 8525 8531 8537 8543 8549 8555 8561 8567
72 8573 8579 8585 8591 8597 8603 8609 8615 862 1 8627
73 8633 8639 8645 865 1 8657 8663 8669 8675 8681 8686
74 8692 8698 8704 8710 8716 8722 8727 8733 8739 8745
75 8751 8756 8762 8768 8774 8779 8785 8791 8797 8802
76 8808 8814 8820 8825 8831 8837 8842 8848 8854 8859
77 8865 8871 8876 8882 8887 8893 8899 8904 8910 8915
78 8921 8927 8932 8938 8943 8949 8954 8960 8965 8971
79 8976 8982 8987 8993 g998 9004 9009 9015 9020 9025
80 9031 9036 9042 9047 9053 9058 9063 9069 9074 9079
81 9085 9090 9096 9101 9106 9112 9117 9122 9128 9133
l 82 913d 9143 9149 9194 9159 9165 9170 9175 9180 9186
83 9191 9196 9201 9206 9212 9217 9222 9227 9232 9238
84 9243 9248 9253 9258 9263 9269 9274 9279 9284 9289
85 9294 9299 9304 9309 9315 9320 9325 9330 9335 9340
I 86 9345 9350 9355 9360 9365 9370 9375 9380 9385 9390
87 9395 9400 9405 9410 9415 9420 9425 9430 9435 9440
9450 9455 9460 9465 9469 9474 9479 9484 9489
9499 9504 9509 9513 9518 9523 9528 9533 9538
9547 9552 9557 9562 9566 9571 9576 9581 9586
9595 9600 9605 9609 9614 9619 9624 9628 9633
9643 9647 9652 9657 9661 9666 967 1 9675 9680
9689 9694 9699 9703 9708 9713 9717 9722 9727
94 9731 9736 9741 9745 9750 9754 9759 9763 9768 9773
95 9777 9782 9786 9791 9795 9800 9805 9809 9814 9818
96 9823 9827 9832 9836 984 1 9845 9850 9854 9859 9863
97 9868 9872 9877 9881 9886 9890 9894 9899 9903 9908
9917 9921 9926 9930 9934 9939 9943 9948 9952
9961 9965 9969 9974 9978 9983 9987 9991 9996
796 TABLE 6 POWERS AND ROOTS

Table 6 Powers and Roots

n n2 6 n3 G I
1 1 1.000 1 1.000 51 2,601 7.141
2 4 1.414 8 1.260 52 2,704 7.211 140,608 3.732
9 1.732 27 1.442 53 2,809 7.280 148,877 3.756
16 2.000 64 1.587 54 2,916 7.348 157,464 3.780
5 25 2.236 125 1.710 55 3,025 7.416
6 36 2.449 216 1.817 56 3,136 7.483
1 7 49 2.646 343 1.913 57 3,249 7.550
8 64 2.828 512 2.000 58 3,364 7.616
9 81 3.000 729 2.080 59 3,481 7.681
10 100 3.162 1,000 2.154 60 3,600 7.746
11 121 3.317 1,331 2.224 61 3,721 7.810
12 144 3.464 1,728 2.289 62 3,844 7.874
13 169 3.606 2,197 2.351 63 3,969 7.937
14 196 3.742 2,744 2.410 64 4,096 8.000
15 225 3.873 3,375 2.466 65 4,225 8.062
16 256 4.000 4,096 2.520 66 4,356 8.124
17 289 4.123 4,913 2.571 67 4,489 8.185
18 324 4.243 5,832 2.621 68 4,624 8.246
19 361 4.359 6,859 2.668 69 4,761 8.307
20 400 4.472 8,000 2.714 70 4,900 8.367 343,000 4.121
I 21 441 4.583 9,261 2.759 71 5,041 8.426 357,911 4.141 ~

22 484 4.690 10,648 2.802 72 5,184 8.485 373,248 4.160


23 529 4.796 12,167 2.844 73 5,329 8.544 389,017 4.179
24 576 4.899 13,824 2.884 74 5,476 8.602 405,224 4.198
25 625 5,000 15,625 2.924 75 5,625 8.660 421,875 4.217
26 676 5.099 17,576 2.962 76 5,776 8.718 438,976 4.236
729 5.196 19,683 3.000 77 5,929 8.775 456,533 4.254
I 28 784 5.291 21,952 3.037 78 6,084 8.832 474,552 4.273
I 29 841 5.385 24,389 3.072 79 6,241 8.888 493,039 4.291
I
30 1 900 5.477 27,000 3.107 80 6,400 8.944 512,000 4.309
1 31 961 5.568 29,791 3.141 81 6,561 9.000 531,441 4.327
32 1,024 5.657 32,768 3.175 82 6,724 9.055 551,368 4.344
33 1,089 5.745 35,937 3.208 83 6,889 9.110 571,787 4.362
34 1,156 5.831 39,304 3.240 84 7,056 9.165 592,704 4.380
35 1,225 5.916 42,875 3.271 85 7,225 9.220 614,125 4.397
36 1,296 6.000 46,656 3.302 86 7,396 9.274 636,056 4.414
37 1,369 6.083 50,653 3.332 87 7,569 9.327 658,503 4.431
38 1,444 6.164 54,872 3.362 88 7,744 9.381 681,472 4.448
39 1,521 6.245 59,319 3.391 89 7,921 9.434 704,969 4.465
40 1,600 6.325 64,000 3.420 90 8,100 9.487 729,000 4.481
41 1,681 6.403 68,921 3.448 91 8,281 9.539 753,571 4.498
42 1,764 6.481 74,088 3.476 92 8,464 9.592 778,688 4.514
43 1,849 6.557 79,507 3.503 93 8,649 9.643 804,357 4.531
44 1,936 6.633 85,184 3.530 94 8,836 9.695 830,584 4.547
45 2,025 6.708 91,125 3.557 95 9,025 9.747 857,375 4.563
46 2,116 6.782 97,336 3.583 96 9,216 9.798 884,736 4.579
912,673 4.595
I 2,209 6.856 103,823 3.609 97 9,409 9.849

::
47
2,304 6.928 110,592 3.634 98 9,604 9.899 941,192 4.610
~ 2,401 7.000 117,649 3.659 99 9.801 9.950 970,299 4.626
50 2,500 7.071 125,000 3.684 100 10,000 io.noo 1,000,000 4.642
L
~ Solutions to Progress Tests
s-2 SOLUTIONS TO PROGRESS TESTS

C h a p t e r 1: Solutions to Progress Tests


See. 1.1 :Progress Test 1

2. (u) x > -1 (6) x 2 -2 (c) 4 <x (d) x 2 -3 (a) (b)


-1 0 -2 0
(c) (4
0 4 -1 -$ 0 1

Sec. 1.1 : Progress Test 2


1 . All x whose dist from - 1 is 25

2. 31x + 31 < 6 lx + 31 < 2 All x whose dist. from -2 is < 2: (-3,i).


8
-3

3. 21x - 31 < 9 = lx - 31 < 4. All x whose dist. from 3 is < 4 (-$, 9)

- 2 - 1 0 1 2 3 4 5 6 7 8
4. No x can have negative dist. from 1, so no x satisfy lx - 1) < 0.

1. 1

t.
c = 3,5
0 B = 2.4
A = 1, 3
1

I e7

4.
6.

7.

Sec. 1.2: Progress Test 2


3-0 -3 2-(-1) 3
1. +l. 2. ---, - 3. ~
- --
- - _1 4. 2. 5 . No. 6. No. 7. -1 (Draw a picture.)
0-4 4 -2-4 -6 2'
-1
8. -- - 1.
(-1)
SECTION 2.1 5-3

See. 1.2: Progress Test 3

1. ( a ) y - 5 = 4 ( x - 3 ) ; (b)y-5=- -
4-1
(x - 3) ory - 9 = 4 ( x - 4); (6) y = -2x + 2. 2. (a) y = 3x +2 rn = 3 , b = 4;
(b) y = j x - 2 3 m = 2, b = -2.

See. 1.3: Progress Test 1


+ ( y - 4)' = 4; ( 6 ) ( x + 3)' + ( y - 3)'= 5; (c) + ( y - a)' = t'.
1.
2.
(a) ( x
(a) ( x
- 1)'
+ 2)' + ( y - 3)' = - 1 + 13; center (-2, 3 ) , r = a=- (X
2fi.
m)'

(b) (x + 3)' + ( y + 6)2 = - 3 + 4 5 ; center ( - 3 , -6), r = a. (c) ( x + 1)' + ( y - 4)' = -40 + 1 - = - y <0, SO no FaPh

See. 1.3: Progress Test 2


1. Y 2. Y 3.

it
t X

1 * X
v 3 , -20) 1
1
1
(-3, -32) 1
(-1, -36) , (0,-35) 1
-2
1 -1
x = - - -
-7 - 7 x = - = - l x = -
2 2
2(-2) 4
y = (x + 7 ) ( x - 5) discriminant = 1 - 8 = - 7 < 0 ,,
y = :-2x + 1):x - 3)

Chapter 2: Solutions to Progress Tests

See. 2.1 :Progress Test 1


1. (a) f ( t ) = 3t2 - 5t + 2: ( b ) f(x +
h) = 3(x +
h)' - 5(x h) + +
2; (c) f(2x) = 3(2x)' - 5 ( 2 x ) 2. +
2. ( a ) f(t) = at + b;
( b ) f ( x + h ) = a(x + h ) + b; (c) f ( 2 x ) = a(2x) + b. 3. ( a ) f(t) = t3; ( b ) f ( x h ) = ( x h)3; +
(c) f ( 2 x +
)=( 2 ~ ) ~ .
4. C(2r) = 2 4 2 7 ) = 4777 = 2C; it doubles also.
5. ( a ) x 2 - x - 2 = ( x + l ) ( x - 2) 2 0 Domain contains all x in (- w , - 11, [ 2 , + w ) ( b ) all x # 8.

----_- (x + 1)
-1 (x - 2)

-1 2
s-4 SOLUTIONS TO PROGRESS TESTS

See. 2.1: Progress Test 2


1. ( a ) All x # 1; ( b )

0 2
3 2
.9 2
.99 2
1 undefined
1.01 2
1.1 2
2 1 2
(c) The function constantly 2 equalsf(?) except at x = 1. ( d ) 3.) =

Gx
(0,2) f(X,

0 (1, - 1 )

2. (u) y = i x + 2 , yes; ( b ) x = f y - 3, yes [graph is straight line, slope 4,y int. (0, 2)].

2y2 - 1
3. ( a ) y = i v z q , n o ( b ) x = -
4
~ yes. 4. ( a ) y1 = m , y z= - ( b ) all x 2 -4 (for both).
3. A

flxj

%---T-- ----- -~
~-
------A
\ I I
-L , y

' I
Sec. 2.1 :Progress Test 3

-- 1
x = -
3
SECTION 2.2 s-5

See. 2.2: Progress Test 1


(c) 1imz+2(9- x2) = 5.

1.59 1 6.75 2.01 4.9599


1.9 ' 5.39 2.001 4.995999
1.99 5.0399
1.999 5.003999

2. (a) x2-9
(c) lim- = 6
x-3 x -3

2.99 5.99
2.999 5.999 3
3 undefined
3. x-1 x- 1 x-1
lim -does not exist; lim -= lim = -1; lim- x - l = lim-= x - 1 1.
8-1- 1 x - 11
~

z-1 Ix - 11 2-1- -(x - 1) z-1+ Ix - 11 c-1- (x - 1)

undefined

1 1 - - - co.
4. lim- does not exist; lim -= + w , lirn -
x-2 2-x x-2- 2 - x x-2+ 2 - x

x = 2
5 . lim fi- fi = lim fi- fi = lim 1
x-3 x -3 x-3 ( G +fi)(&- fi) x-3 & + fi
[(since x # 3)j = -
1
2fi-
- 0.288675 (this could also have been obtained by rationalizing the numerator).

'I
-1
6. Since (x + 3)4 > 0 for all x # -3, l i m p - -1 - -w. x = -3
~

(x + 3)4 < O f o r a l l x # -3and


2--3 (x + 3)4 I f:x: = ~
-1
(x - 314
I
I

See. 2.2: Progress Test 2


3h 3h
1. lim [3(-2 + h, + 21 - [3(-2) + 21 = lim-. But h # 0, so - = 3, and lirn 3 = 3 (3 is unaffected by h).
h-0 h h-0 h h h-0

2. (1+h)+1-- -
- m-fim+fi-
2+h-2 - h
-
1
since h # 0.
h h ' -+ fi- h ( m + f2)- h ( m + fi)--+ fi
Thus lim
f(1 + h) - f ( 1 ) = lim 1 - 1 3. lim
[(t + 2)2 + ( t + 2)] - [22 + 21 = lim- t 2 + 5t = lim(t + 5) = 5
h-0 h h-0 -+ fi- 2 f i ' t-0 t t-0 t f-0

1 1

4. lim
(3 + h)2 32
= lim
9 - (3 + h)2 = lim
-6h - h 2
- limp
-6-h
=-
-6
h-0 h h-0 h * 9 * (3 + h)' h-0 9h(3 + h)2 h-0 9(3 + h)2 81
S-6 SOLUTIONS TO PROGRESS TESTS

See. 2.3: Progress Test 1


64.064016 - 64 63.3616 - 64 '(61 h ) 2 - 16(2)2 = 64
i- + 16h. 2. 64 ft/s = limh,o(64 + 16h).
= 64.016; (6) = 63.84; (c)
(a) ,001 - ,001 h

3. 32 ft/s = lim(32 + 16h) = lim 16(1 + h)' - 16(1)'


h-0 h-0 h

See. 2.3: Progress Test 2


m(x + h ) - m ( x ) -- 2 . 6 ( ~+ h)' - 2 . 6 ~ ' = 5 . 2 ~+ 2.6h; ( b ) m'(x) = lim,,o(5.2x + 2.6h) = 5.22
1. (a)
h h
4 6 + h ) - 4 6 ) = lim 2.6(36 + 12h + h 2 ) - (2.6)(36) = lim
31.2h + 2.6h2 -
2. m'(5.9) = (5.2)(5.9) = 30.68 ft/s. 3. m'(6) = lirn
h-0 h h-0 h h-0 h
limh,,(31.2 + 2.6h) = 31.2. 4. m'(x) = 5 . 2 ~= 24, s o x = 4.6s (approx.), m(4.6) = 2.6(4.6)2 = % . O f t (approx.).

See. 2.3: Progress Test 3

I . d f = lim f ( x + Ax) - A x ) = lim


[3 - ( x + Ax)2] - [3 - x2]
= lim
- 2x Ax - (Ax)2
= lim ( - 2 x - Ax) [(since Ax # O ) ] = -2x.
dx AX-o Ax Ax-0 AX AX-0 Ax Ax-0

x - ( x + Ax)
2. = lim g(x + Ax) - = lim
l/(x + Ax) - l / x = lirn
-AX
= lim-
-1
g'(x)
Ax-0 AX Ax-0 AX Ax-0 Ax Ar-0 x ( x + Ax) Ax AX-0 + A x ) [(since
x(x
-1 -1
A x # o)] = -= -. 3.
4=
- lirn
[(x + AX)' + + Ax)] - [x' + X] = lirn
(X 2x Ax + (Ax)' + Ax = lirn ( 2 x + A x + 1) [(since
X ' X x2 dx AX-o AX Ax-0 AX Ax-0

-1 1
( b ) g'(2) = - =
(2)' 4

See. 2.3: Progress Test 4

1. AA = ( x + Ax)' - (x)' = 2x Ax + (Ax)'. AA


3. - =
2x Ax + ( A x ) 2 = 2x + A x .
A x ; h r iAxl2 AX Ax

X Ax
dA dA dA
4. ( a ) - = lim ( 2 x + Ax) = 2x; ( b ) When x = 2, - = 2 * 2 = 4; ( 6 ) When x = 200,- = 2 * 200 = 400; ( d ) As A x + 0, the growth in
dx Ax-0 dx dx
A , AA, approaches growth in the two edges of length x . Th e same Ax for x small produces a smaller A A than when x is large.

See. 2.4: Progress Test 1

2. g ' ( x ) = lim
g(x + Ax) - g(.) = lim
[(x + Ax)' + c] - [x' + cl = lirn ( 2 x + Ax) = 2 x , which equals the result obtained in Sec. 2.3,
Ax-0 AX AE-0 AX AS-0
SECTION 3.2 s-7

Progress Test 4>forJ(x) = x2. Thus the graph of g ( x ) is merely the graph off(x) = x 2 slid up (if c >
0) or down (if c <
0) by c units. They are
parallel in the sense that for each x, their respective tangents are parallel. For example, if c = 3, we have the following:

See. 2.5: Progress Test 1


[Here LT(a) means Limit Theorem papt ( a ) ; and SL means special limit.)] 1. limz+-l( f i x 4 - 2x) = lim2+l( f i x 4 ) - lim2--l(2x)
[LT(a)] = fi(limz+-l x)(limz+-l x)(limz--l x)(limz+-l x) - 2(1im2--1 x ) [LT(5)] = (fi)(- 1)( - 1)( - 1)( - 1) - (2)( - 1) [(SL)] = fi 2. +
l/(x +
Ax) - l/x -9x - +
2 . Ax-0
lim AX = A+-0
lim x(x +
Ax) Ax [(algebra)] = ki [(Ax # 011 = &l ( - l)/[limAc+ox(x Ax)] [LT(c)] =

+
( - l)/[x lim~~-o(x A x ) ] [SL(a), LT(b)l = ( - 1)/[x(1imA2-ox +
liq,,, A x ) ] [LT(a)] = ( - l)/(x * x ) [SL(b)-note x is const., the variable is Ax].

3. lim
+ +
[ 3 ( x h) - 2(x h ) ] - [ 3 x 2 - 2x1
= lim
(6xh +
3h2 - 2h)
+
[(algebra)] = limh-o(6x 3h - 2)[(h # O ) ] =
h-0 h h-0 h
limh,o 6x + limh+o3h - lim,,o 2 [LT(a)] = ( 6 x - 2 ) + 3 limh,o h [SL(a), LT(b)] = 6x - 2 [SL(b)]

See. 2.5: Progress Test 2


1. x = 3 [(b), (c)]. 2. x = 1 [ ( a ) to (c)]. 3. x = 2 [ ( a ) to (c)]. 4. x = - 3 [ ( a ) to (c)]. 5. cts. everywhere-
including x = 0. 6. ( a ) For C = 2.02, there is no c in [-1, 11 withJ(c) = 2.02, yetJ(-1) = 0.95.J(l) = 3.1 withf(-1) <
C<J(l).
Notef(0) = 2, and for x in [-1, O ) , f ( x ) <
1.95, and for x in (0,l],f(x) >
2.1. (b) [ - 3 , -11, [2, 51. (Any closed intervals not containing x = 0
suffice.)

Chapter 3: Solutions to Progress Tests

See. 3.1: Progress Test 1


1. f ( x ) = 41x40. 2. f(x) = 1. 3. f(x) = 12x2 + 6x + 3. 4. g(x) = 2 1 4 2 + ~ ~5 2~x .
5 . f ( x ) = 8 x 7 + 49x6 + 36x5 + g ( x ) . 6. s(x) = 12t2 - a,7 . p ( r ) = 2 ~ +r 257, 8. (Need to multiply out before
differentiating)J(x) = x 7 + 4x6 - 32 - 12x. Thusf(x) = 7 x 6 + 24x - 6 x - 12.
b-b 0
9. lim ( + -() = lim -= lim - = lim (0) = 0 by (2.5.3). 10. f(x) = 3x2 - 12x = 0 (zero slope) when x = 0, x = 4
,,
Ax-0 Ax A!-0 Ax Az-0 Ax A2-0

See. 3.1: Progress Test 2

( J ) u(x) = (ax + 5x + c ) ( m ) + (mx + d)(2ax + 5). 2. dj/dx = x(2x) + (x2)(1) = 3 2 . 3. ( a ) f(x) = 7(3x2 + lox + 2);
x2(3x2+ 6 ~ -) ( x 3 + 3 ~ + 4)2x
(5) Y(x)= 7(3x2 + 102 + 2) + ( x 3 + 5x2 + 2x - 1)(0). 4. ( a ) f(x) =
[x2I2
(b) f(x) = ( x 3 + 3 x 2 + 4 ) ( - 2 ) ~ - ~+ (x-*)(3x2 + 6 x ) ; (c) the answer to ( a ) simplifies as follow:
x ( 3 x 2 + 6 x ) - ( x 3 + 3x2 + 4)2
= x(3x2 + 6 ~ ) x +
- ~( - Z)(x3 + 3x2 + 4 ) = ~ (3x2 ~+ 6 x ) ~ +
- ~ ( - 2)(x3 + 3x + 4)K3, which equals
x3
the answer to (b). 3. Let u(x + A x ) = U, u(x + Ax) = F, u(x) = u, u ( x ) = u. Then line (1) is
u(U- u ) + u(U - u ) + ( U - u ) ( F - u ) = UU - uu + uU - uu + UU - UZ: - uU + uu = UV - uu, as to be shown.
See. 3.2: Progress Test 1
1. f(x) = 7 ( x 9 +2x4 - x 3 + 41)6(9xs + 8 x 3 - 3x2). 2. g ( x ) = -5(x3 + 7 x 2 + 2x + 1)-6(3x2 + 14x + 2).
3. u(x) = (x + 2x)(4)(50x2 + 6~ - 3)3(100x + 6 ) + ( 5 0 ~ + 6x - 3 ) 4 ( 2 ~+ 2).
$18 SOLUTIONS TO PROGRESS TESTS

4. Y(X)=
+ ~ * ) ~ ( -1 )( x + 4)(8)(3x5 + ~ * ) ~ ( 1 +5 x2x)~
(3x5
(39 + X*)16

(5x2 + 2 ~ ) ~ [ 4 +( x6~x 5 - + 30x4 - Zx)] - ( x 7 + 6.9- ~ * ) ~ [ 7 ( +5 x2x)6(10x


~ + 2)]
8. g(x) =
(5x2 + 2x)14

See. 3.2: Progress Test 2


4 4 Y 4 4 Y 4 4 4
1. (a) x - + y ( l ) = O ; s o - = --; (b) x * 2 y - + y ~ 2 x = 0 , s o - = --; (c) 2 x + 2 y - - 2 x - - ~ + x 3 y ~ - = 0 , s 0
dx dx x dx dx x dx dx dx
4--
- -zx+ 2y-y3( x - Y ) ( l + 4 / d x ) - ( x + Y ) ( l - dY/dx) = 0,so - = d y Y_ , dx dx x
2. ( a ) x . 1 + - = 0 so - = - -;
dx 2y - 2x + 3xy2 (4
-Y Y dx 4 > 4 Y
-.
(2 x
dx dx dx dx dx dx 2y - 2x + 3xy2
, (c) 2x- + 2y - 2x - 2y- + x 3 2 + y 3 - = 0 , so - =
,
( 6 ) x*2y +y*2x-
4 = 0, so -
d Y Y
=
4 4 4 4 - z x + 2y--y3
( 4 ( -y)(dx4 + - ( +y)(dx4 - dx
= 0,so - = _x , 3. (jX-
dx
+ lot - - t-dx = 0, so dx = x - 10t , 4. x -4+ y = o , s o
(x -r)* 4 Y dt dt dt 6x - t dx
4 - -Y 4 -1/2 1 1 1
- --and at x = 2, - = -- -- , T h u s y - - = - - ( x - 2).
dx x dx 2 4 2 4

See. 3.2: Progress Test 3


1 1 3 1 8 3 2x - 3
1. s ( x ) = - +z-x, 1 / 2 2. s(.) = -
z x 1 / 2 - -. 3x1/3 + -
3. g(x) = - 4. =
3x2,3 3x4/3 5x2/5 2(x2 - 3x + 1)1/2
2 - 12t
5. m(t) =
3(3t2 - t)5/3

Sec. 3.3: Progress Test I


1- +
x Ax
- [(. + 1)2 - -1; 1

( e ) Ay - 4= [((3.01) - -)2.01
1 - (3 - -
2
)] - [0.0625] 2 0.0000876 (rounded off in last place).

See. 3.3: Progress Test 2


1. (a) Ay = 277.7 Ax + 57(Ax)*; (b) 4 = 27x A x ; (C) - dY = T ( A X ) * ; I } A x
i

2KX

( d ) area of outside strip is Ay. To bend rectangular strip of width Ax and length ~ T X(circumference of inside circle of radius x ) , little wedges
would have to be cut in the strip (try it!) Ay - 4 is area of all the wedges that would need to be cut in rectangular strip in order to bend
it around circle (impossible in practice). +
2. ( a ) 0 . 2 ~ 0 . 0 0 0 1 ~=. 0.6286; ( b ) 0 . 2 ~2 0.62832; (c) 0 . 0 0 0 1 ~=. 0.00031.
3. (a) AA
lh
= -(x + AX)^
f i =f- x (iA x )
- -x2 + - fi
(Ax)*;
fi
( b ) dA = - x ( A x ) ; (c)
fi
AX)^;
fi
( d ) - - .( Ax) ;
4 4 2 4 2 4 2
Ax
( e ) --Ax fi + -A-4xA xfi = fi
AX)^; ( J ) diKerence between dA and AA is area of triangles I1 and 111. AY - 4
4. y ( x ) = nxn-l and -
4 2 2 4 AX
X*
includes all other terms-including -,
AX

See. 3.3: Progress Test 3

1. 4x (20(4x

dw dw
so solving for -, - =

q z q
3wt ~
w

3
dz
+ 1)4 + 3 x 2 ) d x .

z2 t
~
dz
d
dz
12w3 -
w
+
1 - 6z2 - 36w3-
z

dw
dz
/
-
; ( b ) 3w
2.

m+ 2 9
( 2 ) VFTT
dw
4=

3-dw
(1) 2zdz

= 0. Then solve for - to get same result as ( a ) .


dz
1

+
1

4. ( a ) Choosef(x) =
3.

a = 100,
+
( a ) 3--+

6z2 dz - 36w3 dw = 0 , so dividing through by dz, we get:


dw
dz
3w-
2-
22
+ 62

6,
-3 6 ~ ~=
- 0,
dw
dz
SECTION 4.2 s-9

1 1
Ax = dx = 2 . Thenf(x) = -, so m = f ( l O O + 2 ) ~ f ( 1 0 0 )+f(100)(2) = 10 + - = 10.1; ( b ) choosef(x) = x, a = 1
2?5 10
Ax = dx = -0.02. ThenJ(x) = ( 5 / 2 ) ~ ~ so / ~ (0.98)5/2
, = f ( l + (-0.02)) ~ f ( 1 )+ (5/2)(l)3/2(-0.02) = 1 - (0.05) = 0.95.
5. f ( x ) = x 3 , a = 10, A x = dx = O.l,f(x) = 3x2, so change in volume,f(lO + 0.1) -f(lO) ~ j ( l O ) ( O . l )= (300)(0.1) = 30 cubic units.

See. 3.4: Progress Test 1


1. J(x) = 5x4 + 4x3 + 3 x 2 + l,f(x) = 20x3 + 1 2 2 + 6x + 2,f(x) = 60x2 + 24x + 6 , f 4 ( ~=
) 120%+ 24.
2. f(x) = 22 - S,s(x) = 2,f(x) = 0 = f 4 ( x ) . 3. s ( x ) = - -1
,f(x) = - ,2f ( x ) = --6
,f4(x)
24
= =.
2 x3 x4 22

4. dr=-- X + Y
-d2y- 29 - 2~ dy/dx x2 + 2xy + 3y2 -
-- -2
dx x + 3 dx2 - - [ (x + ?y)2 (x + 3y)3 (x + 3y)3
See. 3.4: Progress Test 2
<
1. f ( t ) = 6(t - 2 ) 0 for t 2,f(t) < > 0 for t > 2. 2. Acceleration is zero when t = 2,J(x) = - 12 is velocity.
3. y(t)= 3t(t - 4) = 0 when t = 0, 4. 4. f ( 0 ) = 32,f(4) = 0.

Chapter 4: Solutions to Progress Tests

See. 4.1: Progress Test I

1. lim
(x - 3) - 5 -
- -a. 2. +a. 3.
x+1
limp = +a. 4. Let Q =
x6 + + 2 . so lirn Q = lirn
x j
Q = -a and
r-3 (x - 3)2 x - 1
r - i (x + 2)
- 3)(x z-3- 1--2-

2(x - 2 ) - 7 - 2x - 11
lim Q = lim Q = + w . 5. Let - P, so lim P = -a.
9-3- m--2+ ( x - 2)2 (x - 2)2 x-2

See. 4.1: Progress Test 2


1. y = l i d . 2. No horiz. asym. 3. y = 0. 4. No horiz. asym. Divide num. and den. by fl,
5 . y = 0. Divide num. and den. by x 3 = w.
6. y = d / 3 , Divide num. and den. by fl.

See. 4.2: Progress Test 2


I . Linear of slope 3, everywhere increasing. 2. Const. fn., zero slope everywhere. 3. J(x) = 2 ( x - 1) < 0 on ( - 00, 1) and > 0 on
(1,+ a).Rel. min. at x = 1. 4. Increasing everywhere sincef(x) = 3 x 2 + 5 > 0 for all x .
s-10 SOLUTIONS TO PROGRESS TESTS

5 - s ( x ) = ( x - 3)(x + 1) incr. on (- 03, 1) and (3, + cc), decr. on (- 1, 3). Rel. max. at x = - 1, rel. min at x = 3

See. 4.2: Progress Test 3


1. y(x)= x 2 - 2 x , y ( x ) = 2 ( x - 1) changes sign at x = 1, a pt. of infl. C.D. on ( - co, l ) , C.U. on (1,
2. f ( x ) = 4x3 - 3 6 x , f ( x ) = 12(x + fi)(x - &) See chart forf: P.I. at x = :+I ifi 41 (-1
__-
+ w).
(+I ~

x
x -
+$
\i3
--$- J3-

See. 4.3: Progress Test 1


1. (1) f(0) = 2; ( 2 ) f ( x ) = 4(x)(x
x= -1/3 i- fi/3 (crit. pts,); (4)
+ 2 ) ( x - 1) = 0 when x
\-,
-
(+I
-_ _
4 1 (-)
---
= 0, -2, 1 (crit. pts.); (3) f ( x ) = 4(3x2 + 2x - 2 ) = 0 when
(.
I\+)+ 2) Rel. max. x = 0. Rel. min. x = -2, 1. (See incr.-decr. onJ
( x - 1)
chart.)

\\\\ I ////I \\\\ I ////*LA

2-\\\\ 0 \\ 1
I \ / ?
See concavity onf chart. P.I. at x = - - i-.
3 3

( - 2 , -12; I

1 1 . 1
2. (1) lim X - - s o 4 = - is horiz. asym. lim = - cc and lim = + c c , so vert. asym. at x = -;f(O) = 0;
2*0:3X -1 3 3 0-1/3 3x - 1 z-1/3+ 3x - 1 3

( 2 ) s(xI = ~
-1
(3.7 - 1)2
< 0 for all x # 1/3; (3) f ( x ) = ~
$6
(3x - 113
> 0 for x # 1/3, andJ(x) never 0; (4) f ( x ) < 0 for all x where defined,
sofis decr. for all x # 1/3; (5)f(x) < 0, hencefC.D. for x in ( - 30, -1/3);f(x) > 0, hencefC.C. for x in (1/3, + w);

or P.I.1

3. f(X) = ( ( 3 ) d P T- (3x + 2) 8x ) / ( 4 2 - 1) =
3(4x2 - 1) - (12x2 + 82) - - 8x +3 defined only in ( - M, - 1/2),
2d G = i (4x2 - 1)dP-T (42 - 1)3/2
(1/2, + >
w ) . f l ( x ) 0 hencefincr. for x - 1/2, andf(x) < < 0, hencefdecr. for x > 1/2. Finally, limz~-l,2-f(x) = +M,
lim2+l12+f(x) = - w (vertical asymptotes). No extremes.
SECTION 4.4 s-11

(+)
----- (-1 (+I ( x + *)
----_.- --- y 1 1 3

Rel. max.
(--2>9 iP)
I/
See. 4.4: Progress Test 1
(These are sample solutions-of course there are many others.)

2. ( a ) f ( x ) = 4x - x 2 . Max. value 4 occurs at x = 2


t

(6) f ( x )
i-
= 4-
1000 forx= 4
Min. value 1/4 occurs at x = 0.
for x <4
s-12 SOLUTIONS TO PROGRESS TESTS

See. 4.4: Progress Test 2


1. ( u ) f(- 1) =f(2) = - 1/3 so, actually, Rolles Theorem applies sincefis everywhere diff,f(x) = x 2 - 1 = 0 when x = il.( b ) Hyp.
fail sincef(0) is undef.
f ( x ) = x3/3 - x - 1

2. 4 f i x ) = ) 2 x - 11 +3 Notef(x) = (2-(2x- - 1) - 3 = -2x for x > 1/2


-2 for x < 1/2
f ( x ) not def. at x = 1/2. No.

Chapter 5: Solutions to Progress Tests

See. 5.1 : Progress Test 1


(Extreme values are underlined.)
+
1. f ( x ) = 6(x 2)(x - 1) = 0 when x = -2, l.J(x) = 122 + 6, sof(-2) < O,f(l) > 0, making x = - 2 a rel. max. and x = 1 a rel.
m i n , f ( l ) = -3,f(-2) = 24,f(-3) = 13,f(3) = 49. 2. f ( x ) = 5x4 + 6 x 2 2 0 on [0, 31, soy(.) is increasing. Hence min. isf(0) = 0 and
max. isf(3) = 297. 3. f ( x ) =
+
4(x - x 2 ) 2x2 242 - x )
- -- +
- 0 when x = 0, 2, so a single rel. extreme on (1, w ) at x = 2. Note
(1 - x)2 (1 - x)2
f(x) > <
0 for x <
2,f(x) 0 for x > 2, so a rel. max. Hencef(2) = -8 is max. for J o n (1, + w ) . Since limz-,l+f(x) = - w , no min. value.

See. 5.1: Progress Test 2


1. 3 x + y = 120 soy = 120 - 3 x . A = xy = x(120 - 3x). dA/dx = 120 - 6x = 0 when x = 20, a rel. max. A ( x ) def. on [0,40], so x = 20,

xJI _ 1.
y = 60, A = 1200.

2. Let one no. be x and the othery. Maximize product x * y = P. Now 3 x + y = 120, so
>
this is same problem as 1. Hence x = 20,y = 60, P = 1200. (Note that P 0 implies the condition 0 5 x 5 40.) If roles of x a n d y are
reversed (with 3y + x = 120), same solution results. 3. 2x + 2y = 120, soy = 60 - x and A = 9 = x(60 - x ) , defined on [0, 601.
dA/dx = 60 - 2x = 0 when x = 30, a rel. max. y = 30 and A = 900 (square pasture 30 x 30).
fl Y If

4. 30 + 2 and A = A ( x ) = -(x
(2) Total area = A = xy. (3) Area of printed p. is ( x - 3 ) ( y - 2) = 30, soy = - 30 + 2).
x - 3 x-3
SECTION 5.2 S-13

(4) Only restrict. is x > +


3, so interval is (3, w ) .
( X - 3)(30) - 3 0 ~ ( 1 ) 2 ( x - 3)2 - 90 - 2x2 - 12x - 72
(5) A(x) = + 2 = -
( x - 3)2 ( x - 3)2 ( x - 3)2

Y
Hence, there is a rel. extreme at x = 3 + +
3 f i (we disregard x = 3 - 3 f i < 0). Since lim,,+,[A(x)] = w , we know this is a rel. min. on
30
+
(3, w ) . (6) T h u s y = - 2 = 2
3\5 + +
Z f i , making dimensions of entire page (3 +
3 6 ) z 9.71 cm by ( 2 +
2 f i ) z 6.47 cm.

5 . ( 2 ) Maximize vol. V of box, which is V ( x ) = 4 1 2 - 2x)(24 - 2 x ) = 288x - 72.9 4x3. +


12
-1 r-

Y
24
(3) (We could have given dimensions using more than 1 variable and then used given horiz. and vert. dimensions to eliminate all but 1.)
(4) 0 2 2 x 5 12, so the interval is [0, 61. (5) V ( x ) = 12(24 - 12.7 + x 2 ) = 0 in [0, 61 when x = 6 - 2 f i . (6) Box will have dimensions
6 - 2~ =. 2.54 cm by 4 f i z 6.93 cm by 18.93 cm.

See. 5.1: Progress Test 3


1. (a) Max. A = xy with 2 x + 2y = 100, soy = 50 - x and A(x) = x(50 - x ) . dA/dx = 50 - 2 x = 0 when x = 25,
05 x 5 50.y = 25 and A = 625 m2, ( b ) A square pasture, perfectly symmetric, with all earlier symmetries

trivially satisfied. (c) 100 = 2nr implies r = 100/2n and A = 77r2 = (100/2a)2 = 5000/.rr z 1591.5 m2. (This is largest possible pasture of any
shape.) +
2. ( a ) Max. A = 9 with 2 ( 5 x ) 2(2y) = 100, soy = (50 - 5 x ) / 2 . A = x(50 - 5 x ) / 2 and dA/dx = 25 - 5x = 0 implies x = 5,
y = 25/2. ( b ) $50. in each dir. 3. Perimeter P = 4x + 4y = 800. Area A = x2 + y 2 = x 2 (200 - x).+

A(x) = 4(x - 100) = 0 at x = 100. But A(x) = 4 >


0 means x = 100 is a rel. min. [A(lOO) = 20,000 m2 with 2 equal plots]. But A(x) is
defined on (0,20O)-endpts. are excluded by given cond. But A(0) = A(200) = 40,000, so max. is achieved at endpts. Hence, to comply with
instructions, captives should make 1 square plot as small as possible and use remaining fence to obtain an area as close as possible to
40,000 m2, One plot is better than two: Put the two plots together along a common side and note the 2 wasted sides are eliminated when 1
plot is used.

Sec. 5.2: Progress Test 1


( b ) Given dxldt = 4, need find dy/dt when x = 9, given that x 2 + y 2 = 225

(a) I l l
,
x-
S-14 SOLUTIONS TO PROGRESS TESTS

(c) x d x / d t + ydy/dt = 0. ( d ) When x = 9 , y = d m = 12, so dy/dt = ( - x / y ) ( d x / d t ) = (-9/12)(4) = - 3 m/s. 2. Let t = 0 be

+ 10) ds 41t + 250


time when plane flies over. 5/t s= d16t2 + ( 5 t + 50) = d41t2 + 500t + 2500. -
dt
=
d41t2 + 500t + 2500
6.29

m/min when t = 20.

h
. r = -.h V = -r2h = - - h = -h3.
3 . 1 = -6 implies
.
18 3
v
3 v(hr
3 3 27
dV
-= -h2-.
dt
v
9
dh
dt
dV
dt
dh
Given - = 16, need -. But
dt

See. 5.3: Progress Test 1


1. s(t) = 3(t - 2 ) ( t - 4), ~ ( t=
) 6(t - 3). Sign of s: (+)
----__-

Sign of s: (-1
I
*t
3
2. s(2) = 20, 4 3 ) = 18, s(4) = 16. 3. -
L --
t=4 -- t=3
- --
I I
-
r

I
t = 2

I I b
16 18 20
4. t< 2, moving right and slowing down; t = 2, stopped; 2 < t < 3, moving left and speeding up;
3 < < 4, moving left and slowing down; t = 4, stopped; t > 4, moving right and speeding up.
t

See. 5.4: Progress Test I-Part 1 (lower demand)


1. p(x) = rnx +
b. with Sp/& = m (since p is linear). We have Ap/hx = lo/-4 = -2/5 = m. Since $4120) = 200, b = 500, so
p(x) = 500 - ( 5 / 2 ) x . 2. R ( x ) = x(500 - ( 5 / 2 ) x ) . 3. ( a ) R(x) = 500 - 5x = 0 implies x = 100, a rel. max. since R(x) = -5 < 0;
(6) $(loo) = 500 - (5/2)(100) = $250; (c) R(100) = $25,000. 4. C(X) = 6 0 ~ (120 - ~ ) ( 4 0 ) 17,000 = 2 0 ~ 17,000. + + +
+
5. ( a ) P ( x ) = (500x - ( 5 / 2 ) x 2 ) - (2Ox 17,000) = 4802 - (5/2)x2 - 21,800. P(x) = 480 - 5 x = 0 implies x = 96, a rel. max. since
P(x) = -5 <
0; ( b ) P(96) = $1240; (c) p(96) = $260.

Part 2 (higher demand)


1. Ap2/hx = lo/-2 = -5 and p2(120) = 240 = (-5)(120) + b implies p 2 ( x ) = 840 - 5x. 2. R,(x) = x(840 - 5x).
3. (a) R ; ( x ) = 840 - lox = 0 implies x = 84; ( b ) p2(84) = $420; ( c ) R2(84) = $35,280.
4. ( a ) P2(x) = (8402 - 5 2 ) - (20x + 21,800) = 820x - 5x2 - 21,800; ( b ) P ; ( x ) = 820 - lox = 0 implies x = 82; (c) P2(82) = $11,820;
( d ) p2(82) = $430.

See. 5.4: Progress Test 2


1. P,(x) = 500x - ( 5 / 2 ) x 2 - 22,800. 2. P;(x) = 500 - 5 x = 0 when x = 100, a max. value. 3. P,(lOO) = $2200.
4. p,(lOO) = 520 - (5/2)100 = $270. 5. 270 - 20 = $250. 6. 100 tenants get (100)(10) = 1000 and profit increase is
2200 - 240 = $1960.
SECTION 6.2
S-15

Chapter 6: Solutions to Progress Tests

See. 6.1: Progress Test 1

wj 2w3
4. J(w2 + 1)dw = J ( w 4 + 2w2 + 1)dw = J w 4 d w + 2Jw2dw + Jdw = - + -+ w
5 3
+ C. 5 . J x 3 d x = - + C,
x4
4

See. 6.1: Progress Test 2


1. ~ ( t =
) -5, u ( t ) = J(-5) dt = - 5 t 20. + 2. h ( t ) = J(-5t +
2O)dt = - f t 2 20t + +
C,, but C, = 0 since h(0) = 0 (object is hurled
from the sufuce of the moon). 3. u ( t ) = - 5 t +
150 = 0 when t = 30, and h(30) = -$(30)2 +
150(30) = 2250 ft (more than f mi).
4. ( a ) h ( t ) = -$t2 +
l5Ot = 0 implies t = 0 and t = 60. Ball takes 1 full min to return to moons surface. ( b ) u(60)= 150 - 5(60) = -150
ft/s (downward). 5 . dy/dx = 6 x 2 2x + +
C, = 10 when x = 1, so C, = 2 . Also,y = 2 x 3 x2 + 2x + +
C, = 3 when x = 0, so C, = 3.

See. 6.1: Progress Test 3


1
1. Let u = 1 - 5 x 2 , so du = -1Oxdx and x d x = - d u / l O . Jx(1 -
2. Let u = x 2
5 ~ 2 ) 7 d x= J.7
(3
- = --Ju7du
10
=

+ 2x + 7, so du = ( 2 x + 2 ) dx = 2 ( x + 1) dx. Thus Bdu = ( x + 1) dx and J(x2 2x + 7)I3(x + 1) dx = Ju1/34d~=


+
_80_+ c = - (
U8
-805x2)s + C

Ju 1/3 du
1 +
= 343 C = i ( x 2 + 2 x + 7)43 + C. du
= J(2v + 1)-32 du. Let u = 2u + 1, so du = 2 dv. Hence + du = du. Thus

I 2. (a) f ( x ) = 2x is increasing on [0, 11, so lower sums based on left side of intervals. &(J) =
f ( 0 ) (1/10) + j ( O . l ) (1/10) + . . . +f(O.S)(l/lO) + + + + + +
= (1/10) [0 0.2 0.4+ 0.6 0.8 1 1.2 1.4 + 1.6 1.81 = 5/10 = 1/2. +

See. 6.2: Progress Test 2


1. We successively substitute 2, 3 , 4, 5 into 2j2 - 1 and add:
10
+ [2(4)? - 11 + [ 2 ( 5 ) 2 - 11 = 7 + 17 + 31 + 49 = 104. (10) = c ~ z ;
[ 2 ( 2 ) 2 - 11 + [ 2 ( 3 ) ? - 11 2 - 1 1 lo 2(2 - 1) 1
2. ( a ) Sl,(f) = E
l
(
,
)
Z Z 1 a=1
S-16 SOLUTIONS TO PROGRESS TESTS

Sec. 6.2: Progress Test 3


1. Maxima are achieved at left endpoints, minima at right endpoints. F n ( f ) - &(f) = if(,^%-^) (-)b - a b-a
- cf(x2) (-) =
t=1 %=I

b-a [ A a ) -f(b)l(b - a ) , 2. lim -f(b)l(b - = 0 since numerator does not depend on n.


[f(xo) -f ( x , ) l y = n n++m
n

3. Essentially Fig. 6.12 inverted.

See. 6.3: Progress Test 1


1 . M = f ( 4 ) = 64/3, m =f(1) = -7/6, b - a = 7: (-7/6)(7) 5 Jt3((1/3)x3+ ( 1 / 2 ) x 2 - 2 x ) d x < (64/3)(7). 2. ( a ) ( - 3 ) ( 5 ) = -15;
( 6 ) JI(1 - x ) d x = St(l - x ) d x + J;(l - x ) d x = 4 + ( - $ ) ( 2 ) ( 2 ) = 3,
2
(c) J?,(x + 1) dx = J:i(x + 1) dx + J l l ( x + 1) dx = -4 + 4(2)(2) = 4; ( d ) 1;: 4dx = ( 4 ) ( 5 ) = 20 (a rectangle of width 5 , height 4)
3. (a) (2)(2)+ (2)(3) + (1)(4) + (1)(1) + (l)(O) + (1)(4)= 19; (6) ( - 4 ) ( 5 ) + (1)(1) + (1)(6) = -13.
Sec. 6.3: Progress Test 2
1. ( a ) Sxdx = x 3 / 3 + C = F ( x ) : F ( 1 ) - F ( 0 ) = (1/3 + C) - ( 0 / 3 + C ) = 1 / 3 ; ( b ) J(x - 2 ) dx = x 2 / 2 - 2~ + C = F ( x ) :
F ( 4 ) - F ( 1 ) = ( 8 - 8 + C) - (1/2 - 2 + C ) = 3/2; (6) J(1 + 2~ - x) dx = x X - x 3 / 3 +
C = F(x): +
F ( 2 ) - F ( 0 ) = ( 2 + 4 - 8 / 3 + C) - (0 + 0 - 0 + C) = 10/3. 2. 56 - 5a. 3. ( a ) 5 x - 5(0) = 5,; (b) A ( x ) = 5x.
A ( x ) = lim 5x + - 5x = lim - 5 Ax = 5 . Here A( + A x ) - A ( x ) is the area of the smaller rectangle shown.
AX+O AX AX-0 A X

t 0.5

I A %+Ax
4. T h e area of the larger trapezoid determined byf(x), x = 0 and x = b is the area of its
+
triangular part (above the dashed line) (4b)(3b 5 - 5 ) = ;b2 plus the area of its rectangular part (below the dashed line), 5b. Similarly, the
area of the smaller trapezoid determined by x = 0 and x = a is $a2 +
5a. Hence the area we are looking for is (4b2 56) - ($az 5a), which + +
turns out to be F ( b ) - F ( a ) , where F is any antiderivative off(x) = 3x +
5 since J(3x 5 ) dx = ( 3 / 2 ) x 2 5x +
C. + +
t 1.1 = 3x +5

l a b

See. 6.3: Progress Test 3


1. JL1(4x3 - 3x2 - 5) dx = [x4 - x 3 - E I X ] L=
~ (1 - 1 - 5) - (1 + 1 + 5 ) = - 12.
2. J 1(x2 + 1)*(2X dx) = -~
+ 1 (x2 8 1
----- - -7 . (Let u = x 2 + 1, so du = 2x dx and x dx = -.2
2 0 2 3 la-6 6 6

3. 13x3 + 3x + 1)-2(x2 + 1) dx = -I 2 ( x 3 + 3x + 1)-12(3x2 + 3 ) dx = -1 (x3 + 3x + 1)12 2 2


3 1/ 2
I 0
= -[(8
3
+6+ - (0 + 0 + 1)1/2] = 3,
2

[Let u = x 3 + 3x + 1, du = (3x2 + 3 ) dx, so ( x 2 + 1) dx = -.


3

See. 6.4: Progress Test 1


1
1. By the M V T for Integrals there is a c between 2 and 7 such thatf(c) = -[f(x) dx = average value. Now
b-a,
x3 215
i 1 ( x 2 - 2x + 1) dx = - - x 2 + x andf(c) = c2 - 2c + 1, so c2 - 2c + 1 = Thus 3c2 - 6c - 40 = 0,which
3
SECTION 7.2 s-19

Chapter 7: Solutions to Progress Tests

See. 7.1: Progress Test 1


1. ( a ) -1/4 rotation, -90"; (6) 3/4 rotation, 270". 2. ( a ) -n/2 rad; ( b ) ( 3 / 2 ) ~rad. 3. 5v rad, 900". 4. ( a ) ~ / rad;
4
( b ) v / 6 rad; (c) ~ / rad;
3 ( d ) (25/18)v rad; ( e ) ( - 5 / 1 2 ) ~ rad. 5. ( a ) 45"; (b) 15"; (c) 20.05"; ( d ) -2349.13"; (e) 3,552,338.33".

See. 7.1: Progress Test 2


cosx x cos x cos x 2. Fig. 7.12, p. 238. 3. 2n
~-
~-

1
0.87
+
@ -0.71
-0.5 -0.5
0
0.71 @ -0.87 0.5
0.5 T -1 0.71
0 @ -0.87 0.87
p -0.71 1

See. 7.1: Progress Test 3


2. 3. All integral multiples of n.
f
1. n 4. 2d, 4th
-l;

5. All odd multiples of v/12. 6. All multiples of v. 7. 1. 8. 1.

See. 7.2: Progress Test 1


1. f ' ( x ) = [ C O S ( X+~3x3 + 2 x + 1)](4x3 + 9x2 + 2). 2. @/dx = [-sin(x5 - 4x3 - 3x + 1)](5x4 - 12x2 - 3).
3. g'(x) = 5 [sin4(3x2 + x - l)](cos(3x2 + x - 1))(6x + 1). 4. t ' ( x ) = (1/2)(cos(x2 + x ) ) - " ~ (-sin(x2 + x ) ) ( 2 x + 1).
(cos3(5x - 2 ) ) * 2 sin(3x + l)(cos(3x + 1)) * 3 - (sin2(3x + 1)) * 3(cos2(5x - 2 ) ) ( -sin(Sx - 2 ) ) 5
a

5. dr/dx =
c0s6(5x - 2 )

See. 7.2: Progress Test 2


1. y' = - csc[(x - 4)j] * cot[(x - 4)5] * 5(x - 4)4.
dh - [-csc'(x~ + 2x)](3x2 + 2 )
2. f'(x) = 3 tan2(3x7 + 2x4 + 3x - 1) * sec2(3x7 + 2x4 + 3x - 1) * (21x6 + 8x3 + 3). 3. - -
dx zdcot(x3 +24
4. f ' ( x ) = (sec(3x + l))(secz(l - x))(-l) +
(tan(1 - x))(sec(3x +
l))(tan(3x 1))(3) +
5. g'(y) = 5 sec4(3y2 - y)4sec[(3yZ- J ) ~ ] tan[(3y2 - J ) ~* ]4(3y2 - * (6y - 1).

sinx(-sinx) - cosxcosx -(sin2x + cos'x)


- = -csc2x.
sin2 x sin2 x

- -csc
- x cot x . 8. Each deriv. of sin ax introduces an Q factor and each deriv. of order a
sin x sin x
multiple of 4 is an sin ax. Since 100 is a multiple of 4, Dpoo'[sin ax] = a1O0 sin ax.
s-30 SOLUTIONS TO PROGRESS TESTS

See. 7.3: Progress Test 1


x + l x + 1 ( 2 x - 1) + 1
1. (a) f ' ( x ) = -; (b) f ( f - l ( x ) ) =f ( T ) = 2(+) - 1= X, f-l(f(~)) = f - 1 ( 2 ~ - 1) = = x.
2 2

2. ( a ) f - ' ( x ) = m;( b ) f ( f - ' ( x ) ) = f ( m )= (m)3


- 1=x + 1- 1 = x,
f - l ( f ( x ) ) = f l ( X 3 - 1) = V(x3 - 1) + 1 = $2= x . 3. (- 1,O).

See. 7.3: Progress Test 2


1. (a) 7; ( 6 ) 0; (c) 10. 2. ( a ) log,(x2 - 1) = 0 , SO x 2 - 1 = 1, x = ifi; ( b ) x + 1 = log3(81) = 4, SO x = 3;
(6) loglo x 2 + loglo x = 3, so loglo x 3 = 3. Thus x 3 = lo3, and so x = 10. 3.

(x2 + x + 4)1'2(3x + I ) ] = -1og,(x2


1
+ x + 4) + logb(3x + 1) - 610g,(x2 + 1);
2
( b ) ( f o g ) ( x ) = b ( ' 0 g b ( 4 x + 5 ) ) = 4x + 5 . 5. ( a ) -3; ( b ) 6; (c) -5; ( d ) 0; (e) 3; (f)2; ( 9 ) 1/2; ( h ) -1; (i) 0; (j)30;

( k ) 2 ; ( I ) 4. 6. ( a ) 3 (log,-) (1 + x y = [-] l + x ; ( 6 ) log,(52z) = 2 ~ ; 210~z[(r2-4)(r-1)3] = ( ~ -


(6) 2 4)(x - 1)s.

See. 7.4: Progress Test 1

1. ( b ) f ' ( x ) = e2-"( -22); (c) g'(x) = eZ2(12x2 + 1) + (4x3 + x ) e Z 2 ( 2 x ) ; 4


( d ) g ' ( t ) = 2t3+2t* (3t2 + 2) * In 2; (e) - = v Z In T;
(a) f ' ( x ) = -e-"; dx
- e"2" In 2 e T ( 1 - In 2)
(f)4 = 7 i x T - l (the Power Rule!);
2C" ex
( 9 ) s'(x) = - = -(I - h 2 ) ; ( h ) t ' ( x ) = er-'ln2(1 - In2).
(2"12 (2"Y 2"

2 I 0.6931 2.7 0.9933


1.0986 2.71 0.9969

1. f'(X) =
+ 4x + 1
12x3
2. h'(x) =
x(l/x) - Inx - 1 - lnx
-- x 2 . 3. f ' ( x ) =-=-,
1l X 1
+ 2x2 + x + 1
3x4 ' xz lnx x*lnx

4. r'(x) = (In 2)2"'""*(1 + h x ) . 5. f ' ( x ) =


12x2 + 2x - 1 1 6. %
?!! Y (x
= 0, s o y ' = - - # 0)
4x3 + x 2 - x In 3 xy

See. 7.5: Progress Test 1

1.
1
lny = - ln(7x -
2
+ 2x +5 2 1
- x - 11 I
SECTION 8.1 s-2 1

1 1 3 1 8
5 ln(x - 5) + 7 mln(3x + 1) - -ln(8x
2
- 1)
1, ThusJ(x) =
3x+1 2 8x-1
3. lnf(x) = x In x , sof(x)/f(x) = x2 ( l / x ) + (In x ) 2 x = x + x(2 In x ) = x(1 + In 2).Hencef(x) = x(.)(x)(l + In x 2 ) = x(s2t1)(l + In 2).
See. 7.5: Progress Test 2
1. f ( x ) = em(x + 1) = 0 when x = - 1 andf(x) = e(x + 2) = 0 when x = - 2. f( - 1) = e > 0, so we have a relative minimum at
x = - 1. The sign off(x) changes from minus on the left of x = - 2 to plus on the right. Hencef(x) is concave down to the left and concave
up to the right of x = - 2 , making x = - 2 a point of inflection. Nowf(- 1) = - l/e andf(-2) = -2/e2. For x < - l,f(x) is decreasing;
that is, as x decreases, f ( x ) increases. f ( x ) . being the product of a negative and a positive number, is bounded above by 0 in ( - cc, 0) and is
asymptotic to the negative x axis

+
2. f ( x ) = 1 cos x 2 0 for all x withY(x) = 0 when
cos x = - 1, that is, when x = v , 3v, - v , in fact all odd multiples of is. These are steps, but not relative extremes sincefis never
decreasing. For such values of x , x = (2k +
l)a,f(x) = (2k +
1)v for any integer k . A

See. 7.5: Progress Test 3


) (0.4)(0.2 cos x ) + 2[4 (0.2 sin x)(0.2 cos x ) ] = 0.04(2 cos x
1. Cross-sectional area equals ~ ( x = +sin x cos x )
o,2 sin xx ~ ( x =
) 0.04( - 2 sin x - sin2 x cos2 +x ) = 0.04( - 2 sin2 x - 2 sin x + 1) = 0 when

0.4
sin x = (fi <
- 1)/2 z 0.366 (ignore negative solution since 0 5 x 5 v/2). Thus x z 0.3747 rad [c(0.3747) 01. 2. Given dx/dt = 50
when x = 300. Need dO/dt. We know x / 3 0 0 = tan 8. Diff. implicitly with respect to t : (1/300) dx/dt = sec 8 (dflldt). When x = 300, 8 = v/4,
and sec8 = 2 / f i . Hence dO/dt = (1/300)(50)(1/2) = 1/12 z 0.0833 rad/s. f t Y
\ X
\

Radar

Chapter 8: Solutions to Progress Tests

See. 8.1: Progress Test 1


I . ( a ) 20,000 e20k = 50,000 implies k = (In 2.5)/20 z 0.0458, so A ( t ) = 20,000 e0.0458t; (6) 20,000 e0.0458t = 100,000 implies t z 35.14 years.
-
2. ( a ) A = 2t-1 = e(h2)(t-1) e0.693(f-1);(6) ( 2 ) converting dollars to cents, solve 100,000,000 = e0.693(f-1) for t , getting
t = [ln(10s)]/0.693 + 1 = (8 In 10)/0.693 + 1 z 26.58 + 1 day-you become a millionaire after the 27th day; (ii) solve l o l l = e0.693t-1 for t ,
getting t z 37.55 days; (izi) t z 47.5 days. 3. A ( t ) = 5en12)t = 5(4)t.
s-22 SOLUTIONS TO PROGRESS TESTS

See. 8.1: Progress Test 2


1. P(1) = 2Co = Coe*-I implies i = In 2 z 0.69 (69 percent). 2. i = 1 yields P ( t ) = Cod z C0(2.7183),with P(1)as needed.
3. 60 = (In 2 ) / k implies k z 1.155 percent.

See. 8.1: Progress Test 3

1. s0
20 50000
50000e-0.08tdt = -[e-1.6 - 11 z $498,815.
- 0.08
2. s
0
20
dt z $209,833.
35000e-0~08t

See. 8.2: Progress Test 1


1. Let u = sin 3x, du = 3 cos 3x dx; then J cos 3x esln3 dx = 3S.P du = QeU + C = $esm3 + C. 2. Let u = eZx, du = 2eZx dx. Then integral
becomes J csc u cot u du/2 = (-csc u ) / 2 +
C = (-csc e2)/2 C. +
3. Let u = sin x, du = cos x.
S(1 - sin2x) cos x dx = S(1 - u2) du = u - u3/3 + C = sin x - (sin3x)/3 + C. 4. Let u = x + 5, du = dx. Then
Jdx/(x +
5) = Jdu/u = In IuJ C = In Ix + +
51. Thus Jj4dx/(x +
5) = In 10 - In 1 = In 10.
5. A = J Z s i n x d x = -cosxiZ= - c o s ~ - ( - c o s O ) = 2 .

See. 8.2: Progress Test 2

1. Let u = 2$, du = (In 2)2* dx, 2 dx =


In 2
~ ( C S C2) dx = s c s c u -du = In /csc2 - cot 2/
In 2 In 2
+ c.
tan2x
2. J tan x(sec2x - 1) dx = -- In lsec X I
2
+ C. 3. Let u = ex, du = ex dx: J tan u du = In /secul + C = In lsec el + C.
4. .f cot x dx = s-
sin x
dx = s$ (with u = sin x) = In 101 + C = In Isin x / + C. General result follows by Chain Rule

See. 8.3: Progress Test 1

1. Case 11. J(
- 7 2x
dx = s(1 - 2 cos 2 x + cos2 2x) dx = 1
1
- 2 cos 2x + -(1
2
+ cos 4x) 1
3
dx = -x
8
1 .
- -sin 2x
4
1 .
+ -sin
32
4x + C.
2. ~ x dx = sin x
Case I. S(1 - sin2x ) cos
2 sin3x
-- 3
sinjx
+- 5
+ c.
du
3. Case I. J(sin 3 ~ ) ~ / -~ sin2
( 1 3x)(cos 3x d x ) u = sin 3x, - = cos 3x dx
3
2 2
JW(I - u2)- du = 2
2 312
- -U 2 7/2
+ c = -sin3/2 3x - -sin7/~ 3x + c,
3 3.3 73 9 21
1-cosx~1+cosx 1
dx = -J(1
4
- COSX) dx = i s ( l - -[1
1
2
+ cos 2x1) dx = -x81 1
- -sm 2x
4
.
+ C.

See. 8.3: Progress Test 2


tan6 x
1. J tan5 x(sec2x - 1) dx = .f tan5 x sec2x - tan5 x dx = -- J tan3 x(sec2x - 1) dx =
6
tan6 x tanex tan4x tanex tan4x tan2x
--
6
J tan3 x sec2x + J tan3 x dx = -- -+ J tan x secz(x - 1) dx = -- -+ -+ In lsecxl
6 4 6 4 2
+ C.
2. .f sec-ll3 x tan2 x(sec x tan x dx) = J ~ e c - l x(sec2
~
3 sec83 x
x - 1) sec x tan x dx = ___ - ___
8
3 sec2/3x
2
c. +
3. J ( 1 + c o t 2 x ) c s c 2 x d x = -cotx-- cot3 x
3
c. +
See. 8.4: Progress Test 1
1. v/6 is in [-57/2, 57/21 and sin 57/6 = 1/2. Thus arcsin 1/2 = 57/6. 2. -v/6 is in [-57/2,57/2] and sin(-n/6) = -1/2, thus
arcsin( - 1/2) = -57/6. 3. Since 57/8 is in [ - m / 2 , n/2], arcsin(sin 57/8) = 57/8. 4. sin(arcsin 0.012) = 0.012, 5. Note that 2 ~ / 3
is not in [ - n / Z , 57/21, but 57/3 is, and sin(2vi3) = sin 57/3; thus arcsin(sin 2v/3) = arcsin(sin ~ / 3 )= n/3.

See. 8.4: Progress Test 2


1. -v/3 is in (-57/2, v/2) and tan( -57/3) = - fi,so arctan(- fi)= -57/3. 2. Needy such that secy = (-2) andy is in [0,57].
But sec 2v/3 = - 2 a n d y is in [O,v],so arcsec(-2) = 257/3. 3. Now s e c ( - ~ / 4 ) = sec.x/4 = d ,and so
arcsec(sec(-57/4)) = arcsec(sec n/4) = 57/4, 4. Xow t a n ( l 9 ~ / 6 )= tan v/6, so arctan(tan 1 9 ~ / 6 )= arctan(tan v/6) = 57/6.
5. arcsec x is def. only for 1x1 2 1, so arcsec(4) is undef. (There is noy such that secy = 1/2 since lsec XI 2 1 for all x.)
SECTION 9.1 S-23

See. 8.4: Progress Test 3

4 -- D,(2x2 - x + 3) - 4x -1
1. -
dx dl - ( 2 x 2 - x + 3)' d -4x4 + 4x3 - 13x2 + 6x - 8

See. 8.5: Progress Test 1


dx dx
1.
1 1 1
) + c. 2. 1 - 2x - x2 = 2 - (x + 1 ) 2 , so
dx dx
= J 42 - (x + 1)2 . Let u = x + 1, du = dx; then
dx

3/2 1
(i)A(2)
-
3 4
1
= -arctan
3

See. 8.5: Progress Test 2


1 2x
1. Let u = e Z X ,du = 2e2" dx. Then = - arcsin JL + c = -1 a r c s i n c + C.
fi 2 6
2. J(x3 + 3x2 - 5x - 17) dx + s3'" + x2
+ 120dx = x4/4
7
+ x3 - j
.
2
'- 17x + (37/2) ln(x2 + 7) + (12O/fl) arctan(x/fi) + C.

See. 8.6: Progress Test 1

1. 1 - t a n h 2 x = 1 - - - cosh2x - sinh2x -
sinhx
(cosh x ) - cosh2x
- (&r = sech2x. 2. DJcoshx) = D, (e"
--
e-") - ; ~
ei ; - sinhx,
e-r
so

[8.6.4(b)] follows by Chain Rule. 3. Result follows since coshx + sinhx = e x . 4.


1
cothx = - = 2, sech2x = 1 - (1/2)2 = 3/4, so
1/ 2

sech x = $ / 2 , (Note sech x


1
> 0 for all x . ) cosh x = - - - sinh x = \/cosh2 x - 1 = m,(Kote that tanh x = 1/2 implies x > 0, so
?5/2 IK
tanh3 x
sinh x > 0.) 5 . J(l - tanh2x ) sech2 x dx = tanh x - -
3
+ c.

See. 8.6: Progress Test 2

1
- arctanh -
3x-2 if 3 x - 2 <2
-dx dx 2 2
2. J---
9x2 - 12x = s 4 - (3x - 2)2 =[ -
1 arccoth -
3x - 2 + C i f 3 x - 2 >2
2 2

3. Here u = sin x , du = cos x dx, a = 1.


Jw du = arcsinh(sin x ) +C

--
Chapter 9: Solutions to Progress Tests

Sec. 9.1: Progress Test 1


1. 01

3
5 2
2. (a) 0 < lx - 21 < 3, ( b )
3. (a) If lx - 2 < 0 1. then
-
5
2

1~
b

6
- 21 < 001, (c) 0 < I X
I(4x - 1)
9
31 < 5
12
+
- 71 = /4x - 81 = 4/u - 21 < 4(0 1) = 04, (6)
S-24 SOLUTIONS TO PROGRESS TESTS

See. 9 . 1 : Progress Test 2


1. Given E > 0, we need 6 > 0 such that 0 < ix - ( - l ) i < 6 implies l(1 - 2 x ) - 31 < E. But l(1 - 2x) - 31 = 2 ) x + 11 = 21x - (-1)1 < E

if / x - ( - 1 ) i < ~ / 2 Choose
. 6 = ~ / 2 [See
. sketch 1.)

iI li Pli,j f ' x ) = 1 - 2x

-1 + s'
2. Suppose E > 0 is given. Then we need 6 >
0 such that 0 <
1% - 21 < 6 implies l ( x 2 - 4)/(x - 2 ) - 41 < But 0 < Ix - 21 implies
E.

x # 2 ; for such x, ( x 2 - 4)/(x - 2 ) = x + 2. so we need 6 >


0 such that .(I + 2 ) - 41 < But l(x + 2 - 4)l = lx - 21, so 6 = works. (See
E. E
sketch

xz - 4
~

x - 2

3. SupposeJ[x) is defined for all x in some open interval (a, c). We say the limit off(x) as x tends to a from the right is L , and we write
lim,,,+f(x) = L , if for every E > >
0, there is a number 6 0 such that a x < <
a + 6 implies If(x) - L ( E. 4. For x 0, < >
f(x) = x + 2.1. If E >
0, then we need 6 >
0 such that 0 x < < + < +
6 implies 1(x 2.1) - 2.11 E . But i(x 2.1) - 2.11 = 1x1, so 6 = E works.

Sec. 9.2: Progress Test 1


1. Let> 0 be given. Since lim2-J(x) = L , there is 6 > 0 such that 0 < ( x - a ( < 6 implies) . ( f i - L ( < But
E E.

llJ(x)l - Ll, so 0 < lx - a1 < S implies Ilf(x)l - ILIl < also.


- (LI))5. ( f I 2. By (1) lim,,,f(x)
E = 0 implies lim2-a l f ( x ) i = 0.
Conversely, assume 1im2-, If(x)l = 0 and let > 0 be given. Then there is 6 > 0 such -
E that 0 5 lx - a1 < 6 implies iif(x)l - 0 1 < But then E.

less than 6, = a / 2 implies (i) x > a / 2 , which implies fi> m; (c) k < 6+ fiimplies -1 > 1
k &+ fi"'
lx - a1 lx - a1 Ek
k
> 6+fi
=1 6- 6 1; ( d ) 0 < J x - a1 < 6, = Ek implies: (ii) lfi - &I <- =
k
E; ( e ) Choose 6 = smaller of 6, and 6, so

both (i) and (ii) hold, proving limz-, fi = 6.

Chapter 10: Solutions to Progress Tests


See. 1 0 . 1 :Progress Test 1
1. ): = 2 x 2 ~ x=
2 i):e x z = 4(&. From Table 1(c) b, = 4,vertex = (0, 0), focus = (0,B).directrix:): = -8.

x
SECTION 10.2 S-25

2. y 2 = 4x = 4(1)x. From Table l(a) p = 1, vertex = (0,0),focus = (1,0), directrix: x = -1.


x = -1

3. y 2 - 4y = -82 - 2 8 0 y 2 - 4y + 4 = - 8 x - 2 4 o ( y - 2)2 = -8(x + 3 ) 0 ( y - 2)' = -4(2)(x + 3)- Y 2 = -4(2)X, where


=Y - 2, = x + 3. From Table l ( b ) p = 2 and in X - Y system, vertex = (0,0), focus = (-2, O), directrix: X = 2. Thus in X - Y system,
vertex = (-3, 2), focus = (-5, 2), directrix: x = 0. 4
,y= -31 ( 4

4. Since the focus is 3 units above the vertex on the line x = 4, the directrix is 3 units below the vertex and so is the liney = -6. Since
p = 3 and the parabola opens upward, its equation is X 2 = 4(3)Y, where X = x - 4 and Y = y +
3. That is, ( x - 4)' = 12(y 3). +

See. 10.2: Progress Test 1


1. Since the length of major axis is 16, 2a = 1 6 = a = 8 and since c = 4, b2 = 64 - 16 = 48. Hence equation is x2/64 +y2/48 = 1.
2. +
( x - 8)2/64 ( y + 6)2/48= 1. 3. Factor out 16 and 25 and then complete square as indicated:
16(x2 - 6x + 9) + 25(y2 - 8y + 16) = 106 + 16 *9+ 25 * 16 = 400, or (x = 3)2/52 + ( y - 4)2/42 = 1. Hence center (3, 4), foci at (6,4),
(0,4).(See sketch.)

See. 10.2: Progress Test 2


9x' - 36x - 4y2 - 89 = 11209(x2 - 4x ) - 4 ( y 2 - 2y + 4) - 4(yZ - 2y + 1) = 112 + 36 - 4 =
) = 1 1 2 0 9 ( x 2 - 4.7
1 4 4 o ( x - 2)2/16 - ( y - 1)2/36 = 1. Thus a = 4, b = 6, and c = d m = m=2 f l . Center (2, l), foci (2 + 2 f l , l),
(2 - 2 m, l ) , vertices (6, l ) , ( - 2 , 1). See sketch next page.
S-26 SOLUTIONS TO PROGRESS TESTS

See. 10.3: Progress Test 1


1. ( f i / Z ) , ( - 1/2), (i,fi/2), (2,O), (v5,l), (( v5 + 1)/2, (v5- 1)/2). 2. Y - 2 = 4x. 3. Y* - fixr +x+ fiY = 10.

See. 10.3: Progress Test 2

Since A = 8, B = -4, and C = 5, we apply (10.3.6.) to obtain cot 2~ = -3/4,


1 - 3/5 1
cos 2a = - 3 / 5 , so sin N =
F~

A
= - and

. Let x = (1/&) ( X - P Y ) , y = (1/&) (22 + Y ) , substitute a n d simplify to get 4X2 + 9 Y 2 = 36, or


X2/9 + Y 2 / 4 = 1. T h e graph is a n ellipse with major axis on the X axis.

Chapter 11: Solutions to Progress Tests

See. 11.1: Progress Test 1


1. u = x , dc = ex dx; du = dx, u = ex: Jxex dx = xex - S e x dx = xex - ex + C.
2. u = x , du = sec2 x dx; du = dx, u = tan x : Jx sec2 x dx = x tan x - J tan x dx = x pan x - In /sec x / + C.
dx
3. u = arcsinx, du = dx: du = ~
x arcsin x + + C.

See. 1 1.1 :Progress Test 2


cos(1n x ) sin(1n x ) dx
1. u = sin(1n x ) , du = d x ; du = ~ dx, u = x : J sin(1n x ) dx = x sin(1n x) - J cos(1n x ) dx; U = cos(1n x ) , = d x ; di2 = -
X
-
u = x : J sin(1n x ) dx = x sin(1n x) - [ x cos(1n x ) + J sin(1n x ) dx] = x sin(1n x ) - x cos(1n x ) - J sin(1n x ) dx. Thus
J sin(1n x ) dx = z [ s i n( l n x ) - cos(1n x)] + C. 2. u = In x . du = 3x3 + x; du = -, dx 3x4
u = - + 5.

s(3r+ a) 3r
2 x 4 2
J(3x3 + x ) In x dx = [- + -
]: 3i4 In x - - [
- dx = - + K 1n x - 16-
3x4 x2 + C. 3. u = x 2 , du = sin 2x dx; du = 2 x dx,
1
u = -- -XZ
cos 2x. J x 2 sin 2x dx = -cos 2 x + -21. cos 2 x dx. U = x , d5 = cos 2 x d x ; d L = dx,U = 1 sin 2 x .
2 2 2 2

x 2 sin 2 x dx =
X2
- - cos
2
2x + 52 sin 2 x - -11sin. 2 x dx = - -
X2
cos 2x + sin 2 x
1
+- cos 2~ + C.
2 2 2 4

See. 1 1 . 2 : Progress Test 1

1. Let x = fisin 8; dx = ficos 8 do, and d m = d7 - 7 sin2 8 = d m = ficos 8.


s- x 3 dx
-
- 7fisin38 ficos8d8
ficose
-

7fiJsin30d0 = lfiJ(1 - cos28)sin0dO = 7 f i


---)+c=
vT=7 (7 - x2)32
- 7v7=-2+ c
fi 3
2. Let x = 2 sin 8, dx = 2 cos 8 d8, and = d4 - 4 sin2 0 = d 4 a = 2 cos 8. x = 0 implies

Ji - x2
SECTION 11.3
s-27

0 = arcsin(0/2) = 0 and x = 2 implies 8 = arcsin(2/2) = rr/2. Thus fi dx = JZ/22 cos 0 * 2 cos 8 d0 = 4J{/2 cosz 8 d0 =
4 / 2 J ; / 2 ( 1 + cos 20) d6 = 2[0 + (1/2) sin 26'];/2 = 2 ( ~ / 2+ (1/2)sin T) - (0 + (1/2)sin 0) = T.

See. 1 1.2: Progress Test 2


1. Let x = 4 sec 8, dx = 4 sec 8 tan 0 d0, d n = 4 tan 0. J d n dx = J 4 t a n 0 4 sec0 t a n 0 d0 = 4J tan2 0 d0 = 4(tan 0 - 0 ) +
X 4 sec 0

4
2. J m d x = J d 9 - ( x - 3 ) 2 d x [ ( u = x - 3 ) ] = J ~ d u [ ( u = 3 s i n 0 , d u = 3 c o s 8 d 0 ,
+
cos 281 d8 = :[8 + 4 sin 201 + C = $[0 + sin 0 cos 6'1 + C =

/Iu
= 3 cos 8)] = 9 1 cos' 8 d8 = 4J[1

4 arcsin (+) +
U r n
+ c = 4 [arcsin (q)
+
(x - 3 ) V , i i c 7
9 l+c.

J c 2
3. Let x = u tan 8, dx = a sec2 0 d0, then x2 + u2 = u2 tan2 0 + u2 = a2(tan28 + 1) = u2 sec28.

See. 11.3: Progress Test 1


x5 - 8x3 - 8x +1 x+l x + l
+ 3)(x - 3 ) . Now x(x + 3)(x - 3 ) -- -x +-x + 3 +--x - 3 -
A, A, A3
1. =x2+1+
x 3 - 9x x(x
A,(x + 3)(x - 3 ) + A ~ ( -x 3 ) + A+(x + 3 ) , x + 1 = A l ( x + 3 ) ( ~- 3 ) + A ~ x ( x- 3 ) + A3x(x + 3). NOW
x ( x + 3)(x - 3 )
SO

J(x2 + 1)dX + J[--.- 1


9 x
1
- -'-1 1
9 x+3
+ -*-]dx
2
9 x-3
1 = x3 1
+ x - -1nJxl 1
- -1nJx 2
+ 31 + -1nIx - 31 + C.
9 9 9
4x + 2 - 2x + 1 2x + 1 + A,(x + 3 )
2.
2 x 2 + 4x - 6 x2 + Z X - 3
-
-
(x + 3)(x - 1 )
-
x
A,
+3
+--x A-, 1 - A,(x ( x-+1)3)(x - 1)
I So 2 x + 1 = A,(x - 1) + A2(x + 3 ) and,
letting x = - 3 , + 1 , we get -5 = 3A,, -1 = 4A,, so A , = 5/4, A , = 3/4, with J F A+4 x + 3
L.']dx
4 x-1
5
= -1nIx
4
3
+ 31 + -1nIx
4 - 11 + C.
~~

See. 11.3: Progress Test 2


x4 + x
1. -- -x+-
x2 +1 . NOW
x2 +
1 - A, A,
- - + 7+ --
A,
-
A,x(x - 1 ) A,(x - 1 ) + + A,x2
, so
x3 - x2 XZ(X - 1)
~

xZ(x - 1 ) x x x - 1 xyx - 1)
x2 + 1 = A,x(x+ A2(x - 1 ) + A3x2. Then x = 0 + A , = - 1, and x = 1 A , = 2. Then for x = - 1 (a number # 0, # l ) , we get
- 1)
-x x2y x +- 11 ) dx = J x dx + ~ [ ~ - ~ +1 _ i ] d2 x = I - 1 n ~ x ~ + - 1+ 2 1 n ~ x l- I + C .
2 = 2A, - 2A2 + A , = 2A, + 4 , so A , = - 1 , J
X2

See. 11.3: Progress Test 3

1.
x+l
-- A ,
-
A2x A , - A,(z2
-
x 1) +
(A, x + + +
A3)(x - 1) + +
, so x 1 = A1(x2 + x + 1 ) + (A,x A3)(x - 1). +
[x - l)[x2 + x + 1 )
I _
x - 1 x2+ x + 1 +
ix - 1xx2 x 1) I~ + +
x = 1 implies 2 = 3A,, so A , = 2 / 3 , with two unknowns yet to determine. Substitute any two numbers # 1 , say x = 0, x = - 1 , to obtain
1 = A , - A,, 0 = A , + ZA, - 2A,. Now A , = 2/3 implies A , = A , - 1 = 1/3, and A , = (2A3 - A,)/2 = -2/3. Thus
( x 1)dX + = 1 - J[5- _.
] d x = ? 2l n l x - l l - - l n 1( x ' + x + l ) + C ,
J, - 1)(x2 + x + 1) 3 x-1 3 x2+x+1 3
5-28 SOLUTIONS TO PROGRESS TESTS

See. 11.3: Progress Test 4

1. j4 + 5x3 + lox2 + 8 x + 1 - A , A,x + A, + A,x + A , -


-
- 1)(x2 + 2x + 2)*
(x x - 1 + x2 + 2x + 2 (x2 + 2x + 2)2

8x + 1 = ( A , + A&, + (4A, + A , + A,)x3 + ( 8 A , + A , + A 4 ) x 2+ ( 8 A , - 2 A , - A, + A & + ( 4 A , - 2A, - A,). Hence

A1 + A* = 1
4A1 +
A, + A3 = 5
8'41 + A, A, += 10
8A1 - 2AZ -A,+Aj= 8
4'41 - 2A3 -A5= 1.

This system solves to yield A , = 1, A , = 0, A , = 1 , A , = 1, A , = 1, S O s (x4 + 5 x 3 + lox2 + 8x + 1 ) dx


(x - 1)(x2 + 2x + 2)*
-

1 1 x + l
J[x+ + 2 + ( x 2 + 2x + 2)2
+ x* 2x
dx = ln/x - 11 t

1
arctan(x + 1) - -
2X2+2X+2
+ c.

See. 11.4: Progress Test 1

1. Let u = m;then u2 = x + 1 , so 2u du = dx and x = u2 -


l. Jx( +q- 2 )
dx - 2udu -
J(,*- l ) ( u - 2) - J,
2u du
+ l)(u - 2)'
- l)(u
2. Let u = (2x + l)Il6; then u6 = 2x + 1 and 6u5 du = 2 dx. Also, u3 = (2x + 1),16 = d m and u2 = ( 2 x + 1),16 = q m . Thus
u8 du

See. 11.4: Progress Test 2

1. Let u = m,
Then u2 = 2 - x2 and -udu = x d x ; -

See. 11.4: Progress Test 3

2. Let u = m.
Then u2 = 1 + ex, u 2 - 1 = e", so ln(u2 - 1) = x and dx = -
-1 u2
du

3. Let u = d m . Then u 3 = 3x5 + 5 , so 3u2 du = 15x4 dx and dx = s f / m x 4 d x


x5
=
u * 1/5 u z d u
S(,3 - 5)/3

See. 11.5: Progress Test 1

Numbers in brackets refers to the appropriate formula. 1. [20] with a = 2, b = 3: -.- I+ C. 2. [73] with

n = 4, u = x :
-sin3x cos x
4
+ qSsin2
3 dx = -sin3 x cos x
4 +-(43 -sin 2 x cos x
+ T1 s d x ) = - sin34x cos x - 3 sin x8 cos x + -83x + c.
SECTION 12.1 93-29

See. 11.6: Progress Test 1

1.
ti
(a) = + 0.8333 + 0.7143 + 0.6250 + 0.5556 + ""1 2
= ;[3.4782]
2
= 0.6956; ( b ) f " ( t ) = -
t3
1.0 1.0000
1.2 0.8333
1.4 0.7143
1.6 0.6250
1.8 0.5556
2.0 0.5000
(113. 2
reaches its max. on [ l , 21 at t = 1, withf"(1) = 2 , so Iln 2 - T51 5 -z 0.0067; (c) to 4 places, In 2 = 0.6931, so Iln 2 - T,l = 0.0025
12 25 9

1
[Notef(t) = - is concave up, so T, is an ouerestimate of In 2.1 2. M = 2 , b - a = 1, so we need a positive integer n such that
t

3. (a) x, 1
,
0.0 1 0.2 0.4 0.6 1 0.8 ~ 1.0 1
1
1.2 1.4 1.6 1.8 2.0

See. 11.6: Progress Test 2


1. ( a ) Heref(t) = l / t , xo = 1, x1 = 1.25, x2 = 1.5, x3 = 1.75, x4 = 2 , so
l n 2 = 1 / ( 3 * 4 ) [ 1 + 4 . 4 + 2 * -2+ 4 * - + +4] = - [ 8 . 3 1 9 015 ] =0.69325; ( ! ~ ) f ' ~ ) = E idecreasingon
s [ 1 , 2 ] , ~ o i M = f ' ~ ) ( l=) 24. By
5 3 7 2 12 t5

(1 1.6.6), Iln 2 - 0.693231 5 --


l 5 * 24
~

180*44 1 5 * 2 * 4 3
1
- 0.00053; (c) (through 5 places) Iln 2 - 0.693251 = 1-0.00010~ 0.00053. <
2. (a)
24
b - a = 1, M = 24, so we need - 0.000005, or ___ < < n4, or 26666.7 < n4, This implies n > 12.8. Since we need an even
180n4 0.000075
24 2 1016
integer, choose n = 14 (considerably less than the n = 183 for the T R ) . ( b ) We need - <5 * or - < n4, or (2.66)1014 < n4 or
180n4 15.5

g*< lo4 n, which makes n > (0.4041)104, or n > 4041. Choose n = 4042. 3. (a) We use thef(x,) from Sec. 11.6, Progress Test 1, to

2 12 * 2 5
obtain J ' e P ' d x
30 180.104
=
z 0.88258; ( b ) error 5 - 0.000213, which implies 3-place accuracy; (c) The T R guarantees 2-place
z -[13.2387]
accurac!. Althoughf(x) = e c Z 2 has a pt. of infl. in [0, 21, so much of the T R error cancels, SR still yields greater accuracy.

Chapter 12: Solutions to Progress Tests

See. 12.1 :Progress Test 1


1. t = x2, soy = x 2 - 4 (a parabola). Since t 2 0, we know x 2 0, so graph is rt. branch of parabola. 2. y = 4 tan' t = 4(sec2 t - 1)
and sec t = x/2, s o y = 4[(x/2)' - 11 = x 2 - 4. Nowy = 4 tan2 t 2 0 for all t. As t increases from t = 0 toward ~ / 2 x, a n d y both increase
without bound from ( 2 , O ) . 4 t t = v / 2 , x a n d y are not defined, but as t increases beyond ~ / to2 t = T, x a n d y decrease, with (x,y) moving
down the left side of the parabola to (-2, 0). 3. t = x + 1, soy = 2 ( x + 1) + 3 = 2 x + 5. No restrictions on t , so graph is entire

I (0,-4)
S-30 SOLUTIONS TO PROGRESS TESTS

See. 12.1:Progress Test 2

1. x(t) = et,y(t) = -&, (%(In3),y(ln 3 ) ) = (eln3, ech3) = (3, 1/3). mo = --


y(h 3) -e-ln3
-
-
-
- 113 = (- f).Equation of tan. line is
x(h 3) eln3 3
y - 1/3 = - ( 1 / 9 ) ( x - 3). 2. dx/dt = - 2 csc2 t , @/dt = 4 sin t cos t = 2 sin 2t. Now dx/dt is never zero, so no vert. tangents, but
< <
= 0 when 2t is a multiple of v , Since 0 t v , t = v / 2 . Now x(v/2) = - 2 , so a horiz. tangent at t = v / 2 , at the pt. (0, 2).
dy/dt
3. At t = 0 x ( t ) , hence x ( t ) , does not exist.

See. 12.2: Progress Test 1


1. x(t) +
= - e t sin t + et cos t, y(t) = et cos t + e t sin t ; [x(t)l2 [y(t)I2= eZt sinZt - 2et sin t cos t + eZt cos2 t + ezt cos2 t + 2et sin t cos t +
eZt sin2 t = 2ezt(sinZt + cos2 t ) = 2eZt. Therefore s = fiJ;et dt = fietl: = fi(es - 1 ) .
2. I34=dx = s1 3 4 8xm d x = s3(:+
1 4 ) d x = 12 2, 3. ~ : j 1[+(ey
+ + e-u)Iz4 =

See. 12.2: Progress Test 2

1 2
2. 2 v ~ z y 3 d m d =y 2v.-.- [(l + 9y4)31z]g= - 11.
0 36 3
3. ZvJg(2 + cos t ) dsin2 t + cos2 t dt = 2v[2t + sin t]: = 4 v 2 .

See. 12.3: Progress Test 1


1. ( a ) Total moment = 36; ( b ) X = 36/26 = 18/13. 2. ( a ) Mu = 36 [from l.(a)]; ( b ) M , = 64; ( c ) X = 18/13 [from l.(b)],
r= 64/26 = 32/13. 3. ( a ) 36 + 600 = 636; ( b ) 636/26 = 318/13. +
4. ( a ) My= ( 1 ) ( 1 ) ( 1 ) ( 2 ) + ( l ) ( 4 )+ ( 1 ) ( 5 )= 12,
+ + +
M , = (1)(4) + ( 1 ) ( 5 ) + ( 1 ) ( 7 )+ (1)(16) = 32, M = 4 , X = 12/4 = 3 , j = 32/4 = 8 ; ( b ) M , = ( 2 ) ( 1 ) ( 2 ) ( 2 ) ( 2 ) ( 4 ) ( 2 ) ( 5 )= 2 - 1 2 = 24,
k.12 k * 32
M,=2.32=64,M=8,X=24/8=3,y=64/8=8; (6) M u = k . 1 2 , M , = k . 3 2 , M = 4 k , x = - - - 3,y= 4 k= 8.
4k

Spe. 12.3: Progress Test 2

1. M , = J 4 x ( 4 x - x z ) dx = 6 4 / 3 ; M , =
0
-s
1
2 0
4 (4x
30
32
- ~ ( 4 x - x 2 ) d x = - , H e n c e x = - = 64/3
- x 2 ) Z = -.5 1 2 A -
3
2,~=--
3213
512/30
32/3
- -8.
5

See. 12.4: Progress Test 1 J d

1. k = 30/2 = 15. W = J:15x dx = 450 in.-lb. 2.

6 [ F @ - y ( 2 ; ;
SECTION 13.1 s-31

( a ) T h e right side boundary of the cross section has equationy = 3x, SO x = y / 3 is the radius of a typical disk,
which thus weighs 50 * (18 - y ) ; (c) W = 18y2(18- y ) 4 = 5 0 ~ ( 9 7 2 )= 192,681.4 ft-lb

3. JA0""(x + 400) dx = 6500 ft-lb. [With x feet remaining to lift, (5x + 400) Ax is approximately work necessary to lift chain and weight Ax
feet.]

See. 1 2 . 4 : Progress Test 2


1. Equation of rt. side is) = 3 x , so x =f(y) = y / 3 , and left side has equationy = - 3 x , so x = g ( y ) = -y/3 Hence total force is
5 0 1 , 6 ( ~ / 3- (-y/3))(6 - ~ ) d y = 50(24) = 12001b. 2. ( a ) 62.4J:E30(18 -y)dy = 62.4(135) = 84241b;
+
(6) key is new depth 18 instead of 15 ft. F = 62.4[JA0 20(18 -9) 4 J";:Zy(18 - y ) 4 + 1::
30(18 - y ) 4 1=
62.4[2600 + + +
1351 = 212,264; (c) 212,264 - (143,000 8424) = 60,840 lb, almost 43% increase.

See. 12.4: Progress Test 3

1. T h e total wt. is 50 times volume = 50 [:fjZ * 181 = 50(216~).T h e centroid is (3/4)h = 3/4 * 18 = 27/2, so oil is lifted 9/2 ft, making total

work = (9/2)(j0)(216~) 152,681.4 ft-lb.


2. Centroid is at (1/4)18 = 9/2, so oil must be lifted 18 - 9/2 = 27/2 ft. Total work = (27/2)(50)(216v) 458,044.2 ft-lb
3. Area = 12 ft2, centroid is a t depth (1/3)(6) = 2 ft. Pressure at 2 ft is (50)(2) = 100, so total force is 1200 lb.

Chapter 13: Solutions to Progress Tests

See. 1 3 . 1 :Progress Test 1

( b ) ( fi,-97/6) = (6, - 97/6 & 2nv) = ( - fi,597/6 i 2 n ~ ) ;


(c) ( - 2, 797/3) = (- 2, v/3 & 2 m ) = (2,497/3 -C 2 m ) ; ( d ) x = ficos( -a/6) = 3 / 2 , y = v'? sin( -97/6) = - &/2 and
x = - 2 cos(77i/3) = - 1 , y = - 2 sin(7v/3) = - fi.So P has (3/2, - fi/2) and Q has (-1, - fi)as r.c. 2. Since x = 1 , y = v'?, we
have r2 = x 2 + y 2 = 4 and t a n 0 = y / x = fi,Since (1, fi)is in 1st quad., take 8 = v / 3 , and so r = 2 . Thus R has p.c. (2, 97/3), or
( - 2,4T/3).

Sec. 13.1: Progress Test 2


1. Since cos( -8) = cos 0, graph is symmetric about x axis and period of cos 0 is 297, so only need consider 0 50 5 7i

0: 0 + T/4 97/4 + T/2 97/2 + 3T/2 3 ~ / 2+


coso: 1 + l / f i 1/*+ 0 o+ -l/fi -l/fi+ -1
r=~+cosO: 3+2+1/fi 2+1/fi+2 2+~-1/fi ~ - 1 / f i + 1
2. Let x 2 + y 2 = r2, x = r cos 0, y = r sin 0 and ( 2+ J ' ) ~= 4xy becomes r4 = 47' cos 8 sin 0. Divide by 7' and use identity
sin 28 = 2 sin 0 cos 0 to obtain r2 = 2 sin 28. This is graphed in a manner similar to Example 5.

(Symmetric about origin, no graph for 4 2 < 0 < n.1


5-32 SOLUTIONS TO PROGRESS TESTS

See. 13.2: Progress Test 1


1. Setting 1 - cos 8 = cos 8 = 2 cos 8 = 1 -
cos 8 = 1/2 -
8 = %i/3, Thus (1/2, 7/3),
( l & -7/3) are pts, of intersection. Also from graphs, curves go through origin.

I = 1 -cose \

If 8 = v/3, then r = 3/2 - cos v/3 = 3/2 - 1/2 = 1. Thus (1, v/3) is on intersection. T h e pt. ( - 2 , a/3) is on 8 = v/3, but these coords. do
not satisfy r = 3/2 - cos 8. However, ( - 2 , v/3) also has coordinates (2,4v/3) and 2 = 3/2 - cos 4v/3, so this pt. is on both graphs.

See. 13.2: Progress Test 2


1. tan 4 = (1 - cos B)/(sin 8) = tan 8/2. 2. If i 0is angle between tangent line at (1, T/Z) and ray v/2. By 1, +o = 8,/2 = ~ / 4 SO
,
do =B0 + i 0= ~ / +
2 7r/4 = 3v/4 and tan bo = tan 377/4 = - 1.

See. 13.3: Progress Test 1


a2 - a2
1. 'JZT a2 dB = -[8]," - -(2v) = va2, 2. Set 2 cos 28 = 1 so cos 28 = 1/2. In 1st and 4th quadrants we have 28 = f v / 3 , SO
2 , 2 2
01 = - 7 / 6 and /3 = + v / 6. Using x axis symmetry, we double area between 0 and v/6. By (13.3.3.),

A = 2 * + [ w ' 6 [ ( 2 cos 28)? - d8 =


%/6
4
(4 cos2 28 - 1) dB =
0
%'tj
[4(
1 cos48
J
2
+
- I ] d8 =
0
7'6
) 1
( 2 cos 46' - 1) d8 = f i / 2 - v/6.

See. 13.3: Progress Test 2


1. (See Fig. 13.6 for cardioid.) f(8) = 1 + cos8,f'(8) = -&no, so ds = d(1+ cos 8)2 + (-sin8)2 d8 = d2 + '2 cos8 do. Now
(1 +
cos 8)/2 = cos2 ( 0 / 2 ) , so 2 + 2 cos 8 = 4 cos2(8/2). Thus s = 21; ds = 2 J ; q m d B = 41; cos(8/2) dB [since cos(8/2) 2 0 in
[0, check!] = 8[sin(8/2)]; = 8. 2. x = J ( S ) cos 8 = (1 + cos 8)cos 8. Using 1, S = 2v1;/2x ds =
+
2~;;S;'~(l cos 0 ) cos 8 2 cos(8/2) dB = 2~1;'?[2 cos2(8/2)][2 cos2(8/2) - 1][2 cos(8/2)] dB (applied cos2(8/2) = (1 +
cos 8)/2).
3. y = f(0) sin 8 = ( I + cos 8) sin 8, ds = d2 + 2 cos 8 d8, so S = 2 ~ 1 ; ' ds ~ ~= 2~1:'~(1 + cos 0) sin 8 q 2 + 2 cos 8 d8.

Chapter 14: Solutions to Progress Tests

See. 14.1: Progress Test 1

[The hypotheses of (14.1.2.) hold in each case.] 1.lim 2' In 2 - es - In 2 - 1 = In 2 - In e = ln(z/e). 2. lim -
sinx = -
0 = 0.
2-0 1 1 8-0 cosx 1
-sin x - 2 sec x sec x tan x 0
3. Since lim(cos x - seczx) = lim 2 x = 0, use (14.1.2.) again: lim = - = 0.
x-0 x-0 0-0 2 2

4. lim
(-1/x2)/(1 l/x) +
= lim -- 1.
1 eh
5. lim- = 1
2-+= - 1/x2 s-+m 1 + l/x h-0 1

See. 1 4 . 1 :Progress Test 2


sec x tan x sin x
[(14.1.4.) applies initially to each problem.] 1. lim ~ = lim cos x -= 1. 2. [Note ln(x - 1) + -SO as x + I+.]
z-(T/z)- sec2 x z-(T/z)- cos x
l/(x - 1)
Differentiate, getting lim
2-1'
= lim[-(x - I)] = 0.
- I & - 1)2 2-11
- 0 since numerator + 1 and denominator +
3. lim --
z-ts 1 + lnx
+ +x
SECTION 15.1 s-33

t See. 14.2: Progress Test 1


sinx - 1
1. (to - co): Consider lim ; (O/O): lirn 3= 0 = 0. 2. ( w - co): Consider
z-(T/~)- (-sin x) 1

lirn
xlnx - x + 1 , ( O / O ) . Differentiate numerator and denominator to get lim
In x
= lim
x In x
again ( O / O ) .
2-1- ( x - 1) l n x s-I+ ( x -1 + x In x ) / x r - ~ +x - 1 + x In x '
Differentiate numerator and denominator to get lirn --
1 = lim
1+lnx -- (L - )'. 3. (cc90). Write as lim
In(sin x )
; (O/O).
2-1~ 2 lnx 2 2-1- x - 1 + lnx 2477/21- cot x
(cos x)/(sin x )
Differentiate to get lirn = lim ( - c o s x s i n x ) = 0 . 1 = 0 = lim tanxln(sinx)
r45/2)- -csc* x Z-(T/Z)- Z-i77/21-

See. 14.2: Progress Test 2

T h e limits are indeterminate of forms l", Oo, 1" respectively.


ln(1 + 2jx) 1/(1 + 2/x)(-2/x2) = lim x + 2
1. lim [ x i n ( F ) ] = lim
z++ x z-1" l/x
[ ( O j O ) ] = lirn
2-+r -I/x2 r-+x 1
2
+ 2,'x
- 2, so lim
r++x (-1 = x e2

In x 1/x (sin2 x ) / x
2. lirn (sin x In x ) = lim -[(.o/w)l lim = l i m p = 0.so lirn xsmz = eo = 1.
x-o- x-O+ l/sinx x-o+ cos x/sin2 x a-0- cos x C-0-

See. 14.3: Progress Test 1

1. Lim
-+=
[F = lirn [(lnx)2]b,= lim (In b ) 2 =
b-+cc b-+z
+ cc. 2. lim J o e x d x = lirn [eo - e n ] = 1 - lirn en = 1 - 0 = 1.
a+-= a--r a+-"

See. 14.3: Progress Test 2

lirn 5 [ ( x - l)1/5]13?s = 5 lim [-c1I5


s+o+ WO+
+ 5 &to+
- ( - 1)1'5]
lirn [32l/j - = 5(1) + 5(2) = 15. B1/j]

3. lirn [ l n ( 4 ~- - In 41 + limL[In 12 - ln(48 + a*)]. Neither limit is finite, so integral is divergent.


E ~ )
r-0' s-0

Chapter 15: Solutions to Progress Tests


See. 15.1: Progress Test 1
1. (u) 0:1/4, 8/27, 81/256; ( b ) 0, 2, 0, 2; (c)l, 0, 2, 0; ( d ) 1/2, 3/4, 7/8, 15/16. 2. (a) Apply [15.1.2(c)] with k = 3 in numerator
and k = 2 in denominator to get a, = - +
'
( b ) each odd-numbered elt. is 1 and even numbered elt. is the reciprocal of the number
n+2'
designating the elt., so a, =
I
1 for n odd ,
l / n for n even'
2n - 1
(c) the numerators consist of the odd integers 2n - 1, and denominators consist of squares of the

even integers (2n)2, so a, = -.


(2nY

See. 15.1: P r o g r e s s Test 2


[ - l)n2n
1. By [15.1.6(e)] with r = -7i we have - e". 2. By [15,1.6(h)] with r = - 2 we have ~ + 0.
n!

3.

In n
4. By [15.1.6(d)] with p = 0.01 > 0, we have 0
n
1+ 0. 5. [15,1,6[a)] applies with k = 2 <3 =j, so limit is 0.

6. sin n7i = 0 for all positive integers n . Thus the seq. is constantly 0 and has limit 0.
s-34 SOLUTIONS TO PROGRESS TESTS

See. 15.2: Progress Test 1

s 1 -- In 1 - In 2 = -In 2; s2 = (In 1 - In 2) + (In 2 - In 3) = In 1 - In 3 = -In 3; and in general


s, = (In 1 - In 2 ) (In 2 - In 3)+ + . . . + ( I n n - ln(n + 1)) = In 1 - In(n + 1) = -ln(n + 1). Since lim s, = lim (-ln(n
,L++" n-+x
+ 1) = - X ,

5
n=1
In (z )
diverges.
n+ 1

See. 15.2: Progress Test 2

1. W'e know
k=l
c- +
"
k2
1
k
= 1 by Example 1, so 1 =
k=l
x-
"
k2+k
1 1
=-
2
+ +
1
x-
"

k=3
l
k2+k Thus x z -
"
k=3
1 -
1----=--.
1
2
1
6
1
3
2. Since

In n n - I 1
lirn - = 0 for p
n--r nP
> 0, we have lirn - =
n--= In n
np
+ w . Hence diverges by (15.2.6.). 3. lirn -= - # 0,
n-+= 2n +5 2

"
2-=2, c-=2-
n-1 " 1 " 1 1 1 " 1 'ii
3:

4. Z-='iic-=-
r

so c,,
7t=1
divergesby(15.2.6.). Since
n=O 2n
2n
l--=-.Thus
2 2
n=2 n=2
2"
2% 2 n=2

See. 15.2: Progress Test 3

1.
n=l
"
2 (3 -
3
gIn-l
= 5
n=l
1
3(1-7;)
n-1
. Series diverges. (lri = l / ( v - 3) > 1). 2.
n=4
" jn+Z
x 3n =
n=4
5
25(_)
n
. Series diverges. (171 = 5/3 > 1).

4. 0.3242424. . . = - + --
3
10
24
1000
24
+-
100,000 + 10,000,000
24
+ . . . = -3
10
+1000
- c[ "-
24 n=O (l:OY1 = 10
3 24
-+-.--- 100 - 107
1000 99 330

See. 15.3: Progress Test 1


1 +m
dx dx Z=n du
1. Letf(u) =
x(In x ) * '
ThenJis pas., cont., and decr. on [2, +a)and
~
= lim J -=
2s: u2

2I:(
u = lnx
) = lim [d+ - = -. 1
Therefore
" 1
-conv. by Int. Test
nlirn
-+=lnx du=dx/x n-+=lnn ln121 In2 k=2
k(ln k ) 2
2. LetJ(x) = x/ex2 = xe-". ClearlyJis pas. and cont. on [1, + w ) . SinceJ'(x) = e+l(l - x 2 ) 1. 0 for x 2 1,Jis decr. on [l, $ 0 0 ) .

J
-r
xe-" dx = lim Jn ~ e - ~dx' = lim
n+L"
[$]I = ;ln~" [G+];
-1 1 1
= -, and so
" k
-conv. by Int. Test
1 n-+r 1 2e k=l ek2

In x
3- LetJ(x) = -. T h e n J i s pos. and cont. on (1, + w ) . SinceY(x) =
X?
< 0 for x > e,Jis decr. on [e, + co) and hence decr. on

[3, + w ) . 3
" lnx
- dx x =
[TI,
:ilm [ l n x dx = ,lAym (In x ) ~ = ily" [T
(In n ) 2
-(In23)2
-1
= + w . Thus
m=3
5% div. by Int. Test

"
4. T h e Int. Test does not apply. But since lirnk-+" cos(k2 -t 1) does not exist, 1cos(k2 + 1) div. by kth Term Test.
k=l
SECTION 16.1 s-35

See. 1 5 . 3 : Progress Test 2

Ink 1 Ink Ink arctan k


k3/. /(z
=
)
+ -
k1/4
0 as k -+ + m by [151.6(d)]. Hence
k=1
-conv. by [15.3.6(b)].
k3/2
2. -,/(l/fi)
m =

arctan k (-) fi
+;*lask+ +w.Hence 2-arctan k
div. by [15.3.6(a)] 3. Div. by (15.3.7.) or by kth Term Test.
V G T
k=l

See. 15.3: Progress Test 3

1. = 2 / k + 0 < 1. The series conv. 2.


ak+l
- =
(k + I)! 3 a6.9. . .3k
-
--
1
+ - < 1. The series conv.
3 * 6 9 .. . 3k(3k + 3 )
+

ak k! 3k+3 3
unfl Zn+l(n + 2) . (n - I)! 3n - 2 +2 n
3. - = - -.- + 0 < 1. The series conv. 4. = + 1. Root Test inconclusive.
an n ! 3n+1 2yn + 1) 3 n(n + 1) n+2
= e-' # 0 by [15.1.6(e)], so the series div. by nth Term Test.

See. 15.4: Progress Test 1

1. lakl = -+ - # 0, hence lim


1 (-l)k(k + 2) # 0, and so series div. by kth Term Test.
+5
+

3k+5 3 k-+m 3k

2. consider bkl/lbkl = Ink 1 In k


= kl/ls + 0 as k --$ +m {by [15.1.6(d)]}. Hence orig. series conv. absolutely by Limit Comp. T

(-l)mhm
3. We know (In m)/m is decreasing for m 2 3. Hence the Alternating Series Test applies to show
m
conv.
m=l

lnm (-1)Inm
However, comparison to the (divergent) harmonic series shows 2-m
div. Hence 2 ~ is conditionally conv.
m=l m=l

See. 15.4: Progress Test 2

1. ( u ~ +=
~ /
1
=
1
4'k+3.We want
1
< or equivalently, 42k+3 > lo4 = 10,000. Now k = 1 yields 4 j = 1024 < lo4, but k = 2
yields 47 = 16,384 > lo4. Thus the sum of the first two terms of this series is within of the actual sum of the series.

Chapter 16: Solutions to Progress Tests

See. 1 6 . 1 :Progress Test 1

1. P&) = 1 - -,
x2
2!
+-
x4

41
- - + -.
6!
x8
8!
x6

2. Note: P4(x) = 1 - -
2
x2
+-
x4
24
= 0 for x 1 1 . 5 9 and x 2 t 3 . 0 8 .

ib)

( 0 , 1) y = 1 = P,[X)
* - x / f ' : x ) = cos x

x2 *3 4 5
3. P , ( x ) = l + x + - + - + X + X ,
2! 3! 4! 5!
S-36 SOLUTIONS TO PROGRESS TESTS

See. 16.1: Progress Test 2

1. Choose n = 3. lf'4i(x)1 = /(~)4(-2)(-5)(-8)~-'"31 z 0.0000002354 at a = 64. I f ' 4 ) ( ~ )5l lf'4i(64)l = M on [a,


651. Then

1 1 -2 1 10 1
IRs(65)I M(1)4z9.8-10-9and $%zP3(65)
5-4!
=4 +----+-a-

3 16 9 1024
+ _.-
27 65536
z 4.0207258. 2. (a) Let n = 4, M = 3; then

lR4(0.1)l 5 -- - 0.00000025.
(0.112 (0.1)3 (0.1)4 3(0.1)5
e0.l = 1 + 0.1 + -
2
+-+-6 24
+ R4(0.1) =. 1.105171 + R 4 ( O ~ l )where
,
120

( b ) IR4(0.3333)1 5 ~
3(0.3333)j
120
- 0.000103. Will try n = 5. lR5(0.3333)1 5 ~
3(0.3333)6
720
- 0.0000057. Hence use P5

z 1.3956. (c) n = 20 yields lRz0(5)1 < 0.000028 < 0.00005,

See. 16.2: Progress Test 1


(- l)n+l(n + 1)xn+l .- 3"
(-l)nnxn
I = lim [3rrlxI]
n-+x
n + l = +,so r = 3. At x = -3: 5
n=O
( - I)%( - 3)n
3%
=
"

n=O
n diverges. At x = 3,

n=0
- c
n=O
( - l ) % diverges (nth term test). I = (-3, 3). 2. lim
n-+=
I (n
Xn+l
+ l)(n + 2) x"'
1x1 = 1x1, so r = 1.

A t x = -1:
n=l
c-------
" (-'In
+ n(n 1)
conv. absolutely by comparison to p series for = 2. Similarly, conv. absolutely at x = 1, so I = [ - 1 , 11.

3. lim 1 (x - 4)"+' n
lx - 41 = lx - 41, so r = 1. Examine endpts. of (4 - 1 , 4 + 1) = (3, 5). At x = 3, c-
n=l
(-1)n

conv. conditionally. At x = 5:
n=l
c
" 1
- div., so I = [3, 5).

See. 16.3: Progress Test 1

n!
1. f'"'(x) = ( 1 - x ) n t l , ~ o f ( ~ ) (=On!) 2. g ( x ) = DZ[=],
1
so (16.3.1) implies g ( x ) = 5
n=l
nxn-l.

See. 16.3: Progress Test 2

[ 5-1 [ (-1)nxn ( - l)nx"+l


1. Replace x by --x in p.s. for er and then multiply through by x : xe-" = x
n=O
=x
n=O 7 1 c7
= n=O
. Rad. of conv.

is infinite. 2. Replace x by x 2 in p.s. for sin x : sin(x2) = c


n=O
( - l)yX2)2n+l

(2n + I)! = n=0 (2nC


( - 1)fix4n+2
+ I)! , Rad. of conv. is infinite.

3. Note ln(1 - x) = s
0
-dt
- so In(-)
1-t'
1 + x
1- x
= ln(1 + x ) - ln(1 - x ) = s ~
dt dt

+- n+1
+ ...I+ [x+-+-+
x2
2
x3
3
. . . +-+
Xn+l
n + l
...I=

See. 16.3: Progress Test 3


SECTION 17.2 5-37

. . . + (-1/2)(-3/2). . . (1/2 - n)(-l)nxn + ., , =


+
x
- I
1.3
x2
1.3-5
+- x3 + . . . +
1 . 3 . . . (2 n - 1 )
xn + . . . [(note: number of
n! 2 (Z2)(2!) (23)3! ( 2n)(n9

negative factors is always even in each term)] = 1 + c


n=l
1 . 3 . . . (2 n -
(2?(n!)
1)
Xn.

Sec. 16.4: Progress Test 1

1. ( a ) x = - 1 ( 6 ) 1 n 2 = 2 *-(1/3)zn+1
3
2-=2-+--
n=0
2n + 1
[; i ( i r
3 3
+ -5i -( 3i r + -71(1)7
-3 + .Now2.z
rj

n=O
(2n
1
+ 1)32n+1-
-

2
r:
-+-
3.33
+-
1
5 . 3
1
2 0.34650
~ 1 ~ 1z 0.69300. Hence three terms are required. (c) n = 5 gives 0.693147, accurate through six places.

c , s'
(- 1)nX4n+2 (- l)nX4n+3
2. (a) sin(x2) =
n=O (2n + l)! so
0
sin(t2)dt =
n=O
(4n + 3)(2n + l ) ! . (6) Hence we have an alternating series whose terms decrease in

absolute value. For n + 1 = 4,- 1 -


19 9!
-
*
1
6,894,720
- 0.00000015 < 10W. Hence
~ sin(t2)dt
1
3
1
z - - -+ -- -
7*3! 11*5!
1 1
15*7!
- 0.310268.

Chapter 17: Solutions to Progress Tests


See. 17.1: Progress Test 1
1. 2 z

J
X Q = (1, - 3 , - 5 )
2. z d(P, Q) = fi, d(P, R ) = fi = 3, d(Q, R ) = fi. Hence d(P, Q)2 + d(P, R ) 2 = d(Q, R ) 2 , so PQR is
T R
rt. triangle. A = (1/2) 6h = (1/2) d(P, Q) d(P, R ) = 3 m / ' 2 .

+
3. ( a ) (x - 2)2 + ( y 3)2 + (t - 1)2 = 0. Graph is single pt. (2, -3, 1).
( b ) Divide by 4 and complete sq.: ( x - 1/Z)2 ( y - 3)2 ( z +
= 9. Sphere centered at + + 3, - 11, radius 3
~

See. 17.2: Progress Test 1


1. Sym. with respect toy+ and x-y planes. Sincey 2 0, traces in planesy = k 0 are circles centered o n y axis of radii Z f i . > Traces in
planes x = k and z = k are parabolas z2 = 4y - k2 and x2 = 4y - k2, respectively.
S-38 SOLUTIONS TO PROGRESS TESTS

2. z

I
z
t
z = k

x = k
mkJ 3x-
4

J x2 +y= b
X

z
t

See. 17.2: Progress Test 2


1. Cyl. with rulings 11 t o y axis and directrix the plane curve in x - t plane with equation z = In x .
2. Cyl. with rulings 11 to z axis and directrix the hyperbola in xy plane with equation x2/16 - y2/4 = 1.
3. Surf, of rev.: revolve plane curve x = 2 6 in x-z plane about z axis.
z
z 4

t
\/\ \ \ \ \ \

<= / x = In x ,J X J

4. Solve for x in terms of z: x = h ( t ) = z213. By Table 17.2e, equation of surf. is x z + y 2 = [ z * ~ ]=~ z413.

See. 17.2: Progress Test 3


1. x2/16 - y2/4 + z2/16 = 1 is of form (172.5) withy and z interchanged, thus a hyperboloid of one sheet envelopingy axis, in fact,
hyperboloid of rev. z <
2. Has form of (17.2.9), with c 0, a hyperbolic paraboloid.
z
SECTION 17.4 $139

+
3. Comp. sq. to get ( x - 1)2 (z - 1)2 = 36(y - l), a translated variqtion of (17.2.8), an elliptic
paraboloid with vertex (1, 1, l), opening upward.

*Y

See. 17.3: Progress Test 1


--+
1. Let Q = (q,, q2). Then A = PQ if and only if ( - 2 , 3) = 3; hence q, = -1, q2 = -1,
so Q = ( - 1, - 1). P

+ *
2. Let S = (sl,sz). Then P Q = ( - 6 - 1, 7 - (-3)) = (-7, 10) a n d R S = (sl 4,s2 + 3). So s, + +4 = - 7 and s2 + 3 = 10; thus
S=(-11,7). 3. j/AI/= d ( 1 ) 2 + ( q 3 ) 2 = f i = Z a n d c o s a = 1 / 2 . S i n c e O l c w l . r r / Z , a = . i r / 3 .

Sec. 17.3: Progress Test 2


1. (A + B) + C = ( ( 2 , -4) + (-3, 1))
C - A = ( - 2 , 3 ) - ( 2 , -4) = ( - 4 7)

UX
$
+ (-2,3) = (-3,0), A - B = (2, -4) - (-3,

A - B
1) = (5, -5);

~x d\
C-A
A
=-

2- ( a ) 3 ( - 3 , 1) +
2(4, - 2 ) - (-1, -1) = (O,O), ( b ) llCll((-3, 1) + (4, -2)) = -d);
(fi,
(6) 113(-3, 1) - 2(4, -2)11 = 11(-17,7)li = q m = v'?%

Sec. 17.4: Progress Test 2


1. Since i, j, and k are unit vectqrs in dir. of pos. x , y , aqd z axes, we apply [17.4.8(a)]: Proji A = ( A * i)/l/ill = a,, Projj A = ( A *j)/lbll = a2,
(2, 1, - 6 ) * ( 3 , -1, 1) ---1
Projk A = (A k)/llkll = as. ProjB 4 =
2.
1 ~ 3- , 1,i)ii - sand
ProjB A = ( -6)
1 ~ 3 -, 1
(3, - 1 , 1 )
, ~
)(3, -1, 1) = (-3/11, 1/11, - l / l l ) . 3. ( l / f i , l / f i ) is a unit vector in dir. of (1, l ) , and so

10(1/*, l/*) = (lo/*, lo/*) = F. Thus W = F (30,O) = 3 0 0 / f i ft-lb


5-40 SOLUTIONS TO PROGRESS TESTS

See. 17.4: Progress Test 3


l/m,
-
1. The vector A/IIAl/ = ( - 2 / m , 3/-) has same dir. as A but length 1. Thus desired vector B is
2A/I/All = ( - 4 / f i , 2/fi, 6/fi). 2. $il= d m = 2, so cos (Y = 1/2, cos ,8 = - 1/2, and cos y = fi/2. Thus (Y = ~ / 3 ,
,8 = 2 ~ / 3 ,y = 71/4. 3. Suffices to show N P,P2 = 0. But
N P,Pz = ( a , b ) ( x , - x1,y2 -yl) = a(x2 - x,) + b(y2 -y,) = ax2 + b, - ( M , + by,). Since PI,Pz are on L , their coords. satisfy
*
ax, + byl = c, ax2 + by, = c, and so N *PIPz= axz + by2 - (axl by,) = c - c = 0. +
See. 17.4: Progress Test 4

triple. 4. Let D = A xC= (1; ,I: - 1; ,I: ;1 :I)= (-2, -17, -12). Then A, C, Dis a rt.-hand triple and Dis I to
plane of A and C. Let D = (2D)/IIDII = ( - 4 / m , - 3 4 / m , -24/m). Then D has length 2, and since it is a pos. mult. of D, it is
I to plane of A, C; and so A, C, D form a rt.-hand triple.

See. 17.5: Progress Test 1


1. ( a ) x = 7 + 3t,y = - 2 + (1/2)t, t = 5 - t ; ( b ) x = 2 - 7t,y = 1 + 2t, z = 6 + t. 2. We have x = 1 - 3t,y = 7 + t ,
t = 11 + 4t; x = 3 + s,y = 9 - s, z = 4 + 3s. Equating 1 - 3t = 3 + s and 7 + t = 9 - s, we solve to obtain t = -2, s = 4. Substituting
into respective t coordinates, we obtain z = 3 and z = 16, respectively. Thus lines do not intersect. Since (-3, 1,4), (1, -1, 3) are not scalar
multiples of one another, lines are not parallel and hence are skew. 3. ( a ) 4(x - 2) - (y - 6) + 2 ( t + 4) = 0 or 4x - y + 22 = -6;
+ + + *
(6) l e t P ~ = ( 1 , 2 , - 3 ) , P ~ = ( 2 , 4 , 6 ) , P z = ( - 2 , - 1 , 1 ) . T h e n P o P , = ( 1 , 2 , 9 ) , P o P 2 = ( - 3 , - 3 , 4 ) . T h e n N = P o P , X P , P 2 = ( 3 5 , - 3 1 , 3 ) i s
normal to n.Since plane contains (1, 2, -3), equation is 35(x - 1) - 31(y - 2) + 3(t + 3) = 0 or 35x - 31y + 3t = -36.

Chapter 18: Solut3ons to Progress Tests

See. 18.1 :Progress Test 1


1. x 2 + y 2 = eZt(cos2t + sin2 t ) = eZt = 9.Hence graph of F lies on cone Since z = et > 0, it lies on nappe.

Cone
x2 + y2 = 2 i
SECTION 18.3 s-4 1

2. F(t) = (et(cos t - sin t ) , et(cos t + sin t ) :e t ) . 3. Tan. line is parallel to F(0) = (1, 1, 1) and goes through F(0) = (1,0, 1). Thus
parametric equations are x = 1 + t,y = t, z = 1 + t.
See. 18.1 : Progress Test 2
1. F(t) = (t3, 2t4, 2t) + (cl, c,, c3). Since F(0) = (4, 6,8), c, = 4,c, = 6, c3 = 8. Now F(t) = ( t 4 / 4 4t, 2 t 5 / 5 +
6t, t2 + + 8 t ) + ( d l , d,, d3),
and since F(0) = 0, dl = d, = d3 = 0. Thus F(t) = (t4/4 4t, 2t5/5 +
6t, t 2 8t). + +
2. IIF(t)lI = dF(t) F(t), SO
F(t) F(t)
Dt(lIF(t)II) = i(F(t) F(t))-/[F(t) F(t) + F(t) F(t)] = ;(F(t) F(t))-[Z F(t) F(t)] =
IIF(0II

See. 18.2: Progress Test 1


1. F(t) = ( - 2 sin 2t, 2 cos Z t ) , 11F(t)Il = 2 , so T(t) = (-sin 2, cos 2 t ) . Now T(t) = ( - 2 cos 2t, - 2 sin 2t) and IIT(t)II = 2, so
N(t) = (-cos 2t, -sin 2t). Thus T(?r/8) = ( - l/@, l / f i ) , N(?r/8) = ( - l / f i , - l/$!).

1 - cost sin t sin t


2. F(t) = ( 1 - cos t, sin t ) , l/F(t)l)= ,
-d SO T(t) =
)=(Jy
v 5 T i G z d2 - 2 cost
T(t) =
(24-I
sin t -cos2 t + 2 cos t + 1
( 2 - 2 c 0 s t ) ~ ~)=(2d2 - 2cost
sin t
, -:JF). Thus IIT(t)II = 1/2 and

N(t) =
2
sin t
- 2 cost
, -iy), and so T(T)= ( l , O ) , N(T) = (0,-1). Y
t (77, 2) T

~~~ ~

See. 18.2: Progress Test 2

See. 18.3: Progress Test 1


1. Since F = mA, if F = 0, then A = 0. Integrating twice, we have V(t) = C, and G(t) = C,t + C,, which has as its graph a single pt. if
C, = 0 and a straight line if C, # 0. 2. V ( t )= w ( a sinh at, a cosh w t ) and A(t) = w z ( a cosh w t , a sinh w t ) = w2G(t). Thus
F(t) = rnw2G(t), and so is a central (repulsive) force.

See. 18.3: Progress Test 2


1. ( a ) y ( t ) = (ao sin a)t - ( g / 2 ) t 2 = t[u, sin a - ( g / 2 ) t ] . Thus y ( t ) = 0 when t = 0 and when i = (2uo sin a ) / g . Since t = 0 corresponds to
initial position, impact occurs after [(2u0 sin a ) / g ] s. ( b ) By ( a ) , flight terminates after t = [(Zoo sin a ) / g ] s. Thus terminal speed is
~~v((zv, sin a)/g)ll = ~ ~ ( cos
u o a, uo sin a - g((20, sin a)/g))/l = I ~ ( Z J cos ~ +
a , - u o sin a)ll = duo2(cos2a sin2 a ) = uo.
S-4 2 SOLUTIONS TO PROGRESS TESTS

(c)y ( t ) = (uo sin a ) t - ( g / 2 ) t 2 gives height at time t. Max. ht. occurs wheny'(t) = uo sin (Y - gt = 0 (that is, when vert. component of V is 0),
or t = (uo sin a ) / g . Thus max. ht. isy((uosin a)/g) = (uo2 sin2a)/(2g). ( d ) By (a), time of impact is (2u0 sin a ) / g , and so range is
x(2uo sin a ) / g ) = uo cos a(2u0sin a ) / g = uO2(2 sin a cos a ) / g = (uo2 sin 2a)/g. Thus max. range occurs when sin 2a = 1; that is, a = n/4.

Chapter 19: Solutions to Progress Tests

S e e . 1 9 . 1 :P r o g r e s s Test 1
1. The pt. (x,y) is in D ( f ) Q xy 2 1. Thus for x > 0 , y 2 l/x, and for x < 0,y 5 l/x.

2. (a) Graph is a plane with x , y , and z intercepts ( 2 , O , 0), ( 0 , 4 , 0 ) and (0, 0 , 4 ) , respectively; 2

\h, 2=4-2x-y

(b) 2 = d3x2+ 4y2 - 12 x2/4 +y2/3 - 9/12 = 1, whose graph is a hyperboloid of one sheet. Graph off is the part lying above x-y plane.
4

See. 19.1: Progress Test 2


1. Pt. ( x , y , z ) is in D ( f ) .a 5 1 and t2 - 1 2 0. This is true o t is in [ - fi,- 11 or [ l , fi].Thus domain consists of two
horizontal plates of thickness fi - 1 and of infinite extent: one above x-y plane between the planes z = 1 and t = fi and one below x-y
plane between the planes z = - 1 and t = - fi,
SECTION 19.4 s-43

Lc=o
2. Y

c = -1 c = l

t c = -1
x2 +3.2

See. 19.2: Progress Test 1


1. ( a ) Let L, be the straight line with equationy = mx. Then ( x , mx) + (0, 0) along L , as x -+ 0. Nowf(x, n x ) = r n x / ( x z m2), which +
tends to 0 as x -+ 0. ( b ) Pts. ( x , x 2 ) -+ ( 0 , 0) alongy = x 2 as x -+ 0. Sincef(x, x 2 ) = 1/2 for x # O , f ( x , y ) -+ 1/2 as ( x , y ) -+ (0,O) along
y = x 2 . (c) No! For lim to exist, lim would have to be the same along any smooth curves through (0,O). 2. Let e 0. Since >
d w < +
5 d(x - a)2 + ( y - b)2, we choose 6 = E . Then 0 d [ ( x , y ) , ( a , b)] = d(x - a ) 2 ( y - b)2 8 implies <
d[f(x,y),f(a, b ) ] = d[x, a ] = -d <
2 d(x - a)2 + ( y - b)2 6 = E .

See. 19.3: Progress Test 1

See. 19.3: Progress Test 2


1. f, = x / d m :f, = y / d m ,sof,(3,4) = 3/5 andf,(3,4) = 4/5. Thus IIT:( 3 / 5 ) ( x - 3) +
(4/5)(y - 4) = z - 5 and N:
x = 3 + (3/5)t,y = 4 + (4/5)t, P = 5 - t. 2. T,(x,y) = -62, and so T,(3, -1) = -18. Temp. is changing at the rate of -18" per unit
of length.

Sec. 19.4: Progress Test 1


1. A t =f(-1.01,3.02) - f ( - l , 3 ) = 2(-1.01)2 - 3(-1.01)(3.02) + (3.02)'- 20 = 0.3112.
dz =fx(-l,3)(-0.01) + f y ( - l , 3)(0.02) = (-13)(-0.01) + g(O.02) = 0.31. Thus lAz - dzl = 0.0012. 2. Letf(x,y) = d m , x = 3,
y = 3, Ax = 0.01, and Ay = -0.03. Thenf(x + Ax,y + Ay) = d(3.01)2 + (2.97)3. Nowf, = x / d m andf, = 3 y 2 / ( 2 d v ) , so
d(3.01)' +
( 2 . 9 7 ) 3 6~ + (1/2)(.01) + (9/4)(-.03) = 5.9375.
5-44 SOLUTIONS TO PROGRESS TESTS

See. 19.5: Progress Test 1

1. ( a ) z7 = t,x7 ] 0 + [ x 3 / ( x 2 + y 2 ) 3 / 2 ] sin 0 =
+ zyy7 = [ y 3 / ( x 2+ y 2 ) 3 / 2cos r3 sin3 0 cos 0
r3
+ r3 C O Sr3~0 sin 0 -- (cos2 0 + sin2 8) cos 0 sin 0 =
cos 0 sin 8; ( b ) dz/dt = z2 dx/dt + tydy/dt + zw dwldt = ( y 2ln(w/x) - y 2 ) ( - l i t 2 ) + (2xy In(w/x))(l) + (xy2/m)(2t + 1 ) = ln(t3 + t 2 ) +
(3t + 2 ) / ( t + 1 ) . 2. V = (v/3)r2h,so dvldt = V,(dr/dt) + V,(dh/dt) = (2mh/3)(dr/dt) + (m2/3)(dh/dt).Thus at given instant
dV/dt = (27: 12 14/3)(2) + ( ~ ( 1 2 ) ~ / 3 ) ( - =
* * 4 ) 32v cm/s.

See. 19.6: Progress Test 1


1. ( a ) vf(~,?) = (6x, By), u = V / ~ I V I I = (I/*, Z/fi), SO D,f(3, -4) = D J ( 3 , -4) = Vf(3, -4) * U = -46/$;
+ +
( b ) Vf(x,Y)= ( y 2 / ( X 2 y 2 ) 3 / 2 ,- V / ( x 2 y2)32),u = v/ll~il = - 1/2), SO D , f ( 3 , 4 ) = D , f ( 3 , 4 ) = V(3,4) u =(16d + 12)/250.
2- D(,,3>f(3, - 2 ) = Vf(3, - 2 ) (1/m 3,/ m ) = 2 . D(,,,)f(3, - 2 ) = V f ( 3 , - 2 ) . (1, 0 ) = 4. Thus
f,(3, -2)(1/m +fy(3,
) - 2 ) ( 3 / f i ) = 2 andf,(3, -2)(1) + f y ( 3 , - 2 ) ( O ) = 4; solving these, we haveh(3, - 2 ) = 4 and
f,(3, - 2 ) = (2m-4)/3. Now u = ( - 3 / m , 2/m)is a unit vector that points from (3, - 2 ) toward (0, 0). Thus
D,f(3, - 2 ) = vf(3, - 2 ) * u = (4m- 44)/(3fi). 3. It should proceed in the direction of Vz(2, 3 ) = (108, 72).

See. 19.7: Progress Test I


1. Sincef(3, fi/Z) = 4 a n d f ( - 2 m , 1 ) = 36, level curves have equations x 2 - 4y2 = 4 and x 2 - 4y2 = 36. V f ( 3 , &/2) = (6, - 4 f i ) ,
1
V f ( - Z a , 1 ) = (-4fi, -8).

+ +
2. Let F ( x , y ,z ) = x3 4y3 z3 - 5xyz. Then F, = 3x2 - 5yz, F, = 1Q2 - 5 . 9 , F, = 32 - 5xy, so FJ2, 1, 2 ) = 2, F,(2, 1, 2 ) = - 8 ,
-
FJ2, 1, 2) = 2. Tangent plane: 2 ( x 2) - 8 ( y - 1 ) + 2(z - 2 ) = 0; normal line: x = 2 + 2t,y = 1 - 8 t , z = 2 + 21. 3. Let
F(x,y,z)=y-x2+z2+2,G(x,y,t)=x2+y2-z-8.Then VF(-2,2,0)=(4,1,0),VG(-2,2,0)=(-4,4,-l).Tan.linetoCat
( - 2 , 2 , O ) = P lies in tan. plane to both surfaces at P. But V F and VG are normals to these planes, so tan. line is parallel to
V F x VG = ( - 1 , 4 , 2 0 ) and hence has equations x = - 2 - t , y = 2 + 4t, z = 20t.

See. 19.8: Progress Test 1


1. f, = 2x + +
2xy = O,fy = Q 2 = 0 have simultaneous solutions (0,0), ( fi,- l ) , ( - fi,- 1 ) . Sincef,, = 2 +
2y,f,, = 2x, fyy = 2 , we
havef,,f,, +
- f Z v 2 = 4 4y - 4x2, which is neg. (= -8) at (%a,
-1) (saddle pts.) and is pos. ( = 4 ) at (0, 0). Sincefso(O, 0) = 2 0, (0, 0) >
is rel. min. 2. f, = 4x3 = O,& = 4y3 = 0 yield crit. pt. at (0, 0) withf(0,O) = 0. Test fails, butf(0 + h, 0 + k ) = h4 k4 0 for all h, + >
k # 0. Hence rel. min. at (0, 0).

See. 19.8: Progress Test 2


1. T, = 8x = 0,T, = 4y + 2 = 0 have solution (0,- 1/2), Let 4
c,: x = ?TqF, -1 < y < 1; A

b X
SECTION 20.1 s-45

C:, x = - m, -1 < J <


1. Then g ( y ) = T ( * e , y ) = -2y2 + +
2y 5 and g ( y ) = -4y + 2 = 0 wheny = 1/2, a rel. max. for g.
Pts. on boundary corresponding t o y = 1/2 a n d y = k 1 (endpts.) are, respectively, ( t f i / 2 , 112) and (0, 2 1 ) . Now T(0,1) = 5,
T(0, -1) = 1, T(0, 1/2) = 5/2, T ( 4 f i / 2 , 1/2) = 11/2. Hence min. temp. ( = 1) occurs at (0, -1) and max. [=(11/2)] at (JT-fi/Z, 1/52),

See. 19.9: Progress Test 1


1 . Solution curves should have this general shape and location. J plates of
spinach)

2. p(plates of
spinach)

. x (hamburgers)

I x +y = 6

See. 19.9: Progress Test 2


1 . (i) f, = 2xy = h4x, (ii) fy = x 2 = X(2y), (iii) 2 x 2 + y 2 = 12. From (i)y = 2h [ x # 0 by (iii)], so x 2 = y 2 by (ii). Hence x = 1 2 by (iii).
Hence extremes at ( 2 , 2 ) , ( - 2, 2 ) , (- 2, - 2), (2, - 2). Computingf at these pts. yields maximum of 8 at ( 2 , 2), (- 2, 2) and minimum of -8
at ( - 2 , - 2 ) , ( 2 , - 2 ) .

Chapter 20: Solutions to Progress Tests

See. 2 0 . 1 :Progress Test 1


S-46 SOLUTIONS TO PROGRESS TESTS

See. 20.1: Progress Test 2 J

1. (a) R = {(X,Y)/O Ix I1 , X Z I r 5 XI,

( b ) I = J1[f(4x
n x2
+ ZJ) 41dx = [[4xy +y2&dx =
-6 (5x2 - 4x3 - x4) dx = 7/15.

I
4 Y

t
j=Jx+9

(0,1:
x = 1 -4-
X
SECTION 20.2 s-4 7

See. 20.2: Progress Test 1


1. Y Since R = {(x,y)/O 5x < 1, x z < y 5 x},

i
2.

0 0
-y2)3'2
3
1: = 1/3. (Note: working with the type I description of R leads to a more difficult

integration problem.)

See. 20.2: Progress Test 2


Now p ( x , y ) = 4, <
and since R = { ( x , y ) / O x 5 1,0 < y 5 1 - x } ,

i
1.
M , = $IRx p(x,y)d A = $A$;-" x ky dx = k$A .9~~/21;-"dx
= ( k / 2 ) X ( x 3 - 2 x 2 + X ) dx = k/24.
5-48 SOLUTIONS TO PROGRESS TESTS

2. Y

Since R = {(x,y)/O5 x 5 2,0 < y 5 x 2 } , Z, = JJEy2p(x,y)dA = J;Jg'y'(x + 2) 4 dx = JX[y3(x+ 2)/3]g2 dx = ( 1 / 3 ) J 6 ( x 7 + 2x6) dx = w.

Sec. 20.3: Progress Test 1

Sec. 20.3: Progress Test 2


1.
SECTION 20.4 s-49

2.
e

See. 20.4: Progress Test 1


7r/4 cos28 7r/4 rZ cos28
1. A(R) = J J rdrd8 = J d8
-7/4 0 -7r/4 0

2. 1
d
- dA. In polar coords.
-1 -m
R = { ( r , 8 ) / 0 5 '6 5 277, 0 5 r 5 1) and d m = r.
211 1 1
so V ( S ) = J J (1 - r)7 d7d8 = -J
2*
d8 = 77/3.
0 0 6 0

+ y 2 = r2 and R = ( ( r , 8 ) / 0 5 8 5 a/2,
3* f Since x 2

0 5 r 5 1) JJ ez2+Y2 dA =
1
44lr/2
e"rdrd8 = (77/4)(e - 1).
R

See. 20.4: Progress Test 2


1. ( a ) r = = 2, tan 8 = - d, and since (- 1, d)
is a pt. in the 2d quad. of the x;v plane, we choose 8 = 2v/3. Thus cyl. coords.
are (2, 2a/3,4). ( 6 ) 7 = = fi,tan 8 = 1; thus, since (1, 1) is in 1st quad., 8 = n/4; so cyl. coords. are (fi, n/4, fi).
2 . ( a ) x 2 - y 2 = r2(cos28 - sin28) = rz cos 28. Thus cyl. equation is r2 cos 28 + z2 = 1. ( b ) x 2 + y 2 = r2, so equation x 2 + y 2 = 9 becomes
rz = 9 in cyl. coord., which is equivalent to 7 = 3 .
s-50 SOLUTIONS TO PROGRESS TESTS

See. 20.4: Progress Test 3


1. (a) p + 3 + 16)li2= 2 f i , tan 6' = - fi,so 6' = 2v/3, + = arccos(2/fi). Thus sph. coord. are ( 2 f i , arccbs(z/fi), 2m/3).
= (1
( b ) p = (1 + 1 + 2 ) l j 2 = 2, tan 0 = 1, so 6' = 7r/4 and + = arccos( */2) = v/4. Thus sph. coord. are (2, v/4,7r/4).
2. ( a ) x 2 - y 2 + z 2 = p2[sin2+(cos26' - sinZ6') + cos2 $1 = p2[kinatp cos 26' + cos241. so sph. equation is $[sin2 + cos 26' + cos2$1 = 1.
(6) x 2 + y z = p2 sin2 +[cos2 6' + sinZ6'1 = p2 sin2 9. Thus sph. eqhation is p2 sin2+ = 9. But p 2 0 and sin + 2 0, so this is equivalent to
p sin + = 3.
See. 20.4: Progress Test 4
4
1.

z
2.

kn(a2 - )'6
--
-
k(a2 - b 2 )
2
[ d6'=
4

Chapter 2 1: Solutions to Pl'ogress Tests

Sec. 2 1 . 1 : Progresd Test 1


1. M y = x 2 # 4y2 = N,.By (21.1.3), F is not conservative. 5. M , = N, i 3gv(x2 Letf, = -gx(x2 +y2)-3/2. Then
f = g(x2 +y2)-1/2+ C ( y ) , soyv = - , g ( x 2 + y 2 ) - 3 / 2+ C ' ( y ) . Setting& = N - C ' ( y ) = 0, so C ( y ) = k , a constant. Thus F = VJ where
f ( x , y ) = g(x2 + Y ~ ) - ' / +
~ k. 3. M , = N, = 4xy/(2 + y 2 ) . Letf, = 2x ln(2 + y 2 ) - 6x2. T h e n f = x21n(2 + y 2 ) - 2x3 + C(y), so
f, = 2x3/(2 + y 2 ) + C ' ( y ) . Setf, = N to conclude C ' ( y ) = 2y, so C ( y ) = y 2 + k. Thud F =' V wheref(x,y) = x 2 ln(2 + y 2 ) - 2x3 + y 2 + k.
SECTION 21.4 s-5 1

See. 2 1.2: Progress Test 2

( 1 , 1:

(41 - 64 fi)/30. Thus Jc(x - y 2 )dx + ( x 2 + y ) dy = (71 - 64 fl)/30.


See. 2 1.3: Progress Test 1
1. M , = N , = 3x2 + 2y (cont. on all of R2). A potential function isf(x,y) = x 3 + x y 2 - x 2 . Thus I = f ( l , 1) -f(O, 0) = 1.
2. M u = N, = -2 siny (cont. on all of R 2 ) . A potential function isf(x,y) = ez cosy. Thus I =f(O, v / 2 ) -f(O, 0) = - 1. 3. Let D be
any simply connected open set containing ( e 2 , 0) and (0, 1) but not (0, 0). Then M , = N, = - 2 x y / ( x 2 + y 2 ) ; M u , N , cont. on D. A potential
function isf(x,y) = (1/2) ln(x2 + y 2 ) .Thus I =f(O, 1) - f ( e 2 , 0) = (- 1/2) ln(e4) = -2.

See. 2 1.4: Progress Test 1


1. Let M = 0, N = x . Then by Greens Theorem & . R x dy = JJR[N,- M u ]dA = JJR dA = A ( R ) . Similarly, if M = -y and N = 0, then
$a.Rydx = JJR[N, - M,] dA = JJR dA = A ( R ) . 2. Let M = xy3/3 + 4y - eZx, N = x y 2 + 2y, and let D be the region inside the rectangle.
Then J,Mdx + Ndy = JJD[N,- M u ]dA = JJD(y2- v 2- 4) dA = J:J!(y2 - xy2 - 4) dx dy = -45/2. 3. Let D be the regular region
outside the circle r of radius 1/2 centered at (- 1,O) (clockwise) and inside K. Since M , = N,, J

t
5-52 SOLUTIONS TO PROGRESS TESTS

$d-Y/((X +
1)' +y') dx ( x + + +
l ) / ( ( x 1)' + y 2 ) 4 = JJ"D 0 dA = 0. Now $aD = JK + Jr = 0 JK = -Jr
3 = J-r, where -r has
parameterization ( - 1 +
( 1 / 2 ) cos t , ( 1 / 2 ) sin t ) , 0 5 t 5 2 ~ Thus
. JK = = J"$"(sin2t + cos2 t ) dt = 27.
See. 2 1.5: Progress Test 1
1. JK F n ds = J"JR div F dA = JJR 3(x2 + y 2 )dA = 3J"X"JX r3 dr dB = 2 4 ~

Appendix A: Solutions to Progress Tests

See. A . l : Progress Test I


3x
1. (Separable) -dx
4 =0
+- Q (3/2) ln(x2 + 4) + lnlyl = ln/cl .a3 ( x 2 + 4 ) = 2c. 2. (Homogeneous)
x2+4 y
y = ux and equation becomes (after simplification) ~

m
dv
=
x
o lnlv +1- = lnlxl + lnlcl O u + = cx 0 ) ~ +d m = cx2.

See. A. 1: Progress Test 2


1. (Linear) P ( x ) = tan x , so eJP(o)dr = sec x and soln. is y sec x = J" cos x sec x dx + e = J dx + c = x + c.
2. (Exact) M y = Nx = 2x sec2y - sec x tan x . Set f, = 2x tany - y sec x tan x + 2 x =f = x 2 tany - y sec x + x 2 + + ( y )'fu
= x z sec2y - sec x + +'(y). Thus @ ' ( y )= 0, so + ( y ) = C,. The soln. is x2tany - y sec x + x 2 + C, = C.

See. A.2: Progress Test 1


1. c(m) = m2 + 4m - 12 = ( m - 2)(m + 6 ) , soyc = cle20 + c2e-6", 2. c(m) = m2 - 10m + 25 = (m - 5)', soyc = clea + c2xe5".
3. c(m) = m2 + m + 3 is irreducible. Now a, = 1 , a2 = 3, so a = -a,/2 = - 1 / 2 and b = d-/2 = d m / 2= m / 2 . Thus
V E X
yc = e-8'2 [el sin-
2
+ c p cos -
2

See. A.2: Progress Test 2


1. c(m) = m2 - 4m + 4 = ( m - 2)2, soyc = cleZs + c2xe2". Lety, = e2' andy2 = .rezr, Theny,yh - y 2 y ; = e2x( e2s + 2xe2") - xe2"(2e2")= e4'.
-xe2"(x2e2") - - 2 3 and ~h = (' 2 2x ) - x 2 . Hence U, = -
ZX -x4 - x4
and u2 = x3/3. Consequently,y, = - ( e z x ) + -(xe2")
x3 x4ezr
= -.
Thus U; = ~

e4x e4" 4 4 3 12
The general solution of the equation isy = cle2" + c p 2 " + x4eZr
12
-, + c2xe2". The at's are e2", 1 , and since e2"
2. As in Prob. I,yc = cleZx
and xeZx satisfy the associated equation, we replace eZr by xze2". Thus y , = Ax2e2" + B ; y; = 2Axe2" + 2Ax2e2";yg = 4Ax2eZr+ 8AxeZ2 + 2Aep5
andy; - 4,u; + 4y, = ( 4 A - 8A + 4A)x2ez" + (8A - 8A)xe2" + 2Ae2" + 4 B = 2Ae2" + 4B. Thus 2A = 1 , 4 B = 1 , so A = 1/2, B = 1/4. The
general solution isy = clezO + c2xe2" + (1/2)x2e2"+ 1/4.
INDEX

Page numbers in parentheses and italic refer Arc (cont'd):


to the exercise(s) on that page. simple, 404, 597 Bernoulli, Johann, 459
(See also Curve) Bessel equation, 541 (44)
Absolute convergence, 505 Arc-: Binomial coefficients, 537
Absolute extremes, of functions of two cosecant, 303 Binomial series, 537
variables, 666, 669 cosine, 300 Binomial Theorem, 74, 537
Absolute value, 4 cotangent, 303 Boundary point, 631
Acceleration, 102, 160 secant, 301 Boundary of a set, 669, 751
normal component of, 613 sine, 299 Bounded sequence, 483
in polar coordinates, 615 tangent, 300 'Bounded set, 669
tangential, 613 (See also Inverse trigonometric functions) Brahe, Tycho, 615
Angle(s), 234 Arc length:
degree measure of, 234 definition, 406 Carbon 14, 282, 287 (9, 1 0 )
direction, of a vector, 575 differential of, 409 Cardioid, 438
of inclination of tangent line in polar of plane curves, 406 Cartesian coordinates:
coordinates, 446 in polar coordinates, 452 in three dimensions, 550
as parameter, 400, 403 of space curves, 597 in two dimensions (see Axes, coordinate)
radian measure of, 234 Arc length function, 598 Cartesian three space, 550
radian versus degree measure of, 248 Archimedes, 185 Catenary, 311
between ray and tangent to polar graph, Archimedean spiral, 443 (62) Cauchy-Schwarz inequality, 573
446 Area, 185, 189 Center of gyration, 430 (22)
between two planes, 589 (4 9 ) between curves, 216 Center of mass, 414
between two vectors, 572 computed using double integrals, 686, 693 of a lamina of nonconstant density, 697
Antiderivative, 176 computed using polar integrals, 713 of a plate of constant density: 416
(See also Antidifferentiation; Integral; interpretations in economics, 220 of a solid of nonconstant density, 711 (27)
Integration) of a parallelogram, 579 of a system of point masses: in the plane,
Antidifferentiation: in polar coordinates, 448, 450 414
application to differential equations, 179 of a surface of revolution, 407 on the x axis, 414
basic rules for, 177 Asymptote: (See also Centroid)
definition of, 176 horizontal, 112 Central force, 610
elementary substitution method, 181 oblique, 115 Centripetal force, 610
substitution method, 181 vertical, 109 Centroid:
(See also Antiderivative; Indefinite Attitude numbers, 583 of a plane curve, 418
integral) Average value of a continuous function, 21 1 of a plane region, 415
Apartment rental, 165, 286, 364 (50),474 Axes: of a solid of revolution, 418
Approximation : coordinate, 9 of a surface of revolution, 418
of e, 522 rotation of, 353 (See also Center of mass)
of functions using Taylor polynomials, translation of, 340 Chain Rule, 85, 335
516 Axis: for antiderivatives, 213
of integrals, 386 major and minor of an ellipse, 346 for definite integrals, 213
of integrals using series, 542 of a parabola, 22, 341 for differentials, 97
linear, 98 polar, 434 for functions of two variables, 649
of sin x , 523 of revolution, 558 general, 652
using alternating series, 507 transverse, of a hyperbola, 349 proof of, 335
using differentials, 98 for triple composition, 681 (15, 16)
using power series, 541 Basis vectors (see Unit coordinate vector) for vector functions, 596
Arc: Beam: Chart method, 7, 122, 127, 144
closed, 404, 597 stiffness of, 154 (28) Circle:
orientation of, 598 strength of, 154 (27) circumference of, 30 ( 4 )

I- 1
1-2 INDEX

Circle (contd), Coordinates (contd): Derivative (contd):


of curvature, 605 rectangular, 433 of b, 263
equation of, 17 transformation of, 340 of a composition of functions (see Chain
involute of, 404 (32) Copernicus, Nicolaus, 616 Rule)
Circular helix, 593 Cosecant function, 236 of a constant function, 74
Circular region, growth of, 55 derivative of, 246 Constant Multiple Rule, 75
Circulation, 757 integral of, 291 of cosecant, 246
integral, 762 inverse of, 303 of cosine, 245
Cissoid of Diodes, 431 (26) Cosine function, 236 of cotangent. 246
Closed interval, 3 derivative of, 245 definition, 52, 71, ( 5 ) , 74
Closed set, 631 integral of, 290 of 8, 262
Coefficient of friction, 276 (96) inverse of, 300 of functions defined implicitly, 87
Comparison Test, 499 Cotangent function, 236 higher order, 100, 123
Competition, 164 derivative of, 246 of hyperbolic functions, 310
Completing the square, 5 integral of, 291 of inverse trigonometric functions, 302
Composition of functions: inverse of, 303 left- and right-hand, 72 (25, 26)
continuity of, 66 Critical number, of a function, 127 of an x , 264
definition of, 36 Critical point: of log, x , 265
derivative of (see Chain Rule) of a function, 127 notations for, 53, 89, 93, 100
Compression of a gas: of two variables, 667 partial, 633
adiabatic, 158 (10) Cross product, 571 of a polynomial function, 76
nonadiabatic, 158 ( 1 1 ) definition, 577 of a power, 74, 80
Concavity: geometric significance of, 579 Power Rule for Functions, 84
definition of, 120 properties of, 577-578 of a product, 77
of a parabola, 22, 103 (20) Curvature, 105 (15), 603 of a quotient, 80
Concavity Theorem, 124 of a circle, 604 of secant, 246
Conchoid, 443 (58-61) circle of, 605 of sine, 241-243
Cone: definition, 603 of a sum, 76
element of, 341 maximum, 606 of tangent, 246
frustrum of, 408 of a polar curve, 608 (26, 27) of a vector function, 594
slant height of, 412 (43) of a space curve, 607 (See also Differentiation; Power Rule for
surface area of, 407, 412 (43) Curve: series)
Conic: closed, 404, 737 Determinant, 577, 763 (16)
central, 562 length of, 404 Differentiability, 52, 74
noncentral, 562 parametric equations for, 398 and continuity, 67, 646
(See also Conic sections) piecewise smooth, 737 of functions: of three variables, 648
Conic sections, 339 reverse orientation of, 738 of two variables, 646
definition, 339 smooth, 404, 737 at a point, 52
degenerate, 339 (See also Arc) on a set, 52
polar equations of, 443 (79-85) Cusp, 119, 132 Differentiable, 52, 74
(See also Conic) Cycloid, 400, 609 (38) infinitely, 101
Connected set, 745 Cylinder, 557 Differential, 93
Conservation of energy, 733-734 directrix of, 557 approximation error, 105
Constant function, 31 generator, 557 approximations, 98, 105 (1-7)
definition of, 31 Cylindrical coordinates, 716 Chain Rule for, 97
derivative of, 61 (13),75, 77 ( 9 ) definitions, 93
limit of, 62 Decreasing function, 121 geometric interpretation, 95
Constant sequence, 481 Decreasing sequence, 483 graphical interpretation, 95
Constraint curves, 672 Definite integral: numerical interpretation, 96
Continuity, 45, 64 definition of, 199 Differential equations, 179, 280, 768
of a composition of functions, 66 properties of, 201 Bernoulli, 773 (20)
and differentiability, 67, 646 (See also Integral; Integration) exact, 771
of a function, 64 Degree measure, 234, 240, 248 first order: linear, 772
on set, 64 Deleted interval, 322 separable, 769
of three variables, 633 (18) Deleted open disk, 629 homogeneous, 770, 771
of two variables, 631 Delta notation, 53 general solution of, 770
and integrability, 200 Demand, 164 particular solution of, 770
of mixed partials, 640 Derivative: implicit solution of, 768
theorems on, 65, 66 applications: to business and ecomomics, integrating factor for, 774 (19)
of a vector function, 594 162 ordinary of order n, 768
Contour curves, 626 to graphing, 117 partial, 642 (31, 32), 652, 656
Coordinate planes, 550 to linear motion problems, 159 power series solution of, 780 (9-16)
Coordinates : to maximum/minimum problems, 146 second order, 12, 180, 774
Cartesian, 550 to related rates, 155 characteristic equation for, 775
INDEX 1-3

Differential equations, second order (contd): Ellipse: Flux, 757


characteristic polynomial for, 775 area of, 753 of a vector field, 760
homogeneous, 774 center of, 346 of a velocity field, 759
linear with constant coefficients, 774 definition of, 346 Force:
nonhomogeneous, 776 equation of, 347 attractive, 610
undetermined coefficienrs method, 778 foci of, 346 central, 610
variation of parameter method, 777 graphs in standard form, 348 centripetal, 6 10
Differentiation: major and minor axes, 346 gravitational, 699
implicit, 87 parametric equations for, 399, 400 normal, 613
logarithmic, 269 reflective properties of, 348 resultant, 568
(See also Derivative) in standard position, 346 tangential, 613
Direction angles, 575 vertices of, 347 Four-leafed rose, 440
Direction cosines, 575 Ellipsis, 489 Friction, coefficient of, 276 (36)
Direction numbers, 582 Ellipsoid, 560 Function(s):
Directional derivative, 656 of revolution, 560 algebraic, 36
computation of, 658 Elliptic cone, 561 analytic, 534
definition, 657 Elliptic helix, 593 arc length, 598
of functions of three variables, 660 Elliptic paraboloid, 561 bounded, 200
geometric interpretations, 656 Elliptical torus, 688 ( 3 ) circular, 310
rate of change interpretation, 658-659 Equation: composition of, 36
Discontinuous function, 64 of a circle, 18 concave down, 120
Discount rate, 285 differential (see Differential eqhations) concave up, 120
(See also Present value) of a line: in the plane, 13 constant, 31
Discriminant, 6 in space, 556, 582 continuity of, 45, 64
Displacement, 48 of a plane, 556, 584 cosecant, 236
Distance: quadratic, 6 cosine, 236
from a point to a line, 576, 585 of a quadric surface, 560 cotangent, 236
from a point to a plane, 585 Equations: decreasing on an interval, 117
Distance between two points. for a line in space, 582 definition of, 28
in a plane, 9 parametric, 396, 398 demand, 220
in space, 552 Euler, Leonhard, 262 difference of, 34
Divergence, definition, 760 Eulers constant, 512 ( 4 7 ) differentiable, 52, 74
Divergence Theorem, 760 Eulers formula, 546 ( 2 4 ) domain of, 28
Division by zero, 2 Evolute of graph of vector function, 609 (37, elementary, 382
Domain: 38) equality of, 31
of a function, 28 Exponential functions, 255 error, 476 (18)
of three variables, 625 of base b, 256 even, 242
of two variables, 622 of base e, 262 exponential: of base b, 256
natural, 28 defined as inverse of logarithm functions, of base e. 262
of a pair of parametric functions, 398 268-269 extreme values on an interval, 133
Dot (scalar) product, 571 derivative of, 262, 263 gamma, 475 ( 3 )
properties of, 572 integral of, 289, 290 greatest integer, 337 (6, 7 )
Double integrals, 684 properties of, 256 (See also Function, largest integer)
applications of, 693 Extreme Value Theorem, 134 hyperbolic, 309
definition, 685 for functions of two variables, 669 implicitly defined, 32, 87
evaluation of, 687 Extreme values subject to constraints, 672 increasing on an interval, 117
as interated integrals, 689 Extremes : infinitely differentiable, 101
properties of, 686 absolute, 669 initial position, 599
Riemann sum leading to, 685 of a function: on an interval, 134 inverse of, 37 ( 3 ) , 251
Double integrals in polar coordinates, 712 of two variables, 669 inverse hyperbolic, 312-314
applications of, 713-716 procedure for determining, 144 inverse trigonometric, 298-301
iterated integrals for, 713, 715 relative, 118 largest integer, 39 (32),47 ( I 3 ) ,69 (25,
Riemann sums for, 712 26)
transforming rectangular integrals to, 715 Factorial, 482 (See also Function, greatest integer)
Doubling time, 281 Fence problem, 146, 172 ( I ) , 672, 678 (26) limit of, 39, 61-62, 324
Dumbo, 617 (24, 25) reversed, 677 (I) linear, 35
Dummy variable, 190 revisited, 149 logarithm base b, 257-258
First Derivative Test, 121 logarithm base e, 263
e: First moment [see Moment (first)] logistic, 287
approximated using power series, 522 First-Term-Neglected-Estimate, 507 monotonic, 194
definition of, 262 Fluid flow, two-dimensional, 732 natural exponential, 262
limit description of, 266 Fluid force, 425 natural logarithm, 263
Elasticity of a spring, 422 Fluid, incompressible, 761 nonelementary, 382, 542
Electrostatic charge, 428 (32) Fluid pressure, 424 odd, 242
1-4 INDEX

Function(s) (cont'd): Graph: Improper integrals:


periodic, 237 of an equation, 9 convergent, 468-469
piecewise continuous, 200 in three variables, 551 divergent, 468-469
piecewise monotonic, 197 of a function, 30 with infinite integrands, 470
piecewise smooth, 737-738 of inverse functions, 253 with infinite limits of integration, 468
polynomial, 35 of parametric equations, 398 involving powers of x , 472
position, 592 of a second degree equation, 354 Increasing-Decreasing Theorem, 121
potential, 733 Graph sketching, step-by-step procedure, 127 Indefinite integral, 208
product of, 35 Gravitational attraction, 699, 733 (See also Antidifferentiation)
proper rational, 370 exerted by a lamina, 700 Indeterminate forms, 458
quadratic, 35 exerted by a solid, 726 (33), 728 (14, 15) O/O, co/w, 458
quotient of, 35 Gravity, acceleration of, 48, 180, 611 0 * co, 03 - M , 464
rational, 36, 370 Green's Theorem, 752 l", coo, Oo, 465
relative extremes of, 118 applications of, 752 Indifference curves, 673
relative maximum of, 118 proof, 754 Inequalities, 4
relative minimum of, 118 Grid: solution using Chart Method, 7
scalar, 592 plane, 684 Infinite product, 512 (41-43)
secant, 236 polar, 712 Infinite sequence, 478
of several variables, 625 solid, 701 bounded, 484
sine, 236 Growth: Cauchy, 512 (39)
sum of, 34 constrained, 286 convergent and divergent, 479
supply, 22 of money, 283, 284 definition, 478
tangent, 236 of populations, 287 elements of, 478
of three variables, 625 in size of geometric objects, 55, 56, 57 general elements of, 478
continuity of, 633 (18) (19, ZO), 72 (27), 77, 83, (37, 46, 47), increasing and decreasing, 483
differentiability of, 643, 648 85, 95-97, 100 (4,105 ( 1 4 ) , 106 (17 limit of, 479
gradient of, 661 Z f ) , 204-205, 209 (17), 637 Limit Theorem for, 480
integrable, 701 Growth and decay, 280, 282 monotonic, 483
limit of, 633 (18) of partial sums, 496
partial derivative of, 639 Half-life, 282 special limits of, 482
total differential of, 648 Hanging cable (catenary curve), 31 1 Squeeze Theorem for, 481
of two variables, 622 Heat-seeking particle, 658, 660, 666 (31) Infinite series, 488
Chain Rule for, 650 Helix: absolutely convergent, 505
composition with one variable function, circular, 593, 606 alternating, 506
630 elliptical, 593 basic facts about, 490
continuity, 631 Higher-order derivatives, 100 binomial, 537
critical points of, 667 interpretations of, 102 Comparison Test, 499
differentiability, 643, 646 Higher-order partial derivatives, 640 conditionally convergent, 507
extreme of, 666 Hooke, Robert, 422 convergent, 488
gradient of, 660 Hyperbola, 310, 348 definition, 488
graph of, 623 central rectangle of, 350 Divergence Test, 491
homogeneous of degree n, 6 definition, 348 divergent, 488
integrable, 685 equation of, 349 First-Term-Neglected Test, 507
limits, 628 Hyperbola graph in standard form, 350 general procedure for convergence testing,
partial derivative of, 633 Hyperbolic functions, 309 508
polynomial, 632 definitions, 309 Generalized Ratio Test, 512 ( 4 4 )
unbounded, 43 derivatives of, 310 geometric, 493
vector valued, 591, 592 identities, 309 harmonic, 492
Fundamental Theorem of Algebra, 371 integrals of, 311 Integral Test, 496
Fundamental Theorem o f Calculus, 175, inverses of, 312 Limit Comparison Test, 500
197, 203, 209, 268, 382 integrals leading to, 314 Maclaurin, 532
Form I, 206 Hyperbolic paraboloid, 562 (See also Maclaurin series)
Form 11, 208 Hyperboloid: partial sums of, 488
Fundamental Theorem of Calculus for Line of one sheet, 560 positive-term, 496
Integrals, 747 of revolution, 560-561 power, 515, 525
of two sheets, 561 (See also Power series)
Good driving, 614 p series, 498
Gradient, 659 Identities: Ratio Test, 501, 525
of functions: of three variables, 661 hyperbolic, 309 Root Test, 503
of two variables, 659 trigonometric, 239-240 sum of, 488
interpretation of, 662-664 Implicit differentiation, 87 Taylor, 532
maximum value of, 659 using the Chain Rule, 654 (See also Taylor series)
minimum value of, 659 using differentials, 98 telescoping, 490
Gradient field, 733 Implicitly defined functions, 32, 87 Infinitely wiggly function, 337
INDEX 1-5

Inflation, 284 Integration (contd): Level curves, 625, 662


Inflection points, 120 using series; 542 Level surfaces, 627, 662
Initial value problem, 180, 770-771 using tables of integrals, 382-383 LHospital, Guillaume Francois, Marquis
Inner partition: (See also Antiderivative; Definite de, 458
of a plane region, 684 integral: Integral) LHospitals Rule, 459, 471
of a polar region, 712 Intercepts, 16 Limacon, 441
of a solid region, 701 Interest: Limit Comparison Test, 500
Inner product, 571 continuously compounded, 283 Limit(s):
(See also Dot product) effective annual rate, 284 and calculators, 45
Integrable, 199-200, 685, 701 Interior point, 631 of a composition, 66
Integral(s), 199 Intermediate Value Theorem. 66 of a constant function, 62
applications, 210, 216, 223, 397, 404, 413, Intermediate variables, 651 definition, 62, 324
422 Intersection: description of e , 266
circulation, 761 of graphs of functions, 217 of a function, 39, 62, 324
convergent and divergent, 468-471 of polar graphs, 443 of three variables, 633 (18)
definite, 199 Interval: of two variables, 628
double, 684 closed, 3 infinite, 109, 119, 334
double iterated, 688, 689 deleted, 322 at infinity, 111, 334
flux, 758 half-open, 3 of integration, 199
improper, 468 infinite, 3 left- and right-hand, 42. 102, 334
line, 741 open, 3 of (an x ) / x and x / e x as x + 0, 272
triple, 701 partition of, 186 one-sided, 42, 334
triple iterated, 703 Inverse function, 251, 298 of a product, 113, 330
of vector fields (see Circulation; Flux; derivative of, 255 of a quotient, 113, 330, 458
Line intergral) existence of, 253, 298 of a sequence, 479
of vector functions, 596 graph of, 253 of (sin A x ) / & , 243, 248
(See also Antiderivative; Definite integral; Inverse hyperbolic functions, 312 special, 62, 327
Integration) Inverse square law, 616, 699, 733 uniqueness of, 328
Integral Test, 496 Inverse trigonometric functions, 298 of a vector function, 594
Integrand, 208 (See also Arc-) Limit Theorem, 62, 330
Integrating factor, 774 (19) Involute of a circle, 404 (32) Line(s):
Integration: Irrational exponents, 256, 268 direction numbers of, 582
approximate (see Integration, numerical) Irrational number, 2 equations of, 13
of cosecant, 291 Irrotational flow, 762 generating, 234
odd powers of, 363 Isotherms, 627 slope, 10
of cosine, 290 Iterated integrals: in space, 582-584
powers of, 293 double, 688 parallel, 582
of cotangent, 291 evaluation using mnemonic device, 690 parametric equations for. 582
powers of, 295 evaluation using strips, 690-691 point of intersection with a plane, 584
elementary substitution method, 181, 183 reversing order of, 791 skew, 583
of exponential functions, 289 triple, 703 slope, 582
of hyperbolic functions, 31 1 changing order of, 707 symmetric equations, 583
involving inverse hyperbolic functions, in cylindrical coordinates, 721 vector equation for, 582
314 evaluation using mnemonic device, 706 Line integral, 737, 741
involving inverse trigonometric functions, in spherical coordinates, 722 along a space curve, 765 ( 1 0 )
305 arc length, 745 (17-20)
of natural logarithm functions, 361 Jerk, 102 Fundamental Theorem of Calculus for,
numerical, 386 747
partial fractions method, 370 Kepler, Johann, 615 independence of path, 746
by parts, 360 Keplers Laws, 615 Lituus, 453 (14)
of powers, 177, 181, 209, 288, 289 Kinetic energy, 734, 740 Logarithm function:
of rational functions (see Partial fractions) kth Term Test for Divergence, 491 algebraic properties of, 258, 268
of rational functions of sine and cosine, base b, 258
380 Lagrange, Joseph Louis, 675 base e, 263
rationalizing substitution method, 378 Lagrange multiplier, 675 definition of, 258
of secant, 291 for functions of three variables, 678 derivative of, 264-265
odd powers of, 363 meaning of, 676 integral of, 361
of sine, 290 Laplaces equation, 643 (55) integral approach to, 268
powers of, 293 Law of equimarginal productivity, 678 (33) integral involving, 280, 289
substitution method, 181, 183 Least squares method, 682 (26) natural (see Logarithm function. base e )
of tangent, 291 Leibnitz notation, 93, 596, 635 table of values, 792-795
powers of, 295 Lemniscate of Bernoulli, 441 Logarithmic differentiation, 269
of trigonometric functions, 290 Length of an arc, 406 Logarithmic spiral, 443 ( 6 4 )
trigonometric substitution method, 383 (See also Arc length) Lottery, 286
INDEX

Maclaurin, Colin, 532 Motion (contd): Parallel planes, 584


Maclaurin series, 532 linear, 159 Parallelepiped:
for arctan x, 536, 540 nonlinear, 592, 599, 600, 609 rectanglar, 532
for cos x, 533, 539 volume of, 579
for cosh x, 540 Natural logarithm function (see Logarithm Parallelogram, 576
for 8 , 539 function, base e) area of, 579
for e-l/*, 538 Natural logarithm power series Parameter, 396, 582, 592
for h ( l + x), 539 representation, 536 elimination of, 398
for sin x, 532, 539 table of values, 792-793 variation of, 777
for sinh x, 540 Nearness, 322 Parametric equations, 396, 582, 592
(See also Infinite series; Power series, Newton Sir Isaac, 186, 616 of an ellipse, 399, 400
Taylor series) Newtons First Law of Motion, 61 1 of a line (in space), 582
Marginal, 162 Newtons Law of Cooling, 288 (23-26) of plane curves, 592
with respect to time, 220 Newtons Law of Gravitation, 428 (30, 31) Partial derivatives, 633
Marginal cost, 163 Newtons Second Law of Motion, 610 lternate notations for, 635
Marginal production cost, 163 Normal curve, 386 Chain Rule, 649
Marginal profit, 162 Normal line, 638 computation of, 634
Marginal revenue, 163 equation of, 664 of functions of three variables, 639
Mass: Normal vector, 583 General Chain Rule, 652
of a lamina of nonconstant density, 697 n-tuples, 625 geometric interpretation of, 634
of a nonhomogeneous solid, 709 outer limit, 758 of higher order, 640
of plate of constant density, 415 mixed, 640
Mathematical model, 143, 146, 149 Octant, 551 rate of change interpretation, 636
Mean value of a continuous function, 2 11 Oil spill, 85 Partial differential equation, 642 ( 3 / , 32),
Mean-Value Theorem, 136 Open ball, 632 652, 656 (31-35, 37)
generalized, 520 Open disk, 629 Partial fractions, 370
for integrals, 212 deleted, 629 Partition of an interval, 185, 195, 199
theoretical applications, 139 Open set, 631 even, 192
Measure of an angle, 234 Optimization, 147 norm of, 195
degree versus radian, 248 under constraints, 672 Pascal, Blaise, 425
radian, 234 involving functions of two variables, 670: Perpendicular bisector, 15
Median of a triangle, 417 672 Perpendicular lines, 12
Midpoint: involving transcendental functions, 272 Perpendicular planes, 584
of a line segment (in the plane), 9 Order properties of real numbers, 2 Perpendicular vectors (see Vector,
of a line segment (in space), 553 Ordered pairs, 9 orthogonal)
Midpoint Rule for integration, 394 (122, Orientation of increasing parameter, 737 Plane(s):
123) Orthogonal trajectory, 781 (17) angle between, 589 (8, 9 )
Mixed partial derivatives, 640 Ovals of Cassini, 455 (12) coordinate, 550
equality of, 640 and cross products, 584
Moment (first), 413 Pappus of Alexandria, 419 definition of, 583
of a lamina of nonconstant density, 697 Pappus Theorems: distance from a point to, 585
of a plane curve, 418 for fluid pressure, 426-427, 430 (86, 87) equation of, 556, 583, 584
of a plane region, 415 for surface areas, 421 (50, 51) graph of, 551, 556, 584
of a plate of constant density, 415 for volumes, 419-420 normal vector to , 583
and the probability density function, 476 for work, 426, 429 (82-85) osculating, 619 (17-19)
(19, 20) Parabola, 22, 341 parellel, 584
of a solid of nonconstant density, 711 (27) axis of, 341 perpendicular, 584
of a solid of revolution, 417 axis of symmetry of, 22 Planetary motion, 615
of a surface of revolution, 418 directrix of, 341 Point of inflection, 120
of a system of point masses: in the plane, equation of, 22, 341 Point-slope form of a line, 13
414 focus of, 341 Polar coordinates, 433, 434
on the x axis, 413 graphs in standard form, 343 arc length in, 452
of a thin wire, 745 (21-25) reflective property, 344-345 area in, 448
Moment of inertia (second moment), 420, vertex of, 341 conversion from rectangular, 436
430 (10-24), 697 Parabolic rule (see Simpsons Rule) conversion to rectangular, 435
about the axes, 697 Paraboloid, 345, 561 double integrals in (see Polar integrals)
computation using spherical integrals, 722 elliptic, 561 graphing in, 436
of a lamina about a line, 701 hyperbolic, 562 intersection of graphs in, 443
and the probability density function, 476 of revolution, 561 surface area in, 452
(21, 22) Paradox, fake, 473 (53) tangent lines in, 445
Moments, higher, 430 (14, 15) Parallel Axis Theorem, 728 (8) Polar equations, 436
Monopoly, 164 Parallel lines: Polar functions, 436
Motion: in the plane, 12 Polar integrals, 712
diagramming, 160 in space, 583 Riemann sum for, 712
INDEX 1-7

Polar moment, 699 Projectile: Rent subsidy, 168


Pole, 434 equations of motion, 61 1 Revenue, 163
Polynomial function, 35 problem, 611 function, 163
degree of. 35 range, 612 marginal, 163
derivative of, 76 p series, 498 Revolution:
higher derivatives of, 101, 515, 516 Pythagorean Theorem, 9, 365, 552 axis of, 223
Positively homogeneous of degree n, 681 (10, solid of, 223
14 Quadrant, 9 volume of, 223 (disks), 226 (shells)
Potential energy, 734 Quadratic equation, 5 surface of, 407
Potential function, 733 Quadratic factors, 371 area of, 407-410
computation of, 734-735 Quadratic formula, 6 unit of angle measure, 276
Power, 424 Quadric surfaces, 560 Riemann, Georg, 185
Power Rule for derivatives: ellipsoid, 560 Riemann sums, 200, 448, 685, 701, 712, 720,
for functions, 84 elliptic cone, 561 739, 741
integer exponents, 80 elliptic paraboloid, 561 Right circular cone, 223, 407, 412 (43)
positive integer exponents, 74 hyperbolic paraboloid, 562 Right circular cylinder, 71 (3),223, 407, 637
rational exponents, 90 hyperboloid: of one sheet, 560 Right-hand triple, 580
real exponents, 269 ( 6 ) , 270 of two sheets, 561 Right-handed system, 550
(See also Derivative) Quotient: Rolle, Michel, 135
Power Rule for integrals, 182 derivative of (Quotient Rule), 79 Rolles Theorem, 135, 520
extended to n = - 1, 289 of functions, 35 Root Test, 503
Power series, 525 limit of, 113, 330, 457, 458 Roots of a quadratic equation, 6
applications: to integration, 542 Rotation of axes, 353
to numerical approximations, 541 Radian measure, 234 Rotation, positive, 234, 276
composition with a function, 534 Radians versus degrees, 248 Ruled surface, 588 ( 4 )
differentiation of, 531 Radium, 319 (3)
integration of, 535 Radius: Saddle point, 562, 667
interval of convergence, 526 of a circle, 18 Scalar product (see Dot product)
radius of convergence, 526 of curvature, 605 Scalar projection, 573
representation of a function, 530 of gyration, 420 Scalar quantity, 563
uniqueness of representation, 534 Range: Secant function, 236
(See also Infinite series; Maclaurin series; of a function, 28 derivative of, 246
Taylor series) of a projectile, 612 integral of, 291
Present value, 285 Rate of change: integral of odd powers of, 363
of future income, 285, 364 (50, 5f),395 average, 48, 54 inverse of, 301
(127) instantaneous, 48, 54 Second Derivative Test for Relative
(See also Discount rate) of position (velocity), 48 Extremes:
Principal, 283 Ratio Test, 501-502, 525 for functions: of one variable, 124
Probability density function, 476 (17-21) generalized, 512 (44,45) of two variables, 668
Procedure: Rational number, 2, 256, 494 Second partials, 640
for convergence testing of series, 508 Real exponents, 256, 269 (4,270 Sequences (see Infinite sequence)
for determining extreme values: on a Real numbers, 2 Series (see Infinite series; Power series)
closed and bounded region, 669 Rectangular coordinates: Shell method, 226, 728 ( J J )
on a closed interval, 144 conversion from polar, 435 Sigma (summation) notation, 190, 488
for determining potential functions, 735 conversion to and from cylindrical, 716 Simply connected set, 746, 749
for graphing a function, 127 conversion to and from spherical, 718 Simpsons Rule, 389, 544
for setting up triple integrals over a solid conversion to polar, 436 error, 391
region, 706 in three dimensions, 550 Sine function, 236
for solving optimization problems, 146 in two dimensions, 9 approximated using Taylor polynomials,
for solving related rate problems, 155 Reduction formulas, 384 517, 523
Product: Reflective property: continuity of, 242
cross (vector), 571, 577 of ellipse, 348 derivative of, 242-245
derivative of (Product Rule), 77 of parabola, 344-345 integral of, 290
dot (scalar), 571 Region: integral of powers of, 293
(See also Product, inner) bounded, 669, 684 inverse of, 299
of functions, 35 regular, 702, 751 Maclaurin series of, 532, 539
inner, 571 in three space, 702 Sink, 761
limit of, 113, 330, 512 (48) type I and 11, 687 Slope:
triple scalar, 579 Related rate problems, 155, 272 of a graph, 58
Profit: procedure for solving, 272 of a line, 10
function, 163 Relative degree, 114, 500 of a line tangent to a graph, 57
marginal, 162 Relative extremes (maxima and minima), of a polar curve, 445
maximizing, 163 118, 666 Slope-intercept form of an equation of a
maximizing over time, 220 Rent control, 169 line, 13
1-8 INDEX

Smooth curve, 402, 404, 737 Taylor polynomials, 517 Vector(s) (contd):
piecewise, 737 Taylor series, 532 base of, 564, 570
Source, 761 (See also Infinite series; Maclaurin Series; components of, 564, 570
Space curve, 582, 592 Power series) coplanar, 580
Speed, 51, 599 Taylors Formula, 519-520 cross product of, 577
Sphere, 223, 552 Taylors Remainder Formula (see Taylors difference of, 566
surface area of, 411 Formula) direction, 564-565
volume of. 57 (19), 224 Taylors Theorem, 532 direction angles of, 575
Spherical coordinates, 717 Temperature, average, 210 direction cosines of, 575
Spiral: Third Derivative Test, 141 (9, 10) direction of, 569
Archimedian, 443 (62) Torus, 420 dot (scalar) product of, 571
logarithmic, 443 (64) elliptical cross section, 588 ( 3 ) equality of, 564
Squeeze Theorem, 332 Total differential, 644 function (see Vector function)
for sequences, 481 approximation using, 645, 648 geometric, 564
Standard orientation, 751 of a function of three variables, 648 head of, 564
Stokes Theorem, 762 geometric interpretation, 647 initial point, 564
Strontium 90, 319 (4, 5) Trajectory of a projectile, 609 inner product of, 571
Substitution method, 181, 183 Transcendental functions, 233 (See also Dot product)
Sum: Transportation costs, minimizing, 151 length (or norm) of, 570
of consecutive integers, 188 Transverse axis of a hyperbola, 349 linear combination of, 569
consecutive squares, 192 Trapezoid approximation, 202 magnitude (norm) of, 564, 570
Sums, upper and lower, 186 (See also Trapezoid Rule) negative of, 566
Surface, 554 Trapezoid Rule, 386, 412 (40), 475 (63), 476 norm (or length) of, 570
symmetries of, 555 (13), 544 normal to a plane, 583
traces of, 556 error, 388 orthogonal, 572, 601
Surface of revolution, 407, 558, 559 (See also Trapezoid approximation) outer unit normal, 758
area of, 409-410 Triangle inequality, 4, 568 parallel, 572, 578
centroid of, 418 Trigonometric functions, 236 parallelogram law of addition of, 568
Surface, quadric, 560 definitions of, 236 perpendicular, 572, 578
(See also Quadric surfaces) derivatives of, 245, 246 position, 592
Surplus : integrals of, 290, 293-297 projection, 573
consumer, 220 inverses of, 298, 299, 303 scalar multiple of, 567
producers, 221 Trihedral, moving, 606 scalar projection of, 573
Symmetry: Triple integrals, 701 standard representative of, 564, 570
in polar coordinates, 437, 439 applications, 704 subtraction of, 568
in three dimensions, 555 in cylindrical coordinates, 720-721 sum of, 566
abouty axis, 21 evaluation of using iterated integrals, tail of, 564
702-703 three-dimensional, 570
Tangent function, 236 Riemann sums for, 701 triangle rule for subtraction of, 568
derivative of, 246 in spherical coordinates, 721 triple scalar product, 579
derivative of powers of, 295 transforming between coordinate systems, two-dimensional, 564
integral of, 291 723 unit, 569
inverse of, 300 Twisted cubic, 595 unit binormal, 606
Tangent line, 57, 58 Type I and Type 11 regions, 687 unit coordinate, 569, 570
angle of inclination in polar coordinates, unit normal, 601
446 Unequal mixed partials, 643 (51, 52) unit tangent, 598, 601, 606
to a circle, 72 (23), 106 ( 1 6 ) Unit binormal vector, 606 velocity, 599
equation of, 82 Unit circle, 240 zero, 564
to a graph given parametrically, 401 Unit coordinate vector, 569, 570 Vector field, 731, 732
perpendicular, 445 Gnit normal vector, 601 circulation of, 761, 762
to polar graphs, 445 Unit tangent vector, 598, 601, 606 conservative, 733
slope of, 88, 402 Unit vector, 569 curl of, 761
vertical, 132, 402 Utility, 673 flux of, 759
Tangent plane, 638 inverse square, 757, 763 (12)
equation of, 664 Variable: solenoidal, 761
failure to exist, 639 dependent, 28 three-dimensional, 737 (21-25), 763 (16)
Tax(es) : dummy, 190 two-dimensional, 732
on energy consumption, 674 independent, 28 Vector function, 592
excise, 166 intermediate, 651 Chain Rule for, 596
real estate, 166 Vector(s): continuity of, 594
Tax rebate, 168 acceleration, 599 curvature of, 603
Taylor, Brook, 517 addition of, 567 derivative of, 594
Taylor approximation, 516 angle between, 572, 578 integral of, 596
error bound for, 521 angle of inclination of, 564 limits of. 594
INDEX 1-9

Vector function (contd): Vertex: Winding number, 765 (7-9)


rules for differentiation of, 595 of an ellipse, 347 Witch of Agnesi, 404 ( 3 4 )
Velocity, 48, 102, 160, 180, 599, 609 of a hyperbola, 349 Work, 422
average, 48 of a parabola, 21, 341 computed using centroids, 426
of a falling body, 48, 180 Vertical asymptotes, 109 done by continuous varying force in
initial, 180 Vertical tangents, 132, 402 straight line in direction of the force,
instantaneous, 49 Volume(s): 422
of a particle moving in space, 599, 609 of a parallelepiped, 579 done by continuously varying force in
of a particle moving in a straight line, under a surface, 684, 686, 695 straight line, 574
160 using disk method, 223 done by force along a curve, 738-739
terminal, 51 using double integrals, 686 done in stretching a spring, 422
Velocity field, 732 using polar integrals, 715
circulation of, 757, 761 using shell method, 226 Zero, division by, 2
flux of, 757 using triple integrals, 703 Zero vector, 564
sink of, 761 in spherical coordinates, 722
source of, 761 von Neumann, John, 496 (45)
35. SUdn du 1
= Qarcsec

31- s Tdu
d m - = a arcsec
l:l
- +c
37. s ( u 2 & a 2 ) 3 / 2du = U(2u2& 5a2)-
d
a

FORMS CONTAINING d a
du - arcsin& +c
a

3a4
47. S ( a 2 - u2)3/2du= -U(2u2 - 5 a 2 ) d m + -arcsin?
8 8 a
+C
48. du - +C
s(,2 - u2)3/2 a2,/&

FORMS CONTAINING ~ U UU
' -
49.

50. 55. +-3a2


2
arccos (1 "+ c

51.

52.

du
53.
S ~m = arccos

FORMS CONTAINING TRIGONOMETRIC FUNCTIOKS


59. Ssin u du = -cos u +C 67. Ssec u tan u du = sec u +C
60. s c o s u du = sin u+C 68. s c s c u cot u du = -csc u +C
61. S t a n u du = lnlsec U I + C 69. Ssin2u du = - 4 sin 2u+C
62. Scot u du = Inlsin U I + C 70. s c o s 2u du = + 4sin 2u + C
sec u du = Inlsec u + tan u / + C = lnItan(4n + &)I + C
63.
64.
65.
f cscu du = lnlcscu - cot uI + C = lnltan$ + C
Ssec2udu = tanu + C
7 1.
72.

73.
S t a n 2u du = tan u - u
Scot2u du = -cot u - u

Ssinn u du = - -
1 sinn-' u cos u
+C
+C
+ e S s i n n - 2 u du
66. Scsc' u du = -cot u + C n n
74. Scos" u du = -
1 cosn-l u sin u + "-lScosn-z u du
n n

You might also like